83
Rishi Aurobindo Memorial Academy 134 P. K. Guha Road DumDum Kol-28 Study Material of 27/4/2020 For Class Nursery to XII Class Nursery Subject Topic Execution English Letter ‘E’ Humpty Dumpty Write in copy Learn Math Number ‘5’ Write in Copy Hindi Letter ‘औ’ Write in copy Bengali औऔऔ औऔऔऔ pg 9 Write in book Class KG I Subject Topic Execution English Reading book Pg 7 Write the letter which comes before and after. - Follow the given worksheet and practice in copy Math Reading book pg 139 Count and circle – Follow the givenworksheet and practice in copy. Bengali औऔऔऔऔऔऔ औऔऔऔऔ Pg ১৬ औ]औऔऔऔ औ औऔऔऔ औऔऔऔऔऔ औऔऔऔ- Write in book Hindi औऔ १० Write in copy Class KG II Subject Topic Execution English Golden Nursery Ryhmes C After a bath – read and learn Math(Term I) Count and Match – Pg 80 Do the given exercise in copy/ worksheet Science (Term I) Pg 122 - Match the parts of the body with their use. Do the given exercise in copy/ worksheet

 · Web viewWe all know that Nouns are divided into two parts: common noun and proper noun.Apart from common and proper noun, we will also study about collective noun and compound

  • Upload
    others

  • View
    7

  • Download
    0

Embed Size (px)

Citation preview

Page 1:  · Web viewWe all know that Nouns are divided into two parts: common noun and proper noun.Apart from common and proper noun, we will also study about collective noun and compound

Rishi Aurobindo Memorial Academy134 P K Guha Road DumDum Kol-28

Study Material of 2742020For Class Nursery to XII

Class NurserySubject Topic Execution

English Letter lsquoErsquo Humpty Dumpty

Write in copy Learn

Math Number lsquo5rsquo Write in CopyHindi Letter lsquoऔrsquo Write in copy

Bengali এসো লিলি pg 9 Write in bookClass KG I

Subject Topic Execution

English Reading book Pg 7 Write the letter which comes before and after

- Follow the given worksheet and practice in copy

Math Reading book pg 139 Count and circle ndash Follow the givenworksheet and practice in copy

Bengali োলিতয মক Pg ১৬ ক]আসর ও পসরর বরণটি বোও- Write in book

Hindi गिनती १ स १० तक Write in copyClass KG II

Subject Topic ExecutionEnglish Golden Nursery Ryhmes C After a bath ndash read and learn

Math(Term I) Count and Match ndash Pg 80 Do the given exercise in copy worksheet

Science (Term I) Pg 122 - Match the parts of the body with their use

Do the given exercise in copy worksheet

Class ISubject English 1

Chapter 1 naming words or noun

Names of people animal things and places are called naming words

Some words name people For example girl and mother

Some words name animals For example dog and cat

Some words name places For example school and park

Some words name things For example ball and moon

Subjects EVS

Chapter 1 My body parts

Our body is made up of different partsEach part of the body has an important role to play

We have two eyes two ears two hands one nose one tongue two legs two knees 10 toes and 10 fingers

Functions of body parts-

I see with my eyes

I taste with my tongue

My head rests on my neck I can turn my head with the help of my neck

I breathe and smell through my nose

I write and hold things with my hands and fingers

I stand walk dance and run on my feet

I eat and speak with my mouth

I bite and chew food with my teeth

I feel with my skin

_______________________________________________________________________________________________

Class II

Subject ndash English language Topic ndash ch 1 NounsDefinition of NounNouns are naming words that tells us the names of people place animal and things For example- boy girl dogcat park zoo basketapple etc In this chapter we learn two types of nouns-

Common noun Proper noun

Common noun ndashNames of people place animal things in general are called common noun Examples ndash

1 I love to play in the park2 The sky is blue3 Cow gives us milk4 The policeman catches the thieves

Proper noun ndashSpecial names given to people placesanimals and things are called Proper nouns It always starts with a capital letter Proper Noun are names of-

Person- Tom Harry Countrycitystates- IndiaKolkataWest Bengal Daysmonths- Monday May BooksfestivalsThe Bible Holi Mountainrivers- Mt Everest the Ganga

Examples- 1 Mr Das is an old man2 Taj Mahal is in Agra3 I live my pet dog Mac verymuch4 My uncle gifted me Titan watch

Difference between ndashCommon noun

1 girl2 car3 park4 aeroplane5 bag

Proper noun1 Devika 2 Maruti3 Nicco Park4 Air India5 Skybag

Subject ndash EVS Topic ndash My FamilyFamily is a group of people living together in the same house and is related to each other There are two main kinds of family-1 Nuclear family- In this family only father mother and their children live together in the same houseIt is also

known as a small or nuclear family2 Joint family ndash In this family grandfather grandmother father mother their children uncles aunt s and

cousins all live together in the same house It is also known as a big or joint familyMembers of the familyGrandfather ndash Father of your father or motherGrandmother- Mother of your father or motherUncle ndash Brother of your father or motherAunt ndash Sister of your father or motherSibling- Your own brother or sisterCousins ndash The children of your uncle and auntMaternal grandparents- Parents of your motherPaternal grandparents- Parents of your father

All members of the family has a common last name that is called surname

______________________________________________________________________________________________

Class III

Subject ndash Eng language Topic ndash ch 3 NOUNS

NOUNS

Nouns are the words that names people place animal and things Around us we see so many things Everything that we see has some name Example- computer blackboard chalk book teacher house cat dog etc

In this chapter we will learn about ndash

Common NounProper NounCollective noun

Common Noun

Common Noun are names of people place animal things in general For example girl boy hospital park ball car etc

Proper noun

Proper nouns are the specific or particular name of person place animal and thingsExamples- Mina Mt Everest Delhi Tommy etc Proper noun is always capitalized

Uses of Proper nouns- They are special names of

people such as RitaJackRam etc Countries citiesstates such as IndiaKolkataWest Bengal Mountains rivers seas lakes such as Mt Everest the Ganga the Indian Ocean Dal

Lake

Subject social studies

Topic My family and I

A family is a group of people who are related to each other by birth Members of a family always give love and support to each other

There are different kinds of families ndash nuclear family joint family and single parent family

Nuclear family means where father mother and kids stay together

Joint family means where many members of the family stay together For example grandfather grandmother father mother uncle aunt cousins live together

Single parent family means where the kids stay with any one of the parent like father or mother

It is always a fun when whole family spends time together example going to a picnic because it creates a strong bond among each other

Class IV

Subject English language Topic Nouns Compound nouns

We all know that Nouns are divided into two parts common noun and proper nounApart from common and proper noun we will also study about collective noun and compound nouns A common or proper noun that is the name of a collection or group of similar things or persons is known as a collective Noun Example crowd committee herd United Nations etc Example Mother has lost her bunch of keysThe word bunch is a collective noun

1 A herd of cows was grazing in the fieldHerd- collective noun

2 All living creatures need food to surviveCreatures- collective noun

3 The Ganga is a river sacred to IndiansIndians- collective noun

Compound noun some common and proper nouns are made up of more than one word They are known as compound nouns or simply compounds Example newspaper (news+paper) ashtray (Ash+tray) plaything (play+thing) etc1 My penfriendlives in a town in South AfricaPenfriend South Africa- compound noun

2 My grandparents live in New DelhiGrandparents New Delhi- compound nouns

3 Please do not park your cars in the drivewayDriveway- compound noun

Subject Social studies Topic A peep into the past

The story of the past is called history It is a journey across time and space that takes us to another period of time where people lived differently It is believed that the way of living eating clothes art were all different from what we have today We need to study history becauseIt tells us about the important people of the pastIt helps us to gain knowledge about the past and the present and help us to understand themHistory guides us to set an aim and warns us not to repeat the same mistakesIt helps us to understand the development of mankindIt helps to developessential skills to become a better person

Division of the pastHistory is so vast that it is divided into two parts ndash BC (Before Christ) and AD (Anna Domini) This helps us to understand about the past clearly Indian history and itrsquos civilisation are the worldrsquos oldest civilisation Indian history is divided into ancient medieval and modernCivilisation mean the stage of human social and cultural development

Subject COMPUTER Chapter-2 (MS WINDOWS)WINDOWS IS AN OPERATING SYSTEM THAT HELPS THE USER TO OPERATE AND CONTROL THE OVERALL ACTIVITIES OF A COMPUTER SOME POPULAR VERSIONS OF WINDOWS ARE WINDOWS 98 WINDOWS 7 WINDOWS 8 AND WINDOWS 10

DESKTOP THE FIRST SCREEN THAT WE SEE WHEN THE OPERATING SYSTEM (WINDOWS)HAS FINISHED LOADING IS KNOWN AS DESKTOP

THE FEATURES OF DESKTOP ARE bullIT IS WHERE ALL THE ICONS OF THE DIFFERENT APPLICATIONS ARE DISPLAYEDbullIT IS THE HOME FOR ALL SHORTCUTS PROGRAMS CAN BE QUICKLY OPENED BY CLICKING ON THE RESPECTIVE SHORTCUTSbullIT HOLDS VARIOUS INFORMATION SUCH AS TIME AND DATE

ICONSICONS ARE SMALL PICTURES OF DIFFERENT APPLICATIONS THAT ARE PRESENT AND INSTALLED IN THE COMPUTER FOR EXAMPLE COMPUTER ICON AND RECYCLE BIN ICON

DESKTOP GADGETS A DESKTOP GADGET IS A SMALL APPLICATION OR A WIDGET THAT RECIDES ON A COMPUTERS DESKTOP FOR EXAMPLE CLOCK CALENDAR ETC_______________________________________________________________________________________________

Class VSubject Socialal studiesTopic Evolution of man

Millions of years ago the earth looked much different as it is seen todayIt was totally different and it was very difficult to live Everything looked very different plants animals and humans everything looked very different from what they are now All the changes took place gradually with the changing environment Earliest ancestors of the humans were known as the Ramapithecus who lived about 14 million years ago They looked like ape that walked upright and weighed about 14kg With the passage of time there was a gradual development in the structure posture features and teeth of the early humansIt is said that the apesthat lived on treetops started living on the ground and stood on two limbs are called hominids They used their limbs to hold things and make tools Finally they changed to homosapiens mean wise human We humans belong to this group Life of early humansThe early men were hunter ndash gatherers They moved from place to place in search of food water and shelter They hunted animals and ate them raw gathered fruits nuts roots and seeds for eating They didnrsquot know how to build houses so they used live on treetops or in the caves or rock shelters In summers they remained uncovered but in winter they used to cover themselves by using bark of trees leaves and animal skins

Subject English language Topic Adjectives Interrogative and Emphasizing

Interrogative adjectiveAn interrogative adjective is used with a noun that it qualifies to ask a question The word interrogative mean to ask questions Example Which part of the world do you come fromThe word lsquoWhich is an interrogative adjectivesEmphasizing adjectivesSometimes we use adjectives to emphasize the effect of what we say Words which are used to emphasize a nounExample Rita came late and had to stand at the very end of the lineThe word Veryis an emphasizing adjective It actually qualifies or emphasizethe noun end

1lsquoDo not listen to what Ravi says for he is a complete foolrsquo said RohanComplete- emphasizing adjective

2Kiran is a great supporter of non violenceGreat- emphasizing adjective

3rsquoWhat crime has the prisoners committedrsquo asked the judgeWhat- interrogative adjective

4Whose book is that lying on the tableWhose- interrogative adjective

Subjects COMPUTER CHAPTER-1 (CHARACTERISTICS OF A COMPUTER)A COMPUTER SYSTEM IS A ONE THAT CONSISTS OF ALL THE SOFTWARES HARDWARES CPU AND MEMORY DEVICES SOME IMPORTANT TERMSTHE COMPUTER WORKS ACCORDING TO THE COMMANDS OR INSTRUCTIONS THAT WE GIVE

A SET OF INSTRUCTIONS IS CALLED THE PROGRAM A SET OF ONE OR MORE PROGRAMS IS CALLED A SOFTWARE THE PHYSICAL COMPONENTS OF A COMPUTER ARE CALLED HARDWARE

FOUR OPERATIONS OF A COMPUTER SYSTEM INPUTINPUT IS SUPPLIED TO THE COMPUTER WITH THE USE OF A KEYBOARD A MOUSE A MICROPHONE

OR OTHER INPUT DEVICES PROCESSING PROCESSING IS DONE INSIDE THE COMPUTER IN THE CPU PROCESSING IS THE CONVERSION

OF THE INPUT INTO THE OUTPUT STORAGE STORAGE REFERS TO THE SAVING OF INFORMATION FOR LATER USE INFORMATION IS STORED

IN THE MEMORY OF A COMPUTER THERE ARE TWO TYPES OF COMPUTER MEMORY PRIMARY MEMORY AND SECONDARY MEMORYPRIMARY MEMORY IS FAST SMALL AND EXPENSIVE THE TWO TYPES OF PRIMARY MEMORY ARE RANDOM ACCESS MEMORY(RAM) AND READ ONLY MEMORY (ROM) RANDOM ACCESS MEMORY (RAM)THIS MEMORY IS VOLATILE IN NATURE WHICH MEANS THAT THE

INFORMATION STORED IN IT IS AUTOMATICALLY ERASED WHEN THE COMPUTER POWER IS TURNED OFF

READ ONLY MEMORY (ROM) THIS TYPE OF MEMORY IS NON VOLATILE IN NATURE WHICH MEANS THAT DATA IS NOT LOST WHEN THE COMPUTER POWER IS TURNED OFF

OUTPUT OUTPUT IS THE RESULT PRODUCED BY A COMPUTEROUTPUT MAY BE VIEWED ON THE MONITOR SCREEN HEARD THROUGH SPEAKERS PRINTED THROUGH PRINTER ON PAPERS OR STORED IN THE MEMORY FOR LATER USE

Class VI

GEOGRAPHY CHAPTER 1 REPRESENTATION OF GEOGRAPHICAL FEATURES

GLOBE AND MAPS

GLOBE- A Globe is a three ndashdimensional model of the Earth

EQUATOR ndash An imaginary line runs through the centre of the earth from east to west

NORTHERN HEMISPHERE ndash the half of the north of the equator is called Northern Hemisphere

SOUTHERN HEMISPHERE ndash the half of the south of the equator is called Southern Hemisphere

LATITUDE - Imaginary parallel lines running on the globe from east to west

LONGITUDE ndash The imaginary lines semicircle in nature running from the North Pole to South Pole Also called meridian

MAP ndash Representation of the Earth as a whole or parts of it drawn on a flat surface to a scale

SKETCH ndash A rough drawing of a place not drawn to a scale

PLAN ndash A plan is prepared to show more details about a small area

COMPUTER - VI CHAPTER-6 (INTRODUCTION TO WINDOWS 10)

THE MOST RECENT VERSION OF WINDOWS IS WINDOWS 10 IT WAS RELEASED IN 2015

WINDOWS 10 IS DESIGNED TO WORK FOR YOU AND NOT YOU WORKING FOR IT THE WINDOWS 10 OPERATING SYSTEM INCLUDES A NUMBER OF NEW FEATURES LIKE IMPROVED PERFORMANCE ON MULTI CORE PROCESSORS IMPROVED G U I DATA SECURITY AND BETTER BOOTING PERFORMANCE

WINDOWS 10 HAS 1187 WITH WEARING FEATURES SETS AND INTENDED HARDWARE

THE VARIOUS EDITIONS AVAILABLE FOR WINDOWS 10 ARE AS FOLLOWS

WINDOWS 10 HOME

WINDOWS 10 HOME IS THE EDITION OF WINDOWS 10 THAT CONTAINS THE LEAST NUMBER OF FEATURESWINDOWS 10 HOME IS THE CONSUMER FOCUSED DESKTOP EDITION WITH SUPPORT FOR BOTH PC AND TOUCH ENABLED TABLETS WINDOWS 10 HOME IS AVAILABLE IN BOTH 32 BIT AND 64 BIT VERSIONS

WINDOWS 10 PRO

WINDOWS 10 PRO HAS FEATURES ESSENTIAL FOR SMALL OR MEDIUM BUSINESSESIT HAS A LARGE NUMBER OF EXTRA FEATURES TO MEET THE NEEDS OF SMALL BUSINESSES

WINDOWS 10 ENTERPRISE

WINDOWS 10 ENTERPRISE IS BUILT ON WINDOWS 10 PROIT HAS FEATURES WHICH ARE DESIGNED TO MEET THE NEEDS OF MEDIUM AND LARGE ORGANISATIONSTHIS WINDOWS EDITION TARGETS THE ENTERPRISE SEGMENT OF THE MARKET

WINDOWS 10 MOBILE

WINDOWS 10 MOBILE IS DESIGNED FOR CUSTOMER ORGANISATION THAT ARE USING THE WINDOWS 10 PLATFORM ON SMARTPHONES AND SMALL TABLETS

WINDOWS 10 EDUCATION

WINDOWS 10 EDUCATION IS DESIGNED TO MEET THE NEEDS OF STAFF ADMINISTRATORS TEACHERS AND STUDENTS OF SCHOOLS

WINDOWS 10T EDITION

WINDOWS 10T EDITION OF WINDOWS 10 IS DESIGNED FOR USE IN SMALL FOOTPRINT AND LOW COST DEVICES

Subject English language

Topic Noun kinds

NOUN KINDS

Noun is a naming word Nouns are names of people places animals or things

For example Ashley Mumbai tiger pencils

Kinds of nouns

Proper noun the name of a particular person place or thing is called proper nounExample

1 The Himalayas stand to the north of India2 Monika is the school captain

Common noun announce that names people place or thing in general is called a common noun Example

1 Kalidas was the greatest dramatist of India2 William Shakespeare is Englands national poet

Collective Noun A collective noun is the name of a collection of people or things taken together and spoken of as a whole Example

1 The feet were completely destroyed in the fierce attack2 The crew revolted against Captain Grand

Abstract Noun An abstract noun is the name of some quality state or idea Example1 Diversity Indias biggest strength2 Patience is a virtue

_____________________________________________________________________________________

Class VII

Subject English 1 Topic Articles

ARTICLES

There are three articles ------ A An The

1 Indefinite articles- The Articles a and an are called Indefinite Articles They do not point to a particular person or thing They are used with singular countable nouns It does not identify a specific noun

Uses of Indefinite articles A

Before a singular noun beginning with a consonant or a vowel with a consonant sound For example a cattle a university a one way track

Before a proper noun which is either unfamiliar or holds a special meaning For example For example A Mahesh Sharma wants to see youNeela wants to be a Tendulkar when she grows up

With a number beginning with a consonant sound For example a ten- rupee note a hundred years

Before half when half follows a whole number or after half when it isnrsquotExample one and a half litres (before) Half a litre (after)

With an expression of quantity For example a lot of time a dozen oranges a little while In exclamation before nouns Example Such a mess What a pity

Uses of Indefinite articles An

Before a singular noun beginning with a vowel sound Example an ant an egg an umbrella Before an abbreviation beginning with a vowel or a consonant with the vowel sound

Example an MP an MA Before the word beginning with a silent h Example an heir an hour an honour

Note We do not say a milk or a lemonade because they are uncountable nouns ever we see a cup of milk or a glass of lemon and

Subject GEOGRAPHY

CHAPTER 1 REPRESENTATION OF GEOGRAPHICAL FEATURES

TOPIC- TOPOGRAPHICAL MAPS

Topographical Maps- topographical maps are small-scale maps with detailed depiction of both natural and human-made features

Conventional colours-different colours used in a map are known as conventional colours

TYPES OF SCALE-

Verbal Statement- Scale written in a statement from like 2 cm to a Km is called verbal statement It means that 2cm on the map represents 1 km on the Earth

Representative Fraction- Scale can also be written as a representative fraction (RF)

Example 150000 it means 1cm on the map represents 50000cm or frac12 km on the ground It shows the ratio between map distance and ground distance

Linear Scale Scale can be drawn on a line to show map distance equivalent to ground distance A line is divided into equal parts Each parts represents the actual distance on the ground in mkm

SUBJECT-COMPUTER

CHAPTER-1 (COMPUTER FUNDAMENTALS)

COMPUTER LANGUAGES

THE THE TERM COMPUTER LANGUAGE REFERS TO A SYSTEM OF RULES AND SYMBOLS THAT ARE DESIGNED TO GIVE INSTRUCTIONS TO A COMPUTERTHE COMPONENTS OF A COMPUTER SYSTEM CANNOT PERFORM A TASK BY THEMSELVES THEREFORE THEY HAVE TO BE GIVEN INSTRUCTIONS TO PERFORM ANY TASKCOMPUTER LANGUAGES ARE USED TO CREATE PROGRAMS USING LOGIC BASED ALGORITHMSTHE RULES OF A COMPUTER LANGUAGE IS

KNOWN AS SYNTAX WHEREAS THE TERM SEMANTICS REFERS TO THE MEANING OF LANGUAGESCOMPUTER CANNOT DEVELOP ANY PROGRAMMING LANGUAGE BY THEMSELVES AND CANNOT THINK INTELLIGENTLY UNLESS THEY ARE ASSISTED BY THE HUMAN BEINGS THEREFORE THE PROGRAMMERS DEVELOP A SET OF METHODS AND TECHNIQUES A SET OF INSTRUCTIONS USED TO PERFORM A SPECIFIC TASK IS CALLED A PROGRAM

A PROGRAMMING LANGUAGE IS ALSO KNOWN AS A COMPUTER LANGUAGE CODED BY PROGRAMMERS TO WRITE INSTRUCTIONS FOR A COMPUTERTHE COMPUTER TAKES THESE INSTRUCTIONS AS INPUT AND PRODUCES THE DESIRED OUTPUT

TYPES OF COMPUTER LANGUAGE

THE COMPUTER LANGUAGE CAN BE BROADLY DIVIDED INTO TWO CATEGORIES

LOW LEVEL LANGUAGES HIGH LEVEL LANGUAGES

LOW LEVEL LANGUAGES

A LOW LEVEL PROGRAMMING LANGUAGE REFERS TO THE LANGUAGE THAT IS UNDERSTOOD BY A COMPUTER DIRECTLYTHE PROGRAMMER MUST HAVE AN IN-DEPTH KNOWLEDGE OF DIFFERENT COMPUTERS TO WRITE PROGRAMS IN A LOW LEVEL LANGUAGE THE TWO TYPES OF LOW LEVEL LANGUAGES ARE MACHINE LANGUAGE AND ASSEMBLY LANGUAGE

MACHINE LANGUAGE

COMPUTER HARDWARE UNDERSTANDS ONLY MACHINE LANGUAGE AS IT IS THE FUNDAMENTAL LANGUAGE OF A COMPUTER WHICH UNDERSTANDS ONLY THE TWO DIGITS 0 AND 1 KNOWN AS THE BINARY DIGITSIT IS A SYSTEM OF INSTRUCTIONS EXECUTED DIRECTLY BY THE CPU WITHOUT ANY TRANSLATION AND HENCE IS EXECUTED QUICKLY AS COMPARED TO PROGRAMS IN OTHER LANGUAGES PROGRAMS IN MACHINE LANGUAGE NEED DIFFERENT BINARY LANGUAGE PROGRAMS TO SOLVE THE COMPLEX TASKSPROGRAMMERS OFTEN FIND IT DIFFICULT TO WRITE PROGRAMS IN THIS LANGUAGE

ASSEMBLY LANGUAGE

AN ASSEMBLY LANGUAGE IS A LOW LEVEL PROGRAMMING LANGUAGE BUT THE INSTRUCTIONS ARE WRITTEN IN THE FORM OF WORDS KNOWN AS MNEMONICS TO CONVERT AN ASSEMBLY LANGUAGE INTO TO MACHINE CODE WE REQUIRE A UTILITY PROGRAM WHICH IS KNOWN AS AN ASSEMBLER ASSEMBLY LANGUAGE PROGRAMS CANNOT BE EXECUTED DIRECTLY BY A COMPUTER THEREFORE THEY ARE SLOWER THAN THE MACHINE LANGUAGE PROGRAMSIT IS EASIER TO WRITE PROGRAMS IN THE MACHINE LANGUAGE THEN AND IN ASSEMBLY LANGUAGE

_______________________________________________________________________________________________

CLASS-VIII

SUBJECT-COMPUTER CHAPTER - Operating system and graphical user interface Role and functions

Need of operating system

It co-ordinates different hardware and software componentsof a computer system It supervises the various actions of the computer system and enables the computer to work in a effective

manner It helps in smooth functioning of various peripherals

In a multi-tasking operating system it determines the order and time to be allowed for each application before giving another application a turn

It sends messages to the system operator about the status of operation or any error that may have occurred while running the application

What is operating system

An operating system is an integrated system of programs that manages various resources and the overall operation of the computer system It is designed to support various activities of computer system in a systematic way

Role of an operating system-

Operating system enables the user to use the system effectively An operating system manages various application that runs on a computer and shares computerrsquos resources User interacts with operating system through command line interface and graphical user interface

Function of an operating system -

Booting the computer it is the start up procedure of a computer system Loading theprograms in the memory - when system is ready the operating system loads certain program

automatically Manages resources it manages between the hardware and software resources Detecting and correcting errors- if the supporting hardware or software doesnrsquot works properly then the

operatingsystem tries to rectify it Ensuring data security programs and data donot interfere with each other Maintaining the internal clock of system - maintain internal clock of system when system is shut down

SUBJECT-GEOGRAPHY CHAPTER 1 REPRESENTATION OF GEOGRAPHICAL FEATURES THROUGH CONTOURSTopographical maps- topographical maps are small-scale maps with detailed depiction of both natural and human-made features

Contour lines- contour lines are imaginary lines drawn on a map joining places having the same height above the mean sea level

Contour linesContour interval- It is an interval at which contour lines are drawnIndex contour- At every 100-m interval a thick brown contour line is called index contour It is drawn for calculating height

Spot heights- The spot heights show heights in metres above the mean sea level

SUBJECT- English Language CHAPTER - The Sentence

Complex sentence-

We have seen that a Complex Sentence consists of a Principal Clause with one or more Subordinate Clauses

We have also learnt that there are three kinds of Subordinate Clauses The Adjective Clause the Adverb Clause and the Noun Clause

1) THE ADJECTIVE CLAUSE

An ADJECTIVE CLAUSE does the work of an ADJECTIVE It qualifies a noun or pronoun In the following examples in each set two commonly patterned sentences are compared one with an ADJECTIVE and the other with an ADJECTIVE CLAUSE ( The Adjective or Adjective Clause qualifies the Noun placed in the box)

a) He is a lazy boy (Adjective)

b) He is a boy who is lazy (Adjective Clause)

a) It is a beautiful place for the tourists (Adjective)

b) It is a place where tourists come for the scenic beauty (Adjective Clause)

a) We have enough funds for the work (Adjective)

b) We have funds which would be enough for the work (Adjective Clause)

Convert the sentence from simple to complex sentence-

1) We believe his honesty Ans- We believe that he is honest

2) This is the birth place of RamaAns- This is the place where Rama was born

____________________________________________________________________________________________

Class IX

Subject English Language

Topic Preposition

Date 270420

PREPOSITIONS

A preposition is a word placed before a noun or a pronoun to show in what relation the person or thing denoted by it stands in regard to something else

EXAMPLE

There is a clock on the wall

The preposition on shoes relationship between clock and the wall

Smitha is afraid of lizards The man jumped off the bus

Here the preposition of shows the relationship between afraid and lizards

The preposition off shows the relation between jumped and bus

The noun or the pronoun which follows a preposition is called its object

So in the first Example wall is the object of the preposition on

A preposition can have more than one object For example

The plane flew overhouses and meadows

Prepositions are used to express a number of relationship including time locationmannermeans quantity purpose and state or condition

Points to remember

Preposition joins a noun to another noun or a pronounThere is a cow in the field

A preposition joints a noun to a verbThe cat runs after the rat

Preposition can have two or more than two objectsThe road runs over hills and plain

A preposition also joins a noun an adjectiveHe is fond of tea

Generally a preposition comes before an object Sometimes it comes even after an object asWhat are you looking atThis is the house I live in

Subject- Computer Application

Chapter 2 Introduction to Java

Java API An application programming interface (API) in the context of Java is a collection of prewritten packages classes and interfaces with their respective methods fields and constructors

Byte Code Java bytecode is the result of the compilation of a Java program an intermediate representation of that program which is machine independent The Java bytecode gets processed by the Java virtual machine (JVM) instead of the processor JVM The Java Virtual Machine (JVM) is the runtime engine of the Java Platform which allows any program written in Java or other language compiled into Java bytecode to run on any computer that has a native JVM

Platform A platform is the hardware or software environment in which a program remains

Java platform The Java platform differs from most other platforms in that itrsquos a software only platform that runs on top of other hardware-based platforms

Applet and Application The fundamental difference between the two Java programs is that an application program is designed to run on a stand-alone machine whereas an applet is a web-version of an application which is used to run a program on a web browser

WORA ldquoWrite once run anywhererdquo (WORA) or sometimes write once run everywhere (WORE) is a slogan created by Sun Microsystems to illustrate the cross-platform benefits of the Java language

Class XSubject Topic Summary Execution

ECONOMICSFACTORS OF PRODUCTION Sub-topiclsquoLABOURrsquo

We shall start our class by discussing the topic taught in the last class

lsquo DIVISION OF LABOURrsquo- By division of labour we mean specialization in workIt refers to splitting up the work of labour involved in the production of a particular commodity into several parts and each part and sub-part is performed by a specialist

Now let us start by the Advantages of Division of labouraIt increases the level

Questions

1 Differentiate between Product-Based division of labour and Process-Based division of labour

Product-Based division of labour

Process-based division of labour

It is also known as simple (or occupational) division of labourUnder it everybody performs a particular occupations The entire is done by the same person

When a person or group of persons undertakes a specialised function which is supplementary to the production of final commodity and service This is also as complex division of labour

It is simple It is complex

of productionbSince the product is produced by an expert workerbest quality of product is producedcIt saves time and toolsd it promotes inventions in the methods and techniques of productioneIt leads to reduction in costs fAll workers get work according to their abilities and choices

Now let us discuss the disadvantages of Division of labour

a Since many workers are involved in the production of a commodity no one has the sense of responsibility

b The constant and repetition of the same work again and again make the work monotonous

c Division of labour facilitates production on large scale Hencethere is fear of over production

d Because of territorial division of labour some areasregions become more developed than others

It is based on labour-intensive techniques of production

It is based on capital-intensive techniques

It is generally found in small enterprises

It is generally found in large enterprises

Example Indian farmers doing all farm activities

Example A modern garmet factory where one person takes the measurementanother does the cuttingsome sew the clothes while a few workers button them and other iron them

2 What are the advantages of Division of labouraIt increases the level of productionbSince the product is produced by an expert workerbest quality of product is producedcIt saves time and toolsd it promotes inventions in the methods and techniques of productioneIt leads to reduction in costs fAll workers get work according to their abilities and choices

3Discuss the disadvantages of Division of laboura Since many workers are involved in the

production of a commodity no one has the sense of responsibility

b The constant and repetition of the same work again and again make the work monotonous

c Division of labour facilitates production on large scale Hence there is fear of over production

d Because of territorial division of labour some areasregions become more developed than others

English 1 Transformation of sentences

Sentences A sentence is a group of words which makes complete sense

a Assertive sentences

Exercise 6Rewrite the following sentences according to the instructions given below without changing their meanings

1 As soon as he saw the beer he jumped into

b Imperative sentences

c Interrogative sentences

d Exclamatory sentences

Sentences can be changed from one grammatical form to another without changing the meaning of the sentence This is known as transformation of sentences

the river ( Begin No sooner)2 None but brave deserve the fair (Begin the

bravehellip)3 This box is too heavy for me to lift ( Use so hellip

That instead of too)4 No one other than a king can live like James

Luxurious ( Begin only James)5 Oh for the wings of a dove (Begin I wishhellip)

Math Topic Commercial MathematicsChapter Shares and Dividends

Study item Discuss about shares and Dividends1) What is share

Ans To start any big business (company or Industry) a large sum of money is needed But it is not possible for an individual to invest such a large amount Then some persons interested in the business join together and from a company They divide the estimated money required into small parts Each such part is called a share

2) What do you mean by the term shareholder

Ans A person who purchases one or more shares is called shareholder3) Some terms related with a share

(i) Nominal value or face value or printed value The original value of a share is called its nominal value or face value or printed value

Note The nominal value of a share always remains same(ii) Market value or cash value The price of a share at any

time is called its market value or cash value

Note The market value of a share changes from time to time(iii) At par If the market value of a share is the same as its

nominal value the share is called at par(iv) At Premium or above Par If the market value of a share

is more than its nominal value the share is called at premium or above par

Example If a share of Rs 100 is selling at Rs 150 then it is said to be selling at a premium of Rs 50 or Rs 50 above par

(v) At Discount or below par If the market value of a share is less than its nominal value the share is called at discount or below par

Example If a share of Rs 100 is selling at Rs80 then it is said to be selling at a discount of Rs 20 or at Rs 20 below par

4) What is Dividend

Ans The profit which a shareholder gets for hisher investment from the company is called dividendNote (i) The dividend is always expressed as the percentage of the face value of the share(ii) The dividend is always given( by the company ) on the face value of the share

irrespective of the market value of the shareBENGALI(2ND LANGUAGE)

ldquoদেবতোর জণমrdquoলিবরোম চকরবত

পরথম লিসর পোঠ-চোর পসথ একটি পোথর লিবপলি ঘটোয় দেক যোতোয়োসতর পসথ পরলিতলিয়ত ওই পোথসর দেো োচট দেসত একলি দেতো দেক দেো োচট দেসয় দেবোমো সয় রোসতোর মোস লি0টসক পস1 লিবপরীত লিক দেথসক আো একটি দেমোটর োলি1 চোসকর কষতোয় পরোসরণ দেবোসচ যো লিকনত পরলিতবোর এমরণ দেৌভোয দেসকর োও সত পোসর তোই লিতলি দেকোো দেজোো1 কসর পোথরটিসক উপস1 দে8স পোথর উপস1 দে8োর ময় এক দেকৌতী জতো দেকসক পরশন কসর দেয লিতলি দেকোসো দেবতোর আস দেপসয়স0রণ লিকো লিকনত দেক বস লিতলি দেকোসো দেবতোর আস পোলি দেক উপলিত ক জতোর উসltসয বস কোরও ইস= স পোথরটি লিসয় দেযসত পোসর এর পর দেথসক দেক দেযসত আসত পোথরটি দেক দেসত পো একলি দেক কষয করস দেকউ পোথরটিসক ধসয়মস0 পলিরসকোর করস0 দেক ওই দেকৌতী জতোসক পোথসরর কোস0 বস থোকসত দেস এ0ো1ো আরও কষয কসর দেক ঠোৎ ওই লি1 পোথরটির োসয় লিোর োো-দেকউ পজো কসরস0 দেকৌতী দেোকটির আঙকো য় যলিসক উ পোথরটিসক লিরসয় দে8স তোর পর ঠোৎ একলি পোথরটির দেোোজ দেই দেক লিসয় দেস0 বো দেকোথোয় দেস0

বদোথ-

দেো োচট ndashচসত লিসয় দেকো লিক0র সE ধোককো দেস পস1 যোবোর উপকরমঅকসমোৎ- ঠোৎআতমমবর- লিসজসক লিয়নতরপ-পো লিপ0স প1োদেসতসসত- লিসপলিউৎোত- দেো1ো দেথসক উপস1 দে8োপরতযয় ndash লিবশবোপরতযোস- টতযোস ndash দৈবোসধসতোধলিসত- পরসপসরর পরলিত ব পরসয়ো করোপরসতরীভত- পোথসর পলিররণলিতবোনতঃকরসরণ- মস পরোসঅলিQৎ- অQো করসত ই=কদেোপ- বধমোর- দেবোস এমইতযোকোর- এইরকমরম- বময়পযসোভী-পসযর জয দেোভ আস0 যোরপোসথয় ঞচয়- পথচোর রচ জমোসোমোমোসরোস- ব ধম ধোসমর সE

তোর লি দেইhelliphellipTo be continued

Hindi 2nd lang सर क पद(सरदास)

सरदास शरी कषण भकति कावय क सरवशरषठ कगिरव ह इनक जनम और मतय क समय तथा सथान का मतभद हसरदास रवातसलय और शरार रस क अनयतम कगिरव ह इनक कावय म बालकषण क सौदय चपपल चषटा और गि7याओ की मनोहर झाकी मिमलती ह कषण और ोगिपयो क अननय परम का कतिचतरण ह सयो शरार की अपकषा उनक कावय म गिरवयो शरार का अमिBक गिरवषय और मारमिमEक कतिचतरण हआ हइन पकतियो म हम सरदास की भकति भारवनाओ का परिरचय मिमलता ह इनका सपण सगरह सरसार म गिनगिहत ह

1 जसोदा हरिर पालन झलारवहलरारवदलराईमलहारव रव जो ईसाई कछ ारव मर लाल को आई निनEदिदया काह ना आगिनसबारव त काह नाही बरवगिह आरवतोको कानहा बलारव

शबदाथ-हलरारव-गिहलती हदलराई - दलार पयार करती हमलहारव-पचकारती हनिनEदरिरया ndashनीदरवगिह-जलदी सअBर-होठमौन-चपसन-सकत

वयाखया- सरदास जी कहत ह गिक यशोदा माता बालक कषण को पालन म झल आती ह रवह उनह गिहलाती ह पयार करती ह मलहार जस कोई ीत ान लती ह और नीद स पछती ह गिक ह नीद तम मर लाल को आकर कयो नही सलाती तझ खाना बला रहा ह कभी कषण आख बद कर लत ह कभी आखफडफडान लत ह उनह सोता हआ जानकर यशोदा माता चप हो जाती ह और इशार म बात करन लती ह इसी बीच अकला कर कषण ज जात हतो गिफर यशोदा माता गिफर स ाना ान लती ह सरदास जी कहत ह गिक भरवान क दशन का सख दरवता और ऋगिष-मगिनयो को भी दलभ ह यही सख माता यशोदा को बडी सहजता स मिमल जा रही ह माता यशोदा बहत ही भागयशाली ह2)Continue to nexthellip

Physics

Chapter 1 Force

(Summary)

Question A body is acted upon by two forces each of magnitude F but in opposite directions State the effect of the forces if

(a) Both forces act at the same point of the body

(b)the two forces act at two different points of the body at a separation r

Solutions

(a) Resultant force acting on the body = 0

F ndash F = 0(b) The forces tend to rotate the body between two forces about the midpoint

Moment of forces = F times rFr

QuestionDefine moment of a couple Write its SI unit

Solutions

Moment of couple is equal to the product of both force and the perpendicular distance between the two forces

The SI unit of moment of couple is NmCommercial Studies

Advertising and sales

Business firms use several methods to

Questions1) What do you mean by advertising

promotion create demand of their product in the market and increase it sales Such methods comprises of advertising sales promotion personal selling and publicityToday we are going to discuss about one of such methods It is advertising

Meaning of advertisingAdvertising is a paid form of non-personal presentation for promotion of Ideas goods and services

Importance or merits of advertising Advertising has importance to manufacturer or traders to customer and to society as a whole

Today we will see how advertisement help the manufacturer or traders

Answer) Advertising is a means of how a company encourages people to buy their products services or ideas It is one element of marketing which also includes design Research and data mining

2) Mention any three features of advertisingAnswer)The main features of advertising are

i) It is impersonal form of presentation for promotion of products and services of Ideas

ii) It is issued by identified sponsor The advertisement contains the name of the advertiser

iii) It is a form of mass communication because the message is directed to a large number of persons simultaneously

3) Mention the main merits or importance of advertisement to manufacturer or tradersAnswer)

i) Introducing new product A business organization can introduce itself and its products to the public through advertising

ii) Increase the sale Advertising leads to increase the sale of existing product by entering into new markets and attracting new customers

iii) Create steady demand Advertising creates sustains regular demand by smoothening out seasonal and other fluctuations It enables regular production for the organisation

iv) Economics of scale Advertising facilitate mass distribution of goods and steady demand which lead to large scale and regular production

v) Goodwill Advertising helps in creating a good image of the firm and reputation for its products

Biology Chapter - 03Genetics

Today wewill start chapter and discuss about Genetics Gregor Mendel is known as father of genetics Before entering into Mendelrsquos experiment on Genetics we must know

Q1 Define the following termsi) Genetics Genetics is the study of

transmissionof body features from parents to offspringand the laws relating to such transmission

ii) Heredity It may be defined as transmissionof genetically based characteristics from parentsto offspring

iii) Character and traits Any heritable

Importance to TraderIntroducing new productIncrease the saleCreate steady demandEconomics of scaleGoodwill

some terms featureis a character The alternative forms of acharacter are called traitsex Character (Hair shape) - Traits (Curly straight)

iv) Homologous chromosomes A pair ofcorresponding chromosomes of the same shapeand size one from each parent

v) Genes Genes are the specific parts (DNA segments) of a chromosome which determinethe hereditary characteristicsNearly 30000genes present in human

vi) Alleles Alternative forms of a gene occupying the same position (locus) on homologouschromosomes and affecting the same characteristicbut in different ways

vii) Genotype ndash PhenotypeGenotype means of genes present in the cells of an organism Phenotype means the observable characteristic which is genetically controlled

viii) Mutation It is a sudden change in one or more genes or in the number or in the structure of chromosomes ex Sickle cell anaemia is a blood disease caused by a gene mutation

CLASS NOTES

Class XSubject Eng Literature (The Merchant of Venice ndash William Shakespeare)Topic Act IV Scene 1 Lines 01 to 34 ( Duke helliphelliphellip We all expect a gentle answer Jew) ate 27th April 2020 (2nd Period)

[Students should read the original play and also the paraphrase given in the school prescribed textbook]Summary Questions amp Answers

This scene may be termed as the catastrophe of the play It is the final unravelling of the complicated events which seem to threaten the happiness of Bassanio Portia and Antonio Right is justified to the fullest degree and malice falls into the trap prepared for others No one suffers here but Shylock but even then he receives a generous measure of mercy

o This is the Court-scene Initially we meet

(1)

DUKE I am sorry for thee thou art come to answer (Line 3-6)A stony adversary an inhuman wretchUncapable of pity void and emptyFrom any dram of mercy

(i) Who is addressed here Where is the person Why is the person there

Antonio is addressed hereAntonio is in the court of justice at VeniceAntoniorsquos trial is scheduled to be held here for his failure to meet the conditions of the bond he signed with Shylock

the Duke Antonio Shylock and Salerio Later we meet Bassanio Portia Gratiano and Nerissa

o The Duke says to Antonio that he has to face a very cruel opponent which Antonio admits and expresses his gratefulness to the Duke for his efforts to soften without result the heart of Shylock in order to be merciful to Antonio Antonio further says that he is ready to accept whatever cruel judgement the Court may award

o When Shylock appears in the court the Duke says that Shylock should change his decision of prosecuting Antonio and demanding the penalty specified in the bond out of consideration of the great misfortunes that Antonio has suffered If this is done by him (Shylock) the whole court would be gladdened by his merciful action

(ii) What is the Duke sorry for

The Duke is unable to change the mind of Shylock from his decision to get the bond forfeited even after he pleaded to Shylock Shylock stands firmly for his bond which when forfeited will allow him to take a pound of flesh from any part of Antoniorsquos body(iii) How does the Duke address Shylock`The Duke calls Shylock an adversary with a heart of stones He calls Shylock as an inhuman wretch without pity Shylock is quite lacking in the slightest quality of mercy (iv) How does Antonio reply to this

Antonio replies that he will meet the revenge of Shylock patiently He has prepared himself to suffer with a quiet spirit the utmost that Shylockrsquos tyranny and rage can do(v) What quality of the Duke is revealed here

The Duke is kind and benevolent He is ready to help Antonio He requests Shylock to free Antonio from the trial(vi) What are the terms of the bond that Antonio has signed

The terms of the bond that Antonio has signed were that if Antonio is unable to repay Shylock a certain sum of money specified on the paper on a certain date and in an agreed place the forfeit has to be paid The forfeiture will be an exact pound of Antoniorsquos flesh which Shylock will be a liberty to take from any part of Antoniorsquos body which pleases him

Class XI

STUDY MATERIAL

Class XISubject Eng Literature (The Tempest ndash William Shakespeare) Topic Act I Scene 2 Lines 88 to 132 (Prospero hellip Me and thy crying self) Date 27th April 2020 (3rd Period)

[Students should read the original play and also the paraphrase given in the school prescribed textbook]Summary Questions amp Answers

o Prospero now tells Miranda that he was the Duke of Milan He had been devoting himself more to studies than the affairs of the State His brother Antonio took advantage of this situation and with the help of Alonso the king of Naples seized upon him and her one midnight and shipped them in a frail bark so that they perished in the sea All this took place

(1)

MIRANDA I should sin (Line 118-132)

To think but nobly of my grandmother

Good wombs have borne bad sonsPROSPERO Now the condition

The King of Naples being an enemyTo me inveterate hearkens my brothers suitWhich was that he in lieu othrsquo premisesOf homage and I know not how much tribute

twelve years back

IMPORTANT PASSAGES EXPLAINED(Line 98-103)

PROSPERO helliphelliphelliphelliphelliphelliphellip

He being thus lorded

Not only with what my revenue yieldedBut what my power might else exact like oneWho having into truth by telling of it Made such a sinner of his memoryTo credit his own lie he did believeHe was indeed the duke

Prospero in telling the narrative of his past life here refers to his brother Antonio Prospero being with a studious bent of mind has left the administration of Milan on his younger brother Now Antonio being thus invested like a lord with all the powers derived from Prosperorsquos wealth and what the exercise of Prosperorsquos authority might secure for him regarded himself as a de facto Duke of Milan It is a well-known fact of psychology that a man who repeatedly tells a lie makes of his memory such a sinner against truth as to credit his own lie by the telling of it So Antonio by repeatedly saying to himself and others that he was the Duke came to believe that he was really the Duke Thus falsehood repeatedly asserted gained the force of truth for Antonio and he truly believed it

Should presently extirpate me and mine Out of the dukedom and confer fair MilanWith all the honours on my brother whereonA treacherous army levied one midnightFated to thrsquo purpose did Antonio openThe gates of Milan and ithrsquo dead of darkness The ministers for thrsquo purpose hurried thenceMe and thy crying self

(i) In the earlier lines of this scene what does Prospero tell about his intense interest What was the demand of his interest

In the earlier lines of this scene Prospero tells Miranda that he had an intense interest in the study of philosophy and magic arts Hence in order to improve his mind with this kind of study he kept himself isolated from worldly and state affairs His study was dearer to him than the applause and esteem that he could win from the public His study demanded too much solitude(ii) What forced Antonio to take an undue advantage over Prospero

Prosperorsquos indifferent attitude towards the statersquos affairs and his having boundless trust in Antonio gave rise to a boundless lust for power in Antoniorsquos mind Antonio felt that he must be the actual Duke instead of the part of the Duke he played Thus Antonio took an undue advantage of the situation to usurp Prosperorsquos dukedom(iii) Explain the following lines ldquoI should sin to think but nobly of my grandmother Good wombs have borne bad sonsrdquo

After hearing the treacherous act of her uncle Antonio Miranda says that Prosperorsquos mother was a noble lady and she cannot dishonour her memory by saying that the person named Antonio cannot be his (Prosperorsquos) brother She says that in honour of her grandmother she also cannot say that Antonio must have been begotten not by her grandfather but by some other man She finally concedes that it is known that good mothers have borne bad sons in their wombs and gave birth to them

(iv) Why did the King of Naples accept Antoniorsquos request to help him in usurping his dukedom What did Antonio propose to Alonso

Alonso who was the king of Naples accepted Antoniorsquos request in usurping Prosperorsquos dukedom because he (Alonso) was a sworn enemy of Prospero Antonio proposed that Alonso should immediately drive him (Prospero) and his offspring out of Milan and should confer the dukedom upon him (Antonio) with all the dignities which go with that

position In return Antonio promised that he shall give an annual tribute and also swore his allegiance to Alonso Also he agreed to hold the Dukedom of Milan as a subordinate to the state of Naples(v) How were Prospero and Miranda carried away from the city of Milan and what was the state of small Miranda at that time

In pursuance of the agreement settled between Antonio and Alonso an army of treacherous men was assembled One midnight when the occasion suited the will of destiny Antonio opened the gates of the city of Milan and in the death like silence of midnight Antoniorsquos agents who had been directed to execute his purpose carried Prospero and small Miranda away from the city in all haste They were then forced into a ship and carried some distance out to sea where they put them on a mere hulk of a boat without any rigging or ship-gear and abandoned them leaving them at the mercy of the roaring sea Miranda was a very small child of three years age and she was crying at that time

CLASS -XIDATE-270420Subject Topic Summary Execution

EVS Chapter 1 ndash Mode of Existence

Impact of mode of existence on resources

Q) Why resources are under pressure

Ans - Increase in the sophistication

of technology enabling natural resources to be extracted quickly and efficiently Eg in the past it could take long hours just to cut down one tree only using saws Due to increased technology rates of deforestation have greatly increased

The number of humans is increasing Cultures of consumerism Materialistic views

lead to the mining of gold and diamonds to produce jewelry unnecessary commodities for human life or advancement Consumerism also leads to extraction of resources for the

production of commodities necessary for human life but in amounts excessive of what is needed because people consume more than is necessary or waste what they have

Lack of awareness among the population is striking People are not aware of ways to reduce depletion and exploitation of materials

Accounts Cash Book Today we are going to start a new topic -Cash Book

The key terms used in this chapter are

bullCash book

bullSimple cash book

bullDouble column cash book bullTriple column cash book

bullPetty cash book

bullCash discount

bullContra entry

Here I will share you the meaning of each key terms

bullCash book Cash Book is a special purpose subsidiary book or journal in which cash received and cash payments are recorded

bullSimple cash book

It is a cash book in which only cash transactions are recorded It has only one column on each side

bullTriple column cash book

It is cash book which has three columns one column for each cash and Bankdiscount on each side of the cash book In this book both cash and Bank transactions are recorded together with discount allowed and received

bullPetty cash book

It is a cash book maintained for recording petty expenses

bullCash discount

Cash discount is the amount of discount received or allowed on cash payments and cash receipts Discount received is an income for the business while discount allowed isan expense

bullContra entry

It means transactions involving both cash and Bank Such transactions though recorded in the cash book are not posted into ledger The letter lsquoC is written in Ledger folio for contra entry

Business Studies

ENTREPRENEURSHIP

Now we shall discuss the second chapter

lsquoENTREPRENEURSHIPrsquo

Today before starting the chapter let us recall what

Questions

1What are the main characteristics of Intrapreneurship

Answer

The main characteristics of Intrapreneurship are

Corporate framework-it occurs within the framework of the same company

Semi-Autonomous-Intrapreneurship

we have read last day

Let s today start the class by recalling the last topic taught

Intrapreneurship is the process of discovering and exploring business opportunities within an existing company It involves launching new business ventures within the framework of a present corporation Intrapreneurship is also known as corporate entrepreneurship or corporate venturing

Now let us start with the characteristics of Intrapreneurship

The main characteristics of Intrapreneurship are

Corporate framework

Semi-Autonomous Lack of ownership Senior position Low risk taking Not own boss

Now let us discuss the meaning of enterprise

Enterprise means an undertaking or adventure that requires some innovation and investment and thus involves riskEnterprise always entails decision making coordination and risk bearing

involves crating amd nurturing a semi-autonomous business unit which may be a subsidiary a strategic business unit or a division

Lack of ownership-the intrapreneur is not the owner of the unitb he creates and nurtures

Senior position-he occupies a senior managerial position in the company

Low risk taking-An intrapreneur does not bear the full risk of failure

Not own boss-An intrapreneur is not his own bosss in legal termsHe enjoys the freedom and gets the required resources and support

2 How is Entrepreneur is different from Intrapreneur

The functions involved in both the entrepreneurship and intrapreneurship are by and large similar however there are several differences between the two

Point of distinction

Entrepreneur

Intrapreneur

status An independent business person

A senior executive within a company

Ownership Owner of

the business

An employeesometimes a share in ownership

Financing Responsible for raising finance for the business

Not responsible for raising the finance

Risk bearing

Bears the risk of the business

Does not bears the risk of the business

Reward Profit which is uncertain and irregularcan be loss

Fixed salary and fringe benefits

Need for security low high

3 What do you understand by enterprise

Answer Enterprise means an undertaking or adventure that requires some innovation and investment and thus involves riskEnterprise always entails decision making coordination and risk bearing

COMMERCE NATURE AND OBJECTIVES OF

BUSINESS

Today let us recall the last other two objectives of business by the chart given in the previous class

Firstly we would discuss Human Objectives

Business is run by people and for people Labour is a valuable business element

Human objectives of business are concerned with the well -being of labour

The human objectives are as follows

Labour welfare Developing human

resources Participative

management Labour

management cooperation

Questions

1 Explain the human objectives of a business enterprise

Answer

Business is run by people and for people Labour is a valuable business element

Human objectives of business are concerned with the well -being of labour

The human objectives are as follows

Labour welfare-Business must recognize the dignity of labour and human factors should be given the recognition

Developing human resources-Employees must be provided the opportunities for developing new skills and attitudes

Participative management-Employees should be allowed to take part in decision making process of business

Labour management cooperation-Business should strive for creating and maintaining cordial employer employee relations so as to ensure peace and progress in industry

Now let us discuss the national objectives of business

Optimum utilization of resources

National self- reliance Development of small

scale industries Development of

backward areas Control over pollution

2Explain the national objectives of a business enterprise

Answer

It is the duty of business to utilize the resources of the country properly the national objectives of business

Optimum utilization of resources ndashBusiness should use the nationrsquos resources in the best possible manner

National self- reliance-It is the duty of the business to help the government in increasing experts and in reducing dependence on imports

Development of small scale industries-Big business firms are expected to encourage growth of small scale industries which are necessary for generating employment

Development of backward areas-Business is expected to give preference to the industrialization of backward regions of the country

ECONOMICS

BASIC ECONOMIC CONCEPTS

SUB

TOPIC

Value

Wealth

Welfare

Today we shall start with a new topic of the same chapter ie lsquoValuersquo

Value of a commodity is defined as the valuation placed by a household on the consumption of this commodity

lsquoValuersquo has two different meanings and these are

a Value -in -use It refers to consumption value of a commodity It expresses the utility derived from the consumption of a particular commodity A necessity like water has a very high value ndashin ndashuse or

Question

1What is value

Answer

Value of a commodity is defined as the valuation placed by a household on the consumption of this commodity

2What is value-in use

Answer It refers to consumption value of a commodity It expresses the utility derived from the consumption of a particular commodity A necessity like water has a very high value ndashin ndashuse or consumption value

3What is value ndashin- exchange

Answer It relates to market value of a commodity

It is the rate at which a particular good or service can be exchanged for money

For example in barter system if a person is prepared to exchange 3 metres of cloth with 1 pair

consumption value

b Value ndashin-exchange It relates to market value of a commodity

It is the rate at which a particular good or service can be exchanged for moneyFor example in barter system if a person is prepared to exchange 3 metres of cloth with 1 pair of shoes then the value in exchange of 3 metres of cloth is 1 pair of shoesValue in exchange is the power of purchasing other goods In modern monetised economies the exchange value of goods are expressed in terms of money as prices

Now let us discuss the term lsquoWealthrsquo

Wealth refers to the stock of all those assets which are a source of income

Wealth is a stock concept

Wealth must possess the following features

a Utility It must possess utility or give some

of shoes then the value in exchange of 3 metres of cloth is 1 pair of shoes

Value in exchange is the power of purchasing other goods In modern monetised economies the exchange value of goods are expressed in terms of money as prices

4 What is wealth

Answer it refers to the stock of assets or goods which are a source of income and have personal or national ownership

5 What are the features of wealth

Answer The features of wealth are as follows

Wealth must possess the following features

a Utility It must possess utility or give some satisfaction

b Scarcity It must be limited in quantityc Transferability it should be transferable its

ownership can be transferred from one person to another person

d Exchange value It must possess exchange value

6 What is welfare

Answer

Welfare is defined as satisfaction and happiness a sense of well- being among the people

satisfactionb Scarcity It must be

limited in quantityc Transferability It

should be transferable its ownership can be transferred from one person to another person

d Exchange value It must possess exchange value

Now let us discuss the term lsquoWelfarersquo

Welfare is defined as satisfaction and happiness a sense of well- being among the people

Welfare is affected by factors like

a Consumption of goods and services

b Environment

c Family relations

d Degree of freedom

e Law and order situation

Mathematics Trigonometric equation

To find the general solution of the equation sinθ=0

When sin θ =0

Then θ= 0 π2π 3π-π -2π -3

i e when θ = 0 or an integral multiple of π

i e when θ= nπ where n is any integer

Therefore the general solution of the equation sin

Example1 Find the general values of θ which satisfy the equation sin2 θ =34

Solution sin2 θ= 34

Or sin θ = +34 or -34

Or sin θ = sin π3 or sin (-π3)

Therefore

θ = [nπ + (-1) n (π3)] or[ nπ+ (-1) n (-π3)]

= nπ +π3 or nπ-π3 where n= any integer

Example 2Find the values of θ which satisfy tan2 θ

θ=0 is θ= nπ where n is any integer

To find the general solution of the equation cos θ=0

When cos θ=0

Then θ=π2 3π2 5π2 -π2 -3π2 -5π3

i e when θ is an odd multiple of π2

i e when θ=(2n+1) π2 where n= any integer

Therefore the general solution of the equation cos θ =0 is θ= (2n+1) π2 where n= any integer

To find the general solution of the equation tan θ = 0

Clearly tan θ =0 implies sin θcos θ =0

Therefore θ = nπ

i e the general solution of the equation tan θ=0 is θ =nπ where n = any integer

To find the general solution of the equation cot θ =0

Clearly cot θ =0 implies (cos θsin θ) = 0

i e cos θ =0

Therefore θ = (2n+1) π2

Therefore the general solution of the equation cot θ =0 is θ = (2n+1) π2

Where n= any integer

To find the general solution of the equation sin θ= k (-

=13 -πleθleπ

Solution tan2 θ =13

Or tan θ = plusmn1radic(3) =tan(plusmnπ6)

θ=nπ plusmn π6 where n =any integer

If n=0 then θ=plusmnπ6

If n=1 then θ= π plusmn π6

If n=-1 then θ= -π plusmn π6

Therefore the required solution in -π le θ le π are θ= π6 5π6 -π6 -5π6

Exercise Find general solution of sin 2θ=cos θ [Hints Use sin 2θ= 2sin θcosθ and then take cosθ

common]

1lek le1)

Determine an angle alpha such that sin =k and -π2le αle π2

Then we have

Sin θ = k = sin α

Or sin θ - sin α =0

Or 2 cos [(θ+α) 2] sin [(θ-α) 2] =0

Therefore either cos [(θ +α) 2] =0 (1)

Or sin [(θ-α) 2] =0 (2)

Now from (1) we get (θ+α) 2= (2m+1) π2)

Or θ = (2m+1) π-α (3)

And from (2) we get (θ-α) 2 =mπ

Or θ= 2mπ+α(4)

Where m = any integer

Clearly the solution (3) amp (4) may be combined in the following form

θ= nπ+(-1) n α where n= any integer

Therefore the general solution of sin θ = sin α is θ = nπ +(-1) n α where n is any integer and -π2 le α le π2

Biology Chapter - 04Kingdom Monera

Today we will discuss about bacterial reproduction and its usefulness

Fig Binary Fission

Fig Conjugation Fig Transformation

Bacterial reproduction is mainly asexual but sexual reproduction

also takes place

Asexual reproduction takes place by i) Binary fission - from one bacteriato

two bacteria are produced in every 20 to 30mins

ii) Buddingiii) Endospore formation - during

unfourable condition

Sexual reproduction by three ways

1) Conjugation - Transfer of genetic material between cells that are in physical contact with one another

2) Transduction - Transfer of genetic materialfrom one cell to another by a bacteriophage

3) Transformation - Transfer of cell-freeor naked DNArsquo from one cell to another

Bacteria causes different diseases inplants animals and human and

it causes food spoilage and waterpollution but it also have some useful

activities

i) Bacteria are helpful in sewage water treatment

ii) It is used in antibiotic (medicine) production

iii) Anaerobic bacteria help in biogas(energy) production

iv) Many household products like yoghurt cheese are manufactured by use of bacteria

v) Rhizobium by symbiotic relationship with leguminous plant increase soil fertility

vi) Besides these bacteria is helpful in genetic engineering degradation of petroleum hydrocarbonand in dairy

industry

Physics Motion in plane Here we will introduce Projectile Motion

Execution

Projectile

Y

usinθ u h

θX

ucosθ

Suppose a body is projected with an angle θ So initial velocity u can be resolved into two components

Horizontal component - ucosθ ( for range)

Vertical component - usinθ ( for height)

usinθ changes during motion and becomes zero at maximum height position but ucosθ remain unchanged

The maximum height of projectile is h

NB If initial is upward then g = -ve and if it is downward then g = +ve Height is +ve if direction of motion does not change ( for ex a body thrown upwards but goes down ultimately then height h = -ve)

The angle of projectile θ is the angle made with horizontal

HISTORY ndash GROWTH OF NATIONALISM

SUB TOPIC- REVOLUTIONARY NATIONALISM Bengal formation of Anushilan Samity and Jugantar Group

The intensification of the Swadeshi movement and Government policy of terror and repression led to outbreak of violence Bombs were manufactured and attempts on the lives of unpopular Government officials became frequent In the gymnasium of Scottish Church College which was known as General Assemblies Institution a secret society was formed known as Anushilan Samity

Aurobindo Ghosh send from Baroda his emissary Jatindranath Banerjee to mobilize the Bengal revolutionaries

Hemchandra Qanungo and Satyen Bose published Journal Jugantar

The Jugantar group planned to assassinate oppressive magistrate Kingsford by Khudiram Bose and Prafulla Chaki in 1908 Prafulla Chaki committed suicide to avoid arrest Khudiram was tried and hanged

Afew days later the police found a bomb factory in Maniktala and arrested a large number of revolutionaries The trial of revolutionaries became famous as the Alipore Bomb Case

In the course of the trial the approver the public prosecuter and a police officer were assassinated

1 Question Name two journals which preached the cult of violence

Answer a) Yugantar edited by Bhupendranath DuttaB) Bandemataram edited by Aurobindo Ghosh2 Question Why was Khudiram arrested and hangedAnswer An attempt was made to assassinate a hated vindictive majistrate named Kingsford by Khudiram Bose and Prafulla Chaki Their attempt failed and the bomb they threw killed two English ladies Khudiram was arrested and put to trial and then hanged3Question Who was Aurubindo GhoshAnswer Aurobindo Ghosh a nationalist revolutionary who was charged for his involvement in the Alipore Bombing Case He was accused of it along with his brother Barindra nath Ghosh But Aurobindo was acquitted because of the brilliant pleading of his counsel Chittaranjan Das Then he became a spiritual reformer introducing his visions on human progress and spiritual evolution4 Qustion Who was KingsfordAnswer Kingsford was an unpopular British chief Magistrate who was the target of the bomb thrown at Muzaffarpur by Khudiram and Prafulla Chaki

Most of the accused were convicted and sentenced to

long term of imprisonmentBut

Aurobindo Ghosh was acquitted mainly owing to the brilliant pleading of his counsel Chittaranjan Das

Political science Topic-Sovereignty

Summary Sovereignty is the full right and power of a governing body over itself without any interference from outside sources or bodies In political theory sovereignty is a substantive term designating supreme legitimate authority over some polity In international law sovereignty is the exercise of power by a state

Internal Sovereignty

Internal sovereignty means supreme authority within ones territory while external sovereignty relates to the recognition on the part of all states that each possesses this power in equal measure

External sovereignty

external sovereignty relates to the recognition on the part of all states that each possesses this power in equal measure

Distinguish between

Execution

Answer the following questions

Short notes-

Sovereignty

Internal Sovereignty

External sovereignty

Homework- learn

external sovereignty and internal sovereigntySovereignty is the principle

of supreme and

unquestionable authority

reflected in the claim by the

state to be the sole author of

laws within its territory

Definition of external vs internal sovereigntyInternal sovereignty refers to

the relationship between a

sovereign power and its

subjects ndash it refers to the

location of the supreme

authority within the state In

the UK for example internal

sovereignty (supposedly)

resides within Parliament

reflected in the

constitutional principle of

parliamentary

sovereigntyBy contrast

external sovereignty refers

to the capacity of the state

to act independently and

autonomously on the world

stage This is what is

sometimes called lsquostate

sovereigntyrsquo or lsquonational

sovereigntyrsquo and implies

that states are legally equal

and that the territorial

integrity and political

independence of a state is

inviolable

Class ndash XII

Date - 2742020 STUDY MATERIALSubject Topic Summary Execution Business Studies

Job Analysis amp Manpower Planning

At first let us recall the chapter what we have discussed till nowbullJob analysisbullJob specification bullJob description bullJob enlargement bullJob enrichment

Today we will do some questions answers from the chapter

Questions 1ldquoJob analysis job description and job specification are interrelatedrdquo Comment Answer) Job analysis is a systematic and detailed examination of a job to collect all the relevant information about it The contents off the job are summarised in the job description The qualification needed for the job are summarised in job specificationThus there is close interrelationship between job analysis job description and job specification

Question 2ldquoJob enlargement is a horizontal extension of a job whereas job enrichment is a vertical extension of a jobrdquo ElucidateAnswer) Job enlargement involves adding one or more task to a job coma where as job enrichment involves adding more autonomy and responsibility to a job Job enlargement is therefore horizontal extension of a job coma whereas job enrichment is a vertical extension of a job

Question 3 )

What is manpower estimation Explain its quantitative and qualitative aspectsAnswer) Manpower estimation is the process by which management determines how an organisation should move from itrsquos current manpower positionto its desired manpower position There are two dimensions of Manpower estimation- quantitative and qualitative

Quantitative aspectThis aspect of Manpower estimation involves estimating the number of employees required in a future time period Workload analysis and workforce analysis are done to estimate the quantity of required manpower

Qualitative aspectThe estimate of the knowledge skills experience etc of required manpower is the qualitative aspect of Manpower estimation The quality of Manpower can be judged on the basis of job analysisand job specification

COMMERCE

CAPITAL-FIXED AND WORKING

Today let us start the class by discussing the sources of finance for different types of business firms

The term lsquocapitalrsquo refers to the investment made in the enterprise for the purpose of earning profits

Requirements of capital and sources of capital for different types of business firms are

1 Capital for sole proprietorship businessA sole proprietor operates at a small scale and thereforerequires a limited amount of capital

2 Capital for partnership firmCapital requirements as well as capital base of a partnership is bigger than that of a sole trader businessThe owned capital is contributed by the partners in an agreed ratio

3 Capital for joint stock companyA joint stock company generally requires large amount of capitalA public company can raise huge capital through issue of shares In addition to share capital it can utilize retained profits

Now let u discuss the meaning of Finance PlanningFinance planning is the process of estimation the financial requirements of an organization specifying the sources of firms and ensuring that enough funds are available at the right time

1 What do you mean by Finance PlanningAnswer Finance planning is the process of estimation the financial requirements of an organization specifying the sources of firms and ensuring that enough funds are available at the right time

2Discuss the role of financial planning of an enterpriseThe role of financial planning are as followsa A sound financial plan helps a business enterprise to avaid the problems of shortage and surplus of fundsbFinancial planning serves as a guide in developing a sound capital structure so as to maximize returns to shareholders c It helps in effective utilization of fundsd It provides policies and procedures for coordinating different functional areas or departments of businesse It enables the management to exercise effective control over the financial activities of an enterprisef It helps the company to prepare for facing business shocks and surprises in future

Mathematics

Continuity and differentiability

Recall Definition of ContinuityLet f(x) be a single valued function of x and x=a be a point in the domain of definition of the function The function is said to be continuous at x=a ifi) f(c) is defined ie f(x) has a definite finite value at x=cii) lim xrarra f(x) exists andiii) lim xrarra f(x) =f(a) In other words f(x) is said to be continuous at x=a if lim xrarra+ f(x)= lim xrarra- f(x) = f(a) Or f(a+0) =f(a-0) =f(a) Or lim hrarr0 f(a+h)= f(a) Algebra of continuous functionsNow we will study some algebra of continuous functions Theorem 1 Suppose f and g be two real functions continuous at a real number c Then(1) f + g is continuous at x = c(2) f ndash g is continuous at x = c(3) f g is continuous at x = c(4) (fg) is continuous at x = c (provided g (c) ne 0)

Example 1 Prove that every rational function is continuousSolution Recall that every rational function f is given byf(x)=[p(x) q(x) ] q(x)ne0where p and q are polynomial functions The domain of f is all real numbers except points at which q is zero Since polynomial functions are continuous f is continuous by (4) of Theorem 1Example 2Discuss the continuity of sine functionSolution To see this we use the following factslim xrarr0 sin x =0Now observe that f (x) = sin x is defined for every real number Let c be a real number Put x = c + h If x rarr c we know that h rarr 0 Therefore lim xrarrc f(x)

= lim xrarrc sin x= lim hrarr0 sin(c+h) =lim hrarr0 [sin c cos h + cos c sin h ]=lim hrarr0 (sin c cos h) + lim hrarr0 (cos c sin h) = sin c +0=sin c = f(c) Thus lim xrarrc f(x) = f(c) and hence f is a constant function Exercise Prove that the function f(x) = x2 +2x is continuous for every real value of x [Hints show that lim xrarra+ f(x) = lim xrarra- f(x) = f(a) ]

Biology Reproduction in Flowering plants We will discuss about megasporoangium

megasporagenesis and female gametophyte

Q4 Describe the structure of megasporangium

Ovule is attached to the placenta by astalk called funicle

Each ovule has one two or three protectivecoverings called integuments

At the tip of integuments a small openingcalled micropyle is organised

Opposite to the micropylar end is the chalaza

Within the integuments a mass of cellsnucellusand inside it embryo sac orfemale gametophyte is present

Q5 Describe a mature embryosacamp its formation

In most of the flowering plants only oneof the 4 megaspores formed as a result ofmegasporogenesis that is functional while theother three degenerate

The

functional megaspore develops into thefemale gametophyte

Formation The nucleus

of the functional megasporedivides mitotically to form two nuclei first andthen two more sequential mitotic nucleardivisions result in the formation of four ampthen eight nucleate stages of embryo sac

Six of the eight nucleus are surrounded bycell walls and organised into cells

The remaining two nuclei called polar nuclei are found below the egg apparatus in the largecentral cell

Three cells consisting of two synergids amp one egg cell present bottom of

embryo sac Three cells

at the chalazal as antipodal cells

Two polar nuclei together present in large central cell

HISTORY

TOWARDS INDEPENDENCE AND PARTITION THE LAST PHASE(1935-1947)SUB TOPIC NATIONAL MOVEMENTS DURING THE SECOND WORLD WAR

Spread of Quit India Movement On 9th August 1942Gandhiji and other Congress leaders were arrested The Congress was declared illegal The news of the arrest of all leaders marked the beginning of a widespread movement of India It was not possible for such a movement to remain peacefulBut the arrest of the all notable congress leaders virtually left the movement in the hands of the mass The movement took the form of violent and militant outbreakBesides congressmen revolutionaries also were very active in the movement The Congress Socialist group also played a prominent role

1 Question Why did the British authority arrest the Congress leaders on 9 th August 1942Answer Congress Working committee adopted the Quit India resolution which was to be ratified at the Bombay AICC meeting in 8th August 1942 They decided to launch a mass struggle on non-violent lines Gandhiji gave a clarion call to all section of the people rdquoKarenge ya Marengerdquo (do or die) Congress leaders gave the call to driving out

the British from IndiaViceroy had taken strong action against the Quit India movement Gandhiji and all the leaders of Congress were arrested

2 Question How did Quit India Movement spread out all over IndiaAnswer The news of the leaders lsquo arrest marked the beginning of a widespread movement to remain peacefulThe movement took form of violent outbreak There were widespread cutting of telephone and Telegraph wires damaging railway lines raising barricades in cities and towns and other forms of violent demonstations

Question Name the leaders of Congress

Socialist group played a prominent part Notable among the Jayprakash Narayan Rammonohar Lohia Aruna Asaf Ali

Political science

Topic-Franchise and Representation

Summary

The election commission

The Election Commission of India is an autonomous constitutional authority responsible for administering Union and State election processes in India The body administers elections to the Lok Sabha Rajya Sabha State Legislative Assemblies in India and the offices of the President and Vice President in the country

Functions of election commission-

India is a sovereign socialist secular democratic republic Democracy runs like a golden thread in the social economic and political fabric woven by the Constitution given by lsquoWe the People of Indiarsquo unto ourselves The concept of democracy as visualised by the Constitution pre-supposes the representation of the people in Parliament and State legislatures by the method of election The Supreme Court has held that democracy is one of the inalienable basic features of the Constitution of India and forms part of its basic structure The Constitution of India adopted a Parliamentary form of government Parliament consists of the President of India and the two Houses mdash Rajya Sabha and Lok Sabha India being a Union of states has separate state legislatures for each state State legislatures consist of the Governor and two Houses mdash Legislative Council and Legislative Assembly mdash in seven states namely Andhra Pradesh Telangana Bihar Jammu amp Kashmir Karnataka Maharashtra and Uttar Pradesh and of the Governor and the state Legislative Assembly in the remaining 22 states Apart from the above two out of the seven Union Territories namely National Capital Territory of Delhi and Puducherry also have their Legislative Assemblies

ExecutionShort notes-Election commissionFunctions of election commission

Homework- Learn

Computer

Science

Computer hardware NAND Gate

A NOT-AND operation is known as NAND operation It has n input (n gt= 2) and one output

Logic diagram

Truth Table

NOR Gate

A NOT-OR operation is known as NOR operation It has n input (n gt= 2) and one output

Logic diagram

Truth Table

XOR Gate

XOR or Ex-OR gate is a special type of gate It can be used in the half

adder full adder and subtractor The exclusive-OR gate is abbreviated as EX-OR gate or sometime as X-OR gate It has n input (n gt= 2) and one output

Logic diagram

Truth Table

XNOR Gate

XNOR gate is a special type of gate It can be used in the half adder full adder and subtractor The exclusive-NOR gate is abbreviated as EX-NOR gate or sometime as X-NOR gate It has n input (n gt= 2) and one output

Logic diagram

Truth Table

Physics

Chapter 1 Electric Field ( Electric Dipole) (Summary)

Here we will derive Expression of electric field at broad side

On position of dipole

Execution

Q With the help of a labelled diagram obtain an expression for the electric field intensity E at any point on the equitorial line ( broad-side on position) of an electric dipole

Ans

E1 E1sinθ

E θ P E1 θ

( r2+L2)12 E2 E

r E2 E2sinθ

-q θ L O L +qA B

Let us consider that the point P is situated on the right bisector of the dipole AB at a distance r meter from its midpoint O

Let E1 and E2 be the electric field intensities of the electric field at P due to charge +q and ndashq of the dipole resp The distance of P from each charge is ( r2+L2)12

So E1 = 14 πϵ q

(r 2+L 2) away from +q

E2 = 14 πϵ q

(r 2+L 2) towards ndashq

The magnitudes of E1 and E2 are equal but directions are different Now resolving E1 and E2 into two components parallel and perpendicular to AB we get

The components perpendicular to AB E1sinθ and E2sinθ cancel each other because they are equal and opposite

The components parallel to AB are E1cosθ and E2 cosθ are in same direction and add up

So resultant intensity of electric field at the point P is

E = E1cosθ + E2 cosθ

E = 14 πϵ q

(r 2+L 2) 2 cosθ

Now from fig we have cosθ =BOBP = L (r2+L2)12

So we get E = 14 πϵ 2qL ( r2+L2)32

Now electric dipole moment p= 2qL

So E = 14 πϵ p ( r2+L2)32

HW Find the expression of Electric field as done here but this time take r gtgt 2L

Also find the expression of torque experience by a dipole

(Hint Electric force experienced by charges of dipole in electric field is qE each Let θ be the angle which dipole makes with electric lines of force then perpendicular distance between two charges is 2Lsinθ Then torque = force x perp distance = qE x 2L sinθ So τ=pE sinθ where p =2qL )

STUDY MATERIAL

Class XIISubject Eng Literature (The Tempest ndash William Shakespeare) Topic Act IV Scene 1 Lines 84 to 133 (Iris hellip A contract of true love Be not too late ) Date 27th April 2020 (4th Period)

[Students should read the original play and also the paraphrase given in the school prescribed textbook]Summary Questions amp Answers

o Ceres soon appears and comes to know that she has been summoned to celebrate the contract of true love

o Ceres expresses her unwillingness to meet Venus and Cupid as she has shunned their company

o Ceres and Juno both bestow their blessings upon Ferdinand and Miranda with June gifting honour riches happiness in marriage and Ceres presents plenty of earthrsquos produce

o Iris summons the water-nymphs and reapers to come and celebrate a contract

(1) IRIS Of her society (Line 91-101)

Be not afraid I met her deity

Cutting the clouds towards Pathos and her sonDove-drawn with her Here thought they to have doneSome wanton charm upon this man and maidWhose vows are that no bed-right shall be paidTill Hymens torch be lightedmdashbut in vainMarss hot minion is returned againHer waspish-headed son has broke his arrowsSwears he will shoot no more but play with sparrowsAnd be a boy right out

(i) Where were Venus and Cupid seen flying How were they travelling Why did they want to join the marriage celebration of Ferdinand and Miranda

of true love

Venus and Cupid were seen flying through the air towards Paphos the famous city which is situated on the island of Cyprus They were travelling by air-borne chariot drawn by doves They certainly wanted to come here in order to play some amorous trick upon Ferdinand and Miranda who are under a vow not to gratify their physical desires till the holy ceremony of their marriage has been performed(ii) What have Venus and Cupid done after failing in their plan

After being failure of their plan Venus who is a very passionate deity and who is the mistress of Mars (the god of war) has gone back while here ill-tempered son Cupid has broken his arrows of love in his state of desperation(iii) What has Cupid firmly decided

Cupid is feeling so disappointed that he has firmly decided to shoot no more arrows to arouse love in human hearts but to spend his time playing with sparrows Thus he would now become just a boy and would give up his original function of shooting arrows on human beings to make them fall in love(iv) What vow had Ceres taken How did Ceres feel at the abduction

After the abduction of her daughter Prosperina by Pluto Ceres had taken a vow to always keep away from the disgraceful company of Venus and her blind son Cupid the god of love Ceres felt deeply distressed when Pluto had carried off her daughter and had made her his wife by force(v) Why has Ceres not forgiven Venus and her blind son For what do Ceres want to be sure

As the abduction had been manipulated by Venus the goddess of beauty and love and her blind son Cupid Ceres has never forgiven them for their part in the whole plot Ceres wants to be sure that she would not have to meet Venus and Cupid who had engineered the abduction of her daughter Prosperina

AS THIS lsquoMASQUErsquo SCENE IS VERY IMPORTANT IN THE PLAY THE PARAPHRASE OF THE ENTIRE PORTION OF MASQUE SCENE (Act IV Lines 58 to 143) IS GIVEN BELOW

IRIS Goddess of RainbowCERES Goddess of Agriculture and all the fruits of the earth

(Nature growth prosperity rebirth ndash notions intimately connected to marriage)JUNO The majestic Queen of Heavens and wife of Jupiter (Jupiter is the king of Gods)

VENUS The Goddess of love CUPID Son of Venus PLUTO God of death (In the play referred by Shakespeare as lsquoDisrsquo which is a Roman name for Pluto)

ORIGINAL TEXT PARAPHRASEPROSPEROWellmdash

PROSPERONow come Ariel Let there be too many rather than too few

Now come my Ariel Bring a corollaryRather than want a spirit Appear and pertly[to Ferdinand and Miranda]No tongue all eyes Be silent

spirits in attendance Appear briskly

[to Ferdinand and Miranda]Look with your eyes but do not say a word

[Soft music] [Soft music][Enter Iris] [Enter Iris]

IRISCeres most bounteous lady thy rich leasOf wheat rye barley vetches oats and peasThy turfy mountains where live nibbling sheepAnd flat meads thatched with stover them to keepThy banks with pioned and twilled brimsWhich spongy April at thy hest betrimsTo make cold nymphs chaste crowns and thybroom-grovesWhose shadow the dismissegraved bachelor lovesBeing lass-lorn thy pole clipped vineyardAnd thy sea-marge sterile and rocky-hardWhere thou thyself dost airmdashthe Queen othrsquoSkyWhose watery arch and messenger am IBids thee leave these and with her sovereign grace[Juno appears] Here on this grass-plot in this very placeTo come and sport Her peacocks fly amainApproach rich Ceres her to entertain

IRISCeres most generous lady you are the cause of rich fields or fertile land where wheat rye barley beans oats and peas grow the grassy mountains where the sheep graze and the flat meadows covered with coarse hay to be used as fodder for cattleYour banks are covered with marsh-marigolds and reeds and the rainy April under your orders brings forth to make for the maids who are not in love beautiful crowns your woods where the broom flourishes and where the bachelor who has been dismissed by the maid he loved lies down being forsaken your vineyard in which the poles are embraced by the vines and the margin of the sea which is barren and rocky where you roam about to enjoy the fresh air ndash the queen of the sky (Juno) whose messenger I am besides being represented as the rainbow bids you leave all these and with her majesty here on this grassy plot in this very place come and sport her peacocks carry her fast in her chariot through the air and are making their way here approach rich Ceres to welcome her

[Enter Ariel as Ceres] [Enter Ariel as Ceres]

CERESHail many-coloured messenger that neerDost disobey the wife of JupiterWho with thy saffron wings upon my flowersDiffusest honey-drops refreshing showersAnd with each end of thy blue bow dost crownMy bosky acres and my unshrubbed downRich scarf to my proud earth Why hath thy queenSummoned me hither to this short-grassed green

CERESWelcome rainbow that never dared disobey Juno the wife of Jupiter who with your orange coloured rays spread honey-drops refreshing showers And with each end of thy blue bow drown my bushy acres and my hilly country which is free from shrubs you thus forming a rich scarf Why has your queen called me here to this place covered with short grass

IRISA contract of true love to celebrateAnd some donation freely to estateOn the blest lovers

IRISI have called you to celebrate a contract of true love and bestow some liberal gift upon the blessed lovers

ORIGINAL TEXT PARAPHRASECERESTell me heavenly bowIf Venus or her son as thou dost knowDo now attend the queen Since they did plotThe means that dusky Dis my daughter gotHer and her blind boys scandaled companyI have forsworn

CERESTell me heavenly bow if Venus the Goddess of love or Cupid her son and pedlar of passion at this time attend the heavenly queen Juno because you are sure to know Since the day they conspired against me and dark Pluto took away my daughter here and Cupidrsquos disgraceful company I have left off

IRISOf her societyBe not afraid I met her deityCutting the clouds towards Pathos and her sonDove-drawn with her Here thought they to have doneSome wanton charm upon this man and miad

IRISBe not afraid of her company I met her deity moving on the clouds towards Paphos the sacred home of Venus on the island of Cyprus along with her son on her chariot drawn by doves Here they contemplated to exercise a charm upon this man and maid producing

Whose vows are that no bed-right shall be paidTill Hymens torch be lightedmdashbut in vainMarss hot minion is returned againHer waspish-headed son has broke his arrowsSwears he will shoot no more but play with sparrowsAnd be a boy right out

wantonness before the actual marriage ceremony but did not succeed Venus has returned her irritable son has broken his arrows and swears that he will give up his practice of trying to inspire love but play with sparrows and be a boy again

[Music is heard] [Music is heard]

CERESHighst queen of stateGreat Juno comes I know her by her gait

CERESHighest queen of state Great Juno there she comes I know here by her gait

[Enter Juno] [Enter Juno]

JUNOHow does my bounteous sister Go with meTo bless this twain that they may Prosperous beAnd honoured in their issue

JUNOHow are you doing my generous sister Come with me to bless this couple so that they may be prosperous and fortunate in their children

[They sing] [They sing]

JUNOHonour riches marriage-blessingLong continuance and increasingHourly joys be still upon youJuno sings her blessings upon you

JUNOMay honour riches happiness in marriage long continuance and increase of those boons ever rest upon you as hourly joys Juno showers down upon you her blessings in song

CERESEarths increase foison plentyBarns and garners never emptyVines and clustring bunches growingPlants and goodly burden bowingSpring come to you at the farthestIn the very end of harvestScarcity and want shall shun youCeresrsquo blessing so is on you

CERESMay you have the plenty of earthrsquos produce Your barns and granaries may never be empty Your vines may grow with clustering bunches Your fruit trees may be heavily laden with their fruit May there be continuous spring and harvest May scantiness and want leave you forever Such is the blessing of Ceres upon you

FERDINANDThis is a most majestic vision andHarmoniously charmingly May I be boldTo think these spirits

FERDINANDThis is a great vision and magically melodious Should I suppose the characters (taking part in the masque) are spirits

PROSPEROSpirits which by mine artI have from their confines calld to enactMy present fancies

PROSPEROYes they are spirits whom I have summoned from the regions to which they are confined to carry into effect my fanciful designs

ORIGINAL TEXT PARAPHRASEFERDINANDLet me live here everSo rare a wondered father and a wifeMakes this place paradise

FERDINANDI should like to live here forever Such a wise and wonderful father makes this place a paradise

[Juno and Ceres whisper and send Iris on employment] [Juno and Ceres whisper and send Iris on employment]

PROSPEROSweet now silence

PROSPEROMy dear Ferdinand speak no more Juno and Ceres are

Juno and Ceres whisper seriouslyTheres something else to do Hush and be muteOr else our spell is marred

whispering with a solemn look There is something else coming Silence Or else our magic will be spoilt

IRISYour nymphs called naiads of the wandering brooksWith your sedged crowns and over-harmless looksLeave your crisp channels and on this green landAnswer your summons Juno does commandCome temperate nymphs and help to celebrateA contract of true love Be not too late

IRISYou nymphs called Naiads denizens (M inhabitants) of the running stream with your chaplets of sedge and ever-helpful looks leave your wrinkled channels and on the green land answer the summons sent to you Juno has ordered some chaste nymphs and help to celebrate a noble and true marriage Donrsquot delay

[Enter certain nymphs] [Enter certain nymphs]You sunburnt sicklemen of August wearyCome hither from the furrow and be merryMake holiday your rye-straw hats put onAnd these fresh nymphs encounter every oneIn country footing

You sunburnt harvesters weary from the effects of the heat in August come here from the furrowed land and rejoice Make holiday with your rye-straw hats upon you and meet these fresh nymphs and join in country dancing

[Enter certain reapers properly habited They join with the nymphs in a graceful dance towards the end whereof Prospero starts suddenly and speaks]

[Enter certain reapers properly habited They join with the nymphs in a graceful dance towards the end whereof Prospero starts suddenly and speaks]

PROSPERO[aside] I had forgot that foul conspiracyOf the beast Caliban and his confederatesAgainst my life The minute of their plotIs almost come [to the spirits]Well done Avoidno more

PROSPERO(Aside)I had forgotten the wicked conspiracy of the beast Caliban and his accomplices against my life the time of their plot has almost arrived ndash (To the Spirits) well done depart no more of this

[To a strange hollow and confused noise the spirits heavily vanish]

[The spirits depart]

ORIGINAL TEXT PARAPHRASEFERDINANDLet me live here everSo rare a wondered father and a wifeMakes this place paradise

FERDINANDI should like to live here forever Such a wise and wonderful father makes this place a paradise

[Juno and Ceres whisper and send Iris on employment] [Juno and Ceres whisper and send Iris on employment]

PROSPEROSweet now silenceJuno and Ceres whisper seriouslyTheres something else to do Hush and be muteOr else our spell is marred

PROSPEROMy dear Ferdinand speak no more Juno and Ceres are whispering with a solemn look There is something else coming Silence Or else our magic will be spoilt

IRISYour nymphs called naiads of the wandering brooksWith your sedged crowns and over-harmless looksLeave your crisp channels and on this green landAnswer your summons Juno does commandCome temperate nymphs and help to celebrateA contract of true love Be not too late

IRISYou nymphs called Naiads denizens (M inhabitants) of the running stream with your chaplets of sedge and ever-helpful looks leave your wrinkled channels and on the green land answer the summons sent to you Juno has ordered some chaste nymphs and help to celebrate a noble and true marriage Donrsquot delay

[Enter certain nymphs] [Enter certain nymphs]You sunburnt sicklemen of August wearyCome hither from the furrow and be merryMake holiday your rye-straw hats put onAnd these fresh nymphs encounter every oneIn country footing

You sunburnt harvesters weary from the effects of the heat in August come here from the furrowed land and rejoice Make holiday with your rye-straw hats upon you and meet these fresh nymphs and join in country dancing

[Enter certain reapers properly habited They join with the nymphs in a graceful dance towards the end whereof Prospero starts suddenly and speaks]

[Enter certain reapers properly habited They join with the nymphs in a graceful dance towards the end whereof Prospero starts suddenly and speaks]

PROSPERO[aside] I had forgot that foul conspiracyOf the beast Caliban and his confederatesAgainst my life The minute of their plotIs almost come [to the spirits]Well done Avoidno more

PROSPERO(Aside)I had forgotten the wicked conspiracy of the beast Caliban and his accomplices against my life the time of their plot has almost arrived ndash (To the Spirits) well done depart no more of this

[To a strange hollow and confused noise the spirits heavily vanish]

[The spirits depart]

Ac-12 27420 topic Revaluation of Assets and Liabilities

REVALUATION OF ASSETS AND LIABILITIES

On admission of a new partner the firm stands reconstituted and consequently the assets are revalued and liabilities are reassessed It is necessary to show the true position of the firm at the time of admission of a new partner If the values of the assets are raised gain will increase the capital of the existing partners Similarly any decrease in the value of assets ie loss will decrease the capital of the existing partners For this purpose alsquoRevaluation Accountrsquo is prepared This account is credited with all increases in the value of assets and decrease in the value of liabilities It is debited with decrease on account of value of assets and increase in the value of liabilities The balance of this account shows a gain or loss on revaluation which is transferred to the existing partnerrsquos capital account in existing profit sharing ratioAccounting for Revaluation of Assets and Liabilities when there is a Changein the Profit Sharing Ratio of Existing PartnersAssets and liabilities of a firm must also be revalued at the time of change in profit sharing ratio of existing partners The reason is that the realisable or actual value of assets and liabilities may be different from those shown in the Balance Sheet It is possible that with the passage of time some of the assets might have appreciated in value while the value of certain other assets might have decreased and no record has been made of such changes in the books of accounts Similarly there may be some unrecorded assets amp libilities that may have to be accounted for Revaluation of assets and reassessments of liabilities becomes necessary because the change in the

value of assets and liabilities belongs to the period to change in profit sharing ratio and hence must be shared by the partners in their old profit sharing ratio Revaluation of assets and reassessment of liabilities may be given effect to in two different ways (a) When revised values are to be recorded in the books and(b) When revised values are not to be recorded in the books

When revised values are to be recorded in the booksIn such a case revaluation of assets and reassessment of liabilities is done with the help of a new account called lsquoRevaluation Accountrsquo Sometimes this account is also called as lsquoProfit amp Loss Adjustment Acrsquo If there is a loss due to revaluation revaluation account is debited and if the revaluation results in a profit the revaluation account is credited The following journal entries made for this purpose are

(i) For increase in the value of assetsAsset Ac Dr (individually)To Revaluation Ac(ii) For decrease in the value of AssetRevaluation Ac Dr (individually)To Asset Ac[Decrease in the value of assets](iii) For increase in the value of LiabilitiesRevaluation Ac Dr (individually)To Liabilities Ac[Increase in the value of Liabilities](iv) For decrease in the value of LiabilitiesLiabilities Ac DrTo Revaluation Ac[Decrease in the value of Liabilities](v) For unrecorded AssetsAsset Ac [unrecorded] DrTo Revaluation Ac[Unrecorded asset recorded at actual value](vi) For unrecorded Liability Revaluation Ac DrTo Liability Ac [unrecorded][Unrecorded Liability recorded at actual value](vii) For transfer of gain on revaluationRevaluation Ac DrTo Existing Partnerrsquos CapitalCurrent Ac[Profit on revaluation transferred to capital account in existing ratio](viii) For transfer of loss on revaluationExisting Partnerrsquos CapitalCurrent Ac DrTo Revaluation Ac[Loss on revaluation transferred to capital account in existing ratio](a) When revaluation account shows gain Revaluation Ac DrTo Partnerrsquos Capital Ac (Old Profit Sharing Ratio)(Profit on revaluation credited to Partnerrsquos Capital Ac)(b) Above entry is reversed when revaluation account shows loss Partners Capital Acs (Old Profit Sharing Ratio) DrTo Revaluation Ac(Loss on revaluation debited to Partnerrsquos Capital Acs)

Proforma of Revaluation Account is given as under

Revaluation Account

Dr Cr Particulars ` Amount Particulars ` Amount To Decrease in value of assets By Increase in value of assets To Increase in value of liabilities By Decrease in value of liabilities To Unrecorded liabilities By Unrecorded assets To Gain on Revaluation (Transferred) By Loss on Revalution (Transferred)

ECO ndash12 2742020Topic- ELASTICITY OF DEMAND

CHAPTER - ELASTICITY OF DEMANDMEANINGDemand for a commodity is affected by many factors such as its price price of related goods income of its buyer tastes and preferences etc Elasticity means degree of response Elasticity of demand means degree of responsiveness of demand Demand for a commodity responds to change in price price of related goods income etc So we have three dimensions of elasticity of demandDIMENSION OF ELASTICITY OF DEMAND TYPES OF ELASTICITY OF DEMAND

Price elasticity of demand Income elasticity of demand Cross Elasticity of demand

Price elasticity of demand Price elasticity of demand means degree of responsiveness of demand for a commodity to the change in its price For example if demand for a commodity rises by 10 due to 5 fall in its price Price elasticity of demand (ep)=Percentage change in quantity demanded Percentage change in price of the commodity = 10 ( -)5 = ( - )2Note that ep will always be negative due to inverse relationship of price and quantity demanded

(ii) Income elasticity of demand Income elasticity of demand refers to the degree of responsiveness of demand for a commodity to the change in income of its buyer Suppose income of buyer rises by 10 and his demand for a commodity rises by 20 then Income elasticity of demand (ey)= change in quantity demanded change in price of the commodity =20 10 = 2

Cross Elasticity of demandCross elasticity of demand means the degree of responsiveness of demand for a commodity to the change in price of its related goods (substitute goods or complementary goods) Suppose demand for a commodity rises by 10 due to 5 rise in price of its substitute good then Cross elasticity of demand (ec) = change in quantity demanded change in price of related good = 10 2 = 5 (Tastes and preferences cannot be expressed numerically So elasticity ofdemand cannot be numerically expressed)

  • Chapter 1 Force (Summary)
  • Distinguish between external sovereignty and internal sovereignty
    • NAND Gate
      • Logic diagram
      • Truth Table
        • NOR Gate
          • Logic diagram
          • Truth Table
            • XOR Gate
              • Logic diagram
              • Truth Table
                • XNOR Gate
                  • Logic diagram
                  • Truth Table
                      • Physics
                      • Chapter 1 Electric Field ( Electric Dipole) (Summary)
Page 2:  · Web viewWe all know that Nouns are divided into two parts: common noun and proper noun.Apart from common and proper noun, we will also study about collective noun and compound

Class ISubject English 1

Chapter 1 naming words or noun

Names of people animal things and places are called naming words

Some words name people For example girl and mother

Some words name animals For example dog and cat

Some words name places For example school and park

Some words name things For example ball and moon

Subjects EVS

Chapter 1 My body parts

Our body is made up of different partsEach part of the body has an important role to play

We have two eyes two ears two hands one nose one tongue two legs two knees 10 toes and 10 fingers

Functions of body parts-

I see with my eyes

I taste with my tongue

My head rests on my neck I can turn my head with the help of my neck

I breathe and smell through my nose

I write and hold things with my hands and fingers

I stand walk dance and run on my feet

I eat and speak with my mouth

I bite and chew food with my teeth

I feel with my skin

_______________________________________________________________________________________________

Class II

Subject ndash English language Topic ndash ch 1 NounsDefinition of NounNouns are naming words that tells us the names of people place animal and things For example- boy girl dogcat park zoo basketapple etc In this chapter we learn two types of nouns-

Common noun Proper noun

Common noun ndashNames of people place animal things in general are called common noun Examples ndash

1 I love to play in the park2 The sky is blue3 Cow gives us milk4 The policeman catches the thieves

Proper noun ndashSpecial names given to people placesanimals and things are called Proper nouns It always starts with a capital letter Proper Noun are names of-

Person- Tom Harry Countrycitystates- IndiaKolkataWest Bengal Daysmonths- Monday May BooksfestivalsThe Bible Holi Mountainrivers- Mt Everest the Ganga

Examples- 1 Mr Das is an old man2 Taj Mahal is in Agra3 I live my pet dog Mac verymuch4 My uncle gifted me Titan watch

Difference between ndashCommon noun

1 girl2 car3 park4 aeroplane5 bag

Proper noun1 Devika 2 Maruti3 Nicco Park4 Air India5 Skybag

Subject ndash EVS Topic ndash My FamilyFamily is a group of people living together in the same house and is related to each other There are two main kinds of family-1 Nuclear family- In this family only father mother and their children live together in the same houseIt is also

known as a small or nuclear family2 Joint family ndash In this family grandfather grandmother father mother their children uncles aunt s and

cousins all live together in the same house It is also known as a big or joint familyMembers of the familyGrandfather ndash Father of your father or motherGrandmother- Mother of your father or motherUncle ndash Brother of your father or motherAunt ndash Sister of your father or motherSibling- Your own brother or sisterCousins ndash The children of your uncle and auntMaternal grandparents- Parents of your motherPaternal grandparents- Parents of your father

All members of the family has a common last name that is called surname

______________________________________________________________________________________________

Class III

Subject ndash Eng language Topic ndash ch 3 NOUNS

NOUNS

Nouns are the words that names people place animal and things Around us we see so many things Everything that we see has some name Example- computer blackboard chalk book teacher house cat dog etc

In this chapter we will learn about ndash

Common NounProper NounCollective noun

Common Noun

Common Noun are names of people place animal things in general For example girl boy hospital park ball car etc

Proper noun

Proper nouns are the specific or particular name of person place animal and thingsExamples- Mina Mt Everest Delhi Tommy etc Proper noun is always capitalized

Uses of Proper nouns- They are special names of

people such as RitaJackRam etc Countries citiesstates such as IndiaKolkataWest Bengal Mountains rivers seas lakes such as Mt Everest the Ganga the Indian Ocean Dal

Lake

Subject social studies

Topic My family and I

A family is a group of people who are related to each other by birth Members of a family always give love and support to each other

There are different kinds of families ndash nuclear family joint family and single parent family

Nuclear family means where father mother and kids stay together

Joint family means where many members of the family stay together For example grandfather grandmother father mother uncle aunt cousins live together

Single parent family means where the kids stay with any one of the parent like father or mother

It is always a fun when whole family spends time together example going to a picnic because it creates a strong bond among each other

Class IV

Subject English language Topic Nouns Compound nouns

We all know that Nouns are divided into two parts common noun and proper nounApart from common and proper noun we will also study about collective noun and compound nouns A common or proper noun that is the name of a collection or group of similar things or persons is known as a collective Noun Example crowd committee herd United Nations etc Example Mother has lost her bunch of keysThe word bunch is a collective noun

1 A herd of cows was grazing in the fieldHerd- collective noun

2 All living creatures need food to surviveCreatures- collective noun

3 The Ganga is a river sacred to IndiansIndians- collective noun

Compound noun some common and proper nouns are made up of more than one word They are known as compound nouns or simply compounds Example newspaper (news+paper) ashtray (Ash+tray) plaything (play+thing) etc1 My penfriendlives in a town in South AfricaPenfriend South Africa- compound noun

2 My grandparents live in New DelhiGrandparents New Delhi- compound nouns

3 Please do not park your cars in the drivewayDriveway- compound noun

Subject Social studies Topic A peep into the past

The story of the past is called history It is a journey across time and space that takes us to another period of time where people lived differently It is believed that the way of living eating clothes art were all different from what we have today We need to study history becauseIt tells us about the important people of the pastIt helps us to gain knowledge about the past and the present and help us to understand themHistory guides us to set an aim and warns us not to repeat the same mistakesIt helps us to understand the development of mankindIt helps to developessential skills to become a better person

Division of the pastHistory is so vast that it is divided into two parts ndash BC (Before Christ) and AD (Anna Domini) This helps us to understand about the past clearly Indian history and itrsquos civilisation are the worldrsquos oldest civilisation Indian history is divided into ancient medieval and modernCivilisation mean the stage of human social and cultural development

Subject COMPUTER Chapter-2 (MS WINDOWS)WINDOWS IS AN OPERATING SYSTEM THAT HELPS THE USER TO OPERATE AND CONTROL THE OVERALL ACTIVITIES OF A COMPUTER SOME POPULAR VERSIONS OF WINDOWS ARE WINDOWS 98 WINDOWS 7 WINDOWS 8 AND WINDOWS 10

DESKTOP THE FIRST SCREEN THAT WE SEE WHEN THE OPERATING SYSTEM (WINDOWS)HAS FINISHED LOADING IS KNOWN AS DESKTOP

THE FEATURES OF DESKTOP ARE bullIT IS WHERE ALL THE ICONS OF THE DIFFERENT APPLICATIONS ARE DISPLAYEDbullIT IS THE HOME FOR ALL SHORTCUTS PROGRAMS CAN BE QUICKLY OPENED BY CLICKING ON THE RESPECTIVE SHORTCUTSbullIT HOLDS VARIOUS INFORMATION SUCH AS TIME AND DATE

ICONSICONS ARE SMALL PICTURES OF DIFFERENT APPLICATIONS THAT ARE PRESENT AND INSTALLED IN THE COMPUTER FOR EXAMPLE COMPUTER ICON AND RECYCLE BIN ICON

DESKTOP GADGETS A DESKTOP GADGET IS A SMALL APPLICATION OR A WIDGET THAT RECIDES ON A COMPUTERS DESKTOP FOR EXAMPLE CLOCK CALENDAR ETC_______________________________________________________________________________________________

Class VSubject Socialal studiesTopic Evolution of man

Millions of years ago the earth looked much different as it is seen todayIt was totally different and it was very difficult to live Everything looked very different plants animals and humans everything looked very different from what they are now All the changes took place gradually with the changing environment Earliest ancestors of the humans were known as the Ramapithecus who lived about 14 million years ago They looked like ape that walked upright and weighed about 14kg With the passage of time there was a gradual development in the structure posture features and teeth of the early humansIt is said that the apesthat lived on treetops started living on the ground and stood on two limbs are called hominids They used their limbs to hold things and make tools Finally they changed to homosapiens mean wise human We humans belong to this group Life of early humansThe early men were hunter ndash gatherers They moved from place to place in search of food water and shelter They hunted animals and ate them raw gathered fruits nuts roots and seeds for eating They didnrsquot know how to build houses so they used live on treetops or in the caves or rock shelters In summers they remained uncovered but in winter they used to cover themselves by using bark of trees leaves and animal skins

Subject English language Topic Adjectives Interrogative and Emphasizing

Interrogative adjectiveAn interrogative adjective is used with a noun that it qualifies to ask a question The word interrogative mean to ask questions Example Which part of the world do you come fromThe word lsquoWhich is an interrogative adjectivesEmphasizing adjectivesSometimes we use adjectives to emphasize the effect of what we say Words which are used to emphasize a nounExample Rita came late and had to stand at the very end of the lineThe word Veryis an emphasizing adjective It actually qualifies or emphasizethe noun end

1lsquoDo not listen to what Ravi says for he is a complete foolrsquo said RohanComplete- emphasizing adjective

2Kiran is a great supporter of non violenceGreat- emphasizing adjective

3rsquoWhat crime has the prisoners committedrsquo asked the judgeWhat- interrogative adjective

4Whose book is that lying on the tableWhose- interrogative adjective

Subjects COMPUTER CHAPTER-1 (CHARACTERISTICS OF A COMPUTER)A COMPUTER SYSTEM IS A ONE THAT CONSISTS OF ALL THE SOFTWARES HARDWARES CPU AND MEMORY DEVICES SOME IMPORTANT TERMSTHE COMPUTER WORKS ACCORDING TO THE COMMANDS OR INSTRUCTIONS THAT WE GIVE

A SET OF INSTRUCTIONS IS CALLED THE PROGRAM A SET OF ONE OR MORE PROGRAMS IS CALLED A SOFTWARE THE PHYSICAL COMPONENTS OF A COMPUTER ARE CALLED HARDWARE

FOUR OPERATIONS OF A COMPUTER SYSTEM INPUTINPUT IS SUPPLIED TO THE COMPUTER WITH THE USE OF A KEYBOARD A MOUSE A MICROPHONE

OR OTHER INPUT DEVICES PROCESSING PROCESSING IS DONE INSIDE THE COMPUTER IN THE CPU PROCESSING IS THE CONVERSION

OF THE INPUT INTO THE OUTPUT STORAGE STORAGE REFERS TO THE SAVING OF INFORMATION FOR LATER USE INFORMATION IS STORED

IN THE MEMORY OF A COMPUTER THERE ARE TWO TYPES OF COMPUTER MEMORY PRIMARY MEMORY AND SECONDARY MEMORYPRIMARY MEMORY IS FAST SMALL AND EXPENSIVE THE TWO TYPES OF PRIMARY MEMORY ARE RANDOM ACCESS MEMORY(RAM) AND READ ONLY MEMORY (ROM) RANDOM ACCESS MEMORY (RAM)THIS MEMORY IS VOLATILE IN NATURE WHICH MEANS THAT THE

INFORMATION STORED IN IT IS AUTOMATICALLY ERASED WHEN THE COMPUTER POWER IS TURNED OFF

READ ONLY MEMORY (ROM) THIS TYPE OF MEMORY IS NON VOLATILE IN NATURE WHICH MEANS THAT DATA IS NOT LOST WHEN THE COMPUTER POWER IS TURNED OFF

OUTPUT OUTPUT IS THE RESULT PRODUCED BY A COMPUTEROUTPUT MAY BE VIEWED ON THE MONITOR SCREEN HEARD THROUGH SPEAKERS PRINTED THROUGH PRINTER ON PAPERS OR STORED IN THE MEMORY FOR LATER USE

Class VI

GEOGRAPHY CHAPTER 1 REPRESENTATION OF GEOGRAPHICAL FEATURES

GLOBE AND MAPS

GLOBE- A Globe is a three ndashdimensional model of the Earth

EQUATOR ndash An imaginary line runs through the centre of the earth from east to west

NORTHERN HEMISPHERE ndash the half of the north of the equator is called Northern Hemisphere

SOUTHERN HEMISPHERE ndash the half of the south of the equator is called Southern Hemisphere

LATITUDE - Imaginary parallel lines running on the globe from east to west

LONGITUDE ndash The imaginary lines semicircle in nature running from the North Pole to South Pole Also called meridian

MAP ndash Representation of the Earth as a whole or parts of it drawn on a flat surface to a scale

SKETCH ndash A rough drawing of a place not drawn to a scale

PLAN ndash A plan is prepared to show more details about a small area

COMPUTER - VI CHAPTER-6 (INTRODUCTION TO WINDOWS 10)

THE MOST RECENT VERSION OF WINDOWS IS WINDOWS 10 IT WAS RELEASED IN 2015

WINDOWS 10 IS DESIGNED TO WORK FOR YOU AND NOT YOU WORKING FOR IT THE WINDOWS 10 OPERATING SYSTEM INCLUDES A NUMBER OF NEW FEATURES LIKE IMPROVED PERFORMANCE ON MULTI CORE PROCESSORS IMPROVED G U I DATA SECURITY AND BETTER BOOTING PERFORMANCE

WINDOWS 10 HAS 1187 WITH WEARING FEATURES SETS AND INTENDED HARDWARE

THE VARIOUS EDITIONS AVAILABLE FOR WINDOWS 10 ARE AS FOLLOWS

WINDOWS 10 HOME

WINDOWS 10 HOME IS THE EDITION OF WINDOWS 10 THAT CONTAINS THE LEAST NUMBER OF FEATURESWINDOWS 10 HOME IS THE CONSUMER FOCUSED DESKTOP EDITION WITH SUPPORT FOR BOTH PC AND TOUCH ENABLED TABLETS WINDOWS 10 HOME IS AVAILABLE IN BOTH 32 BIT AND 64 BIT VERSIONS

WINDOWS 10 PRO

WINDOWS 10 PRO HAS FEATURES ESSENTIAL FOR SMALL OR MEDIUM BUSINESSESIT HAS A LARGE NUMBER OF EXTRA FEATURES TO MEET THE NEEDS OF SMALL BUSINESSES

WINDOWS 10 ENTERPRISE

WINDOWS 10 ENTERPRISE IS BUILT ON WINDOWS 10 PROIT HAS FEATURES WHICH ARE DESIGNED TO MEET THE NEEDS OF MEDIUM AND LARGE ORGANISATIONSTHIS WINDOWS EDITION TARGETS THE ENTERPRISE SEGMENT OF THE MARKET

WINDOWS 10 MOBILE

WINDOWS 10 MOBILE IS DESIGNED FOR CUSTOMER ORGANISATION THAT ARE USING THE WINDOWS 10 PLATFORM ON SMARTPHONES AND SMALL TABLETS

WINDOWS 10 EDUCATION

WINDOWS 10 EDUCATION IS DESIGNED TO MEET THE NEEDS OF STAFF ADMINISTRATORS TEACHERS AND STUDENTS OF SCHOOLS

WINDOWS 10T EDITION

WINDOWS 10T EDITION OF WINDOWS 10 IS DESIGNED FOR USE IN SMALL FOOTPRINT AND LOW COST DEVICES

Subject English language

Topic Noun kinds

NOUN KINDS

Noun is a naming word Nouns are names of people places animals or things

For example Ashley Mumbai tiger pencils

Kinds of nouns

Proper noun the name of a particular person place or thing is called proper nounExample

1 The Himalayas stand to the north of India2 Monika is the school captain

Common noun announce that names people place or thing in general is called a common noun Example

1 Kalidas was the greatest dramatist of India2 William Shakespeare is Englands national poet

Collective Noun A collective noun is the name of a collection of people or things taken together and spoken of as a whole Example

1 The feet were completely destroyed in the fierce attack2 The crew revolted against Captain Grand

Abstract Noun An abstract noun is the name of some quality state or idea Example1 Diversity Indias biggest strength2 Patience is a virtue

_____________________________________________________________________________________

Class VII

Subject English 1 Topic Articles

ARTICLES

There are three articles ------ A An The

1 Indefinite articles- The Articles a and an are called Indefinite Articles They do not point to a particular person or thing They are used with singular countable nouns It does not identify a specific noun

Uses of Indefinite articles A

Before a singular noun beginning with a consonant or a vowel with a consonant sound For example a cattle a university a one way track

Before a proper noun which is either unfamiliar or holds a special meaning For example For example A Mahesh Sharma wants to see youNeela wants to be a Tendulkar when she grows up

With a number beginning with a consonant sound For example a ten- rupee note a hundred years

Before half when half follows a whole number or after half when it isnrsquotExample one and a half litres (before) Half a litre (after)

With an expression of quantity For example a lot of time a dozen oranges a little while In exclamation before nouns Example Such a mess What a pity

Uses of Indefinite articles An

Before a singular noun beginning with a vowel sound Example an ant an egg an umbrella Before an abbreviation beginning with a vowel or a consonant with the vowel sound

Example an MP an MA Before the word beginning with a silent h Example an heir an hour an honour

Note We do not say a milk or a lemonade because they are uncountable nouns ever we see a cup of milk or a glass of lemon and

Subject GEOGRAPHY

CHAPTER 1 REPRESENTATION OF GEOGRAPHICAL FEATURES

TOPIC- TOPOGRAPHICAL MAPS

Topographical Maps- topographical maps are small-scale maps with detailed depiction of both natural and human-made features

Conventional colours-different colours used in a map are known as conventional colours

TYPES OF SCALE-

Verbal Statement- Scale written in a statement from like 2 cm to a Km is called verbal statement It means that 2cm on the map represents 1 km on the Earth

Representative Fraction- Scale can also be written as a representative fraction (RF)

Example 150000 it means 1cm on the map represents 50000cm or frac12 km on the ground It shows the ratio between map distance and ground distance

Linear Scale Scale can be drawn on a line to show map distance equivalent to ground distance A line is divided into equal parts Each parts represents the actual distance on the ground in mkm

SUBJECT-COMPUTER

CHAPTER-1 (COMPUTER FUNDAMENTALS)

COMPUTER LANGUAGES

THE THE TERM COMPUTER LANGUAGE REFERS TO A SYSTEM OF RULES AND SYMBOLS THAT ARE DESIGNED TO GIVE INSTRUCTIONS TO A COMPUTERTHE COMPONENTS OF A COMPUTER SYSTEM CANNOT PERFORM A TASK BY THEMSELVES THEREFORE THEY HAVE TO BE GIVEN INSTRUCTIONS TO PERFORM ANY TASKCOMPUTER LANGUAGES ARE USED TO CREATE PROGRAMS USING LOGIC BASED ALGORITHMSTHE RULES OF A COMPUTER LANGUAGE IS

KNOWN AS SYNTAX WHEREAS THE TERM SEMANTICS REFERS TO THE MEANING OF LANGUAGESCOMPUTER CANNOT DEVELOP ANY PROGRAMMING LANGUAGE BY THEMSELVES AND CANNOT THINK INTELLIGENTLY UNLESS THEY ARE ASSISTED BY THE HUMAN BEINGS THEREFORE THE PROGRAMMERS DEVELOP A SET OF METHODS AND TECHNIQUES A SET OF INSTRUCTIONS USED TO PERFORM A SPECIFIC TASK IS CALLED A PROGRAM

A PROGRAMMING LANGUAGE IS ALSO KNOWN AS A COMPUTER LANGUAGE CODED BY PROGRAMMERS TO WRITE INSTRUCTIONS FOR A COMPUTERTHE COMPUTER TAKES THESE INSTRUCTIONS AS INPUT AND PRODUCES THE DESIRED OUTPUT

TYPES OF COMPUTER LANGUAGE

THE COMPUTER LANGUAGE CAN BE BROADLY DIVIDED INTO TWO CATEGORIES

LOW LEVEL LANGUAGES HIGH LEVEL LANGUAGES

LOW LEVEL LANGUAGES

A LOW LEVEL PROGRAMMING LANGUAGE REFERS TO THE LANGUAGE THAT IS UNDERSTOOD BY A COMPUTER DIRECTLYTHE PROGRAMMER MUST HAVE AN IN-DEPTH KNOWLEDGE OF DIFFERENT COMPUTERS TO WRITE PROGRAMS IN A LOW LEVEL LANGUAGE THE TWO TYPES OF LOW LEVEL LANGUAGES ARE MACHINE LANGUAGE AND ASSEMBLY LANGUAGE

MACHINE LANGUAGE

COMPUTER HARDWARE UNDERSTANDS ONLY MACHINE LANGUAGE AS IT IS THE FUNDAMENTAL LANGUAGE OF A COMPUTER WHICH UNDERSTANDS ONLY THE TWO DIGITS 0 AND 1 KNOWN AS THE BINARY DIGITSIT IS A SYSTEM OF INSTRUCTIONS EXECUTED DIRECTLY BY THE CPU WITHOUT ANY TRANSLATION AND HENCE IS EXECUTED QUICKLY AS COMPARED TO PROGRAMS IN OTHER LANGUAGES PROGRAMS IN MACHINE LANGUAGE NEED DIFFERENT BINARY LANGUAGE PROGRAMS TO SOLVE THE COMPLEX TASKSPROGRAMMERS OFTEN FIND IT DIFFICULT TO WRITE PROGRAMS IN THIS LANGUAGE

ASSEMBLY LANGUAGE

AN ASSEMBLY LANGUAGE IS A LOW LEVEL PROGRAMMING LANGUAGE BUT THE INSTRUCTIONS ARE WRITTEN IN THE FORM OF WORDS KNOWN AS MNEMONICS TO CONVERT AN ASSEMBLY LANGUAGE INTO TO MACHINE CODE WE REQUIRE A UTILITY PROGRAM WHICH IS KNOWN AS AN ASSEMBLER ASSEMBLY LANGUAGE PROGRAMS CANNOT BE EXECUTED DIRECTLY BY A COMPUTER THEREFORE THEY ARE SLOWER THAN THE MACHINE LANGUAGE PROGRAMSIT IS EASIER TO WRITE PROGRAMS IN THE MACHINE LANGUAGE THEN AND IN ASSEMBLY LANGUAGE

_______________________________________________________________________________________________

CLASS-VIII

SUBJECT-COMPUTER CHAPTER - Operating system and graphical user interface Role and functions

Need of operating system

It co-ordinates different hardware and software componentsof a computer system It supervises the various actions of the computer system and enables the computer to work in a effective

manner It helps in smooth functioning of various peripherals

In a multi-tasking operating system it determines the order and time to be allowed for each application before giving another application a turn

It sends messages to the system operator about the status of operation or any error that may have occurred while running the application

What is operating system

An operating system is an integrated system of programs that manages various resources and the overall operation of the computer system It is designed to support various activities of computer system in a systematic way

Role of an operating system-

Operating system enables the user to use the system effectively An operating system manages various application that runs on a computer and shares computerrsquos resources User interacts with operating system through command line interface and graphical user interface

Function of an operating system -

Booting the computer it is the start up procedure of a computer system Loading theprograms in the memory - when system is ready the operating system loads certain program

automatically Manages resources it manages between the hardware and software resources Detecting and correcting errors- if the supporting hardware or software doesnrsquot works properly then the

operatingsystem tries to rectify it Ensuring data security programs and data donot interfere with each other Maintaining the internal clock of system - maintain internal clock of system when system is shut down

SUBJECT-GEOGRAPHY CHAPTER 1 REPRESENTATION OF GEOGRAPHICAL FEATURES THROUGH CONTOURSTopographical maps- topographical maps are small-scale maps with detailed depiction of both natural and human-made features

Contour lines- contour lines are imaginary lines drawn on a map joining places having the same height above the mean sea level

Contour linesContour interval- It is an interval at which contour lines are drawnIndex contour- At every 100-m interval a thick brown contour line is called index contour It is drawn for calculating height

Spot heights- The spot heights show heights in metres above the mean sea level

SUBJECT- English Language CHAPTER - The Sentence

Complex sentence-

We have seen that a Complex Sentence consists of a Principal Clause with one or more Subordinate Clauses

We have also learnt that there are three kinds of Subordinate Clauses The Adjective Clause the Adverb Clause and the Noun Clause

1) THE ADJECTIVE CLAUSE

An ADJECTIVE CLAUSE does the work of an ADJECTIVE It qualifies a noun or pronoun In the following examples in each set two commonly patterned sentences are compared one with an ADJECTIVE and the other with an ADJECTIVE CLAUSE ( The Adjective or Adjective Clause qualifies the Noun placed in the box)

a) He is a lazy boy (Adjective)

b) He is a boy who is lazy (Adjective Clause)

a) It is a beautiful place for the tourists (Adjective)

b) It is a place where tourists come for the scenic beauty (Adjective Clause)

a) We have enough funds for the work (Adjective)

b) We have funds which would be enough for the work (Adjective Clause)

Convert the sentence from simple to complex sentence-

1) We believe his honesty Ans- We believe that he is honest

2) This is the birth place of RamaAns- This is the place where Rama was born

____________________________________________________________________________________________

Class IX

Subject English Language

Topic Preposition

Date 270420

PREPOSITIONS

A preposition is a word placed before a noun or a pronoun to show in what relation the person or thing denoted by it stands in regard to something else

EXAMPLE

There is a clock on the wall

The preposition on shoes relationship between clock and the wall

Smitha is afraid of lizards The man jumped off the bus

Here the preposition of shows the relationship between afraid and lizards

The preposition off shows the relation between jumped and bus

The noun or the pronoun which follows a preposition is called its object

So in the first Example wall is the object of the preposition on

A preposition can have more than one object For example

The plane flew overhouses and meadows

Prepositions are used to express a number of relationship including time locationmannermeans quantity purpose and state or condition

Points to remember

Preposition joins a noun to another noun or a pronounThere is a cow in the field

A preposition joints a noun to a verbThe cat runs after the rat

Preposition can have two or more than two objectsThe road runs over hills and plain

A preposition also joins a noun an adjectiveHe is fond of tea

Generally a preposition comes before an object Sometimes it comes even after an object asWhat are you looking atThis is the house I live in

Subject- Computer Application

Chapter 2 Introduction to Java

Java API An application programming interface (API) in the context of Java is a collection of prewritten packages classes and interfaces with their respective methods fields and constructors

Byte Code Java bytecode is the result of the compilation of a Java program an intermediate representation of that program which is machine independent The Java bytecode gets processed by the Java virtual machine (JVM) instead of the processor JVM The Java Virtual Machine (JVM) is the runtime engine of the Java Platform which allows any program written in Java or other language compiled into Java bytecode to run on any computer that has a native JVM

Platform A platform is the hardware or software environment in which a program remains

Java platform The Java platform differs from most other platforms in that itrsquos a software only platform that runs on top of other hardware-based platforms

Applet and Application The fundamental difference between the two Java programs is that an application program is designed to run on a stand-alone machine whereas an applet is a web-version of an application which is used to run a program on a web browser

WORA ldquoWrite once run anywhererdquo (WORA) or sometimes write once run everywhere (WORE) is a slogan created by Sun Microsystems to illustrate the cross-platform benefits of the Java language

Class XSubject Topic Summary Execution

ECONOMICSFACTORS OF PRODUCTION Sub-topiclsquoLABOURrsquo

We shall start our class by discussing the topic taught in the last class

lsquo DIVISION OF LABOURrsquo- By division of labour we mean specialization in workIt refers to splitting up the work of labour involved in the production of a particular commodity into several parts and each part and sub-part is performed by a specialist

Now let us start by the Advantages of Division of labouraIt increases the level

Questions

1 Differentiate between Product-Based division of labour and Process-Based division of labour

Product-Based division of labour

Process-based division of labour

It is also known as simple (or occupational) division of labourUnder it everybody performs a particular occupations The entire is done by the same person

When a person or group of persons undertakes a specialised function which is supplementary to the production of final commodity and service This is also as complex division of labour

It is simple It is complex

of productionbSince the product is produced by an expert workerbest quality of product is producedcIt saves time and toolsd it promotes inventions in the methods and techniques of productioneIt leads to reduction in costs fAll workers get work according to their abilities and choices

Now let us discuss the disadvantages of Division of labour

a Since many workers are involved in the production of a commodity no one has the sense of responsibility

b The constant and repetition of the same work again and again make the work monotonous

c Division of labour facilitates production on large scale Hencethere is fear of over production

d Because of territorial division of labour some areasregions become more developed than others

It is based on labour-intensive techniques of production

It is based on capital-intensive techniques

It is generally found in small enterprises

It is generally found in large enterprises

Example Indian farmers doing all farm activities

Example A modern garmet factory where one person takes the measurementanother does the cuttingsome sew the clothes while a few workers button them and other iron them

2 What are the advantages of Division of labouraIt increases the level of productionbSince the product is produced by an expert workerbest quality of product is producedcIt saves time and toolsd it promotes inventions in the methods and techniques of productioneIt leads to reduction in costs fAll workers get work according to their abilities and choices

3Discuss the disadvantages of Division of laboura Since many workers are involved in the

production of a commodity no one has the sense of responsibility

b The constant and repetition of the same work again and again make the work monotonous

c Division of labour facilitates production on large scale Hence there is fear of over production

d Because of territorial division of labour some areasregions become more developed than others

English 1 Transformation of sentences

Sentences A sentence is a group of words which makes complete sense

a Assertive sentences

Exercise 6Rewrite the following sentences according to the instructions given below without changing their meanings

1 As soon as he saw the beer he jumped into

b Imperative sentences

c Interrogative sentences

d Exclamatory sentences

Sentences can be changed from one grammatical form to another without changing the meaning of the sentence This is known as transformation of sentences

the river ( Begin No sooner)2 None but brave deserve the fair (Begin the

bravehellip)3 This box is too heavy for me to lift ( Use so hellip

That instead of too)4 No one other than a king can live like James

Luxurious ( Begin only James)5 Oh for the wings of a dove (Begin I wishhellip)

Math Topic Commercial MathematicsChapter Shares and Dividends

Study item Discuss about shares and Dividends1) What is share

Ans To start any big business (company or Industry) a large sum of money is needed But it is not possible for an individual to invest such a large amount Then some persons interested in the business join together and from a company They divide the estimated money required into small parts Each such part is called a share

2) What do you mean by the term shareholder

Ans A person who purchases one or more shares is called shareholder3) Some terms related with a share

(i) Nominal value or face value or printed value The original value of a share is called its nominal value or face value or printed value

Note The nominal value of a share always remains same(ii) Market value or cash value The price of a share at any

time is called its market value or cash value

Note The market value of a share changes from time to time(iii) At par If the market value of a share is the same as its

nominal value the share is called at par(iv) At Premium or above Par If the market value of a share

is more than its nominal value the share is called at premium or above par

Example If a share of Rs 100 is selling at Rs 150 then it is said to be selling at a premium of Rs 50 or Rs 50 above par

(v) At Discount or below par If the market value of a share is less than its nominal value the share is called at discount or below par

Example If a share of Rs 100 is selling at Rs80 then it is said to be selling at a discount of Rs 20 or at Rs 20 below par

4) What is Dividend

Ans The profit which a shareholder gets for hisher investment from the company is called dividendNote (i) The dividend is always expressed as the percentage of the face value of the share(ii) The dividend is always given( by the company ) on the face value of the share

irrespective of the market value of the shareBENGALI(2ND LANGUAGE)

ldquoদেবতোর জণমrdquoলিবরোম চকরবত

পরথম লিসর পোঠ-চোর পসথ একটি পোথর লিবপলি ঘটোয় দেক যোতোয়োসতর পসথ পরলিতলিয়ত ওই পোথসর দেো োচট দেসত একলি দেতো দেক দেো োচট দেসয় দেবোমো সয় রোসতোর মোস লি0টসক পস1 লিবপরীত লিক দেথসক আো একটি দেমোটর োলি1 চোসকর কষতোয় পরোসরণ দেবোসচ যো লিকনত পরলিতবোর এমরণ দেৌভোয দেসকর োও সত পোসর তোই লিতলি দেকোো দেজোো1 কসর পোথরটিসক উপস1 দে8স পোথর উপস1 দে8োর ময় এক দেকৌতী জতো দেকসক পরশন কসর দেয লিতলি দেকোসো দেবতোর আস দেপসয়স0রণ লিকো লিকনত দেক বস লিতলি দেকোসো দেবতোর আস পোলি দেক উপলিত ক জতোর উসltসয বস কোরও ইস= স পোথরটি লিসয় দেযসত পোসর এর পর দেথসক দেক দেযসত আসত পোথরটি দেক দেসত পো একলি দেক কষয করস দেকউ পোথরটিসক ধসয়মস0 পলিরসকোর করস0 দেক ওই দেকৌতী জতোসক পোথসরর কোস0 বস থোকসত দেস এ0ো1ো আরও কষয কসর দেক ঠোৎ ওই লি1 পোথরটির োসয় লিোর োো-দেকউ পজো কসরস0 দেকৌতী দেোকটির আঙকো য় যলিসক উ পোথরটিসক লিরসয় দে8স তোর পর ঠোৎ একলি পোথরটির দেোোজ দেই দেক লিসয় দেস0 বো দেকোথোয় দেস0

বদোথ-

দেো োচট ndashচসত লিসয় দেকো লিক0র সE ধোককো দেস পস1 যোবোর উপকরমঅকসমোৎ- ঠোৎআতমমবর- লিসজসক লিয়নতরপ-পো লিপ0স প1োদেসতসসত- লিসপলিউৎোত- দেো1ো দেথসক উপস1 দে8োপরতযয় ndash লিবশবোপরতযোস- টতযোস ndash দৈবোসধসতোধলিসত- পরসপসরর পরলিত ব পরসয়ো করোপরসতরীভত- পোথসর পলিররণলিতবোনতঃকরসরণ- মস পরোসঅলিQৎ- অQো করসত ই=কদেোপ- বধমোর- দেবোস এমইতযোকোর- এইরকমরম- বময়পযসোভী-পসযর জয দেোভ আস0 যোরপোসথয় ঞচয়- পথচোর রচ জমোসোমোমোসরোস- ব ধম ধোসমর সE

তোর লি দেইhelliphellipTo be continued

Hindi 2nd lang सर क पद(सरदास)

सरदास शरी कषण भकति कावय क सरवशरषठ कगिरव ह इनक जनम और मतय क समय तथा सथान का मतभद हसरदास रवातसलय और शरार रस क अनयतम कगिरव ह इनक कावय म बालकषण क सौदय चपपल चषटा और गि7याओ की मनोहर झाकी मिमलती ह कषण और ोगिपयो क अननय परम का कतिचतरण ह सयो शरार की अपकषा उनक कावय म गिरवयो शरार का अमिBक गिरवषय और मारमिमEक कतिचतरण हआ हइन पकतियो म हम सरदास की भकति भारवनाओ का परिरचय मिमलता ह इनका सपण सगरह सरसार म गिनगिहत ह

1 जसोदा हरिर पालन झलारवहलरारवदलराईमलहारव रव जो ईसाई कछ ारव मर लाल को आई निनEदिदया काह ना आगिनसबारव त काह नाही बरवगिह आरवतोको कानहा बलारव

शबदाथ-हलरारव-गिहलती हदलराई - दलार पयार करती हमलहारव-पचकारती हनिनEदरिरया ndashनीदरवगिह-जलदी सअBर-होठमौन-चपसन-सकत

वयाखया- सरदास जी कहत ह गिक यशोदा माता बालक कषण को पालन म झल आती ह रवह उनह गिहलाती ह पयार करती ह मलहार जस कोई ीत ान लती ह और नीद स पछती ह गिक ह नीद तम मर लाल को आकर कयो नही सलाती तझ खाना बला रहा ह कभी कषण आख बद कर लत ह कभी आखफडफडान लत ह उनह सोता हआ जानकर यशोदा माता चप हो जाती ह और इशार म बात करन लती ह इसी बीच अकला कर कषण ज जात हतो गिफर यशोदा माता गिफर स ाना ान लती ह सरदास जी कहत ह गिक भरवान क दशन का सख दरवता और ऋगिष-मगिनयो को भी दलभ ह यही सख माता यशोदा को बडी सहजता स मिमल जा रही ह माता यशोदा बहत ही भागयशाली ह2)Continue to nexthellip

Physics

Chapter 1 Force

(Summary)

Question A body is acted upon by two forces each of magnitude F but in opposite directions State the effect of the forces if

(a) Both forces act at the same point of the body

(b)the two forces act at two different points of the body at a separation r

Solutions

(a) Resultant force acting on the body = 0

F ndash F = 0(b) The forces tend to rotate the body between two forces about the midpoint

Moment of forces = F times rFr

QuestionDefine moment of a couple Write its SI unit

Solutions

Moment of couple is equal to the product of both force and the perpendicular distance between the two forces

The SI unit of moment of couple is NmCommercial Studies

Advertising and sales

Business firms use several methods to

Questions1) What do you mean by advertising

promotion create demand of their product in the market and increase it sales Such methods comprises of advertising sales promotion personal selling and publicityToday we are going to discuss about one of such methods It is advertising

Meaning of advertisingAdvertising is a paid form of non-personal presentation for promotion of Ideas goods and services

Importance or merits of advertising Advertising has importance to manufacturer or traders to customer and to society as a whole

Today we will see how advertisement help the manufacturer or traders

Answer) Advertising is a means of how a company encourages people to buy their products services or ideas It is one element of marketing which also includes design Research and data mining

2) Mention any three features of advertisingAnswer)The main features of advertising are

i) It is impersonal form of presentation for promotion of products and services of Ideas

ii) It is issued by identified sponsor The advertisement contains the name of the advertiser

iii) It is a form of mass communication because the message is directed to a large number of persons simultaneously

3) Mention the main merits or importance of advertisement to manufacturer or tradersAnswer)

i) Introducing new product A business organization can introduce itself and its products to the public through advertising

ii) Increase the sale Advertising leads to increase the sale of existing product by entering into new markets and attracting new customers

iii) Create steady demand Advertising creates sustains regular demand by smoothening out seasonal and other fluctuations It enables regular production for the organisation

iv) Economics of scale Advertising facilitate mass distribution of goods and steady demand which lead to large scale and regular production

v) Goodwill Advertising helps in creating a good image of the firm and reputation for its products

Biology Chapter - 03Genetics

Today wewill start chapter and discuss about Genetics Gregor Mendel is known as father of genetics Before entering into Mendelrsquos experiment on Genetics we must know

Q1 Define the following termsi) Genetics Genetics is the study of

transmissionof body features from parents to offspringand the laws relating to such transmission

ii) Heredity It may be defined as transmissionof genetically based characteristics from parentsto offspring

iii) Character and traits Any heritable

Importance to TraderIntroducing new productIncrease the saleCreate steady demandEconomics of scaleGoodwill

some terms featureis a character The alternative forms of acharacter are called traitsex Character (Hair shape) - Traits (Curly straight)

iv) Homologous chromosomes A pair ofcorresponding chromosomes of the same shapeand size one from each parent

v) Genes Genes are the specific parts (DNA segments) of a chromosome which determinethe hereditary characteristicsNearly 30000genes present in human

vi) Alleles Alternative forms of a gene occupying the same position (locus) on homologouschromosomes and affecting the same characteristicbut in different ways

vii) Genotype ndash PhenotypeGenotype means of genes present in the cells of an organism Phenotype means the observable characteristic which is genetically controlled

viii) Mutation It is a sudden change in one or more genes or in the number or in the structure of chromosomes ex Sickle cell anaemia is a blood disease caused by a gene mutation

CLASS NOTES

Class XSubject Eng Literature (The Merchant of Venice ndash William Shakespeare)Topic Act IV Scene 1 Lines 01 to 34 ( Duke helliphelliphellip We all expect a gentle answer Jew) ate 27th April 2020 (2nd Period)

[Students should read the original play and also the paraphrase given in the school prescribed textbook]Summary Questions amp Answers

This scene may be termed as the catastrophe of the play It is the final unravelling of the complicated events which seem to threaten the happiness of Bassanio Portia and Antonio Right is justified to the fullest degree and malice falls into the trap prepared for others No one suffers here but Shylock but even then he receives a generous measure of mercy

o This is the Court-scene Initially we meet

(1)

DUKE I am sorry for thee thou art come to answer (Line 3-6)A stony adversary an inhuman wretchUncapable of pity void and emptyFrom any dram of mercy

(i) Who is addressed here Where is the person Why is the person there

Antonio is addressed hereAntonio is in the court of justice at VeniceAntoniorsquos trial is scheduled to be held here for his failure to meet the conditions of the bond he signed with Shylock

the Duke Antonio Shylock and Salerio Later we meet Bassanio Portia Gratiano and Nerissa

o The Duke says to Antonio that he has to face a very cruel opponent which Antonio admits and expresses his gratefulness to the Duke for his efforts to soften without result the heart of Shylock in order to be merciful to Antonio Antonio further says that he is ready to accept whatever cruel judgement the Court may award

o When Shylock appears in the court the Duke says that Shylock should change his decision of prosecuting Antonio and demanding the penalty specified in the bond out of consideration of the great misfortunes that Antonio has suffered If this is done by him (Shylock) the whole court would be gladdened by his merciful action

(ii) What is the Duke sorry for

The Duke is unable to change the mind of Shylock from his decision to get the bond forfeited even after he pleaded to Shylock Shylock stands firmly for his bond which when forfeited will allow him to take a pound of flesh from any part of Antoniorsquos body(iii) How does the Duke address Shylock`The Duke calls Shylock an adversary with a heart of stones He calls Shylock as an inhuman wretch without pity Shylock is quite lacking in the slightest quality of mercy (iv) How does Antonio reply to this

Antonio replies that he will meet the revenge of Shylock patiently He has prepared himself to suffer with a quiet spirit the utmost that Shylockrsquos tyranny and rage can do(v) What quality of the Duke is revealed here

The Duke is kind and benevolent He is ready to help Antonio He requests Shylock to free Antonio from the trial(vi) What are the terms of the bond that Antonio has signed

The terms of the bond that Antonio has signed were that if Antonio is unable to repay Shylock a certain sum of money specified on the paper on a certain date and in an agreed place the forfeit has to be paid The forfeiture will be an exact pound of Antoniorsquos flesh which Shylock will be a liberty to take from any part of Antoniorsquos body which pleases him

Class XI

STUDY MATERIAL

Class XISubject Eng Literature (The Tempest ndash William Shakespeare) Topic Act I Scene 2 Lines 88 to 132 (Prospero hellip Me and thy crying self) Date 27th April 2020 (3rd Period)

[Students should read the original play and also the paraphrase given in the school prescribed textbook]Summary Questions amp Answers

o Prospero now tells Miranda that he was the Duke of Milan He had been devoting himself more to studies than the affairs of the State His brother Antonio took advantage of this situation and with the help of Alonso the king of Naples seized upon him and her one midnight and shipped them in a frail bark so that they perished in the sea All this took place

(1)

MIRANDA I should sin (Line 118-132)

To think but nobly of my grandmother

Good wombs have borne bad sonsPROSPERO Now the condition

The King of Naples being an enemyTo me inveterate hearkens my brothers suitWhich was that he in lieu othrsquo premisesOf homage and I know not how much tribute

twelve years back

IMPORTANT PASSAGES EXPLAINED(Line 98-103)

PROSPERO helliphelliphelliphelliphelliphelliphellip

He being thus lorded

Not only with what my revenue yieldedBut what my power might else exact like oneWho having into truth by telling of it Made such a sinner of his memoryTo credit his own lie he did believeHe was indeed the duke

Prospero in telling the narrative of his past life here refers to his brother Antonio Prospero being with a studious bent of mind has left the administration of Milan on his younger brother Now Antonio being thus invested like a lord with all the powers derived from Prosperorsquos wealth and what the exercise of Prosperorsquos authority might secure for him regarded himself as a de facto Duke of Milan It is a well-known fact of psychology that a man who repeatedly tells a lie makes of his memory such a sinner against truth as to credit his own lie by the telling of it So Antonio by repeatedly saying to himself and others that he was the Duke came to believe that he was really the Duke Thus falsehood repeatedly asserted gained the force of truth for Antonio and he truly believed it

Should presently extirpate me and mine Out of the dukedom and confer fair MilanWith all the honours on my brother whereonA treacherous army levied one midnightFated to thrsquo purpose did Antonio openThe gates of Milan and ithrsquo dead of darkness The ministers for thrsquo purpose hurried thenceMe and thy crying self

(i) In the earlier lines of this scene what does Prospero tell about his intense interest What was the demand of his interest

In the earlier lines of this scene Prospero tells Miranda that he had an intense interest in the study of philosophy and magic arts Hence in order to improve his mind with this kind of study he kept himself isolated from worldly and state affairs His study was dearer to him than the applause and esteem that he could win from the public His study demanded too much solitude(ii) What forced Antonio to take an undue advantage over Prospero

Prosperorsquos indifferent attitude towards the statersquos affairs and his having boundless trust in Antonio gave rise to a boundless lust for power in Antoniorsquos mind Antonio felt that he must be the actual Duke instead of the part of the Duke he played Thus Antonio took an undue advantage of the situation to usurp Prosperorsquos dukedom(iii) Explain the following lines ldquoI should sin to think but nobly of my grandmother Good wombs have borne bad sonsrdquo

After hearing the treacherous act of her uncle Antonio Miranda says that Prosperorsquos mother was a noble lady and she cannot dishonour her memory by saying that the person named Antonio cannot be his (Prosperorsquos) brother She says that in honour of her grandmother she also cannot say that Antonio must have been begotten not by her grandfather but by some other man She finally concedes that it is known that good mothers have borne bad sons in their wombs and gave birth to them

(iv) Why did the King of Naples accept Antoniorsquos request to help him in usurping his dukedom What did Antonio propose to Alonso

Alonso who was the king of Naples accepted Antoniorsquos request in usurping Prosperorsquos dukedom because he (Alonso) was a sworn enemy of Prospero Antonio proposed that Alonso should immediately drive him (Prospero) and his offspring out of Milan and should confer the dukedom upon him (Antonio) with all the dignities which go with that

position In return Antonio promised that he shall give an annual tribute and also swore his allegiance to Alonso Also he agreed to hold the Dukedom of Milan as a subordinate to the state of Naples(v) How were Prospero and Miranda carried away from the city of Milan and what was the state of small Miranda at that time

In pursuance of the agreement settled between Antonio and Alonso an army of treacherous men was assembled One midnight when the occasion suited the will of destiny Antonio opened the gates of the city of Milan and in the death like silence of midnight Antoniorsquos agents who had been directed to execute his purpose carried Prospero and small Miranda away from the city in all haste They were then forced into a ship and carried some distance out to sea where they put them on a mere hulk of a boat without any rigging or ship-gear and abandoned them leaving them at the mercy of the roaring sea Miranda was a very small child of three years age and she was crying at that time

CLASS -XIDATE-270420Subject Topic Summary Execution

EVS Chapter 1 ndash Mode of Existence

Impact of mode of existence on resources

Q) Why resources are under pressure

Ans - Increase in the sophistication

of technology enabling natural resources to be extracted quickly and efficiently Eg in the past it could take long hours just to cut down one tree only using saws Due to increased technology rates of deforestation have greatly increased

The number of humans is increasing Cultures of consumerism Materialistic views

lead to the mining of gold and diamonds to produce jewelry unnecessary commodities for human life or advancement Consumerism also leads to extraction of resources for the

production of commodities necessary for human life but in amounts excessive of what is needed because people consume more than is necessary or waste what they have

Lack of awareness among the population is striking People are not aware of ways to reduce depletion and exploitation of materials

Accounts Cash Book Today we are going to start a new topic -Cash Book

The key terms used in this chapter are

bullCash book

bullSimple cash book

bullDouble column cash book bullTriple column cash book

bullPetty cash book

bullCash discount

bullContra entry

Here I will share you the meaning of each key terms

bullCash book Cash Book is a special purpose subsidiary book or journal in which cash received and cash payments are recorded

bullSimple cash book

It is a cash book in which only cash transactions are recorded It has only one column on each side

bullTriple column cash book

It is cash book which has three columns one column for each cash and Bankdiscount on each side of the cash book In this book both cash and Bank transactions are recorded together with discount allowed and received

bullPetty cash book

It is a cash book maintained for recording petty expenses

bullCash discount

Cash discount is the amount of discount received or allowed on cash payments and cash receipts Discount received is an income for the business while discount allowed isan expense

bullContra entry

It means transactions involving both cash and Bank Such transactions though recorded in the cash book are not posted into ledger The letter lsquoC is written in Ledger folio for contra entry

Business Studies

ENTREPRENEURSHIP

Now we shall discuss the second chapter

lsquoENTREPRENEURSHIPrsquo

Today before starting the chapter let us recall what

Questions

1What are the main characteristics of Intrapreneurship

Answer

The main characteristics of Intrapreneurship are

Corporate framework-it occurs within the framework of the same company

Semi-Autonomous-Intrapreneurship

we have read last day

Let s today start the class by recalling the last topic taught

Intrapreneurship is the process of discovering and exploring business opportunities within an existing company It involves launching new business ventures within the framework of a present corporation Intrapreneurship is also known as corporate entrepreneurship or corporate venturing

Now let us start with the characteristics of Intrapreneurship

The main characteristics of Intrapreneurship are

Corporate framework

Semi-Autonomous Lack of ownership Senior position Low risk taking Not own boss

Now let us discuss the meaning of enterprise

Enterprise means an undertaking or adventure that requires some innovation and investment and thus involves riskEnterprise always entails decision making coordination and risk bearing

involves crating amd nurturing a semi-autonomous business unit which may be a subsidiary a strategic business unit or a division

Lack of ownership-the intrapreneur is not the owner of the unitb he creates and nurtures

Senior position-he occupies a senior managerial position in the company

Low risk taking-An intrapreneur does not bear the full risk of failure

Not own boss-An intrapreneur is not his own bosss in legal termsHe enjoys the freedom and gets the required resources and support

2 How is Entrepreneur is different from Intrapreneur

The functions involved in both the entrepreneurship and intrapreneurship are by and large similar however there are several differences between the two

Point of distinction

Entrepreneur

Intrapreneur

status An independent business person

A senior executive within a company

Ownership Owner of

the business

An employeesometimes a share in ownership

Financing Responsible for raising finance for the business

Not responsible for raising the finance

Risk bearing

Bears the risk of the business

Does not bears the risk of the business

Reward Profit which is uncertain and irregularcan be loss

Fixed salary and fringe benefits

Need for security low high

3 What do you understand by enterprise

Answer Enterprise means an undertaking or adventure that requires some innovation and investment and thus involves riskEnterprise always entails decision making coordination and risk bearing

COMMERCE NATURE AND OBJECTIVES OF

BUSINESS

Today let us recall the last other two objectives of business by the chart given in the previous class

Firstly we would discuss Human Objectives

Business is run by people and for people Labour is a valuable business element

Human objectives of business are concerned with the well -being of labour

The human objectives are as follows

Labour welfare Developing human

resources Participative

management Labour

management cooperation

Questions

1 Explain the human objectives of a business enterprise

Answer

Business is run by people and for people Labour is a valuable business element

Human objectives of business are concerned with the well -being of labour

The human objectives are as follows

Labour welfare-Business must recognize the dignity of labour and human factors should be given the recognition

Developing human resources-Employees must be provided the opportunities for developing new skills and attitudes

Participative management-Employees should be allowed to take part in decision making process of business

Labour management cooperation-Business should strive for creating and maintaining cordial employer employee relations so as to ensure peace and progress in industry

Now let us discuss the national objectives of business

Optimum utilization of resources

National self- reliance Development of small

scale industries Development of

backward areas Control over pollution

2Explain the national objectives of a business enterprise

Answer

It is the duty of business to utilize the resources of the country properly the national objectives of business

Optimum utilization of resources ndashBusiness should use the nationrsquos resources in the best possible manner

National self- reliance-It is the duty of the business to help the government in increasing experts and in reducing dependence on imports

Development of small scale industries-Big business firms are expected to encourage growth of small scale industries which are necessary for generating employment

Development of backward areas-Business is expected to give preference to the industrialization of backward regions of the country

ECONOMICS

BASIC ECONOMIC CONCEPTS

SUB

TOPIC

Value

Wealth

Welfare

Today we shall start with a new topic of the same chapter ie lsquoValuersquo

Value of a commodity is defined as the valuation placed by a household on the consumption of this commodity

lsquoValuersquo has two different meanings and these are

a Value -in -use It refers to consumption value of a commodity It expresses the utility derived from the consumption of a particular commodity A necessity like water has a very high value ndashin ndashuse or

Question

1What is value

Answer

Value of a commodity is defined as the valuation placed by a household on the consumption of this commodity

2What is value-in use

Answer It refers to consumption value of a commodity It expresses the utility derived from the consumption of a particular commodity A necessity like water has a very high value ndashin ndashuse or consumption value

3What is value ndashin- exchange

Answer It relates to market value of a commodity

It is the rate at which a particular good or service can be exchanged for money

For example in barter system if a person is prepared to exchange 3 metres of cloth with 1 pair

consumption value

b Value ndashin-exchange It relates to market value of a commodity

It is the rate at which a particular good or service can be exchanged for moneyFor example in barter system if a person is prepared to exchange 3 metres of cloth with 1 pair of shoes then the value in exchange of 3 metres of cloth is 1 pair of shoesValue in exchange is the power of purchasing other goods In modern monetised economies the exchange value of goods are expressed in terms of money as prices

Now let us discuss the term lsquoWealthrsquo

Wealth refers to the stock of all those assets which are a source of income

Wealth is a stock concept

Wealth must possess the following features

a Utility It must possess utility or give some

of shoes then the value in exchange of 3 metres of cloth is 1 pair of shoes

Value in exchange is the power of purchasing other goods In modern monetised economies the exchange value of goods are expressed in terms of money as prices

4 What is wealth

Answer it refers to the stock of assets or goods which are a source of income and have personal or national ownership

5 What are the features of wealth

Answer The features of wealth are as follows

Wealth must possess the following features

a Utility It must possess utility or give some satisfaction

b Scarcity It must be limited in quantityc Transferability it should be transferable its

ownership can be transferred from one person to another person

d Exchange value It must possess exchange value

6 What is welfare

Answer

Welfare is defined as satisfaction and happiness a sense of well- being among the people

satisfactionb Scarcity It must be

limited in quantityc Transferability It

should be transferable its ownership can be transferred from one person to another person

d Exchange value It must possess exchange value

Now let us discuss the term lsquoWelfarersquo

Welfare is defined as satisfaction and happiness a sense of well- being among the people

Welfare is affected by factors like

a Consumption of goods and services

b Environment

c Family relations

d Degree of freedom

e Law and order situation

Mathematics Trigonometric equation

To find the general solution of the equation sinθ=0

When sin θ =0

Then θ= 0 π2π 3π-π -2π -3

i e when θ = 0 or an integral multiple of π

i e when θ= nπ where n is any integer

Therefore the general solution of the equation sin

Example1 Find the general values of θ which satisfy the equation sin2 θ =34

Solution sin2 θ= 34

Or sin θ = +34 or -34

Or sin θ = sin π3 or sin (-π3)

Therefore

θ = [nπ + (-1) n (π3)] or[ nπ+ (-1) n (-π3)]

= nπ +π3 or nπ-π3 where n= any integer

Example 2Find the values of θ which satisfy tan2 θ

θ=0 is θ= nπ where n is any integer

To find the general solution of the equation cos θ=0

When cos θ=0

Then θ=π2 3π2 5π2 -π2 -3π2 -5π3

i e when θ is an odd multiple of π2

i e when θ=(2n+1) π2 where n= any integer

Therefore the general solution of the equation cos θ =0 is θ= (2n+1) π2 where n= any integer

To find the general solution of the equation tan θ = 0

Clearly tan θ =0 implies sin θcos θ =0

Therefore θ = nπ

i e the general solution of the equation tan θ=0 is θ =nπ where n = any integer

To find the general solution of the equation cot θ =0

Clearly cot θ =0 implies (cos θsin θ) = 0

i e cos θ =0

Therefore θ = (2n+1) π2

Therefore the general solution of the equation cot θ =0 is θ = (2n+1) π2

Where n= any integer

To find the general solution of the equation sin θ= k (-

=13 -πleθleπ

Solution tan2 θ =13

Or tan θ = plusmn1radic(3) =tan(plusmnπ6)

θ=nπ plusmn π6 where n =any integer

If n=0 then θ=plusmnπ6

If n=1 then θ= π plusmn π6

If n=-1 then θ= -π plusmn π6

Therefore the required solution in -π le θ le π are θ= π6 5π6 -π6 -5π6

Exercise Find general solution of sin 2θ=cos θ [Hints Use sin 2θ= 2sin θcosθ and then take cosθ

common]

1lek le1)

Determine an angle alpha such that sin =k and -π2le αle π2

Then we have

Sin θ = k = sin α

Or sin θ - sin α =0

Or 2 cos [(θ+α) 2] sin [(θ-α) 2] =0

Therefore either cos [(θ +α) 2] =0 (1)

Or sin [(θ-α) 2] =0 (2)

Now from (1) we get (θ+α) 2= (2m+1) π2)

Or θ = (2m+1) π-α (3)

And from (2) we get (θ-α) 2 =mπ

Or θ= 2mπ+α(4)

Where m = any integer

Clearly the solution (3) amp (4) may be combined in the following form

θ= nπ+(-1) n α where n= any integer

Therefore the general solution of sin θ = sin α is θ = nπ +(-1) n α where n is any integer and -π2 le α le π2

Biology Chapter - 04Kingdom Monera

Today we will discuss about bacterial reproduction and its usefulness

Fig Binary Fission

Fig Conjugation Fig Transformation

Bacterial reproduction is mainly asexual but sexual reproduction

also takes place

Asexual reproduction takes place by i) Binary fission - from one bacteriato

two bacteria are produced in every 20 to 30mins

ii) Buddingiii) Endospore formation - during

unfourable condition

Sexual reproduction by three ways

1) Conjugation - Transfer of genetic material between cells that are in physical contact with one another

2) Transduction - Transfer of genetic materialfrom one cell to another by a bacteriophage

3) Transformation - Transfer of cell-freeor naked DNArsquo from one cell to another

Bacteria causes different diseases inplants animals and human and

it causes food spoilage and waterpollution but it also have some useful

activities

i) Bacteria are helpful in sewage water treatment

ii) It is used in antibiotic (medicine) production

iii) Anaerobic bacteria help in biogas(energy) production

iv) Many household products like yoghurt cheese are manufactured by use of bacteria

v) Rhizobium by symbiotic relationship with leguminous plant increase soil fertility

vi) Besides these bacteria is helpful in genetic engineering degradation of petroleum hydrocarbonand in dairy

industry

Physics Motion in plane Here we will introduce Projectile Motion

Execution

Projectile

Y

usinθ u h

θX

ucosθ

Suppose a body is projected with an angle θ So initial velocity u can be resolved into two components

Horizontal component - ucosθ ( for range)

Vertical component - usinθ ( for height)

usinθ changes during motion and becomes zero at maximum height position but ucosθ remain unchanged

The maximum height of projectile is h

NB If initial is upward then g = -ve and if it is downward then g = +ve Height is +ve if direction of motion does not change ( for ex a body thrown upwards but goes down ultimately then height h = -ve)

The angle of projectile θ is the angle made with horizontal

HISTORY ndash GROWTH OF NATIONALISM

SUB TOPIC- REVOLUTIONARY NATIONALISM Bengal formation of Anushilan Samity and Jugantar Group

The intensification of the Swadeshi movement and Government policy of terror and repression led to outbreak of violence Bombs were manufactured and attempts on the lives of unpopular Government officials became frequent In the gymnasium of Scottish Church College which was known as General Assemblies Institution a secret society was formed known as Anushilan Samity

Aurobindo Ghosh send from Baroda his emissary Jatindranath Banerjee to mobilize the Bengal revolutionaries

Hemchandra Qanungo and Satyen Bose published Journal Jugantar

The Jugantar group planned to assassinate oppressive magistrate Kingsford by Khudiram Bose and Prafulla Chaki in 1908 Prafulla Chaki committed suicide to avoid arrest Khudiram was tried and hanged

Afew days later the police found a bomb factory in Maniktala and arrested a large number of revolutionaries The trial of revolutionaries became famous as the Alipore Bomb Case

In the course of the trial the approver the public prosecuter and a police officer were assassinated

1 Question Name two journals which preached the cult of violence

Answer a) Yugantar edited by Bhupendranath DuttaB) Bandemataram edited by Aurobindo Ghosh2 Question Why was Khudiram arrested and hangedAnswer An attempt was made to assassinate a hated vindictive majistrate named Kingsford by Khudiram Bose and Prafulla Chaki Their attempt failed and the bomb they threw killed two English ladies Khudiram was arrested and put to trial and then hanged3Question Who was Aurubindo GhoshAnswer Aurobindo Ghosh a nationalist revolutionary who was charged for his involvement in the Alipore Bombing Case He was accused of it along with his brother Barindra nath Ghosh But Aurobindo was acquitted because of the brilliant pleading of his counsel Chittaranjan Das Then he became a spiritual reformer introducing his visions on human progress and spiritual evolution4 Qustion Who was KingsfordAnswer Kingsford was an unpopular British chief Magistrate who was the target of the bomb thrown at Muzaffarpur by Khudiram and Prafulla Chaki

Most of the accused were convicted and sentenced to

long term of imprisonmentBut

Aurobindo Ghosh was acquitted mainly owing to the brilliant pleading of his counsel Chittaranjan Das

Political science Topic-Sovereignty

Summary Sovereignty is the full right and power of a governing body over itself without any interference from outside sources or bodies In political theory sovereignty is a substantive term designating supreme legitimate authority over some polity In international law sovereignty is the exercise of power by a state

Internal Sovereignty

Internal sovereignty means supreme authority within ones territory while external sovereignty relates to the recognition on the part of all states that each possesses this power in equal measure

External sovereignty

external sovereignty relates to the recognition on the part of all states that each possesses this power in equal measure

Distinguish between

Execution

Answer the following questions

Short notes-

Sovereignty

Internal Sovereignty

External sovereignty

Homework- learn

external sovereignty and internal sovereigntySovereignty is the principle

of supreme and

unquestionable authority

reflected in the claim by the

state to be the sole author of

laws within its territory

Definition of external vs internal sovereigntyInternal sovereignty refers to

the relationship between a

sovereign power and its

subjects ndash it refers to the

location of the supreme

authority within the state In

the UK for example internal

sovereignty (supposedly)

resides within Parliament

reflected in the

constitutional principle of

parliamentary

sovereigntyBy contrast

external sovereignty refers

to the capacity of the state

to act independently and

autonomously on the world

stage This is what is

sometimes called lsquostate

sovereigntyrsquo or lsquonational

sovereigntyrsquo and implies

that states are legally equal

and that the territorial

integrity and political

independence of a state is

inviolable

Class ndash XII

Date - 2742020 STUDY MATERIALSubject Topic Summary Execution Business Studies

Job Analysis amp Manpower Planning

At first let us recall the chapter what we have discussed till nowbullJob analysisbullJob specification bullJob description bullJob enlargement bullJob enrichment

Today we will do some questions answers from the chapter

Questions 1ldquoJob analysis job description and job specification are interrelatedrdquo Comment Answer) Job analysis is a systematic and detailed examination of a job to collect all the relevant information about it The contents off the job are summarised in the job description The qualification needed for the job are summarised in job specificationThus there is close interrelationship between job analysis job description and job specification

Question 2ldquoJob enlargement is a horizontal extension of a job whereas job enrichment is a vertical extension of a jobrdquo ElucidateAnswer) Job enlargement involves adding one or more task to a job coma where as job enrichment involves adding more autonomy and responsibility to a job Job enlargement is therefore horizontal extension of a job coma whereas job enrichment is a vertical extension of a job

Question 3 )

What is manpower estimation Explain its quantitative and qualitative aspectsAnswer) Manpower estimation is the process by which management determines how an organisation should move from itrsquos current manpower positionto its desired manpower position There are two dimensions of Manpower estimation- quantitative and qualitative

Quantitative aspectThis aspect of Manpower estimation involves estimating the number of employees required in a future time period Workload analysis and workforce analysis are done to estimate the quantity of required manpower

Qualitative aspectThe estimate of the knowledge skills experience etc of required manpower is the qualitative aspect of Manpower estimation The quality of Manpower can be judged on the basis of job analysisand job specification

COMMERCE

CAPITAL-FIXED AND WORKING

Today let us start the class by discussing the sources of finance for different types of business firms

The term lsquocapitalrsquo refers to the investment made in the enterprise for the purpose of earning profits

Requirements of capital and sources of capital for different types of business firms are

1 Capital for sole proprietorship businessA sole proprietor operates at a small scale and thereforerequires a limited amount of capital

2 Capital for partnership firmCapital requirements as well as capital base of a partnership is bigger than that of a sole trader businessThe owned capital is contributed by the partners in an agreed ratio

3 Capital for joint stock companyA joint stock company generally requires large amount of capitalA public company can raise huge capital through issue of shares In addition to share capital it can utilize retained profits

Now let u discuss the meaning of Finance PlanningFinance planning is the process of estimation the financial requirements of an organization specifying the sources of firms and ensuring that enough funds are available at the right time

1 What do you mean by Finance PlanningAnswer Finance planning is the process of estimation the financial requirements of an organization specifying the sources of firms and ensuring that enough funds are available at the right time

2Discuss the role of financial planning of an enterpriseThe role of financial planning are as followsa A sound financial plan helps a business enterprise to avaid the problems of shortage and surplus of fundsbFinancial planning serves as a guide in developing a sound capital structure so as to maximize returns to shareholders c It helps in effective utilization of fundsd It provides policies and procedures for coordinating different functional areas or departments of businesse It enables the management to exercise effective control over the financial activities of an enterprisef It helps the company to prepare for facing business shocks and surprises in future

Mathematics

Continuity and differentiability

Recall Definition of ContinuityLet f(x) be a single valued function of x and x=a be a point in the domain of definition of the function The function is said to be continuous at x=a ifi) f(c) is defined ie f(x) has a definite finite value at x=cii) lim xrarra f(x) exists andiii) lim xrarra f(x) =f(a) In other words f(x) is said to be continuous at x=a if lim xrarra+ f(x)= lim xrarra- f(x) = f(a) Or f(a+0) =f(a-0) =f(a) Or lim hrarr0 f(a+h)= f(a) Algebra of continuous functionsNow we will study some algebra of continuous functions Theorem 1 Suppose f and g be two real functions continuous at a real number c Then(1) f + g is continuous at x = c(2) f ndash g is continuous at x = c(3) f g is continuous at x = c(4) (fg) is continuous at x = c (provided g (c) ne 0)

Example 1 Prove that every rational function is continuousSolution Recall that every rational function f is given byf(x)=[p(x) q(x) ] q(x)ne0where p and q are polynomial functions The domain of f is all real numbers except points at which q is zero Since polynomial functions are continuous f is continuous by (4) of Theorem 1Example 2Discuss the continuity of sine functionSolution To see this we use the following factslim xrarr0 sin x =0Now observe that f (x) = sin x is defined for every real number Let c be a real number Put x = c + h If x rarr c we know that h rarr 0 Therefore lim xrarrc f(x)

= lim xrarrc sin x= lim hrarr0 sin(c+h) =lim hrarr0 [sin c cos h + cos c sin h ]=lim hrarr0 (sin c cos h) + lim hrarr0 (cos c sin h) = sin c +0=sin c = f(c) Thus lim xrarrc f(x) = f(c) and hence f is a constant function Exercise Prove that the function f(x) = x2 +2x is continuous for every real value of x [Hints show that lim xrarra+ f(x) = lim xrarra- f(x) = f(a) ]

Biology Reproduction in Flowering plants We will discuss about megasporoangium

megasporagenesis and female gametophyte

Q4 Describe the structure of megasporangium

Ovule is attached to the placenta by astalk called funicle

Each ovule has one two or three protectivecoverings called integuments

At the tip of integuments a small openingcalled micropyle is organised

Opposite to the micropylar end is the chalaza

Within the integuments a mass of cellsnucellusand inside it embryo sac orfemale gametophyte is present

Q5 Describe a mature embryosacamp its formation

In most of the flowering plants only oneof the 4 megaspores formed as a result ofmegasporogenesis that is functional while theother three degenerate

The

functional megaspore develops into thefemale gametophyte

Formation The nucleus

of the functional megasporedivides mitotically to form two nuclei first andthen two more sequential mitotic nucleardivisions result in the formation of four ampthen eight nucleate stages of embryo sac

Six of the eight nucleus are surrounded bycell walls and organised into cells

The remaining two nuclei called polar nuclei are found below the egg apparatus in the largecentral cell

Three cells consisting of two synergids amp one egg cell present bottom of

embryo sac Three cells

at the chalazal as antipodal cells

Two polar nuclei together present in large central cell

HISTORY

TOWARDS INDEPENDENCE AND PARTITION THE LAST PHASE(1935-1947)SUB TOPIC NATIONAL MOVEMENTS DURING THE SECOND WORLD WAR

Spread of Quit India Movement On 9th August 1942Gandhiji and other Congress leaders were arrested The Congress was declared illegal The news of the arrest of all leaders marked the beginning of a widespread movement of India It was not possible for such a movement to remain peacefulBut the arrest of the all notable congress leaders virtually left the movement in the hands of the mass The movement took the form of violent and militant outbreakBesides congressmen revolutionaries also were very active in the movement The Congress Socialist group also played a prominent role

1 Question Why did the British authority arrest the Congress leaders on 9 th August 1942Answer Congress Working committee adopted the Quit India resolution which was to be ratified at the Bombay AICC meeting in 8th August 1942 They decided to launch a mass struggle on non-violent lines Gandhiji gave a clarion call to all section of the people rdquoKarenge ya Marengerdquo (do or die) Congress leaders gave the call to driving out

the British from IndiaViceroy had taken strong action against the Quit India movement Gandhiji and all the leaders of Congress were arrested

2 Question How did Quit India Movement spread out all over IndiaAnswer The news of the leaders lsquo arrest marked the beginning of a widespread movement to remain peacefulThe movement took form of violent outbreak There were widespread cutting of telephone and Telegraph wires damaging railway lines raising barricades in cities and towns and other forms of violent demonstations

Question Name the leaders of Congress

Socialist group played a prominent part Notable among the Jayprakash Narayan Rammonohar Lohia Aruna Asaf Ali

Political science

Topic-Franchise and Representation

Summary

The election commission

The Election Commission of India is an autonomous constitutional authority responsible for administering Union and State election processes in India The body administers elections to the Lok Sabha Rajya Sabha State Legislative Assemblies in India and the offices of the President and Vice President in the country

Functions of election commission-

India is a sovereign socialist secular democratic republic Democracy runs like a golden thread in the social economic and political fabric woven by the Constitution given by lsquoWe the People of Indiarsquo unto ourselves The concept of democracy as visualised by the Constitution pre-supposes the representation of the people in Parliament and State legislatures by the method of election The Supreme Court has held that democracy is one of the inalienable basic features of the Constitution of India and forms part of its basic structure The Constitution of India adopted a Parliamentary form of government Parliament consists of the President of India and the two Houses mdash Rajya Sabha and Lok Sabha India being a Union of states has separate state legislatures for each state State legislatures consist of the Governor and two Houses mdash Legislative Council and Legislative Assembly mdash in seven states namely Andhra Pradesh Telangana Bihar Jammu amp Kashmir Karnataka Maharashtra and Uttar Pradesh and of the Governor and the state Legislative Assembly in the remaining 22 states Apart from the above two out of the seven Union Territories namely National Capital Territory of Delhi and Puducherry also have their Legislative Assemblies

ExecutionShort notes-Election commissionFunctions of election commission

Homework- Learn

Computer

Science

Computer hardware NAND Gate

A NOT-AND operation is known as NAND operation It has n input (n gt= 2) and one output

Logic diagram

Truth Table

NOR Gate

A NOT-OR operation is known as NOR operation It has n input (n gt= 2) and one output

Logic diagram

Truth Table

XOR Gate

XOR or Ex-OR gate is a special type of gate It can be used in the half

adder full adder and subtractor The exclusive-OR gate is abbreviated as EX-OR gate or sometime as X-OR gate It has n input (n gt= 2) and one output

Logic diagram

Truth Table

XNOR Gate

XNOR gate is a special type of gate It can be used in the half adder full adder and subtractor The exclusive-NOR gate is abbreviated as EX-NOR gate or sometime as X-NOR gate It has n input (n gt= 2) and one output

Logic diagram

Truth Table

Physics

Chapter 1 Electric Field ( Electric Dipole) (Summary)

Here we will derive Expression of electric field at broad side

On position of dipole

Execution

Q With the help of a labelled diagram obtain an expression for the electric field intensity E at any point on the equitorial line ( broad-side on position) of an electric dipole

Ans

E1 E1sinθ

E θ P E1 θ

( r2+L2)12 E2 E

r E2 E2sinθ

-q θ L O L +qA B

Let us consider that the point P is situated on the right bisector of the dipole AB at a distance r meter from its midpoint O

Let E1 and E2 be the electric field intensities of the electric field at P due to charge +q and ndashq of the dipole resp The distance of P from each charge is ( r2+L2)12

So E1 = 14 πϵ q

(r 2+L 2) away from +q

E2 = 14 πϵ q

(r 2+L 2) towards ndashq

The magnitudes of E1 and E2 are equal but directions are different Now resolving E1 and E2 into two components parallel and perpendicular to AB we get

The components perpendicular to AB E1sinθ and E2sinθ cancel each other because they are equal and opposite

The components parallel to AB are E1cosθ and E2 cosθ are in same direction and add up

So resultant intensity of electric field at the point P is

E = E1cosθ + E2 cosθ

E = 14 πϵ q

(r 2+L 2) 2 cosθ

Now from fig we have cosθ =BOBP = L (r2+L2)12

So we get E = 14 πϵ 2qL ( r2+L2)32

Now electric dipole moment p= 2qL

So E = 14 πϵ p ( r2+L2)32

HW Find the expression of Electric field as done here but this time take r gtgt 2L

Also find the expression of torque experience by a dipole

(Hint Electric force experienced by charges of dipole in electric field is qE each Let θ be the angle which dipole makes with electric lines of force then perpendicular distance between two charges is 2Lsinθ Then torque = force x perp distance = qE x 2L sinθ So τ=pE sinθ where p =2qL )

STUDY MATERIAL

Class XIISubject Eng Literature (The Tempest ndash William Shakespeare) Topic Act IV Scene 1 Lines 84 to 133 (Iris hellip A contract of true love Be not too late ) Date 27th April 2020 (4th Period)

[Students should read the original play and also the paraphrase given in the school prescribed textbook]Summary Questions amp Answers

o Ceres soon appears and comes to know that she has been summoned to celebrate the contract of true love

o Ceres expresses her unwillingness to meet Venus and Cupid as she has shunned their company

o Ceres and Juno both bestow their blessings upon Ferdinand and Miranda with June gifting honour riches happiness in marriage and Ceres presents plenty of earthrsquos produce

o Iris summons the water-nymphs and reapers to come and celebrate a contract

(1) IRIS Of her society (Line 91-101)

Be not afraid I met her deity

Cutting the clouds towards Pathos and her sonDove-drawn with her Here thought they to have doneSome wanton charm upon this man and maidWhose vows are that no bed-right shall be paidTill Hymens torch be lightedmdashbut in vainMarss hot minion is returned againHer waspish-headed son has broke his arrowsSwears he will shoot no more but play with sparrowsAnd be a boy right out

(i) Where were Venus and Cupid seen flying How were they travelling Why did they want to join the marriage celebration of Ferdinand and Miranda

of true love

Venus and Cupid were seen flying through the air towards Paphos the famous city which is situated on the island of Cyprus They were travelling by air-borne chariot drawn by doves They certainly wanted to come here in order to play some amorous trick upon Ferdinand and Miranda who are under a vow not to gratify their physical desires till the holy ceremony of their marriage has been performed(ii) What have Venus and Cupid done after failing in their plan

After being failure of their plan Venus who is a very passionate deity and who is the mistress of Mars (the god of war) has gone back while here ill-tempered son Cupid has broken his arrows of love in his state of desperation(iii) What has Cupid firmly decided

Cupid is feeling so disappointed that he has firmly decided to shoot no more arrows to arouse love in human hearts but to spend his time playing with sparrows Thus he would now become just a boy and would give up his original function of shooting arrows on human beings to make them fall in love(iv) What vow had Ceres taken How did Ceres feel at the abduction

After the abduction of her daughter Prosperina by Pluto Ceres had taken a vow to always keep away from the disgraceful company of Venus and her blind son Cupid the god of love Ceres felt deeply distressed when Pluto had carried off her daughter and had made her his wife by force(v) Why has Ceres not forgiven Venus and her blind son For what do Ceres want to be sure

As the abduction had been manipulated by Venus the goddess of beauty and love and her blind son Cupid Ceres has never forgiven them for their part in the whole plot Ceres wants to be sure that she would not have to meet Venus and Cupid who had engineered the abduction of her daughter Prosperina

AS THIS lsquoMASQUErsquo SCENE IS VERY IMPORTANT IN THE PLAY THE PARAPHRASE OF THE ENTIRE PORTION OF MASQUE SCENE (Act IV Lines 58 to 143) IS GIVEN BELOW

IRIS Goddess of RainbowCERES Goddess of Agriculture and all the fruits of the earth

(Nature growth prosperity rebirth ndash notions intimately connected to marriage)JUNO The majestic Queen of Heavens and wife of Jupiter (Jupiter is the king of Gods)

VENUS The Goddess of love CUPID Son of Venus PLUTO God of death (In the play referred by Shakespeare as lsquoDisrsquo which is a Roman name for Pluto)

ORIGINAL TEXT PARAPHRASEPROSPEROWellmdash

PROSPERONow come Ariel Let there be too many rather than too few

Now come my Ariel Bring a corollaryRather than want a spirit Appear and pertly[to Ferdinand and Miranda]No tongue all eyes Be silent

spirits in attendance Appear briskly

[to Ferdinand and Miranda]Look with your eyes but do not say a word

[Soft music] [Soft music][Enter Iris] [Enter Iris]

IRISCeres most bounteous lady thy rich leasOf wheat rye barley vetches oats and peasThy turfy mountains where live nibbling sheepAnd flat meads thatched with stover them to keepThy banks with pioned and twilled brimsWhich spongy April at thy hest betrimsTo make cold nymphs chaste crowns and thybroom-grovesWhose shadow the dismissegraved bachelor lovesBeing lass-lorn thy pole clipped vineyardAnd thy sea-marge sterile and rocky-hardWhere thou thyself dost airmdashthe Queen othrsquoSkyWhose watery arch and messenger am IBids thee leave these and with her sovereign grace[Juno appears] Here on this grass-plot in this very placeTo come and sport Her peacocks fly amainApproach rich Ceres her to entertain

IRISCeres most generous lady you are the cause of rich fields or fertile land where wheat rye barley beans oats and peas grow the grassy mountains where the sheep graze and the flat meadows covered with coarse hay to be used as fodder for cattleYour banks are covered with marsh-marigolds and reeds and the rainy April under your orders brings forth to make for the maids who are not in love beautiful crowns your woods where the broom flourishes and where the bachelor who has been dismissed by the maid he loved lies down being forsaken your vineyard in which the poles are embraced by the vines and the margin of the sea which is barren and rocky where you roam about to enjoy the fresh air ndash the queen of the sky (Juno) whose messenger I am besides being represented as the rainbow bids you leave all these and with her majesty here on this grassy plot in this very place come and sport her peacocks carry her fast in her chariot through the air and are making their way here approach rich Ceres to welcome her

[Enter Ariel as Ceres] [Enter Ariel as Ceres]

CERESHail many-coloured messenger that neerDost disobey the wife of JupiterWho with thy saffron wings upon my flowersDiffusest honey-drops refreshing showersAnd with each end of thy blue bow dost crownMy bosky acres and my unshrubbed downRich scarf to my proud earth Why hath thy queenSummoned me hither to this short-grassed green

CERESWelcome rainbow that never dared disobey Juno the wife of Jupiter who with your orange coloured rays spread honey-drops refreshing showers And with each end of thy blue bow drown my bushy acres and my hilly country which is free from shrubs you thus forming a rich scarf Why has your queen called me here to this place covered with short grass

IRISA contract of true love to celebrateAnd some donation freely to estateOn the blest lovers

IRISI have called you to celebrate a contract of true love and bestow some liberal gift upon the blessed lovers

ORIGINAL TEXT PARAPHRASECERESTell me heavenly bowIf Venus or her son as thou dost knowDo now attend the queen Since they did plotThe means that dusky Dis my daughter gotHer and her blind boys scandaled companyI have forsworn

CERESTell me heavenly bow if Venus the Goddess of love or Cupid her son and pedlar of passion at this time attend the heavenly queen Juno because you are sure to know Since the day they conspired against me and dark Pluto took away my daughter here and Cupidrsquos disgraceful company I have left off

IRISOf her societyBe not afraid I met her deityCutting the clouds towards Pathos and her sonDove-drawn with her Here thought they to have doneSome wanton charm upon this man and miad

IRISBe not afraid of her company I met her deity moving on the clouds towards Paphos the sacred home of Venus on the island of Cyprus along with her son on her chariot drawn by doves Here they contemplated to exercise a charm upon this man and maid producing

Whose vows are that no bed-right shall be paidTill Hymens torch be lightedmdashbut in vainMarss hot minion is returned againHer waspish-headed son has broke his arrowsSwears he will shoot no more but play with sparrowsAnd be a boy right out

wantonness before the actual marriage ceremony but did not succeed Venus has returned her irritable son has broken his arrows and swears that he will give up his practice of trying to inspire love but play with sparrows and be a boy again

[Music is heard] [Music is heard]

CERESHighst queen of stateGreat Juno comes I know her by her gait

CERESHighest queen of state Great Juno there she comes I know here by her gait

[Enter Juno] [Enter Juno]

JUNOHow does my bounteous sister Go with meTo bless this twain that they may Prosperous beAnd honoured in their issue

JUNOHow are you doing my generous sister Come with me to bless this couple so that they may be prosperous and fortunate in their children

[They sing] [They sing]

JUNOHonour riches marriage-blessingLong continuance and increasingHourly joys be still upon youJuno sings her blessings upon you

JUNOMay honour riches happiness in marriage long continuance and increase of those boons ever rest upon you as hourly joys Juno showers down upon you her blessings in song

CERESEarths increase foison plentyBarns and garners never emptyVines and clustring bunches growingPlants and goodly burden bowingSpring come to you at the farthestIn the very end of harvestScarcity and want shall shun youCeresrsquo blessing so is on you

CERESMay you have the plenty of earthrsquos produce Your barns and granaries may never be empty Your vines may grow with clustering bunches Your fruit trees may be heavily laden with their fruit May there be continuous spring and harvest May scantiness and want leave you forever Such is the blessing of Ceres upon you

FERDINANDThis is a most majestic vision andHarmoniously charmingly May I be boldTo think these spirits

FERDINANDThis is a great vision and magically melodious Should I suppose the characters (taking part in the masque) are spirits

PROSPEROSpirits which by mine artI have from their confines calld to enactMy present fancies

PROSPEROYes they are spirits whom I have summoned from the regions to which they are confined to carry into effect my fanciful designs

ORIGINAL TEXT PARAPHRASEFERDINANDLet me live here everSo rare a wondered father and a wifeMakes this place paradise

FERDINANDI should like to live here forever Such a wise and wonderful father makes this place a paradise

[Juno and Ceres whisper and send Iris on employment] [Juno and Ceres whisper and send Iris on employment]

PROSPEROSweet now silence

PROSPEROMy dear Ferdinand speak no more Juno and Ceres are

Juno and Ceres whisper seriouslyTheres something else to do Hush and be muteOr else our spell is marred

whispering with a solemn look There is something else coming Silence Or else our magic will be spoilt

IRISYour nymphs called naiads of the wandering brooksWith your sedged crowns and over-harmless looksLeave your crisp channels and on this green landAnswer your summons Juno does commandCome temperate nymphs and help to celebrateA contract of true love Be not too late

IRISYou nymphs called Naiads denizens (M inhabitants) of the running stream with your chaplets of sedge and ever-helpful looks leave your wrinkled channels and on the green land answer the summons sent to you Juno has ordered some chaste nymphs and help to celebrate a noble and true marriage Donrsquot delay

[Enter certain nymphs] [Enter certain nymphs]You sunburnt sicklemen of August wearyCome hither from the furrow and be merryMake holiday your rye-straw hats put onAnd these fresh nymphs encounter every oneIn country footing

You sunburnt harvesters weary from the effects of the heat in August come here from the furrowed land and rejoice Make holiday with your rye-straw hats upon you and meet these fresh nymphs and join in country dancing

[Enter certain reapers properly habited They join with the nymphs in a graceful dance towards the end whereof Prospero starts suddenly and speaks]

[Enter certain reapers properly habited They join with the nymphs in a graceful dance towards the end whereof Prospero starts suddenly and speaks]

PROSPERO[aside] I had forgot that foul conspiracyOf the beast Caliban and his confederatesAgainst my life The minute of their plotIs almost come [to the spirits]Well done Avoidno more

PROSPERO(Aside)I had forgotten the wicked conspiracy of the beast Caliban and his accomplices against my life the time of their plot has almost arrived ndash (To the Spirits) well done depart no more of this

[To a strange hollow and confused noise the spirits heavily vanish]

[The spirits depart]

ORIGINAL TEXT PARAPHRASEFERDINANDLet me live here everSo rare a wondered father and a wifeMakes this place paradise

FERDINANDI should like to live here forever Such a wise and wonderful father makes this place a paradise

[Juno and Ceres whisper and send Iris on employment] [Juno and Ceres whisper and send Iris on employment]

PROSPEROSweet now silenceJuno and Ceres whisper seriouslyTheres something else to do Hush and be muteOr else our spell is marred

PROSPEROMy dear Ferdinand speak no more Juno and Ceres are whispering with a solemn look There is something else coming Silence Or else our magic will be spoilt

IRISYour nymphs called naiads of the wandering brooksWith your sedged crowns and over-harmless looksLeave your crisp channels and on this green landAnswer your summons Juno does commandCome temperate nymphs and help to celebrateA contract of true love Be not too late

IRISYou nymphs called Naiads denizens (M inhabitants) of the running stream with your chaplets of sedge and ever-helpful looks leave your wrinkled channels and on the green land answer the summons sent to you Juno has ordered some chaste nymphs and help to celebrate a noble and true marriage Donrsquot delay

[Enter certain nymphs] [Enter certain nymphs]You sunburnt sicklemen of August wearyCome hither from the furrow and be merryMake holiday your rye-straw hats put onAnd these fresh nymphs encounter every oneIn country footing

You sunburnt harvesters weary from the effects of the heat in August come here from the furrowed land and rejoice Make holiday with your rye-straw hats upon you and meet these fresh nymphs and join in country dancing

[Enter certain reapers properly habited They join with the nymphs in a graceful dance towards the end whereof Prospero starts suddenly and speaks]

[Enter certain reapers properly habited They join with the nymphs in a graceful dance towards the end whereof Prospero starts suddenly and speaks]

PROSPERO[aside] I had forgot that foul conspiracyOf the beast Caliban and his confederatesAgainst my life The minute of their plotIs almost come [to the spirits]Well done Avoidno more

PROSPERO(Aside)I had forgotten the wicked conspiracy of the beast Caliban and his accomplices against my life the time of their plot has almost arrived ndash (To the Spirits) well done depart no more of this

[To a strange hollow and confused noise the spirits heavily vanish]

[The spirits depart]

Ac-12 27420 topic Revaluation of Assets and Liabilities

REVALUATION OF ASSETS AND LIABILITIES

On admission of a new partner the firm stands reconstituted and consequently the assets are revalued and liabilities are reassessed It is necessary to show the true position of the firm at the time of admission of a new partner If the values of the assets are raised gain will increase the capital of the existing partners Similarly any decrease in the value of assets ie loss will decrease the capital of the existing partners For this purpose alsquoRevaluation Accountrsquo is prepared This account is credited with all increases in the value of assets and decrease in the value of liabilities It is debited with decrease on account of value of assets and increase in the value of liabilities The balance of this account shows a gain or loss on revaluation which is transferred to the existing partnerrsquos capital account in existing profit sharing ratioAccounting for Revaluation of Assets and Liabilities when there is a Changein the Profit Sharing Ratio of Existing PartnersAssets and liabilities of a firm must also be revalued at the time of change in profit sharing ratio of existing partners The reason is that the realisable or actual value of assets and liabilities may be different from those shown in the Balance Sheet It is possible that with the passage of time some of the assets might have appreciated in value while the value of certain other assets might have decreased and no record has been made of such changes in the books of accounts Similarly there may be some unrecorded assets amp libilities that may have to be accounted for Revaluation of assets and reassessments of liabilities becomes necessary because the change in the

value of assets and liabilities belongs to the period to change in profit sharing ratio and hence must be shared by the partners in their old profit sharing ratio Revaluation of assets and reassessment of liabilities may be given effect to in two different ways (a) When revised values are to be recorded in the books and(b) When revised values are not to be recorded in the books

When revised values are to be recorded in the booksIn such a case revaluation of assets and reassessment of liabilities is done with the help of a new account called lsquoRevaluation Accountrsquo Sometimes this account is also called as lsquoProfit amp Loss Adjustment Acrsquo If there is a loss due to revaluation revaluation account is debited and if the revaluation results in a profit the revaluation account is credited The following journal entries made for this purpose are

(i) For increase in the value of assetsAsset Ac Dr (individually)To Revaluation Ac(ii) For decrease in the value of AssetRevaluation Ac Dr (individually)To Asset Ac[Decrease in the value of assets](iii) For increase in the value of LiabilitiesRevaluation Ac Dr (individually)To Liabilities Ac[Increase in the value of Liabilities](iv) For decrease in the value of LiabilitiesLiabilities Ac DrTo Revaluation Ac[Decrease in the value of Liabilities](v) For unrecorded AssetsAsset Ac [unrecorded] DrTo Revaluation Ac[Unrecorded asset recorded at actual value](vi) For unrecorded Liability Revaluation Ac DrTo Liability Ac [unrecorded][Unrecorded Liability recorded at actual value](vii) For transfer of gain on revaluationRevaluation Ac DrTo Existing Partnerrsquos CapitalCurrent Ac[Profit on revaluation transferred to capital account in existing ratio](viii) For transfer of loss on revaluationExisting Partnerrsquos CapitalCurrent Ac DrTo Revaluation Ac[Loss on revaluation transferred to capital account in existing ratio](a) When revaluation account shows gain Revaluation Ac DrTo Partnerrsquos Capital Ac (Old Profit Sharing Ratio)(Profit on revaluation credited to Partnerrsquos Capital Ac)(b) Above entry is reversed when revaluation account shows loss Partners Capital Acs (Old Profit Sharing Ratio) DrTo Revaluation Ac(Loss on revaluation debited to Partnerrsquos Capital Acs)

Proforma of Revaluation Account is given as under

Revaluation Account

Dr Cr Particulars ` Amount Particulars ` Amount To Decrease in value of assets By Increase in value of assets To Increase in value of liabilities By Decrease in value of liabilities To Unrecorded liabilities By Unrecorded assets To Gain on Revaluation (Transferred) By Loss on Revalution (Transferred)

ECO ndash12 2742020Topic- ELASTICITY OF DEMAND

CHAPTER - ELASTICITY OF DEMANDMEANINGDemand for a commodity is affected by many factors such as its price price of related goods income of its buyer tastes and preferences etc Elasticity means degree of response Elasticity of demand means degree of responsiveness of demand Demand for a commodity responds to change in price price of related goods income etc So we have three dimensions of elasticity of demandDIMENSION OF ELASTICITY OF DEMAND TYPES OF ELASTICITY OF DEMAND

Price elasticity of demand Income elasticity of demand Cross Elasticity of demand

Price elasticity of demand Price elasticity of demand means degree of responsiveness of demand for a commodity to the change in its price For example if demand for a commodity rises by 10 due to 5 fall in its price Price elasticity of demand (ep)=Percentage change in quantity demanded Percentage change in price of the commodity = 10 ( -)5 = ( - )2Note that ep will always be negative due to inverse relationship of price and quantity demanded

(ii) Income elasticity of demand Income elasticity of demand refers to the degree of responsiveness of demand for a commodity to the change in income of its buyer Suppose income of buyer rises by 10 and his demand for a commodity rises by 20 then Income elasticity of demand (ey)= change in quantity demanded change in price of the commodity =20 10 = 2

Cross Elasticity of demandCross elasticity of demand means the degree of responsiveness of demand for a commodity to the change in price of its related goods (substitute goods or complementary goods) Suppose demand for a commodity rises by 10 due to 5 rise in price of its substitute good then Cross elasticity of demand (ec) = change in quantity demanded change in price of related good = 10 2 = 5 (Tastes and preferences cannot be expressed numerically So elasticity ofdemand cannot be numerically expressed)

  • Chapter 1 Force (Summary)
  • Distinguish between external sovereignty and internal sovereignty
    • NAND Gate
      • Logic diagram
      • Truth Table
        • NOR Gate
          • Logic diagram
          • Truth Table
            • XOR Gate
              • Logic diagram
              • Truth Table
                • XNOR Gate
                  • Logic diagram
                  • Truth Table
                      • Physics
                      • Chapter 1 Electric Field ( Electric Dipole) (Summary)
Page 3:  · Web viewWe all know that Nouns are divided into two parts: common noun and proper noun.Apart from common and proper noun, we will also study about collective noun and compound

Class II

Subject ndash English language Topic ndash ch 1 NounsDefinition of NounNouns are naming words that tells us the names of people place animal and things For example- boy girl dogcat park zoo basketapple etc In this chapter we learn two types of nouns-

Common noun Proper noun

Common noun ndashNames of people place animal things in general are called common noun Examples ndash

1 I love to play in the park2 The sky is blue3 Cow gives us milk4 The policeman catches the thieves

Proper noun ndashSpecial names given to people placesanimals and things are called Proper nouns It always starts with a capital letter Proper Noun are names of-

Person- Tom Harry Countrycitystates- IndiaKolkataWest Bengal Daysmonths- Monday May BooksfestivalsThe Bible Holi Mountainrivers- Mt Everest the Ganga

Examples- 1 Mr Das is an old man2 Taj Mahal is in Agra3 I live my pet dog Mac verymuch4 My uncle gifted me Titan watch

Difference between ndashCommon noun

1 girl2 car3 park4 aeroplane5 bag

Proper noun1 Devika 2 Maruti3 Nicco Park4 Air India5 Skybag

Subject ndash EVS Topic ndash My FamilyFamily is a group of people living together in the same house and is related to each other There are two main kinds of family-1 Nuclear family- In this family only father mother and their children live together in the same houseIt is also

known as a small or nuclear family2 Joint family ndash In this family grandfather grandmother father mother their children uncles aunt s and

cousins all live together in the same house It is also known as a big or joint familyMembers of the familyGrandfather ndash Father of your father or motherGrandmother- Mother of your father or motherUncle ndash Brother of your father or motherAunt ndash Sister of your father or motherSibling- Your own brother or sisterCousins ndash The children of your uncle and auntMaternal grandparents- Parents of your motherPaternal grandparents- Parents of your father

All members of the family has a common last name that is called surname

______________________________________________________________________________________________

Class III

Subject ndash Eng language Topic ndash ch 3 NOUNS

NOUNS

Nouns are the words that names people place animal and things Around us we see so many things Everything that we see has some name Example- computer blackboard chalk book teacher house cat dog etc

In this chapter we will learn about ndash

Common NounProper NounCollective noun

Common Noun

Common Noun are names of people place animal things in general For example girl boy hospital park ball car etc

Proper noun

Proper nouns are the specific or particular name of person place animal and thingsExamples- Mina Mt Everest Delhi Tommy etc Proper noun is always capitalized

Uses of Proper nouns- They are special names of

people such as RitaJackRam etc Countries citiesstates such as IndiaKolkataWest Bengal Mountains rivers seas lakes such as Mt Everest the Ganga the Indian Ocean Dal

Lake

Subject social studies

Topic My family and I

A family is a group of people who are related to each other by birth Members of a family always give love and support to each other

There are different kinds of families ndash nuclear family joint family and single parent family

Nuclear family means where father mother and kids stay together

Joint family means where many members of the family stay together For example grandfather grandmother father mother uncle aunt cousins live together

Single parent family means where the kids stay with any one of the parent like father or mother

It is always a fun when whole family spends time together example going to a picnic because it creates a strong bond among each other

Class IV

Subject English language Topic Nouns Compound nouns

We all know that Nouns are divided into two parts common noun and proper nounApart from common and proper noun we will also study about collective noun and compound nouns A common or proper noun that is the name of a collection or group of similar things or persons is known as a collective Noun Example crowd committee herd United Nations etc Example Mother has lost her bunch of keysThe word bunch is a collective noun

1 A herd of cows was grazing in the fieldHerd- collective noun

2 All living creatures need food to surviveCreatures- collective noun

3 The Ganga is a river sacred to IndiansIndians- collective noun

Compound noun some common and proper nouns are made up of more than one word They are known as compound nouns or simply compounds Example newspaper (news+paper) ashtray (Ash+tray) plaything (play+thing) etc1 My penfriendlives in a town in South AfricaPenfriend South Africa- compound noun

2 My grandparents live in New DelhiGrandparents New Delhi- compound nouns

3 Please do not park your cars in the drivewayDriveway- compound noun

Subject Social studies Topic A peep into the past

The story of the past is called history It is a journey across time and space that takes us to another period of time where people lived differently It is believed that the way of living eating clothes art were all different from what we have today We need to study history becauseIt tells us about the important people of the pastIt helps us to gain knowledge about the past and the present and help us to understand themHistory guides us to set an aim and warns us not to repeat the same mistakesIt helps us to understand the development of mankindIt helps to developessential skills to become a better person

Division of the pastHistory is so vast that it is divided into two parts ndash BC (Before Christ) and AD (Anna Domini) This helps us to understand about the past clearly Indian history and itrsquos civilisation are the worldrsquos oldest civilisation Indian history is divided into ancient medieval and modernCivilisation mean the stage of human social and cultural development

Subject COMPUTER Chapter-2 (MS WINDOWS)WINDOWS IS AN OPERATING SYSTEM THAT HELPS THE USER TO OPERATE AND CONTROL THE OVERALL ACTIVITIES OF A COMPUTER SOME POPULAR VERSIONS OF WINDOWS ARE WINDOWS 98 WINDOWS 7 WINDOWS 8 AND WINDOWS 10

DESKTOP THE FIRST SCREEN THAT WE SEE WHEN THE OPERATING SYSTEM (WINDOWS)HAS FINISHED LOADING IS KNOWN AS DESKTOP

THE FEATURES OF DESKTOP ARE bullIT IS WHERE ALL THE ICONS OF THE DIFFERENT APPLICATIONS ARE DISPLAYEDbullIT IS THE HOME FOR ALL SHORTCUTS PROGRAMS CAN BE QUICKLY OPENED BY CLICKING ON THE RESPECTIVE SHORTCUTSbullIT HOLDS VARIOUS INFORMATION SUCH AS TIME AND DATE

ICONSICONS ARE SMALL PICTURES OF DIFFERENT APPLICATIONS THAT ARE PRESENT AND INSTALLED IN THE COMPUTER FOR EXAMPLE COMPUTER ICON AND RECYCLE BIN ICON

DESKTOP GADGETS A DESKTOP GADGET IS A SMALL APPLICATION OR A WIDGET THAT RECIDES ON A COMPUTERS DESKTOP FOR EXAMPLE CLOCK CALENDAR ETC_______________________________________________________________________________________________

Class VSubject Socialal studiesTopic Evolution of man

Millions of years ago the earth looked much different as it is seen todayIt was totally different and it was very difficult to live Everything looked very different plants animals and humans everything looked very different from what they are now All the changes took place gradually with the changing environment Earliest ancestors of the humans were known as the Ramapithecus who lived about 14 million years ago They looked like ape that walked upright and weighed about 14kg With the passage of time there was a gradual development in the structure posture features and teeth of the early humansIt is said that the apesthat lived on treetops started living on the ground and stood on two limbs are called hominids They used their limbs to hold things and make tools Finally they changed to homosapiens mean wise human We humans belong to this group Life of early humansThe early men were hunter ndash gatherers They moved from place to place in search of food water and shelter They hunted animals and ate them raw gathered fruits nuts roots and seeds for eating They didnrsquot know how to build houses so they used live on treetops or in the caves or rock shelters In summers they remained uncovered but in winter they used to cover themselves by using bark of trees leaves and animal skins

Subject English language Topic Adjectives Interrogative and Emphasizing

Interrogative adjectiveAn interrogative adjective is used with a noun that it qualifies to ask a question The word interrogative mean to ask questions Example Which part of the world do you come fromThe word lsquoWhich is an interrogative adjectivesEmphasizing adjectivesSometimes we use adjectives to emphasize the effect of what we say Words which are used to emphasize a nounExample Rita came late and had to stand at the very end of the lineThe word Veryis an emphasizing adjective It actually qualifies or emphasizethe noun end

1lsquoDo not listen to what Ravi says for he is a complete foolrsquo said RohanComplete- emphasizing adjective

2Kiran is a great supporter of non violenceGreat- emphasizing adjective

3rsquoWhat crime has the prisoners committedrsquo asked the judgeWhat- interrogative adjective

4Whose book is that lying on the tableWhose- interrogative adjective

Subjects COMPUTER CHAPTER-1 (CHARACTERISTICS OF A COMPUTER)A COMPUTER SYSTEM IS A ONE THAT CONSISTS OF ALL THE SOFTWARES HARDWARES CPU AND MEMORY DEVICES SOME IMPORTANT TERMSTHE COMPUTER WORKS ACCORDING TO THE COMMANDS OR INSTRUCTIONS THAT WE GIVE

A SET OF INSTRUCTIONS IS CALLED THE PROGRAM A SET OF ONE OR MORE PROGRAMS IS CALLED A SOFTWARE THE PHYSICAL COMPONENTS OF A COMPUTER ARE CALLED HARDWARE

FOUR OPERATIONS OF A COMPUTER SYSTEM INPUTINPUT IS SUPPLIED TO THE COMPUTER WITH THE USE OF A KEYBOARD A MOUSE A MICROPHONE

OR OTHER INPUT DEVICES PROCESSING PROCESSING IS DONE INSIDE THE COMPUTER IN THE CPU PROCESSING IS THE CONVERSION

OF THE INPUT INTO THE OUTPUT STORAGE STORAGE REFERS TO THE SAVING OF INFORMATION FOR LATER USE INFORMATION IS STORED

IN THE MEMORY OF A COMPUTER THERE ARE TWO TYPES OF COMPUTER MEMORY PRIMARY MEMORY AND SECONDARY MEMORYPRIMARY MEMORY IS FAST SMALL AND EXPENSIVE THE TWO TYPES OF PRIMARY MEMORY ARE RANDOM ACCESS MEMORY(RAM) AND READ ONLY MEMORY (ROM) RANDOM ACCESS MEMORY (RAM)THIS MEMORY IS VOLATILE IN NATURE WHICH MEANS THAT THE

INFORMATION STORED IN IT IS AUTOMATICALLY ERASED WHEN THE COMPUTER POWER IS TURNED OFF

READ ONLY MEMORY (ROM) THIS TYPE OF MEMORY IS NON VOLATILE IN NATURE WHICH MEANS THAT DATA IS NOT LOST WHEN THE COMPUTER POWER IS TURNED OFF

OUTPUT OUTPUT IS THE RESULT PRODUCED BY A COMPUTEROUTPUT MAY BE VIEWED ON THE MONITOR SCREEN HEARD THROUGH SPEAKERS PRINTED THROUGH PRINTER ON PAPERS OR STORED IN THE MEMORY FOR LATER USE

Class VI

GEOGRAPHY CHAPTER 1 REPRESENTATION OF GEOGRAPHICAL FEATURES

GLOBE AND MAPS

GLOBE- A Globe is a three ndashdimensional model of the Earth

EQUATOR ndash An imaginary line runs through the centre of the earth from east to west

NORTHERN HEMISPHERE ndash the half of the north of the equator is called Northern Hemisphere

SOUTHERN HEMISPHERE ndash the half of the south of the equator is called Southern Hemisphere

LATITUDE - Imaginary parallel lines running on the globe from east to west

LONGITUDE ndash The imaginary lines semicircle in nature running from the North Pole to South Pole Also called meridian

MAP ndash Representation of the Earth as a whole or parts of it drawn on a flat surface to a scale

SKETCH ndash A rough drawing of a place not drawn to a scale

PLAN ndash A plan is prepared to show more details about a small area

COMPUTER - VI CHAPTER-6 (INTRODUCTION TO WINDOWS 10)

THE MOST RECENT VERSION OF WINDOWS IS WINDOWS 10 IT WAS RELEASED IN 2015

WINDOWS 10 IS DESIGNED TO WORK FOR YOU AND NOT YOU WORKING FOR IT THE WINDOWS 10 OPERATING SYSTEM INCLUDES A NUMBER OF NEW FEATURES LIKE IMPROVED PERFORMANCE ON MULTI CORE PROCESSORS IMPROVED G U I DATA SECURITY AND BETTER BOOTING PERFORMANCE

WINDOWS 10 HAS 1187 WITH WEARING FEATURES SETS AND INTENDED HARDWARE

THE VARIOUS EDITIONS AVAILABLE FOR WINDOWS 10 ARE AS FOLLOWS

WINDOWS 10 HOME

WINDOWS 10 HOME IS THE EDITION OF WINDOWS 10 THAT CONTAINS THE LEAST NUMBER OF FEATURESWINDOWS 10 HOME IS THE CONSUMER FOCUSED DESKTOP EDITION WITH SUPPORT FOR BOTH PC AND TOUCH ENABLED TABLETS WINDOWS 10 HOME IS AVAILABLE IN BOTH 32 BIT AND 64 BIT VERSIONS

WINDOWS 10 PRO

WINDOWS 10 PRO HAS FEATURES ESSENTIAL FOR SMALL OR MEDIUM BUSINESSESIT HAS A LARGE NUMBER OF EXTRA FEATURES TO MEET THE NEEDS OF SMALL BUSINESSES

WINDOWS 10 ENTERPRISE

WINDOWS 10 ENTERPRISE IS BUILT ON WINDOWS 10 PROIT HAS FEATURES WHICH ARE DESIGNED TO MEET THE NEEDS OF MEDIUM AND LARGE ORGANISATIONSTHIS WINDOWS EDITION TARGETS THE ENTERPRISE SEGMENT OF THE MARKET

WINDOWS 10 MOBILE

WINDOWS 10 MOBILE IS DESIGNED FOR CUSTOMER ORGANISATION THAT ARE USING THE WINDOWS 10 PLATFORM ON SMARTPHONES AND SMALL TABLETS

WINDOWS 10 EDUCATION

WINDOWS 10 EDUCATION IS DESIGNED TO MEET THE NEEDS OF STAFF ADMINISTRATORS TEACHERS AND STUDENTS OF SCHOOLS

WINDOWS 10T EDITION

WINDOWS 10T EDITION OF WINDOWS 10 IS DESIGNED FOR USE IN SMALL FOOTPRINT AND LOW COST DEVICES

Subject English language

Topic Noun kinds

NOUN KINDS

Noun is a naming word Nouns are names of people places animals or things

For example Ashley Mumbai tiger pencils

Kinds of nouns

Proper noun the name of a particular person place or thing is called proper nounExample

1 The Himalayas stand to the north of India2 Monika is the school captain

Common noun announce that names people place or thing in general is called a common noun Example

1 Kalidas was the greatest dramatist of India2 William Shakespeare is Englands national poet

Collective Noun A collective noun is the name of a collection of people or things taken together and spoken of as a whole Example

1 The feet were completely destroyed in the fierce attack2 The crew revolted against Captain Grand

Abstract Noun An abstract noun is the name of some quality state or idea Example1 Diversity Indias biggest strength2 Patience is a virtue

_____________________________________________________________________________________

Class VII

Subject English 1 Topic Articles

ARTICLES

There are three articles ------ A An The

1 Indefinite articles- The Articles a and an are called Indefinite Articles They do not point to a particular person or thing They are used with singular countable nouns It does not identify a specific noun

Uses of Indefinite articles A

Before a singular noun beginning with a consonant or a vowel with a consonant sound For example a cattle a university a one way track

Before a proper noun which is either unfamiliar or holds a special meaning For example For example A Mahesh Sharma wants to see youNeela wants to be a Tendulkar when she grows up

With a number beginning with a consonant sound For example a ten- rupee note a hundred years

Before half when half follows a whole number or after half when it isnrsquotExample one and a half litres (before) Half a litre (after)

With an expression of quantity For example a lot of time a dozen oranges a little while In exclamation before nouns Example Such a mess What a pity

Uses of Indefinite articles An

Before a singular noun beginning with a vowel sound Example an ant an egg an umbrella Before an abbreviation beginning with a vowel or a consonant with the vowel sound

Example an MP an MA Before the word beginning with a silent h Example an heir an hour an honour

Note We do not say a milk or a lemonade because they are uncountable nouns ever we see a cup of milk or a glass of lemon and

Subject GEOGRAPHY

CHAPTER 1 REPRESENTATION OF GEOGRAPHICAL FEATURES

TOPIC- TOPOGRAPHICAL MAPS

Topographical Maps- topographical maps are small-scale maps with detailed depiction of both natural and human-made features

Conventional colours-different colours used in a map are known as conventional colours

TYPES OF SCALE-

Verbal Statement- Scale written in a statement from like 2 cm to a Km is called verbal statement It means that 2cm on the map represents 1 km on the Earth

Representative Fraction- Scale can also be written as a representative fraction (RF)

Example 150000 it means 1cm on the map represents 50000cm or frac12 km on the ground It shows the ratio between map distance and ground distance

Linear Scale Scale can be drawn on a line to show map distance equivalent to ground distance A line is divided into equal parts Each parts represents the actual distance on the ground in mkm

SUBJECT-COMPUTER

CHAPTER-1 (COMPUTER FUNDAMENTALS)

COMPUTER LANGUAGES

THE THE TERM COMPUTER LANGUAGE REFERS TO A SYSTEM OF RULES AND SYMBOLS THAT ARE DESIGNED TO GIVE INSTRUCTIONS TO A COMPUTERTHE COMPONENTS OF A COMPUTER SYSTEM CANNOT PERFORM A TASK BY THEMSELVES THEREFORE THEY HAVE TO BE GIVEN INSTRUCTIONS TO PERFORM ANY TASKCOMPUTER LANGUAGES ARE USED TO CREATE PROGRAMS USING LOGIC BASED ALGORITHMSTHE RULES OF A COMPUTER LANGUAGE IS

KNOWN AS SYNTAX WHEREAS THE TERM SEMANTICS REFERS TO THE MEANING OF LANGUAGESCOMPUTER CANNOT DEVELOP ANY PROGRAMMING LANGUAGE BY THEMSELVES AND CANNOT THINK INTELLIGENTLY UNLESS THEY ARE ASSISTED BY THE HUMAN BEINGS THEREFORE THE PROGRAMMERS DEVELOP A SET OF METHODS AND TECHNIQUES A SET OF INSTRUCTIONS USED TO PERFORM A SPECIFIC TASK IS CALLED A PROGRAM

A PROGRAMMING LANGUAGE IS ALSO KNOWN AS A COMPUTER LANGUAGE CODED BY PROGRAMMERS TO WRITE INSTRUCTIONS FOR A COMPUTERTHE COMPUTER TAKES THESE INSTRUCTIONS AS INPUT AND PRODUCES THE DESIRED OUTPUT

TYPES OF COMPUTER LANGUAGE

THE COMPUTER LANGUAGE CAN BE BROADLY DIVIDED INTO TWO CATEGORIES

LOW LEVEL LANGUAGES HIGH LEVEL LANGUAGES

LOW LEVEL LANGUAGES

A LOW LEVEL PROGRAMMING LANGUAGE REFERS TO THE LANGUAGE THAT IS UNDERSTOOD BY A COMPUTER DIRECTLYTHE PROGRAMMER MUST HAVE AN IN-DEPTH KNOWLEDGE OF DIFFERENT COMPUTERS TO WRITE PROGRAMS IN A LOW LEVEL LANGUAGE THE TWO TYPES OF LOW LEVEL LANGUAGES ARE MACHINE LANGUAGE AND ASSEMBLY LANGUAGE

MACHINE LANGUAGE

COMPUTER HARDWARE UNDERSTANDS ONLY MACHINE LANGUAGE AS IT IS THE FUNDAMENTAL LANGUAGE OF A COMPUTER WHICH UNDERSTANDS ONLY THE TWO DIGITS 0 AND 1 KNOWN AS THE BINARY DIGITSIT IS A SYSTEM OF INSTRUCTIONS EXECUTED DIRECTLY BY THE CPU WITHOUT ANY TRANSLATION AND HENCE IS EXECUTED QUICKLY AS COMPARED TO PROGRAMS IN OTHER LANGUAGES PROGRAMS IN MACHINE LANGUAGE NEED DIFFERENT BINARY LANGUAGE PROGRAMS TO SOLVE THE COMPLEX TASKSPROGRAMMERS OFTEN FIND IT DIFFICULT TO WRITE PROGRAMS IN THIS LANGUAGE

ASSEMBLY LANGUAGE

AN ASSEMBLY LANGUAGE IS A LOW LEVEL PROGRAMMING LANGUAGE BUT THE INSTRUCTIONS ARE WRITTEN IN THE FORM OF WORDS KNOWN AS MNEMONICS TO CONVERT AN ASSEMBLY LANGUAGE INTO TO MACHINE CODE WE REQUIRE A UTILITY PROGRAM WHICH IS KNOWN AS AN ASSEMBLER ASSEMBLY LANGUAGE PROGRAMS CANNOT BE EXECUTED DIRECTLY BY A COMPUTER THEREFORE THEY ARE SLOWER THAN THE MACHINE LANGUAGE PROGRAMSIT IS EASIER TO WRITE PROGRAMS IN THE MACHINE LANGUAGE THEN AND IN ASSEMBLY LANGUAGE

_______________________________________________________________________________________________

CLASS-VIII

SUBJECT-COMPUTER CHAPTER - Operating system and graphical user interface Role and functions

Need of operating system

It co-ordinates different hardware and software componentsof a computer system It supervises the various actions of the computer system and enables the computer to work in a effective

manner It helps in smooth functioning of various peripherals

In a multi-tasking operating system it determines the order and time to be allowed for each application before giving another application a turn

It sends messages to the system operator about the status of operation or any error that may have occurred while running the application

What is operating system

An operating system is an integrated system of programs that manages various resources and the overall operation of the computer system It is designed to support various activities of computer system in a systematic way

Role of an operating system-

Operating system enables the user to use the system effectively An operating system manages various application that runs on a computer and shares computerrsquos resources User interacts with operating system through command line interface and graphical user interface

Function of an operating system -

Booting the computer it is the start up procedure of a computer system Loading theprograms in the memory - when system is ready the operating system loads certain program

automatically Manages resources it manages between the hardware and software resources Detecting and correcting errors- if the supporting hardware or software doesnrsquot works properly then the

operatingsystem tries to rectify it Ensuring data security programs and data donot interfere with each other Maintaining the internal clock of system - maintain internal clock of system when system is shut down

SUBJECT-GEOGRAPHY CHAPTER 1 REPRESENTATION OF GEOGRAPHICAL FEATURES THROUGH CONTOURSTopographical maps- topographical maps are small-scale maps with detailed depiction of both natural and human-made features

Contour lines- contour lines are imaginary lines drawn on a map joining places having the same height above the mean sea level

Contour linesContour interval- It is an interval at which contour lines are drawnIndex contour- At every 100-m interval a thick brown contour line is called index contour It is drawn for calculating height

Spot heights- The spot heights show heights in metres above the mean sea level

SUBJECT- English Language CHAPTER - The Sentence

Complex sentence-

We have seen that a Complex Sentence consists of a Principal Clause with one or more Subordinate Clauses

We have also learnt that there are three kinds of Subordinate Clauses The Adjective Clause the Adverb Clause and the Noun Clause

1) THE ADJECTIVE CLAUSE

An ADJECTIVE CLAUSE does the work of an ADJECTIVE It qualifies a noun or pronoun In the following examples in each set two commonly patterned sentences are compared one with an ADJECTIVE and the other with an ADJECTIVE CLAUSE ( The Adjective or Adjective Clause qualifies the Noun placed in the box)

a) He is a lazy boy (Adjective)

b) He is a boy who is lazy (Adjective Clause)

a) It is a beautiful place for the tourists (Adjective)

b) It is a place where tourists come for the scenic beauty (Adjective Clause)

a) We have enough funds for the work (Adjective)

b) We have funds which would be enough for the work (Adjective Clause)

Convert the sentence from simple to complex sentence-

1) We believe his honesty Ans- We believe that he is honest

2) This is the birth place of RamaAns- This is the place where Rama was born

____________________________________________________________________________________________

Class IX

Subject English Language

Topic Preposition

Date 270420

PREPOSITIONS

A preposition is a word placed before a noun or a pronoun to show in what relation the person or thing denoted by it stands in regard to something else

EXAMPLE

There is a clock on the wall

The preposition on shoes relationship between clock and the wall

Smitha is afraid of lizards The man jumped off the bus

Here the preposition of shows the relationship between afraid and lizards

The preposition off shows the relation between jumped and bus

The noun or the pronoun which follows a preposition is called its object

So in the first Example wall is the object of the preposition on

A preposition can have more than one object For example

The plane flew overhouses and meadows

Prepositions are used to express a number of relationship including time locationmannermeans quantity purpose and state or condition

Points to remember

Preposition joins a noun to another noun or a pronounThere is a cow in the field

A preposition joints a noun to a verbThe cat runs after the rat

Preposition can have two or more than two objectsThe road runs over hills and plain

A preposition also joins a noun an adjectiveHe is fond of tea

Generally a preposition comes before an object Sometimes it comes even after an object asWhat are you looking atThis is the house I live in

Subject- Computer Application

Chapter 2 Introduction to Java

Java API An application programming interface (API) in the context of Java is a collection of prewritten packages classes and interfaces with their respective methods fields and constructors

Byte Code Java bytecode is the result of the compilation of a Java program an intermediate representation of that program which is machine independent The Java bytecode gets processed by the Java virtual machine (JVM) instead of the processor JVM The Java Virtual Machine (JVM) is the runtime engine of the Java Platform which allows any program written in Java or other language compiled into Java bytecode to run on any computer that has a native JVM

Platform A platform is the hardware or software environment in which a program remains

Java platform The Java platform differs from most other platforms in that itrsquos a software only platform that runs on top of other hardware-based platforms

Applet and Application The fundamental difference between the two Java programs is that an application program is designed to run on a stand-alone machine whereas an applet is a web-version of an application which is used to run a program on a web browser

WORA ldquoWrite once run anywhererdquo (WORA) or sometimes write once run everywhere (WORE) is a slogan created by Sun Microsystems to illustrate the cross-platform benefits of the Java language

Class XSubject Topic Summary Execution

ECONOMICSFACTORS OF PRODUCTION Sub-topiclsquoLABOURrsquo

We shall start our class by discussing the topic taught in the last class

lsquo DIVISION OF LABOURrsquo- By division of labour we mean specialization in workIt refers to splitting up the work of labour involved in the production of a particular commodity into several parts and each part and sub-part is performed by a specialist

Now let us start by the Advantages of Division of labouraIt increases the level

Questions

1 Differentiate between Product-Based division of labour and Process-Based division of labour

Product-Based division of labour

Process-based division of labour

It is also known as simple (or occupational) division of labourUnder it everybody performs a particular occupations The entire is done by the same person

When a person or group of persons undertakes a specialised function which is supplementary to the production of final commodity and service This is also as complex division of labour

It is simple It is complex

of productionbSince the product is produced by an expert workerbest quality of product is producedcIt saves time and toolsd it promotes inventions in the methods and techniques of productioneIt leads to reduction in costs fAll workers get work according to their abilities and choices

Now let us discuss the disadvantages of Division of labour

a Since many workers are involved in the production of a commodity no one has the sense of responsibility

b The constant and repetition of the same work again and again make the work monotonous

c Division of labour facilitates production on large scale Hencethere is fear of over production

d Because of territorial division of labour some areasregions become more developed than others

It is based on labour-intensive techniques of production

It is based on capital-intensive techniques

It is generally found in small enterprises

It is generally found in large enterprises

Example Indian farmers doing all farm activities

Example A modern garmet factory where one person takes the measurementanother does the cuttingsome sew the clothes while a few workers button them and other iron them

2 What are the advantages of Division of labouraIt increases the level of productionbSince the product is produced by an expert workerbest quality of product is producedcIt saves time and toolsd it promotes inventions in the methods and techniques of productioneIt leads to reduction in costs fAll workers get work according to their abilities and choices

3Discuss the disadvantages of Division of laboura Since many workers are involved in the

production of a commodity no one has the sense of responsibility

b The constant and repetition of the same work again and again make the work monotonous

c Division of labour facilitates production on large scale Hence there is fear of over production

d Because of territorial division of labour some areasregions become more developed than others

English 1 Transformation of sentences

Sentences A sentence is a group of words which makes complete sense

a Assertive sentences

Exercise 6Rewrite the following sentences according to the instructions given below without changing their meanings

1 As soon as he saw the beer he jumped into

b Imperative sentences

c Interrogative sentences

d Exclamatory sentences

Sentences can be changed from one grammatical form to another without changing the meaning of the sentence This is known as transformation of sentences

the river ( Begin No sooner)2 None but brave deserve the fair (Begin the

bravehellip)3 This box is too heavy for me to lift ( Use so hellip

That instead of too)4 No one other than a king can live like James

Luxurious ( Begin only James)5 Oh for the wings of a dove (Begin I wishhellip)

Math Topic Commercial MathematicsChapter Shares and Dividends

Study item Discuss about shares and Dividends1) What is share

Ans To start any big business (company or Industry) a large sum of money is needed But it is not possible for an individual to invest such a large amount Then some persons interested in the business join together and from a company They divide the estimated money required into small parts Each such part is called a share

2) What do you mean by the term shareholder

Ans A person who purchases one or more shares is called shareholder3) Some terms related with a share

(i) Nominal value or face value or printed value The original value of a share is called its nominal value or face value or printed value

Note The nominal value of a share always remains same(ii) Market value or cash value The price of a share at any

time is called its market value or cash value

Note The market value of a share changes from time to time(iii) At par If the market value of a share is the same as its

nominal value the share is called at par(iv) At Premium or above Par If the market value of a share

is more than its nominal value the share is called at premium or above par

Example If a share of Rs 100 is selling at Rs 150 then it is said to be selling at a premium of Rs 50 or Rs 50 above par

(v) At Discount or below par If the market value of a share is less than its nominal value the share is called at discount or below par

Example If a share of Rs 100 is selling at Rs80 then it is said to be selling at a discount of Rs 20 or at Rs 20 below par

4) What is Dividend

Ans The profit which a shareholder gets for hisher investment from the company is called dividendNote (i) The dividend is always expressed as the percentage of the face value of the share(ii) The dividend is always given( by the company ) on the face value of the share

irrespective of the market value of the shareBENGALI(2ND LANGUAGE)

ldquoদেবতোর জণমrdquoলিবরোম চকরবত

পরথম লিসর পোঠ-চোর পসথ একটি পোথর লিবপলি ঘটোয় দেক যোতোয়োসতর পসথ পরলিতলিয়ত ওই পোথসর দেো োচট দেসত একলি দেতো দেক দেো োচট দেসয় দেবোমো সয় রোসতোর মোস লি0টসক পস1 লিবপরীত লিক দেথসক আো একটি দেমোটর োলি1 চোসকর কষতোয় পরোসরণ দেবোসচ যো লিকনত পরলিতবোর এমরণ দেৌভোয দেসকর োও সত পোসর তোই লিতলি দেকোো দেজোো1 কসর পোথরটিসক উপস1 দে8স পোথর উপস1 দে8োর ময় এক দেকৌতী জতো দেকসক পরশন কসর দেয লিতলি দেকোসো দেবতোর আস দেপসয়স0রণ লিকো লিকনত দেক বস লিতলি দেকোসো দেবতোর আস পোলি দেক উপলিত ক জতোর উসltসয বস কোরও ইস= স পোথরটি লিসয় দেযসত পোসর এর পর দেথসক দেক দেযসত আসত পোথরটি দেক দেসত পো একলি দেক কষয করস দেকউ পোথরটিসক ধসয়মস0 পলিরসকোর করস0 দেক ওই দেকৌতী জতোসক পোথসরর কোস0 বস থোকসত দেস এ0ো1ো আরও কষয কসর দেক ঠোৎ ওই লি1 পোথরটির োসয় লিোর োো-দেকউ পজো কসরস0 দেকৌতী দেোকটির আঙকো য় যলিসক উ পোথরটিসক লিরসয় দে8স তোর পর ঠোৎ একলি পোথরটির দেোোজ দেই দেক লিসয় দেস0 বো দেকোথোয় দেস0

বদোথ-

দেো োচট ndashচসত লিসয় দেকো লিক0র সE ধোককো দেস পস1 যোবোর উপকরমঅকসমোৎ- ঠোৎআতমমবর- লিসজসক লিয়নতরপ-পো লিপ0স প1োদেসতসসত- লিসপলিউৎোত- দেো1ো দেথসক উপস1 দে8োপরতযয় ndash লিবশবোপরতযোস- টতযোস ndash দৈবোসধসতোধলিসত- পরসপসরর পরলিত ব পরসয়ো করোপরসতরীভত- পোথসর পলিররণলিতবোনতঃকরসরণ- মস পরোসঅলিQৎ- অQো করসত ই=কদেোপ- বধমোর- দেবোস এমইতযোকোর- এইরকমরম- বময়পযসোভী-পসযর জয দেোভ আস0 যোরপোসথয় ঞচয়- পথচোর রচ জমোসোমোমোসরোস- ব ধম ধোসমর সE

তোর লি দেইhelliphellipTo be continued

Hindi 2nd lang सर क पद(सरदास)

सरदास शरी कषण भकति कावय क सरवशरषठ कगिरव ह इनक जनम और मतय क समय तथा सथान का मतभद हसरदास रवातसलय और शरार रस क अनयतम कगिरव ह इनक कावय म बालकषण क सौदय चपपल चषटा और गि7याओ की मनोहर झाकी मिमलती ह कषण और ोगिपयो क अननय परम का कतिचतरण ह सयो शरार की अपकषा उनक कावय म गिरवयो शरार का अमिBक गिरवषय और मारमिमEक कतिचतरण हआ हइन पकतियो म हम सरदास की भकति भारवनाओ का परिरचय मिमलता ह इनका सपण सगरह सरसार म गिनगिहत ह

1 जसोदा हरिर पालन झलारवहलरारवदलराईमलहारव रव जो ईसाई कछ ारव मर लाल को आई निनEदिदया काह ना आगिनसबारव त काह नाही बरवगिह आरवतोको कानहा बलारव

शबदाथ-हलरारव-गिहलती हदलराई - दलार पयार करती हमलहारव-पचकारती हनिनEदरिरया ndashनीदरवगिह-जलदी सअBर-होठमौन-चपसन-सकत

वयाखया- सरदास जी कहत ह गिक यशोदा माता बालक कषण को पालन म झल आती ह रवह उनह गिहलाती ह पयार करती ह मलहार जस कोई ीत ान लती ह और नीद स पछती ह गिक ह नीद तम मर लाल को आकर कयो नही सलाती तझ खाना बला रहा ह कभी कषण आख बद कर लत ह कभी आखफडफडान लत ह उनह सोता हआ जानकर यशोदा माता चप हो जाती ह और इशार म बात करन लती ह इसी बीच अकला कर कषण ज जात हतो गिफर यशोदा माता गिफर स ाना ान लती ह सरदास जी कहत ह गिक भरवान क दशन का सख दरवता और ऋगिष-मगिनयो को भी दलभ ह यही सख माता यशोदा को बडी सहजता स मिमल जा रही ह माता यशोदा बहत ही भागयशाली ह2)Continue to nexthellip

Physics

Chapter 1 Force

(Summary)

Question A body is acted upon by two forces each of magnitude F but in opposite directions State the effect of the forces if

(a) Both forces act at the same point of the body

(b)the two forces act at two different points of the body at a separation r

Solutions

(a) Resultant force acting on the body = 0

F ndash F = 0(b) The forces tend to rotate the body between two forces about the midpoint

Moment of forces = F times rFr

QuestionDefine moment of a couple Write its SI unit

Solutions

Moment of couple is equal to the product of both force and the perpendicular distance between the two forces

The SI unit of moment of couple is NmCommercial Studies

Advertising and sales

Business firms use several methods to

Questions1) What do you mean by advertising

promotion create demand of their product in the market and increase it sales Such methods comprises of advertising sales promotion personal selling and publicityToday we are going to discuss about one of such methods It is advertising

Meaning of advertisingAdvertising is a paid form of non-personal presentation for promotion of Ideas goods and services

Importance or merits of advertising Advertising has importance to manufacturer or traders to customer and to society as a whole

Today we will see how advertisement help the manufacturer or traders

Answer) Advertising is a means of how a company encourages people to buy their products services or ideas It is one element of marketing which also includes design Research and data mining

2) Mention any three features of advertisingAnswer)The main features of advertising are

i) It is impersonal form of presentation for promotion of products and services of Ideas

ii) It is issued by identified sponsor The advertisement contains the name of the advertiser

iii) It is a form of mass communication because the message is directed to a large number of persons simultaneously

3) Mention the main merits or importance of advertisement to manufacturer or tradersAnswer)

i) Introducing new product A business organization can introduce itself and its products to the public through advertising

ii) Increase the sale Advertising leads to increase the sale of existing product by entering into new markets and attracting new customers

iii) Create steady demand Advertising creates sustains regular demand by smoothening out seasonal and other fluctuations It enables regular production for the organisation

iv) Economics of scale Advertising facilitate mass distribution of goods and steady demand which lead to large scale and regular production

v) Goodwill Advertising helps in creating a good image of the firm and reputation for its products

Biology Chapter - 03Genetics

Today wewill start chapter and discuss about Genetics Gregor Mendel is known as father of genetics Before entering into Mendelrsquos experiment on Genetics we must know

Q1 Define the following termsi) Genetics Genetics is the study of

transmissionof body features from parents to offspringand the laws relating to such transmission

ii) Heredity It may be defined as transmissionof genetically based characteristics from parentsto offspring

iii) Character and traits Any heritable

Importance to TraderIntroducing new productIncrease the saleCreate steady demandEconomics of scaleGoodwill

some terms featureis a character The alternative forms of acharacter are called traitsex Character (Hair shape) - Traits (Curly straight)

iv) Homologous chromosomes A pair ofcorresponding chromosomes of the same shapeand size one from each parent

v) Genes Genes are the specific parts (DNA segments) of a chromosome which determinethe hereditary characteristicsNearly 30000genes present in human

vi) Alleles Alternative forms of a gene occupying the same position (locus) on homologouschromosomes and affecting the same characteristicbut in different ways

vii) Genotype ndash PhenotypeGenotype means of genes present in the cells of an organism Phenotype means the observable characteristic which is genetically controlled

viii) Mutation It is a sudden change in one or more genes or in the number or in the structure of chromosomes ex Sickle cell anaemia is a blood disease caused by a gene mutation

CLASS NOTES

Class XSubject Eng Literature (The Merchant of Venice ndash William Shakespeare)Topic Act IV Scene 1 Lines 01 to 34 ( Duke helliphelliphellip We all expect a gentle answer Jew) ate 27th April 2020 (2nd Period)

[Students should read the original play and also the paraphrase given in the school prescribed textbook]Summary Questions amp Answers

This scene may be termed as the catastrophe of the play It is the final unravelling of the complicated events which seem to threaten the happiness of Bassanio Portia and Antonio Right is justified to the fullest degree and malice falls into the trap prepared for others No one suffers here but Shylock but even then he receives a generous measure of mercy

o This is the Court-scene Initially we meet

(1)

DUKE I am sorry for thee thou art come to answer (Line 3-6)A stony adversary an inhuman wretchUncapable of pity void and emptyFrom any dram of mercy

(i) Who is addressed here Where is the person Why is the person there

Antonio is addressed hereAntonio is in the court of justice at VeniceAntoniorsquos trial is scheduled to be held here for his failure to meet the conditions of the bond he signed with Shylock

the Duke Antonio Shylock and Salerio Later we meet Bassanio Portia Gratiano and Nerissa

o The Duke says to Antonio that he has to face a very cruel opponent which Antonio admits and expresses his gratefulness to the Duke for his efforts to soften without result the heart of Shylock in order to be merciful to Antonio Antonio further says that he is ready to accept whatever cruel judgement the Court may award

o When Shylock appears in the court the Duke says that Shylock should change his decision of prosecuting Antonio and demanding the penalty specified in the bond out of consideration of the great misfortunes that Antonio has suffered If this is done by him (Shylock) the whole court would be gladdened by his merciful action

(ii) What is the Duke sorry for

The Duke is unable to change the mind of Shylock from his decision to get the bond forfeited even after he pleaded to Shylock Shylock stands firmly for his bond which when forfeited will allow him to take a pound of flesh from any part of Antoniorsquos body(iii) How does the Duke address Shylock`The Duke calls Shylock an adversary with a heart of stones He calls Shylock as an inhuman wretch without pity Shylock is quite lacking in the slightest quality of mercy (iv) How does Antonio reply to this

Antonio replies that he will meet the revenge of Shylock patiently He has prepared himself to suffer with a quiet spirit the utmost that Shylockrsquos tyranny and rage can do(v) What quality of the Duke is revealed here

The Duke is kind and benevolent He is ready to help Antonio He requests Shylock to free Antonio from the trial(vi) What are the terms of the bond that Antonio has signed

The terms of the bond that Antonio has signed were that if Antonio is unable to repay Shylock a certain sum of money specified on the paper on a certain date and in an agreed place the forfeit has to be paid The forfeiture will be an exact pound of Antoniorsquos flesh which Shylock will be a liberty to take from any part of Antoniorsquos body which pleases him

Class XI

STUDY MATERIAL

Class XISubject Eng Literature (The Tempest ndash William Shakespeare) Topic Act I Scene 2 Lines 88 to 132 (Prospero hellip Me and thy crying self) Date 27th April 2020 (3rd Period)

[Students should read the original play and also the paraphrase given in the school prescribed textbook]Summary Questions amp Answers

o Prospero now tells Miranda that he was the Duke of Milan He had been devoting himself more to studies than the affairs of the State His brother Antonio took advantage of this situation and with the help of Alonso the king of Naples seized upon him and her one midnight and shipped them in a frail bark so that they perished in the sea All this took place

(1)

MIRANDA I should sin (Line 118-132)

To think but nobly of my grandmother

Good wombs have borne bad sonsPROSPERO Now the condition

The King of Naples being an enemyTo me inveterate hearkens my brothers suitWhich was that he in lieu othrsquo premisesOf homage and I know not how much tribute

twelve years back

IMPORTANT PASSAGES EXPLAINED(Line 98-103)

PROSPERO helliphelliphelliphelliphelliphelliphellip

He being thus lorded

Not only with what my revenue yieldedBut what my power might else exact like oneWho having into truth by telling of it Made such a sinner of his memoryTo credit his own lie he did believeHe was indeed the duke

Prospero in telling the narrative of his past life here refers to his brother Antonio Prospero being with a studious bent of mind has left the administration of Milan on his younger brother Now Antonio being thus invested like a lord with all the powers derived from Prosperorsquos wealth and what the exercise of Prosperorsquos authority might secure for him regarded himself as a de facto Duke of Milan It is a well-known fact of psychology that a man who repeatedly tells a lie makes of his memory such a sinner against truth as to credit his own lie by the telling of it So Antonio by repeatedly saying to himself and others that he was the Duke came to believe that he was really the Duke Thus falsehood repeatedly asserted gained the force of truth for Antonio and he truly believed it

Should presently extirpate me and mine Out of the dukedom and confer fair MilanWith all the honours on my brother whereonA treacherous army levied one midnightFated to thrsquo purpose did Antonio openThe gates of Milan and ithrsquo dead of darkness The ministers for thrsquo purpose hurried thenceMe and thy crying self

(i) In the earlier lines of this scene what does Prospero tell about his intense interest What was the demand of his interest

In the earlier lines of this scene Prospero tells Miranda that he had an intense interest in the study of philosophy and magic arts Hence in order to improve his mind with this kind of study he kept himself isolated from worldly and state affairs His study was dearer to him than the applause and esteem that he could win from the public His study demanded too much solitude(ii) What forced Antonio to take an undue advantage over Prospero

Prosperorsquos indifferent attitude towards the statersquos affairs and his having boundless trust in Antonio gave rise to a boundless lust for power in Antoniorsquos mind Antonio felt that he must be the actual Duke instead of the part of the Duke he played Thus Antonio took an undue advantage of the situation to usurp Prosperorsquos dukedom(iii) Explain the following lines ldquoI should sin to think but nobly of my grandmother Good wombs have borne bad sonsrdquo

After hearing the treacherous act of her uncle Antonio Miranda says that Prosperorsquos mother was a noble lady and she cannot dishonour her memory by saying that the person named Antonio cannot be his (Prosperorsquos) brother She says that in honour of her grandmother she also cannot say that Antonio must have been begotten not by her grandfather but by some other man She finally concedes that it is known that good mothers have borne bad sons in their wombs and gave birth to them

(iv) Why did the King of Naples accept Antoniorsquos request to help him in usurping his dukedom What did Antonio propose to Alonso

Alonso who was the king of Naples accepted Antoniorsquos request in usurping Prosperorsquos dukedom because he (Alonso) was a sworn enemy of Prospero Antonio proposed that Alonso should immediately drive him (Prospero) and his offspring out of Milan and should confer the dukedom upon him (Antonio) with all the dignities which go with that

position In return Antonio promised that he shall give an annual tribute and also swore his allegiance to Alonso Also he agreed to hold the Dukedom of Milan as a subordinate to the state of Naples(v) How were Prospero and Miranda carried away from the city of Milan and what was the state of small Miranda at that time

In pursuance of the agreement settled between Antonio and Alonso an army of treacherous men was assembled One midnight when the occasion suited the will of destiny Antonio opened the gates of the city of Milan and in the death like silence of midnight Antoniorsquos agents who had been directed to execute his purpose carried Prospero and small Miranda away from the city in all haste They were then forced into a ship and carried some distance out to sea where they put them on a mere hulk of a boat without any rigging or ship-gear and abandoned them leaving them at the mercy of the roaring sea Miranda was a very small child of three years age and she was crying at that time

CLASS -XIDATE-270420Subject Topic Summary Execution

EVS Chapter 1 ndash Mode of Existence

Impact of mode of existence on resources

Q) Why resources are under pressure

Ans - Increase in the sophistication

of technology enabling natural resources to be extracted quickly and efficiently Eg in the past it could take long hours just to cut down one tree only using saws Due to increased technology rates of deforestation have greatly increased

The number of humans is increasing Cultures of consumerism Materialistic views

lead to the mining of gold and diamonds to produce jewelry unnecessary commodities for human life or advancement Consumerism also leads to extraction of resources for the

production of commodities necessary for human life but in amounts excessive of what is needed because people consume more than is necessary or waste what they have

Lack of awareness among the population is striking People are not aware of ways to reduce depletion and exploitation of materials

Accounts Cash Book Today we are going to start a new topic -Cash Book

The key terms used in this chapter are

bullCash book

bullSimple cash book

bullDouble column cash book bullTriple column cash book

bullPetty cash book

bullCash discount

bullContra entry

Here I will share you the meaning of each key terms

bullCash book Cash Book is a special purpose subsidiary book or journal in which cash received and cash payments are recorded

bullSimple cash book

It is a cash book in which only cash transactions are recorded It has only one column on each side

bullTriple column cash book

It is cash book which has three columns one column for each cash and Bankdiscount on each side of the cash book In this book both cash and Bank transactions are recorded together with discount allowed and received

bullPetty cash book

It is a cash book maintained for recording petty expenses

bullCash discount

Cash discount is the amount of discount received or allowed on cash payments and cash receipts Discount received is an income for the business while discount allowed isan expense

bullContra entry

It means transactions involving both cash and Bank Such transactions though recorded in the cash book are not posted into ledger The letter lsquoC is written in Ledger folio for contra entry

Business Studies

ENTREPRENEURSHIP

Now we shall discuss the second chapter

lsquoENTREPRENEURSHIPrsquo

Today before starting the chapter let us recall what

Questions

1What are the main characteristics of Intrapreneurship

Answer

The main characteristics of Intrapreneurship are

Corporate framework-it occurs within the framework of the same company

Semi-Autonomous-Intrapreneurship

we have read last day

Let s today start the class by recalling the last topic taught

Intrapreneurship is the process of discovering and exploring business opportunities within an existing company It involves launching new business ventures within the framework of a present corporation Intrapreneurship is also known as corporate entrepreneurship or corporate venturing

Now let us start with the characteristics of Intrapreneurship

The main characteristics of Intrapreneurship are

Corporate framework

Semi-Autonomous Lack of ownership Senior position Low risk taking Not own boss

Now let us discuss the meaning of enterprise

Enterprise means an undertaking or adventure that requires some innovation and investment and thus involves riskEnterprise always entails decision making coordination and risk bearing

involves crating amd nurturing a semi-autonomous business unit which may be a subsidiary a strategic business unit or a division

Lack of ownership-the intrapreneur is not the owner of the unitb he creates and nurtures

Senior position-he occupies a senior managerial position in the company

Low risk taking-An intrapreneur does not bear the full risk of failure

Not own boss-An intrapreneur is not his own bosss in legal termsHe enjoys the freedom and gets the required resources and support

2 How is Entrepreneur is different from Intrapreneur

The functions involved in both the entrepreneurship and intrapreneurship are by and large similar however there are several differences between the two

Point of distinction

Entrepreneur

Intrapreneur

status An independent business person

A senior executive within a company

Ownership Owner of

the business

An employeesometimes a share in ownership

Financing Responsible for raising finance for the business

Not responsible for raising the finance

Risk bearing

Bears the risk of the business

Does not bears the risk of the business

Reward Profit which is uncertain and irregularcan be loss

Fixed salary and fringe benefits

Need for security low high

3 What do you understand by enterprise

Answer Enterprise means an undertaking or adventure that requires some innovation and investment and thus involves riskEnterprise always entails decision making coordination and risk bearing

COMMERCE NATURE AND OBJECTIVES OF

BUSINESS

Today let us recall the last other two objectives of business by the chart given in the previous class

Firstly we would discuss Human Objectives

Business is run by people and for people Labour is a valuable business element

Human objectives of business are concerned with the well -being of labour

The human objectives are as follows

Labour welfare Developing human

resources Participative

management Labour

management cooperation

Questions

1 Explain the human objectives of a business enterprise

Answer

Business is run by people and for people Labour is a valuable business element

Human objectives of business are concerned with the well -being of labour

The human objectives are as follows

Labour welfare-Business must recognize the dignity of labour and human factors should be given the recognition

Developing human resources-Employees must be provided the opportunities for developing new skills and attitudes

Participative management-Employees should be allowed to take part in decision making process of business

Labour management cooperation-Business should strive for creating and maintaining cordial employer employee relations so as to ensure peace and progress in industry

Now let us discuss the national objectives of business

Optimum utilization of resources

National self- reliance Development of small

scale industries Development of

backward areas Control over pollution

2Explain the national objectives of a business enterprise

Answer

It is the duty of business to utilize the resources of the country properly the national objectives of business

Optimum utilization of resources ndashBusiness should use the nationrsquos resources in the best possible manner

National self- reliance-It is the duty of the business to help the government in increasing experts and in reducing dependence on imports

Development of small scale industries-Big business firms are expected to encourage growth of small scale industries which are necessary for generating employment

Development of backward areas-Business is expected to give preference to the industrialization of backward regions of the country

ECONOMICS

BASIC ECONOMIC CONCEPTS

SUB

TOPIC

Value

Wealth

Welfare

Today we shall start with a new topic of the same chapter ie lsquoValuersquo

Value of a commodity is defined as the valuation placed by a household on the consumption of this commodity

lsquoValuersquo has two different meanings and these are

a Value -in -use It refers to consumption value of a commodity It expresses the utility derived from the consumption of a particular commodity A necessity like water has a very high value ndashin ndashuse or

Question

1What is value

Answer

Value of a commodity is defined as the valuation placed by a household on the consumption of this commodity

2What is value-in use

Answer It refers to consumption value of a commodity It expresses the utility derived from the consumption of a particular commodity A necessity like water has a very high value ndashin ndashuse or consumption value

3What is value ndashin- exchange

Answer It relates to market value of a commodity

It is the rate at which a particular good or service can be exchanged for money

For example in barter system if a person is prepared to exchange 3 metres of cloth with 1 pair

consumption value

b Value ndashin-exchange It relates to market value of a commodity

It is the rate at which a particular good or service can be exchanged for moneyFor example in barter system if a person is prepared to exchange 3 metres of cloth with 1 pair of shoes then the value in exchange of 3 metres of cloth is 1 pair of shoesValue in exchange is the power of purchasing other goods In modern monetised economies the exchange value of goods are expressed in terms of money as prices

Now let us discuss the term lsquoWealthrsquo

Wealth refers to the stock of all those assets which are a source of income

Wealth is a stock concept

Wealth must possess the following features

a Utility It must possess utility or give some

of shoes then the value in exchange of 3 metres of cloth is 1 pair of shoes

Value in exchange is the power of purchasing other goods In modern monetised economies the exchange value of goods are expressed in terms of money as prices

4 What is wealth

Answer it refers to the stock of assets or goods which are a source of income and have personal or national ownership

5 What are the features of wealth

Answer The features of wealth are as follows

Wealth must possess the following features

a Utility It must possess utility or give some satisfaction

b Scarcity It must be limited in quantityc Transferability it should be transferable its

ownership can be transferred from one person to another person

d Exchange value It must possess exchange value

6 What is welfare

Answer

Welfare is defined as satisfaction and happiness a sense of well- being among the people

satisfactionb Scarcity It must be

limited in quantityc Transferability It

should be transferable its ownership can be transferred from one person to another person

d Exchange value It must possess exchange value

Now let us discuss the term lsquoWelfarersquo

Welfare is defined as satisfaction and happiness a sense of well- being among the people

Welfare is affected by factors like

a Consumption of goods and services

b Environment

c Family relations

d Degree of freedom

e Law and order situation

Mathematics Trigonometric equation

To find the general solution of the equation sinθ=0

When sin θ =0

Then θ= 0 π2π 3π-π -2π -3

i e when θ = 0 or an integral multiple of π

i e when θ= nπ where n is any integer

Therefore the general solution of the equation sin

Example1 Find the general values of θ which satisfy the equation sin2 θ =34

Solution sin2 θ= 34

Or sin θ = +34 or -34

Or sin θ = sin π3 or sin (-π3)

Therefore

θ = [nπ + (-1) n (π3)] or[ nπ+ (-1) n (-π3)]

= nπ +π3 or nπ-π3 where n= any integer

Example 2Find the values of θ which satisfy tan2 θ

θ=0 is θ= nπ where n is any integer

To find the general solution of the equation cos θ=0

When cos θ=0

Then θ=π2 3π2 5π2 -π2 -3π2 -5π3

i e when θ is an odd multiple of π2

i e when θ=(2n+1) π2 where n= any integer

Therefore the general solution of the equation cos θ =0 is θ= (2n+1) π2 where n= any integer

To find the general solution of the equation tan θ = 0

Clearly tan θ =0 implies sin θcos θ =0

Therefore θ = nπ

i e the general solution of the equation tan θ=0 is θ =nπ where n = any integer

To find the general solution of the equation cot θ =0

Clearly cot θ =0 implies (cos θsin θ) = 0

i e cos θ =0

Therefore θ = (2n+1) π2

Therefore the general solution of the equation cot θ =0 is θ = (2n+1) π2

Where n= any integer

To find the general solution of the equation sin θ= k (-

=13 -πleθleπ

Solution tan2 θ =13

Or tan θ = plusmn1radic(3) =tan(plusmnπ6)

θ=nπ plusmn π6 where n =any integer

If n=0 then θ=plusmnπ6

If n=1 then θ= π plusmn π6

If n=-1 then θ= -π plusmn π6

Therefore the required solution in -π le θ le π are θ= π6 5π6 -π6 -5π6

Exercise Find general solution of sin 2θ=cos θ [Hints Use sin 2θ= 2sin θcosθ and then take cosθ

common]

1lek le1)

Determine an angle alpha such that sin =k and -π2le αle π2

Then we have

Sin θ = k = sin α

Or sin θ - sin α =0

Or 2 cos [(θ+α) 2] sin [(θ-α) 2] =0

Therefore either cos [(θ +α) 2] =0 (1)

Or sin [(θ-α) 2] =0 (2)

Now from (1) we get (θ+α) 2= (2m+1) π2)

Or θ = (2m+1) π-α (3)

And from (2) we get (θ-α) 2 =mπ

Or θ= 2mπ+α(4)

Where m = any integer

Clearly the solution (3) amp (4) may be combined in the following form

θ= nπ+(-1) n α where n= any integer

Therefore the general solution of sin θ = sin α is θ = nπ +(-1) n α where n is any integer and -π2 le α le π2

Biology Chapter - 04Kingdom Monera

Today we will discuss about bacterial reproduction and its usefulness

Fig Binary Fission

Fig Conjugation Fig Transformation

Bacterial reproduction is mainly asexual but sexual reproduction

also takes place

Asexual reproduction takes place by i) Binary fission - from one bacteriato

two bacteria are produced in every 20 to 30mins

ii) Buddingiii) Endospore formation - during

unfourable condition

Sexual reproduction by three ways

1) Conjugation - Transfer of genetic material between cells that are in physical contact with one another

2) Transduction - Transfer of genetic materialfrom one cell to another by a bacteriophage

3) Transformation - Transfer of cell-freeor naked DNArsquo from one cell to another

Bacteria causes different diseases inplants animals and human and

it causes food spoilage and waterpollution but it also have some useful

activities

i) Bacteria are helpful in sewage water treatment

ii) It is used in antibiotic (medicine) production

iii) Anaerobic bacteria help in biogas(energy) production

iv) Many household products like yoghurt cheese are manufactured by use of bacteria

v) Rhizobium by symbiotic relationship with leguminous plant increase soil fertility

vi) Besides these bacteria is helpful in genetic engineering degradation of petroleum hydrocarbonand in dairy

industry

Physics Motion in plane Here we will introduce Projectile Motion

Execution

Projectile

Y

usinθ u h

θX

ucosθ

Suppose a body is projected with an angle θ So initial velocity u can be resolved into two components

Horizontal component - ucosθ ( for range)

Vertical component - usinθ ( for height)

usinθ changes during motion and becomes zero at maximum height position but ucosθ remain unchanged

The maximum height of projectile is h

NB If initial is upward then g = -ve and if it is downward then g = +ve Height is +ve if direction of motion does not change ( for ex a body thrown upwards but goes down ultimately then height h = -ve)

The angle of projectile θ is the angle made with horizontal

HISTORY ndash GROWTH OF NATIONALISM

SUB TOPIC- REVOLUTIONARY NATIONALISM Bengal formation of Anushilan Samity and Jugantar Group

The intensification of the Swadeshi movement and Government policy of terror and repression led to outbreak of violence Bombs were manufactured and attempts on the lives of unpopular Government officials became frequent In the gymnasium of Scottish Church College which was known as General Assemblies Institution a secret society was formed known as Anushilan Samity

Aurobindo Ghosh send from Baroda his emissary Jatindranath Banerjee to mobilize the Bengal revolutionaries

Hemchandra Qanungo and Satyen Bose published Journal Jugantar

The Jugantar group planned to assassinate oppressive magistrate Kingsford by Khudiram Bose and Prafulla Chaki in 1908 Prafulla Chaki committed suicide to avoid arrest Khudiram was tried and hanged

Afew days later the police found a bomb factory in Maniktala and arrested a large number of revolutionaries The trial of revolutionaries became famous as the Alipore Bomb Case

In the course of the trial the approver the public prosecuter and a police officer were assassinated

1 Question Name two journals which preached the cult of violence

Answer a) Yugantar edited by Bhupendranath DuttaB) Bandemataram edited by Aurobindo Ghosh2 Question Why was Khudiram arrested and hangedAnswer An attempt was made to assassinate a hated vindictive majistrate named Kingsford by Khudiram Bose and Prafulla Chaki Their attempt failed and the bomb they threw killed two English ladies Khudiram was arrested and put to trial and then hanged3Question Who was Aurubindo GhoshAnswer Aurobindo Ghosh a nationalist revolutionary who was charged for his involvement in the Alipore Bombing Case He was accused of it along with his brother Barindra nath Ghosh But Aurobindo was acquitted because of the brilliant pleading of his counsel Chittaranjan Das Then he became a spiritual reformer introducing his visions on human progress and spiritual evolution4 Qustion Who was KingsfordAnswer Kingsford was an unpopular British chief Magistrate who was the target of the bomb thrown at Muzaffarpur by Khudiram and Prafulla Chaki

Most of the accused were convicted and sentenced to

long term of imprisonmentBut

Aurobindo Ghosh was acquitted mainly owing to the brilliant pleading of his counsel Chittaranjan Das

Political science Topic-Sovereignty

Summary Sovereignty is the full right and power of a governing body over itself without any interference from outside sources or bodies In political theory sovereignty is a substantive term designating supreme legitimate authority over some polity In international law sovereignty is the exercise of power by a state

Internal Sovereignty

Internal sovereignty means supreme authority within ones territory while external sovereignty relates to the recognition on the part of all states that each possesses this power in equal measure

External sovereignty

external sovereignty relates to the recognition on the part of all states that each possesses this power in equal measure

Distinguish between

Execution

Answer the following questions

Short notes-

Sovereignty

Internal Sovereignty

External sovereignty

Homework- learn

external sovereignty and internal sovereigntySovereignty is the principle

of supreme and

unquestionable authority

reflected in the claim by the

state to be the sole author of

laws within its territory

Definition of external vs internal sovereigntyInternal sovereignty refers to

the relationship between a

sovereign power and its

subjects ndash it refers to the

location of the supreme

authority within the state In

the UK for example internal

sovereignty (supposedly)

resides within Parliament

reflected in the

constitutional principle of

parliamentary

sovereigntyBy contrast

external sovereignty refers

to the capacity of the state

to act independently and

autonomously on the world

stage This is what is

sometimes called lsquostate

sovereigntyrsquo or lsquonational

sovereigntyrsquo and implies

that states are legally equal

and that the territorial

integrity and political

independence of a state is

inviolable

Class ndash XII

Date - 2742020 STUDY MATERIALSubject Topic Summary Execution Business Studies

Job Analysis amp Manpower Planning

At first let us recall the chapter what we have discussed till nowbullJob analysisbullJob specification bullJob description bullJob enlargement bullJob enrichment

Today we will do some questions answers from the chapter

Questions 1ldquoJob analysis job description and job specification are interrelatedrdquo Comment Answer) Job analysis is a systematic and detailed examination of a job to collect all the relevant information about it The contents off the job are summarised in the job description The qualification needed for the job are summarised in job specificationThus there is close interrelationship between job analysis job description and job specification

Question 2ldquoJob enlargement is a horizontal extension of a job whereas job enrichment is a vertical extension of a jobrdquo ElucidateAnswer) Job enlargement involves adding one or more task to a job coma where as job enrichment involves adding more autonomy and responsibility to a job Job enlargement is therefore horizontal extension of a job coma whereas job enrichment is a vertical extension of a job

Question 3 )

What is manpower estimation Explain its quantitative and qualitative aspectsAnswer) Manpower estimation is the process by which management determines how an organisation should move from itrsquos current manpower positionto its desired manpower position There are two dimensions of Manpower estimation- quantitative and qualitative

Quantitative aspectThis aspect of Manpower estimation involves estimating the number of employees required in a future time period Workload analysis and workforce analysis are done to estimate the quantity of required manpower

Qualitative aspectThe estimate of the knowledge skills experience etc of required manpower is the qualitative aspect of Manpower estimation The quality of Manpower can be judged on the basis of job analysisand job specification

COMMERCE

CAPITAL-FIXED AND WORKING

Today let us start the class by discussing the sources of finance for different types of business firms

The term lsquocapitalrsquo refers to the investment made in the enterprise for the purpose of earning profits

Requirements of capital and sources of capital for different types of business firms are

1 Capital for sole proprietorship businessA sole proprietor operates at a small scale and thereforerequires a limited amount of capital

2 Capital for partnership firmCapital requirements as well as capital base of a partnership is bigger than that of a sole trader businessThe owned capital is contributed by the partners in an agreed ratio

3 Capital for joint stock companyA joint stock company generally requires large amount of capitalA public company can raise huge capital through issue of shares In addition to share capital it can utilize retained profits

Now let u discuss the meaning of Finance PlanningFinance planning is the process of estimation the financial requirements of an organization specifying the sources of firms and ensuring that enough funds are available at the right time

1 What do you mean by Finance PlanningAnswer Finance planning is the process of estimation the financial requirements of an organization specifying the sources of firms and ensuring that enough funds are available at the right time

2Discuss the role of financial planning of an enterpriseThe role of financial planning are as followsa A sound financial plan helps a business enterprise to avaid the problems of shortage and surplus of fundsbFinancial planning serves as a guide in developing a sound capital structure so as to maximize returns to shareholders c It helps in effective utilization of fundsd It provides policies and procedures for coordinating different functional areas or departments of businesse It enables the management to exercise effective control over the financial activities of an enterprisef It helps the company to prepare for facing business shocks and surprises in future

Mathematics

Continuity and differentiability

Recall Definition of ContinuityLet f(x) be a single valued function of x and x=a be a point in the domain of definition of the function The function is said to be continuous at x=a ifi) f(c) is defined ie f(x) has a definite finite value at x=cii) lim xrarra f(x) exists andiii) lim xrarra f(x) =f(a) In other words f(x) is said to be continuous at x=a if lim xrarra+ f(x)= lim xrarra- f(x) = f(a) Or f(a+0) =f(a-0) =f(a) Or lim hrarr0 f(a+h)= f(a) Algebra of continuous functionsNow we will study some algebra of continuous functions Theorem 1 Suppose f and g be two real functions continuous at a real number c Then(1) f + g is continuous at x = c(2) f ndash g is continuous at x = c(3) f g is continuous at x = c(4) (fg) is continuous at x = c (provided g (c) ne 0)

Example 1 Prove that every rational function is continuousSolution Recall that every rational function f is given byf(x)=[p(x) q(x) ] q(x)ne0where p and q are polynomial functions The domain of f is all real numbers except points at which q is zero Since polynomial functions are continuous f is continuous by (4) of Theorem 1Example 2Discuss the continuity of sine functionSolution To see this we use the following factslim xrarr0 sin x =0Now observe that f (x) = sin x is defined for every real number Let c be a real number Put x = c + h If x rarr c we know that h rarr 0 Therefore lim xrarrc f(x)

= lim xrarrc sin x= lim hrarr0 sin(c+h) =lim hrarr0 [sin c cos h + cos c sin h ]=lim hrarr0 (sin c cos h) + lim hrarr0 (cos c sin h) = sin c +0=sin c = f(c) Thus lim xrarrc f(x) = f(c) and hence f is a constant function Exercise Prove that the function f(x) = x2 +2x is continuous for every real value of x [Hints show that lim xrarra+ f(x) = lim xrarra- f(x) = f(a) ]

Biology Reproduction in Flowering plants We will discuss about megasporoangium

megasporagenesis and female gametophyte

Q4 Describe the structure of megasporangium

Ovule is attached to the placenta by astalk called funicle

Each ovule has one two or three protectivecoverings called integuments

At the tip of integuments a small openingcalled micropyle is organised

Opposite to the micropylar end is the chalaza

Within the integuments a mass of cellsnucellusand inside it embryo sac orfemale gametophyte is present

Q5 Describe a mature embryosacamp its formation

In most of the flowering plants only oneof the 4 megaspores formed as a result ofmegasporogenesis that is functional while theother three degenerate

The

functional megaspore develops into thefemale gametophyte

Formation The nucleus

of the functional megasporedivides mitotically to form two nuclei first andthen two more sequential mitotic nucleardivisions result in the formation of four ampthen eight nucleate stages of embryo sac

Six of the eight nucleus are surrounded bycell walls and organised into cells

The remaining two nuclei called polar nuclei are found below the egg apparatus in the largecentral cell

Three cells consisting of two synergids amp one egg cell present bottom of

embryo sac Three cells

at the chalazal as antipodal cells

Two polar nuclei together present in large central cell

HISTORY

TOWARDS INDEPENDENCE AND PARTITION THE LAST PHASE(1935-1947)SUB TOPIC NATIONAL MOVEMENTS DURING THE SECOND WORLD WAR

Spread of Quit India Movement On 9th August 1942Gandhiji and other Congress leaders were arrested The Congress was declared illegal The news of the arrest of all leaders marked the beginning of a widespread movement of India It was not possible for such a movement to remain peacefulBut the arrest of the all notable congress leaders virtually left the movement in the hands of the mass The movement took the form of violent and militant outbreakBesides congressmen revolutionaries also were very active in the movement The Congress Socialist group also played a prominent role

1 Question Why did the British authority arrest the Congress leaders on 9 th August 1942Answer Congress Working committee adopted the Quit India resolution which was to be ratified at the Bombay AICC meeting in 8th August 1942 They decided to launch a mass struggle on non-violent lines Gandhiji gave a clarion call to all section of the people rdquoKarenge ya Marengerdquo (do or die) Congress leaders gave the call to driving out

the British from IndiaViceroy had taken strong action against the Quit India movement Gandhiji and all the leaders of Congress were arrested

2 Question How did Quit India Movement spread out all over IndiaAnswer The news of the leaders lsquo arrest marked the beginning of a widespread movement to remain peacefulThe movement took form of violent outbreak There were widespread cutting of telephone and Telegraph wires damaging railway lines raising barricades in cities and towns and other forms of violent demonstations

Question Name the leaders of Congress

Socialist group played a prominent part Notable among the Jayprakash Narayan Rammonohar Lohia Aruna Asaf Ali

Political science

Topic-Franchise and Representation

Summary

The election commission

The Election Commission of India is an autonomous constitutional authority responsible for administering Union and State election processes in India The body administers elections to the Lok Sabha Rajya Sabha State Legislative Assemblies in India and the offices of the President and Vice President in the country

Functions of election commission-

India is a sovereign socialist secular democratic republic Democracy runs like a golden thread in the social economic and political fabric woven by the Constitution given by lsquoWe the People of Indiarsquo unto ourselves The concept of democracy as visualised by the Constitution pre-supposes the representation of the people in Parliament and State legislatures by the method of election The Supreme Court has held that democracy is one of the inalienable basic features of the Constitution of India and forms part of its basic structure The Constitution of India adopted a Parliamentary form of government Parliament consists of the President of India and the two Houses mdash Rajya Sabha and Lok Sabha India being a Union of states has separate state legislatures for each state State legislatures consist of the Governor and two Houses mdash Legislative Council and Legislative Assembly mdash in seven states namely Andhra Pradesh Telangana Bihar Jammu amp Kashmir Karnataka Maharashtra and Uttar Pradesh and of the Governor and the state Legislative Assembly in the remaining 22 states Apart from the above two out of the seven Union Territories namely National Capital Territory of Delhi and Puducherry also have their Legislative Assemblies

ExecutionShort notes-Election commissionFunctions of election commission

Homework- Learn

Computer

Science

Computer hardware NAND Gate

A NOT-AND operation is known as NAND operation It has n input (n gt= 2) and one output

Logic diagram

Truth Table

NOR Gate

A NOT-OR operation is known as NOR operation It has n input (n gt= 2) and one output

Logic diagram

Truth Table

XOR Gate

XOR or Ex-OR gate is a special type of gate It can be used in the half

adder full adder and subtractor The exclusive-OR gate is abbreviated as EX-OR gate or sometime as X-OR gate It has n input (n gt= 2) and one output

Logic diagram

Truth Table

XNOR Gate

XNOR gate is a special type of gate It can be used in the half adder full adder and subtractor The exclusive-NOR gate is abbreviated as EX-NOR gate or sometime as X-NOR gate It has n input (n gt= 2) and one output

Logic diagram

Truth Table

Physics

Chapter 1 Electric Field ( Electric Dipole) (Summary)

Here we will derive Expression of electric field at broad side

On position of dipole

Execution

Q With the help of a labelled diagram obtain an expression for the electric field intensity E at any point on the equitorial line ( broad-side on position) of an electric dipole

Ans

E1 E1sinθ

E θ P E1 θ

( r2+L2)12 E2 E

r E2 E2sinθ

-q θ L O L +qA B

Let us consider that the point P is situated on the right bisector of the dipole AB at a distance r meter from its midpoint O

Let E1 and E2 be the electric field intensities of the electric field at P due to charge +q and ndashq of the dipole resp The distance of P from each charge is ( r2+L2)12

So E1 = 14 πϵ q

(r 2+L 2) away from +q

E2 = 14 πϵ q

(r 2+L 2) towards ndashq

The magnitudes of E1 and E2 are equal but directions are different Now resolving E1 and E2 into two components parallel and perpendicular to AB we get

The components perpendicular to AB E1sinθ and E2sinθ cancel each other because they are equal and opposite

The components parallel to AB are E1cosθ and E2 cosθ are in same direction and add up

So resultant intensity of electric field at the point P is

E = E1cosθ + E2 cosθ

E = 14 πϵ q

(r 2+L 2) 2 cosθ

Now from fig we have cosθ =BOBP = L (r2+L2)12

So we get E = 14 πϵ 2qL ( r2+L2)32

Now electric dipole moment p= 2qL

So E = 14 πϵ p ( r2+L2)32

HW Find the expression of Electric field as done here but this time take r gtgt 2L

Also find the expression of torque experience by a dipole

(Hint Electric force experienced by charges of dipole in electric field is qE each Let θ be the angle which dipole makes with electric lines of force then perpendicular distance between two charges is 2Lsinθ Then torque = force x perp distance = qE x 2L sinθ So τ=pE sinθ where p =2qL )

STUDY MATERIAL

Class XIISubject Eng Literature (The Tempest ndash William Shakespeare) Topic Act IV Scene 1 Lines 84 to 133 (Iris hellip A contract of true love Be not too late ) Date 27th April 2020 (4th Period)

[Students should read the original play and also the paraphrase given in the school prescribed textbook]Summary Questions amp Answers

o Ceres soon appears and comes to know that she has been summoned to celebrate the contract of true love

o Ceres expresses her unwillingness to meet Venus and Cupid as she has shunned their company

o Ceres and Juno both bestow their blessings upon Ferdinand and Miranda with June gifting honour riches happiness in marriage and Ceres presents plenty of earthrsquos produce

o Iris summons the water-nymphs and reapers to come and celebrate a contract

(1) IRIS Of her society (Line 91-101)

Be not afraid I met her deity

Cutting the clouds towards Pathos and her sonDove-drawn with her Here thought they to have doneSome wanton charm upon this man and maidWhose vows are that no bed-right shall be paidTill Hymens torch be lightedmdashbut in vainMarss hot minion is returned againHer waspish-headed son has broke his arrowsSwears he will shoot no more but play with sparrowsAnd be a boy right out

(i) Where were Venus and Cupid seen flying How were they travelling Why did they want to join the marriage celebration of Ferdinand and Miranda

of true love

Venus and Cupid were seen flying through the air towards Paphos the famous city which is situated on the island of Cyprus They were travelling by air-borne chariot drawn by doves They certainly wanted to come here in order to play some amorous trick upon Ferdinand and Miranda who are under a vow not to gratify their physical desires till the holy ceremony of their marriage has been performed(ii) What have Venus and Cupid done after failing in their plan

After being failure of their plan Venus who is a very passionate deity and who is the mistress of Mars (the god of war) has gone back while here ill-tempered son Cupid has broken his arrows of love in his state of desperation(iii) What has Cupid firmly decided

Cupid is feeling so disappointed that he has firmly decided to shoot no more arrows to arouse love in human hearts but to spend his time playing with sparrows Thus he would now become just a boy and would give up his original function of shooting arrows on human beings to make them fall in love(iv) What vow had Ceres taken How did Ceres feel at the abduction

After the abduction of her daughter Prosperina by Pluto Ceres had taken a vow to always keep away from the disgraceful company of Venus and her blind son Cupid the god of love Ceres felt deeply distressed when Pluto had carried off her daughter and had made her his wife by force(v) Why has Ceres not forgiven Venus and her blind son For what do Ceres want to be sure

As the abduction had been manipulated by Venus the goddess of beauty and love and her blind son Cupid Ceres has never forgiven them for their part in the whole plot Ceres wants to be sure that she would not have to meet Venus and Cupid who had engineered the abduction of her daughter Prosperina

AS THIS lsquoMASQUErsquo SCENE IS VERY IMPORTANT IN THE PLAY THE PARAPHRASE OF THE ENTIRE PORTION OF MASQUE SCENE (Act IV Lines 58 to 143) IS GIVEN BELOW

IRIS Goddess of RainbowCERES Goddess of Agriculture and all the fruits of the earth

(Nature growth prosperity rebirth ndash notions intimately connected to marriage)JUNO The majestic Queen of Heavens and wife of Jupiter (Jupiter is the king of Gods)

VENUS The Goddess of love CUPID Son of Venus PLUTO God of death (In the play referred by Shakespeare as lsquoDisrsquo which is a Roman name for Pluto)

ORIGINAL TEXT PARAPHRASEPROSPEROWellmdash

PROSPERONow come Ariel Let there be too many rather than too few

Now come my Ariel Bring a corollaryRather than want a spirit Appear and pertly[to Ferdinand and Miranda]No tongue all eyes Be silent

spirits in attendance Appear briskly

[to Ferdinand and Miranda]Look with your eyes but do not say a word

[Soft music] [Soft music][Enter Iris] [Enter Iris]

IRISCeres most bounteous lady thy rich leasOf wheat rye barley vetches oats and peasThy turfy mountains where live nibbling sheepAnd flat meads thatched with stover them to keepThy banks with pioned and twilled brimsWhich spongy April at thy hest betrimsTo make cold nymphs chaste crowns and thybroom-grovesWhose shadow the dismissegraved bachelor lovesBeing lass-lorn thy pole clipped vineyardAnd thy sea-marge sterile and rocky-hardWhere thou thyself dost airmdashthe Queen othrsquoSkyWhose watery arch and messenger am IBids thee leave these and with her sovereign grace[Juno appears] Here on this grass-plot in this very placeTo come and sport Her peacocks fly amainApproach rich Ceres her to entertain

IRISCeres most generous lady you are the cause of rich fields or fertile land where wheat rye barley beans oats and peas grow the grassy mountains where the sheep graze and the flat meadows covered with coarse hay to be used as fodder for cattleYour banks are covered with marsh-marigolds and reeds and the rainy April under your orders brings forth to make for the maids who are not in love beautiful crowns your woods where the broom flourishes and where the bachelor who has been dismissed by the maid he loved lies down being forsaken your vineyard in which the poles are embraced by the vines and the margin of the sea which is barren and rocky where you roam about to enjoy the fresh air ndash the queen of the sky (Juno) whose messenger I am besides being represented as the rainbow bids you leave all these and with her majesty here on this grassy plot in this very place come and sport her peacocks carry her fast in her chariot through the air and are making their way here approach rich Ceres to welcome her

[Enter Ariel as Ceres] [Enter Ariel as Ceres]

CERESHail many-coloured messenger that neerDost disobey the wife of JupiterWho with thy saffron wings upon my flowersDiffusest honey-drops refreshing showersAnd with each end of thy blue bow dost crownMy bosky acres and my unshrubbed downRich scarf to my proud earth Why hath thy queenSummoned me hither to this short-grassed green

CERESWelcome rainbow that never dared disobey Juno the wife of Jupiter who with your orange coloured rays spread honey-drops refreshing showers And with each end of thy blue bow drown my bushy acres and my hilly country which is free from shrubs you thus forming a rich scarf Why has your queen called me here to this place covered with short grass

IRISA contract of true love to celebrateAnd some donation freely to estateOn the blest lovers

IRISI have called you to celebrate a contract of true love and bestow some liberal gift upon the blessed lovers

ORIGINAL TEXT PARAPHRASECERESTell me heavenly bowIf Venus or her son as thou dost knowDo now attend the queen Since they did plotThe means that dusky Dis my daughter gotHer and her blind boys scandaled companyI have forsworn

CERESTell me heavenly bow if Venus the Goddess of love or Cupid her son and pedlar of passion at this time attend the heavenly queen Juno because you are sure to know Since the day they conspired against me and dark Pluto took away my daughter here and Cupidrsquos disgraceful company I have left off

IRISOf her societyBe not afraid I met her deityCutting the clouds towards Pathos and her sonDove-drawn with her Here thought they to have doneSome wanton charm upon this man and miad

IRISBe not afraid of her company I met her deity moving on the clouds towards Paphos the sacred home of Venus on the island of Cyprus along with her son on her chariot drawn by doves Here they contemplated to exercise a charm upon this man and maid producing

Whose vows are that no bed-right shall be paidTill Hymens torch be lightedmdashbut in vainMarss hot minion is returned againHer waspish-headed son has broke his arrowsSwears he will shoot no more but play with sparrowsAnd be a boy right out

wantonness before the actual marriage ceremony but did not succeed Venus has returned her irritable son has broken his arrows and swears that he will give up his practice of trying to inspire love but play with sparrows and be a boy again

[Music is heard] [Music is heard]

CERESHighst queen of stateGreat Juno comes I know her by her gait

CERESHighest queen of state Great Juno there she comes I know here by her gait

[Enter Juno] [Enter Juno]

JUNOHow does my bounteous sister Go with meTo bless this twain that they may Prosperous beAnd honoured in their issue

JUNOHow are you doing my generous sister Come with me to bless this couple so that they may be prosperous and fortunate in their children

[They sing] [They sing]

JUNOHonour riches marriage-blessingLong continuance and increasingHourly joys be still upon youJuno sings her blessings upon you

JUNOMay honour riches happiness in marriage long continuance and increase of those boons ever rest upon you as hourly joys Juno showers down upon you her blessings in song

CERESEarths increase foison plentyBarns and garners never emptyVines and clustring bunches growingPlants and goodly burden bowingSpring come to you at the farthestIn the very end of harvestScarcity and want shall shun youCeresrsquo blessing so is on you

CERESMay you have the plenty of earthrsquos produce Your barns and granaries may never be empty Your vines may grow with clustering bunches Your fruit trees may be heavily laden with their fruit May there be continuous spring and harvest May scantiness and want leave you forever Such is the blessing of Ceres upon you

FERDINANDThis is a most majestic vision andHarmoniously charmingly May I be boldTo think these spirits

FERDINANDThis is a great vision and magically melodious Should I suppose the characters (taking part in the masque) are spirits

PROSPEROSpirits which by mine artI have from their confines calld to enactMy present fancies

PROSPEROYes they are spirits whom I have summoned from the regions to which they are confined to carry into effect my fanciful designs

ORIGINAL TEXT PARAPHRASEFERDINANDLet me live here everSo rare a wondered father and a wifeMakes this place paradise

FERDINANDI should like to live here forever Such a wise and wonderful father makes this place a paradise

[Juno and Ceres whisper and send Iris on employment] [Juno and Ceres whisper and send Iris on employment]

PROSPEROSweet now silence

PROSPEROMy dear Ferdinand speak no more Juno and Ceres are

Juno and Ceres whisper seriouslyTheres something else to do Hush and be muteOr else our spell is marred

whispering with a solemn look There is something else coming Silence Or else our magic will be spoilt

IRISYour nymphs called naiads of the wandering brooksWith your sedged crowns and over-harmless looksLeave your crisp channels and on this green landAnswer your summons Juno does commandCome temperate nymphs and help to celebrateA contract of true love Be not too late

IRISYou nymphs called Naiads denizens (M inhabitants) of the running stream with your chaplets of sedge and ever-helpful looks leave your wrinkled channels and on the green land answer the summons sent to you Juno has ordered some chaste nymphs and help to celebrate a noble and true marriage Donrsquot delay

[Enter certain nymphs] [Enter certain nymphs]You sunburnt sicklemen of August wearyCome hither from the furrow and be merryMake holiday your rye-straw hats put onAnd these fresh nymphs encounter every oneIn country footing

You sunburnt harvesters weary from the effects of the heat in August come here from the furrowed land and rejoice Make holiday with your rye-straw hats upon you and meet these fresh nymphs and join in country dancing

[Enter certain reapers properly habited They join with the nymphs in a graceful dance towards the end whereof Prospero starts suddenly and speaks]

[Enter certain reapers properly habited They join with the nymphs in a graceful dance towards the end whereof Prospero starts suddenly and speaks]

PROSPERO[aside] I had forgot that foul conspiracyOf the beast Caliban and his confederatesAgainst my life The minute of their plotIs almost come [to the spirits]Well done Avoidno more

PROSPERO(Aside)I had forgotten the wicked conspiracy of the beast Caliban and his accomplices against my life the time of their plot has almost arrived ndash (To the Spirits) well done depart no more of this

[To a strange hollow and confused noise the spirits heavily vanish]

[The spirits depart]

ORIGINAL TEXT PARAPHRASEFERDINANDLet me live here everSo rare a wondered father and a wifeMakes this place paradise

FERDINANDI should like to live here forever Such a wise and wonderful father makes this place a paradise

[Juno and Ceres whisper and send Iris on employment] [Juno and Ceres whisper and send Iris on employment]

PROSPEROSweet now silenceJuno and Ceres whisper seriouslyTheres something else to do Hush and be muteOr else our spell is marred

PROSPEROMy dear Ferdinand speak no more Juno and Ceres are whispering with a solemn look There is something else coming Silence Or else our magic will be spoilt

IRISYour nymphs called naiads of the wandering brooksWith your sedged crowns and over-harmless looksLeave your crisp channels and on this green landAnswer your summons Juno does commandCome temperate nymphs and help to celebrateA contract of true love Be not too late

IRISYou nymphs called Naiads denizens (M inhabitants) of the running stream with your chaplets of sedge and ever-helpful looks leave your wrinkled channels and on the green land answer the summons sent to you Juno has ordered some chaste nymphs and help to celebrate a noble and true marriage Donrsquot delay

[Enter certain nymphs] [Enter certain nymphs]You sunburnt sicklemen of August wearyCome hither from the furrow and be merryMake holiday your rye-straw hats put onAnd these fresh nymphs encounter every oneIn country footing

You sunburnt harvesters weary from the effects of the heat in August come here from the furrowed land and rejoice Make holiday with your rye-straw hats upon you and meet these fresh nymphs and join in country dancing

[Enter certain reapers properly habited They join with the nymphs in a graceful dance towards the end whereof Prospero starts suddenly and speaks]

[Enter certain reapers properly habited They join with the nymphs in a graceful dance towards the end whereof Prospero starts suddenly and speaks]

PROSPERO[aside] I had forgot that foul conspiracyOf the beast Caliban and his confederatesAgainst my life The minute of their plotIs almost come [to the spirits]Well done Avoidno more

PROSPERO(Aside)I had forgotten the wicked conspiracy of the beast Caliban and his accomplices against my life the time of their plot has almost arrived ndash (To the Spirits) well done depart no more of this

[To a strange hollow and confused noise the spirits heavily vanish]

[The spirits depart]

Ac-12 27420 topic Revaluation of Assets and Liabilities

REVALUATION OF ASSETS AND LIABILITIES

On admission of a new partner the firm stands reconstituted and consequently the assets are revalued and liabilities are reassessed It is necessary to show the true position of the firm at the time of admission of a new partner If the values of the assets are raised gain will increase the capital of the existing partners Similarly any decrease in the value of assets ie loss will decrease the capital of the existing partners For this purpose alsquoRevaluation Accountrsquo is prepared This account is credited with all increases in the value of assets and decrease in the value of liabilities It is debited with decrease on account of value of assets and increase in the value of liabilities The balance of this account shows a gain or loss on revaluation which is transferred to the existing partnerrsquos capital account in existing profit sharing ratioAccounting for Revaluation of Assets and Liabilities when there is a Changein the Profit Sharing Ratio of Existing PartnersAssets and liabilities of a firm must also be revalued at the time of change in profit sharing ratio of existing partners The reason is that the realisable or actual value of assets and liabilities may be different from those shown in the Balance Sheet It is possible that with the passage of time some of the assets might have appreciated in value while the value of certain other assets might have decreased and no record has been made of such changes in the books of accounts Similarly there may be some unrecorded assets amp libilities that may have to be accounted for Revaluation of assets and reassessments of liabilities becomes necessary because the change in the

value of assets and liabilities belongs to the period to change in profit sharing ratio and hence must be shared by the partners in their old profit sharing ratio Revaluation of assets and reassessment of liabilities may be given effect to in two different ways (a) When revised values are to be recorded in the books and(b) When revised values are not to be recorded in the books

When revised values are to be recorded in the booksIn such a case revaluation of assets and reassessment of liabilities is done with the help of a new account called lsquoRevaluation Accountrsquo Sometimes this account is also called as lsquoProfit amp Loss Adjustment Acrsquo If there is a loss due to revaluation revaluation account is debited and if the revaluation results in a profit the revaluation account is credited The following journal entries made for this purpose are

(i) For increase in the value of assetsAsset Ac Dr (individually)To Revaluation Ac(ii) For decrease in the value of AssetRevaluation Ac Dr (individually)To Asset Ac[Decrease in the value of assets](iii) For increase in the value of LiabilitiesRevaluation Ac Dr (individually)To Liabilities Ac[Increase in the value of Liabilities](iv) For decrease in the value of LiabilitiesLiabilities Ac DrTo Revaluation Ac[Decrease in the value of Liabilities](v) For unrecorded AssetsAsset Ac [unrecorded] DrTo Revaluation Ac[Unrecorded asset recorded at actual value](vi) For unrecorded Liability Revaluation Ac DrTo Liability Ac [unrecorded][Unrecorded Liability recorded at actual value](vii) For transfer of gain on revaluationRevaluation Ac DrTo Existing Partnerrsquos CapitalCurrent Ac[Profit on revaluation transferred to capital account in existing ratio](viii) For transfer of loss on revaluationExisting Partnerrsquos CapitalCurrent Ac DrTo Revaluation Ac[Loss on revaluation transferred to capital account in existing ratio](a) When revaluation account shows gain Revaluation Ac DrTo Partnerrsquos Capital Ac (Old Profit Sharing Ratio)(Profit on revaluation credited to Partnerrsquos Capital Ac)(b) Above entry is reversed when revaluation account shows loss Partners Capital Acs (Old Profit Sharing Ratio) DrTo Revaluation Ac(Loss on revaluation debited to Partnerrsquos Capital Acs)

Proforma of Revaluation Account is given as under

Revaluation Account

Dr Cr Particulars ` Amount Particulars ` Amount To Decrease in value of assets By Increase in value of assets To Increase in value of liabilities By Decrease in value of liabilities To Unrecorded liabilities By Unrecorded assets To Gain on Revaluation (Transferred) By Loss on Revalution (Transferred)

ECO ndash12 2742020Topic- ELASTICITY OF DEMAND

CHAPTER - ELASTICITY OF DEMANDMEANINGDemand for a commodity is affected by many factors such as its price price of related goods income of its buyer tastes and preferences etc Elasticity means degree of response Elasticity of demand means degree of responsiveness of demand Demand for a commodity responds to change in price price of related goods income etc So we have three dimensions of elasticity of demandDIMENSION OF ELASTICITY OF DEMAND TYPES OF ELASTICITY OF DEMAND

Price elasticity of demand Income elasticity of demand Cross Elasticity of demand

Price elasticity of demand Price elasticity of demand means degree of responsiveness of demand for a commodity to the change in its price For example if demand for a commodity rises by 10 due to 5 fall in its price Price elasticity of demand (ep)=Percentage change in quantity demanded Percentage change in price of the commodity = 10 ( -)5 = ( - )2Note that ep will always be negative due to inverse relationship of price and quantity demanded

(ii) Income elasticity of demand Income elasticity of demand refers to the degree of responsiveness of demand for a commodity to the change in income of its buyer Suppose income of buyer rises by 10 and his demand for a commodity rises by 20 then Income elasticity of demand (ey)= change in quantity demanded change in price of the commodity =20 10 = 2

Cross Elasticity of demandCross elasticity of demand means the degree of responsiveness of demand for a commodity to the change in price of its related goods (substitute goods or complementary goods) Suppose demand for a commodity rises by 10 due to 5 rise in price of its substitute good then Cross elasticity of demand (ec) = change in quantity demanded change in price of related good = 10 2 = 5 (Tastes and preferences cannot be expressed numerically So elasticity ofdemand cannot be numerically expressed)

  • Chapter 1 Force (Summary)
  • Distinguish between external sovereignty and internal sovereignty
    • NAND Gate
      • Logic diagram
      • Truth Table
        • NOR Gate
          • Logic diagram
          • Truth Table
            • XOR Gate
              • Logic diagram
              • Truth Table
                • XNOR Gate
                  • Logic diagram
                  • Truth Table
                      • Physics
                      • Chapter 1 Electric Field ( Electric Dipole) (Summary)
Page 4:  · Web viewWe all know that Nouns are divided into two parts: common noun and proper noun.Apart from common and proper noun, we will also study about collective noun and compound

Class III

Subject ndash Eng language Topic ndash ch 3 NOUNS

NOUNS

Nouns are the words that names people place animal and things Around us we see so many things Everything that we see has some name Example- computer blackboard chalk book teacher house cat dog etc

In this chapter we will learn about ndash

Common NounProper NounCollective noun

Common Noun

Common Noun are names of people place animal things in general For example girl boy hospital park ball car etc

Proper noun

Proper nouns are the specific or particular name of person place animal and thingsExamples- Mina Mt Everest Delhi Tommy etc Proper noun is always capitalized

Uses of Proper nouns- They are special names of

people such as RitaJackRam etc Countries citiesstates such as IndiaKolkataWest Bengal Mountains rivers seas lakes such as Mt Everest the Ganga the Indian Ocean Dal

Lake

Subject social studies

Topic My family and I

A family is a group of people who are related to each other by birth Members of a family always give love and support to each other

There are different kinds of families ndash nuclear family joint family and single parent family

Nuclear family means where father mother and kids stay together

Joint family means where many members of the family stay together For example grandfather grandmother father mother uncle aunt cousins live together

Single parent family means where the kids stay with any one of the parent like father or mother

It is always a fun when whole family spends time together example going to a picnic because it creates a strong bond among each other

Class IV

Subject English language Topic Nouns Compound nouns

We all know that Nouns are divided into two parts common noun and proper nounApart from common and proper noun we will also study about collective noun and compound nouns A common or proper noun that is the name of a collection or group of similar things or persons is known as a collective Noun Example crowd committee herd United Nations etc Example Mother has lost her bunch of keysThe word bunch is a collective noun

1 A herd of cows was grazing in the fieldHerd- collective noun

2 All living creatures need food to surviveCreatures- collective noun

3 The Ganga is a river sacred to IndiansIndians- collective noun

Compound noun some common and proper nouns are made up of more than one word They are known as compound nouns or simply compounds Example newspaper (news+paper) ashtray (Ash+tray) plaything (play+thing) etc1 My penfriendlives in a town in South AfricaPenfriend South Africa- compound noun

2 My grandparents live in New DelhiGrandparents New Delhi- compound nouns

3 Please do not park your cars in the drivewayDriveway- compound noun

Subject Social studies Topic A peep into the past

The story of the past is called history It is a journey across time and space that takes us to another period of time where people lived differently It is believed that the way of living eating clothes art were all different from what we have today We need to study history becauseIt tells us about the important people of the pastIt helps us to gain knowledge about the past and the present and help us to understand themHistory guides us to set an aim and warns us not to repeat the same mistakesIt helps us to understand the development of mankindIt helps to developessential skills to become a better person

Division of the pastHistory is so vast that it is divided into two parts ndash BC (Before Christ) and AD (Anna Domini) This helps us to understand about the past clearly Indian history and itrsquos civilisation are the worldrsquos oldest civilisation Indian history is divided into ancient medieval and modernCivilisation mean the stage of human social and cultural development

Subject COMPUTER Chapter-2 (MS WINDOWS)WINDOWS IS AN OPERATING SYSTEM THAT HELPS THE USER TO OPERATE AND CONTROL THE OVERALL ACTIVITIES OF A COMPUTER SOME POPULAR VERSIONS OF WINDOWS ARE WINDOWS 98 WINDOWS 7 WINDOWS 8 AND WINDOWS 10

DESKTOP THE FIRST SCREEN THAT WE SEE WHEN THE OPERATING SYSTEM (WINDOWS)HAS FINISHED LOADING IS KNOWN AS DESKTOP

THE FEATURES OF DESKTOP ARE bullIT IS WHERE ALL THE ICONS OF THE DIFFERENT APPLICATIONS ARE DISPLAYEDbullIT IS THE HOME FOR ALL SHORTCUTS PROGRAMS CAN BE QUICKLY OPENED BY CLICKING ON THE RESPECTIVE SHORTCUTSbullIT HOLDS VARIOUS INFORMATION SUCH AS TIME AND DATE

ICONSICONS ARE SMALL PICTURES OF DIFFERENT APPLICATIONS THAT ARE PRESENT AND INSTALLED IN THE COMPUTER FOR EXAMPLE COMPUTER ICON AND RECYCLE BIN ICON

DESKTOP GADGETS A DESKTOP GADGET IS A SMALL APPLICATION OR A WIDGET THAT RECIDES ON A COMPUTERS DESKTOP FOR EXAMPLE CLOCK CALENDAR ETC_______________________________________________________________________________________________

Class VSubject Socialal studiesTopic Evolution of man

Millions of years ago the earth looked much different as it is seen todayIt was totally different and it was very difficult to live Everything looked very different plants animals and humans everything looked very different from what they are now All the changes took place gradually with the changing environment Earliest ancestors of the humans were known as the Ramapithecus who lived about 14 million years ago They looked like ape that walked upright and weighed about 14kg With the passage of time there was a gradual development in the structure posture features and teeth of the early humansIt is said that the apesthat lived on treetops started living on the ground and stood on two limbs are called hominids They used their limbs to hold things and make tools Finally they changed to homosapiens mean wise human We humans belong to this group Life of early humansThe early men were hunter ndash gatherers They moved from place to place in search of food water and shelter They hunted animals and ate them raw gathered fruits nuts roots and seeds for eating They didnrsquot know how to build houses so they used live on treetops or in the caves or rock shelters In summers they remained uncovered but in winter they used to cover themselves by using bark of trees leaves and animal skins

Subject English language Topic Adjectives Interrogative and Emphasizing

Interrogative adjectiveAn interrogative adjective is used with a noun that it qualifies to ask a question The word interrogative mean to ask questions Example Which part of the world do you come fromThe word lsquoWhich is an interrogative adjectivesEmphasizing adjectivesSometimes we use adjectives to emphasize the effect of what we say Words which are used to emphasize a nounExample Rita came late and had to stand at the very end of the lineThe word Veryis an emphasizing adjective It actually qualifies or emphasizethe noun end

1lsquoDo not listen to what Ravi says for he is a complete foolrsquo said RohanComplete- emphasizing adjective

2Kiran is a great supporter of non violenceGreat- emphasizing adjective

3rsquoWhat crime has the prisoners committedrsquo asked the judgeWhat- interrogative adjective

4Whose book is that lying on the tableWhose- interrogative adjective

Subjects COMPUTER CHAPTER-1 (CHARACTERISTICS OF A COMPUTER)A COMPUTER SYSTEM IS A ONE THAT CONSISTS OF ALL THE SOFTWARES HARDWARES CPU AND MEMORY DEVICES SOME IMPORTANT TERMSTHE COMPUTER WORKS ACCORDING TO THE COMMANDS OR INSTRUCTIONS THAT WE GIVE

A SET OF INSTRUCTIONS IS CALLED THE PROGRAM A SET OF ONE OR MORE PROGRAMS IS CALLED A SOFTWARE THE PHYSICAL COMPONENTS OF A COMPUTER ARE CALLED HARDWARE

FOUR OPERATIONS OF A COMPUTER SYSTEM INPUTINPUT IS SUPPLIED TO THE COMPUTER WITH THE USE OF A KEYBOARD A MOUSE A MICROPHONE

OR OTHER INPUT DEVICES PROCESSING PROCESSING IS DONE INSIDE THE COMPUTER IN THE CPU PROCESSING IS THE CONVERSION

OF THE INPUT INTO THE OUTPUT STORAGE STORAGE REFERS TO THE SAVING OF INFORMATION FOR LATER USE INFORMATION IS STORED

IN THE MEMORY OF A COMPUTER THERE ARE TWO TYPES OF COMPUTER MEMORY PRIMARY MEMORY AND SECONDARY MEMORYPRIMARY MEMORY IS FAST SMALL AND EXPENSIVE THE TWO TYPES OF PRIMARY MEMORY ARE RANDOM ACCESS MEMORY(RAM) AND READ ONLY MEMORY (ROM) RANDOM ACCESS MEMORY (RAM)THIS MEMORY IS VOLATILE IN NATURE WHICH MEANS THAT THE

INFORMATION STORED IN IT IS AUTOMATICALLY ERASED WHEN THE COMPUTER POWER IS TURNED OFF

READ ONLY MEMORY (ROM) THIS TYPE OF MEMORY IS NON VOLATILE IN NATURE WHICH MEANS THAT DATA IS NOT LOST WHEN THE COMPUTER POWER IS TURNED OFF

OUTPUT OUTPUT IS THE RESULT PRODUCED BY A COMPUTEROUTPUT MAY BE VIEWED ON THE MONITOR SCREEN HEARD THROUGH SPEAKERS PRINTED THROUGH PRINTER ON PAPERS OR STORED IN THE MEMORY FOR LATER USE

Class VI

GEOGRAPHY CHAPTER 1 REPRESENTATION OF GEOGRAPHICAL FEATURES

GLOBE AND MAPS

GLOBE- A Globe is a three ndashdimensional model of the Earth

EQUATOR ndash An imaginary line runs through the centre of the earth from east to west

NORTHERN HEMISPHERE ndash the half of the north of the equator is called Northern Hemisphere

SOUTHERN HEMISPHERE ndash the half of the south of the equator is called Southern Hemisphere

LATITUDE - Imaginary parallel lines running on the globe from east to west

LONGITUDE ndash The imaginary lines semicircle in nature running from the North Pole to South Pole Also called meridian

MAP ndash Representation of the Earth as a whole or parts of it drawn on a flat surface to a scale

SKETCH ndash A rough drawing of a place not drawn to a scale

PLAN ndash A plan is prepared to show more details about a small area

COMPUTER - VI CHAPTER-6 (INTRODUCTION TO WINDOWS 10)

THE MOST RECENT VERSION OF WINDOWS IS WINDOWS 10 IT WAS RELEASED IN 2015

WINDOWS 10 IS DESIGNED TO WORK FOR YOU AND NOT YOU WORKING FOR IT THE WINDOWS 10 OPERATING SYSTEM INCLUDES A NUMBER OF NEW FEATURES LIKE IMPROVED PERFORMANCE ON MULTI CORE PROCESSORS IMPROVED G U I DATA SECURITY AND BETTER BOOTING PERFORMANCE

WINDOWS 10 HAS 1187 WITH WEARING FEATURES SETS AND INTENDED HARDWARE

THE VARIOUS EDITIONS AVAILABLE FOR WINDOWS 10 ARE AS FOLLOWS

WINDOWS 10 HOME

WINDOWS 10 HOME IS THE EDITION OF WINDOWS 10 THAT CONTAINS THE LEAST NUMBER OF FEATURESWINDOWS 10 HOME IS THE CONSUMER FOCUSED DESKTOP EDITION WITH SUPPORT FOR BOTH PC AND TOUCH ENABLED TABLETS WINDOWS 10 HOME IS AVAILABLE IN BOTH 32 BIT AND 64 BIT VERSIONS

WINDOWS 10 PRO

WINDOWS 10 PRO HAS FEATURES ESSENTIAL FOR SMALL OR MEDIUM BUSINESSESIT HAS A LARGE NUMBER OF EXTRA FEATURES TO MEET THE NEEDS OF SMALL BUSINESSES

WINDOWS 10 ENTERPRISE

WINDOWS 10 ENTERPRISE IS BUILT ON WINDOWS 10 PROIT HAS FEATURES WHICH ARE DESIGNED TO MEET THE NEEDS OF MEDIUM AND LARGE ORGANISATIONSTHIS WINDOWS EDITION TARGETS THE ENTERPRISE SEGMENT OF THE MARKET

WINDOWS 10 MOBILE

WINDOWS 10 MOBILE IS DESIGNED FOR CUSTOMER ORGANISATION THAT ARE USING THE WINDOWS 10 PLATFORM ON SMARTPHONES AND SMALL TABLETS

WINDOWS 10 EDUCATION

WINDOWS 10 EDUCATION IS DESIGNED TO MEET THE NEEDS OF STAFF ADMINISTRATORS TEACHERS AND STUDENTS OF SCHOOLS

WINDOWS 10T EDITION

WINDOWS 10T EDITION OF WINDOWS 10 IS DESIGNED FOR USE IN SMALL FOOTPRINT AND LOW COST DEVICES

Subject English language

Topic Noun kinds

NOUN KINDS

Noun is a naming word Nouns are names of people places animals or things

For example Ashley Mumbai tiger pencils

Kinds of nouns

Proper noun the name of a particular person place or thing is called proper nounExample

1 The Himalayas stand to the north of India2 Monika is the school captain

Common noun announce that names people place or thing in general is called a common noun Example

1 Kalidas was the greatest dramatist of India2 William Shakespeare is Englands national poet

Collective Noun A collective noun is the name of a collection of people or things taken together and spoken of as a whole Example

1 The feet were completely destroyed in the fierce attack2 The crew revolted against Captain Grand

Abstract Noun An abstract noun is the name of some quality state or idea Example1 Diversity Indias biggest strength2 Patience is a virtue

_____________________________________________________________________________________

Class VII

Subject English 1 Topic Articles

ARTICLES

There are three articles ------ A An The

1 Indefinite articles- The Articles a and an are called Indefinite Articles They do not point to a particular person or thing They are used with singular countable nouns It does not identify a specific noun

Uses of Indefinite articles A

Before a singular noun beginning with a consonant or a vowel with a consonant sound For example a cattle a university a one way track

Before a proper noun which is either unfamiliar or holds a special meaning For example For example A Mahesh Sharma wants to see youNeela wants to be a Tendulkar when she grows up

With a number beginning with a consonant sound For example a ten- rupee note a hundred years

Before half when half follows a whole number or after half when it isnrsquotExample one and a half litres (before) Half a litre (after)

With an expression of quantity For example a lot of time a dozen oranges a little while In exclamation before nouns Example Such a mess What a pity

Uses of Indefinite articles An

Before a singular noun beginning with a vowel sound Example an ant an egg an umbrella Before an abbreviation beginning with a vowel or a consonant with the vowel sound

Example an MP an MA Before the word beginning with a silent h Example an heir an hour an honour

Note We do not say a milk or a lemonade because they are uncountable nouns ever we see a cup of milk or a glass of lemon and

Subject GEOGRAPHY

CHAPTER 1 REPRESENTATION OF GEOGRAPHICAL FEATURES

TOPIC- TOPOGRAPHICAL MAPS

Topographical Maps- topographical maps are small-scale maps with detailed depiction of both natural and human-made features

Conventional colours-different colours used in a map are known as conventional colours

TYPES OF SCALE-

Verbal Statement- Scale written in a statement from like 2 cm to a Km is called verbal statement It means that 2cm on the map represents 1 km on the Earth

Representative Fraction- Scale can also be written as a representative fraction (RF)

Example 150000 it means 1cm on the map represents 50000cm or frac12 km on the ground It shows the ratio between map distance and ground distance

Linear Scale Scale can be drawn on a line to show map distance equivalent to ground distance A line is divided into equal parts Each parts represents the actual distance on the ground in mkm

SUBJECT-COMPUTER

CHAPTER-1 (COMPUTER FUNDAMENTALS)

COMPUTER LANGUAGES

THE THE TERM COMPUTER LANGUAGE REFERS TO A SYSTEM OF RULES AND SYMBOLS THAT ARE DESIGNED TO GIVE INSTRUCTIONS TO A COMPUTERTHE COMPONENTS OF A COMPUTER SYSTEM CANNOT PERFORM A TASK BY THEMSELVES THEREFORE THEY HAVE TO BE GIVEN INSTRUCTIONS TO PERFORM ANY TASKCOMPUTER LANGUAGES ARE USED TO CREATE PROGRAMS USING LOGIC BASED ALGORITHMSTHE RULES OF A COMPUTER LANGUAGE IS

KNOWN AS SYNTAX WHEREAS THE TERM SEMANTICS REFERS TO THE MEANING OF LANGUAGESCOMPUTER CANNOT DEVELOP ANY PROGRAMMING LANGUAGE BY THEMSELVES AND CANNOT THINK INTELLIGENTLY UNLESS THEY ARE ASSISTED BY THE HUMAN BEINGS THEREFORE THE PROGRAMMERS DEVELOP A SET OF METHODS AND TECHNIQUES A SET OF INSTRUCTIONS USED TO PERFORM A SPECIFIC TASK IS CALLED A PROGRAM

A PROGRAMMING LANGUAGE IS ALSO KNOWN AS A COMPUTER LANGUAGE CODED BY PROGRAMMERS TO WRITE INSTRUCTIONS FOR A COMPUTERTHE COMPUTER TAKES THESE INSTRUCTIONS AS INPUT AND PRODUCES THE DESIRED OUTPUT

TYPES OF COMPUTER LANGUAGE

THE COMPUTER LANGUAGE CAN BE BROADLY DIVIDED INTO TWO CATEGORIES

LOW LEVEL LANGUAGES HIGH LEVEL LANGUAGES

LOW LEVEL LANGUAGES

A LOW LEVEL PROGRAMMING LANGUAGE REFERS TO THE LANGUAGE THAT IS UNDERSTOOD BY A COMPUTER DIRECTLYTHE PROGRAMMER MUST HAVE AN IN-DEPTH KNOWLEDGE OF DIFFERENT COMPUTERS TO WRITE PROGRAMS IN A LOW LEVEL LANGUAGE THE TWO TYPES OF LOW LEVEL LANGUAGES ARE MACHINE LANGUAGE AND ASSEMBLY LANGUAGE

MACHINE LANGUAGE

COMPUTER HARDWARE UNDERSTANDS ONLY MACHINE LANGUAGE AS IT IS THE FUNDAMENTAL LANGUAGE OF A COMPUTER WHICH UNDERSTANDS ONLY THE TWO DIGITS 0 AND 1 KNOWN AS THE BINARY DIGITSIT IS A SYSTEM OF INSTRUCTIONS EXECUTED DIRECTLY BY THE CPU WITHOUT ANY TRANSLATION AND HENCE IS EXECUTED QUICKLY AS COMPARED TO PROGRAMS IN OTHER LANGUAGES PROGRAMS IN MACHINE LANGUAGE NEED DIFFERENT BINARY LANGUAGE PROGRAMS TO SOLVE THE COMPLEX TASKSPROGRAMMERS OFTEN FIND IT DIFFICULT TO WRITE PROGRAMS IN THIS LANGUAGE

ASSEMBLY LANGUAGE

AN ASSEMBLY LANGUAGE IS A LOW LEVEL PROGRAMMING LANGUAGE BUT THE INSTRUCTIONS ARE WRITTEN IN THE FORM OF WORDS KNOWN AS MNEMONICS TO CONVERT AN ASSEMBLY LANGUAGE INTO TO MACHINE CODE WE REQUIRE A UTILITY PROGRAM WHICH IS KNOWN AS AN ASSEMBLER ASSEMBLY LANGUAGE PROGRAMS CANNOT BE EXECUTED DIRECTLY BY A COMPUTER THEREFORE THEY ARE SLOWER THAN THE MACHINE LANGUAGE PROGRAMSIT IS EASIER TO WRITE PROGRAMS IN THE MACHINE LANGUAGE THEN AND IN ASSEMBLY LANGUAGE

_______________________________________________________________________________________________

CLASS-VIII

SUBJECT-COMPUTER CHAPTER - Operating system and graphical user interface Role and functions

Need of operating system

It co-ordinates different hardware and software componentsof a computer system It supervises the various actions of the computer system and enables the computer to work in a effective

manner It helps in smooth functioning of various peripherals

In a multi-tasking operating system it determines the order and time to be allowed for each application before giving another application a turn

It sends messages to the system operator about the status of operation or any error that may have occurred while running the application

What is operating system

An operating system is an integrated system of programs that manages various resources and the overall operation of the computer system It is designed to support various activities of computer system in a systematic way

Role of an operating system-

Operating system enables the user to use the system effectively An operating system manages various application that runs on a computer and shares computerrsquos resources User interacts with operating system through command line interface and graphical user interface

Function of an operating system -

Booting the computer it is the start up procedure of a computer system Loading theprograms in the memory - when system is ready the operating system loads certain program

automatically Manages resources it manages between the hardware and software resources Detecting and correcting errors- if the supporting hardware or software doesnrsquot works properly then the

operatingsystem tries to rectify it Ensuring data security programs and data donot interfere with each other Maintaining the internal clock of system - maintain internal clock of system when system is shut down

SUBJECT-GEOGRAPHY CHAPTER 1 REPRESENTATION OF GEOGRAPHICAL FEATURES THROUGH CONTOURSTopographical maps- topographical maps are small-scale maps with detailed depiction of both natural and human-made features

Contour lines- contour lines are imaginary lines drawn on a map joining places having the same height above the mean sea level

Contour linesContour interval- It is an interval at which contour lines are drawnIndex contour- At every 100-m interval a thick brown contour line is called index contour It is drawn for calculating height

Spot heights- The spot heights show heights in metres above the mean sea level

SUBJECT- English Language CHAPTER - The Sentence

Complex sentence-

We have seen that a Complex Sentence consists of a Principal Clause with one or more Subordinate Clauses

We have also learnt that there are three kinds of Subordinate Clauses The Adjective Clause the Adverb Clause and the Noun Clause

1) THE ADJECTIVE CLAUSE

An ADJECTIVE CLAUSE does the work of an ADJECTIVE It qualifies a noun or pronoun In the following examples in each set two commonly patterned sentences are compared one with an ADJECTIVE and the other with an ADJECTIVE CLAUSE ( The Adjective or Adjective Clause qualifies the Noun placed in the box)

a) He is a lazy boy (Adjective)

b) He is a boy who is lazy (Adjective Clause)

a) It is a beautiful place for the tourists (Adjective)

b) It is a place where tourists come for the scenic beauty (Adjective Clause)

a) We have enough funds for the work (Adjective)

b) We have funds which would be enough for the work (Adjective Clause)

Convert the sentence from simple to complex sentence-

1) We believe his honesty Ans- We believe that he is honest

2) This is the birth place of RamaAns- This is the place where Rama was born

____________________________________________________________________________________________

Class IX

Subject English Language

Topic Preposition

Date 270420

PREPOSITIONS

A preposition is a word placed before a noun or a pronoun to show in what relation the person or thing denoted by it stands in regard to something else

EXAMPLE

There is a clock on the wall

The preposition on shoes relationship between clock and the wall

Smitha is afraid of lizards The man jumped off the bus

Here the preposition of shows the relationship between afraid and lizards

The preposition off shows the relation between jumped and bus

The noun or the pronoun which follows a preposition is called its object

So in the first Example wall is the object of the preposition on

A preposition can have more than one object For example

The plane flew overhouses and meadows

Prepositions are used to express a number of relationship including time locationmannermeans quantity purpose and state or condition

Points to remember

Preposition joins a noun to another noun or a pronounThere is a cow in the field

A preposition joints a noun to a verbThe cat runs after the rat

Preposition can have two or more than two objectsThe road runs over hills and plain

A preposition also joins a noun an adjectiveHe is fond of tea

Generally a preposition comes before an object Sometimes it comes even after an object asWhat are you looking atThis is the house I live in

Subject- Computer Application

Chapter 2 Introduction to Java

Java API An application programming interface (API) in the context of Java is a collection of prewritten packages classes and interfaces with their respective methods fields and constructors

Byte Code Java bytecode is the result of the compilation of a Java program an intermediate representation of that program which is machine independent The Java bytecode gets processed by the Java virtual machine (JVM) instead of the processor JVM The Java Virtual Machine (JVM) is the runtime engine of the Java Platform which allows any program written in Java or other language compiled into Java bytecode to run on any computer that has a native JVM

Platform A platform is the hardware or software environment in which a program remains

Java platform The Java platform differs from most other platforms in that itrsquos a software only platform that runs on top of other hardware-based platforms

Applet and Application The fundamental difference between the two Java programs is that an application program is designed to run on a stand-alone machine whereas an applet is a web-version of an application which is used to run a program on a web browser

WORA ldquoWrite once run anywhererdquo (WORA) or sometimes write once run everywhere (WORE) is a slogan created by Sun Microsystems to illustrate the cross-platform benefits of the Java language

Class XSubject Topic Summary Execution

ECONOMICSFACTORS OF PRODUCTION Sub-topiclsquoLABOURrsquo

We shall start our class by discussing the topic taught in the last class

lsquo DIVISION OF LABOURrsquo- By division of labour we mean specialization in workIt refers to splitting up the work of labour involved in the production of a particular commodity into several parts and each part and sub-part is performed by a specialist

Now let us start by the Advantages of Division of labouraIt increases the level

Questions

1 Differentiate between Product-Based division of labour and Process-Based division of labour

Product-Based division of labour

Process-based division of labour

It is also known as simple (or occupational) division of labourUnder it everybody performs a particular occupations The entire is done by the same person

When a person or group of persons undertakes a specialised function which is supplementary to the production of final commodity and service This is also as complex division of labour

It is simple It is complex

of productionbSince the product is produced by an expert workerbest quality of product is producedcIt saves time and toolsd it promotes inventions in the methods and techniques of productioneIt leads to reduction in costs fAll workers get work according to their abilities and choices

Now let us discuss the disadvantages of Division of labour

a Since many workers are involved in the production of a commodity no one has the sense of responsibility

b The constant and repetition of the same work again and again make the work monotonous

c Division of labour facilitates production on large scale Hencethere is fear of over production

d Because of territorial division of labour some areasregions become more developed than others

It is based on labour-intensive techniques of production

It is based on capital-intensive techniques

It is generally found in small enterprises

It is generally found in large enterprises

Example Indian farmers doing all farm activities

Example A modern garmet factory where one person takes the measurementanother does the cuttingsome sew the clothes while a few workers button them and other iron them

2 What are the advantages of Division of labouraIt increases the level of productionbSince the product is produced by an expert workerbest quality of product is producedcIt saves time and toolsd it promotes inventions in the methods and techniques of productioneIt leads to reduction in costs fAll workers get work according to their abilities and choices

3Discuss the disadvantages of Division of laboura Since many workers are involved in the

production of a commodity no one has the sense of responsibility

b The constant and repetition of the same work again and again make the work monotonous

c Division of labour facilitates production on large scale Hence there is fear of over production

d Because of territorial division of labour some areasregions become more developed than others

English 1 Transformation of sentences

Sentences A sentence is a group of words which makes complete sense

a Assertive sentences

Exercise 6Rewrite the following sentences according to the instructions given below without changing their meanings

1 As soon as he saw the beer he jumped into

b Imperative sentences

c Interrogative sentences

d Exclamatory sentences

Sentences can be changed from one grammatical form to another without changing the meaning of the sentence This is known as transformation of sentences

the river ( Begin No sooner)2 None but brave deserve the fair (Begin the

bravehellip)3 This box is too heavy for me to lift ( Use so hellip

That instead of too)4 No one other than a king can live like James

Luxurious ( Begin only James)5 Oh for the wings of a dove (Begin I wishhellip)

Math Topic Commercial MathematicsChapter Shares and Dividends

Study item Discuss about shares and Dividends1) What is share

Ans To start any big business (company or Industry) a large sum of money is needed But it is not possible for an individual to invest such a large amount Then some persons interested in the business join together and from a company They divide the estimated money required into small parts Each such part is called a share

2) What do you mean by the term shareholder

Ans A person who purchases one or more shares is called shareholder3) Some terms related with a share

(i) Nominal value or face value or printed value The original value of a share is called its nominal value or face value or printed value

Note The nominal value of a share always remains same(ii) Market value or cash value The price of a share at any

time is called its market value or cash value

Note The market value of a share changes from time to time(iii) At par If the market value of a share is the same as its

nominal value the share is called at par(iv) At Premium or above Par If the market value of a share

is more than its nominal value the share is called at premium or above par

Example If a share of Rs 100 is selling at Rs 150 then it is said to be selling at a premium of Rs 50 or Rs 50 above par

(v) At Discount or below par If the market value of a share is less than its nominal value the share is called at discount or below par

Example If a share of Rs 100 is selling at Rs80 then it is said to be selling at a discount of Rs 20 or at Rs 20 below par

4) What is Dividend

Ans The profit which a shareholder gets for hisher investment from the company is called dividendNote (i) The dividend is always expressed as the percentage of the face value of the share(ii) The dividend is always given( by the company ) on the face value of the share

irrespective of the market value of the shareBENGALI(2ND LANGUAGE)

ldquoদেবতোর জণমrdquoলিবরোম চকরবত

পরথম লিসর পোঠ-চোর পসথ একটি পোথর লিবপলি ঘটোয় দেক যোতোয়োসতর পসথ পরলিতলিয়ত ওই পোথসর দেো োচট দেসত একলি দেতো দেক দেো োচট দেসয় দেবোমো সয় রোসতোর মোস লি0টসক পস1 লিবপরীত লিক দেথসক আো একটি দেমোটর োলি1 চোসকর কষতোয় পরোসরণ দেবোসচ যো লিকনত পরলিতবোর এমরণ দেৌভোয দেসকর োও সত পোসর তোই লিতলি দেকোো দেজোো1 কসর পোথরটিসক উপস1 দে8স পোথর উপস1 দে8োর ময় এক দেকৌতী জতো দেকসক পরশন কসর দেয লিতলি দেকোসো দেবতোর আস দেপসয়স0রণ লিকো লিকনত দেক বস লিতলি দেকোসো দেবতোর আস পোলি দেক উপলিত ক জতোর উসltসয বস কোরও ইস= স পোথরটি লিসয় দেযসত পোসর এর পর দেথসক দেক দেযসত আসত পোথরটি দেক দেসত পো একলি দেক কষয করস দেকউ পোথরটিসক ধসয়মস0 পলিরসকোর করস0 দেক ওই দেকৌতী জতোসক পোথসরর কোস0 বস থোকসত দেস এ0ো1ো আরও কষয কসর দেক ঠোৎ ওই লি1 পোথরটির োসয় লিোর োো-দেকউ পজো কসরস0 দেকৌতী দেোকটির আঙকো য় যলিসক উ পোথরটিসক লিরসয় দে8স তোর পর ঠোৎ একলি পোথরটির দেোোজ দেই দেক লিসয় দেস0 বো দেকোথোয় দেস0

বদোথ-

দেো োচট ndashচসত লিসয় দেকো লিক0র সE ধোককো দেস পস1 যোবোর উপকরমঅকসমোৎ- ঠোৎআতমমবর- লিসজসক লিয়নতরপ-পো লিপ0স প1োদেসতসসত- লিসপলিউৎোত- দেো1ো দেথসক উপস1 দে8োপরতযয় ndash লিবশবোপরতযোস- টতযোস ndash দৈবোসধসতোধলিসত- পরসপসরর পরলিত ব পরসয়ো করোপরসতরীভত- পোথসর পলিররণলিতবোনতঃকরসরণ- মস পরোসঅলিQৎ- অQো করসত ই=কদেোপ- বধমোর- দেবোস এমইতযোকোর- এইরকমরম- বময়পযসোভী-পসযর জয দেোভ আস0 যোরপোসথয় ঞচয়- পথচোর রচ জমোসোমোমোসরোস- ব ধম ধোসমর সE

তোর লি দেইhelliphellipTo be continued

Hindi 2nd lang सर क पद(सरदास)

सरदास शरी कषण भकति कावय क सरवशरषठ कगिरव ह इनक जनम और मतय क समय तथा सथान का मतभद हसरदास रवातसलय और शरार रस क अनयतम कगिरव ह इनक कावय म बालकषण क सौदय चपपल चषटा और गि7याओ की मनोहर झाकी मिमलती ह कषण और ोगिपयो क अननय परम का कतिचतरण ह सयो शरार की अपकषा उनक कावय म गिरवयो शरार का अमिBक गिरवषय और मारमिमEक कतिचतरण हआ हइन पकतियो म हम सरदास की भकति भारवनाओ का परिरचय मिमलता ह इनका सपण सगरह सरसार म गिनगिहत ह

1 जसोदा हरिर पालन झलारवहलरारवदलराईमलहारव रव जो ईसाई कछ ारव मर लाल को आई निनEदिदया काह ना आगिनसबारव त काह नाही बरवगिह आरवतोको कानहा बलारव

शबदाथ-हलरारव-गिहलती हदलराई - दलार पयार करती हमलहारव-पचकारती हनिनEदरिरया ndashनीदरवगिह-जलदी सअBर-होठमौन-चपसन-सकत

वयाखया- सरदास जी कहत ह गिक यशोदा माता बालक कषण को पालन म झल आती ह रवह उनह गिहलाती ह पयार करती ह मलहार जस कोई ीत ान लती ह और नीद स पछती ह गिक ह नीद तम मर लाल को आकर कयो नही सलाती तझ खाना बला रहा ह कभी कषण आख बद कर लत ह कभी आखफडफडान लत ह उनह सोता हआ जानकर यशोदा माता चप हो जाती ह और इशार म बात करन लती ह इसी बीच अकला कर कषण ज जात हतो गिफर यशोदा माता गिफर स ाना ान लती ह सरदास जी कहत ह गिक भरवान क दशन का सख दरवता और ऋगिष-मगिनयो को भी दलभ ह यही सख माता यशोदा को बडी सहजता स मिमल जा रही ह माता यशोदा बहत ही भागयशाली ह2)Continue to nexthellip

Physics

Chapter 1 Force

(Summary)

Question A body is acted upon by two forces each of magnitude F but in opposite directions State the effect of the forces if

(a) Both forces act at the same point of the body

(b)the two forces act at two different points of the body at a separation r

Solutions

(a) Resultant force acting on the body = 0

F ndash F = 0(b) The forces tend to rotate the body between two forces about the midpoint

Moment of forces = F times rFr

QuestionDefine moment of a couple Write its SI unit

Solutions

Moment of couple is equal to the product of both force and the perpendicular distance between the two forces

The SI unit of moment of couple is NmCommercial Studies

Advertising and sales

Business firms use several methods to

Questions1) What do you mean by advertising

promotion create demand of their product in the market and increase it sales Such methods comprises of advertising sales promotion personal selling and publicityToday we are going to discuss about one of such methods It is advertising

Meaning of advertisingAdvertising is a paid form of non-personal presentation for promotion of Ideas goods and services

Importance or merits of advertising Advertising has importance to manufacturer or traders to customer and to society as a whole

Today we will see how advertisement help the manufacturer or traders

Answer) Advertising is a means of how a company encourages people to buy their products services or ideas It is one element of marketing which also includes design Research and data mining

2) Mention any three features of advertisingAnswer)The main features of advertising are

i) It is impersonal form of presentation for promotion of products and services of Ideas

ii) It is issued by identified sponsor The advertisement contains the name of the advertiser

iii) It is a form of mass communication because the message is directed to a large number of persons simultaneously

3) Mention the main merits or importance of advertisement to manufacturer or tradersAnswer)

i) Introducing new product A business organization can introduce itself and its products to the public through advertising

ii) Increase the sale Advertising leads to increase the sale of existing product by entering into new markets and attracting new customers

iii) Create steady demand Advertising creates sustains regular demand by smoothening out seasonal and other fluctuations It enables regular production for the organisation

iv) Economics of scale Advertising facilitate mass distribution of goods and steady demand which lead to large scale and regular production

v) Goodwill Advertising helps in creating a good image of the firm and reputation for its products

Biology Chapter - 03Genetics

Today wewill start chapter and discuss about Genetics Gregor Mendel is known as father of genetics Before entering into Mendelrsquos experiment on Genetics we must know

Q1 Define the following termsi) Genetics Genetics is the study of

transmissionof body features from parents to offspringand the laws relating to such transmission

ii) Heredity It may be defined as transmissionof genetically based characteristics from parentsto offspring

iii) Character and traits Any heritable

Importance to TraderIntroducing new productIncrease the saleCreate steady demandEconomics of scaleGoodwill

some terms featureis a character The alternative forms of acharacter are called traitsex Character (Hair shape) - Traits (Curly straight)

iv) Homologous chromosomes A pair ofcorresponding chromosomes of the same shapeand size one from each parent

v) Genes Genes are the specific parts (DNA segments) of a chromosome which determinethe hereditary characteristicsNearly 30000genes present in human

vi) Alleles Alternative forms of a gene occupying the same position (locus) on homologouschromosomes and affecting the same characteristicbut in different ways

vii) Genotype ndash PhenotypeGenotype means of genes present in the cells of an organism Phenotype means the observable characteristic which is genetically controlled

viii) Mutation It is a sudden change in one or more genes or in the number or in the structure of chromosomes ex Sickle cell anaemia is a blood disease caused by a gene mutation

CLASS NOTES

Class XSubject Eng Literature (The Merchant of Venice ndash William Shakespeare)Topic Act IV Scene 1 Lines 01 to 34 ( Duke helliphelliphellip We all expect a gentle answer Jew) ate 27th April 2020 (2nd Period)

[Students should read the original play and also the paraphrase given in the school prescribed textbook]Summary Questions amp Answers

This scene may be termed as the catastrophe of the play It is the final unravelling of the complicated events which seem to threaten the happiness of Bassanio Portia and Antonio Right is justified to the fullest degree and malice falls into the trap prepared for others No one suffers here but Shylock but even then he receives a generous measure of mercy

o This is the Court-scene Initially we meet

(1)

DUKE I am sorry for thee thou art come to answer (Line 3-6)A stony adversary an inhuman wretchUncapable of pity void and emptyFrom any dram of mercy

(i) Who is addressed here Where is the person Why is the person there

Antonio is addressed hereAntonio is in the court of justice at VeniceAntoniorsquos trial is scheduled to be held here for his failure to meet the conditions of the bond he signed with Shylock

the Duke Antonio Shylock and Salerio Later we meet Bassanio Portia Gratiano and Nerissa

o The Duke says to Antonio that he has to face a very cruel opponent which Antonio admits and expresses his gratefulness to the Duke for his efforts to soften without result the heart of Shylock in order to be merciful to Antonio Antonio further says that he is ready to accept whatever cruel judgement the Court may award

o When Shylock appears in the court the Duke says that Shylock should change his decision of prosecuting Antonio and demanding the penalty specified in the bond out of consideration of the great misfortunes that Antonio has suffered If this is done by him (Shylock) the whole court would be gladdened by his merciful action

(ii) What is the Duke sorry for

The Duke is unable to change the mind of Shylock from his decision to get the bond forfeited even after he pleaded to Shylock Shylock stands firmly for his bond which when forfeited will allow him to take a pound of flesh from any part of Antoniorsquos body(iii) How does the Duke address Shylock`The Duke calls Shylock an adversary with a heart of stones He calls Shylock as an inhuman wretch without pity Shylock is quite lacking in the slightest quality of mercy (iv) How does Antonio reply to this

Antonio replies that he will meet the revenge of Shylock patiently He has prepared himself to suffer with a quiet spirit the utmost that Shylockrsquos tyranny and rage can do(v) What quality of the Duke is revealed here

The Duke is kind and benevolent He is ready to help Antonio He requests Shylock to free Antonio from the trial(vi) What are the terms of the bond that Antonio has signed

The terms of the bond that Antonio has signed were that if Antonio is unable to repay Shylock a certain sum of money specified on the paper on a certain date and in an agreed place the forfeit has to be paid The forfeiture will be an exact pound of Antoniorsquos flesh which Shylock will be a liberty to take from any part of Antoniorsquos body which pleases him

Class XI

STUDY MATERIAL

Class XISubject Eng Literature (The Tempest ndash William Shakespeare) Topic Act I Scene 2 Lines 88 to 132 (Prospero hellip Me and thy crying self) Date 27th April 2020 (3rd Period)

[Students should read the original play and also the paraphrase given in the school prescribed textbook]Summary Questions amp Answers

o Prospero now tells Miranda that he was the Duke of Milan He had been devoting himself more to studies than the affairs of the State His brother Antonio took advantage of this situation and with the help of Alonso the king of Naples seized upon him and her one midnight and shipped them in a frail bark so that they perished in the sea All this took place

(1)

MIRANDA I should sin (Line 118-132)

To think but nobly of my grandmother

Good wombs have borne bad sonsPROSPERO Now the condition

The King of Naples being an enemyTo me inveterate hearkens my brothers suitWhich was that he in lieu othrsquo premisesOf homage and I know not how much tribute

twelve years back

IMPORTANT PASSAGES EXPLAINED(Line 98-103)

PROSPERO helliphelliphelliphelliphelliphelliphellip

He being thus lorded

Not only with what my revenue yieldedBut what my power might else exact like oneWho having into truth by telling of it Made such a sinner of his memoryTo credit his own lie he did believeHe was indeed the duke

Prospero in telling the narrative of his past life here refers to his brother Antonio Prospero being with a studious bent of mind has left the administration of Milan on his younger brother Now Antonio being thus invested like a lord with all the powers derived from Prosperorsquos wealth and what the exercise of Prosperorsquos authority might secure for him regarded himself as a de facto Duke of Milan It is a well-known fact of psychology that a man who repeatedly tells a lie makes of his memory such a sinner against truth as to credit his own lie by the telling of it So Antonio by repeatedly saying to himself and others that he was the Duke came to believe that he was really the Duke Thus falsehood repeatedly asserted gained the force of truth for Antonio and he truly believed it

Should presently extirpate me and mine Out of the dukedom and confer fair MilanWith all the honours on my brother whereonA treacherous army levied one midnightFated to thrsquo purpose did Antonio openThe gates of Milan and ithrsquo dead of darkness The ministers for thrsquo purpose hurried thenceMe and thy crying self

(i) In the earlier lines of this scene what does Prospero tell about his intense interest What was the demand of his interest

In the earlier lines of this scene Prospero tells Miranda that he had an intense interest in the study of philosophy and magic arts Hence in order to improve his mind with this kind of study he kept himself isolated from worldly and state affairs His study was dearer to him than the applause and esteem that he could win from the public His study demanded too much solitude(ii) What forced Antonio to take an undue advantage over Prospero

Prosperorsquos indifferent attitude towards the statersquos affairs and his having boundless trust in Antonio gave rise to a boundless lust for power in Antoniorsquos mind Antonio felt that he must be the actual Duke instead of the part of the Duke he played Thus Antonio took an undue advantage of the situation to usurp Prosperorsquos dukedom(iii) Explain the following lines ldquoI should sin to think but nobly of my grandmother Good wombs have borne bad sonsrdquo

After hearing the treacherous act of her uncle Antonio Miranda says that Prosperorsquos mother was a noble lady and she cannot dishonour her memory by saying that the person named Antonio cannot be his (Prosperorsquos) brother She says that in honour of her grandmother she also cannot say that Antonio must have been begotten not by her grandfather but by some other man She finally concedes that it is known that good mothers have borne bad sons in their wombs and gave birth to them

(iv) Why did the King of Naples accept Antoniorsquos request to help him in usurping his dukedom What did Antonio propose to Alonso

Alonso who was the king of Naples accepted Antoniorsquos request in usurping Prosperorsquos dukedom because he (Alonso) was a sworn enemy of Prospero Antonio proposed that Alonso should immediately drive him (Prospero) and his offspring out of Milan and should confer the dukedom upon him (Antonio) with all the dignities which go with that

position In return Antonio promised that he shall give an annual tribute and also swore his allegiance to Alonso Also he agreed to hold the Dukedom of Milan as a subordinate to the state of Naples(v) How were Prospero and Miranda carried away from the city of Milan and what was the state of small Miranda at that time

In pursuance of the agreement settled between Antonio and Alonso an army of treacherous men was assembled One midnight when the occasion suited the will of destiny Antonio opened the gates of the city of Milan and in the death like silence of midnight Antoniorsquos agents who had been directed to execute his purpose carried Prospero and small Miranda away from the city in all haste They were then forced into a ship and carried some distance out to sea where they put them on a mere hulk of a boat without any rigging or ship-gear and abandoned them leaving them at the mercy of the roaring sea Miranda was a very small child of three years age and she was crying at that time

CLASS -XIDATE-270420Subject Topic Summary Execution

EVS Chapter 1 ndash Mode of Existence

Impact of mode of existence on resources

Q) Why resources are under pressure

Ans - Increase in the sophistication

of technology enabling natural resources to be extracted quickly and efficiently Eg in the past it could take long hours just to cut down one tree only using saws Due to increased technology rates of deforestation have greatly increased

The number of humans is increasing Cultures of consumerism Materialistic views

lead to the mining of gold and diamonds to produce jewelry unnecessary commodities for human life or advancement Consumerism also leads to extraction of resources for the

production of commodities necessary for human life but in amounts excessive of what is needed because people consume more than is necessary or waste what they have

Lack of awareness among the population is striking People are not aware of ways to reduce depletion and exploitation of materials

Accounts Cash Book Today we are going to start a new topic -Cash Book

The key terms used in this chapter are

bullCash book

bullSimple cash book

bullDouble column cash book bullTriple column cash book

bullPetty cash book

bullCash discount

bullContra entry

Here I will share you the meaning of each key terms

bullCash book Cash Book is a special purpose subsidiary book or journal in which cash received and cash payments are recorded

bullSimple cash book

It is a cash book in which only cash transactions are recorded It has only one column on each side

bullTriple column cash book

It is cash book which has three columns one column for each cash and Bankdiscount on each side of the cash book In this book both cash and Bank transactions are recorded together with discount allowed and received

bullPetty cash book

It is a cash book maintained for recording petty expenses

bullCash discount

Cash discount is the amount of discount received or allowed on cash payments and cash receipts Discount received is an income for the business while discount allowed isan expense

bullContra entry

It means transactions involving both cash and Bank Such transactions though recorded in the cash book are not posted into ledger The letter lsquoC is written in Ledger folio for contra entry

Business Studies

ENTREPRENEURSHIP

Now we shall discuss the second chapter

lsquoENTREPRENEURSHIPrsquo

Today before starting the chapter let us recall what

Questions

1What are the main characteristics of Intrapreneurship

Answer

The main characteristics of Intrapreneurship are

Corporate framework-it occurs within the framework of the same company

Semi-Autonomous-Intrapreneurship

we have read last day

Let s today start the class by recalling the last topic taught

Intrapreneurship is the process of discovering and exploring business opportunities within an existing company It involves launching new business ventures within the framework of a present corporation Intrapreneurship is also known as corporate entrepreneurship or corporate venturing

Now let us start with the characteristics of Intrapreneurship

The main characteristics of Intrapreneurship are

Corporate framework

Semi-Autonomous Lack of ownership Senior position Low risk taking Not own boss

Now let us discuss the meaning of enterprise

Enterprise means an undertaking or adventure that requires some innovation and investment and thus involves riskEnterprise always entails decision making coordination and risk bearing

involves crating amd nurturing a semi-autonomous business unit which may be a subsidiary a strategic business unit or a division

Lack of ownership-the intrapreneur is not the owner of the unitb he creates and nurtures

Senior position-he occupies a senior managerial position in the company

Low risk taking-An intrapreneur does not bear the full risk of failure

Not own boss-An intrapreneur is not his own bosss in legal termsHe enjoys the freedom and gets the required resources and support

2 How is Entrepreneur is different from Intrapreneur

The functions involved in both the entrepreneurship and intrapreneurship are by and large similar however there are several differences between the two

Point of distinction

Entrepreneur

Intrapreneur

status An independent business person

A senior executive within a company

Ownership Owner of

the business

An employeesometimes a share in ownership

Financing Responsible for raising finance for the business

Not responsible for raising the finance

Risk bearing

Bears the risk of the business

Does not bears the risk of the business

Reward Profit which is uncertain and irregularcan be loss

Fixed salary and fringe benefits

Need for security low high

3 What do you understand by enterprise

Answer Enterprise means an undertaking or adventure that requires some innovation and investment and thus involves riskEnterprise always entails decision making coordination and risk bearing

COMMERCE NATURE AND OBJECTIVES OF

BUSINESS

Today let us recall the last other two objectives of business by the chart given in the previous class

Firstly we would discuss Human Objectives

Business is run by people and for people Labour is a valuable business element

Human objectives of business are concerned with the well -being of labour

The human objectives are as follows

Labour welfare Developing human

resources Participative

management Labour

management cooperation

Questions

1 Explain the human objectives of a business enterprise

Answer

Business is run by people and for people Labour is a valuable business element

Human objectives of business are concerned with the well -being of labour

The human objectives are as follows

Labour welfare-Business must recognize the dignity of labour and human factors should be given the recognition

Developing human resources-Employees must be provided the opportunities for developing new skills and attitudes

Participative management-Employees should be allowed to take part in decision making process of business

Labour management cooperation-Business should strive for creating and maintaining cordial employer employee relations so as to ensure peace and progress in industry

Now let us discuss the national objectives of business

Optimum utilization of resources

National self- reliance Development of small

scale industries Development of

backward areas Control over pollution

2Explain the national objectives of a business enterprise

Answer

It is the duty of business to utilize the resources of the country properly the national objectives of business

Optimum utilization of resources ndashBusiness should use the nationrsquos resources in the best possible manner

National self- reliance-It is the duty of the business to help the government in increasing experts and in reducing dependence on imports

Development of small scale industries-Big business firms are expected to encourage growth of small scale industries which are necessary for generating employment

Development of backward areas-Business is expected to give preference to the industrialization of backward regions of the country

ECONOMICS

BASIC ECONOMIC CONCEPTS

SUB

TOPIC

Value

Wealth

Welfare

Today we shall start with a new topic of the same chapter ie lsquoValuersquo

Value of a commodity is defined as the valuation placed by a household on the consumption of this commodity

lsquoValuersquo has two different meanings and these are

a Value -in -use It refers to consumption value of a commodity It expresses the utility derived from the consumption of a particular commodity A necessity like water has a very high value ndashin ndashuse or

Question

1What is value

Answer

Value of a commodity is defined as the valuation placed by a household on the consumption of this commodity

2What is value-in use

Answer It refers to consumption value of a commodity It expresses the utility derived from the consumption of a particular commodity A necessity like water has a very high value ndashin ndashuse or consumption value

3What is value ndashin- exchange

Answer It relates to market value of a commodity

It is the rate at which a particular good or service can be exchanged for money

For example in barter system if a person is prepared to exchange 3 metres of cloth with 1 pair

consumption value

b Value ndashin-exchange It relates to market value of a commodity

It is the rate at which a particular good or service can be exchanged for moneyFor example in barter system if a person is prepared to exchange 3 metres of cloth with 1 pair of shoes then the value in exchange of 3 metres of cloth is 1 pair of shoesValue in exchange is the power of purchasing other goods In modern monetised economies the exchange value of goods are expressed in terms of money as prices

Now let us discuss the term lsquoWealthrsquo

Wealth refers to the stock of all those assets which are a source of income

Wealth is a stock concept

Wealth must possess the following features

a Utility It must possess utility or give some

of shoes then the value in exchange of 3 metres of cloth is 1 pair of shoes

Value in exchange is the power of purchasing other goods In modern monetised economies the exchange value of goods are expressed in terms of money as prices

4 What is wealth

Answer it refers to the stock of assets or goods which are a source of income and have personal or national ownership

5 What are the features of wealth

Answer The features of wealth are as follows

Wealth must possess the following features

a Utility It must possess utility or give some satisfaction

b Scarcity It must be limited in quantityc Transferability it should be transferable its

ownership can be transferred from one person to another person

d Exchange value It must possess exchange value

6 What is welfare

Answer

Welfare is defined as satisfaction and happiness a sense of well- being among the people

satisfactionb Scarcity It must be

limited in quantityc Transferability It

should be transferable its ownership can be transferred from one person to another person

d Exchange value It must possess exchange value

Now let us discuss the term lsquoWelfarersquo

Welfare is defined as satisfaction and happiness a sense of well- being among the people

Welfare is affected by factors like

a Consumption of goods and services

b Environment

c Family relations

d Degree of freedom

e Law and order situation

Mathematics Trigonometric equation

To find the general solution of the equation sinθ=0

When sin θ =0

Then θ= 0 π2π 3π-π -2π -3

i e when θ = 0 or an integral multiple of π

i e when θ= nπ where n is any integer

Therefore the general solution of the equation sin

Example1 Find the general values of θ which satisfy the equation sin2 θ =34

Solution sin2 θ= 34

Or sin θ = +34 or -34

Or sin θ = sin π3 or sin (-π3)

Therefore

θ = [nπ + (-1) n (π3)] or[ nπ+ (-1) n (-π3)]

= nπ +π3 or nπ-π3 where n= any integer

Example 2Find the values of θ which satisfy tan2 θ

θ=0 is θ= nπ where n is any integer

To find the general solution of the equation cos θ=0

When cos θ=0

Then θ=π2 3π2 5π2 -π2 -3π2 -5π3

i e when θ is an odd multiple of π2

i e when θ=(2n+1) π2 where n= any integer

Therefore the general solution of the equation cos θ =0 is θ= (2n+1) π2 where n= any integer

To find the general solution of the equation tan θ = 0

Clearly tan θ =0 implies sin θcos θ =0

Therefore θ = nπ

i e the general solution of the equation tan θ=0 is θ =nπ where n = any integer

To find the general solution of the equation cot θ =0

Clearly cot θ =0 implies (cos θsin θ) = 0

i e cos θ =0

Therefore θ = (2n+1) π2

Therefore the general solution of the equation cot θ =0 is θ = (2n+1) π2

Where n= any integer

To find the general solution of the equation sin θ= k (-

=13 -πleθleπ

Solution tan2 θ =13

Or tan θ = plusmn1radic(3) =tan(plusmnπ6)

θ=nπ plusmn π6 where n =any integer

If n=0 then θ=plusmnπ6

If n=1 then θ= π plusmn π6

If n=-1 then θ= -π plusmn π6

Therefore the required solution in -π le θ le π are θ= π6 5π6 -π6 -5π6

Exercise Find general solution of sin 2θ=cos θ [Hints Use sin 2θ= 2sin θcosθ and then take cosθ

common]

1lek le1)

Determine an angle alpha such that sin =k and -π2le αle π2

Then we have

Sin θ = k = sin α

Or sin θ - sin α =0

Or 2 cos [(θ+α) 2] sin [(θ-α) 2] =0

Therefore either cos [(θ +α) 2] =0 (1)

Or sin [(θ-α) 2] =0 (2)

Now from (1) we get (θ+α) 2= (2m+1) π2)

Or θ = (2m+1) π-α (3)

And from (2) we get (θ-α) 2 =mπ

Or θ= 2mπ+α(4)

Where m = any integer

Clearly the solution (3) amp (4) may be combined in the following form

θ= nπ+(-1) n α where n= any integer

Therefore the general solution of sin θ = sin α is θ = nπ +(-1) n α where n is any integer and -π2 le α le π2

Biology Chapter - 04Kingdom Monera

Today we will discuss about bacterial reproduction and its usefulness

Fig Binary Fission

Fig Conjugation Fig Transformation

Bacterial reproduction is mainly asexual but sexual reproduction

also takes place

Asexual reproduction takes place by i) Binary fission - from one bacteriato

two bacteria are produced in every 20 to 30mins

ii) Buddingiii) Endospore formation - during

unfourable condition

Sexual reproduction by three ways

1) Conjugation - Transfer of genetic material between cells that are in physical contact with one another

2) Transduction - Transfer of genetic materialfrom one cell to another by a bacteriophage

3) Transformation - Transfer of cell-freeor naked DNArsquo from one cell to another

Bacteria causes different diseases inplants animals and human and

it causes food spoilage and waterpollution but it also have some useful

activities

i) Bacteria are helpful in sewage water treatment

ii) It is used in antibiotic (medicine) production

iii) Anaerobic bacteria help in biogas(energy) production

iv) Many household products like yoghurt cheese are manufactured by use of bacteria

v) Rhizobium by symbiotic relationship with leguminous plant increase soil fertility

vi) Besides these bacteria is helpful in genetic engineering degradation of petroleum hydrocarbonand in dairy

industry

Physics Motion in plane Here we will introduce Projectile Motion

Execution

Projectile

Y

usinθ u h

θX

ucosθ

Suppose a body is projected with an angle θ So initial velocity u can be resolved into two components

Horizontal component - ucosθ ( for range)

Vertical component - usinθ ( for height)

usinθ changes during motion and becomes zero at maximum height position but ucosθ remain unchanged

The maximum height of projectile is h

NB If initial is upward then g = -ve and if it is downward then g = +ve Height is +ve if direction of motion does not change ( for ex a body thrown upwards but goes down ultimately then height h = -ve)

The angle of projectile θ is the angle made with horizontal

HISTORY ndash GROWTH OF NATIONALISM

SUB TOPIC- REVOLUTIONARY NATIONALISM Bengal formation of Anushilan Samity and Jugantar Group

The intensification of the Swadeshi movement and Government policy of terror and repression led to outbreak of violence Bombs were manufactured and attempts on the lives of unpopular Government officials became frequent In the gymnasium of Scottish Church College which was known as General Assemblies Institution a secret society was formed known as Anushilan Samity

Aurobindo Ghosh send from Baroda his emissary Jatindranath Banerjee to mobilize the Bengal revolutionaries

Hemchandra Qanungo and Satyen Bose published Journal Jugantar

The Jugantar group planned to assassinate oppressive magistrate Kingsford by Khudiram Bose and Prafulla Chaki in 1908 Prafulla Chaki committed suicide to avoid arrest Khudiram was tried and hanged

Afew days later the police found a bomb factory in Maniktala and arrested a large number of revolutionaries The trial of revolutionaries became famous as the Alipore Bomb Case

In the course of the trial the approver the public prosecuter and a police officer were assassinated

1 Question Name two journals which preached the cult of violence

Answer a) Yugantar edited by Bhupendranath DuttaB) Bandemataram edited by Aurobindo Ghosh2 Question Why was Khudiram arrested and hangedAnswer An attempt was made to assassinate a hated vindictive majistrate named Kingsford by Khudiram Bose and Prafulla Chaki Their attempt failed and the bomb they threw killed two English ladies Khudiram was arrested and put to trial and then hanged3Question Who was Aurubindo GhoshAnswer Aurobindo Ghosh a nationalist revolutionary who was charged for his involvement in the Alipore Bombing Case He was accused of it along with his brother Barindra nath Ghosh But Aurobindo was acquitted because of the brilliant pleading of his counsel Chittaranjan Das Then he became a spiritual reformer introducing his visions on human progress and spiritual evolution4 Qustion Who was KingsfordAnswer Kingsford was an unpopular British chief Magistrate who was the target of the bomb thrown at Muzaffarpur by Khudiram and Prafulla Chaki

Most of the accused were convicted and sentenced to

long term of imprisonmentBut

Aurobindo Ghosh was acquitted mainly owing to the brilliant pleading of his counsel Chittaranjan Das

Political science Topic-Sovereignty

Summary Sovereignty is the full right and power of a governing body over itself without any interference from outside sources or bodies In political theory sovereignty is a substantive term designating supreme legitimate authority over some polity In international law sovereignty is the exercise of power by a state

Internal Sovereignty

Internal sovereignty means supreme authority within ones territory while external sovereignty relates to the recognition on the part of all states that each possesses this power in equal measure

External sovereignty

external sovereignty relates to the recognition on the part of all states that each possesses this power in equal measure

Distinguish between

Execution

Answer the following questions

Short notes-

Sovereignty

Internal Sovereignty

External sovereignty

Homework- learn

external sovereignty and internal sovereigntySovereignty is the principle

of supreme and

unquestionable authority

reflected in the claim by the

state to be the sole author of

laws within its territory

Definition of external vs internal sovereigntyInternal sovereignty refers to

the relationship between a

sovereign power and its

subjects ndash it refers to the

location of the supreme

authority within the state In

the UK for example internal

sovereignty (supposedly)

resides within Parliament

reflected in the

constitutional principle of

parliamentary

sovereigntyBy contrast

external sovereignty refers

to the capacity of the state

to act independently and

autonomously on the world

stage This is what is

sometimes called lsquostate

sovereigntyrsquo or lsquonational

sovereigntyrsquo and implies

that states are legally equal

and that the territorial

integrity and political

independence of a state is

inviolable

Class ndash XII

Date - 2742020 STUDY MATERIALSubject Topic Summary Execution Business Studies

Job Analysis amp Manpower Planning

At first let us recall the chapter what we have discussed till nowbullJob analysisbullJob specification bullJob description bullJob enlargement bullJob enrichment

Today we will do some questions answers from the chapter

Questions 1ldquoJob analysis job description and job specification are interrelatedrdquo Comment Answer) Job analysis is a systematic and detailed examination of a job to collect all the relevant information about it The contents off the job are summarised in the job description The qualification needed for the job are summarised in job specificationThus there is close interrelationship between job analysis job description and job specification

Question 2ldquoJob enlargement is a horizontal extension of a job whereas job enrichment is a vertical extension of a jobrdquo ElucidateAnswer) Job enlargement involves adding one or more task to a job coma where as job enrichment involves adding more autonomy and responsibility to a job Job enlargement is therefore horizontal extension of a job coma whereas job enrichment is a vertical extension of a job

Question 3 )

What is manpower estimation Explain its quantitative and qualitative aspectsAnswer) Manpower estimation is the process by which management determines how an organisation should move from itrsquos current manpower positionto its desired manpower position There are two dimensions of Manpower estimation- quantitative and qualitative

Quantitative aspectThis aspect of Manpower estimation involves estimating the number of employees required in a future time period Workload analysis and workforce analysis are done to estimate the quantity of required manpower

Qualitative aspectThe estimate of the knowledge skills experience etc of required manpower is the qualitative aspect of Manpower estimation The quality of Manpower can be judged on the basis of job analysisand job specification

COMMERCE

CAPITAL-FIXED AND WORKING

Today let us start the class by discussing the sources of finance for different types of business firms

The term lsquocapitalrsquo refers to the investment made in the enterprise for the purpose of earning profits

Requirements of capital and sources of capital for different types of business firms are

1 Capital for sole proprietorship businessA sole proprietor operates at a small scale and thereforerequires a limited amount of capital

2 Capital for partnership firmCapital requirements as well as capital base of a partnership is bigger than that of a sole trader businessThe owned capital is contributed by the partners in an agreed ratio

3 Capital for joint stock companyA joint stock company generally requires large amount of capitalA public company can raise huge capital through issue of shares In addition to share capital it can utilize retained profits

Now let u discuss the meaning of Finance PlanningFinance planning is the process of estimation the financial requirements of an organization specifying the sources of firms and ensuring that enough funds are available at the right time

1 What do you mean by Finance PlanningAnswer Finance planning is the process of estimation the financial requirements of an organization specifying the sources of firms and ensuring that enough funds are available at the right time

2Discuss the role of financial planning of an enterpriseThe role of financial planning are as followsa A sound financial plan helps a business enterprise to avaid the problems of shortage and surplus of fundsbFinancial planning serves as a guide in developing a sound capital structure so as to maximize returns to shareholders c It helps in effective utilization of fundsd It provides policies and procedures for coordinating different functional areas or departments of businesse It enables the management to exercise effective control over the financial activities of an enterprisef It helps the company to prepare for facing business shocks and surprises in future

Mathematics

Continuity and differentiability

Recall Definition of ContinuityLet f(x) be a single valued function of x and x=a be a point in the domain of definition of the function The function is said to be continuous at x=a ifi) f(c) is defined ie f(x) has a definite finite value at x=cii) lim xrarra f(x) exists andiii) lim xrarra f(x) =f(a) In other words f(x) is said to be continuous at x=a if lim xrarra+ f(x)= lim xrarra- f(x) = f(a) Or f(a+0) =f(a-0) =f(a) Or lim hrarr0 f(a+h)= f(a) Algebra of continuous functionsNow we will study some algebra of continuous functions Theorem 1 Suppose f and g be two real functions continuous at a real number c Then(1) f + g is continuous at x = c(2) f ndash g is continuous at x = c(3) f g is continuous at x = c(4) (fg) is continuous at x = c (provided g (c) ne 0)

Example 1 Prove that every rational function is continuousSolution Recall that every rational function f is given byf(x)=[p(x) q(x) ] q(x)ne0where p and q are polynomial functions The domain of f is all real numbers except points at which q is zero Since polynomial functions are continuous f is continuous by (4) of Theorem 1Example 2Discuss the continuity of sine functionSolution To see this we use the following factslim xrarr0 sin x =0Now observe that f (x) = sin x is defined for every real number Let c be a real number Put x = c + h If x rarr c we know that h rarr 0 Therefore lim xrarrc f(x)

= lim xrarrc sin x= lim hrarr0 sin(c+h) =lim hrarr0 [sin c cos h + cos c sin h ]=lim hrarr0 (sin c cos h) + lim hrarr0 (cos c sin h) = sin c +0=sin c = f(c) Thus lim xrarrc f(x) = f(c) and hence f is a constant function Exercise Prove that the function f(x) = x2 +2x is continuous for every real value of x [Hints show that lim xrarra+ f(x) = lim xrarra- f(x) = f(a) ]

Biology Reproduction in Flowering plants We will discuss about megasporoangium

megasporagenesis and female gametophyte

Q4 Describe the structure of megasporangium

Ovule is attached to the placenta by astalk called funicle

Each ovule has one two or three protectivecoverings called integuments

At the tip of integuments a small openingcalled micropyle is organised

Opposite to the micropylar end is the chalaza

Within the integuments a mass of cellsnucellusand inside it embryo sac orfemale gametophyte is present

Q5 Describe a mature embryosacamp its formation

In most of the flowering plants only oneof the 4 megaspores formed as a result ofmegasporogenesis that is functional while theother three degenerate

The

functional megaspore develops into thefemale gametophyte

Formation The nucleus

of the functional megasporedivides mitotically to form two nuclei first andthen two more sequential mitotic nucleardivisions result in the formation of four ampthen eight nucleate stages of embryo sac

Six of the eight nucleus are surrounded bycell walls and organised into cells

The remaining two nuclei called polar nuclei are found below the egg apparatus in the largecentral cell

Three cells consisting of two synergids amp one egg cell present bottom of

embryo sac Three cells

at the chalazal as antipodal cells

Two polar nuclei together present in large central cell

HISTORY

TOWARDS INDEPENDENCE AND PARTITION THE LAST PHASE(1935-1947)SUB TOPIC NATIONAL MOVEMENTS DURING THE SECOND WORLD WAR

Spread of Quit India Movement On 9th August 1942Gandhiji and other Congress leaders were arrested The Congress was declared illegal The news of the arrest of all leaders marked the beginning of a widespread movement of India It was not possible for such a movement to remain peacefulBut the arrest of the all notable congress leaders virtually left the movement in the hands of the mass The movement took the form of violent and militant outbreakBesides congressmen revolutionaries also were very active in the movement The Congress Socialist group also played a prominent role

1 Question Why did the British authority arrest the Congress leaders on 9 th August 1942Answer Congress Working committee adopted the Quit India resolution which was to be ratified at the Bombay AICC meeting in 8th August 1942 They decided to launch a mass struggle on non-violent lines Gandhiji gave a clarion call to all section of the people rdquoKarenge ya Marengerdquo (do or die) Congress leaders gave the call to driving out

the British from IndiaViceroy had taken strong action against the Quit India movement Gandhiji and all the leaders of Congress were arrested

2 Question How did Quit India Movement spread out all over IndiaAnswer The news of the leaders lsquo arrest marked the beginning of a widespread movement to remain peacefulThe movement took form of violent outbreak There were widespread cutting of telephone and Telegraph wires damaging railway lines raising barricades in cities and towns and other forms of violent demonstations

Question Name the leaders of Congress

Socialist group played a prominent part Notable among the Jayprakash Narayan Rammonohar Lohia Aruna Asaf Ali

Political science

Topic-Franchise and Representation

Summary

The election commission

The Election Commission of India is an autonomous constitutional authority responsible for administering Union and State election processes in India The body administers elections to the Lok Sabha Rajya Sabha State Legislative Assemblies in India and the offices of the President and Vice President in the country

Functions of election commission-

India is a sovereign socialist secular democratic republic Democracy runs like a golden thread in the social economic and political fabric woven by the Constitution given by lsquoWe the People of Indiarsquo unto ourselves The concept of democracy as visualised by the Constitution pre-supposes the representation of the people in Parliament and State legislatures by the method of election The Supreme Court has held that democracy is one of the inalienable basic features of the Constitution of India and forms part of its basic structure The Constitution of India adopted a Parliamentary form of government Parliament consists of the President of India and the two Houses mdash Rajya Sabha and Lok Sabha India being a Union of states has separate state legislatures for each state State legislatures consist of the Governor and two Houses mdash Legislative Council and Legislative Assembly mdash in seven states namely Andhra Pradesh Telangana Bihar Jammu amp Kashmir Karnataka Maharashtra and Uttar Pradesh and of the Governor and the state Legislative Assembly in the remaining 22 states Apart from the above two out of the seven Union Territories namely National Capital Territory of Delhi and Puducherry also have their Legislative Assemblies

ExecutionShort notes-Election commissionFunctions of election commission

Homework- Learn

Computer

Science

Computer hardware NAND Gate

A NOT-AND operation is known as NAND operation It has n input (n gt= 2) and one output

Logic diagram

Truth Table

NOR Gate

A NOT-OR operation is known as NOR operation It has n input (n gt= 2) and one output

Logic diagram

Truth Table

XOR Gate

XOR or Ex-OR gate is a special type of gate It can be used in the half

adder full adder and subtractor The exclusive-OR gate is abbreviated as EX-OR gate or sometime as X-OR gate It has n input (n gt= 2) and one output

Logic diagram

Truth Table

XNOR Gate

XNOR gate is a special type of gate It can be used in the half adder full adder and subtractor The exclusive-NOR gate is abbreviated as EX-NOR gate or sometime as X-NOR gate It has n input (n gt= 2) and one output

Logic diagram

Truth Table

Physics

Chapter 1 Electric Field ( Electric Dipole) (Summary)

Here we will derive Expression of electric field at broad side

On position of dipole

Execution

Q With the help of a labelled diagram obtain an expression for the electric field intensity E at any point on the equitorial line ( broad-side on position) of an electric dipole

Ans

E1 E1sinθ

E θ P E1 θ

( r2+L2)12 E2 E

r E2 E2sinθ

-q θ L O L +qA B

Let us consider that the point P is situated on the right bisector of the dipole AB at a distance r meter from its midpoint O

Let E1 and E2 be the electric field intensities of the electric field at P due to charge +q and ndashq of the dipole resp The distance of P from each charge is ( r2+L2)12

So E1 = 14 πϵ q

(r 2+L 2) away from +q

E2 = 14 πϵ q

(r 2+L 2) towards ndashq

The magnitudes of E1 and E2 are equal but directions are different Now resolving E1 and E2 into two components parallel and perpendicular to AB we get

The components perpendicular to AB E1sinθ and E2sinθ cancel each other because they are equal and opposite

The components parallel to AB are E1cosθ and E2 cosθ are in same direction and add up

So resultant intensity of electric field at the point P is

E = E1cosθ + E2 cosθ

E = 14 πϵ q

(r 2+L 2) 2 cosθ

Now from fig we have cosθ =BOBP = L (r2+L2)12

So we get E = 14 πϵ 2qL ( r2+L2)32

Now electric dipole moment p= 2qL

So E = 14 πϵ p ( r2+L2)32

HW Find the expression of Electric field as done here but this time take r gtgt 2L

Also find the expression of torque experience by a dipole

(Hint Electric force experienced by charges of dipole in electric field is qE each Let θ be the angle which dipole makes with electric lines of force then perpendicular distance between two charges is 2Lsinθ Then torque = force x perp distance = qE x 2L sinθ So τ=pE sinθ where p =2qL )

STUDY MATERIAL

Class XIISubject Eng Literature (The Tempest ndash William Shakespeare) Topic Act IV Scene 1 Lines 84 to 133 (Iris hellip A contract of true love Be not too late ) Date 27th April 2020 (4th Period)

[Students should read the original play and also the paraphrase given in the school prescribed textbook]Summary Questions amp Answers

o Ceres soon appears and comes to know that she has been summoned to celebrate the contract of true love

o Ceres expresses her unwillingness to meet Venus and Cupid as she has shunned their company

o Ceres and Juno both bestow their blessings upon Ferdinand and Miranda with June gifting honour riches happiness in marriage and Ceres presents plenty of earthrsquos produce

o Iris summons the water-nymphs and reapers to come and celebrate a contract

(1) IRIS Of her society (Line 91-101)

Be not afraid I met her deity

Cutting the clouds towards Pathos and her sonDove-drawn with her Here thought they to have doneSome wanton charm upon this man and maidWhose vows are that no bed-right shall be paidTill Hymens torch be lightedmdashbut in vainMarss hot minion is returned againHer waspish-headed son has broke his arrowsSwears he will shoot no more but play with sparrowsAnd be a boy right out

(i) Where were Venus and Cupid seen flying How were they travelling Why did they want to join the marriage celebration of Ferdinand and Miranda

of true love

Venus and Cupid were seen flying through the air towards Paphos the famous city which is situated on the island of Cyprus They were travelling by air-borne chariot drawn by doves They certainly wanted to come here in order to play some amorous trick upon Ferdinand and Miranda who are under a vow not to gratify their physical desires till the holy ceremony of their marriage has been performed(ii) What have Venus and Cupid done after failing in their plan

After being failure of their plan Venus who is a very passionate deity and who is the mistress of Mars (the god of war) has gone back while here ill-tempered son Cupid has broken his arrows of love in his state of desperation(iii) What has Cupid firmly decided

Cupid is feeling so disappointed that he has firmly decided to shoot no more arrows to arouse love in human hearts but to spend his time playing with sparrows Thus he would now become just a boy and would give up his original function of shooting arrows on human beings to make them fall in love(iv) What vow had Ceres taken How did Ceres feel at the abduction

After the abduction of her daughter Prosperina by Pluto Ceres had taken a vow to always keep away from the disgraceful company of Venus and her blind son Cupid the god of love Ceres felt deeply distressed when Pluto had carried off her daughter and had made her his wife by force(v) Why has Ceres not forgiven Venus and her blind son For what do Ceres want to be sure

As the abduction had been manipulated by Venus the goddess of beauty and love and her blind son Cupid Ceres has never forgiven them for their part in the whole plot Ceres wants to be sure that she would not have to meet Venus and Cupid who had engineered the abduction of her daughter Prosperina

AS THIS lsquoMASQUErsquo SCENE IS VERY IMPORTANT IN THE PLAY THE PARAPHRASE OF THE ENTIRE PORTION OF MASQUE SCENE (Act IV Lines 58 to 143) IS GIVEN BELOW

IRIS Goddess of RainbowCERES Goddess of Agriculture and all the fruits of the earth

(Nature growth prosperity rebirth ndash notions intimately connected to marriage)JUNO The majestic Queen of Heavens and wife of Jupiter (Jupiter is the king of Gods)

VENUS The Goddess of love CUPID Son of Venus PLUTO God of death (In the play referred by Shakespeare as lsquoDisrsquo which is a Roman name for Pluto)

ORIGINAL TEXT PARAPHRASEPROSPEROWellmdash

PROSPERONow come Ariel Let there be too many rather than too few

Now come my Ariel Bring a corollaryRather than want a spirit Appear and pertly[to Ferdinand and Miranda]No tongue all eyes Be silent

spirits in attendance Appear briskly

[to Ferdinand and Miranda]Look with your eyes but do not say a word

[Soft music] [Soft music][Enter Iris] [Enter Iris]

IRISCeres most bounteous lady thy rich leasOf wheat rye barley vetches oats and peasThy turfy mountains where live nibbling sheepAnd flat meads thatched with stover them to keepThy banks with pioned and twilled brimsWhich spongy April at thy hest betrimsTo make cold nymphs chaste crowns and thybroom-grovesWhose shadow the dismissegraved bachelor lovesBeing lass-lorn thy pole clipped vineyardAnd thy sea-marge sterile and rocky-hardWhere thou thyself dost airmdashthe Queen othrsquoSkyWhose watery arch and messenger am IBids thee leave these and with her sovereign grace[Juno appears] Here on this grass-plot in this very placeTo come and sport Her peacocks fly amainApproach rich Ceres her to entertain

IRISCeres most generous lady you are the cause of rich fields or fertile land where wheat rye barley beans oats and peas grow the grassy mountains where the sheep graze and the flat meadows covered with coarse hay to be used as fodder for cattleYour banks are covered with marsh-marigolds and reeds and the rainy April under your orders brings forth to make for the maids who are not in love beautiful crowns your woods where the broom flourishes and where the bachelor who has been dismissed by the maid he loved lies down being forsaken your vineyard in which the poles are embraced by the vines and the margin of the sea which is barren and rocky where you roam about to enjoy the fresh air ndash the queen of the sky (Juno) whose messenger I am besides being represented as the rainbow bids you leave all these and with her majesty here on this grassy plot in this very place come and sport her peacocks carry her fast in her chariot through the air and are making their way here approach rich Ceres to welcome her

[Enter Ariel as Ceres] [Enter Ariel as Ceres]

CERESHail many-coloured messenger that neerDost disobey the wife of JupiterWho with thy saffron wings upon my flowersDiffusest honey-drops refreshing showersAnd with each end of thy blue bow dost crownMy bosky acres and my unshrubbed downRich scarf to my proud earth Why hath thy queenSummoned me hither to this short-grassed green

CERESWelcome rainbow that never dared disobey Juno the wife of Jupiter who with your orange coloured rays spread honey-drops refreshing showers And with each end of thy blue bow drown my bushy acres and my hilly country which is free from shrubs you thus forming a rich scarf Why has your queen called me here to this place covered with short grass

IRISA contract of true love to celebrateAnd some donation freely to estateOn the blest lovers

IRISI have called you to celebrate a contract of true love and bestow some liberal gift upon the blessed lovers

ORIGINAL TEXT PARAPHRASECERESTell me heavenly bowIf Venus or her son as thou dost knowDo now attend the queen Since they did plotThe means that dusky Dis my daughter gotHer and her blind boys scandaled companyI have forsworn

CERESTell me heavenly bow if Venus the Goddess of love or Cupid her son and pedlar of passion at this time attend the heavenly queen Juno because you are sure to know Since the day they conspired against me and dark Pluto took away my daughter here and Cupidrsquos disgraceful company I have left off

IRISOf her societyBe not afraid I met her deityCutting the clouds towards Pathos and her sonDove-drawn with her Here thought they to have doneSome wanton charm upon this man and miad

IRISBe not afraid of her company I met her deity moving on the clouds towards Paphos the sacred home of Venus on the island of Cyprus along with her son on her chariot drawn by doves Here they contemplated to exercise a charm upon this man and maid producing

Whose vows are that no bed-right shall be paidTill Hymens torch be lightedmdashbut in vainMarss hot minion is returned againHer waspish-headed son has broke his arrowsSwears he will shoot no more but play with sparrowsAnd be a boy right out

wantonness before the actual marriage ceremony but did not succeed Venus has returned her irritable son has broken his arrows and swears that he will give up his practice of trying to inspire love but play with sparrows and be a boy again

[Music is heard] [Music is heard]

CERESHighst queen of stateGreat Juno comes I know her by her gait

CERESHighest queen of state Great Juno there she comes I know here by her gait

[Enter Juno] [Enter Juno]

JUNOHow does my bounteous sister Go with meTo bless this twain that they may Prosperous beAnd honoured in their issue

JUNOHow are you doing my generous sister Come with me to bless this couple so that they may be prosperous and fortunate in their children

[They sing] [They sing]

JUNOHonour riches marriage-blessingLong continuance and increasingHourly joys be still upon youJuno sings her blessings upon you

JUNOMay honour riches happiness in marriage long continuance and increase of those boons ever rest upon you as hourly joys Juno showers down upon you her blessings in song

CERESEarths increase foison plentyBarns and garners never emptyVines and clustring bunches growingPlants and goodly burden bowingSpring come to you at the farthestIn the very end of harvestScarcity and want shall shun youCeresrsquo blessing so is on you

CERESMay you have the plenty of earthrsquos produce Your barns and granaries may never be empty Your vines may grow with clustering bunches Your fruit trees may be heavily laden with their fruit May there be continuous spring and harvest May scantiness and want leave you forever Such is the blessing of Ceres upon you

FERDINANDThis is a most majestic vision andHarmoniously charmingly May I be boldTo think these spirits

FERDINANDThis is a great vision and magically melodious Should I suppose the characters (taking part in the masque) are spirits

PROSPEROSpirits which by mine artI have from their confines calld to enactMy present fancies

PROSPEROYes they are spirits whom I have summoned from the regions to which they are confined to carry into effect my fanciful designs

ORIGINAL TEXT PARAPHRASEFERDINANDLet me live here everSo rare a wondered father and a wifeMakes this place paradise

FERDINANDI should like to live here forever Such a wise and wonderful father makes this place a paradise

[Juno and Ceres whisper and send Iris on employment] [Juno and Ceres whisper and send Iris on employment]

PROSPEROSweet now silence

PROSPEROMy dear Ferdinand speak no more Juno and Ceres are

Juno and Ceres whisper seriouslyTheres something else to do Hush and be muteOr else our spell is marred

whispering with a solemn look There is something else coming Silence Or else our magic will be spoilt

IRISYour nymphs called naiads of the wandering brooksWith your sedged crowns and over-harmless looksLeave your crisp channels and on this green landAnswer your summons Juno does commandCome temperate nymphs and help to celebrateA contract of true love Be not too late

IRISYou nymphs called Naiads denizens (M inhabitants) of the running stream with your chaplets of sedge and ever-helpful looks leave your wrinkled channels and on the green land answer the summons sent to you Juno has ordered some chaste nymphs and help to celebrate a noble and true marriage Donrsquot delay

[Enter certain nymphs] [Enter certain nymphs]You sunburnt sicklemen of August wearyCome hither from the furrow and be merryMake holiday your rye-straw hats put onAnd these fresh nymphs encounter every oneIn country footing

You sunburnt harvesters weary from the effects of the heat in August come here from the furrowed land and rejoice Make holiday with your rye-straw hats upon you and meet these fresh nymphs and join in country dancing

[Enter certain reapers properly habited They join with the nymphs in a graceful dance towards the end whereof Prospero starts suddenly and speaks]

[Enter certain reapers properly habited They join with the nymphs in a graceful dance towards the end whereof Prospero starts suddenly and speaks]

PROSPERO[aside] I had forgot that foul conspiracyOf the beast Caliban and his confederatesAgainst my life The minute of their plotIs almost come [to the spirits]Well done Avoidno more

PROSPERO(Aside)I had forgotten the wicked conspiracy of the beast Caliban and his accomplices against my life the time of their plot has almost arrived ndash (To the Spirits) well done depart no more of this

[To a strange hollow and confused noise the spirits heavily vanish]

[The spirits depart]

ORIGINAL TEXT PARAPHRASEFERDINANDLet me live here everSo rare a wondered father and a wifeMakes this place paradise

FERDINANDI should like to live here forever Such a wise and wonderful father makes this place a paradise

[Juno and Ceres whisper and send Iris on employment] [Juno and Ceres whisper and send Iris on employment]

PROSPEROSweet now silenceJuno and Ceres whisper seriouslyTheres something else to do Hush and be muteOr else our spell is marred

PROSPEROMy dear Ferdinand speak no more Juno and Ceres are whispering with a solemn look There is something else coming Silence Or else our magic will be spoilt

IRISYour nymphs called naiads of the wandering brooksWith your sedged crowns and over-harmless looksLeave your crisp channels and on this green landAnswer your summons Juno does commandCome temperate nymphs and help to celebrateA contract of true love Be not too late

IRISYou nymphs called Naiads denizens (M inhabitants) of the running stream with your chaplets of sedge and ever-helpful looks leave your wrinkled channels and on the green land answer the summons sent to you Juno has ordered some chaste nymphs and help to celebrate a noble and true marriage Donrsquot delay

[Enter certain nymphs] [Enter certain nymphs]You sunburnt sicklemen of August wearyCome hither from the furrow and be merryMake holiday your rye-straw hats put onAnd these fresh nymphs encounter every oneIn country footing

You sunburnt harvesters weary from the effects of the heat in August come here from the furrowed land and rejoice Make holiday with your rye-straw hats upon you and meet these fresh nymphs and join in country dancing

[Enter certain reapers properly habited They join with the nymphs in a graceful dance towards the end whereof Prospero starts suddenly and speaks]

[Enter certain reapers properly habited They join with the nymphs in a graceful dance towards the end whereof Prospero starts suddenly and speaks]

PROSPERO[aside] I had forgot that foul conspiracyOf the beast Caliban and his confederatesAgainst my life The minute of their plotIs almost come [to the spirits]Well done Avoidno more

PROSPERO(Aside)I had forgotten the wicked conspiracy of the beast Caliban and his accomplices against my life the time of their plot has almost arrived ndash (To the Spirits) well done depart no more of this

[To a strange hollow and confused noise the spirits heavily vanish]

[The spirits depart]

Ac-12 27420 topic Revaluation of Assets and Liabilities

REVALUATION OF ASSETS AND LIABILITIES

On admission of a new partner the firm stands reconstituted and consequently the assets are revalued and liabilities are reassessed It is necessary to show the true position of the firm at the time of admission of a new partner If the values of the assets are raised gain will increase the capital of the existing partners Similarly any decrease in the value of assets ie loss will decrease the capital of the existing partners For this purpose alsquoRevaluation Accountrsquo is prepared This account is credited with all increases in the value of assets and decrease in the value of liabilities It is debited with decrease on account of value of assets and increase in the value of liabilities The balance of this account shows a gain or loss on revaluation which is transferred to the existing partnerrsquos capital account in existing profit sharing ratioAccounting for Revaluation of Assets and Liabilities when there is a Changein the Profit Sharing Ratio of Existing PartnersAssets and liabilities of a firm must also be revalued at the time of change in profit sharing ratio of existing partners The reason is that the realisable or actual value of assets and liabilities may be different from those shown in the Balance Sheet It is possible that with the passage of time some of the assets might have appreciated in value while the value of certain other assets might have decreased and no record has been made of such changes in the books of accounts Similarly there may be some unrecorded assets amp libilities that may have to be accounted for Revaluation of assets and reassessments of liabilities becomes necessary because the change in the

value of assets and liabilities belongs to the period to change in profit sharing ratio and hence must be shared by the partners in their old profit sharing ratio Revaluation of assets and reassessment of liabilities may be given effect to in two different ways (a) When revised values are to be recorded in the books and(b) When revised values are not to be recorded in the books

When revised values are to be recorded in the booksIn such a case revaluation of assets and reassessment of liabilities is done with the help of a new account called lsquoRevaluation Accountrsquo Sometimes this account is also called as lsquoProfit amp Loss Adjustment Acrsquo If there is a loss due to revaluation revaluation account is debited and if the revaluation results in a profit the revaluation account is credited The following journal entries made for this purpose are

(i) For increase in the value of assetsAsset Ac Dr (individually)To Revaluation Ac(ii) For decrease in the value of AssetRevaluation Ac Dr (individually)To Asset Ac[Decrease in the value of assets](iii) For increase in the value of LiabilitiesRevaluation Ac Dr (individually)To Liabilities Ac[Increase in the value of Liabilities](iv) For decrease in the value of LiabilitiesLiabilities Ac DrTo Revaluation Ac[Decrease in the value of Liabilities](v) For unrecorded AssetsAsset Ac [unrecorded] DrTo Revaluation Ac[Unrecorded asset recorded at actual value](vi) For unrecorded Liability Revaluation Ac DrTo Liability Ac [unrecorded][Unrecorded Liability recorded at actual value](vii) For transfer of gain on revaluationRevaluation Ac DrTo Existing Partnerrsquos CapitalCurrent Ac[Profit on revaluation transferred to capital account in existing ratio](viii) For transfer of loss on revaluationExisting Partnerrsquos CapitalCurrent Ac DrTo Revaluation Ac[Loss on revaluation transferred to capital account in existing ratio](a) When revaluation account shows gain Revaluation Ac DrTo Partnerrsquos Capital Ac (Old Profit Sharing Ratio)(Profit on revaluation credited to Partnerrsquos Capital Ac)(b) Above entry is reversed when revaluation account shows loss Partners Capital Acs (Old Profit Sharing Ratio) DrTo Revaluation Ac(Loss on revaluation debited to Partnerrsquos Capital Acs)

Proforma of Revaluation Account is given as under

Revaluation Account

Dr Cr Particulars ` Amount Particulars ` Amount To Decrease in value of assets By Increase in value of assets To Increase in value of liabilities By Decrease in value of liabilities To Unrecorded liabilities By Unrecorded assets To Gain on Revaluation (Transferred) By Loss on Revalution (Transferred)

ECO ndash12 2742020Topic- ELASTICITY OF DEMAND

CHAPTER - ELASTICITY OF DEMANDMEANINGDemand for a commodity is affected by many factors such as its price price of related goods income of its buyer tastes and preferences etc Elasticity means degree of response Elasticity of demand means degree of responsiveness of demand Demand for a commodity responds to change in price price of related goods income etc So we have three dimensions of elasticity of demandDIMENSION OF ELASTICITY OF DEMAND TYPES OF ELASTICITY OF DEMAND

Price elasticity of demand Income elasticity of demand Cross Elasticity of demand

Price elasticity of demand Price elasticity of demand means degree of responsiveness of demand for a commodity to the change in its price For example if demand for a commodity rises by 10 due to 5 fall in its price Price elasticity of demand (ep)=Percentage change in quantity demanded Percentage change in price of the commodity = 10 ( -)5 = ( - )2Note that ep will always be negative due to inverse relationship of price and quantity demanded

(ii) Income elasticity of demand Income elasticity of demand refers to the degree of responsiveness of demand for a commodity to the change in income of its buyer Suppose income of buyer rises by 10 and his demand for a commodity rises by 20 then Income elasticity of demand (ey)= change in quantity demanded change in price of the commodity =20 10 = 2

Cross Elasticity of demandCross elasticity of demand means the degree of responsiveness of demand for a commodity to the change in price of its related goods (substitute goods or complementary goods) Suppose demand for a commodity rises by 10 due to 5 rise in price of its substitute good then Cross elasticity of demand (ec) = change in quantity demanded change in price of related good = 10 2 = 5 (Tastes and preferences cannot be expressed numerically So elasticity ofdemand cannot be numerically expressed)

  • Chapter 1 Force (Summary)
  • Distinguish between external sovereignty and internal sovereignty
    • NAND Gate
      • Logic diagram
      • Truth Table
        • NOR Gate
          • Logic diagram
          • Truth Table
            • XOR Gate
              • Logic diagram
              • Truth Table
                • XNOR Gate
                  • Logic diagram
                  • Truth Table
                      • Physics
                      • Chapter 1 Electric Field ( Electric Dipole) (Summary)
Page 5:  · Web viewWe all know that Nouns are divided into two parts: common noun and proper noun.Apart from common and proper noun, we will also study about collective noun and compound

Class IV

Subject English language Topic Nouns Compound nouns

We all know that Nouns are divided into two parts common noun and proper nounApart from common and proper noun we will also study about collective noun and compound nouns A common or proper noun that is the name of a collection or group of similar things or persons is known as a collective Noun Example crowd committee herd United Nations etc Example Mother has lost her bunch of keysThe word bunch is a collective noun

1 A herd of cows was grazing in the fieldHerd- collective noun

2 All living creatures need food to surviveCreatures- collective noun

3 The Ganga is a river sacred to IndiansIndians- collective noun

Compound noun some common and proper nouns are made up of more than one word They are known as compound nouns or simply compounds Example newspaper (news+paper) ashtray (Ash+tray) plaything (play+thing) etc1 My penfriendlives in a town in South AfricaPenfriend South Africa- compound noun

2 My grandparents live in New DelhiGrandparents New Delhi- compound nouns

3 Please do not park your cars in the drivewayDriveway- compound noun

Subject Social studies Topic A peep into the past

The story of the past is called history It is a journey across time and space that takes us to another period of time where people lived differently It is believed that the way of living eating clothes art were all different from what we have today We need to study history becauseIt tells us about the important people of the pastIt helps us to gain knowledge about the past and the present and help us to understand themHistory guides us to set an aim and warns us not to repeat the same mistakesIt helps us to understand the development of mankindIt helps to developessential skills to become a better person

Division of the pastHistory is so vast that it is divided into two parts ndash BC (Before Christ) and AD (Anna Domini) This helps us to understand about the past clearly Indian history and itrsquos civilisation are the worldrsquos oldest civilisation Indian history is divided into ancient medieval and modernCivilisation mean the stage of human social and cultural development

Subject COMPUTER Chapter-2 (MS WINDOWS)WINDOWS IS AN OPERATING SYSTEM THAT HELPS THE USER TO OPERATE AND CONTROL THE OVERALL ACTIVITIES OF A COMPUTER SOME POPULAR VERSIONS OF WINDOWS ARE WINDOWS 98 WINDOWS 7 WINDOWS 8 AND WINDOWS 10

DESKTOP THE FIRST SCREEN THAT WE SEE WHEN THE OPERATING SYSTEM (WINDOWS)HAS FINISHED LOADING IS KNOWN AS DESKTOP

THE FEATURES OF DESKTOP ARE bullIT IS WHERE ALL THE ICONS OF THE DIFFERENT APPLICATIONS ARE DISPLAYEDbullIT IS THE HOME FOR ALL SHORTCUTS PROGRAMS CAN BE QUICKLY OPENED BY CLICKING ON THE RESPECTIVE SHORTCUTSbullIT HOLDS VARIOUS INFORMATION SUCH AS TIME AND DATE

ICONSICONS ARE SMALL PICTURES OF DIFFERENT APPLICATIONS THAT ARE PRESENT AND INSTALLED IN THE COMPUTER FOR EXAMPLE COMPUTER ICON AND RECYCLE BIN ICON

DESKTOP GADGETS A DESKTOP GADGET IS A SMALL APPLICATION OR A WIDGET THAT RECIDES ON A COMPUTERS DESKTOP FOR EXAMPLE CLOCK CALENDAR ETC_______________________________________________________________________________________________

Class VSubject Socialal studiesTopic Evolution of man

Millions of years ago the earth looked much different as it is seen todayIt was totally different and it was very difficult to live Everything looked very different plants animals and humans everything looked very different from what they are now All the changes took place gradually with the changing environment Earliest ancestors of the humans were known as the Ramapithecus who lived about 14 million years ago They looked like ape that walked upright and weighed about 14kg With the passage of time there was a gradual development in the structure posture features and teeth of the early humansIt is said that the apesthat lived on treetops started living on the ground and stood on two limbs are called hominids They used their limbs to hold things and make tools Finally they changed to homosapiens mean wise human We humans belong to this group Life of early humansThe early men were hunter ndash gatherers They moved from place to place in search of food water and shelter They hunted animals and ate them raw gathered fruits nuts roots and seeds for eating They didnrsquot know how to build houses so they used live on treetops or in the caves or rock shelters In summers they remained uncovered but in winter they used to cover themselves by using bark of trees leaves and animal skins

Subject English language Topic Adjectives Interrogative and Emphasizing

Interrogative adjectiveAn interrogative adjective is used with a noun that it qualifies to ask a question The word interrogative mean to ask questions Example Which part of the world do you come fromThe word lsquoWhich is an interrogative adjectivesEmphasizing adjectivesSometimes we use adjectives to emphasize the effect of what we say Words which are used to emphasize a nounExample Rita came late and had to stand at the very end of the lineThe word Veryis an emphasizing adjective It actually qualifies or emphasizethe noun end

1lsquoDo not listen to what Ravi says for he is a complete foolrsquo said RohanComplete- emphasizing adjective

2Kiran is a great supporter of non violenceGreat- emphasizing adjective

3rsquoWhat crime has the prisoners committedrsquo asked the judgeWhat- interrogative adjective

4Whose book is that lying on the tableWhose- interrogative adjective

Subjects COMPUTER CHAPTER-1 (CHARACTERISTICS OF A COMPUTER)A COMPUTER SYSTEM IS A ONE THAT CONSISTS OF ALL THE SOFTWARES HARDWARES CPU AND MEMORY DEVICES SOME IMPORTANT TERMSTHE COMPUTER WORKS ACCORDING TO THE COMMANDS OR INSTRUCTIONS THAT WE GIVE

A SET OF INSTRUCTIONS IS CALLED THE PROGRAM A SET OF ONE OR MORE PROGRAMS IS CALLED A SOFTWARE THE PHYSICAL COMPONENTS OF A COMPUTER ARE CALLED HARDWARE

FOUR OPERATIONS OF A COMPUTER SYSTEM INPUTINPUT IS SUPPLIED TO THE COMPUTER WITH THE USE OF A KEYBOARD A MOUSE A MICROPHONE

OR OTHER INPUT DEVICES PROCESSING PROCESSING IS DONE INSIDE THE COMPUTER IN THE CPU PROCESSING IS THE CONVERSION

OF THE INPUT INTO THE OUTPUT STORAGE STORAGE REFERS TO THE SAVING OF INFORMATION FOR LATER USE INFORMATION IS STORED

IN THE MEMORY OF A COMPUTER THERE ARE TWO TYPES OF COMPUTER MEMORY PRIMARY MEMORY AND SECONDARY MEMORYPRIMARY MEMORY IS FAST SMALL AND EXPENSIVE THE TWO TYPES OF PRIMARY MEMORY ARE RANDOM ACCESS MEMORY(RAM) AND READ ONLY MEMORY (ROM) RANDOM ACCESS MEMORY (RAM)THIS MEMORY IS VOLATILE IN NATURE WHICH MEANS THAT THE

INFORMATION STORED IN IT IS AUTOMATICALLY ERASED WHEN THE COMPUTER POWER IS TURNED OFF

READ ONLY MEMORY (ROM) THIS TYPE OF MEMORY IS NON VOLATILE IN NATURE WHICH MEANS THAT DATA IS NOT LOST WHEN THE COMPUTER POWER IS TURNED OFF

OUTPUT OUTPUT IS THE RESULT PRODUCED BY A COMPUTEROUTPUT MAY BE VIEWED ON THE MONITOR SCREEN HEARD THROUGH SPEAKERS PRINTED THROUGH PRINTER ON PAPERS OR STORED IN THE MEMORY FOR LATER USE

Class VI

GEOGRAPHY CHAPTER 1 REPRESENTATION OF GEOGRAPHICAL FEATURES

GLOBE AND MAPS

GLOBE- A Globe is a three ndashdimensional model of the Earth

EQUATOR ndash An imaginary line runs through the centre of the earth from east to west

NORTHERN HEMISPHERE ndash the half of the north of the equator is called Northern Hemisphere

SOUTHERN HEMISPHERE ndash the half of the south of the equator is called Southern Hemisphere

LATITUDE - Imaginary parallel lines running on the globe from east to west

LONGITUDE ndash The imaginary lines semicircle in nature running from the North Pole to South Pole Also called meridian

MAP ndash Representation of the Earth as a whole or parts of it drawn on a flat surface to a scale

SKETCH ndash A rough drawing of a place not drawn to a scale

PLAN ndash A plan is prepared to show more details about a small area

COMPUTER - VI CHAPTER-6 (INTRODUCTION TO WINDOWS 10)

THE MOST RECENT VERSION OF WINDOWS IS WINDOWS 10 IT WAS RELEASED IN 2015

WINDOWS 10 IS DESIGNED TO WORK FOR YOU AND NOT YOU WORKING FOR IT THE WINDOWS 10 OPERATING SYSTEM INCLUDES A NUMBER OF NEW FEATURES LIKE IMPROVED PERFORMANCE ON MULTI CORE PROCESSORS IMPROVED G U I DATA SECURITY AND BETTER BOOTING PERFORMANCE

WINDOWS 10 HAS 1187 WITH WEARING FEATURES SETS AND INTENDED HARDWARE

THE VARIOUS EDITIONS AVAILABLE FOR WINDOWS 10 ARE AS FOLLOWS

WINDOWS 10 HOME

WINDOWS 10 HOME IS THE EDITION OF WINDOWS 10 THAT CONTAINS THE LEAST NUMBER OF FEATURESWINDOWS 10 HOME IS THE CONSUMER FOCUSED DESKTOP EDITION WITH SUPPORT FOR BOTH PC AND TOUCH ENABLED TABLETS WINDOWS 10 HOME IS AVAILABLE IN BOTH 32 BIT AND 64 BIT VERSIONS

WINDOWS 10 PRO

WINDOWS 10 PRO HAS FEATURES ESSENTIAL FOR SMALL OR MEDIUM BUSINESSESIT HAS A LARGE NUMBER OF EXTRA FEATURES TO MEET THE NEEDS OF SMALL BUSINESSES

WINDOWS 10 ENTERPRISE

WINDOWS 10 ENTERPRISE IS BUILT ON WINDOWS 10 PROIT HAS FEATURES WHICH ARE DESIGNED TO MEET THE NEEDS OF MEDIUM AND LARGE ORGANISATIONSTHIS WINDOWS EDITION TARGETS THE ENTERPRISE SEGMENT OF THE MARKET

WINDOWS 10 MOBILE

WINDOWS 10 MOBILE IS DESIGNED FOR CUSTOMER ORGANISATION THAT ARE USING THE WINDOWS 10 PLATFORM ON SMARTPHONES AND SMALL TABLETS

WINDOWS 10 EDUCATION

WINDOWS 10 EDUCATION IS DESIGNED TO MEET THE NEEDS OF STAFF ADMINISTRATORS TEACHERS AND STUDENTS OF SCHOOLS

WINDOWS 10T EDITION

WINDOWS 10T EDITION OF WINDOWS 10 IS DESIGNED FOR USE IN SMALL FOOTPRINT AND LOW COST DEVICES

Subject English language

Topic Noun kinds

NOUN KINDS

Noun is a naming word Nouns are names of people places animals or things

For example Ashley Mumbai tiger pencils

Kinds of nouns

Proper noun the name of a particular person place or thing is called proper nounExample

1 The Himalayas stand to the north of India2 Monika is the school captain

Common noun announce that names people place or thing in general is called a common noun Example

1 Kalidas was the greatest dramatist of India2 William Shakespeare is Englands national poet

Collective Noun A collective noun is the name of a collection of people or things taken together and spoken of as a whole Example

1 The feet were completely destroyed in the fierce attack2 The crew revolted against Captain Grand

Abstract Noun An abstract noun is the name of some quality state or idea Example1 Diversity Indias biggest strength2 Patience is a virtue

_____________________________________________________________________________________

Class VII

Subject English 1 Topic Articles

ARTICLES

There are three articles ------ A An The

1 Indefinite articles- The Articles a and an are called Indefinite Articles They do not point to a particular person or thing They are used with singular countable nouns It does not identify a specific noun

Uses of Indefinite articles A

Before a singular noun beginning with a consonant or a vowel with a consonant sound For example a cattle a university a one way track

Before a proper noun which is either unfamiliar or holds a special meaning For example For example A Mahesh Sharma wants to see youNeela wants to be a Tendulkar when she grows up

With a number beginning with a consonant sound For example a ten- rupee note a hundred years

Before half when half follows a whole number or after half when it isnrsquotExample one and a half litres (before) Half a litre (after)

With an expression of quantity For example a lot of time a dozen oranges a little while In exclamation before nouns Example Such a mess What a pity

Uses of Indefinite articles An

Before a singular noun beginning with a vowel sound Example an ant an egg an umbrella Before an abbreviation beginning with a vowel or a consonant with the vowel sound

Example an MP an MA Before the word beginning with a silent h Example an heir an hour an honour

Note We do not say a milk or a lemonade because they are uncountable nouns ever we see a cup of milk or a glass of lemon and

Subject GEOGRAPHY

CHAPTER 1 REPRESENTATION OF GEOGRAPHICAL FEATURES

TOPIC- TOPOGRAPHICAL MAPS

Topographical Maps- topographical maps are small-scale maps with detailed depiction of both natural and human-made features

Conventional colours-different colours used in a map are known as conventional colours

TYPES OF SCALE-

Verbal Statement- Scale written in a statement from like 2 cm to a Km is called verbal statement It means that 2cm on the map represents 1 km on the Earth

Representative Fraction- Scale can also be written as a representative fraction (RF)

Example 150000 it means 1cm on the map represents 50000cm or frac12 km on the ground It shows the ratio between map distance and ground distance

Linear Scale Scale can be drawn on a line to show map distance equivalent to ground distance A line is divided into equal parts Each parts represents the actual distance on the ground in mkm

SUBJECT-COMPUTER

CHAPTER-1 (COMPUTER FUNDAMENTALS)

COMPUTER LANGUAGES

THE THE TERM COMPUTER LANGUAGE REFERS TO A SYSTEM OF RULES AND SYMBOLS THAT ARE DESIGNED TO GIVE INSTRUCTIONS TO A COMPUTERTHE COMPONENTS OF A COMPUTER SYSTEM CANNOT PERFORM A TASK BY THEMSELVES THEREFORE THEY HAVE TO BE GIVEN INSTRUCTIONS TO PERFORM ANY TASKCOMPUTER LANGUAGES ARE USED TO CREATE PROGRAMS USING LOGIC BASED ALGORITHMSTHE RULES OF A COMPUTER LANGUAGE IS

KNOWN AS SYNTAX WHEREAS THE TERM SEMANTICS REFERS TO THE MEANING OF LANGUAGESCOMPUTER CANNOT DEVELOP ANY PROGRAMMING LANGUAGE BY THEMSELVES AND CANNOT THINK INTELLIGENTLY UNLESS THEY ARE ASSISTED BY THE HUMAN BEINGS THEREFORE THE PROGRAMMERS DEVELOP A SET OF METHODS AND TECHNIQUES A SET OF INSTRUCTIONS USED TO PERFORM A SPECIFIC TASK IS CALLED A PROGRAM

A PROGRAMMING LANGUAGE IS ALSO KNOWN AS A COMPUTER LANGUAGE CODED BY PROGRAMMERS TO WRITE INSTRUCTIONS FOR A COMPUTERTHE COMPUTER TAKES THESE INSTRUCTIONS AS INPUT AND PRODUCES THE DESIRED OUTPUT

TYPES OF COMPUTER LANGUAGE

THE COMPUTER LANGUAGE CAN BE BROADLY DIVIDED INTO TWO CATEGORIES

LOW LEVEL LANGUAGES HIGH LEVEL LANGUAGES

LOW LEVEL LANGUAGES

A LOW LEVEL PROGRAMMING LANGUAGE REFERS TO THE LANGUAGE THAT IS UNDERSTOOD BY A COMPUTER DIRECTLYTHE PROGRAMMER MUST HAVE AN IN-DEPTH KNOWLEDGE OF DIFFERENT COMPUTERS TO WRITE PROGRAMS IN A LOW LEVEL LANGUAGE THE TWO TYPES OF LOW LEVEL LANGUAGES ARE MACHINE LANGUAGE AND ASSEMBLY LANGUAGE

MACHINE LANGUAGE

COMPUTER HARDWARE UNDERSTANDS ONLY MACHINE LANGUAGE AS IT IS THE FUNDAMENTAL LANGUAGE OF A COMPUTER WHICH UNDERSTANDS ONLY THE TWO DIGITS 0 AND 1 KNOWN AS THE BINARY DIGITSIT IS A SYSTEM OF INSTRUCTIONS EXECUTED DIRECTLY BY THE CPU WITHOUT ANY TRANSLATION AND HENCE IS EXECUTED QUICKLY AS COMPARED TO PROGRAMS IN OTHER LANGUAGES PROGRAMS IN MACHINE LANGUAGE NEED DIFFERENT BINARY LANGUAGE PROGRAMS TO SOLVE THE COMPLEX TASKSPROGRAMMERS OFTEN FIND IT DIFFICULT TO WRITE PROGRAMS IN THIS LANGUAGE

ASSEMBLY LANGUAGE

AN ASSEMBLY LANGUAGE IS A LOW LEVEL PROGRAMMING LANGUAGE BUT THE INSTRUCTIONS ARE WRITTEN IN THE FORM OF WORDS KNOWN AS MNEMONICS TO CONVERT AN ASSEMBLY LANGUAGE INTO TO MACHINE CODE WE REQUIRE A UTILITY PROGRAM WHICH IS KNOWN AS AN ASSEMBLER ASSEMBLY LANGUAGE PROGRAMS CANNOT BE EXECUTED DIRECTLY BY A COMPUTER THEREFORE THEY ARE SLOWER THAN THE MACHINE LANGUAGE PROGRAMSIT IS EASIER TO WRITE PROGRAMS IN THE MACHINE LANGUAGE THEN AND IN ASSEMBLY LANGUAGE

_______________________________________________________________________________________________

CLASS-VIII

SUBJECT-COMPUTER CHAPTER - Operating system and graphical user interface Role and functions

Need of operating system

It co-ordinates different hardware and software componentsof a computer system It supervises the various actions of the computer system and enables the computer to work in a effective

manner It helps in smooth functioning of various peripherals

In a multi-tasking operating system it determines the order and time to be allowed for each application before giving another application a turn

It sends messages to the system operator about the status of operation or any error that may have occurred while running the application

What is operating system

An operating system is an integrated system of programs that manages various resources and the overall operation of the computer system It is designed to support various activities of computer system in a systematic way

Role of an operating system-

Operating system enables the user to use the system effectively An operating system manages various application that runs on a computer and shares computerrsquos resources User interacts with operating system through command line interface and graphical user interface

Function of an operating system -

Booting the computer it is the start up procedure of a computer system Loading theprograms in the memory - when system is ready the operating system loads certain program

automatically Manages resources it manages between the hardware and software resources Detecting and correcting errors- if the supporting hardware or software doesnrsquot works properly then the

operatingsystem tries to rectify it Ensuring data security programs and data donot interfere with each other Maintaining the internal clock of system - maintain internal clock of system when system is shut down

SUBJECT-GEOGRAPHY CHAPTER 1 REPRESENTATION OF GEOGRAPHICAL FEATURES THROUGH CONTOURSTopographical maps- topographical maps are small-scale maps with detailed depiction of both natural and human-made features

Contour lines- contour lines are imaginary lines drawn on a map joining places having the same height above the mean sea level

Contour linesContour interval- It is an interval at which contour lines are drawnIndex contour- At every 100-m interval a thick brown contour line is called index contour It is drawn for calculating height

Spot heights- The spot heights show heights in metres above the mean sea level

SUBJECT- English Language CHAPTER - The Sentence

Complex sentence-

We have seen that a Complex Sentence consists of a Principal Clause with one or more Subordinate Clauses

We have also learnt that there are three kinds of Subordinate Clauses The Adjective Clause the Adverb Clause and the Noun Clause

1) THE ADJECTIVE CLAUSE

An ADJECTIVE CLAUSE does the work of an ADJECTIVE It qualifies a noun or pronoun In the following examples in each set two commonly patterned sentences are compared one with an ADJECTIVE and the other with an ADJECTIVE CLAUSE ( The Adjective or Adjective Clause qualifies the Noun placed in the box)

a) He is a lazy boy (Adjective)

b) He is a boy who is lazy (Adjective Clause)

a) It is a beautiful place for the tourists (Adjective)

b) It is a place where tourists come for the scenic beauty (Adjective Clause)

a) We have enough funds for the work (Adjective)

b) We have funds which would be enough for the work (Adjective Clause)

Convert the sentence from simple to complex sentence-

1) We believe his honesty Ans- We believe that he is honest

2) This is the birth place of RamaAns- This is the place where Rama was born

____________________________________________________________________________________________

Class IX

Subject English Language

Topic Preposition

Date 270420

PREPOSITIONS

A preposition is a word placed before a noun or a pronoun to show in what relation the person or thing denoted by it stands in regard to something else

EXAMPLE

There is a clock on the wall

The preposition on shoes relationship between clock and the wall

Smitha is afraid of lizards The man jumped off the bus

Here the preposition of shows the relationship between afraid and lizards

The preposition off shows the relation between jumped and bus

The noun or the pronoun which follows a preposition is called its object

So in the first Example wall is the object of the preposition on

A preposition can have more than one object For example

The plane flew overhouses and meadows

Prepositions are used to express a number of relationship including time locationmannermeans quantity purpose and state or condition

Points to remember

Preposition joins a noun to another noun or a pronounThere is a cow in the field

A preposition joints a noun to a verbThe cat runs after the rat

Preposition can have two or more than two objectsThe road runs over hills and plain

A preposition also joins a noun an adjectiveHe is fond of tea

Generally a preposition comes before an object Sometimes it comes even after an object asWhat are you looking atThis is the house I live in

Subject- Computer Application

Chapter 2 Introduction to Java

Java API An application programming interface (API) in the context of Java is a collection of prewritten packages classes and interfaces with their respective methods fields and constructors

Byte Code Java bytecode is the result of the compilation of a Java program an intermediate representation of that program which is machine independent The Java bytecode gets processed by the Java virtual machine (JVM) instead of the processor JVM The Java Virtual Machine (JVM) is the runtime engine of the Java Platform which allows any program written in Java or other language compiled into Java bytecode to run on any computer that has a native JVM

Platform A platform is the hardware or software environment in which a program remains

Java platform The Java platform differs from most other platforms in that itrsquos a software only platform that runs on top of other hardware-based platforms

Applet and Application The fundamental difference between the two Java programs is that an application program is designed to run on a stand-alone machine whereas an applet is a web-version of an application which is used to run a program on a web browser

WORA ldquoWrite once run anywhererdquo (WORA) or sometimes write once run everywhere (WORE) is a slogan created by Sun Microsystems to illustrate the cross-platform benefits of the Java language

Class XSubject Topic Summary Execution

ECONOMICSFACTORS OF PRODUCTION Sub-topiclsquoLABOURrsquo

We shall start our class by discussing the topic taught in the last class

lsquo DIVISION OF LABOURrsquo- By division of labour we mean specialization in workIt refers to splitting up the work of labour involved in the production of a particular commodity into several parts and each part and sub-part is performed by a specialist

Now let us start by the Advantages of Division of labouraIt increases the level

Questions

1 Differentiate between Product-Based division of labour and Process-Based division of labour

Product-Based division of labour

Process-based division of labour

It is also known as simple (or occupational) division of labourUnder it everybody performs a particular occupations The entire is done by the same person

When a person or group of persons undertakes a specialised function which is supplementary to the production of final commodity and service This is also as complex division of labour

It is simple It is complex

of productionbSince the product is produced by an expert workerbest quality of product is producedcIt saves time and toolsd it promotes inventions in the methods and techniques of productioneIt leads to reduction in costs fAll workers get work according to their abilities and choices

Now let us discuss the disadvantages of Division of labour

a Since many workers are involved in the production of a commodity no one has the sense of responsibility

b The constant and repetition of the same work again and again make the work monotonous

c Division of labour facilitates production on large scale Hencethere is fear of over production

d Because of territorial division of labour some areasregions become more developed than others

It is based on labour-intensive techniques of production

It is based on capital-intensive techniques

It is generally found in small enterprises

It is generally found in large enterprises

Example Indian farmers doing all farm activities

Example A modern garmet factory where one person takes the measurementanother does the cuttingsome sew the clothes while a few workers button them and other iron them

2 What are the advantages of Division of labouraIt increases the level of productionbSince the product is produced by an expert workerbest quality of product is producedcIt saves time and toolsd it promotes inventions in the methods and techniques of productioneIt leads to reduction in costs fAll workers get work according to their abilities and choices

3Discuss the disadvantages of Division of laboura Since many workers are involved in the

production of a commodity no one has the sense of responsibility

b The constant and repetition of the same work again and again make the work monotonous

c Division of labour facilitates production on large scale Hence there is fear of over production

d Because of territorial division of labour some areasregions become more developed than others

English 1 Transformation of sentences

Sentences A sentence is a group of words which makes complete sense

a Assertive sentences

Exercise 6Rewrite the following sentences according to the instructions given below without changing their meanings

1 As soon as he saw the beer he jumped into

b Imperative sentences

c Interrogative sentences

d Exclamatory sentences

Sentences can be changed from one grammatical form to another without changing the meaning of the sentence This is known as transformation of sentences

the river ( Begin No sooner)2 None but brave deserve the fair (Begin the

bravehellip)3 This box is too heavy for me to lift ( Use so hellip

That instead of too)4 No one other than a king can live like James

Luxurious ( Begin only James)5 Oh for the wings of a dove (Begin I wishhellip)

Math Topic Commercial MathematicsChapter Shares and Dividends

Study item Discuss about shares and Dividends1) What is share

Ans To start any big business (company or Industry) a large sum of money is needed But it is not possible for an individual to invest such a large amount Then some persons interested in the business join together and from a company They divide the estimated money required into small parts Each such part is called a share

2) What do you mean by the term shareholder

Ans A person who purchases one or more shares is called shareholder3) Some terms related with a share

(i) Nominal value or face value or printed value The original value of a share is called its nominal value or face value or printed value

Note The nominal value of a share always remains same(ii) Market value or cash value The price of a share at any

time is called its market value or cash value

Note The market value of a share changes from time to time(iii) At par If the market value of a share is the same as its

nominal value the share is called at par(iv) At Premium or above Par If the market value of a share

is more than its nominal value the share is called at premium or above par

Example If a share of Rs 100 is selling at Rs 150 then it is said to be selling at a premium of Rs 50 or Rs 50 above par

(v) At Discount or below par If the market value of a share is less than its nominal value the share is called at discount or below par

Example If a share of Rs 100 is selling at Rs80 then it is said to be selling at a discount of Rs 20 or at Rs 20 below par

4) What is Dividend

Ans The profit which a shareholder gets for hisher investment from the company is called dividendNote (i) The dividend is always expressed as the percentage of the face value of the share(ii) The dividend is always given( by the company ) on the face value of the share

irrespective of the market value of the shareBENGALI(2ND LANGUAGE)

ldquoদেবতোর জণমrdquoলিবরোম চকরবত

পরথম লিসর পোঠ-চোর পসথ একটি পোথর লিবপলি ঘটোয় দেক যোতোয়োসতর পসথ পরলিতলিয়ত ওই পোথসর দেো োচট দেসত একলি দেতো দেক দেো োচট দেসয় দেবোমো সয় রোসতোর মোস লি0টসক পস1 লিবপরীত লিক দেথসক আো একটি দেমোটর োলি1 চোসকর কষতোয় পরোসরণ দেবোসচ যো লিকনত পরলিতবোর এমরণ দেৌভোয দেসকর োও সত পোসর তোই লিতলি দেকোো দেজোো1 কসর পোথরটিসক উপস1 দে8স পোথর উপস1 দে8োর ময় এক দেকৌতী জতো দেকসক পরশন কসর দেয লিতলি দেকোসো দেবতোর আস দেপসয়স0রণ লিকো লিকনত দেক বস লিতলি দেকোসো দেবতোর আস পোলি দেক উপলিত ক জতোর উসltসয বস কোরও ইস= স পোথরটি লিসয় দেযসত পোসর এর পর দেথসক দেক দেযসত আসত পোথরটি দেক দেসত পো একলি দেক কষয করস দেকউ পোথরটিসক ধসয়মস0 পলিরসকোর করস0 দেক ওই দেকৌতী জতোসক পোথসরর কোস0 বস থোকসত দেস এ0ো1ো আরও কষয কসর দেক ঠোৎ ওই লি1 পোথরটির োসয় লিোর োো-দেকউ পজো কসরস0 দেকৌতী দেোকটির আঙকো য় যলিসক উ পোথরটিসক লিরসয় দে8স তোর পর ঠোৎ একলি পোথরটির দেোোজ দেই দেক লিসয় দেস0 বো দেকোথোয় দেস0

বদোথ-

দেো োচট ndashচসত লিসয় দেকো লিক0র সE ধোককো দেস পস1 যোবোর উপকরমঅকসমোৎ- ঠোৎআতমমবর- লিসজসক লিয়নতরপ-পো লিপ0স প1োদেসতসসত- লিসপলিউৎোত- দেো1ো দেথসক উপস1 দে8োপরতযয় ndash লিবশবোপরতযোস- টতযোস ndash দৈবোসধসতোধলিসত- পরসপসরর পরলিত ব পরসয়ো করোপরসতরীভত- পোথসর পলিররণলিতবোনতঃকরসরণ- মস পরোসঅলিQৎ- অQো করসত ই=কদেোপ- বধমোর- দেবোস এমইতযোকোর- এইরকমরম- বময়পযসোভী-পসযর জয দেোভ আস0 যোরপোসথয় ঞচয়- পথচোর রচ জমোসোমোমোসরোস- ব ধম ধোসমর সE

তোর লি দেইhelliphellipTo be continued

Hindi 2nd lang सर क पद(सरदास)

सरदास शरी कषण भकति कावय क सरवशरषठ कगिरव ह इनक जनम और मतय क समय तथा सथान का मतभद हसरदास रवातसलय और शरार रस क अनयतम कगिरव ह इनक कावय म बालकषण क सौदय चपपल चषटा और गि7याओ की मनोहर झाकी मिमलती ह कषण और ोगिपयो क अननय परम का कतिचतरण ह सयो शरार की अपकषा उनक कावय म गिरवयो शरार का अमिBक गिरवषय और मारमिमEक कतिचतरण हआ हइन पकतियो म हम सरदास की भकति भारवनाओ का परिरचय मिमलता ह इनका सपण सगरह सरसार म गिनगिहत ह

1 जसोदा हरिर पालन झलारवहलरारवदलराईमलहारव रव जो ईसाई कछ ारव मर लाल को आई निनEदिदया काह ना आगिनसबारव त काह नाही बरवगिह आरवतोको कानहा बलारव

शबदाथ-हलरारव-गिहलती हदलराई - दलार पयार करती हमलहारव-पचकारती हनिनEदरिरया ndashनीदरवगिह-जलदी सअBर-होठमौन-चपसन-सकत

वयाखया- सरदास जी कहत ह गिक यशोदा माता बालक कषण को पालन म झल आती ह रवह उनह गिहलाती ह पयार करती ह मलहार जस कोई ीत ान लती ह और नीद स पछती ह गिक ह नीद तम मर लाल को आकर कयो नही सलाती तझ खाना बला रहा ह कभी कषण आख बद कर लत ह कभी आखफडफडान लत ह उनह सोता हआ जानकर यशोदा माता चप हो जाती ह और इशार म बात करन लती ह इसी बीच अकला कर कषण ज जात हतो गिफर यशोदा माता गिफर स ाना ान लती ह सरदास जी कहत ह गिक भरवान क दशन का सख दरवता और ऋगिष-मगिनयो को भी दलभ ह यही सख माता यशोदा को बडी सहजता स मिमल जा रही ह माता यशोदा बहत ही भागयशाली ह2)Continue to nexthellip

Physics

Chapter 1 Force

(Summary)

Question A body is acted upon by two forces each of magnitude F but in opposite directions State the effect of the forces if

(a) Both forces act at the same point of the body

(b)the two forces act at two different points of the body at a separation r

Solutions

(a) Resultant force acting on the body = 0

F ndash F = 0(b) The forces tend to rotate the body between two forces about the midpoint

Moment of forces = F times rFr

QuestionDefine moment of a couple Write its SI unit

Solutions

Moment of couple is equal to the product of both force and the perpendicular distance between the two forces

The SI unit of moment of couple is NmCommercial Studies

Advertising and sales

Business firms use several methods to

Questions1) What do you mean by advertising

promotion create demand of their product in the market and increase it sales Such methods comprises of advertising sales promotion personal selling and publicityToday we are going to discuss about one of such methods It is advertising

Meaning of advertisingAdvertising is a paid form of non-personal presentation for promotion of Ideas goods and services

Importance or merits of advertising Advertising has importance to manufacturer or traders to customer and to society as a whole

Today we will see how advertisement help the manufacturer or traders

Answer) Advertising is a means of how a company encourages people to buy their products services or ideas It is one element of marketing which also includes design Research and data mining

2) Mention any three features of advertisingAnswer)The main features of advertising are

i) It is impersonal form of presentation for promotion of products and services of Ideas

ii) It is issued by identified sponsor The advertisement contains the name of the advertiser

iii) It is a form of mass communication because the message is directed to a large number of persons simultaneously

3) Mention the main merits or importance of advertisement to manufacturer or tradersAnswer)

i) Introducing new product A business organization can introduce itself and its products to the public through advertising

ii) Increase the sale Advertising leads to increase the sale of existing product by entering into new markets and attracting new customers

iii) Create steady demand Advertising creates sustains regular demand by smoothening out seasonal and other fluctuations It enables regular production for the organisation

iv) Economics of scale Advertising facilitate mass distribution of goods and steady demand which lead to large scale and regular production

v) Goodwill Advertising helps in creating a good image of the firm and reputation for its products

Biology Chapter - 03Genetics

Today wewill start chapter and discuss about Genetics Gregor Mendel is known as father of genetics Before entering into Mendelrsquos experiment on Genetics we must know

Q1 Define the following termsi) Genetics Genetics is the study of

transmissionof body features from parents to offspringand the laws relating to such transmission

ii) Heredity It may be defined as transmissionof genetically based characteristics from parentsto offspring

iii) Character and traits Any heritable

Importance to TraderIntroducing new productIncrease the saleCreate steady demandEconomics of scaleGoodwill

some terms featureis a character The alternative forms of acharacter are called traitsex Character (Hair shape) - Traits (Curly straight)

iv) Homologous chromosomes A pair ofcorresponding chromosomes of the same shapeand size one from each parent

v) Genes Genes are the specific parts (DNA segments) of a chromosome which determinethe hereditary characteristicsNearly 30000genes present in human

vi) Alleles Alternative forms of a gene occupying the same position (locus) on homologouschromosomes and affecting the same characteristicbut in different ways

vii) Genotype ndash PhenotypeGenotype means of genes present in the cells of an organism Phenotype means the observable characteristic which is genetically controlled

viii) Mutation It is a sudden change in one or more genes or in the number or in the structure of chromosomes ex Sickle cell anaemia is a blood disease caused by a gene mutation

CLASS NOTES

Class XSubject Eng Literature (The Merchant of Venice ndash William Shakespeare)Topic Act IV Scene 1 Lines 01 to 34 ( Duke helliphelliphellip We all expect a gentle answer Jew) ate 27th April 2020 (2nd Period)

[Students should read the original play and also the paraphrase given in the school prescribed textbook]Summary Questions amp Answers

This scene may be termed as the catastrophe of the play It is the final unravelling of the complicated events which seem to threaten the happiness of Bassanio Portia and Antonio Right is justified to the fullest degree and malice falls into the trap prepared for others No one suffers here but Shylock but even then he receives a generous measure of mercy

o This is the Court-scene Initially we meet

(1)

DUKE I am sorry for thee thou art come to answer (Line 3-6)A stony adversary an inhuman wretchUncapable of pity void and emptyFrom any dram of mercy

(i) Who is addressed here Where is the person Why is the person there

Antonio is addressed hereAntonio is in the court of justice at VeniceAntoniorsquos trial is scheduled to be held here for his failure to meet the conditions of the bond he signed with Shylock

the Duke Antonio Shylock and Salerio Later we meet Bassanio Portia Gratiano and Nerissa

o The Duke says to Antonio that he has to face a very cruel opponent which Antonio admits and expresses his gratefulness to the Duke for his efforts to soften without result the heart of Shylock in order to be merciful to Antonio Antonio further says that he is ready to accept whatever cruel judgement the Court may award

o When Shylock appears in the court the Duke says that Shylock should change his decision of prosecuting Antonio and demanding the penalty specified in the bond out of consideration of the great misfortunes that Antonio has suffered If this is done by him (Shylock) the whole court would be gladdened by his merciful action

(ii) What is the Duke sorry for

The Duke is unable to change the mind of Shylock from his decision to get the bond forfeited even after he pleaded to Shylock Shylock stands firmly for his bond which when forfeited will allow him to take a pound of flesh from any part of Antoniorsquos body(iii) How does the Duke address Shylock`The Duke calls Shylock an adversary with a heart of stones He calls Shylock as an inhuman wretch without pity Shylock is quite lacking in the slightest quality of mercy (iv) How does Antonio reply to this

Antonio replies that he will meet the revenge of Shylock patiently He has prepared himself to suffer with a quiet spirit the utmost that Shylockrsquos tyranny and rage can do(v) What quality of the Duke is revealed here

The Duke is kind and benevolent He is ready to help Antonio He requests Shylock to free Antonio from the trial(vi) What are the terms of the bond that Antonio has signed

The terms of the bond that Antonio has signed were that if Antonio is unable to repay Shylock a certain sum of money specified on the paper on a certain date and in an agreed place the forfeit has to be paid The forfeiture will be an exact pound of Antoniorsquos flesh which Shylock will be a liberty to take from any part of Antoniorsquos body which pleases him

Class XI

STUDY MATERIAL

Class XISubject Eng Literature (The Tempest ndash William Shakespeare) Topic Act I Scene 2 Lines 88 to 132 (Prospero hellip Me and thy crying self) Date 27th April 2020 (3rd Period)

[Students should read the original play and also the paraphrase given in the school prescribed textbook]Summary Questions amp Answers

o Prospero now tells Miranda that he was the Duke of Milan He had been devoting himself more to studies than the affairs of the State His brother Antonio took advantage of this situation and with the help of Alonso the king of Naples seized upon him and her one midnight and shipped them in a frail bark so that they perished in the sea All this took place

(1)

MIRANDA I should sin (Line 118-132)

To think but nobly of my grandmother

Good wombs have borne bad sonsPROSPERO Now the condition

The King of Naples being an enemyTo me inveterate hearkens my brothers suitWhich was that he in lieu othrsquo premisesOf homage and I know not how much tribute

twelve years back

IMPORTANT PASSAGES EXPLAINED(Line 98-103)

PROSPERO helliphelliphelliphelliphelliphelliphellip

He being thus lorded

Not only with what my revenue yieldedBut what my power might else exact like oneWho having into truth by telling of it Made such a sinner of his memoryTo credit his own lie he did believeHe was indeed the duke

Prospero in telling the narrative of his past life here refers to his brother Antonio Prospero being with a studious bent of mind has left the administration of Milan on his younger brother Now Antonio being thus invested like a lord with all the powers derived from Prosperorsquos wealth and what the exercise of Prosperorsquos authority might secure for him regarded himself as a de facto Duke of Milan It is a well-known fact of psychology that a man who repeatedly tells a lie makes of his memory such a sinner against truth as to credit his own lie by the telling of it So Antonio by repeatedly saying to himself and others that he was the Duke came to believe that he was really the Duke Thus falsehood repeatedly asserted gained the force of truth for Antonio and he truly believed it

Should presently extirpate me and mine Out of the dukedom and confer fair MilanWith all the honours on my brother whereonA treacherous army levied one midnightFated to thrsquo purpose did Antonio openThe gates of Milan and ithrsquo dead of darkness The ministers for thrsquo purpose hurried thenceMe and thy crying self

(i) In the earlier lines of this scene what does Prospero tell about his intense interest What was the demand of his interest

In the earlier lines of this scene Prospero tells Miranda that he had an intense interest in the study of philosophy and magic arts Hence in order to improve his mind with this kind of study he kept himself isolated from worldly and state affairs His study was dearer to him than the applause and esteem that he could win from the public His study demanded too much solitude(ii) What forced Antonio to take an undue advantage over Prospero

Prosperorsquos indifferent attitude towards the statersquos affairs and his having boundless trust in Antonio gave rise to a boundless lust for power in Antoniorsquos mind Antonio felt that he must be the actual Duke instead of the part of the Duke he played Thus Antonio took an undue advantage of the situation to usurp Prosperorsquos dukedom(iii) Explain the following lines ldquoI should sin to think but nobly of my grandmother Good wombs have borne bad sonsrdquo

After hearing the treacherous act of her uncle Antonio Miranda says that Prosperorsquos mother was a noble lady and she cannot dishonour her memory by saying that the person named Antonio cannot be his (Prosperorsquos) brother She says that in honour of her grandmother she also cannot say that Antonio must have been begotten not by her grandfather but by some other man She finally concedes that it is known that good mothers have borne bad sons in their wombs and gave birth to them

(iv) Why did the King of Naples accept Antoniorsquos request to help him in usurping his dukedom What did Antonio propose to Alonso

Alonso who was the king of Naples accepted Antoniorsquos request in usurping Prosperorsquos dukedom because he (Alonso) was a sworn enemy of Prospero Antonio proposed that Alonso should immediately drive him (Prospero) and his offspring out of Milan and should confer the dukedom upon him (Antonio) with all the dignities which go with that

position In return Antonio promised that he shall give an annual tribute and also swore his allegiance to Alonso Also he agreed to hold the Dukedom of Milan as a subordinate to the state of Naples(v) How were Prospero and Miranda carried away from the city of Milan and what was the state of small Miranda at that time

In pursuance of the agreement settled between Antonio and Alonso an army of treacherous men was assembled One midnight when the occasion suited the will of destiny Antonio opened the gates of the city of Milan and in the death like silence of midnight Antoniorsquos agents who had been directed to execute his purpose carried Prospero and small Miranda away from the city in all haste They were then forced into a ship and carried some distance out to sea where they put them on a mere hulk of a boat without any rigging or ship-gear and abandoned them leaving them at the mercy of the roaring sea Miranda was a very small child of three years age and she was crying at that time

CLASS -XIDATE-270420Subject Topic Summary Execution

EVS Chapter 1 ndash Mode of Existence

Impact of mode of existence on resources

Q) Why resources are under pressure

Ans - Increase in the sophistication

of technology enabling natural resources to be extracted quickly and efficiently Eg in the past it could take long hours just to cut down one tree only using saws Due to increased technology rates of deforestation have greatly increased

The number of humans is increasing Cultures of consumerism Materialistic views

lead to the mining of gold and diamonds to produce jewelry unnecessary commodities for human life or advancement Consumerism also leads to extraction of resources for the

production of commodities necessary for human life but in amounts excessive of what is needed because people consume more than is necessary or waste what they have

Lack of awareness among the population is striking People are not aware of ways to reduce depletion and exploitation of materials

Accounts Cash Book Today we are going to start a new topic -Cash Book

The key terms used in this chapter are

bullCash book

bullSimple cash book

bullDouble column cash book bullTriple column cash book

bullPetty cash book

bullCash discount

bullContra entry

Here I will share you the meaning of each key terms

bullCash book Cash Book is a special purpose subsidiary book or journal in which cash received and cash payments are recorded

bullSimple cash book

It is a cash book in which only cash transactions are recorded It has only one column on each side

bullTriple column cash book

It is cash book which has three columns one column for each cash and Bankdiscount on each side of the cash book In this book both cash and Bank transactions are recorded together with discount allowed and received

bullPetty cash book

It is a cash book maintained for recording petty expenses

bullCash discount

Cash discount is the amount of discount received or allowed on cash payments and cash receipts Discount received is an income for the business while discount allowed isan expense

bullContra entry

It means transactions involving both cash and Bank Such transactions though recorded in the cash book are not posted into ledger The letter lsquoC is written in Ledger folio for contra entry

Business Studies

ENTREPRENEURSHIP

Now we shall discuss the second chapter

lsquoENTREPRENEURSHIPrsquo

Today before starting the chapter let us recall what

Questions

1What are the main characteristics of Intrapreneurship

Answer

The main characteristics of Intrapreneurship are

Corporate framework-it occurs within the framework of the same company

Semi-Autonomous-Intrapreneurship

we have read last day

Let s today start the class by recalling the last topic taught

Intrapreneurship is the process of discovering and exploring business opportunities within an existing company It involves launching new business ventures within the framework of a present corporation Intrapreneurship is also known as corporate entrepreneurship or corporate venturing

Now let us start with the characteristics of Intrapreneurship

The main characteristics of Intrapreneurship are

Corporate framework

Semi-Autonomous Lack of ownership Senior position Low risk taking Not own boss

Now let us discuss the meaning of enterprise

Enterprise means an undertaking or adventure that requires some innovation and investment and thus involves riskEnterprise always entails decision making coordination and risk bearing

involves crating amd nurturing a semi-autonomous business unit which may be a subsidiary a strategic business unit or a division

Lack of ownership-the intrapreneur is not the owner of the unitb he creates and nurtures

Senior position-he occupies a senior managerial position in the company

Low risk taking-An intrapreneur does not bear the full risk of failure

Not own boss-An intrapreneur is not his own bosss in legal termsHe enjoys the freedom and gets the required resources and support

2 How is Entrepreneur is different from Intrapreneur

The functions involved in both the entrepreneurship and intrapreneurship are by and large similar however there are several differences between the two

Point of distinction

Entrepreneur

Intrapreneur

status An independent business person

A senior executive within a company

Ownership Owner of

the business

An employeesometimes a share in ownership

Financing Responsible for raising finance for the business

Not responsible for raising the finance

Risk bearing

Bears the risk of the business

Does not bears the risk of the business

Reward Profit which is uncertain and irregularcan be loss

Fixed salary and fringe benefits

Need for security low high

3 What do you understand by enterprise

Answer Enterprise means an undertaking or adventure that requires some innovation and investment and thus involves riskEnterprise always entails decision making coordination and risk bearing

COMMERCE NATURE AND OBJECTIVES OF

BUSINESS

Today let us recall the last other two objectives of business by the chart given in the previous class

Firstly we would discuss Human Objectives

Business is run by people and for people Labour is a valuable business element

Human objectives of business are concerned with the well -being of labour

The human objectives are as follows

Labour welfare Developing human

resources Participative

management Labour

management cooperation

Questions

1 Explain the human objectives of a business enterprise

Answer

Business is run by people and for people Labour is a valuable business element

Human objectives of business are concerned with the well -being of labour

The human objectives are as follows

Labour welfare-Business must recognize the dignity of labour and human factors should be given the recognition

Developing human resources-Employees must be provided the opportunities for developing new skills and attitudes

Participative management-Employees should be allowed to take part in decision making process of business

Labour management cooperation-Business should strive for creating and maintaining cordial employer employee relations so as to ensure peace and progress in industry

Now let us discuss the national objectives of business

Optimum utilization of resources

National self- reliance Development of small

scale industries Development of

backward areas Control over pollution

2Explain the national objectives of a business enterprise

Answer

It is the duty of business to utilize the resources of the country properly the national objectives of business

Optimum utilization of resources ndashBusiness should use the nationrsquos resources in the best possible manner

National self- reliance-It is the duty of the business to help the government in increasing experts and in reducing dependence on imports

Development of small scale industries-Big business firms are expected to encourage growth of small scale industries which are necessary for generating employment

Development of backward areas-Business is expected to give preference to the industrialization of backward regions of the country

ECONOMICS

BASIC ECONOMIC CONCEPTS

SUB

TOPIC

Value

Wealth

Welfare

Today we shall start with a new topic of the same chapter ie lsquoValuersquo

Value of a commodity is defined as the valuation placed by a household on the consumption of this commodity

lsquoValuersquo has two different meanings and these are

a Value -in -use It refers to consumption value of a commodity It expresses the utility derived from the consumption of a particular commodity A necessity like water has a very high value ndashin ndashuse or

Question

1What is value

Answer

Value of a commodity is defined as the valuation placed by a household on the consumption of this commodity

2What is value-in use

Answer It refers to consumption value of a commodity It expresses the utility derived from the consumption of a particular commodity A necessity like water has a very high value ndashin ndashuse or consumption value

3What is value ndashin- exchange

Answer It relates to market value of a commodity

It is the rate at which a particular good or service can be exchanged for money

For example in barter system if a person is prepared to exchange 3 metres of cloth with 1 pair

consumption value

b Value ndashin-exchange It relates to market value of a commodity

It is the rate at which a particular good or service can be exchanged for moneyFor example in barter system if a person is prepared to exchange 3 metres of cloth with 1 pair of shoes then the value in exchange of 3 metres of cloth is 1 pair of shoesValue in exchange is the power of purchasing other goods In modern monetised economies the exchange value of goods are expressed in terms of money as prices

Now let us discuss the term lsquoWealthrsquo

Wealth refers to the stock of all those assets which are a source of income

Wealth is a stock concept

Wealth must possess the following features

a Utility It must possess utility or give some

of shoes then the value in exchange of 3 metres of cloth is 1 pair of shoes

Value in exchange is the power of purchasing other goods In modern monetised economies the exchange value of goods are expressed in terms of money as prices

4 What is wealth

Answer it refers to the stock of assets or goods which are a source of income and have personal or national ownership

5 What are the features of wealth

Answer The features of wealth are as follows

Wealth must possess the following features

a Utility It must possess utility or give some satisfaction

b Scarcity It must be limited in quantityc Transferability it should be transferable its

ownership can be transferred from one person to another person

d Exchange value It must possess exchange value

6 What is welfare

Answer

Welfare is defined as satisfaction and happiness a sense of well- being among the people

satisfactionb Scarcity It must be

limited in quantityc Transferability It

should be transferable its ownership can be transferred from one person to another person

d Exchange value It must possess exchange value

Now let us discuss the term lsquoWelfarersquo

Welfare is defined as satisfaction and happiness a sense of well- being among the people

Welfare is affected by factors like

a Consumption of goods and services

b Environment

c Family relations

d Degree of freedom

e Law and order situation

Mathematics Trigonometric equation

To find the general solution of the equation sinθ=0

When sin θ =0

Then θ= 0 π2π 3π-π -2π -3

i e when θ = 0 or an integral multiple of π

i e when θ= nπ where n is any integer

Therefore the general solution of the equation sin

Example1 Find the general values of θ which satisfy the equation sin2 θ =34

Solution sin2 θ= 34

Or sin θ = +34 or -34

Or sin θ = sin π3 or sin (-π3)

Therefore

θ = [nπ + (-1) n (π3)] or[ nπ+ (-1) n (-π3)]

= nπ +π3 or nπ-π3 where n= any integer

Example 2Find the values of θ which satisfy tan2 θ

θ=0 is θ= nπ where n is any integer

To find the general solution of the equation cos θ=0

When cos θ=0

Then θ=π2 3π2 5π2 -π2 -3π2 -5π3

i e when θ is an odd multiple of π2

i e when θ=(2n+1) π2 where n= any integer

Therefore the general solution of the equation cos θ =0 is θ= (2n+1) π2 where n= any integer

To find the general solution of the equation tan θ = 0

Clearly tan θ =0 implies sin θcos θ =0

Therefore θ = nπ

i e the general solution of the equation tan θ=0 is θ =nπ where n = any integer

To find the general solution of the equation cot θ =0

Clearly cot θ =0 implies (cos θsin θ) = 0

i e cos θ =0

Therefore θ = (2n+1) π2

Therefore the general solution of the equation cot θ =0 is θ = (2n+1) π2

Where n= any integer

To find the general solution of the equation sin θ= k (-

=13 -πleθleπ

Solution tan2 θ =13

Or tan θ = plusmn1radic(3) =tan(plusmnπ6)

θ=nπ plusmn π6 where n =any integer

If n=0 then θ=plusmnπ6

If n=1 then θ= π plusmn π6

If n=-1 then θ= -π plusmn π6

Therefore the required solution in -π le θ le π are θ= π6 5π6 -π6 -5π6

Exercise Find general solution of sin 2θ=cos θ [Hints Use sin 2θ= 2sin θcosθ and then take cosθ

common]

1lek le1)

Determine an angle alpha such that sin =k and -π2le αle π2

Then we have

Sin θ = k = sin α

Or sin θ - sin α =0

Or 2 cos [(θ+α) 2] sin [(θ-α) 2] =0

Therefore either cos [(θ +α) 2] =0 (1)

Or sin [(θ-α) 2] =0 (2)

Now from (1) we get (θ+α) 2= (2m+1) π2)

Or θ = (2m+1) π-α (3)

And from (2) we get (θ-α) 2 =mπ

Or θ= 2mπ+α(4)

Where m = any integer

Clearly the solution (3) amp (4) may be combined in the following form

θ= nπ+(-1) n α where n= any integer

Therefore the general solution of sin θ = sin α is θ = nπ +(-1) n α where n is any integer and -π2 le α le π2

Biology Chapter - 04Kingdom Monera

Today we will discuss about bacterial reproduction and its usefulness

Fig Binary Fission

Fig Conjugation Fig Transformation

Bacterial reproduction is mainly asexual but sexual reproduction

also takes place

Asexual reproduction takes place by i) Binary fission - from one bacteriato

two bacteria are produced in every 20 to 30mins

ii) Buddingiii) Endospore formation - during

unfourable condition

Sexual reproduction by three ways

1) Conjugation - Transfer of genetic material between cells that are in physical contact with one another

2) Transduction - Transfer of genetic materialfrom one cell to another by a bacteriophage

3) Transformation - Transfer of cell-freeor naked DNArsquo from one cell to another

Bacteria causes different diseases inplants animals and human and

it causes food spoilage and waterpollution but it also have some useful

activities

i) Bacteria are helpful in sewage water treatment

ii) It is used in antibiotic (medicine) production

iii) Anaerobic bacteria help in biogas(energy) production

iv) Many household products like yoghurt cheese are manufactured by use of bacteria

v) Rhizobium by symbiotic relationship with leguminous plant increase soil fertility

vi) Besides these bacteria is helpful in genetic engineering degradation of petroleum hydrocarbonand in dairy

industry

Physics Motion in plane Here we will introduce Projectile Motion

Execution

Projectile

Y

usinθ u h

θX

ucosθ

Suppose a body is projected with an angle θ So initial velocity u can be resolved into two components

Horizontal component - ucosθ ( for range)

Vertical component - usinθ ( for height)

usinθ changes during motion and becomes zero at maximum height position but ucosθ remain unchanged

The maximum height of projectile is h

NB If initial is upward then g = -ve and if it is downward then g = +ve Height is +ve if direction of motion does not change ( for ex a body thrown upwards but goes down ultimately then height h = -ve)

The angle of projectile θ is the angle made with horizontal

HISTORY ndash GROWTH OF NATIONALISM

SUB TOPIC- REVOLUTIONARY NATIONALISM Bengal formation of Anushilan Samity and Jugantar Group

The intensification of the Swadeshi movement and Government policy of terror and repression led to outbreak of violence Bombs were manufactured and attempts on the lives of unpopular Government officials became frequent In the gymnasium of Scottish Church College which was known as General Assemblies Institution a secret society was formed known as Anushilan Samity

Aurobindo Ghosh send from Baroda his emissary Jatindranath Banerjee to mobilize the Bengal revolutionaries

Hemchandra Qanungo and Satyen Bose published Journal Jugantar

The Jugantar group planned to assassinate oppressive magistrate Kingsford by Khudiram Bose and Prafulla Chaki in 1908 Prafulla Chaki committed suicide to avoid arrest Khudiram was tried and hanged

Afew days later the police found a bomb factory in Maniktala and arrested a large number of revolutionaries The trial of revolutionaries became famous as the Alipore Bomb Case

In the course of the trial the approver the public prosecuter and a police officer were assassinated

1 Question Name two journals which preached the cult of violence

Answer a) Yugantar edited by Bhupendranath DuttaB) Bandemataram edited by Aurobindo Ghosh2 Question Why was Khudiram arrested and hangedAnswer An attempt was made to assassinate a hated vindictive majistrate named Kingsford by Khudiram Bose and Prafulla Chaki Their attempt failed and the bomb they threw killed two English ladies Khudiram was arrested and put to trial and then hanged3Question Who was Aurubindo GhoshAnswer Aurobindo Ghosh a nationalist revolutionary who was charged for his involvement in the Alipore Bombing Case He was accused of it along with his brother Barindra nath Ghosh But Aurobindo was acquitted because of the brilliant pleading of his counsel Chittaranjan Das Then he became a spiritual reformer introducing his visions on human progress and spiritual evolution4 Qustion Who was KingsfordAnswer Kingsford was an unpopular British chief Magistrate who was the target of the bomb thrown at Muzaffarpur by Khudiram and Prafulla Chaki

Most of the accused were convicted and sentenced to

long term of imprisonmentBut

Aurobindo Ghosh was acquitted mainly owing to the brilliant pleading of his counsel Chittaranjan Das

Political science Topic-Sovereignty

Summary Sovereignty is the full right and power of a governing body over itself without any interference from outside sources or bodies In political theory sovereignty is a substantive term designating supreme legitimate authority over some polity In international law sovereignty is the exercise of power by a state

Internal Sovereignty

Internal sovereignty means supreme authority within ones territory while external sovereignty relates to the recognition on the part of all states that each possesses this power in equal measure

External sovereignty

external sovereignty relates to the recognition on the part of all states that each possesses this power in equal measure

Distinguish between

Execution

Answer the following questions

Short notes-

Sovereignty

Internal Sovereignty

External sovereignty

Homework- learn

external sovereignty and internal sovereigntySovereignty is the principle

of supreme and

unquestionable authority

reflected in the claim by the

state to be the sole author of

laws within its territory

Definition of external vs internal sovereigntyInternal sovereignty refers to

the relationship between a

sovereign power and its

subjects ndash it refers to the

location of the supreme

authority within the state In

the UK for example internal

sovereignty (supposedly)

resides within Parliament

reflected in the

constitutional principle of

parliamentary

sovereigntyBy contrast

external sovereignty refers

to the capacity of the state

to act independently and

autonomously on the world

stage This is what is

sometimes called lsquostate

sovereigntyrsquo or lsquonational

sovereigntyrsquo and implies

that states are legally equal

and that the territorial

integrity and political

independence of a state is

inviolable

Class ndash XII

Date - 2742020 STUDY MATERIALSubject Topic Summary Execution Business Studies

Job Analysis amp Manpower Planning

At first let us recall the chapter what we have discussed till nowbullJob analysisbullJob specification bullJob description bullJob enlargement bullJob enrichment

Today we will do some questions answers from the chapter

Questions 1ldquoJob analysis job description and job specification are interrelatedrdquo Comment Answer) Job analysis is a systematic and detailed examination of a job to collect all the relevant information about it The contents off the job are summarised in the job description The qualification needed for the job are summarised in job specificationThus there is close interrelationship between job analysis job description and job specification

Question 2ldquoJob enlargement is a horizontal extension of a job whereas job enrichment is a vertical extension of a jobrdquo ElucidateAnswer) Job enlargement involves adding one or more task to a job coma where as job enrichment involves adding more autonomy and responsibility to a job Job enlargement is therefore horizontal extension of a job coma whereas job enrichment is a vertical extension of a job

Question 3 )

What is manpower estimation Explain its quantitative and qualitative aspectsAnswer) Manpower estimation is the process by which management determines how an organisation should move from itrsquos current manpower positionto its desired manpower position There are two dimensions of Manpower estimation- quantitative and qualitative

Quantitative aspectThis aspect of Manpower estimation involves estimating the number of employees required in a future time period Workload analysis and workforce analysis are done to estimate the quantity of required manpower

Qualitative aspectThe estimate of the knowledge skills experience etc of required manpower is the qualitative aspect of Manpower estimation The quality of Manpower can be judged on the basis of job analysisand job specification

COMMERCE

CAPITAL-FIXED AND WORKING

Today let us start the class by discussing the sources of finance for different types of business firms

The term lsquocapitalrsquo refers to the investment made in the enterprise for the purpose of earning profits

Requirements of capital and sources of capital for different types of business firms are

1 Capital for sole proprietorship businessA sole proprietor operates at a small scale and thereforerequires a limited amount of capital

2 Capital for partnership firmCapital requirements as well as capital base of a partnership is bigger than that of a sole trader businessThe owned capital is contributed by the partners in an agreed ratio

3 Capital for joint stock companyA joint stock company generally requires large amount of capitalA public company can raise huge capital through issue of shares In addition to share capital it can utilize retained profits

Now let u discuss the meaning of Finance PlanningFinance planning is the process of estimation the financial requirements of an organization specifying the sources of firms and ensuring that enough funds are available at the right time

1 What do you mean by Finance PlanningAnswer Finance planning is the process of estimation the financial requirements of an organization specifying the sources of firms and ensuring that enough funds are available at the right time

2Discuss the role of financial planning of an enterpriseThe role of financial planning are as followsa A sound financial plan helps a business enterprise to avaid the problems of shortage and surplus of fundsbFinancial planning serves as a guide in developing a sound capital structure so as to maximize returns to shareholders c It helps in effective utilization of fundsd It provides policies and procedures for coordinating different functional areas or departments of businesse It enables the management to exercise effective control over the financial activities of an enterprisef It helps the company to prepare for facing business shocks and surprises in future

Mathematics

Continuity and differentiability

Recall Definition of ContinuityLet f(x) be a single valued function of x and x=a be a point in the domain of definition of the function The function is said to be continuous at x=a ifi) f(c) is defined ie f(x) has a definite finite value at x=cii) lim xrarra f(x) exists andiii) lim xrarra f(x) =f(a) In other words f(x) is said to be continuous at x=a if lim xrarra+ f(x)= lim xrarra- f(x) = f(a) Or f(a+0) =f(a-0) =f(a) Or lim hrarr0 f(a+h)= f(a) Algebra of continuous functionsNow we will study some algebra of continuous functions Theorem 1 Suppose f and g be two real functions continuous at a real number c Then(1) f + g is continuous at x = c(2) f ndash g is continuous at x = c(3) f g is continuous at x = c(4) (fg) is continuous at x = c (provided g (c) ne 0)

Example 1 Prove that every rational function is continuousSolution Recall that every rational function f is given byf(x)=[p(x) q(x) ] q(x)ne0where p and q are polynomial functions The domain of f is all real numbers except points at which q is zero Since polynomial functions are continuous f is continuous by (4) of Theorem 1Example 2Discuss the continuity of sine functionSolution To see this we use the following factslim xrarr0 sin x =0Now observe that f (x) = sin x is defined for every real number Let c be a real number Put x = c + h If x rarr c we know that h rarr 0 Therefore lim xrarrc f(x)

= lim xrarrc sin x= lim hrarr0 sin(c+h) =lim hrarr0 [sin c cos h + cos c sin h ]=lim hrarr0 (sin c cos h) + lim hrarr0 (cos c sin h) = sin c +0=sin c = f(c) Thus lim xrarrc f(x) = f(c) and hence f is a constant function Exercise Prove that the function f(x) = x2 +2x is continuous for every real value of x [Hints show that lim xrarra+ f(x) = lim xrarra- f(x) = f(a) ]

Biology Reproduction in Flowering plants We will discuss about megasporoangium

megasporagenesis and female gametophyte

Q4 Describe the structure of megasporangium

Ovule is attached to the placenta by astalk called funicle

Each ovule has one two or three protectivecoverings called integuments

At the tip of integuments a small openingcalled micropyle is organised

Opposite to the micropylar end is the chalaza

Within the integuments a mass of cellsnucellusand inside it embryo sac orfemale gametophyte is present

Q5 Describe a mature embryosacamp its formation

In most of the flowering plants only oneof the 4 megaspores formed as a result ofmegasporogenesis that is functional while theother three degenerate

The

functional megaspore develops into thefemale gametophyte

Formation The nucleus

of the functional megasporedivides mitotically to form two nuclei first andthen two more sequential mitotic nucleardivisions result in the formation of four ampthen eight nucleate stages of embryo sac

Six of the eight nucleus are surrounded bycell walls and organised into cells

The remaining two nuclei called polar nuclei are found below the egg apparatus in the largecentral cell

Three cells consisting of two synergids amp one egg cell present bottom of

embryo sac Three cells

at the chalazal as antipodal cells

Two polar nuclei together present in large central cell

HISTORY

TOWARDS INDEPENDENCE AND PARTITION THE LAST PHASE(1935-1947)SUB TOPIC NATIONAL MOVEMENTS DURING THE SECOND WORLD WAR

Spread of Quit India Movement On 9th August 1942Gandhiji and other Congress leaders were arrested The Congress was declared illegal The news of the arrest of all leaders marked the beginning of a widespread movement of India It was not possible for such a movement to remain peacefulBut the arrest of the all notable congress leaders virtually left the movement in the hands of the mass The movement took the form of violent and militant outbreakBesides congressmen revolutionaries also were very active in the movement The Congress Socialist group also played a prominent role

1 Question Why did the British authority arrest the Congress leaders on 9 th August 1942Answer Congress Working committee adopted the Quit India resolution which was to be ratified at the Bombay AICC meeting in 8th August 1942 They decided to launch a mass struggle on non-violent lines Gandhiji gave a clarion call to all section of the people rdquoKarenge ya Marengerdquo (do or die) Congress leaders gave the call to driving out

the British from IndiaViceroy had taken strong action against the Quit India movement Gandhiji and all the leaders of Congress were arrested

2 Question How did Quit India Movement spread out all over IndiaAnswer The news of the leaders lsquo arrest marked the beginning of a widespread movement to remain peacefulThe movement took form of violent outbreak There were widespread cutting of telephone and Telegraph wires damaging railway lines raising barricades in cities and towns and other forms of violent demonstations

Question Name the leaders of Congress

Socialist group played a prominent part Notable among the Jayprakash Narayan Rammonohar Lohia Aruna Asaf Ali

Political science

Topic-Franchise and Representation

Summary

The election commission

The Election Commission of India is an autonomous constitutional authority responsible for administering Union and State election processes in India The body administers elections to the Lok Sabha Rajya Sabha State Legislative Assemblies in India and the offices of the President and Vice President in the country

Functions of election commission-

India is a sovereign socialist secular democratic republic Democracy runs like a golden thread in the social economic and political fabric woven by the Constitution given by lsquoWe the People of Indiarsquo unto ourselves The concept of democracy as visualised by the Constitution pre-supposes the representation of the people in Parliament and State legislatures by the method of election The Supreme Court has held that democracy is one of the inalienable basic features of the Constitution of India and forms part of its basic structure The Constitution of India adopted a Parliamentary form of government Parliament consists of the President of India and the two Houses mdash Rajya Sabha and Lok Sabha India being a Union of states has separate state legislatures for each state State legislatures consist of the Governor and two Houses mdash Legislative Council and Legislative Assembly mdash in seven states namely Andhra Pradesh Telangana Bihar Jammu amp Kashmir Karnataka Maharashtra and Uttar Pradesh and of the Governor and the state Legislative Assembly in the remaining 22 states Apart from the above two out of the seven Union Territories namely National Capital Territory of Delhi and Puducherry also have their Legislative Assemblies

ExecutionShort notes-Election commissionFunctions of election commission

Homework- Learn

Computer

Science

Computer hardware NAND Gate

A NOT-AND operation is known as NAND operation It has n input (n gt= 2) and one output

Logic diagram

Truth Table

NOR Gate

A NOT-OR operation is known as NOR operation It has n input (n gt= 2) and one output

Logic diagram

Truth Table

XOR Gate

XOR or Ex-OR gate is a special type of gate It can be used in the half

adder full adder and subtractor The exclusive-OR gate is abbreviated as EX-OR gate or sometime as X-OR gate It has n input (n gt= 2) and one output

Logic diagram

Truth Table

XNOR Gate

XNOR gate is a special type of gate It can be used in the half adder full adder and subtractor The exclusive-NOR gate is abbreviated as EX-NOR gate or sometime as X-NOR gate It has n input (n gt= 2) and one output

Logic diagram

Truth Table

Physics

Chapter 1 Electric Field ( Electric Dipole) (Summary)

Here we will derive Expression of electric field at broad side

On position of dipole

Execution

Q With the help of a labelled diagram obtain an expression for the electric field intensity E at any point on the equitorial line ( broad-side on position) of an electric dipole

Ans

E1 E1sinθ

E θ P E1 θ

( r2+L2)12 E2 E

r E2 E2sinθ

-q θ L O L +qA B

Let us consider that the point P is situated on the right bisector of the dipole AB at a distance r meter from its midpoint O

Let E1 and E2 be the electric field intensities of the electric field at P due to charge +q and ndashq of the dipole resp The distance of P from each charge is ( r2+L2)12

So E1 = 14 πϵ q

(r 2+L 2) away from +q

E2 = 14 πϵ q

(r 2+L 2) towards ndashq

The magnitudes of E1 and E2 are equal but directions are different Now resolving E1 and E2 into two components parallel and perpendicular to AB we get

The components perpendicular to AB E1sinθ and E2sinθ cancel each other because they are equal and opposite

The components parallel to AB are E1cosθ and E2 cosθ are in same direction and add up

So resultant intensity of electric field at the point P is

E = E1cosθ + E2 cosθ

E = 14 πϵ q

(r 2+L 2) 2 cosθ

Now from fig we have cosθ =BOBP = L (r2+L2)12

So we get E = 14 πϵ 2qL ( r2+L2)32

Now electric dipole moment p= 2qL

So E = 14 πϵ p ( r2+L2)32

HW Find the expression of Electric field as done here but this time take r gtgt 2L

Also find the expression of torque experience by a dipole

(Hint Electric force experienced by charges of dipole in electric field is qE each Let θ be the angle which dipole makes with electric lines of force then perpendicular distance between two charges is 2Lsinθ Then torque = force x perp distance = qE x 2L sinθ So τ=pE sinθ where p =2qL )

STUDY MATERIAL

Class XIISubject Eng Literature (The Tempest ndash William Shakespeare) Topic Act IV Scene 1 Lines 84 to 133 (Iris hellip A contract of true love Be not too late ) Date 27th April 2020 (4th Period)

[Students should read the original play and also the paraphrase given in the school prescribed textbook]Summary Questions amp Answers

o Ceres soon appears and comes to know that she has been summoned to celebrate the contract of true love

o Ceres expresses her unwillingness to meet Venus and Cupid as she has shunned their company

o Ceres and Juno both bestow their blessings upon Ferdinand and Miranda with June gifting honour riches happiness in marriage and Ceres presents plenty of earthrsquos produce

o Iris summons the water-nymphs and reapers to come and celebrate a contract

(1) IRIS Of her society (Line 91-101)

Be not afraid I met her deity

Cutting the clouds towards Pathos and her sonDove-drawn with her Here thought they to have doneSome wanton charm upon this man and maidWhose vows are that no bed-right shall be paidTill Hymens torch be lightedmdashbut in vainMarss hot minion is returned againHer waspish-headed son has broke his arrowsSwears he will shoot no more but play with sparrowsAnd be a boy right out

(i) Where were Venus and Cupid seen flying How were they travelling Why did they want to join the marriage celebration of Ferdinand and Miranda

of true love

Venus and Cupid were seen flying through the air towards Paphos the famous city which is situated on the island of Cyprus They were travelling by air-borne chariot drawn by doves They certainly wanted to come here in order to play some amorous trick upon Ferdinand and Miranda who are under a vow not to gratify their physical desires till the holy ceremony of their marriage has been performed(ii) What have Venus and Cupid done after failing in their plan

After being failure of their plan Venus who is a very passionate deity and who is the mistress of Mars (the god of war) has gone back while here ill-tempered son Cupid has broken his arrows of love in his state of desperation(iii) What has Cupid firmly decided

Cupid is feeling so disappointed that he has firmly decided to shoot no more arrows to arouse love in human hearts but to spend his time playing with sparrows Thus he would now become just a boy and would give up his original function of shooting arrows on human beings to make them fall in love(iv) What vow had Ceres taken How did Ceres feel at the abduction

After the abduction of her daughter Prosperina by Pluto Ceres had taken a vow to always keep away from the disgraceful company of Venus and her blind son Cupid the god of love Ceres felt deeply distressed when Pluto had carried off her daughter and had made her his wife by force(v) Why has Ceres not forgiven Venus and her blind son For what do Ceres want to be sure

As the abduction had been manipulated by Venus the goddess of beauty and love and her blind son Cupid Ceres has never forgiven them for their part in the whole plot Ceres wants to be sure that she would not have to meet Venus and Cupid who had engineered the abduction of her daughter Prosperina

AS THIS lsquoMASQUErsquo SCENE IS VERY IMPORTANT IN THE PLAY THE PARAPHRASE OF THE ENTIRE PORTION OF MASQUE SCENE (Act IV Lines 58 to 143) IS GIVEN BELOW

IRIS Goddess of RainbowCERES Goddess of Agriculture and all the fruits of the earth

(Nature growth prosperity rebirth ndash notions intimately connected to marriage)JUNO The majestic Queen of Heavens and wife of Jupiter (Jupiter is the king of Gods)

VENUS The Goddess of love CUPID Son of Venus PLUTO God of death (In the play referred by Shakespeare as lsquoDisrsquo which is a Roman name for Pluto)

ORIGINAL TEXT PARAPHRASEPROSPEROWellmdash

PROSPERONow come Ariel Let there be too many rather than too few

Now come my Ariel Bring a corollaryRather than want a spirit Appear and pertly[to Ferdinand and Miranda]No tongue all eyes Be silent

spirits in attendance Appear briskly

[to Ferdinand and Miranda]Look with your eyes but do not say a word

[Soft music] [Soft music][Enter Iris] [Enter Iris]

IRISCeres most bounteous lady thy rich leasOf wheat rye barley vetches oats and peasThy turfy mountains where live nibbling sheepAnd flat meads thatched with stover them to keepThy banks with pioned and twilled brimsWhich spongy April at thy hest betrimsTo make cold nymphs chaste crowns and thybroom-grovesWhose shadow the dismissegraved bachelor lovesBeing lass-lorn thy pole clipped vineyardAnd thy sea-marge sterile and rocky-hardWhere thou thyself dost airmdashthe Queen othrsquoSkyWhose watery arch and messenger am IBids thee leave these and with her sovereign grace[Juno appears] Here on this grass-plot in this very placeTo come and sport Her peacocks fly amainApproach rich Ceres her to entertain

IRISCeres most generous lady you are the cause of rich fields or fertile land where wheat rye barley beans oats and peas grow the grassy mountains where the sheep graze and the flat meadows covered with coarse hay to be used as fodder for cattleYour banks are covered with marsh-marigolds and reeds and the rainy April under your orders brings forth to make for the maids who are not in love beautiful crowns your woods where the broom flourishes and where the bachelor who has been dismissed by the maid he loved lies down being forsaken your vineyard in which the poles are embraced by the vines and the margin of the sea which is barren and rocky where you roam about to enjoy the fresh air ndash the queen of the sky (Juno) whose messenger I am besides being represented as the rainbow bids you leave all these and with her majesty here on this grassy plot in this very place come and sport her peacocks carry her fast in her chariot through the air and are making their way here approach rich Ceres to welcome her

[Enter Ariel as Ceres] [Enter Ariel as Ceres]

CERESHail many-coloured messenger that neerDost disobey the wife of JupiterWho with thy saffron wings upon my flowersDiffusest honey-drops refreshing showersAnd with each end of thy blue bow dost crownMy bosky acres and my unshrubbed downRich scarf to my proud earth Why hath thy queenSummoned me hither to this short-grassed green

CERESWelcome rainbow that never dared disobey Juno the wife of Jupiter who with your orange coloured rays spread honey-drops refreshing showers And with each end of thy blue bow drown my bushy acres and my hilly country which is free from shrubs you thus forming a rich scarf Why has your queen called me here to this place covered with short grass

IRISA contract of true love to celebrateAnd some donation freely to estateOn the blest lovers

IRISI have called you to celebrate a contract of true love and bestow some liberal gift upon the blessed lovers

ORIGINAL TEXT PARAPHRASECERESTell me heavenly bowIf Venus or her son as thou dost knowDo now attend the queen Since they did plotThe means that dusky Dis my daughter gotHer and her blind boys scandaled companyI have forsworn

CERESTell me heavenly bow if Venus the Goddess of love or Cupid her son and pedlar of passion at this time attend the heavenly queen Juno because you are sure to know Since the day they conspired against me and dark Pluto took away my daughter here and Cupidrsquos disgraceful company I have left off

IRISOf her societyBe not afraid I met her deityCutting the clouds towards Pathos and her sonDove-drawn with her Here thought they to have doneSome wanton charm upon this man and miad

IRISBe not afraid of her company I met her deity moving on the clouds towards Paphos the sacred home of Venus on the island of Cyprus along with her son on her chariot drawn by doves Here they contemplated to exercise a charm upon this man and maid producing

Whose vows are that no bed-right shall be paidTill Hymens torch be lightedmdashbut in vainMarss hot minion is returned againHer waspish-headed son has broke his arrowsSwears he will shoot no more but play with sparrowsAnd be a boy right out

wantonness before the actual marriage ceremony but did not succeed Venus has returned her irritable son has broken his arrows and swears that he will give up his practice of trying to inspire love but play with sparrows and be a boy again

[Music is heard] [Music is heard]

CERESHighst queen of stateGreat Juno comes I know her by her gait

CERESHighest queen of state Great Juno there she comes I know here by her gait

[Enter Juno] [Enter Juno]

JUNOHow does my bounteous sister Go with meTo bless this twain that they may Prosperous beAnd honoured in their issue

JUNOHow are you doing my generous sister Come with me to bless this couple so that they may be prosperous and fortunate in their children

[They sing] [They sing]

JUNOHonour riches marriage-blessingLong continuance and increasingHourly joys be still upon youJuno sings her blessings upon you

JUNOMay honour riches happiness in marriage long continuance and increase of those boons ever rest upon you as hourly joys Juno showers down upon you her blessings in song

CERESEarths increase foison plentyBarns and garners never emptyVines and clustring bunches growingPlants and goodly burden bowingSpring come to you at the farthestIn the very end of harvestScarcity and want shall shun youCeresrsquo blessing so is on you

CERESMay you have the plenty of earthrsquos produce Your barns and granaries may never be empty Your vines may grow with clustering bunches Your fruit trees may be heavily laden with their fruit May there be continuous spring and harvest May scantiness and want leave you forever Such is the blessing of Ceres upon you

FERDINANDThis is a most majestic vision andHarmoniously charmingly May I be boldTo think these spirits

FERDINANDThis is a great vision and magically melodious Should I suppose the characters (taking part in the masque) are spirits

PROSPEROSpirits which by mine artI have from their confines calld to enactMy present fancies

PROSPEROYes they are spirits whom I have summoned from the regions to which they are confined to carry into effect my fanciful designs

ORIGINAL TEXT PARAPHRASEFERDINANDLet me live here everSo rare a wondered father and a wifeMakes this place paradise

FERDINANDI should like to live here forever Such a wise and wonderful father makes this place a paradise

[Juno and Ceres whisper and send Iris on employment] [Juno and Ceres whisper and send Iris on employment]

PROSPEROSweet now silence

PROSPEROMy dear Ferdinand speak no more Juno and Ceres are

Juno and Ceres whisper seriouslyTheres something else to do Hush and be muteOr else our spell is marred

whispering with a solemn look There is something else coming Silence Or else our magic will be spoilt

IRISYour nymphs called naiads of the wandering brooksWith your sedged crowns and over-harmless looksLeave your crisp channels and on this green landAnswer your summons Juno does commandCome temperate nymphs and help to celebrateA contract of true love Be not too late

IRISYou nymphs called Naiads denizens (M inhabitants) of the running stream with your chaplets of sedge and ever-helpful looks leave your wrinkled channels and on the green land answer the summons sent to you Juno has ordered some chaste nymphs and help to celebrate a noble and true marriage Donrsquot delay

[Enter certain nymphs] [Enter certain nymphs]You sunburnt sicklemen of August wearyCome hither from the furrow and be merryMake holiday your rye-straw hats put onAnd these fresh nymphs encounter every oneIn country footing

You sunburnt harvesters weary from the effects of the heat in August come here from the furrowed land and rejoice Make holiday with your rye-straw hats upon you and meet these fresh nymphs and join in country dancing

[Enter certain reapers properly habited They join with the nymphs in a graceful dance towards the end whereof Prospero starts suddenly and speaks]

[Enter certain reapers properly habited They join with the nymphs in a graceful dance towards the end whereof Prospero starts suddenly and speaks]

PROSPERO[aside] I had forgot that foul conspiracyOf the beast Caliban and his confederatesAgainst my life The minute of their plotIs almost come [to the spirits]Well done Avoidno more

PROSPERO(Aside)I had forgotten the wicked conspiracy of the beast Caliban and his accomplices against my life the time of their plot has almost arrived ndash (To the Spirits) well done depart no more of this

[To a strange hollow and confused noise the spirits heavily vanish]

[The spirits depart]

ORIGINAL TEXT PARAPHRASEFERDINANDLet me live here everSo rare a wondered father and a wifeMakes this place paradise

FERDINANDI should like to live here forever Such a wise and wonderful father makes this place a paradise

[Juno and Ceres whisper and send Iris on employment] [Juno and Ceres whisper and send Iris on employment]

PROSPEROSweet now silenceJuno and Ceres whisper seriouslyTheres something else to do Hush and be muteOr else our spell is marred

PROSPEROMy dear Ferdinand speak no more Juno and Ceres are whispering with a solemn look There is something else coming Silence Or else our magic will be spoilt

IRISYour nymphs called naiads of the wandering brooksWith your sedged crowns and over-harmless looksLeave your crisp channels and on this green landAnswer your summons Juno does commandCome temperate nymphs and help to celebrateA contract of true love Be not too late

IRISYou nymphs called Naiads denizens (M inhabitants) of the running stream with your chaplets of sedge and ever-helpful looks leave your wrinkled channels and on the green land answer the summons sent to you Juno has ordered some chaste nymphs and help to celebrate a noble and true marriage Donrsquot delay

[Enter certain nymphs] [Enter certain nymphs]You sunburnt sicklemen of August wearyCome hither from the furrow and be merryMake holiday your rye-straw hats put onAnd these fresh nymphs encounter every oneIn country footing

You sunburnt harvesters weary from the effects of the heat in August come here from the furrowed land and rejoice Make holiday with your rye-straw hats upon you and meet these fresh nymphs and join in country dancing

[Enter certain reapers properly habited They join with the nymphs in a graceful dance towards the end whereof Prospero starts suddenly and speaks]

[Enter certain reapers properly habited They join with the nymphs in a graceful dance towards the end whereof Prospero starts suddenly and speaks]

PROSPERO[aside] I had forgot that foul conspiracyOf the beast Caliban and his confederatesAgainst my life The minute of their plotIs almost come [to the spirits]Well done Avoidno more

PROSPERO(Aside)I had forgotten the wicked conspiracy of the beast Caliban and his accomplices against my life the time of their plot has almost arrived ndash (To the Spirits) well done depart no more of this

[To a strange hollow and confused noise the spirits heavily vanish]

[The spirits depart]

Ac-12 27420 topic Revaluation of Assets and Liabilities

REVALUATION OF ASSETS AND LIABILITIES

On admission of a new partner the firm stands reconstituted and consequently the assets are revalued and liabilities are reassessed It is necessary to show the true position of the firm at the time of admission of a new partner If the values of the assets are raised gain will increase the capital of the existing partners Similarly any decrease in the value of assets ie loss will decrease the capital of the existing partners For this purpose alsquoRevaluation Accountrsquo is prepared This account is credited with all increases in the value of assets and decrease in the value of liabilities It is debited with decrease on account of value of assets and increase in the value of liabilities The balance of this account shows a gain or loss on revaluation which is transferred to the existing partnerrsquos capital account in existing profit sharing ratioAccounting for Revaluation of Assets and Liabilities when there is a Changein the Profit Sharing Ratio of Existing PartnersAssets and liabilities of a firm must also be revalued at the time of change in profit sharing ratio of existing partners The reason is that the realisable or actual value of assets and liabilities may be different from those shown in the Balance Sheet It is possible that with the passage of time some of the assets might have appreciated in value while the value of certain other assets might have decreased and no record has been made of such changes in the books of accounts Similarly there may be some unrecorded assets amp libilities that may have to be accounted for Revaluation of assets and reassessments of liabilities becomes necessary because the change in the

value of assets and liabilities belongs to the period to change in profit sharing ratio and hence must be shared by the partners in their old profit sharing ratio Revaluation of assets and reassessment of liabilities may be given effect to in two different ways (a) When revised values are to be recorded in the books and(b) When revised values are not to be recorded in the books

When revised values are to be recorded in the booksIn such a case revaluation of assets and reassessment of liabilities is done with the help of a new account called lsquoRevaluation Accountrsquo Sometimes this account is also called as lsquoProfit amp Loss Adjustment Acrsquo If there is a loss due to revaluation revaluation account is debited and if the revaluation results in a profit the revaluation account is credited The following journal entries made for this purpose are

(i) For increase in the value of assetsAsset Ac Dr (individually)To Revaluation Ac(ii) For decrease in the value of AssetRevaluation Ac Dr (individually)To Asset Ac[Decrease in the value of assets](iii) For increase in the value of LiabilitiesRevaluation Ac Dr (individually)To Liabilities Ac[Increase in the value of Liabilities](iv) For decrease in the value of LiabilitiesLiabilities Ac DrTo Revaluation Ac[Decrease in the value of Liabilities](v) For unrecorded AssetsAsset Ac [unrecorded] DrTo Revaluation Ac[Unrecorded asset recorded at actual value](vi) For unrecorded Liability Revaluation Ac DrTo Liability Ac [unrecorded][Unrecorded Liability recorded at actual value](vii) For transfer of gain on revaluationRevaluation Ac DrTo Existing Partnerrsquos CapitalCurrent Ac[Profit on revaluation transferred to capital account in existing ratio](viii) For transfer of loss on revaluationExisting Partnerrsquos CapitalCurrent Ac DrTo Revaluation Ac[Loss on revaluation transferred to capital account in existing ratio](a) When revaluation account shows gain Revaluation Ac DrTo Partnerrsquos Capital Ac (Old Profit Sharing Ratio)(Profit on revaluation credited to Partnerrsquos Capital Ac)(b) Above entry is reversed when revaluation account shows loss Partners Capital Acs (Old Profit Sharing Ratio) DrTo Revaluation Ac(Loss on revaluation debited to Partnerrsquos Capital Acs)

Proforma of Revaluation Account is given as under

Revaluation Account

Dr Cr Particulars ` Amount Particulars ` Amount To Decrease in value of assets By Increase in value of assets To Increase in value of liabilities By Decrease in value of liabilities To Unrecorded liabilities By Unrecorded assets To Gain on Revaluation (Transferred) By Loss on Revalution (Transferred)

ECO ndash12 2742020Topic- ELASTICITY OF DEMAND

CHAPTER - ELASTICITY OF DEMANDMEANINGDemand for a commodity is affected by many factors such as its price price of related goods income of its buyer tastes and preferences etc Elasticity means degree of response Elasticity of demand means degree of responsiveness of demand Demand for a commodity responds to change in price price of related goods income etc So we have three dimensions of elasticity of demandDIMENSION OF ELASTICITY OF DEMAND TYPES OF ELASTICITY OF DEMAND

Price elasticity of demand Income elasticity of demand Cross Elasticity of demand

Price elasticity of demand Price elasticity of demand means degree of responsiveness of demand for a commodity to the change in its price For example if demand for a commodity rises by 10 due to 5 fall in its price Price elasticity of demand (ep)=Percentage change in quantity demanded Percentage change in price of the commodity = 10 ( -)5 = ( - )2Note that ep will always be negative due to inverse relationship of price and quantity demanded

(ii) Income elasticity of demand Income elasticity of demand refers to the degree of responsiveness of demand for a commodity to the change in income of its buyer Suppose income of buyer rises by 10 and his demand for a commodity rises by 20 then Income elasticity of demand (ey)= change in quantity demanded change in price of the commodity =20 10 = 2

Cross Elasticity of demandCross elasticity of demand means the degree of responsiveness of demand for a commodity to the change in price of its related goods (substitute goods or complementary goods) Suppose demand for a commodity rises by 10 due to 5 rise in price of its substitute good then Cross elasticity of demand (ec) = change in quantity demanded change in price of related good = 10 2 = 5 (Tastes and preferences cannot be expressed numerically So elasticity ofdemand cannot be numerically expressed)

  • Chapter 1 Force (Summary)
  • Distinguish between external sovereignty and internal sovereignty
    • NAND Gate
      • Logic diagram
      • Truth Table
        • NOR Gate
          • Logic diagram
          • Truth Table
            • XOR Gate
              • Logic diagram
              • Truth Table
                • XNOR Gate
                  • Logic diagram
                  • Truth Table
                      • Physics
                      • Chapter 1 Electric Field ( Electric Dipole) (Summary)
Page 6:  · Web viewWe all know that Nouns are divided into two parts: common noun and proper noun.Apart from common and proper noun, we will also study about collective noun and compound

DESKTOP THE FIRST SCREEN THAT WE SEE WHEN THE OPERATING SYSTEM (WINDOWS)HAS FINISHED LOADING IS KNOWN AS DESKTOP

THE FEATURES OF DESKTOP ARE bullIT IS WHERE ALL THE ICONS OF THE DIFFERENT APPLICATIONS ARE DISPLAYEDbullIT IS THE HOME FOR ALL SHORTCUTS PROGRAMS CAN BE QUICKLY OPENED BY CLICKING ON THE RESPECTIVE SHORTCUTSbullIT HOLDS VARIOUS INFORMATION SUCH AS TIME AND DATE

ICONSICONS ARE SMALL PICTURES OF DIFFERENT APPLICATIONS THAT ARE PRESENT AND INSTALLED IN THE COMPUTER FOR EXAMPLE COMPUTER ICON AND RECYCLE BIN ICON

DESKTOP GADGETS A DESKTOP GADGET IS A SMALL APPLICATION OR A WIDGET THAT RECIDES ON A COMPUTERS DESKTOP FOR EXAMPLE CLOCK CALENDAR ETC_______________________________________________________________________________________________

Class VSubject Socialal studiesTopic Evolution of man

Millions of years ago the earth looked much different as it is seen todayIt was totally different and it was very difficult to live Everything looked very different plants animals and humans everything looked very different from what they are now All the changes took place gradually with the changing environment Earliest ancestors of the humans were known as the Ramapithecus who lived about 14 million years ago They looked like ape that walked upright and weighed about 14kg With the passage of time there was a gradual development in the structure posture features and teeth of the early humansIt is said that the apesthat lived on treetops started living on the ground and stood on two limbs are called hominids They used their limbs to hold things and make tools Finally they changed to homosapiens mean wise human We humans belong to this group Life of early humansThe early men were hunter ndash gatherers They moved from place to place in search of food water and shelter They hunted animals and ate them raw gathered fruits nuts roots and seeds for eating They didnrsquot know how to build houses so they used live on treetops or in the caves or rock shelters In summers they remained uncovered but in winter they used to cover themselves by using bark of trees leaves and animal skins

Subject English language Topic Adjectives Interrogative and Emphasizing

Interrogative adjectiveAn interrogative adjective is used with a noun that it qualifies to ask a question The word interrogative mean to ask questions Example Which part of the world do you come fromThe word lsquoWhich is an interrogative adjectivesEmphasizing adjectivesSometimes we use adjectives to emphasize the effect of what we say Words which are used to emphasize a nounExample Rita came late and had to stand at the very end of the lineThe word Veryis an emphasizing adjective It actually qualifies or emphasizethe noun end

1lsquoDo not listen to what Ravi says for he is a complete foolrsquo said RohanComplete- emphasizing adjective

2Kiran is a great supporter of non violenceGreat- emphasizing adjective

3rsquoWhat crime has the prisoners committedrsquo asked the judgeWhat- interrogative adjective

4Whose book is that lying on the tableWhose- interrogative adjective

Subjects COMPUTER CHAPTER-1 (CHARACTERISTICS OF A COMPUTER)A COMPUTER SYSTEM IS A ONE THAT CONSISTS OF ALL THE SOFTWARES HARDWARES CPU AND MEMORY DEVICES SOME IMPORTANT TERMSTHE COMPUTER WORKS ACCORDING TO THE COMMANDS OR INSTRUCTIONS THAT WE GIVE

A SET OF INSTRUCTIONS IS CALLED THE PROGRAM A SET OF ONE OR MORE PROGRAMS IS CALLED A SOFTWARE THE PHYSICAL COMPONENTS OF A COMPUTER ARE CALLED HARDWARE

FOUR OPERATIONS OF A COMPUTER SYSTEM INPUTINPUT IS SUPPLIED TO THE COMPUTER WITH THE USE OF A KEYBOARD A MOUSE A MICROPHONE

OR OTHER INPUT DEVICES PROCESSING PROCESSING IS DONE INSIDE THE COMPUTER IN THE CPU PROCESSING IS THE CONVERSION

OF THE INPUT INTO THE OUTPUT STORAGE STORAGE REFERS TO THE SAVING OF INFORMATION FOR LATER USE INFORMATION IS STORED

IN THE MEMORY OF A COMPUTER THERE ARE TWO TYPES OF COMPUTER MEMORY PRIMARY MEMORY AND SECONDARY MEMORYPRIMARY MEMORY IS FAST SMALL AND EXPENSIVE THE TWO TYPES OF PRIMARY MEMORY ARE RANDOM ACCESS MEMORY(RAM) AND READ ONLY MEMORY (ROM) RANDOM ACCESS MEMORY (RAM)THIS MEMORY IS VOLATILE IN NATURE WHICH MEANS THAT THE

INFORMATION STORED IN IT IS AUTOMATICALLY ERASED WHEN THE COMPUTER POWER IS TURNED OFF

READ ONLY MEMORY (ROM) THIS TYPE OF MEMORY IS NON VOLATILE IN NATURE WHICH MEANS THAT DATA IS NOT LOST WHEN THE COMPUTER POWER IS TURNED OFF

OUTPUT OUTPUT IS THE RESULT PRODUCED BY A COMPUTEROUTPUT MAY BE VIEWED ON THE MONITOR SCREEN HEARD THROUGH SPEAKERS PRINTED THROUGH PRINTER ON PAPERS OR STORED IN THE MEMORY FOR LATER USE

Class VI

GEOGRAPHY CHAPTER 1 REPRESENTATION OF GEOGRAPHICAL FEATURES

GLOBE AND MAPS

GLOBE- A Globe is a three ndashdimensional model of the Earth

EQUATOR ndash An imaginary line runs through the centre of the earth from east to west

NORTHERN HEMISPHERE ndash the half of the north of the equator is called Northern Hemisphere

SOUTHERN HEMISPHERE ndash the half of the south of the equator is called Southern Hemisphere

LATITUDE - Imaginary parallel lines running on the globe from east to west

LONGITUDE ndash The imaginary lines semicircle in nature running from the North Pole to South Pole Also called meridian

MAP ndash Representation of the Earth as a whole or parts of it drawn on a flat surface to a scale

SKETCH ndash A rough drawing of a place not drawn to a scale

PLAN ndash A plan is prepared to show more details about a small area

COMPUTER - VI CHAPTER-6 (INTRODUCTION TO WINDOWS 10)

THE MOST RECENT VERSION OF WINDOWS IS WINDOWS 10 IT WAS RELEASED IN 2015

WINDOWS 10 IS DESIGNED TO WORK FOR YOU AND NOT YOU WORKING FOR IT THE WINDOWS 10 OPERATING SYSTEM INCLUDES A NUMBER OF NEW FEATURES LIKE IMPROVED PERFORMANCE ON MULTI CORE PROCESSORS IMPROVED G U I DATA SECURITY AND BETTER BOOTING PERFORMANCE

WINDOWS 10 HAS 1187 WITH WEARING FEATURES SETS AND INTENDED HARDWARE

THE VARIOUS EDITIONS AVAILABLE FOR WINDOWS 10 ARE AS FOLLOWS

WINDOWS 10 HOME

WINDOWS 10 HOME IS THE EDITION OF WINDOWS 10 THAT CONTAINS THE LEAST NUMBER OF FEATURESWINDOWS 10 HOME IS THE CONSUMER FOCUSED DESKTOP EDITION WITH SUPPORT FOR BOTH PC AND TOUCH ENABLED TABLETS WINDOWS 10 HOME IS AVAILABLE IN BOTH 32 BIT AND 64 BIT VERSIONS

WINDOWS 10 PRO

WINDOWS 10 PRO HAS FEATURES ESSENTIAL FOR SMALL OR MEDIUM BUSINESSESIT HAS A LARGE NUMBER OF EXTRA FEATURES TO MEET THE NEEDS OF SMALL BUSINESSES

WINDOWS 10 ENTERPRISE

WINDOWS 10 ENTERPRISE IS BUILT ON WINDOWS 10 PROIT HAS FEATURES WHICH ARE DESIGNED TO MEET THE NEEDS OF MEDIUM AND LARGE ORGANISATIONSTHIS WINDOWS EDITION TARGETS THE ENTERPRISE SEGMENT OF THE MARKET

WINDOWS 10 MOBILE

WINDOWS 10 MOBILE IS DESIGNED FOR CUSTOMER ORGANISATION THAT ARE USING THE WINDOWS 10 PLATFORM ON SMARTPHONES AND SMALL TABLETS

WINDOWS 10 EDUCATION

WINDOWS 10 EDUCATION IS DESIGNED TO MEET THE NEEDS OF STAFF ADMINISTRATORS TEACHERS AND STUDENTS OF SCHOOLS

WINDOWS 10T EDITION

WINDOWS 10T EDITION OF WINDOWS 10 IS DESIGNED FOR USE IN SMALL FOOTPRINT AND LOW COST DEVICES

Subject English language

Topic Noun kinds

NOUN KINDS

Noun is a naming word Nouns are names of people places animals or things

For example Ashley Mumbai tiger pencils

Kinds of nouns

Proper noun the name of a particular person place or thing is called proper nounExample

1 The Himalayas stand to the north of India2 Monika is the school captain

Common noun announce that names people place or thing in general is called a common noun Example

1 Kalidas was the greatest dramatist of India2 William Shakespeare is Englands national poet

Collective Noun A collective noun is the name of a collection of people or things taken together and spoken of as a whole Example

1 The feet were completely destroyed in the fierce attack2 The crew revolted against Captain Grand

Abstract Noun An abstract noun is the name of some quality state or idea Example1 Diversity Indias biggest strength2 Patience is a virtue

_____________________________________________________________________________________

Class VII

Subject English 1 Topic Articles

ARTICLES

There are three articles ------ A An The

1 Indefinite articles- The Articles a and an are called Indefinite Articles They do not point to a particular person or thing They are used with singular countable nouns It does not identify a specific noun

Uses of Indefinite articles A

Before a singular noun beginning with a consonant or a vowel with a consonant sound For example a cattle a university a one way track

Before a proper noun which is either unfamiliar or holds a special meaning For example For example A Mahesh Sharma wants to see youNeela wants to be a Tendulkar when she grows up

With a number beginning with a consonant sound For example a ten- rupee note a hundred years

Before half when half follows a whole number or after half when it isnrsquotExample one and a half litres (before) Half a litre (after)

With an expression of quantity For example a lot of time a dozen oranges a little while In exclamation before nouns Example Such a mess What a pity

Uses of Indefinite articles An

Before a singular noun beginning with a vowel sound Example an ant an egg an umbrella Before an abbreviation beginning with a vowel or a consonant with the vowel sound

Example an MP an MA Before the word beginning with a silent h Example an heir an hour an honour

Note We do not say a milk or a lemonade because they are uncountable nouns ever we see a cup of milk or a glass of lemon and

Subject GEOGRAPHY

CHAPTER 1 REPRESENTATION OF GEOGRAPHICAL FEATURES

TOPIC- TOPOGRAPHICAL MAPS

Topographical Maps- topographical maps are small-scale maps with detailed depiction of both natural and human-made features

Conventional colours-different colours used in a map are known as conventional colours

TYPES OF SCALE-

Verbal Statement- Scale written in a statement from like 2 cm to a Km is called verbal statement It means that 2cm on the map represents 1 km on the Earth

Representative Fraction- Scale can also be written as a representative fraction (RF)

Example 150000 it means 1cm on the map represents 50000cm or frac12 km on the ground It shows the ratio between map distance and ground distance

Linear Scale Scale can be drawn on a line to show map distance equivalent to ground distance A line is divided into equal parts Each parts represents the actual distance on the ground in mkm

SUBJECT-COMPUTER

CHAPTER-1 (COMPUTER FUNDAMENTALS)

COMPUTER LANGUAGES

THE THE TERM COMPUTER LANGUAGE REFERS TO A SYSTEM OF RULES AND SYMBOLS THAT ARE DESIGNED TO GIVE INSTRUCTIONS TO A COMPUTERTHE COMPONENTS OF A COMPUTER SYSTEM CANNOT PERFORM A TASK BY THEMSELVES THEREFORE THEY HAVE TO BE GIVEN INSTRUCTIONS TO PERFORM ANY TASKCOMPUTER LANGUAGES ARE USED TO CREATE PROGRAMS USING LOGIC BASED ALGORITHMSTHE RULES OF A COMPUTER LANGUAGE IS

KNOWN AS SYNTAX WHEREAS THE TERM SEMANTICS REFERS TO THE MEANING OF LANGUAGESCOMPUTER CANNOT DEVELOP ANY PROGRAMMING LANGUAGE BY THEMSELVES AND CANNOT THINK INTELLIGENTLY UNLESS THEY ARE ASSISTED BY THE HUMAN BEINGS THEREFORE THE PROGRAMMERS DEVELOP A SET OF METHODS AND TECHNIQUES A SET OF INSTRUCTIONS USED TO PERFORM A SPECIFIC TASK IS CALLED A PROGRAM

A PROGRAMMING LANGUAGE IS ALSO KNOWN AS A COMPUTER LANGUAGE CODED BY PROGRAMMERS TO WRITE INSTRUCTIONS FOR A COMPUTERTHE COMPUTER TAKES THESE INSTRUCTIONS AS INPUT AND PRODUCES THE DESIRED OUTPUT

TYPES OF COMPUTER LANGUAGE

THE COMPUTER LANGUAGE CAN BE BROADLY DIVIDED INTO TWO CATEGORIES

LOW LEVEL LANGUAGES HIGH LEVEL LANGUAGES

LOW LEVEL LANGUAGES

A LOW LEVEL PROGRAMMING LANGUAGE REFERS TO THE LANGUAGE THAT IS UNDERSTOOD BY A COMPUTER DIRECTLYTHE PROGRAMMER MUST HAVE AN IN-DEPTH KNOWLEDGE OF DIFFERENT COMPUTERS TO WRITE PROGRAMS IN A LOW LEVEL LANGUAGE THE TWO TYPES OF LOW LEVEL LANGUAGES ARE MACHINE LANGUAGE AND ASSEMBLY LANGUAGE

MACHINE LANGUAGE

COMPUTER HARDWARE UNDERSTANDS ONLY MACHINE LANGUAGE AS IT IS THE FUNDAMENTAL LANGUAGE OF A COMPUTER WHICH UNDERSTANDS ONLY THE TWO DIGITS 0 AND 1 KNOWN AS THE BINARY DIGITSIT IS A SYSTEM OF INSTRUCTIONS EXECUTED DIRECTLY BY THE CPU WITHOUT ANY TRANSLATION AND HENCE IS EXECUTED QUICKLY AS COMPARED TO PROGRAMS IN OTHER LANGUAGES PROGRAMS IN MACHINE LANGUAGE NEED DIFFERENT BINARY LANGUAGE PROGRAMS TO SOLVE THE COMPLEX TASKSPROGRAMMERS OFTEN FIND IT DIFFICULT TO WRITE PROGRAMS IN THIS LANGUAGE

ASSEMBLY LANGUAGE

AN ASSEMBLY LANGUAGE IS A LOW LEVEL PROGRAMMING LANGUAGE BUT THE INSTRUCTIONS ARE WRITTEN IN THE FORM OF WORDS KNOWN AS MNEMONICS TO CONVERT AN ASSEMBLY LANGUAGE INTO TO MACHINE CODE WE REQUIRE A UTILITY PROGRAM WHICH IS KNOWN AS AN ASSEMBLER ASSEMBLY LANGUAGE PROGRAMS CANNOT BE EXECUTED DIRECTLY BY A COMPUTER THEREFORE THEY ARE SLOWER THAN THE MACHINE LANGUAGE PROGRAMSIT IS EASIER TO WRITE PROGRAMS IN THE MACHINE LANGUAGE THEN AND IN ASSEMBLY LANGUAGE

_______________________________________________________________________________________________

CLASS-VIII

SUBJECT-COMPUTER CHAPTER - Operating system and graphical user interface Role and functions

Need of operating system

It co-ordinates different hardware and software componentsof a computer system It supervises the various actions of the computer system and enables the computer to work in a effective

manner It helps in smooth functioning of various peripherals

In a multi-tasking operating system it determines the order and time to be allowed for each application before giving another application a turn

It sends messages to the system operator about the status of operation or any error that may have occurred while running the application

What is operating system

An operating system is an integrated system of programs that manages various resources and the overall operation of the computer system It is designed to support various activities of computer system in a systematic way

Role of an operating system-

Operating system enables the user to use the system effectively An operating system manages various application that runs on a computer and shares computerrsquos resources User interacts with operating system through command line interface and graphical user interface

Function of an operating system -

Booting the computer it is the start up procedure of a computer system Loading theprograms in the memory - when system is ready the operating system loads certain program

automatically Manages resources it manages between the hardware and software resources Detecting and correcting errors- if the supporting hardware or software doesnrsquot works properly then the

operatingsystem tries to rectify it Ensuring data security programs and data donot interfere with each other Maintaining the internal clock of system - maintain internal clock of system when system is shut down

SUBJECT-GEOGRAPHY CHAPTER 1 REPRESENTATION OF GEOGRAPHICAL FEATURES THROUGH CONTOURSTopographical maps- topographical maps are small-scale maps with detailed depiction of both natural and human-made features

Contour lines- contour lines are imaginary lines drawn on a map joining places having the same height above the mean sea level

Contour linesContour interval- It is an interval at which contour lines are drawnIndex contour- At every 100-m interval a thick brown contour line is called index contour It is drawn for calculating height

Spot heights- The spot heights show heights in metres above the mean sea level

SUBJECT- English Language CHAPTER - The Sentence

Complex sentence-

We have seen that a Complex Sentence consists of a Principal Clause with one or more Subordinate Clauses

We have also learnt that there are three kinds of Subordinate Clauses The Adjective Clause the Adverb Clause and the Noun Clause

1) THE ADJECTIVE CLAUSE

An ADJECTIVE CLAUSE does the work of an ADJECTIVE It qualifies a noun or pronoun In the following examples in each set two commonly patterned sentences are compared one with an ADJECTIVE and the other with an ADJECTIVE CLAUSE ( The Adjective or Adjective Clause qualifies the Noun placed in the box)

a) He is a lazy boy (Adjective)

b) He is a boy who is lazy (Adjective Clause)

a) It is a beautiful place for the tourists (Adjective)

b) It is a place where tourists come for the scenic beauty (Adjective Clause)

a) We have enough funds for the work (Adjective)

b) We have funds which would be enough for the work (Adjective Clause)

Convert the sentence from simple to complex sentence-

1) We believe his honesty Ans- We believe that he is honest

2) This is the birth place of RamaAns- This is the place where Rama was born

____________________________________________________________________________________________

Class IX

Subject English Language

Topic Preposition

Date 270420

PREPOSITIONS

A preposition is a word placed before a noun or a pronoun to show in what relation the person or thing denoted by it stands in regard to something else

EXAMPLE

There is a clock on the wall

The preposition on shoes relationship between clock and the wall

Smitha is afraid of lizards The man jumped off the bus

Here the preposition of shows the relationship between afraid and lizards

The preposition off shows the relation between jumped and bus

The noun or the pronoun which follows a preposition is called its object

So in the first Example wall is the object of the preposition on

A preposition can have more than one object For example

The plane flew overhouses and meadows

Prepositions are used to express a number of relationship including time locationmannermeans quantity purpose and state or condition

Points to remember

Preposition joins a noun to another noun or a pronounThere is a cow in the field

A preposition joints a noun to a verbThe cat runs after the rat

Preposition can have two or more than two objectsThe road runs over hills and plain

A preposition also joins a noun an adjectiveHe is fond of tea

Generally a preposition comes before an object Sometimes it comes even after an object asWhat are you looking atThis is the house I live in

Subject- Computer Application

Chapter 2 Introduction to Java

Java API An application programming interface (API) in the context of Java is a collection of prewritten packages classes and interfaces with their respective methods fields and constructors

Byte Code Java bytecode is the result of the compilation of a Java program an intermediate representation of that program which is machine independent The Java bytecode gets processed by the Java virtual machine (JVM) instead of the processor JVM The Java Virtual Machine (JVM) is the runtime engine of the Java Platform which allows any program written in Java or other language compiled into Java bytecode to run on any computer that has a native JVM

Platform A platform is the hardware or software environment in which a program remains

Java platform The Java platform differs from most other platforms in that itrsquos a software only platform that runs on top of other hardware-based platforms

Applet and Application The fundamental difference between the two Java programs is that an application program is designed to run on a stand-alone machine whereas an applet is a web-version of an application which is used to run a program on a web browser

WORA ldquoWrite once run anywhererdquo (WORA) or sometimes write once run everywhere (WORE) is a slogan created by Sun Microsystems to illustrate the cross-platform benefits of the Java language

Class XSubject Topic Summary Execution

ECONOMICSFACTORS OF PRODUCTION Sub-topiclsquoLABOURrsquo

We shall start our class by discussing the topic taught in the last class

lsquo DIVISION OF LABOURrsquo- By division of labour we mean specialization in workIt refers to splitting up the work of labour involved in the production of a particular commodity into several parts and each part and sub-part is performed by a specialist

Now let us start by the Advantages of Division of labouraIt increases the level

Questions

1 Differentiate between Product-Based division of labour and Process-Based division of labour

Product-Based division of labour

Process-based division of labour

It is also known as simple (or occupational) division of labourUnder it everybody performs a particular occupations The entire is done by the same person

When a person or group of persons undertakes a specialised function which is supplementary to the production of final commodity and service This is also as complex division of labour

It is simple It is complex

of productionbSince the product is produced by an expert workerbest quality of product is producedcIt saves time and toolsd it promotes inventions in the methods and techniques of productioneIt leads to reduction in costs fAll workers get work according to their abilities and choices

Now let us discuss the disadvantages of Division of labour

a Since many workers are involved in the production of a commodity no one has the sense of responsibility

b The constant and repetition of the same work again and again make the work monotonous

c Division of labour facilitates production on large scale Hencethere is fear of over production

d Because of territorial division of labour some areasregions become more developed than others

It is based on labour-intensive techniques of production

It is based on capital-intensive techniques

It is generally found in small enterprises

It is generally found in large enterprises

Example Indian farmers doing all farm activities

Example A modern garmet factory where one person takes the measurementanother does the cuttingsome sew the clothes while a few workers button them and other iron them

2 What are the advantages of Division of labouraIt increases the level of productionbSince the product is produced by an expert workerbest quality of product is producedcIt saves time and toolsd it promotes inventions in the methods and techniques of productioneIt leads to reduction in costs fAll workers get work according to their abilities and choices

3Discuss the disadvantages of Division of laboura Since many workers are involved in the

production of a commodity no one has the sense of responsibility

b The constant and repetition of the same work again and again make the work monotonous

c Division of labour facilitates production on large scale Hence there is fear of over production

d Because of territorial division of labour some areasregions become more developed than others

English 1 Transformation of sentences

Sentences A sentence is a group of words which makes complete sense

a Assertive sentences

Exercise 6Rewrite the following sentences according to the instructions given below without changing their meanings

1 As soon as he saw the beer he jumped into

b Imperative sentences

c Interrogative sentences

d Exclamatory sentences

Sentences can be changed from one grammatical form to another without changing the meaning of the sentence This is known as transformation of sentences

the river ( Begin No sooner)2 None but brave deserve the fair (Begin the

bravehellip)3 This box is too heavy for me to lift ( Use so hellip

That instead of too)4 No one other than a king can live like James

Luxurious ( Begin only James)5 Oh for the wings of a dove (Begin I wishhellip)

Math Topic Commercial MathematicsChapter Shares and Dividends

Study item Discuss about shares and Dividends1) What is share

Ans To start any big business (company or Industry) a large sum of money is needed But it is not possible for an individual to invest such a large amount Then some persons interested in the business join together and from a company They divide the estimated money required into small parts Each such part is called a share

2) What do you mean by the term shareholder

Ans A person who purchases one or more shares is called shareholder3) Some terms related with a share

(i) Nominal value or face value or printed value The original value of a share is called its nominal value or face value or printed value

Note The nominal value of a share always remains same(ii) Market value or cash value The price of a share at any

time is called its market value or cash value

Note The market value of a share changes from time to time(iii) At par If the market value of a share is the same as its

nominal value the share is called at par(iv) At Premium or above Par If the market value of a share

is more than its nominal value the share is called at premium or above par

Example If a share of Rs 100 is selling at Rs 150 then it is said to be selling at a premium of Rs 50 or Rs 50 above par

(v) At Discount or below par If the market value of a share is less than its nominal value the share is called at discount or below par

Example If a share of Rs 100 is selling at Rs80 then it is said to be selling at a discount of Rs 20 or at Rs 20 below par

4) What is Dividend

Ans The profit which a shareholder gets for hisher investment from the company is called dividendNote (i) The dividend is always expressed as the percentage of the face value of the share(ii) The dividend is always given( by the company ) on the face value of the share

irrespective of the market value of the shareBENGALI(2ND LANGUAGE)

ldquoদেবতোর জণমrdquoলিবরোম চকরবত

পরথম লিসর পোঠ-চোর পসথ একটি পোথর লিবপলি ঘটোয় দেক যোতোয়োসতর পসথ পরলিতলিয়ত ওই পোথসর দেো োচট দেসত একলি দেতো দেক দেো োচট দেসয় দেবোমো সয় রোসতোর মোস লি0টসক পস1 লিবপরীত লিক দেথসক আো একটি দেমোটর োলি1 চোসকর কষতোয় পরোসরণ দেবোসচ যো লিকনত পরলিতবোর এমরণ দেৌভোয দেসকর োও সত পোসর তোই লিতলি দেকোো দেজোো1 কসর পোথরটিসক উপস1 দে8স পোথর উপস1 দে8োর ময় এক দেকৌতী জতো দেকসক পরশন কসর দেয লিতলি দেকোসো দেবতোর আস দেপসয়স0রণ লিকো লিকনত দেক বস লিতলি দেকোসো দেবতোর আস পোলি দেক উপলিত ক জতোর উসltসয বস কোরও ইস= স পোথরটি লিসয় দেযসত পোসর এর পর দেথসক দেক দেযসত আসত পোথরটি দেক দেসত পো একলি দেক কষয করস দেকউ পোথরটিসক ধসয়মস0 পলিরসকোর করস0 দেক ওই দেকৌতী জতোসক পোথসরর কোস0 বস থোকসত দেস এ0ো1ো আরও কষয কসর দেক ঠোৎ ওই লি1 পোথরটির োসয় লিোর োো-দেকউ পজো কসরস0 দেকৌতী দেোকটির আঙকো য় যলিসক উ পোথরটিসক লিরসয় দে8স তোর পর ঠোৎ একলি পোথরটির দেোোজ দেই দেক লিসয় দেস0 বো দেকোথোয় দেস0

বদোথ-

দেো োচট ndashচসত লিসয় দেকো লিক0র সE ধোককো দেস পস1 যোবোর উপকরমঅকসমোৎ- ঠোৎআতমমবর- লিসজসক লিয়নতরপ-পো লিপ0স প1োদেসতসসত- লিসপলিউৎোত- দেো1ো দেথসক উপস1 দে8োপরতযয় ndash লিবশবোপরতযোস- টতযোস ndash দৈবোসধসতোধলিসত- পরসপসরর পরলিত ব পরসয়ো করোপরসতরীভত- পোথসর পলিররণলিতবোনতঃকরসরণ- মস পরোসঅলিQৎ- অQো করসত ই=কদেোপ- বধমোর- দেবোস এমইতযোকোর- এইরকমরম- বময়পযসোভী-পসযর জয দেোভ আস0 যোরপোসথয় ঞচয়- পথচোর রচ জমোসোমোমোসরোস- ব ধম ধোসমর সE

তোর লি দেইhelliphellipTo be continued

Hindi 2nd lang सर क पद(सरदास)

सरदास शरी कषण भकति कावय क सरवशरषठ कगिरव ह इनक जनम और मतय क समय तथा सथान का मतभद हसरदास रवातसलय और शरार रस क अनयतम कगिरव ह इनक कावय म बालकषण क सौदय चपपल चषटा और गि7याओ की मनोहर झाकी मिमलती ह कषण और ोगिपयो क अननय परम का कतिचतरण ह सयो शरार की अपकषा उनक कावय म गिरवयो शरार का अमिBक गिरवषय और मारमिमEक कतिचतरण हआ हइन पकतियो म हम सरदास की भकति भारवनाओ का परिरचय मिमलता ह इनका सपण सगरह सरसार म गिनगिहत ह

1 जसोदा हरिर पालन झलारवहलरारवदलराईमलहारव रव जो ईसाई कछ ारव मर लाल को आई निनEदिदया काह ना आगिनसबारव त काह नाही बरवगिह आरवतोको कानहा बलारव

शबदाथ-हलरारव-गिहलती हदलराई - दलार पयार करती हमलहारव-पचकारती हनिनEदरिरया ndashनीदरवगिह-जलदी सअBर-होठमौन-चपसन-सकत

वयाखया- सरदास जी कहत ह गिक यशोदा माता बालक कषण को पालन म झल आती ह रवह उनह गिहलाती ह पयार करती ह मलहार जस कोई ीत ान लती ह और नीद स पछती ह गिक ह नीद तम मर लाल को आकर कयो नही सलाती तझ खाना बला रहा ह कभी कषण आख बद कर लत ह कभी आखफडफडान लत ह उनह सोता हआ जानकर यशोदा माता चप हो जाती ह और इशार म बात करन लती ह इसी बीच अकला कर कषण ज जात हतो गिफर यशोदा माता गिफर स ाना ान लती ह सरदास जी कहत ह गिक भरवान क दशन का सख दरवता और ऋगिष-मगिनयो को भी दलभ ह यही सख माता यशोदा को बडी सहजता स मिमल जा रही ह माता यशोदा बहत ही भागयशाली ह2)Continue to nexthellip

Physics

Chapter 1 Force

(Summary)

Question A body is acted upon by two forces each of magnitude F but in opposite directions State the effect of the forces if

(a) Both forces act at the same point of the body

(b)the two forces act at two different points of the body at a separation r

Solutions

(a) Resultant force acting on the body = 0

F ndash F = 0(b) The forces tend to rotate the body between two forces about the midpoint

Moment of forces = F times rFr

QuestionDefine moment of a couple Write its SI unit

Solutions

Moment of couple is equal to the product of both force and the perpendicular distance between the two forces

The SI unit of moment of couple is NmCommercial Studies

Advertising and sales

Business firms use several methods to

Questions1) What do you mean by advertising

promotion create demand of their product in the market and increase it sales Such methods comprises of advertising sales promotion personal selling and publicityToday we are going to discuss about one of such methods It is advertising

Meaning of advertisingAdvertising is a paid form of non-personal presentation for promotion of Ideas goods and services

Importance or merits of advertising Advertising has importance to manufacturer or traders to customer and to society as a whole

Today we will see how advertisement help the manufacturer or traders

Answer) Advertising is a means of how a company encourages people to buy their products services or ideas It is one element of marketing which also includes design Research and data mining

2) Mention any three features of advertisingAnswer)The main features of advertising are

i) It is impersonal form of presentation for promotion of products and services of Ideas

ii) It is issued by identified sponsor The advertisement contains the name of the advertiser

iii) It is a form of mass communication because the message is directed to a large number of persons simultaneously

3) Mention the main merits or importance of advertisement to manufacturer or tradersAnswer)

i) Introducing new product A business organization can introduce itself and its products to the public through advertising

ii) Increase the sale Advertising leads to increase the sale of existing product by entering into new markets and attracting new customers

iii) Create steady demand Advertising creates sustains regular demand by smoothening out seasonal and other fluctuations It enables regular production for the organisation

iv) Economics of scale Advertising facilitate mass distribution of goods and steady demand which lead to large scale and regular production

v) Goodwill Advertising helps in creating a good image of the firm and reputation for its products

Biology Chapter - 03Genetics

Today wewill start chapter and discuss about Genetics Gregor Mendel is known as father of genetics Before entering into Mendelrsquos experiment on Genetics we must know

Q1 Define the following termsi) Genetics Genetics is the study of

transmissionof body features from parents to offspringand the laws relating to such transmission

ii) Heredity It may be defined as transmissionof genetically based characteristics from parentsto offspring

iii) Character and traits Any heritable

Importance to TraderIntroducing new productIncrease the saleCreate steady demandEconomics of scaleGoodwill

some terms featureis a character The alternative forms of acharacter are called traitsex Character (Hair shape) - Traits (Curly straight)

iv) Homologous chromosomes A pair ofcorresponding chromosomes of the same shapeand size one from each parent

v) Genes Genes are the specific parts (DNA segments) of a chromosome which determinethe hereditary characteristicsNearly 30000genes present in human

vi) Alleles Alternative forms of a gene occupying the same position (locus) on homologouschromosomes and affecting the same characteristicbut in different ways

vii) Genotype ndash PhenotypeGenotype means of genes present in the cells of an organism Phenotype means the observable characteristic which is genetically controlled

viii) Mutation It is a sudden change in one or more genes or in the number or in the structure of chromosomes ex Sickle cell anaemia is a blood disease caused by a gene mutation

CLASS NOTES

Class XSubject Eng Literature (The Merchant of Venice ndash William Shakespeare)Topic Act IV Scene 1 Lines 01 to 34 ( Duke helliphelliphellip We all expect a gentle answer Jew) ate 27th April 2020 (2nd Period)

[Students should read the original play and also the paraphrase given in the school prescribed textbook]Summary Questions amp Answers

This scene may be termed as the catastrophe of the play It is the final unravelling of the complicated events which seem to threaten the happiness of Bassanio Portia and Antonio Right is justified to the fullest degree and malice falls into the trap prepared for others No one suffers here but Shylock but even then he receives a generous measure of mercy

o This is the Court-scene Initially we meet

(1)

DUKE I am sorry for thee thou art come to answer (Line 3-6)A stony adversary an inhuman wretchUncapable of pity void and emptyFrom any dram of mercy

(i) Who is addressed here Where is the person Why is the person there

Antonio is addressed hereAntonio is in the court of justice at VeniceAntoniorsquos trial is scheduled to be held here for his failure to meet the conditions of the bond he signed with Shylock

the Duke Antonio Shylock and Salerio Later we meet Bassanio Portia Gratiano and Nerissa

o The Duke says to Antonio that he has to face a very cruel opponent which Antonio admits and expresses his gratefulness to the Duke for his efforts to soften without result the heart of Shylock in order to be merciful to Antonio Antonio further says that he is ready to accept whatever cruel judgement the Court may award

o When Shylock appears in the court the Duke says that Shylock should change his decision of prosecuting Antonio and demanding the penalty specified in the bond out of consideration of the great misfortunes that Antonio has suffered If this is done by him (Shylock) the whole court would be gladdened by his merciful action

(ii) What is the Duke sorry for

The Duke is unable to change the mind of Shylock from his decision to get the bond forfeited even after he pleaded to Shylock Shylock stands firmly for his bond which when forfeited will allow him to take a pound of flesh from any part of Antoniorsquos body(iii) How does the Duke address Shylock`The Duke calls Shylock an adversary with a heart of stones He calls Shylock as an inhuman wretch without pity Shylock is quite lacking in the slightest quality of mercy (iv) How does Antonio reply to this

Antonio replies that he will meet the revenge of Shylock patiently He has prepared himself to suffer with a quiet spirit the utmost that Shylockrsquos tyranny and rage can do(v) What quality of the Duke is revealed here

The Duke is kind and benevolent He is ready to help Antonio He requests Shylock to free Antonio from the trial(vi) What are the terms of the bond that Antonio has signed

The terms of the bond that Antonio has signed were that if Antonio is unable to repay Shylock a certain sum of money specified on the paper on a certain date and in an agreed place the forfeit has to be paid The forfeiture will be an exact pound of Antoniorsquos flesh which Shylock will be a liberty to take from any part of Antoniorsquos body which pleases him

Class XI

STUDY MATERIAL

Class XISubject Eng Literature (The Tempest ndash William Shakespeare) Topic Act I Scene 2 Lines 88 to 132 (Prospero hellip Me and thy crying self) Date 27th April 2020 (3rd Period)

[Students should read the original play and also the paraphrase given in the school prescribed textbook]Summary Questions amp Answers

o Prospero now tells Miranda that he was the Duke of Milan He had been devoting himself more to studies than the affairs of the State His brother Antonio took advantage of this situation and with the help of Alonso the king of Naples seized upon him and her one midnight and shipped them in a frail bark so that they perished in the sea All this took place

(1)

MIRANDA I should sin (Line 118-132)

To think but nobly of my grandmother

Good wombs have borne bad sonsPROSPERO Now the condition

The King of Naples being an enemyTo me inveterate hearkens my brothers suitWhich was that he in lieu othrsquo premisesOf homage and I know not how much tribute

twelve years back

IMPORTANT PASSAGES EXPLAINED(Line 98-103)

PROSPERO helliphelliphelliphelliphelliphelliphellip

He being thus lorded

Not only with what my revenue yieldedBut what my power might else exact like oneWho having into truth by telling of it Made such a sinner of his memoryTo credit his own lie he did believeHe was indeed the duke

Prospero in telling the narrative of his past life here refers to his brother Antonio Prospero being with a studious bent of mind has left the administration of Milan on his younger brother Now Antonio being thus invested like a lord with all the powers derived from Prosperorsquos wealth and what the exercise of Prosperorsquos authority might secure for him regarded himself as a de facto Duke of Milan It is a well-known fact of psychology that a man who repeatedly tells a lie makes of his memory such a sinner against truth as to credit his own lie by the telling of it So Antonio by repeatedly saying to himself and others that he was the Duke came to believe that he was really the Duke Thus falsehood repeatedly asserted gained the force of truth for Antonio and he truly believed it

Should presently extirpate me and mine Out of the dukedom and confer fair MilanWith all the honours on my brother whereonA treacherous army levied one midnightFated to thrsquo purpose did Antonio openThe gates of Milan and ithrsquo dead of darkness The ministers for thrsquo purpose hurried thenceMe and thy crying self

(i) In the earlier lines of this scene what does Prospero tell about his intense interest What was the demand of his interest

In the earlier lines of this scene Prospero tells Miranda that he had an intense interest in the study of philosophy and magic arts Hence in order to improve his mind with this kind of study he kept himself isolated from worldly and state affairs His study was dearer to him than the applause and esteem that he could win from the public His study demanded too much solitude(ii) What forced Antonio to take an undue advantage over Prospero

Prosperorsquos indifferent attitude towards the statersquos affairs and his having boundless trust in Antonio gave rise to a boundless lust for power in Antoniorsquos mind Antonio felt that he must be the actual Duke instead of the part of the Duke he played Thus Antonio took an undue advantage of the situation to usurp Prosperorsquos dukedom(iii) Explain the following lines ldquoI should sin to think but nobly of my grandmother Good wombs have borne bad sonsrdquo

After hearing the treacherous act of her uncle Antonio Miranda says that Prosperorsquos mother was a noble lady and she cannot dishonour her memory by saying that the person named Antonio cannot be his (Prosperorsquos) brother She says that in honour of her grandmother she also cannot say that Antonio must have been begotten not by her grandfather but by some other man She finally concedes that it is known that good mothers have borne bad sons in their wombs and gave birth to them

(iv) Why did the King of Naples accept Antoniorsquos request to help him in usurping his dukedom What did Antonio propose to Alonso

Alonso who was the king of Naples accepted Antoniorsquos request in usurping Prosperorsquos dukedom because he (Alonso) was a sworn enemy of Prospero Antonio proposed that Alonso should immediately drive him (Prospero) and his offspring out of Milan and should confer the dukedom upon him (Antonio) with all the dignities which go with that

position In return Antonio promised that he shall give an annual tribute and also swore his allegiance to Alonso Also he agreed to hold the Dukedom of Milan as a subordinate to the state of Naples(v) How were Prospero and Miranda carried away from the city of Milan and what was the state of small Miranda at that time

In pursuance of the agreement settled between Antonio and Alonso an army of treacherous men was assembled One midnight when the occasion suited the will of destiny Antonio opened the gates of the city of Milan and in the death like silence of midnight Antoniorsquos agents who had been directed to execute his purpose carried Prospero and small Miranda away from the city in all haste They were then forced into a ship and carried some distance out to sea where they put them on a mere hulk of a boat without any rigging or ship-gear and abandoned them leaving them at the mercy of the roaring sea Miranda was a very small child of three years age and she was crying at that time

CLASS -XIDATE-270420Subject Topic Summary Execution

EVS Chapter 1 ndash Mode of Existence

Impact of mode of existence on resources

Q) Why resources are under pressure

Ans - Increase in the sophistication

of technology enabling natural resources to be extracted quickly and efficiently Eg in the past it could take long hours just to cut down one tree only using saws Due to increased technology rates of deforestation have greatly increased

The number of humans is increasing Cultures of consumerism Materialistic views

lead to the mining of gold and diamonds to produce jewelry unnecessary commodities for human life or advancement Consumerism also leads to extraction of resources for the

production of commodities necessary for human life but in amounts excessive of what is needed because people consume more than is necessary or waste what they have

Lack of awareness among the population is striking People are not aware of ways to reduce depletion and exploitation of materials

Accounts Cash Book Today we are going to start a new topic -Cash Book

The key terms used in this chapter are

bullCash book

bullSimple cash book

bullDouble column cash book bullTriple column cash book

bullPetty cash book

bullCash discount

bullContra entry

Here I will share you the meaning of each key terms

bullCash book Cash Book is a special purpose subsidiary book or journal in which cash received and cash payments are recorded

bullSimple cash book

It is a cash book in which only cash transactions are recorded It has only one column on each side

bullTriple column cash book

It is cash book which has three columns one column for each cash and Bankdiscount on each side of the cash book In this book both cash and Bank transactions are recorded together with discount allowed and received

bullPetty cash book

It is a cash book maintained for recording petty expenses

bullCash discount

Cash discount is the amount of discount received or allowed on cash payments and cash receipts Discount received is an income for the business while discount allowed isan expense

bullContra entry

It means transactions involving both cash and Bank Such transactions though recorded in the cash book are not posted into ledger The letter lsquoC is written in Ledger folio for contra entry

Business Studies

ENTREPRENEURSHIP

Now we shall discuss the second chapter

lsquoENTREPRENEURSHIPrsquo

Today before starting the chapter let us recall what

Questions

1What are the main characteristics of Intrapreneurship

Answer

The main characteristics of Intrapreneurship are

Corporate framework-it occurs within the framework of the same company

Semi-Autonomous-Intrapreneurship

we have read last day

Let s today start the class by recalling the last topic taught

Intrapreneurship is the process of discovering and exploring business opportunities within an existing company It involves launching new business ventures within the framework of a present corporation Intrapreneurship is also known as corporate entrepreneurship or corporate venturing

Now let us start with the characteristics of Intrapreneurship

The main characteristics of Intrapreneurship are

Corporate framework

Semi-Autonomous Lack of ownership Senior position Low risk taking Not own boss

Now let us discuss the meaning of enterprise

Enterprise means an undertaking or adventure that requires some innovation and investment and thus involves riskEnterprise always entails decision making coordination and risk bearing

involves crating amd nurturing a semi-autonomous business unit which may be a subsidiary a strategic business unit or a division

Lack of ownership-the intrapreneur is not the owner of the unitb he creates and nurtures

Senior position-he occupies a senior managerial position in the company

Low risk taking-An intrapreneur does not bear the full risk of failure

Not own boss-An intrapreneur is not his own bosss in legal termsHe enjoys the freedom and gets the required resources and support

2 How is Entrepreneur is different from Intrapreneur

The functions involved in both the entrepreneurship and intrapreneurship are by and large similar however there are several differences between the two

Point of distinction

Entrepreneur

Intrapreneur

status An independent business person

A senior executive within a company

Ownership Owner of

the business

An employeesometimes a share in ownership

Financing Responsible for raising finance for the business

Not responsible for raising the finance

Risk bearing

Bears the risk of the business

Does not bears the risk of the business

Reward Profit which is uncertain and irregularcan be loss

Fixed salary and fringe benefits

Need for security low high

3 What do you understand by enterprise

Answer Enterprise means an undertaking or adventure that requires some innovation and investment and thus involves riskEnterprise always entails decision making coordination and risk bearing

COMMERCE NATURE AND OBJECTIVES OF

BUSINESS

Today let us recall the last other two objectives of business by the chart given in the previous class

Firstly we would discuss Human Objectives

Business is run by people and for people Labour is a valuable business element

Human objectives of business are concerned with the well -being of labour

The human objectives are as follows

Labour welfare Developing human

resources Participative

management Labour

management cooperation

Questions

1 Explain the human objectives of a business enterprise

Answer

Business is run by people and for people Labour is a valuable business element

Human objectives of business are concerned with the well -being of labour

The human objectives are as follows

Labour welfare-Business must recognize the dignity of labour and human factors should be given the recognition

Developing human resources-Employees must be provided the opportunities for developing new skills and attitudes

Participative management-Employees should be allowed to take part in decision making process of business

Labour management cooperation-Business should strive for creating and maintaining cordial employer employee relations so as to ensure peace and progress in industry

Now let us discuss the national objectives of business

Optimum utilization of resources

National self- reliance Development of small

scale industries Development of

backward areas Control over pollution

2Explain the national objectives of a business enterprise

Answer

It is the duty of business to utilize the resources of the country properly the national objectives of business

Optimum utilization of resources ndashBusiness should use the nationrsquos resources in the best possible manner

National self- reliance-It is the duty of the business to help the government in increasing experts and in reducing dependence on imports

Development of small scale industries-Big business firms are expected to encourage growth of small scale industries which are necessary for generating employment

Development of backward areas-Business is expected to give preference to the industrialization of backward regions of the country

ECONOMICS

BASIC ECONOMIC CONCEPTS

SUB

TOPIC

Value

Wealth

Welfare

Today we shall start with a new topic of the same chapter ie lsquoValuersquo

Value of a commodity is defined as the valuation placed by a household on the consumption of this commodity

lsquoValuersquo has two different meanings and these are

a Value -in -use It refers to consumption value of a commodity It expresses the utility derived from the consumption of a particular commodity A necessity like water has a very high value ndashin ndashuse or

Question

1What is value

Answer

Value of a commodity is defined as the valuation placed by a household on the consumption of this commodity

2What is value-in use

Answer It refers to consumption value of a commodity It expresses the utility derived from the consumption of a particular commodity A necessity like water has a very high value ndashin ndashuse or consumption value

3What is value ndashin- exchange

Answer It relates to market value of a commodity

It is the rate at which a particular good or service can be exchanged for money

For example in barter system if a person is prepared to exchange 3 metres of cloth with 1 pair

consumption value

b Value ndashin-exchange It relates to market value of a commodity

It is the rate at which a particular good or service can be exchanged for moneyFor example in barter system if a person is prepared to exchange 3 metres of cloth with 1 pair of shoes then the value in exchange of 3 metres of cloth is 1 pair of shoesValue in exchange is the power of purchasing other goods In modern monetised economies the exchange value of goods are expressed in terms of money as prices

Now let us discuss the term lsquoWealthrsquo

Wealth refers to the stock of all those assets which are a source of income

Wealth is a stock concept

Wealth must possess the following features

a Utility It must possess utility or give some

of shoes then the value in exchange of 3 metres of cloth is 1 pair of shoes

Value in exchange is the power of purchasing other goods In modern monetised economies the exchange value of goods are expressed in terms of money as prices

4 What is wealth

Answer it refers to the stock of assets or goods which are a source of income and have personal or national ownership

5 What are the features of wealth

Answer The features of wealth are as follows

Wealth must possess the following features

a Utility It must possess utility or give some satisfaction

b Scarcity It must be limited in quantityc Transferability it should be transferable its

ownership can be transferred from one person to another person

d Exchange value It must possess exchange value

6 What is welfare

Answer

Welfare is defined as satisfaction and happiness a sense of well- being among the people

satisfactionb Scarcity It must be

limited in quantityc Transferability It

should be transferable its ownership can be transferred from one person to another person

d Exchange value It must possess exchange value

Now let us discuss the term lsquoWelfarersquo

Welfare is defined as satisfaction and happiness a sense of well- being among the people

Welfare is affected by factors like

a Consumption of goods and services

b Environment

c Family relations

d Degree of freedom

e Law and order situation

Mathematics Trigonometric equation

To find the general solution of the equation sinθ=0

When sin θ =0

Then θ= 0 π2π 3π-π -2π -3

i e when θ = 0 or an integral multiple of π

i e when θ= nπ where n is any integer

Therefore the general solution of the equation sin

Example1 Find the general values of θ which satisfy the equation sin2 θ =34

Solution sin2 θ= 34

Or sin θ = +34 or -34

Or sin θ = sin π3 or sin (-π3)

Therefore

θ = [nπ + (-1) n (π3)] or[ nπ+ (-1) n (-π3)]

= nπ +π3 or nπ-π3 where n= any integer

Example 2Find the values of θ which satisfy tan2 θ

θ=0 is θ= nπ where n is any integer

To find the general solution of the equation cos θ=0

When cos θ=0

Then θ=π2 3π2 5π2 -π2 -3π2 -5π3

i e when θ is an odd multiple of π2

i e when θ=(2n+1) π2 where n= any integer

Therefore the general solution of the equation cos θ =0 is θ= (2n+1) π2 where n= any integer

To find the general solution of the equation tan θ = 0

Clearly tan θ =0 implies sin θcos θ =0

Therefore θ = nπ

i e the general solution of the equation tan θ=0 is θ =nπ where n = any integer

To find the general solution of the equation cot θ =0

Clearly cot θ =0 implies (cos θsin θ) = 0

i e cos θ =0

Therefore θ = (2n+1) π2

Therefore the general solution of the equation cot θ =0 is θ = (2n+1) π2

Where n= any integer

To find the general solution of the equation sin θ= k (-

=13 -πleθleπ

Solution tan2 θ =13

Or tan θ = plusmn1radic(3) =tan(plusmnπ6)

θ=nπ plusmn π6 where n =any integer

If n=0 then θ=plusmnπ6

If n=1 then θ= π plusmn π6

If n=-1 then θ= -π plusmn π6

Therefore the required solution in -π le θ le π are θ= π6 5π6 -π6 -5π6

Exercise Find general solution of sin 2θ=cos θ [Hints Use sin 2θ= 2sin θcosθ and then take cosθ

common]

1lek le1)

Determine an angle alpha such that sin =k and -π2le αle π2

Then we have

Sin θ = k = sin α

Or sin θ - sin α =0

Or 2 cos [(θ+α) 2] sin [(θ-α) 2] =0

Therefore either cos [(θ +α) 2] =0 (1)

Or sin [(θ-α) 2] =0 (2)

Now from (1) we get (θ+α) 2= (2m+1) π2)

Or θ = (2m+1) π-α (3)

And from (2) we get (θ-α) 2 =mπ

Or θ= 2mπ+α(4)

Where m = any integer

Clearly the solution (3) amp (4) may be combined in the following form

θ= nπ+(-1) n α where n= any integer

Therefore the general solution of sin θ = sin α is θ = nπ +(-1) n α where n is any integer and -π2 le α le π2

Biology Chapter - 04Kingdom Monera

Today we will discuss about bacterial reproduction and its usefulness

Fig Binary Fission

Fig Conjugation Fig Transformation

Bacterial reproduction is mainly asexual but sexual reproduction

also takes place

Asexual reproduction takes place by i) Binary fission - from one bacteriato

two bacteria are produced in every 20 to 30mins

ii) Buddingiii) Endospore formation - during

unfourable condition

Sexual reproduction by three ways

1) Conjugation - Transfer of genetic material between cells that are in physical contact with one another

2) Transduction - Transfer of genetic materialfrom one cell to another by a bacteriophage

3) Transformation - Transfer of cell-freeor naked DNArsquo from one cell to another

Bacteria causes different diseases inplants animals and human and

it causes food spoilage and waterpollution but it also have some useful

activities

i) Bacteria are helpful in sewage water treatment

ii) It is used in antibiotic (medicine) production

iii) Anaerobic bacteria help in biogas(energy) production

iv) Many household products like yoghurt cheese are manufactured by use of bacteria

v) Rhizobium by symbiotic relationship with leguminous plant increase soil fertility

vi) Besides these bacteria is helpful in genetic engineering degradation of petroleum hydrocarbonand in dairy

industry

Physics Motion in plane Here we will introduce Projectile Motion

Execution

Projectile

Y

usinθ u h

θX

ucosθ

Suppose a body is projected with an angle θ So initial velocity u can be resolved into two components

Horizontal component - ucosθ ( for range)

Vertical component - usinθ ( for height)

usinθ changes during motion and becomes zero at maximum height position but ucosθ remain unchanged

The maximum height of projectile is h

NB If initial is upward then g = -ve and if it is downward then g = +ve Height is +ve if direction of motion does not change ( for ex a body thrown upwards but goes down ultimately then height h = -ve)

The angle of projectile θ is the angle made with horizontal

HISTORY ndash GROWTH OF NATIONALISM

SUB TOPIC- REVOLUTIONARY NATIONALISM Bengal formation of Anushilan Samity and Jugantar Group

The intensification of the Swadeshi movement and Government policy of terror and repression led to outbreak of violence Bombs were manufactured and attempts on the lives of unpopular Government officials became frequent In the gymnasium of Scottish Church College which was known as General Assemblies Institution a secret society was formed known as Anushilan Samity

Aurobindo Ghosh send from Baroda his emissary Jatindranath Banerjee to mobilize the Bengal revolutionaries

Hemchandra Qanungo and Satyen Bose published Journal Jugantar

The Jugantar group planned to assassinate oppressive magistrate Kingsford by Khudiram Bose and Prafulla Chaki in 1908 Prafulla Chaki committed suicide to avoid arrest Khudiram was tried and hanged

Afew days later the police found a bomb factory in Maniktala and arrested a large number of revolutionaries The trial of revolutionaries became famous as the Alipore Bomb Case

In the course of the trial the approver the public prosecuter and a police officer were assassinated

1 Question Name two journals which preached the cult of violence

Answer a) Yugantar edited by Bhupendranath DuttaB) Bandemataram edited by Aurobindo Ghosh2 Question Why was Khudiram arrested and hangedAnswer An attempt was made to assassinate a hated vindictive majistrate named Kingsford by Khudiram Bose and Prafulla Chaki Their attempt failed and the bomb they threw killed two English ladies Khudiram was arrested and put to trial and then hanged3Question Who was Aurubindo GhoshAnswer Aurobindo Ghosh a nationalist revolutionary who was charged for his involvement in the Alipore Bombing Case He was accused of it along with his brother Barindra nath Ghosh But Aurobindo was acquitted because of the brilliant pleading of his counsel Chittaranjan Das Then he became a spiritual reformer introducing his visions on human progress and spiritual evolution4 Qustion Who was KingsfordAnswer Kingsford was an unpopular British chief Magistrate who was the target of the bomb thrown at Muzaffarpur by Khudiram and Prafulla Chaki

Most of the accused were convicted and sentenced to

long term of imprisonmentBut

Aurobindo Ghosh was acquitted mainly owing to the brilliant pleading of his counsel Chittaranjan Das

Political science Topic-Sovereignty

Summary Sovereignty is the full right and power of a governing body over itself without any interference from outside sources or bodies In political theory sovereignty is a substantive term designating supreme legitimate authority over some polity In international law sovereignty is the exercise of power by a state

Internal Sovereignty

Internal sovereignty means supreme authority within ones territory while external sovereignty relates to the recognition on the part of all states that each possesses this power in equal measure

External sovereignty

external sovereignty relates to the recognition on the part of all states that each possesses this power in equal measure

Distinguish between

Execution

Answer the following questions

Short notes-

Sovereignty

Internal Sovereignty

External sovereignty

Homework- learn

external sovereignty and internal sovereigntySovereignty is the principle

of supreme and

unquestionable authority

reflected in the claim by the

state to be the sole author of

laws within its territory

Definition of external vs internal sovereigntyInternal sovereignty refers to

the relationship between a

sovereign power and its

subjects ndash it refers to the

location of the supreme

authority within the state In

the UK for example internal

sovereignty (supposedly)

resides within Parliament

reflected in the

constitutional principle of

parliamentary

sovereigntyBy contrast

external sovereignty refers

to the capacity of the state

to act independently and

autonomously on the world

stage This is what is

sometimes called lsquostate

sovereigntyrsquo or lsquonational

sovereigntyrsquo and implies

that states are legally equal

and that the territorial

integrity and political

independence of a state is

inviolable

Class ndash XII

Date - 2742020 STUDY MATERIALSubject Topic Summary Execution Business Studies

Job Analysis amp Manpower Planning

At first let us recall the chapter what we have discussed till nowbullJob analysisbullJob specification bullJob description bullJob enlargement bullJob enrichment

Today we will do some questions answers from the chapter

Questions 1ldquoJob analysis job description and job specification are interrelatedrdquo Comment Answer) Job analysis is a systematic and detailed examination of a job to collect all the relevant information about it The contents off the job are summarised in the job description The qualification needed for the job are summarised in job specificationThus there is close interrelationship between job analysis job description and job specification

Question 2ldquoJob enlargement is a horizontal extension of a job whereas job enrichment is a vertical extension of a jobrdquo ElucidateAnswer) Job enlargement involves adding one or more task to a job coma where as job enrichment involves adding more autonomy and responsibility to a job Job enlargement is therefore horizontal extension of a job coma whereas job enrichment is a vertical extension of a job

Question 3 )

What is manpower estimation Explain its quantitative and qualitative aspectsAnswer) Manpower estimation is the process by which management determines how an organisation should move from itrsquos current manpower positionto its desired manpower position There are two dimensions of Manpower estimation- quantitative and qualitative

Quantitative aspectThis aspect of Manpower estimation involves estimating the number of employees required in a future time period Workload analysis and workforce analysis are done to estimate the quantity of required manpower

Qualitative aspectThe estimate of the knowledge skills experience etc of required manpower is the qualitative aspect of Manpower estimation The quality of Manpower can be judged on the basis of job analysisand job specification

COMMERCE

CAPITAL-FIXED AND WORKING

Today let us start the class by discussing the sources of finance for different types of business firms

The term lsquocapitalrsquo refers to the investment made in the enterprise for the purpose of earning profits

Requirements of capital and sources of capital for different types of business firms are

1 Capital for sole proprietorship businessA sole proprietor operates at a small scale and thereforerequires a limited amount of capital

2 Capital for partnership firmCapital requirements as well as capital base of a partnership is bigger than that of a sole trader businessThe owned capital is contributed by the partners in an agreed ratio

3 Capital for joint stock companyA joint stock company generally requires large amount of capitalA public company can raise huge capital through issue of shares In addition to share capital it can utilize retained profits

Now let u discuss the meaning of Finance PlanningFinance planning is the process of estimation the financial requirements of an organization specifying the sources of firms and ensuring that enough funds are available at the right time

1 What do you mean by Finance PlanningAnswer Finance planning is the process of estimation the financial requirements of an organization specifying the sources of firms and ensuring that enough funds are available at the right time

2Discuss the role of financial planning of an enterpriseThe role of financial planning are as followsa A sound financial plan helps a business enterprise to avaid the problems of shortage and surplus of fundsbFinancial planning serves as a guide in developing a sound capital structure so as to maximize returns to shareholders c It helps in effective utilization of fundsd It provides policies and procedures for coordinating different functional areas or departments of businesse It enables the management to exercise effective control over the financial activities of an enterprisef It helps the company to prepare for facing business shocks and surprises in future

Mathematics

Continuity and differentiability

Recall Definition of ContinuityLet f(x) be a single valued function of x and x=a be a point in the domain of definition of the function The function is said to be continuous at x=a ifi) f(c) is defined ie f(x) has a definite finite value at x=cii) lim xrarra f(x) exists andiii) lim xrarra f(x) =f(a) In other words f(x) is said to be continuous at x=a if lim xrarra+ f(x)= lim xrarra- f(x) = f(a) Or f(a+0) =f(a-0) =f(a) Or lim hrarr0 f(a+h)= f(a) Algebra of continuous functionsNow we will study some algebra of continuous functions Theorem 1 Suppose f and g be two real functions continuous at a real number c Then(1) f + g is continuous at x = c(2) f ndash g is continuous at x = c(3) f g is continuous at x = c(4) (fg) is continuous at x = c (provided g (c) ne 0)

Example 1 Prove that every rational function is continuousSolution Recall that every rational function f is given byf(x)=[p(x) q(x) ] q(x)ne0where p and q are polynomial functions The domain of f is all real numbers except points at which q is zero Since polynomial functions are continuous f is continuous by (4) of Theorem 1Example 2Discuss the continuity of sine functionSolution To see this we use the following factslim xrarr0 sin x =0Now observe that f (x) = sin x is defined for every real number Let c be a real number Put x = c + h If x rarr c we know that h rarr 0 Therefore lim xrarrc f(x)

= lim xrarrc sin x= lim hrarr0 sin(c+h) =lim hrarr0 [sin c cos h + cos c sin h ]=lim hrarr0 (sin c cos h) + lim hrarr0 (cos c sin h) = sin c +0=sin c = f(c) Thus lim xrarrc f(x) = f(c) and hence f is a constant function Exercise Prove that the function f(x) = x2 +2x is continuous for every real value of x [Hints show that lim xrarra+ f(x) = lim xrarra- f(x) = f(a) ]

Biology Reproduction in Flowering plants We will discuss about megasporoangium

megasporagenesis and female gametophyte

Q4 Describe the structure of megasporangium

Ovule is attached to the placenta by astalk called funicle

Each ovule has one two or three protectivecoverings called integuments

At the tip of integuments a small openingcalled micropyle is organised

Opposite to the micropylar end is the chalaza

Within the integuments a mass of cellsnucellusand inside it embryo sac orfemale gametophyte is present

Q5 Describe a mature embryosacamp its formation

In most of the flowering plants only oneof the 4 megaspores formed as a result ofmegasporogenesis that is functional while theother three degenerate

The

functional megaspore develops into thefemale gametophyte

Formation The nucleus

of the functional megasporedivides mitotically to form two nuclei first andthen two more sequential mitotic nucleardivisions result in the formation of four ampthen eight nucleate stages of embryo sac

Six of the eight nucleus are surrounded bycell walls and organised into cells

The remaining two nuclei called polar nuclei are found below the egg apparatus in the largecentral cell

Three cells consisting of two synergids amp one egg cell present bottom of

embryo sac Three cells

at the chalazal as antipodal cells

Two polar nuclei together present in large central cell

HISTORY

TOWARDS INDEPENDENCE AND PARTITION THE LAST PHASE(1935-1947)SUB TOPIC NATIONAL MOVEMENTS DURING THE SECOND WORLD WAR

Spread of Quit India Movement On 9th August 1942Gandhiji and other Congress leaders were arrested The Congress was declared illegal The news of the arrest of all leaders marked the beginning of a widespread movement of India It was not possible for such a movement to remain peacefulBut the arrest of the all notable congress leaders virtually left the movement in the hands of the mass The movement took the form of violent and militant outbreakBesides congressmen revolutionaries also were very active in the movement The Congress Socialist group also played a prominent role

1 Question Why did the British authority arrest the Congress leaders on 9 th August 1942Answer Congress Working committee adopted the Quit India resolution which was to be ratified at the Bombay AICC meeting in 8th August 1942 They decided to launch a mass struggle on non-violent lines Gandhiji gave a clarion call to all section of the people rdquoKarenge ya Marengerdquo (do or die) Congress leaders gave the call to driving out

the British from IndiaViceroy had taken strong action against the Quit India movement Gandhiji and all the leaders of Congress were arrested

2 Question How did Quit India Movement spread out all over IndiaAnswer The news of the leaders lsquo arrest marked the beginning of a widespread movement to remain peacefulThe movement took form of violent outbreak There were widespread cutting of telephone and Telegraph wires damaging railway lines raising barricades in cities and towns and other forms of violent demonstations

Question Name the leaders of Congress

Socialist group played a prominent part Notable among the Jayprakash Narayan Rammonohar Lohia Aruna Asaf Ali

Political science

Topic-Franchise and Representation

Summary

The election commission

The Election Commission of India is an autonomous constitutional authority responsible for administering Union and State election processes in India The body administers elections to the Lok Sabha Rajya Sabha State Legislative Assemblies in India and the offices of the President and Vice President in the country

Functions of election commission-

India is a sovereign socialist secular democratic republic Democracy runs like a golden thread in the social economic and political fabric woven by the Constitution given by lsquoWe the People of Indiarsquo unto ourselves The concept of democracy as visualised by the Constitution pre-supposes the representation of the people in Parliament and State legislatures by the method of election The Supreme Court has held that democracy is one of the inalienable basic features of the Constitution of India and forms part of its basic structure The Constitution of India adopted a Parliamentary form of government Parliament consists of the President of India and the two Houses mdash Rajya Sabha and Lok Sabha India being a Union of states has separate state legislatures for each state State legislatures consist of the Governor and two Houses mdash Legislative Council and Legislative Assembly mdash in seven states namely Andhra Pradesh Telangana Bihar Jammu amp Kashmir Karnataka Maharashtra and Uttar Pradesh and of the Governor and the state Legislative Assembly in the remaining 22 states Apart from the above two out of the seven Union Territories namely National Capital Territory of Delhi and Puducherry also have their Legislative Assemblies

ExecutionShort notes-Election commissionFunctions of election commission

Homework- Learn

Computer

Science

Computer hardware NAND Gate

A NOT-AND operation is known as NAND operation It has n input (n gt= 2) and one output

Logic diagram

Truth Table

NOR Gate

A NOT-OR operation is known as NOR operation It has n input (n gt= 2) and one output

Logic diagram

Truth Table

XOR Gate

XOR or Ex-OR gate is a special type of gate It can be used in the half

adder full adder and subtractor The exclusive-OR gate is abbreviated as EX-OR gate or sometime as X-OR gate It has n input (n gt= 2) and one output

Logic diagram

Truth Table

XNOR Gate

XNOR gate is a special type of gate It can be used in the half adder full adder and subtractor The exclusive-NOR gate is abbreviated as EX-NOR gate or sometime as X-NOR gate It has n input (n gt= 2) and one output

Logic diagram

Truth Table

Physics

Chapter 1 Electric Field ( Electric Dipole) (Summary)

Here we will derive Expression of electric field at broad side

On position of dipole

Execution

Q With the help of a labelled diagram obtain an expression for the electric field intensity E at any point on the equitorial line ( broad-side on position) of an electric dipole

Ans

E1 E1sinθ

E θ P E1 θ

( r2+L2)12 E2 E

r E2 E2sinθ

-q θ L O L +qA B

Let us consider that the point P is situated on the right bisector of the dipole AB at a distance r meter from its midpoint O

Let E1 and E2 be the electric field intensities of the electric field at P due to charge +q and ndashq of the dipole resp The distance of P from each charge is ( r2+L2)12

So E1 = 14 πϵ q

(r 2+L 2) away from +q

E2 = 14 πϵ q

(r 2+L 2) towards ndashq

The magnitudes of E1 and E2 are equal but directions are different Now resolving E1 and E2 into two components parallel and perpendicular to AB we get

The components perpendicular to AB E1sinθ and E2sinθ cancel each other because they are equal and opposite

The components parallel to AB are E1cosθ and E2 cosθ are in same direction and add up

So resultant intensity of electric field at the point P is

E = E1cosθ + E2 cosθ

E = 14 πϵ q

(r 2+L 2) 2 cosθ

Now from fig we have cosθ =BOBP = L (r2+L2)12

So we get E = 14 πϵ 2qL ( r2+L2)32

Now electric dipole moment p= 2qL

So E = 14 πϵ p ( r2+L2)32

HW Find the expression of Electric field as done here but this time take r gtgt 2L

Also find the expression of torque experience by a dipole

(Hint Electric force experienced by charges of dipole in electric field is qE each Let θ be the angle which dipole makes with electric lines of force then perpendicular distance between two charges is 2Lsinθ Then torque = force x perp distance = qE x 2L sinθ So τ=pE sinθ where p =2qL )

STUDY MATERIAL

Class XIISubject Eng Literature (The Tempest ndash William Shakespeare) Topic Act IV Scene 1 Lines 84 to 133 (Iris hellip A contract of true love Be not too late ) Date 27th April 2020 (4th Period)

[Students should read the original play and also the paraphrase given in the school prescribed textbook]Summary Questions amp Answers

o Ceres soon appears and comes to know that she has been summoned to celebrate the contract of true love

o Ceres expresses her unwillingness to meet Venus and Cupid as she has shunned their company

o Ceres and Juno both bestow their blessings upon Ferdinand and Miranda with June gifting honour riches happiness in marriage and Ceres presents plenty of earthrsquos produce

o Iris summons the water-nymphs and reapers to come and celebrate a contract

(1) IRIS Of her society (Line 91-101)

Be not afraid I met her deity

Cutting the clouds towards Pathos and her sonDove-drawn with her Here thought they to have doneSome wanton charm upon this man and maidWhose vows are that no bed-right shall be paidTill Hymens torch be lightedmdashbut in vainMarss hot minion is returned againHer waspish-headed son has broke his arrowsSwears he will shoot no more but play with sparrowsAnd be a boy right out

(i) Where were Venus and Cupid seen flying How were they travelling Why did they want to join the marriage celebration of Ferdinand and Miranda

of true love

Venus and Cupid were seen flying through the air towards Paphos the famous city which is situated on the island of Cyprus They were travelling by air-borne chariot drawn by doves They certainly wanted to come here in order to play some amorous trick upon Ferdinand and Miranda who are under a vow not to gratify their physical desires till the holy ceremony of their marriage has been performed(ii) What have Venus and Cupid done after failing in their plan

After being failure of their plan Venus who is a very passionate deity and who is the mistress of Mars (the god of war) has gone back while here ill-tempered son Cupid has broken his arrows of love in his state of desperation(iii) What has Cupid firmly decided

Cupid is feeling so disappointed that he has firmly decided to shoot no more arrows to arouse love in human hearts but to spend his time playing with sparrows Thus he would now become just a boy and would give up his original function of shooting arrows on human beings to make them fall in love(iv) What vow had Ceres taken How did Ceres feel at the abduction

After the abduction of her daughter Prosperina by Pluto Ceres had taken a vow to always keep away from the disgraceful company of Venus and her blind son Cupid the god of love Ceres felt deeply distressed when Pluto had carried off her daughter and had made her his wife by force(v) Why has Ceres not forgiven Venus and her blind son For what do Ceres want to be sure

As the abduction had been manipulated by Venus the goddess of beauty and love and her blind son Cupid Ceres has never forgiven them for their part in the whole plot Ceres wants to be sure that she would not have to meet Venus and Cupid who had engineered the abduction of her daughter Prosperina

AS THIS lsquoMASQUErsquo SCENE IS VERY IMPORTANT IN THE PLAY THE PARAPHRASE OF THE ENTIRE PORTION OF MASQUE SCENE (Act IV Lines 58 to 143) IS GIVEN BELOW

IRIS Goddess of RainbowCERES Goddess of Agriculture and all the fruits of the earth

(Nature growth prosperity rebirth ndash notions intimately connected to marriage)JUNO The majestic Queen of Heavens and wife of Jupiter (Jupiter is the king of Gods)

VENUS The Goddess of love CUPID Son of Venus PLUTO God of death (In the play referred by Shakespeare as lsquoDisrsquo which is a Roman name for Pluto)

ORIGINAL TEXT PARAPHRASEPROSPEROWellmdash

PROSPERONow come Ariel Let there be too many rather than too few

Now come my Ariel Bring a corollaryRather than want a spirit Appear and pertly[to Ferdinand and Miranda]No tongue all eyes Be silent

spirits in attendance Appear briskly

[to Ferdinand and Miranda]Look with your eyes but do not say a word

[Soft music] [Soft music][Enter Iris] [Enter Iris]

IRISCeres most bounteous lady thy rich leasOf wheat rye barley vetches oats and peasThy turfy mountains where live nibbling sheepAnd flat meads thatched with stover them to keepThy banks with pioned and twilled brimsWhich spongy April at thy hest betrimsTo make cold nymphs chaste crowns and thybroom-grovesWhose shadow the dismissegraved bachelor lovesBeing lass-lorn thy pole clipped vineyardAnd thy sea-marge sterile and rocky-hardWhere thou thyself dost airmdashthe Queen othrsquoSkyWhose watery arch and messenger am IBids thee leave these and with her sovereign grace[Juno appears] Here on this grass-plot in this very placeTo come and sport Her peacocks fly amainApproach rich Ceres her to entertain

IRISCeres most generous lady you are the cause of rich fields or fertile land where wheat rye barley beans oats and peas grow the grassy mountains where the sheep graze and the flat meadows covered with coarse hay to be used as fodder for cattleYour banks are covered with marsh-marigolds and reeds and the rainy April under your orders brings forth to make for the maids who are not in love beautiful crowns your woods where the broom flourishes and where the bachelor who has been dismissed by the maid he loved lies down being forsaken your vineyard in which the poles are embraced by the vines and the margin of the sea which is barren and rocky where you roam about to enjoy the fresh air ndash the queen of the sky (Juno) whose messenger I am besides being represented as the rainbow bids you leave all these and with her majesty here on this grassy plot in this very place come and sport her peacocks carry her fast in her chariot through the air and are making their way here approach rich Ceres to welcome her

[Enter Ariel as Ceres] [Enter Ariel as Ceres]

CERESHail many-coloured messenger that neerDost disobey the wife of JupiterWho with thy saffron wings upon my flowersDiffusest honey-drops refreshing showersAnd with each end of thy blue bow dost crownMy bosky acres and my unshrubbed downRich scarf to my proud earth Why hath thy queenSummoned me hither to this short-grassed green

CERESWelcome rainbow that never dared disobey Juno the wife of Jupiter who with your orange coloured rays spread honey-drops refreshing showers And with each end of thy blue bow drown my bushy acres and my hilly country which is free from shrubs you thus forming a rich scarf Why has your queen called me here to this place covered with short grass

IRISA contract of true love to celebrateAnd some donation freely to estateOn the blest lovers

IRISI have called you to celebrate a contract of true love and bestow some liberal gift upon the blessed lovers

ORIGINAL TEXT PARAPHRASECERESTell me heavenly bowIf Venus or her son as thou dost knowDo now attend the queen Since they did plotThe means that dusky Dis my daughter gotHer and her blind boys scandaled companyI have forsworn

CERESTell me heavenly bow if Venus the Goddess of love or Cupid her son and pedlar of passion at this time attend the heavenly queen Juno because you are sure to know Since the day they conspired against me and dark Pluto took away my daughter here and Cupidrsquos disgraceful company I have left off

IRISOf her societyBe not afraid I met her deityCutting the clouds towards Pathos and her sonDove-drawn with her Here thought they to have doneSome wanton charm upon this man and miad

IRISBe not afraid of her company I met her deity moving on the clouds towards Paphos the sacred home of Venus on the island of Cyprus along with her son on her chariot drawn by doves Here they contemplated to exercise a charm upon this man and maid producing

Whose vows are that no bed-right shall be paidTill Hymens torch be lightedmdashbut in vainMarss hot minion is returned againHer waspish-headed son has broke his arrowsSwears he will shoot no more but play with sparrowsAnd be a boy right out

wantonness before the actual marriage ceremony but did not succeed Venus has returned her irritable son has broken his arrows and swears that he will give up his practice of trying to inspire love but play with sparrows and be a boy again

[Music is heard] [Music is heard]

CERESHighst queen of stateGreat Juno comes I know her by her gait

CERESHighest queen of state Great Juno there she comes I know here by her gait

[Enter Juno] [Enter Juno]

JUNOHow does my bounteous sister Go with meTo bless this twain that they may Prosperous beAnd honoured in their issue

JUNOHow are you doing my generous sister Come with me to bless this couple so that they may be prosperous and fortunate in their children

[They sing] [They sing]

JUNOHonour riches marriage-blessingLong continuance and increasingHourly joys be still upon youJuno sings her blessings upon you

JUNOMay honour riches happiness in marriage long continuance and increase of those boons ever rest upon you as hourly joys Juno showers down upon you her blessings in song

CERESEarths increase foison plentyBarns and garners never emptyVines and clustring bunches growingPlants and goodly burden bowingSpring come to you at the farthestIn the very end of harvestScarcity and want shall shun youCeresrsquo blessing so is on you

CERESMay you have the plenty of earthrsquos produce Your barns and granaries may never be empty Your vines may grow with clustering bunches Your fruit trees may be heavily laden with their fruit May there be continuous spring and harvest May scantiness and want leave you forever Such is the blessing of Ceres upon you

FERDINANDThis is a most majestic vision andHarmoniously charmingly May I be boldTo think these spirits

FERDINANDThis is a great vision and magically melodious Should I suppose the characters (taking part in the masque) are spirits

PROSPEROSpirits which by mine artI have from their confines calld to enactMy present fancies

PROSPEROYes they are spirits whom I have summoned from the regions to which they are confined to carry into effect my fanciful designs

ORIGINAL TEXT PARAPHRASEFERDINANDLet me live here everSo rare a wondered father and a wifeMakes this place paradise

FERDINANDI should like to live here forever Such a wise and wonderful father makes this place a paradise

[Juno and Ceres whisper and send Iris on employment] [Juno and Ceres whisper and send Iris on employment]

PROSPEROSweet now silence

PROSPEROMy dear Ferdinand speak no more Juno and Ceres are

Juno and Ceres whisper seriouslyTheres something else to do Hush and be muteOr else our spell is marred

whispering with a solemn look There is something else coming Silence Or else our magic will be spoilt

IRISYour nymphs called naiads of the wandering brooksWith your sedged crowns and over-harmless looksLeave your crisp channels and on this green landAnswer your summons Juno does commandCome temperate nymphs and help to celebrateA contract of true love Be not too late

IRISYou nymphs called Naiads denizens (M inhabitants) of the running stream with your chaplets of sedge and ever-helpful looks leave your wrinkled channels and on the green land answer the summons sent to you Juno has ordered some chaste nymphs and help to celebrate a noble and true marriage Donrsquot delay

[Enter certain nymphs] [Enter certain nymphs]You sunburnt sicklemen of August wearyCome hither from the furrow and be merryMake holiday your rye-straw hats put onAnd these fresh nymphs encounter every oneIn country footing

You sunburnt harvesters weary from the effects of the heat in August come here from the furrowed land and rejoice Make holiday with your rye-straw hats upon you and meet these fresh nymphs and join in country dancing

[Enter certain reapers properly habited They join with the nymphs in a graceful dance towards the end whereof Prospero starts suddenly and speaks]

[Enter certain reapers properly habited They join with the nymphs in a graceful dance towards the end whereof Prospero starts suddenly and speaks]

PROSPERO[aside] I had forgot that foul conspiracyOf the beast Caliban and his confederatesAgainst my life The minute of their plotIs almost come [to the spirits]Well done Avoidno more

PROSPERO(Aside)I had forgotten the wicked conspiracy of the beast Caliban and his accomplices against my life the time of their plot has almost arrived ndash (To the Spirits) well done depart no more of this

[To a strange hollow and confused noise the spirits heavily vanish]

[The spirits depart]

ORIGINAL TEXT PARAPHRASEFERDINANDLet me live here everSo rare a wondered father and a wifeMakes this place paradise

FERDINANDI should like to live here forever Such a wise and wonderful father makes this place a paradise

[Juno and Ceres whisper and send Iris on employment] [Juno and Ceres whisper and send Iris on employment]

PROSPEROSweet now silenceJuno and Ceres whisper seriouslyTheres something else to do Hush and be muteOr else our spell is marred

PROSPEROMy dear Ferdinand speak no more Juno and Ceres are whispering with a solemn look There is something else coming Silence Or else our magic will be spoilt

IRISYour nymphs called naiads of the wandering brooksWith your sedged crowns and over-harmless looksLeave your crisp channels and on this green landAnswer your summons Juno does commandCome temperate nymphs and help to celebrateA contract of true love Be not too late

IRISYou nymphs called Naiads denizens (M inhabitants) of the running stream with your chaplets of sedge and ever-helpful looks leave your wrinkled channels and on the green land answer the summons sent to you Juno has ordered some chaste nymphs and help to celebrate a noble and true marriage Donrsquot delay

[Enter certain nymphs] [Enter certain nymphs]You sunburnt sicklemen of August wearyCome hither from the furrow and be merryMake holiday your rye-straw hats put onAnd these fresh nymphs encounter every oneIn country footing

You sunburnt harvesters weary from the effects of the heat in August come here from the furrowed land and rejoice Make holiday with your rye-straw hats upon you and meet these fresh nymphs and join in country dancing

[Enter certain reapers properly habited They join with the nymphs in a graceful dance towards the end whereof Prospero starts suddenly and speaks]

[Enter certain reapers properly habited They join with the nymphs in a graceful dance towards the end whereof Prospero starts suddenly and speaks]

PROSPERO[aside] I had forgot that foul conspiracyOf the beast Caliban and his confederatesAgainst my life The minute of their plotIs almost come [to the spirits]Well done Avoidno more

PROSPERO(Aside)I had forgotten the wicked conspiracy of the beast Caliban and his accomplices against my life the time of their plot has almost arrived ndash (To the Spirits) well done depart no more of this

[To a strange hollow and confused noise the spirits heavily vanish]

[The spirits depart]

Ac-12 27420 topic Revaluation of Assets and Liabilities

REVALUATION OF ASSETS AND LIABILITIES

On admission of a new partner the firm stands reconstituted and consequently the assets are revalued and liabilities are reassessed It is necessary to show the true position of the firm at the time of admission of a new partner If the values of the assets are raised gain will increase the capital of the existing partners Similarly any decrease in the value of assets ie loss will decrease the capital of the existing partners For this purpose alsquoRevaluation Accountrsquo is prepared This account is credited with all increases in the value of assets and decrease in the value of liabilities It is debited with decrease on account of value of assets and increase in the value of liabilities The balance of this account shows a gain or loss on revaluation which is transferred to the existing partnerrsquos capital account in existing profit sharing ratioAccounting for Revaluation of Assets and Liabilities when there is a Changein the Profit Sharing Ratio of Existing PartnersAssets and liabilities of a firm must also be revalued at the time of change in profit sharing ratio of existing partners The reason is that the realisable or actual value of assets and liabilities may be different from those shown in the Balance Sheet It is possible that with the passage of time some of the assets might have appreciated in value while the value of certain other assets might have decreased and no record has been made of such changes in the books of accounts Similarly there may be some unrecorded assets amp libilities that may have to be accounted for Revaluation of assets and reassessments of liabilities becomes necessary because the change in the

value of assets and liabilities belongs to the period to change in profit sharing ratio and hence must be shared by the partners in their old profit sharing ratio Revaluation of assets and reassessment of liabilities may be given effect to in two different ways (a) When revised values are to be recorded in the books and(b) When revised values are not to be recorded in the books

When revised values are to be recorded in the booksIn such a case revaluation of assets and reassessment of liabilities is done with the help of a new account called lsquoRevaluation Accountrsquo Sometimes this account is also called as lsquoProfit amp Loss Adjustment Acrsquo If there is a loss due to revaluation revaluation account is debited and if the revaluation results in a profit the revaluation account is credited The following journal entries made for this purpose are

(i) For increase in the value of assetsAsset Ac Dr (individually)To Revaluation Ac(ii) For decrease in the value of AssetRevaluation Ac Dr (individually)To Asset Ac[Decrease in the value of assets](iii) For increase in the value of LiabilitiesRevaluation Ac Dr (individually)To Liabilities Ac[Increase in the value of Liabilities](iv) For decrease in the value of LiabilitiesLiabilities Ac DrTo Revaluation Ac[Decrease in the value of Liabilities](v) For unrecorded AssetsAsset Ac [unrecorded] DrTo Revaluation Ac[Unrecorded asset recorded at actual value](vi) For unrecorded Liability Revaluation Ac DrTo Liability Ac [unrecorded][Unrecorded Liability recorded at actual value](vii) For transfer of gain on revaluationRevaluation Ac DrTo Existing Partnerrsquos CapitalCurrent Ac[Profit on revaluation transferred to capital account in existing ratio](viii) For transfer of loss on revaluationExisting Partnerrsquos CapitalCurrent Ac DrTo Revaluation Ac[Loss on revaluation transferred to capital account in existing ratio](a) When revaluation account shows gain Revaluation Ac DrTo Partnerrsquos Capital Ac (Old Profit Sharing Ratio)(Profit on revaluation credited to Partnerrsquos Capital Ac)(b) Above entry is reversed when revaluation account shows loss Partners Capital Acs (Old Profit Sharing Ratio) DrTo Revaluation Ac(Loss on revaluation debited to Partnerrsquos Capital Acs)

Proforma of Revaluation Account is given as under

Revaluation Account

Dr Cr Particulars ` Amount Particulars ` Amount To Decrease in value of assets By Increase in value of assets To Increase in value of liabilities By Decrease in value of liabilities To Unrecorded liabilities By Unrecorded assets To Gain on Revaluation (Transferred) By Loss on Revalution (Transferred)

ECO ndash12 2742020Topic- ELASTICITY OF DEMAND

CHAPTER - ELASTICITY OF DEMANDMEANINGDemand for a commodity is affected by many factors such as its price price of related goods income of its buyer tastes and preferences etc Elasticity means degree of response Elasticity of demand means degree of responsiveness of demand Demand for a commodity responds to change in price price of related goods income etc So we have three dimensions of elasticity of demandDIMENSION OF ELASTICITY OF DEMAND TYPES OF ELASTICITY OF DEMAND

Price elasticity of demand Income elasticity of demand Cross Elasticity of demand

Price elasticity of demand Price elasticity of demand means degree of responsiveness of demand for a commodity to the change in its price For example if demand for a commodity rises by 10 due to 5 fall in its price Price elasticity of demand (ep)=Percentage change in quantity demanded Percentage change in price of the commodity = 10 ( -)5 = ( - )2Note that ep will always be negative due to inverse relationship of price and quantity demanded

(ii) Income elasticity of demand Income elasticity of demand refers to the degree of responsiveness of demand for a commodity to the change in income of its buyer Suppose income of buyer rises by 10 and his demand for a commodity rises by 20 then Income elasticity of demand (ey)= change in quantity demanded change in price of the commodity =20 10 = 2

Cross Elasticity of demandCross elasticity of demand means the degree of responsiveness of demand for a commodity to the change in price of its related goods (substitute goods or complementary goods) Suppose demand for a commodity rises by 10 due to 5 rise in price of its substitute good then Cross elasticity of demand (ec) = change in quantity demanded change in price of related good = 10 2 = 5 (Tastes and preferences cannot be expressed numerically So elasticity ofdemand cannot be numerically expressed)

  • Chapter 1 Force (Summary)
  • Distinguish between external sovereignty and internal sovereignty
    • NAND Gate
      • Logic diagram
      • Truth Table
        • NOR Gate
          • Logic diagram
          • Truth Table
            • XOR Gate
              • Logic diagram
              • Truth Table
                • XNOR Gate
                  • Logic diagram
                  • Truth Table
                      • Physics
                      • Chapter 1 Electric Field ( Electric Dipole) (Summary)
Page 7:  · Web viewWe all know that Nouns are divided into two parts: common noun and proper noun.Apart from common and proper noun, we will also study about collective noun and compound

3rsquoWhat crime has the prisoners committedrsquo asked the judgeWhat- interrogative adjective

4Whose book is that lying on the tableWhose- interrogative adjective

Subjects COMPUTER CHAPTER-1 (CHARACTERISTICS OF A COMPUTER)A COMPUTER SYSTEM IS A ONE THAT CONSISTS OF ALL THE SOFTWARES HARDWARES CPU AND MEMORY DEVICES SOME IMPORTANT TERMSTHE COMPUTER WORKS ACCORDING TO THE COMMANDS OR INSTRUCTIONS THAT WE GIVE

A SET OF INSTRUCTIONS IS CALLED THE PROGRAM A SET OF ONE OR MORE PROGRAMS IS CALLED A SOFTWARE THE PHYSICAL COMPONENTS OF A COMPUTER ARE CALLED HARDWARE

FOUR OPERATIONS OF A COMPUTER SYSTEM INPUTINPUT IS SUPPLIED TO THE COMPUTER WITH THE USE OF A KEYBOARD A MOUSE A MICROPHONE

OR OTHER INPUT DEVICES PROCESSING PROCESSING IS DONE INSIDE THE COMPUTER IN THE CPU PROCESSING IS THE CONVERSION

OF THE INPUT INTO THE OUTPUT STORAGE STORAGE REFERS TO THE SAVING OF INFORMATION FOR LATER USE INFORMATION IS STORED

IN THE MEMORY OF A COMPUTER THERE ARE TWO TYPES OF COMPUTER MEMORY PRIMARY MEMORY AND SECONDARY MEMORYPRIMARY MEMORY IS FAST SMALL AND EXPENSIVE THE TWO TYPES OF PRIMARY MEMORY ARE RANDOM ACCESS MEMORY(RAM) AND READ ONLY MEMORY (ROM) RANDOM ACCESS MEMORY (RAM)THIS MEMORY IS VOLATILE IN NATURE WHICH MEANS THAT THE

INFORMATION STORED IN IT IS AUTOMATICALLY ERASED WHEN THE COMPUTER POWER IS TURNED OFF

READ ONLY MEMORY (ROM) THIS TYPE OF MEMORY IS NON VOLATILE IN NATURE WHICH MEANS THAT DATA IS NOT LOST WHEN THE COMPUTER POWER IS TURNED OFF

OUTPUT OUTPUT IS THE RESULT PRODUCED BY A COMPUTEROUTPUT MAY BE VIEWED ON THE MONITOR SCREEN HEARD THROUGH SPEAKERS PRINTED THROUGH PRINTER ON PAPERS OR STORED IN THE MEMORY FOR LATER USE

Class VI

GEOGRAPHY CHAPTER 1 REPRESENTATION OF GEOGRAPHICAL FEATURES

GLOBE AND MAPS

GLOBE- A Globe is a three ndashdimensional model of the Earth

EQUATOR ndash An imaginary line runs through the centre of the earth from east to west

NORTHERN HEMISPHERE ndash the half of the north of the equator is called Northern Hemisphere

SOUTHERN HEMISPHERE ndash the half of the south of the equator is called Southern Hemisphere

LATITUDE - Imaginary parallel lines running on the globe from east to west

LONGITUDE ndash The imaginary lines semicircle in nature running from the North Pole to South Pole Also called meridian

MAP ndash Representation of the Earth as a whole or parts of it drawn on a flat surface to a scale

SKETCH ndash A rough drawing of a place not drawn to a scale

PLAN ndash A plan is prepared to show more details about a small area

COMPUTER - VI CHAPTER-6 (INTRODUCTION TO WINDOWS 10)

THE MOST RECENT VERSION OF WINDOWS IS WINDOWS 10 IT WAS RELEASED IN 2015

WINDOWS 10 IS DESIGNED TO WORK FOR YOU AND NOT YOU WORKING FOR IT THE WINDOWS 10 OPERATING SYSTEM INCLUDES A NUMBER OF NEW FEATURES LIKE IMPROVED PERFORMANCE ON MULTI CORE PROCESSORS IMPROVED G U I DATA SECURITY AND BETTER BOOTING PERFORMANCE

WINDOWS 10 HAS 1187 WITH WEARING FEATURES SETS AND INTENDED HARDWARE

THE VARIOUS EDITIONS AVAILABLE FOR WINDOWS 10 ARE AS FOLLOWS

WINDOWS 10 HOME

WINDOWS 10 HOME IS THE EDITION OF WINDOWS 10 THAT CONTAINS THE LEAST NUMBER OF FEATURESWINDOWS 10 HOME IS THE CONSUMER FOCUSED DESKTOP EDITION WITH SUPPORT FOR BOTH PC AND TOUCH ENABLED TABLETS WINDOWS 10 HOME IS AVAILABLE IN BOTH 32 BIT AND 64 BIT VERSIONS

WINDOWS 10 PRO

WINDOWS 10 PRO HAS FEATURES ESSENTIAL FOR SMALL OR MEDIUM BUSINESSESIT HAS A LARGE NUMBER OF EXTRA FEATURES TO MEET THE NEEDS OF SMALL BUSINESSES

WINDOWS 10 ENTERPRISE

WINDOWS 10 ENTERPRISE IS BUILT ON WINDOWS 10 PROIT HAS FEATURES WHICH ARE DESIGNED TO MEET THE NEEDS OF MEDIUM AND LARGE ORGANISATIONSTHIS WINDOWS EDITION TARGETS THE ENTERPRISE SEGMENT OF THE MARKET

WINDOWS 10 MOBILE

WINDOWS 10 MOBILE IS DESIGNED FOR CUSTOMER ORGANISATION THAT ARE USING THE WINDOWS 10 PLATFORM ON SMARTPHONES AND SMALL TABLETS

WINDOWS 10 EDUCATION

WINDOWS 10 EDUCATION IS DESIGNED TO MEET THE NEEDS OF STAFF ADMINISTRATORS TEACHERS AND STUDENTS OF SCHOOLS

WINDOWS 10T EDITION

WINDOWS 10T EDITION OF WINDOWS 10 IS DESIGNED FOR USE IN SMALL FOOTPRINT AND LOW COST DEVICES

Subject English language

Topic Noun kinds

NOUN KINDS

Noun is a naming word Nouns are names of people places animals or things

For example Ashley Mumbai tiger pencils

Kinds of nouns

Proper noun the name of a particular person place or thing is called proper nounExample

1 The Himalayas stand to the north of India2 Monika is the school captain

Common noun announce that names people place or thing in general is called a common noun Example

1 Kalidas was the greatest dramatist of India2 William Shakespeare is Englands national poet

Collective Noun A collective noun is the name of a collection of people or things taken together and spoken of as a whole Example

1 The feet were completely destroyed in the fierce attack2 The crew revolted against Captain Grand

Abstract Noun An abstract noun is the name of some quality state or idea Example1 Diversity Indias biggest strength2 Patience is a virtue

_____________________________________________________________________________________

Class VII

Subject English 1 Topic Articles

ARTICLES

There are three articles ------ A An The

1 Indefinite articles- The Articles a and an are called Indefinite Articles They do not point to a particular person or thing They are used with singular countable nouns It does not identify a specific noun

Uses of Indefinite articles A

Before a singular noun beginning with a consonant or a vowel with a consonant sound For example a cattle a university a one way track

Before a proper noun which is either unfamiliar or holds a special meaning For example For example A Mahesh Sharma wants to see youNeela wants to be a Tendulkar when she grows up

With a number beginning with a consonant sound For example a ten- rupee note a hundred years

Before half when half follows a whole number or after half when it isnrsquotExample one and a half litres (before) Half a litre (after)

With an expression of quantity For example a lot of time a dozen oranges a little while In exclamation before nouns Example Such a mess What a pity

Uses of Indefinite articles An

Before a singular noun beginning with a vowel sound Example an ant an egg an umbrella Before an abbreviation beginning with a vowel or a consonant with the vowel sound

Example an MP an MA Before the word beginning with a silent h Example an heir an hour an honour

Note We do not say a milk or a lemonade because they are uncountable nouns ever we see a cup of milk or a glass of lemon and

Subject GEOGRAPHY

CHAPTER 1 REPRESENTATION OF GEOGRAPHICAL FEATURES

TOPIC- TOPOGRAPHICAL MAPS

Topographical Maps- topographical maps are small-scale maps with detailed depiction of both natural and human-made features

Conventional colours-different colours used in a map are known as conventional colours

TYPES OF SCALE-

Verbal Statement- Scale written in a statement from like 2 cm to a Km is called verbal statement It means that 2cm on the map represents 1 km on the Earth

Representative Fraction- Scale can also be written as a representative fraction (RF)

Example 150000 it means 1cm on the map represents 50000cm or frac12 km on the ground It shows the ratio between map distance and ground distance

Linear Scale Scale can be drawn on a line to show map distance equivalent to ground distance A line is divided into equal parts Each parts represents the actual distance on the ground in mkm

SUBJECT-COMPUTER

CHAPTER-1 (COMPUTER FUNDAMENTALS)

COMPUTER LANGUAGES

THE THE TERM COMPUTER LANGUAGE REFERS TO A SYSTEM OF RULES AND SYMBOLS THAT ARE DESIGNED TO GIVE INSTRUCTIONS TO A COMPUTERTHE COMPONENTS OF A COMPUTER SYSTEM CANNOT PERFORM A TASK BY THEMSELVES THEREFORE THEY HAVE TO BE GIVEN INSTRUCTIONS TO PERFORM ANY TASKCOMPUTER LANGUAGES ARE USED TO CREATE PROGRAMS USING LOGIC BASED ALGORITHMSTHE RULES OF A COMPUTER LANGUAGE IS

KNOWN AS SYNTAX WHEREAS THE TERM SEMANTICS REFERS TO THE MEANING OF LANGUAGESCOMPUTER CANNOT DEVELOP ANY PROGRAMMING LANGUAGE BY THEMSELVES AND CANNOT THINK INTELLIGENTLY UNLESS THEY ARE ASSISTED BY THE HUMAN BEINGS THEREFORE THE PROGRAMMERS DEVELOP A SET OF METHODS AND TECHNIQUES A SET OF INSTRUCTIONS USED TO PERFORM A SPECIFIC TASK IS CALLED A PROGRAM

A PROGRAMMING LANGUAGE IS ALSO KNOWN AS A COMPUTER LANGUAGE CODED BY PROGRAMMERS TO WRITE INSTRUCTIONS FOR A COMPUTERTHE COMPUTER TAKES THESE INSTRUCTIONS AS INPUT AND PRODUCES THE DESIRED OUTPUT

TYPES OF COMPUTER LANGUAGE

THE COMPUTER LANGUAGE CAN BE BROADLY DIVIDED INTO TWO CATEGORIES

LOW LEVEL LANGUAGES HIGH LEVEL LANGUAGES

LOW LEVEL LANGUAGES

A LOW LEVEL PROGRAMMING LANGUAGE REFERS TO THE LANGUAGE THAT IS UNDERSTOOD BY A COMPUTER DIRECTLYTHE PROGRAMMER MUST HAVE AN IN-DEPTH KNOWLEDGE OF DIFFERENT COMPUTERS TO WRITE PROGRAMS IN A LOW LEVEL LANGUAGE THE TWO TYPES OF LOW LEVEL LANGUAGES ARE MACHINE LANGUAGE AND ASSEMBLY LANGUAGE

MACHINE LANGUAGE

COMPUTER HARDWARE UNDERSTANDS ONLY MACHINE LANGUAGE AS IT IS THE FUNDAMENTAL LANGUAGE OF A COMPUTER WHICH UNDERSTANDS ONLY THE TWO DIGITS 0 AND 1 KNOWN AS THE BINARY DIGITSIT IS A SYSTEM OF INSTRUCTIONS EXECUTED DIRECTLY BY THE CPU WITHOUT ANY TRANSLATION AND HENCE IS EXECUTED QUICKLY AS COMPARED TO PROGRAMS IN OTHER LANGUAGES PROGRAMS IN MACHINE LANGUAGE NEED DIFFERENT BINARY LANGUAGE PROGRAMS TO SOLVE THE COMPLEX TASKSPROGRAMMERS OFTEN FIND IT DIFFICULT TO WRITE PROGRAMS IN THIS LANGUAGE

ASSEMBLY LANGUAGE

AN ASSEMBLY LANGUAGE IS A LOW LEVEL PROGRAMMING LANGUAGE BUT THE INSTRUCTIONS ARE WRITTEN IN THE FORM OF WORDS KNOWN AS MNEMONICS TO CONVERT AN ASSEMBLY LANGUAGE INTO TO MACHINE CODE WE REQUIRE A UTILITY PROGRAM WHICH IS KNOWN AS AN ASSEMBLER ASSEMBLY LANGUAGE PROGRAMS CANNOT BE EXECUTED DIRECTLY BY A COMPUTER THEREFORE THEY ARE SLOWER THAN THE MACHINE LANGUAGE PROGRAMSIT IS EASIER TO WRITE PROGRAMS IN THE MACHINE LANGUAGE THEN AND IN ASSEMBLY LANGUAGE

_______________________________________________________________________________________________

CLASS-VIII

SUBJECT-COMPUTER CHAPTER - Operating system and graphical user interface Role and functions

Need of operating system

It co-ordinates different hardware and software componentsof a computer system It supervises the various actions of the computer system and enables the computer to work in a effective

manner It helps in smooth functioning of various peripherals

In a multi-tasking operating system it determines the order and time to be allowed for each application before giving another application a turn

It sends messages to the system operator about the status of operation or any error that may have occurred while running the application

What is operating system

An operating system is an integrated system of programs that manages various resources and the overall operation of the computer system It is designed to support various activities of computer system in a systematic way

Role of an operating system-

Operating system enables the user to use the system effectively An operating system manages various application that runs on a computer and shares computerrsquos resources User interacts with operating system through command line interface and graphical user interface

Function of an operating system -

Booting the computer it is the start up procedure of a computer system Loading theprograms in the memory - when system is ready the operating system loads certain program

automatically Manages resources it manages between the hardware and software resources Detecting and correcting errors- if the supporting hardware or software doesnrsquot works properly then the

operatingsystem tries to rectify it Ensuring data security programs and data donot interfere with each other Maintaining the internal clock of system - maintain internal clock of system when system is shut down

SUBJECT-GEOGRAPHY CHAPTER 1 REPRESENTATION OF GEOGRAPHICAL FEATURES THROUGH CONTOURSTopographical maps- topographical maps are small-scale maps with detailed depiction of both natural and human-made features

Contour lines- contour lines are imaginary lines drawn on a map joining places having the same height above the mean sea level

Contour linesContour interval- It is an interval at which contour lines are drawnIndex contour- At every 100-m interval a thick brown contour line is called index contour It is drawn for calculating height

Spot heights- The spot heights show heights in metres above the mean sea level

SUBJECT- English Language CHAPTER - The Sentence

Complex sentence-

We have seen that a Complex Sentence consists of a Principal Clause with one or more Subordinate Clauses

We have also learnt that there are three kinds of Subordinate Clauses The Adjective Clause the Adverb Clause and the Noun Clause

1) THE ADJECTIVE CLAUSE

An ADJECTIVE CLAUSE does the work of an ADJECTIVE It qualifies a noun or pronoun In the following examples in each set two commonly patterned sentences are compared one with an ADJECTIVE and the other with an ADJECTIVE CLAUSE ( The Adjective or Adjective Clause qualifies the Noun placed in the box)

a) He is a lazy boy (Adjective)

b) He is a boy who is lazy (Adjective Clause)

a) It is a beautiful place for the tourists (Adjective)

b) It is a place where tourists come for the scenic beauty (Adjective Clause)

a) We have enough funds for the work (Adjective)

b) We have funds which would be enough for the work (Adjective Clause)

Convert the sentence from simple to complex sentence-

1) We believe his honesty Ans- We believe that he is honest

2) This is the birth place of RamaAns- This is the place where Rama was born

____________________________________________________________________________________________

Class IX

Subject English Language

Topic Preposition

Date 270420

PREPOSITIONS

A preposition is a word placed before a noun or a pronoun to show in what relation the person or thing denoted by it stands in regard to something else

EXAMPLE

There is a clock on the wall

The preposition on shoes relationship between clock and the wall

Smitha is afraid of lizards The man jumped off the bus

Here the preposition of shows the relationship between afraid and lizards

The preposition off shows the relation between jumped and bus

The noun or the pronoun which follows a preposition is called its object

So in the first Example wall is the object of the preposition on

A preposition can have more than one object For example

The plane flew overhouses and meadows

Prepositions are used to express a number of relationship including time locationmannermeans quantity purpose and state or condition

Points to remember

Preposition joins a noun to another noun or a pronounThere is a cow in the field

A preposition joints a noun to a verbThe cat runs after the rat

Preposition can have two or more than two objectsThe road runs over hills and plain

A preposition also joins a noun an adjectiveHe is fond of tea

Generally a preposition comes before an object Sometimes it comes even after an object asWhat are you looking atThis is the house I live in

Subject- Computer Application

Chapter 2 Introduction to Java

Java API An application programming interface (API) in the context of Java is a collection of prewritten packages classes and interfaces with their respective methods fields and constructors

Byte Code Java bytecode is the result of the compilation of a Java program an intermediate representation of that program which is machine independent The Java bytecode gets processed by the Java virtual machine (JVM) instead of the processor JVM The Java Virtual Machine (JVM) is the runtime engine of the Java Platform which allows any program written in Java or other language compiled into Java bytecode to run on any computer that has a native JVM

Platform A platform is the hardware or software environment in which a program remains

Java platform The Java platform differs from most other platforms in that itrsquos a software only platform that runs on top of other hardware-based platforms

Applet and Application The fundamental difference between the two Java programs is that an application program is designed to run on a stand-alone machine whereas an applet is a web-version of an application which is used to run a program on a web browser

WORA ldquoWrite once run anywhererdquo (WORA) or sometimes write once run everywhere (WORE) is a slogan created by Sun Microsystems to illustrate the cross-platform benefits of the Java language

Class XSubject Topic Summary Execution

ECONOMICSFACTORS OF PRODUCTION Sub-topiclsquoLABOURrsquo

We shall start our class by discussing the topic taught in the last class

lsquo DIVISION OF LABOURrsquo- By division of labour we mean specialization in workIt refers to splitting up the work of labour involved in the production of a particular commodity into several parts and each part and sub-part is performed by a specialist

Now let us start by the Advantages of Division of labouraIt increases the level

Questions

1 Differentiate between Product-Based division of labour and Process-Based division of labour

Product-Based division of labour

Process-based division of labour

It is also known as simple (or occupational) division of labourUnder it everybody performs a particular occupations The entire is done by the same person

When a person or group of persons undertakes a specialised function which is supplementary to the production of final commodity and service This is also as complex division of labour

It is simple It is complex

of productionbSince the product is produced by an expert workerbest quality of product is producedcIt saves time and toolsd it promotes inventions in the methods and techniques of productioneIt leads to reduction in costs fAll workers get work according to their abilities and choices

Now let us discuss the disadvantages of Division of labour

a Since many workers are involved in the production of a commodity no one has the sense of responsibility

b The constant and repetition of the same work again and again make the work monotonous

c Division of labour facilitates production on large scale Hencethere is fear of over production

d Because of territorial division of labour some areasregions become more developed than others

It is based on labour-intensive techniques of production

It is based on capital-intensive techniques

It is generally found in small enterprises

It is generally found in large enterprises

Example Indian farmers doing all farm activities

Example A modern garmet factory where one person takes the measurementanother does the cuttingsome sew the clothes while a few workers button them and other iron them

2 What are the advantages of Division of labouraIt increases the level of productionbSince the product is produced by an expert workerbest quality of product is producedcIt saves time and toolsd it promotes inventions in the methods and techniques of productioneIt leads to reduction in costs fAll workers get work according to their abilities and choices

3Discuss the disadvantages of Division of laboura Since many workers are involved in the

production of a commodity no one has the sense of responsibility

b The constant and repetition of the same work again and again make the work monotonous

c Division of labour facilitates production on large scale Hence there is fear of over production

d Because of territorial division of labour some areasregions become more developed than others

English 1 Transformation of sentences

Sentences A sentence is a group of words which makes complete sense

a Assertive sentences

Exercise 6Rewrite the following sentences according to the instructions given below without changing their meanings

1 As soon as he saw the beer he jumped into

b Imperative sentences

c Interrogative sentences

d Exclamatory sentences

Sentences can be changed from one grammatical form to another without changing the meaning of the sentence This is known as transformation of sentences

the river ( Begin No sooner)2 None but brave deserve the fair (Begin the

bravehellip)3 This box is too heavy for me to lift ( Use so hellip

That instead of too)4 No one other than a king can live like James

Luxurious ( Begin only James)5 Oh for the wings of a dove (Begin I wishhellip)

Math Topic Commercial MathematicsChapter Shares and Dividends

Study item Discuss about shares and Dividends1) What is share

Ans To start any big business (company or Industry) a large sum of money is needed But it is not possible for an individual to invest such a large amount Then some persons interested in the business join together and from a company They divide the estimated money required into small parts Each such part is called a share

2) What do you mean by the term shareholder

Ans A person who purchases one or more shares is called shareholder3) Some terms related with a share

(i) Nominal value or face value or printed value The original value of a share is called its nominal value or face value or printed value

Note The nominal value of a share always remains same(ii) Market value or cash value The price of a share at any

time is called its market value or cash value

Note The market value of a share changes from time to time(iii) At par If the market value of a share is the same as its

nominal value the share is called at par(iv) At Premium or above Par If the market value of a share

is more than its nominal value the share is called at premium or above par

Example If a share of Rs 100 is selling at Rs 150 then it is said to be selling at a premium of Rs 50 or Rs 50 above par

(v) At Discount or below par If the market value of a share is less than its nominal value the share is called at discount or below par

Example If a share of Rs 100 is selling at Rs80 then it is said to be selling at a discount of Rs 20 or at Rs 20 below par

4) What is Dividend

Ans The profit which a shareholder gets for hisher investment from the company is called dividendNote (i) The dividend is always expressed as the percentage of the face value of the share(ii) The dividend is always given( by the company ) on the face value of the share

irrespective of the market value of the shareBENGALI(2ND LANGUAGE)

ldquoদেবতোর জণমrdquoলিবরোম চকরবত

পরথম লিসর পোঠ-চোর পসথ একটি পোথর লিবপলি ঘটোয় দেক যোতোয়োসতর পসথ পরলিতলিয়ত ওই পোথসর দেো োচট দেসত একলি দেতো দেক দেো োচট দেসয় দেবোমো সয় রোসতোর মোস লি0টসক পস1 লিবপরীত লিক দেথসক আো একটি দেমোটর োলি1 চোসকর কষতোয় পরোসরণ দেবোসচ যো লিকনত পরলিতবোর এমরণ দেৌভোয দেসকর োও সত পোসর তোই লিতলি দেকোো দেজোো1 কসর পোথরটিসক উপস1 দে8স পোথর উপস1 দে8োর ময় এক দেকৌতী জতো দেকসক পরশন কসর দেয লিতলি দেকোসো দেবতোর আস দেপসয়স0রণ লিকো লিকনত দেক বস লিতলি দেকোসো দেবতোর আস পোলি দেক উপলিত ক জতোর উসltসয বস কোরও ইস= স পোথরটি লিসয় দেযসত পোসর এর পর দেথসক দেক দেযসত আসত পোথরটি দেক দেসত পো একলি দেক কষয করস দেকউ পোথরটিসক ধসয়মস0 পলিরসকোর করস0 দেক ওই দেকৌতী জতোসক পোথসরর কোস0 বস থোকসত দেস এ0ো1ো আরও কষয কসর দেক ঠোৎ ওই লি1 পোথরটির োসয় লিোর োো-দেকউ পজো কসরস0 দেকৌতী দেোকটির আঙকো য় যলিসক উ পোথরটিসক লিরসয় দে8স তোর পর ঠোৎ একলি পোথরটির দেোোজ দেই দেক লিসয় দেস0 বো দেকোথোয় দেস0

বদোথ-

দেো োচট ndashচসত লিসয় দেকো লিক0র সE ধোককো দেস পস1 যোবোর উপকরমঅকসমোৎ- ঠোৎআতমমবর- লিসজসক লিয়নতরপ-পো লিপ0স প1োদেসতসসত- লিসপলিউৎোত- দেো1ো দেথসক উপস1 দে8োপরতযয় ndash লিবশবোপরতযোস- টতযোস ndash দৈবোসধসতোধলিসত- পরসপসরর পরলিত ব পরসয়ো করোপরসতরীভত- পোথসর পলিররণলিতবোনতঃকরসরণ- মস পরোসঅলিQৎ- অQো করসত ই=কদেোপ- বধমোর- দেবোস এমইতযোকোর- এইরকমরম- বময়পযসোভী-পসযর জয দেোভ আস0 যোরপোসথয় ঞচয়- পথচোর রচ জমোসোমোমোসরোস- ব ধম ধোসমর সE

তোর লি দেইhelliphellipTo be continued

Hindi 2nd lang सर क पद(सरदास)

सरदास शरी कषण भकति कावय क सरवशरषठ कगिरव ह इनक जनम और मतय क समय तथा सथान का मतभद हसरदास रवातसलय और शरार रस क अनयतम कगिरव ह इनक कावय म बालकषण क सौदय चपपल चषटा और गि7याओ की मनोहर झाकी मिमलती ह कषण और ोगिपयो क अननय परम का कतिचतरण ह सयो शरार की अपकषा उनक कावय म गिरवयो शरार का अमिBक गिरवषय और मारमिमEक कतिचतरण हआ हइन पकतियो म हम सरदास की भकति भारवनाओ का परिरचय मिमलता ह इनका सपण सगरह सरसार म गिनगिहत ह

1 जसोदा हरिर पालन झलारवहलरारवदलराईमलहारव रव जो ईसाई कछ ारव मर लाल को आई निनEदिदया काह ना आगिनसबारव त काह नाही बरवगिह आरवतोको कानहा बलारव

शबदाथ-हलरारव-गिहलती हदलराई - दलार पयार करती हमलहारव-पचकारती हनिनEदरिरया ndashनीदरवगिह-जलदी सअBर-होठमौन-चपसन-सकत

वयाखया- सरदास जी कहत ह गिक यशोदा माता बालक कषण को पालन म झल आती ह रवह उनह गिहलाती ह पयार करती ह मलहार जस कोई ीत ान लती ह और नीद स पछती ह गिक ह नीद तम मर लाल को आकर कयो नही सलाती तझ खाना बला रहा ह कभी कषण आख बद कर लत ह कभी आखफडफडान लत ह उनह सोता हआ जानकर यशोदा माता चप हो जाती ह और इशार म बात करन लती ह इसी बीच अकला कर कषण ज जात हतो गिफर यशोदा माता गिफर स ाना ान लती ह सरदास जी कहत ह गिक भरवान क दशन का सख दरवता और ऋगिष-मगिनयो को भी दलभ ह यही सख माता यशोदा को बडी सहजता स मिमल जा रही ह माता यशोदा बहत ही भागयशाली ह2)Continue to nexthellip

Physics

Chapter 1 Force

(Summary)

Question A body is acted upon by two forces each of magnitude F but in opposite directions State the effect of the forces if

(a) Both forces act at the same point of the body

(b)the two forces act at two different points of the body at a separation r

Solutions

(a) Resultant force acting on the body = 0

F ndash F = 0(b) The forces tend to rotate the body between two forces about the midpoint

Moment of forces = F times rFr

QuestionDefine moment of a couple Write its SI unit

Solutions

Moment of couple is equal to the product of both force and the perpendicular distance between the two forces

The SI unit of moment of couple is NmCommercial Studies

Advertising and sales

Business firms use several methods to

Questions1) What do you mean by advertising

promotion create demand of their product in the market and increase it sales Such methods comprises of advertising sales promotion personal selling and publicityToday we are going to discuss about one of such methods It is advertising

Meaning of advertisingAdvertising is a paid form of non-personal presentation for promotion of Ideas goods and services

Importance or merits of advertising Advertising has importance to manufacturer or traders to customer and to society as a whole

Today we will see how advertisement help the manufacturer or traders

Answer) Advertising is a means of how a company encourages people to buy their products services or ideas It is one element of marketing which also includes design Research and data mining

2) Mention any three features of advertisingAnswer)The main features of advertising are

i) It is impersonal form of presentation for promotion of products and services of Ideas

ii) It is issued by identified sponsor The advertisement contains the name of the advertiser

iii) It is a form of mass communication because the message is directed to a large number of persons simultaneously

3) Mention the main merits or importance of advertisement to manufacturer or tradersAnswer)

i) Introducing new product A business organization can introduce itself and its products to the public through advertising

ii) Increase the sale Advertising leads to increase the sale of existing product by entering into new markets and attracting new customers

iii) Create steady demand Advertising creates sustains regular demand by smoothening out seasonal and other fluctuations It enables regular production for the organisation

iv) Economics of scale Advertising facilitate mass distribution of goods and steady demand which lead to large scale and regular production

v) Goodwill Advertising helps in creating a good image of the firm and reputation for its products

Biology Chapter - 03Genetics

Today wewill start chapter and discuss about Genetics Gregor Mendel is known as father of genetics Before entering into Mendelrsquos experiment on Genetics we must know

Q1 Define the following termsi) Genetics Genetics is the study of

transmissionof body features from parents to offspringand the laws relating to such transmission

ii) Heredity It may be defined as transmissionof genetically based characteristics from parentsto offspring

iii) Character and traits Any heritable

Importance to TraderIntroducing new productIncrease the saleCreate steady demandEconomics of scaleGoodwill

some terms featureis a character The alternative forms of acharacter are called traitsex Character (Hair shape) - Traits (Curly straight)

iv) Homologous chromosomes A pair ofcorresponding chromosomes of the same shapeand size one from each parent

v) Genes Genes are the specific parts (DNA segments) of a chromosome which determinethe hereditary characteristicsNearly 30000genes present in human

vi) Alleles Alternative forms of a gene occupying the same position (locus) on homologouschromosomes and affecting the same characteristicbut in different ways

vii) Genotype ndash PhenotypeGenotype means of genes present in the cells of an organism Phenotype means the observable characteristic which is genetically controlled

viii) Mutation It is a sudden change in one or more genes or in the number or in the structure of chromosomes ex Sickle cell anaemia is a blood disease caused by a gene mutation

CLASS NOTES

Class XSubject Eng Literature (The Merchant of Venice ndash William Shakespeare)Topic Act IV Scene 1 Lines 01 to 34 ( Duke helliphelliphellip We all expect a gentle answer Jew) ate 27th April 2020 (2nd Period)

[Students should read the original play and also the paraphrase given in the school prescribed textbook]Summary Questions amp Answers

This scene may be termed as the catastrophe of the play It is the final unravelling of the complicated events which seem to threaten the happiness of Bassanio Portia and Antonio Right is justified to the fullest degree and malice falls into the trap prepared for others No one suffers here but Shylock but even then he receives a generous measure of mercy

o This is the Court-scene Initially we meet

(1)

DUKE I am sorry for thee thou art come to answer (Line 3-6)A stony adversary an inhuman wretchUncapable of pity void and emptyFrom any dram of mercy

(i) Who is addressed here Where is the person Why is the person there

Antonio is addressed hereAntonio is in the court of justice at VeniceAntoniorsquos trial is scheduled to be held here for his failure to meet the conditions of the bond he signed with Shylock

the Duke Antonio Shylock and Salerio Later we meet Bassanio Portia Gratiano and Nerissa

o The Duke says to Antonio that he has to face a very cruel opponent which Antonio admits and expresses his gratefulness to the Duke for his efforts to soften without result the heart of Shylock in order to be merciful to Antonio Antonio further says that he is ready to accept whatever cruel judgement the Court may award

o When Shylock appears in the court the Duke says that Shylock should change his decision of prosecuting Antonio and demanding the penalty specified in the bond out of consideration of the great misfortunes that Antonio has suffered If this is done by him (Shylock) the whole court would be gladdened by his merciful action

(ii) What is the Duke sorry for

The Duke is unable to change the mind of Shylock from his decision to get the bond forfeited even after he pleaded to Shylock Shylock stands firmly for his bond which when forfeited will allow him to take a pound of flesh from any part of Antoniorsquos body(iii) How does the Duke address Shylock`The Duke calls Shylock an adversary with a heart of stones He calls Shylock as an inhuman wretch without pity Shylock is quite lacking in the slightest quality of mercy (iv) How does Antonio reply to this

Antonio replies that he will meet the revenge of Shylock patiently He has prepared himself to suffer with a quiet spirit the utmost that Shylockrsquos tyranny and rage can do(v) What quality of the Duke is revealed here

The Duke is kind and benevolent He is ready to help Antonio He requests Shylock to free Antonio from the trial(vi) What are the terms of the bond that Antonio has signed

The terms of the bond that Antonio has signed were that if Antonio is unable to repay Shylock a certain sum of money specified on the paper on a certain date and in an agreed place the forfeit has to be paid The forfeiture will be an exact pound of Antoniorsquos flesh which Shylock will be a liberty to take from any part of Antoniorsquos body which pleases him

Class XI

STUDY MATERIAL

Class XISubject Eng Literature (The Tempest ndash William Shakespeare) Topic Act I Scene 2 Lines 88 to 132 (Prospero hellip Me and thy crying self) Date 27th April 2020 (3rd Period)

[Students should read the original play and also the paraphrase given in the school prescribed textbook]Summary Questions amp Answers

o Prospero now tells Miranda that he was the Duke of Milan He had been devoting himself more to studies than the affairs of the State His brother Antonio took advantage of this situation and with the help of Alonso the king of Naples seized upon him and her one midnight and shipped them in a frail bark so that they perished in the sea All this took place

(1)

MIRANDA I should sin (Line 118-132)

To think but nobly of my grandmother

Good wombs have borne bad sonsPROSPERO Now the condition

The King of Naples being an enemyTo me inveterate hearkens my brothers suitWhich was that he in lieu othrsquo premisesOf homage and I know not how much tribute

twelve years back

IMPORTANT PASSAGES EXPLAINED(Line 98-103)

PROSPERO helliphelliphelliphelliphelliphelliphellip

He being thus lorded

Not only with what my revenue yieldedBut what my power might else exact like oneWho having into truth by telling of it Made such a sinner of his memoryTo credit his own lie he did believeHe was indeed the duke

Prospero in telling the narrative of his past life here refers to his brother Antonio Prospero being with a studious bent of mind has left the administration of Milan on his younger brother Now Antonio being thus invested like a lord with all the powers derived from Prosperorsquos wealth and what the exercise of Prosperorsquos authority might secure for him regarded himself as a de facto Duke of Milan It is a well-known fact of psychology that a man who repeatedly tells a lie makes of his memory such a sinner against truth as to credit his own lie by the telling of it So Antonio by repeatedly saying to himself and others that he was the Duke came to believe that he was really the Duke Thus falsehood repeatedly asserted gained the force of truth for Antonio and he truly believed it

Should presently extirpate me and mine Out of the dukedom and confer fair MilanWith all the honours on my brother whereonA treacherous army levied one midnightFated to thrsquo purpose did Antonio openThe gates of Milan and ithrsquo dead of darkness The ministers for thrsquo purpose hurried thenceMe and thy crying self

(i) In the earlier lines of this scene what does Prospero tell about his intense interest What was the demand of his interest

In the earlier lines of this scene Prospero tells Miranda that he had an intense interest in the study of philosophy and magic arts Hence in order to improve his mind with this kind of study he kept himself isolated from worldly and state affairs His study was dearer to him than the applause and esteem that he could win from the public His study demanded too much solitude(ii) What forced Antonio to take an undue advantage over Prospero

Prosperorsquos indifferent attitude towards the statersquos affairs and his having boundless trust in Antonio gave rise to a boundless lust for power in Antoniorsquos mind Antonio felt that he must be the actual Duke instead of the part of the Duke he played Thus Antonio took an undue advantage of the situation to usurp Prosperorsquos dukedom(iii) Explain the following lines ldquoI should sin to think but nobly of my grandmother Good wombs have borne bad sonsrdquo

After hearing the treacherous act of her uncle Antonio Miranda says that Prosperorsquos mother was a noble lady and she cannot dishonour her memory by saying that the person named Antonio cannot be his (Prosperorsquos) brother She says that in honour of her grandmother she also cannot say that Antonio must have been begotten not by her grandfather but by some other man She finally concedes that it is known that good mothers have borne bad sons in their wombs and gave birth to them

(iv) Why did the King of Naples accept Antoniorsquos request to help him in usurping his dukedom What did Antonio propose to Alonso

Alonso who was the king of Naples accepted Antoniorsquos request in usurping Prosperorsquos dukedom because he (Alonso) was a sworn enemy of Prospero Antonio proposed that Alonso should immediately drive him (Prospero) and his offspring out of Milan and should confer the dukedom upon him (Antonio) with all the dignities which go with that

position In return Antonio promised that he shall give an annual tribute and also swore his allegiance to Alonso Also he agreed to hold the Dukedom of Milan as a subordinate to the state of Naples(v) How were Prospero and Miranda carried away from the city of Milan and what was the state of small Miranda at that time

In pursuance of the agreement settled between Antonio and Alonso an army of treacherous men was assembled One midnight when the occasion suited the will of destiny Antonio opened the gates of the city of Milan and in the death like silence of midnight Antoniorsquos agents who had been directed to execute his purpose carried Prospero and small Miranda away from the city in all haste They were then forced into a ship and carried some distance out to sea where they put them on a mere hulk of a boat without any rigging or ship-gear and abandoned them leaving them at the mercy of the roaring sea Miranda was a very small child of three years age and she was crying at that time

CLASS -XIDATE-270420Subject Topic Summary Execution

EVS Chapter 1 ndash Mode of Existence

Impact of mode of existence on resources

Q) Why resources are under pressure

Ans - Increase in the sophistication

of technology enabling natural resources to be extracted quickly and efficiently Eg in the past it could take long hours just to cut down one tree only using saws Due to increased technology rates of deforestation have greatly increased

The number of humans is increasing Cultures of consumerism Materialistic views

lead to the mining of gold and diamonds to produce jewelry unnecessary commodities for human life or advancement Consumerism also leads to extraction of resources for the

production of commodities necessary for human life but in amounts excessive of what is needed because people consume more than is necessary or waste what they have

Lack of awareness among the population is striking People are not aware of ways to reduce depletion and exploitation of materials

Accounts Cash Book Today we are going to start a new topic -Cash Book

The key terms used in this chapter are

bullCash book

bullSimple cash book

bullDouble column cash book bullTriple column cash book

bullPetty cash book

bullCash discount

bullContra entry

Here I will share you the meaning of each key terms

bullCash book Cash Book is a special purpose subsidiary book or journal in which cash received and cash payments are recorded

bullSimple cash book

It is a cash book in which only cash transactions are recorded It has only one column on each side

bullTriple column cash book

It is cash book which has three columns one column for each cash and Bankdiscount on each side of the cash book In this book both cash and Bank transactions are recorded together with discount allowed and received

bullPetty cash book

It is a cash book maintained for recording petty expenses

bullCash discount

Cash discount is the amount of discount received or allowed on cash payments and cash receipts Discount received is an income for the business while discount allowed isan expense

bullContra entry

It means transactions involving both cash and Bank Such transactions though recorded in the cash book are not posted into ledger The letter lsquoC is written in Ledger folio for contra entry

Business Studies

ENTREPRENEURSHIP

Now we shall discuss the second chapter

lsquoENTREPRENEURSHIPrsquo

Today before starting the chapter let us recall what

Questions

1What are the main characteristics of Intrapreneurship

Answer

The main characteristics of Intrapreneurship are

Corporate framework-it occurs within the framework of the same company

Semi-Autonomous-Intrapreneurship

we have read last day

Let s today start the class by recalling the last topic taught

Intrapreneurship is the process of discovering and exploring business opportunities within an existing company It involves launching new business ventures within the framework of a present corporation Intrapreneurship is also known as corporate entrepreneurship or corporate venturing

Now let us start with the characteristics of Intrapreneurship

The main characteristics of Intrapreneurship are

Corporate framework

Semi-Autonomous Lack of ownership Senior position Low risk taking Not own boss

Now let us discuss the meaning of enterprise

Enterprise means an undertaking or adventure that requires some innovation and investment and thus involves riskEnterprise always entails decision making coordination and risk bearing

involves crating amd nurturing a semi-autonomous business unit which may be a subsidiary a strategic business unit or a division

Lack of ownership-the intrapreneur is not the owner of the unitb he creates and nurtures

Senior position-he occupies a senior managerial position in the company

Low risk taking-An intrapreneur does not bear the full risk of failure

Not own boss-An intrapreneur is not his own bosss in legal termsHe enjoys the freedom and gets the required resources and support

2 How is Entrepreneur is different from Intrapreneur

The functions involved in both the entrepreneurship and intrapreneurship are by and large similar however there are several differences between the two

Point of distinction

Entrepreneur

Intrapreneur

status An independent business person

A senior executive within a company

Ownership Owner of

the business

An employeesometimes a share in ownership

Financing Responsible for raising finance for the business

Not responsible for raising the finance

Risk bearing

Bears the risk of the business

Does not bears the risk of the business

Reward Profit which is uncertain and irregularcan be loss

Fixed salary and fringe benefits

Need for security low high

3 What do you understand by enterprise

Answer Enterprise means an undertaking or adventure that requires some innovation and investment and thus involves riskEnterprise always entails decision making coordination and risk bearing

COMMERCE NATURE AND OBJECTIVES OF

BUSINESS

Today let us recall the last other two objectives of business by the chart given in the previous class

Firstly we would discuss Human Objectives

Business is run by people and for people Labour is a valuable business element

Human objectives of business are concerned with the well -being of labour

The human objectives are as follows

Labour welfare Developing human

resources Participative

management Labour

management cooperation

Questions

1 Explain the human objectives of a business enterprise

Answer

Business is run by people and for people Labour is a valuable business element

Human objectives of business are concerned with the well -being of labour

The human objectives are as follows

Labour welfare-Business must recognize the dignity of labour and human factors should be given the recognition

Developing human resources-Employees must be provided the opportunities for developing new skills and attitudes

Participative management-Employees should be allowed to take part in decision making process of business

Labour management cooperation-Business should strive for creating and maintaining cordial employer employee relations so as to ensure peace and progress in industry

Now let us discuss the national objectives of business

Optimum utilization of resources

National self- reliance Development of small

scale industries Development of

backward areas Control over pollution

2Explain the national objectives of a business enterprise

Answer

It is the duty of business to utilize the resources of the country properly the national objectives of business

Optimum utilization of resources ndashBusiness should use the nationrsquos resources in the best possible manner

National self- reliance-It is the duty of the business to help the government in increasing experts and in reducing dependence on imports

Development of small scale industries-Big business firms are expected to encourage growth of small scale industries which are necessary for generating employment

Development of backward areas-Business is expected to give preference to the industrialization of backward regions of the country

ECONOMICS

BASIC ECONOMIC CONCEPTS

SUB

TOPIC

Value

Wealth

Welfare

Today we shall start with a new topic of the same chapter ie lsquoValuersquo

Value of a commodity is defined as the valuation placed by a household on the consumption of this commodity

lsquoValuersquo has two different meanings and these are

a Value -in -use It refers to consumption value of a commodity It expresses the utility derived from the consumption of a particular commodity A necessity like water has a very high value ndashin ndashuse or

Question

1What is value

Answer

Value of a commodity is defined as the valuation placed by a household on the consumption of this commodity

2What is value-in use

Answer It refers to consumption value of a commodity It expresses the utility derived from the consumption of a particular commodity A necessity like water has a very high value ndashin ndashuse or consumption value

3What is value ndashin- exchange

Answer It relates to market value of a commodity

It is the rate at which a particular good or service can be exchanged for money

For example in barter system if a person is prepared to exchange 3 metres of cloth with 1 pair

consumption value

b Value ndashin-exchange It relates to market value of a commodity

It is the rate at which a particular good or service can be exchanged for moneyFor example in barter system if a person is prepared to exchange 3 metres of cloth with 1 pair of shoes then the value in exchange of 3 metres of cloth is 1 pair of shoesValue in exchange is the power of purchasing other goods In modern monetised economies the exchange value of goods are expressed in terms of money as prices

Now let us discuss the term lsquoWealthrsquo

Wealth refers to the stock of all those assets which are a source of income

Wealth is a stock concept

Wealth must possess the following features

a Utility It must possess utility or give some

of shoes then the value in exchange of 3 metres of cloth is 1 pair of shoes

Value in exchange is the power of purchasing other goods In modern monetised economies the exchange value of goods are expressed in terms of money as prices

4 What is wealth

Answer it refers to the stock of assets or goods which are a source of income and have personal or national ownership

5 What are the features of wealth

Answer The features of wealth are as follows

Wealth must possess the following features

a Utility It must possess utility or give some satisfaction

b Scarcity It must be limited in quantityc Transferability it should be transferable its

ownership can be transferred from one person to another person

d Exchange value It must possess exchange value

6 What is welfare

Answer

Welfare is defined as satisfaction and happiness a sense of well- being among the people

satisfactionb Scarcity It must be

limited in quantityc Transferability It

should be transferable its ownership can be transferred from one person to another person

d Exchange value It must possess exchange value

Now let us discuss the term lsquoWelfarersquo

Welfare is defined as satisfaction and happiness a sense of well- being among the people

Welfare is affected by factors like

a Consumption of goods and services

b Environment

c Family relations

d Degree of freedom

e Law and order situation

Mathematics Trigonometric equation

To find the general solution of the equation sinθ=0

When sin θ =0

Then θ= 0 π2π 3π-π -2π -3

i e when θ = 0 or an integral multiple of π

i e when θ= nπ where n is any integer

Therefore the general solution of the equation sin

Example1 Find the general values of θ which satisfy the equation sin2 θ =34

Solution sin2 θ= 34

Or sin θ = +34 or -34

Or sin θ = sin π3 or sin (-π3)

Therefore

θ = [nπ + (-1) n (π3)] or[ nπ+ (-1) n (-π3)]

= nπ +π3 or nπ-π3 where n= any integer

Example 2Find the values of θ which satisfy tan2 θ

θ=0 is θ= nπ where n is any integer

To find the general solution of the equation cos θ=0

When cos θ=0

Then θ=π2 3π2 5π2 -π2 -3π2 -5π3

i e when θ is an odd multiple of π2

i e when θ=(2n+1) π2 where n= any integer

Therefore the general solution of the equation cos θ =0 is θ= (2n+1) π2 where n= any integer

To find the general solution of the equation tan θ = 0

Clearly tan θ =0 implies sin θcos θ =0

Therefore θ = nπ

i e the general solution of the equation tan θ=0 is θ =nπ where n = any integer

To find the general solution of the equation cot θ =0

Clearly cot θ =0 implies (cos θsin θ) = 0

i e cos θ =0

Therefore θ = (2n+1) π2

Therefore the general solution of the equation cot θ =0 is θ = (2n+1) π2

Where n= any integer

To find the general solution of the equation sin θ= k (-

=13 -πleθleπ

Solution tan2 θ =13

Or tan θ = plusmn1radic(3) =tan(plusmnπ6)

θ=nπ plusmn π6 where n =any integer

If n=0 then θ=plusmnπ6

If n=1 then θ= π plusmn π6

If n=-1 then θ= -π plusmn π6

Therefore the required solution in -π le θ le π are θ= π6 5π6 -π6 -5π6

Exercise Find general solution of sin 2θ=cos θ [Hints Use sin 2θ= 2sin θcosθ and then take cosθ

common]

1lek le1)

Determine an angle alpha such that sin =k and -π2le αle π2

Then we have

Sin θ = k = sin α

Or sin θ - sin α =0

Or 2 cos [(θ+α) 2] sin [(θ-α) 2] =0

Therefore either cos [(θ +α) 2] =0 (1)

Or sin [(θ-α) 2] =0 (2)

Now from (1) we get (θ+α) 2= (2m+1) π2)

Or θ = (2m+1) π-α (3)

And from (2) we get (θ-α) 2 =mπ

Or θ= 2mπ+α(4)

Where m = any integer

Clearly the solution (3) amp (4) may be combined in the following form

θ= nπ+(-1) n α where n= any integer

Therefore the general solution of sin θ = sin α is θ = nπ +(-1) n α where n is any integer and -π2 le α le π2

Biology Chapter - 04Kingdom Monera

Today we will discuss about bacterial reproduction and its usefulness

Fig Binary Fission

Fig Conjugation Fig Transformation

Bacterial reproduction is mainly asexual but sexual reproduction

also takes place

Asexual reproduction takes place by i) Binary fission - from one bacteriato

two bacteria are produced in every 20 to 30mins

ii) Buddingiii) Endospore formation - during

unfourable condition

Sexual reproduction by three ways

1) Conjugation - Transfer of genetic material between cells that are in physical contact with one another

2) Transduction - Transfer of genetic materialfrom one cell to another by a bacteriophage

3) Transformation - Transfer of cell-freeor naked DNArsquo from one cell to another

Bacteria causes different diseases inplants animals and human and

it causes food spoilage and waterpollution but it also have some useful

activities

i) Bacteria are helpful in sewage water treatment

ii) It is used in antibiotic (medicine) production

iii) Anaerobic bacteria help in biogas(energy) production

iv) Many household products like yoghurt cheese are manufactured by use of bacteria

v) Rhizobium by symbiotic relationship with leguminous plant increase soil fertility

vi) Besides these bacteria is helpful in genetic engineering degradation of petroleum hydrocarbonand in dairy

industry

Physics Motion in plane Here we will introduce Projectile Motion

Execution

Projectile

Y

usinθ u h

θX

ucosθ

Suppose a body is projected with an angle θ So initial velocity u can be resolved into two components

Horizontal component - ucosθ ( for range)

Vertical component - usinθ ( for height)

usinθ changes during motion and becomes zero at maximum height position but ucosθ remain unchanged

The maximum height of projectile is h

NB If initial is upward then g = -ve and if it is downward then g = +ve Height is +ve if direction of motion does not change ( for ex a body thrown upwards but goes down ultimately then height h = -ve)

The angle of projectile θ is the angle made with horizontal

HISTORY ndash GROWTH OF NATIONALISM

SUB TOPIC- REVOLUTIONARY NATIONALISM Bengal formation of Anushilan Samity and Jugantar Group

The intensification of the Swadeshi movement and Government policy of terror and repression led to outbreak of violence Bombs were manufactured and attempts on the lives of unpopular Government officials became frequent In the gymnasium of Scottish Church College which was known as General Assemblies Institution a secret society was formed known as Anushilan Samity

Aurobindo Ghosh send from Baroda his emissary Jatindranath Banerjee to mobilize the Bengal revolutionaries

Hemchandra Qanungo and Satyen Bose published Journal Jugantar

The Jugantar group planned to assassinate oppressive magistrate Kingsford by Khudiram Bose and Prafulla Chaki in 1908 Prafulla Chaki committed suicide to avoid arrest Khudiram was tried and hanged

Afew days later the police found a bomb factory in Maniktala and arrested a large number of revolutionaries The trial of revolutionaries became famous as the Alipore Bomb Case

In the course of the trial the approver the public prosecuter and a police officer were assassinated

1 Question Name two journals which preached the cult of violence

Answer a) Yugantar edited by Bhupendranath DuttaB) Bandemataram edited by Aurobindo Ghosh2 Question Why was Khudiram arrested and hangedAnswer An attempt was made to assassinate a hated vindictive majistrate named Kingsford by Khudiram Bose and Prafulla Chaki Their attempt failed and the bomb they threw killed two English ladies Khudiram was arrested and put to trial and then hanged3Question Who was Aurubindo GhoshAnswer Aurobindo Ghosh a nationalist revolutionary who was charged for his involvement in the Alipore Bombing Case He was accused of it along with his brother Barindra nath Ghosh But Aurobindo was acquitted because of the brilliant pleading of his counsel Chittaranjan Das Then he became a spiritual reformer introducing his visions on human progress and spiritual evolution4 Qustion Who was KingsfordAnswer Kingsford was an unpopular British chief Magistrate who was the target of the bomb thrown at Muzaffarpur by Khudiram and Prafulla Chaki

Most of the accused were convicted and sentenced to

long term of imprisonmentBut

Aurobindo Ghosh was acquitted mainly owing to the brilliant pleading of his counsel Chittaranjan Das

Political science Topic-Sovereignty

Summary Sovereignty is the full right and power of a governing body over itself without any interference from outside sources or bodies In political theory sovereignty is a substantive term designating supreme legitimate authority over some polity In international law sovereignty is the exercise of power by a state

Internal Sovereignty

Internal sovereignty means supreme authority within ones territory while external sovereignty relates to the recognition on the part of all states that each possesses this power in equal measure

External sovereignty

external sovereignty relates to the recognition on the part of all states that each possesses this power in equal measure

Distinguish between

Execution

Answer the following questions

Short notes-

Sovereignty

Internal Sovereignty

External sovereignty

Homework- learn

external sovereignty and internal sovereigntySovereignty is the principle

of supreme and

unquestionable authority

reflected in the claim by the

state to be the sole author of

laws within its territory

Definition of external vs internal sovereigntyInternal sovereignty refers to

the relationship between a

sovereign power and its

subjects ndash it refers to the

location of the supreme

authority within the state In

the UK for example internal

sovereignty (supposedly)

resides within Parliament

reflected in the

constitutional principle of

parliamentary

sovereigntyBy contrast

external sovereignty refers

to the capacity of the state

to act independently and

autonomously on the world

stage This is what is

sometimes called lsquostate

sovereigntyrsquo or lsquonational

sovereigntyrsquo and implies

that states are legally equal

and that the territorial

integrity and political

independence of a state is

inviolable

Class ndash XII

Date - 2742020 STUDY MATERIALSubject Topic Summary Execution Business Studies

Job Analysis amp Manpower Planning

At first let us recall the chapter what we have discussed till nowbullJob analysisbullJob specification bullJob description bullJob enlargement bullJob enrichment

Today we will do some questions answers from the chapter

Questions 1ldquoJob analysis job description and job specification are interrelatedrdquo Comment Answer) Job analysis is a systematic and detailed examination of a job to collect all the relevant information about it The contents off the job are summarised in the job description The qualification needed for the job are summarised in job specificationThus there is close interrelationship between job analysis job description and job specification

Question 2ldquoJob enlargement is a horizontal extension of a job whereas job enrichment is a vertical extension of a jobrdquo ElucidateAnswer) Job enlargement involves adding one or more task to a job coma where as job enrichment involves adding more autonomy and responsibility to a job Job enlargement is therefore horizontal extension of a job coma whereas job enrichment is a vertical extension of a job

Question 3 )

What is manpower estimation Explain its quantitative and qualitative aspectsAnswer) Manpower estimation is the process by which management determines how an organisation should move from itrsquos current manpower positionto its desired manpower position There are two dimensions of Manpower estimation- quantitative and qualitative

Quantitative aspectThis aspect of Manpower estimation involves estimating the number of employees required in a future time period Workload analysis and workforce analysis are done to estimate the quantity of required manpower

Qualitative aspectThe estimate of the knowledge skills experience etc of required manpower is the qualitative aspect of Manpower estimation The quality of Manpower can be judged on the basis of job analysisand job specification

COMMERCE

CAPITAL-FIXED AND WORKING

Today let us start the class by discussing the sources of finance for different types of business firms

The term lsquocapitalrsquo refers to the investment made in the enterprise for the purpose of earning profits

Requirements of capital and sources of capital for different types of business firms are

1 Capital for sole proprietorship businessA sole proprietor operates at a small scale and thereforerequires a limited amount of capital

2 Capital for partnership firmCapital requirements as well as capital base of a partnership is bigger than that of a sole trader businessThe owned capital is contributed by the partners in an agreed ratio

3 Capital for joint stock companyA joint stock company generally requires large amount of capitalA public company can raise huge capital through issue of shares In addition to share capital it can utilize retained profits

Now let u discuss the meaning of Finance PlanningFinance planning is the process of estimation the financial requirements of an organization specifying the sources of firms and ensuring that enough funds are available at the right time

1 What do you mean by Finance PlanningAnswer Finance planning is the process of estimation the financial requirements of an organization specifying the sources of firms and ensuring that enough funds are available at the right time

2Discuss the role of financial planning of an enterpriseThe role of financial planning are as followsa A sound financial plan helps a business enterprise to avaid the problems of shortage and surplus of fundsbFinancial planning serves as a guide in developing a sound capital structure so as to maximize returns to shareholders c It helps in effective utilization of fundsd It provides policies and procedures for coordinating different functional areas or departments of businesse It enables the management to exercise effective control over the financial activities of an enterprisef It helps the company to prepare for facing business shocks and surprises in future

Mathematics

Continuity and differentiability

Recall Definition of ContinuityLet f(x) be a single valued function of x and x=a be a point in the domain of definition of the function The function is said to be continuous at x=a ifi) f(c) is defined ie f(x) has a definite finite value at x=cii) lim xrarra f(x) exists andiii) lim xrarra f(x) =f(a) In other words f(x) is said to be continuous at x=a if lim xrarra+ f(x)= lim xrarra- f(x) = f(a) Or f(a+0) =f(a-0) =f(a) Or lim hrarr0 f(a+h)= f(a) Algebra of continuous functionsNow we will study some algebra of continuous functions Theorem 1 Suppose f and g be two real functions continuous at a real number c Then(1) f + g is continuous at x = c(2) f ndash g is continuous at x = c(3) f g is continuous at x = c(4) (fg) is continuous at x = c (provided g (c) ne 0)

Example 1 Prove that every rational function is continuousSolution Recall that every rational function f is given byf(x)=[p(x) q(x) ] q(x)ne0where p and q are polynomial functions The domain of f is all real numbers except points at which q is zero Since polynomial functions are continuous f is continuous by (4) of Theorem 1Example 2Discuss the continuity of sine functionSolution To see this we use the following factslim xrarr0 sin x =0Now observe that f (x) = sin x is defined for every real number Let c be a real number Put x = c + h If x rarr c we know that h rarr 0 Therefore lim xrarrc f(x)

= lim xrarrc sin x= lim hrarr0 sin(c+h) =lim hrarr0 [sin c cos h + cos c sin h ]=lim hrarr0 (sin c cos h) + lim hrarr0 (cos c sin h) = sin c +0=sin c = f(c) Thus lim xrarrc f(x) = f(c) and hence f is a constant function Exercise Prove that the function f(x) = x2 +2x is continuous for every real value of x [Hints show that lim xrarra+ f(x) = lim xrarra- f(x) = f(a) ]

Biology Reproduction in Flowering plants We will discuss about megasporoangium

megasporagenesis and female gametophyte

Q4 Describe the structure of megasporangium

Ovule is attached to the placenta by astalk called funicle

Each ovule has one two or three protectivecoverings called integuments

At the tip of integuments a small openingcalled micropyle is organised

Opposite to the micropylar end is the chalaza

Within the integuments a mass of cellsnucellusand inside it embryo sac orfemale gametophyte is present

Q5 Describe a mature embryosacamp its formation

In most of the flowering plants only oneof the 4 megaspores formed as a result ofmegasporogenesis that is functional while theother three degenerate

The

functional megaspore develops into thefemale gametophyte

Formation The nucleus

of the functional megasporedivides mitotically to form two nuclei first andthen two more sequential mitotic nucleardivisions result in the formation of four ampthen eight nucleate stages of embryo sac

Six of the eight nucleus are surrounded bycell walls and organised into cells

The remaining two nuclei called polar nuclei are found below the egg apparatus in the largecentral cell

Three cells consisting of two synergids amp one egg cell present bottom of

embryo sac Three cells

at the chalazal as antipodal cells

Two polar nuclei together present in large central cell

HISTORY

TOWARDS INDEPENDENCE AND PARTITION THE LAST PHASE(1935-1947)SUB TOPIC NATIONAL MOVEMENTS DURING THE SECOND WORLD WAR

Spread of Quit India Movement On 9th August 1942Gandhiji and other Congress leaders were arrested The Congress was declared illegal The news of the arrest of all leaders marked the beginning of a widespread movement of India It was not possible for such a movement to remain peacefulBut the arrest of the all notable congress leaders virtually left the movement in the hands of the mass The movement took the form of violent and militant outbreakBesides congressmen revolutionaries also were very active in the movement The Congress Socialist group also played a prominent role

1 Question Why did the British authority arrest the Congress leaders on 9 th August 1942Answer Congress Working committee adopted the Quit India resolution which was to be ratified at the Bombay AICC meeting in 8th August 1942 They decided to launch a mass struggle on non-violent lines Gandhiji gave a clarion call to all section of the people rdquoKarenge ya Marengerdquo (do or die) Congress leaders gave the call to driving out

the British from IndiaViceroy had taken strong action against the Quit India movement Gandhiji and all the leaders of Congress were arrested

2 Question How did Quit India Movement spread out all over IndiaAnswer The news of the leaders lsquo arrest marked the beginning of a widespread movement to remain peacefulThe movement took form of violent outbreak There were widespread cutting of telephone and Telegraph wires damaging railway lines raising barricades in cities and towns and other forms of violent demonstations

Question Name the leaders of Congress

Socialist group played a prominent part Notable among the Jayprakash Narayan Rammonohar Lohia Aruna Asaf Ali

Political science

Topic-Franchise and Representation

Summary

The election commission

The Election Commission of India is an autonomous constitutional authority responsible for administering Union and State election processes in India The body administers elections to the Lok Sabha Rajya Sabha State Legislative Assemblies in India and the offices of the President and Vice President in the country

Functions of election commission-

India is a sovereign socialist secular democratic republic Democracy runs like a golden thread in the social economic and political fabric woven by the Constitution given by lsquoWe the People of Indiarsquo unto ourselves The concept of democracy as visualised by the Constitution pre-supposes the representation of the people in Parliament and State legislatures by the method of election The Supreme Court has held that democracy is one of the inalienable basic features of the Constitution of India and forms part of its basic structure The Constitution of India adopted a Parliamentary form of government Parliament consists of the President of India and the two Houses mdash Rajya Sabha and Lok Sabha India being a Union of states has separate state legislatures for each state State legislatures consist of the Governor and two Houses mdash Legislative Council and Legislative Assembly mdash in seven states namely Andhra Pradesh Telangana Bihar Jammu amp Kashmir Karnataka Maharashtra and Uttar Pradesh and of the Governor and the state Legislative Assembly in the remaining 22 states Apart from the above two out of the seven Union Territories namely National Capital Territory of Delhi and Puducherry also have their Legislative Assemblies

ExecutionShort notes-Election commissionFunctions of election commission

Homework- Learn

Computer

Science

Computer hardware NAND Gate

A NOT-AND operation is known as NAND operation It has n input (n gt= 2) and one output

Logic diagram

Truth Table

NOR Gate

A NOT-OR operation is known as NOR operation It has n input (n gt= 2) and one output

Logic diagram

Truth Table

XOR Gate

XOR or Ex-OR gate is a special type of gate It can be used in the half

adder full adder and subtractor The exclusive-OR gate is abbreviated as EX-OR gate or sometime as X-OR gate It has n input (n gt= 2) and one output

Logic diagram

Truth Table

XNOR Gate

XNOR gate is a special type of gate It can be used in the half adder full adder and subtractor The exclusive-NOR gate is abbreviated as EX-NOR gate or sometime as X-NOR gate It has n input (n gt= 2) and one output

Logic diagram

Truth Table

Physics

Chapter 1 Electric Field ( Electric Dipole) (Summary)

Here we will derive Expression of electric field at broad side

On position of dipole

Execution

Q With the help of a labelled diagram obtain an expression for the electric field intensity E at any point on the equitorial line ( broad-side on position) of an electric dipole

Ans

E1 E1sinθ

E θ P E1 θ

( r2+L2)12 E2 E

r E2 E2sinθ

-q θ L O L +qA B

Let us consider that the point P is situated on the right bisector of the dipole AB at a distance r meter from its midpoint O

Let E1 and E2 be the electric field intensities of the electric field at P due to charge +q and ndashq of the dipole resp The distance of P from each charge is ( r2+L2)12

So E1 = 14 πϵ q

(r 2+L 2) away from +q

E2 = 14 πϵ q

(r 2+L 2) towards ndashq

The magnitudes of E1 and E2 are equal but directions are different Now resolving E1 and E2 into two components parallel and perpendicular to AB we get

The components perpendicular to AB E1sinθ and E2sinθ cancel each other because they are equal and opposite

The components parallel to AB are E1cosθ and E2 cosθ are in same direction and add up

So resultant intensity of electric field at the point P is

E = E1cosθ + E2 cosθ

E = 14 πϵ q

(r 2+L 2) 2 cosθ

Now from fig we have cosθ =BOBP = L (r2+L2)12

So we get E = 14 πϵ 2qL ( r2+L2)32

Now electric dipole moment p= 2qL

So E = 14 πϵ p ( r2+L2)32

HW Find the expression of Electric field as done here but this time take r gtgt 2L

Also find the expression of torque experience by a dipole

(Hint Electric force experienced by charges of dipole in electric field is qE each Let θ be the angle which dipole makes with electric lines of force then perpendicular distance between two charges is 2Lsinθ Then torque = force x perp distance = qE x 2L sinθ So τ=pE sinθ where p =2qL )

STUDY MATERIAL

Class XIISubject Eng Literature (The Tempest ndash William Shakespeare) Topic Act IV Scene 1 Lines 84 to 133 (Iris hellip A contract of true love Be not too late ) Date 27th April 2020 (4th Period)

[Students should read the original play and also the paraphrase given in the school prescribed textbook]Summary Questions amp Answers

o Ceres soon appears and comes to know that she has been summoned to celebrate the contract of true love

o Ceres expresses her unwillingness to meet Venus and Cupid as she has shunned their company

o Ceres and Juno both bestow their blessings upon Ferdinand and Miranda with June gifting honour riches happiness in marriage and Ceres presents plenty of earthrsquos produce

o Iris summons the water-nymphs and reapers to come and celebrate a contract

(1) IRIS Of her society (Line 91-101)

Be not afraid I met her deity

Cutting the clouds towards Pathos and her sonDove-drawn with her Here thought they to have doneSome wanton charm upon this man and maidWhose vows are that no bed-right shall be paidTill Hymens torch be lightedmdashbut in vainMarss hot minion is returned againHer waspish-headed son has broke his arrowsSwears he will shoot no more but play with sparrowsAnd be a boy right out

(i) Where were Venus and Cupid seen flying How were they travelling Why did they want to join the marriage celebration of Ferdinand and Miranda

of true love

Venus and Cupid were seen flying through the air towards Paphos the famous city which is situated on the island of Cyprus They were travelling by air-borne chariot drawn by doves They certainly wanted to come here in order to play some amorous trick upon Ferdinand and Miranda who are under a vow not to gratify their physical desires till the holy ceremony of their marriage has been performed(ii) What have Venus and Cupid done after failing in their plan

After being failure of their plan Venus who is a very passionate deity and who is the mistress of Mars (the god of war) has gone back while here ill-tempered son Cupid has broken his arrows of love in his state of desperation(iii) What has Cupid firmly decided

Cupid is feeling so disappointed that he has firmly decided to shoot no more arrows to arouse love in human hearts but to spend his time playing with sparrows Thus he would now become just a boy and would give up his original function of shooting arrows on human beings to make them fall in love(iv) What vow had Ceres taken How did Ceres feel at the abduction

After the abduction of her daughter Prosperina by Pluto Ceres had taken a vow to always keep away from the disgraceful company of Venus and her blind son Cupid the god of love Ceres felt deeply distressed when Pluto had carried off her daughter and had made her his wife by force(v) Why has Ceres not forgiven Venus and her blind son For what do Ceres want to be sure

As the abduction had been manipulated by Venus the goddess of beauty and love and her blind son Cupid Ceres has never forgiven them for their part in the whole plot Ceres wants to be sure that she would not have to meet Venus and Cupid who had engineered the abduction of her daughter Prosperina

AS THIS lsquoMASQUErsquo SCENE IS VERY IMPORTANT IN THE PLAY THE PARAPHRASE OF THE ENTIRE PORTION OF MASQUE SCENE (Act IV Lines 58 to 143) IS GIVEN BELOW

IRIS Goddess of RainbowCERES Goddess of Agriculture and all the fruits of the earth

(Nature growth prosperity rebirth ndash notions intimately connected to marriage)JUNO The majestic Queen of Heavens and wife of Jupiter (Jupiter is the king of Gods)

VENUS The Goddess of love CUPID Son of Venus PLUTO God of death (In the play referred by Shakespeare as lsquoDisrsquo which is a Roman name for Pluto)

ORIGINAL TEXT PARAPHRASEPROSPEROWellmdash

PROSPERONow come Ariel Let there be too many rather than too few

Now come my Ariel Bring a corollaryRather than want a spirit Appear and pertly[to Ferdinand and Miranda]No tongue all eyes Be silent

spirits in attendance Appear briskly

[to Ferdinand and Miranda]Look with your eyes but do not say a word

[Soft music] [Soft music][Enter Iris] [Enter Iris]

IRISCeres most bounteous lady thy rich leasOf wheat rye barley vetches oats and peasThy turfy mountains where live nibbling sheepAnd flat meads thatched with stover them to keepThy banks with pioned and twilled brimsWhich spongy April at thy hest betrimsTo make cold nymphs chaste crowns and thybroom-grovesWhose shadow the dismissegraved bachelor lovesBeing lass-lorn thy pole clipped vineyardAnd thy sea-marge sterile and rocky-hardWhere thou thyself dost airmdashthe Queen othrsquoSkyWhose watery arch and messenger am IBids thee leave these and with her sovereign grace[Juno appears] Here on this grass-plot in this very placeTo come and sport Her peacocks fly amainApproach rich Ceres her to entertain

IRISCeres most generous lady you are the cause of rich fields or fertile land where wheat rye barley beans oats and peas grow the grassy mountains where the sheep graze and the flat meadows covered with coarse hay to be used as fodder for cattleYour banks are covered with marsh-marigolds and reeds and the rainy April under your orders brings forth to make for the maids who are not in love beautiful crowns your woods where the broom flourishes and where the bachelor who has been dismissed by the maid he loved lies down being forsaken your vineyard in which the poles are embraced by the vines and the margin of the sea which is barren and rocky where you roam about to enjoy the fresh air ndash the queen of the sky (Juno) whose messenger I am besides being represented as the rainbow bids you leave all these and with her majesty here on this grassy plot in this very place come and sport her peacocks carry her fast in her chariot through the air and are making their way here approach rich Ceres to welcome her

[Enter Ariel as Ceres] [Enter Ariel as Ceres]

CERESHail many-coloured messenger that neerDost disobey the wife of JupiterWho with thy saffron wings upon my flowersDiffusest honey-drops refreshing showersAnd with each end of thy blue bow dost crownMy bosky acres and my unshrubbed downRich scarf to my proud earth Why hath thy queenSummoned me hither to this short-grassed green

CERESWelcome rainbow that never dared disobey Juno the wife of Jupiter who with your orange coloured rays spread honey-drops refreshing showers And with each end of thy blue bow drown my bushy acres and my hilly country which is free from shrubs you thus forming a rich scarf Why has your queen called me here to this place covered with short grass

IRISA contract of true love to celebrateAnd some donation freely to estateOn the blest lovers

IRISI have called you to celebrate a contract of true love and bestow some liberal gift upon the blessed lovers

ORIGINAL TEXT PARAPHRASECERESTell me heavenly bowIf Venus or her son as thou dost knowDo now attend the queen Since they did plotThe means that dusky Dis my daughter gotHer and her blind boys scandaled companyI have forsworn

CERESTell me heavenly bow if Venus the Goddess of love or Cupid her son and pedlar of passion at this time attend the heavenly queen Juno because you are sure to know Since the day they conspired against me and dark Pluto took away my daughter here and Cupidrsquos disgraceful company I have left off

IRISOf her societyBe not afraid I met her deityCutting the clouds towards Pathos and her sonDove-drawn with her Here thought they to have doneSome wanton charm upon this man and miad

IRISBe not afraid of her company I met her deity moving on the clouds towards Paphos the sacred home of Venus on the island of Cyprus along with her son on her chariot drawn by doves Here they contemplated to exercise a charm upon this man and maid producing

Whose vows are that no bed-right shall be paidTill Hymens torch be lightedmdashbut in vainMarss hot minion is returned againHer waspish-headed son has broke his arrowsSwears he will shoot no more but play with sparrowsAnd be a boy right out

wantonness before the actual marriage ceremony but did not succeed Venus has returned her irritable son has broken his arrows and swears that he will give up his practice of trying to inspire love but play with sparrows and be a boy again

[Music is heard] [Music is heard]

CERESHighst queen of stateGreat Juno comes I know her by her gait

CERESHighest queen of state Great Juno there she comes I know here by her gait

[Enter Juno] [Enter Juno]

JUNOHow does my bounteous sister Go with meTo bless this twain that they may Prosperous beAnd honoured in their issue

JUNOHow are you doing my generous sister Come with me to bless this couple so that they may be prosperous and fortunate in their children

[They sing] [They sing]

JUNOHonour riches marriage-blessingLong continuance and increasingHourly joys be still upon youJuno sings her blessings upon you

JUNOMay honour riches happiness in marriage long continuance and increase of those boons ever rest upon you as hourly joys Juno showers down upon you her blessings in song

CERESEarths increase foison plentyBarns and garners never emptyVines and clustring bunches growingPlants and goodly burden bowingSpring come to you at the farthestIn the very end of harvestScarcity and want shall shun youCeresrsquo blessing so is on you

CERESMay you have the plenty of earthrsquos produce Your barns and granaries may never be empty Your vines may grow with clustering bunches Your fruit trees may be heavily laden with their fruit May there be continuous spring and harvest May scantiness and want leave you forever Such is the blessing of Ceres upon you

FERDINANDThis is a most majestic vision andHarmoniously charmingly May I be boldTo think these spirits

FERDINANDThis is a great vision and magically melodious Should I suppose the characters (taking part in the masque) are spirits

PROSPEROSpirits which by mine artI have from their confines calld to enactMy present fancies

PROSPEROYes they are spirits whom I have summoned from the regions to which they are confined to carry into effect my fanciful designs

ORIGINAL TEXT PARAPHRASEFERDINANDLet me live here everSo rare a wondered father and a wifeMakes this place paradise

FERDINANDI should like to live here forever Such a wise and wonderful father makes this place a paradise

[Juno and Ceres whisper and send Iris on employment] [Juno and Ceres whisper and send Iris on employment]

PROSPEROSweet now silence

PROSPEROMy dear Ferdinand speak no more Juno and Ceres are

Juno and Ceres whisper seriouslyTheres something else to do Hush and be muteOr else our spell is marred

whispering with a solemn look There is something else coming Silence Or else our magic will be spoilt

IRISYour nymphs called naiads of the wandering brooksWith your sedged crowns and over-harmless looksLeave your crisp channels and on this green landAnswer your summons Juno does commandCome temperate nymphs and help to celebrateA contract of true love Be not too late

IRISYou nymphs called Naiads denizens (M inhabitants) of the running stream with your chaplets of sedge and ever-helpful looks leave your wrinkled channels and on the green land answer the summons sent to you Juno has ordered some chaste nymphs and help to celebrate a noble and true marriage Donrsquot delay

[Enter certain nymphs] [Enter certain nymphs]You sunburnt sicklemen of August wearyCome hither from the furrow and be merryMake holiday your rye-straw hats put onAnd these fresh nymphs encounter every oneIn country footing

You sunburnt harvesters weary from the effects of the heat in August come here from the furrowed land and rejoice Make holiday with your rye-straw hats upon you and meet these fresh nymphs and join in country dancing

[Enter certain reapers properly habited They join with the nymphs in a graceful dance towards the end whereof Prospero starts suddenly and speaks]

[Enter certain reapers properly habited They join with the nymphs in a graceful dance towards the end whereof Prospero starts suddenly and speaks]

PROSPERO[aside] I had forgot that foul conspiracyOf the beast Caliban and his confederatesAgainst my life The minute of their plotIs almost come [to the spirits]Well done Avoidno more

PROSPERO(Aside)I had forgotten the wicked conspiracy of the beast Caliban and his accomplices against my life the time of their plot has almost arrived ndash (To the Spirits) well done depart no more of this

[To a strange hollow and confused noise the spirits heavily vanish]

[The spirits depart]

ORIGINAL TEXT PARAPHRASEFERDINANDLet me live here everSo rare a wondered father and a wifeMakes this place paradise

FERDINANDI should like to live here forever Such a wise and wonderful father makes this place a paradise

[Juno and Ceres whisper and send Iris on employment] [Juno and Ceres whisper and send Iris on employment]

PROSPEROSweet now silenceJuno and Ceres whisper seriouslyTheres something else to do Hush and be muteOr else our spell is marred

PROSPEROMy dear Ferdinand speak no more Juno and Ceres are whispering with a solemn look There is something else coming Silence Or else our magic will be spoilt

IRISYour nymphs called naiads of the wandering brooksWith your sedged crowns and over-harmless looksLeave your crisp channels and on this green landAnswer your summons Juno does commandCome temperate nymphs and help to celebrateA contract of true love Be not too late

IRISYou nymphs called Naiads denizens (M inhabitants) of the running stream with your chaplets of sedge and ever-helpful looks leave your wrinkled channels and on the green land answer the summons sent to you Juno has ordered some chaste nymphs and help to celebrate a noble and true marriage Donrsquot delay

[Enter certain nymphs] [Enter certain nymphs]You sunburnt sicklemen of August wearyCome hither from the furrow and be merryMake holiday your rye-straw hats put onAnd these fresh nymphs encounter every oneIn country footing

You sunburnt harvesters weary from the effects of the heat in August come here from the furrowed land and rejoice Make holiday with your rye-straw hats upon you and meet these fresh nymphs and join in country dancing

[Enter certain reapers properly habited They join with the nymphs in a graceful dance towards the end whereof Prospero starts suddenly and speaks]

[Enter certain reapers properly habited They join with the nymphs in a graceful dance towards the end whereof Prospero starts suddenly and speaks]

PROSPERO[aside] I had forgot that foul conspiracyOf the beast Caliban and his confederatesAgainst my life The minute of their plotIs almost come [to the spirits]Well done Avoidno more

PROSPERO(Aside)I had forgotten the wicked conspiracy of the beast Caliban and his accomplices against my life the time of their plot has almost arrived ndash (To the Spirits) well done depart no more of this

[To a strange hollow and confused noise the spirits heavily vanish]

[The spirits depart]

Ac-12 27420 topic Revaluation of Assets and Liabilities

REVALUATION OF ASSETS AND LIABILITIES

On admission of a new partner the firm stands reconstituted and consequently the assets are revalued and liabilities are reassessed It is necessary to show the true position of the firm at the time of admission of a new partner If the values of the assets are raised gain will increase the capital of the existing partners Similarly any decrease in the value of assets ie loss will decrease the capital of the existing partners For this purpose alsquoRevaluation Accountrsquo is prepared This account is credited with all increases in the value of assets and decrease in the value of liabilities It is debited with decrease on account of value of assets and increase in the value of liabilities The balance of this account shows a gain or loss on revaluation which is transferred to the existing partnerrsquos capital account in existing profit sharing ratioAccounting for Revaluation of Assets and Liabilities when there is a Changein the Profit Sharing Ratio of Existing PartnersAssets and liabilities of a firm must also be revalued at the time of change in profit sharing ratio of existing partners The reason is that the realisable or actual value of assets and liabilities may be different from those shown in the Balance Sheet It is possible that with the passage of time some of the assets might have appreciated in value while the value of certain other assets might have decreased and no record has been made of such changes in the books of accounts Similarly there may be some unrecorded assets amp libilities that may have to be accounted for Revaluation of assets and reassessments of liabilities becomes necessary because the change in the

value of assets and liabilities belongs to the period to change in profit sharing ratio and hence must be shared by the partners in their old profit sharing ratio Revaluation of assets and reassessment of liabilities may be given effect to in two different ways (a) When revised values are to be recorded in the books and(b) When revised values are not to be recorded in the books

When revised values are to be recorded in the booksIn such a case revaluation of assets and reassessment of liabilities is done with the help of a new account called lsquoRevaluation Accountrsquo Sometimes this account is also called as lsquoProfit amp Loss Adjustment Acrsquo If there is a loss due to revaluation revaluation account is debited and if the revaluation results in a profit the revaluation account is credited The following journal entries made for this purpose are

(i) For increase in the value of assetsAsset Ac Dr (individually)To Revaluation Ac(ii) For decrease in the value of AssetRevaluation Ac Dr (individually)To Asset Ac[Decrease in the value of assets](iii) For increase in the value of LiabilitiesRevaluation Ac Dr (individually)To Liabilities Ac[Increase in the value of Liabilities](iv) For decrease in the value of LiabilitiesLiabilities Ac DrTo Revaluation Ac[Decrease in the value of Liabilities](v) For unrecorded AssetsAsset Ac [unrecorded] DrTo Revaluation Ac[Unrecorded asset recorded at actual value](vi) For unrecorded Liability Revaluation Ac DrTo Liability Ac [unrecorded][Unrecorded Liability recorded at actual value](vii) For transfer of gain on revaluationRevaluation Ac DrTo Existing Partnerrsquos CapitalCurrent Ac[Profit on revaluation transferred to capital account in existing ratio](viii) For transfer of loss on revaluationExisting Partnerrsquos CapitalCurrent Ac DrTo Revaluation Ac[Loss on revaluation transferred to capital account in existing ratio](a) When revaluation account shows gain Revaluation Ac DrTo Partnerrsquos Capital Ac (Old Profit Sharing Ratio)(Profit on revaluation credited to Partnerrsquos Capital Ac)(b) Above entry is reversed when revaluation account shows loss Partners Capital Acs (Old Profit Sharing Ratio) DrTo Revaluation Ac(Loss on revaluation debited to Partnerrsquos Capital Acs)

Proforma of Revaluation Account is given as under

Revaluation Account

Dr Cr Particulars ` Amount Particulars ` Amount To Decrease in value of assets By Increase in value of assets To Increase in value of liabilities By Decrease in value of liabilities To Unrecorded liabilities By Unrecorded assets To Gain on Revaluation (Transferred) By Loss on Revalution (Transferred)

ECO ndash12 2742020Topic- ELASTICITY OF DEMAND

CHAPTER - ELASTICITY OF DEMANDMEANINGDemand for a commodity is affected by many factors such as its price price of related goods income of its buyer tastes and preferences etc Elasticity means degree of response Elasticity of demand means degree of responsiveness of demand Demand for a commodity responds to change in price price of related goods income etc So we have three dimensions of elasticity of demandDIMENSION OF ELASTICITY OF DEMAND TYPES OF ELASTICITY OF DEMAND

Price elasticity of demand Income elasticity of demand Cross Elasticity of demand

Price elasticity of demand Price elasticity of demand means degree of responsiveness of demand for a commodity to the change in its price For example if demand for a commodity rises by 10 due to 5 fall in its price Price elasticity of demand (ep)=Percentage change in quantity demanded Percentage change in price of the commodity = 10 ( -)5 = ( - )2Note that ep will always be negative due to inverse relationship of price and quantity demanded

(ii) Income elasticity of demand Income elasticity of demand refers to the degree of responsiveness of demand for a commodity to the change in income of its buyer Suppose income of buyer rises by 10 and his demand for a commodity rises by 20 then Income elasticity of demand (ey)= change in quantity demanded change in price of the commodity =20 10 = 2

Cross Elasticity of demandCross elasticity of demand means the degree of responsiveness of demand for a commodity to the change in price of its related goods (substitute goods or complementary goods) Suppose demand for a commodity rises by 10 due to 5 rise in price of its substitute good then Cross elasticity of demand (ec) = change in quantity demanded change in price of related good = 10 2 = 5 (Tastes and preferences cannot be expressed numerically So elasticity ofdemand cannot be numerically expressed)

  • Chapter 1 Force (Summary)
  • Distinguish between external sovereignty and internal sovereignty
    • NAND Gate
      • Logic diagram
      • Truth Table
        • NOR Gate
          • Logic diagram
          • Truth Table
            • XOR Gate
              • Logic diagram
              • Truth Table
                • XNOR Gate
                  • Logic diagram
                  • Truth Table
                      • Physics
                      • Chapter 1 Electric Field ( Electric Dipole) (Summary)
Page 8:  · Web viewWe all know that Nouns are divided into two parts: common noun and proper noun.Apart from common and proper noun, we will also study about collective noun and compound

MAP ndash Representation of the Earth as a whole or parts of it drawn on a flat surface to a scale

SKETCH ndash A rough drawing of a place not drawn to a scale

PLAN ndash A plan is prepared to show more details about a small area

COMPUTER - VI CHAPTER-6 (INTRODUCTION TO WINDOWS 10)

THE MOST RECENT VERSION OF WINDOWS IS WINDOWS 10 IT WAS RELEASED IN 2015

WINDOWS 10 IS DESIGNED TO WORK FOR YOU AND NOT YOU WORKING FOR IT THE WINDOWS 10 OPERATING SYSTEM INCLUDES A NUMBER OF NEW FEATURES LIKE IMPROVED PERFORMANCE ON MULTI CORE PROCESSORS IMPROVED G U I DATA SECURITY AND BETTER BOOTING PERFORMANCE

WINDOWS 10 HAS 1187 WITH WEARING FEATURES SETS AND INTENDED HARDWARE

THE VARIOUS EDITIONS AVAILABLE FOR WINDOWS 10 ARE AS FOLLOWS

WINDOWS 10 HOME

WINDOWS 10 HOME IS THE EDITION OF WINDOWS 10 THAT CONTAINS THE LEAST NUMBER OF FEATURESWINDOWS 10 HOME IS THE CONSUMER FOCUSED DESKTOP EDITION WITH SUPPORT FOR BOTH PC AND TOUCH ENABLED TABLETS WINDOWS 10 HOME IS AVAILABLE IN BOTH 32 BIT AND 64 BIT VERSIONS

WINDOWS 10 PRO

WINDOWS 10 PRO HAS FEATURES ESSENTIAL FOR SMALL OR MEDIUM BUSINESSESIT HAS A LARGE NUMBER OF EXTRA FEATURES TO MEET THE NEEDS OF SMALL BUSINESSES

WINDOWS 10 ENTERPRISE

WINDOWS 10 ENTERPRISE IS BUILT ON WINDOWS 10 PROIT HAS FEATURES WHICH ARE DESIGNED TO MEET THE NEEDS OF MEDIUM AND LARGE ORGANISATIONSTHIS WINDOWS EDITION TARGETS THE ENTERPRISE SEGMENT OF THE MARKET

WINDOWS 10 MOBILE

WINDOWS 10 MOBILE IS DESIGNED FOR CUSTOMER ORGANISATION THAT ARE USING THE WINDOWS 10 PLATFORM ON SMARTPHONES AND SMALL TABLETS

WINDOWS 10 EDUCATION

WINDOWS 10 EDUCATION IS DESIGNED TO MEET THE NEEDS OF STAFF ADMINISTRATORS TEACHERS AND STUDENTS OF SCHOOLS

WINDOWS 10T EDITION

WINDOWS 10T EDITION OF WINDOWS 10 IS DESIGNED FOR USE IN SMALL FOOTPRINT AND LOW COST DEVICES

Subject English language

Topic Noun kinds

NOUN KINDS

Noun is a naming word Nouns are names of people places animals or things

For example Ashley Mumbai tiger pencils

Kinds of nouns

Proper noun the name of a particular person place or thing is called proper nounExample

1 The Himalayas stand to the north of India2 Monika is the school captain

Common noun announce that names people place or thing in general is called a common noun Example

1 Kalidas was the greatest dramatist of India2 William Shakespeare is Englands national poet

Collective Noun A collective noun is the name of a collection of people or things taken together and spoken of as a whole Example

1 The feet were completely destroyed in the fierce attack2 The crew revolted against Captain Grand

Abstract Noun An abstract noun is the name of some quality state or idea Example1 Diversity Indias biggest strength2 Patience is a virtue

_____________________________________________________________________________________

Class VII

Subject English 1 Topic Articles

ARTICLES

There are three articles ------ A An The

1 Indefinite articles- The Articles a and an are called Indefinite Articles They do not point to a particular person or thing They are used with singular countable nouns It does not identify a specific noun

Uses of Indefinite articles A

Before a singular noun beginning with a consonant or a vowel with a consonant sound For example a cattle a university a one way track

Before a proper noun which is either unfamiliar or holds a special meaning For example For example A Mahesh Sharma wants to see youNeela wants to be a Tendulkar when she grows up

With a number beginning with a consonant sound For example a ten- rupee note a hundred years

Before half when half follows a whole number or after half when it isnrsquotExample one and a half litres (before) Half a litre (after)

With an expression of quantity For example a lot of time a dozen oranges a little while In exclamation before nouns Example Such a mess What a pity

Uses of Indefinite articles An

Before a singular noun beginning with a vowel sound Example an ant an egg an umbrella Before an abbreviation beginning with a vowel or a consonant with the vowel sound

Example an MP an MA Before the word beginning with a silent h Example an heir an hour an honour

Note We do not say a milk or a lemonade because they are uncountable nouns ever we see a cup of milk or a glass of lemon and

Subject GEOGRAPHY

CHAPTER 1 REPRESENTATION OF GEOGRAPHICAL FEATURES

TOPIC- TOPOGRAPHICAL MAPS

Topographical Maps- topographical maps are small-scale maps with detailed depiction of both natural and human-made features

Conventional colours-different colours used in a map are known as conventional colours

TYPES OF SCALE-

Verbal Statement- Scale written in a statement from like 2 cm to a Km is called verbal statement It means that 2cm on the map represents 1 km on the Earth

Representative Fraction- Scale can also be written as a representative fraction (RF)

Example 150000 it means 1cm on the map represents 50000cm or frac12 km on the ground It shows the ratio between map distance and ground distance

Linear Scale Scale can be drawn on a line to show map distance equivalent to ground distance A line is divided into equal parts Each parts represents the actual distance on the ground in mkm

SUBJECT-COMPUTER

CHAPTER-1 (COMPUTER FUNDAMENTALS)

COMPUTER LANGUAGES

THE THE TERM COMPUTER LANGUAGE REFERS TO A SYSTEM OF RULES AND SYMBOLS THAT ARE DESIGNED TO GIVE INSTRUCTIONS TO A COMPUTERTHE COMPONENTS OF A COMPUTER SYSTEM CANNOT PERFORM A TASK BY THEMSELVES THEREFORE THEY HAVE TO BE GIVEN INSTRUCTIONS TO PERFORM ANY TASKCOMPUTER LANGUAGES ARE USED TO CREATE PROGRAMS USING LOGIC BASED ALGORITHMSTHE RULES OF A COMPUTER LANGUAGE IS

KNOWN AS SYNTAX WHEREAS THE TERM SEMANTICS REFERS TO THE MEANING OF LANGUAGESCOMPUTER CANNOT DEVELOP ANY PROGRAMMING LANGUAGE BY THEMSELVES AND CANNOT THINK INTELLIGENTLY UNLESS THEY ARE ASSISTED BY THE HUMAN BEINGS THEREFORE THE PROGRAMMERS DEVELOP A SET OF METHODS AND TECHNIQUES A SET OF INSTRUCTIONS USED TO PERFORM A SPECIFIC TASK IS CALLED A PROGRAM

A PROGRAMMING LANGUAGE IS ALSO KNOWN AS A COMPUTER LANGUAGE CODED BY PROGRAMMERS TO WRITE INSTRUCTIONS FOR A COMPUTERTHE COMPUTER TAKES THESE INSTRUCTIONS AS INPUT AND PRODUCES THE DESIRED OUTPUT

TYPES OF COMPUTER LANGUAGE

THE COMPUTER LANGUAGE CAN BE BROADLY DIVIDED INTO TWO CATEGORIES

LOW LEVEL LANGUAGES HIGH LEVEL LANGUAGES

LOW LEVEL LANGUAGES

A LOW LEVEL PROGRAMMING LANGUAGE REFERS TO THE LANGUAGE THAT IS UNDERSTOOD BY A COMPUTER DIRECTLYTHE PROGRAMMER MUST HAVE AN IN-DEPTH KNOWLEDGE OF DIFFERENT COMPUTERS TO WRITE PROGRAMS IN A LOW LEVEL LANGUAGE THE TWO TYPES OF LOW LEVEL LANGUAGES ARE MACHINE LANGUAGE AND ASSEMBLY LANGUAGE

MACHINE LANGUAGE

COMPUTER HARDWARE UNDERSTANDS ONLY MACHINE LANGUAGE AS IT IS THE FUNDAMENTAL LANGUAGE OF A COMPUTER WHICH UNDERSTANDS ONLY THE TWO DIGITS 0 AND 1 KNOWN AS THE BINARY DIGITSIT IS A SYSTEM OF INSTRUCTIONS EXECUTED DIRECTLY BY THE CPU WITHOUT ANY TRANSLATION AND HENCE IS EXECUTED QUICKLY AS COMPARED TO PROGRAMS IN OTHER LANGUAGES PROGRAMS IN MACHINE LANGUAGE NEED DIFFERENT BINARY LANGUAGE PROGRAMS TO SOLVE THE COMPLEX TASKSPROGRAMMERS OFTEN FIND IT DIFFICULT TO WRITE PROGRAMS IN THIS LANGUAGE

ASSEMBLY LANGUAGE

AN ASSEMBLY LANGUAGE IS A LOW LEVEL PROGRAMMING LANGUAGE BUT THE INSTRUCTIONS ARE WRITTEN IN THE FORM OF WORDS KNOWN AS MNEMONICS TO CONVERT AN ASSEMBLY LANGUAGE INTO TO MACHINE CODE WE REQUIRE A UTILITY PROGRAM WHICH IS KNOWN AS AN ASSEMBLER ASSEMBLY LANGUAGE PROGRAMS CANNOT BE EXECUTED DIRECTLY BY A COMPUTER THEREFORE THEY ARE SLOWER THAN THE MACHINE LANGUAGE PROGRAMSIT IS EASIER TO WRITE PROGRAMS IN THE MACHINE LANGUAGE THEN AND IN ASSEMBLY LANGUAGE

_______________________________________________________________________________________________

CLASS-VIII

SUBJECT-COMPUTER CHAPTER - Operating system and graphical user interface Role and functions

Need of operating system

It co-ordinates different hardware and software componentsof a computer system It supervises the various actions of the computer system and enables the computer to work in a effective

manner It helps in smooth functioning of various peripherals

In a multi-tasking operating system it determines the order and time to be allowed for each application before giving another application a turn

It sends messages to the system operator about the status of operation or any error that may have occurred while running the application

What is operating system

An operating system is an integrated system of programs that manages various resources and the overall operation of the computer system It is designed to support various activities of computer system in a systematic way

Role of an operating system-

Operating system enables the user to use the system effectively An operating system manages various application that runs on a computer and shares computerrsquos resources User interacts with operating system through command line interface and graphical user interface

Function of an operating system -

Booting the computer it is the start up procedure of a computer system Loading theprograms in the memory - when system is ready the operating system loads certain program

automatically Manages resources it manages between the hardware and software resources Detecting and correcting errors- if the supporting hardware or software doesnrsquot works properly then the

operatingsystem tries to rectify it Ensuring data security programs and data donot interfere with each other Maintaining the internal clock of system - maintain internal clock of system when system is shut down

SUBJECT-GEOGRAPHY CHAPTER 1 REPRESENTATION OF GEOGRAPHICAL FEATURES THROUGH CONTOURSTopographical maps- topographical maps are small-scale maps with detailed depiction of both natural and human-made features

Contour lines- contour lines are imaginary lines drawn on a map joining places having the same height above the mean sea level

Contour linesContour interval- It is an interval at which contour lines are drawnIndex contour- At every 100-m interval a thick brown contour line is called index contour It is drawn for calculating height

Spot heights- The spot heights show heights in metres above the mean sea level

SUBJECT- English Language CHAPTER - The Sentence

Complex sentence-

We have seen that a Complex Sentence consists of a Principal Clause with one or more Subordinate Clauses

We have also learnt that there are three kinds of Subordinate Clauses The Adjective Clause the Adverb Clause and the Noun Clause

1) THE ADJECTIVE CLAUSE

An ADJECTIVE CLAUSE does the work of an ADJECTIVE It qualifies a noun or pronoun In the following examples in each set two commonly patterned sentences are compared one with an ADJECTIVE and the other with an ADJECTIVE CLAUSE ( The Adjective or Adjective Clause qualifies the Noun placed in the box)

a) He is a lazy boy (Adjective)

b) He is a boy who is lazy (Adjective Clause)

a) It is a beautiful place for the tourists (Adjective)

b) It is a place where tourists come for the scenic beauty (Adjective Clause)

a) We have enough funds for the work (Adjective)

b) We have funds which would be enough for the work (Adjective Clause)

Convert the sentence from simple to complex sentence-

1) We believe his honesty Ans- We believe that he is honest

2) This is the birth place of RamaAns- This is the place where Rama was born

____________________________________________________________________________________________

Class IX

Subject English Language

Topic Preposition

Date 270420

PREPOSITIONS

A preposition is a word placed before a noun or a pronoun to show in what relation the person or thing denoted by it stands in regard to something else

EXAMPLE

There is a clock on the wall

The preposition on shoes relationship between clock and the wall

Smitha is afraid of lizards The man jumped off the bus

Here the preposition of shows the relationship between afraid and lizards

The preposition off shows the relation between jumped and bus

The noun or the pronoun which follows a preposition is called its object

So in the first Example wall is the object of the preposition on

A preposition can have more than one object For example

The plane flew overhouses and meadows

Prepositions are used to express a number of relationship including time locationmannermeans quantity purpose and state or condition

Points to remember

Preposition joins a noun to another noun or a pronounThere is a cow in the field

A preposition joints a noun to a verbThe cat runs after the rat

Preposition can have two or more than two objectsThe road runs over hills and plain

A preposition also joins a noun an adjectiveHe is fond of tea

Generally a preposition comes before an object Sometimes it comes even after an object asWhat are you looking atThis is the house I live in

Subject- Computer Application

Chapter 2 Introduction to Java

Java API An application programming interface (API) in the context of Java is a collection of prewritten packages classes and interfaces with their respective methods fields and constructors

Byte Code Java bytecode is the result of the compilation of a Java program an intermediate representation of that program which is machine independent The Java bytecode gets processed by the Java virtual machine (JVM) instead of the processor JVM The Java Virtual Machine (JVM) is the runtime engine of the Java Platform which allows any program written in Java or other language compiled into Java bytecode to run on any computer that has a native JVM

Platform A platform is the hardware or software environment in which a program remains

Java platform The Java platform differs from most other platforms in that itrsquos a software only platform that runs on top of other hardware-based platforms

Applet and Application The fundamental difference between the two Java programs is that an application program is designed to run on a stand-alone machine whereas an applet is a web-version of an application which is used to run a program on a web browser

WORA ldquoWrite once run anywhererdquo (WORA) or sometimes write once run everywhere (WORE) is a slogan created by Sun Microsystems to illustrate the cross-platform benefits of the Java language

Class XSubject Topic Summary Execution

ECONOMICSFACTORS OF PRODUCTION Sub-topiclsquoLABOURrsquo

We shall start our class by discussing the topic taught in the last class

lsquo DIVISION OF LABOURrsquo- By division of labour we mean specialization in workIt refers to splitting up the work of labour involved in the production of a particular commodity into several parts and each part and sub-part is performed by a specialist

Now let us start by the Advantages of Division of labouraIt increases the level

Questions

1 Differentiate between Product-Based division of labour and Process-Based division of labour

Product-Based division of labour

Process-based division of labour

It is also known as simple (or occupational) division of labourUnder it everybody performs a particular occupations The entire is done by the same person

When a person or group of persons undertakes a specialised function which is supplementary to the production of final commodity and service This is also as complex division of labour

It is simple It is complex

of productionbSince the product is produced by an expert workerbest quality of product is producedcIt saves time and toolsd it promotes inventions in the methods and techniques of productioneIt leads to reduction in costs fAll workers get work according to their abilities and choices

Now let us discuss the disadvantages of Division of labour

a Since many workers are involved in the production of a commodity no one has the sense of responsibility

b The constant and repetition of the same work again and again make the work monotonous

c Division of labour facilitates production on large scale Hencethere is fear of over production

d Because of territorial division of labour some areasregions become more developed than others

It is based on labour-intensive techniques of production

It is based on capital-intensive techniques

It is generally found in small enterprises

It is generally found in large enterprises

Example Indian farmers doing all farm activities

Example A modern garmet factory where one person takes the measurementanother does the cuttingsome sew the clothes while a few workers button them and other iron them

2 What are the advantages of Division of labouraIt increases the level of productionbSince the product is produced by an expert workerbest quality of product is producedcIt saves time and toolsd it promotes inventions in the methods and techniques of productioneIt leads to reduction in costs fAll workers get work according to their abilities and choices

3Discuss the disadvantages of Division of laboura Since many workers are involved in the

production of a commodity no one has the sense of responsibility

b The constant and repetition of the same work again and again make the work monotonous

c Division of labour facilitates production on large scale Hence there is fear of over production

d Because of territorial division of labour some areasregions become more developed than others

English 1 Transformation of sentences

Sentences A sentence is a group of words which makes complete sense

a Assertive sentences

Exercise 6Rewrite the following sentences according to the instructions given below without changing their meanings

1 As soon as he saw the beer he jumped into

b Imperative sentences

c Interrogative sentences

d Exclamatory sentences

Sentences can be changed from one grammatical form to another without changing the meaning of the sentence This is known as transformation of sentences

the river ( Begin No sooner)2 None but brave deserve the fair (Begin the

bravehellip)3 This box is too heavy for me to lift ( Use so hellip

That instead of too)4 No one other than a king can live like James

Luxurious ( Begin only James)5 Oh for the wings of a dove (Begin I wishhellip)

Math Topic Commercial MathematicsChapter Shares and Dividends

Study item Discuss about shares and Dividends1) What is share

Ans To start any big business (company or Industry) a large sum of money is needed But it is not possible for an individual to invest such a large amount Then some persons interested in the business join together and from a company They divide the estimated money required into small parts Each such part is called a share

2) What do you mean by the term shareholder

Ans A person who purchases one or more shares is called shareholder3) Some terms related with a share

(i) Nominal value or face value or printed value The original value of a share is called its nominal value or face value or printed value

Note The nominal value of a share always remains same(ii) Market value or cash value The price of a share at any

time is called its market value or cash value

Note The market value of a share changes from time to time(iii) At par If the market value of a share is the same as its

nominal value the share is called at par(iv) At Premium or above Par If the market value of a share

is more than its nominal value the share is called at premium or above par

Example If a share of Rs 100 is selling at Rs 150 then it is said to be selling at a premium of Rs 50 or Rs 50 above par

(v) At Discount or below par If the market value of a share is less than its nominal value the share is called at discount or below par

Example If a share of Rs 100 is selling at Rs80 then it is said to be selling at a discount of Rs 20 or at Rs 20 below par

4) What is Dividend

Ans The profit which a shareholder gets for hisher investment from the company is called dividendNote (i) The dividend is always expressed as the percentage of the face value of the share(ii) The dividend is always given( by the company ) on the face value of the share

irrespective of the market value of the shareBENGALI(2ND LANGUAGE)

ldquoদেবতোর জণমrdquoলিবরোম চকরবত

পরথম লিসর পোঠ-চোর পসথ একটি পোথর লিবপলি ঘটোয় দেক যোতোয়োসতর পসথ পরলিতলিয়ত ওই পোথসর দেো োচট দেসত একলি দেতো দেক দেো োচট দেসয় দেবোমো সয় রোসতোর মোস লি0টসক পস1 লিবপরীত লিক দেথসক আো একটি দেমোটর োলি1 চোসকর কষতোয় পরোসরণ দেবোসচ যো লিকনত পরলিতবোর এমরণ দেৌভোয দেসকর োও সত পোসর তোই লিতলি দেকোো দেজোো1 কসর পোথরটিসক উপস1 দে8স পোথর উপস1 দে8োর ময় এক দেকৌতী জতো দেকসক পরশন কসর দেয লিতলি দেকোসো দেবতোর আস দেপসয়স0রণ লিকো লিকনত দেক বস লিতলি দেকোসো দেবতোর আস পোলি দেক উপলিত ক জতোর উসltসয বস কোরও ইস= স পোথরটি লিসয় দেযসত পোসর এর পর দেথসক দেক দেযসত আসত পোথরটি দেক দেসত পো একলি দেক কষয করস দেকউ পোথরটিসক ধসয়মস0 পলিরসকোর করস0 দেক ওই দেকৌতী জতোসক পোথসরর কোস0 বস থোকসত দেস এ0ো1ো আরও কষয কসর দেক ঠোৎ ওই লি1 পোথরটির োসয় লিোর োো-দেকউ পজো কসরস0 দেকৌতী দেোকটির আঙকো য় যলিসক উ পোথরটিসক লিরসয় দে8স তোর পর ঠোৎ একলি পোথরটির দেোোজ দেই দেক লিসয় দেস0 বো দেকোথোয় দেস0

বদোথ-

দেো োচট ndashচসত লিসয় দেকো লিক0র সE ধোককো দেস পস1 যোবোর উপকরমঅকসমোৎ- ঠোৎআতমমবর- লিসজসক লিয়নতরপ-পো লিপ0স প1োদেসতসসত- লিসপলিউৎোত- দেো1ো দেথসক উপস1 দে8োপরতযয় ndash লিবশবোপরতযোস- টতযোস ndash দৈবোসধসতোধলিসত- পরসপসরর পরলিত ব পরসয়ো করোপরসতরীভত- পোথসর পলিররণলিতবোনতঃকরসরণ- মস পরোসঅলিQৎ- অQো করসত ই=কদেোপ- বধমোর- দেবোস এমইতযোকোর- এইরকমরম- বময়পযসোভী-পসযর জয দেোভ আস0 যোরপোসথয় ঞচয়- পথচোর রচ জমোসোমোমোসরোস- ব ধম ধোসমর সE

তোর লি দেইhelliphellipTo be continued

Hindi 2nd lang सर क पद(सरदास)

सरदास शरी कषण भकति कावय क सरवशरषठ कगिरव ह इनक जनम और मतय क समय तथा सथान का मतभद हसरदास रवातसलय और शरार रस क अनयतम कगिरव ह इनक कावय म बालकषण क सौदय चपपल चषटा और गि7याओ की मनोहर झाकी मिमलती ह कषण और ोगिपयो क अननय परम का कतिचतरण ह सयो शरार की अपकषा उनक कावय म गिरवयो शरार का अमिBक गिरवषय और मारमिमEक कतिचतरण हआ हइन पकतियो म हम सरदास की भकति भारवनाओ का परिरचय मिमलता ह इनका सपण सगरह सरसार म गिनगिहत ह

1 जसोदा हरिर पालन झलारवहलरारवदलराईमलहारव रव जो ईसाई कछ ारव मर लाल को आई निनEदिदया काह ना आगिनसबारव त काह नाही बरवगिह आरवतोको कानहा बलारव

शबदाथ-हलरारव-गिहलती हदलराई - दलार पयार करती हमलहारव-पचकारती हनिनEदरिरया ndashनीदरवगिह-जलदी सअBर-होठमौन-चपसन-सकत

वयाखया- सरदास जी कहत ह गिक यशोदा माता बालक कषण को पालन म झल आती ह रवह उनह गिहलाती ह पयार करती ह मलहार जस कोई ीत ान लती ह और नीद स पछती ह गिक ह नीद तम मर लाल को आकर कयो नही सलाती तझ खाना बला रहा ह कभी कषण आख बद कर लत ह कभी आखफडफडान लत ह उनह सोता हआ जानकर यशोदा माता चप हो जाती ह और इशार म बात करन लती ह इसी बीच अकला कर कषण ज जात हतो गिफर यशोदा माता गिफर स ाना ान लती ह सरदास जी कहत ह गिक भरवान क दशन का सख दरवता और ऋगिष-मगिनयो को भी दलभ ह यही सख माता यशोदा को बडी सहजता स मिमल जा रही ह माता यशोदा बहत ही भागयशाली ह2)Continue to nexthellip

Physics

Chapter 1 Force

(Summary)

Question A body is acted upon by two forces each of magnitude F but in opposite directions State the effect of the forces if

(a) Both forces act at the same point of the body

(b)the two forces act at two different points of the body at a separation r

Solutions

(a) Resultant force acting on the body = 0

F ndash F = 0(b) The forces tend to rotate the body between two forces about the midpoint

Moment of forces = F times rFr

QuestionDefine moment of a couple Write its SI unit

Solutions

Moment of couple is equal to the product of both force and the perpendicular distance between the two forces

The SI unit of moment of couple is NmCommercial Studies

Advertising and sales

Business firms use several methods to

Questions1) What do you mean by advertising

promotion create demand of their product in the market and increase it sales Such methods comprises of advertising sales promotion personal selling and publicityToday we are going to discuss about one of such methods It is advertising

Meaning of advertisingAdvertising is a paid form of non-personal presentation for promotion of Ideas goods and services

Importance or merits of advertising Advertising has importance to manufacturer or traders to customer and to society as a whole

Today we will see how advertisement help the manufacturer or traders

Answer) Advertising is a means of how a company encourages people to buy their products services or ideas It is one element of marketing which also includes design Research and data mining

2) Mention any three features of advertisingAnswer)The main features of advertising are

i) It is impersonal form of presentation for promotion of products and services of Ideas

ii) It is issued by identified sponsor The advertisement contains the name of the advertiser

iii) It is a form of mass communication because the message is directed to a large number of persons simultaneously

3) Mention the main merits or importance of advertisement to manufacturer or tradersAnswer)

i) Introducing new product A business organization can introduce itself and its products to the public through advertising

ii) Increase the sale Advertising leads to increase the sale of existing product by entering into new markets and attracting new customers

iii) Create steady demand Advertising creates sustains regular demand by smoothening out seasonal and other fluctuations It enables regular production for the organisation

iv) Economics of scale Advertising facilitate mass distribution of goods and steady demand which lead to large scale and regular production

v) Goodwill Advertising helps in creating a good image of the firm and reputation for its products

Biology Chapter - 03Genetics

Today wewill start chapter and discuss about Genetics Gregor Mendel is known as father of genetics Before entering into Mendelrsquos experiment on Genetics we must know

Q1 Define the following termsi) Genetics Genetics is the study of

transmissionof body features from parents to offspringand the laws relating to such transmission

ii) Heredity It may be defined as transmissionof genetically based characteristics from parentsto offspring

iii) Character and traits Any heritable

Importance to TraderIntroducing new productIncrease the saleCreate steady demandEconomics of scaleGoodwill

some terms featureis a character The alternative forms of acharacter are called traitsex Character (Hair shape) - Traits (Curly straight)

iv) Homologous chromosomes A pair ofcorresponding chromosomes of the same shapeand size one from each parent

v) Genes Genes are the specific parts (DNA segments) of a chromosome which determinethe hereditary characteristicsNearly 30000genes present in human

vi) Alleles Alternative forms of a gene occupying the same position (locus) on homologouschromosomes and affecting the same characteristicbut in different ways

vii) Genotype ndash PhenotypeGenotype means of genes present in the cells of an organism Phenotype means the observable characteristic which is genetically controlled

viii) Mutation It is a sudden change in one or more genes or in the number or in the structure of chromosomes ex Sickle cell anaemia is a blood disease caused by a gene mutation

CLASS NOTES

Class XSubject Eng Literature (The Merchant of Venice ndash William Shakespeare)Topic Act IV Scene 1 Lines 01 to 34 ( Duke helliphelliphellip We all expect a gentle answer Jew) ate 27th April 2020 (2nd Period)

[Students should read the original play and also the paraphrase given in the school prescribed textbook]Summary Questions amp Answers

This scene may be termed as the catastrophe of the play It is the final unravelling of the complicated events which seem to threaten the happiness of Bassanio Portia and Antonio Right is justified to the fullest degree and malice falls into the trap prepared for others No one suffers here but Shylock but even then he receives a generous measure of mercy

o This is the Court-scene Initially we meet

(1)

DUKE I am sorry for thee thou art come to answer (Line 3-6)A stony adversary an inhuman wretchUncapable of pity void and emptyFrom any dram of mercy

(i) Who is addressed here Where is the person Why is the person there

Antonio is addressed hereAntonio is in the court of justice at VeniceAntoniorsquos trial is scheduled to be held here for his failure to meet the conditions of the bond he signed with Shylock

the Duke Antonio Shylock and Salerio Later we meet Bassanio Portia Gratiano and Nerissa

o The Duke says to Antonio that he has to face a very cruel opponent which Antonio admits and expresses his gratefulness to the Duke for his efforts to soften without result the heart of Shylock in order to be merciful to Antonio Antonio further says that he is ready to accept whatever cruel judgement the Court may award

o When Shylock appears in the court the Duke says that Shylock should change his decision of prosecuting Antonio and demanding the penalty specified in the bond out of consideration of the great misfortunes that Antonio has suffered If this is done by him (Shylock) the whole court would be gladdened by his merciful action

(ii) What is the Duke sorry for

The Duke is unable to change the mind of Shylock from his decision to get the bond forfeited even after he pleaded to Shylock Shylock stands firmly for his bond which when forfeited will allow him to take a pound of flesh from any part of Antoniorsquos body(iii) How does the Duke address Shylock`The Duke calls Shylock an adversary with a heart of stones He calls Shylock as an inhuman wretch without pity Shylock is quite lacking in the slightest quality of mercy (iv) How does Antonio reply to this

Antonio replies that he will meet the revenge of Shylock patiently He has prepared himself to suffer with a quiet spirit the utmost that Shylockrsquos tyranny and rage can do(v) What quality of the Duke is revealed here

The Duke is kind and benevolent He is ready to help Antonio He requests Shylock to free Antonio from the trial(vi) What are the terms of the bond that Antonio has signed

The terms of the bond that Antonio has signed were that if Antonio is unable to repay Shylock a certain sum of money specified on the paper on a certain date and in an agreed place the forfeit has to be paid The forfeiture will be an exact pound of Antoniorsquos flesh which Shylock will be a liberty to take from any part of Antoniorsquos body which pleases him

Class XI

STUDY MATERIAL

Class XISubject Eng Literature (The Tempest ndash William Shakespeare) Topic Act I Scene 2 Lines 88 to 132 (Prospero hellip Me and thy crying self) Date 27th April 2020 (3rd Period)

[Students should read the original play and also the paraphrase given in the school prescribed textbook]Summary Questions amp Answers

o Prospero now tells Miranda that he was the Duke of Milan He had been devoting himself more to studies than the affairs of the State His brother Antonio took advantage of this situation and with the help of Alonso the king of Naples seized upon him and her one midnight and shipped them in a frail bark so that they perished in the sea All this took place

(1)

MIRANDA I should sin (Line 118-132)

To think but nobly of my grandmother

Good wombs have borne bad sonsPROSPERO Now the condition

The King of Naples being an enemyTo me inveterate hearkens my brothers suitWhich was that he in lieu othrsquo premisesOf homage and I know not how much tribute

twelve years back

IMPORTANT PASSAGES EXPLAINED(Line 98-103)

PROSPERO helliphelliphelliphelliphelliphelliphellip

He being thus lorded

Not only with what my revenue yieldedBut what my power might else exact like oneWho having into truth by telling of it Made such a sinner of his memoryTo credit his own lie he did believeHe was indeed the duke

Prospero in telling the narrative of his past life here refers to his brother Antonio Prospero being with a studious bent of mind has left the administration of Milan on his younger brother Now Antonio being thus invested like a lord with all the powers derived from Prosperorsquos wealth and what the exercise of Prosperorsquos authority might secure for him regarded himself as a de facto Duke of Milan It is a well-known fact of psychology that a man who repeatedly tells a lie makes of his memory such a sinner against truth as to credit his own lie by the telling of it So Antonio by repeatedly saying to himself and others that he was the Duke came to believe that he was really the Duke Thus falsehood repeatedly asserted gained the force of truth for Antonio and he truly believed it

Should presently extirpate me and mine Out of the dukedom and confer fair MilanWith all the honours on my brother whereonA treacherous army levied one midnightFated to thrsquo purpose did Antonio openThe gates of Milan and ithrsquo dead of darkness The ministers for thrsquo purpose hurried thenceMe and thy crying self

(i) In the earlier lines of this scene what does Prospero tell about his intense interest What was the demand of his interest

In the earlier lines of this scene Prospero tells Miranda that he had an intense interest in the study of philosophy and magic arts Hence in order to improve his mind with this kind of study he kept himself isolated from worldly and state affairs His study was dearer to him than the applause and esteem that he could win from the public His study demanded too much solitude(ii) What forced Antonio to take an undue advantage over Prospero

Prosperorsquos indifferent attitude towards the statersquos affairs and his having boundless trust in Antonio gave rise to a boundless lust for power in Antoniorsquos mind Antonio felt that he must be the actual Duke instead of the part of the Duke he played Thus Antonio took an undue advantage of the situation to usurp Prosperorsquos dukedom(iii) Explain the following lines ldquoI should sin to think but nobly of my grandmother Good wombs have borne bad sonsrdquo

After hearing the treacherous act of her uncle Antonio Miranda says that Prosperorsquos mother was a noble lady and she cannot dishonour her memory by saying that the person named Antonio cannot be his (Prosperorsquos) brother She says that in honour of her grandmother she also cannot say that Antonio must have been begotten not by her grandfather but by some other man She finally concedes that it is known that good mothers have borne bad sons in their wombs and gave birth to them

(iv) Why did the King of Naples accept Antoniorsquos request to help him in usurping his dukedom What did Antonio propose to Alonso

Alonso who was the king of Naples accepted Antoniorsquos request in usurping Prosperorsquos dukedom because he (Alonso) was a sworn enemy of Prospero Antonio proposed that Alonso should immediately drive him (Prospero) and his offspring out of Milan and should confer the dukedom upon him (Antonio) with all the dignities which go with that

position In return Antonio promised that he shall give an annual tribute and also swore his allegiance to Alonso Also he agreed to hold the Dukedom of Milan as a subordinate to the state of Naples(v) How were Prospero and Miranda carried away from the city of Milan and what was the state of small Miranda at that time

In pursuance of the agreement settled between Antonio and Alonso an army of treacherous men was assembled One midnight when the occasion suited the will of destiny Antonio opened the gates of the city of Milan and in the death like silence of midnight Antoniorsquos agents who had been directed to execute his purpose carried Prospero and small Miranda away from the city in all haste They were then forced into a ship and carried some distance out to sea where they put them on a mere hulk of a boat without any rigging or ship-gear and abandoned them leaving them at the mercy of the roaring sea Miranda was a very small child of three years age and she was crying at that time

CLASS -XIDATE-270420Subject Topic Summary Execution

EVS Chapter 1 ndash Mode of Existence

Impact of mode of existence on resources

Q) Why resources are under pressure

Ans - Increase in the sophistication

of technology enabling natural resources to be extracted quickly and efficiently Eg in the past it could take long hours just to cut down one tree only using saws Due to increased technology rates of deforestation have greatly increased

The number of humans is increasing Cultures of consumerism Materialistic views

lead to the mining of gold and diamonds to produce jewelry unnecessary commodities for human life or advancement Consumerism also leads to extraction of resources for the

production of commodities necessary for human life but in amounts excessive of what is needed because people consume more than is necessary or waste what they have

Lack of awareness among the population is striking People are not aware of ways to reduce depletion and exploitation of materials

Accounts Cash Book Today we are going to start a new topic -Cash Book

The key terms used in this chapter are

bullCash book

bullSimple cash book

bullDouble column cash book bullTriple column cash book

bullPetty cash book

bullCash discount

bullContra entry

Here I will share you the meaning of each key terms

bullCash book Cash Book is a special purpose subsidiary book or journal in which cash received and cash payments are recorded

bullSimple cash book

It is a cash book in which only cash transactions are recorded It has only one column on each side

bullTriple column cash book

It is cash book which has three columns one column for each cash and Bankdiscount on each side of the cash book In this book both cash and Bank transactions are recorded together with discount allowed and received

bullPetty cash book

It is a cash book maintained for recording petty expenses

bullCash discount

Cash discount is the amount of discount received or allowed on cash payments and cash receipts Discount received is an income for the business while discount allowed isan expense

bullContra entry

It means transactions involving both cash and Bank Such transactions though recorded in the cash book are not posted into ledger The letter lsquoC is written in Ledger folio for contra entry

Business Studies

ENTREPRENEURSHIP

Now we shall discuss the second chapter

lsquoENTREPRENEURSHIPrsquo

Today before starting the chapter let us recall what

Questions

1What are the main characteristics of Intrapreneurship

Answer

The main characteristics of Intrapreneurship are

Corporate framework-it occurs within the framework of the same company

Semi-Autonomous-Intrapreneurship

we have read last day

Let s today start the class by recalling the last topic taught

Intrapreneurship is the process of discovering and exploring business opportunities within an existing company It involves launching new business ventures within the framework of a present corporation Intrapreneurship is also known as corporate entrepreneurship or corporate venturing

Now let us start with the characteristics of Intrapreneurship

The main characteristics of Intrapreneurship are

Corporate framework

Semi-Autonomous Lack of ownership Senior position Low risk taking Not own boss

Now let us discuss the meaning of enterprise

Enterprise means an undertaking or adventure that requires some innovation and investment and thus involves riskEnterprise always entails decision making coordination and risk bearing

involves crating amd nurturing a semi-autonomous business unit which may be a subsidiary a strategic business unit or a division

Lack of ownership-the intrapreneur is not the owner of the unitb he creates and nurtures

Senior position-he occupies a senior managerial position in the company

Low risk taking-An intrapreneur does not bear the full risk of failure

Not own boss-An intrapreneur is not his own bosss in legal termsHe enjoys the freedom and gets the required resources and support

2 How is Entrepreneur is different from Intrapreneur

The functions involved in both the entrepreneurship and intrapreneurship are by and large similar however there are several differences between the two

Point of distinction

Entrepreneur

Intrapreneur

status An independent business person

A senior executive within a company

Ownership Owner of

the business

An employeesometimes a share in ownership

Financing Responsible for raising finance for the business

Not responsible for raising the finance

Risk bearing

Bears the risk of the business

Does not bears the risk of the business

Reward Profit which is uncertain and irregularcan be loss

Fixed salary and fringe benefits

Need for security low high

3 What do you understand by enterprise

Answer Enterprise means an undertaking or adventure that requires some innovation and investment and thus involves riskEnterprise always entails decision making coordination and risk bearing

COMMERCE NATURE AND OBJECTIVES OF

BUSINESS

Today let us recall the last other two objectives of business by the chart given in the previous class

Firstly we would discuss Human Objectives

Business is run by people and for people Labour is a valuable business element

Human objectives of business are concerned with the well -being of labour

The human objectives are as follows

Labour welfare Developing human

resources Participative

management Labour

management cooperation

Questions

1 Explain the human objectives of a business enterprise

Answer

Business is run by people and for people Labour is a valuable business element

Human objectives of business are concerned with the well -being of labour

The human objectives are as follows

Labour welfare-Business must recognize the dignity of labour and human factors should be given the recognition

Developing human resources-Employees must be provided the opportunities for developing new skills and attitudes

Participative management-Employees should be allowed to take part in decision making process of business

Labour management cooperation-Business should strive for creating and maintaining cordial employer employee relations so as to ensure peace and progress in industry

Now let us discuss the national objectives of business

Optimum utilization of resources

National self- reliance Development of small

scale industries Development of

backward areas Control over pollution

2Explain the national objectives of a business enterprise

Answer

It is the duty of business to utilize the resources of the country properly the national objectives of business

Optimum utilization of resources ndashBusiness should use the nationrsquos resources in the best possible manner

National self- reliance-It is the duty of the business to help the government in increasing experts and in reducing dependence on imports

Development of small scale industries-Big business firms are expected to encourage growth of small scale industries which are necessary for generating employment

Development of backward areas-Business is expected to give preference to the industrialization of backward regions of the country

ECONOMICS

BASIC ECONOMIC CONCEPTS

SUB

TOPIC

Value

Wealth

Welfare

Today we shall start with a new topic of the same chapter ie lsquoValuersquo

Value of a commodity is defined as the valuation placed by a household on the consumption of this commodity

lsquoValuersquo has two different meanings and these are

a Value -in -use It refers to consumption value of a commodity It expresses the utility derived from the consumption of a particular commodity A necessity like water has a very high value ndashin ndashuse or

Question

1What is value

Answer

Value of a commodity is defined as the valuation placed by a household on the consumption of this commodity

2What is value-in use

Answer It refers to consumption value of a commodity It expresses the utility derived from the consumption of a particular commodity A necessity like water has a very high value ndashin ndashuse or consumption value

3What is value ndashin- exchange

Answer It relates to market value of a commodity

It is the rate at which a particular good or service can be exchanged for money

For example in barter system if a person is prepared to exchange 3 metres of cloth with 1 pair

consumption value

b Value ndashin-exchange It relates to market value of a commodity

It is the rate at which a particular good or service can be exchanged for moneyFor example in barter system if a person is prepared to exchange 3 metres of cloth with 1 pair of shoes then the value in exchange of 3 metres of cloth is 1 pair of shoesValue in exchange is the power of purchasing other goods In modern monetised economies the exchange value of goods are expressed in terms of money as prices

Now let us discuss the term lsquoWealthrsquo

Wealth refers to the stock of all those assets which are a source of income

Wealth is a stock concept

Wealth must possess the following features

a Utility It must possess utility or give some

of shoes then the value in exchange of 3 metres of cloth is 1 pair of shoes

Value in exchange is the power of purchasing other goods In modern monetised economies the exchange value of goods are expressed in terms of money as prices

4 What is wealth

Answer it refers to the stock of assets or goods which are a source of income and have personal or national ownership

5 What are the features of wealth

Answer The features of wealth are as follows

Wealth must possess the following features

a Utility It must possess utility or give some satisfaction

b Scarcity It must be limited in quantityc Transferability it should be transferable its

ownership can be transferred from one person to another person

d Exchange value It must possess exchange value

6 What is welfare

Answer

Welfare is defined as satisfaction and happiness a sense of well- being among the people

satisfactionb Scarcity It must be

limited in quantityc Transferability It

should be transferable its ownership can be transferred from one person to another person

d Exchange value It must possess exchange value

Now let us discuss the term lsquoWelfarersquo

Welfare is defined as satisfaction and happiness a sense of well- being among the people

Welfare is affected by factors like

a Consumption of goods and services

b Environment

c Family relations

d Degree of freedom

e Law and order situation

Mathematics Trigonometric equation

To find the general solution of the equation sinθ=0

When sin θ =0

Then θ= 0 π2π 3π-π -2π -3

i e when θ = 0 or an integral multiple of π

i e when θ= nπ where n is any integer

Therefore the general solution of the equation sin

Example1 Find the general values of θ which satisfy the equation sin2 θ =34

Solution sin2 θ= 34

Or sin θ = +34 or -34

Or sin θ = sin π3 or sin (-π3)

Therefore

θ = [nπ + (-1) n (π3)] or[ nπ+ (-1) n (-π3)]

= nπ +π3 or nπ-π3 where n= any integer

Example 2Find the values of θ which satisfy tan2 θ

θ=0 is θ= nπ where n is any integer

To find the general solution of the equation cos θ=0

When cos θ=0

Then θ=π2 3π2 5π2 -π2 -3π2 -5π3

i e when θ is an odd multiple of π2

i e when θ=(2n+1) π2 where n= any integer

Therefore the general solution of the equation cos θ =0 is θ= (2n+1) π2 where n= any integer

To find the general solution of the equation tan θ = 0

Clearly tan θ =0 implies sin θcos θ =0

Therefore θ = nπ

i e the general solution of the equation tan θ=0 is θ =nπ where n = any integer

To find the general solution of the equation cot θ =0

Clearly cot θ =0 implies (cos θsin θ) = 0

i e cos θ =0

Therefore θ = (2n+1) π2

Therefore the general solution of the equation cot θ =0 is θ = (2n+1) π2

Where n= any integer

To find the general solution of the equation sin θ= k (-

=13 -πleθleπ

Solution tan2 θ =13

Or tan θ = plusmn1radic(3) =tan(plusmnπ6)

θ=nπ plusmn π6 where n =any integer

If n=0 then θ=plusmnπ6

If n=1 then θ= π plusmn π6

If n=-1 then θ= -π plusmn π6

Therefore the required solution in -π le θ le π are θ= π6 5π6 -π6 -5π6

Exercise Find general solution of sin 2θ=cos θ [Hints Use sin 2θ= 2sin θcosθ and then take cosθ

common]

1lek le1)

Determine an angle alpha such that sin =k and -π2le αle π2

Then we have

Sin θ = k = sin α

Or sin θ - sin α =0

Or 2 cos [(θ+α) 2] sin [(θ-α) 2] =0

Therefore either cos [(θ +α) 2] =0 (1)

Or sin [(θ-α) 2] =0 (2)

Now from (1) we get (θ+α) 2= (2m+1) π2)

Or θ = (2m+1) π-α (3)

And from (2) we get (θ-α) 2 =mπ

Or θ= 2mπ+α(4)

Where m = any integer

Clearly the solution (3) amp (4) may be combined in the following form

θ= nπ+(-1) n α where n= any integer

Therefore the general solution of sin θ = sin α is θ = nπ +(-1) n α where n is any integer and -π2 le α le π2

Biology Chapter - 04Kingdom Monera

Today we will discuss about bacterial reproduction and its usefulness

Fig Binary Fission

Fig Conjugation Fig Transformation

Bacterial reproduction is mainly asexual but sexual reproduction

also takes place

Asexual reproduction takes place by i) Binary fission - from one bacteriato

two bacteria are produced in every 20 to 30mins

ii) Buddingiii) Endospore formation - during

unfourable condition

Sexual reproduction by three ways

1) Conjugation - Transfer of genetic material between cells that are in physical contact with one another

2) Transduction - Transfer of genetic materialfrom one cell to another by a bacteriophage

3) Transformation - Transfer of cell-freeor naked DNArsquo from one cell to another

Bacteria causes different diseases inplants animals and human and

it causes food spoilage and waterpollution but it also have some useful

activities

i) Bacteria are helpful in sewage water treatment

ii) It is used in antibiotic (medicine) production

iii) Anaerobic bacteria help in biogas(energy) production

iv) Many household products like yoghurt cheese are manufactured by use of bacteria

v) Rhizobium by symbiotic relationship with leguminous plant increase soil fertility

vi) Besides these bacteria is helpful in genetic engineering degradation of petroleum hydrocarbonand in dairy

industry

Physics Motion in plane Here we will introduce Projectile Motion

Execution

Projectile

Y

usinθ u h

θX

ucosθ

Suppose a body is projected with an angle θ So initial velocity u can be resolved into two components

Horizontal component - ucosθ ( for range)

Vertical component - usinθ ( for height)

usinθ changes during motion and becomes zero at maximum height position but ucosθ remain unchanged

The maximum height of projectile is h

NB If initial is upward then g = -ve and if it is downward then g = +ve Height is +ve if direction of motion does not change ( for ex a body thrown upwards but goes down ultimately then height h = -ve)

The angle of projectile θ is the angle made with horizontal

HISTORY ndash GROWTH OF NATIONALISM

SUB TOPIC- REVOLUTIONARY NATIONALISM Bengal formation of Anushilan Samity and Jugantar Group

The intensification of the Swadeshi movement and Government policy of terror and repression led to outbreak of violence Bombs were manufactured and attempts on the lives of unpopular Government officials became frequent In the gymnasium of Scottish Church College which was known as General Assemblies Institution a secret society was formed known as Anushilan Samity

Aurobindo Ghosh send from Baroda his emissary Jatindranath Banerjee to mobilize the Bengal revolutionaries

Hemchandra Qanungo and Satyen Bose published Journal Jugantar

The Jugantar group planned to assassinate oppressive magistrate Kingsford by Khudiram Bose and Prafulla Chaki in 1908 Prafulla Chaki committed suicide to avoid arrest Khudiram was tried and hanged

Afew days later the police found a bomb factory in Maniktala and arrested a large number of revolutionaries The trial of revolutionaries became famous as the Alipore Bomb Case

In the course of the trial the approver the public prosecuter and a police officer were assassinated

1 Question Name two journals which preached the cult of violence

Answer a) Yugantar edited by Bhupendranath DuttaB) Bandemataram edited by Aurobindo Ghosh2 Question Why was Khudiram arrested and hangedAnswer An attempt was made to assassinate a hated vindictive majistrate named Kingsford by Khudiram Bose and Prafulla Chaki Their attempt failed and the bomb they threw killed two English ladies Khudiram was arrested and put to trial and then hanged3Question Who was Aurubindo GhoshAnswer Aurobindo Ghosh a nationalist revolutionary who was charged for his involvement in the Alipore Bombing Case He was accused of it along with his brother Barindra nath Ghosh But Aurobindo was acquitted because of the brilliant pleading of his counsel Chittaranjan Das Then he became a spiritual reformer introducing his visions on human progress and spiritual evolution4 Qustion Who was KingsfordAnswer Kingsford was an unpopular British chief Magistrate who was the target of the bomb thrown at Muzaffarpur by Khudiram and Prafulla Chaki

Most of the accused were convicted and sentenced to

long term of imprisonmentBut

Aurobindo Ghosh was acquitted mainly owing to the brilliant pleading of his counsel Chittaranjan Das

Political science Topic-Sovereignty

Summary Sovereignty is the full right and power of a governing body over itself without any interference from outside sources or bodies In political theory sovereignty is a substantive term designating supreme legitimate authority over some polity In international law sovereignty is the exercise of power by a state

Internal Sovereignty

Internal sovereignty means supreme authority within ones territory while external sovereignty relates to the recognition on the part of all states that each possesses this power in equal measure

External sovereignty

external sovereignty relates to the recognition on the part of all states that each possesses this power in equal measure

Distinguish between

Execution

Answer the following questions

Short notes-

Sovereignty

Internal Sovereignty

External sovereignty

Homework- learn

external sovereignty and internal sovereigntySovereignty is the principle

of supreme and

unquestionable authority

reflected in the claim by the

state to be the sole author of

laws within its territory

Definition of external vs internal sovereigntyInternal sovereignty refers to

the relationship between a

sovereign power and its

subjects ndash it refers to the

location of the supreme

authority within the state In

the UK for example internal

sovereignty (supposedly)

resides within Parliament

reflected in the

constitutional principle of

parliamentary

sovereigntyBy contrast

external sovereignty refers

to the capacity of the state

to act independently and

autonomously on the world

stage This is what is

sometimes called lsquostate

sovereigntyrsquo or lsquonational

sovereigntyrsquo and implies

that states are legally equal

and that the territorial

integrity and political

independence of a state is

inviolable

Class ndash XII

Date - 2742020 STUDY MATERIALSubject Topic Summary Execution Business Studies

Job Analysis amp Manpower Planning

At first let us recall the chapter what we have discussed till nowbullJob analysisbullJob specification bullJob description bullJob enlargement bullJob enrichment

Today we will do some questions answers from the chapter

Questions 1ldquoJob analysis job description and job specification are interrelatedrdquo Comment Answer) Job analysis is a systematic and detailed examination of a job to collect all the relevant information about it The contents off the job are summarised in the job description The qualification needed for the job are summarised in job specificationThus there is close interrelationship between job analysis job description and job specification

Question 2ldquoJob enlargement is a horizontal extension of a job whereas job enrichment is a vertical extension of a jobrdquo ElucidateAnswer) Job enlargement involves adding one or more task to a job coma where as job enrichment involves adding more autonomy and responsibility to a job Job enlargement is therefore horizontal extension of a job coma whereas job enrichment is a vertical extension of a job

Question 3 )

What is manpower estimation Explain its quantitative and qualitative aspectsAnswer) Manpower estimation is the process by which management determines how an organisation should move from itrsquos current manpower positionto its desired manpower position There are two dimensions of Manpower estimation- quantitative and qualitative

Quantitative aspectThis aspect of Manpower estimation involves estimating the number of employees required in a future time period Workload analysis and workforce analysis are done to estimate the quantity of required manpower

Qualitative aspectThe estimate of the knowledge skills experience etc of required manpower is the qualitative aspect of Manpower estimation The quality of Manpower can be judged on the basis of job analysisand job specification

COMMERCE

CAPITAL-FIXED AND WORKING

Today let us start the class by discussing the sources of finance for different types of business firms

The term lsquocapitalrsquo refers to the investment made in the enterprise for the purpose of earning profits

Requirements of capital and sources of capital for different types of business firms are

1 Capital for sole proprietorship businessA sole proprietor operates at a small scale and thereforerequires a limited amount of capital

2 Capital for partnership firmCapital requirements as well as capital base of a partnership is bigger than that of a sole trader businessThe owned capital is contributed by the partners in an agreed ratio

3 Capital for joint stock companyA joint stock company generally requires large amount of capitalA public company can raise huge capital through issue of shares In addition to share capital it can utilize retained profits

Now let u discuss the meaning of Finance PlanningFinance planning is the process of estimation the financial requirements of an organization specifying the sources of firms and ensuring that enough funds are available at the right time

1 What do you mean by Finance PlanningAnswer Finance planning is the process of estimation the financial requirements of an organization specifying the sources of firms and ensuring that enough funds are available at the right time

2Discuss the role of financial planning of an enterpriseThe role of financial planning are as followsa A sound financial plan helps a business enterprise to avaid the problems of shortage and surplus of fundsbFinancial planning serves as a guide in developing a sound capital structure so as to maximize returns to shareholders c It helps in effective utilization of fundsd It provides policies and procedures for coordinating different functional areas or departments of businesse It enables the management to exercise effective control over the financial activities of an enterprisef It helps the company to prepare for facing business shocks and surprises in future

Mathematics

Continuity and differentiability

Recall Definition of ContinuityLet f(x) be a single valued function of x and x=a be a point in the domain of definition of the function The function is said to be continuous at x=a ifi) f(c) is defined ie f(x) has a definite finite value at x=cii) lim xrarra f(x) exists andiii) lim xrarra f(x) =f(a) In other words f(x) is said to be continuous at x=a if lim xrarra+ f(x)= lim xrarra- f(x) = f(a) Or f(a+0) =f(a-0) =f(a) Or lim hrarr0 f(a+h)= f(a) Algebra of continuous functionsNow we will study some algebra of continuous functions Theorem 1 Suppose f and g be two real functions continuous at a real number c Then(1) f + g is continuous at x = c(2) f ndash g is continuous at x = c(3) f g is continuous at x = c(4) (fg) is continuous at x = c (provided g (c) ne 0)

Example 1 Prove that every rational function is continuousSolution Recall that every rational function f is given byf(x)=[p(x) q(x) ] q(x)ne0where p and q are polynomial functions The domain of f is all real numbers except points at which q is zero Since polynomial functions are continuous f is continuous by (4) of Theorem 1Example 2Discuss the continuity of sine functionSolution To see this we use the following factslim xrarr0 sin x =0Now observe that f (x) = sin x is defined for every real number Let c be a real number Put x = c + h If x rarr c we know that h rarr 0 Therefore lim xrarrc f(x)

= lim xrarrc sin x= lim hrarr0 sin(c+h) =lim hrarr0 [sin c cos h + cos c sin h ]=lim hrarr0 (sin c cos h) + lim hrarr0 (cos c sin h) = sin c +0=sin c = f(c) Thus lim xrarrc f(x) = f(c) and hence f is a constant function Exercise Prove that the function f(x) = x2 +2x is continuous for every real value of x [Hints show that lim xrarra+ f(x) = lim xrarra- f(x) = f(a) ]

Biology Reproduction in Flowering plants We will discuss about megasporoangium

megasporagenesis and female gametophyte

Q4 Describe the structure of megasporangium

Ovule is attached to the placenta by astalk called funicle

Each ovule has one two or three protectivecoverings called integuments

At the tip of integuments a small openingcalled micropyle is organised

Opposite to the micropylar end is the chalaza

Within the integuments a mass of cellsnucellusand inside it embryo sac orfemale gametophyte is present

Q5 Describe a mature embryosacamp its formation

In most of the flowering plants only oneof the 4 megaspores formed as a result ofmegasporogenesis that is functional while theother three degenerate

The

functional megaspore develops into thefemale gametophyte

Formation The nucleus

of the functional megasporedivides mitotically to form two nuclei first andthen two more sequential mitotic nucleardivisions result in the formation of four ampthen eight nucleate stages of embryo sac

Six of the eight nucleus are surrounded bycell walls and organised into cells

The remaining two nuclei called polar nuclei are found below the egg apparatus in the largecentral cell

Three cells consisting of two synergids amp one egg cell present bottom of

embryo sac Three cells

at the chalazal as antipodal cells

Two polar nuclei together present in large central cell

HISTORY

TOWARDS INDEPENDENCE AND PARTITION THE LAST PHASE(1935-1947)SUB TOPIC NATIONAL MOVEMENTS DURING THE SECOND WORLD WAR

Spread of Quit India Movement On 9th August 1942Gandhiji and other Congress leaders were arrested The Congress was declared illegal The news of the arrest of all leaders marked the beginning of a widespread movement of India It was not possible for such a movement to remain peacefulBut the arrest of the all notable congress leaders virtually left the movement in the hands of the mass The movement took the form of violent and militant outbreakBesides congressmen revolutionaries also were very active in the movement The Congress Socialist group also played a prominent role

1 Question Why did the British authority arrest the Congress leaders on 9 th August 1942Answer Congress Working committee adopted the Quit India resolution which was to be ratified at the Bombay AICC meeting in 8th August 1942 They decided to launch a mass struggle on non-violent lines Gandhiji gave a clarion call to all section of the people rdquoKarenge ya Marengerdquo (do or die) Congress leaders gave the call to driving out

the British from IndiaViceroy had taken strong action against the Quit India movement Gandhiji and all the leaders of Congress were arrested

2 Question How did Quit India Movement spread out all over IndiaAnswer The news of the leaders lsquo arrest marked the beginning of a widespread movement to remain peacefulThe movement took form of violent outbreak There were widespread cutting of telephone and Telegraph wires damaging railway lines raising barricades in cities and towns and other forms of violent demonstations

Question Name the leaders of Congress

Socialist group played a prominent part Notable among the Jayprakash Narayan Rammonohar Lohia Aruna Asaf Ali

Political science

Topic-Franchise and Representation

Summary

The election commission

The Election Commission of India is an autonomous constitutional authority responsible for administering Union and State election processes in India The body administers elections to the Lok Sabha Rajya Sabha State Legislative Assemblies in India and the offices of the President and Vice President in the country

Functions of election commission-

India is a sovereign socialist secular democratic republic Democracy runs like a golden thread in the social economic and political fabric woven by the Constitution given by lsquoWe the People of Indiarsquo unto ourselves The concept of democracy as visualised by the Constitution pre-supposes the representation of the people in Parliament and State legislatures by the method of election The Supreme Court has held that democracy is one of the inalienable basic features of the Constitution of India and forms part of its basic structure The Constitution of India adopted a Parliamentary form of government Parliament consists of the President of India and the two Houses mdash Rajya Sabha and Lok Sabha India being a Union of states has separate state legislatures for each state State legislatures consist of the Governor and two Houses mdash Legislative Council and Legislative Assembly mdash in seven states namely Andhra Pradesh Telangana Bihar Jammu amp Kashmir Karnataka Maharashtra and Uttar Pradesh and of the Governor and the state Legislative Assembly in the remaining 22 states Apart from the above two out of the seven Union Territories namely National Capital Territory of Delhi and Puducherry also have their Legislative Assemblies

ExecutionShort notes-Election commissionFunctions of election commission

Homework- Learn

Computer

Science

Computer hardware NAND Gate

A NOT-AND operation is known as NAND operation It has n input (n gt= 2) and one output

Logic diagram

Truth Table

NOR Gate

A NOT-OR operation is known as NOR operation It has n input (n gt= 2) and one output

Logic diagram

Truth Table

XOR Gate

XOR or Ex-OR gate is a special type of gate It can be used in the half

adder full adder and subtractor The exclusive-OR gate is abbreviated as EX-OR gate or sometime as X-OR gate It has n input (n gt= 2) and one output

Logic diagram

Truth Table

XNOR Gate

XNOR gate is a special type of gate It can be used in the half adder full adder and subtractor The exclusive-NOR gate is abbreviated as EX-NOR gate or sometime as X-NOR gate It has n input (n gt= 2) and one output

Logic diagram

Truth Table

Physics

Chapter 1 Electric Field ( Electric Dipole) (Summary)

Here we will derive Expression of electric field at broad side

On position of dipole

Execution

Q With the help of a labelled diagram obtain an expression for the electric field intensity E at any point on the equitorial line ( broad-side on position) of an electric dipole

Ans

E1 E1sinθ

E θ P E1 θ

( r2+L2)12 E2 E

r E2 E2sinθ

-q θ L O L +qA B

Let us consider that the point P is situated on the right bisector of the dipole AB at a distance r meter from its midpoint O

Let E1 and E2 be the electric field intensities of the electric field at P due to charge +q and ndashq of the dipole resp The distance of P from each charge is ( r2+L2)12

So E1 = 14 πϵ q

(r 2+L 2) away from +q

E2 = 14 πϵ q

(r 2+L 2) towards ndashq

The magnitudes of E1 and E2 are equal but directions are different Now resolving E1 and E2 into two components parallel and perpendicular to AB we get

The components perpendicular to AB E1sinθ and E2sinθ cancel each other because they are equal and opposite

The components parallel to AB are E1cosθ and E2 cosθ are in same direction and add up

So resultant intensity of electric field at the point P is

E = E1cosθ + E2 cosθ

E = 14 πϵ q

(r 2+L 2) 2 cosθ

Now from fig we have cosθ =BOBP = L (r2+L2)12

So we get E = 14 πϵ 2qL ( r2+L2)32

Now electric dipole moment p= 2qL

So E = 14 πϵ p ( r2+L2)32

HW Find the expression of Electric field as done here but this time take r gtgt 2L

Also find the expression of torque experience by a dipole

(Hint Electric force experienced by charges of dipole in electric field is qE each Let θ be the angle which dipole makes with electric lines of force then perpendicular distance between two charges is 2Lsinθ Then torque = force x perp distance = qE x 2L sinθ So τ=pE sinθ where p =2qL )

STUDY MATERIAL

Class XIISubject Eng Literature (The Tempest ndash William Shakespeare) Topic Act IV Scene 1 Lines 84 to 133 (Iris hellip A contract of true love Be not too late ) Date 27th April 2020 (4th Period)

[Students should read the original play and also the paraphrase given in the school prescribed textbook]Summary Questions amp Answers

o Ceres soon appears and comes to know that she has been summoned to celebrate the contract of true love

o Ceres expresses her unwillingness to meet Venus and Cupid as she has shunned their company

o Ceres and Juno both bestow their blessings upon Ferdinand and Miranda with June gifting honour riches happiness in marriage and Ceres presents plenty of earthrsquos produce

o Iris summons the water-nymphs and reapers to come and celebrate a contract

(1) IRIS Of her society (Line 91-101)

Be not afraid I met her deity

Cutting the clouds towards Pathos and her sonDove-drawn with her Here thought they to have doneSome wanton charm upon this man and maidWhose vows are that no bed-right shall be paidTill Hymens torch be lightedmdashbut in vainMarss hot minion is returned againHer waspish-headed son has broke his arrowsSwears he will shoot no more but play with sparrowsAnd be a boy right out

(i) Where were Venus and Cupid seen flying How were they travelling Why did they want to join the marriage celebration of Ferdinand and Miranda

of true love

Venus and Cupid were seen flying through the air towards Paphos the famous city which is situated on the island of Cyprus They were travelling by air-borne chariot drawn by doves They certainly wanted to come here in order to play some amorous trick upon Ferdinand and Miranda who are under a vow not to gratify their physical desires till the holy ceremony of their marriage has been performed(ii) What have Venus and Cupid done after failing in their plan

After being failure of their plan Venus who is a very passionate deity and who is the mistress of Mars (the god of war) has gone back while here ill-tempered son Cupid has broken his arrows of love in his state of desperation(iii) What has Cupid firmly decided

Cupid is feeling so disappointed that he has firmly decided to shoot no more arrows to arouse love in human hearts but to spend his time playing with sparrows Thus he would now become just a boy and would give up his original function of shooting arrows on human beings to make them fall in love(iv) What vow had Ceres taken How did Ceres feel at the abduction

After the abduction of her daughter Prosperina by Pluto Ceres had taken a vow to always keep away from the disgraceful company of Venus and her blind son Cupid the god of love Ceres felt deeply distressed when Pluto had carried off her daughter and had made her his wife by force(v) Why has Ceres not forgiven Venus and her blind son For what do Ceres want to be sure

As the abduction had been manipulated by Venus the goddess of beauty and love and her blind son Cupid Ceres has never forgiven them for their part in the whole plot Ceres wants to be sure that she would not have to meet Venus and Cupid who had engineered the abduction of her daughter Prosperina

AS THIS lsquoMASQUErsquo SCENE IS VERY IMPORTANT IN THE PLAY THE PARAPHRASE OF THE ENTIRE PORTION OF MASQUE SCENE (Act IV Lines 58 to 143) IS GIVEN BELOW

IRIS Goddess of RainbowCERES Goddess of Agriculture and all the fruits of the earth

(Nature growth prosperity rebirth ndash notions intimately connected to marriage)JUNO The majestic Queen of Heavens and wife of Jupiter (Jupiter is the king of Gods)

VENUS The Goddess of love CUPID Son of Venus PLUTO God of death (In the play referred by Shakespeare as lsquoDisrsquo which is a Roman name for Pluto)

ORIGINAL TEXT PARAPHRASEPROSPEROWellmdash

PROSPERONow come Ariel Let there be too many rather than too few

Now come my Ariel Bring a corollaryRather than want a spirit Appear and pertly[to Ferdinand and Miranda]No tongue all eyes Be silent

spirits in attendance Appear briskly

[to Ferdinand and Miranda]Look with your eyes but do not say a word

[Soft music] [Soft music][Enter Iris] [Enter Iris]

IRISCeres most bounteous lady thy rich leasOf wheat rye barley vetches oats and peasThy turfy mountains where live nibbling sheepAnd flat meads thatched with stover them to keepThy banks with pioned and twilled brimsWhich spongy April at thy hest betrimsTo make cold nymphs chaste crowns and thybroom-grovesWhose shadow the dismissegraved bachelor lovesBeing lass-lorn thy pole clipped vineyardAnd thy sea-marge sterile and rocky-hardWhere thou thyself dost airmdashthe Queen othrsquoSkyWhose watery arch and messenger am IBids thee leave these and with her sovereign grace[Juno appears] Here on this grass-plot in this very placeTo come and sport Her peacocks fly amainApproach rich Ceres her to entertain

IRISCeres most generous lady you are the cause of rich fields or fertile land where wheat rye barley beans oats and peas grow the grassy mountains where the sheep graze and the flat meadows covered with coarse hay to be used as fodder for cattleYour banks are covered with marsh-marigolds and reeds and the rainy April under your orders brings forth to make for the maids who are not in love beautiful crowns your woods where the broom flourishes and where the bachelor who has been dismissed by the maid he loved lies down being forsaken your vineyard in which the poles are embraced by the vines and the margin of the sea which is barren and rocky where you roam about to enjoy the fresh air ndash the queen of the sky (Juno) whose messenger I am besides being represented as the rainbow bids you leave all these and with her majesty here on this grassy plot in this very place come and sport her peacocks carry her fast in her chariot through the air and are making their way here approach rich Ceres to welcome her

[Enter Ariel as Ceres] [Enter Ariel as Ceres]

CERESHail many-coloured messenger that neerDost disobey the wife of JupiterWho with thy saffron wings upon my flowersDiffusest honey-drops refreshing showersAnd with each end of thy blue bow dost crownMy bosky acres and my unshrubbed downRich scarf to my proud earth Why hath thy queenSummoned me hither to this short-grassed green

CERESWelcome rainbow that never dared disobey Juno the wife of Jupiter who with your orange coloured rays spread honey-drops refreshing showers And with each end of thy blue bow drown my bushy acres and my hilly country which is free from shrubs you thus forming a rich scarf Why has your queen called me here to this place covered with short grass

IRISA contract of true love to celebrateAnd some donation freely to estateOn the blest lovers

IRISI have called you to celebrate a contract of true love and bestow some liberal gift upon the blessed lovers

ORIGINAL TEXT PARAPHRASECERESTell me heavenly bowIf Venus or her son as thou dost knowDo now attend the queen Since they did plotThe means that dusky Dis my daughter gotHer and her blind boys scandaled companyI have forsworn

CERESTell me heavenly bow if Venus the Goddess of love or Cupid her son and pedlar of passion at this time attend the heavenly queen Juno because you are sure to know Since the day they conspired against me and dark Pluto took away my daughter here and Cupidrsquos disgraceful company I have left off

IRISOf her societyBe not afraid I met her deityCutting the clouds towards Pathos and her sonDove-drawn with her Here thought they to have doneSome wanton charm upon this man and miad

IRISBe not afraid of her company I met her deity moving on the clouds towards Paphos the sacred home of Venus on the island of Cyprus along with her son on her chariot drawn by doves Here they contemplated to exercise a charm upon this man and maid producing

Whose vows are that no bed-right shall be paidTill Hymens torch be lightedmdashbut in vainMarss hot minion is returned againHer waspish-headed son has broke his arrowsSwears he will shoot no more but play with sparrowsAnd be a boy right out

wantonness before the actual marriage ceremony but did not succeed Venus has returned her irritable son has broken his arrows and swears that he will give up his practice of trying to inspire love but play with sparrows and be a boy again

[Music is heard] [Music is heard]

CERESHighst queen of stateGreat Juno comes I know her by her gait

CERESHighest queen of state Great Juno there she comes I know here by her gait

[Enter Juno] [Enter Juno]

JUNOHow does my bounteous sister Go with meTo bless this twain that they may Prosperous beAnd honoured in their issue

JUNOHow are you doing my generous sister Come with me to bless this couple so that they may be prosperous and fortunate in their children

[They sing] [They sing]

JUNOHonour riches marriage-blessingLong continuance and increasingHourly joys be still upon youJuno sings her blessings upon you

JUNOMay honour riches happiness in marriage long continuance and increase of those boons ever rest upon you as hourly joys Juno showers down upon you her blessings in song

CERESEarths increase foison plentyBarns and garners never emptyVines and clustring bunches growingPlants and goodly burden bowingSpring come to you at the farthestIn the very end of harvestScarcity and want shall shun youCeresrsquo blessing so is on you

CERESMay you have the plenty of earthrsquos produce Your barns and granaries may never be empty Your vines may grow with clustering bunches Your fruit trees may be heavily laden with their fruit May there be continuous spring and harvest May scantiness and want leave you forever Such is the blessing of Ceres upon you

FERDINANDThis is a most majestic vision andHarmoniously charmingly May I be boldTo think these spirits

FERDINANDThis is a great vision and magically melodious Should I suppose the characters (taking part in the masque) are spirits

PROSPEROSpirits which by mine artI have from their confines calld to enactMy present fancies

PROSPEROYes they are spirits whom I have summoned from the regions to which they are confined to carry into effect my fanciful designs

ORIGINAL TEXT PARAPHRASEFERDINANDLet me live here everSo rare a wondered father and a wifeMakes this place paradise

FERDINANDI should like to live here forever Such a wise and wonderful father makes this place a paradise

[Juno and Ceres whisper and send Iris on employment] [Juno and Ceres whisper and send Iris on employment]

PROSPEROSweet now silence

PROSPEROMy dear Ferdinand speak no more Juno and Ceres are

Juno and Ceres whisper seriouslyTheres something else to do Hush and be muteOr else our spell is marred

whispering with a solemn look There is something else coming Silence Or else our magic will be spoilt

IRISYour nymphs called naiads of the wandering brooksWith your sedged crowns and over-harmless looksLeave your crisp channels and on this green landAnswer your summons Juno does commandCome temperate nymphs and help to celebrateA contract of true love Be not too late

IRISYou nymphs called Naiads denizens (M inhabitants) of the running stream with your chaplets of sedge and ever-helpful looks leave your wrinkled channels and on the green land answer the summons sent to you Juno has ordered some chaste nymphs and help to celebrate a noble and true marriage Donrsquot delay

[Enter certain nymphs] [Enter certain nymphs]You sunburnt sicklemen of August wearyCome hither from the furrow and be merryMake holiday your rye-straw hats put onAnd these fresh nymphs encounter every oneIn country footing

You sunburnt harvesters weary from the effects of the heat in August come here from the furrowed land and rejoice Make holiday with your rye-straw hats upon you and meet these fresh nymphs and join in country dancing

[Enter certain reapers properly habited They join with the nymphs in a graceful dance towards the end whereof Prospero starts suddenly and speaks]

[Enter certain reapers properly habited They join with the nymphs in a graceful dance towards the end whereof Prospero starts suddenly and speaks]

PROSPERO[aside] I had forgot that foul conspiracyOf the beast Caliban and his confederatesAgainst my life The minute of their plotIs almost come [to the spirits]Well done Avoidno more

PROSPERO(Aside)I had forgotten the wicked conspiracy of the beast Caliban and his accomplices against my life the time of their plot has almost arrived ndash (To the Spirits) well done depart no more of this

[To a strange hollow and confused noise the spirits heavily vanish]

[The spirits depart]

ORIGINAL TEXT PARAPHRASEFERDINANDLet me live here everSo rare a wondered father and a wifeMakes this place paradise

FERDINANDI should like to live here forever Such a wise and wonderful father makes this place a paradise

[Juno and Ceres whisper and send Iris on employment] [Juno and Ceres whisper and send Iris on employment]

PROSPEROSweet now silenceJuno and Ceres whisper seriouslyTheres something else to do Hush and be muteOr else our spell is marred

PROSPEROMy dear Ferdinand speak no more Juno and Ceres are whispering with a solemn look There is something else coming Silence Or else our magic will be spoilt

IRISYour nymphs called naiads of the wandering brooksWith your sedged crowns and over-harmless looksLeave your crisp channels and on this green landAnswer your summons Juno does commandCome temperate nymphs and help to celebrateA contract of true love Be not too late

IRISYou nymphs called Naiads denizens (M inhabitants) of the running stream with your chaplets of sedge and ever-helpful looks leave your wrinkled channels and on the green land answer the summons sent to you Juno has ordered some chaste nymphs and help to celebrate a noble and true marriage Donrsquot delay

[Enter certain nymphs] [Enter certain nymphs]You sunburnt sicklemen of August wearyCome hither from the furrow and be merryMake holiday your rye-straw hats put onAnd these fresh nymphs encounter every oneIn country footing

You sunburnt harvesters weary from the effects of the heat in August come here from the furrowed land and rejoice Make holiday with your rye-straw hats upon you and meet these fresh nymphs and join in country dancing

[Enter certain reapers properly habited They join with the nymphs in a graceful dance towards the end whereof Prospero starts suddenly and speaks]

[Enter certain reapers properly habited They join with the nymphs in a graceful dance towards the end whereof Prospero starts suddenly and speaks]

PROSPERO[aside] I had forgot that foul conspiracyOf the beast Caliban and his confederatesAgainst my life The minute of their plotIs almost come [to the spirits]Well done Avoidno more

PROSPERO(Aside)I had forgotten the wicked conspiracy of the beast Caliban and his accomplices against my life the time of their plot has almost arrived ndash (To the Spirits) well done depart no more of this

[To a strange hollow and confused noise the spirits heavily vanish]

[The spirits depart]

Ac-12 27420 topic Revaluation of Assets and Liabilities

REVALUATION OF ASSETS AND LIABILITIES

On admission of a new partner the firm stands reconstituted and consequently the assets are revalued and liabilities are reassessed It is necessary to show the true position of the firm at the time of admission of a new partner If the values of the assets are raised gain will increase the capital of the existing partners Similarly any decrease in the value of assets ie loss will decrease the capital of the existing partners For this purpose alsquoRevaluation Accountrsquo is prepared This account is credited with all increases in the value of assets and decrease in the value of liabilities It is debited with decrease on account of value of assets and increase in the value of liabilities The balance of this account shows a gain or loss on revaluation which is transferred to the existing partnerrsquos capital account in existing profit sharing ratioAccounting for Revaluation of Assets and Liabilities when there is a Changein the Profit Sharing Ratio of Existing PartnersAssets and liabilities of a firm must also be revalued at the time of change in profit sharing ratio of existing partners The reason is that the realisable or actual value of assets and liabilities may be different from those shown in the Balance Sheet It is possible that with the passage of time some of the assets might have appreciated in value while the value of certain other assets might have decreased and no record has been made of such changes in the books of accounts Similarly there may be some unrecorded assets amp libilities that may have to be accounted for Revaluation of assets and reassessments of liabilities becomes necessary because the change in the

value of assets and liabilities belongs to the period to change in profit sharing ratio and hence must be shared by the partners in their old profit sharing ratio Revaluation of assets and reassessment of liabilities may be given effect to in two different ways (a) When revised values are to be recorded in the books and(b) When revised values are not to be recorded in the books

When revised values are to be recorded in the booksIn such a case revaluation of assets and reassessment of liabilities is done with the help of a new account called lsquoRevaluation Accountrsquo Sometimes this account is also called as lsquoProfit amp Loss Adjustment Acrsquo If there is a loss due to revaluation revaluation account is debited and if the revaluation results in a profit the revaluation account is credited The following journal entries made for this purpose are

(i) For increase in the value of assetsAsset Ac Dr (individually)To Revaluation Ac(ii) For decrease in the value of AssetRevaluation Ac Dr (individually)To Asset Ac[Decrease in the value of assets](iii) For increase in the value of LiabilitiesRevaluation Ac Dr (individually)To Liabilities Ac[Increase in the value of Liabilities](iv) For decrease in the value of LiabilitiesLiabilities Ac DrTo Revaluation Ac[Decrease in the value of Liabilities](v) For unrecorded AssetsAsset Ac [unrecorded] DrTo Revaluation Ac[Unrecorded asset recorded at actual value](vi) For unrecorded Liability Revaluation Ac DrTo Liability Ac [unrecorded][Unrecorded Liability recorded at actual value](vii) For transfer of gain on revaluationRevaluation Ac DrTo Existing Partnerrsquos CapitalCurrent Ac[Profit on revaluation transferred to capital account in existing ratio](viii) For transfer of loss on revaluationExisting Partnerrsquos CapitalCurrent Ac DrTo Revaluation Ac[Loss on revaluation transferred to capital account in existing ratio](a) When revaluation account shows gain Revaluation Ac DrTo Partnerrsquos Capital Ac (Old Profit Sharing Ratio)(Profit on revaluation credited to Partnerrsquos Capital Ac)(b) Above entry is reversed when revaluation account shows loss Partners Capital Acs (Old Profit Sharing Ratio) DrTo Revaluation Ac(Loss on revaluation debited to Partnerrsquos Capital Acs)

Proforma of Revaluation Account is given as under

Revaluation Account

Dr Cr Particulars ` Amount Particulars ` Amount To Decrease in value of assets By Increase in value of assets To Increase in value of liabilities By Decrease in value of liabilities To Unrecorded liabilities By Unrecorded assets To Gain on Revaluation (Transferred) By Loss on Revalution (Transferred)

ECO ndash12 2742020Topic- ELASTICITY OF DEMAND

CHAPTER - ELASTICITY OF DEMANDMEANINGDemand for a commodity is affected by many factors such as its price price of related goods income of its buyer tastes and preferences etc Elasticity means degree of response Elasticity of demand means degree of responsiveness of demand Demand for a commodity responds to change in price price of related goods income etc So we have three dimensions of elasticity of demandDIMENSION OF ELASTICITY OF DEMAND TYPES OF ELASTICITY OF DEMAND

Price elasticity of demand Income elasticity of demand Cross Elasticity of demand

Price elasticity of demand Price elasticity of demand means degree of responsiveness of demand for a commodity to the change in its price For example if demand for a commodity rises by 10 due to 5 fall in its price Price elasticity of demand (ep)=Percentage change in quantity demanded Percentage change in price of the commodity = 10 ( -)5 = ( - )2Note that ep will always be negative due to inverse relationship of price and quantity demanded

(ii) Income elasticity of demand Income elasticity of demand refers to the degree of responsiveness of demand for a commodity to the change in income of its buyer Suppose income of buyer rises by 10 and his demand for a commodity rises by 20 then Income elasticity of demand (ey)= change in quantity demanded change in price of the commodity =20 10 = 2

Cross Elasticity of demandCross elasticity of demand means the degree of responsiveness of demand for a commodity to the change in price of its related goods (substitute goods or complementary goods) Suppose demand for a commodity rises by 10 due to 5 rise in price of its substitute good then Cross elasticity of demand (ec) = change in quantity demanded change in price of related good = 10 2 = 5 (Tastes and preferences cannot be expressed numerically So elasticity ofdemand cannot be numerically expressed)

  • Chapter 1 Force (Summary)
  • Distinguish between external sovereignty and internal sovereignty
    • NAND Gate
      • Logic diagram
      • Truth Table
        • NOR Gate
          • Logic diagram
          • Truth Table
            • XOR Gate
              • Logic diagram
              • Truth Table
                • XNOR Gate
                  • Logic diagram
                  • Truth Table
                      • Physics
                      • Chapter 1 Electric Field ( Electric Dipole) (Summary)
Page 9:  · Web viewWe all know that Nouns are divided into two parts: common noun and proper noun.Apart from common and proper noun, we will also study about collective noun and compound

WINDOWS 10 HAS 1187 WITH WEARING FEATURES SETS AND INTENDED HARDWARE

THE VARIOUS EDITIONS AVAILABLE FOR WINDOWS 10 ARE AS FOLLOWS

WINDOWS 10 HOME

WINDOWS 10 HOME IS THE EDITION OF WINDOWS 10 THAT CONTAINS THE LEAST NUMBER OF FEATURESWINDOWS 10 HOME IS THE CONSUMER FOCUSED DESKTOP EDITION WITH SUPPORT FOR BOTH PC AND TOUCH ENABLED TABLETS WINDOWS 10 HOME IS AVAILABLE IN BOTH 32 BIT AND 64 BIT VERSIONS

WINDOWS 10 PRO

WINDOWS 10 PRO HAS FEATURES ESSENTIAL FOR SMALL OR MEDIUM BUSINESSESIT HAS A LARGE NUMBER OF EXTRA FEATURES TO MEET THE NEEDS OF SMALL BUSINESSES

WINDOWS 10 ENTERPRISE

WINDOWS 10 ENTERPRISE IS BUILT ON WINDOWS 10 PROIT HAS FEATURES WHICH ARE DESIGNED TO MEET THE NEEDS OF MEDIUM AND LARGE ORGANISATIONSTHIS WINDOWS EDITION TARGETS THE ENTERPRISE SEGMENT OF THE MARKET

WINDOWS 10 MOBILE

WINDOWS 10 MOBILE IS DESIGNED FOR CUSTOMER ORGANISATION THAT ARE USING THE WINDOWS 10 PLATFORM ON SMARTPHONES AND SMALL TABLETS

WINDOWS 10 EDUCATION

WINDOWS 10 EDUCATION IS DESIGNED TO MEET THE NEEDS OF STAFF ADMINISTRATORS TEACHERS AND STUDENTS OF SCHOOLS

WINDOWS 10T EDITION

WINDOWS 10T EDITION OF WINDOWS 10 IS DESIGNED FOR USE IN SMALL FOOTPRINT AND LOW COST DEVICES

Subject English language

Topic Noun kinds

NOUN KINDS

Noun is a naming word Nouns are names of people places animals or things

For example Ashley Mumbai tiger pencils

Kinds of nouns

Proper noun the name of a particular person place or thing is called proper nounExample

1 The Himalayas stand to the north of India2 Monika is the school captain

Common noun announce that names people place or thing in general is called a common noun Example

1 Kalidas was the greatest dramatist of India2 William Shakespeare is Englands national poet

Collective Noun A collective noun is the name of a collection of people or things taken together and spoken of as a whole Example

1 The feet were completely destroyed in the fierce attack2 The crew revolted against Captain Grand

Abstract Noun An abstract noun is the name of some quality state or idea Example1 Diversity Indias biggest strength2 Patience is a virtue

_____________________________________________________________________________________

Class VII

Subject English 1 Topic Articles

ARTICLES

There are three articles ------ A An The

1 Indefinite articles- The Articles a and an are called Indefinite Articles They do not point to a particular person or thing They are used with singular countable nouns It does not identify a specific noun

Uses of Indefinite articles A

Before a singular noun beginning with a consonant or a vowel with a consonant sound For example a cattle a university a one way track

Before a proper noun which is either unfamiliar or holds a special meaning For example For example A Mahesh Sharma wants to see youNeela wants to be a Tendulkar when she grows up

With a number beginning with a consonant sound For example a ten- rupee note a hundred years

Before half when half follows a whole number or after half when it isnrsquotExample one and a half litres (before) Half a litre (after)

With an expression of quantity For example a lot of time a dozen oranges a little while In exclamation before nouns Example Such a mess What a pity

Uses of Indefinite articles An

Before a singular noun beginning with a vowel sound Example an ant an egg an umbrella Before an abbreviation beginning with a vowel or a consonant with the vowel sound

Example an MP an MA Before the word beginning with a silent h Example an heir an hour an honour

Note We do not say a milk or a lemonade because they are uncountable nouns ever we see a cup of milk or a glass of lemon and

Subject GEOGRAPHY

CHAPTER 1 REPRESENTATION OF GEOGRAPHICAL FEATURES

TOPIC- TOPOGRAPHICAL MAPS

Topographical Maps- topographical maps are small-scale maps with detailed depiction of both natural and human-made features

Conventional colours-different colours used in a map are known as conventional colours

TYPES OF SCALE-

Verbal Statement- Scale written in a statement from like 2 cm to a Km is called verbal statement It means that 2cm on the map represents 1 km on the Earth

Representative Fraction- Scale can also be written as a representative fraction (RF)

Example 150000 it means 1cm on the map represents 50000cm or frac12 km on the ground It shows the ratio between map distance and ground distance

Linear Scale Scale can be drawn on a line to show map distance equivalent to ground distance A line is divided into equal parts Each parts represents the actual distance on the ground in mkm

SUBJECT-COMPUTER

CHAPTER-1 (COMPUTER FUNDAMENTALS)

COMPUTER LANGUAGES

THE THE TERM COMPUTER LANGUAGE REFERS TO A SYSTEM OF RULES AND SYMBOLS THAT ARE DESIGNED TO GIVE INSTRUCTIONS TO A COMPUTERTHE COMPONENTS OF A COMPUTER SYSTEM CANNOT PERFORM A TASK BY THEMSELVES THEREFORE THEY HAVE TO BE GIVEN INSTRUCTIONS TO PERFORM ANY TASKCOMPUTER LANGUAGES ARE USED TO CREATE PROGRAMS USING LOGIC BASED ALGORITHMSTHE RULES OF A COMPUTER LANGUAGE IS

KNOWN AS SYNTAX WHEREAS THE TERM SEMANTICS REFERS TO THE MEANING OF LANGUAGESCOMPUTER CANNOT DEVELOP ANY PROGRAMMING LANGUAGE BY THEMSELVES AND CANNOT THINK INTELLIGENTLY UNLESS THEY ARE ASSISTED BY THE HUMAN BEINGS THEREFORE THE PROGRAMMERS DEVELOP A SET OF METHODS AND TECHNIQUES A SET OF INSTRUCTIONS USED TO PERFORM A SPECIFIC TASK IS CALLED A PROGRAM

A PROGRAMMING LANGUAGE IS ALSO KNOWN AS A COMPUTER LANGUAGE CODED BY PROGRAMMERS TO WRITE INSTRUCTIONS FOR A COMPUTERTHE COMPUTER TAKES THESE INSTRUCTIONS AS INPUT AND PRODUCES THE DESIRED OUTPUT

TYPES OF COMPUTER LANGUAGE

THE COMPUTER LANGUAGE CAN BE BROADLY DIVIDED INTO TWO CATEGORIES

LOW LEVEL LANGUAGES HIGH LEVEL LANGUAGES

LOW LEVEL LANGUAGES

A LOW LEVEL PROGRAMMING LANGUAGE REFERS TO THE LANGUAGE THAT IS UNDERSTOOD BY A COMPUTER DIRECTLYTHE PROGRAMMER MUST HAVE AN IN-DEPTH KNOWLEDGE OF DIFFERENT COMPUTERS TO WRITE PROGRAMS IN A LOW LEVEL LANGUAGE THE TWO TYPES OF LOW LEVEL LANGUAGES ARE MACHINE LANGUAGE AND ASSEMBLY LANGUAGE

MACHINE LANGUAGE

COMPUTER HARDWARE UNDERSTANDS ONLY MACHINE LANGUAGE AS IT IS THE FUNDAMENTAL LANGUAGE OF A COMPUTER WHICH UNDERSTANDS ONLY THE TWO DIGITS 0 AND 1 KNOWN AS THE BINARY DIGITSIT IS A SYSTEM OF INSTRUCTIONS EXECUTED DIRECTLY BY THE CPU WITHOUT ANY TRANSLATION AND HENCE IS EXECUTED QUICKLY AS COMPARED TO PROGRAMS IN OTHER LANGUAGES PROGRAMS IN MACHINE LANGUAGE NEED DIFFERENT BINARY LANGUAGE PROGRAMS TO SOLVE THE COMPLEX TASKSPROGRAMMERS OFTEN FIND IT DIFFICULT TO WRITE PROGRAMS IN THIS LANGUAGE

ASSEMBLY LANGUAGE

AN ASSEMBLY LANGUAGE IS A LOW LEVEL PROGRAMMING LANGUAGE BUT THE INSTRUCTIONS ARE WRITTEN IN THE FORM OF WORDS KNOWN AS MNEMONICS TO CONVERT AN ASSEMBLY LANGUAGE INTO TO MACHINE CODE WE REQUIRE A UTILITY PROGRAM WHICH IS KNOWN AS AN ASSEMBLER ASSEMBLY LANGUAGE PROGRAMS CANNOT BE EXECUTED DIRECTLY BY A COMPUTER THEREFORE THEY ARE SLOWER THAN THE MACHINE LANGUAGE PROGRAMSIT IS EASIER TO WRITE PROGRAMS IN THE MACHINE LANGUAGE THEN AND IN ASSEMBLY LANGUAGE

_______________________________________________________________________________________________

CLASS-VIII

SUBJECT-COMPUTER CHAPTER - Operating system and graphical user interface Role and functions

Need of operating system

It co-ordinates different hardware and software componentsof a computer system It supervises the various actions of the computer system and enables the computer to work in a effective

manner It helps in smooth functioning of various peripherals

In a multi-tasking operating system it determines the order and time to be allowed for each application before giving another application a turn

It sends messages to the system operator about the status of operation or any error that may have occurred while running the application

What is operating system

An operating system is an integrated system of programs that manages various resources and the overall operation of the computer system It is designed to support various activities of computer system in a systematic way

Role of an operating system-

Operating system enables the user to use the system effectively An operating system manages various application that runs on a computer and shares computerrsquos resources User interacts with operating system through command line interface and graphical user interface

Function of an operating system -

Booting the computer it is the start up procedure of a computer system Loading theprograms in the memory - when system is ready the operating system loads certain program

automatically Manages resources it manages between the hardware and software resources Detecting and correcting errors- if the supporting hardware or software doesnrsquot works properly then the

operatingsystem tries to rectify it Ensuring data security programs and data donot interfere with each other Maintaining the internal clock of system - maintain internal clock of system when system is shut down

SUBJECT-GEOGRAPHY CHAPTER 1 REPRESENTATION OF GEOGRAPHICAL FEATURES THROUGH CONTOURSTopographical maps- topographical maps are small-scale maps with detailed depiction of both natural and human-made features

Contour lines- contour lines are imaginary lines drawn on a map joining places having the same height above the mean sea level

Contour linesContour interval- It is an interval at which contour lines are drawnIndex contour- At every 100-m interval a thick brown contour line is called index contour It is drawn for calculating height

Spot heights- The spot heights show heights in metres above the mean sea level

SUBJECT- English Language CHAPTER - The Sentence

Complex sentence-

We have seen that a Complex Sentence consists of a Principal Clause with one or more Subordinate Clauses

We have also learnt that there are three kinds of Subordinate Clauses The Adjective Clause the Adverb Clause and the Noun Clause

1) THE ADJECTIVE CLAUSE

An ADJECTIVE CLAUSE does the work of an ADJECTIVE It qualifies a noun or pronoun In the following examples in each set two commonly patterned sentences are compared one with an ADJECTIVE and the other with an ADJECTIVE CLAUSE ( The Adjective or Adjective Clause qualifies the Noun placed in the box)

a) He is a lazy boy (Adjective)

b) He is a boy who is lazy (Adjective Clause)

a) It is a beautiful place for the tourists (Adjective)

b) It is a place where tourists come for the scenic beauty (Adjective Clause)

a) We have enough funds for the work (Adjective)

b) We have funds which would be enough for the work (Adjective Clause)

Convert the sentence from simple to complex sentence-

1) We believe his honesty Ans- We believe that he is honest

2) This is the birth place of RamaAns- This is the place where Rama was born

____________________________________________________________________________________________

Class IX

Subject English Language

Topic Preposition

Date 270420

PREPOSITIONS

A preposition is a word placed before a noun or a pronoun to show in what relation the person or thing denoted by it stands in regard to something else

EXAMPLE

There is a clock on the wall

The preposition on shoes relationship between clock and the wall

Smitha is afraid of lizards The man jumped off the bus

Here the preposition of shows the relationship between afraid and lizards

The preposition off shows the relation between jumped and bus

The noun or the pronoun which follows a preposition is called its object

So in the first Example wall is the object of the preposition on

A preposition can have more than one object For example

The plane flew overhouses and meadows

Prepositions are used to express a number of relationship including time locationmannermeans quantity purpose and state or condition

Points to remember

Preposition joins a noun to another noun or a pronounThere is a cow in the field

A preposition joints a noun to a verbThe cat runs after the rat

Preposition can have two or more than two objectsThe road runs over hills and plain

A preposition also joins a noun an adjectiveHe is fond of tea

Generally a preposition comes before an object Sometimes it comes even after an object asWhat are you looking atThis is the house I live in

Subject- Computer Application

Chapter 2 Introduction to Java

Java API An application programming interface (API) in the context of Java is a collection of prewritten packages classes and interfaces with their respective methods fields and constructors

Byte Code Java bytecode is the result of the compilation of a Java program an intermediate representation of that program which is machine independent The Java bytecode gets processed by the Java virtual machine (JVM) instead of the processor JVM The Java Virtual Machine (JVM) is the runtime engine of the Java Platform which allows any program written in Java or other language compiled into Java bytecode to run on any computer that has a native JVM

Platform A platform is the hardware or software environment in which a program remains

Java platform The Java platform differs from most other platforms in that itrsquos a software only platform that runs on top of other hardware-based platforms

Applet and Application The fundamental difference between the two Java programs is that an application program is designed to run on a stand-alone machine whereas an applet is a web-version of an application which is used to run a program on a web browser

WORA ldquoWrite once run anywhererdquo (WORA) or sometimes write once run everywhere (WORE) is a slogan created by Sun Microsystems to illustrate the cross-platform benefits of the Java language

Class XSubject Topic Summary Execution

ECONOMICSFACTORS OF PRODUCTION Sub-topiclsquoLABOURrsquo

We shall start our class by discussing the topic taught in the last class

lsquo DIVISION OF LABOURrsquo- By division of labour we mean specialization in workIt refers to splitting up the work of labour involved in the production of a particular commodity into several parts and each part and sub-part is performed by a specialist

Now let us start by the Advantages of Division of labouraIt increases the level

Questions

1 Differentiate between Product-Based division of labour and Process-Based division of labour

Product-Based division of labour

Process-based division of labour

It is also known as simple (or occupational) division of labourUnder it everybody performs a particular occupations The entire is done by the same person

When a person or group of persons undertakes a specialised function which is supplementary to the production of final commodity and service This is also as complex division of labour

It is simple It is complex

of productionbSince the product is produced by an expert workerbest quality of product is producedcIt saves time and toolsd it promotes inventions in the methods and techniques of productioneIt leads to reduction in costs fAll workers get work according to their abilities and choices

Now let us discuss the disadvantages of Division of labour

a Since many workers are involved in the production of a commodity no one has the sense of responsibility

b The constant and repetition of the same work again and again make the work monotonous

c Division of labour facilitates production on large scale Hencethere is fear of over production

d Because of territorial division of labour some areasregions become more developed than others

It is based on labour-intensive techniques of production

It is based on capital-intensive techniques

It is generally found in small enterprises

It is generally found in large enterprises

Example Indian farmers doing all farm activities

Example A modern garmet factory where one person takes the measurementanother does the cuttingsome sew the clothes while a few workers button them and other iron them

2 What are the advantages of Division of labouraIt increases the level of productionbSince the product is produced by an expert workerbest quality of product is producedcIt saves time and toolsd it promotes inventions in the methods and techniques of productioneIt leads to reduction in costs fAll workers get work according to their abilities and choices

3Discuss the disadvantages of Division of laboura Since many workers are involved in the

production of a commodity no one has the sense of responsibility

b The constant and repetition of the same work again and again make the work monotonous

c Division of labour facilitates production on large scale Hence there is fear of over production

d Because of territorial division of labour some areasregions become more developed than others

English 1 Transformation of sentences

Sentences A sentence is a group of words which makes complete sense

a Assertive sentences

Exercise 6Rewrite the following sentences according to the instructions given below without changing their meanings

1 As soon as he saw the beer he jumped into

b Imperative sentences

c Interrogative sentences

d Exclamatory sentences

Sentences can be changed from one grammatical form to another without changing the meaning of the sentence This is known as transformation of sentences

the river ( Begin No sooner)2 None but brave deserve the fair (Begin the

bravehellip)3 This box is too heavy for me to lift ( Use so hellip

That instead of too)4 No one other than a king can live like James

Luxurious ( Begin only James)5 Oh for the wings of a dove (Begin I wishhellip)

Math Topic Commercial MathematicsChapter Shares and Dividends

Study item Discuss about shares and Dividends1) What is share

Ans To start any big business (company or Industry) a large sum of money is needed But it is not possible for an individual to invest such a large amount Then some persons interested in the business join together and from a company They divide the estimated money required into small parts Each such part is called a share

2) What do you mean by the term shareholder

Ans A person who purchases one or more shares is called shareholder3) Some terms related with a share

(i) Nominal value or face value or printed value The original value of a share is called its nominal value or face value or printed value

Note The nominal value of a share always remains same(ii) Market value or cash value The price of a share at any

time is called its market value or cash value

Note The market value of a share changes from time to time(iii) At par If the market value of a share is the same as its

nominal value the share is called at par(iv) At Premium or above Par If the market value of a share

is more than its nominal value the share is called at premium or above par

Example If a share of Rs 100 is selling at Rs 150 then it is said to be selling at a premium of Rs 50 or Rs 50 above par

(v) At Discount or below par If the market value of a share is less than its nominal value the share is called at discount or below par

Example If a share of Rs 100 is selling at Rs80 then it is said to be selling at a discount of Rs 20 or at Rs 20 below par

4) What is Dividend

Ans The profit which a shareholder gets for hisher investment from the company is called dividendNote (i) The dividend is always expressed as the percentage of the face value of the share(ii) The dividend is always given( by the company ) on the face value of the share

irrespective of the market value of the shareBENGALI(2ND LANGUAGE)

ldquoদেবতোর জণমrdquoলিবরোম চকরবত

পরথম লিসর পোঠ-চোর পসথ একটি পোথর লিবপলি ঘটোয় দেক যোতোয়োসতর পসথ পরলিতলিয়ত ওই পোথসর দেো োচট দেসত একলি দেতো দেক দেো োচট দেসয় দেবোমো সয় রোসতোর মোস লি0টসক পস1 লিবপরীত লিক দেথসক আো একটি দেমোটর োলি1 চোসকর কষতোয় পরোসরণ দেবোসচ যো লিকনত পরলিতবোর এমরণ দেৌভোয দেসকর োও সত পোসর তোই লিতলি দেকোো দেজোো1 কসর পোথরটিসক উপস1 দে8স পোথর উপস1 দে8োর ময় এক দেকৌতী জতো দেকসক পরশন কসর দেয লিতলি দেকোসো দেবতোর আস দেপসয়স0রণ লিকো লিকনত দেক বস লিতলি দেকোসো দেবতোর আস পোলি দেক উপলিত ক জতোর উসltসয বস কোরও ইস= স পোথরটি লিসয় দেযসত পোসর এর পর দেথসক দেক দেযসত আসত পোথরটি দেক দেসত পো একলি দেক কষয করস দেকউ পোথরটিসক ধসয়মস0 পলিরসকোর করস0 দেক ওই দেকৌতী জতোসক পোথসরর কোস0 বস থোকসত দেস এ0ো1ো আরও কষয কসর দেক ঠোৎ ওই লি1 পোথরটির োসয় লিোর োো-দেকউ পজো কসরস0 দেকৌতী দেোকটির আঙকো য় যলিসক উ পোথরটিসক লিরসয় দে8স তোর পর ঠোৎ একলি পোথরটির দেোোজ দেই দেক লিসয় দেস0 বো দেকোথোয় দেস0

বদোথ-

দেো োচট ndashচসত লিসয় দেকো লিক0র সE ধোককো দেস পস1 যোবোর উপকরমঅকসমোৎ- ঠোৎআতমমবর- লিসজসক লিয়নতরপ-পো লিপ0স প1োদেসতসসত- লিসপলিউৎোত- দেো1ো দেথসক উপস1 দে8োপরতযয় ndash লিবশবোপরতযোস- টতযোস ndash দৈবোসধসতোধলিসত- পরসপসরর পরলিত ব পরসয়ো করোপরসতরীভত- পোথসর পলিররণলিতবোনতঃকরসরণ- মস পরোসঅলিQৎ- অQো করসত ই=কদেোপ- বধমোর- দেবোস এমইতযোকোর- এইরকমরম- বময়পযসোভী-পসযর জয দেোভ আস0 যোরপোসথয় ঞচয়- পথচোর রচ জমোসোমোমোসরোস- ব ধম ধোসমর সE

তোর লি দেইhelliphellipTo be continued

Hindi 2nd lang सर क पद(सरदास)

सरदास शरी कषण भकति कावय क सरवशरषठ कगिरव ह इनक जनम और मतय क समय तथा सथान का मतभद हसरदास रवातसलय और शरार रस क अनयतम कगिरव ह इनक कावय म बालकषण क सौदय चपपल चषटा और गि7याओ की मनोहर झाकी मिमलती ह कषण और ोगिपयो क अननय परम का कतिचतरण ह सयो शरार की अपकषा उनक कावय म गिरवयो शरार का अमिBक गिरवषय और मारमिमEक कतिचतरण हआ हइन पकतियो म हम सरदास की भकति भारवनाओ का परिरचय मिमलता ह इनका सपण सगरह सरसार म गिनगिहत ह

1 जसोदा हरिर पालन झलारवहलरारवदलराईमलहारव रव जो ईसाई कछ ारव मर लाल को आई निनEदिदया काह ना आगिनसबारव त काह नाही बरवगिह आरवतोको कानहा बलारव

शबदाथ-हलरारव-गिहलती हदलराई - दलार पयार करती हमलहारव-पचकारती हनिनEदरिरया ndashनीदरवगिह-जलदी सअBर-होठमौन-चपसन-सकत

वयाखया- सरदास जी कहत ह गिक यशोदा माता बालक कषण को पालन म झल आती ह रवह उनह गिहलाती ह पयार करती ह मलहार जस कोई ीत ान लती ह और नीद स पछती ह गिक ह नीद तम मर लाल को आकर कयो नही सलाती तझ खाना बला रहा ह कभी कषण आख बद कर लत ह कभी आखफडफडान लत ह उनह सोता हआ जानकर यशोदा माता चप हो जाती ह और इशार म बात करन लती ह इसी बीच अकला कर कषण ज जात हतो गिफर यशोदा माता गिफर स ाना ान लती ह सरदास जी कहत ह गिक भरवान क दशन का सख दरवता और ऋगिष-मगिनयो को भी दलभ ह यही सख माता यशोदा को बडी सहजता स मिमल जा रही ह माता यशोदा बहत ही भागयशाली ह2)Continue to nexthellip

Physics

Chapter 1 Force

(Summary)

Question A body is acted upon by two forces each of magnitude F but in opposite directions State the effect of the forces if

(a) Both forces act at the same point of the body

(b)the two forces act at two different points of the body at a separation r

Solutions

(a) Resultant force acting on the body = 0

F ndash F = 0(b) The forces tend to rotate the body between two forces about the midpoint

Moment of forces = F times rFr

QuestionDefine moment of a couple Write its SI unit

Solutions

Moment of couple is equal to the product of both force and the perpendicular distance between the two forces

The SI unit of moment of couple is NmCommercial Studies

Advertising and sales

Business firms use several methods to

Questions1) What do you mean by advertising

promotion create demand of their product in the market and increase it sales Such methods comprises of advertising sales promotion personal selling and publicityToday we are going to discuss about one of such methods It is advertising

Meaning of advertisingAdvertising is a paid form of non-personal presentation for promotion of Ideas goods and services

Importance or merits of advertising Advertising has importance to manufacturer or traders to customer and to society as a whole

Today we will see how advertisement help the manufacturer or traders

Answer) Advertising is a means of how a company encourages people to buy their products services or ideas It is one element of marketing which also includes design Research and data mining

2) Mention any three features of advertisingAnswer)The main features of advertising are

i) It is impersonal form of presentation for promotion of products and services of Ideas

ii) It is issued by identified sponsor The advertisement contains the name of the advertiser

iii) It is a form of mass communication because the message is directed to a large number of persons simultaneously

3) Mention the main merits or importance of advertisement to manufacturer or tradersAnswer)

i) Introducing new product A business organization can introduce itself and its products to the public through advertising

ii) Increase the sale Advertising leads to increase the sale of existing product by entering into new markets and attracting new customers

iii) Create steady demand Advertising creates sustains regular demand by smoothening out seasonal and other fluctuations It enables regular production for the organisation

iv) Economics of scale Advertising facilitate mass distribution of goods and steady demand which lead to large scale and regular production

v) Goodwill Advertising helps in creating a good image of the firm and reputation for its products

Biology Chapter - 03Genetics

Today wewill start chapter and discuss about Genetics Gregor Mendel is known as father of genetics Before entering into Mendelrsquos experiment on Genetics we must know

Q1 Define the following termsi) Genetics Genetics is the study of

transmissionof body features from parents to offspringand the laws relating to such transmission

ii) Heredity It may be defined as transmissionof genetically based characteristics from parentsto offspring

iii) Character and traits Any heritable

Importance to TraderIntroducing new productIncrease the saleCreate steady demandEconomics of scaleGoodwill

some terms featureis a character The alternative forms of acharacter are called traitsex Character (Hair shape) - Traits (Curly straight)

iv) Homologous chromosomes A pair ofcorresponding chromosomes of the same shapeand size one from each parent

v) Genes Genes are the specific parts (DNA segments) of a chromosome which determinethe hereditary characteristicsNearly 30000genes present in human

vi) Alleles Alternative forms of a gene occupying the same position (locus) on homologouschromosomes and affecting the same characteristicbut in different ways

vii) Genotype ndash PhenotypeGenotype means of genes present in the cells of an organism Phenotype means the observable characteristic which is genetically controlled

viii) Mutation It is a sudden change in one or more genes or in the number or in the structure of chromosomes ex Sickle cell anaemia is a blood disease caused by a gene mutation

CLASS NOTES

Class XSubject Eng Literature (The Merchant of Venice ndash William Shakespeare)Topic Act IV Scene 1 Lines 01 to 34 ( Duke helliphelliphellip We all expect a gentle answer Jew) ate 27th April 2020 (2nd Period)

[Students should read the original play and also the paraphrase given in the school prescribed textbook]Summary Questions amp Answers

This scene may be termed as the catastrophe of the play It is the final unravelling of the complicated events which seem to threaten the happiness of Bassanio Portia and Antonio Right is justified to the fullest degree and malice falls into the trap prepared for others No one suffers here but Shylock but even then he receives a generous measure of mercy

o This is the Court-scene Initially we meet

(1)

DUKE I am sorry for thee thou art come to answer (Line 3-6)A stony adversary an inhuman wretchUncapable of pity void and emptyFrom any dram of mercy

(i) Who is addressed here Where is the person Why is the person there

Antonio is addressed hereAntonio is in the court of justice at VeniceAntoniorsquos trial is scheduled to be held here for his failure to meet the conditions of the bond he signed with Shylock

the Duke Antonio Shylock and Salerio Later we meet Bassanio Portia Gratiano and Nerissa

o The Duke says to Antonio that he has to face a very cruel opponent which Antonio admits and expresses his gratefulness to the Duke for his efforts to soften without result the heart of Shylock in order to be merciful to Antonio Antonio further says that he is ready to accept whatever cruel judgement the Court may award

o When Shylock appears in the court the Duke says that Shylock should change his decision of prosecuting Antonio and demanding the penalty specified in the bond out of consideration of the great misfortunes that Antonio has suffered If this is done by him (Shylock) the whole court would be gladdened by his merciful action

(ii) What is the Duke sorry for

The Duke is unable to change the mind of Shylock from his decision to get the bond forfeited even after he pleaded to Shylock Shylock stands firmly for his bond which when forfeited will allow him to take a pound of flesh from any part of Antoniorsquos body(iii) How does the Duke address Shylock`The Duke calls Shylock an adversary with a heart of stones He calls Shylock as an inhuman wretch without pity Shylock is quite lacking in the slightest quality of mercy (iv) How does Antonio reply to this

Antonio replies that he will meet the revenge of Shylock patiently He has prepared himself to suffer with a quiet spirit the utmost that Shylockrsquos tyranny and rage can do(v) What quality of the Duke is revealed here

The Duke is kind and benevolent He is ready to help Antonio He requests Shylock to free Antonio from the trial(vi) What are the terms of the bond that Antonio has signed

The terms of the bond that Antonio has signed were that if Antonio is unable to repay Shylock a certain sum of money specified on the paper on a certain date and in an agreed place the forfeit has to be paid The forfeiture will be an exact pound of Antoniorsquos flesh which Shylock will be a liberty to take from any part of Antoniorsquos body which pleases him

Class XI

STUDY MATERIAL

Class XISubject Eng Literature (The Tempest ndash William Shakespeare) Topic Act I Scene 2 Lines 88 to 132 (Prospero hellip Me and thy crying self) Date 27th April 2020 (3rd Period)

[Students should read the original play and also the paraphrase given in the school prescribed textbook]Summary Questions amp Answers

o Prospero now tells Miranda that he was the Duke of Milan He had been devoting himself more to studies than the affairs of the State His brother Antonio took advantage of this situation and with the help of Alonso the king of Naples seized upon him and her one midnight and shipped them in a frail bark so that they perished in the sea All this took place

(1)

MIRANDA I should sin (Line 118-132)

To think but nobly of my grandmother

Good wombs have borne bad sonsPROSPERO Now the condition

The King of Naples being an enemyTo me inveterate hearkens my brothers suitWhich was that he in lieu othrsquo premisesOf homage and I know not how much tribute

twelve years back

IMPORTANT PASSAGES EXPLAINED(Line 98-103)

PROSPERO helliphelliphelliphelliphelliphelliphellip

He being thus lorded

Not only with what my revenue yieldedBut what my power might else exact like oneWho having into truth by telling of it Made such a sinner of his memoryTo credit his own lie he did believeHe was indeed the duke

Prospero in telling the narrative of his past life here refers to his brother Antonio Prospero being with a studious bent of mind has left the administration of Milan on his younger brother Now Antonio being thus invested like a lord with all the powers derived from Prosperorsquos wealth and what the exercise of Prosperorsquos authority might secure for him regarded himself as a de facto Duke of Milan It is a well-known fact of psychology that a man who repeatedly tells a lie makes of his memory such a sinner against truth as to credit his own lie by the telling of it So Antonio by repeatedly saying to himself and others that he was the Duke came to believe that he was really the Duke Thus falsehood repeatedly asserted gained the force of truth for Antonio and he truly believed it

Should presently extirpate me and mine Out of the dukedom and confer fair MilanWith all the honours on my brother whereonA treacherous army levied one midnightFated to thrsquo purpose did Antonio openThe gates of Milan and ithrsquo dead of darkness The ministers for thrsquo purpose hurried thenceMe and thy crying self

(i) In the earlier lines of this scene what does Prospero tell about his intense interest What was the demand of his interest

In the earlier lines of this scene Prospero tells Miranda that he had an intense interest in the study of philosophy and magic arts Hence in order to improve his mind with this kind of study he kept himself isolated from worldly and state affairs His study was dearer to him than the applause and esteem that he could win from the public His study demanded too much solitude(ii) What forced Antonio to take an undue advantage over Prospero

Prosperorsquos indifferent attitude towards the statersquos affairs and his having boundless trust in Antonio gave rise to a boundless lust for power in Antoniorsquos mind Antonio felt that he must be the actual Duke instead of the part of the Duke he played Thus Antonio took an undue advantage of the situation to usurp Prosperorsquos dukedom(iii) Explain the following lines ldquoI should sin to think but nobly of my grandmother Good wombs have borne bad sonsrdquo

After hearing the treacherous act of her uncle Antonio Miranda says that Prosperorsquos mother was a noble lady and she cannot dishonour her memory by saying that the person named Antonio cannot be his (Prosperorsquos) brother She says that in honour of her grandmother she also cannot say that Antonio must have been begotten not by her grandfather but by some other man She finally concedes that it is known that good mothers have borne bad sons in their wombs and gave birth to them

(iv) Why did the King of Naples accept Antoniorsquos request to help him in usurping his dukedom What did Antonio propose to Alonso

Alonso who was the king of Naples accepted Antoniorsquos request in usurping Prosperorsquos dukedom because he (Alonso) was a sworn enemy of Prospero Antonio proposed that Alonso should immediately drive him (Prospero) and his offspring out of Milan and should confer the dukedom upon him (Antonio) with all the dignities which go with that

position In return Antonio promised that he shall give an annual tribute and also swore his allegiance to Alonso Also he agreed to hold the Dukedom of Milan as a subordinate to the state of Naples(v) How were Prospero and Miranda carried away from the city of Milan and what was the state of small Miranda at that time

In pursuance of the agreement settled between Antonio and Alonso an army of treacherous men was assembled One midnight when the occasion suited the will of destiny Antonio opened the gates of the city of Milan and in the death like silence of midnight Antoniorsquos agents who had been directed to execute his purpose carried Prospero and small Miranda away from the city in all haste They were then forced into a ship and carried some distance out to sea where they put them on a mere hulk of a boat without any rigging or ship-gear and abandoned them leaving them at the mercy of the roaring sea Miranda was a very small child of three years age and she was crying at that time

CLASS -XIDATE-270420Subject Topic Summary Execution

EVS Chapter 1 ndash Mode of Existence

Impact of mode of existence on resources

Q) Why resources are under pressure

Ans - Increase in the sophistication

of technology enabling natural resources to be extracted quickly and efficiently Eg in the past it could take long hours just to cut down one tree only using saws Due to increased technology rates of deforestation have greatly increased

The number of humans is increasing Cultures of consumerism Materialistic views

lead to the mining of gold and diamonds to produce jewelry unnecessary commodities for human life or advancement Consumerism also leads to extraction of resources for the

production of commodities necessary for human life but in amounts excessive of what is needed because people consume more than is necessary or waste what they have

Lack of awareness among the population is striking People are not aware of ways to reduce depletion and exploitation of materials

Accounts Cash Book Today we are going to start a new topic -Cash Book

The key terms used in this chapter are

bullCash book

bullSimple cash book

bullDouble column cash book bullTriple column cash book

bullPetty cash book

bullCash discount

bullContra entry

Here I will share you the meaning of each key terms

bullCash book Cash Book is a special purpose subsidiary book or journal in which cash received and cash payments are recorded

bullSimple cash book

It is a cash book in which only cash transactions are recorded It has only one column on each side

bullTriple column cash book

It is cash book which has three columns one column for each cash and Bankdiscount on each side of the cash book In this book both cash and Bank transactions are recorded together with discount allowed and received

bullPetty cash book

It is a cash book maintained for recording petty expenses

bullCash discount

Cash discount is the amount of discount received or allowed on cash payments and cash receipts Discount received is an income for the business while discount allowed isan expense

bullContra entry

It means transactions involving both cash and Bank Such transactions though recorded in the cash book are not posted into ledger The letter lsquoC is written in Ledger folio for contra entry

Business Studies

ENTREPRENEURSHIP

Now we shall discuss the second chapter

lsquoENTREPRENEURSHIPrsquo

Today before starting the chapter let us recall what

Questions

1What are the main characteristics of Intrapreneurship

Answer

The main characteristics of Intrapreneurship are

Corporate framework-it occurs within the framework of the same company

Semi-Autonomous-Intrapreneurship

we have read last day

Let s today start the class by recalling the last topic taught

Intrapreneurship is the process of discovering and exploring business opportunities within an existing company It involves launching new business ventures within the framework of a present corporation Intrapreneurship is also known as corporate entrepreneurship or corporate venturing

Now let us start with the characteristics of Intrapreneurship

The main characteristics of Intrapreneurship are

Corporate framework

Semi-Autonomous Lack of ownership Senior position Low risk taking Not own boss

Now let us discuss the meaning of enterprise

Enterprise means an undertaking or adventure that requires some innovation and investment and thus involves riskEnterprise always entails decision making coordination and risk bearing

involves crating amd nurturing a semi-autonomous business unit which may be a subsidiary a strategic business unit or a division

Lack of ownership-the intrapreneur is not the owner of the unitb he creates and nurtures

Senior position-he occupies a senior managerial position in the company

Low risk taking-An intrapreneur does not bear the full risk of failure

Not own boss-An intrapreneur is not his own bosss in legal termsHe enjoys the freedom and gets the required resources and support

2 How is Entrepreneur is different from Intrapreneur

The functions involved in both the entrepreneurship and intrapreneurship are by and large similar however there are several differences between the two

Point of distinction

Entrepreneur

Intrapreneur

status An independent business person

A senior executive within a company

Ownership Owner of

the business

An employeesometimes a share in ownership

Financing Responsible for raising finance for the business

Not responsible for raising the finance

Risk bearing

Bears the risk of the business

Does not bears the risk of the business

Reward Profit which is uncertain and irregularcan be loss

Fixed salary and fringe benefits

Need for security low high

3 What do you understand by enterprise

Answer Enterprise means an undertaking or adventure that requires some innovation and investment and thus involves riskEnterprise always entails decision making coordination and risk bearing

COMMERCE NATURE AND OBJECTIVES OF

BUSINESS

Today let us recall the last other two objectives of business by the chart given in the previous class

Firstly we would discuss Human Objectives

Business is run by people and for people Labour is a valuable business element

Human objectives of business are concerned with the well -being of labour

The human objectives are as follows

Labour welfare Developing human

resources Participative

management Labour

management cooperation

Questions

1 Explain the human objectives of a business enterprise

Answer

Business is run by people and for people Labour is a valuable business element

Human objectives of business are concerned with the well -being of labour

The human objectives are as follows

Labour welfare-Business must recognize the dignity of labour and human factors should be given the recognition

Developing human resources-Employees must be provided the opportunities for developing new skills and attitudes

Participative management-Employees should be allowed to take part in decision making process of business

Labour management cooperation-Business should strive for creating and maintaining cordial employer employee relations so as to ensure peace and progress in industry

Now let us discuss the national objectives of business

Optimum utilization of resources

National self- reliance Development of small

scale industries Development of

backward areas Control over pollution

2Explain the national objectives of a business enterprise

Answer

It is the duty of business to utilize the resources of the country properly the national objectives of business

Optimum utilization of resources ndashBusiness should use the nationrsquos resources in the best possible manner

National self- reliance-It is the duty of the business to help the government in increasing experts and in reducing dependence on imports

Development of small scale industries-Big business firms are expected to encourage growth of small scale industries which are necessary for generating employment

Development of backward areas-Business is expected to give preference to the industrialization of backward regions of the country

ECONOMICS

BASIC ECONOMIC CONCEPTS

SUB

TOPIC

Value

Wealth

Welfare

Today we shall start with a new topic of the same chapter ie lsquoValuersquo

Value of a commodity is defined as the valuation placed by a household on the consumption of this commodity

lsquoValuersquo has two different meanings and these are

a Value -in -use It refers to consumption value of a commodity It expresses the utility derived from the consumption of a particular commodity A necessity like water has a very high value ndashin ndashuse or

Question

1What is value

Answer

Value of a commodity is defined as the valuation placed by a household on the consumption of this commodity

2What is value-in use

Answer It refers to consumption value of a commodity It expresses the utility derived from the consumption of a particular commodity A necessity like water has a very high value ndashin ndashuse or consumption value

3What is value ndashin- exchange

Answer It relates to market value of a commodity

It is the rate at which a particular good or service can be exchanged for money

For example in barter system if a person is prepared to exchange 3 metres of cloth with 1 pair

consumption value

b Value ndashin-exchange It relates to market value of a commodity

It is the rate at which a particular good or service can be exchanged for moneyFor example in barter system if a person is prepared to exchange 3 metres of cloth with 1 pair of shoes then the value in exchange of 3 metres of cloth is 1 pair of shoesValue in exchange is the power of purchasing other goods In modern monetised economies the exchange value of goods are expressed in terms of money as prices

Now let us discuss the term lsquoWealthrsquo

Wealth refers to the stock of all those assets which are a source of income

Wealth is a stock concept

Wealth must possess the following features

a Utility It must possess utility or give some

of shoes then the value in exchange of 3 metres of cloth is 1 pair of shoes

Value in exchange is the power of purchasing other goods In modern monetised economies the exchange value of goods are expressed in terms of money as prices

4 What is wealth

Answer it refers to the stock of assets or goods which are a source of income and have personal or national ownership

5 What are the features of wealth

Answer The features of wealth are as follows

Wealth must possess the following features

a Utility It must possess utility or give some satisfaction

b Scarcity It must be limited in quantityc Transferability it should be transferable its

ownership can be transferred from one person to another person

d Exchange value It must possess exchange value

6 What is welfare

Answer

Welfare is defined as satisfaction and happiness a sense of well- being among the people

satisfactionb Scarcity It must be

limited in quantityc Transferability It

should be transferable its ownership can be transferred from one person to another person

d Exchange value It must possess exchange value

Now let us discuss the term lsquoWelfarersquo

Welfare is defined as satisfaction and happiness a sense of well- being among the people

Welfare is affected by factors like

a Consumption of goods and services

b Environment

c Family relations

d Degree of freedom

e Law and order situation

Mathematics Trigonometric equation

To find the general solution of the equation sinθ=0

When sin θ =0

Then θ= 0 π2π 3π-π -2π -3

i e when θ = 0 or an integral multiple of π

i e when θ= nπ where n is any integer

Therefore the general solution of the equation sin

Example1 Find the general values of θ which satisfy the equation sin2 θ =34

Solution sin2 θ= 34

Or sin θ = +34 or -34

Or sin θ = sin π3 or sin (-π3)

Therefore

θ = [nπ + (-1) n (π3)] or[ nπ+ (-1) n (-π3)]

= nπ +π3 or nπ-π3 where n= any integer

Example 2Find the values of θ which satisfy tan2 θ

θ=0 is θ= nπ where n is any integer

To find the general solution of the equation cos θ=0

When cos θ=0

Then θ=π2 3π2 5π2 -π2 -3π2 -5π3

i e when θ is an odd multiple of π2

i e when θ=(2n+1) π2 where n= any integer

Therefore the general solution of the equation cos θ =0 is θ= (2n+1) π2 where n= any integer

To find the general solution of the equation tan θ = 0

Clearly tan θ =0 implies sin θcos θ =0

Therefore θ = nπ

i e the general solution of the equation tan θ=0 is θ =nπ where n = any integer

To find the general solution of the equation cot θ =0

Clearly cot θ =0 implies (cos θsin θ) = 0

i e cos θ =0

Therefore θ = (2n+1) π2

Therefore the general solution of the equation cot θ =0 is θ = (2n+1) π2

Where n= any integer

To find the general solution of the equation sin θ= k (-

=13 -πleθleπ

Solution tan2 θ =13

Or tan θ = plusmn1radic(3) =tan(plusmnπ6)

θ=nπ plusmn π6 where n =any integer

If n=0 then θ=plusmnπ6

If n=1 then θ= π plusmn π6

If n=-1 then θ= -π plusmn π6

Therefore the required solution in -π le θ le π are θ= π6 5π6 -π6 -5π6

Exercise Find general solution of sin 2θ=cos θ [Hints Use sin 2θ= 2sin θcosθ and then take cosθ

common]

1lek le1)

Determine an angle alpha such that sin =k and -π2le αle π2

Then we have

Sin θ = k = sin α

Or sin θ - sin α =0

Or 2 cos [(θ+α) 2] sin [(θ-α) 2] =0

Therefore either cos [(θ +α) 2] =0 (1)

Or sin [(θ-α) 2] =0 (2)

Now from (1) we get (θ+α) 2= (2m+1) π2)

Or θ = (2m+1) π-α (3)

And from (2) we get (θ-α) 2 =mπ

Or θ= 2mπ+α(4)

Where m = any integer

Clearly the solution (3) amp (4) may be combined in the following form

θ= nπ+(-1) n α where n= any integer

Therefore the general solution of sin θ = sin α is θ = nπ +(-1) n α where n is any integer and -π2 le α le π2

Biology Chapter - 04Kingdom Monera

Today we will discuss about bacterial reproduction and its usefulness

Fig Binary Fission

Fig Conjugation Fig Transformation

Bacterial reproduction is mainly asexual but sexual reproduction

also takes place

Asexual reproduction takes place by i) Binary fission - from one bacteriato

two bacteria are produced in every 20 to 30mins

ii) Buddingiii) Endospore formation - during

unfourable condition

Sexual reproduction by three ways

1) Conjugation - Transfer of genetic material between cells that are in physical contact with one another

2) Transduction - Transfer of genetic materialfrom one cell to another by a bacteriophage

3) Transformation - Transfer of cell-freeor naked DNArsquo from one cell to another

Bacteria causes different diseases inplants animals and human and

it causes food spoilage and waterpollution but it also have some useful

activities

i) Bacteria are helpful in sewage water treatment

ii) It is used in antibiotic (medicine) production

iii) Anaerobic bacteria help in biogas(energy) production

iv) Many household products like yoghurt cheese are manufactured by use of bacteria

v) Rhizobium by symbiotic relationship with leguminous plant increase soil fertility

vi) Besides these bacteria is helpful in genetic engineering degradation of petroleum hydrocarbonand in dairy

industry

Physics Motion in plane Here we will introduce Projectile Motion

Execution

Projectile

Y

usinθ u h

θX

ucosθ

Suppose a body is projected with an angle θ So initial velocity u can be resolved into two components

Horizontal component - ucosθ ( for range)

Vertical component - usinθ ( for height)

usinθ changes during motion and becomes zero at maximum height position but ucosθ remain unchanged

The maximum height of projectile is h

NB If initial is upward then g = -ve and if it is downward then g = +ve Height is +ve if direction of motion does not change ( for ex a body thrown upwards but goes down ultimately then height h = -ve)

The angle of projectile θ is the angle made with horizontal

HISTORY ndash GROWTH OF NATIONALISM

SUB TOPIC- REVOLUTIONARY NATIONALISM Bengal formation of Anushilan Samity and Jugantar Group

The intensification of the Swadeshi movement and Government policy of terror and repression led to outbreak of violence Bombs were manufactured and attempts on the lives of unpopular Government officials became frequent In the gymnasium of Scottish Church College which was known as General Assemblies Institution a secret society was formed known as Anushilan Samity

Aurobindo Ghosh send from Baroda his emissary Jatindranath Banerjee to mobilize the Bengal revolutionaries

Hemchandra Qanungo and Satyen Bose published Journal Jugantar

The Jugantar group planned to assassinate oppressive magistrate Kingsford by Khudiram Bose and Prafulla Chaki in 1908 Prafulla Chaki committed suicide to avoid arrest Khudiram was tried and hanged

Afew days later the police found a bomb factory in Maniktala and arrested a large number of revolutionaries The trial of revolutionaries became famous as the Alipore Bomb Case

In the course of the trial the approver the public prosecuter and a police officer were assassinated

1 Question Name two journals which preached the cult of violence

Answer a) Yugantar edited by Bhupendranath DuttaB) Bandemataram edited by Aurobindo Ghosh2 Question Why was Khudiram arrested and hangedAnswer An attempt was made to assassinate a hated vindictive majistrate named Kingsford by Khudiram Bose and Prafulla Chaki Their attempt failed and the bomb they threw killed two English ladies Khudiram was arrested and put to trial and then hanged3Question Who was Aurubindo GhoshAnswer Aurobindo Ghosh a nationalist revolutionary who was charged for his involvement in the Alipore Bombing Case He was accused of it along with his brother Barindra nath Ghosh But Aurobindo was acquitted because of the brilliant pleading of his counsel Chittaranjan Das Then he became a spiritual reformer introducing his visions on human progress and spiritual evolution4 Qustion Who was KingsfordAnswer Kingsford was an unpopular British chief Magistrate who was the target of the bomb thrown at Muzaffarpur by Khudiram and Prafulla Chaki

Most of the accused were convicted and sentenced to

long term of imprisonmentBut

Aurobindo Ghosh was acquitted mainly owing to the brilliant pleading of his counsel Chittaranjan Das

Political science Topic-Sovereignty

Summary Sovereignty is the full right and power of a governing body over itself without any interference from outside sources or bodies In political theory sovereignty is a substantive term designating supreme legitimate authority over some polity In international law sovereignty is the exercise of power by a state

Internal Sovereignty

Internal sovereignty means supreme authority within ones territory while external sovereignty relates to the recognition on the part of all states that each possesses this power in equal measure

External sovereignty

external sovereignty relates to the recognition on the part of all states that each possesses this power in equal measure

Distinguish between

Execution

Answer the following questions

Short notes-

Sovereignty

Internal Sovereignty

External sovereignty

Homework- learn

external sovereignty and internal sovereigntySovereignty is the principle

of supreme and

unquestionable authority

reflected in the claim by the

state to be the sole author of

laws within its territory

Definition of external vs internal sovereigntyInternal sovereignty refers to

the relationship between a

sovereign power and its

subjects ndash it refers to the

location of the supreme

authority within the state In

the UK for example internal

sovereignty (supposedly)

resides within Parliament

reflected in the

constitutional principle of

parliamentary

sovereigntyBy contrast

external sovereignty refers

to the capacity of the state

to act independently and

autonomously on the world

stage This is what is

sometimes called lsquostate

sovereigntyrsquo or lsquonational

sovereigntyrsquo and implies

that states are legally equal

and that the territorial

integrity and political

independence of a state is

inviolable

Class ndash XII

Date - 2742020 STUDY MATERIALSubject Topic Summary Execution Business Studies

Job Analysis amp Manpower Planning

At first let us recall the chapter what we have discussed till nowbullJob analysisbullJob specification bullJob description bullJob enlargement bullJob enrichment

Today we will do some questions answers from the chapter

Questions 1ldquoJob analysis job description and job specification are interrelatedrdquo Comment Answer) Job analysis is a systematic and detailed examination of a job to collect all the relevant information about it The contents off the job are summarised in the job description The qualification needed for the job are summarised in job specificationThus there is close interrelationship between job analysis job description and job specification

Question 2ldquoJob enlargement is a horizontal extension of a job whereas job enrichment is a vertical extension of a jobrdquo ElucidateAnswer) Job enlargement involves adding one or more task to a job coma where as job enrichment involves adding more autonomy and responsibility to a job Job enlargement is therefore horizontal extension of a job coma whereas job enrichment is a vertical extension of a job

Question 3 )

What is manpower estimation Explain its quantitative and qualitative aspectsAnswer) Manpower estimation is the process by which management determines how an organisation should move from itrsquos current manpower positionto its desired manpower position There are two dimensions of Manpower estimation- quantitative and qualitative

Quantitative aspectThis aspect of Manpower estimation involves estimating the number of employees required in a future time period Workload analysis and workforce analysis are done to estimate the quantity of required manpower

Qualitative aspectThe estimate of the knowledge skills experience etc of required manpower is the qualitative aspect of Manpower estimation The quality of Manpower can be judged on the basis of job analysisand job specification

COMMERCE

CAPITAL-FIXED AND WORKING

Today let us start the class by discussing the sources of finance for different types of business firms

The term lsquocapitalrsquo refers to the investment made in the enterprise for the purpose of earning profits

Requirements of capital and sources of capital for different types of business firms are

1 Capital for sole proprietorship businessA sole proprietor operates at a small scale and thereforerequires a limited amount of capital

2 Capital for partnership firmCapital requirements as well as capital base of a partnership is bigger than that of a sole trader businessThe owned capital is contributed by the partners in an agreed ratio

3 Capital for joint stock companyA joint stock company generally requires large amount of capitalA public company can raise huge capital through issue of shares In addition to share capital it can utilize retained profits

Now let u discuss the meaning of Finance PlanningFinance planning is the process of estimation the financial requirements of an organization specifying the sources of firms and ensuring that enough funds are available at the right time

1 What do you mean by Finance PlanningAnswer Finance planning is the process of estimation the financial requirements of an organization specifying the sources of firms and ensuring that enough funds are available at the right time

2Discuss the role of financial planning of an enterpriseThe role of financial planning are as followsa A sound financial plan helps a business enterprise to avaid the problems of shortage and surplus of fundsbFinancial planning serves as a guide in developing a sound capital structure so as to maximize returns to shareholders c It helps in effective utilization of fundsd It provides policies and procedures for coordinating different functional areas or departments of businesse It enables the management to exercise effective control over the financial activities of an enterprisef It helps the company to prepare for facing business shocks and surprises in future

Mathematics

Continuity and differentiability

Recall Definition of ContinuityLet f(x) be a single valued function of x and x=a be a point in the domain of definition of the function The function is said to be continuous at x=a ifi) f(c) is defined ie f(x) has a definite finite value at x=cii) lim xrarra f(x) exists andiii) lim xrarra f(x) =f(a) In other words f(x) is said to be continuous at x=a if lim xrarra+ f(x)= lim xrarra- f(x) = f(a) Or f(a+0) =f(a-0) =f(a) Or lim hrarr0 f(a+h)= f(a) Algebra of continuous functionsNow we will study some algebra of continuous functions Theorem 1 Suppose f and g be two real functions continuous at a real number c Then(1) f + g is continuous at x = c(2) f ndash g is continuous at x = c(3) f g is continuous at x = c(4) (fg) is continuous at x = c (provided g (c) ne 0)

Example 1 Prove that every rational function is continuousSolution Recall that every rational function f is given byf(x)=[p(x) q(x) ] q(x)ne0where p and q are polynomial functions The domain of f is all real numbers except points at which q is zero Since polynomial functions are continuous f is continuous by (4) of Theorem 1Example 2Discuss the continuity of sine functionSolution To see this we use the following factslim xrarr0 sin x =0Now observe that f (x) = sin x is defined for every real number Let c be a real number Put x = c + h If x rarr c we know that h rarr 0 Therefore lim xrarrc f(x)

= lim xrarrc sin x= lim hrarr0 sin(c+h) =lim hrarr0 [sin c cos h + cos c sin h ]=lim hrarr0 (sin c cos h) + lim hrarr0 (cos c sin h) = sin c +0=sin c = f(c) Thus lim xrarrc f(x) = f(c) and hence f is a constant function Exercise Prove that the function f(x) = x2 +2x is continuous for every real value of x [Hints show that lim xrarra+ f(x) = lim xrarra- f(x) = f(a) ]

Biology Reproduction in Flowering plants We will discuss about megasporoangium

megasporagenesis and female gametophyte

Q4 Describe the structure of megasporangium

Ovule is attached to the placenta by astalk called funicle

Each ovule has one two or three protectivecoverings called integuments

At the tip of integuments a small openingcalled micropyle is organised

Opposite to the micropylar end is the chalaza

Within the integuments a mass of cellsnucellusand inside it embryo sac orfemale gametophyte is present

Q5 Describe a mature embryosacamp its formation

In most of the flowering plants only oneof the 4 megaspores formed as a result ofmegasporogenesis that is functional while theother three degenerate

The

functional megaspore develops into thefemale gametophyte

Formation The nucleus

of the functional megasporedivides mitotically to form two nuclei first andthen two more sequential mitotic nucleardivisions result in the formation of four ampthen eight nucleate stages of embryo sac

Six of the eight nucleus are surrounded bycell walls and organised into cells

The remaining two nuclei called polar nuclei are found below the egg apparatus in the largecentral cell

Three cells consisting of two synergids amp one egg cell present bottom of

embryo sac Three cells

at the chalazal as antipodal cells

Two polar nuclei together present in large central cell

HISTORY

TOWARDS INDEPENDENCE AND PARTITION THE LAST PHASE(1935-1947)SUB TOPIC NATIONAL MOVEMENTS DURING THE SECOND WORLD WAR

Spread of Quit India Movement On 9th August 1942Gandhiji and other Congress leaders were arrested The Congress was declared illegal The news of the arrest of all leaders marked the beginning of a widespread movement of India It was not possible for such a movement to remain peacefulBut the arrest of the all notable congress leaders virtually left the movement in the hands of the mass The movement took the form of violent and militant outbreakBesides congressmen revolutionaries also were very active in the movement The Congress Socialist group also played a prominent role

1 Question Why did the British authority arrest the Congress leaders on 9 th August 1942Answer Congress Working committee adopted the Quit India resolution which was to be ratified at the Bombay AICC meeting in 8th August 1942 They decided to launch a mass struggle on non-violent lines Gandhiji gave a clarion call to all section of the people rdquoKarenge ya Marengerdquo (do or die) Congress leaders gave the call to driving out

the British from IndiaViceroy had taken strong action against the Quit India movement Gandhiji and all the leaders of Congress were arrested

2 Question How did Quit India Movement spread out all over IndiaAnswer The news of the leaders lsquo arrest marked the beginning of a widespread movement to remain peacefulThe movement took form of violent outbreak There were widespread cutting of telephone and Telegraph wires damaging railway lines raising barricades in cities and towns and other forms of violent demonstations

Question Name the leaders of Congress

Socialist group played a prominent part Notable among the Jayprakash Narayan Rammonohar Lohia Aruna Asaf Ali

Political science

Topic-Franchise and Representation

Summary

The election commission

The Election Commission of India is an autonomous constitutional authority responsible for administering Union and State election processes in India The body administers elections to the Lok Sabha Rajya Sabha State Legislative Assemblies in India and the offices of the President and Vice President in the country

Functions of election commission-

India is a sovereign socialist secular democratic republic Democracy runs like a golden thread in the social economic and political fabric woven by the Constitution given by lsquoWe the People of Indiarsquo unto ourselves The concept of democracy as visualised by the Constitution pre-supposes the representation of the people in Parliament and State legislatures by the method of election The Supreme Court has held that democracy is one of the inalienable basic features of the Constitution of India and forms part of its basic structure The Constitution of India adopted a Parliamentary form of government Parliament consists of the President of India and the two Houses mdash Rajya Sabha and Lok Sabha India being a Union of states has separate state legislatures for each state State legislatures consist of the Governor and two Houses mdash Legislative Council and Legislative Assembly mdash in seven states namely Andhra Pradesh Telangana Bihar Jammu amp Kashmir Karnataka Maharashtra and Uttar Pradesh and of the Governor and the state Legislative Assembly in the remaining 22 states Apart from the above two out of the seven Union Territories namely National Capital Territory of Delhi and Puducherry also have their Legislative Assemblies

ExecutionShort notes-Election commissionFunctions of election commission

Homework- Learn

Computer

Science

Computer hardware NAND Gate

A NOT-AND operation is known as NAND operation It has n input (n gt= 2) and one output

Logic diagram

Truth Table

NOR Gate

A NOT-OR operation is known as NOR operation It has n input (n gt= 2) and one output

Logic diagram

Truth Table

XOR Gate

XOR or Ex-OR gate is a special type of gate It can be used in the half

adder full adder and subtractor The exclusive-OR gate is abbreviated as EX-OR gate or sometime as X-OR gate It has n input (n gt= 2) and one output

Logic diagram

Truth Table

XNOR Gate

XNOR gate is a special type of gate It can be used in the half adder full adder and subtractor The exclusive-NOR gate is abbreviated as EX-NOR gate or sometime as X-NOR gate It has n input (n gt= 2) and one output

Logic diagram

Truth Table

Physics

Chapter 1 Electric Field ( Electric Dipole) (Summary)

Here we will derive Expression of electric field at broad side

On position of dipole

Execution

Q With the help of a labelled diagram obtain an expression for the electric field intensity E at any point on the equitorial line ( broad-side on position) of an electric dipole

Ans

E1 E1sinθ

E θ P E1 θ

( r2+L2)12 E2 E

r E2 E2sinθ

-q θ L O L +qA B

Let us consider that the point P is situated on the right bisector of the dipole AB at a distance r meter from its midpoint O

Let E1 and E2 be the electric field intensities of the electric field at P due to charge +q and ndashq of the dipole resp The distance of P from each charge is ( r2+L2)12

So E1 = 14 πϵ q

(r 2+L 2) away from +q

E2 = 14 πϵ q

(r 2+L 2) towards ndashq

The magnitudes of E1 and E2 are equal but directions are different Now resolving E1 and E2 into two components parallel and perpendicular to AB we get

The components perpendicular to AB E1sinθ and E2sinθ cancel each other because they are equal and opposite

The components parallel to AB are E1cosθ and E2 cosθ are in same direction and add up

So resultant intensity of electric field at the point P is

E = E1cosθ + E2 cosθ

E = 14 πϵ q

(r 2+L 2) 2 cosθ

Now from fig we have cosθ =BOBP = L (r2+L2)12

So we get E = 14 πϵ 2qL ( r2+L2)32

Now electric dipole moment p= 2qL

So E = 14 πϵ p ( r2+L2)32

HW Find the expression of Electric field as done here but this time take r gtgt 2L

Also find the expression of torque experience by a dipole

(Hint Electric force experienced by charges of dipole in electric field is qE each Let θ be the angle which dipole makes with electric lines of force then perpendicular distance between two charges is 2Lsinθ Then torque = force x perp distance = qE x 2L sinθ So τ=pE sinθ where p =2qL )

STUDY MATERIAL

Class XIISubject Eng Literature (The Tempest ndash William Shakespeare) Topic Act IV Scene 1 Lines 84 to 133 (Iris hellip A contract of true love Be not too late ) Date 27th April 2020 (4th Period)

[Students should read the original play and also the paraphrase given in the school prescribed textbook]Summary Questions amp Answers

o Ceres soon appears and comes to know that she has been summoned to celebrate the contract of true love

o Ceres expresses her unwillingness to meet Venus and Cupid as she has shunned their company

o Ceres and Juno both bestow their blessings upon Ferdinand and Miranda with June gifting honour riches happiness in marriage and Ceres presents plenty of earthrsquos produce

o Iris summons the water-nymphs and reapers to come and celebrate a contract

(1) IRIS Of her society (Line 91-101)

Be not afraid I met her deity

Cutting the clouds towards Pathos and her sonDove-drawn with her Here thought they to have doneSome wanton charm upon this man and maidWhose vows are that no bed-right shall be paidTill Hymens torch be lightedmdashbut in vainMarss hot minion is returned againHer waspish-headed son has broke his arrowsSwears he will shoot no more but play with sparrowsAnd be a boy right out

(i) Where were Venus and Cupid seen flying How were they travelling Why did they want to join the marriage celebration of Ferdinand and Miranda

of true love

Venus and Cupid were seen flying through the air towards Paphos the famous city which is situated on the island of Cyprus They were travelling by air-borne chariot drawn by doves They certainly wanted to come here in order to play some amorous trick upon Ferdinand and Miranda who are under a vow not to gratify their physical desires till the holy ceremony of their marriage has been performed(ii) What have Venus and Cupid done after failing in their plan

After being failure of their plan Venus who is a very passionate deity and who is the mistress of Mars (the god of war) has gone back while here ill-tempered son Cupid has broken his arrows of love in his state of desperation(iii) What has Cupid firmly decided

Cupid is feeling so disappointed that he has firmly decided to shoot no more arrows to arouse love in human hearts but to spend his time playing with sparrows Thus he would now become just a boy and would give up his original function of shooting arrows on human beings to make them fall in love(iv) What vow had Ceres taken How did Ceres feel at the abduction

After the abduction of her daughter Prosperina by Pluto Ceres had taken a vow to always keep away from the disgraceful company of Venus and her blind son Cupid the god of love Ceres felt deeply distressed when Pluto had carried off her daughter and had made her his wife by force(v) Why has Ceres not forgiven Venus and her blind son For what do Ceres want to be sure

As the abduction had been manipulated by Venus the goddess of beauty and love and her blind son Cupid Ceres has never forgiven them for their part in the whole plot Ceres wants to be sure that she would not have to meet Venus and Cupid who had engineered the abduction of her daughter Prosperina

AS THIS lsquoMASQUErsquo SCENE IS VERY IMPORTANT IN THE PLAY THE PARAPHRASE OF THE ENTIRE PORTION OF MASQUE SCENE (Act IV Lines 58 to 143) IS GIVEN BELOW

IRIS Goddess of RainbowCERES Goddess of Agriculture and all the fruits of the earth

(Nature growth prosperity rebirth ndash notions intimately connected to marriage)JUNO The majestic Queen of Heavens and wife of Jupiter (Jupiter is the king of Gods)

VENUS The Goddess of love CUPID Son of Venus PLUTO God of death (In the play referred by Shakespeare as lsquoDisrsquo which is a Roman name for Pluto)

ORIGINAL TEXT PARAPHRASEPROSPEROWellmdash

PROSPERONow come Ariel Let there be too many rather than too few

Now come my Ariel Bring a corollaryRather than want a spirit Appear and pertly[to Ferdinand and Miranda]No tongue all eyes Be silent

spirits in attendance Appear briskly

[to Ferdinand and Miranda]Look with your eyes but do not say a word

[Soft music] [Soft music][Enter Iris] [Enter Iris]

IRISCeres most bounteous lady thy rich leasOf wheat rye barley vetches oats and peasThy turfy mountains where live nibbling sheepAnd flat meads thatched with stover them to keepThy banks with pioned and twilled brimsWhich spongy April at thy hest betrimsTo make cold nymphs chaste crowns and thybroom-grovesWhose shadow the dismissegraved bachelor lovesBeing lass-lorn thy pole clipped vineyardAnd thy sea-marge sterile and rocky-hardWhere thou thyself dost airmdashthe Queen othrsquoSkyWhose watery arch and messenger am IBids thee leave these and with her sovereign grace[Juno appears] Here on this grass-plot in this very placeTo come and sport Her peacocks fly amainApproach rich Ceres her to entertain

IRISCeres most generous lady you are the cause of rich fields or fertile land where wheat rye barley beans oats and peas grow the grassy mountains where the sheep graze and the flat meadows covered with coarse hay to be used as fodder for cattleYour banks are covered with marsh-marigolds and reeds and the rainy April under your orders brings forth to make for the maids who are not in love beautiful crowns your woods where the broom flourishes and where the bachelor who has been dismissed by the maid he loved lies down being forsaken your vineyard in which the poles are embraced by the vines and the margin of the sea which is barren and rocky where you roam about to enjoy the fresh air ndash the queen of the sky (Juno) whose messenger I am besides being represented as the rainbow bids you leave all these and with her majesty here on this grassy plot in this very place come and sport her peacocks carry her fast in her chariot through the air and are making their way here approach rich Ceres to welcome her

[Enter Ariel as Ceres] [Enter Ariel as Ceres]

CERESHail many-coloured messenger that neerDost disobey the wife of JupiterWho with thy saffron wings upon my flowersDiffusest honey-drops refreshing showersAnd with each end of thy blue bow dost crownMy bosky acres and my unshrubbed downRich scarf to my proud earth Why hath thy queenSummoned me hither to this short-grassed green

CERESWelcome rainbow that never dared disobey Juno the wife of Jupiter who with your orange coloured rays spread honey-drops refreshing showers And with each end of thy blue bow drown my bushy acres and my hilly country which is free from shrubs you thus forming a rich scarf Why has your queen called me here to this place covered with short grass

IRISA contract of true love to celebrateAnd some donation freely to estateOn the blest lovers

IRISI have called you to celebrate a contract of true love and bestow some liberal gift upon the blessed lovers

ORIGINAL TEXT PARAPHRASECERESTell me heavenly bowIf Venus or her son as thou dost knowDo now attend the queen Since they did plotThe means that dusky Dis my daughter gotHer and her blind boys scandaled companyI have forsworn

CERESTell me heavenly bow if Venus the Goddess of love or Cupid her son and pedlar of passion at this time attend the heavenly queen Juno because you are sure to know Since the day they conspired against me and dark Pluto took away my daughter here and Cupidrsquos disgraceful company I have left off

IRISOf her societyBe not afraid I met her deityCutting the clouds towards Pathos and her sonDove-drawn with her Here thought they to have doneSome wanton charm upon this man and miad

IRISBe not afraid of her company I met her deity moving on the clouds towards Paphos the sacred home of Venus on the island of Cyprus along with her son on her chariot drawn by doves Here they contemplated to exercise a charm upon this man and maid producing

Whose vows are that no bed-right shall be paidTill Hymens torch be lightedmdashbut in vainMarss hot minion is returned againHer waspish-headed son has broke his arrowsSwears he will shoot no more but play with sparrowsAnd be a boy right out

wantonness before the actual marriage ceremony but did not succeed Venus has returned her irritable son has broken his arrows and swears that he will give up his practice of trying to inspire love but play with sparrows and be a boy again

[Music is heard] [Music is heard]

CERESHighst queen of stateGreat Juno comes I know her by her gait

CERESHighest queen of state Great Juno there she comes I know here by her gait

[Enter Juno] [Enter Juno]

JUNOHow does my bounteous sister Go with meTo bless this twain that they may Prosperous beAnd honoured in their issue

JUNOHow are you doing my generous sister Come with me to bless this couple so that they may be prosperous and fortunate in their children

[They sing] [They sing]

JUNOHonour riches marriage-blessingLong continuance and increasingHourly joys be still upon youJuno sings her blessings upon you

JUNOMay honour riches happiness in marriage long continuance and increase of those boons ever rest upon you as hourly joys Juno showers down upon you her blessings in song

CERESEarths increase foison plentyBarns and garners never emptyVines and clustring bunches growingPlants and goodly burden bowingSpring come to you at the farthestIn the very end of harvestScarcity and want shall shun youCeresrsquo blessing so is on you

CERESMay you have the plenty of earthrsquos produce Your barns and granaries may never be empty Your vines may grow with clustering bunches Your fruit trees may be heavily laden with their fruit May there be continuous spring and harvest May scantiness and want leave you forever Such is the blessing of Ceres upon you

FERDINANDThis is a most majestic vision andHarmoniously charmingly May I be boldTo think these spirits

FERDINANDThis is a great vision and magically melodious Should I suppose the characters (taking part in the masque) are spirits

PROSPEROSpirits which by mine artI have from their confines calld to enactMy present fancies

PROSPEROYes they are spirits whom I have summoned from the regions to which they are confined to carry into effect my fanciful designs

ORIGINAL TEXT PARAPHRASEFERDINANDLet me live here everSo rare a wondered father and a wifeMakes this place paradise

FERDINANDI should like to live here forever Such a wise and wonderful father makes this place a paradise

[Juno and Ceres whisper and send Iris on employment] [Juno and Ceres whisper and send Iris on employment]

PROSPEROSweet now silence

PROSPEROMy dear Ferdinand speak no more Juno and Ceres are

Juno and Ceres whisper seriouslyTheres something else to do Hush and be muteOr else our spell is marred

whispering with a solemn look There is something else coming Silence Or else our magic will be spoilt

IRISYour nymphs called naiads of the wandering brooksWith your sedged crowns and over-harmless looksLeave your crisp channels and on this green landAnswer your summons Juno does commandCome temperate nymphs and help to celebrateA contract of true love Be not too late

IRISYou nymphs called Naiads denizens (M inhabitants) of the running stream with your chaplets of sedge and ever-helpful looks leave your wrinkled channels and on the green land answer the summons sent to you Juno has ordered some chaste nymphs and help to celebrate a noble and true marriage Donrsquot delay

[Enter certain nymphs] [Enter certain nymphs]You sunburnt sicklemen of August wearyCome hither from the furrow and be merryMake holiday your rye-straw hats put onAnd these fresh nymphs encounter every oneIn country footing

You sunburnt harvesters weary from the effects of the heat in August come here from the furrowed land and rejoice Make holiday with your rye-straw hats upon you and meet these fresh nymphs and join in country dancing

[Enter certain reapers properly habited They join with the nymphs in a graceful dance towards the end whereof Prospero starts suddenly and speaks]

[Enter certain reapers properly habited They join with the nymphs in a graceful dance towards the end whereof Prospero starts suddenly and speaks]

PROSPERO[aside] I had forgot that foul conspiracyOf the beast Caliban and his confederatesAgainst my life The minute of their plotIs almost come [to the spirits]Well done Avoidno more

PROSPERO(Aside)I had forgotten the wicked conspiracy of the beast Caliban and his accomplices against my life the time of their plot has almost arrived ndash (To the Spirits) well done depart no more of this

[To a strange hollow and confused noise the spirits heavily vanish]

[The spirits depart]

ORIGINAL TEXT PARAPHRASEFERDINANDLet me live here everSo rare a wondered father and a wifeMakes this place paradise

FERDINANDI should like to live here forever Such a wise and wonderful father makes this place a paradise

[Juno and Ceres whisper and send Iris on employment] [Juno and Ceres whisper and send Iris on employment]

PROSPEROSweet now silenceJuno and Ceres whisper seriouslyTheres something else to do Hush and be muteOr else our spell is marred

PROSPEROMy dear Ferdinand speak no more Juno and Ceres are whispering with a solemn look There is something else coming Silence Or else our magic will be spoilt

IRISYour nymphs called naiads of the wandering brooksWith your sedged crowns and over-harmless looksLeave your crisp channels and on this green landAnswer your summons Juno does commandCome temperate nymphs and help to celebrateA contract of true love Be not too late

IRISYou nymphs called Naiads denizens (M inhabitants) of the running stream with your chaplets of sedge and ever-helpful looks leave your wrinkled channels and on the green land answer the summons sent to you Juno has ordered some chaste nymphs and help to celebrate a noble and true marriage Donrsquot delay

[Enter certain nymphs] [Enter certain nymphs]You sunburnt sicklemen of August wearyCome hither from the furrow and be merryMake holiday your rye-straw hats put onAnd these fresh nymphs encounter every oneIn country footing

You sunburnt harvesters weary from the effects of the heat in August come here from the furrowed land and rejoice Make holiday with your rye-straw hats upon you and meet these fresh nymphs and join in country dancing

[Enter certain reapers properly habited They join with the nymphs in a graceful dance towards the end whereof Prospero starts suddenly and speaks]

[Enter certain reapers properly habited They join with the nymphs in a graceful dance towards the end whereof Prospero starts suddenly and speaks]

PROSPERO[aside] I had forgot that foul conspiracyOf the beast Caliban and his confederatesAgainst my life The minute of their plotIs almost come [to the spirits]Well done Avoidno more

PROSPERO(Aside)I had forgotten the wicked conspiracy of the beast Caliban and his accomplices against my life the time of their plot has almost arrived ndash (To the Spirits) well done depart no more of this

[To a strange hollow and confused noise the spirits heavily vanish]

[The spirits depart]

Ac-12 27420 topic Revaluation of Assets and Liabilities

REVALUATION OF ASSETS AND LIABILITIES

On admission of a new partner the firm stands reconstituted and consequently the assets are revalued and liabilities are reassessed It is necessary to show the true position of the firm at the time of admission of a new partner If the values of the assets are raised gain will increase the capital of the existing partners Similarly any decrease in the value of assets ie loss will decrease the capital of the existing partners For this purpose alsquoRevaluation Accountrsquo is prepared This account is credited with all increases in the value of assets and decrease in the value of liabilities It is debited with decrease on account of value of assets and increase in the value of liabilities The balance of this account shows a gain or loss on revaluation which is transferred to the existing partnerrsquos capital account in existing profit sharing ratioAccounting for Revaluation of Assets and Liabilities when there is a Changein the Profit Sharing Ratio of Existing PartnersAssets and liabilities of a firm must also be revalued at the time of change in profit sharing ratio of existing partners The reason is that the realisable or actual value of assets and liabilities may be different from those shown in the Balance Sheet It is possible that with the passage of time some of the assets might have appreciated in value while the value of certain other assets might have decreased and no record has been made of such changes in the books of accounts Similarly there may be some unrecorded assets amp libilities that may have to be accounted for Revaluation of assets and reassessments of liabilities becomes necessary because the change in the

value of assets and liabilities belongs to the period to change in profit sharing ratio and hence must be shared by the partners in their old profit sharing ratio Revaluation of assets and reassessment of liabilities may be given effect to in two different ways (a) When revised values are to be recorded in the books and(b) When revised values are not to be recorded in the books

When revised values are to be recorded in the booksIn such a case revaluation of assets and reassessment of liabilities is done with the help of a new account called lsquoRevaluation Accountrsquo Sometimes this account is also called as lsquoProfit amp Loss Adjustment Acrsquo If there is a loss due to revaluation revaluation account is debited and if the revaluation results in a profit the revaluation account is credited The following journal entries made for this purpose are

(i) For increase in the value of assetsAsset Ac Dr (individually)To Revaluation Ac(ii) For decrease in the value of AssetRevaluation Ac Dr (individually)To Asset Ac[Decrease in the value of assets](iii) For increase in the value of LiabilitiesRevaluation Ac Dr (individually)To Liabilities Ac[Increase in the value of Liabilities](iv) For decrease in the value of LiabilitiesLiabilities Ac DrTo Revaluation Ac[Decrease in the value of Liabilities](v) For unrecorded AssetsAsset Ac [unrecorded] DrTo Revaluation Ac[Unrecorded asset recorded at actual value](vi) For unrecorded Liability Revaluation Ac DrTo Liability Ac [unrecorded][Unrecorded Liability recorded at actual value](vii) For transfer of gain on revaluationRevaluation Ac DrTo Existing Partnerrsquos CapitalCurrent Ac[Profit on revaluation transferred to capital account in existing ratio](viii) For transfer of loss on revaluationExisting Partnerrsquos CapitalCurrent Ac DrTo Revaluation Ac[Loss on revaluation transferred to capital account in existing ratio](a) When revaluation account shows gain Revaluation Ac DrTo Partnerrsquos Capital Ac (Old Profit Sharing Ratio)(Profit on revaluation credited to Partnerrsquos Capital Ac)(b) Above entry is reversed when revaluation account shows loss Partners Capital Acs (Old Profit Sharing Ratio) DrTo Revaluation Ac(Loss on revaluation debited to Partnerrsquos Capital Acs)

Proforma of Revaluation Account is given as under

Revaluation Account

Dr Cr Particulars ` Amount Particulars ` Amount To Decrease in value of assets By Increase in value of assets To Increase in value of liabilities By Decrease in value of liabilities To Unrecorded liabilities By Unrecorded assets To Gain on Revaluation (Transferred) By Loss on Revalution (Transferred)

ECO ndash12 2742020Topic- ELASTICITY OF DEMAND

CHAPTER - ELASTICITY OF DEMANDMEANINGDemand for a commodity is affected by many factors such as its price price of related goods income of its buyer tastes and preferences etc Elasticity means degree of response Elasticity of demand means degree of responsiveness of demand Demand for a commodity responds to change in price price of related goods income etc So we have three dimensions of elasticity of demandDIMENSION OF ELASTICITY OF DEMAND TYPES OF ELASTICITY OF DEMAND

Price elasticity of demand Income elasticity of demand Cross Elasticity of demand

Price elasticity of demand Price elasticity of demand means degree of responsiveness of demand for a commodity to the change in its price For example if demand for a commodity rises by 10 due to 5 fall in its price Price elasticity of demand (ep)=Percentage change in quantity demanded Percentage change in price of the commodity = 10 ( -)5 = ( - )2Note that ep will always be negative due to inverse relationship of price and quantity demanded

(ii) Income elasticity of demand Income elasticity of demand refers to the degree of responsiveness of demand for a commodity to the change in income of its buyer Suppose income of buyer rises by 10 and his demand for a commodity rises by 20 then Income elasticity of demand (ey)= change in quantity demanded change in price of the commodity =20 10 = 2

Cross Elasticity of demandCross elasticity of demand means the degree of responsiveness of demand for a commodity to the change in price of its related goods (substitute goods or complementary goods) Suppose demand for a commodity rises by 10 due to 5 rise in price of its substitute good then Cross elasticity of demand (ec) = change in quantity demanded change in price of related good = 10 2 = 5 (Tastes and preferences cannot be expressed numerically So elasticity ofdemand cannot be numerically expressed)

  • Chapter 1 Force (Summary)
  • Distinguish between external sovereignty and internal sovereignty
    • NAND Gate
      • Logic diagram
      • Truth Table
        • NOR Gate
          • Logic diagram
          • Truth Table
            • XOR Gate
              • Logic diagram
              • Truth Table
                • XNOR Gate
                  • Logic diagram
                  • Truth Table
                      • Physics
                      • Chapter 1 Electric Field ( Electric Dipole) (Summary)
Page 10:  · Web viewWe all know that Nouns are divided into two parts: common noun and proper noun.Apart from common and proper noun, we will also study about collective noun and compound

Collective Noun A collective noun is the name of a collection of people or things taken together and spoken of as a whole Example

1 The feet were completely destroyed in the fierce attack2 The crew revolted against Captain Grand

Abstract Noun An abstract noun is the name of some quality state or idea Example1 Diversity Indias biggest strength2 Patience is a virtue

_____________________________________________________________________________________

Class VII

Subject English 1 Topic Articles

ARTICLES

There are three articles ------ A An The

1 Indefinite articles- The Articles a and an are called Indefinite Articles They do not point to a particular person or thing They are used with singular countable nouns It does not identify a specific noun

Uses of Indefinite articles A

Before a singular noun beginning with a consonant or a vowel with a consonant sound For example a cattle a university a one way track

Before a proper noun which is either unfamiliar or holds a special meaning For example For example A Mahesh Sharma wants to see youNeela wants to be a Tendulkar when she grows up

With a number beginning with a consonant sound For example a ten- rupee note a hundred years

Before half when half follows a whole number or after half when it isnrsquotExample one and a half litres (before) Half a litre (after)

With an expression of quantity For example a lot of time a dozen oranges a little while In exclamation before nouns Example Such a mess What a pity

Uses of Indefinite articles An

Before a singular noun beginning with a vowel sound Example an ant an egg an umbrella Before an abbreviation beginning with a vowel or a consonant with the vowel sound

Example an MP an MA Before the word beginning with a silent h Example an heir an hour an honour

Note We do not say a milk or a lemonade because they are uncountable nouns ever we see a cup of milk or a glass of lemon and

Subject GEOGRAPHY

CHAPTER 1 REPRESENTATION OF GEOGRAPHICAL FEATURES

TOPIC- TOPOGRAPHICAL MAPS

Topographical Maps- topographical maps are small-scale maps with detailed depiction of both natural and human-made features

Conventional colours-different colours used in a map are known as conventional colours

TYPES OF SCALE-

Verbal Statement- Scale written in a statement from like 2 cm to a Km is called verbal statement It means that 2cm on the map represents 1 km on the Earth

Representative Fraction- Scale can also be written as a representative fraction (RF)

Example 150000 it means 1cm on the map represents 50000cm or frac12 km on the ground It shows the ratio between map distance and ground distance

Linear Scale Scale can be drawn on a line to show map distance equivalent to ground distance A line is divided into equal parts Each parts represents the actual distance on the ground in mkm

SUBJECT-COMPUTER

CHAPTER-1 (COMPUTER FUNDAMENTALS)

COMPUTER LANGUAGES

THE THE TERM COMPUTER LANGUAGE REFERS TO A SYSTEM OF RULES AND SYMBOLS THAT ARE DESIGNED TO GIVE INSTRUCTIONS TO A COMPUTERTHE COMPONENTS OF A COMPUTER SYSTEM CANNOT PERFORM A TASK BY THEMSELVES THEREFORE THEY HAVE TO BE GIVEN INSTRUCTIONS TO PERFORM ANY TASKCOMPUTER LANGUAGES ARE USED TO CREATE PROGRAMS USING LOGIC BASED ALGORITHMSTHE RULES OF A COMPUTER LANGUAGE IS

KNOWN AS SYNTAX WHEREAS THE TERM SEMANTICS REFERS TO THE MEANING OF LANGUAGESCOMPUTER CANNOT DEVELOP ANY PROGRAMMING LANGUAGE BY THEMSELVES AND CANNOT THINK INTELLIGENTLY UNLESS THEY ARE ASSISTED BY THE HUMAN BEINGS THEREFORE THE PROGRAMMERS DEVELOP A SET OF METHODS AND TECHNIQUES A SET OF INSTRUCTIONS USED TO PERFORM A SPECIFIC TASK IS CALLED A PROGRAM

A PROGRAMMING LANGUAGE IS ALSO KNOWN AS A COMPUTER LANGUAGE CODED BY PROGRAMMERS TO WRITE INSTRUCTIONS FOR A COMPUTERTHE COMPUTER TAKES THESE INSTRUCTIONS AS INPUT AND PRODUCES THE DESIRED OUTPUT

TYPES OF COMPUTER LANGUAGE

THE COMPUTER LANGUAGE CAN BE BROADLY DIVIDED INTO TWO CATEGORIES

LOW LEVEL LANGUAGES HIGH LEVEL LANGUAGES

LOW LEVEL LANGUAGES

A LOW LEVEL PROGRAMMING LANGUAGE REFERS TO THE LANGUAGE THAT IS UNDERSTOOD BY A COMPUTER DIRECTLYTHE PROGRAMMER MUST HAVE AN IN-DEPTH KNOWLEDGE OF DIFFERENT COMPUTERS TO WRITE PROGRAMS IN A LOW LEVEL LANGUAGE THE TWO TYPES OF LOW LEVEL LANGUAGES ARE MACHINE LANGUAGE AND ASSEMBLY LANGUAGE

MACHINE LANGUAGE

COMPUTER HARDWARE UNDERSTANDS ONLY MACHINE LANGUAGE AS IT IS THE FUNDAMENTAL LANGUAGE OF A COMPUTER WHICH UNDERSTANDS ONLY THE TWO DIGITS 0 AND 1 KNOWN AS THE BINARY DIGITSIT IS A SYSTEM OF INSTRUCTIONS EXECUTED DIRECTLY BY THE CPU WITHOUT ANY TRANSLATION AND HENCE IS EXECUTED QUICKLY AS COMPARED TO PROGRAMS IN OTHER LANGUAGES PROGRAMS IN MACHINE LANGUAGE NEED DIFFERENT BINARY LANGUAGE PROGRAMS TO SOLVE THE COMPLEX TASKSPROGRAMMERS OFTEN FIND IT DIFFICULT TO WRITE PROGRAMS IN THIS LANGUAGE

ASSEMBLY LANGUAGE

AN ASSEMBLY LANGUAGE IS A LOW LEVEL PROGRAMMING LANGUAGE BUT THE INSTRUCTIONS ARE WRITTEN IN THE FORM OF WORDS KNOWN AS MNEMONICS TO CONVERT AN ASSEMBLY LANGUAGE INTO TO MACHINE CODE WE REQUIRE A UTILITY PROGRAM WHICH IS KNOWN AS AN ASSEMBLER ASSEMBLY LANGUAGE PROGRAMS CANNOT BE EXECUTED DIRECTLY BY A COMPUTER THEREFORE THEY ARE SLOWER THAN THE MACHINE LANGUAGE PROGRAMSIT IS EASIER TO WRITE PROGRAMS IN THE MACHINE LANGUAGE THEN AND IN ASSEMBLY LANGUAGE

_______________________________________________________________________________________________

CLASS-VIII

SUBJECT-COMPUTER CHAPTER - Operating system and graphical user interface Role and functions

Need of operating system

It co-ordinates different hardware and software componentsof a computer system It supervises the various actions of the computer system and enables the computer to work in a effective

manner It helps in smooth functioning of various peripherals

In a multi-tasking operating system it determines the order and time to be allowed for each application before giving another application a turn

It sends messages to the system operator about the status of operation or any error that may have occurred while running the application

What is operating system

An operating system is an integrated system of programs that manages various resources and the overall operation of the computer system It is designed to support various activities of computer system in a systematic way

Role of an operating system-

Operating system enables the user to use the system effectively An operating system manages various application that runs on a computer and shares computerrsquos resources User interacts with operating system through command line interface and graphical user interface

Function of an operating system -

Booting the computer it is the start up procedure of a computer system Loading theprograms in the memory - when system is ready the operating system loads certain program

automatically Manages resources it manages between the hardware and software resources Detecting and correcting errors- if the supporting hardware or software doesnrsquot works properly then the

operatingsystem tries to rectify it Ensuring data security programs and data donot interfere with each other Maintaining the internal clock of system - maintain internal clock of system when system is shut down

SUBJECT-GEOGRAPHY CHAPTER 1 REPRESENTATION OF GEOGRAPHICAL FEATURES THROUGH CONTOURSTopographical maps- topographical maps are small-scale maps with detailed depiction of both natural and human-made features

Contour lines- contour lines are imaginary lines drawn on a map joining places having the same height above the mean sea level

Contour linesContour interval- It is an interval at which contour lines are drawnIndex contour- At every 100-m interval a thick brown contour line is called index contour It is drawn for calculating height

Spot heights- The spot heights show heights in metres above the mean sea level

SUBJECT- English Language CHAPTER - The Sentence

Complex sentence-

We have seen that a Complex Sentence consists of a Principal Clause with one or more Subordinate Clauses

We have also learnt that there are three kinds of Subordinate Clauses The Adjective Clause the Adverb Clause and the Noun Clause

1) THE ADJECTIVE CLAUSE

An ADJECTIVE CLAUSE does the work of an ADJECTIVE It qualifies a noun or pronoun In the following examples in each set two commonly patterned sentences are compared one with an ADJECTIVE and the other with an ADJECTIVE CLAUSE ( The Adjective or Adjective Clause qualifies the Noun placed in the box)

a) He is a lazy boy (Adjective)

b) He is a boy who is lazy (Adjective Clause)

a) It is a beautiful place for the tourists (Adjective)

b) It is a place where tourists come for the scenic beauty (Adjective Clause)

a) We have enough funds for the work (Adjective)

b) We have funds which would be enough for the work (Adjective Clause)

Convert the sentence from simple to complex sentence-

1) We believe his honesty Ans- We believe that he is honest

2) This is the birth place of RamaAns- This is the place where Rama was born

____________________________________________________________________________________________

Class IX

Subject English Language

Topic Preposition

Date 270420

PREPOSITIONS

A preposition is a word placed before a noun or a pronoun to show in what relation the person or thing denoted by it stands in regard to something else

EXAMPLE

There is a clock on the wall

The preposition on shoes relationship between clock and the wall

Smitha is afraid of lizards The man jumped off the bus

Here the preposition of shows the relationship between afraid and lizards

The preposition off shows the relation between jumped and bus

The noun or the pronoun which follows a preposition is called its object

So in the first Example wall is the object of the preposition on

A preposition can have more than one object For example

The plane flew overhouses and meadows

Prepositions are used to express a number of relationship including time locationmannermeans quantity purpose and state or condition

Points to remember

Preposition joins a noun to another noun or a pronounThere is a cow in the field

A preposition joints a noun to a verbThe cat runs after the rat

Preposition can have two or more than two objectsThe road runs over hills and plain

A preposition also joins a noun an adjectiveHe is fond of tea

Generally a preposition comes before an object Sometimes it comes even after an object asWhat are you looking atThis is the house I live in

Subject- Computer Application

Chapter 2 Introduction to Java

Java API An application programming interface (API) in the context of Java is a collection of prewritten packages classes and interfaces with their respective methods fields and constructors

Byte Code Java bytecode is the result of the compilation of a Java program an intermediate representation of that program which is machine independent The Java bytecode gets processed by the Java virtual machine (JVM) instead of the processor JVM The Java Virtual Machine (JVM) is the runtime engine of the Java Platform which allows any program written in Java or other language compiled into Java bytecode to run on any computer that has a native JVM

Platform A platform is the hardware or software environment in which a program remains

Java platform The Java platform differs from most other platforms in that itrsquos a software only platform that runs on top of other hardware-based platforms

Applet and Application The fundamental difference between the two Java programs is that an application program is designed to run on a stand-alone machine whereas an applet is a web-version of an application which is used to run a program on a web browser

WORA ldquoWrite once run anywhererdquo (WORA) or sometimes write once run everywhere (WORE) is a slogan created by Sun Microsystems to illustrate the cross-platform benefits of the Java language

Class XSubject Topic Summary Execution

ECONOMICSFACTORS OF PRODUCTION Sub-topiclsquoLABOURrsquo

We shall start our class by discussing the topic taught in the last class

lsquo DIVISION OF LABOURrsquo- By division of labour we mean specialization in workIt refers to splitting up the work of labour involved in the production of a particular commodity into several parts and each part and sub-part is performed by a specialist

Now let us start by the Advantages of Division of labouraIt increases the level

Questions

1 Differentiate between Product-Based division of labour and Process-Based division of labour

Product-Based division of labour

Process-based division of labour

It is also known as simple (or occupational) division of labourUnder it everybody performs a particular occupations The entire is done by the same person

When a person or group of persons undertakes a specialised function which is supplementary to the production of final commodity and service This is also as complex division of labour

It is simple It is complex

of productionbSince the product is produced by an expert workerbest quality of product is producedcIt saves time and toolsd it promotes inventions in the methods and techniques of productioneIt leads to reduction in costs fAll workers get work according to their abilities and choices

Now let us discuss the disadvantages of Division of labour

a Since many workers are involved in the production of a commodity no one has the sense of responsibility

b The constant and repetition of the same work again and again make the work monotonous

c Division of labour facilitates production on large scale Hencethere is fear of over production

d Because of territorial division of labour some areasregions become more developed than others

It is based on labour-intensive techniques of production

It is based on capital-intensive techniques

It is generally found in small enterprises

It is generally found in large enterprises

Example Indian farmers doing all farm activities

Example A modern garmet factory where one person takes the measurementanother does the cuttingsome sew the clothes while a few workers button them and other iron them

2 What are the advantages of Division of labouraIt increases the level of productionbSince the product is produced by an expert workerbest quality of product is producedcIt saves time and toolsd it promotes inventions in the methods and techniques of productioneIt leads to reduction in costs fAll workers get work according to their abilities and choices

3Discuss the disadvantages of Division of laboura Since many workers are involved in the

production of a commodity no one has the sense of responsibility

b The constant and repetition of the same work again and again make the work monotonous

c Division of labour facilitates production on large scale Hence there is fear of over production

d Because of territorial division of labour some areasregions become more developed than others

English 1 Transformation of sentences

Sentences A sentence is a group of words which makes complete sense

a Assertive sentences

Exercise 6Rewrite the following sentences according to the instructions given below without changing their meanings

1 As soon as he saw the beer he jumped into

b Imperative sentences

c Interrogative sentences

d Exclamatory sentences

Sentences can be changed from one grammatical form to another without changing the meaning of the sentence This is known as transformation of sentences

the river ( Begin No sooner)2 None but brave deserve the fair (Begin the

bravehellip)3 This box is too heavy for me to lift ( Use so hellip

That instead of too)4 No one other than a king can live like James

Luxurious ( Begin only James)5 Oh for the wings of a dove (Begin I wishhellip)

Math Topic Commercial MathematicsChapter Shares and Dividends

Study item Discuss about shares and Dividends1) What is share

Ans To start any big business (company or Industry) a large sum of money is needed But it is not possible for an individual to invest such a large amount Then some persons interested in the business join together and from a company They divide the estimated money required into small parts Each such part is called a share

2) What do you mean by the term shareholder

Ans A person who purchases one or more shares is called shareholder3) Some terms related with a share

(i) Nominal value or face value or printed value The original value of a share is called its nominal value or face value or printed value

Note The nominal value of a share always remains same(ii) Market value or cash value The price of a share at any

time is called its market value or cash value

Note The market value of a share changes from time to time(iii) At par If the market value of a share is the same as its

nominal value the share is called at par(iv) At Premium or above Par If the market value of a share

is more than its nominal value the share is called at premium or above par

Example If a share of Rs 100 is selling at Rs 150 then it is said to be selling at a premium of Rs 50 or Rs 50 above par

(v) At Discount or below par If the market value of a share is less than its nominal value the share is called at discount or below par

Example If a share of Rs 100 is selling at Rs80 then it is said to be selling at a discount of Rs 20 or at Rs 20 below par

4) What is Dividend

Ans The profit which a shareholder gets for hisher investment from the company is called dividendNote (i) The dividend is always expressed as the percentage of the face value of the share(ii) The dividend is always given( by the company ) on the face value of the share

irrespective of the market value of the shareBENGALI(2ND LANGUAGE)

ldquoদেবতোর জণমrdquoলিবরোম চকরবত

পরথম লিসর পোঠ-চোর পসথ একটি পোথর লিবপলি ঘটোয় দেক যোতোয়োসতর পসথ পরলিতলিয়ত ওই পোথসর দেো োচট দেসত একলি দেতো দেক দেো োচট দেসয় দেবোমো সয় রোসতোর মোস লি0টসক পস1 লিবপরীত লিক দেথসক আো একটি দেমোটর োলি1 চোসকর কষতোয় পরোসরণ দেবোসচ যো লিকনত পরলিতবোর এমরণ দেৌভোয দেসকর োও সত পোসর তোই লিতলি দেকোো দেজোো1 কসর পোথরটিসক উপস1 দে8স পোথর উপস1 দে8োর ময় এক দেকৌতী জতো দেকসক পরশন কসর দেয লিতলি দেকোসো দেবতোর আস দেপসয়স0রণ লিকো লিকনত দেক বস লিতলি দেকোসো দেবতোর আস পোলি দেক উপলিত ক জতোর উসltসয বস কোরও ইস= স পোথরটি লিসয় দেযসত পোসর এর পর দেথসক দেক দেযসত আসত পোথরটি দেক দেসত পো একলি দেক কষয করস দেকউ পোথরটিসক ধসয়মস0 পলিরসকোর করস0 দেক ওই দেকৌতী জতোসক পোথসরর কোস0 বস থোকসত দেস এ0ো1ো আরও কষয কসর দেক ঠোৎ ওই লি1 পোথরটির োসয় লিোর োো-দেকউ পজো কসরস0 দেকৌতী দেোকটির আঙকো য় যলিসক উ পোথরটিসক লিরসয় দে8স তোর পর ঠোৎ একলি পোথরটির দেোোজ দেই দেক লিসয় দেস0 বো দেকোথোয় দেস0

বদোথ-

দেো োচট ndashচসত লিসয় দেকো লিক0র সE ধোককো দেস পস1 যোবোর উপকরমঅকসমোৎ- ঠোৎআতমমবর- লিসজসক লিয়নতরপ-পো লিপ0স প1োদেসতসসত- লিসপলিউৎোত- দেো1ো দেথসক উপস1 দে8োপরতযয় ndash লিবশবোপরতযোস- টতযোস ndash দৈবোসধসতোধলিসত- পরসপসরর পরলিত ব পরসয়ো করোপরসতরীভত- পোথসর পলিররণলিতবোনতঃকরসরণ- মস পরোসঅলিQৎ- অQো করসত ই=কদেোপ- বধমোর- দেবোস এমইতযোকোর- এইরকমরম- বময়পযসোভী-পসযর জয দেোভ আস0 যোরপোসথয় ঞচয়- পথচোর রচ জমোসোমোমোসরোস- ব ধম ধোসমর সE

তোর লি দেইhelliphellipTo be continued

Hindi 2nd lang सर क पद(सरदास)

सरदास शरी कषण भकति कावय क सरवशरषठ कगिरव ह इनक जनम और मतय क समय तथा सथान का मतभद हसरदास रवातसलय और शरार रस क अनयतम कगिरव ह इनक कावय म बालकषण क सौदय चपपल चषटा और गि7याओ की मनोहर झाकी मिमलती ह कषण और ोगिपयो क अननय परम का कतिचतरण ह सयो शरार की अपकषा उनक कावय म गिरवयो शरार का अमिBक गिरवषय और मारमिमEक कतिचतरण हआ हइन पकतियो म हम सरदास की भकति भारवनाओ का परिरचय मिमलता ह इनका सपण सगरह सरसार म गिनगिहत ह

1 जसोदा हरिर पालन झलारवहलरारवदलराईमलहारव रव जो ईसाई कछ ारव मर लाल को आई निनEदिदया काह ना आगिनसबारव त काह नाही बरवगिह आरवतोको कानहा बलारव

शबदाथ-हलरारव-गिहलती हदलराई - दलार पयार करती हमलहारव-पचकारती हनिनEदरिरया ndashनीदरवगिह-जलदी सअBर-होठमौन-चपसन-सकत

वयाखया- सरदास जी कहत ह गिक यशोदा माता बालक कषण को पालन म झल आती ह रवह उनह गिहलाती ह पयार करती ह मलहार जस कोई ीत ान लती ह और नीद स पछती ह गिक ह नीद तम मर लाल को आकर कयो नही सलाती तझ खाना बला रहा ह कभी कषण आख बद कर लत ह कभी आखफडफडान लत ह उनह सोता हआ जानकर यशोदा माता चप हो जाती ह और इशार म बात करन लती ह इसी बीच अकला कर कषण ज जात हतो गिफर यशोदा माता गिफर स ाना ान लती ह सरदास जी कहत ह गिक भरवान क दशन का सख दरवता और ऋगिष-मगिनयो को भी दलभ ह यही सख माता यशोदा को बडी सहजता स मिमल जा रही ह माता यशोदा बहत ही भागयशाली ह2)Continue to nexthellip

Physics

Chapter 1 Force

(Summary)

Question A body is acted upon by two forces each of magnitude F but in opposite directions State the effect of the forces if

(a) Both forces act at the same point of the body

(b)the two forces act at two different points of the body at a separation r

Solutions

(a) Resultant force acting on the body = 0

F ndash F = 0(b) The forces tend to rotate the body between two forces about the midpoint

Moment of forces = F times rFr

QuestionDefine moment of a couple Write its SI unit

Solutions

Moment of couple is equal to the product of both force and the perpendicular distance between the two forces

The SI unit of moment of couple is NmCommercial Studies

Advertising and sales

Business firms use several methods to

Questions1) What do you mean by advertising

promotion create demand of their product in the market and increase it sales Such methods comprises of advertising sales promotion personal selling and publicityToday we are going to discuss about one of such methods It is advertising

Meaning of advertisingAdvertising is a paid form of non-personal presentation for promotion of Ideas goods and services

Importance or merits of advertising Advertising has importance to manufacturer or traders to customer and to society as a whole

Today we will see how advertisement help the manufacturer or traders

Answer) Advertising is a means of how a company encourages people to buy their products services or ideas It is one element of marketing which also includes design Research and data mining

2) Mention any three features of advertisingAnswer)The main features of advertising are

i) It is impersonal form of presentation for promotion of products and services of Ideas

ii) It is issued by identified sponsor The advertisement contains the name of the advertiser

iii) It is a form of mass communication because the message is directed to a large number of persons simultaneously

3) Mention the main merits or importance of advertisement to manufacturer or tradersAnswer)

i) Introducing new product A business organization can introduce itself and its products to the public through advertising

ii) Increase the sale Advertising leads to increase the sale of existing product by entering into new markets and attracting new customers

iii) Create steady demand Advertising creates sustains regular demand by smoothening out seasonal and other fluctuations It enables regular production for the organisation

iv) Economics of scale Advertising facilitate mass distribution of goods and steady demand which lead to large scale and regular production

v) Goodwill Advertising helps in creating a good image of the firm and reputation for its products

Biology Chapter - 03Genetics

Today wewill start chapter and discuss about Genetics Gregor Mendel is known as father of genetics Before entering into Mendelrsquos experiment on Genetics we must know

Q1 Define the following termsi) Genetics Genetics is the study of

transmissionof body features from parents to offspringand the laws relating to such transmission

ii) Heredity It may be defined as transmissionof genetically based characteristics from parentsto offspring

iii) Character and traits Any heritable

Importance to TraderIntroducing new productIncrease the saleCreate steady demandEconomics of scaleGoodwill

some terms featureis a character The alternative forms of acharacter are called traitsex Character (Hair shape) - Traits (Curly straight)

iv) Homologous chromosomes A pair ofcorresponding chromosomes of the same shapeand size one from each parent

v) Genes Genes are the specific parts (DNA segments) of a chromosome which determinethe hereditary characteristicsNearly 30000genes present in human

vi) Alleles Alternative forms of a gene occupying the same position (locus) on homologouschromosomes and affecting the same characteristicbut in different ways

vii) Genotype ndash PhenotypeGenotype means of genes present in the cells of an organism Phenotype means the observable characteristic which is genetically controlled

viii) Mutation It is a sudden change in one or more genes or in the number or in the structure of chromosomes ex Sickle cell anaemia is a blood disease caused by a gene mutation

CLASS NOTES

Class XSubject Eng Literature (The Merchant of Venice ndash William Shakespeare)Topic Act IV Scene 1 Lines 01 to 34 ( Duke helliphelliphellip We all expect a gentle answer Jew) ate 27th April 2020 (2nd Period)

[Students should read the original play and also the paraphrase given in the school prescribed textbook]Summary Questions amp Answers

This scene may be termed as the catastrophe of the play It is the final unravelling of the complicated events which seem to threaten the happiness of Bassanio Portia and Antonio Right is justified to the fullest degree and malice falls into the trap prepared for others No one suffers here but Shylock but even then he receives a generous measure of mercy

o This is the Court-scene Initially we meet

(1)

DUKE I am sorry for thee thou art come to answer (Line 3-6)A stony adversary an inhuman wretchUncapable of pity void and emptyFrom any dram of mercy

(i) Who is addressed here Where is the person Why is the person there

Antonio is addressed hereAntonio is in the court of justice at VeniceAntoniorsquos trial is scheduled to be held here for his failure to meet the conditions of the bond he signed with Shylock

the Duke Antonio Shylock and Salerio Later we meet Bassanio Portia Gratiano and Nerissa

o The Duke says to Antonio that he has to face a very cruel opponent which Antonio admits and expresses his gratefulness to the Duke for his efforts to soften without result the heart of Shylock in order to be merciful to Antonio Antonio further says that he is ready to accept whatever cruel judgement the Court may award

o When Shylock appears in the court the Duke says that Shylock should change his decision of prosecuting Antonio and demanding the penalty specified in the bond out of consideration of the great misfortunes that Antonio has suffered If this is done by him (Shylock) the whole court would be gladdened by his merciful action

(ii) What is the Duke sorry for

The Duke is unable to change the mind of Shylock from his decision to get the bond forfeited even after he pleaded to Shylock Shylock stands firmly for his bond which when forfeited will allow him to take a pound of flesh from any part of Antoniorsquos body(iii) How does the Duke address Shylock`The Duke calls Shylock an adversary with a heart of stones He calls Shylock as an inhuman wretch without pity Shylock is quite lacking in the slightest quality of mercy (iv) How does Antonio reply to this

Antonio replies that he will meet the revenge of Shylock patiently He has prepared himself to suffer with a quiet spirit the utmost that Shylockrsquos tyranny and rage can do(v) What quality of the Duke is revealed here

The Duke is kind and benevolent He is ready to help Antonio He requests Shylock to free Antonio from the trial(vi) What are the terms of the bond that Antonio has signed

The terms of the bond that Antonio has signed were that if Antonio is unable to repay Shylock a certain sum of money specified on the paper on a certain date and in an agreed place the forfeit has to be paid The forfeiture will be an exact pound of Antoniorsquos flesh which Shylock will be a liberty to take from any part of Antoniorsquos body which pleases him

Class XI

STUDY MATERIAL

Class XISubject Eng Literature (The Tempest ndash William Shakespeare) Topic Act I Scene 2 Lines 88 to 132 (Prospero hellip Me and thy crying self) Date 27th April 2020 (3rd Period)

[Students should read the original play and also the paraphrase given in the school prescribed textbook]Summary Questions amp Answers

o Prospero now tells Miranda that he was the Duke of Milan He had been devoting himself more to studies than the affairs of the State His brother Antonio took advantage of this situation and with the help of Alonso the king of Naples seized upon him and her one midnight and shipped them in a frail bark so that they perished in the sea All this took place

(1)

MIRANDA I should sin (Line 118-132)

To think but nobly of my grandmother

Good wombs have borne bad sonsPROSPERO Now the condition

The King of Naples being an enemyTo me inveterate hearkens my brothers suitWhich was that he in lieu othrsquo premisesOf homage and I know not how much tribute

twelve years back

IMPORTANT PASSAGES EXPLAINED(Line 98-103)

PROSPERO helliphelliphelliphelliphelliphelliphellip

He being thus lorded

Not only with what my revenue yieldedBut what my power might else exact like oneWho having into truth by telling of it Made such a sinner of his memoryTo credit his own lie he did believeHe was indeed the duke

Prospero in telling the narrative of his past life here refers to his brother Antonio Prospero being with a studious bent of mind has left the administration of Milan on his younger brother Now Antonio being thus invested like a lord with all the powers derived from Prosperorsquos wealth and what the exercise of Prosperorsquos authority might secure for him regarded himself as a de facto Duke of Milan It is a well-known fact of psychology that a man who repeatedly tells a lie makes of his memory such a sinner against truth as to credit his own lie by the telling of it So Antonio by repeatedly saying to himself and others that he was the Duke came to believe that he was really the Duke Thus falsehood repeatedly asserted gained the force of truth for Antonio and he truly believed it

Should presently extirpate me and mine Out of the dukedom and confer fair MilanWith all the honours on my brother whereonA treacherous army levied one midnightFated to thrsquo purpose did Antonio openThe gates of Milan and ithrsquo dead of darkness The ministers for thrsquo purpose hurried thenceMe and thy crying self

(i) In the earlier lines of this scene what does Prospero tell about his intense interest What was the demand of his interest

In the earlier lines of this scene Prospero tells Miranda that he had an intense interest in the study of philosophy and magic arts Hence in order to improve his mind with this kind of study he kept himself isolated from worldly and state affairs His study was dearer to him than the applause and esteem that he could win from the public His study demanded too much solitude(ii) What forced Antonio to take an undue advantage over Prospero

Prosperorsquos indifferent attitude towards the statersquos affairs and his having boundless trust in Antonio gave rise to a boundless lust for power in Antoniorsquos mind Antonio felt that he must be the actual Duke instead of the part of the Duke he played Thus Antonio took an undue advantage of the situation to usurp Prosperorsquos dukedom(iii) Explain the following lines ldquoI should sin to think but nobly of my grandmother Good wombs have borne bad sonsrdquo

After hearing the treacherous act of her uncle Antonio Miranda says that Prosperorsquos mother was a noble lady and she cannot dishonour her memory by saying that the person named Antonio cannot be his (Prosperorsquos) brother She says that in honour of her grandmother she also cannot say that Antonio must have been begotten not by her grandfather but by some other man She finally concedes that it is known that good mothers have borne bad sons in their wombs and gave birth to them

(iv) Why did the King of Naples accept Antoniorsquos request to help him in usurping his dukedom What did Antonio propose to Alonso

Alonso who was the king of Naples accepted Antoniorsquos request in usurping Prosperorsquos dukedom because he (Alonso) was a sworn enemy of Prospero Antonio proposed that Alonso should immediately drive him (Prospero) and his offspring out of Milan and should confer the dukedom upon him (Antonio) with all the dignities which go with that

position In return Antonio promised that he shall give an annual tribute and also swore his allegiance to Alonso Also he agreed to hold the Dukedom of Milan as a subordinate to the state of Naples(v) How were Prospero and Miranda carried away from the city of Milan and what was the state of small Miranda at that time

In pursuance of the agreement settled between Antonio and Alonso an army of treacherous men was assembled One midnight when the occasion suited the will of destiny Antonio opened the gates of the city of Milan and in the death like silence of midnight Antoniorsquos agents who had been directed to execute his purpose carried Prospero and small Miranda away from the city in all haste They were then forced into a ship and carried some distance out to sea where they put them on a mere hulk of a boat without any rigging or ship-gear and abandoned them leaving them at the mercy of the roaring sea Miranda was a very small child of three years age and she was crying at that time

CLASS -XIDATE-270420Subject Topic Summary Execution

EVS Chapter 1 ndash Mode of Existence

Impact of mode of existence on resources

Q) Why resources are under pressure

Ans - Increase in the sophistication

of technology enabling natural resources to be extracted quickly and efficiently Eg in the past it could take long hours just to cut down one tree only using saws Due to increased technology rates of deforestation have greatly increased

The number of humans is increasing Cultures of consumerism Materialistic views

lead to the mining of gold and diamonds to produce jewelry unnecessary commodities for human life or advancement Consumerism also leads to extraction of resources for the

production of commodities necessary for human life but in amounts excessive of what is needed because people consume more than is necessary or waste what they have

Lack of awareness among the population is striking People are not aware of ways to reduce depletion and exploitation of materials

Accounts Cash Book Today we are going to start a new topic -Cash Book

The key terms used in this chapter are

bullCash book

bullSimple cash book

bullDouble column cash book bullTriple column cash book

bullPetty cash book

bullCash discount

bullContra entry

Here I will share you the meaning of each key terms

bullCash book Cash Book is a special purpose subsidiary book or journal in which cash received and cash payments are recorded

bullSimple cash book

It is a cash book in which only cash transactions are recorded It has only one column on each side

bullTriple column cash book

It is cash book which has three columns one column for each cash and Bankdiscount on each side of the cash book In this book both cash and Bank transactions are recorded together with discount allowed and received

bullPetty cash book

It is a cash book maintained for recording petty expenses

bullCash discount

Cash discount is the amount of discount received or allowed on cash payments and cash receipts Discount received is an income for the business while discount allowed isan expense

bullContra entry

It means transactions involving both cash and Bank Such transactions though recorded in the cash book are not posted into ledger The letter lsquoC is written in Ledger folio for contra entry

Business Studies

ENTREPRENEURSHIP

Now we shall discuss the second chapter

lsquoENTREPRENEURSHIPrsquo

Today before starting the chapter let us recall what

Questions

1What are the main characteristics of Intrapreneurship

Answer

The main characteristics of Intrapreneurship are

Corporate framework-it occurs within the framework of the same company

Semi-Autonomous-Intrapreneurship

we have read last day

Let s today start the class by recalling the last topic taught

Intrapreneurship is the process of discovering and exploring business opportunities within an existing company It involves launching new business ventures within the framework of a present corporation Intrapreneurship is also known as corporate entrepreneurship or corporate venturing

Now let us start with the characteristics of Intrapreneurship

The main characteristics of Intrapreneurship are

Corporate framework

Semi-Autonomous Lack of ownership Senior position Low risk taking Not own boss

Now let us discuss the meaning of enterprise

Enterprise means an undertaking or adventure that requires some innovation and investment and thus involves riskEnterprise always entails decision making coordination and risk bearing

involves crating amd nurturing a semi-autonomous business unit which may be a subsidiary a strategic business unit or a division

Lack of ownership-the intrapreneur is not the owner of the unitb he creates and nurtures

Senior position-he occupies a senior managerial position in the company

Low risk taking-An intrapreneur does not bear the full risk of failure

Not own boss-An intrapreneur is not his own bosss in legal termsHe enjoys the freedom and gets the required resources and support

2 How is Entrepreneur is different from Intrapreneur

The functions involved in both the entrepreneurship and intrapreneurship are by and large similar however there are several differences between the two

Point of distinction

Entrepreneur

Intrapreneur

status An independent business person

A senior executive within a company

Ownership Owner of

the business

An employeesometimes a share in ownership

Financing Responsible for raising finance for the business

Not responsible for raising the finance

Risk bearing

Bears the risk of the business

Does not bears the risk of the business

Reward Profit which is uncertain and irregularcan be loss

Fixed salary and fringe benefits

Need for security low high

3 What do you understand by enterprise

Answer Enterprise means an undertaking or adventure that requires some innovation and investment and thus involves riskEnterprise always entails decision making coordination and risk bearing

COMMERCE NATURE AND OBJECTIVES OF

BUSINESS

Today let us recall the last other two objectives of business by the chart given in the previous class

Firstly we would discuss Human Objectives

Business is run by people and for people Labour is a valuable business element

Human objectives of business are concerned with the well -being of labour

The human objectives are as follows

Labour welfare Developing human

resources Participative

management Labour

management cooperation

Questions

1 Explain the human objectives of a business enterprise

Answer

Business is run by people and for people Labour is a valuable business element

Human objectives of business are concerned with the well -being of labour

The human objectives are as follows

Labour welfare-Business must recognize the dignity of labour and human factors should be given the recognition

Developing human resources-Employees must be provided the opportunities for developing new skills and attitudes

Participative management-Employees should be allowed to take part in decision making process of business

Labour management cooperation-Business should strive for creating and maintaining cordial employer employee relations so as to ensure peace and progress in industry

Now let us discuss the national objectives of business

Optimum utilization of resources

National self- reliance Development of small

scale industries Development of

backward areas Control over pollution

2Explain the national objectives of a business enterprise

Answer

It is the duty of business to utilize the resources of the country properly the national objectives of business

Optimum utilization of resources ndashBusiness should use the nationrsquos resources in the best possible manner

National self- reliance-It is the duty of the business to help the government in increasing experts and in reducing dependence on imports

Development of small scale industries-Big business firms are expected to encourage growth of small scale industries which are necessary for generating employment

Development of backward areas-Business is expected to give preference to the industrialization of backward regions of the country

ECONOMICS

BASIC ECONOMIC CONCEPTS

SUB

TOPIC

Value

Wealth

Welfare

Today we shall start with a new topic of the same chapter ie lsquoValuersquo

Value of a commodity is defined as the valuation placed by a household on the consumption of this commodity

lsquoValuersquo has two different meanings and these are

a Value -in -use It refers to consumption value of a commodity It expresses the utility derived from the consumption of a particular commodity A necessity like water has a very high value ndashin ndashuse or

Question

1What is value

Answer

Value of a commodity is defined as the valuation placed by a household on the consumption of this commodity

2What is value-in use

Answer It refers to consumption value of a commodity It expresses the utility derived from the consumption of a particular commodity A necessity like water has a very high value ndashin ndashuse or consumption value

3What is value ndashin- exchange

Answer It relates to market value of a commodity

It is the rate at which a particular good or service can be exchanged for money

For example in barter system if a person is prepared to exchange 3 metres of cloth with 1 pair

consumption value

b Value ndashin-exchange It relates to market value of a commodity

It is the rate at which a particular good or service can be exchanged for moneyFor example in barter system if a person is prepared to exchange 3 metres of cloth with 1 pair of shoes then the value in exchange of 3 metres of cloth is 1 pair of shoesValue in exchange is the power of purchasing other goods In modern monetised economies the exchange value of goods are expressed in terms of money as prices

Now let us discuss the term lsquoWealthrsquo

Wealth refers to the stock of all those assets which are a source of income

Wealth is a stock concept

Wealth must possess the following features

a Utility It must possess utility or give some

of shoes then the value in exchange of 3 metres of cloth is 1 pair of shoes

Value in exchange is the power of purchasing other goods In modern monetised economies the exchange value of goods are expressed in terms of money as prices

4 What is wealth

Answer it refers to the stock of assets or goods which are a source of income and have personal or national ownership

5 What are the features of wealth

Answer The features of wealth are as follows

Wealth must possess the following features

a Utility It must possess utility or give some satisfaction

b Scarcity It must be limited in quantityc Transferability it should be transferable its

ownership can be transferred from one person to another person

d Exchange value It must possess exchange value

6 What is welfare

Answer

Welfare is defined as satisfaction and happiness a sense of well- being among the people

satisfactionb Scarcity It must be

limited in quantityc Transferability It

should be transferable its ownership can be transferred from one person to another person

d Exchange value It must possess exchange value

Now let us discuss the term lsquoWelfarersquo

Welfare is defined as satisfaction and happiness a sense of well- being among the people

Welfare is affected by factors like

a Consumption of goods and services

b Environment

c Family relations

d Degree of freedom

e Law and order situation

Mathematics Trigonometric equation

To find the general solution of the equation sinθ=0

When sin θ =0

Then θ= 0 π2π 3π-π -2π -3

i e when θ = 0 or an integral multiple of π

i e when θ= nπ where n is any integer

Therefore the general solution of the equation sin

Example1 Find the general values of θ which satisfy the equation sin2 θ =34

Solution sin2 θ= 34

Or sin θ = +34 or -34

Or sin θ = sin π3 or sin (-π3)

Therefore

θ = [nπ + (-1) n (π3)] or[ nπ+ (-1) n (-π3)]

= nπ +π3 or nπ-π3 where n= any integer

Example 2Find the values of θ which satisfy tan2 θ

θ=0 is θ= nπ where n is any integer

To find the general solution of the equation cos θ=0

When cos θ=0

Then θ=π2 3π2 5π2 -π2 -3π2 -5π3

i e when θ is an odd multiple of π2

i e when θ=(2n+1) π2 where n= any integer

Therefore the general solution of the equation cos θ =0 is θ= (2n+1) π2 where n= any integer

To find the general solution of the equation tan θ = 0

Clearly tan θ =0 implies sin θcos θ =0

Therefore θ = nπ

i e the general solution of the equation tan θ=0 is θ =nπ where n = any integer

To find the general solution of the equation cot θ =0

Clearly cot θ =0 implies (cos θsin θ) = 0

i e cos θ =0

Therefore θ = (2n+1) π2

Therefore the general solution of the equation cot θ =0 is θ = (2n+1) π2

Where n= any integer

To find the general solution of the equation sin θ= k (-

=13 -πleθleπ

Solution tan2 θ =13

Or tan θ = plusmn1radic(3) =tan(plusmnπ6)

θ=nπ plusmn π6 where n =any integer

If n=0 then θ=plusmnπ6

If n=1 then θ= π plusmn π6

If n=-1 then θ= -π plusmn π6

Therefore the required solution in -π le θ le π are θ= π6 5π6 -π6 -5π6

Exercise Find general solution of sin 2θ=cos θ [Hints Use sin 2θ= 2sin θcosθ and then take cosθ

common]

1lek le1)

Determine an angle alpha such that sin =k and -π2le αle π2

Then we have

Sin θ = k = sin α

Or sin θ - sin α =0

Or 2 cos [(θ+α) 2] sin [(θ-α) 2] =0

Therefore either cos [(θ +α) 2] =0 (1)

Or sin [(θ-α) 2] =0 (2)

Now from (1) we get (θ+α) 2= (2m+1) π2)

Or θ = (2m+1) π-α (3)

And from (2) we get (θ-α) 2 =mπ

Or θ= 2mπ+α(4)

Where m = any integer

Clearly the solution (3) amp (4) may be combined in the following form

θ= nπ+(-1) n α where n= any integer

Therefore the general solution of sin θ = sin α is θ = nπ +(-1) n α where n is any integer and -π2 le α le π2

Biology Chapter - 04Kingdom Monera

Today we will discuss about bacterial reproduction and its usefulness

Fig Binary Fission

Fig Conjugation Fig Transformation

Bacterial reproduction is mainly asexual but sexual reproduction

also takes place

Asexual reproduction takes place by i) Binary fission - from one bacteriato

two bacteria are produced in every 20 to 30mins

ii) Buddingiii) Endospore formation - during

unfourable condition

Sexual reproduction by three ways

1) Conjugation - Transfer of genetic material between cells that are in physical contact with one another

2) Transduction - Transfer of genetic materialfrom one cell to another by a bacteriophage

3) Transformation - Transfer of cell-freeor naked DNArsquo from one cell to another

Bacteria causes different diseases inplants animals and human and

it causes food spoilage and waterpollution but it also have some useful

activities

i) Bacteria are helpful in sewage water treatment

ii) It is used in antibiotic (medicine) production

iii) Anaerobic bacteria help in biogas(energy) production

iv) Many household products like yoghurt cheese are manufactured by use of bacteria

v) Rhizobium by symbiotic relationship with leguminous plant increase soil fertility

vi) Besides these bacteria is helpful in genetic engineering degradation of petroleum hydrocarbonand in dairy

industry

Physics Motion in plane Here we will introduce Projectile Motion

Execution

Projectile

Y

usinθ u h

θX

ucosθ

Suppose a body is projected with an angle θ So initial velocity u can be resolved into two components

Horizontal component - ucosθ ( for range)

Vertical component - usinθ ( for height)

usinθ changes during motion and becomes zero at maximum height position but ucosθ remain unchanged

The maximum height of projectile is h

NB If initial is upward then g = -ve and if it is downward then g = +ve Height is +ve if direction of motion does not change ( for ex a body thrown upwards but goes down ultimately then height h = -ve)

The angle of projectile θ is the angle made with horizontal

HISTORY ndash GROWTH OF NATIONALISM

SUB TOPIC- REVOLUTIONARY NATIONALISM Bengal formation of Anushilan Samity and Jugantar Group

The intensification of the Swadeshi movement and Government policy of terror and repression led to outbreak of violence Bombs were manufactured and attempts on the lives of unpopular Government officials became frequent In the gymnasium of Scottish Church College which was known as General Assemblies Institution a secret society was formed known as Anushilan Samity

Aurobindo Ghosh send from Baroda his emissary Jatindranath Banerjee to mobilize the Bengal revolutionaries

Hemchandra Qanungo and Satyen Bose published Journal Jugantar

The Jugantar group planned to assassinate oppressive magistrate Kingsford by Khudiram Bose and Prafulla Chaki in 1908 Prafulla Chaki committed suicide to avoid arrest Khudiram was tried and hanged

Afew days later the police found a bomb factory in Maniktala and arrested a large number of revolutionaries The trial of revolutionaries became famous as the Alipore Bomb Case

In the course of the trial the approver the public prosecuter and a police officer were assassinated

1 Question Name two journals which preached the cult of violence

Answer a) Yugantar edited by Bhupendranath DuttaB) Bandemataram edited by Aurobindo Ghosh2 Question Why was Khudiram arrested and hangedAnswer An attempt was made to assassinate a hated vindictive majistrate named Kingsford by Khudiram Bose and Prafulla Chaki Their attempt failed and the bomb they threw killed two English ladies Khudiram was arrested and put to trial and then hanged3Question Who was Aurubindo GhoshAnswer Aurobindo Ghosh a nationalist revolutionary who was charged for his involvement in the Alipore Bombing Case He was accused of it along with his brother Barindra nath Ghosh But Aurobindo was acquitted because of the brilliant pleading of his counsel Chittaranjan Das Then he became a spiritual reformer introducing his visions on human progress and spiritual evolution4 Qustion Who was KingsfordAnswer Kingsford was an unpopular British chief Magistrate who was the target of the bomb thrown at Muzaffarpur by Khudiram and Prafulla Chaki

Most of the accused were convicted and sentenced to

long term of imprisonmentBut

Aurobindo Ghosh was acquitted mainly owing to the brilliant pleading of his counsel Chittaranjan Das

Political science Topic-Sovereignty

Summary Sovereignty is the full right and power of a governing body over itself without any interference from outside sources or bodies In political theory sovereignty is a substantive term designating supreme legitimate authority over some polity In international law sovereignty is the exercise of power by a state

Internal Sovereignty

Internal sovereignty means supreme authority within ones territory while external sovereignty relates to the recognition on the part of all states that each possesses this power in equal measure

External sovereignty

external sovereignty relates to the recognition on the part of all states that each possesses this power in equal measure

Distinguish between

Execution

Answer the following questions

Short notes-

Sovereignty

Internal Sovereignty

External sovereignty

Homework- learn

external sovereignty and internal sovereigntySovereignty is the principle

of supreme and

unquestionable authority

reflected in the claim by the

state to be the sole author of

laws within its territory

Definition of external vs internal sovereigntyInternal sovereignty refers to

the relationship between a

sovereign power and its

subjects ndash it refers to the

location of the supreme

authority within the state In

the UK for example internal

sovereignty (supposedly)

resides within Parliament

reflected in the

constitutional principle of

parliamentary

sovereigntyBy contrast

external sovereignty refers

to the capacity of the state

to act independently and

autonomously on the world

stage This is what is

sometimes called lsquostate

sovereigntyrsquo or lsquonational

sovereigntyrsquo and implies

that states are legally equal

and that the territorial

integrity and political

independence of a state is

inviolable

Class ndash XII

Date - 2742020 STUDY MATERIALSubject Topic Summary Execution Business Studies

Job Analysis amp Manpower Planning

At first let us recall the chapter what we have discussed till nowbullJob analysisbullJob specification bullJob description bullJob enlargement bullJob enrichment

Today we will do some questions answers from the chapter

Questions 1ldquoJob analysis job description and job specification are interrelatedrdquo Comment Answer) Job analysis is a systematic and detailed examination of a job to collect all the relevant information about it The contents off the job are summarised in the job description The qualification needed for the job are summarised in job specificationThus there is close interrelationship between job analysis job description and job specification

Question 2ldquoJob enlargement is a horizontal extension of a job whereas job enrichment is a vertical extension of a jobrdquo ElucidateAnswer) Job enlargement involves adding one or more task to a job coma where as job enrichment involves adding more autonomy and responsibility to a job Job enlargement is therefore horizontal extension of a job coma whereas job enrichment is a vertical extension of a job

Question 3 )

What is manpower estimation Explain its quantitative and qualitative aspectsAnswer) Manpower estimation is the process by which management determines how an organisation should move from itrsquos current manpower positionto its desired manpower position There are two dimensions of Manpower estimation- quantitative and qualitative

Quantitative aspectThis aspect of Manpower estimation involves estimating the number of employees required in a future time period Workload analysis and workforce analysis are done to estimate the quantity of required manpower

Qualitative aspectThe estimate of the knowledge skills experience etc of required manpower is the qualitative aspect of Manpower estimation The quality of Manpower can be judged on the basis of job analysisand job specification

COMMERCE

CAPITAL-FIXED AND WORKING

Today let us start the class by discussing the sources of finance for different types of business firms

The term lsquocapitalrsquo refers to the investment made in the enterprise for the purpose of earning profits

Requirements of capital and sources of capital for different types of business firms are

1 Capital for sole proprietorship businessA sole proprietor operates at a small scale and thereforerequires a limited amount of capital

2 Capital for partnership firmCapital requirements as well as capital base of a partnership is bigger than that of a sole trader businessThe owned capital is contributed by the partners in an agreed ratio

3 Capital for joint stock companyA joint stock company generally requires large amount of capitalA public company can raise huge capital through issue of shares In addition to share capital it can utilize retained profits

Now let u discuss the meaning of Finance PlanningFinance planning is the process of estimation the financial requirements of an organization specifying the sources of firms and ensuring that enough funds are available at the right time

1 What do you mean by Finance PlanningAnswer Finance planning is the process of estimation the financial requirements of an organization specifying the sources of firms and ensuring that enough funds are available at the right time

2Discuss the role of financial planning of an enterpriseThe role of financial planning are as followsa A sound financial plan helps a business enterprise to avaid the problems of shortage and surplus of fundsbFinancial planning serves as a guide in developing a sound capital structure so as to maximize returns to shareholders c It helps in effective utilization of fundsd It provides policies and procedures for coordinating different functional areas or departments of businesse It enables the management to exercise effective control over the financial activities of an enterprisef It helps the company to prepare for facing business shocks and surprises in future

Mathematics

Continuity and differentiability

Recall Definition of ContinuityLet f(x) be a single valued function of x and x=a be a point in the domain of definition of the function The function is said to be continuous at x=a ifi) f(c) is defined ie f(x) has a definite finite value at x=cii) lim xrarra f(x) exists andiii) lim xrarra f(x) =f(a) In other words f(x) is said to be continuous at x=a if lim xrarra+ f(x)= lim xrarra- f(x) = f(a) Or f(a+0) =f(a-0) =f(a) Or lim hrarr0 f(a+h)= f(a) Algebra of continuous functionsNow we will study some algebra of continuous functions Theorem 1 Suppose f and g be two real functions continuous at a real number c Then(1) f + g is continuous at x = c(2) f ndash g is continuous at x = c(3) f g is continuous at x = c(4) (fg) is continuous at x = c (provided g (c) ne 0)

Example 1 Prove that every rational function is continuousSolution Recall that every rational function f is given byf(x)=[p(x) q(x) ] q(x)ne0where p and q are polynomial functions The domain of f is all real numbers except points at which q is zero Since polynomial functions are continuous f is continuous by (4) of Theorem 1Example 2Discuss the continuity of sine functionSolution To see this we use the following factslim xrarr0 sin x =0Now observe that f (x) = sin x is defined for every real number Let c be a real number Put x = c + h If x rarr c we know that h rarr 0 Therefore lim xrarrc f(x)

= lim xrarrc sin x= lim hrarr0 sin(c+h) =lim hrarr0 [sin c cos h + cos c sin h ]=lim hrarr0 (sin c cos h) + lim hrarr0 (cos c sin h) = sin c +0=sin c = f(c) Thus lim xrarrc f(x) = f(c) and hence f is a constant function Exercise Prove that the function f(x) = x2 +2x is continuous for every real value of x [Hints show that lim xrarra+ f(x) = lim xrarra- f(x) = f(a) ]

Biology Reproduction in Flowering plants We will discuss about megasporoangium

megasporagenesis and female gametophyte

Q4 Describe the structure of megasporangium

Ovule is attached to the placenta by astalk called funicle

Each ovule has one two or three protectivecoverings called integuments

At the tip of integuments a small openingcalled micropyle is organised

Opposite to the micropylar end is the chalaza

Within the integuments a mass of cellsnucellusand inside it embryo sac orfemale gametophyte is present

Q5 Describe a mature embryosacamp its formation

In most of the flowering plants only oneof the 4 megaspores formed as a result ofmegasporogenesis that is functional while theother three degenerate

The

functional megaspore develops into thefemale gametophyte

Formation The nucleus

of the functional megasporedivides mitotically to form two nuclei first andthen two more sequential mitotic nucleardivisions result in the formation of four ampthen eight nucleate stages of embryo sac

Six of the eight nucleus are surrounded bycell walls and organised into cells

The remaining two nuclei called polar nuclei are found below the egg apparatus in the largecentral cell

Three cells consisting of two synergids amp one egg cell present bottom of

embryo sac Three cells

at the chalazal as antipodal cells

Two polar nuclei together present in large central cell

HISTORY

TOWARDS INDEPENDENCE AND PARTITION THE LAST PHASE(1935-1947)SUB TOPIC NATIONAL MOVEMENTS DURING THE SECOND WORLD WAR

Spread of Quit India Movement On 9th August 1942Gandhiji and other Congress leaders were arrested The Congress was declared illegal The news of the arrest of all leaders marked the beginning of a widespread movement of India It was not possible for such a movement to remain peacefulBut the arrest of the all notable congress leaders virtually left the movement in the hands of the mass The movement took the form of violent and militant outbreakBesides congressmen revolutionaries also were very active in the movement The Congress Socialist group also played a prominent role

1 Question Why did the British authority arrest the Congress leaders on 9 th August 1942Answer Congress Working committee adopted the Quit India resolution which was to be ratified at the Bombay AICC meeting in 8th August 1942 They decided to launch a mass struggle on non-violent lines Gandhiji gave a clarion call to all section of the people rdquoKarenge ya Marengerdquo (do or die) Congress leaders gave the call to driving out

the British from IndiaViceroy had taken strong action against the Quit India movement Gandhiji and all the leaders of Congress were arrested

2 Question How did Quit India Movement spread out all over IndiaAnswer The news of the leaders lsquo arrest marked the beginning of a widespread movement to remain peacefulThe movement took form of violent outbreak There were widespread cutting of telephone and Telegraph wires damaging railway lines raising barricades in cities and towns and other forms of violent demonstations

Question Name the leaders of Congress

Socialist group played a prominent part Notable among the Jayprakash Narayan Rammonohar Lohia Aruna Asaf Ali

Political science

Topic-Franchise and Representation

Summary

The election commission

The Election Commission of India is an autonomous constitutional authority responsible for administering Union and State election processes in India The body administers elections to the Lok Sabha Rajya Sabha State Legislative Assemblies in India and the offices of the President and Vice President in the country

Functions of election commission-

India is a sovereign socialist secular democratic republic Democracy runs like a golden thread in the social economic and political fabric woven by the Constitution given by lsquoWe the People of Indiarsquo unto ourselves The concept of democracy as visualised by the Constitution pre-supposes the representation of the people in Parliament and State legislatures by the method of election The Supreme Court has held that democracy is one of the inalienable basic features of the Constitution of India and forms part of its basic structure The Constitution of India adopted a Parliamentary form of government Parliament consists of the President of India and the two Houses mdash Rajya Sabha and Lok Sabha India being a Union of states has separate state legislatures for each state State legislatures consist of the Governor and two Houses mdash Legislative Council and Legislative Assembly mdash in seven states namely Andhra Pradesh Telangana Bihar Jammu amp Kashmir Karnataka Maharashtra and Uttar Pradesh and of the Governor and the state Legislative Assembly in the remaining 22 states Apart from the above two out of the seven Union Territories namely National Capital Territory of Delhi and Puducherry also have their Legislative Assemblies

ExecutionShort notes-Election commissionFunctions of election commission

Homework- Learn

Computer

Science

Computer hardware NAND Gate

A NOT-AND operation is known as NAND operation It has n input (n gt= 2) and one output

Logic diagram

Truth Table

NOR Gate

A NOT-OR operation is known as NOR operation It has n input (n gt= 2) and one output

Logic diagram

Truth Table

XOR Gate

XOR or Ex-OR gate is a special type of gate It can be used in the half

adder full adder and subtractor The exclusive-OR gate is abbreviated as EX-OR gate or sometime as X-OR gate It has n input (n gt= 2) and one output

Logic diagram

Truth Table

XNOR Gate

XNOR gate is a special type of gate It can be used in the half adder full adder and subtractor The exclusive-NOR gate is abbreviated as EX-NOR gate or sometime as X-NOR gate It has n input (n gt= 2) and one output

Logic diagram

Truth Table

Physics

Chapter 1 Electric Field ( Electric Dipole) (Summary)

Here we will derive Expression of electric field at broad side

On position of dipole

Execution

Q With the help of a labelled diagram obtain an expression for the electric field intensity E at any point on the equitorial line ( broad-side on position) of an electric dipole

Ans

E1 E1sinθ

E θ P E1 θ

( r2+L2)12 E2 E

r E2 E2sinθ

-q θ L O L +qA B

Let us consider that the point P is situated on the right bisector of the dipole AB at a distance r meter from its midpoint O

Let E1 and E2 be the electric field intensities of the electric field at P due to charge +q and ndashq of the dipole resp The distance of P from each charge is ( r2+L2)12

So E1 = 14 πϵ q

(r 2+L 2) away from +q

E2 = 14 πϵ q

(r 2+L 2) towards ndashq

The magnitudes of E1 and E2 are equal but directions are different Now resolving E1 and E2 into two components parallel and perpendicular to AB we get

The components perpendicular to AB E1sinθ and E2sinθ cancel each other because they are equal and opposite

The components parallel to AB are E1cosθ and E2 cosθ are in same direction and add up

So resultant intensity of electric field at the point P is

E = E1cosθ + E2 cosθ

E = 14 πϵ q

(r 2+L 2) 2 cosθ

Now from fig we have cosθ =BOBP = L (r2+L2)12

So we get E = 14 πϵ 2qL ( r2+L2)32

Now electric dipole moment p= 2qL

So E = 14 πϵ p ( r2+L2)32

HW Find the expression of Electric field as done here but this time take r gtgt 2L

Also find the expression of torque experience by a dipole

(Hint Electric force experienced by charges of dipole in electric field is qE each Let θ be the angle which dipole makes with electric lines of force then perpendicular distance between two charges is 2Lsinθ Then torque = force x perp distance = qE x 2L sinθ So τ=pE sinθ where p =2qL )

STUDY MATERIAL

Class XIISubject Eng Literature (The Tempest ndash William Shakespeare) Topic Act IV Scene 1 Lines 84 to 133 (Iris hellip A contract of true love Be not too late ) Date 27th April 2020 (4th Period)

[Students should read the original play and also the paraphrase given in the school prescribed textbook]Summary Questions amp Answers

o Ceres soon appears and comes to know that she has been summoned to celebrate the contract of true love

o Ceres expresses her unwillingness to meet Venus and Cupid as she has shunned their company

o Ceres and Juno both bestow their blessings upon Ferdinand and Miranda with June gifting honour riches happiness in marriage and Ceres presents plenty of earthrsquos produce

o Iris summons the water-nymphs and reapers to come and celebrate a contract

(1) IRIS Of her society (Line 91-101)

Be not afraid I met her deity

Cutting the clouds towards Pathos and her sonDove-drawn with her Here thought they to have doneSome wanton charm upon this man and maidWhose vows are that no bed-right shall be paidTill Hymens torch be lightedmdashbut in vainMarss hot minion is returned againHer waspish-headed son has broke his arrowsSwears he will shoot no more but play with sparrowsAnd be a boy right out

(i) Where were Venus and Cupid seen flying How were they travelling Why did they want to join the marriage celebration of Ferdinand and Miranda

of true love

Venus and Cupid were seen flying through the air towards Paphos the famous city which is situated on the island of Cyprus They were travelling by air-borne chariot drawn by doves They certainly wanted to come here in order to play some amorous trick upon Ferdinand and Miranda who are under a vow not to gratify their physical desires till the holy ceremony of their marriage has been performed(ii) What have Venus and Cupid done after failing in their plan

After being failure of their plan Venus who is a very passionate deity and who is the mistress of Mars (the god of war) has gone back while here ill-tempered son Cupid has broken his arrows of love in his state of desperation(iii) What has Cupid firmly decided

Cupid is feeling so disappointed that he has firmly decided to shoot no more arrows to arouse love in human hearts but to spend his time playing with sparrows Thus he would now become just a boy and would give up his original function of shooting arrows on human beings to make them fall in love(iv) What vow had Ceres taken How did Ceres feel at the abduction

After the abduction of her daughter Prosperina by Pluto Ceres had taken a vow to always keep away from the disgraceful company of Venus and her blind son Cupid the god of love Ceres felt deeply distressed when Pluto had carried off her daughter and had made her his wife by force(v) Why has Ceres not forgiven Venus and her blind son For what do Ceres want to be sure

As the abduction had been manipulated by Venus the goddess of beauty and love and her blind son Cupid Ceres has never forgiven them for their part in the whole plot Ceres wants to be sure that she would not have to meet Venus and Cupid who had engineered the abduction of her daughter Prosperina

AS THIS lsquoMASQUErsquo SCENE IS VERY IMPORTANT IN THE PLAY THE PARAPHRASE OF THE ENTIRE PORTION OF MASQUE SCENE (Act IV Lines 58 to 143) IS GIVEN BELOW

IRIS Goddess of RainbowCERES Goddess of Agriculture and all the fruits of the earth

(Nature growth prosperity rebirth ndash notions intimately connected to marriage)JUNO The majestic Queen of Heavens and wife of Jupiter (Jupiter is the king of Gods)

VENUS The Goddess of love CUPID Son of Venus PLUTO God of death (In the play referred by Shakespeare as lsquoDisrsquo which is a Roman name for Pluto)

ORIGINAL TEXT PARAPHRASEPROSPEROWellmdash

PROSPERONow come Ariel Let there be too many rather than too few

Now come my Ariel Bring a corollaryRather than want a spirit Appear and pertly[to Ferdinand and Miranda]No tongue all eyes Be silent

spirits in attendance Appear briskly

[to Ferdinand and Miranda]Look with your eyes but do not say a word

[Soft music] [Soft music][Enter Iris] [Enter Iris]

IRISCeres most bounteous lady thy rich leasOf wheat rye barley vetches oats and peasThy turfy mountains where live nibbling sheepAnd flat meads thatched with stover them to keepThy banks with pioned and twilled brimsWhich spongy April at thy hest betrimsTo make cold nymphs chaste crowns and thybroom-grovesWhose shadow the dismissegraved bachelor lovesBeing lass-lorn thy pole clipped vineyardAnd thy sea-marge sterile and rocky-hardWhere thou thyself dost airmdashthe Queen othrsquoSkyWhose watery arch and messenger am IBids thee leave these and with her sovereign grace[Juno appears] Here on this grass-plot in this very placeTo come and sport Her peacocks fly amainApproach rich Ceres her to entertain

IRISCeres most generous lady you are the cause of rich fields or fertile land where wheat rye barley beans oats and peas grow the grassy mountains where the sheep graze and the flat meadows covered with coarse hay to be used as fodder for cattleYour banks are covered with marsh-marigolds and reeds and the rainy April under your orders brings forth to make for the maids who are not in love beautiful crowns your woods where the broom flourishes and where the bachelor who has been dismissed by the maid he loved lies down being forsaken your vineyard in which the poles are embraced by the vines and the margin of the sea which is barren and rocky where you roam about to enjoy the fresh air ndash the queen of the sky (Juno) whose messenger I am besides being represented as the rainbow bids you leave all these and with her majesty here on this grassy plot in this very place come and sport her peacocks carry her fast in her chariot through the air and are making their way here approach rich Ceres to welcome her

[Enter Ariel as Ceres] [Enter Ariel as Ceres]

CERESHail many-coloured messenger that neerDost disobey the wife of JupiterWho with thy saffron wings upon my flowersDiffusest honey-drops refreshing showersAnd with each end of thy blue bow dost crownMy bosky acres and my unshrubbed downRich scarf to my proud earth Why hath thy queenSummoned me hither to this short-grassed green

CERESWelcome rainbow that never dared disobey Juno the wife of Jupiter who with your orange coloured rays spread honey-drops refreshing showers And with each end of thy blue bow drown my bushy acres and my hilly country which is free from shrubs you thus forming a rich scarf Why has your queen called me here to this place covered with short grass

IRISA contract of true love to celebrateAnd some donation freely to estateOn the blest lovers

IRISI have called you to celebrate a contract of true love and bestow some liberal gift upon the blessed lovers

ORIGINAL TEXT PARAPHRASECERESTell me heavenly bowIf Venus or her son as thou dost knowDo now attend the queen Since they did plotThe means that dusky Dis my daughter gotHer and her blind boys scandaled companyI have forsworn

CERESTell me heavenly bow if Venus the Goddess of love or Cupid her son and pedlar of passion at this time attend the heavenly queen Juno because you are sure to know Since the day they conspired against me and dark Pluto took away my daughter here and Cupidrsquos disgraceful company I have left off

IRISOf her societyBe not afraid I met her deityCutting the clouds towards Pathos and her sonDove-drawn with her Here thought they to have doneSome wanton charm upon this man and miad

IRISBe not afraid of her company I met her deity moving on the clouds towards Paphos the sacred home of Venus on the island of Cyprus along with her son on her chariot drawn by doves Here they contemplated to exercise a charm upon this man and maid producing

Whose vows are that no bed-right shall be paidTill Hymens torch be lightedmdashbut in vainMarss hot minion is returned againHer waspish-headed son has broke his arrowsSwears he will shoot no more but play with sparrowsAnd be a boy right out

wantonness before the actual marriage ceremony but did not succeed Venus has returned her irritable son has broken his arrows and swears that he will give up his practice of trying to inspire love but play with sparrows and be a boy again

[Music is heard] [Music is heard]

CERESHighst queen of stateGreat Juno comes I know her by her gait

CERESHighest queen of state Great Juno there she comes I know here by her gait

[Enter Juno] [Enter Juno]

JUNOHow does my bounteous sister Go with meTo bless this twain that they may Prosperous beAnd honoured in their issue

JUNOHow are you doing my generous sister Come with me to bless this couple so that they may be prosperous and fortunate in their children

[They sing] [They sing]

JUNOHonour riches marriage-blessingLong continuance and increasingHourly joys be still upon youJuno sings her blessings upon you

JUNOMay honour riches happiness in marriage long continuance and increase of those boons ever rest upon you as hourly joys Juno showers down upon you her blessings in song

CERESEarths increase foison plentyBarns and garners never emptyVines and clustring bunches growingPlants and goodly burden bowingSpring come to you at the farthestIn the very end of harvestScarcity and want shall shun youCeresrsquo blessing so is on you

CERESMay you have the plenty of earthrsquos produce Your barns and granaries may never be empty Your vines may grow with clustering bunches Your fruit trees may be heavily laden with their fruit May there be continuous spring and harvest May scantiness and want leave you forever Such is the blessing of Ceres upon you

FERDINANDThis is a most majestic vision andHarmoniously charmingly May I be boldTo think these spirits

FERDINANDThis is a great vision and magically melodious Should I suppose the characters (taking part in the masque) are spirits

PROSPEROSpirits which by mine artI have from their confines calld to enactMy present fancies

PROSPEROYes they are spirits whom I have summoned from the regions to which they are confined to carry into effect my fanciful designs

ORIGINAL TEXT PARAPHRASEFERDINANDLet me live here everSo rare a wondered father and a wifeMakes this place paradise

FERDINANDI should like to live here forever Such a wise and wonderful father makes this place a paradise

[Juno and Ceres whisper and send Iris on employment] [Juno and Ceres whisper and send Iris on employment]

PROSPEROSweet now silence

PROSPEROMy dear Ferdinand speak no more Juno and Ceres are

Juno and Ceres whisper seriouslyTheres something else to do Hush and be muteOr else our spell is marred

whispering with a solemn look There is something else coming Silence Or else our magic will be spoilt

IRISYour nymphs called naiads of the wandering brooksWith your sedged crowns and over-harmless looksLeave your crisp channels and on this green landAnswer your summons Juno does commandCome temperate nymphs and help to celebrateA contract of true love Be not too late

IRISYou nymphs called Naiads denizens (M inhabitants) of the running stream with your chaplets of sedge and ever-helpful looks leave your wrinkled channels and on the green land answer the summons sent to you Juno has ordered some chaste nymphs and help to celebrate a noble and true marriage Donrsquot delay

[Enter certain nymphs] [Enter certain nymphs]You sunburnt sicklemen of August wearyCome hither from the furrow and be merryMake holiday your rye-straw hats put onAnd these fresh nymphs encounter every oneIn country footing

You sunburnt harvesters weary from the effects of the heat in August come here from the furrowed land and rejoice Make holiday with your rye-straw hats upon you and meet these fresh nymphs and join in country dancing

[Enter certain reapers properly habited They join with the nymphs in a graceful dance towards the end whereof Prospero starts suddenly and speaks]

[Enter certain reapers properly habited They join with the nymphs in a graceful dance towards the end whereof Prospero starts suddenly and speaks]

PROSPERO[aside] I had forgot that foul conspiracyOf the beast Caliban and his confederatesAgainst my life The minute of their plotIs almost come [to the spirits]Well done Avoidno more

PROSPERO(Aside)I had forgotten the wicked conspiracy of the beast Caliban and his accomplices against my life the time of their plot has almost arrived ndash (To the Spirits) well done depart no more of this

[To a strange hollow and confused noise the spirits heavily vanish]

[The spirits depart]

ORIGINAL TEXT PARAPHRASEFERDINANDLet me live here everSo rare a wondered father and a wifeMakes this place paradise

FERDINANDI should like to live here forever Such a wise and wonderful father makes this place a paradise

[Juno and Ceres whisper and send Iris on employment] [Juno and Ceres whisper and send Iris on employment]

PROSPEROSweet now silenceJuno and Ceres whisper seriouslyTheres something else to do Hush and be muteOr else our spell is marred

PROSPEROMy dear Ferdinand speak no more Juno and Ceres are whispering with a solemn look There is something else coming Silence Or else our magic will be spoilt

IRISYour nymphs called naiads of the wandering brooksWith your sedged crowns and over-harmless looksLeave your crisp channels and on this green landAnswer your summons Juno does commandCome temperate nymphs and help to celebrateA contract of true love Be not too late

IRISYou nymphs called Naiads denizens (M inhabitants) of the running stream with your chaplets of sedge and ever-helpful looks leave your wrinkled channels and on the green land answer the summons sent to you Juno has ordered some chaste nymphs and help to celebrate a noble and true marriage Donrsquot delay

[Enter certain nymphs] [Enter certain nymphs]You sunburnt sicklemen of August wearyCome hither from the furrow and be merryMake holiday your rye-straw hats put onAnd these fresh nymphs encounter every oneIn country footing

You sunburnt harvesters weary from the effects of the heat in August come here from the furrowed land and rejoice Make holiday with your rye-straw hats upon you and meet these fresh nymphs and join in country dancing

[Enter certain reapers properly habited They join with the nymphs in a graceful dance towards the end whereof Prospero starts suddenly and speaks]

[Enter certain reapers properly habited They join with the nymphs in a graceful dance towards the end whereof Prospero starts suddenly and speaks]

PROSPERO[aside] I had forgot that foul conspiracyOf the beast Caliban and his confederatesAgainst my life The minute of their plotIs almost come [to the spirits]Well done Avoidno more

PROSPERO(Aside)I had forgotten the wicked conspiracy of the beast Caliban and his accomplices against my life the time of their plot has almost arrived ndash (To the Spirits) well done depart no more of this

[To a strange hollow and confused noise the spirits heavily vanish]

[The spirits depart]

Ac-12 27420 topic Revaluation of Assets and Liabilities

REVALUATION OF ASSETS AND LIABILITIES

On admission of a new partner the firm stands reconstituted and consequently the assets are revalued and liabilities are reassessed It is necessary to show the true position of the firm at the time of admission of a new partner If the values of the assets are raised gain will increase the capital of the existing partners Similarly any decrease in the value of assets ie loss will decrease the capital of the existing partners For this purpose alsquoRevaluation Accountrsquo is prepared This account is credited with all increases in the value of assets and decrease in the value of liabilities It is debited with decrease on account of value of assets and increase in the value of liabilities The balance of this account shows a gain or loss on revaluation which is transferred to the existing partnerrsquos capital account in existing profit sharing ratioAccounting for Revaluation of Assets and Liabilities when there is a Changein the Profit Sharing Ratio of Existing PartnersAssets and liabilities of a firm must also be revalued at the time of change in profit sharing ratio of existing partners The reason is that the realisable or actual value of assets and liabilities may be different from those shown in the Balance Sheet It is possible that with the passage of time some of the assets might have appreciated in value while the value of certain other assets might have decreased and no record has been made of such changes in the books of accounts Similarly there may be some unrecorded assets amp libilities that may have to be accounted for Revaluation of assets and reassessments of liabilities becomes necessary because the change in the

value of assets and liabilities belongs to the period to change in profit sharing ratio and hence must be shared by the partners in their old profit sharing ratio Revaluation of assets and reassessment of liabilities may be given effect to in two different ways (a) When revised values are to be recorded in the books and(b) When revised values are not to be recorded in the books

When revised values are to be recorded in the booksIn such a case revaluation of assets and reassessment of liabilities is done with the help of a new account called lsquoRevaluation Accountrsquo Sometimes this account is also called as lsquoProfit amp Loss Adjustment Acrsquo If there is a loss due to revaluation revaluation account is debited and if the revaluation results in a profit the revaluation account is credited The following journal entries made for this purpose are

(i) For increase in the value of assetsAsset Ac Dr (individually)To Revaluation Ac(ii) For decrease in the value of AssetRevaluation Ac Dr (individually)To Asset Ac[Decrease in the value of assets](iii) For increase in the value of LiabilitiesRevaluation Ac Dr (individually)To Liabilities Ac[Increase in the value of Liabilities](iv) For decrease in the value of LiabilitiesLiabilities Ac DrTo Revaluation Ac[Decrease in the value of Liabilities](v) For unrecorded AssetsAsset Ac [unrecorded] DrTo Revaluation Ac[Unrecorded asset recorded at actual value](vi) For unrecorded Liability Revaluation Ac DrTo Liability Ac [unrecorded][Unrecorded Liability recorded at actual value](vii) For transfer of gain on revaluationRevaluation Ac DrTo Existing Partnerrsquos CapitalCurrent Ac[Profit on revaluation transferred to capital account in existing ratio](viii) For transfer of loss on revaluationExisting Partnerrsquos CapitalCurrent Ac DrTo Revaluation Ac[Loss on revaluation transferred to capital account in existing ratio](a) When revaluation account shows gain Revaluation Ac DrTo Partnerrsquos Capital Ac (Old Profit Sharing Ratio)(Profit on revaluation credited to Partnerrsquos Capital Ac)(b) Above entry is reversed when revaluation account shows loss Partners Capital Acs (Old Profit Sharing Ratio) DrTo Revaluation Ac(Loss on revaluation debited to Partnerrsquos Capital Acs)

Proforma of Revaluation Account is given as under

Revaluation Account

Dr Cr Particulars ` Amount Particulars ` Amount To Decrease in value of assets By Increase in value of assets To Increase in value of liabilities By Decrease in value of liabilities To Unrecorded liabilities By Unrecorded assets To Gain on Revaluation (Transferred) By Loss on Revalution (Transferred)

ECO ndash12 2742020Topic- ELASTICITY OF DEMAND

CHAPTER - ELASTICITY OF DEMANDMEANINGDemand for a commodity is affected by many factors such as its price price of related goods income of its buyer tastes and preferences etc Elasticity means degree of response Elasticity of demand means degree of responsiveness of demand Demand for a commodity responds to change in price price of related goods income etc So we have three dimensions of elasticity of demandDIMENSION OF ELASTICITY OF DEMAND TYPES OF ELASTICITY OF DEMAND

Price elasticity of demand Income elasticity of demand Cross Elasticity of demand

Price elasticity of demand Price elasticity of demand means degree of responsiveness of demand for a commodity to the change in its price For example if demand for a commodity rises by 10 due to 5 fall in its price Price elasticity of demand (ep)=Percentage change in quantity demanded Percentage change in price of the commodity = 10 ( -)5 = ( - )2Note that ep will always be negative due to inverse relationship of price and quantity demanded

(ii) Income elasticity of demand Income elasticity of demand refers to the degree of responsiveness of demand for a commodity to the change in income of its buyer Suppose income of buyer rises by 10 and his demand for a commodity rises by 20 then Income elasticity of demand (ey)= change in quantity demanded change in price of the commodity =20 10 = 2

Cross Elasticity of demandCross elasticity of demand means the degree of responsiveness of demand for a commodity to the change in price of its related goods (substitute goods or complementary goods) Suppose demand for a commodity rises by 10 due to 5 rise in price of its substitute good then Cross elasticity of demand (ec) = change in quantity demanded change in price of related good = 10 2 = 5 (Tastes and preferences cannot be expressed numerically So elasticity ofdemand cannot be numerically expressed)

  • Chapter 1 Force (Summary)
  • Distinguish between external sovereignty and internal sovereignty
    • NAND Gate
      • Logic diagram
      • Truth Table
        • NOR Gate
          • Logic diagram
          • Truth Table
            • XOR Gate
              • Logic diagram
              • Truth Table
                • XNOR Gate
                  • Logic diagram
                  • Truth Table
                      • Physics
                      • Chapter 1 Electric Field ( Electric Dipole) (Summary)
Page 11:  · Web viewWe all know that Nouns are divided into two parts: common noun and proper noun.Apart from common and proper noun, we will also study about collective noun and compound

TOPIC- TOPOGRAPHICAL MAPS

Topographical Maps- topographical maps are small-scale maps with detailed depiction of both natural and human-made features

Conventional colours-different colours used in a map are known as conventional colours

TYPES OF SCALE-

Verbal Statement- Scale written in a statement from like 2 cm to a Km is called verbal statement It means that 2cm on the map represents 1 km on the Earth

Representative Fraction- Scale can also be written as a representative fraction (RF)

Example 150000 it means 1cm on the map represents 50000cm or frac12 km on the ground It shows the ratio between map distance and ground distance

Linear Scale Scale can be drawn on a line to show map distance equivalent to ground distance A line is divided into equal parts Each parts represents the actual distance on the ground in mkm

SUBJECT-COMPUTER

CHAPTER-1 (COMPUTER FUNDAMENTALS)

COMPUTER LANGUAGES

THE THE TERM COMPUTER LANGUAGE REFERS TO A SYSTEM OF RULES AND SYMBOLS THAT ARE DESIGNED TO GIVE INSTRUCTIONS TO A COMPUTERTHE COMPONENTS OF A COMPUTER SYSTEM CANNOT PERFORM A TASK BY THEMSELVES THEREFORE THEY HAVE TO BE GIVEN INSTRUCTIONS TO PERFORM ANY TASKCOMPUTER LANGUAGES ARE USED TO CREATE PROGRAMS USING LOGIC BASED ALGORITHMSTHE RULES OF A COMPUTER LANGUAGE IS

KNOWN AS SYNTAX WHEREAS THE TERM SEMANTICS REFERS TO THE MEANING OF LANGUAGESCOMPUTER CANNOT DEVELOP ANY PROGRAMMING LANGUAGE BY THEMSELVES AND CANNOT THINK INTELLIGENTLY UNLESS THEY ARE ASSISTED BY THE HUMAN BEINGS THEREFORE THE PROGRAMMERS DEVELOP A SET OF METHODS AND TECHNIQUES A SET OF INSTRUCTIONS USED TO PERFORM A SPECIFIC TASK IS CALLED A PROGRAM

A PROGRAMMING LANGUAGE IS ALSO KNOWN AS A COMPUTER LANGUAGE CODED BY PROGRAMMERS TO WRITE INSTRUCTIONS FOR A COMPUTERTHE COMPUTER TAKES THESE INSTRUCTIONS AS INPUT AND PRODUCES THE DESIRED OUTPUT

TYPES OF COMPUTER LANGUAGE

THE COMPUTER LANGUAGE CAN BE BROADLY DIVIDED INTO TWO CATEGORIES

LOW LEVEL LANGUAGES HIGH LEVEL LANGUAGES

LOW LEVEL LANGUAGES

A LOW LEVEL PROGRAMMING LANGUAGE REFERS TO THE LANGUAGE THAT IS UNDERSTOOD BY A COMPUTER DIRECTLYTHE PROGRAMMER MUST HAVE AN IN-DEPTH KNOWLEDGE OF DIFFERENT COMPUTERS TO WRITE PROGRAMS IN A LOW LEVEL LANGUAGE THE TWO TYPES OF LOW LEVEL LANGUAGES ARE MACHINE LANGUAGE AND ASSEMBLY LANGUAGE

MACHINE LANGUAGE

COMPUTER HARDWARE UNDERSTANDS ONLY MACHINE LANGUAGE AS IT IS THE FUNDAMENTAL LANGUAGE OF A COMPUTER WHICH UNDERSTANDS ONLY THE TWO DIGITS 0 AND 1 KNOWN AS THE BINARY DIGITSIT IS A SYSTEM OF INSTRUCTIONS EXECUTED DIRECTLY BY THE CPU WITHOUT ANY TRANSLATION AND HENCE IS EXECUTED QUICKLY AS COMPARED TO PROGRAMS IN OTHER LANGUAGES PROGRAMS IN MACHINE LANGUAGE NEED DIFFERENT BINARY LANGUAGE PROGRAMS TO SOLVE THE COMPLEX TASKSPROGRAMMERS OFTEN FIND IT DIFFICULT TO WRITE PROGRAMS IN THIS LANGUAGE

ASSEMBLY LANGUAGE

AN ASSEMBLY LANGUAGE IS A LOW LEVEL PROGRAMMING LANGUAGE BUT THE INSTRUCTIONS ARE WRITTEN IN THE FORM OF WORDS KNOWN AS MNEMONICS TO CONVERT AN ASSEMBLY LANGUAGE INTO TO MACHINE CODE WE REQUIRE A UTILITY PROGRAM WHICH IS KNOWN AS AN ASSEMBLER ASSEMBLY LANGUAGE PROGRAMS CANNOT BE EXECUTED DIRECTLY BY A COMPUTER THEREFORE THEY ARE SLOWER THAN THE MACHINE LANGUAGE PROGRAMSIT IS EASIER TO WRITE PROGRAMS IN THE MACHINE LANGUAGE THEN AND IN ASSEMBLY LANGUAGE

_______________________________________________________________________________________________

CLASS-VIII

SUBJECT-COMPUTER CHAPTER - Operating system and graphical user interface Role and functions

Need of operating system

It co-ordinates different hardware and software componentsof a computer system It supervises the various actions of the computer system and enables the computer to work in a effective

manner It helps in smooth functioning of various peripherals

In a multi-tasking operating system it determines the order and time to be allowed for each application before giving another application a turn

It sends messages to the system operator about the status of operation or any error that may have occurred while running the application

What is operating system

An operating system is an integrated system of programs that manages various resources and the overall operation of the computer system It is designed to support various activities of computer system in a systematic way

Role of an operating system-

Operating system enables the user to use the system effectively An operating system manages various application that runs on a computer and shares computerrsquos resources User interacts with operating system through command line interface and graphical user interface

Function of an operating system -

Booting the computer it is the start up procedure of a computer system Loading theprograms in the memory - when system is ready the operating system loads certain program

automatically Manages resources it manages between the hardware and software resources Detecting and correcting errors- if the supporting hardware or software doesnrsquot works properly then the

operatingsystem tries to rectify it Ensuring data security programs and data donot interfere with each other Maintaining the internal clock of system - maintain internal clock of system when system is shut down

SUBJECT-GEOGRAPHY CHAPTER 1 REPRESENTATION OF GEOGRAPHICAL FEATURES THROUGH CONTOURSTopographical maps- topographical maps are small-scale maps with detailed depiction of both natural and human-made features

Contour lines- contour lines are imaginary lines drawn on a map joining places having the same height above the mean sea level

Contour linesContour interval- It is an interval at which contour lines are drawnIndex contour- At every 100-m interval a thick brown contour line is called index contour It is drawn for calculating height

Spot heights- The spot heights show heights in metres above the mean sea level

SUBJECT- English Language CHAPTER - The Sentence

Complex sentence-

We have seen that a Complex Sentence consists of a Principal Clause with one or more Subordinate Clauses

We have also learnt that there are three kinds of Subordinate Clauses The Adjective Clause the Adverb Clause and the Noun Clause

1) THE ADJECTIVE CLAUSE

An ADJECTIVE CLAUSE does the work of an ADJECTIVE It qualifies a noun or pronoun In the following examples in each set two commonly patterned sentences are compared one with an ADJECTIVE and the other with an ADJECTIVE CLAUSE ( The Adjective or Adjective Clause qualifies the Noun placed in the box)

a) He is a lazy boy (Adjective)

b) He is a boy who is lazy (Adjective Clause)

a) It is a beautiful place for the tourists (Adjective)

b) It is a place where tourists come for the scenic beauty (Adjective Clause)

a) We have enough funds for the work (Adjective)

b) We have funds which would be enough for the work (Adjective Clause)

Convert the sentence from simple to complex sentence-

1) We believe his honesty Ans- We believe that he is honest

2) This is the birth place of RamaAns- This is the place where Rama was born

____________________________________________________________________________________________

Class IX

Subject English Language

Topic Preposition

Date 270420

PREPOSITIONS

A preposition is a word placed before a noun or a pronoun to show in what relation the person or thing denoted by it stands in regard to something else

EXAMPLE

There is a clock on the wall

The preposition on shoes relationship between clock and the wall

Smitha is afraid of lizards The man jumped off the bus

Here the preposition of shows the relationship between afraid and lizards

The preposition off shows the relation between jumped and bus

The noun or the pronoun which follows a preposition is called its object

So in the first Example wall is the object of the preposition on

A preposition can have more than one object For example

The plane flew overhouses and meadows

Prepositions are used to express a number of relationship including time locationmannermeans quantity purpose and state or condition

Points to remember

Preposition joins a noun to another noun or a pronounThere is a cow in the field

A preposition joints a noun to a verbThe cat runs after the rat

Preposition can have two or more than two objectsThe road runs over hills and plain

A preposition also joins a noun an adjectiveHe is fond of tea

Generally a preposition comes before an object Sometimes it comes even after an object asWhat are you looking atThis is the house I live in

Subject- Computer Application

Chapter 2 Introduction to Java

Java API An application programming interface (API) in the context of Java is a collection of prewritten packages classes and interfaces with their respective methods fields and constructors

Byte Code Java bytecode is the result of the compilation of a Java program an intermediate representation of that program which is machine independent The Java bytecode gets processed by the Java virtual machine (JVM) instead of the processor JVM The Java Virtual Machine (JVM) is the runtime engine of the Java Platform which allows any program written in Java or other language compiled into Java bytecode to run on any computer that has a native JVM

Platform A platform is the hardware or software environment in which a program remains

Java platform The Java platform differs from most other platforms in that itrsquos a software only platform that runs on top of other hardware-based platforms

Applet and Application The fundamental difference between the two Java programs is that an application program is designed to run on a stand-alone machine whereas an applet is a web-version of an application which is used to run a program on a web browser

WORA ldquoWrite once run anywhererdquo (WORA) or sometimes write once run everywhere (WORE) is a slogan created by Sun Microsystems to illustrate the cross-platform benefits of the Java language

Class XSubject Topic Summary Execution

ECONOMICSFACTORS OF PRODUCTION Sub-topiclsquoLABOURrsquo

We shall start our class by discussing the topic taught in the last class

lsquo DIVISION OF LABOURrsquo- By division of labour we mean specialization in workIt refers to splitting up the work of labour involved in the production of a particular commodity into several parts and each part and sub-part is performed by a specialist

Now let us start by the Advantages of Division of labouraIt increases the level

Questions

1 Differentiate between Product-Based division of labour and Process-Based division of labour

Product-Based division of labour

Process-based division of labour

It is also known as simple (or occupational) division of labourUnder it everybody performs a particular occupations The entire is done by the same person

When a person or group of persons undertakes a specialised function which is supplementary to the production of final commodity and service This is also as complex division of labour

It is simple It is complex

of productionbSince the product is produced by an expert workerbest quality of product is producedcIt saves time and toolsd it promotes inventions in the methods and techniques of productioneIt leads to reduction in costs fAll workers get work according to their abilities and choices

Now let us discuss the disadvantages of Division of labour

a Since many workers are involved in the production of a commodity no one has the sense of responsibility

b The constant and repetition of the same work again and again make the work monotonous

c Division of labour facilitates production on large scale Hencethere is fear of over production

d Because of territorial division of labour some areasregions become more developed than others

It is based on labour-intensive techniques of production

It is based on capital-intensive techniques

It is generally found in small enterprises

It is generally found in large enterprises

Example Indian farmers doing all farm activities

Example A modern garmet factory where one person takes the measurementanother does the cuttingsome sew the clothes while a few workers button them and other iron them

2 What are the advantages of Division of labouraIt increases the level of productionbSince the product is produced by an expert workerbest quality of product is producedcIt saves time and toolsd it promotes inventions in the methods and techniques of productioneIt leads to reduction in costs fAll workers get work according to their abilities and choices

3Discuss the disadvantages of Division of laboura Since many workers are involved in the

production of a commodity no one has the sense of responsibility

b The constant and repetition of the same work again and again make the work monotonous

c Division of labour facilitates production on large scale Hence there is fear of over production

d Because of territorial division of labour some areasregions become more developed than others

English 1 Transformation of sentences

Sentences A sentence is a group of words which makes complete sense

a Assertive sentences

Exercise 6Rewrite the following sentences according to the instructions given below without changing their meanings

1 As soon as he saw the beer he jumped into

b Imperative sentences

c Interrogative sentences

d Exclamatory sentences

Sentences can be changed from one grammatical form to another without changing the meaning of the sentence This is known as transformation of sentences

the river ( Begin No sooner)2 None but brave deserve the fair (Begin the

bravehellip)3 This box is too heavy for me to lift ( Use so hellip

That instead of too)4 No one other than a king can live like James

Luxurious ( Begin only James)5 Oh for the wings of a dove (Begin I wishhellip)

Math Topic Commercial MathematicsChapter Shares and Dividends

Study item Discuss about shares and Dividends1) What is share

Ans To start any big business (company or Industry) a large sum of money is needed But it is not possible for an individual to invest such a large amount Then some persons interested in the business join together and from a company They divide the estimated money required into small parts Each such part is called a share

2) What do you mean by the term shareholder

Ans A person who purchases one or more shares is called shareholder3) Some terms related with a share

(i) Nominal value or face value or printed value The original value of a share is called its nominal value or face value or printed value

Note The nominal value of a share always remains same(ii) Market value or cash value The price of a share at any

time is called its market value or cash value

Note The market value of a share changes from time to time(iii) At par If the market value of a share is the same as its

nominal value the share is called at par(iv) At Premium or above Par If the market value of a share

is more than its nominal value the share is called at premium or above par

Example If a share of Rs 100 is selling at Rs 150 then it is said to be selling at a premium of Rs 50 or Rs 50 above par

(v) At Discount or below par If the market value of a share is less than its nominal value the share is called at discount or below par

Example If a share of Rs 100 is selling at Rs80 then it is said to be selling at a discount of Rs 20 or at Rs 20 below par

4) What is Dividend

Ans The profit which a shareholder gets for hisher investment from the company is called dividendNote (i) The dividend is always expressed as the percentage of the face value of the share(ii) The dividend is always given( by the company ) on the face value of the share

irrespective of the market value of the shareBENGALI(2ND LANGUAGE)

ldquoদেবতোর জণমrdquoলিবরোম চকরবত

পরথম লিসর পোঠ-চোর পসথ একটি পোথর লিবপলি ঘটোয় দেক যোতোয়োসতর পসথ পরলিতলিয়ত ওই পোথসর দেো োচট দেসত একলি দেতো দেক দেো োচট দেসয় দেবোমো সয় রোসতোর মোস লি0টসক পস1 লিবপরীত লিক দেথসক আো একটি দেমোটর োলি1 চোসকর কষতোয় পরোসরণ দেবোসচ যো লিকনত পরলিতবোর এমরণ দেৌভোয দেসকর োও সত পোসর তোই লিতলি দেকোো দেজোো1 কসর পোথরটিসক উপস1 দে8স পোথর উপস1 দে8োর ময় এক দেকৌতী জতো দেকসক পরশন কসর দেয লিতলি দেকোসো দেবতোর আস দেপসয়স0রণ লিকো লিকনত দেক বস লিতলি দেকোসো দেবতোর আস পোলি দেক উপলিত ক জতোর উসltসয বস কোরও ইস= স পোথরটি লিসয় দেযসত পোসর এর পর দেথসক দেক দেযসত আসত পোথরটি দেক দেসত পো একলি দেক কষয করস দেকউ পোথরটিসক ধসয়মস0 পলিরসকোর করস0 দেক ওই দেকৌতী জতোসক পোথসরর কোস0 বস থোকসত দেস এ0ো1ো আরও কষয কসর দেক ঠোৎ ওই লি1 পোথরটির োসয় লিোর োো-দেকউ পজো কসরস0 দেকৌতী দেোকটির আঙকো য় যলিসক উ পোথরটিসক লিরসয় দে8স তোর পর ঠোৎ একলি পোথরটির দেোোজ দেই দেক লিসয় দেস0 বো দেকোথোয় দেস0

বদোথ-

দেো োচট ndashচসত লিসয় দেকো লিক0র সE ধোককো দেস পস1 যোবোর উপকরমঅকসমোৎ- ঠোৎআতমমবর- লিসজসক লিয়নতরপ-পো লিপ0স প1োদেসতসসত- লিসপলিউৎোত- দেো1ো দেথসক উপস1 দে8োপরতযয় ndash লিবশবোপরতযোস- টতযোস ndash দৈবোসধসতোধলিসত- পরসপসরর পরলিত ব পরসয়ো করোপরসতরীভত- পোথসর পলিররণলিতবোনতঃকরসরণ- মস পরোসঅলিQৎ- অQো করসত ই=কদেোপ- বধমোর- দেবোস এমইতযোকোর- এইরকমরম- বময়পযসোভী-পসযর জয দেোভ আস0 যোরপোসথয় ঞচয়- পথচোর রচ জমোসোমোমোসরোস- ব ধম ধোসমর সE

তোর লি দেইhelliphellipTo be continued

Hindi 2nd lang सर क पद(सरदास)

सरदास शरी कषण भकति कावय क सरवशरषठ कगिरव ह इनक जनम और मतय क समय तथा सथान का मतभद हसरदास रवातसलय और शरार रस क अनयतम कगिरव ह इनक कावय म बालकषण क सौदय चपपल चषटा और गि7याओ की मनोहर झाकी मिमलती ह कषण और ोगिपयो क अननय परम का कतिचतरण ह सयो शरार की अपकषा उनक कावय म गिरवयो शरार का अमिBक गिरवषय और मारमिमEक कतिचतरण हआ हइन पकतियो म हम सरदास की भकति भारवनाओ का परिरचय मिमलता ह इनका सपण सगरह सरसार म गिनगिहत ह

1 जसोदा हरिर पालन झलारवहलरारवदलराईमलहारव रव जो ईसाई कछ ारव मर लाल को आई निनEदिदया काह ना आगिनसबारव त काह नाही बरवगिह आरवतोको कानहा बलारव

शबदाथ-हलरारव-गिहलती हदलराई - दलार पयार करती हमलहारव-पचकारती हनिनEदरिरया ndashनीदरवगिह-जलदी सअBर-होठमौन-चपसन-सकत

वयाखया- सरदास जी कहत ह गिक यशोदा माता बालक कषण को पालन म झल आती ह रवह उनह गिहलाती ह पयार करती ह मलहार जस कोई ीत ान लती ह और नीद स पछती ह गिक ह नीद तम मर लाल को आकर कयो नही सलाती तझ खाना बला रहा ह कभी कषण आख बद कर लत ह कभी आखफडफडान लत ह उनह सोता हआ जानकर यशोदा माता चप हो जाती ह और इशार म बात करन लती ह इसी बीच अकला कर कषण ज जात हतो गिफर यशोदा माता गिफर स ाना ान लती ह सरदास जी कहत ह गिक भरवान क दशन का सख दरवता और ऋगिष-मगिनयो को भी दलभ ह यही सख माता यशोदा को बडी सहजता स मिमल जा रही ह माता यशोदा बहत ही भागयशाली ह2)Continue to nexthellip

Physics

Chapter 1 Force

(Summary)

Question A body is acted upon by two forces each of magnitude F but in opposite directions State the effect of the forces if

(a) Both forces act at the same point of the body

(b)the two forces act at two different points of the body at a separation r

Solutions

(a) Resultant force acting on the body = 0

F ndash F = 0(b) The forces tend to rotate the body between two forces about the midpoint

Moment of forces = F times rFr

QuestionDefine moment of a couple Write its SI unit

Solutions

Moment of couple is equal to the product of both force and the perpendicular distance between the two forces

The SI unit of moment of couple is NmCommercial Studies

Advertising and sales

Business firms use several methods to

Questions1) What do you mean by advertising

promotion create demand of their product in the market and increase it sales Such methods comprises of advertising sales promotion personal selling and publicityToday we are going to discuss about one of such methods It is advertising

Meaning of advertisingAdvertising is a paid form of non-personal presentation for promotion of Ideas goods and services

Importance or merits of advertising Advertising has importance to manufacturer or traders to customer and to society as a whole

Today we will see how advertisement help the manufacturer or traders

Answer) Advertising is a means of how a company encourages people to buy their products services or ideas It is one element of marketing which also includes design Research and data mining

2) Mention any three features of advertisingAnswer)The main features of advertising are

i) It is impersonal form of presentation for promotion of products and services of Ideas

ii) It is issued by identified sponsor The advertisement contains the name of the advertiser

iii) It is a form of mass communication because the message is directed to a large number of persons simultaneously

3) Mention the main merits or importance of advertisement to manufacturer or tradersAnswer)

i) Introducing new product A business organization can introduce itself and its products to the public through advertising

ii) Increase the sale Advertising leads to increase the sale of existing product by entering into new markets and attracting new customers

iii) Create steady demand Advertising creates sustains regular demand by smoothening out seasonal and other fluctuations It enables regular production for the organisation

iv) Economics of scale Advertising facilitate mass distribution of goods and steady demand which lead to large scale and regular production

v) Goodwill Advertising helps in creating a good image of the firm and reputation for its products

Biology Chapter - 03Genetics

Today wewill start chapter and discuss about Genetics Gregor Mendel is known as father of genetics Before entering into Mendelrsquos experiment on Genetics we must know

Q1 Define the following termsi) Genetics Genetics is the study of

transmissionof body features from parents to offspringand the laws relating to such transmission

ii) Heredity It may be defined as transmissionof genetically based characteristics from parentsto offspring

iii) Character and traits Any heritable

Importance to TraderIntroducing new productIncrease the saleCreate steady demandEconomics of scaleGoodwill

some terms featureis a character The alternative forms of acharacter are called traitsex Character (Hair shape) - Traits (Curly straight)

iv) Homologous chromosomes A pair ofcorresponding chromosomes of the same shapeand size one from each parent

v) Genes Genes are the specific parts (DNA segments) of a chromosome which determinethe hereditary characteristicsNearly 30000genes present in human

vi) Alleles Alternative forms of a gene occupying the same position (locus) on homologouschromosomes and affecting the same characteristicbut in different ways

vii) Genotype ndash PhenotypeGenotype means of genes present in the cells of an organism Phenotype means the observable characteristic which is genetically controlled

viii) Mutation It is a sudden change in one or more genes or in the number or in the structure of chromosomes ex Sickle cell anaemia is a blood disease caused by a gene mutation

CLASS NOTES

Class XSubject Eng Literature (The Merchant of Venice ndash William Shakespeare)Topic Act IV Scene 1 Lines 01 to 34 ( Duke helliphelliphellip We all expect a gentle answer Jew) ate 27th April 2020 (2nd Period)

[Students should read the original play and also the paraphrase given in the school prescribed textbook]Summary Questions amp Answers

This scene may be termed as the catastrophe of the play It is the final unravelling of the complicated events which seem to threaten the happiness of Bassanio Portia and Antonio Right is justified to the fullest degree and malice falls into the trap prepared for others No one suffers here but Shylock but even then he receives a generous measure of mercy

o This is the Court-scene Initially we meet

(1)

DUKE I am sorry for thee thou art come to answer (Line 3-6)A stony adversary an inhuman wretchUncapable of pity void and emptyFrom any dram of mercy

(i) Who is addressed here Where is the person Why is the person there

Antonio is addressed hereAntonio is in the court of justice at VeniceAntoniorsquos trial is scheduled to be held here for his failure to meet the conditions of the bond he signed with Shylock

the Duke Antonio Shylock and Salerio Later we meet Bassanio Portia Gratiano and Nerissa

o The Duke says to Antonio that he has to face a very cruel opponent which Antonio admits and expresses his gratefulness to the Duke for his efforts to soften without result the heart of Shylock in order to be merciful to Antonio Antonio further says that he is ready to accept whatever cruel judgement the Court may award

o When Shylock appears in the court the Duke says that Shylock should change his decision of prosecuting Antonio and demanding the penalty specified in the bond out of consideration of the great misfortunes that Antonio has suffered If this is done by him (Shylock) the whole court would be gladdened by his merciful action

(ii) What is the Duke sorry for

The Duke is unable to change the mind of Shylock from his decision to get the bond forfeited even after he pleaded to Shylock Shylock stands firmly for his bond which when forfeited will allow him to take a pound of flesh from any part of Antoniorsquos body(iii) How does the Duke address Shylock`The Duke calls Shylock an adversary with a heart of stones He calls Shylock as an inhuman wretch without pity Shylock is quite lacking in the slightest quality of mercy (iv) How does Antonio reply to this

Antonio replies that he will meet the revenge of Shylock patiently He has prepared himself to suffer with a quiet spirit the utmost that Shylockrsquos tyranny and rage can do(v) What quality of the Duke is revealed here

The Duke is kind and benevolent He is ready to help Antonio He requests Shylock to free Antonio from the trial(vi) What are the terms of the bond that Antonio has signed

The terms of the bond that Antonio has signed were that if Antonio is unable to repay Shylock a certain sum of money specified on the paper on a certain date and in an agreed place the forfeit has to be paid The forfeiture will be an exact pound of Antoniorsquos flesh which Shylock will be a liberty to take from any part of Antoniorsquos body which pleases him

Class XI

STUDY MATERIAL

Class XISubject Eng Literature (The Tempest ndash William Shakespeare) Topic Act I Scene 2 Lines 88 to 132 (Prospero hellip Me and thy crying self) Date 27th April 2020 (3rd Period)

[Students should read the original play and also the paraphrase given in the school prescribed textbook]Summary Questions amp Answers

o Prospero now tells Miranda that he was the Duke of Milan He had been devoting himself more to studies than the affairs of the State His brother Antonio took advantage of this situation and with the help of Alonso the king of Naples seized upon him and her one midnight and shipped them in a frail bark so that they perished in the sea All this took place

(1)

MIRANDA I should sin (Line 118-132)

To think but nobly of my grandmother

Good wombs have borne bad sonsPROSPERO Now the condition

The King of Naples being an enemyTo me inveterate hearkens my brothers suitWhich was that he in lieu othrsquo premisesOf homage and I know not how much tribute

twelve years back

IMPORTANT PASSAGES EXPLAINED(Line 98-103)

PROSPERO helliphelliphelliphelliphelliphelliphellip

He being thus lorded

Not only with what my revenue yieldedBut what my power might else exact like oneWho having into truth by telling of it Made such a sinner of his memoryTo credit his own lie he did believeHe was indeed the duke

Prospero in telling the narrative of his past life here refers to his brother Antonio Prospero being with a studious bent of mind has left the administration of Milan on his younger brother Now Antonio being thus invested like a lord with all the powers derived from Prosperorsquos wealth and what the exercise of Prosperorsquos authority might secure for him regarded himself as a de facto Duke of Milan It is a well-known fact of psychology that a man who repeatedly tells a lie makes of his memory such a sinner against truth as to credit his own lie by the telling of it So Antonio by repeatedly saying to himself and others that he was the Duke came to believe that he was really the Duke Thus falsehood repeatedly asserted gained the force of truth for Antonio and he truly believed it

Should presently extirpate me and mine Out of the dukedom and confer fair MilanWith all the honours on my brother whereonA treacherous army levied one midnightFated to thrsquo purpose did Antonio openThe gates of Milan and ithrsquo dead of darkness The ministers for thrsquo purpose hurried thenceMe and thy crying self

(i) In the earlier lines of this scene what does Prospero tell about his intense interest What was the demand of his interest

In the earlier lines of this scene Prospero tells Miranda that he had an intense interest in the study of philosophy and magic arts Hence in order to improve his mind with this kind of study he kept himself isolated from worldly and state affairs His study was dearer to him than the applause and esteem that he could win from the public His study demanded too much solitude(ii) What forced Antonio to take an undue advantage over Prospero

Prosperorsquos indifferent attitude towards the statersquos affairs and his having boundless trust in Antonio gave rise to a boundless lust for power in Antoniorsquos mind Antonio felt that he must be the actual Duke instead of the part of the Duke he played Thus Antonio took an undue advantage of the situation to usurp Prosperorsquos dukedom(iii) Explain the following lines ldquoI should sin to think but nobly of my grandmother Good wombs have borne bad sonsrdquo

After hearing the treacherous act of her uncle Antonio Miranda says that Prosperorsquos mother was a noble lady and she cannot dishonour her memory by saying that the person named Antonio cannot be his (Prosperorsquos) brother She says that in honour of her grandmother she also cannot say that Antonio must have been begotten not by her grandfather but by some other man She finally concedes that it is known that good mothers have borne bad sons in their wombs and gave birth to them

(iv) Why did the King of Naples accept Antoniorsquos request to help him in usurping his dukedom What did Antonio propose to Alonso

Alonso who was the king of Naples accepted Antoniorsquos request in usurping Prosperorsquos dukedom because he (Alonso) was a sworn enemy of Prospero Antonio proposed that Alonso should immediately drive him (Prospero) and his offspring out of Milan and should confer the dukedom upon him (Antonio) with all the dignities which go with that

position In return Antonio promised that he shall give an annual tribute and also swore his allegiance to Alonso Also he agreed to hold the Dukedom of Milan as a subordinate to the state of Naples(v) How were Prospero and Miranda carried away from the city of Milan and what was the state of small Miranda at that time

In pursuance of the agreement settled between Antonio and Alonso an army of treacherous men was assembled One midnight when the occasion suited the will of destiny Antonio opened the gates of the city of Milan and in the death like silence of midnight Antoniorsquos agents who had been directed to execute his purpose carried Prospero and small Miranda away from the city in all haste They were then forced into a ship and carried some distance out to sea where they put them on a mere hulk of a boat without any rigging or ship-gear and abandoned them leaving them at the mercy of the roaring sea Miranda was a very small child of three years age and she was crying at that time

CLASS -XIDATE-270420Subject Topic Summary Execution

EVS Chapter 1 ndash Mode of Existence

Impact of mode of existence on resources

Q) Why resources are under pressure

Ans - Increase in the sophistication

of technology enabling natural resources to be extracted quickly and efficiently Eg in the past it could take long hours just to cut down one tree only using saws Due to increased technology rates of deforestation have greatly increased

The number of humans is increasing Cultures of consumerism Materialistic views

lead to the mining of gold and diamonds to produce jewelry unnecessary commodities for human life or advancement Consumerism also leads to extraction of resources for the

production of commodities necessary for human life but in amounts excessive of what is needed because people consume more than is necessary or waste what they have

Lack of awareness among the population is striking People are not aware of ways to reduce depletion and exploitation of materials

Accounts Cash Book Today we are going to start a new topic -Cash Book

The key terms used in this chapter are

bullCash book

bullSimple cash book

bullDouble column cash book bullTriple column cash book

bullPetty cash book

bullCash discount

bullContra entry

Here I will share you the meaning of each key terms

bullCash book Cash Book is a special purpose subsidiary book or journal in which cash received and cash payments are recorded

bullSimple cash book

It is a cash book in which only cash transactions are recorded It has only one column on each side

bullTriple column cash book

It is cash book which has three columns one column for each cash and Bankdiscount on each side of the cash book In this book both cash and Bank transactions are recorded together with discount allowed and received

bullPetty cash book

It is a cash book maintained for recording petty expenses

bullCash discount

Cash discount is the amount of discount received or allowed on cash payments and cash receipts Discount received is an income for the business while discount allowed isan expense

bullContra entry

It means transactions involving both cash and Bank Such transactions though recorded in the cash book are not posted into ledger The letter lsquoC is written in Ledger folio for contra entry

Business Studies

ENTREPRENEURSHIP

Now we shall discuss the second chapter

lsquoENTREPRENEURSHIPrsquo

Today before starting the chapter let us recall what

Questions

1What are the main characteristics of Intrapreneurship

Answer

The main characteristics of Intrapreneurship are

Corporate framework-it occurs within the framework of the same company

Semi-Autonomous-Intrapreneurship

we have read last day

Let s today start the class by recalling the last topic taught

Intrapreneurship is the process of discovering and exploring business opportunities within an existing company It involves launching new business ventures within the framework of a present corporation Intrapreneurship is also known as corporate entrepreneurship or corporate venturing

Now let us start with the characteristics of Intrapreneurship

The main characteristics of Intrapreneurship are

Corporate framework

Semi-Autonomous Lack of ownership Senior position Low risk taking Not own boss

Now let us discuss the meaning of enterprise

Enterprise means an undertaking or adventure that requires some innovation and investment and thus involves riskEnterprise always entails decision making coordination and risk bearing

involves crating amd nurturing a semi-autonomous business unit which may be a subsidiary a strategic business unit or a division

Lack of ownership-the intrapreneur is not the owner of the unitb he creates and nurtures

Senior position-he occupies a senior managerial position in the company

Low risk taking-An intrapreneur does not bear the full risk of failure

Not own boss-An intrapreneur is not his own bosss in legal termsHe enjoys the freedom and gets the required resources and support

2 How is Entrepreneur is different from Intrapreneur

The functions involved in both the entrepreneurship and intrapreneurship are by and large similar however there are several differences between the two

Point of distinction

Entrepreneur

Intrapreneur

status An independent business person

A senior executive within a company

Ownership Owner of

the business

An employeesometimes a share in ownership

Financing Responsible for raising finance for the business

Not responsible for raising the finance

Risk bearing

Bears the risk of the business

Does not bears the risk of the business

Reward Profit which is uncertain and irregularcan be loss

Fixed salary and fringe benefits

Need for security low high

3 What do you understand by enterprise

Answer Enterprise means an undertaking or adventure that requires some innovation and investment and thus involves riskEnterprise always entails decision making coordination and risk bearing

COMMERCE NATURE AND OBJECTIVES OF

BUSINESS

Today let us recall the last other two objectives of business by the chart given in the previous class

Firstly we would discuss Human Objectives

Business is run by people and for people Labour is a valuable business element

Human objectives of business are concerned with the well -being of labour

The human objectives are as follows

Labour welfare Developing human

resources Participative

management Labour

management cooperation

Questions

1 Explain the human objectives of a business enterprise

Answer

Business is run by people and for people Labour is a valuable business element

Human objectives of business are concerned with the well -being of labour

The human objectives are as follows

Labour welfare-Business must recognize the dignity of labour and human factors should be given the recognition

Developing human resources-Employees must be provided the opportunities for developing new skills and attitudes

Participative management-Employees should be allowed to take part in decision making process of business

Labour management cooperation-Business should strive for creating and maintaining cordial employer employee relations so as to ensure peace and progress in industry

Now let us discuss the national objectives of business

Optimum utilization of resources

National self- reliance Development of small

scale industries Development of

backward areas Control over pollution

2Explain the national objectives of a business enterprise

Answer

It is the duty of business to utilize the resources of the country properly the national objectives of business

Optimum utilization of resources ndashBusiness should use the nationrsquos resources in the best possible manner

National self- reliance-It is the duty of the business to help the government in increasing experts and in reducing dependence on imports

Development of small scale industries-Big business firms are expected to encourage growth of small scale industries which are necessary for generating employment

Development of backward areas-Business is expected to give preference to the industrialization of backward regions of the country

ECONOMICS

BASIC ECONOMIC CONCEPTS

SUB

TOPIC

Value

Wealth

Welfare

Today we shall start with a new topic of the same chapter ie lsquoValuersquo

Value of a commodity is defined as the valuation placed by a household on the consumption of this commodity

lsquoValuersquo has two different meanings and these are

a Value -in -use It refers to consumption value of a commodity It expresses the utility derived from the consumption of a particular commodity A necessity like water has a very high value ndashin ndashuse or

Question

1What is value

Answer

Value of a commodity is defined as the valuation placed by a household on the consumption of this commodity

2What is value-in use

Answer It refers to consumption value of a commodity It expresses the utility derived from the consumption of a particular commodity A necessity like water has a very high value ndashin ndashuse or consumption value

3What is value ndashin- exchange

Answer It relates to market value of a commodity

It is the rate at which a particular good or service can be exchanged for money

For example in barter system if a person is prepared to exchange 3 metres of cloth with 1 pair

consumption value

b Value ndashin-exchange It relates to market value of a commodity

It is the rate at which a particular good or service can be exchanged for moneyFor example in barter system if a person is prepared to exchange 3 metres of cloth with 1 pair of shoes then the value in exchange of 3 metres of cloth is 1 pair of shoesValue in exchange is the power of purchasing other goods In modern monetised economies the exchange value of goods are expressed in terms of money as prices

Now let us discuss the term lsquoWealthrsquo

Wealth refers to the stock of all those assets which are a source of income

Wealth is a stock concept

Wealth must possess the following features

a Utility It must possess utility or give some

of shoes then the value in exchange of 3 metres of cloth is 1 pair of shoes

Value in exchange is the power of purchasing other goods In modern monetised economies the exchange value of goods are expressed in terms of money as prices

4 What is wealth

Answer it refers to the stock of assets or goods which are a source of income and have personal or national ownership

5 What are the features of wealth

Answer The features of wealth are as follows

Wealth must possess the following features

a Utility It must possess utility or give some satisfaction

b Scarcity It must be limited in quantityc Transferability it should be transferable its

ownership can be transferred from one person to another person

d Exchange value It must possess exchange value

6 What is welfare

Answer

Welfare is defined as satisfaction and happiness a sense of well- being among the people

satisfactionb Scarcity It must be

limited in quantityc Transferability It

should be transferable its ownership can be transferred from one person to another person

d Exchange value It must possess exchange value

Now let us discuss the term lsquoWelfarersquo

Welfare is defined as satisfaction and happiness a sense of well- being among the people

Welfare is affected by factors like

a Consumption of goods and services

b Environment

c Family relations

d Degree of freedom

e Law and order situation

Mathematics Trigonometric equation

To find the general solution of the equation sinθ=0

When sin θ =0

Then θ= 0 π2π 3π-π -2π -3

i e when θ = 0 or an integral multiple of π

i e when θ= nπ where n is any integer

Therefore the general solution of the equation sin

Example1 Find the general values of θ which satisfy the equation sin2 θ =34

Solution sin2 θ= 34

Or sin θ = +34 or -34

Or sin θ = sin π3 or sin (-π3)

Therefore

θ = [nπ + (-1) n (π3)] or[ nπ+ (-1) n (-π3)]

= nπ +π3 or nπ-π3 where n= any integer

Example 2Find the values of θ which satisfy tan2 θ

θ=0 is θ= nπ where n is any integer

To find the general solution of the equation cos θ=0

When cos θ=0

Then θ=π2 3π2 5π2 -π2 -3π2 -5π3

i e when θ is an odd multiple of π2

i e when θ=(2n+1) π2 where n= any integer

Therefore the general solution of the equation cos θ =0 is θ= (2n+1) π2 where n= any integer

To find the general solution of the equation tan θ = 0

Clearly tan θ =0 implies sin θcos θ =0

Therefore θ = nπ

i e the general solution of the equation tan θ=0 is θ =nπ where n = any integer

To find the general solution of the equation cot θ =0

Clearly cot θ =0 implies (cos θsin θ) = 0

i e cos θ =0

Therefore θ = (2n+1) π2

Therefore the general solution of the equation cot θ =0 is θ = (2n+1) π2

Where n= any integer

To find the general solution of the equation sin θ= k (-

=13 -πleθleπ

Solution tan2 θ =13

Or tan θ = plusmn1radic(3) =tan(plusmnπ6)

θ=nπ plusmn π6 where n =any integer

If n=0 then θ=plusmnπ6

If n=1 then θ= π plusmn π6

If n=-1 then θ= -π plusmn π6

Therefore the required solution in -π le θ le π are θ= π6 5π6 -π6 -5π6

Exercise Find general solution of sin 2θ=cos θ [Hints Use sin 2θ= 2sin θcosθ and then take cosθ

common]

1lek le1)

Determine an angle alpha such that sin =k and -π2le αle π2

Then we have

Sin θ = k = sin α

Or sin θ - sin α =0

Or 2 cos [(θ+α) 2] sin [(θ-α) 2] =0

Therefore either cos [(θ +α) 2] =0 (1)

Or sin [(θ-α) 2] =0 (2)

Now from (1) we get (θ+α) 2= (2m+1) π2)

Or θ = (2m+1) π-α (3)

And from (2) we get (θ-α) 2 =mπ

Or θ= 2mπ+α(4)

Where m = any integer

Clearly the solution (3) amp (4) may be combined in the following form

θ= nπ+(-1) n α where n= any integer

Therefore the general solution of sin θ = sin α is θ = nπ +(-1) n α where n is any integer and -π2 le α le π2

Biology Chapter - 04Kingdom Monera

Today we will discuss about bacterial reproduction and its usefulness

Fig Binary Fission

Fig Conjugation Fig Transformation

Bacterial reproduction is mainly asexual but sexual reproduction

also takes place

Asexual reproduction takes place by i) Binary fission - from one bacteriato

two bacteria are produced in every 20 to 30mins

ii) Buddingiii) Endospore formation - during

unfourable condition

Sexual reproduction by three ways

1) Conjugation - Transfer of genetic material between cells that are in physical contact with one another

2) Transduction - Transfer of genetic materialfrom one cell to another by a bacteriophage

3) Transformation - Transfer of cell-freeor naked DNArsquo from one cell to another

Bacteria causes different diseases inplants animals and human and

it causes food spoilage and waterpollution but it also have some useful

activities

i) Bacteria are helpful in sewage water treatment

ii) It is used in antibiotic (medicine) production

iii) Anaerobic bacteria help in biogas(energy) production

iv) Many household products like yoghurt cheese are manufactured by use of bacteria

v) Rhizobium by symbiotic relationship with leguminous plant increase soil fertility

vi) Besides these bacteria is helpful in genetic engineering degradation of petroleum hydrocarbonand in dairy

industry

Physics Motion in plane Here we will introduce Projectile Motion

Execution

Projectile

Y

usinθ u h

θX

ucosθ

Suppose a body is projected with an angle θ So initial velocity u can be resolved into two components

Horizontal component - ucosθ ( for range)

Vertical component - usinθ ( for height)

usinθ changes during motion and becomes zero at maximum height position but ucosθ remain unchanged

The maximum height of projectile is h

NB If initial is upward then g = -ve and if it is downward then g = +ve Height is +ve if direction of motion does not change ( for ex a body thrown upwards but goes down ultimately then height h = -ve)

The angle of projectile θ is the angle made with horizontal

HISTORY ndash GROWTH OF NATIONALISM

SUB TOPIC- REVOLUTIONARY NATIONALISM Bengal formation of Anushilan Samity and Jugantar Group

The intensification of the Swadeshi movement and Government policy of terror and repression led to outbreak of violence Bombs were manufactured and attempts on the lives of unpopular Government officials became frequent In the gymnasium of Scottish Church College which was known as General Assemblies Institution a secret society was formed known as Anushilan Samity

Aurobindo Ghosh send from Baroda his emissary Jatindranath Banerjee to mobilize the Bengal revolutionaries

Hemchandra Qanungo and Satyen Bose published Journal Jugantar

The Jugantar group planned to assassinate oppressive magistrate Kingsford by Khudiram Bose and Prafulla Chaki in 1908 Prafulla Chaki committed suicide to avoid arrest Khudiram was tried and hanged

Afew days later the police found a bomb factory in Maniktala and arrested a large number of revolutionaries The trial of revolutionaries became famous as the Alipore Bomb Case

In the course of the trial the approver the public prosecuter and a police officer were assassinated

1 Question Name two journals which preached the cult of violence

Answer a) Yugantar edited by Bhupendranath DuttaB) Bandemataram edited by Aurobindo Ghosh2 Question Why was Khudiram arrested and hangedAnswer An attempt was made to assassinate a hated vindictive majistrate named Kingsford by Khudiram Bose and Prafulla Chaki Their attempt failed and the bomb they threw killed two English ladies Khudiram was arrested and put to trial and then hanged3Question Who was Aurubindo GhoshAnswer Aurobindo Ghosh a nationalist revolutionary who was charged for his involvement in the Alipore Bombing Case He was accused of it along with his brother Barindra nath Ghosh But Aurobindo was acquitted because of the brilliant pleading of his counsel Chittaranjan Das Then he became a spiritual reformer introducing his visions on human progress and spiritual evolution4 Qustion Who was KingsfordAnswer Kingsford was an unpopular British chief Magistrate who was the target of the bomb thrown at Muzaffarpur by Khudiram and Prafulla Chaki

Most of the accused were convicted and sentenced to

long term of imprisonmentBut

Aurobindo Ghosh was acquitted mainly owing to the brilliant pleading of his counsel Chittaranjan Das

Political science Topic-Sovereignty

Summary Sovereignty is the full right and power of a governing body over itself without any interference from outside sources or bodies In political theory sovereignty is a substantive term designating supreme legitimate authority over some polity In international law sovereignty is the exercise of power by a state

Internal Sovereignty

Internal sovereignty means supreme authority within ones territory while external sovereignty relates to the recognition on the part of all states that each possesses this power in equal measure

External sovereignty

external sovereignty relates to the recognition on the part of all states that each possesses this power in equal measure

Distinguish between

Execution

Answer the following questions

Short notes-

Sovereignty

Internal Sovereignty

External sovereignty

Homework- learn

external sovereignty and internal sovereigntySovereignty is the principle

of supreme and

unquestionable authority

reflected in the claim by the

state to be the sole author of

laws within its territory

Definition of external vs internal sovereigntyInternal sovereignty refers to

the relationship between a

sovereign power and its

subjects ndash it refers to the

location of the supreme

authority within the state In

the UK for example internal

sovereignty (supposedly)

resides within Parliament

reflected in the

constitutional principle of

parliamentary

sovereigntyBy contrast

external sovereignty refers

to the capacity of the state

to act independently and

autonomously on the world

stage This is what is

sometimes called lsquostate

sovereigntyrsquo or lsquonational

sovereigntyrsquo and implies

that states are legally equal

and that the territorial

integrity and political

independence of a state is

inviolable

Class ndash XII

Date - 2742020 STUDY MATERIALSubject Topic Summary Execution Business Studies

Job Analysis amp Manpower Planning

At first let us recall the chapter what we have discussed till nowbullJob analysisbullJob specification bullJob description bullJob enlargement bullJob enrichment

Today we will do some questions answers from the chapter

Questions 1ldquoJob analysis job description and job specification are interrelatedrdquo Comment Answer) Job analysis is a systematic and detailed examination of a job to collect all the relevant information about it The contents off the job are summarised in the job description The qualification needed for the job are summarised in job specificationThus there is close interrelationship between job analysis job description and job specification

Question 2ldquoJob enlargement is a horizontal extension of a job whereas job enrichment is a vertical extension of a jobrdquo ElucidateAnswer) Job enlargement involves adding one or more task to a job coma where as job enrichment involves adding more autonomy and responsibility to a job Job enlargement is therefore horizontal extension of a job coma whereas job enrichment is a vertical extension of a job

Question 3 )

What is manpower estimation Explain its quantitative and qualitative aspectsAnswer) Manpower estimation is the process by which management determines how an organisation should move from itrsquos current manpower positionto its desired manpower position There are two dimensions of Manpower estimation- quantitative and qualitative

Quantitative aspectThis aspect of Manpower estimation involves estimating the number of employees required in a future time period Workload analysis and workforce analysis are done to estimate the quantity of required manpower

Qualitative aspectThe estimate of the knowledge skills experience etc of required manpower is the qualitative aspect of Manpower estimation The quality of Manpower can be judged on the basis of job analysisand job specification

COMMERCE

CAPITAL-FIXED AND WORKING

Today let us start the class by discussing the sources of finance for different types of business firms

The term lsquocapitalrsquo refers to the investment made in the enterprise for the purpose of earning profits

Requirements of capital and sources of capital for different types of business firms are

1 Capital for sole proprietorship businessA sole proprietor operates at a small scale and thereforerequires a limited amount of capital

2 Capital for partnership firmCapital requirements as well as capital base of a partnership is bigger than that of a sole trader businessThe owned capital is contributed by the partners in an agreed ratio

3 Capital for joint stock companyA joint stock company generally requires large amount of capitalA public company can raise huge capital through issue of shares In addition to share capital it can utilize retained profits

Now let u discuss the meaning of Finance PlanningFinance planning is the process of estimation the financial requirements of an organization specifying the sources of firms and ensuring that enough funds are available at the right time

1 What do you mean by Finance PlanningAnswer Finance planning is the process of estimation the financial requirements of an organization specifying the sources of firms and ensuring that enough funds are available at the right time

2Discuss the role of financial planning of an enterpriseThe role of financial planning are as followsa A sound financial plan helps a business enterprise to avaid the problems of shortage and surplus of fundsbFinancial planning serves as a guide in developing a sound capital structure so as to maximize returns to shareholders c It helps in effective utilization of fundsd It provides policies and procedures for coordinating different functional areas or departments of businesse It enables the management to exercise effective control over the financial activities of an enterprisef It helps the company to prepare for facing business shocks and surprises in future

Mathematics

Continuity and differentiability

Recall Definition of ContinuityLet f(x) be a single valued function of x and x=a be a point in the domain of definition of the function The function is said to be continuous at x=a ifi) f(c) is defined ie f(x) has a definite finite value at x=cii) lim xrarra f(x) exists andiii) lim xrarra f(x) =f(a) In other words f(x) is said to be continuous at x=a if lim xrarra+ f(x)= lim xrarra- f(x) = f(a) Or f(a+0) =f(a-0) =f(a) Or lim hrarr0 f(a+h)= f(a) Algebra of continuous functionsNow we will study some algebra of continuous functions Theorem 1 Suppose f and g be two real functions continuous at a real number c Then(1) f + g is continuous at x = c(2) f ndash g is continuous at x = c(3) f g is continuous at x = c(4) (fg) is continuous at x = c (provided g (c) ne 0)

Example 1 Prove that every rational function is continuousSolution Recall that every rational function f is given byf(x)=[p(x) q(x) ] q(x)ne0where p and q are polynomial functions The domain of f is all real numbers except points at which q is zero Since polynomial functions are continuous f is continuous by (4) of Theorem 1Example 2Discuss the continuity of sine functionSolution To see this we use the following factslim xrarr0 sin x =0Now observe that f (x) = sin x is defined for every real number Let c be a real number Put x = c + h If x rarr c we know that h rarr 0 Therefore lim xrarrc f(x)

= lim xrarrc sin x= lim hrarr0 sin(c+h) =lim hrarr0 [sin c cos h + cos c sin h ]=lim hrarr0 (sin c cos h) + lim hrarr0 (cos c sin h) = sin c +0=sin c = f(c) Thus lim xrarrc f(x) = f(c) and hence f is a constant function Exercise Prove that the function f(x) = x2 +2x is continuous for every real value of x [Hints show that lim xrarra+ f(x) = lim xrarra- f(x) = f(a) ]

Biology Reproduction in Flowering plants We will discuss about megasporoangium

megasporagenesis and female gametophyte

Q4 Describe the structure of megasporangium

Ovule is attached to the placenta by astalk called funicle

Each ovule has one two or three protectivecoverings called integuments

At the tip of integuments a small openingcalled micropyle is organised

Opposite to the micropylar end is the chalaza

Within the integuments a mass of cellsnucellusand inside it embryo sac orfemale gametophyte is present

Q5 Describe a mature embryosacamp its formation

In most of the flowering plants only oneof the 4 megaspores formed as a result ofmegasporogenesis that is functional while theother three degenerate

The

functional megaspore develops into thefemale gametophyte

Formation The nucleus

of the functional megasporedivides mitotically to form two nuclei first andthen two more sequential mitotic nucleardivisions result in the formation of four ampthen eight nucleate stages of embryo sac

Six of the eight nucleus are surrounded bycell walls and organised into cells

The remaining two nuclei called polar nuclei are found below the egg apparatus in the largecentral cell

Three cells consisting of two synergids amp one egg cell present bottom of

embryo sac Three cells

at the chalazal as antipodal cells

Two polar nuclei together present in large central cell

HISTORY

TOWARDS INDEPENDENCE AND PARTITION THE LAST PHASE(1935-1947)SUB TOPIC NATIONAL MOVEMENTS DURING THE SECOND WORLD WAR

Spread of Quit India Movement On 9th August 1942Gandhiji and other Congress leaders were arrested The Congress was declared illegal The news of the arrest of all leaders marked the beginning of a widespread movement of India It was not possible for such a movement to remain peacefulBut the arrest of the all notable congress leaders virtually left the movement in the hands of the mass The movement took the form of violent and militant outbreakBesides congressmen revolutionaries also were very active in the movement The Congress Socialist group also played a prominent role

1 Question Why did the British authority arrest the Congress leaders on 9 th August 1942Answer Congress Working committee adopted the Quit India resolution which was to be ratified at the Bombay AICC meeting in 8th August 1942 They decided to launch a mass struggle on non-violent lines Gandhiji gave a clarion call to all section of the people rdquoKarenge ya Marengerdquo (do or die) Congress leaders gave the call to driving out

the British from IndiaViceroy had taken strong action against the Quit India movement Gandhiji and all the leaders of Congress were arrested

2 Question How did Quit India Movement spread out all over IndiaAnswer The news of the leaders lsquo arrest marked the beginning of a widespread movement to remain peacefulThe movement took form of violent outbreak There were widespread cutting of telephone and Telegraph wires damaging railway lines raising barricades in cities and towns and other forms of violent demonstations

Question Name the leaders of Congress

Socialist group played a prominent part Notable among the Jayprakash Narayan Rammonohar Lohia Aruna Asaf Ali

Political science

Topic-Franchise and Representation

Summary

The election commission

The Election Commission of India is an autonomous constitutional authority responsible for administering Union and State election processes in India The body administers elections to the Lok Sabha Rajya Sabha State Legislative Assemblies in India and the offices of the President and Vice President in the country

Functions of election commission-

India is a sovereign socialist secular democratic republic Democracy runs like a golden thread in the social economic and political fabric woven by the Constitution given by lsquoWe the People of Indiarsquo unto ourselves The concept of democracy as visualised by the Constitution pre-supposes the representation of the people in Parliament and State legislatures by the method of election The Supreme Court has held that democracy is one of the inalienable basic features of the Constitution of India and forms part of its basic structure The Constitution of India adopted a Parliamentary form of government Parliament consists of the President of India and the two Houses mdash Rajya Sabha and Lok Sabha India being a Union of states has separate state legislatures for each state State legislatures consist of the Governor and two Houses mdash Legislative Council and Legislative Assembly mdash in seven states namely Andhra Pradesh Telangana Bihar Jammu amp Kashmir Karnataka Maharashtra and Uttar Pradesh and of the Governor and the state Legislative Assembly in the remaining 22 states Apart from the above two out of the seven Union Territories namely National Capital Territory of Delhi and Puducherry also have their Legislative Assemblies

ExecutionShort notes-Election commissionFunctions of election commission

Homework- Learn

Computer

Science

Computer hardware NAND Gate

A NOT-AND operation is known as NAND operation It has n input (n gt= 2) and one output

Logic diagram

Truth Table

NOR Gate

A NOT-OR operation is known as NOR operation It has n input (n gt= 2) and one output

Logic diagram

Truth Table

XOR Gate

XOR or Ex-OR gate is a special type of gate It can be used in the half

adder full adder and subtractor The exclusive-OR gate is abbreviated as EX-OR gate or sometime as X-OR gate It has n input (n gt= 2) and one output

Logic diagram

Truth Table

XNOR Gate

XNOR gate is a special type of gate It can be used in the half adder full adder and subtractor The exclusive-NOR gate is abbreviated as EX-NOR gate or sometime as X-NOR gate It has n input (n gt= 2) and one output

Logic diagram

Truth Table

Physics

Chapter 1 Electric Field ( Electric Dipole) (Summary)

Here we will derive Expression of electric field at broad side

On position of dipole

Execution

Q With the help of a labelled diagram obtain an expression for the electric field intensity E at any point on the equitorial line ( broad-side on position) of an electric dipole

Ans

E1 E1sinθ

E θ P E1 θ

( r2+L2)12 E2 E

r E2 E2sinθ

-q θ L O L +qA B

Let us consider that the point P is situated on the right bisector of the dipole AB at a distance r meter from its midpoint O

Let E1 and E2 be the electric field intensities of the electric field at P due to charge +q and ndashq of the dipole resp The distance of P from each charge is ( r2+L2)12

So E1 = 14 πϵ q

(r 2+L 2) away from +q

E2 = 14 πϵ q

(r 2+L 2) towards ndashq

The magnitudes of E1 and E2 are equal but directions are different Now resolving E1 and E2 into two components parallel and perpendicular to AB we get

The components perpendicular to AB E1sinθ and E2sinθ cancel each other because they are equal and opposite

The components parallel to AB are E1cosθ and E2 cosθ are in same direction and add up

So resultant intensity of electric field at the point P is

E = E1cosθ + E2 cosθ

E = 14 πϵ q

(r 2+L 2) 2 cosθ

Now from fig we have cosθ =BOBP = L (r2+L2)12

So we get E = 14 πϵ 2qL ( r2+L2)32

Now electric dipole moment p= 2qL

So E = 14 πϵ p ( r2+L2)32

HW Find the expression of Electric field as done here but this time take r gtgt 2L

Also find the expression of torque experience by a dipole

(Hint Electric force experienced by charges of dipole in electric field is qE each Let θ be the angle which dipole makes with electric lines of force then perpendicular distance between two charges is 2Lsinθ Then torque = force x perp distance = qE x 2L sinθ So τ=pE sinθ where p =2qL )

STUDY MATERIAL

Class XIISubject Eng Literature (The Tempest ndash William Shakespeare) Topic Act IV Scene 1 Lines 84 to 133 (Iris hellip A contract of true love Be not too late ) Date 27th April 2020 (4th Period)

[Students should read the original play and also the paraphrase given in the school prescribed textbook]Summary Questions amp Answers

o Ceres soon appears and comes to know that she has been summoned to celebrate the contract of true love

o Ceres expresses her unwillingness to meet Venus and Cupid as she has shunned their company

o Ceres and Juno both bestow their blessings upon Ferdinand and Miranda with June gifting honour riches happiness in marriage and Ceres presents plenty of earthrsquos produce

o Iris summons the water-nymphs and reapers to come and celebrate a contract

(1) IRIS Of her society (Line 91-101)

Be not afraid I met her deity

Cutting the clouds towards Pathos and her sonDove-drawn with her Here thought they to have doneSome wanton charm upon this man and maidWhose vows are that no bed-right shall be paidTill Hymens torch be lightedmdashbut in vainMarss hot minion is returned againHer waspish-headed son has broke his arrowsSwears he will shoot no more but play with sparrowsAnd be a boy right out

(i) Where were Venus and Cupid seen flying How were they travelling Why did they want to join the marriage celebration of Ferdinand and Miranda

of true love

Venus and Cupid were seen flying through the air towards Paphos the famous city which is situated on the island of Cyprus They were travelling by air-borne chariot drawn by doves They certainly wanted to come here in order to play some amorous trick upon Ferdinand and Miranda who are under a vow not to gratify their physical desires till the holy ceremony of their marriage has been performed(ii) What have Venus and Cupid done after failing in their plan

After being failure of their plan Venus who is a very passionate deity and who is the mistress of Mars (the god of war) has gone back while here ill-tempered son Cupid has broken his arrows of love in his state of desperation(iii) What has Cupid firmly decided

Cupid is feeling so disappointed that he has firmly decided to shoot no more arrows to arouse love in human hearts but to spend his time playing with sparrows Thus he would now become just a boy and would give up his original function of shooting arrows on human beings to make them fall in love(iv) What vow had Ceres taken How did Ceres feel at the abduction

After the abduction of her daughter Prosperina by Pluto Ceres had taken a vow to always keep away from the disgraceful company of Venus and her blind son Cupid the god of love Ceres felt deeply distressed when Pluto had carried off her daughter and had made her his wife by force(v) Why has Ceres not forgiven Venus and her blind son For what do Ceres want to be sure

As the abduction had been manipulated by Venus the goddess of beauty and love and her blind son Cupid Ceres has never forgiven them for their part in the whole plot Ceres wants to be sure that she would not have to meet Venus and Cupid who had engineered the abduction of her daughter Prosperina

AS THIS lsquoMASQUErsquo SCENE IS VERY IMPORTANT IN THE PLAY THE PARAPHRASE OF THE ENTIRE PORTION OF MASQUE SCENE (Act IV Lines 58 to 143) IS GIVEN BELOW

IRIS Goddess of RainbowCERES Goddess of Agriculture and all the fruits of the earth

(Nature growth prosperity rebirth ndash notions intimately connected to marriage)JUNO The majestic Queen of Heavens and wife of Jupiter (Jupiter is the king of Gods)

VENUS The Goddess of love CUPID Son of Venus PLUTO God of death (In the play referred by Shakespeare as lsquoDisrsquo which is a Roman name for Pluto)

ORIGINAL TEXT PARAPHRASEPROSPEROWellmdash

PROSPERONow come Ariel Let there be too many rather than too few

Now come my Ariel Bring a corollaryRather than want a spirit Appear and pertly[to Ferdinand and Miranda]No tongue all eyes Be silent

spirits in attendance Appear briskly

[to Ferdinand and Miranda]Look with your eyes but do not say a word

[Soft music] [Soft music][Enter Iris] [Enter Iris]

IRISCeres most bounteous lady thy rich leasOf wheat rye barley vetches oats and peasThy turfy mountains where live nibbling sheepAnd flat meads thatched with stover them to keepThy banks with pioned and twilled brimsWhich spongy April at thy hest betrimsTo make cold nymphs chaste crowns and thybroom-grovesWhose shadow the dismissegraved bachelor lovesBeing lass-lorn thy pole clipped vineyardAnd thy sea-marge sterile and rocky-hardWhere thou thyself dost airmdashthe Queen othrsquoSkyWhose watery arch and messenger am IBids thee leave these and with her sovereign grace[Juno appears] Here on this grass-plot in this very placeTo come and sport Her peacocks fly amainApproach rich Ceres her to entertain

IRISCeres most generous lady you are the cause of rich fields or fertile land where wheat rye barley beans oats and peas grow the grassy mountains where the sheep graze and the flat meadows covered with coarse hay to be used as fodder for cattleYour banks are covered with marsh-marigolds and reeds and the rainy April under your orders brings forth to make for the maids who are not in love beautiful crowns your woods where the broom flourishes and where the bachelor who has been dismissed by the maid he loved lies down being forsaken your vineyard in which the poles are embraced by the vines and the margin of the sea which is barren and rocky where you roam about to enjoy the fresh air ndash the queen of the sky (Juno) whose messenger I am besides being represented as the rainbow bids you leave all these and with her majesty here on this grassy plot in this very place come and sport her peacocks carry her fast in her chariot through the air and are making their way here approach rich Ceres to welcome her

[Enter Ariel as Ceres] [Enter Ariel as Ceres]

CERESHail many-coloured messenger that neerDost disobey the wife of JupiterWho with thy saffron wings upon my flowersDiffusest honey-drops refreshing showersAnd with each end of thy blue bow dost crownMy bosky acres and my unshrubbed downRich scarf to my proud earth Why hath thy queenSummoned me hither to this short-grassed green

CERESWelcome rainbow that never dared disobey Juno the wife of Jupiter who with your orange coloured rays spread honey-drops refreshing showers And with each end of thy blue bow drown my bushy acres and my hilly country which is free from shrubs you thus forming a rich scarf Why has your queen called me here to this place covered with short grass

IRISA contract of true love to celebrateAnd some donation freely to estateOn the blest lovers

IRISI have called you to celebrate a contract of true love and bestow some liberal gift upon the blessed lovers

ORIGINAL TEXT PARAPHRASECERESTell me heavenly bowIf Venus or her son as thou dost knowDo now attend the queen Since they did plotThe means that dusky Dis my daughter gotHer and her blind boys scandaled companyI have forsworn

CERESTell me heavenly bow if Venus the Goddess of love or Cupid her son and pedlar of passion at this time attend the heavenly queen Juno because you are sure to know Since the day they conspired against me and dark Pluto took away my daughter here and Cupidrsquos disgraceful company I have left off

IRISOf her societyBe not afraid I met her deityCutting the clouds towards Pathos and her sonDove-drawn with her Here thought they to have doneSome wanton charm upon this man and miad

IRISBe not afraid of her company I met her deity moving on the clouds towards Paphos the sacred home of Venus on the island of Cyprus along with her son on her chariot drawn by doves Here they contemplated to exercise a charm upon this man and maid producing

Whose vows are that no bed-right shall be paidTill Hymens torch be lightedmdashbut in vainMarss hot minion is returned againHer waspish-headed son has broke his arrowsSwears he will shoot no more but play with sparrowsAnd be a boy right out

wantonness before the actual marriage ceremony but did not succeed Venus has returned her irritable son has broken his arrows and swears that he will give up his practice of trying to inspire love but play with sparrows and be a boy again

[Music is heard] [Music is heard]

CERESHighst queen of stateGreat Juno comes I know her by her gait

CERESHighest queen of state Great Juno there she comes I know here by her gait

[Enter Juno] [Enter Juno]

JUNOHow does my bounteous sister Go with meTo bless this twain that they may Prosperous beAnd honoured in their issue

JUNOHow are you doing my generous sister Come with me to bless this couple so that they may be prosperous and fortunate in their children

[They sing] [They sing]

JUNOHonour riches marriage-blessingLong continuance and increasingHourly joys be still upon youJuno sings her blessings upon you

JUNOMay honour riches happiness in marriage long continuance and increase of those boons ever rest upon you as hourly joys Juno showers down upon you her blessings in song

CERESEarths increase foison plentyBarns and garners never emptyVines and clustring bunches growingPlants and goodly burden bowingSpring come to you at the farthestIn the very end of harvestScarcity and want shall shun youCeresrsquo blessing so is on you

CERESMay you have the plenty of earthrsquos produce Your barns and granaries may never be empty Your vines may grow with clustering bunches Your fruit trees may be heavily laden with their fruit May there be continuous spring and harvest May scantiness and want leave you forever Such is the blessing of Ceres upon you

FERDINANDThis is a most majestic vision andHarmoniously charmingly May I be boldTo think these spirits

FERDINANDThis is a great vision and magically melodious Should I suppose the characters (taking part in the masque) are spirits

PROSPEROSpirits which by mine artI have from their confines calld to enactMy present fancies

PROSPEROYes they are spirits whom I have summoned from the regions to which they are confined to carry into effect my fanciful designs

ORIGINAL TEXT PARAPHRASEFERDINANDLet me live here everSo rare a wondered father and a wifeMakes this place paradise

FERDINANDI should like to live here forever Such a wise and wonderful father makes this place a paradise

[Juno and Ceres whisper and send Iris on employment] [Juno and Ceres whisper and send Iris on employment]

PROSPEROSweet now silence

PROSPEROMy dear Ferdinand speak no more Juno and Ceres are

Juno and Ceres whisper seriouslyTheres something else to do Hush and be muteOr else our spell is marred

whispering with a solemn look There is something else coming Silence Or else our magic will be spoilt

IRISYour nymphs called naiads of the wandering brooksWith your sedged crowns and over-harmless looksLeave your crisp channels and on this green landAnswer your summons Juno does commandCome temperate nymphs and help to celebrateA contract of true love Be not too late

IRISYou nymphs called Naiads denizens (M inhabitants) of the running stream with your chaplets of sedge and ever-helpful looks leave your wrinkled channels and on the green land answer the summons sent to you Juno has ordered some chaste nymphs and help to celebrate a noble and true marriage Donrsquot delay

[Enter certain nymphs] [Enter certain nymphs]You sunburnt sicklemen of August wearyCome hither from the furrow and be merryMake holiday your rye-straw hats put onAnd these fresh nymphs encounter every oneIn country footing

You sunburnt harvesters weary from the effects of the heat in August come here from the furrowed land and rejoice Make holiday with your rye-straw hats upon you and meet these fresh nymphs and join in country dancing

[Enter certain reapers properly habited They join with the nymphs in a graceful dance towards the end whereof Prospero starts suddenly and speaks]

[Enter certain reapers properly habited They join with the nymphs in a graceful dance towards the end whereof Prospero starts suddenly and speaks]

PROSPERO[aside] I had forgot that foul conspiracyOf the beast Caliban and his confederatesAgainst my life The minute of their plotIs almost come [to the spirits]Well done Avoidno more

PROSPERO(Aside)I had forgotten the wicked conspiracy of the beast Caliban and his accomplices against my life the time of their plot has almost arrived ndash (To the Spirits) well done depart no more of this

[To a strange hollow and confused noise the spirits heavily vanish]

[The spirits depart]

ORIGINAL TEXT PARAPHRASEFERDINANDLet me live here everSo rare a wondered father and a wifeMakes this place paradise

FERDINANDI should like to live here forever Such a wise and wonderful father makes this place a paradise

[Juno and Ceres whisper and send Iris on employment] [Juno and Ceres whisper and send Iris on employment]

PROSPEROSweet now silenceJuno and Ceres whisper seriouslyTheres something else to do Hush and be muteOr else our spell is marred

PROSPEROMy dear Ferdinand speak no more Juno and Ceres are whispering with a solemn look There is something else coming Silence Or else our magic will be spoilt

IRISYour nymphs called naiads of the wandering brooksWith your sedged crowns and over-harmless looksLeave your crisp channels and on this green landAnswer your summons Juno does commandCome temperate nymphs and help to celebrateA contract of true love Be not too late

IRISYou nymphs called Naiads denizens (M inhabitants) of the running stream with your chaplets of sedge and ever-helpful looks leave your wrinkled channels and on the green land answer the summons sent to you Juno has ordered some chaste nymphs and help to celebrate a noble and true marriage Donrsquot delay

[Enter certain nymphs] [Enter certain nymphs]You sunburnt sicklemen of August wearyCome hither from the furrow and be merryMake holiday your rye-straw hats put onAnd these fresh nymphs encounter every oneIn country footing

You sunburnt harvesters weary from the effects of the heat in August come here from the furrowed land and rejoice Make holiday with your rye-straw hats upon you and meet these fresh nymphs and join in country dancing

[Enter certain reapers properly habited They join with the nymphs in a graceful dance towards the end whereof Prospero starts suddenly and speaks]

[Enter certain reapers properly habited They join with the nymphs in a graceful dance towards the end whereof Prospero starts suddenly and speaks]

PROSPERO[aside] I had forgot that foul conspiracyOf the beast Caliban and his confederatesAgainst my life The minute of their plotIs almost come [to the spirits]Well done Avoidno more

PROSPERO(Aside)I had forgotten the wicked conspiracy of the beast Caliban and his accomplices against my life the time of their plot has almost arrived ndash (To the Spirits) well done depart no more of this

[To a strange hollow and confused noise the spirits heavily vanish]

[The spirits depart]

Ac-12 27420 topic Revaluation of Assets and Liabilities

REVALUATION OF ASSETS AND LIABILITIES

On admission of a new partner the firm stands reconstituted and consequently the assets are revalued and liabilities are reassessed It is necessary to show the true position of the firm at the time of admission of a new partner If the values of the assets are raised gain will increase the capital of the existing partners Similarly any decrease in the value of assets ie loss will decrease the capital of the existing partners For this purpose alsquoRevaluation Accountrsquo is prepared This account is credited with all increases in the value of assets and decrease in the value of liabilities It is debited with decrease on account of value of assets and increase in the value of liabilities The balance of this account shows a gain or loss on revaluation which is transferred to the existing partnerrsquos capital account in existing profit sharing ratioAccounting for Revaluation of Assets and Liabilities when there is a Changein the Profit Sharing Ratio of Existing PartnersAssets and liabilities of a firm must also be revalued at the time of change in profit sharing ratio of existing partners The reason is that the realisable or actual value of assets and liabilities may be different from those shown in the Balance Sheet It is possible that with the passage of time some of the assets might have appreciated in value while the value of certain other assets might have decreased and no record has been made of such changes in the books of accounts Similarly there may be some unrecorded assets amp libilities that may have to be accounted for Revaluation of assets and reassessments of liabilities becomes necessary because the change in the

value of assets and liabilities belongs to the period to change in profit sharing ratio and hence must be shared by the partners in their old profit sharing ratio Revaluation of assets and reassessment of liabilities may be given effect to in two different ways (a) When revised values are to be recorded in the books and(b) When revised values are not to be recorded in the books

When revised values are to be recorded in the booksIn such a case revaluation of assets and reassessment of liabilities is done with the help of a new account called lsquoRevaluation Accountrsquo Sometimes this account is also called as lsquoProfit amp Loss Adjustment Acrsquo If there is a loss due to revaluation revaluation account is debited and if the revaluation results in a profit the revaluation account is credited The following journal entries made for this purpose are

(i) For increase in the value of assetsAsset Ac Dr (individually)To Revaluation Ac(ii) For decrease in the value of AssetRevaluation Ac Dr (individually)To Asset Ac[Decrease in the value of assets](iii) For increase in the value of LiabilitiesRevaluation Ac Dr (individually)To Liabilities Ac[Increase in the value of Liabilities](iv) For decrease in the value of LiabilitiesLiabilities Ac DrTo Revaluation Ac[Decrease in the value of Liabilities](v) For unrecorded AssetsAsset Ac [unrecorded] DrTo Revaluation Ac[Unrecorded asset recorded at actual value](vi) For unrecorded Liability Revaluation Ac DrTo Liability Ac [unrecorded][Unrecorded Liability recorded at actual value](vii) For transfer of gain on revaluationRevaluation Ac DrTo Existing Partnerrsquos CapitalCurrent Ac[Profit on revaluation transferred to capital account in existing ratio](viii) For transfer of loss on revaluationExisting Partnerrsquos CapitalCurrent Ac DrTo Revaluation Ac[Loss on revaluation transferred to capital account in existing ratio](a) When revaluation account shows gain Revaluation Ac DrTo Partnerrsquos Capital Ac (Old Profit Sharing Ratio)(Profit on revaluation credited to Partnerrsquos Capital Ac)(b) Above entry is reversed when revaluation account shows loss Partners Capital Acs (Old Profit Sharing Ratio) DrTo Revaluation Ac(Loss on revaluation debited to Partnerrsquos Capital Acs)

Proforma of Revaluation Account is given as under

Revaluation Account

Dr Cr Particulars ` Amount Particulars ` Amount To Decrease in value of assets By Increase in value of assets To Increase in value of liabilities By Decrease in value of liabilities To Unrecorded liabilities By Unrecorded assets To Gain on Revaluation (Transferred) By Loss on Revalution (Transferred)

ECO ndash12 2742020Topic- ELASTICITY OF DEMAND

CHAPTER - ELASTICITY OF DEMANDMEANINGDemand for a commodity is affected by many factors such as its price price of related goods income of its buyer tastes and preferences etc Elasticity means degree of response Elasticity of demand means degree of responsiveness of demand Demand for a commodity responds to change in price price of related goods income etc So we have three dimensions of elasticity of demandDIMENSION OF ELASTICITY OF DEMAND TYPES OF ELASTICITY OF DEMAND

Price elasticity of demand Income elasticity of demand Cross Elasticity of demand

Price elasticity of demand Price elasticity of demand means degree of responsiveness of demand for a commodity to the change in its price For example if demand for a commodity rises by 10 due to 5 fall in its price Price elasticity of demand (ep)=Percentage change in quantity demanded Percentage change in price of the commodity = 10 ( -)5 = ( - )2Note that ep will always be negative due to inverse relationship of price and quantity demanded

(ii) Income elasticity of demand Income elasticity of demand refers to the degree of responsiveness of demand for a commodity to the change in income of its buyer Suppose income of buyer rises by 10 and his demand for a commodity rises by 20 then Income elasticity of demand (ey)= change in quantity demanded change in price of the commodity =20 10 = 2

Cross Elasticity of demandCross elasticity of demand means the degree of responsiveness of demand for a commodity to the change in price of its related goods (substitute goods or complementary goods) Suppose demand for a commodity rises by 10 due to 5 rise in price of its substitute good then Cross elasticity of demand (ec) = change in quantity demanded change in price of related good = 10 2 = 5 (Tastes and preferences cannot be expressed numerically So elasticity ofdemand cannot be numerically expressed)

  • Chapter 1 Force (Summary)
  • Distinguish between external sovereignty and internal sovereignty
    • NAND Gate
      • Logic diagram
      • Truth Table
        • NOR Gate
          • Logic diagram
          • Truth Table
            • XOR Gate
              • Logic diagram
              • Truth Table
                • XNOR Gate
                  • Logic diagram
                  • Truth Table
                      • Physics
                      • Chapter 1 Electric Field ( Electric Dipole) (Summary)
Page 12:  · Web viewWe all know that Nouns are divided into two parts: common noun and proper noun.Apart from common and proper noun, we will also study about collective noun and compound

KNOWN AS SYNTAX WHEREAS THE TERM SEMANTICS REFERS TO THE MEANING OF LANGUAGESCOMPUTER CANNOT DEVELOP ANY PROGRAMMING LANGUAGE BY THEMSELVES AND CANNOT THINK INTELLIGENTLY UNLESS THEY ARE ASSISTED BY THE HUMAN BEINGS THEREFORE THE PROGRAMMERS DEVELOP A SET OF METHODS AND TECHNIQUES A SET OF INSTRUCTIONS USED TO PERFORM A SPECIFIC TASK IS CALLED A PROGRAM

A PROGRAMMING LANGUAGE IS ALSO KNOWN AS A COMPUTER LANGUAGE CODED BY PROGRAMMERS TO WRITE INSTRUCTIONS FOR A COMPUTERTHE COMPUTER TAKES THESE INSTRUCTIONS AS INPUT AND PRODUCES THE DESIRED OUTPUT

TYPES OF COMPUTER LANGUAGE

THE COMPUTER LANGUAGE CAN BE BROADLY DIVIDED INTO TWO CATEGORIES

LOW LEVEL LANGUAGES HIGH LEVEL LANGUAGES

LOW LEVEL LANGUAGES

A LOW LEVEL PROGRAMMING LANGUAGE REFERS TO THE LANGUAGE THAT IS UNDERSTOOD BY A COMPUTER DIRECTLYTHE PROGRAMMER MUST HAVE AN IN-DEPTH KNOWLEDGE OF DIFFERENT COMPUTERS TO WRITE PROGRAMS IN A LOW LEVEL LANGUAGE THE TWO TYPES OF LOW LEVEL LANGUAGES ARE MACHINE LANGUAGE AND ASSEMBLY LANGUAGE

MACHINE LANGUAGE

COMPUTER HARDWARE UNDERSTANDS ONLY MACHINE LANGUAGE AS IT IS THE FUNDAMENTAL LANGUAGE OF A COMPUTER WHICH UNDERSTANDS ONLY THE TWO DIGITS 0 AND 1 KNOWN AS THE BINARY DIGITSIT IS A SYSTEM OF INSTRUCTIONS EXECUTED DIRECTLY BY THE CPU WITHOUT ANY TRANSLATION AND HENCE IS EXECUTED QUICKLY AS COMPARED TO PROGRAMS IN OTHER LANGUAGES PROGRAMS IN MACHINE LANGUAGE NEED DIFFERENT BINARY LANGUAGE PROGRAMS TO SOLVE THE COMPLEX TASKSPROGRAMMERS OFTEN FIND IT DIFFICULT TO WRITE PROGRAMS IN THIS LANGUAGE

ASSEMBLY LANGUAGE

AN ASSEMBLY LANGUAGE IS A LOW LEVEL PROGRAMMING LANGUAGE BUT THE INSTRUCTIONS ARE WRITTEN IN THE FORM OF WORDS KNOWN AS MNEMONICS TO CONVERT AN ASSEMBLY LANGUAGE INTO TO MACHINE CODE WE REQUIRE A UTILITY PROGRAM WHICH IS KNOWN AS AN ASSEMBLER ASSEMBLY LANGUAGE PROGRAMS CANNOT BE EXECUTED DIRECTLY BY A COMPUTER THEREFORE THEY ARE SLOWER THAN THE MACHINE LANGUAGE PROGRAMSIT IS EASIER TO WRITE PROGRAMS IN THE MACHINE LANGUAGE THEN AND IN ASSEMBLY LANGUAGE

_______________________________________________________________________________________________

CLASS-VIII

SUBJECT-COMPUTER CHAPTER - Operating system and graphical user interface Role and functions

Need of operating system

It co-ordinates different hardware and software componentsof a computer system It supervises the various actions of the computer system and enables the computer to work in a effective

manner It helps in smooth functioning of various peripherals

In a multi-tasking operating system it determines the order and time to be allowed for each application before giving another application a turn

It sends messages to the system operator about the status of operation or any error that may have occurred while running the application

What is operating system

An operating system is an integrated system of programs that manages various resources and the overall operation of the computer system It is designed to support various activities of computer system in a systematic way

Role of an operating system-

Operating system enables the user to use the system effectively An operating system manages various application that runs on a computer and shares computerrsquos resources User interacts with operating system through command line interface and graphical user interface

Function of an operating system -

Booting the computer it is the start up procedure of a computer system Loading theprograms in the memory - when system is ready the operating system loads certain program

automatically Manages resources it manages between the hardware and software resources Detecting and correcting errors- if the supporting hardware or software doesnrsquot works properly then the

operatingsystem tries to rectify it Ensuring data security programs and data donot interfere with each other Maintaining the internal clock of system - maintain internal clock of system when system is shut down

SUBJECT-GEOGRAPHY CHAPTER 1 REPRESENTATION OF GEOGRAPHICAL FEATURES THROUGH CONTOURSTopographical maps- topographical maps are small-scale maps with detailed depiction of both natural and human-made features

Contour lines- contour lines are imaginary lines drawn on a map joining places having the same height above the mean sea level

Contour linesContour interval- It is an interval at which contour lines are drawnIndex contour- At every 100-m interval a thick brown contour line is called index contour It is drawn for calculating height

Spot heights- The spot heights show heights in metres above the mean sea level

SUBJECT- English Language CHAPTER - The Sentence

Complex sentence-

We have seen that a Complex Sentence consists of a Principal Clause with one or more Subordinate Clauses

We have also learnt that there are three kinds of Subordinate Clauses The Adjective Clause the Adverb Clause and the Noun Clause

1) THE ADJECTIVE CLAUSE

An ADJECTIVE CLAUSE does the work of an ADJECTIVE It qualifies a noun or pronoun In the following examples in each set two commonly patterned sentences are compared one with an ADJECTIVE and the other with an ADJECTIVE CLAUSE ( The Adjective or Adjective Clause qualifies the Noun placed in the box)

a) He is a lazy boy (Adjective)

b) He is a boy who is lazy (Adjective Clause)

a) It is a beautiful place for the tourists (Adjective)

b) It is a place where tourists come for the scenic beauty (Adjective Clause)

a) We have enough funds for the work (Adjective)

b) We have funds which would be enough for the work (Adjective Clause)

Convert the sentence from simple to complex sentence-

1) We believe his honesty Ans- We believe that he is honest

2) This is the birth place of RamaAns- This is the place where Rama was born

____________________________________________________________________________________________

Class IX

Subject English Language

Topic Preposition

Date 270420

PREPOSITIONS

A preposition is a word placed before a noun or a pronoun to show in what relation the person or thing denoted by it stands in regard to something else

EXAMPLE

There is a clock on the wall

The preposition on shoes relationship between clock and the wall

Smitha is afraid of lizards The man jumped off the bus

Here the preposition of shows the relationship between afraid and lizards

The preposition off shows the relation between jumped and bus

The noun or the pronoun which follows a preposition is called its object

So in the first Example wall is the object of the preposition on

A preposition can have more than one object For example

The plane flew overhouses and meadows

Prepositions are used to express a number of relationship including time locationmannermeans quantity purpose and state or condition

Points to remember

Preposition joins a noun to another noun or a pronounThere is a cow in the field

A preposition joints a noun to a verbThe cat runs after the rat

Preposition can have two or more than two objectsThe road runs over hills and plain

A preposition also joins a noun an adjectiveHe is fond of tea

Generally a preposition comes before an object Sometimes it comes even after an object asWhat are you looking atThis is the house I live in

Subject- Computer Application

Chapter 2 Introduction to Java

Java API An application programming interface (API) in the context of Java is a collection of prewritten packages classes and interfaces with their respective methods fields and constructors

Byte Code Java bytecode is the result of the compilation of a Java program an intermediate representation of that program which is machine independent The Java bytecode gets processed by the Java virtual machine (JVM) instead of the processor JVM The Java Virtual Machine (JVM) is the runtime engine of the Java Platform which allows any program written in Java or other language compiled into Java bytecode to run on any computer that has a native JVM

Platform A platform is the hardware or software environment in which a program remains

Java platform The Java platform differs from most other platforms in that itrsquos a software only platform that runs on top of other hardware-based platforms

Applet and Application The fundamental difference between the two Java programs is that an application program is designed to run on a stand-alone machine whereas an applet is a web-version of an application which is used to run a program on a web browser

WORA ldquoWrite once run anywhererdquo (WORA) or sometimes write once run everywhere (WORE) is a slogan created by Sun Microsystems to illustrate the cross-platform benefits of the Java language

Class XSubject Topic Summary Execution

ECONOMICSFACTORS OF PRODUCTION Sub-topiclsquoLABOURrsquo

We shall start our class by discussing the topic taught in the last class

lsquo DIVISION OF LABOURrsquo- By division of labour we mean specialization in workIt refers to splitting up the work of labour involved in the production of a particular commodity into several parts and each part and sub-part is performed by a specialist

Now let us start by the Advantages of Division of labouraIt increases the level

Questions

1 Differentiate between Product-Based division of labour and Process-Based division of labour

Product-Based division of labour

Process-based division of labour

It is also known as simple (or occupational) division of labourUnder it everybody performs a particular occupations The entire is done by the same person

When a person or group of persons undertakes a specialised function which is supplementary to the production of final commodity and service This is also as complex division of labour

It is simple It is complex

of productionbSince the product is produced by an expert workerbest quality of product is producedcIt saves time and toolsd it promotes inventions in the methods and techniques of productioneIt leads to reduction in costs fAll workers get work according to their abilities and choices

Now let us discuss the disadvantages of Division of labour

a Since many workers are involved in the production of a commodity no one has the sense of responsibility

b The constant and repetition of the same work again and again make the work monotonous

c Division of labour facilitates production on large scale Hencethere is fear of over production

d Because of territorial division of labour some areasregions become more developed than others

It is based on labour-intensive techniques of production

It is based on capital-intensive techniques

It is generally found in small enterprises

It is generally found in large enterprises

Example Indian farmers doing all farm activities

Example A modern garmet factory where one person takes the measurementanother does the cuttingsome sew the clothes while a few workers button them and other iron them

2 What are the advantages of Division of labouraIt increases the level of productionbSince the product is produced by an expert workerbest quality of product is producedcIt saves time and toolsd it promotes inventions in the methods and techniques of productioneIt leads to reduction in costs fAll workers get work according to their abilities and choices

3Discuss the disadvantages of Division of laboura Since many workers are involved in the

production of a commodity no one has the sense of responsibility

b The constant and repetition of the same work again and again make the work monotonous

c Division of labour facilitates production on large scale Hence there is fear of over production

d Because of territorial division of labour some areasregions become more developed than others

English 1 Transformation of sentences

Sentences A sentence is a group of words which makes complete sense

a Assertive sentences

Exercise 6Rewrite the following sentences according to the instructions given below without changing their meanings

1 As soon as he saw the beer he jumped into

b Imperative sentences

c Interrogative sentences

d Exclamatory sentences

Sentences can be changed from one grammatical form to another without changing the meaning of the sentence This is known as transformation of sentences

the river ( Begin No sooner)2 None but brave deserve the fair (Begin the

bravehellip)3 This box is too heavy for me to lift ( Use so hellip

That instead of too)4 No one other than a king can live like James

Luxurious ( Begin only James)5 Oh for the wings of a dove (Begin I wishhellip)

Math Topic Commercial MathematicsChapter Shares and Dividends

Study item Discuss about shares and Dividends1) What is share

Ans To start any big business (company or Industry) a large sum of money is needed But it is not possible for an individual to invest such a large amount Then some persons interested in the business join together and from a company They divide the estimated money required into small parts Each such part is called a share

2) What do you mean by the term shareholder

Ans A person who purchases one or more shares is called shareholder3) Some terms related with a share

(i) Nominal value or face value or printed value The original value of a share is called its nominal value or face value or printed value

Note The nominal value of a share always remains same(ii) Market value or cash value The price of a share at any

time is called its market value or cash value

Note The market value of a share changes from time to time(iii) At par If the market value of a share is the same as its

nominal value the share is called at par(iv) At Premium or above Par If the market value of a share

is more than its nominal value the share is called at premium or above par

Example If a share of Rs 100 is selling at Rs 150 then it is said to be selling at a premium of Rs 50 or Rs 50 above par

(v) At Discount or below par If the market value of a share is less than its nominal value the share is called at discount or below par

Example If a share of Rs 100 is selling at Rs80 then it is said to be selling at a discount of Rs 20 or at Rs 20 below par

4) What is Dividend

Ans The profit which a shareholder gets for hisher investment from the company is called dividendNote (i) The dividend is always expressed as the percentage of the face value of the share(ii) The dividend is always given( by the company ) on the face value of the share

irrespective of the market value of the shareBENGALI(2ND LANGUAGE)

ldquoদেবতোর জণমrdquoলিবরোম চকরবত

পরথম লিসর পোঠ-চোর পসথ একটি পোথর লিবপলি ঘটোয় দেক যোতোয়োসতর পসথ পরলিতলিয়ত ওই পোথসর দেো োচট দেসত একলি দেতো দেক দেো োচট দেসয় দেবোমো সয় রোসতোর মোস লি0টসক পস1 লিবপরীত লিক দেথসক আো একটি দেমোটর োলি1 চোসকর কষতোয় পরোসরণ দেবোসচ যো লিকনত পরলিতবোর এমরণ দেৌভোয দেসকর োও সত পোসর তোই লিতলি দেকোো দেজোো1 কসর পোথরটিসক উপস1 দে8স পোথর উপস1 দে8োর ময় এক দেকৌতী জতো দেকসক পরশন কসর দেয লিতলি দেকোসো দেবতোর আস দেপসয়স0রণ লিকো লিকনত দেক বস লিতলি দেকোসো দেবতোর আস পোলি দেক উপলিত ক জতোর উসltসয বস কোরও ইস= স পোথরটি লিসয় দেযসত পোসর এর পর দেথসক দেক দেযসত আসত পোথরটি দেক দেসত পো একলি দেক কষয করস দেকউ পোথরটিসক ধসয়মস0 পলিরসকোর করস0 দেক ওই দেকৌতী জতোসক পোথসরর কোস0 বস থোকসত দেস এ0ো1ো আরও কষয কসর দেক ঠোৎ ওই লি1 পোথরটির োসয় লিোর োো-দেকউ পজো কসরস0 দেকৌতী দেোকটির আঙকো য় যলিসক উ পোথরটিসক লিরসয় দে8স তোর পর ঠোৎ একলি পোথরটির দেোোজ দেই দেক লিসয় দেস0 বো দেকোথোয় দেস0

বদোথ-

দেো োচট ndashচসত লিসয় দেকো লিক0র সE ধোককো দেস পস1 যোবোর উপকরমঅকসমোৎ- ঠোৎআতমমবর- লিসজসক লিয়নতরপ-পো লিপ0স প1োদেসতসসত- লিসপলিউৎোত- দেো1ো দেথসক উপস1 দে8োপরতযয় ndash লিবশবোপরতযোস- টতযোস ndash দৈবোসধসতোধলিসত- পরসপসরর পরলিত ব পরসয়ো করোপরসতরীভত- পোথসর পলিররণলিতবোনতঃকরসরণ- মস পরোসঅলিQৎ- অQো করসত ই=কদেোপ- বধমোর- দেবোস এমইতযোকোর- এইরকমরম- বময়পযসোভী-পসযর জয দেোভ আস0 যোরপোসথয় ঞচয়- পথচোর রচ জমোসোমোমোসরোস- ব ধম ধোসমর সE

তোর লি দেইhelliphellipTo be continued

Hindi 2nd lang सर क पद(सरदास)

सरदास शरी कषण भकति कावय क सरवशरषठ कगिरव ह इनक जनम और मतय क समय तथा सथान का मतभद हसरदास रवातसलय और शरार रस क अनयतम कगिरव ह इनक कावय म बालकषण क सौदय चपपल चषटा और गि7याओ की मनोहर झाकी मिमलती ह कषण और ोगिपयो क अननय परम का कतिचतरण ह सयो शरार की अपकषा उनक कावय म गिरवयो शरार का अमिBक गिरवषय और मारमिमEक कतिचतरण हआ हइन पकतियो म हम सरदास की भकति भारवनाओ का परिरचय मिमलता ह इनका सपण सगरह सरसार म गिनगिहत ह

1 जसोदा हरिर पालन झलारवहलरारवदलराईमलहारव रव जो ईसाई कछ ारव मर लाल को आई निनEदिदया काह ना आगिनसबारव त काह नाही बरवगिह आरवतोको कानहा बलारव

शबदाथ-हलरारव-गिहलती हदलराई - दलार पयार करती हमलहारव-पचकारती हनिनEदरिरया ndashनीदरवगिह-जलदी सअBर-होठमौन-चपसन-सकत

वयाखया- सरदास जी कहत ह गिक यशोदा माता बालक कषण को पालन म झल आती ह रवह उनह गिहलाती ह पयार करती ह मलहार जस कोई ीत ान लती ह और नीद स पछती ह गिक ह नीद तम मर लाल को आकर कयो नही सलाती तझ खाना बला रहा ह कभी कषण आख बद कर लत ह कभी आखफडफडान लत ह उनह सोता हआ जानकर यशोदा माता चप हो जाती ह और इशार म बात करन लती ह इसी बीच अकला कर कषण ज जात हतो गिफर यशोदा माता गिफर स ाना ान लती ह सरदास जी कहत ह गिक भरवान क दशन का सख दरवता और ऋगिष-मगिनयो को भी दलभ ह यही सख माता यशोदा को बडी सहजता स मिमल जा रही ह माता यशोदा बहत ही भागयशाली ह2)Continue to nexthellip

Physics

Chapter 1 Force

(Summary)

Question A body is acted upon by two forces each of magnitude F but in opposite directions State the effect of the forces if

(a) Both forces act at the same point of the body

(b)the two forces act at two different points of the body at a separation r

Solutions

(a) Resultant force acting on the body = 0

F ndash F = 0(b) The forces tend to rotate the body between two forces about the midpoint

Moment of forces = F times rFr

QuestionDefine moment of a couple Write its SI unit

Solutions

Moment of couple is equal to the product of both force and the perpendicular distance between the two forces

The SI unit of moment of couple is NmCommercial Studies

Advertising and sales

Business firms use several methods to

Questions1) What do you mean by advertising

promotion create demand of their product in the market and increase it sales Such methods comprises of advertising sales promotion personal selling and publicityToday we are going to discuss about one of such methods It is advertising

Meaning of advertisingAdvertising is a paid form of non-personal presentation for promotion of Ideas goods and services

Importance or merits of advertising Advertising has importance to manufacturer or traders to customer and to society as a whole

Today we will see how advertisement help the manufacturer or traders

Answer) Advertising is a means of how a company encourages people to buy their products services or ideas It is one element of marketing which also includes design Research and data mining

2) Mention any three features of advertisingAnswer)The main features of advertising are

i) It is impersonal form of presentation for promotion of products and services of Ideas

ii) It is issued by identified sponsor The advertisement contains the name of the advertiser

iii) It is a form of mass communication because the message is directed to a large number of persons simultaneously

3) Mention the main merits or importance of advertisement to manufacturer or tradersAnswer)

i) Introducing new product A business organization can introduce itself and its products to the public through advertising

ii) Increase the sale Advertising leads to increase the sale of existing product by entering into new markets and attracting new customers

iii) Create steady demand Advertising creates sustains regular demand by smoothening out seasonal and other fluctuations It enables regular production for the organisation

iv) Economics of scale Advertising facilitate mass distribution of goods and steady demand which lead to large scale and regular production

v) Goodwill Advertising helps in creating a good image of the firm and reputation for its products

Biology Chapter - 03Genetics

Today wewill start chapter and discuss about Genetics Gregor Mendel is known as father of genetics Before entering into Mendelrsquos experiment on Genetics we must know

Q1 Define the following termsi) Genetics Genetics is the study of

transmissionof body features from parents to offspringand the laws relating to such transmission

ii) Heredity It may be defined as transmissionof genetically based characteristics from parentsto offspring

iii) Character and traits Any heritable

Importance to TraderIntroducing new productIncrease the saleCreate steady demandEconomics of scaleGoodwill

some terms featureis a character The alternative forms of acharacter are called traitsex Character (Hair shape) - Traits (Curly straight)

iv) Homologous chromosomes A pair ofcorresponding chromosomes of the same shapeand size one from each parent

v) Genes Genes are the specific parts (DNA segments) of a chromosome which determinethe hereditary characteristicsNearly 30000genes present in human

vi) Alleles Alternative forms of a gene occupying the same position (locus) on homologouschromosomes and affecting the same characteristicbut in different ways

vii) Genotype ndash PhenotypeGenotype means of genes present in the cells of an organism Phenotype means the observable characteristic which is genetically controlled

viii) Mutation It is a sudden change in one or more genes or in the number or in the structure of chromosomes ex Sickle cell anaemia is a blood disease caused by a gene mutation

CLASS NOTES

Class XSubject Eng Literature (The Merchant of Venice ndash William Shakespeare)Topic Act IV Scene 1 Lines 01 to 34 ( Duke helliphelliphellip We all expect a gentle answer Jew) ate 27th April 2020 (2nd Period)

[Students should read the original play and also the paraphrase given in the school prescribed textbook]Summary Questions amp Answers

This scene may be termed as the catastrophe of the play It is the final unravelling of the complicated events which seem to threaten the happiness of Bassanio Portia and Antonio Right is justified to the fullest degree and malice falls into the trap prepared for others No one suffers here but Shylock but even then he receives a generous measure of mercy

o This is the Court-scene Initially we meet

(1)

DUKE I am sorry for thee thou art come to answer (Line 3-6)A stony adversary an inhuman wretchUncapable of pity void and emptyFrom any dram of mercy

(i) Who is addressed here Where is the person Why is the person there

Antonio is addressed hereAntonio is in the court of justice at VeniceAntoniorsquos trial is scheduled to be held here for his failure to meet the conditions of the bond he signed with Shylock

the Duke Antonio Shylock and Salerio Later we meet Bassanio Portia Gratiano and Nerissa

o The Duke says to Antonio that he has to face a very cruel opponent which Antonio admits and expresses his gratefulness to the Duke for his efforts to soften without result the heart of Shylock in order to be merciful to Antonio Antonio further says that he is ready to accept whatever cruel judgement the Court may award

o When Shylock appears in the court the Duke says that Shylock should change his decision of prosecuting Antonio and demanding the penalty specified in the bond out of consideration of the great misfortunes that Antonio has suffered If this is done by him (Shylock) the whole court would be gladdened by his merciful action

(ii) What is the Duke sorry for

The Duke is unable to change the mind of Shylock from his decision to get the bond forfeited even after he pleaded to Shylock Shylock stands firmly for his bond which when forfeited will allow him to take a pound of flesh from any part of Antoniorsquos body(iii) How does the Duke address Shylock`The Duke calls Shylock an adversary with a heart of stones He calls Shylock as an inhuman wretch without pity Shylock is quite lacking in the slightest quality of mercy (iv) How does Antonio reply to this

Antonio replies that he will meet the revenge of Shylock patiently He has prepared himself to suffer with a quiet spirit the utmost that Shylockrsquos tyranny and rage can do(v) What quality of the Duke is revealed here

The Duke is kind and benevolent He is ready to help Antonio He requests Shylock to free Antonio from the trial(vi) What are the terms of the bond that Antonio has signed

The terms of the bond that Antonio has signed were that if Antonio is unable to repay Shylock a certain sum of money specified on the paper on a certain date and in an agreed place the forfeit has to be paid The forfeiture will be an exact pound of Antoniorsquos flesh which Shylock will be a liberty to take from any part of Antoniorsquos body which pleases him

Class XI

STUDY MATERIAL

Class XISubject Eng Literature (The Tempest ndash William Shakespeare) Topic Act I Scene 2 Lines 88 to 132 (Prospero hellip Me and thy crying self) Date 27th April 2020 (3rd Period)

[Students should read the original play and also the paraphrase given in the school prescribed textbook]Summary Questions amp Answers

o Prospero now tells Miranda that he was the Duke of Milan He had been devoting himself more to studies than the affairs of the State His brother Antonio took advantage of this situation and with the help of Alonso the king of Naples seized upon him and her one midnight and shipped them in a frail bark so that they perished in the sea All this took place

(1)

MIRANDA I should sin (Line 118-132)

To think but nobly of my grandmother

Good wombs have borne bad sonsPROSPERO Now the condition

The King of Naples being an enemyTo me inveterate hearkens my brothers suitWhich was that he in lieu othrsquo premisesOf homage and I know not how much tribute

twelve years back

IMPORTANT PASSAGES EXPLAINED(Line 98-103)

PROSPERO helliphelliphelliphelliphelliphelliphellip

He being thus lorded

Not only with what my revenue yieldedBut what my power might else exact like oneWho having into truth by telling of it Made such a sinner of his memoryTo credit his own lie he did believeHe was indeed the duke

Prospero in telling the narrative of his past life here refers to his brother Antonio Prospero being with a studious bent of mind has left the administration of Milan on his younger brother Now Antonio being thus invested like a lord with all the powers derived from Prosperorsquos wealth and what the exercise of Prosperorsquos authority might secure for him regarded himself as a de facto Duke of Milan It is a well-known fact of psychology that a man who repeatedly tells a lie makes of his memory such a sinner against truth as to credit his own lie by the telling of it So Antonio by repeatedly saying to himself and others that he was the Duke came to believe that he was really the Duke Thus falsehood repeatedly asserted gained the force of truth for Antonio and he truly believed it

Should presently extirpate me and mine Out of the dukedom and confer fair MilanWith all the honours on my brother whereonA treacherous army levied one midnightFated to thrsquo purpose did Antonio openThe gates of Milan and ithrsquo dead of darkness The ministers for thrsquo purpose hurried thenceMe and thy crying self

(i) In the earlier lines of this scene what does Prospero tell about his intense interest What was the demand of his interest

In the earlier lines of this scene Prospero tells Miranda that he had an intense interest in the study of philosophy and magic arts Hence in order to improve his mind with this kind of study he kept himself isolated from worldly and state affairs His study was dearer to him than the applause and esteem that he could win from the public His study demanded too much solitude(ii) What forced Antonio to take an undue advantage over Prospero

Prosperorsquos indifferent attitude towards the statersquos affairs and his having boundless trust in Antonio gave rise to a boundless lust for power in Antoniorsquos mind Antonio felt that he must be the actual Duke instead of the part of the Duke he played Thus Antonio took an undue advantage of the situation to usurp Prosperorsquos dukedom(iii) Explain the following lines ldquoI should sin to think but nobly of my grandmother Good wombs have borne bad sonsrdquo

After hearing the treacherous act of her uncle Antonio Miranda says that Prosperorsquos mother was a noble lady and she cannot dishonour her memory by saying that the person named Antonio cannot be his (Prosperorsquos) brother She says that in honour of her grandmother she also cannot say that Antonio must have been begotten not by her grandfather but by some other man She finally concedes that it is known that good mothers have borne bad sons in their wombs and gave birth to them

(iv) Why did the King of Naples accept Antoniorsquos request to help him in usurping his dukedom What did Antonio propose to Alonso

Alonso who was the king of Naples accepted Antoniorsquos request in usurping Prosperorsquos dukedom because he (Alonso) was a sworn enemy of Prospero Antonio proposed that Alonso should immediately drive him (Prospero) and his offspring out of Milan and should confer the dukedom upon him (Antonio) with all the dignities which go with that

position In return Antonio promised that he shall give an annual tribute and also swore his allegiance to Alonso Also he agreed to hold the Dukedom of Milan as a subordinate to the state of Naples(v) How were Prospero and Miranda carried away from the city of Milan and what was the state of small Miranda at that time

In pursuance of the agreement settled between Antonio and Alonso an army of treacherous men was assembled One midnight when the occasion suited the will of destiny Antonio opened the gates of the city of Milan and in the death like silence of midnight Antoniorsquos agents who had been directed to execute his purpose carried Prospero and small Miranda away from the city in all haste They were then forced into a ship and carried some distance out to sea where they put them on a mere hulk of a boat without any rigging or ship-gear and abandoned them leaving them at the mercy of the roaring sea Miranda was a very small child of three years age and she was crying at that time

CLASS -XIDATE-270420Subject Topic Summary Execution

EVS Chapter 1 ndash Mode of Existence

Impact of mode of existence on resources

Q) Why resources are under pressure

Ans - Increase in the sophistication

of technology enabling natural resources to be extracted quickly and efficiently Eg in the past it could take long hours just to cut down one tree only using saws Due to increased technology rates of deforestation have greatly increased

The number of humans is increasing Cultures of consumerism Materialistic views

lead to the mining of gold and diamonds to produce jewelry unnecessary commodities for human life or advancement Consumerism also leads to extraction of resources for the

production of commodities necessary for human life but in amounts excessive of what is needed because people consume more than is necessary or waste what they have

Lack of awareness among the population is striking People are not aware of ways to reduce depletion and exploitation of materials

Accounts Cash Book Today we are going to start a new topic -Cash Book

The key terms used in this chapter are

bullCash book

bullSimple cash book

bullDouble column cash book bullTriple column cash book

bullPetty cash book

bullCash discount

bullContra entry

Here I will share you the meaning of each key terms

bullCash book Cash Book is a special purpose subsidiary book or journal in which cash received and cash payments are recorded

bullSimple cash book

It is a cash book in which only cash transactions are recorded It has only one column on each side

bullTriple column cash book

It is cash book which has three columns one column for each cash and Bankdiscount on each side of the cash book In this book both cash and Bank transactions are recorded together with discount allowed and received

bullPetty cash book

It is a cash book maintained for recording petty expenses

bullCash discount

Cash discount is the amount of discount received or allowed on cash payments and cash receipts Discount received is an income for the business while discount allowed isan expense

bullContra entry

It means transactions involving both cash and Bank Such transactions though recorded in the cash book are not posted into ledger The letter lsquoC is written in Ledger folio for contra entry

Business Studies

ENTREPRENEURSHIP

Now we shall discuss the second chapter

lsquoENTREPRENEURSHIPrsquo

Today before starting the chapter let us recall what

Questions

1What are the main characteristics of Intrapreneurship

Answer

The main characteristics of Intrapreneurship are

Corporate framework-it occurs within the framework of the same company

Semi-Autonomous-Intrapreneurship

we have read last day

Let s today start the class by recalling the last topic taught

Intrapreneurship is the process of discovering and exploring business opportunities within an existing company It involves launching new business ventures within the framework of a present corporation Intrapreneurship is also known as corporate entrepreneurship or corporate venturing

Now let us start with the characteristics of Intrapreneurship

The main characteristics of Intrapreneurship are

Corporate framework

Semi-Autonomous Lack of ownership Senior position Low risk taking Not own boss

Now let us discuss the meaning of enterprise

Enterprise means an undertaking or adventure that requires some innovation and investment and thus involves riskEnterprise always entails decision making coordination and risk bearing

involves crating amd nurturing a semi-autonomous business unit which may be a subsidiary a strategic business unit or a division

Lack of ownership-the intrapreneur is not the owner of the unitb he creates and nurtures

Senior position-he occupies a senior managerial position in the company

Low risk taking-An intrapreneur does not bear the full risk of failure

Not own boss-An intrapreneur is not his own bosss in legal termsHe enjoys the freedom and gets the required resources and support

2 How is Entrepreneur is different from Intrapreneur

The functions involved in both the entrepreneurship and intrapreneurship are by and large similar however there are several differences between the two

Point of distinction

Entrepreneur

Intrapreneur

status An independent business person

A senior executive within a company

Ownership Owner of

the business

An employeesometimes a share in ownership

Financing Responsible for raising finance for the business

Not responsible for raising the finance

Risk bearing

Bears the risk of the business

Does not bears the risk of the business

Reward Profit which is uncertain and irregularcan be loss

Fixed salary and fringe benefits

Need for security low high

3 What do you understand by enterprise

Answer Enterprise means an undertaking or adventure that requires some innovation and investment and thus involves riskEnterprise always entails decision making coordination and risk bearing

COMMERCE NATURE AND OBJECTIVES OF

BUSINESS

Today let us recall the last other two objectives of business by the chart given in the previous class

Firstly we would discuss Human Objectives

Business is run by people and for people Labour is a valuable business element

Human objectives of business are concerned with the well -being of labour

The human objectives are as follows

Labour welfare Developing human

resources Participative

management Labour

management cooperation

Questions

1 Explain the human objectives of a business enterprise

Answer

Business is run by people and for people Labour is a valuable business element

Human objectives of business are concerned with the well -being of labour

The human objectives are as follows

Labour welfare-Business must recognize the dignity of labour and human factors should be given the recognition

Developing human resources-Employees must be provided the opportunities for developing new skills and attitudes

Participative management-Employees should be allowed to take part in decision making process of business

Labour management cooperation-Business should strive for creating and maintaining cordial employer employee relations so as to ensure peace and progress in industry

Now let us discuss the national objectives of business

Optimum utilization of resources

National self- reliance Development of small

scale industries Development of

backward areas Control over pollution

2Explain the national objectives of a business enterprise

Answer

It is the duty of business to utilize the resources of the country properly the national objectives of business

Optimum utilization of resources ndashBusiness should use the nationrsquos resources in the best possible manner

National self- reliance-It is the duty of the business to help the government in increasing experts and in reducing dependence on imports

Development of small scale industries-Big business firms are expected to encourage growth of small scale industries which are necessary for generating employment

Development of backward areas-Business is expected to give preference to the industrialization of backward regions of the country

ECONOMICS

BASIC ECONOMIC CONCEPTS

SUB

TOPIC

Value

Wealth

Welfare

Today we shall start with a new topic of the same chapter ie lsquoValuersquo

Value of a commodity is defined as the valuation placed by a household on the consumption of this commodity

lsquoValuersquo has two different meanings and these are

a Value -in -use It refers to consumption value of a commodity It expresses the utility derived from the consumption of a particular commodity A necessity like water has a very high value ndashin ndashuse or

Question

1What is value

Answer

Value of a commodity is defined as the valuation placed by a household on the consumption of this commodity

2What is value-in use

Answer It refers to consumption value of a commodity It expresses the utility derived from the consumption of a particular commodity A necessity like water has a very high value ndashin ndashuse or consumption value

3What is value ndashin- exchange

Answer It relates to market value of a commodity

It is the rate at which a particular good or service can be exchanged for money

For example in barter system if a person is prepared to exchange 3 metres of cloth with 1 pair

consumption value

b Value ndashin-exchange It relates to market value of a commodity

It is the rate at which a particular good or service can be exchanged for moneyFor example in barter system if a person is prepared to exchange 3 metres of cloth with 1 pair of shoes then the value in exchange of 3 metres of cloth is 1 pair of shoesValue in exchange is the power of purchasing other goods In modern monetised economies the exchange value of goods are expressed in terms of money as prices

Now let us discuss the term lsquoWealthrsquo

Wealth refers to the stock of all those assets which are a source of income

Wealth is a stock concept

Wealth must possess the following features

a Utility It must possess utility or give some

of shoes then the value in exchange of 3 metres of cloth is 1 pair of shoes

Value in exchange is the power of purchasing other goods In modern monetised economies the exchange value of goods are expressed in terms of money as prices

4 What is wealth

Answer it refers to the stock of assets or goods which are a source of income and have personal or national ownership

5 What are the features of wealth

Answer The features of wealth are as follows

Wealth must possess the following features

a Utility It must possess utility or give some satisfaction

b Scarcity It must be limited in quantityc Transferability it should be transferable its

ownership can be transferred from one person to another person

d Exchange value It must possess exchange value

6 What is welfare

Answer

Welfare is defined as satisfaction and happiness a sense of well- being among the people

satisfactionb Scarcity It must be

limited in quantityc Transferability It

should be transferable its ownership can be transferred from one person to another person

d Exchange value It must possess exchange value

Now let us discuss the term lsquoWelfarersquo

Welfare is defined as satisfaction and happiness a sense of well- being among the people

Welfare is affected by factors like

a Consumption of goods and services

b Environment

c Family relations

d Degree of freedom

e Law and order situation

Mathematics Trigonometric equation

To find the general solution of the equation sinθ=0

When sin θ =0

Then θ= 0 π2π 3π-π -2π -3

i e when θ = 0 or an integral multiple of π

i e when θ= nπ where n is any integer

Therefore the general solution of the equation sin

Example1 Find the general values of θ which satisfy the equation sin2 θ =34

Solution sin2 θ= 34

Or sin θ = +34 or -34

Or sin θ = sin π3 or sin (-π3)

Therefore

θ = [nπ + (-1) n (π3)] or[ nπ+ (-1) n (-π3)]

= nπ +π3 or nπ-π3 where n= any integer

Example 2Find the values of θ which satisfy tan2 θ

θ=0 is θ= nπ where n is any integer

To find the general solution of the equation cos θ=0

When cos θ=0

Then θ=π2 3π2 5π2 -π2 -3π2 -5π3

i e when θ is an odd multiple of π2

i e when θ=(2n+1) π2 where n= any integer

Therefore the general solution of the equation cos θ =0 is θ= (2n+1) π2 where n= any integer

To find the general solution of the equation tan θ = 0

Clearly tan θ =0 implies sin θcos θ =0

Therefore θ = nπ

i e the general solution of the equation tan θ=0 is θ =nπ where n = any integer

To find the general solution of the equation cot θ =0

Clearly cot θ =0 implies (cos θsin θ) = 0

i e cos θ =0

Therefore θ = (2n+1) π2

Therefore the general solution of the equation cot θ =0 is θ = (2n+1) π2

Where n= any integer

To find the general solution of the equation sin θ= k (-

=13 -πleθleπ

Solution tan2 θ =13

Or tan θ = plusmn1radic(3) =tan(plusmnπ6)

θ=nπ plusmn π6 where n =any integer

If n=0 then θ=plusmnπ6

If n=1 then θ= π plusmn π6

If n=-1 then θ= -π plusmn π6

Therefore the required solution in -π le θ le π are θ= π6 5π6 -π6 -5π6

Exercise Find general solution of sin 2θ=cos θ [Hints Use sin 2θ= 2sin θcosθ and then take cosθ

common]

1lek le1)

Determine an angle alpha such that sin =k and -π2le αle π2

Then we have

Sin θ = k = sin α

Or sin θ - sin α =0

Or 2 cos [(θ+α) 2] sin [(θ-α) 2] =0

Therefore either cos [(θ +α) 2] =0 (1)

Or sin [(θ-α) 2] =0 (2)

Now from (1) we get (θ+α) 2= (2m+1) π2)

Or θ = (2m+1) π-α (3)

And from (2) we get (θ-α) 2 =mπ

Or θ= 2mπ+α(4)

Where m = any integer

Clearly the solution (3) amp (4) may be combined in the following form

θ= nπ+(-1) n α where n= any integer

Therefore the general solution of sin θ = sin α is θ = nπ +(-1) n α where n is any integer and -π2 le α le π2

Biology Chapter - 04Kingdom Monera

Today we will discuss about bacterial reproduction and its usefulness

Fig Binary Fission

Fig Conjugation Fig Transformation

Bacterial reproduction is mainly asexual but sexual reproduction

also takes place

Asexual reproduction takes place by i) Binary fission - from one bacteriato

two bacteria are produced in every 20 to 30mins

ii) Buddingiii) Endospore formation - during

unfourable condition

Sexual reproduction by three ways

1) Conjugation - Transfer of genetic material between cells that are in physical contact with one another

2) Transduction - Transfer of genetic materialfrom one cell to another by a bacteriophage

3) Transformation - Transfer of cell-freeor naked DNArsquo from one cell to another

Bacteria causes different diseases inplants animals and human and

it causes food spoilage and waterpollution but it also have some useful

activities

i) Bacteria are helpful in sewage water treatment

ii) It is used in antibiotic (medicine) production

iii) Anaerobic bacteria help in biogas(energy) production

iv) Many household products like yoghurt cheese are manufactured by use of bacteria

v) Rhizobium by symbiotic relationship with leguminous plant increase soil fertility

vi) Besides these bacteria is helpful in genetic engineering degradation of petroleum hydrocarbonand in dairy

industry

Physics Motion in plane Here we will introduce Projectile Motion

Execution

Projectile

Y

usinθ u h

θX

ucosθ

Suppose a body is projected with an angle θ So initial velocity u can be resolved into two components

Horizontal component - ucosθ ( for range)

Vertical component - usinθ ( for height)

usinθ changes during motion and becomes zero at maximum height position but ucosθ remain unchanged

The maximum height of projectile is h

NB If initial is upward then g = -ve and if it is downward then g = +ve Height is +ve if direction of motion does not change ( for ex a body thrown upwards but goes down ultimately then height h = -ve)

The angle of projectile θ is the angle made with horizontal

HISTORY ndash GROWTH OF NATIONALISM

SUB TOPIC- REVOLUTIONARY NATIONALISM Bengal formation of Anushilan Samity and Jugantar Group

The intensification of the Swadeshi movement and Government policy of terror and repression led to outbreak of violence Bombs were manufactured and attempts on the lives of unpopular Government officials became frequent In the gymnasium of Scottish Church College which was known as General Assemblies Institution a secret society was formed known as Anushilan Samity

Aurobindo Ghosh send from Baroda his emissary Jatindranath Banerjee to mobilize the Bengal revolutionaries

Hemchandra Qanungo and Satyen Bose published Journal Jugantar

The Jugantar group planned to assassinate oppressive magistrate Kingsford by Khudiram Bose and Prafulla Chaki in 1908 Prafulla Chaki committed suicide to avoid arrest Khudiram was tried and hanged

Afew days later the police found a bomb factory in Maniktala and arrested a large number of revolutionaries The trial of revolutionaries became famous as the Alipore Bomb Case

In the course of the trial the approver the public prosecuter and a police officer were assassinated

1 Question Name two journals which preached the cult of violence

Answer a) Yugantar edited by Bhupendranath DuttaB) Bandemataram edited by Aurobindo Ghosh2 Question Why was Khudiram arrested and hangedAnswer An attempt was made to assassinate a hated vindictive majistrate named Kingsford by Khudiram Bose and Prafulla Chaki Their attempt failed and the bomb they threw killed two English ladies Khudiram was arrested and put to trial and then hanged3Question Who was Aurubindo GhoshAnswer Aurobindo Ghosh a nationalist revolutionary who was charged for his involvement in the Alipore Bombing Case He was accused of it along with his brother Barindra nath Ghosh But Aurobindo was acquitted because of the brilliant pleading of his counsel Chittaranjan Das Then he became a spiritual reformer introducing his visions on human progress and spiritual evolution4 Qustion Who was KingsfordAnswer Kingsford was an unpopular British chief Magistrate who was the target of the bomb thrown at Muzaffarpur by Khudiram and Prafulla Chaki

Most of the accused were convicted and sentenced to

long term of imprisonmentBut

Aurobindo Ghosh was acquitted mainly owing to the brilliant pleading of his counsel Chittaranjan Das

Political science Topic-Sovereignty

Summary Sovereignty is the full right and power of a governing body over itself without any interference from outside sources or bodies In political theory sovereignty is a substantive term designating supreme legitimate authority over some polity In international law sovereignty is the exercise of power by a state

Internal Sovereignty

Internal sovereignty means supreme authority within ones territory while external sovereignty relates to the recognition on the part of all states that each possesses this power in equal measure

External sovereignty

external sovereignty relates to the recognition on the part of all states that each possesses this power in equal measure

Distinguish between

Execution

Answer the following questions

Short notes-

Sovereignty

Internal Sovereignty

External sovereignty

Homework- learn

external sovereignty and internal sovereigntySovereignty is the principle

of supreme and

unquestionable authority

reflected in the claim by the

state to be the sole author of

laws within its territory

Definition of external vs internal sovereigntyInternal sovereignty refers to

the relationship between a

sovereign power and its

subjects ndash it refers to the

location of the supreme

authority within the state In

the UK for example internal

sovereignty (supposedly)

resides within Parliament

reflected in the

constitutional principle of

parliamentary

sovereigntyBy contrast

external sovereignty refers

to the capacity of the state

to act independently and

autonomously on the world

stage This is what is

sometimes called lsquostate

sovereigntyrsquo or lsquonational

sovereigntyrsquo and implies

that states are legally equal

and that the territorial

integrity and political

independence of a state is

inviolable

Class ndash XII

Date - 2742020 STUDY MATERIALSubject Topic Summary Execution Business Studies

Job Analysis amp Manpower Planning

At first let us recall the chapter what we have discussed till nowbullJob analysisbullJob specification bullJob description bullJob enlargement bullJob enrichment

Today we will do some questions answers from the chapter

Questions 1ldquoJob analysis job description and job specification are interrelatedrdquo Comment Answer) Job analysis is a systematic and detailed examination of a job to collect all the relevant information about it The contents off the job are summarised in the job description The qualification needed for the job are summarised in job specificationThus there is close interrelationship between job analysis job description and job specification

Question 2ldquoJob enlargement is a horizontal extension of a job whereas job enrichment is a vertical extension of a jobrdquo ElucidateAnswer) Job enlargement involves adding one or more task to a job coma where as job enrichment involves adding more autonomy and responsibility to a job Job enlargement is therefore horizontal extension of a job coma whereas job enrichment is a vertical extension of a job

Question 3 )

What is manpower estimation Explain its quantitative and qualitative aspectsAnswer) Manpower estimation is the process by which management determines how an organisation should move from itrsquos current manpower positionto its desired manpower position There are two dimensions of Manpower estimation- quantitative and qualitative

Quantitative aspectThis aspect of Manpower estimation involves estimating the number of employees required in a future time period Workload analysis and workforce analysis are done to estimate the quantity of required manpower

Qualitative aspectThe estimate of the knowledge skills experience etc of required manpower is the qualitative aspect of Manpower estimation The quality of Manpower can be judged on the basis of job analysisand job specification

COMMERCE

CAPITAL-FIXED AND WORKING

Today let us start the class by discussing the sources of finance for different types of business firms

The term lsquocapitalrsquo refers to the investment made in the enterprise for the purpose of earning profits

Requirements of capital and sources of capital for different types of business firms are

1 Capital for sole proprietorship businessA sole proprietor operates at a small scale and thereforerequires a limited amount of capital

2 Capital for partnership firmCapital requirements as well as capital base of a partnership is bigger than that of a sole trader businessThe owned capital is contributed by the partners in an agreed ratio

3 Capital for joint stock companyA joint stock company generally requires large amount of capitalA public company can raise huge capital through issue of shares In addition to share capital it can utilize retained profits

Now let u discuss the meaning of Finance PlanningFinance planning is the process of estimation the financial requirements of an organization specifying the sources of firms and ensuring that enough funds are available at the right time

1 What do you mean by Finance PlanningAnswer Finance planning is the process of estimation the financial requirements of an organization specifying the sources of firms and ensuring that enough funds are available at the right time

2Discuss the role of financial planning of an enterpriseThe role of financial planning are as followsa A sound financial plan helps a business enterprise to avaid the problems of shortage and surplus of fundsbFinancial planning serves as a guide in developing a sound capital structure so as to maximize returns to shareholders c It helps in effective utilization of fundsd It provides policies and procedures for coordinating different functional areas or departments of businesse It enables the management to exercise effective control over the financial activities of an enterprisef It helps the company to prepare for facing business shocks and surprises in future

Mathematics

Continuity and differentiability

Recall Definition of ContinuityLet f(x) be a single valued function of x and x=a be a point in the domain of definition of the function The function is said to be continuous at x=a ifi) f(c) is defined ie f(x) has a definite finite value at x=cii) lim xrarra f(x) exists andiii) lim xrarra f(x) =f(a) In other words f(x) is said to be continuous at x=a if lim xrarra+ f(x)= lim xrarra- f(x) = f(a) Or f(a+0) =f(a-0) =f(a) Or lim hrarr0 f(a+h)= f(a) Algebra of continuous functionsNow we will study some algebra of continuous functions Theorem 1 Suppose f and g be two real functions continuous at a real number c Then(1) f + g is continuous at x = c(2) f ndash g is continuous at x = c(3) f g is continuous at x = c(4) (fg) is continuous at x = c (provided g (c) ne 0)

Example 1 Prove that every rational function is continuousSolution Recall that every rational function f is given byf(x)=[p(x) q(x) ] q(x)ne0where p and q are polynomial functions The domain of f is all real numbers except points at which q is zero Since polynomial functions are continuous f is continuous by (4) of Theorem 1Example 2Discuss the continuity of sine functionSolution To see this we use the following factslim xrarr0 sin x =0Now observe that f (x) = sin x is defined for every real number Let c be a real number Put x = c + h If x rarr c we know that h rarr 0 Therefore lim xrarrc f(x)

= lim xrarrc sin x= lim hrarr0 sin(c+h) =lim hrarr0 [sin c cos h + cos c sin h ]=lim hrarr0 (sin c cos h) + lim hrarr0 (cos c sin h) = sin c +0=sin c = f(c) Thus lim xrarrc f(x) = f(c) and hence f is a constant function Exercise Prove that the function f(x) = x2 +2x is continuous for every real value of x [Hints show that lim xrarra+ f(x) = lim xrarra- f(x) = f(a) ]

Biology Reproduction in Flowering plants We will discuss about megasporoangium

megasporagenesis and female gametophyte

Q4 Describe the structure of megasporangium

Ovule is attached to the placenta by astalk called funicle

Each ovule has one two or three protectivecoverings called integuments

At the tip of integuments a small openingcalled micropyle is organised

Opposite to the micropylar end is the chalaza

Within the integuments a mass of cellsnucellusand inside it embryo sac orfemale gametophyte is present

Q5 Describe a mature embryosacamp its formation

In most of the flowering plants only oneof the 4 megaspores formed as a result ofmegasporogenesis that is functional while theother three degenerate

The

functional megaspore develops into thefemale gametophyte

Formation The nucleus

of the functional megasporedivides mitotically to form two nuclei first andthen two more sequential mitotic nucleardivisions result in the formation of four ampthen eight nucleate stages of embryo sac

Six of the eight nucleus are surrounded bycell walls and organised into cells

The remaining two nuclei called polar nuclei are found below the egg apparatus in the largecentral cell

Three cells consisting of two synergids amp one egg cell present bottom of

embryo sac Three cells

at the chalazal as antipodal cells

Two polar nuclei together present in large central cell

HISTORY

TOWARDS INDEPENDENCE AND PARTITION THE LAST PHASE(1935-1947)SUB TOPIC NATIONAL MOVEMENTS DURING THE SECOND WORLD WAR

Spread of Quit India Movement On 9th August 1942Gandhiji and other Congress leaders were arrested The Congress was declared illegal The news of the arrest of all leaders marked the beginning of a widespread movement of India It was not possible for such a movement to remain peacefulBut the arrest of the all notable congress leaders virtually left the movement in the hands of the mass The movement took the form of violent and militant outbreakBesides congressmen revolutionaries also were very active in the movement The Congress Socialist group also played a prominent role

1 Question Why did the British authority arrest the Congress leaders on 9 th August 1942Answer Congress Working committee adopted the Quit India resolution which was to be ratified at the Bombay AICC meeting in 8th August 1942 They decided to launch a mass struggle on non-violent lines Gandhiji gave a clarion call to all section of the people rdquoKarenge ya Marengerdquo (do or die) Congress leaders gave the call to driving out

the British from IndiaViceroy had taken strong action against the Quit India movement Gandhiji and all the leaders of Congress were arrested

2 Question How did Quit India Movement spread out all over IndiaAnswer The news of the leaders lsquo arrest marked the beginning of a widespread movement to remain peacefulThe movement took form of violent outbreak There were widespread cutting of telephone and Telegraph wires damaging railway lines raising barricades in cities and towns and other forms of violent demonstations

Question Name the leaders of Congress

Socialist group played a prominent part Notable among the Jayprakash Narayan Rammonohar Lohia Aruna Asaf Ali

Political science

Topic-Franchise and Representation

Summary

The election commission

The Election Commission of India is an autonomous constitutional authority responsible for administering Union and State election processes in India The body administers elections to the Lok Sabha Rajya Sabha State Legislative Assemblies in India and the offices of the President and Vice President in the country

Functions of election commission-

India is a sovereign socialist secular democratic republic Democracy runs like a golden thread in the social economic and political fabric woven by the Constitution given by lsquoWe the People of Indiarsquo unto ourselves The concept of democracy as visualised by the Constitution pre-supposes the representation of the people in Parliament and State legislatures by the method of election The Supreme Court has held that democracy is one of the inalienable basic features of the Constitution of India and forms part of its basic structure The Constitution of India adopted a Parliamentary form of government Parliament consists of the President of India and the two Houses mdash Rajya Sabha and Lok Sabha India being a Union of states has separate state legislatures for each state State legislatures consist of the Governor and two Houses mdash Legislative Council and Legislative Assembly mdash in seven states namely Andhra Pradesh Telangana Bihar Jammu amp Kashmir Karnataka Maharashtra and Uttar Pradesh and of the Governor and the state Legislative Assembly in the remaining 22 states Apart from the above two out of the seven Union Territories namely National Capital Territory of Delhi and Puducherry also have their Legislative Assemblies

ExecutionShort notes-Election commissionFunctions of election commission

Homework- Learn

Computer

Science

Computer hardware NAND Gate

A NOT-AND operation is known as NAND operation It has n input (n gt= 2) and one output

Logic diagram

Truth Table

NOR Gate

A NOT-OR operation is known as NOR operation It has n input (n gt= 2) and one output

Logic diagram

Truth Table

XOR Gate

XOR or Ex-OR gate is a special type of gate It can be used in the half

adder full adder and subtractor The exclusive-OR gate is abbreviated as EX-OR gate or sometime as X-OR gate It has n input (n gt= 2) and one output

Logic diagram

Truth Table

XNOR Gate

XNOR gate is a special type of gate It can be used in the half adder full adder and subtractor The exclusive-NOR gate is abbreviated as EX-NOR gate or sometime as X-NOR gate It has n input (n gt= 2) and one output

Logic diagram

Truth Table

Physics

Chapter 1 Electric Field ( Electric Dipole) (Summary)

Here we will derive Expression of electric field at broad side

On position of dipole

Execution

Q With the help of a labelled diagram obtain an expression for the electric field intensity E at any point on the equitorial line ( broad-side on position) of an electric dipole

Ans

E1 E1sinθ

E θ P E1 θ

( r2+L2)12 E2 E

r E2 E2sinθ

-q θ L O L +qA B

Let us consider that the point P is situated on the right bisector of the dipole AB at a distance r meter from its midpoint O

Let E1 and E2 be the electric field intensities of the electric field at P due to charge +q and ndashq of the dipole resp The distance of P from each charge is ( r2+L2)12

So E1 = 14 πϵ q

(r 2+L 2) away from +q

E2 = 14 πϵ q

(r 2+L 2) towards ndashq

The magnitudes of E1 and E2 are equal but directions are different Now resolving E1 and E2 into two components parallel and perpendicular to AB we get

The components perpendicular to AB E1sinθ and E2sinθ cancel each other because they are equal and opposite

The components parallel to AB are E1cosθ and E2 cosθ are in same direction and add up

So resultant intensity of electric field at the point P is

E = E1cosθ + E2 cosθ

E = 14 πϵ q

(r 2+L 2) 2 cosθ

Now from fig we have cosθ =BOBP = L (r2+L2)12

So we get E = 14 πϵ 2qL ( r2+L2)32

Now electric dipole moment p= 2qL

So E = 14 πϵ p ( r2+L2)32

HW Find the expression of Electric field as done here but this time take r gtgt 2L

Also find the expression of torque experience by a dipole

(Hint Electric force experienced by charges of dipole in electric field is qE each Let θ be the angle which dipole makes with electric lines of force then perpendicular distance between two charges is 2Lsinθ Then torque = force x perp distance = qE x 2L sinθ So τ=pE sinθ where p =2qL )

STUDY MATERIAL

Class XIISubject Eng Literature (The Tempest ndash William Shakespeare) Topic Act IV Scene 1 Lines 84 to 133 (Iris hellip A contract of true love Be not too late ) Date 27th April 2020 (4th Period)

[Students should read the original play and also the paraphrase given in the school prescribed textbook]Summary Questions amp Answers

o Ceres soon appears and comes to know that she has been summoned to celebrate the contract of true love

o Ceres expresses her unwillingness to meet Venus and Cupid as she has shunned their company

o Ceres and Juno both bestow their blessings upon Ferdinand and Miranda with June gifting honour riches happiness in marriage and Ceres presents plenty of earthrsquos produce

o Iris summons the water-nymphs and reapers to come and celebrate a contract

(1) IRIS Of her society (Line 91-101)

Be not afraid I met her deity

Cutting the clouds towards Pathos and her sonDove-drawn with her Here thought they to have doneSome wanton charm upon this man and maidWhose vows are that no bed-right shall be paidTill Hymens torch be lightedmdashbut in vainMarss hot minion is returned againHer waspish-headed son has broke his arrowsSwears he will shoot no more but play with sparrowsAnd be a boy right out

(i) Where were Venus and Cupid seen flying How were they travelling Why did they want to join the marriage celebration of Ferdinand and Miranda

of true love

Venus and Cupid were seen flying through the air towards Paphos the famous city which is situated on the island of Cyprus They were travelling by air-borne chariot drawn by doves They certainly wanted to come here in order to play some amorous trick upon Ferdinand and Miranda who are under a vow not to gratify their physical desires till the holy ceremony of their marriage has been performed(ii) What have Venus and Cupid done after failing in their plan

After being failure of their plan Venus who is a very passionate deity and who is the mistress of Mars (the god of war) has gone back while here ill-tempered son Cupid has broken his arrows of love in his state of desperation(iii) What has Cupid firmly decided

Cupid is feeling so disappointed that he has firmly decided to shoot no more arrows to arouse love in human hearts but to spend his time playing with sparrows Thus he would now become just a boy and would give up his original function of shooting arrows on human beings to make them fall in love(iv) What vow had Ceres taken How did Ceres feel at the abduction

After the abduction of her daughter Prosperina by Pluto Ceres had taken a vow to always keep away from the disgraceful company of Venus and her blind son Cupid the god of love Ceres felt deeply distressed when Pluto had carried off her daughter and had made her his wife by force(v) Why has Ceres not forgiven Venus and her blind son For what do Ceres want to be sure

As the abduction had been manipulated by Venus the goddess of beauty and love and her blind son Cupid Ceres has never forgiven them for their part in the whole plot Ceres wants to be sure that she would not have to meet Venus and Cupid who had engineered the abduction of her daughter Prosperina

AS THIS lsquoMASQUErsquo SCENE IS VERY IMPORTANT IN THE PLAY THE PARAPHRASE OF THE ENTIRE PORTION OF MASQUE SCENE (Act IV Lines 58 to 143) IS GIVEN BELOW

IRIS Goddess of RainbowCERES Goddess of Agriculture and all the fruits of the earth

(Nature growth prosperity rebirth ndash notions intimately connected to marriage)JUNO The majestic Queen of Heavens and wife of Jupiter (Jupiter is the king of Gods)

VENUS The Goddess of love CUPID Son of Venus PLUTO God of death (In the play referred by Shakespeare as lsquoDisrsquo which is a Roman name for Pluto)

ORIGINAL TEXT PARAPHRASEPROSPEROWellmdash

PROSPERONow come Ariel Let there be too many rather than too few

Now come my Ariel Bring a corollaryRather than want a spirit Appear and pertly[to Ferdinand and Miranda]No tongue all eyes Be silent

spirits in attendance Appear briskly

[to Ferdinand and Miranda]Look with your eyes but do not say a word

[Soft music] [Soft music][Enter Iris] [Enter Iris]

IRISCeres most bounteous lady thy rich leasOf wheat rye barley vetches oats and peasThy turfy mountains where live nibbling sheepAnd flat meads thatched with stover them to keepThy banks with pioned and twilled brimsWhich spongy April at thy hest betrimsTo make cold nymphs chaste crowns and thybroom-grovesWhose shadow the dismissegraved bachelor lovesBeing lass-lorn thy pole clipped vineyardAnd thy sea-marge sterile and rocky-hardWhere thou thyself dost airmdashthe Queen othrsquoSkyWhose watery arch and messenger am IBids thee leave these and with her sovereign grace[Juno appears] Here on this grass-plot in this very placeTo come and sport Her peacocks fly amainApproach rich Ceres her to entertain

IRISCeres most generous lady you are the cause of rich fields or fertile land where wheat rye barley beans oats and peas grow the grassy mountains where the sheep graze and the flat meadows covered with coarse hay to be used as fodder for cattleYour banks are covered with marsh-marigolds and reeds and the rainy April under your orders brings forth to make for the maids who are not in love beautiful crowns your woods where the broom flourishes and where the bachelor who has been dismissed by the maid he loved lies down being forsaken your vineyard in which the poles are embraced by the vines and the margin of the sea which is barren and rocky where you roam about to enjoy the fresh air ndash the queen of the sky (Juno) whose messenger I am besides being represented as the rainbow bids you leave all these and with her majesty here on this grassy plot in this very place come and sport her peacocks carry her fast in her chariot through the air and are making their way here approach rich Ceres to welcome her

[Enter Ariel as Ceres] [Enter Ariel as Ceres]

CERESHail many-coloured messenger that neerDost disobey the wife of JupiterWho with thy saffron wings upon my flowersDiffusest honey-drops refreshing showersAnd with each end of thy blue bow dost crownMy bosky acres and my unshrubbed downRich scarf to my proud earth Why hath thy queenSummoned me hither to this short-grassed green

CERESWelcome rainbow that never dared disobey Juno the wife of Jupiter who with your orange coloured rays spread honey-drops refreshing showers And with each end of thy blue bow drown my bushy acres and my hilly country which is free from shrubs you thus forming a rich scarf Why has your queen called me here to this place covered with short grass

IRISA contract of true love to celebrateAnd some donation freely to estateOn the blest lovers

IRISI have called you to celebrate a contract of true love and bestow some liberal gift upon the blessed lovers

ORIGINAL TEXT PARAPHRASECERESTell me heavenly bowIf Venus or her son as thou dost knowDo now attend the queen Since they did plotThe means that dusky Dis my daughter gotHer and her blind boys scandaled companyI have forsworn

CERESTell me heavenly bow if Venus the Goddess of love or Cupid her son and pedlar of passion at this time attend the heavenly queen Juno because you are sure to know Since the day they conspired against me and dark Pluto took away my daughter here and Cupidrsquos disgraceful company I have left off

IRISOf her societyBe not afraid I met her deityCutting the clouds towards Pathos and her sonDove-drawn with her Here thought they to have doneSome wanton charm upon this man and miad

IRISBe not afraid of her company I met her deity moving on the clouds towards Paphos the sacred home of Venus on the island of Cyprus along with her son on her chariot drawn by doves Here they contemplated to exercise a charm upon this man and maid producing

Whose vows are that no bed-right shall be paidTill Hymens torch be lightedmdashbut in vainMarss hot minion is returned againHer waspish-headed son has broke his arrowsSwears he will shoot no more but play with sparrowsAnd be a boy right out

wantonness before the actual marriage ceremony but did not succeed Venus has returned her irritable son has broken his arrows and swears that he will give up his practice of trying to inspire love but play with sparrows and be a boy again

[Music is heard] [Music is heard]

CERESHighst queen of stateGreat Juno comes I know her by her gait

CERESHighest queen of state Great Juno there she comes I know here by her gait

[Enter Juno] [Enter Juno]

JUNOHow does my bounteous sister Go with meTo bless this twain that they may Prosperous beAnd honoured in their issue

JUNOHow are you doing my generous sister Come with me to bless this couple so that they may be prosperous and fortunate in their children

[They sing] [They sing]

JUNOHonour riches marriage-blessingLong continuance and increasingHourly joys be still upon youJuno sings her blessings upon you

JUNOMay honour riches happiness in marriage long continuance and increase of those boons ever rest upon you as hourly joys Juno showers down upon you her blessings in song

CERESEarths increase foison plentyBarns and garners never emptyVines and clustring bunches growingPlants and goodly burden bowingSpring come to you at the farthestIn the very end of harvestScarcity and want shall shun youCeresrsquo blessing so is on you

CERESMay you have the plenty of earthrsquos produce Your barns and granaries may never be empty Your vines may grow with clustering bunches Your fruit trees may be heavily laden with their fruit May there be continuous spring and harvest May scantiness and want leave you forever Such is the blessing of Ceres upon you

FERDINANDThis is a most majestic vision andHarmoniously charmingly May I be boldTo think these spirits

FERDINANDThis is a great vision and magically melodious Should I suppose the characters (taking part in the masque) are spirits

PROSPEROSpirits which by mine artI have from their confines calld to enactMy present fancies

PROSPEROYes they are spirits whom I have summoned from the regions to which they are confined to carry into effect my fanciful designs

ORIGINAL TEXT PARAPHRASEFERDINANDLet me live here everSo rare a wondered father and a wifeMakes this place paradise

FERDINANDI should like to live here forever Such a wise and wonderful father makes this place a paradise

[Juno and Ceres whisper and send Iris on employment] [Juno and Ceres whisper and send Iris on employment]

PROSPEROSweet now silence

PROSPEROMy dear Ferdinand speak no more Juno and Ceres are

Juno and Ceres whisper seriouslyTheres something else to do Hush and be muteOr else our spell is marred

whispering with a solemn look There is something else coming Silence Or else our magic will be spoilt

IRISYour nymphs called naiads of the wandering brooksWith your sedged crowns and over-harmless looksLeave your crisp channels and on this green landAnswer your summons Juno does commandCome temperate nymphs and help to celebrateA contract of true love Be not too late

IRISYou nymphs called Naiads denizens (M inhabitants) of the running stream with your chaplets of sedge and ever-helpful looks leave your wrinkled channels and on the green land answer the summons sent to you Juno has ordered some chaste nymphs and help to celebrate a noble and true marriage Donrsquot delay

[Enter certain nymphs] [Enter certain nymphs]You sunburnt sicklemen of August wearyCome hither from the furrow and be merryMake holiday your rye-straw hats put onAnd these fresh nymphs encounter every oneIn country footing

You sunburnt harvesters weary from the effects of the heat in August come here from the furrowed land and rejoice Make holiday with your rye-straw hats upon you and meet these fresh nymphs and join in country dancing

[Enter certain reapers properly habited They join with the nymphs in a graceful dance towards the end whereof Prospero starts suddenly and speaks]

[Enter certain reapers properly habited They join with the nymphs in a graceful dance towards the end whereof Prospero starts suddenly and speaks]

PROSPERO[aside] I had forgot that foul conspiracyOf the beast Caliban and his confederatesAgainst my life The minute of their plotIs almost come [to the spirits]Well done Avoidno more

PROSPERO(Aside)I had forgotten the wicked conspiracy of the beast Caliban and his accomplices against my life the time of their plot has almost arrived ndash (To the Spirits) well done depart no more of this

[To a strange hollow and confused noise the spirits heavily vanish]

[The spirits depart]

ORIGINAL TEXT PARAPHRASEFERDINANDLet me live here everSo rare a wondered father and a wifeMakes this place paradise

FERDINANDI should like to live here forever Such a wise and wonderful father makes this place a paradise

[Juno and Ceres whisper and send Iris on employment] [Juno and Ceres whisper and send Iris on employment]

PROSPEROSweet now silenceJuno and Ceres whisper seriouslyTheres something else to do Hush and be muteOr else our spell is marred

PROSPEROMy dear Ferdinand speak no more Juno and Ceres are whispering with a solemn look There is something else coming Silence Or else our magic will be spoilt

IRISYour nymphs called naiads of the wandering brooksWith your sedged crowns and over-harmless looksLeave your crisp channels and on this green landAnswer your summons Juno does commandCome temperate nymphs and help to celebrateA contract of true love Be not too late

IRISYou nymphs called Naiads denizens (M inhabitants) of the running stream with your chaplets of sedge and ever-helpful looks leave your wrinkled channels and on the green land answer the summons sent to you Juno has ordered some chaste nymphs and help to celebrate a noble and true marriage Donrsquot delay

[Enter certain nymphs] [Enter certain nymphs]You sunburnt sicklemen of August wearyCome hither from the furrow and be merryMake holiday your rye-straw hats put onAnd these fresh nymphs encounter every oneIn country footing

You sunburnt harvesters weary from the effects of the heat in August come here from the furrowed land and rejoice Make holiday with your rye-straw hats upon you and meet these fresh nymphs and join in country dancing

[Enter certain reapers properly habited They join with the nymphs in a graceful dance towards the end whereof Prospero starts suddenly and speaks]

[Enter certain reapers properly habited They join with the nymphs in a graceful dance towards the end whereof Prospero starts suddenly and speaks]

PROSPERO[aside] I had forgot that foul conspiracyOf the beast Caliban and his confederatesAgainst my life The minute of their plotIs almost come [to the spirits]Well done Avoidno more

PROSPERO(Aside)I had forgotten the wicked conspiracy of the beast Caliban and his accomplices against my life the time of their plot has almost arrived ndash (To the Spirits) well done depart no more of this

[To a strange hollow and confused noise the spirits heavily vanish]

[The spirits depart]

Ac-12 27420 topic Revaluation of Assets and Liabilities

REVALUATION OF ASSETS AND LIABILITIES

On admission of a new partner the firm stands reconstituted and consequently the assets are revalued and liabilities are reassessed It is necessary to show the true position of the firm at the time of admission of a new partner If the values of the assets are raised gain will increase the capital of the existing partners Similarly any decrease in the value of assets ie loss will decrease the capital of the existing partners For this purpose alsquoRevaluation Accountrsquo is prepared This account is credited with all increases in the value of assets and decrease in the value of liabilities It is debited with decrease on account of value of assets and increase in the value of liabilities The balance of this account shows a gain or loss on revaluation which is transferred to the existing partnerrsquos capital account in existing profit sharing ratioAccounting for Revaluation of Assets and Liabilities when there is a Changein the Profit Sharing Ratio of Existing PartnersAssets and liabilities of a firm must also be revalued at the time of change in profit sharing ratio of existing partners The reason is that the realisable or actual value of assets and liabilities may be different from those shown in the Balance Sheet It is possible that with the passage of time some of the assets might have appreciated in value while the value of certain other assets might have decreased and no record has been made of such changes in the books of accounts Similarly there may be some unrecorded assets amp libilities that may have to be accounted for Revaluation of assets and reassessments of liabilities becomes necessary because the change in the

value of assets and liabilities belongs to the period to change in profit sharing ratio and hence must be shared by the partners in their old profit sharing ratio Revaluation of assets and reassessment of liabilities may be given effect to in two different ways (a) When revised values are to be recorded in the books and(b) When revised values are not to be recorded in the books

When revised values are to be recorded in the booksIn such a case revaluation of assets and reassessment of liabilities is done with the help of a new account called lsquoRevaluation Accountrsquo Sometimes this account is also called as lsquoProfit amp Loss Adjustment Acrsquo If there is a loss due to revaluation revaluation account is debited and if the revaluation results in a profit the revaluation account is credited The following journal entries made for this purpose are

(i) For increase in the value of assetsAsset Ac Dr (individually)To Revaluation Ac(ii) For decrease in the value of AssetRevaluation Ac Dr (individually)To Asset Ac[Decrease in the value of assets](iii) For increase in the value of LiabilitiesRevaluation Ac Dr (individually)To Liabilities Ac[Increase in the value of Liabilities](iv) For decrease in the value of LiabilitiesLiabilities Ac DrTo Revaluation Ac[Decrease in the value of Liabilities](v) For unrecorded AssetsAsset Ac [unrecorded] DrTo Revaluation Ac[Unrecorded asset recorded at actual value](vi) For unrecorded Liability Revaluation Ac DrTo Liability Ac [unrecorded][Unrecorded Liability recorded at actual value](vii) For transfer of gain on revaluationRevaluation Ac DrTo Existing Partnerrsquos CapitalCurrent Ac[Profit on revaluation transferred to capital account in existing ratio](viii) For transfer of loss on revaluationExisting Partnerrsquos CapitalCurrent Ac DrTo Revaluation Ac[Loss on revaluation transferred to capital account in existing ratio](a) When revaluation account shows gain Revaluation Ac DrTo Partnerrsquos Capital Ac (Old Profit Sharing Ratio)(Profit on revaluation credited to Partnerrsquos Capital Ac)(b) Above entry is reversed when revaluation account shows loss Partners Capital Acs (Old Profit Sharing Ratio) DrTo Revaluation Ac(Loss on revaluation debited to Partnerrsquos Capital Acs)

Proforma of Revaluation Account is given as under

Revaluation Account

Dr Cr Particulars ` Amount Particulars ` Amount To Decrease in value of assets By Increase in value of assets To Increase in value of liabilities By Decrease in value of liabilities To Unrecorded liabilities By Unrecorded assets To Gain on Revaluation (Transferred) By Loss on Revalution (Transferred)

ECO ndash12 2742020Topic- ELASTICITY OF DEMAND

CHAPTER - ELASTICITY OF DEMANDMEANINGDemand for a commodity is affected by many factors such as its price price of related goods income of its buyer tastes and preferences etc Elasticity means degree of response Elasticity of demand means degree of responsiveness of demand Demand for a commodity responds to change in price price of related goods income etc So we have three dimensions of elasticity of demandDIMENSION OF ELASTICITY OF DEMAND TYPES OF ELASTICITY OF DEMAND

Price elasticity of demand Income elasticity of demand Cross Elasticity of demand

Price elasticity of demand Price elasticity of demand means degree of responsiveness of demand for a commodity to the change in its price For example if demand for a commodity rises by 10 due to 5 fall in its price Price elasticity of demand (ep)=Percentage change in quantity demanded Percentage change in price of the commodity = 10 ( -)5 = ( - )2Note that ep will always be negative due to inverse relationship of price and quantity demanded

(ii) Income elasticity of demand Income elasticity of demand refers to the degree of responsiveness of demand for a commodity to the change in income of its buyer Suppose income of buyer rises by 10 and his demand for a commodity rises by 20 then Income elasticity of demand (ey)= change in quantity demanded change in price of the commodity =20 10 = 2

Cross Elasticity of demandCross elasticity of demand means the degree of responsiveness of demand for a commodity to the change in price of its related goods (substitute goods or complementary goods) Suppose demand for a commodity rises by 10 due to 5 rise in price of its substitute good then Cross elasticity of demand (ec) = change in quantity demanded change in price of related good = 10 2 = 5 (Tastes and preferences cannot be expressed numerically So elasticity ofdemand cannot be numerically expressed)

  • Chapter 1 Force (Summary)
  • Distinguish between external sovereignty and internal sovereignty
    • NAND Gate
      • Logic diagram
      • Truth Table
        • NOR Gate
          • Logic diagram
          • Truth Table
            • XOR Gate
              • Logic diagram
              • Truth Table
                • XNOR Gate
                  • Logic diagram
                  • Truth Table
                      • Physics
                      • Chapter 1 Electric Field ( Electric Dipole) (Summary)
Page 13:  · Web viewWe all know that Nouns are divided into two parts: common noun and proper noun.Apart from common and proper noun, we will also study about collective noun and compound

In a multi-tasking operating system it determines the order and time to be allowed for each application before giving another application a turn

It sends messages to the system operator about the status of operation or any error that may have occurred while running the application

What is operating system

An operating system is an integrated system of programs that manages various resources and the overall operation of the computer system It is designed to support various activities of computer system in a systematic way

Role of an operating system-

Operating system enables the user to use the system effectively An operating system manages various application that runs on a computer and shares computerrsquos resources User interacts with operating system through command line interface and graphical user interface

Function of an operating system -

Booting the computer it is the start up procedure of a computer system Loading theprograms in the memory - when system is ready the operating system loads certain program

automatically Manages resources it manages between the hardware and software resources Detecting and correcting errors- if the supporting hardware or software doesnrsquot works properly then the

operatingsystem tries to rectify it Ensuring data security programs and data donot interfere with each other Maintaining the internal clock of system - maintain internal clock of system when system is shut down

SUBJECT-GEOGRAPHY CHAPTER 1 REPRESENTATION OF GEOGRAPHICAL FEATURES THROUGH CONTOURSTopographical maps- topographical maps are small-scale maps with detailed depiction of both natural and human-made features

Contour lines- contour lines are imaginary lines drawn on a map joining places having the same height above the mean sea level

Contour linesContour interval- It is an interval at which contour lines are drawnIndex contour- At every 100-m interval a thick brown contour line is called index contour It is drawn for calculating height

Spot heights- The spot heights show heights in metres above the mean sea level

SUBJECT- English Language CHAPTER - The Sentence

Complex sentence-

We have seen that a Complex Sentence consists of a Principal Clause with one or more Subordinate Clauses

We have also learnt that there are three kinds of Subordinate Clauses The Adjective Clause the Adverb Clause and the Noun Clause

1) THE ADJECTIVE CLAUSE

An ADJECTIVE CLAUSE does the work of an ADJECTIVE It qualifies a noun or pronoun In the following examples in each set two commonly patterned sentences are compared one with an ADJECTIVE and the other with an ADJECTIVE CLAUSE ( The Adjective or Adjective Clause qualifies the Noun placed in the box)

a) He is a lazy boy (Adjective)

b) He is a boy who is lazy (Adjective Clause)

a) It is a beautiful place for the tourists (Adjective)

b) It is a place where tourists come for the scenic beauty (Adjective Clause)

a) We have enough funds for the work (Adjective)

b) We have funds which would be enough for the work (Adjective Clause)

Convert the sentence from simple to complex sentence-

1) We believe his honesty Ans- We believe that he is honest

2) This is the birth place of RamaAns- This is the place where Rama was born

____________________________________________________________________________________________

Class IX

Subject English Language

Topic Preposition

Date 270420

PREPOSITIONS

A preposition is a word placed before a noun or a pronoun to show in what relation the person or thing denoted by it stands in regard to something else

EXAMPLE

There is a clock on the wall

The preposition on shoes relationship between clock and the wall

Smitha is afraid of lizards The man jumped off the bus

Here the preposition of shows the relationship between afraid and lizards

The preposition off shows the relation between jumped and bus

The noun or the pronoun which follows a preposition is called its object

So in the first Example wall is the object of the preposition on

A preposition can have more than one object For example

The plane flew overhouses and meadows

Prepositions are used to express a number of relationship including time locationmannermeans quantity purpose and state or condition

Points to remember

Preposition joins a noun to another noun or a pronounThere is a cow in the field

A preposition joints a noun to a verbThe cat runs after the rat

Preposition can have two or more than two objectsThe road runs over hills and plain

A preposition also joins a noun an adjectiveHe is fond of tea

Generally a preposition comes before an object Sometimes it comes even after an object asWhat are you looking atThis is the house I live in

Subject- Computer Application

Chapter 2 Introduction to Java

Java API An application programming interface (API) in the context of Java is a collection of prewritten packages classes and interfaces with their respective methods fields and constructors

Byte Code Java bytecode is the result of the compilation of a Java program an intermediate representation of that program which is machine independent The Java bytecode gets processed by the Java virtual machine (JVM) instead of the processor JVM The Java Virtual Machine (JVM) is the runtime engine of the Java Platform which allows any program written in Java or other language compiled into Java bytecode to run on any computer that has a native JVM

Platform A platform is the hardware or software environment in which a program remains

Java platform The Java platform differs from most other platforms in that itrsquos a software only platform that runs on top of other hardware-based platforms

Applet and Application The fundamental difference between the two Java programs is that an application program is designed to run on a stand-alone machine whereas an applet is a web-version of an application which is used to run a program on a web browser

WORA ldquoWrite once run anywhererdquo (WORA) or sometimes write once run everywhere (WORE) is a slogan created by Sun Microsystems to illustrate the cross-platform benefits of the Java language

Class XSubject Topic Summary Execution

ECONOMICSFACTORS OF PRODUCTION Sub-topiclsquoLABOURrsquo

We shall start our class by discussing the topic taught in the last class

lsquo DIVISION OF LABOURrsquo- By division of labour we mean specialization in workIt refers to splitting up the work of labour involved in the production of a particular commodity into several parts and each part and sub-part is performed by a specialist

Now let us start by the Advantages of Division of labouraIt increases the level

Questions

1 Differentiate between Product-Based division of labour and Process-Based division of labour

Product-Based division of labour

Process-based division of labour

It is also known as simple (or occupational) division of labourUnder it everybody performs a particular occupations The entire is done by the same person

When a person or group of persons undertakes a specialised function which is supplementary to the production of final commodity and service This is also as complex division of labour

It is simple It is complex

of productionbSince the product is produced by an expert workerbest quality of product is producedcIt saves time and toolsd it promotes inventions in the methods and techniques of productioneIt leads to reduction in costs fAll workers get work according to their abilities and choices

Now let us discuss the disadvantages of Division of labour

a Since many workers are involved in the production of a commodity no one has the sense of responsibility

b The constant and repetition of the same work again and again make the work monotonous

c Division of labour facilitates production on large scale Hencethere is fear of over production

d Because of territorial division of labour some areasregions become more developed than others

It is based on labour-intensive techniques of production

It is based on capital-intensive techniques

It is generally found in small enterprises

It is generally found in large enterprises

Example Indian farmers doing all farm activities

Example A modern garmet factory where one person takes the measurementanother does the cuttingsome sew the clothes while a few workers button them and other iron them

2 What are the advantages of Division of labouraIt increases the level of productionbSince the product is produced by an expert workerbest quality of product is producedcIt saves time and toolsd it promotes inventions in the methods and techniques of productioneIt leads to reduction in costs fAll workers get work according to their abilities and choices

3Discuss the disadvantages of Division of laboura Since many workers are involved in the

production of a commodity no one has the sense of responsibility

b The constant and repetition of the same work again and again make the work monotonous

c Division of labour facilitates production on large scale Hence there is fear of over production

d Because of territorial division of labour some areasregions become more developed than others

English 1 Transformation of sentences

Sentences A sentence is a group of words which makes complete sense

a Assertive sentences

Exercise 6Rewrite the following sentences according to the instructions given below without changing their meanings

1 As soon as he saw the beer he jumped into

b Imperative sentences

c Interrogative sentences

d Exclamatory sentences

Sentences can be changed from one grammatical form to another without changing the meaning of the sentence This is known as transformation of sentences

the river ( Begin No sooner)2 None but brave deserve the fair (Begin the

bravehellip)3 This box is too heavy for me to lift ( Use so hellip

That instead of too)4 No one other than a king can live like James

Luxurious ( Begin only James)5 Oh for the wings of a dove (Begin I wishhellip)

Math Topic Commercial MathematicsChapter Shares and Dividends

Study item Discuss about shares and Dividends1) What is share

Ans To start any big business (company or Industry) a large sum of money is needed But it is not possible for an individual to invest such a large amount Then some persons interested in the business join together and from a company They divide the estimated money required into small parts Each such part is called a share

2) What do you mean by the term shareholder

Ans A person who purchases one or more shares is called shareholder3) Some terms related with a share

(i) Nominal value or face value or printed value The original value of a share is called its nominal value or face value or printed value

Note The nominal value of a share always remains same(ii) Market value or cash value The price of a share at any

time is called its market value or cash value

Note The market value of a share changes from time to time(iii) At par If the market value of a share is the same as its

nominal value the share is called at par(iv) At Premium or above Par If the market value of a share

is more than its nominal value the share is called at premium or above par

Example If a share of Rs 100 is selling at Rs 150 then it is said to be selling at a premium of Rs 50 or Rs 50 above par

(v) At Discount or below par If the market value of a share is less than its nominal value the share is called at discount or below par

Example If a share of Rs 100 is selling at Rs80 then it is said to be selling at a discount of Rs 20 or at Rs 20 below par

4) What is Dividend

Ans The profit which a shareholder gets for hisher investment from the company is called dividendNote (i) The dividend is always expressed as the percentage of the face value of the share(ii) The dividend is always given( by the company ) on the face value of the share

irrespective of the market value of the shareBENGALI(2ND LANGUAGE)

ldquoদেবতোর জণমrdquoলিবরোম চকরবত

পরথম লিসর পোঠ-চোর পসথ একটি পোথর লিবপলি ঘটোয় দেক যোতোয়োসতর পসথ পরলিতলিয়ত ওই পোথসর দেো োচট দেসত একলি দেতো দেক দেো োচট দেসয় দেবোমো সয় রোসতোর মোস লি0টসক পস1 লিবপরীত লিক দেথসক আো একটি দেমোটর োলি1 চোসকর কষতোয় পরোসরণ দেবোসচ যো লিকনত পরলিতবোর এমরণ দেৌভোয দেসকর োও সত পোসর তোই লিতলি দেকোো দেজোো1 কসর পোথরটিসক উপস1 দে8স পোথর উপস1 দে8োর ময় এক দেকৌতী জতো দেকসক পরশন কসর দেয লিতলি দেকোসো দেবতোর আস দেপসয়স0রণ লিকো লিকনত দেক বস লিতলি দেকোসো দেবতোর আস পোলি দেক উপলিত ক জতোর উসltসয বস কোরও ইস= স পোথরটি লিসয় দেযসত পোসর এর পর দেথসক দেক দেযসত আসত পোথরটি দেক দেসত পো একলি দেক কষয করস দেকউ পোথরটিসক ধসয়মস0 পলিরসকোর করস0 দেক ওই দেকৌতী জতোসক পোথসরর কোস0 বস থোকসত দেস এ0ো1ো আরও কষয কসর দেক ঠোৎ ওই লি1 পোথরটির োসয় লিোর োো-দেকউ পজো কসরস0 দেকৌতী দেোকটির আঙকো য় যলিসক উ পোথরটিসক লিরসয় দে8স তোর পর ঠোৎ একলি পোথরটির দেোোজ দেই দেক লিসয় দেস0 বো দেকোথোয় দেস0

বদোথ-

দেো োচট ndashচসত লিসয় দেকো লিক0র সE ধোককো দেস পস1 যোবোর উপকরমঅকসমোৎ- ঠোৎআতমমবর- লিসজসক লিয়নতরপ-পো লিপ0স প1োদেসতসসত- লিসপলিউৎোত- দেো1ো দেথসক উপস1 দে8োপরতযয় ndash লিবশবোপরতযোস- টতযোস ndash দৈবোসধসতোধলিসত- পরসপসরর পরলিত ব পরসয়ো করোপরসতরীভত- পোথসর পলিররণলিতবোনতঃকরসরণ- মস পরোসঅলিQৎ- অQো করসত ই=কদেোপ- বধমোর- দেবোস এমইতযোকোর- এইরকমরম- বময়পযসোভী-পসযর জয দেোভ আস0 যোরপোসথয় ঞচয়- পথচোর রচ জমোসোমোমোসরোস- ব ধম ধোসমর সE

তোর লি দেইhelliphellipTo be continued

Hindi 2nd lang सर क पद(सरदास)

सरदास शरी कषण भकति कावय क सरवशरषठ कगिरव ह इनक जनम और मतय क समय तथा सथान का मतभद हसरदास रवातसलय और शरार रस क अनयतम कगिरव ह इनक कावय म बालकषण क सौदय चपपल चषटा और गि7याओ की मनोहर झाकी मिमलती ह कषण और ोगिपयो क अननय परम का कतिचतरण ह सयो शरार की अपकषा उनक कावय म गिरवयो शरार का अमिBक गिरवषय और मारमिमEक कतिचतरण हआ हइन पकतियो म हम सरदास की भकति भारवनाओ का परिरचय मिमलता ह इनका सपण सगरह सरसार म गिनगिहत ह

1 जसोदा हरिर पालन झलारवहलरारवदलराईमलहारव रव जो ईसाई कछ ारव मर लाल को आई निनEदिदया काह ना आगिनसबारव त काह नाही बरवगिह आरवतोको कानहा बलारव

शबदाथ-हलरारव-गिहलती हदलराई - दलार पयार करती हमलहारव-पचकारती हनिनEदरिरया ndashनीदरवगिह-जलदी सअBर-होठमौन-चपसन-सकत

वयाखया- सरदास जी कहत ह गिक यशोदा माता बालक कषण को पालन म झल आती ह रवह उनह गिहलाती ह पयार करती ह मलहार जस कोई ीत ान लती ह और नीद स पछती ह गिक ह नीद तम मर लाल को आकर कयो नही सलाती तझ खाना बला रहा ह कभी कषण आख बद कर लत ह कभी आखफडफडान लत ह उनह सोता हआ जानकर यशोदा माता चप हो जाती ह और इशार म बात करन लती ह इसी बीच अकला कर कषण ज जात हतो गिफर यशोदा माता गिफर स ाना ान लती ह सरदास जी कहत ह गिक भरवान क दशन का सख दरवता और ऋगिष-मगिनयो को भी दलभ ह यही सख माता यशोदा को बडी सहजता स मिमल जा रही ह माता यशोदा बहत ही भागयशाली ह2)Continue to nexthellip

Physics

Chapter 1 Force

(Summary)

Question A body is acted upon by two forces each of magnitude F but in opposite directions State the effect of the forces if

(a) Both forces act at the same point of the body

(b)the two forces act at two different points of the body at a separation r

Solutions

(a) Resultant force acting on the body = 0

F ndash F = 0(b) The forces tend to rotate the body between two forces about the midpoint

Moment of forces = F times rFr

QuestionDefine moment of a couple Write its SI unit

Solutions

Moment of couple is equal to the product of both force and the perpendicular distance between the two forces

The SI unit of moment of couple is NmCommercial Studies

Advertising and sales

Business firms use several methods to

Questions1) What do you mean by advertising

promotion create demand of their product in the market and increase it sales Such methods comprises of advertising sales promotion personal selling and publicityToday we are going to discuss about one of such methods It is advertising

Meaning of advertisingAdvertising is a paid form of non-personal presentation for promotion of Ideas goods and services

Importance or merits of advertising Advertising has importance to manufacturer or traders to customer and to society as a whole

Today we will see how advertisement help the manufacturer or traders

Answer) Advertising is a means of how a company encourages people to buy their products services or ideas It is one element of marketing which also includes design Research and data mining

2) Mention any three features of advertisingAnswer)The main features of advertising are

i) It is impersonal form of presentation for promotion of products and services of Ideas

ii) It is issued by identified sponsor The advertisement contains the name of the advertiser

iii) It is a form of mass communication because the message is directed to a large number of persons simultaneously

3) Mention the main merits or importance of advertisement to manufacturer or tradersAnswer)

i) Introducing new product A business organization can introduce itself and its products to the public through advertising

ii) Increase the sale Advertising leads to increase the sale of existing product by entering into new markets and attracting new customers

iii) Create steady demand Advertising creates sustains regular demand by smoothening out seasonal and other fluctuations It enables regular production for the organisation

iv) Economics of scale Advertising facilitate mass distribution of goods and steady demand which lead to large scale and regular production

v) Goodwill Advertising helps in creating a good image of the firm and reputation for its products

Biology Chapter - 03Genetics

Today wewill start chapter and discuss about Genetics Gregor Mendel is known as father of genetics Before entering into Mendelrsquos experiment on Genetics we must know

Q1 Define the following termsi) Genetics Genetics is the study of

transmissionof body features from parents to offspringand the laws relating to such transmission

ii) Heredity It may be defined as transmissionof genetically based characteristics from parentsto offspring

iii) Character and traits Any heritable

Importance to TraderIntroducing new productIncrease the saleCreate steady demandEconomics of scaleGoodwill

some terms featureis a character The alternative forms of acharacter are called traitsex Character (Hair shape) - Traits (Curly straight)

iv) Homologous chromosomes A pair ofcorresponding chromosomes of the same shapeand size one from each parent

v) Genes Genes are the specific parts (DNA segments) of a chromosome which determinethe hereditary characteristicsNearly 30000genes present in human

vi) Alleles Alternative forms of a gene occupying the same position (locus) on homologouschromosomes and affecting the same characteristicbut in different ways

vii) Genotype ndash PhenotypeGenotype means of genes present in the cells of an organism Phenotype means the observable characteristic which is genetically controlled

viii) Mutation It is a sudden change in one or more genes or in the number or in the structure of chromosomes ex Sickle cell anaemia is a blood disease caused by a gene mutation

CLASS NOTES

Class XSubject Eng Literature (The Merchant of Venice ndash William Shakespeare)Topic Act IV Scene 1 Lines 01 to 34 ( Duke helliphelliphellip We all expect a gentle answer Jew) ate 27th April 2020 (2nd Period)

[Students should read the original play and also the paraphrase given in the school prescribed textbook]Summary Questions amp Answers

This scene may be termed as the catastrophe of the play It is the final unravelling of the complicated events which seem to threaten the happiness of Bassanio Portia and Antonio Right is justified to the fullest degree and malice falls into the trap prepared for others No one suffers here but Shylock but even then he receives a generous measure of mercy

o This is the Court-scene Initially we meet

(1)

DUKE I am sorry for thee thou art come to answer (Line 3-6)A stony adversary an inhuman wretchUncapable of pity void and emptyFrom any dram of mercy

(i) Who is addressed here Where is the person Why is the person there

Antonio is addressed hereAntonio is in the court of justice at VeniceAntoniorsquos trial is scheduled to be held here for his failure to meet the conditions of the bond he signed with Shylock

the Duke Antonio Shylock and Salerio Later we meet Bassanio Portia Gratiano and Nerissa

o The Duke says to Antonio that he has to face a very cruel opponent which Antonio admits and expresses his gratefulness to the Duke for his efforts to soften without result the heart of Shylock in order to be merciful to Antonio Antonio further says that he is ready to accept whatever cruel judgement the Court may award

o When Shylock appears in the court the Duke says that Shylock should change his decision of prosecuting Antonio and demanding the penalty specified in the bond out of consideration of the great misfortunes that Antonio has suffered If this is done by him (Shylock) the whole court would be gladdened by his merciful action

(ii) What is the Duke sorry for

The Duke is unable to change the mind of Shylock from his decision to get the bond forfeited even after he pleaded to Shylock Shylock stands firmly for his bond which when forfeited will allow him to take a pound of flesh from any part of Antoniorsquos body(iii) How does the Duke address Shylock`The Duke calls Shylock an adversary with a heart of stones He calls Shylock as an inhuman wretch without pity Shylock is quite lacking in the slightest quality of mercy (iv) How does Antonio reply to this

Antonio replies that he will meet the revenge of Shylock patiently He has prepared himself to suffer with a quiet spirit the utmost that Shylockrsquos tyranny and rage can do(v) What quality of the Duke is revealed here

The Duke is kind and benevolent He is ready to help Antonio He requests Shylock to free Antonio from the trial(vi) What are the terms of the bond that Antonio has signed

The terms of the bond that Antonio has signed were that if Antonio is unable to repay Shylock a certain sum of money specified on the paper on a certain date and in an agreed place the forfeit has to be paid The forfeiture will be an exact pound of Antoniorsquos flesh which Shylock will be a liberty to take from any part of Antoniorsquos body which pleases him

Class XI

STUDY MATERIAL

Class XISubject Eng Literature (The Tempest ndash William Shakespeare) Topic Act I Scene 2 Lines 88 to 132 (Prospero hellip Me and thy crying self) Date 27th April 2020 (3rd Period)

[Students should read the original play and also the paraphrase given in the school prescribed textbook]Summary Questions amp Answers

o Prospero now tells Miranda that he was the Duke of Milan He had been devoting himself more to studies than the affairs of the State His brother Antonio took advantage of this situation and with the help of Alonso the king of Naples seized upon him and her one midnight and shipped them in a frail bark so that they perished in the sea All this took place

(1)

MIRANDA I should sin (Line 118-132)

To think but nobly of my grandmother

Good wombs have borne bad sonsPROSPERO Now the condition

The King of Naples being an enemyTo me inveterate hearkens my brothers suitWhich was that he in lieu othrsquo premisesOf homage and I know not how much tribute

twelve years back

IMPORTANT PASSAGES EXPLAINED(Line 98-103)

PROSPERO helliphelliphelliphelliphelliphelliphellip

He being thus lorded

Not only with what my revenue yieldedBut what my power might else exact like oneWho having into truth by telling of it Made such a sinner of his memoryTo credit his own lie he did believeHe was indeed the duke

Prospero in telling the narrative of his past life here refers to his brother Antonio Prospero being with a studious bent of mind has left the administration of Milan on his younger brother Now Antonio being thus invested like a lord with all the powers derived from Prosperorsquos wealth and what the exercise of Prosperorsquos authority might secure for him regarded himself as a de facto Duke of Milan It is a well-known fact of psychology that a man who repeatedly tells a lie makes of his memory such a sinner against truth as to credit his own lie by the telling of it So Antonio by repeatedly saying to himself and others that he was the Duke came to believe that he was really the Duke Thus falsehood repeatedly asserted gained the force of truth for Antonio and he truly believed it

Should presently extirpate me and mine Out of the dukedom and confer fair MilanWith all the honours on my brother whereonA treacherous army levied one midnightFated to thrsquo purpose did Antonio openThe gates of Milan and ithrsquo dead of darkness The ministers for thrsquo purpose hurried thenceMe and thy crying self

(i) In the earlier lines of this scene what does Prospero tell about his intense interest What was the demand of his interest

In the earlier lines of this scene Prospero tells Miranda that he had an intense interest in the study of philosophy and magic arts Hence in order to improve his mind with this kind of study he kept himself isolated from worldly and state affairs His study was dearer to him than the applause and esteem that he could win from the public His study demanded too much solitude(ii) What forced Antonio to take an undue advantage over Prospero

Prosperorsquos indifferent attitude towards the statersquos affairs and his having boundless trust in Antonio gave rise to a boundless lust for power in Antoniorsquos mind Antonio felt that he must be the actual Duke instead of the part of the Duke he played Thus Antonio took an undue advantage of the situation to usurp Prosperorsquos dukedom(iii) Explain the following lines ldquoI should sin to think but nobly of my grandmother Good wombs have borne bad sonsrdquo

After hearing the treacherous act of her uncle Antonio Miranda says that Prosperorsquos mother was a noble lady and she cannot dishonour her memory by saying that the person named Antonio cannot be his (Prosperorsquos) brother She says that in honour of her grandmother she also cannot say that Antonio must have been begotten not by her grandfather but by some other man She finally concedes that it is known that good mothers have borne bad sons in their wombs and gave birth to them

(iv) Why did the King of Naples accept Antoniorsquos request to help him in usurping his dukedom What did Antonio propose to Alonso

Alonso who was the king of Naples accepted Antoniorsquos request in usurping Prosperorsquos dukedom because he (Alonso) was a sworn enemy of Prospero Antonio proposed that Alonso should immediately drive him (Prospero) and his offspring out of Milan and should confer the dukedom upon him (Antonio) with all the dignities which go with that

position In return Antonio promised that he shall give an annual tribute and also swore his allegiance to Alonso Also he agreed to hold the Dukedom of Milan as a subordinate to the state of Naples(v) How were Prospero and Miranda carried away from the city of Milan and what was the state of small Miranda at that time

In pursuance of the agreement settled between Antonio and Alonso an army of treacherous men was assembled One midnight when the occasion suited the will of destiny Antonio opened the gates of the city of Milan and in the death like silence of midnight Antoniorsquos agents who had been directed to execute his purpose carried Prospero and small Miranda away from the city in all haste They were then forced into a ship and carried some distance out to sea where they put them on a mere hulk of a boat without any rigging or ship-gear and abandoned them leaving them at the mercy of the roaring sea Miranda was a very small child of three years age and she was crying at that time

CLASS -XIDATE-270420Subject Topic Summary Execution

EVS Chapter 1 ndash Mode of Existence

Impact of mode of existence on resources

Q) Why resources are under pressure

Ans - Increase in the sophistication

of technology enabling natural resources to be extracted quickly and efficiently Eg in the past it could take long hours just to cut down one tree only using saws Due to increased technology rates of deforestation have greatly increased

The number of humans is increasing Cultures of consumerism Materialistic views

lead to the mining of gold and diamonds to produce jewelry unnecessary commodities for human life or advancement Consumerism also leads to extraction of resources for the

production of commodities necessary for human life but in amounts excessive of what is needed because people consume more than is necessary or waste what they have

Lack of awareness among the population is striking People are not aware of ways to reduce depletion and exploitation of materials

Accounts Cash Book Today we are going to start a new topic -Cash Book

The key terms used in this chapter are

bullCash book

bullSimple cash book

bullDouble column cash book bullTriple column cash book

bullPetty cash book

bullCash discount

bullContra entry

Here I will share you the meaning of each key terms

bullCash book Cash Book is a special purpose subsidiary book or journal in which cash received and cash payments are recorded

bullSimple cash book

It is a cash book in which only cash transactions are recorded It has only one column on each side

bullTriple column cash book

It is cash book which has three columns one column for each cash and Bankdiscount on each side of the cash book In this book both cash and Bank transactions are recorded together with discount allowed and received

bullPetty cash book

It is a cash book maintained for recording petty expenses

bullCash discount

Cash discount is the amount of discount received or allowed on cash payments and cash receipts Discount received is an income for the business while discount allowed isan expense

bullContra entry

It means transactions involving both cash and Bank Such transactions though recorded in the cash book are not posted into ledger The letter lsquoC is written in Ledger folio for contra entry

Business Studies

ENTREPRENEURSHIP

Now we shall discuss the second chapter

lsquoENTREPRENEURSHIPrsquo

Today before starting the chapter let us recall what

Questions

1What are the main characteristics of Intrapreneurship

Answer

The main characteristics of Intrapreneurship are

Corporate framework-it occurs within the framework of the same company

Semi-Autonomous-Intrapreneurship

we have read last day

Let s today start the class by recalling the last topic taught

Intrapreneurship is the process of discovering and exploring business opportunities within an existing company It involves launching new business ventures within the framework of a present corporation Intrapreneurship is also known as corporate entrepreneurship or corporate venturing

Now let us start with the characteristics of Intrapreneurship

The main characteristics of Intrapreneurship are

Corporate framework

Semi-Autonomous Lack of ownership Senior position Low risk taking Not own boss

Now let us discuss the meaning of enterprise

Enterprise means an undertaking or adventure that requires some innovation and investment and thus involves riskEnterprise always entails decision making coordination and risk bearing

involves crating amd nurturing a semi-autonomous business unit which may be a subsidiary a strategic business unit or a division

Lack of ownership-the intrapreneur is not the owner of the unitb he creates and nurtures

Senior position-he occupies a senior managerial position in the company

Low risk taking-An intrapreneur does not bear the full risk of failure

Not own boss-An intrapreneur is not his own bosss in legal termsHe enjoys the freedom and gets the required resources and support

2 How is Entrepreneur is different from Intrapreneur

The functions involved in both the entrepreneurship and intrapreneurship are by and large similar however there are several differences between the two

Point of distinction

Entrepreneur

Intrapreneur

status An independent business person

A senior executive within a company

Ownership Owner of

the business

An employeesometimes a share in ownership

Financing Responsible for raising finance for the business

Not responsible for raising the finance

Risk bearing

Bears the risk of the business

Does not bears the risk of the business

Reward Profit which is uncertain and irregularcan be loss

Fixed salary and fringe benefits

Need for security low high

3 What do you understand by enterprise

Answer Enterprise means an undertaking or adventure that requires some innovation and investment and thus involves riskEnterprise always entails decision making coordination and risk bearing

COMMERCE NATURE AND OBJECTIVES OF

BUSINESS

Today let us recall the last other two objectives of business by the chart given in the previous class

Firstly we would discuss Human Objectives

Business is run by people and for people Labour is a valuable business element

Human objectives of business are concerned with the well -being of labour

The human objectives are as follows

Labour welfare Developing human

resources Participative

management Labour

management cooperation

Questions

1 Explain the human objectives of a business enterprise

Answer

Business is run by people and for people Labour is a valuable business element

Human objectives of business are concerned with the well -being of labour

The human objectives are as follows

Labour welfare-Business must recognize the dignity of labour and human factors should be given the recognition

Developing human resources-Employees must be provided the opportunities for developing new skills and attitudes

Participative management-Employees should be allowed to take part in decision making process of business

Labour management cooperation-Business should strive for creating and maintaining cordial employer employee relations so as to ensure peace and progress in industry

Now let us discuss the national objectives of business

Optimum utilization of resources

National self- reliance Development of small

scale industries Development of

backward areas Control over pollution

2Explain the national objectives of a business enterprise

Answer

It is the duty of business to utilize the resources of the country properly the national objectives of business

Optimum utilization of resources ndashBusiness should use the nationrsquos resources in the best possible manner

National self- reliance-It is the duty of the business to help the government in increasing experts and in reducing dependence on imports

Development of small scale industries-Big business firms are expected to encourage growth of small scale industries which are necessary for generating employment

Development of backward areas-Business is expected to give preference to the industrialization of backward regions of the country

ECONOMICS

BASIC ECONOMIC CONCEPTS

SUB

TOPIC

Value

Wealth

Welfare

Today we shall start with a new topic of the same chapter ie lsquoValuersquo

Value of a commodity is defined as the valuation placed by a household on the consumption of this commodity

lsquoValuersquo has two different meanings and these are

a Value -in -use It refers to consumption value of a commodity It expresses the utility derived from the consumption of a particular commodity A necessity like water has a very high value ndashin ndashuse or

Question

1What is value

Answer

Value of a commodity is defined as the valuation placed by a household on the consumption of this commodity

2What is value-in use

Answer It refers to consumption value of a commodity It expresses the utility derived from the consumption of a particular commodity A necessity like water has a very high value ndashin ndashuse or consumption value

3What is value ndashin- exchange

Answer It relates to market value of a commodity

It is the rate at which a particular good or service can be exchanged for money

For example in barter system if a person is prepared to exchange 3 metres of cloth with 1 pair

consumption value

b Value ndashin-exchange It relates to market value of a commodity

It is the rate at which a particular good or service can be exchanged for moneyFor example in barter system if a person is prepared to exchange 3 metres of cloth with 1 pair of shoes then the value in exchange of 3 metres of cloth is 1 pair of shoesValue in exchange is the power of purchasing other goods In modern monetised economies the exchange value of goods are expressed in terms of money as prices

Now let us discuss the term lsquoWealthrsquo

Wealth refers to the stock of all those assets which are a source of income

Wealth is a stock concept

Wealth must possess the following features

a Utility It must possess utility or give some

of shoes then the value in exchange of 3 metres of cloth is 1 pair of shoes

Value in exchange is the power of purchasing other goods In modern monetised economies the exchange value of goods are expressed in terms of money as prices

4 What is wealth

Answer it refers to the stock of assets or goods which are a source of income and have personal or national ownership

5 What are the features of wealth

Answer The features of wealth are as follows

Wealth must possess the following features

a Utility It must possess utility or give some satisfaction

b Scarcity It must be limited in quantityc Transferability it should be transferable its

ownership can be transferred from one person to another person

d Exchange value It must possess exchange value

6 What is welfare

Answer

Welfare is defined as satisfaction and happiness a sense of well- being among the people

satisfactionb Scarcity It must be

limited in quantityc Transferability It

should be transferable its ownership can be transferred from one person to another person

d Exchange value It must possess exchange value

Now let us discuss the term lsquoWelfarersquo

Welfare is defined as satisfaction and happiness a sense of well- being among the people

Welfare is affected by factors like

a Consumption of goods and services

b Environment

c Family relations

d Degree of freedom

e Law and order situation

Mathematics Trigonometric equation

To find the general solution of the equation sinθ=0

When sin θ =0

Then θ= 0 π2π 3π-π -2π -3

i e when θ = 0 or an integral multiple of π

i e when θ= nπ where n is any integer

Therefore the general solution of the equation sin

Example1 Find the general values of θ which satisfy the equation sin2 θ =34

Solution sin2 θ= 34

Or sin θ = +34 or -34

Or sin θ = sin π3 or sin (-π3)

Therefore

θ = [nπ + (-1) n (π3)] or[ nπ+ (-1) n (-π3)]

= nπ +π3 or nπ-π3 where n= any integer

Example 2Find the values of θ which satisfy tan2 θ

θ=0 is θ= nπ where n is any integer

To find the general solution of the equation cos θ=0

When cos θ=0

Then θ=π2 3π2 5π2 -π2 -3π2 -5π3

i e when θ is an odd multiple of π2

i e when θ=(2n+1) π2 where n= any integer

Therefore the general solution of the equation cos θ =0 is θ= (2n+1) π2 where n= any integer

To find the general solution of the equation tan θ = 0

Clearly tan θ =0 implies sin θcos θ =0

Therefore θ = nπ

i e the general solution of the equation tan θ=0 is θ =nπ where n = any integer

To find the general solution of the equation cot θ =0

Clearly cot θ =0 implies (cos θsin θ) = 0

i e cos θ =0

Therefore θ = (2n+1) π2

Therefore the general solution of the equation cot θ =0 is θ = (2n+1) π2

Where n= any integer

To find the general solution of the equation sin θ= k (-

=13 -πleθleπ

Solution tan2 θ =13

Or tan θ = plusmn1radic(3) =tan(plusmnπ6)

θ=nπ plusmn π6 where n =any integer

If n=0 then θ=plusmnπ6

If n=1 then θ= π plusmn π6

If n=-1 then θ= -π plusmn π6

Therefore the required solution in -π le θ le π are θ= π6 5π6 -π6 -5π6

Exercise Find general solution of sin 2θ=cos θ [Hints Use sin 2θ= 2sin θcosθ and then take cosθ

common]

1lek le1)

Determine an angle alpha such that sin =k and -π2le αle π2

Then we have

Sin θ = k = sin α

Or sin θ - sin α =0

Or 2 cos [(θ+α) 2] sin [(θ-α) 2] =0

Therefore either cos [(θ +α) 2] =0 (1)

Or sin [(θ-α) 2] =0 (2)

Now from (1) we get (θ+α) 2= (2m+1) π2)

Or θ = (2m+1) π-α (3)

And from (2) we get (θ-α) 2 =mπ

Or θ= 2mπ+α(4)

Where m = any integer

Clearly the solution (3) amp (4) may be combined in the following form

θ= nπ+(-1) n α where n= any integer

Therefore the general solution of sin θ = sin α is θ = nπ +(-1) n α where n is any integer and -π2 le α le π2

Biology Chapter - 04Kingdom Monera

Today we will discuss about bacterial reproduction and its usefulness

Fig Binary Fission

Fig Conjugation Fig Transformation

Bacterial reproduction is mainly asexual but sexual reproduction

also takes place

Asexual reproduction takes place by i) Binary fission - from one bacteriato

two bacteria are produced in every 20 to 30mins

ii) Buddingiii) Endospore formation - during

unfourable condition

Sexual reproduction by three ways

1) Conjugation - Transfer of genetic material between cells that are in physical contact with one another

2) Transduction - Transfer of genetic materialfrom one cell to another by a bacteriophage

3) Transformation - Transfer of cell-freeor naked DNArsquo from one cell to another

Bacteria causes different diseases inplants animals and human and

it causes food spoilage and waterpollution but it also have some useful

activities

i) Bacteria are helpful in sewage water treatment

ii) It is used in antibiotic (medicine) production

iii) Anaerobic bacteria help in biogas(energy) production

iv) Many household products like yoghurt cheese are manufactured by use of bacteria

v) Rhizobium by symbiotic relationship with leguminous plant increase soil fertility

vi) Besides these bacteria is helpful in genetic engineering degradation of petroleum hydrocarbonand in dairy

industry

Physics Motion in plane Here we will introduce Projectile Motion

Execution

Projectile

Y

usinθ u h

θX

ucosθ

Suppose a body is projected with an angle θ So initial velocity u can be resolved into two components

Horizontal component - ucosθ ( for range)

Vertical component - usinθ ( for height)

usinθ changes during motion and becomes zero at maximum height position but ucosθ remain unchanged

The maximum height of projectile is h

NB If initial is upward then g = -ve and if it is downward then g = +ve Height is +ve if direction of motion does not change ( for ex a body thrown upwards but goes down ultimately then height h = -ve)

The angle of projectile θ is the angle made with horizontal

HISTORY ndash GROWTH OF NATIONALISM

SUB TOPIC- REVOLUTIONARY NATIONALISM Bengal formation of Anushilan Samity and Jugantar Group

The intensification of the Swadeshi movement and Government policy of terror and repression led to outbreak of violence Bombs were manufactured and attempts on the lives of unpopular Government officials became frequent In the gymnasium of Scottish Church College which was known as General Assemblies Institution a secret society was formed known as Anushilan Samity

Aurobindo Ghosh send from Baroda his emissary Jatindranath Banerjee to mobilize the Bengal revolutionaries

Hemchandra Qanungo and Satyen Bose published Journal Jugantar

The Jugantar group planned to assassinate oppressive magistrate Kingsford by Khudiram Bose and Prafulla Chaki in 1908 Prafulla Chaki committed suicide to avoid arrest Khudiram was tried and hanged

Afew days later the police found a bomb factory in Maniktala and arrested a large number of revolutionaries The trial of revolutionaries became famous as the Alipore Bomb Case

In the course of the trial the approver the public prosecuter and a police officer were assassinated

1 Question Name two journals which preached the cult of violence

Answer a) Yugantar edited by Bhupendranath DuttaB) Bandemataram edited by Aurobindo Ghosh2 Question Why was Khudiram arrested and hangedAnswer An attempt was made to assassinate a hated vindictive majistrate named Kingsford by Khudiram Bose and Prafulla Chaki Their attempt failed and the bomb they threw killed two English ladies Khudiram was arrested and put to trial and then hanged3Question Who was Aurubindo GhoshAnswer Aurobindo Ghosh a nationalist revolutionary who was charged for his involvement in the Alipore Bombing Case He was accused of it along with his brother Barindra nath Ghosh But Aurobindo was acquitted because of the brilliant pleading of his counsel Chittaranjan Das Then he became a spiritual reformer introducing his visions on human progress and spiritual evolution4 Qustion Who was KingsfordAnswer Kingsford was an unpopular British chief Magistrate who was the target of the bomb thrown at Muzaffarpur by Khudiram and Prafulla Chaki

Most of the accused were convicted and sentenced to

long term of imprisonmentBut

Aurobindo Ghosh was acquitted mainly owing to the brilliant pleading of his counsel Chittaranjan Das

Political science Topic-Sovereignty

Summary Sovereignty is the full right and power of a governing body over itself without any interference from outside sources or bodies In political theory sovereignty is a substantive term designating supreme legitimate authority over some polity In international law sovereignty is the exercise of power by a state

Internal Sovereignty

Internal sovereignty means supreme authority within ones territory while external sovereignty relates to the recognition on the part of all states that each possesses this power in equal measure

External sovereignty

external sovereignty relates to the recognition on the part of all states that each possesses this power in equal measure

Distinguish between

Execution

Answer the following questions

Short notes-

Sovereignty

Internal Sovereignty

External sovereignty

Homework- learn

external sovereignty and internal sovereigntySovereignty is the principle

of supreme and

unquestionable authority

reflected in the claim by the

state to be the sole author of

laws within its territory

Definition of external vs internal sovereigntyInternal sovereignty refers to

the relationship between a

sovereign power and its

subjects ndash it refers to the

location of the supreme

authority within the state In

the UK for example internal

sovereignty (supposedly)

resides within Parliament

reflected in the

constitutional principle of

parliamentary

sovereigntyBy contrast

external sovereignty refers

to the capacity of the state

to act independently and

autonomously on the world

stage This is what is

sometimes called lsquostate

sovereigntyrsquo or lsquonational

sovereigntyrsquo and implies

that states are legally equal

and that the territorial

integrity and political

independence of a state is

inviolable

Class ndash XII

Date - 2742020 STUDY MATERIALSubject Topic Summary Execution Business Studies

Job Analysis amp Manpower Planning

At first let us recall the chapter what we have discussed till nowbullJob analysisbullJob specification bullJob description bullJob enlargement bullJob enrichment

Today we will do some questions answers from the chapter

Questions 1ldquoJob analysis job description and job specification are interrelatedrdquo Comment Answer) Job analysis is a systematic and detailed examination of a job to collect all the relevant information about it The contents off the job are summarised in the job description The qualification needed for the job are summarised in job specificationThus there is close interrelationship between job analysis job description and job specification

Question 2ldquoJob enlargement is a horizontal extension of a job whereas job enrichment is a vertical extension of a jobrdquo ElucidateAnswer) Job enlargement involves adding one or more task to a job coma where as job enrichment involves adding more autonomy and responsibility to a job Job enlargement is therefore horizontal extension of a job coma whereas job enrichment is a vertical extension of a job

Question 3 )

What is manpower estimation Explain its quantitative and qualitative aspectsAnswer) Manpower estimation is the process by which management determines how an organisation should move from itrsquos current manpower positionto its desired manpower position There are two dimensions of Manpower estimation- quantitative and qualitative

Quantitative aspectThis aspect of Manpower estimation involves estimating the number of employees required in a future time period Workload analysis and workforce analysis are done to estimate the quantity of required manpower

Qualitative aspectThe estimate of the knowledge skills experience etc of required manpower is the qualitative aspect of Manpower estimation The quality of Manpower can be judged on the basis of job analysisand job specification

COMMERCE

CAPITAL-FIXED AND WORKING

Today let us start the class by discussing the sources of finance for different types of business firms

The term lsquocapitalrsquo refers to the investment made in the enterprise for the purpose of earning profits

Requirements of capital and sources of capital for different types of business firms are

1 Capital for sole proprietorship businessA sole proprietor operates at a small scale and thereforerequires a limited amount of capital

2 Capital for partnership firmCapital requirements as well as capital base of a partnership is bigger than that of a sole trader businessThe owned capital is contributed by the partners in an agreed ratio

3 Capital for joint stock companyA joint stock company generally requires large amount of capitalA public company can raise huge capital through issue of shares In addition to share capital it can utilize retained profits

Now let u discuss the meaning of Finance PlanningFinance planning is the process of estimation the financial requirements of an organization specifying the sources of firms and ensuring that enough funds are available at the right time

1 What do you mean by Finance PlanningAnswer Finance planning is the process of estimation the financial requirements of an organization specifying the sources of firms and ensuring that enough funds are available at the right time

2Discuss the role of financial planning of an enterpriseThe role of financial planning are as followsa A sound financial plan helps a business enterprise to avaid the problems of shortage and surplus of fundsbFinancial planning serves as a guide in developing a sound capital structure so as to maximize returns to shareholders c It helps in effective utilization of fundsd It provides policies and procedures for coordinating different functional areas or departments of businesse It enables the management to exercise effective control over the financial activities of an enterprisef It helps the company to prepare for facing business shocks and surprises in future

Mathematics

Continuity and differentiability

Recall Definition of ContinuityLet f(x) be a single valued function of x and x=a be a point in the domain of definition of the function The function is said to be continuous at x=a ifi) f(c) is defined ie f(x) has a definite finite value at x=cii) lim xrarra f(x) exists andiii) lim xrarra f(x) =f(a) In other words f(x) is said to be continuous at x=a if lim xrarra+ f(x)= lim xrarra- f(x) = f(a) Or f(a+0) =f(a-0) =f(a) Or lim hrarr0 f(a+h)= f(a) Algebra of continuous functionsNow we will study some algebra of continuous functions Theorem 1 Suppose f and g be two real functions continuous at a real number c Then(1) f + g is continuous at x = c(2) f ndash g is continuous at x = c(3) f g is continuous at x = c(4) (fg) is continuous at x = c (provided g (c) ne 0)

Example 1 Prove that every rational function is continuousSolution Recall that every rational function f is given byf(x)=[p(x) q(x) ] q(x)ne0where p and q are polynomial functions The domain of f is all real numbers except points at which q is zero Since polynomial functions are continuous f is continuous by (4) of Theorem 1Example 2Discuss the continuity of sine functionSolution To see this we use the following factslim xrarr0 sin x =0Now observe that f (x) = sin x is defined for every real number Let c be a real number Put x = c + h If x rarr c we know that h rarr 0 Therefore lim xrarrc f(x)

= lim xrarrc sin x= lim hrarr0 sin(c+h) =lim hrarr0 [sin c cos h + cos c sin h ]=lim hrarr0 (sin c cos h) + lim hrarr0 (cos c sin h) = sin c +0=sin c = f(c) Thus lim xrarrc f(x) = f(c) and hence f is a constant function Exercise Prove that the function f(x) = x2 +2x is continuous for every real value of x [Hints show that lim xrarra+ f(x) = lim xrarra- f(x) = f(a) ]

Biology Reproduction in Flowering plants We will discuss about megasporoangium

megasporagenesis and female gametophyte

Q4 Describe the structure of megasporangium

Ovule is attached to the placenta by astalk called funicle

Each ovule has one two or three protectivecoverings called integuments

At the tip of integuments a small openingcalled micropyle is organised

Opposite to the micropylar end is the chalaza

Within the integuments a mass of cellsnucellusand inside it embryo sac orfemale gametophyte is present

Q5 Describe a mature embryosacamp its formation

In most of the flowering plants only oneof the 4 megaspores formed as a result ofmegasporogenesis that is functional while theother three degenerate

The

functional megaspore develops into thefemale gametophyte

Formation The nucleus

of the functional megasporedivides mitotically to form two nuclei first andthen two more sequential mitotic nucleardivisions result in the formation of four ampthen eight nucleate stages of embryo sac

Six of the eight nucleus are surrounded bycell walls and organised into cells

The remaining two nuclei called polar nuclei are found below the egg apparatus in the largecentral cell

Three cells consisting of two synergids amp one egg cell present bottom of

embryo sac Three cells

at the chalazal as antipodal cells

Two polar nuclei together present in large central cell

HISTORY

TOWARDS INDEPENDENCE AND PARTITION THE LAST PHASE(1935-1947)SUB TOPIC NATIONAL MOVEMENTS DURING THE SECOND WORLD WAR

Spread of Quit India Movement On 9th August 1942Gandhiji and other Congress leaders were arrested The Congress was declared illegal The news of the arrest of all leaders marked the beginning of a widespread movement of India It was not possible for such a movement to remain peacefulBut the arrest of the all notable congress leaders virtually left the movement in the hands of the mass The movement took the form of violent and militant outbreakBesides congressmen revolutionaries also were very active in the movement The Congress Socialist group also played a prominent role

1 Question Why did the British authority arrest the Congress leaders on 9 th August 1942Answer Congress Working committee adopted the Quit India resolution which was to be ratified at the Bombay AICC meeting in 8th August 1942 They decided to launch a mass struggle on non-violent lines Gandhiji gave a clarion call to all section of the people rdquoKarenge ya Marengerdquo (do or die) Congress leaders gave the call to driving out

the British from IndiaViceroy had taken strong action against the Quit India movement Gandhiji and all the leaders of Congress were arrested

2 Question How did Quit India Movement spread out all over IndiaAnswer The news of the leaders lsquo arrest marked the beginning of a widespread movement to remain peacefulThe movement took form of violent outbreak There were widespread cutting of telephone and Telegraph wires damaging railway lines raising barricades in cities and towns and other forms of violent demonstations

Question Name the leaders of Congress

Socialist group played a prominent part Notable among the Jayprakash Narayan Rammonohar Lohia Aruna Asaf Ali

Political science

Topic-Franchise and Representation

Summary

The election commission

The Election Commission of India is an autonomous constitutional authority responsible for administering Union and State election processes in India The body administers elections to the Lok Sabha Rajya Sabha State Legislative Assemblies in India and the offices of the President and Vice President in the country

Functions of election commission-

India is a sovereign socialist secular democratic republic Democracy runs like a golden thread in the social economic and political fabric woven by the Constitution given by lsquoWe the People of Indiarsquo unto ourselves The concept of democracy as visualised by the Constitution pre-supposes the representation of the people in Parliament and State legislatures by the method of election The Supreme Court has held that democracy is one of the inalienable basic features of the Constitution of India and forms part of its basic structure The Constitution of India adopted a Parliamentary form of government Parliament consists of the President of India and the two Houses mdash Rajya Sabha and Lok Sabha India being a Union of states has separate state legislatures for each state State legislatures consist of the Governor and two Houses mdash Legislative Council and Legislative Assembly mdash in seven states namely Andhra Pradesh Telangana Bihar Jammu amp Kashmir Karnataka Maharashtra and Uttar Pradesh and of the Governor and the state Legislative Assembly in the remaining 22 states Apart from the above two out of the seven Union Territories namely National Capital Territory of Delhi and Puducherry also have their Legislative Assemblies

ExecutionShort notes-Election commissionFunctions of election commission

Homework- Learn

Computer

Science

Computer hardware NAND Gate

A NOT-AND operation is known as NAND operation It has n input (n gt= 2) and one output

Logic diagram

Truth Table

NOR Gate

A NOT-OR operation is known as NOR operation It has n input (n gt= 2) and one output

Logic diagram

Truth Table

XOR Gate

XOR or Ex-OR gate is a special type of gate It can be used in the half

adder full adder and subtractor The exclusive-OR gate is abbreviated as EX-OR gate or sometime as X-OR gate It has n input (n gt= 2) and one output

Logic diagram

Truth Table

XNOR Gate

XNOR gate is a special type of gate It can be used in the half adder full adder and subtractor The exclusive-NOR gate is abbreviated as EX-NOR gate or sometime as X-NOR gate It has n input (n gt= 2) and one output

Logic diagram

Truth Table

Physics

Chapter 1 Electric Field ( Electric Dipole) (Summary)

Here we will derive Expression of electric field at broad side

On position of dipole

Execution

Q With the help of a labelled diagram obtain an expression for the electric field intensity E at any point on the equitorial line ( broad-side on position) of an electric dipole

Ans

E1 E1sinθ

E θ P E1 θ

( r2+L2)12 E2 E

r E2 E2sinθ

-q θ L O L +qA B

Let us consider that the point P is situated on the right bisector of the dipole AB at a distance r meter from its midpoint O

Let E1 and E2 be the electric field intensities of the electric field at P due to charge +q and ndashq of the dipole resp The distance of P from each charge is ( r2+L2)12

So E1 = 14 πϵ q

(r 2+L 2) away from +q

E2 = 14 πϵ q

(r 2+L 2) towards ndashq

The magnitudes of E1 and E2 are equal but directions are different Now resolving E1 and E2 into two components parallel and perpendicular to AB we get

The components perpendicular to AB E1sinθ and E2sinθ cancel each other because they are equal and opposite

The components parallel to AB are E1cosθ and E2 cosθ are in same direction and add up

So resultant intensity of electric field at the point P is

E = E1cosθ + E2 cosθ

E = 14 πϵ q

(r 2+L 2) 2 cosθ

Now from fig we have cosθ =BOBP = L (r2+L2)12

So we get E = 14 πϵ 2qL ( r2+L2)32

Now electric dipole moment p= 2qL

So E = 14 πϵ p ( r2+L2)32

HW Find the expression of Electric field as done here but this time take r gtgt 2L

Also find the expression of torque experience by a dipole

(Hint Electric force experienced by charges of dipole in electric field is qE each Let θ be the angle which dipole makes with electric lines of force then perpendicular distance between two charges is 2Lsinθ Then torque = force x perp distance = qE x 2L sinθ So τ=pE sinθ where p =2qL )

STUDY MATERIAL

Class XIISubject Eng Literature (The Tempest ndash William Shakespeare) Topic Act IV Scene 1 Lines 84 to 133 (Iris hellip A contract of true love Be not too late ) Date 27th April 2020 (4th Period)

[Students should read the original play and also the paraphrase given in the school prescribed textbook]Summary Questions amp Answers

o Ceres soon appears and comes to know that she has been summoned to celebrate the contract of true love

o Ceres expresses her unwillingness to meet Venus and Cupid as she has shunned their company

o Ceres and Juno both bestow their blessings upon Ferdinand and Miranda with June gifting honour riches happiness in marriage and Ceres presents plenty of earthrsquos produce

o Iris summons the water-nymphs and reapers to come and celebrate a contract

(1) IRIS Of her society (Line 91-101)

Be not afraid I met her deity

Cutting the clouds towards Pathos and her sonDove-drawn with her Here thought they to have doneSome wanton charm upon this man and maidWhose vows are that no bed-right shall be paidTill Hymens torch be lightedmdashbut in vainMarss hot minion is returned againHer waspish-headed son has broke his arrowsSwears he will shoot no more but play with sparrowsAnd be a boy right out

(i) Where were Venus and Cupid seen flying How were they travelling Why did they want to join the marriage celebration of Ferdinand and Miranda

of true love

Venus and Cupid were seen flying through the air towards Paphos the famous city which is situated on the island of Cyprus They were travelling by air-borne chariot drawn by doves They certainly wanted to come here in order to play some amorous trick upon Ferdinand and Miranda who are under a vow not to gratify their physical desires till the holy ceremony of their marriage has been performed(ii) What have Venus and Cupid done after failing in their plan

After being failure of their plan Venus who is a very passionate deity and who is the mistress of Mars (the god of war) has gone back while here ill-tempered son Cupid has broken his arrows of love in his state of desperation(iii) What has Cupid firmly decided

Cupid is feeling so disappointed that he has firmly decided to shoot no more arrows to arouse love in human hearts but to spend his time playing with sparrows Thus he would now become just a boy and would give up his original function of shooting arrows on human beings to make them fall in love(iv) What vow had Ceres taken How did Ceres feel at the abduction

After the abduction of her daughter Prosperina by Pluto Ceres had taken a vow to always keep away from the disgraceful company of Venus and her blind son Cupid the god of love Ceres felt deeply distressed when Pluto had carried off her daughter and had made her his wife by force(v) Why has Ceres not forgiven Venus and her blind son For what do Ceres want to be sure

As the abduction had been manipulated by Venus the goddess of beauty and love and her blind son Cupid Ceres has never forgiven them for their part in the whole plot Ceres wants to be sure that she would not have to meet Venus and Cupid who had engineered the abduction of her daughter Prosperina

AS THIS lsquoMASQUErsquo SCENE IS VERY IMPORTANT IN THE PLAY THE PARAPHRASE OF THE ENTIRE PORTION OF MASQUE SCENE (Act IV Lines 58 to 143) IS GIVEN BELOW

IRIS Goddess of RainbowCERES Goddess of Agriculture and all the fruits of the earth

(Nature growth prosperity rebirth ndash notions intimately connected to marriage)JUNO The majestic Queen of Heavens and wife of Jupiter (Jupiter is the king of Gods)

VENUS The Goddess of love CUPID Son of Venus PLUTO God of death (In the play referred by Shakespeare as lsquoDisrsquo which is a Roman name for Pluto)

ORIGINAL TEXT PARAPHRASEPROSPEROWellmdash

PROSPERONow come Ariel Let there be too many rather than too few

Now come my Ariel Bring a corollaryRather than want a spirit Appear and pertly[to Ferdinand and Miranda]No tongue all eyes Be silent

spirits in attendance Appear briskly

[to Ferdinand and Miranda]Look with your eyes but do not say a word

[Soft music] [Soft music][Enter Iris] [Enter Iris]

IRISCeres most bounteous lady thy rich leasOf wheat rye barley vetches oats and peasThy turfy mountains where live nibbling sheepAnd flat meads thatched with stover them to keepThy banks with pioned and twilled brimsWhich spongy April at thy hest betrimsTo make cold nymphs chaste crowns and thybroom-grovesWhose shadow the dismissegraved bachelor lovesBeing lass-lorn thy pole clipped vineyardAnd thy sea-marge sterile and rocky-hardWhere thou thyself dost airmdashthe Queen othrsquoSkyWhose watery arch and messenger am IBids thee leave these and with her sovereign grace[Juno appears] Here on this grass-plot in this very placeTo come and sport Her peacocks fly amainApproach rich Ceres her to entertain

IRISCeres most generous lady you are the cause of rich fields or fertile land where wheat rye barley beans oats and peas grow the grassy mountains where the sheep graze and the flat meadows covered with coarse hay to be used as fodder for cattleYour banks are covered with marsh-marigolds and reeds and the rainy April under your orders brings forth to make for the maids who are not in love beautiful crowns your woods where the broom flourishes and where the bachelor who has been dismissed by the maid he loved lies down being forsaken your vineyard in which the poles are embraced by the vines and the margin of the sea which is barren and rocky where you roam about to enjoy the fresh air ndash the queen of the sky (Juno) whose messenger I am besides being represented as the rainbow bids you leave all these and with her majesty here on this grassy plot in this very place come and sport her peacocks carry her fast in her chariot through the air and are making their way here approach rich Ceres to welcome her

[Enter Ariel as Ceres] [Enter Ariel as Ceres]

CERESHail many-coloured messenger that neerDost disobey the wife of JupiterWho with thy saffron wings upon my flowersDiffusest honey-drops refreshing showersAnd with each end of thy blue bow dost crownMy bosky acres and my unshrubbed downRich scarf to my proud earth Why hath thy queenSummoned me hither to this short-grassed green

CERESWelcome rainbow that never dared disobey Juno the wife of Jupiter who with your orange coloured rays spread honey-drops refreshing showers And with each end of thy blue bow drown my bushy acres and my hilly country which is free from shrubs you thus forming a rich scarf Why has your queen called me here to this place covered with short grass

IRISA contract of true love to celebrateAnd some donation freely to estateOn the blest lovers

IRISI have called you to celebrate a contract of true love and bestow some liberal gift upon the blessed lovers

ORIGINAL TEXT PARAPHRASECERESTell me heavenly bowIf Venus or her son as thou dost knowDo now attend the queen Since they did plotThe means that dusky Dis my daughter gotHer and her blind boys scandaled companyI have forsworn

CERESTell me heavenly bow if Venus the Goddess of love or Cupid her son and pedlar of passion at this time attend the heavenly queen Juno because you are sure to know Since the day they conspired against me and dark Pluto took away my daughter here and Cupidrsquos disgraceful company I have left off

IRISOf her societyBe not afraid I met her deityCutting the clouds towards Pathos and her sonDove-drawn with her Here thought they to have doneSome wanton charm upon this man and miad

IRISBe not afraid of her company I met her deity moving on the clouds towards Paphos the sacred home of Venus on the island of Cyprus along with her son on her chariot drawn by doves Here they contemplated to exercise a charm upon this man and maid producing

Whose vows are that no bed-right shall be paidTill Hymens torch be lightedmdashbut in vainMarss hot minion is returned againHer waspish-headed son has broke his arrowsSwears he will shoot no more but play with sparrowsAnd be a boy right out

wantonness before the actual marriage ceremony but did not succeed Venus has returned her irritable son has broken his arrows and swears that he will give up his practice of trying to inspire love but play with sparrows and be a boy again

[Music is heard] [Music is heard]

CERESHighst queen of stateGreat Juno comes I know her by her gait

CERESHighest queen of state Great Juno there she comes I know here by her gait

[Enter Juno] [Enter Juno]

JUNOHow does my bounteous sister Go with meTo bless this twain that they may Prosperous beAnd honoured in their issue

JUNOHow are you doing my generous sister Come with me to bless this couple so that they may be prosperous and fortunate in their children

[They sing] [They sing]

JUNOHonour riches marriage-blessingLong continuance and increasingHourly joys be still upon youJuno sings her blessings upon you

JUNOMay honour riches happiness in marriage long continuance and increase of those boons ever rest upon you as hourly joys Juno showers down upon you her blessings in song

CERESEarths increase foison plentyBarns and garners never emptyVines and clustring bunches growingPlants and goodly burden bowingSpring come to you at the farthestIn the very end of harvestScarcity and want shall shun youCeresrsquo blessing so is on you

CERESMay you have the plenty of earthrsquos produce Your barns and granaries may never be empty Your vines may grow with clustering bunches Your fruit trees may be heavily laden with their fruit May there be continuous spring and harvest May scantiness and want leave you forever Such is the blessing of Ceres upon you

FERDINANDThis is a most majestic vision andHarmoniously charmingly May I be boldTo think these spirits

FERDINANDThis is a great vision and magically melodious Should I suppose the characters (taking part in the masque) are spirits

PROSPEROSpirits which by mine artI have from their confines calld to enactMy present fancies

PROSPEROYes they are spirits whom I have summoned from the regions to which they are confined to carry into effect my fanciful designs

ORIGINAL TEXT PARAPHRASEFERDINANDLet me live here everSo rare a wondered father and a wifeMakes this place paradise

FERDINANDI should like to live here forever Such a wise and wonderful father makes this place a paradise

[Juno and Ceres whisper and send Iris on employment] [Juno and Ceres whisper and send Iris on employment]

PROSPEROSweet now silence

PROSPEROMy dear Ferdinand speak no more Juno and Ceres are

Juno and Ceres whisper seriouslyTheres something else to do Hush and be muteOr else our spell is marred

whispering with a solemn look There is something else coming Silence Or else our magic will be spoilt

IRISYour nymphs called naiads of the wandering brooksWith your sedged crowns and over-harmless looksLeave your crisp channels and on this green landAnswer your summons Juno does commandCome temperate nymphs and help to celebrateA contract of true love Be not too late

IRISYou nymphs called Naiads denizens (M inhabitants) of the running stream with your chaplets of sedge and ever-helpful looks leave your wrinkled channels and on the green land answer the summons sent to you Juno has ordered some chaste nymphs and help to celebrate a noble and true marriage Donrsquot delay

[Enter certain nymphs] [Enter certain nymphs]You sunburnt sicklemen of August wearyCome hither from the furrow and be merryMake holiday your rye-straw hats put onAnd these fresh nymphs encounter every oneIn country footing

You sunburnt harvesters weary from the effects of the heat in August come here from the furrowed land and rejoice Make holiday with your rye-straw hats upon you and meet these fresh nymphs and join in country dancing

[Enter certain reapers properly habited They join with the nymphs in a graceful dance towards the end whereof Prospero starts suddenly and speaks]

[Enter certain reapers properly habited They join with the nymphs in a graceful dance towards the end whereof Prospero starts suddenly and speaks]

PROSPERO[aside] I had forgot that foul conspiracyOf the beast Caliban and his confederatesAgainst my life The minute of their plotIs almost come [to the spirits]Well done Avoidno more

PROSPERO(Aside)I had forgotten the wicked conspiracy of the beast Caliban and his accomplices against my life the time of their plot has almost arrived ndash (To the Spirits) well done depart no more of this

[To a strange hollow and confused noise the spirits heavily vanish]

[The spirits depart]

ORIGINAL TEXT PARAPHRASEFERDINANDLet me live here everSo rare a wondered father and a wifeMakes this place paradise

FERDINANDI should like to live here forever Such a wise and wonderful father makes this place a paradise

[Juno and Ceres whisper and send Iris on employment] [Juno and Ceres whisper and send Iris on employment]

PROSPEROSweet now silenceJuno and Ceres whisper seriouslyTheres something else to do Hush and be muteOr else our spell is marred

PROSPEROMy dear Ferdinand speak no more Juno and Ceres are whispering with a solemn look There is something else coming Silence Or else our magic will be spoilt

IRISYour nymphs called naiads of the wandering brooksWith your sedged crowns and over-harmless looksLeave your crisp channels and on this green landAnswer your summons Juno does commandCome temperate nymphs and help to celebrateA contract of true love Be not too late

IRISYou nymphs called Naiads denizens (M inhabitants) of the running stream with your chaplets of sedge and ever-helpful looks leave your wrinkled channels and on the green land answer the summons sent to you Juno has ordered some chaste nymphs and help to celebrate a noble and true marriage Donrsquot delay

[Enter certain nymphs] [Enter certain nymphs]You sunburnt sicklemen of August wearyCome hither from the furrow and be merryMake holiday your rye-straw hats put onAnd these fresh nymphs encounter every oneIn country footing

You sunburnt harvesters weary from the effects of the heat in August come here from the furrowed land and rejoice Make holiday with your rye-straw hats upon you and meet these fresh nymphs and join in country dancing

[Enter certain reapers properly habited They join with the nymphs in a graceful dance towards the end whereof Prospero starts suddenly and speaks]

[Enter certain reapers properly habited They join with the nymphs in a graceful dance towards the end whereof Prospero starts suddenly and speaks]

PROSPERO[aside] I had forgot that foul conspiracyOf the beast Caliban and his confederatesAgainst my life The minute of their plotIs almost come [to the spirits]Well done Avoidno more

PROSPERO(Aside)I had forgotten the wicked conspiracy of the beast Caliban and his accomplices against my life the time of their plot has almost arrived ndash (To the Spirits) well done depart no more of this

[To a strange hollow and confused noise the spirits heavily vanish]

[The spirits depart]

Ac-12 27420 topic Revaluation of Assets and Liabilities

REVALUATION OF ASSETS AND LIABILITIES

On admission of a new partner the firm stands reconstituted and consequently the assets are revalued and liabilities are reassessed It is necessary to show the true position of the firm at the time of admission of a new partner If the values of the assets are raised gain will increase the capital of the existing partners Similarly any decrease in the value of assets ie loss will decrease the capital of the existing partners For this purpose alsquoRevaluation Accountrsquo is prepared This account is credited with all increases in the value of assets and decrease in the value of liabilities It is debited with decrease on account of value of assets and increase in the value of liabilities The balance of this account shows a gain or loss on revaluation which is transferred to the existing partnerrsquos capital account in existing profit sharing ratioAccounting for Revaluation of Assets and Liabilities when there is a Changein the Profit Sharing Ratio of Existing PartnersAssets and liabilities of a firm must also be revalued at the time of change in profit sharing ratio of existing partners The reason is that the realisable or actual value of assets and liabilities may be different from those shown in the Balance Sheet It is possible that with the passage of time some of the assets might have appreciated in value while the value of certain other assets might have decreased and no record has been made of such changes in the books of accounts Similarly there may be some unrecorded assets amp libilities that may have to be accounted for Revaluation of assets and reassessments of liabilities becomes necessary because the change in the

value of assets and liabilities belongs to the period to change in profit sharing ratio and hence must be shared by the partners in their old profit sharing ratio Revaluation of assets and reassessment of liabilities may be given effect to in two different ways (a) When revised values are to be recorded in the books and(b) When revised values are not to be recorded in the books

When revised values are to be recorded in the booksIn such a case revaluation of assets and reassessment of liabilities is done with the help of a new account called lsquoRevaluation Accountrsquo Sometimes this account is also called as lsquoProfit amp Loss Adjustment Acrsquo If there is a loss due to revaluation revaluation account is debited and if the revaluation results in a profit the revaluation account is credited The following journal entries made for this purpose are

(i) For increase in the value of assetsAsset Ac Dr (individually)To Revaluation Ac(ii) For decrease in the value of AssetRevaluation Ac Dr (individually)To Asset Ac[Decrease in the value of assets](iii) For increase in the value of LiabilitiesRevaluation Ac Dr (individually)To Liabilities Ac[Increase in the value of Liabilities](iv) For decrease in the value of LiabilitiesLiabilities Ac DrTo Revaluation Ac[Decrease in the value of Liabilities](v) For unrecorded AssetsAsset Ac [unrecorded] DrTo Revaluation Ac[Unrecorded asset recorded at actual value](vi) For unrecorded Liability Revaluation Ac DrTo Liability Ac [unrecorded][Unrecorded Liability recorded at actual value](vii) For transfer of gain on revaluationRevaluation Ac DrTo Existing Partnerrsquos CapitalCurrent Ac[Profit on revaluation transferred to capital account in existing ratio](viii) For transfer of loss on revaluationExisting Partnerrsquos CapitalCurrent Ac DrTo Revaluation Ac[Loss on revaluation transferred to capital account in existing ratio](a) When revaluation account shows gain Revaluation Ac DrTo Partnerrsquos Capital Ac (Old Profit Sharing Ratio)(Profit on revaluation credited to Partnerrsquos Capital Ac)(b) Above entry is reversed when revaluation account shows loss Partners Capital Acs (Old Profit Sharing Ratio) DrTo Revaluation Ac(Loss on revaluation debited to Partnerrsquos Capital Acs)

Proforma of Revaluation Account is given as under

Revaluation Account

Dr Cr Particulars ` Amount Particulars ` Amount To Decrease in value of assets By Increase in value of assets To Increase in value of liabilities By Decrease in value of liabilities To Unrecorded liabilities By Unrecorded assets To Gain on Revaluation (Transferred) By Loss on Revalution (Transferred)

ECO ndash12 2742020Topic- ELASTICITY OF DEMAND

CHAPTER - ELASTICITY OF DEMANDMEANINGDemand for a commodity is affected by many factors such as its price price of related goods income of its buyer tastes and preferences etc Elasticity means degree of response Elasticity of demand means degree of responsiveness of demand Demand for a commodity responds to change in price price of related goods income etc So we have three dimensions of elasticity of demandDIMENSION OF ELASTICITY OF DEMAND TYPES OF ELASTICITY OF DEMAND

Price elasticity of demand Income elasticity of demand Cross Elasticity of demand

Price elasticity of demand Price elasticity of demand means degree of responsiveness of demand for a commodity to the change in its price For example if demand for a commodity rises by 10 due to 5 fall in its price Price elasticity of demand (ep)=Percentage change in quantity demanded Percentage change in price of the commodity = 10 ( -)5 = ( - )2Note that ep will always be negative due to inverse relationship of price and quantity demanded

(ii) Income elasticity of demand Income elasticity of demand refers to the degree of responsiveness of demand for a commodity to the change in income of its buyer Suppose income of buyer rises by 10 and his demand for a commodity rises by 20 then Income elasticity of demand (ey)= change in quantity demanded change in price of the commodity =20 10 = 2

Cross Elasticity of demandCross elasticity of demand means the degree of responsiveness of demand for a commodity to the change in price of its related goods (substitute goods or complementary goods) Suppose demand for a commodity rises by 10 due to 5 rise in price of its substitute good then Cross elasticity of demand (ec) = change in quantity demanded change in price of related good = 10 2 = 5 (Tastes and preferences cannot be expressed numerically So elasticity ofdemand cannot be numerically expressed)

  • Chapter 1 Force (Summary)
  • Distinguish between external sovereignty and internal sovereignty
    • NAND Gate
      • Logic diagram
      • Truth Table
        • NOR Gate
          • Logic diagram
          • Truth Table
            • XOR Gate
              • Logic diagram
              • Truth Table
                • XNOR Gate
                  • Logic diagram
                  • Truth Table
                      • Physics
                      • Chapter 1 Electric Field ( Electric Dipole) (Summary)
Page 14:  · Web viewWe all know that Nouns are divided into two parts: common noun and proper noun.Apart from common and proper noun, we will also study about collective noun and compound

Spot heights- The spot heights show heights in metres above the mean sea level

SUBJECT- English Language CHAPTER - The Sentence

Complex sentence-

We have seen that a Complex Sentence consists of a Principal Clause with one or more Subordinate Clauses

We have also learnt that there are three kinds of Subordinate Clauses The Adjective Clause the Adverb Clause and the Noun Clause

1) THE ADJECTIVE CLAUSE

An ADJECTIVE CLAUSE does the work of an ADJECTIVE It qualifies a noun or pronoun In the following examples in each set two commonly patterned sentences are compared one with an ADJECTIVE and the other with an ADJECTIVE CLAUSE ( The Adjective or Adjective Clause qualifies the Noun placed in the box)

a) He is a lazy boy (Adjective)

b) He is a boy who is lazy (Adjective Clause)

a) It is a beautiful place for the tourists (Adjective)

b) It is a place where tourists come for the scenic beauty (Adjective Clause)

a) We have enough funds for the work (Adjective)

b) We have funds which would be enough for the work (Adjective Clause)

Convert the sentence from simple to complex sentence-

1) We believe his honesty Ans- We believe that he is honest

2) This is the birth place of RamaAns- This is the place where Rama was born

____________________________________________________________________________________________

Class IX

Subject English Language

Topic Preposition

Date 270420

PREPOSITIONS

A preposition is a word placed before a noun or a pronoun to show in what relation the person or thing denoted by it stands in regard to something else

EXAMPLE

There is a clock on the wall

The preposition on shoes relationship between clock and the wall

Smitha is afraid of lizards The man jumped off the bus

Here the preposition of shows the relationship between afraid and lizards

The preposition off shows the relation between jumped and bus

The noun or the pronoun which follows a preposition is called its object

So in the first Example wall is the object of the preposition on

A preposition can have more than one object For example

The plane flew overhouses and meadows

Prepositions are used to express a number of relationship including time locationmannermeans quantity purpose and state or condition

Points to remember

Preposition joins a noun to another noun or a pronounThere is a cow in the field

A preposition joints a noun to a verbThe cat runs after the rat

Preposition can have two or more than two objectsThe road runs over hills and plain

A preposition also joins a noun an adjectiveHe is fond of tea

Generally a preposition comes before an object Sometimes it comes even after an object asWhat are you looking atThis is the house I live in

Subject- Computer Application

Chapter 2 Introduction to Java

Java API An application programming interface (API) in the context of Java is a collection of prewritten packages classes and interfaces with their respective methods fields and constructors

Byte Code Java bytecode is the result of the compilation of a Java program an intermediate representation of that program which is machine independent The Java bytecode gets processed by the Java virtual machine (JVM) instead of the processor JVM The Java Virtual Machine (JVM) is the runtime engine of the Java Platform which allows any program written in Java or other language compiled into Java bytecode to run on any computer that has a native JVM

Platform A platform is the hardware or software environment in which a program remains

Java platform The Java platform differs from most other platforms in that itrsquos a software only platform that runs on top of other hardware-based platforms

Applet and Application The fundamental difference between the two Java programs is that an application program is designed to run on a stand-alone machine whereas an applet is a web-version of an application which is used to run a program on a web browser

WORA ldquoWrite once run anywhererdquo (WORA) or sometimes write once run everywhere (WORE) is a slogan created by Sun Microsystems to illustrate the cross-platform benefits of the Java language

Class XSubject Topic Summary Execution

ECONOMICSFACTORS OF PRODUCTION Sub-topiclsquoLABOURrsquo

We shall start our class by discussing the topic taught in the last class

lsquo DIVISION OF LABOURrsquo- By division of labour we mean specialization in workIt refers to splitting up the work of labour involved in the production of a particular commodity into several parts and each part and sub-part is performed by a specialist

Now let us start by the Advantages of Division of labouraIt increases the level

Questions

1 Differentiate between Product-Based division of labour and Process-Based division of labour

Product-Based division of labour

Process-based division of labour

It is also known as simple (or occupational) division of labourUnder it everybody performs a particular occupations The entire is done by the same person

When a person or group of persons undertakes a specialised function which is supplementary to the production of final commodity and service This is also as complex division of labour

It is simple It is complex

of productionbSince the product is produced by an expert workerbest quality of product is producedcIt saves time and toolsd it promotes inventions in the methods and techniques of productioneIt leads to reduction in costs fAll workers get work according to their abilities and choices

Now let us discuss the disadvantages of Division of labour

a Since many workers are involved in the production of a commodity no one has the sense of responsibility

b The constant and repetition of the same work again and again make the work monotonous

c Division of labour facilitates production on large scale Hencethere is fear of over production

d Because of territorial division of labour some areasregions become more developed than others

It is based on labour-intensive techniques of production

It is based on capital-intensive techniques

It is generally found in small enterprises

It is generally found in large enterprises

Example Indian farmers doing all farm activities

Example A modern garmet factory where one person takes the measurementanother does the cuttingsome sew the clothes while a few workers button them and other iron them

2 What are the advantages of Division of labouraIt increases the level of productionbSince the product is produced by an expert workerbest quality of product is producedcIt saves time and toolsd it promotes inventions in the methods and techniques of productioneIt leads to reduction in costs fAll workers get work according to their abilities and choices

3Discuss the disadvantages of Division of laboura Since many workers are involved in the

production of a commodity no one has the sense of responsibility

b The constant and repetition of the same work again and again make the work monotonous

c Division of labour facilitates production on large scale Hence there is fear of over production

d Because of territorial division of labour some areasregions become more developed than others

English 1 Transformation of sentences

Sentences A sentence is a group of words which makes complete sense

a Assertive sentences

Exercise 6Rewrite the following sentences according to the instructions given below without changing their meanings

1 As soon as he saw the beer he jumped into

b Imperative sentences

c Interrogative sentences

d Exclamatory sentences

Sentences can be changed from one grammatical form to another without changing the meaning of the sentence This is known as transformation of sentences

the river ( Begin No sooner)2 None but brave deserve the fair (Begin the

bravehellip)3 This box is too heavy for me to lift ( Use so hellip

That instead of too)4 No one other than a king can live like James

Luxurious ( Begin only James)5 Oh for the wings of a dove (Begin I wishhellip)

Math Topic Commercial MathematicsChapter Shares and Dividends

Study item Discuss about shares and Dividends1) What is share

Ans To start any big business (company or Industry) a large sum of money is needed But it is not possible for an individual to invest such a large amount Then some persons interested in the business join together and from a company They divide the estimated money required into small parts Each such part is called a share

2) What do you mean by the term shareholder

Ans A person who purchases one or more shares is called shareholder3) Some terms related with a share

(i) Nominal value or face value or printed value The original value of a share is called its nominal value or face value or printed value

Note The nominal value of a share always remains same(ii) Market value or cash value The price of a share at any

time is called its market value or cash value

Note The market value of a share changes from time to time(iii) At par If the market value of a share is the same as its

nominal value the share is called at par(iv) At Premium or above Par If the market value of a share

is more than its nominal value the share is called at premium or above par

Example If a share of Rs 100 is selling at Rs 150 then it is said to be selling at a premium of Rs 50 or Rs 50 above par

(v) At Discount or below par If the market value of a share is less than its nominal value the share is called at discount or below par

Example If a share of Rs 100 is selling at Rs80 then it is said to be selling at a discount of Rs 20 or at Rs 20 below par

4) What is Dividend

Ans The profit which a shareholder gets for hisher investment from the company is called dividendNote (i) The dividend is always expressed as the percentage of the face value of the share(ii) The dividend is always given( by the company ) on the face value of the share

irrespective of the market value of the shareBENGALI(2ND LANGUAGE)

ldquoদেবতোর জণমrdquoলিবরোম চকরবত

পরথম লিসর পোঠ-চোর পসথ একটি পোথর লিবপলি ঘটোয় দেক যোতোয়োসতর পসথ পরলিতলিয়ত ওই পোথসর দেো োচট দেসত একলি দেতো দেক দেো োচট দেসয় দেবোমো সয় রোসতোর মোস লি0টসক পস1 লিবপরীত লিক দেথসক আো একটি দেমোটর োলি1 চোসকর কষতোয় পরোসরণ দেবোসচ যো লিকনত পরলিতবোর এমরণ দেৌভোয দেসকর োও সত পোসর তোই লিতলি দেকোো দেজোো1 কসর পোথরটিসক উপস1 দে8স পোথর উপস1 দে8োর ময় এক দেকৌতী জতো দেকসক পরশন কসর দেয লিতলি দেকোসো দেবতোর আস দেপসয়স0রণ লিকো লিকনত দেক বস লিতলি দেকোসো দেবতোর আস পোলি দেক উপলিত ক জতোর উসltসয বস কোরও ইস= স পোথরটি লিসয় দেযসত পোসর এর পর দেথসক দেক দেযসত আসত পোথরটি দেক দেসত পো একলি দেক কষয করস দেকউ পোথরটিসক ধসয়মস0 পলিরসকোর করস0 দেক ওই দেকৌতী জতোসক পোথসরর কোস0 বস থোকসত দেস এ0ো1ো আরও কষয কসর দেক ঠোৎ ওই লি1 পোথরটির োসয় লিোর োো-দেকউ পজো কসরস0 দেকৌতী দেোকটির আঙকো য় যলিসক উ পোথরটিসক লিরসয় দে8স তোর পর ঠোৎ একলি পোথরটির দেোোজ দেই দেক লিসয় দেস0 বো দেকোথোয় দেস0

বদোথ-

দেো োচট ndashচসত লিসয় দেকো লিক0র সE ধোককো দেস পস1 যোবোর উপকরমঅকসমোৎ- ঠোৎআতমমবর- লিসজসক লিয়নতরপ-পো লিপ0স প1োদেসতসসত- লিসপলিউৎোত- দেো1ো দেথসক উপস1 দে8োপরতযয় ndash লিবশবোপরতযোস- টতযোস ndash দৈবোসধসতোধলিসত- পরসপসরর পরলিত ব পরসয়ো করোপরসতরীভত- পোথসর পলিররণলিতবোনতঃকরসরণ- মস পরোসঅলিQৎ- অQো করসত ই=কদেোপ- বধমোর- দেবোস এমইতযোকোর- এইরকমরম- বময়পযসোভী-পসযর জয দেোভ আস0 যোরপোসথয় ঞচয়- পথচোর রচ জমোসোমোমোসরোস- ব ধম ধোসমর সE

তোর লি দেইhelliphellipTo be continued

Hindi 2nd lang सर क पद(सरदास)

सरदास शरी कषण भकति कावय क सरवशरषठ कगिरव ह इनक जनम और मतय क समय तथा सथान का मतभद हसरदास रवातसलय और शरार रस क अनयतम कगिरव ह इनक कावय म बालकषण क सौदय चपपल चषटा और गि7याओ की मनोहर झाकी मिमलती ह कषण और ोगिपयो क अननय परम का कतिचतरण ह सयो शरार की अपकषा उनक कावय म गिरवयो शरार का अमिBक गिरवषय और मारमिमEक कतिचतरण हआ हइन पकतियो म हम सरदास की भकति भारवनाओ का परिरचय मिमलता ह इनका सपण सगरह सरसार म गिनगिहत ह

1 जसोदा हरिर पालन झलारवहलरारवदलराईमलहारव रव जो ईसाई कछ ारव मर लाल को आई निनEदिदया काह ना आगिनसबारव त काह नाही बरवगिह आरवतोको कानहा बलारव

शबदाथ-हलरारव-गिहलती हदलराई - दलार पयार करती हमलहारव-पचकारती हनिनEदरिरया ndashनीदरवगिह-जलदी सअBर-होठमौन-चपसन-सकत

वयाखया- सरदास जी कहत ह गिक यशोदा माता बालक कषण को पालन म झल आती ह रवह उनह गिहलाती ह पयार करती ह मलहार जस कोई ीत ान लती ह और नीद स पछती ह गिक ह नीद तम मर लाल को आकर कयो नही सलाती तझ खाना बला रहा ह कभी कषण आख बद कर लत ह कभी आखफडफडान लत ह उनह सोता हआ जानकर यशोदा माता चप हो जाती ह और इशार म बात करन लती ह इसी बीच अकला कर कषण ज जात हतो गिफर यशोदा माता गिफर स ाना ान लती ह सरदास जी कहत ह गिक भरवान क दशन का सख दरवता और ऋगिष-मगिनयो को भी दलभ ह यही सख माता यशोदा को बडी सहजता स मिमल जा रही ह माता यशोदा बहत ही भागयशाली ह2)Continue to nexthellip

Physics

Chapter 1 Force

(Summary)

Question A body is acted upon by two forces each of magnitude F but in opposite directions State the effect of the forces if

(a) Both forces act at the same point of the body

(b)the two forces act at two different points of the body at a separation r

Solutions

(a) Resultant force acting on the body = 0

F ndash F = 0(b) The forces tend to rotate the body between two forces about the midpoint

Moment of forces = F times rFr

QuestionDefine moment of a couple Write its SI unit

Solutions

Moment of couple is equal to the product of both force and the perpendicular distance between the two forces

The SI unit of moment of couple is NmCommercial Studies

Advertising and sales

Business firms use several methods to

Questions1) What do you mean by advertising

promotion create demand of their product in the market and increase it sales Such methods comprises of advertising sales promotion personal selling and publicityToday we are going to discuss about one of such methods It is advertising

Meaning of advertisingAdvertising is a paid form of non-personal presentation for promotion of Ideas goods and services

Importance or merits of advertising Advertising has importance to manufacturer or traders to customer and to society as a whole

Today we will see how advertisement help the manufacturer or traders

Answer) Advertising is a means of how a company encourages people to buy their products services or ideas It is one element of marketing which also includes design Research and data mining

2) Mention any three features of advertisingAnswer)The main features of advertising are

i) It is impersonal form of presentation for promotion of products and services of Ideas

ii) It is issued by identified sponsor The advertisement contains the name of the advertiser

iii) It is a form of mass communication because the message is directed to a large number of persons simultaneously

3) Mention the main merits or importance of advertisement to manufacturer or tradersAnswer)

i) Introducing new product A business organization can introduce itself and its products to the public through advertising

ii) Increase the sale Advertising leads to increase the sale of existing product by entering into new markets and attracting new customers

iii) Create steady demand Advertising creates sustains regular demand by smoothening out seasonal and other fluctuations It enables regular production for the organisation

iv) Economics of scale Advertising facilitate mass distribution of goods and steady demand which lead to large scale and regular production

v) Goodwill Advertising helps in creating a good image of the firm and reputation for its products

Biology Chapter - 03Genetics

Today wewill start chapter and discuss about Genetics Gregor Mendel is known as father of genetics Before entering into Mendelrsquos experiment on Genetics we must know

Q1 Define the following termsi) Genetics Genetics is the study of

transmissionof body features from parents to offspringand the laws relating to such transmission

ii) Heredity It may be defined as transmissionof genetically based characteristics from parentsto offspring

iii) Character and traits Any heritable

Importance to TraderIntroducing new productIncrease the saleCreate steady demandEconomics of scaleGoodwill

some terms featureis a character The alternative forms of acharacter are called traitsex Character (Hair shape) - Traits (Curly straight)

iv) Homologous chromosomes A pair ofcorresponding chromosomes of the same shapeand size one from each parent

v) Genes Genes are the specific parts (DNA segments) of a chromosome which determinethe hereditary characteristicsNearly 30000genes present in human

vi) Alleles Alternative forms of a gene occupying the same position (locus) on homologouschromosomes and affecting the same characteristicbut in different ways

vii) Genotype ndash PhenotypeGenotype means of genes present in the cells of an organism Phenotype means the observable characteristic which is genetically controlled

viii) Mutation It is a sudden change in one or more genes or in the number or in the structure of chromosomes ex Sickle cell anaemia is a blood disease caused by a gene mutation

CLASS NOTES

Class XSubject Eng Literature (The Merchant of Venice ndash William Shakespeare)Topic Act IV Scene 1 Lines 01 to 34 ( Duke helliphelliphellip We all expect a gentle answer Jew) ate 27th April 2020 (2nd Period)

[Students should read the original play and also the paraphrase given in the school prescribed textbook]Summary Questions amp Answers

This scene may be termed as the catastrophe of the play It is the final unravelling of the complicated events which seem to threaten the happiness of Bassanio Portia and Antonio Right is justified to the fullest degree and malice falls into the trap prepared for others No one suffers here but Shylock but even then he receives a generous measure of mercy

o This is the Court-scene Initially we meet

(1)

DUKE I am sorry for thee thou art come to answer (Line 3-6)A stony adversary an inhuman wretchUncapable of pity void and emptyFrom any dram of mercy

(i) Who is addressed here Where is the person Why is the person there

Antonio is addressed hereAntonio is in the court of justice at VeniceAntoniorsquos trial is scheduled to be held here for his failure to meet the conditions of the bond he signed with Shylock

the Duke Antonio Shylock and Salerio Later we meet Bassanio Portia Gratiano and Nerissa

o The Duke says to Antonio that he has to face a very cruel opponent which Antonio admits and expresses his gratefulness to the Duke for his efforts to soften without result the heart of Shylock in order to be merciful to Antonio Antonio further says that he is ready to accept whatever cruel judgement the Court may award

o When Shylock appears in the court the Duke says that Shylock should change his decision of prosecuting Antonio and demanding the penalty specified in the bond out of consideration of the great misfortunes that Antonio has suffered If this is done by him (Shylock) the whole court would be gladdened by his merciful action

(ii) What is the Duke sorry for

The Duke is unable to change the mind of Shylock from his decision to get the bond forfeited even after he pleaded to Shylock Shylock stands firmly for his bond which when forfeited will allow him to take a pound of flesh from any part of Antoniorsquos body(iii) How does the Duke address Shylock`The Duke calls Shylock an adversary with a heart of stones He calls Shylock as an inhuman wretch without pity Shylock is quite lacking in the slightest quality of mercy (iv) How does Antonio reply to this

Antonio replies that he will meet the revenge of Shylock patiently He has prepared himself to suffer with a quiet spirit the utmost that Shylockrsquos tyranny and rage can do(v) What quality of the Duke is revealed here

The Duke is kind and benevolent He is ready to help Antonio He requests Shylock to free Antonio from the trial(vi) What are the terms of the bond that Antonio has signed

The terms of the bond that Antonio has signed were that if Antonio is unable to repay Shylock a certain sum of money specified on the paper on a certain date and in an agreed place the forfeit has to be paid The forfeiture will be an exact pound of Antoniorsquos flesh which Shylock will be a liberty to take from any part of Antoniorsquos body which pleases him

Class XI

STUDY MATERIAL

Class XISubject Eng Literature (The Tempest ndash William Shakespeare) Topic Act I Scene 2 Lines 88 to 132 (Prospero hellip Me and thy crying self) Date 27th April 2020 (3rd Period)

[Students should read the original play and also the paraphrase given in the school prescribed textbook]Summary Questions amp Answers

o Prospero now tells Miranda that he was the Duke of Milan He had been devoting himself more to studies than the affairs of the State His brother Antonio took advantage of this situation and with the help of Alonso the king of Naples seized upon him and her one midnight and shipped them in a frail bark so that they perished in the sea All this took place

(1)

MIRANDA I should sin (Line 118-132)

To think but nobly of my grandmother

Good wombs have borne bad sonsPROSPERO Now the condition

The King of Naples being an enemyTo me inveterate hearkens my brothers suitWhich was that he in lieu othrsquo premisesOf homage and I know not how much tribute

twelve years back

IMPORTANT PASSAGES EXPLAINED(Line 98-103)

PROSPERO helliphelliphelliphelliphelliphelliphellip

He being thus lorded

Not only with what my revenue yieldedBut what my power might else exact like oneWho having into truth by telling of it Made such a sinner of his memoryTo credit his own lie he did believeHe was indeed the duke

Prospero in telling the narrative of his past life here refers to his brother Antonio Prospero being with a studious bent of mind has left the administration of Milan on his younger brother Now Antonio being thus invested like a lord with all the powers derived from Prosperorsquos wealth and what the exercise of Prosperorsquos authority might secure for him regarded himself as a de facto Duke of Milan It is a well-known fact of psychology that a man who repeatedly tells a lie makes of his memory such a sinner against truth as to credit his own lie by the telling of it So Antonio by repeatedly saying to himself and others that he was the Duke came to believe that he was really the Duke Thus falsehood repeatedly asserted gained the force of truth for Antonio and he truly believed it

Should presently extirpate me and mine Out of the dukedom and confer fair MilanWith all the honours on my brother whereonA treacherous army levied one midnightFated to thrsquo purpose did Antonio openThe gates of Milan and ithrsquo dead of darkness The ministers for thrsquo purpose hurried thenceMe and thy crying self

(i) In the earlier lines of this scene what does Prospero tell about his intense interest What was the demand of his interest

In the earlier lines of this scene Prospero tells Miranda that he had an intense interest in the study of philosophy and magic arts Hence in order to improve his mind with this kind of study he kept himself isolated from worldly and state affairs His study was dearer to him than the applause and esteem that he could win from the public His study demanded too much solitude(ii) What forced Antonio to take an undue advantage over Prospero

Prosperorsquos indifferent attitude towards the statersquos affairs and his having boundless trust in Antonio gave rise to a boundless lust for power in Antoniorsquos mind Antonio felt that he must be the actual Duke instead of the part of the Duke he played Thus Antonio took an undue advantage of the situation to usurp Prosperorsquos dukedom(iii) Explain the following lines ldquoI should sin to think but nobly of my grandmother Good wombs have borne bad sonsrdquo

After hearing the treacherous act of her uncle Antonio Miranda says that Prosperorsquos mother was a noble lady and she cannot dishonour her memory by saying that the person named Antonio cannot be his (Prosperorsquos) brother She says that in honour of her grandmother she also cannot say that Antonio must have been begotten not by her grandfather but by some other man She finally concedes that it is known that good mothers have borne bad sons in their wombs and gave birth to them

(iv) Why did the King of Naples accept Antoniorsquos request to help him in usurping his dukedom What did Antonio propose to Alonso

Alonso who was the king of Naples accepted Antoniorsquos request in usurping Prosperorsquos dukedom because he (Alonso) was a sworn enemy of Prospero Antonio proposed that Alonso should immediately drive him (Prospero) and his offspring out of Milan and should confer the dukedom upon him (Antonio) with all the dignities which go with that

position In return Antonio promised that he shall give an annual tribute and also swore his allegiance to Alonso Also he agreed to hold the Dukedom of Milan as a subordinate to the state of Naples(v) How were Prospero and Miranda carried away from the city of Milan and what was the state of small Miranda at that time

In pursuance of the agreement settled between Antonio and Alonso an army of treacherous men was assembled One midnight when the occasion suited the will of destiny Antonio opened the gates of the city of Milan and in the death like silence of midnight Antoniorsquos agents who had been directed to execute his purpose carried Prospero and small Miranda away from the city in all haste They were then forced into a ship and carried some distance out to sea where they put them on a mere hulk of a boat without any rigging or ship-gear and abandoned them leaving them at the mercy of the roaring sea Miranda was a very small child of three years age and she was crying at that time

CLASS -XIDATE-270420Subject Topic Summary Execution

EVS Chapter 1 ndash Mode of Existence

Impact of mode of existence on resources

Q) Why resources are under pressure

Ans - Increase in the sophistication

of technology enabling natural resources to be extracted quickly and efficiently Eg in the past it could take long hours just to cut down one tree only using saws Due to increased technology rates of deforestation have greatly increased

The number of humans is increasing Cultures of consumerism Materialistic views

lead to the mining of gold and diamonds to produce jewelry unnecessary commodities for human life or advancement Consumerism also leads to extraction of resources for the

production of commodities necessary for human life but in amounts excessive of what is needed because people consume more than is necessary or waste what they have

Lack of awareness among the population is striking People are not aware of ways to reduce depletion and exploitation of materials

Accounts Cash Book Today we are going to start a new topic -Cash Book

The key terms used in this chapter are

bullCash book

bullSimple cash book

bullDouble column cash book bullTriple column cash book

bullPetty cash book

bullCash discount

bullContra entry

Here I will share you the meaning of each key terms

bullCash book Cash Book is a special purpose subsidiary book or journal in which cash received and cash payments are recorded

bullSimple cash book

It is a cash book in which only cash transactions are recorded It has only one column on each side

bullTriple column cash book

It is cash book which has three columns one column for each cash and Bankdiscount on each side of the cash book In this book both cash and Bank transactions are recorded together with discount allowed and received

bullPetty cash book

It is a cash book maintained for recording petty expenses

bullCash discount

Cash discount is the amount of discount received or allowed on cash payments and cash receipts Discount received is an income for the business while discount allowed isan expense

bullContra entry

It means transactions involving both cash and Bank Such transactions though recorded in the cash book are not posted into ledger The letter lsquoC is written in Ledger folio for contra entry

Business Studies

ENTREPRENEURSHIP

Now we shall discuss the second chapter

lsquoENTREPRENEURSHIPrsquo

Today before starting the chapter let us recall what

Questions

1What are the main characteristics of Intrapreneurship

Answer

The main characteristics of Intrapreneurship are

Corporate framework-it occurs within the framework of the same company

Semi-Autonomous-Intrapreneurship

we have read last day

Let s today start the class by recalling the last topic taught

Intrapreneurship is the process of discovering and exploring business opportunities within an existing company It involves launching new business ventures within the framework of a present corporation Intrapreneurship is also known as corporate entrepreneurship or corporate venturing

Now let us start with the characteristics of Intrapreneurship

The main characteristics of Intrapreneurship are

Corporate framework

Semi-Autonomous Lack of ownership Senior position Low risk taking Not own boss

Now let us discuss the meaning of enterprise

Enterprise means an undertaking or adventure that requires some innovation and investment and thus involves riskEnterprise always entails decision making coordination and risk bearing

involves crating amd nurturing a semi-autonomous business unit which may be a subsidiary a strategic business unit or a division

Lack of ownership-the intrapreneur is not the owner of the unitb he creates and nurtures

Senior position-he occupies a senior managerial position in the company

Low risk taking-An intrapreneur does not bear the full risk of failure

Not own boss-An intrapreneur is not his own bosss in legal termsHe enjoys the freedom and gets the required resources and support

2 How is Entrepreneur is different from Intrapreneur

The functions involved in both the entrepreneurship and intrapreneurship are by and large similar however there are several differences between the two

Point of distinction

Entrepreneur

Intrapreneur

status An independent business person

A senior executive within a company

Ownership Owner of

the business

An employeesometimes a share in ownership

Financing Responsible for raising finance for the business

Not responsible for raising the finance

Risk bearing

Bears the risk of the business

Does not bears the risk of the business

Reward Profit which is uncertain and irregularcan be loss

Fixed salary and fringe benefits

Need for security low high

3 What do you understand by enterprise

Answer Enterprise means an undertaking or adventure that requires some innovation and investment and thus involves riskEnterprise always entails decision making coordination and risk bearing

COMMERCE NATURE AND OBJECTIVES OF

BUSINESS

Today let us recall the last other two objectives of business by the chart given in the previous class

Firstly we would discuss Human Objectives

Business is run by people and for people Labour is a valuable business element

Human objectives of business are concerned with the well -being of labour

The human objectives are as follows

Labour welfare Developing human

resources Participative

management Labour

management cooperation

Questions

1 Explain the human objectives of a business enterprise

Answer

Business is run by people and for people Labour is a valuable business element

Human objectives of business are concerned with the well -being of labour

The human objectives are as follows

Labour welfare-Business must recognize the dignity of labour and human factors should be given the recognition

Developing human resources-Employees must be provided the opportunities for developing new skills and attitudes

Participative management-Employees should be allowed to take part in decision making process of business

Labour management cooperation-Business should strive for creating and maintaining cordial employer employee relations so as to ensure peace and progress in industry

Now let us discuss the national objectives of business

Optimum utilization of resources

National self- reliance Development of small

scale industries Development of

backward areas Control over pollution

2Explain the national objectives of a business enterprise

Answer

It is the duty of business to utilize the resources of the country properly the national objectives of business

Optimum utilization of resources ndashBusiness should use the nationrsquos resources in the best possible manner

National self- reliance-It is the duty of the business to help the government in increasing experts and in reducing dependence on imports

Development of small scale industries-Big business firms are expected to encourage growth of small scale industries which are necessary for generating employment

Development of backward areas-Business is expected to give preference to the industrialization of backward regions of the country

ECONOMICS

BASIC ECONOMIC CONCEPTS

SUB

TOPIC

Value

Wealth

Welfare

Today we shall start with a new topic of the same chapter ie lsquoValuersquo

Value of a commodity is defined as the valuation placed by a household on the consumption of this commodity

lsquoValuersquo has two different meanings and these are

a Value -in -use It refers to consumption value of a commodity It expresses the utility derived from the consumption of a particular commodity A necessity like water has a very high value ndashin ndashuse or

Question

1What is value

Answer

Value of a commodity is defined as the valuation placed by a household on the consumption of this commodity

2What is value-in use

Answer It refers to consumption value of a commodity It expresses the utility derived from the consumption of a particular commodity A necessity like water has a very high value ndashin ndashuse or consumption value

3What is value ndashin- exchange

Answer It relates to market value of a commodity

It is the rate at which a particular good or service can be exchanged for money

For example in barter system if a person is prepared to exchange 3 metres of cloth with 1 pair

consumption value

b Value ndashin-exchange It relates to market value of a commodity

It is the rate at which a particular good or service can be exchanged for moneyFor example in barter system if a person is prepared to exchange 3 metres of cloth with 1 pair of shoes then the value in exchange of 3 metres of cloth is 1 pair of shoesValue in exchange is the power of purchasing other goods In modern monetised economies the exchange value of goods are expressed in terms of money as prices

Now let us discuss the term lsquoWealthrsquo

Wealth refers to the stock of all those assets which are a source of income

Wealth is a stock concept

Wealth must possess the following features

a Utility It must possess utility or give some

of shoes then the value in exchange of 3 metres of cloth is 1 pair of shoes

Value in exchange is the power of purchasing other goods In modern monetised economies the exchange value of goods are expressed in terms of money as prices

4 What is wealth

Answer it refers to the stock of assets or goods which are a source of income and have personal or national ownership

5 What are the features of wealth

Answer The features of wealth are as follows

Wealth must possess the following features

a Utility It must possess utility or give some satisfaction

b Scarcity It must be limited in quantityc Transferability it should be transferable its

ownership can be transferred from one person to another person

d Exchange value It must possess exchange value

6 What is welfare

Answer

Welfare is defined as satisfaction and happiness a sense of well- being among the people

satisfactionb Scarcity It must be

limited in quantityc Transferability It

should be transferable its ownership can be transferred from one person to another person

d Exchange value It must possess exchange value

Now let us discuss the term lsquoWelfarersquo

Welfare is defined as satisfaction and happiness a sense of well- being among the people

Welfare is affected by factors like

a Consumption of goods and services

b Environment

c Family relations

d Degree of freedom

e Law and order situation

Mathematics Trigonometric equation

To find the general solution of the equation sinθ=0

When sin θ =0

Then θ= 0 π2π 3π-π -2π -3

i e when θ = 0 or an integral multiple of π

i e when θ= nπ where n is any integer

Therefore the general solution of the equation sin

Example1 Find the general values of θ which satisfy the equation sin2 θ =34

Solution sin2 θ= 34

Or sin θ = +34 or -34

Or sin θ = sin π3 or sin (-π3)

Therefore

θ = [nπ + (-1) n (π3)] or[ nπ+ (-1) n (-π3)]

= nπ +π3 or nπ-π3 where n= any integer

Example 2Find the values of θ which satisfy tan2 θ

θ=0 is θ= nπ where n is any integer

To find the general solution of the equation cos θ=0

When cos θ=0

Then θ=π2 3π2 5π2 -π2 -3π2 -5π3

i e when θ is an odd multiple of π2

i e when θ=(2n+1) π2 where n= any integer

Therefore the general solution of the equation cos θ =0 is θ= (2n+1) π2 where n= any integer

To find the general solution of the equation tan θ = 0

Clearly tan θ =0 implies sin θcos θ =0

Therefore θ = nπ

i e the general solution of the equation tan θ=0 is θ =nπ where n = any integer

To find the general solution of the equation cot θ =0

Clearly cot θ =0 implies (cos θsin θ) = 0

i e cos θ =0

Therefore θ = (2n+1) π2

Therefore the general solution of the equation cot θ =0 is θ = (2n+1) π2

Where n= any integer

To find the general solution of the equation sin θ= k (-

=13 -πleθleπ

Solution tan2 θ =13

Or tan θ = plusmn1radic(3) =tan(plusmnπ6)

θ=nπ plusmn π6 where n =any integer

If n=0 then θ=plusmnπ6

If n=1 then θ= π plusmn π6

If n=-1 then θ= -π plusmn π6

Therefore the required solution in -π le θ le π are θ= π6 5π6 -π6 -5π6

Exercise Find general solution of sin 2θ=cos θ [Hints Use sin 2θ= 2sin θcosθ and then take cosθ

common]

1lek le1)

Determine an angle alpha such that sin =k and -π2le αle π2

Then we have

Sin θ = k = sin α

Or sin θ - sin α =0

Or 2 cos [(θ+α) 2] sin [(θ-α) 2] =0

Therefore either cos [(θ +α) 2] =0 (1)

Or sin [(θ-α) 2] =0 (2)

Now from (1) we get (θ+α) 2= (2m+1) π2)

Or θ = (2m+1) π-α (3)

And from (2) we get (θ-α) 2 =mπ

Or θ= 2mπ+α(4)

Where m = any integer

Clearly the solution (3) amp (4) may be combined in the following form

θ= nπ+(-1) n α where n= any integer

Therefore the general solution of sin θ = sin α is θ = nπ +(-1) n α where n is any integer and -π2 le α le π2

Biology Chapter - 04Kingdom Monera

Today we will discuss about bacterial reproduction and its usefulness

Fig Binary Fission

Fig Conjugation Fig Transformation

Bacterial reproduction is mainly asexual but sexual reproduction

also takes place

Asexual reproduction takes place by i) Binary fission - from one bacteriato

two bacteria are produced in every 20 to 30mins

ii) Buddingiii) Endospore formation - during

unfourable condition

Sexual reproduction by three ways

1) Conjugation - Transfer of genetic material between cells that are in physical contact with one another

2) Transduction - Transfer of genetic materialfrom one cell to another by a bacteriophage

3) Transformation - Transfer of cell-freeor naked DNArsquo from one cell to another

Bacteria causes different diseases inplants animals and human and

it causes food spoilage and waterpollution but it also have some useful

activities

i) Bacteria are helpful in sewage water treatment

ii) It is used in antibiotic (medicine) production

iii) Anaerobic bacteria help in biogas(energy) production

iv) Many household products like yoghurt cheese are manufactured by use of bacteria

v) Rhizobium by symbiotic relationship with leguminous plant increase soil fertility

vi) Besides these bacteria is helpful in genetic engineering degradation of petroleum hydrocarbonand in dairy

industry

Physics Motion in plane Here we will introduce Projectile Motion

Execution

Projectile

Y

usinθ u h

θX

ucosθ

Suppose a body is projected with an angle θ So initial velocity u can be resolved into two components

Horizontal component - ucosθ ( for range)

Vertical component - usinθ ( for height)

usinθ changes during motion and becomes zero at maximum height position but ucosθ remain unchanged

The maximum height of projectile is h

NB If initial is upward then g = -ve and if it is downward then g = +ve Height is +ve if direction of motion does not change ( for ex a body thrown upwards but goes down ultimately then height h = -ve)

The angle of projectile θ is the angle made with horizontal

HISTORY ndash GROWTH OF NATIONALISM

SUB TOPIC- REVOLUTIONARY NATIONALISM Bengal formation of Anushilan Samity and Jugantar Group

The intensification of the Swadeshi movement and Government policy of terror and repression led to outbreak of violence Bombs were manufactured and attempts on the lives of unpopular Government officials became frequent In the gymnasium of Scottish Church College which was known as General Assemblies Institution a secret society was formed known as Anushilan Samity

Aurobindo Ghosh send from Baroda his emissary Jatindranath Banerjee to mobilize the Bengal revolutionaries

Hemchandra Qanungo and Satyen Bose published Journal Jugantar

The Jugantar group planned to assassinate oppressive magistrate Kingsford by Khudiram Bose and Prafulla Chaki in 1908 Prafulla Chaki committed suicide to avoid arrest Khudiram was tried and hanged

Afew days later the police found a bomb factory in Maniktala and arrested a large number of revolutionaries The trial of revolutionaries became famous as the Alipore Bomb Case

In the course of the trial the approver the public prosecuter and a police officer were assassinated

1 Question Name two journals which preached the cult of violence

Answer a) Yugantar edited by Bhupendranath DuttaB) Bandemataram edited by Aurobindo Ghosh2 Question Why was Khudiram arrested and hangedAnswer An attempt was made to assassinate a hated vindictive majistrate named Kingsford by Khudiram Bose and Prafulla Chaki Their attempt failed and the bomb they threw killed two English ladies Khudiram was arrested and put to trial and then hanged3Question Who was Aurubindo GhoshAnswer Aurobindo Ghosh a nationalist revolutionary who was charged for his involvement in the Alipore Bombing Case He was accused of it along with his brother Barindra nath Ghosh But Aurobindo was acquitted because of the brilliant pleading of his counsel Chittaranjan Das Then he became a spiritual reformer introducing his visions on human progress and spiritual evolution4 Qustion Who was KingsfordAnswer Kingsford was an unpopular British chief Magistrate who was the target of the bomb thrown at Muzaffarpur by Khudiram and Prafulla Chaki

Most of the accused were convicted and sentenced to

long term of imprisonmentBut

Aurobindo Ghosh was acquitted mainly owing to the brilliant pleading of his counsel Chittaranjan Das

Political science Topic-Sovereignty

Summary Sovereignty is the full right and power of a governing body over itself without any interference from outside sources or bodies In political theory sovereignty is a substantive term designating supreme legitimate authority over some polity In international law sovereignty is the exercise of power by a state

Internal Sovereignty

Internal sovereignty means supreme authority within ones territory while external sovereignty relates to the recognition on the part of all states that each possesses this power in equal measure

External sovereignty

external sovereignty relates to the recognition on the part of all states that each possesses this power in equal measure

Distinguish between

Execution

Answer the following questions

Short notes-

Sovereignty

Internal Sovereignty

External sovereignty

Homework- learn

external sovereignty and internal sovereigntySovereignty is the principle

of supreme and

unquestionable authority

reflected in the claim by the

state to be the sole author of

laws within its territory

Definition of external vs internal sovereigntyInternal sovereignty refers to

the relationship between a

sovereign power and its

subjects ndash it refers to the

location of the supreme

authority within the state In

the UK for example internal

sovereignty (supposedly)

resides within Parliament

reflected in the

constitutional principle of

parliamentary

sovereigntyBy contrast

external sovereignty refers

to the capacity of the state

to act independently and

autonomously on the world

stage This is what is

sometimes called lsquostate

sovereigntyrsquo or lsquonational

sovereigntyrsquo and implies

that states are legally equal

and that the territorial

integrity and political

independence of a state is

inviolable

Class ndash XII

Date - 2742020 STUDY MATERIALSubject Topic Summary Execution Business Studies

Job Analysis amp Manpower Planning

At first let us recall the chapter what we have discussed till nowbullJob analysisbullJob specification bullJob description bullJob enlargement bullJob enrichment

Today we will do some questions answers from the chapter

Questions 1ldquoJob analysis job description and job specification are interrelatedrdquo Comment Answer) Job analysis is a systematic and detailed examination of a job to collect all the relevant information about it The contents off the job are summarised in the job description The qualification needed for the job are summarised in job specificationThus there is close interrelationship between job analysis job description and job specification

Question 2ldquoJob enlargement is a horizontal extension of a job whereas job enrichment is a vertical extension of a jobrdquo ElucidateAnswer) Job enlargement involves adding one or more task to a job coma where as job enrichment involves adding more autonomy and responsibility to a job Job enlargement is therefore horizontal extension of a job coma whereas job enrichment is a vertical extension of a job

Question 3 )

What is manpower estimation Explain its quantitative and qualitative aspectsAnswer) Manpower estimation is the process by which management determines how an organisation should move from itrsquos current manpower positionto its desired manpower position There are two dimensions of Manpower estimation- quantitative and qualitative

Quantitative aspectThis aspect of Manpower estimation involves estimating the number of employees required in a future time period Workload analysis and workforce analysis are done to estimate the quantity of required manpower

Qualitative aspectThe estimate of the knowledge skills experience etc of required manpower is the qualitative aspect of Manpower estimation The quality of Manpower can be judged on the basis of job analysisand job specification

COMMERCE

CAPITAL-FIXED AND WORKING

Today let us start the class by discussing the sources of finance for different types of business firms

The term lsquocapitalrsquo refers to the investment made in the enterprise for the purpose of earning profits

Requirements of capital and sources of capital for different types of business firms are

1 Capital for sole proprietorship businessA sole proprietor operates at a small scale and thereforerequires a limited amount of capital

2 Capital for partnership firmCapital requirements as well as capital base of a partnership is bigger than that of a sole trader businessThe owned capital is contributed by the partners in an agreed ratio

3 Capital for joint stock companyA joint stock company generally requires large amount of capitalA public company can raise huge capital through issue of shares In addition to share capital it can utilize retained profits

Now let u discuss the meaning of Finance PlanningFinance planning is the process of estimation the financial requirements of an organization specifying the sources of firms and ensuring that enough funds are available at the right time

1 What do you mean by Finance PlanningAnswer Finance planning is the process of estimation the financial requirements of an organization specifying the sources of firms and ensuring that enough funds are available at the right time

2Discuss the role of financial planning of an enterpriseThe role of financial planning are as followsa A sound financial plan helps a business enterprise to avaid the problems of shortage and surplus of fundsbFinancial planning serves as a guide in developing a sound capital structure so as to maximize returns to shareholders c It helps in effective utilization of fundsd It provides policies and procedures for coordinating different functional areas or departments of businesse It enables the management to exercise effective control over the financial activities of an enterprisef It helps the company to prepare for facing business shocks and surprises in future

Mathematics

Continuity and differentiability

Recall Definition of ContinuityLet f(x) be a single valued function of x and x=a be a point in the domain of definition of the function The function is said to be continuous at x=a ifi) f(c) is defined ie f(x) has a definite finite value at x=cii) lim xrarra f(x) exists andiii) lim xrarra f(x) =f(a) In other words f(x) is said to be continuous at x=a if lim xrarra+ f(x)= lim xrarra- f(x) = f(a) Or f(a+0) =f(a-0) =f(a) Or lim hrarr0 f(a+h)= f(a) Algebra of continuous functionsNow we will study some algebra of continuous functions Theorem 1 Suppose f and g be two real functions continuous at a real number c Then(1) f + g is continuous at x = c(2) f ndash g is continuous at x = c(3) f g is continuous at x = c(4) (fg) is continuous at x = c (provided g (c) ne 0)

Example 1 Prove that every rational function is continuousSolution Recall that every rational function f is given byf(x)=[p(x) q(x) ] q(x)ne0where p and q are polynomial functions The domain of f is all real numbers except points at which q is zero Since polynomial functions are continuous f is continuous by (4) of Theorem 1Example 2Discuss the continuity of sine functionSolution To see this we use the following factslim xrarr0 sin x =0Now observe that f (x) = sin x is defined for every real number Let c be a real number Put x = c + h If x rarr c we know that h rarr 0 Therefore lim xrarrc f(x)

= lim xrarrc sin x= lim hrarr0 sin(c+h) =lim hrarr0 [sin c cos h + cos c sin h ]=lim hrarr0 (sin c cos h) + lim hrarr0 (cos c sin h) = sin c +0=sin c = f(c) Thus lim xrarrc f(x) = f(c) and hence f is a constant function Exercise Prove that the function f(x) = x2 +2x is continuous for every real value of x [Hints show that lim xrarra+ f(x) = lim xrarra- f(x) = f(a) ]

Biology Reproduction in Flowering plants We will discuss about megasporoangium

megasporagenesis and female gametophyte

Q4 Describe the structure of megasporangium

Ovule is attached to the placenta by astalk called funicle

Each ovule has one two or three protectivecoverings called integuments

At the tip of integuments a small openingcalled micropyle is organised

Opposite to the micropylar end is the chalaza

Within the integuments a mass of cellsnucellusand inside it embryo sac orfemale gametophyte is present

Q5 Describe a mature embryosacamp its formation

In most of the flowering plants only oneof the 4 megaspores formed as a result ofmegasporogenesis that is functional while theother three degenerate

The

functional megaspore develops into thefemale gametophyte

Formation The nucleus

of the functional megasporedivides mitotically to form two nuclei first andthen two more sequential mitotic nucleardivisions result in the formation of four ampthen eight nucleate stages of embryo sac

Six of the eight nucleus are surrounded bycell walls and organised into cells

The remaining two nuclei called polar nuclei are found below the egg apparatus in the largecentral cell

Three cells consisting of two synergids amp one egg cell present bottom of

embryo sac Three cells

at the chalazal as antipodal cells

Two polar nuclei together present in large central cell

HISTORY

TOWARDS INDEPENDENCE AND PARTITION THE LAST PHASE(1935-1947)SUB TOPIC NATIONAL MOVEMENTS DURING THE SECOND WORLD WAR

Spread of Quit India Movement On 9th August 1942Gandhiji and other Congress leaders were arrested The Congress was declared illegal The news of the arrest of all leaders marked the beginning of a widespread movement of India It was not possible for such a movement to remain peacefulBut the arrest of the all notable congress leaders virtually left the movement in the hands of the mass The movement took the form of violent and militant outbreakBesides congressmen revolutionaries also were very active in the movement The Congress Socialist group also played a prominent role

1 Question Why did the British authority arrest the Congress leaders on 9 th August 1942Answer Congress Working committee adopted the Quit India resolution which was to be ratified at the Bombay AICC meeting in 8th August 1942 They decided to launch a mass struggle on non-violent lines Gandhiji gave a clarion call to all section of the people rdquoKarenge ya Marengerdquo (do or die) Congress leaders gave the call to driving out

the British from IndiaViceroy had taken strong action against the Quit India movement Gandhiji and all the leaders of Congress were arrested

2 Question How did Quit India Movement spread out all over IndiaAnswer The news of the leaders lsquo arrest marked the beginning of a widespread movement to remain peacefulThe movement took form of violent outbreak There were widespread cutting of telephone and Telegraph wires damaging railway lines raising barricades in cities and towns and other forms of violent demonstations

Question Name the leaders of Congress

Socialist group played a prominent part Notable among the Jayprakash Narayan Rammonohar Lohia Aruna Asaf Ali

Political science

Topic-Franchise and Representation

Summary

The election commission

The Election Commission of India is an autonomous constitutional authority responsible for administering Union and State election processes in India The body administers elections to the Lok Sabha Rajya Sabha State Legislative Assemblies in India and the offices of the President and Vice President in the country

Functions of election commission-

India is a sovereign socialist secular democratic republic Democracy runs like a golden thread in the social economic and political fabric woven by the Constitution given by lsquoWe the People of Indiarsquo unto ourselves The concept of democracy as visualised by the Constitution pre-supposes the representation of the people in Parliament and State legislatures by the method of election The Supreme Court has held that democracy is one of the inalienable basic features of the Constitution of India and forms part of its basic structure The Constitution of India adopted a Parliamentary form of government Parliament consists of the President of India and the two Houses mdash Rajya Sabha and Lok Sabha India being a Union of states has separate state legislatures for each state State legislatures consist of the Governor and two Houses mdash Legislative Council and Legislative Assembly mdash in seven states namely Andhra Pradesh Telangana Bihar Jammu amp Kashmir Karnataka Maharashtra and Uttar Pradesh and of the Governor and the state Legislative Assembly in the remaining 22 states Apart from the above two out of the seven Union Territories namely National Capital Territory of Delhi and Puducherry also have their Legislative Assemblies

ExecutionShort notes-Election commissionFunctions of election commission

Homework- Learn

Computer

Science

Computer hardware NAND Gate

A NOT-AND operation is known as NAND operation It has n input (n gt= 2) and one output

Logic diagram

Truth Table

NOR Gate

A NOT-OR operation is known as NOR operation It has n input (n gt= 2) and one output

Logic diagram

Truth Table

XOR Gate

XOR or Ex-OR gate is a special type of gate It can be used in the half

adder full adder and subtractor The exclusive-OR gate is abbreviated as EX-OR gate or sometime as X-OR gate It has n input (n gt= 2) and one output

Logic diagram

Truth Table

XNOR Gate

XNOR gate is a special type of gate It can be used in the half adder full adder and subtractor The exclusive-NOR gate is abbreviated as EX-NOR gate or sometime as X-NOR gate It has n input (n gt= 2) and one output

Logic diagram

Truth Table

Physics

Chapter 1 Electric Field ( Electric Dipole) (Summary)

Here we will derive Expression of electric field at broad side

On position of dipole

Execution

Q With the help of a labelled diagram obtain an expression for the electric field intensity E at any point on the equitorial line ( broad-side on position) of an electric dipole

Ans

E1 E1sinθ

E θ P E1 θ

( r2+L2)12 E2 E

r E2 E2sinθ

-q θ L O L +qA B

Let us consider that the point P is situated on the right bisector of the dipole AB at a distance r meter from its midpoint O

Let E1 and E2 be the electric field intensities of the electric field at P due to charge +q and ndashq of the dipole resp The distance of P from each charge is ( r2+L2)12

So E1 = 14 πϵ q

(r 2+L 2) away from +q

E2 = 14 πϵ q

(r 2+L 2) towards ndashq

The magnitudes of E1 and E2 are equal but directions are different Now resolving E1 and E2 into two components parallel and perpendicular to AB we get

The components perpendicular to AB E1sinθ and E2sinθ cancel each other because they are equal and opposite

The components parallel to AB are E1cosθ and E2 cosθ are in same direction and add up

So resultant intensity of electric field at the point P is

E = E1cosθ + E2 cosθ

E = 14 πϵ q

(r 2+L 2) 2 cosθ

Now from fig we have cosθ =BOBP = L (r2+L2)12

So we get E = 14 πϵ 2qL ( r2+L2)32

Now electric dipole moment p= 2qL

So E = 14 πϵ p ( r2+L2)32

HW Find the expression of Electric field as done here but this time take r gtgt 2L

Also find the expression of torque experience by a dipole

(Hint Electric force experienced by charges of dipole in electric field is qE each Let θ be the angle which dipole makes with electric lines of force then perpendicular distance between two charges is 2Lsinθ Then torque = force x perp distance = qE x 2L sinθ So τ=pE sinθ where p =2qL )

STUDY MATERIAL

Class XIISubject Eng Literature (The Tempest ndash William Shakespeare) Topic Act IV Scene 1 Lines 84 to 133 (Iris hellip A contract of true love Be not too late ) Date 27th April 2020 (4th Period)

[Students should read the original play and also the paraphrase given in the school prescribed textbook]Summary Questions amp Answers

o Ceres soon appears and comes to know that she has been summoned to celebrate the contract of true love

o Ceres expresses her unwillingness to meet Venus and Cupid as she has shunned their company

o Ceres and Juno both bestow their blessings upon Ferdinand and Miranda with June gifting honour riches happiness in marriage and Ceres presents plenty of earthrsquos produce

o Iris summons the water-nymphs and reapers to come and celebrate a contract

(1) IRIS Of her society (Line 91-101)

Be not afraid I met her deity

Cutting the clouds towards Pathos and her sonDove-drawn with her Here thought they to have doneSome wanton charm upon this man and maidWhose vows are that no bed-right shall be paidTill Hymens torch be lightedmdashbut in vainMarss hot minion is returned againHer waspish-headed son has broke his arrowsSwears he will shoot no more but play with sparrowsAnd be a boy right out

(i) Where were Venus and Cupid seen flying How were they travelling Why did they want to join the marriage celebration of Ferdinand and Miranda

of true love

Venus and Cupid were seen flying through the air towards Paphos the famous city which is situated on the island of Cyprus They were travelling by air-borne chariot drawn by doves They certainly wanted to come here in order to play some amorous trick upon Ferdinand and Miranda who are under a vow not to gratify their physical desires till the holy ceremony of their marriage has been performed(ii) What have Venus and Cupid done after failing in their plan

After being failure of their plan Venus who is a very passionate deity and who is the mistress of Mars (the god of war) has gone back while here ill-tempered son Cupid has broken his arrows of love in his state of desperation(iii) What has Cupid firmly decided

Cupid is feeling so disappointed that he has firmly decided to shoot no more arrows to arouse love in human hearts but to spend his time playing with sparrows Thus he would now become just a boy and would give up his original function of shooting arrows on human beings to make them fall in love(iv) What vow had Ceres taken How did Ceres feel at the abduction

After the abduction of her daughter Prosperina by Pluto Ceres had taken a vow to always keep away from the disgraceful company of Venus and her blind son Cupid the god of love Ceres felt deeply distressed when Pluto had carried off her daughter and had made her his wife by force(v) Why has Ceres not forgiven Venus and her blind son For what do Ceres want to be sure

As the abduction had been manipulated by Venus the goddess of beauty and love and her blind son Cupid Ceres has never forgiven them for their part in the whole plot Ceres wants to be sure that she would not have to meet Venus and Cupid who had engineered the abduction of her daughter Prosperina

AS THIS lsquoMASQUErsquo SCENE IS VERY IMPORTANT IN THE PLAY THE PARAPHRASE OF THE ENTIRE PORTION OF MASQUE SCENE (Act IV Lines 58 to 143) IS GIVEN BELOW

IRIS Goddess of RainbowCERES Goddess of Agriculture and all the fruits of the earth

(Nature growth prosperity rebirth ndash notions intimately connected to marriage)JUNO The majestic Queen of Heavens and wife of Jupiter (Jupiter is the king of Gods)

VENUS The Goddess of love CUPID Son of Venus PLUTO God of death (In the play referred by Shakespeare as lsquoDisrsquo which is a Roman name for Pluto)

ORIGINAL TEXT PARAPHRASEPROSPEROWellmdash

PROSPERONow come Ariel Let there be too many rather than too few

Now come my Ariel Bring a corollaryRather than want a spirit Appear and pertly[to Ferdinand and Miranda]No tongue all eyes Be silent

spirits in attendance Appear briskly

[to Ferdinand and Miranda]Look with your eyes but do not say a word

[Soft music] [Soft music][Enter Iris] [Enter Iris]

IRISCeres most bounteous lady thy rich leasOf wheat rye barley vetches oats and peasThy turfy mountains where live nibbling sheepAnd flat meads thatched with stover them to keepThy banks with pioned and twilled brimsWhich spongy April at thy hest betrimsTo make cold nymphs chaste crowns and thybroom-grovesWhose shadow the dismissegraved bachelor lovesBeing lass-lorn thy pole clipped vineyardAnd thy sea-marge sterile and rocky-hardWhere thou thyself dost airmdashthe Queen othrsquoSkyWhose watery arch and messenger am IBids thee leave these and with her sovereign grace[Juno appears] Here on this grass-plot in this very placeTo come and sport Her peacocks fly amainApproach rich Ceres her to entertain

IRISCeres most generous lady you are the cause of rich fields or fertile land where wheat rye barley beans oats and peas grow the grassy mountains where the sheep graze and the flat meadows covered with coarse hay to be used as fodder for cattleYour banks are covered with marsh-marigolds and reeds and the rainy April under your orders brings forth to make for the maids who are not in love beautiful crowns your woods where the broom flourishes and where the bachelor who has been dismissed by the maid he loved lies down being forsaken your vineyard in which the poles are embraced by the vines and the margin of the sea which is barren and rocky where you roam about to enjoy the fresh air ndash the queen of the sky (Juno) whose messenger I am besides being represented as the rainbow bids you leave all these and with her majesty here on this grassy plot in this very place come and sport her peacocks carry her fast in her chariot through the air and are making their way here approach rich Ceres to welcome her

[Enter Ariel as Ceres] [Enter Ariel as Ceres]

CERESHail many-coloured messenger that neerDost disobey the wife of JupiterWho with thy saffron wings upon my flowersDiffusest honey-drops refreshing showersAnd with each end of thy blue bow dost crownMy bosky acres and my unshrubbed downRich scarf to my proud earth Why hath thy queenSummoned me hither to this short-grassed green

CERESWelcome rainbow that never dared disobey Juno the wife of Jupiter who with your orange coloured rays spread honey-drops refreshing showers And with each end of thy blue bow drown my bushy acres and my hilly country which is free from shrubs you thus forming a rich scarf Why has your queen called me here to this place covered with short grass

IRISA contract of true love to celebrateAnd some donation freely to estateOn the blest lovers

IRISI have called you to celebrate a contract of true love and bestow some liberal gift upon the blessed lovers

ORIGINAL TEXT PARAPHRASECERESTell me heavenly bowIf Venus or her son as thou dost knowDo now attend the queen Since they did plotThe means that dusky Dis my daughter gotHer and her blind boys scandaled companyI have forsworn

CERESTell me heavenly bow if Venus the Goddess of love or Cupid her son and pedlar of passion at this time attend the heavenly queen Juno because you are sure to know Since the day they conspired against me and dark Pluto took away my daughter here and Cupidrsquos disgraceful company I have left off

IRISOf her societyBe not afraid I met her deityCutting the clouds towards Pathos and her sonDove-drawn with her Here thought they to have doneSome wanton charm upon this man and miad

IRISBe not afraid of her company I met her deity moving on the clouds towards Paphos the sacred home of Venus on the island of Cyprus along with her son on her chariot drawn by doves Here they contemplated to exercise a charm upon this man and maid producing

Whose vows are that no bed-right shall be paidTill Hymens torch be lightedmdashbut in vainMarss hot minion is returned againHer waspish-headed son has broke his arrowsSwears he will shoot no more but play with sparrowsAnd be a boy right out

wantonness before the actual marriage ceremony but did not succeed Venus has returned her irritable son has broken his arrows and swears that he will give up his practice of trying to inspire love but play with sparrows and be a boy again

[Music is heard] [Music is heard]

CERESHighst queen of stateGreat Juno comes I know her by her gait

CERESHighest queen of state Great Juno there she comes I know here by her gait

[Enter Juno] [Enter Juno]

JUNOHow does my bounteous sister Go with meTo bless this twain that they may Prosperous beAnd honoured in their issue

JUNOHow are you doing my generous sister Come with me to bless this couple so that they may be prosperous and fortunate in their children

[They sing] [They sing]

JUNOHonour riches marriage-blessingLong continuance and increasingHourly joys be still upon youJuno sings her blessings upon you

JUNOMay honour riches happiness in marriage long continuance and increase of those boons ever rest upon you as hourly joys Juno showers down upon you her blessings in song

CERESEarths increase foison plentyBarns and garners never emptyVines and clustring bunches growingPlants and goodly burden bowingSpring come to you at the farthestIn the very end of harvestScarcity and want shall shun youCeresrsquo blessing so is on you

CERESMay you have the plenty of earthrsquos produce Your barns and granaries may never be empty Your vines may grow with clustering bunches Your fruit trees may be heavily laden with their fruit May there be continuous spring and harvest May scantiness and want leave you forever Such is the blessing of Ceres upon you

FERDINANDThis is a most majestic vision andHarmoniously charmingly May I be boldTo think these spirits

FERDINANDThis is a great vision and magically melodious Should I suppose the characters (taking part in the masque) are spirits

PROSPEROSpirits which by mine artI have from their confines calld to enactMy present fancies

PROSPEROYes they are spirits whom I have summoned from the regions to which they are confined to carry into effect my fanciful designs

ORIGINAL TEXT PARAPHRASEFERDINANDLet me live here everSo rare a wondered father and a wifeMakes this place paradise

FERDINANDI should like to live here forever Such a wise and wonderful father makes this place a paradise

[Juno and Ceres whisper and send Iris on employment] [Juno and Ceres whisper and send Iris on employment]

PROSPEROSweet now silence

PROSPEROMy dear Ferdinand speak no more Juno and Ceres are

Juno and Ceres whisper seriouslyTheres something else to do Hush and be muteOr else our spell is marred

whispering with a solemn look There is something else coming Silence Or else our magic will be spoilt

IRISYour nymphs called naiads of the wandering brooksWith your sedged crowns and over-harmless looksLeave your crisp channels and on this green landAnswer your summons Juno does commandCome temperate nymphs and help to celebrateA contract of true love Be not too late

IRISYou nymphs called Naiads denizens (M inhabitants) of the running stream with your chaplets of sedge and ever-helpful looks leave your wrinkled channels and on the green land answer the summons sent to you Juno has ordered some chaste nymphs and help to celebrate a noble and true marriage Donrsquot delay

[Enter certain nymphs] [Enter certain nymphs]You sunburnt sicklemen of August wearyCome hither from the furrow and be merryMake holiday your rye-straw hats put onAnd these fresh nymphs encounter every oneIn country footing

You sunburnt harvesters weary from the effects of the heat in August come here from the furrowed land and rejoice Make holiday with your rye-straw hats upon you and meet these fresh nymphs and join in country dancing

[Enter certain reapers properly habited They join with the nymphs in a graceful dance towards the end whereof Prospero starts suddenly and speaks]

[Enter certain reapers properly habited They join with the nymphs in a graceful dance towards the end whereof Prospero starts suddenly and speaks]

PROSPERO[aside] I had forgot that foul conspiracyOf the beast Caliban and his confederatesAgainst my life The minute of their plotIs almost come [to the spirits]Well done Avoidno more

PROSPERO(Aside)I had forgotten the wicked conspiracy of the beast Caliban and his accomplices against my life the time of their plot has almost arrived ndash (To the Spirits) well done depart no more of this

[To a strange hollow and confused noise the spirits heavily vanish]

[The spirits depart]

ORIGINAL TEXT PARAPHRASEFERDINANDLet me live here everSo rare a wondered father and a wifeMakes this place paradise

FERDINANDI should like to live here forever Such a wise and wonderful father makes this place a paradise

[Juno and Ceres whisper and send Iris on employment] [Juno and Ceres whisper and send Iris on employment]

PROSPEROSweet now silenceJuno and Ceres whisper seriouslyTheres something else to do Hush and be muteOr else our spell is marred

PROSPEROMy dear Ferdinand speak no more Juno and Ceres are whispering with a solemn look There is something else coming Silence Or else our magic will be spoilt

IRISYour nymphs called naiads of the wandering brooksWith your sedged crowns and over-harmless looksLeave your crisp channels and on this green landAnswer your summons Juno does commandCome temperate nymphs and help to celebrateA contract of true love Be not too late

IRISYou nymphs called Naiads denizens (M inhabitants) of the running stream with your chaplets of sedge and ever-helpful looks leave your wrinkled channels and on the green land answer the summons sent to you Juno has ordered some chaste nymphs and help to celebrate a noble and true marriage Donrsquot delay

[Enter certain nymphs] [Enter certain nymphs]You sunburnt sicklemen of August wearyCome hither from the furrow and be merryMake holiday your rye-straw hats put onAnd these fresh nymphs encounter every oneIn country footing

You sunburnt harvesters weary from the effects of the heat in August come here from the furrowed land and rejoice Make holiday with your rye-straw hats upon you and meet these fresh nymphs and join in country dancing

[Enter certain reapers properly habited They join with the nymphs in a graceful dance towards the end whereof Prospero starts suddenly and speaks]

[Enter certain reapers properly habited They join with the nymphs in a graceful dance towards the end whereof Prospero starts suddenly and speaks]

PROSPERO[aside] I had forgot that foul conspiracyOf the beast Caliban and his confederatesAgainst my life The minute of their plotIs almost come [to the spirits]Well done Avoidno more

PROSPERO(Aside)I had forgotten the wicked conspiracy of the beast Caliban and his accomplices against my life the time of their plot has almost arrived ndash (To the Spirits) well done depart no more of this

[To a strange hollow and confused noise the spirits heavily vanish]

[The spirits depart]

Ac-12 27420 topic Revaluation of Assets and Liabilities

REVALUATION OF ASSETS AND LIABILITIES

On admission of a new partner the firm stands reconstituted and consequently the assets are revalued and liabilities are reassessed It is necessary to show the true position of the firm at the time of admission of a new partner If the values of the assets are raised gain will increase the capital of the existing partners Similarly any decrease in the value of assets ie loss will decrease the capital of the existing partners For this purpose alsquoRevaluation Accountrsquo is prepared This account is credited with all increases in the value of assets and decrease in the value of liabilities It is debited with decrease on account of value of assets and increase in the value of liabilities The balance of this account shows a gain or loss on revaluation which is transferred to the existing partnerrsquos capital account in existing profit sharing ratioAccounting for Revaluation of Assets and Liabilities when there is a Changein the Profit Sharing Ratio of Existing PartnersAssets and liabilities of a firm must also be revalued at the time of change in profit sharing ratio of existing partners The reason is that the realisable or actual value of assets and liabilities may be different from those shown in the Balance Sheet It is possible that with the passage of time some of the assets might have appreciated in value while the value of certain other assets might have decreased and no record has been made of such changes in the books of accounts Similarly there may be some unrecorded assets amp libilities that may have to be accounted for Revaluation of assets and reassessments of liabilities becomes necessary because the change in the

value of assets and liabilities belongs to the period to change in profit sharing ratio and hence must be shared by the partners in their old profit sharing ratio Revaluation of assets and reassessment of liabilities may be given effect to in two different ways (a) When revised values are to be recorded in the books and(b) When revised values are not to be recorded in the books

When revised values are to be recorded in the booksIn such a case revaluation of assets and reassessment of liabilities is done with the help of a new account called lsquoRevaluation Accountrsquo Sometimes this account is also called as lsquoProfit amp Loss Adjustment Acrsquo If there is a loss due to revaluation revaluation account is debited and if the revaluation results in a profit the revaluation account is credited The following journal entries made for this purpose are

(i) For increase in the value of assetsAsset Ac Dr (individually)To Revaluation Ac(ii) For decrease in the value of AssetRevaluation Ac Dr (individually)To Asset Ac[Decrease in the value of assets](iii) For increase in the value of LiabilitiesRevaluation Ac Dr (individually)To Liabilities Ac[Increase in the value of Liabilities](iv) For decrease in the value of LiabilitiesLiabilities Ac DrTo Revaluation Ac[Decrease in the value of Liabilities](v) For unrecorded AssetsAsset Ac [unrecorded] DrTo Revaluation Ac[Unrecorded asset recorded at actual value](vi) For unrecorded Liability Revaluation Ac DrTo Liability Ac [unrecorded][Unrecorded Liability recorded at actual value](vii) For transfer of gain on revaluationRevaluation Ac DrTo Existing Partnerrsquos CapitalCurrent Ac[Profit on revaluation transferred to capital account in existing ratio](viii) For transfer of loss on revaluationExisting Partnerrsquos CapitalCurrent Ac DrTo Revaluation Ac[Loss on revaluation transferred to capital account in existing ratio](a) When revaluation account shows gain Revaluation Ac DrTo Partnerrsquos Capital Ac (Old Profit Sharing Ratio)(Profit on revaluation credited to Partnerrsquos Capital Ac)(b) Above entry is reversed when revaluation account shows loss Partners Capital Acs (Old Profit Sharing Ratio) DrTo Revaluation Ac(Loss on revaluation debited to Partnerrsquos Capital Acs)

Proforma of Revaluation Account is given as under

Revaluation Account

Dr Cr Particulars ` Amount Particulars ` Amount To Decrease in value of assets By Increase in value of assets To Increase in value of liabilities By Decrease in value of liabilities To Unrecorded liabilities By Unrecorded assets To Gain on Revaluation (Transferred) By Loss on Revalution (Transferred)

ECO ndash12 2742020Topic- ELASTICITY OF DEMAND

CHAPTER - ELASTICITY OF DEMANDMEANINGDemand for a commodity is affected by many factors such as its price price of related goods income of its buyer tastes and preferences etc Elasticity means degree of response Elasticity of demand means degree of responsiveness of demand Demand for a commodity responds to change in price price of related goods income etc So we have three dimensions of elasticity of demandDIMENSION OF ELASTICITY OF DEMAND TYPES OF ELASTICITY OF DEMAND

Price elasticity of demand Income elasticity of demand Cross Elasticity of demand

Price elasticity of demand Price elasticity of demand means degree of responsiveness of demand for a commodity to the change in its price For example if demand for a commodity rises by 10 due to 5 fall in its price Price elasticity of demand (ep)=Percentage change in quantity demanded Percentage change in price of the commodity = 10 ( -)5 = ( - )2Note that ep will always be negative due to inverse relationship of price and quantity demanded

(ii) Income elasticity of demand Income elasticity of demand refers to the degree of responsiveness of demand for a commodity to the change in income of its buyer Suppose income of buyer rises by 10 and his demand for a commodity rises by 20 then Income elasticity of demand (ey)= change in quantity demanded change in price of the commodity =20 10 = 2

Cross Elasticity of demandCross elasticity of demand means the degree of responsiveness of demand for a commodity to the change in price of its related goods (substitute goods or complementary goods) Suppose demand for a commodity rises by 10 due to 5 rise in price of its substitute good then Cross elasticity of demand (ec) = change in quantity demanded change in price of related good = 10 2 = 5 (Tastes and preferences cannot be expressed numerically So elasticity ofdemand cannot be numerically expressed)

  • Chapter 1 Force (Summary)
  • Distinguish between external sovereignty and internal sovereignty
    • NAND Gate
      • Logic diagram
      • Truth Table
        • NOR Gate
          • Logic diagram
          • Truth Table
            • XOR Gate
              • Logic diagram
              • Truth Table
                • XNOR Gate
                  • Logic diagram
                  • Truth Table
                      • Physics
                      • Chapter 1 Electric Field ( Electric Dipole) (Summary)
Page 15:  · Web viewWe all know that Nouns are divided into two parts: common noun and proper noun.Apart from common and proper noun, we will also study about collective noun and compound

Date 270420

PREPOSITIONS

A preposition is a word placed before a noun or a pronoun to show in what relation the person or thing denoted by it stands in regard to something else

EXAMPLE

There is a clock on the wall

The preposition on shoes relationship between clock and the wall

Smitha is afraid of lizards The man jumped off the bus

Here the preposition of shows the relationship between afraid and lizards

The preposition off shows the relation between jumped and bus

The noun or the pronoun which follows a preposition is called its object

So in the first Example wall is the object of the preposition on

A preposition can have more than one object For example

The plane flew overhouses and meadows

Prepositions are used to express a number of relationship including time locationmannermeans quantity purpose and state or condition

Points to remember

Preposition joins a noun to another noun or a pronounThere is a cow in the field

A preposition joints a noun to a verbThe cat runs after the rat

Preposition can have two or more than two objectsThe road runs over hills and plain

A preposition also joins a noun an adjectiveHe is fond of tea

Generally a preposition comes before an object Sometimes it comes even after an object asWhat are you looking atThis is the house I live in

Subject- Computer Application

Chapter 2 Introduction to Java

Java API An application programming interface (API) in the context of Java is a collection of prewritten packages classes and interfaces with their respective methods fields and constructors

Byte Code Java bytecode is the result of the compilation of a Java program an intermediate representation of that program which is machine independent The Java bytecode gets processed by the Java virtual machine (JVM) instead of the processor JVM The Java Virtual Machine (JVM) is the runtime engine of the Java Platform which allows any program written in Java or other language compiled into Java bytecode to run on any computer that has a native JVM

Platform A platform is the hardware or software environment in which a program remains

Java platform The Java platform differs from most other platforms in that itrsquos a software only platform that runs on top of other hardware-based platforms

Applet and Application The fundamental difference between the two Java programs is that an application program is designed to run on a stand-alone machine whereas an applet is a web-version of an application which is used to run a program on a web browser

WORA ldquoWrite once run anywhererdquo (WORA) or sometimes write once run everywhere (WORE) is a slogan created by Sun Microsystems to illustrate the cross-platform benefits of the Java language

Class XSubject Topic Summary Execution

ECONOMICSFACTORS OF PRODUCTION Sub-topiclsquoLABOURrsquo

We shall start our class by discussing the topic taught in the last class

lsquo DIVISION OF LABOURrsquo- By division of labour we mean specialization in workIt refers to splitting up the work of labour involved in the production of a particular commodity into several parts and each part and sub-part is performed by a specialist

Now let us start by the Advantages of Division of labouraIt increases the level

Questions

1 Differentiate between Product-Based division of labour and Process-Based division of labour

Product-Based division of labour

Process-based division of labour

It is also known as simple (or occupational) division of labourUnder it everybody performs a particular occupations The entire is done by the same person

When a person or group of persons undertakes a specialised function which is supplementary to the production of final commodity and service This is also as complex division of labour

It is simple It is complex

of productionbSince the product is produced by an expert workerbest quality of product is producedcIt saves time and toolsd it promotes inventions in the methods and techniques of productioneIt leads to reduction in costs fAll workers get work according to their abilities and choices

Now let us discuss the disadvantages of Division of labour

a Since many workers are involved in the production of a commodity no one has the sense of responsibility

b The constant and repetition of the same work again and again make the work monotonous

c Division of labour facilitates production on large scale Hencethere is fear of over production

d Because of territorial division of labour some areasregions become more developed than others

It is based on labour-intensive techniques of production

It is based on capital-intensive techniques

It is generally found in small enterprises

It is generally found in large enterprises

Example Indian farmers doing all farm activities

Example A modern garmet factory where one person takes the measurementanother does the cuttingsome sew the clothes while a few workers button them and other iron them

2 What are the advantages of Division of labouraIt increases the level of productionbSince the product is produced by an expert workerbest quality of product is producedcIt saves time and toolsd it promotes inventions in the methods and techniques of productioneIt leads to reduction in costs fAll workers get work according to their abilities and choices

3Discuss the disadvantages of Division of laboura Since many workers are involved in the

production of a commodity no one has the sense of responsibility

b The constant and repetition of the same work again and again make the work monotonous

c Division of labour facilitates production on large scale Hence there is fear of over production

d Because of territorial division of labour some areasregions become more developed than others

English 1 Transformation of sentences

Sentences A sentence is a group of words which makes complete sense

a Assertive sentences

Exercise 6Rewrite the following sentences according to the instructions given below without changing their meanings

1 As soon as he saw the beer he jumped into

b Imperative sentences

c Interrogative sentences

d Exclamatory sentences

Sentences can be changed from one grammatical form to another without changing the meaning of the sentence This is known as transformation of sentences

the river ( Begin No sooner)2 None but brave deserve the fair (Begin the

bravehellip)3 This box is too heavy for me to lift ( Use so hellip

That instead of too)4 No one other than a king can live like James

Luxurious ( Begin only James)5 Oh for the wings of a dove (Begin I wishhellip)

Math Topic Commercial MathematicsChapter Shares and Dividends

Study item Discuss about shares and Dividends1) What is share

Ans To start any big business (company or Industry) a large sum of money is needed But it is not possible for an individual to invest such a large amount Then some persons interested in the business join together and from a company They divide the estimated money required into small parts Each such part is called a share

2) What do you mean by the term shareholder

Ans A person who purchases one or more shares is called shareholder3) Some terms related with a share

(i) Nominal value or face value or printed value The original value of a share is called its nominal value or face value or printed value

Note The nominal value of a share always remains same(ii) Market value or cash value The price of a share at any

time is called its market value or cash value

Note The market value of a share changes from time to time(iii) At par If the market value of a share is the same as its

nominal value the share is called at par(iv) At Premium or above Par If the market value of a share

is more than its nominal value the share is called at premium or above par

Example If a share of Rs 100 is selling at Rs 150 then it is said to be selling at a premium of Rs 50 or Rs 50 above par

(v) At Discount or below par If the market value of a share is less than its nominal value the share is called at discount or below par

Example If a share of Rs 100 is selling at Rs80 then it is said to be selling at a discount of Rs 20 or at Rs 20 below par

4) What is Dividend

Ans The profit which a shareholder gets for hisher investment from the company is called dividendNote (i) The dividend is always expressed as the percentage of the face value of the share(ii) The dividend is always given( by the company ) on the face value of the share

irrespective of the market value of the shareBENGALI(2ND LANGUAGE)

ldquoদেবতোর জণমrdquoলিবরোম চকরবত

পরথম লিসর পোঠ-চোর পসথ একটি পোথর লিবপলি ঘটোয় দেক যোতোয়োসতর পসথ পরলিতলিয়ত ওই পোথসর দেো োচট দেসত একলি দেতো দেক দেো োচট দেসয় দেবোমো সয় রোসতোর মোস লি0টসক পস1 লিবপরীত লিক দেথসক আো একটি দেমোটর োলি1 চোসকর কষতোয় পরোসরণ দেবোসচ যো লিকনত পরলিতবোর এমরণ দেৌভোয দেসকর োও সত পোসর তোই লিতলি দেকোো দেজোো1 কসর পোথরটিসক উপস1 দে8স পোথর উপস1 দে8োর ময় এক দেকৌতী জতো দেকসক পরশন কসর দেয লিতলি দেকোসো দেবতোর আস দেপসয়স0রণ লিকো লিকনত দেক বস লিতলি দেকোসো দেবতোর আস পোলি দেক উপলিত ক জতোর উসltসয বস কোরও ইস= স পোথরটি লিসয় দেযসত পোসর এর পর দেথসক দেক দেযসত আসত পোথরটি দেক দেসত পো একলি দেক কষয করস দেকউ পোথরটিসক ধসয়মস0 পলিরসকোর করস0 দেক ওই দেকৌতী জতোসক পোথসরর কোস0 বস থোকসত দেস এ0ো1ো আরও কষয কসর দেক ঠোৎ ওই লি1 পোথরটির োসয় লিোর োো-দেকউ পজো কসরস0 দেকৌতী দেোকটির আঙকো য় যলিসক উ পোথরটিসক লিরসয় দে8স তোর পর ঠোৎ একলি পোথরটির দেোোজ দেই দেক লিসয় দেস0 বো দেকোথোয় দেস0

বদোথ-

দেো োচট ndashচসত লিসয় দেকো লিক0র সE ধোককো দেস পস1 যোবোর উপকরমঅকসমোৎ- ঠোৎআতমমবর- লিসজসক লিয়নতরপ-পো লিপ0স প1োদেসতসসত- লিসপলিউৎোত- দেো1ো দেথসক উপস1 দে8োপরতযয় ndash লিবশবোপরতযোস- টতযোস ndash দৈবোসধসতোধলিসত- পরসপসরর পরলিত ব পরসয়ো করোপরসতরীভত- পোথসর পলিররণলিতবোনতঃকরসরণ- মস পরোসঅলিQৎ- অQো করসত ই=কদেোপ- বধমোর- দেবোস এমইতযোকোর- এইরকমরম- বময়পযসোভী-পসযর জয দেোভ আস0 যোরপোসথয় ঞচয়- পথচোর রচ জমোসোমোমোসরোস- ব ধম ধোসমর সE

তোর লি দেইhelliphellipTo be continued

Hindi 2nd lang सर क पद(सरदास)

सरदास शरी कषण भकति कावय क सरवशरषठ कगिरव ह इनक जनम और मतय क समय तथा सथान का मतभद हसरदास रवातसलय और शरार रस क अनयतम कगिरव ह इनक कावय म बालकषण क सौदय चपपल चषटा और गि7याओ की मनोहर झाकी मिमलती ह कषण और ोगिपयो क अननय परम का कतिचतरण ह सयो शरार की अपकषा उनक कावय म गिरवयो शरार का अमिBक गिरवषय और मारमिमEक कतिचतरण हआ हइन पकतियो म हम सरदास की भकति भारवनाओ का परिरचय मिमलता ह इनका सपण सगरह सरसार म गिनगिहत ह

1 जसोदा हरिर पालन झलारवहलरारवदलराईमलहारव रव जो ईसाई कछ ारव मर लाल को आई निनEदिदया काह ना आगिनसबारव त काह नाही बरवगिह आरवतोको कानहा बलारव

शबदाथ-हलरारव-गिहलती हदलराई - दलार पयार करती हमलहारव-पचकारती हनिनEदरिरया ndashनीदरवगिह-जलदी सअBर-होठमौन-चपसन-सकत

वयाखया- सरदास जी कहत ह गिक यशोदा माता बालक कषण को पालन म झल आती ह रवह उनह गिहलाती ह पयार करती ह मलहार जस कोई ीत ान लती ह और नीद स पछती ह गिक ह नीद तम मर लाल को आकर कयो नही सलाती तझ खाना बला रहा ह कभी कषण आख बद कर लत ह कभी आखफडफडान लत ह उनह सोता हआ जानकर यशोदा माता चप हो जाती ह और इशार म बात करन लती ह इसी बीच अकला कर कषण ज जात हतो गिफर यशोदा माता गिफर स ाना ान लती ह सरदास जी कहत ह गिक भरवान क दशन का सख दरवता और ऋगिष-मगिनयो को भी दलभ ह यही सख माता यशोदा को बडी सहजता स मिमल जा रही ह माता यशोदा बहत ही भागयशाली ह2)Continue to nexthellip

Physics

Chapter 1 Force

(Summary)

Question A body is acted upon by two forces each of magnitude F but in opposite directions State the effect of the forces if

(a) Both forces act at the same point of the body

(b)the two forces act at two different points of the body at a separation r

Solutions

(a) Resultant force acting on the body = 0

F ndash F = 0(b) The forces tend to rotate the body between two forces about the midpoint

Moment of forces = F times rFr

QuestionDefine moment of a couple Write its SI unit

Solutions

Moment of couple is equal to the product of both force and the perpendicular distance between the two forces

The SI unit of moment of couple is NmCommercial Studies

Advertising and sales

Business firms use several methods to

Questions1) What do you mean by advertising

promotion create demand of their product in the market and increase it sales Such methods comprises of advertising sales promotion personal selling and publicityToday we are going to discuss about one of such methods It is advertising

Meaning of advertisingAdvertising is a paid form of non-personal presentation for promotion of Ideas goods and services

Importance or merits of advertising Advertising has importance to manufacturer or traders to customer and to society as a whole

Today we will see how advertisement help the manufacturer or traders

Answer) Advertising is a means of how a company encourages people to buy their products services or ideas It is one element of marketing which also includes design Research and data mining

2) Mention any three features of advertisingAnswer)The main features of advertising are

i) It is impersonal form of presentation for promotion of products and services of Ideas

ii) It is issued by identified sponsor The advertisement contains the name of the advertiser

iii) It is a form of mass communication because the message is directed to a large number of persons simultaneously

3) Mention the main merits or importance of advertisement to manufacturer or tradersAnswer)

i) Introducing new product A business organization can introduce itself and its products to the public through advertising

ii) Increase the sale Advertising leads to increase the sale of existing product by entering into new markets and attracting new customers

iii) Create steady demand Advertising creates sustains regular demand by smoothening out seasonal and other fluctuations It enables regular production for the organisation

iv) Economics of scale Advertising facilitate mass distribution of goods and steady demand which lead to large scale and regular production

v) Goodwill Advertising helps in creating a good image of the firm and reputation for its products

Biology Chapter - 03Genetics

Today wewill start chapter and discuss about Genetics Gregor Mendel is known as father of genetics Before entering into Mendelrsquos experiment on Genetics we must know

Q1 Define the following termsi) Genetics Genetics is the study of

transmissionof body features from parents to offspringand the laws relating to such transmission

ii) Heredity It may be defined as transmissionof genetically based characteristics from parentsto offspring

iii) Character and traits Any heritable

Importance to TraderIntroducing new productIncrease the saleCreate steady demandEconomics of scaleGoodwill

some terms featureis a character The alternative forms of acharacter are called traitsex Character (Hair shape) - Traits (Curly straight)

iv) Homologous chromosomes A pair ofcorresponding chromosomes of the same shapeand size one from each parent

v) Genes Genes are the specific parts (DNA segments) of a chromosome which determinethe hereditary characteristicsNearly 30000genes present in human

vi) Alleles Alternative forms of a gene occupying the same position (locus) on homologouschromosomes and affecting the same characteristicbut in different ways

vii) Genotype ndash PhenotypeGenotype means of genes present in the cells of an organism Phenotype means the observable characteristic which is genetically controlled

viii) Mutation It is a sudden change in one or more genes or in the number or in the structure of chromosomes ex Sickle cell anaemia is a blood disease caused by a gene mutation

CLASS NOTES

Class XSubject Eng Literature (The Merchant of Venice ndash William Shakespeare)Topic Act IV Scene 1 Lines 01 to 34 ( Duke helliphelliphellip We all expect a gentle answer Jew) ate 27th April 2020 (2nd Period)

[Students should read the original play and also the paraphrase given in the school prescribed textbook]Summary Questions amp Answers

This scene may be termed as the catastrophe of the play It is the final unravelling of the complicated events which seem to threaten the happiness of Bassanio Portia and Antonio Right is justified to the fullest degree and malice falls into the trap prepared for others No one suffers here but Shylock but even then he receives a generous measure of mercy

o This is the Court-scene Initially we meet

(1)

DUKE I am sorry for thee thou art come to answer (Line 3-6)A stony adversary an inhuman wretchUncapable of pity void and emptyFrom any dram of mercy

(i) Who is addressed here Where is the person Why is the person there

Antonio is addressed hereAntonio is in the court of justice at VeniceAntoniorsquos trial is scheduled to be held here for his failure to meet the conditions of the bond he signed with Shylock

the Duke Antonio Shylock and Salerio Later we meet Bassanio Portia Gratiano and Nerissa

o The Duke says to Antonio that he has to face a very cruel opponent which Antonio admits and expresses his gratefulness to the Duke for his efforts to soften without result the heart of Shylock in order to be merciful to Antonio Antonio further says that he is ready to accept whatever cruel judgement the Court may award

o When Shylock appears in the court the Duke says that Shylock should change his decision of prosecuting Antonio and demanding the penalty specified in the bond out of consideration of the great misfortunes that Antonio has suffered If this is done by him (Shylock) the whole court would be gladdened by his merciful action

(ii) What is the Duke sorry for

The Duke is unable to change the mind of Shylock from his decision to get the bond forfeited even after he pleaded to Shylock Shylock stands firmly for his bond which when forfeited will allow him to take a pound of flesh from any part of Antoniorsquos body(iii) How does the Duke address Shylock`The Duke calls Shylock an adversary with a heart of stones He calls Shylock as an inhuman wretch without pity Shylock is quite lacking in the slightest quality of mercy (iv) How does Antonio reply to this

Antonio replies that he will meet the revenge of Shylock patiently He has prepared himself to suffer with a quiet spirit the utmost that Shylockrsquos tyranny and rage can do(v) What quality of the Duke is revealed here

The Duke is kind and benevolent He is ready to help Antonio He requests Shylock to free Antonio from the trial(vi) What are the terms of the bond that Antonio has signed

The terms of the bond that Antonio has signed were that if Antonio is unable to repay Shylock a certain sum of money specified on the paper on a certain date and in an agreed place the forfeit has to be paid The forfeiture will be an exact pound of Antoniorsquos flesh which Shylock will be a liberty to take from any part of Antoniorsquos body which pleases him

Class XI

STUDY MATERIAL

Class XISubject Eng Literature (The Tempest ndash William Shakespeare) Topic Act I Scene 2 Lines 88 to 132 (Prospero hellip Me and thy crying self) Date 27th April 2020 (3rd Period)

[Students should read the original play and also the paraphrase given in the school prescribed textbook]Summary Questions amp Answers

o Prospero now tells Miranda that he was the Duke of Milan He had been devoting himself more to studies than the affairs of the State His brother Antonio took advantage of this situation and with the help of Alonso the king of Naples seized upon him and her one midnight and shipped them in a frail bark so that they perished in the sea All this took place

(1)

MIRANDA I should sin (Line 118-132)

To think but nobly of my grandmother

Good wombs have borne bad sonsPROSPERO Now the condition

The King of Naples being an enemyTo me inveterate hearkens my brothers suitWhich was that he in lieu othrsquo premisesOf homage and I know not how much tribute

twelve years back

IMPORTANT PASSAGES EXPLAINED(Line 98-103)

PROSPERO helliphelliphelliphelliphelliphelliphellip

He being thus lorded

Not only with what my revenue yieldedBut what my power might else exact like oneWho having into truth by telling of it Made such a sinner of his memoryTo credit his own lie he did believeHe was indeed the duke

Prospero in telling the narrative of his past life here refers to his brother Antonio Prospero being with a studious bent of mind has left the administration of Milan on his younger brother Now Antonio being thus invested like a lord with all the powers derived from Prosperorsquos wealth and what the exercise of Prosperorsquos authority might secure for him regarded himself as a de facto Duke of Milan It is a well-known fact of psychology that a man who repeatedly tells a lie makes of his memory such a sinner against truth as to credit his own lie by the telling of it So Antonio by repeatedly saying to himself and others that he was the Duke came to believe that he was really the Duke Thus falsehood repeatedly asserted gained the force of truth for Antonio and he truly believed it

Should presently extirpate me and mine Out of the dukedom and confer fair MilanWith all the honours on my brother whereonA treacherous army levied one midnightFated to thrsquo purpose did Antonio openThe gates of Milan and ithrsquo dead of darkness The ministers for thrsquo purpose hurried thenceMe and thy crying self

(i) In the earlier lines of this scene what does Prospero tell about his intense interest What was the demand of his interest

In the earlier lines of this scene Prospero tells Miranda that he had an intense interest in the study of philosophy and magic arts Hence in order to improve his mind with this kind of study he kept himself isolated from worldly and state affairs His study was dearer to him than the applause and esteem that he could win from the public His study demanded too much solitude(ii) What forced Antonio to take an undue advantage over Prospero

Prosperorsquos indifferent attitude towards the statersquos affairs and his having boundless trust in Antonio gave rise to a boundless lust for power in Antoniorsquos mind Antonio felt that he must be the actual Duke instead of the part of the Duke he played Thus Antonio took an undue advantage of the situation to usurp Prosperorsquos dukedom(iii) Explain the following lines ldquoI should sin to think but nobly of my grandmother Good wombs have borne bad sonsrdquo

After hearing the treacherous act of her uncle Antonio Miranda says that Prosperorsquos mother was a noble lady and she cannot dishonour her memory by saying that the person named Antonio cannot be his (Prosperorsquos) brother She says that in honour of her grandmother she also cannot say that Antonio must have been begotten not by her grandfather but by some other man She finally concedes that it is known that good mothers have borne bad sons in their wombs and gave birth to them

(iv) Why did the King of Naples accept Antoniorsquos request to help him in usurping his dukedom What did Antonio propose to Alonso

Alonso who was the king of Naples accepted Antoniorsquos request in usurping Prosperorsquos dukedom because he (Alonso) was a sworn enemy of Prospero Antonio proposed that Alonso should immediately drive him (Prospero) and his offspring out of Milan and should confer the dukedom upon him (Antonio) with all the dignities which go with that

position In return Antonio promised that he shall give an annual tribute and also swore his allegiance to Alonso Also he agreed to hold the Dukedom of Milan as a subordinate to the state of Naples(v) How were Prospero and Miranda carried away from the city of Milan and what was the state of small Miranda at that time

In pursuance of the agreement settled between Antonio and Alonso an army of treacherous men was assembled One midnight when the occasion suited the will of destiny Antonio opened the gates of the city of Milan and in the death like silence of midnight Antoniorsquos agents who had been directed to execute his purpose carried Prospero and small Miranda away from the city in all haste They were then forced into a ship and carried some distance out to sea where they put them on a mere hulk of a boat without any rigging or ship-gear and abandoned them leaving them at the mercy of the roaring sea Miranda was a very small child of three years age and she was crying at that time

CLASS -XIDATE-270420Subject Topic Summary Execution

EVS Chapter 1 ndash Mode of Existence

Impact of mode of existence on resources

Q) Why resources are under pressure

Ans - Increase in the sophistication

of technology enabling natural resources to be extracted quickly and efficiently Eg in the past it could take long hours just to cut down one tree only using saws Due to increased technology rates of deforestation have greatly increased

The number of humans is increasing Cultures of consumerism Materialistic views

lead to the mining of gold and diamonds to produce jewelry unnecessary commodities for human life or advancement Consumerism also leads to extraction of resources for the

production of commodities necessary for human life but in amounts excessive of what is needed because people consume more than is necessary or waste what they have

Lack of awareness among the population is striking People are not aware of ways to reduce depletion and exploitation of materials

Accounts Cash Book Today we are going to start a new topic -Cash Book

The key terms used in this chapter are

bullCash book

bullSimple cash book

bullDouble column cash book bullTriple column cash book

bullPetty cash book

bullCash discount

bullContra entry

Here I will share you the meaning of each key terms

bullCash book Cash Book is a special purpose subsidiary book or journal in which cash received and cash payments are recorded

bullSimple cash book

It is a cash book in which only cash transactions are recorded It has only one column on each side

bullTriple column cash book

It is cash book which has three columns one column for each cash and Bankdiscount on each side of the cash book In this book both cash and Bank transactions are recorded together with discount allowed and received

bullPetty cash book

It is a cash book maintained for recording petty expenses

bullCash discount

Cash discount is the amount of discount received or allowed on cash payments and cash receipts Discount received is an income for the business while discount allowed isan expense

bullContra entry

It means transactions involving both cash and Bank Such transactions though recorded in the cash book are not posted into ledger The letter lsquoC is written in Ledger folio for contra entry

Business Studies

ENTREPRENEURSHIP

Now we shall discuss the second chapter

lsquoENTREPRENEURSHIPrsquo

Today before starting the chapter let us recall what

Questions

1What are the main characteristics of Intrapreneurship

Answer

The main characteristics of Intrapreneurship are

Corporate framework-it occurs within the framework of the same company

Semi-Autonomous-Intrapreneurship

we have read last day

Let s today start the class by recalling the last topic taught

Intrapreneurship is the process of discovering and exploring business opportunities within an existing company It involves launching new business ventures within the framework of a present corporation Intrapreneurship is also known as corporate entrepreneurship or corporate venturing

Now let us start with the characteristics of Intrapreneurship

The main characteristics of Intrapreneurship are

Corporate framework

Semi-Autonomous Lack of ownership Senior position Low risk taking Not own boss

Now let us discuss the meaning of enterprise

Enterprise means an undertaking or adventure that requires some innovation and investment and thus involves riskEnterprise always entails decision making coordination and risk bearing

involves crating amd nurturing a semi-autonomous business unit which may be a subsidiary a strategic business unit or a division

Lack of ownership-the intrapreneur is not the owner of the unitb he creates and nurtures

Senior position-he occupies a senior managerial position in the company

Low risk taking-An intrapreneur does not bear the full risk of failure

Not own boss-An intrapreneur is not his own bosss in legal termsHe enjoys the freedom and gets the required resources and support

2 How is Entrepreneur is different from Intrapreneur

The functions involved in both the entrepreneurship and intrapreneurship are by and large similar however there are several differences between the two

Point of distinction

Entrepreneur

Intrapreneur

status An independent business person

A senior executive within a company

Ownership Owner of

the business

An employeesometimes a share in ownership

Financing Responsible for raising finance for the business

Not responsible for raising the finance

Risk bearing

Bears the risk of the business

Does not bears the risk of the business

Reward Profit which is uncertain and irregularcan be loss

Fixed salary and fringe benefits

Need for security low high

3 What do you understand by enterprise

Answer Enterprise means an undertaking or adventure that requires some innovation and investment and thus involves riskEnterprise always entails decision making coordination and risk bearing

COMMERCE NATURE AND OBJECTIVES OF

BUSINESS

Today let us recall the last other two objectives of business by the chart given in the previous class

Firstly we would discuss Human Objectives

Business is run by people and for people Labour is a valuable business element

Human objectives of business are concerned with the well -being of labour

The human objectives are as follows

Labour welfare Developing human

resources Participative

management Labour

management cooperation

Questions

1 Explain the human objectives of a business enterprise

Answer

Business is run by people and for people Labour is a valuable business element

Human objectives of business are concerned with the well -being of labour

The human objectives are as follows

Labour welfare-Business must recognize the dignity of labour and human factors should be given the recognition

Developing human resources-Employees must be provided the opportunities for developing new skills and attitudes

Participative management-Employees should be allowed to take part in decision making process of business

Labour management cooperation-Business should strive for creating and maintaining cordial employer employee relations so as to ensure peace and progress in industry

Now let us discuss the national objectives of business

Optimum utilization of resources

National self- reliance Development of small

scale industries Development of

backward areas Control over pollution

2Explain the national objectives of a business enterprise

Answer

It is the duty of business to utilize the resources of the country properly the national objectives of business

Optimum utilization of resources ndashBusiness should use the nationrsquos resources in the best possible manner

National self- reliance-It is the duty of the business to help the government in increasing experts and in reducing dependence on imports

Development of small scale industries-Big business firms are expected to encourage growth of small scale industries which are necessary for generating employment

Development of backward areas-Business is expected to give preference to the industrialization of backward regions of the country

ECONOMICS

BASIC ECONOMIC CONCEPTS

SUB

TOPIC

Value

Wealth

Welfare

Today we shall start with a new topic of the same chapter ie lsquoValuersquo

Value of a commodity is defined as the valuation placed by a household on the consumption of this commodity

lsquoValuersquo has two different meanings and these are

a Value -in -use It refers to consumption value of a commodity It expresses the utility derived from the consumption of a particular commodity A necessity like water has a very high value ndashin ndashuse or

Question

1What is value

Answer

Value of a commodity is defined as the valuation placed by a household on the consumption of this commodity

2What is value-in use

Answer It refers to consumption value of a commodity It expresses the utility derived from the consumption of a particular commodity A necessity like water has a very high value ndashin ndashuse or consumption value

3What is value ndashin- exchange

Answer It relates to market value of a commodity

It is the rate at which a particular good or service can be exchanged for money

For example in barter system if a person is prepared to exchange 3 metres of cloth with 1 pair

consumption value

b Value ndashin-exchange It relates to market value of a commodity

It is the rate at which a particular good or service can be exchanged for moneyFor example in barter system if a person is prepared to exchange 3 metres of cloth with 1 pair of shoes then the value in exchange of 3 metres of cloth is 1 pair of shoesValue in exchange is the power of purchasing other goods In modern monetised economies the exchange value of goods are expressed in terms of money as prices

Now let us discuss the term lsquoWealthrsquo

Wealth refers to the stock of all those assets which are a source of income

Wealth is a stock concept

Wealth must possess the following features

a Utility It must possess utility or give some

of shoes then the value in exchange of 3 metres of cloth is 1 pair of shoes

Value in exchange is the power of purchasing other goods In modern monetised economies the exchange value of goods are expressed in terms of money as prices

4 What is wealth

Answer it refers to the stock of assets or goods which are a source of income and have personal or national ownership

5 What are the features of wealth

Answer The features of wealth are as follows

Wealth must possess the following features

a Utility It must possess utility or give some satisfaction

b Scarcity It must be limited in quantityc Transferability it should be transferable its

ownership can be transferred from one person to another person

d Exchange value It must possess exchange value

6 What is welfare

Answer

Welfare is defined as satisfaction and happiness a sense of well- being among the people

satisfactionb Scarcity It must be

limited in quantityc Transferability It

should be transferable its ownership can be transferred from one person to another person

d Exchange value It must possess exchange value

Now let us discuss the term lsquoWelfarersquo

Welfare is defined as satisfaction and happiness a sense of well- being among the people

Welfare is affected by factors like

a Consumption of goods and services

b Environment

c Family relations

d Degree of freedom

e Law and order situation

Mathematics Trigonometric equation

To find the general solution of the equation sinθ=0

When sin θ =0

Then θ= 0 π2π 3π-π -2π -3

i e when θ = 0 or an integral multiple of π

i e when θ= nπ where n is any integer

Therefore the general solution of the equation sin

Example1 Find the general values of θ which satisfy the equation sin2 θ =34

Solution sin2 θ= 34

Or sin θ = +34 or -34

Or sin θ = sin π3 or sin (-π3)

Therefore

θ = [nπ + (-1) n (π3)] or[ nπ+ (-1) n (-π3)]

= nπ +π3 or nπ-π3 where n= any integer

Example 2Find the values of θ which satisfy tan2 θ

θ=0 is θ= nπ where n is any integer

To find the general solution of the equation cos θ=0

When cos θ=0

Then θ=π2 3π2 5π2 -π2 -3π2 -5π3

i e when θ is an odd multiple of π2

i e when θ=(2n+1) π2 where n= any integer

Therefore the general solution of the equation cos θ =0 is θ= (2n+1) π2 where n= any integer

To find the general solution of the equation tan θ = 0

Clearly tan θ =0 implies sin θcos θ =0

Therefore θ = nπ

i e the general solution of the equation tan θ=0 is θ =nπ where n = any integer

To find the general solution of the equation cot θ =0

Clearly cot θ =0 implies (cos θsin θ) = 0

i e cos θ =0

Therefore θ = (2n+1) π2

Therefore the general solution of the equation cot θ =0 is θ = (2n+1) π2

Where n= any integer

To find the general solution of the equation sin θ= k (-

=13 -πleθleπ

Solution tan2 θ =13

Or tan θ = plusmn1radic(3) =tan(plusmnπ6)

θ=nπ plusmn π6 where n =any integer

If n=0 then θ=plusmnπ6

If n=1 then θ= π plusmn π6

If n=-1 then θ= -π plusmn π6

Therefore the required solution in -π le θ le π are θ= π6 5π6 -π6 -5π6

Exercise Find general solution of sin 2θ=cos θ [Hints Use sin 2θ= 2sin θcosθ and then take cosθ

common]

1lek le1)

Determine an angle alpha such that sin =k and -π2le αle π2

Then we have

Sin θ = k = sin α

Or sin θ - sin α =0

Or 2 cos [(θ+α) 2] sin [(θ-α) 2] =0

Therefore either cos [(θ +α) 2] =0 (1)

Or sin [(θ-α) 2] =0 (2)

Now from (1) we get (θ+α) 2= (2m+1) π2)

Or θ = (2m+1) π-α (3)

And from (2) we get (θ-α) 2 =mπ

Or θ= 2mπ+α(4)

Where m = any integer

Clearly the solution (3) amp (4) may be combined in the following form

θ= nπ+(-1) n α where n= any integer

Therefore the general solution of sin θ = sin α is θ = nπ +(-1) n α where n is any integer and -π2 le α le π2

Biology Chapter - 04Kingdom Monera

Today we will discuss about bacterial reproduction and its usefulness

Fig Binary Fission

Fig Conjugation Fig Transformation

Bacterial reproduction is mainly asexual but sexual reproduction

also takes place

Asexual reproduction takes place by i) Binary fission - from one bacteriato

two bacteria are produced in every 20 to 30mins

ii) Buddingiii) Endospore formation - during

unfourable condition

Sexual reproduction by three ways

1) Conjugation - Transfer of genetic material between cells that are in physical contact with one another

2) Transduction - Transfer of genetic materialfrom one cell to another by a bacteriophage

3) Transformation - Transfer of cell-freeor naked DNArsquo from one cell to another

Bacteria causes different diseases inplants animals and human and

it causes food spoilage and waterpollution but it also have some useful

activities

i) Bacteria are helpful in sewage water treatment

ii) It is used in antibiotic (medicine) production

iii) Anaerobic bacteria help in biogas(energy) production

iv) Many household products like yoghurt cheese are manufactured by use of bacteria

v) Rhizobium by symbiotic relationship with leguminous plant increase soil fertility

vi) Besides these bacteria is helpful in genetic engineering degradation of petroleum hydrocarbonand in dairy

industry

Physics Motion in plane Here we will introduce Projectile Motion

Execution

Projectile

Y

usinθ u h

θX

ucosθ

Suppose a body is projected with an angle θ So initial velocity u can be resolved into two components

Horizontal component - ucosθ ( for range)

Vertical component - usinθ ( for height)

usinθ changes during motion and becomes zero at maximum height position but ucosθ remain unchanged

The maximum height of projectile is h

NB If initial is upward then g = -ve and if it is downward then g = +ve Height is +ve if direction of motion does not change ( for ex a body thrown upwards but goes down ultimately then height h = -ve)

The angle of projectile θ is the angle made with horizontal

HISTORY ndash GROWTH OF NATIONALISM

SUB TOPIC- REVOLUTIONARY NATIONALISM Bengal formation of Anushilan Samity and Jugantar Group

The intensification of the Swadeshi movement and Government policy of terror and repression led to outbreak of violence Bombs were manufactured and attempts on the lives of unpopular Government officials became frequent In the gymnasium of Scottish Church College which was known as General Assemblies Institution a secret society was formed known as Anushilan Samity

Aurobindo Ghosh send from Baroda his emissary Jatindranath Banerjee to mobilize the Bengal revolutionaries

Hemchandra Qanungo and Satyen Bose published Journal Jugantar

The Jugantar group planned to assassinate oppressive magistrate Kingsford by Khudiram Bose and Prafulla Chaki in 1908 Prafulla Chaki committed suicide to avoid arrest Khudiram was tried and hanged

Afew days later the police found a bomb factory in Maniktala and arrested a large number of revolutionaries The trial of revolutionaries became famous as the Alipore Bomb Case

In the course of the trial the approver the public prosecuter and a police officer were assassinated

1 Question Name two journals which preached the cult of violence

Answer a) Yugantar edited by Bhupendranath DuttaB) Bandemataram edited by Aurobindo Ghosh2 Question Why was Khudiram arrested and hangedAnswer An attempt was made to assassinate a hated vindictive majistrate named Kingsford by Khudiram Bose and Prafulla Chaki Their attempt failed and the bomb they threw killed two English ladies Khudiram was arrested and put to trial and then hanged3Question Who was Aurubindo GhoshAnswer Aurobindo Ghosh a nationalist revolutionary who was charged for his involvement in the Alipore Bombing Case He was accused of it along with his brother Barindra nath Ghosh But Aurobindo was acquitted because of the brilliant pleading of his counsel Chittaranjan Das Then he became a spiritual reformer introducing his visions on human progress and spiritual evolution4 Qustion Who was KingsfordAnswer Kingsford was an unpopular British chief Magistrate who was the target of the bomb thrown at Muzaffarpur by Khudiram and Prafulla Chaki

Most of the accused were convicted and sentenced to

long term of imprisonmentBut

Aurobindo Ghosh was acquitted mainly owing to the brilliant pleading of his counsel Chittaranjan Das

Political science Topic-Sovereignty

Summary Sovereignty is the full right and power of a governing body over itself without any interference from outside sources or bodies In political theory sovereignty is a substantive term designating supreme legitimate authority over some polity In international law sovereignty is the exercise of power by a state

Internal Sovereignty

Internal sovereignty means supreme authority within ones territory while external sovereignty relates to the recognition on the part of all states that each possesses this power in equal measure

External sovereignty

external sovereignty relates to the recognition on the part of all states that each possesses this power in equal measure

Distinguish between

Execution

Answer the following questions

Short notes-

Sovereignty

Internal Sovereignty

External sovereignty

Homework- learn

external sovereignty and internal sovereigntySovereignty is the principle

of supreme and

unquestionable authority

reflected in the claim by the

state to be the sole author of

laws within its territory

Definition of external vs internal sovereigntyInternal sovereignty refers to

the relationship between a

sovereign power and its

subjects ndash it refers to the

location of the supreme

authority within the state In

the UK for example internal

sovereignty (supposedly)

resides within Parliament

reflected in the

constitutional principle of

parliamentary

sovereigntyBy contrast

external sovereignty refers

to the capacity of the state

to act independently and

autonomously on the world

stage This is what is

sometimes called lsquostate

sovereigntyrsquo or lsquonational

sovereigntyrsquo and implies

that states are legally equal

and that the territorial

integrity and political

independence of a state is

inviolable

Class ndash XII

Date - 2742020 STUDY MATERIALSubject Topic Summary Execution Business Studies

Job Analysis amp Manpower Planning

At first let us recall the chapter what we have discussed till nowbullJob analysisbullJob specification bullJob description bullJob enlargement bullJob enrichment

Today we will do some questions answers from the chapter

Questions 1ldquoJob analysis job description and job specification are interrelatedrdquo Comment Answer) Job analysis is a systematic and detailed examination of a job to collect all the relevant information about it The contents off the job are summarised in the job description The qualification needed for the job are summarised in job specificationThus there is close interrelationship between job analysis job description and job specification

Question 2ldquoJob enlargement is a horizontal extension of a job whereas job enrichment is a vertical extension of a jobrdquo ElucidateAnswer) Job enlargement involves adding one or more task to a job coma where as job enrichment involves adding more autonomy and responsibility to a job Job enlargement is therefore horizontal extension of a job coma whereas job enrichment is a vertical extension of a job

Question 3 )

What is manpower estimation Explain its quantitative and qualitative aspectsAnswer) Manpower estimation is the process by which management determines how an organisation should move from itrsquos current manpower positionto its desired manpower position There are two dimensions of Manpower estimation- quantitative and qualitative

Quantitative aspectThis aspect of Manpower estimation involves estimating the number of employees required in a future time period Workload analysis and workforce analysis are done to estimate the quantity of required manpower

Qualitative aspectThe estimate of the knowledge skills experience etc of required manpower is the qualitative aspect of Manpower estimation The quality of Manpower can be judged on the basis of job analysisand job specification

COMMERCE

CAPITAL-FIXED AND WORKING

Today let us start the class by discussing the sources of finance for different types of business firms

The term lsquocapitalrsquo refers to the investment made in the enterprise for the purpose of earning profits

Requirements of capital and sources of capital for different types of business firms are

1 Capital for sole proprietorship businessA sole proprietor operates at a small scale and thereforerequires a limited amount of capital

2 Capital for partnership firmCapital requirements as well as capital base of a partnership is bigger than that of a sole trader businessThe owned capital is contributed by the partners in an agreed ratio

3 Capital for joint stock companyA joint stock company generally requires large amount of capitalA public company can raise huge capital through issue of shares In addition to share capital it can utilize retained profits

Now let u discuss the meaning of Finance PlanningFinance planning is the process of estimation the financial requirements of an organization specifying the sources of firms and ensuring that enough funds are available at the right time

1 What do you mean by Finance PlanningAnswer Finance planning is the process of estimation the financial requirements of an organization specifying the sources of firms and ensuring that enough funds are available at the right time

2Discuss the role of financial planning of an enterpriseThe role of financial planning are as followsa A sound financial plan helps a business enterprise to avaid the problems of shortage and surplus of fundsbFinancial planning serves as a guide in developing a sound capital structure so as to maximize returns to shareholders c It helps in effective utilization of fundsd It provides policies and procedures for coordinating different functional areas or departments of businesse It enables the management to exercise effective control over the financial activities of an enterprisef It helps the company to prepare for facing business shocks and surprises in future

Mathematics

Continuity and differentiability

Recall Definition of ContinuityLet f(x) be a single valued function of x and x=a be a point in the domain of definition of the function The function is said to be continuous at x=a ifi) f(c) is defined ie f(x) has a definite finite value at x=cii) lim xrarra f(x) exists andiii) lim xrarra f(x) =f(a) In other words f(x) is said to be continuous at x=a if lim xrarra+ f(x)= lim xrarra- f(x) = f(a) Or f(a+0) =f(a-0) =f(a) Or lim hrarr0 f(a+h)= f(a) Algebra of continuous functionsNow we will study some algebra of continuous functions Theorem 1 Suppose f and g be two real functions continuous at a real number c Then(1) f + g is continuous at x = c(2) f ndash g is continuous at x = c(3) f g is continuous at x = c(4) (fg) is continuous at x = c (provided g (c) ne 0)

Example 1 Prove that every rational function is continuousSolution Recall that every rational function f is given byf(x)=[p(x) q(x) ] q(x)ne0where p and q are polynomial functions The domain of f is all real numbers except points at which q is zero Since polynomial functions are continuous f is continuous by (4) of Theorem 1Example 2Discuss the continuity of sine functionSolution To see this we use the following factslim xrarr0 sin x =0Now observe that f (x) = sin x is defined for every real number Let c be a real number Put x = c + h If x rarr c we know that h rarr 0 Therefore lim xrarrc f(x)

= lim xrarrc sin x= lim hrarr0 sin(c+h) =lim hrarr0 [sin c cos h + cos c sin h ]=lim hrarr0 (sin c cos h) + lim hrarr0 (cos c sin h) = sin c +0=sin c = f(c) Thus lim xrarrc f(x) = f(c) and hence f is a constant function Exercise Prove that the function f(x) = x2 +2x is continuous for every real value of x [Hints show that lim xrarra+ f(x) = lim xrarra- f(x) = f(a) ]

Biology Reproduction in Flowering plants We will discuss about megasporoangium

megasporagenesis and female gametophyte

Q4 Describe the structure of megasporangium

Ovule is attached to the placenta by astalk called funicle

Each ovule has one two or three protectivecoverings called integuments

At the tip of integuments a small openingcalled micropyle is organised

Opposite to the micropylar end is the chalaza

Within the integuments a mass of cellsnucellusand inside it embryo sac orfemale gametophyte is present

Q5 Describe a mature embryosacamp its formation

In most of the flowering plants only oneof the 4 megaspores formed as a result ofmegasporogenesis that is functional while theother three degenerate

The

functional megaspore develops into thefemale gametophyte

Formation The nucleus

of the functional megasporedivides mitotically to form two nuclei first andthen two more sequential mitotic nucleardivisions result in the formation of four ampthen eight nucleate stages of embryo sac

Six of the eight nucleus are surrounded bycell walls and organised into cells

The remaining two nuclei called polar nuclei are found below the egg apparatus in the largecentral cell

Three cells consisting of two synergids amp one egg cell present bottom of

embryo sac Three cells

at the chalazal as antipodal cells

Two polar nuclei together present in large central cell

HISTORY

TOWARDS INDEPENDENCE AND PARTITION THE LAST PHASE(1935-1947)SUB TOPIC NATIONAL MOVEMENTS DURING THE SECOND WORLD WAR

Spread of Quit India Movement On 9th August 1942Gandhiji and other Congress leaders were arrested The Congress was declared illegal The news of the arrest of all leaders marked the beginning of a widespread movement of India It was not possible for such a movement to remain peacefulBut the arrest of the all notable congress leaders virtually left the movement in the hands of the mass The movement took the form of violent and militant outbreakBesides congressmen revolutionaries also were very active in the movement The Congress Socialist group also played a prominent role

1 Question Why did the British authority arrest the Congress leaders on 9 th August 1942Answer Congress Working committee adopted the Quit India resolution which was to be ratified at the Bombay AICC meeting in 8th August 1942 They decided to launch a mass struggle on non-violent lines Gandhiji gave a clarion call to all section of the people rdquoKarenge ya Marengerdquo (do or die) Congress leaders gave the call to driving out

the British from IndiaViceroy had taken strong action against the Quit India movement Gandhiji and all the leaders of Congress were arrested

2 Question How did Quit India Movement spread out all over IndiaAnswer The news of the leaders lsquo arrest marked the beginning of a widespread movement to remain peacefulThe movement took form of violent outbreak There were widespread cutting of telephone and Telegraph wires damaging railway lines raising barricades in cities and towns and other forms of violent demonstations

Question Name the leaders of Congress

Socialist group played a prominent part Notable among the Jayprakash Narayan Rammonohar Lohia Aruna Asaf Ali

Political science

Topic-Franchise and Representation

Summary

The election commission

The Election Commission of India is an autonomous constitutional authority responsible for administering Union and State election processes in India The body administers elections to the Lok Sabha Rajya Sabha State Legislative Assemblies in India and the offices of the President and Vice President in the country

Functions of election commission-

India is a sovereign socialist secular democratic republic Democracy runs like a golden thread in the social economic and political fabric woven by the Constitution given by lsquoWe the People of Indiarsquo unto ourselves The concept of democracy as visualised by the Constitution pre-supposes the representation of the people in Parliament and State legislatures by the method of election The Supreme Court has held that democracy is one of the inalienable basic features of the Constitution of India and forms part of its basic structure The Constitution of India adopted a Parliamentary form of government Parliament consists of the President of India and the two Houses mdash Rajya Sabha and Lok Sabha India being a Union of states has separate state legislatures for each state State legislatures consist of the Governor and two Houses mdash Legislative Council and Legislative Assembly mdash in seven states namely Andhra Pradesh Telangana Bihar Jammu amp Kashmir Karnataka Maharashtra and Uttar Pradesh and of the Governor and the state Legislative Assembly in the remaining 22 states Apart from the above two out of the seven Union Territories namely National Capital Territory of Delhi and Puducherry also have their Legislative Assemblies

ExecutionShort notes-Election commissionFunctions of election commission

Homework- Learn

Computer

Science

Computer hardware NAND Gate

A NOT-AND operation is known as NAND operation It has n input (n gt= 2) and one output

Logic diagram

Truth Table

NOR Gate

A NOT-OR operation is known as NOR operation It has n input (n gt= 2) and one output

Logic diagram

Truth Table

XOR Gate

XOR or Ex-OR gate is a special type of gate It can be used in the half

adder full adder and subtractor The exclusive-OR gate is abbreviated as EX-OR gate or sometime as X-OR gate It has n input (n gt= 2) and one output

Logic diagram

Truth Table

XNOR Gate

XNOR gate is a special type of gate It can be used in the half adder full adder and subtractor The exclusive-NOR gate is abbreviated as EX-NOR gate or sometime as X-NOR gate It has n input (n gt= 2) and one output

Logic diagram

Truth Table

Physics

Chapter 1 Electric Field ( Electric Dipole) (Summary)

Here we will derive Expression of electric field at broad side

On position of dipole

Execution

Q With the help of a labelled diagram obtain an expression for the electric field intensity E at any point on the equitorial line ( broad-side on position) of an electric dipole

Ans

E1 E1sinθ

E θ P E1 θ

( r2+L2)12 E2 E

r E2 E2sinθ

-q θ L O L +qA B

Let us consider that the point P is situated on the right bisector of the dipole AB at a distance r meter from its midpoint O

Let E1 and E2 be the electric field intensities of the electric field at P due to charge +q and ndashq of the dipole resp The distance of P from each charge is ( r2+L2)12

So E1 = 14 πϵ q

(r 2+L 2) away from +q

E2 = 14 πϵ q

(r 2+L 2) towards ndashq

The magnitudes of E1 and E2 are equal but directions are different Now resolving E1 and E2 into two components parallel and perpendicular to AB we get

The components perpendicular to AB E1sinθ and E2sinθ cancel each other because they are equal and opposite

The components parallel to AB are E1cosθ and E2 cosθ are in same direction and add up

So resultant intensity of electric field at the point P is

E = E1cosθ + E2 cosθ

E = 14 πϵ q

(r 2+L 2) 2 cosθ

Now from fig we have cosθ =BOBP = L (r2+L2)12

So we get E = 14 πϵ 2qL ( r2+L2)32

Now electric dipole moment p= 2qL

So E = 14 πϵ p ( r2+L2)32

HW Find the expression of Electric field as done here but this time take r gtgt 2L

Also find the expression of torque experience by a dipole

(Hint Electric force experienced by charges of dipole in electric field is qE each Let θ be the angle which dipole makes with electric lines of force then perpendicular distance between two charges is 2Lsinθ Then torque = force x perp distance = qE x 2L sinθ So τ=pE sinθ where p =2qL )

STUDY MATERIAL

Class XIISubject Eng Literature (The Tempest ndash William Shakespeare) Topic Act IV Scene 1 Lines 84 to 133 (Iris hellip A contract of true love Be not too late ) Date 27th April 2020 (4th Period)

[Students should read the original play and also the paraphrase given in the school prescribed textbook]Summary Questions amp Answers

o Ceres soon appears and comes to know that she has been summoned to celebrate the contract of true love

o Ceres expresses her unwillingness to meet Venus and Cupid as she has shunned their company

o Ceres and Juno both bestow their blessings upon Ferdinand and Miranda with June gifting honour riches happiness in marriage and Ceres presents plenty of earthrsquos produce

o Iris summons the water-nymphs and reapers to come and celebrate a contract

(1) IRIS Of her society (Line 91-101)

Be not afraid I met her deity

Cutting the clouds towards Pathos and her sonDove-drawn with her Here thought they to have doneSome wanton charm upon this man and maidWhose vows are that no bed-right shall be paidTill Hymens torch be lightedmdashbut in vainMarss hot minion is returned againHer waspish-headed son has broke his arrowsSwears he will shoot no more but play with sparrowsAnd be a boy right out

(i) Where were Venus and Cupid seen flying How were they travelling Why did they want to join the marriage celebration of Ferdinand and Miranda

of true love

Venus and Cupid were seen flying through the air towards Paphos the famous city which is situated on the island of Cyprus They were travelling by air-borne chariot drawn by doves They certainly wanted to come here in order to play some amorous trick upon Ferdinand and Miranda who are under a vow not to gratify their physical desires till the holy ceremony of their marriage has been performed(ii) What have Venus and Cupid done after failing in their plan

After being failure of their plan Venus who is a very passionate deity and who is the mistress of Mars (the god of war) has gone back while here ill-tempered son Cupid has broken his arrows of love in his state of desperation(iii) What has Cupid firmly decided

Cupid is feeling so disappointed that he has firmly decided to shoot no more arrows to arouse love in human hearts but to spend his time playing with sparrows Thus he would now become just a boy and would give up his original function of shooting arrows on human beings to make them fall in love(iv) What vow had Ceres taken How did Ceres feel at the abduction

After the abduction of her daughter Prosperina by Pluto Ceres had taken a vow to always keep away from the disgraceful company of Venus and her blind son Cupid the god of love Ceres felt deeply distressed when Pluto had carried off her daughter and had made her his wife by force(v) Why has Ceres not forgiven Venus and her blind son For what do Ceres want to be sure

As the abduction had been manipulated by Venus the goddess of beauty and love and her blind son Cupid Ceres has never forgiven them for their part in the whole plot Ceres wants to be sure that she would not have to meet Venus and Cupid who had engineered the abduction of her daughter Prosperina

AS THIS lsquoMASQUErsquo SCENE IS VERY IMPORTANT IN THE PLAY THE PARAPHRASE OF THE ENTIRE PORTION OF MASQUE SCENE (Act IV Lines 58 to 143) IS GIVEN BELOW

IRIS Goddess of RainbowCERES Goddess of Agriculture and all the fruits of the earth

(Nature growth prosperity rebirth ndash notions intimately connected to marriage)JUNO The majestic Queen of Heavens and wife of Jupiter (Jupiter is the king of Gods)

VENUS The Goddess of love CUPID Son of Venus PLUTO God of death (In the play referred by Shakespeare as lsquoDisrsquo which is a Roman name for Pluto)

ORIGINAL TEXT PARAPHRASEPROSPEROWellmdash

PROSPERONow come Ariel Let there be too many rather than too few

Now come my Ariel Bring a corollaryRather than want a spirit Appear and pertly[to Ferdinand and Miranda]No tongue all eyes Be silent

spirits in attendance Appear briskly

[to Ferdinand and Miranda]Look with your eyes but do not say a word

[Soft music] [Soft music][Enter Iris] [Enter Iris]

IRISCeres most bounteous lady thy rich leasOf wheat rye barley vetches oats and peasThy turfy mountains where live nibbling sheepAnd flat meads thatched with stover them to keepThy banks with pioned and twilled brimsWhich spongy April at thy hest betrimsTo make cold nymphs chaste crowns and thybroom-grovesWhose shadow the dismissegraved bachelor lovesBeing lass-lorn thy pole clipped vineyardAnd thy sea-marge sterile and rocky-hardWhere thou thyself dost airmdashthe Queen othrsquoSkyWhose watery arch and messenger am IBids thee leave these and with her sovereign grace[Juno appears] Here on this grass-plot in this very placeTo come and sport Her peacocks fly amainApproach rich Ceres her to entertain

IRISCeres most generous lady you are the cause of rich fields or fertile land where wheat rye barley beans oats and peas grow the grassy mountains where the sheep graze and the flat meadows covered with coarse hay to be used as fodder for cattleYour banks are covered with marsh-marigolds and reeds and the rainy April under your orders brings forth to make for the maids who are not in love beautiful crowns your woods where the broom flourishes and where the bachelor who has been dismissed by the maid he loved lies down being forsaken your vineyard in which the poles are embraced by the vines and the margin of the sea which is barren and rocky where you roam about to enjoy the fresh air ndash the queen of the sky (Juno) whose messenger I am besides being represented as the rainbow bids you leave all these and with her majesty here on this grassy plot in this very place come and sport her peacocks carry her fast in her chariot through the air and are making their way here approach rich Ceres to welcome her

[Enter Ariel as Ceres] [Enter Ariel as Ceres]

CERESHail many-coloured messenger that neerDost disobey the wife of JupiterWho with thy saffron wings upon my flowersDiffusest honey-drops refreshing showersAnd with each end of thy blue bow dost crownMy bosky acres and my unshrubbed downRich scarf to my proud earth Why hath thy queenSummoned me hither to this short-grassed green

CERESWelcome rainbow that never dared disobey Juno the wife of Jupiter who with your orange coloured rays spread honey-drops refreshing showers And with each end of thy blue bow drown my bushy acres and my hilly country which is free from shrubs you thus forming a rich scarf Why has your queen called me here to this place covered with short grass

IRISA contract of true love to celebrateAnd some donation freely to estateOn the blest lovers

IRISI have called you to celebrate a contract of true love and bestow some liberal gift upon the blessed lovers

ORIGINAL TEXT PARAPHRASECERESTell me heavenly bowIf Venus or her son as thou dost knowDo now attend the queen Since they did plotThe means that dusky Dis my daughter gotHer and her blind boys scandaled companyI have forsworn

CERESTell me heavenly bow if Venus the Goddess of love or Cupid her son and pedlar of passion at this time attend the heavenly queen Juno because you are sure to know Since the day they conspired against me and dark Pluto took away my daughter here and Cupidrsquos disgraceful company I have left off

IRISOf her societyBe not afraid I met her deityCutting the clouds towards Pathos and her sonDove-drawn with her Here thought they to have doneSome wanton charm upon this man and miad

IRISBe not afraid of her company I met her deity moving on the clouds towards Paphos the sacred home of Venus on the island of Cyprus along with her son on her chariot drawn by doves Here they contemplated to exercise a charm upon this man and maid producing

Whose vows are that no bed-right shall be paidTill Hymens torch be lightedmdashbut in vainMarss hot minion is returned againHer waspish-headed son has broke his arrowsSwears he will shoot no more but play with sparrowsAnd be a boy right out

wantonness before the actual marriage ceremony but did not succeed Venus has returned her irritable son has broken his arrows and swears that he will give up his practice of trying to inspire love but play with sparrows and be a boy again

[Music is heard] [Music is heard]

CERESHighst queen of stateGreat Juno comes I know her by her gait

CERESHighest queen of state Great Juno there she comes I know here by her gait

[Enter Juno] [Enter Juno]

JUNOHow does my bounteous sister Go with meTo bless this twain that they may Prosperous beAnd honoured in their issue

JUNOHow are you doing my generous sister Come with me to bless this couple so that they may be prosperous and fortunate in their children

[They sing] [They sing]

JUNOHonour riches marriage-blessingLong continuance and increasingHourly joys be still upon youJuno sings her blessings upon you

JUNOMay honour riches happiness in marriage long continuance and increase of those boons ever rest upon you as hourly joys Juno showers down upon you her blessings in song

CERESEarths increase foison plentyBarns and garners never emptyVines and clustring bunches growingPlants and goodly burden bowingSpring come to you at the farthestIn the very end of harvestScarcity and want shall shun youCeresrsquo blessing so is on you

CERESMay you have the plenty of earthrsquos produce Your barns and granaries may never be empty Your vines may grow with clustering bunches Your fruit trees may be heavily laden with their fruit May there be continuous spring and harvest May scantiness and want leave you forever Such is the blessing of Ceres upon you

FERDINANDThis is a most majestic vision andHarmoniously charmingly May I be boldTo think these spirits

FERDINANDThis is a great vision and magically melodious Should I suppose the characters (taking part in the masque) are spirits

PROSPEROSpirits which by mine artI have from their confines calld to enactMy present fancies

PROSPEROYes they are spirits whom I have summoned from the regions to which they are confined to carry into effect my fanciful designs

ORIGINAL TEXT PARAPHRASEFERDINANDLet me live here everSo rare a wondered father and a wifeMakes this place paradise

FERDINANDI should like to live here forever Such a wise and wonderful father makes this place a paradise

[Juno and Ceres whisper and send Iris on employment] [Juno and Ceres whisper and send Iris on employment]

PROSPEROSweet now silence

PROSPEROMy dear Ferdinand speak no more Juno and Ceres are

Juno and Ceres whisper seriouslyTheres something else to do Hush and be muteOr else our spell is marred

whispering with a solemn look There is something else coming Silence Or else our magic will be spoilt

IRISYour nymphs called naiads of the wandering brooksWith your sedged crowns and over-harmless looksLeave your crisp channels and on this green landAnswer your summons Juno does commandCome temperate nymphs and help to celebrateA contract of true love Be not too late

IRISYou nymphs called Naiads denizens (M inhabitants) of the running stream with your chaplets of sedge and ever-helpful looks leave your wrinkled channels and on the green land answer the summons sent to you Juno has ordered some chaste nymphs and help to celebrate a noble and true marriage Donrsquot delay

[Enter certain nymphs] [Enter certain nymphs]You sunburnt sicklemen of August wearyCome hither from the furrow and be merryMake holiday your rye-straw hats put onAnd these fresh nymphs encounter every oneIn country footing

You sunburnt harvesters weary from the effects of the heat in August come here from the furrowed land and rejoice Make holiday with your rye-straw hats upon you and meet these fresh nymphs and join in country dancing

[Enter certain reapers properly habited They join with the nymphs in a graceful dance towards the end whereof Prospero starts suddenly and speaks]

[Enter certain reapers properly habited They join with the nymphs in a graceful dance towards the end whereof Prospero starts suddenly and speaks]

PROSPERO[aside] I had forgot that foul conspiracyOf the beast Caliban and his confederatesAgainst my life The minute of their plotIs almost come [to the spirits]Well done Avoidno more

PROSPERO(Aside)I had forgotten the wicked conspiracy of the beast Caliban and his accomplices against my life the time of their plot has almost arrived ndash (To the Spirits) well done depart no more of this

[To a strange hollow and confused noise the spirits heavily vanish]

[The spirits depart]

ORIGINAL TEXT PARAPHRASEFERDINANDLet me live here everSo rare a wondered father and a wifeMakes this place paradise

FERDINANDI should like to live here forever Such a wise and wonderful father makes this place a paradise

[Juno and Ceres whisper and send Iris on employment] [Juno and Ceres whisper and send Iris on employment]

PROSPEROSweet now silenceJuno and Ceres whisper seriouslyTheres something else to do Hush and be muteOr else our spell is marred

PROSPEROMy dear Ferdinand speak no more Juno and Ceres are whispering with a solemn look There is something else coming Silence Or else our magic will be spoilt

IRISYour nymphs called naiads of the wandering brooksWith your sedged crowns and over-harmless looksLeave your crisp channels and on this green landAnswer your summons Juno does commandCome temperate nymphs and help to celebrateA contract of true love Be not too late

IRISYou nymphs called Naiads denizens (M inhabitants) of the running stream with your chaplets of sedge and ever-helpful looks leave your wrinkled channels and on the green land answer the summons sent to you Juno has ordered some chaste nymphs and help to celebrate a noble and true marriage Donrsquot delay

[Enter certain nymphs] [Enter certain nymphs]You sunburnt sicklemen of August wearyCome hither from the furrow and be merryMake holiday your rye-straw hats put onAnd these fresh nymphs encounter every oneIn country footing

You sunburnt harvesters weary from the effects of the heat in August come here from the furrowed land and rejoice Make holiday with your rye-straw hats upon you and meet these fresh nymphs and join in country dancing

[Enter certain reapers properly habited They join with the nymphs in a graceful dance towards the end whereof Prospero starts suddenly and speaks]

[Enter certain reapers properly habited They join with the nymphs in a graceful dance towards the end whereof Prospero starts suddenly and speaks]

PROSPERO[aside] I had forgot that foul conspiracyOf the beast Caliban and his confederatesAgainst my life The minute of their plotIs almost come [to the spirits]Well done Avoidno more

PROSPERO(Aside)I had forgotten the wicked conspiracy of the beast Caliban and his accomplices against my life the time of their plot has almost arrived ndash (To the Spirits) well done depart no more of this

[To a strange hollow and confused noise the spirits heavily vanish]

[The spirits depart]

Ac-12 27420 topic Revaluation of Assets and Liabilities

REVALUATION OF ASSETS AND LIABILITIES

On admission of a new partner the firm stands reconstituted and consequently the assets are revalued and liabilities are reassessed It is necessary to show the true position of the firm at the time of admission of a new partner If the values of the assets are raised gain will increase the capital of the existing partners Similarly any decrease in the value of assets ie loss will decrease the capital of the existing partners For this purpose alsquoRevaluation Accountrsquo is prepared This account is credited with all increases in the value of assets and decrease in the value of liabilities It is debited with decrease on account of value of assets and increase in the value of liabilities The balance of this account shows a gain or loss on revaluation which is transferred to the existing partnerrsquos capital account in existing profit sharing ratioAccounting for Revaluation of Assets and Liabilities when there is a Changein the Profit Sharing Ratio of Existing PartnersAssets and liabilities of a firm must also be revalued at the time of change in profit sharing ratio of existing partners The reason is that the realisable or actual value of assets and liabilities may be different from those shown in the Balance Sheet It is possible that with the passage of time some of the assets might have appreciated in value while the value of certain other assets might have decreased and no record has been made of such changes in the books of accounts Similarly there may be some unrecorded assets amp libilities that may have to be accounted for Revaluation of assets and reassessments of liabilities becomes necessary because the change in the

value of assets and liabilities belongs to the period to change in profit sharing ratio and hence must be shared by the partners in their old profit sharing ratio Revaluation of assets and reassessment of liabilities may be given effect to in two different ways (a) When revised values are to be recorded in the books and(b) When revised values are not to be recorded in the books

When revised values are to be recorded in the booksIn such a case revaluation of assets and reassessment of liabilities is done with the help of a new account called lsquoRevaluation Accountrsquo Sometimes this account is also called as lsquoProfit amp Loss Adjustment Acrsquo If there is a loss due to revaluation revaluation account is debited and if the revaluation results in a profit the revaluation account is credited The following journal entries made for this purpose are

(i) For increase in the value of assetsAsset Ac Dr (individually)To Revaluation Ac(ii) For decrease in the value of AssetRevaluation Ac Dr (individually)To Asset Ac[Decrease in the value of assets](iii) For increase in the value of LiabilitiesRevaluation Ac Dr (individually)To Liabilities Ac[Increase in the value of Liabilities](iv) For decrease in the value of LiabilitiesLiabilities Ac DrTo Revaluation Ac[Decrease in the value of Liabilities](v) For unrecorded AssetsAsset Ac [unrecorded] DrTo Revaluation Ac[Unrecorded asset recorded at actual value](vi) For unrecorded Liability Revaluation Ac DrTo Liability Ac [unrecorded][Unrecorded Liability recorded at actual value](vii) For transfer of gain on revaluationRevaluation Ac DrTo Existing Partnerrsquos CapitalCurrent Ac[Profit on revaluation transferred to capital account in existing ratio](viii) For transfer of loss on revaluationExisting Partnerrsquos CapitalCurrent Ac DrTo Revaluation Ac[Loss on revaluation transferred to capital account in existing ratio](a) When revaluation account shows gain Revaluation Ac DrTo Partnerrsquos Capital Ac (Old Profit Sharing Ratio)(Profit on revaluation credited to Partnerrsquos Capital Ac)(b) Above entry is reversed when revaluation account shows loss Partners Capital Acs (Old Profit Sharing Ratio) DrTo Revaluation Ac(Loss on revaluation debited to Partnerrsquos Capital Acs)

Proforma of Revaluation Account is given as under

Revaluation Account

Dr Cr Particulars ` Amount Particulars ` Amount To Decrease in value of assets By Increase in value of assets To Increase in value of liabilities By Decrease in value of liabilities To Unrecorded liabilities By Unrecorded assets To Gain on Revaluation (Transferred) By Loss on Revalution (Transferred)

ECO ndash12 2742020Topic- ELASTICITY OF DEMAND

CHAPTER - ELASTICITY OF DEMANDMEANINGDemand for a commodity is affected by many factors such as its price price of related goods income of its buyer tastes and preferences etc Elasticity means degree of response Elasticity of demand means degree of responsiveness of demand Demand for a commodity responds to change in price price of related goods income etc So we have three dimensions of elasticity of demandDIMENSION OF ELASTICITY OF DEMAND TYPES OF ELASTICITY OF DEMAND

Price elasticity of demand Income elasticity of demand Cross Elasticity of demand

Price elasticity of demand Price elasticity of demand means degree of responsiveness of demand for a commodity to the change in its price For example if demand for a commodity rises by 10 due to 5 fall in its price Price elasticity of demand (ep)=Percentage change in quantity demanded Percentage change in price of the commodity = 10 ( -)5 = ( - )2Note that ep will always be negative due to inverse relationship of price and quantity demanded

(ii) Income elasticity of demand Income elasticity of demand refers to the degree of responsiveness of demand for a commodity to the change in income of its buyer Suppose income of buyer rises by 10 and his demand for a commodity rises by 20 then Income elasticity of demand (ey)= change in quantity demanded change in price of the commodity =20 10 = 2

Cross Elasticity of demandCross elasticity of demand means the degree of responsiveness of demand for a commodity to the change in price of its related goods (substitute goods or complementary goods) Suppose demand for a commodity rises by 10 due to 5 rise in price of its substitute good then Cross elasticity of demand (ec) = change in quantity demanded change in price of related good = 10 2 = 5 (Tastes and preferences cannot be expressed numerically So elasticity ofdemand cannot be numerically expressed)

  • Chapter 1 Force (Summary)
  • Distinguish between external sovereignty and internal sovereignty
    • NAND Gate
      • Logic diagram
      • Truth Table
        • NOR Gate
          • Logic diagram
          • Truth Table
            • XOR Gate
              • Logic diagram
              • Truth Table
                • XNOR Gate
                  • Logic diagram
                  • Truth Table
                      • Physics
                      • Chapter 1 Electric Field ( Electric Dipole) (Summary)
Page 16:  · Web viewWe all know that Nouns are divided into two parts: common noun and proper noun.Apart from common and proper noun, we will also study about collective noun and compound

Generally a preposition comes before an object Sometimes it comes even after an object asWhat are you looking atThis is the house I live in

Subject- Computer Application

Chapter 2 Introduction to Java

Java API An application programming interface (API) in the context of Java is a collection of prewritten packages classes and interfaces with their respective methods fields and constructors

Byte Code Java bytecode is the result of the compilation of a Java program an intermediate representation of that program which is machine independent The Java bytecode gets processed by the Java virtual machine (JVM) instead of the processor JVM The Java Virtual Machine (JVM) is the runtime engine of the Java Platform which allows any program written in Java or other language compiled into Java bytecode to run on any computer that has a native JVM

Platform A platform is the hardware or software environment in which a program remains

Java platform The Java platform differs from most other platforms in that itrsquos a software only platform that runs on top of other hardware-based platforms

Applet and Application The fundamental difference between the two Java programs is that an application program is designed to run on a stand-alone machine whereas an applet is a web-version of an application which is used to run a program on a web browser

WORA ldquoWrite once run anywhererdquo (WORA) or sometimes write once run everywhere (WORE) is a slogan created by Sun Microsystems to illustrate the cross-platform benefits of the Java language

Class XSubject Topic Summary Execution

ECONOMICSFACTORS OF PRODUCTION Sub-topiclsquoLABOURrsquo

We shall start our class by discussing the topic taught in the last class

lsquo DIVISION OF LABOURrsquo- By division of labour we mean specialization in workIt refers to splitting up the work of labour involved in the production of a particular commodity into several parts and each part and sub-part is performed by a specialist

Now let us start by the Advantages of Division of labouraIt increases the level

Questions

1 Differentiate between Product-Based division of labour and Process-Based division of labour

Product-Based division of labour

Process-based division of labour

It is also known as simple (or occupational) division of labourUnder it everybody performs a particular occupations The entire is done by the same person

When a person or group of persons undertakes a specialised function which is supplementary to the production of final commodity and service This is also as complex division of labour

It is simple It is complex

of productionbSince the product is produced by an expert workerbest quality of product is producedcIt saves time and toolsd it promotes inventions in the methods and techniques of productioneIt leads to reduction in costs fAll workers get work according to their abilities and choices

Now let us discuss the disadvantages of Division of labour

a Since many workers are involved in the production of a commodity no one has the sense of responsibility

b The constant and repetition of the same work again and again make the work monotonous

c Division of labour facilitates production on large scale Hencethere is fear of over production

d Because of territorial division of labour some areasregions become more developed than others

It is based on labour-intensive techniques of production

It is based on capital-intensive techniques

It is generally found in small enterprises

It is generally found in large enterprises

Example Indian farmers doing all farm activities

Example A modern garmet factory where one person takes the measurementanother does the cuttingsome sew the clothes while a few workers button them and other iron them

2 What are the advantages of Division of labouraIt increases the level of productionbSince the product is produced by an expert workerbest quality of product is producedcIt saves time and toolsd it promotes inventions in the methods and techniques of productioneIt leads to reduction in costs fAll workers get work according to their abilities and choices

3Discuss the disadvantages of Division of laboura Since many workers are involved in the

production of a commodity no one has the sense of responsibility

b The constant and repetition of the same work again and again make the work monotonous

c Division of labour facilitates production on large scale Hence there is fear of over production

d Because of territorial division of labour some areasregions become more developed than others

English 1 Transformation of sentences

Sentences A sentence is a group of words which makes complete sense

a Assertive sentences

Exercise 6Rewrite the following sentences according to the instructions given below without changing their meanings

1 As soon as he saw the beer he jumped into

b Imperative sentences

c Interrogative sentences

d Exclamatory sentences

Sentences can be changed from one grammatical form to another without changing the meaning of the sentence This is known as transformation of sentences

the river ( Begin No sooner)2 None but brave deserve the fair (Begin the

bravehellip)3 This box is too heavy for me to lift ( Use so hellip

That instead of too)4 No one other than a king can live like James

Luxurious ( Begin only James)5 Oh for the wings of a dove (Begin I wishhellip)

Math Topic Commercial MathematicsChapter Shares and Dividends

Study item Discuss about shares and Dividends1) What is share

Ans To start any big business (company or Industry) a large sum of money is needed But it is not possible for an individual to invest such a large amount Then some persons interested in the business join together and from a company They divide the estimated money required into small parts Each such part is called a share

2) What do you mean by the term shareholder

Ans A person who purchases one or more shares is called shareholder3) Some terms related with a share

(i) Nominal value or face value or printed value The original value of a share is called its nominal value or face value or printed value

Note The nominal value of a share always remains same(ii) Market value or cash value The price of a share at any

time is called its market value or cash value

Note The market value of a share changes from time to time(iii) At par If the market value of a share is the same as its

nominal value the share is called at par(iv) At Premium or above Par If the market value of a share

is more than its nominal value the share is called at premium or above par

Example If a share of Rs 100 is selling at Rs 150 then it is said to be selling at a premium of Rs 50 or Rs 50 above par

(v) At Discount or below par If the market value of a share is less than its nominal value the share is called at discount or below par

Example If a share of Rs 100 is selling at Rs80 then it is said to be selling at a discount of Rs 20 or at Rs 20 below par

4) What is Dividend

Ans The profit which a shareholder gets for hisher investment from the company is called dividendNote (i) The dividend is always expressed as the percentage of the face value of the share(ii) The dividend is always given( by the company ) on the face value of the share

irrespective of the market value of the shareBENGALI(2ND LANGUAGE)

ldquoদেবতোর জণমrdquoলিবরোম চকরবত

পরথম লিসর পোঠ-চোর পসথ একটি পোথর লিবপলি ঘটোয় দেক যোতোয়োসতর পসথ পরলিতলিয়ত ওই পোথসর দেো োচট দেসত একলি দেতো দেক দেো োচট দেসয় দেবোমো সয় রোসতোর মোস লি0টসক পস1 লিবপরীত লিক দেথসক আো একটি দেমোটর োলি1 চোসকর কষতোয় পরোসরণ দেবোসচ যো লিকনত পরলিতবোর এমরণ দেৌভোয দেসকর োও সত পোসর তোই লিতলি দেকোো দেজোো1 কসর পোথরটিসক উপস1 দে8স পোথর উপস1 দে8োর ময় এক দেকৌতী জতো দেকসক পরশন কসর দেয লিতলি দেকোসো দেবতোর আস দেপসয়স0রণ লিকো লিকনত দেক বস লিতলি দেকোসো দেবতোর আস পোলি দেক উপলিত ক জতোর উসltসয বস কোরও ইস= স পোথরটি লিসয় দেযসত পোসর এর পর দেথসক দেক দেযসত আসত পোথরটি দেক দেসত পো একলি দেক কষয করস দেকউ পোথরটিসক ধসয়মস0 পলিরসকোর করস0 দেক ওই দেকৌতী জতোসক পোথসরর কোস0 বস থোকসত দেস এ0ো1ো আরও কষয কসর দেক ঠোৎ ওই লি1 পোথরটির োসয় লিোর োো-দেকউ পজো কসরস0 দেকৌতী দেোকটির আঙকো য় যলিসক উ পোথরটিসক লিরসয় দে8স তোর পর ঠোৎ একলি পোথরটির দেোোজ দেই দেক লিসয় দেস0 বো দেকোথোয় দেস0

বদোথ-

দেো োচট ndashচসত লিসয় দেকো লিক0র সE ধোককো দেস পস1 যোবোর উপকরমঅকসমোৎ- ঠোৎআতমমবর- লিসজসক লিয়নতরপ-পো লিপ0স প1োদেসতসসত- লিসপলিউৎোত- দেো1ো দেথসক উপস1 দে8োপরতযয় ndash লিবশবোপরতযোস- টতযোস ndash দৈবোসধসতোধলিসত- পরসপসরর পরলিত ব পরসয়ো করোপরসতরীভত- পোথসর পলিররণলিতবোনতঃকরসরণ- মস পরোসঅলিQৎ- অQো করসত ই=কদেোপ- বধমোর- দেবোস এমইতযোকোর- এইরকমরম- বময়পযসোভী-পসযর জয দেোভ আস0 যোরপোসথয় ঞচয়- পথচোর রচ জমোসোমোমোসরোস- ব ধম ধোসমর সE

তোর লি দেইhelliphellipTo be continued

Hindi 2nd lang सर क पद(सरदास)

सरदास शरी कषण भकति कावय क सरवशरषठ कगिरव ह इनक जनम और मतय क समय तथा सथान का मतभद हसरदास रवातसलय और शरार रस क अनयतम कगिरव ह इनक कावय म बालकषण क सौदय चपपल चषटा और गि7याओ की मनोहर झाकी मिमलती ह कषण और ोगिपयो क अननय परम का कतिचतरण ह सयो शरार की अपकषा उनक कावय म गिरवयो शरार का अमिBक गिरवषय और मारमिमEक कतिचतरण हआ हइन पकतियो म हम सरदास की भकति भारवनाओ का परिरचय मिमलता ह इनका सपण सगरह सरसार म गिनगिहत ह

1 जसोदा हरिर पालन झलारवहलरारवदलराईमलहारव रव जो ईसाई कछ ारव मर लाल को आई निनEदिदया काह ना आगिनसबारव त काह नाही बरवगिह आरवतोको कानहा बलारव

शबदाथ-हलरारव-गिहलती हदलराई - दलार पयार करती हमलहारव-पचकारती हनिनEदरिरया ndashनीदरवगिह-जलदी सअBर-होठमौन-चपसन-सकत

वयाखया- सरदास जी कहत ह गिक यशोदा माता बालक कषण को पालन म झल आती ह रवह उनह गिहलाती ह पयार करती ह मलहार जस कोई ीत ान लती ह और नीद स पछती ह गिक ह नीद तम मर लाल को आकर कयो नही सलाती तझ खाना बला रहा ह कभी कषण आख बद कर लत ह कभी आखफडफडान लत ह उनह सोता हआ जानकर यशोदा माता चप हो जाती ह और इशार म बात करन लती ह इसी बीच अकला कर कषण ज जात हतो गिफर यशोदा माता गिफर स ाना ान लती ह सरदास जी कहत ह गिक भरवान क दशन का सख दरवता और ऋगिष-मगिनयो को भी दलभ ह यही सख माता यशोदा को बडी सहजता स मिमल जा रही ह माता यशोदा बहत ही भागयशाली ह2)Continue to nexthellip

Physics

Chapter 1 Force

(Summary)

Question A body is acted upon by two forces each of magnitude F but in opposite directions State the effect of the forces if

(a) Both forces act at the same point of the body

(b)the two forces act at two different points of the body at a separation r

Solutions

(a) Resultant force acting on the body = 0

F ndash F = 0(b) The forces tend to rotate the body between two forces about the midpoint

Moment of forces = F times rFr

QuestionDefine moment of a couple Write its SI unit

Solutions

Moment of couple is equal to the product of both force and the perpendicular distance between the two forces

The SI unit of moment of couple is NmCommercial Studies

Advertising and sales

Business firms use several methods to

Questions1) What do you mean by advertising

promotion create demand of their product in the market and increase it sales Such methods comprises of advertising sales promotion personal selling and publicityToday we are going to discuss about one of such methods It is advertising

Meaning of advertisingAdvertising is a paid form of non-personal presentation for promotion of Ideas goods and services

Importance or merits of advertising Advertising has importance to manufacturer or traders to customer and to society as a whole

Today we will see how advertisement help the manufacturer or traders

Answer) Advertising is a means of how a company encourages people to buy their products services or ideas It is one element of marketing which also includes design Research and data mining

2) Mention any three features of advertisingAnswer)The main features of advertising are

i) It is impersonal form of presentation for promotion of products and services of Ideas

ii) It is issued by identified sponsor The advertisement contains the name of the advertiser

iii) It is a form of mass communication because the message is directed to a large number of persons simultaneously

3) Mention the main merits or importance of advertisement to manufacturer or tradersAnswer)

i) Introducing new product A business organization can introduce itself and its products to the public through advertising

ii) Increase the sale Advertising leads to increase the sale of existing product by entering into new markets and attracting new customers

iii) Create steady demand Advertising creates sustains regular demand by smoothening out seasonal and other fluctuations It enables regular production for the organisation

iv) Economics of scale Advertising facilitate mass distribution of goods and steady demand which lead to large scale and regular production

v) Goodwill Advertising helps in creating a good image of the firm and reputation for its products

Biology Chapter - 03Genetics

Today wewill start chapter and discuss about Genetics Gregor Mendel is known as father of genetics Before entering into Mendelrsquos experiment on Genetics we must know

Q1 Define the following termsi) Genetics Genetics is the study of

transmissionof body features from parents to offspringand the laws relating to such transmission

ii) Heredity It may be defined as transmissionof genetically based characteristics from parentsto offspring

iii) Character and traits Any heritable

Importance to TraderIntroducing new productIncrease the saleCreate steady demandEconomics of scaleGoodwill

some terms featureis a character The alternative forms of acharacter are called traitsex Character (Hair shape) - Traits (Curly straight)

iv) Homologous chromosomes A pair ofcorresponding chromosomes of the same shapeand size one from each parent

v) Genes Genes are the specific parts (DNA segments) of a chromosome which determinethe hereditary characteristicsNearly 30000genes present in human

vi) Alleles Alternative forms of a gene occupying the same position (locus) on homologouschromosomes and affecting the same characteristicbut in different ways

vii) Genotype ndash PhenotypeGenotype means of genes present in the cells of an organism Phenotype means the observable characteristic which is genetically controlled

viii) Mutation It is a sudden change in one or more genes or in the number or in the structure of chromosomes ex Sickle cell anaemia is a blood disease caused by a gene mutation

CLASS NOTES

Class XSubject Eng Literature (The Merchant of Venice ndash William Shakespeare)Topic Act IV Scene 1 Lines 01 to 34 ( Duke helliphelliphellip We all expect a gentle answer Jew) ate 27th April 2020 (2nd Period)

[Students should read the original play and also the paraphrase given in the school prescribed textbook]Summary Questions amp Answers

This scene may be termed as the catastrophe of the play It is the final unravelling of the complicated events which seem to threaten the happiness of Bassanio Portia and Antonio Right is justified to the fullest degree and malice falls into the trap prepared for others No one suffers here but Shylock but even then he receives a generous measure of mercy

o This is the Court-scene Initially we meet

(1)

DUKE I am sorry for thee thou art come to answer (Line 3-6)A stony adversary an inhuman wretchUncapable of pity void and emptyFrom any dram of mercy

(i) Who is addressed here Where is the person Why is the person there

Antonio is addressed hereAntonio is in the court of justice at VeniceAntoniorsquos trial is scheduled to be held here for his failure to meet the conditions of the bond he signed with Shylock

the Duke Antonio Shylock and Salerio Later we meet Bassanio Portia Gratiano and Nerissa

o The Duke says to Antonio that he has to face a very cruel opponent which Antonio admits and expresses his gratefulness to the Duke for his efforts to soften without result the heart of Shylock in order to be merciful to Antonio Antonio further says that he is ready to accept whatever cruel judgement the Court may award

o When Shylock appears in the court the Duke says that Shylock should change his decision of prosecuting Antonio and demanding the penalty specified in the bond out of consideration of the great misfortunes that Antonio has suffered If this is done by him (Shylock) the whole court would be gladdened by his merciful action

(ii) What is the Duke sorry for

The Duke is unable to change the mind of Shylock from his decision to get the bond forfeited even after he pleaded to Shylock Shylock stands firmly for his bond which when forfeited will allow him to take a pound of flesh from any part of Antoniorsquos body(iii) How does the Duke address Shylock`The Duke calls Shylock an adversary with a heart of stones He calls Shylock as an inhuman wretch without pity Shylock is quite lacking in the slightest quality of mercy (iv) How does Antonio reply to this

Antonio replies that he will meet the revenge of Shylock patiently He has prepared himself to suffer with a quiet spirit the utmost that Shylockrsquos tyranny and rage can do(v) What quality of the Duke is revealed here

The Duke is kind and benevolent He is ready to help Antonio He requests Shylock to free Antonio from the trial(vi) What are the terms of the bond that Antonio has signed

The terms of the bond that Antonio has signed were that if Antonio is unable to repay Shylock a certain sum of money specified on the paper on a certain date and in an agreed place the forfeit has to be paid The forfeiture will be an exact pound of Antoniorsquos flesh which Shylock will be a liberty to take from any part of Antoniorsquos body which pleases him

Class XI

STUDY MATERIAL

Class XISubject Eng Literature (The Tempest ndash William Shakespeare) Topic Act I Scene 2 Lines 88 to 132 (Prospero hellip Me and thy crying self) Date 27th April 2020 (3rd Period)

[Students should read the original play and also the paraphrase given in the school prescribed textbook]Summary Questions amp Answers

o Prospero now tells Miranda that he was the Duke of Milan He had been devoting himself more to studies than the affairs of the State His brother Antonio took advantage of this situation and with the help of Alonso the king of Naples seized upon him and her one midnight and shipped them in a frail bark so that they perished in the sea All this took place

(1)

MIRANDA I should sin (Line 118-132)

To think but nobly of my grandmother

Good wombs have borne bad sonsPROSPERO Now the condition

The King of Naples being an enemyTo me inveterate hearkens my brothers suitWhich was that he in lieu othrsquo premisesOf homage and I know not how much tribute

twelve years back

IMPORTANT PASSAGES EXPLAINED(Line 98-103)

PROSPERO helliphelliphelliphelliphelliphelliphellip

He being thus lorded

Not only with what my revenue yieldedBut what my power might else exact like oneWho having into truth by telling of it Made such a sinner of his memoryTo credit his own lie he did believeHe was indeed the duke

Prospero in telling the narrative of his past life here refers to his brother Antonio Prospero being with a studious bent of mind has left the administration of Milan on his younger brother Now Antonio being thus invested like a lord with all the powers derived from Prosperorsquos wealth and what the exercise of Prosperorsquos authority might secure for him regarded himself as a de facto Duke of Milan It is a well-known fact of psychology that a man who repeatedly tells a lie makes of his memory such a sinner against truth as to credit his own lie by the telling of it So Antonio by repeatedly saying to himself and others that he was the Duke came to believe that he was really the Duke Thus falsehood repeatedly asserted gained the force of truth for Antonio and he truly believed it

Should presently extirpate me and mine Out of the dukedom and confer fair MilanWith all the honours on my brother whereonA treacherous army levied one midnightFated to thrsquo purpose did Antonio openThe gates of Milan and ithrsquo dead of darkness The ministers for thrsquo purpose hurried thenceMe and thy crying self

(i) In the earlier lines of this scene what does Prospero tell about his intense interest What was the demand of his interest

In the earlier lines of this scene Prospero tells Miranda that he had an intense interest in the study of philosophy and magic arts Hence in order to improve his mind with this kind of study he kept himself isolated from worldly and state affairs His study was dearer to him than the applause and esteem that he could win from the public His study demanded too much solitude(ii) What forced Antonio to take an undue advantage over Prospero

Prosperorsquos indifferent attitude towards the statersquos affairs and his having boundless trust in Antonio gave rise to a boundless lust for power in Antoniorsquos mind Antonio felt that he must be the actual Duke instead of the part of the Duke he played Thus Antonio took an undue advantage of the situation to usurp Prosperorsquos dukedom(iii) Explain the following lines ldquoI should sin to think but nobly of my grandmother Good wombs have borne bad sonsrdquo

After hearing the treacherous act of her uncle Antonio Miranda says that Prosperorsquos mother was a noble lady and she cannot dishonour her memory by saying that the person named Antonio cannot be his (Prosperorsquos) brother She says that in honour of her grandmother she also cannot say that Antonio must have been begotten not by her grandfather but by some other man She finally concedes that it is known that good mothers have borne bad sons in their wombs and gave birth to them

(iv) Why did the King of Naples accept Antoniorsquos request to help him in usurping his dukedom What did Antonio propose to Alonso

Alonso who was the king of Naples accepted Antoniorsquos request in usurping Prosperorsquos dukedom because he (Alonso) was a sworn enemy of Prospero Antonio proposed that Alonso should immediately drive him (Prospero) and his offspring out of Milan and should confer the dukedom upon him (Antonio) with all the dignities which go with that

position In return Antonio promised that he shall give an annual tribute and also swore his allegiance to Alonso Also he agreed to hold the Dukedom of Milan as a subordinate to the state of Naples(v) How were Prospero and Miranda carried away from the city of Milan and what was the state of small Miranda at that time

In pursuance of the agreement settled between Antonio and Alonso an army of treacherous men was assembled One midnight when the occasion suited the will of destiny Antonio opened the gates of the city of Milan and in the death like silence of midnight Antoniorsquos agents who had been directed to execute his purpose carried Prospero and small Miranda away from the city in all haste They were then forced into a ship and carried some distance out to sea where they put them on a mere hulk of a boat without any rigging or ship-gear and abandoned them leaving them at the mercy of the roaring sea Miranda was a very small child of three years age and she was crying at that time

CLASS -XIDATE-270420Subject Topic Summary Execution

EVS Chapter 1 ndash Mode of Existence

Impact of mode of existence on resources

Q) Why resources are under pressure

Ans - Increase in the sophistication

of technology enabling natural resources to be extracted quickly and efficiently Eg in the past it could take long hours just to cut down one tree only using saws Due to increased technology rates of deforestation have greatly increased

The number of humans is increasing Cultures of consumerism Materialistic views

lead to the mining of gold and diamonds to produce jewelry unnecessary commodities for human life or advancement Consumerism also leads to extraction of resources for the

production of commodities necessary for human life but in amounts excessive of what is needed because people consume more than is necessary or waste what they have

Lack of awareness among the population is striking People are not aware of ways to reduce depletion and exploitation of materials

Accounts Cash Book Today we are going to start a new topic -Cash Book

The key terms used in this chapter are

bullCash book

bullSimple cash book

bullDouble column cash book bullTriple column cash book

bullPetty cash book

bullCash discount

bullContra entry

Here I will share you the meaning of each key terms

bullCash book Cash Book is a special purpose subsidiary book or journal in which cash received and cash payments are recorded

bullSimple cash book

It is a cash book in which only cash transactions are recorded It has only one column on each side

bullTriple column cash book

It is cash book which has three columns one column for each cash and Bankdiscount on each side of the cash book In this book both cash and Bank transactions are recorded together with discount allowed and received

bullPetty cash book

It is a cash book maintained for recording petty expenses

bullCash discount

Cash discount is the amount of discount received or allowed on cash payments and cash receipts Discount received is an income for the business while discount allowed isan expense

bullContra entry

It means transactions involving both cash and Bank Such transactions though recorded in the cash book are not posted into ledger The letter lsquoC is written in Ledger folio for contra entry

Business Studies

ENTREPRENEURSHIP

Now we shall discuss the second chapter

lsquoENTREPRENEURSHIPrsquo

Today before starting the chapter let us recall what

Questions

1What are the main characteristics of Intrapreneurship

Answer

The main characteristics of Intrapreneurship are

Corporate framework-it occurs within the framework of the same company

Semi-Autonomous-Intrapreneurship

we have read last day

Let s today start the class by recalling the last topic taught

Intrapreneurship is the process of discovering and exploring business opportunities within an existing company It involves launching new business ventures within the framework of a present corporation Intrapreneurship is also known as corporate entrepreneurship or corporate venturing

Now let us start with the characteristics of Intrapreneurship

The main characteristics of Intrapreneurship are

Corporate framework

Semi-Autonomous Lack of ownership Senior position Low risk taking Not own boss

Now let us discuss the meaning of enterprise

Enterprise means an undertaking or adventure that requires some innovation and investment and thus involves riskEnterprise always entails decision making coordination and risk bearing

involves crating amd nurturing a semi-autonomous business unit which may be a subsidiary a strategic business unit or a division

Lack of ownership-the intrapreneur is not the owner of the unitb he creates and nurtures

Senior position-he occupies a senior managerial position in the company

Low risk taking-An intrapreneur does not bear the full risk of failure

Not own boss-An intrapreneur is not his own bosss in legal termsHe enjoys the freedom and gets the required resources and support

2 How is Entrepreneur is different from Intrapreneur

The functions involved in both the entrepreneurship and intrapreneurship are by and large similar however there are several differences between the two

Point of distinction

Entrepreneur

Intrapreneur

status An independent business person

A senior executive within a company

Ownership Owner of

the business

An employeesometimes a share in ownership

Financing Responsible for raising finance for the business

Not responsible for raising the finance

Risk bearing

Bears the risk of the business

Does not bears the risk of the business

Reward Profit which is uncertain and irregularcan be loss

Fixed salary and fringe benefits

Need for security low high

3 What do you understand by enterprise

Answer Enterprise means an undertaking or adventure that requires some innovation and investment and thus involves riskEnterprise always entails decision making coordination and risk bearing

COMMERCE NATURE AND OBJECTIVES OF

BUSINESS

Today let us recall the last other two objectives of business by the chart given in the previous class

Firstly we would discuss Human Objectives

Business is run by people and for people Labour is a valuable business element

Human objectives of business are concerned with the well -being of labour

The human objectives are as follows

Labour welfare Developing human

resources Participative

management Labour

management cooperation

Questions

1 Explain the human objectives of a business enterprise

Answer

Business is run by people and for people Labour is a valuable business element

Human objectives of business are concerned with the well -being of labour

The human objectives are as follows

Labour welfare-Business must recognize the dignity of labour and human factors should be given the recognition

Developing human resources-Employees must be provided the opportunities for developing new skills and attitudes

Participative management-Employees should be allowed to take part in decision making process of business

Labour management cooperation-Business should strive for creating and maintaining cordial employer employee relations so as to ensure peace and progress in industry

Now let us discuss the national objectives of business

Optimum utilization of resources

National self- reliance Development of small

scale industries Development of

backward areas Control over pollution

2Explain the national objectives of a business enterprise

Answer

It is the duty of business to utilize the resources of the country properly the national objectives of business

Optimum utilization of resources ndashBusiness should use the nationrsquos resources in the best possible manner

National self- reliance-It is the duty of the business to help the government in increasing experts and in reducing dependence on imports

Development of small scale industries-Big business firms are expected to encourage growth of small scale industries which are necessary for generating employment

Development of backward areas-Business is expected to give preference to the industrialization of backward regions of the country

ECONOMICS

BASIC ECONOMIC CONCEPTS

SUB

TOPIC

Value

Wealth

Welfare

Today we shall start with a new topic of the same chapter ie lsquoValuersquo

Value of a commodity is defined as the valuation placed by a household on the consumption of this commodity

lsquoValuersquo has two different meanings and these are

a Value -in -use It refers to consumption value of a commodity It expresses the utility derived from the consumption of a particular commodity A necessity like water has a very high value ndashin ndashuse or

Question

1What is value

Answer

Value of a commodity is defined as the valuation placed by a household on the consumption of this commodity

2What is value-in use

Answer It refers to consumption value of a commodity It expresses the utility derived from the consumption of a particular commodity A necessity like water has a very high value ndashin ndashuse or consumption value

3What is value ndashin- exchange

Answer It relates to market value of a commodity

It is the rate at which a particular good or service can be exchanged for money

For example in barter system if a person is prepared to exchange 3 metres of cloth with 1 pair

consumption value

b Value ndashin-exchange It relates to market value of a commodity

It is the rate at which a particular good or service can be exchanged for moneyFor example in barter system if a person is prepared to exchange 3 metres of cloth with 1 pair of shoes then the value in exchange of 3 metres of cloth is 1 pair of shoesValue in exchange is the power of purchasing other goods In modern monetised economies the exchange value of goods are expressed in terms of money as prices

Now let us discuss the term lsquoWealthrsquo

Wealth refers to the stock of all those assets which are a source of income

Wealth is a stock concept

Wealth must possess the following features

a Utility It must possess utility or give some

of shoes then the value in exchange of 3 metres of cloth is 1 pair of shoes

Value in exchange is the power of purchasing other goods In modern monetised economies the exchange value of goods are expressed in terms of money as prices

4 What is wealth

Answer it refers to the stock of assets or goods which are a source of income and have personal or national ownership

5 What are the features of wealth

Answer The features of wealth are as follows

Wealth must possess the following features

a Utility It must possess utility or give some satisfaction

b Scarcity It must be limited in quantityc Transferability it should be transferable its

ownership can be transferred from one person to another person

d Exchange value It must possess exchange value

6 What is welfare

Answer

Welfare is defined as satisfaction and happiness a sense of well- being among the people

satisfactionb Scarcity It must be

limited in quantityc Transferability It

should be transferable its ownership can be transferred from one person to another person

d Exchange value It must possess exchange value

Now let us discuss the term lsquoWelfarersquo

Welfare is defined as satisfaction and happiness a sense of well- being among the people

Welfare is affected by factors like

a Consumption of goods and services

b Environment

c Family relations

d Degree of freedom

e Law and order situation

Mathematics Trigonometric equation

To find the general solution of the equation sinθ=0

When sin θ =0

Then θ= 0 π2π 3π-π -2π -3

i e when θ = 0 or an integral multiple of π

i e when θ= nπ where n is any integer

Therefore the general solution of the equation sin

Example1 Find the general values of θ which satisfy the equation sin2 θ =34

Solution sin2 θ= 34

Or sin θ = +34 or -34

Or sin θ = sin π3 or sin (-π3)

Therefore

θ = [nπ + (-1) n (π3)] or[ nπ+ (-1) n (-π3)]

= nπ +π3 or nπ-π3 where n= any integer

Example 2Find the values of θ which satisfy tan2 θ

θ=0 is θ= nπ where n is any integer

To find the general solution of the equation cos θ=0

When cos θ=0

Then θ=π2 3π2 5π2 -π2 -3π2 -5π3

i e when θ is an odd multiple of π2

i e when θ=(2n+1) π2 where n= any integer

Therefore the general solution of the equation cos θ =0 is θ= (2n+1) π2 where n= any integer

To find the general solution of the equation tan θ = 0

Clearly tan θ =0 implies sin θcos θ =0

Therefore θ = nπ

i e the general solution of the equation tan θ=0 is θ =nπ where n = any integer

To find the general solution of the equation cot θ =0

Clearly cot θ =0 implies (cos θsin θ) = 0

i e cos θ =0

Therefore θ = (2n+1) π2

Therefore the general solution of the equation cot θ =0 is θ = (2n+1) π2

Where n= any integer

To find the general solution of the equation sin θ= k (-

=13 -πleθleπ

Solution tan2 θ =13

Or tan θ = plusmn1radic(3) =tan(plusmnπ6)

θ=nπ plusmn π6 where n =any integer

If n=0 then θ=plusmnπ6

If n=1 then θ= π plusmn π6

If n=-1 then θ= -π plusmn π6

Therefore the required solution in -π le θ le π are θ= π6 5π6 -π6 -5π6

Exercise Find general solution of sin 2θ=cos θ [Hints Use sin 2θ= 2sin θcosθ and then take cosθ

common]

1lek le1)

Determine an angle alpha such that sin =k and -π2le αle π2

Then we have

Sin θ = k = sin α

Or sin θ - sin α =0

Or 2 cos [(θ+α) 2] sin [(θ-α) 2] =0

Therefore either cos [(θ +α) 2] =0 (1)

Or sin [(θ-α) 2] =0 (2)

Now from (1) we get (θ+α) 2= (2m+1) π2)

Or θ = (2m+1) π-α (3)

And from (2) we get (θ-α) 2 =mπ

Or θ= 2mπ+α(4)

Where m = any integer

Clearly the solution (3) amp (4) may be combined in the following form

θ= nπ+(-1) n α where n= any integer

Therefore the general solution of sin θ = sin α is θ = nπ +(-1) n α where n is any integer and -π2 le α le π2

Biology Chapter - 04Kingdom Monera

Today we will discuss about bacterial reproduction and its usefulness

Fig Binary Fission

Fig Conjugation Fig Transformation

Bacterial reproduction is mainly asexual but sexual reproduction

also takes place

Asexual reproduction takes place by i) Binary fission - from one bacteriato

two bacteria are produced in every 20 to 30mins

ii) Buddingiii) Endospore formation - during

unfourable condition

Sexual reproduction by three ways

1) Conjugation - Transfer of genetic material between cells that are in physical contact with one another

2) Transduction - Transfer of genetic materialfrom one cell to another by a bacteriophage

3) Transformation - Transfer of cell-freeor naked DNArsquo from one cell to another

Bacteria causes different diseases inplants animals and human and

it causes food spoilage and waterpollution but it also have some useful

activities

i) Bacteria are helpful in sewage water treatment

ii) It is used in antibiotic (medicine) production

iii) Anaerobic bacteria help in biogas(energy) production

iv) Many household products like yoghurt cheese are manufactured by use of bacteria

v) Rhizobium by symbiotic relationship with leguminous plant increase soil fertility

vi) Besides these bacteria is helpful in genetic engineering degradation of petroleum hydrocarbonand in dairy

industry

Physics Motion in plane Here we will introduce Projectile Motion

Execution

Projectile

Y

usinθ u h

θX

ucosθ

Suppose a body is projected with an angle θ So initial velocity u can be resolved into two components

Horizontal component - ucosθ ( for range)

Vertical component - usinθ ( for height)

usinθ changes during motion and becomes zero at maximum height position but ucosθ remain unchanged

The maximum height of projectile is h

NB If initial is upward then g = -ve and if it is downward then g = +ve Height is +ve if direction of motion does not change ( for ex a body thrown upwards but goes down ultimately then height h = -ve)

The angle of projectile θ is the angle made with horizontal

HISTORY ndash GROWTH OF NATIONALISM

SUB TOPIC- REVOLUTIONARY NATIONALISM Bengal formation of Anushilan Samity and Jugantar Group

The intensification of the Swadeshi movement and Government policy of terror and repression led to outbreak of violence Bombs were manufactured and attempts on the lives of unpopular Government officials became frequent In the gymnasium of Scottish Church College which was known as General Assemblies Institution a secret society was formed known as Anushilan Samity

Aurobindo Ghosh send from Baroda his emissary Jatindranath Banerjee to mobilize the Bengal revolutionaries

Hemchandra Qanungo and Satyen Bose published Journal Jugantar

The Jugantar group planned to assassinate oppressive magistrate Kingsford by Khudiram Bose and Prafulla Chaki in 1908 Prafulla Chaki committed suicide to avoid arrest Khudiram was tried and hanged

Afew days later the police found a bomb factory in Maniktala and arrested a large number of revolutionaries The trial of revolutionaries became famous as the Alipore Bomb Case

In the course of the trial the approver the public prosecuter and a police officer were assassinated

1 Question Name two journals which preached the cult of violence

Answer a) Yugantar edited by Bhupendranath DuttaB) Bandemataram edited by Aurobindo Ghosh2 Question Why was Khudiram arrested and hangedAnswer An attempt was made to assassinate a hated vindictive majistrate named Kingsford by Khudiram Bose and Prafulla Chaki Their attempt failed and the bomb they threw killed two English ladies Khudiram was arrested and put to trial and then hanged3Question Who was Aurubindo GhoshAnswer Aurobindo Ghosh a nationalist revolutionary who was charged for his involvement in the Alipore Bombing Case He was accused of it along with his brother Barindra nath Ghosh But Aurobindo was acquitted because of the brilliant pleading of his counsel Chittaranjan Das Then he became a spiritual reformer introducing his visions on human progress and spiritual evolution4 Qustion Who was KingsfordAnswer Kingsford was an unpopular British chief Magistrate who was the target of the bomb thrown at Muzaffarpur by Khudiram and Prafulla Chaki

Most of the accused were convicted and sentenced to

long term of imprisonmentBut

Aurobindo Ghosh was acquitted mainly owing to the brilliant pleading of his counsel Chittaranjan Das

Political science Topic-Sovereignty

Summary Sovereignty is the full right and power of a governing body over itself without any interference from outside sources or bodies In political theory sovereignty is a substantive term designating supreme legitimate authority over some polity In international law sovereignty is the exercise of power by a state

Internal Sovereignty

Internal sovereignty means supreme authority within ones territory while external sovereignty relates to the recognition on the part of all states that each possesses this power in equal measure

External sovereignty

external sovereignty relates to the recognition on the part of all states that each possesses this power in equal measure

Distinguish between

Execution

Answer the following questions

Short notes-

Sovereignty

Internal Sovereignty

External sovereignty

Homework- learn

external sovereignty and internal sovereigntySovereignty is the principle

of supreme and

unquestionable authority

reflected in the claim by the

state to be the sole author of

laws within its territory

Definition of external vs internal sovereigntyInternal sovereignty refers to

the relationship between a

sovereign power and its

subjects ndash it refers to the

location of the supreme

authority within the state In

the UK for example internal

sovereignty (supposedly)

resides within Parliament

reflected in the

constitutional principle of

parliamentary

sovereigntyBy contrast

external sovereignty refers

to the capacity of the state

to act independently and

autonomously on the world

stage This is what is

sometimes called lsquostate

sovereigntyrsquo or lsquonational

sovereigntyrsquo and implies

that states are legally equal

and that the territorial

integrity and political

independence of a state is

inviolable

Class ndash XII

Date - 2742020 STUDY MATERIALSubject Topic Summary Execution Business Studies

Job Analysis amp Manpower Planning

At first let us recall the chapter what we have discussed till nowbullJob analysisbullJob specification bullJob description bullJob enlargement bullJob enrichment

Today we will do some questions answers from the chapter

Questions 1ldquoJob analysis job description and job specification are interrelatedrdquo Comment Answer) Job analysis is a systematic and detailed examination of a job to collect all the relevant information about it The contents off the job are summarised in the job description The qualification needed for the job are summarised in job specificationThus there is close interrelationship between job analysis job description and job specification

Question 2ldquoJob enlargement is a horizontal extension of a job whereas job enrichment is a vertical extension of a jobrdquo ElucidateAnswer) Job enlargement involves adding one or more task to a job coma where as job enrichment involves adding more autonomy and responsibility to a job Job enlargement is therefore horizontal extension of a job coma whereas job enrichment is a vertical extension of a job

Question 3 )

What is manpower estimation Explain its quantitative and qualitative aspectsAnswer) Manpower estimation is the process by which management determines how an organisation should move from itrsquos current manpower positionto its desired manpower position There are two dimensions of Manpower estimation- quantitative and qualitative

Quantitative aspectThis aspect of Manpower estimation involves estimating the number of employees required in a future time period Workload analysis and workforce analysis are done to estimate the quantity of required manpower

Qualitative aspectThe estimate of the knowledge skills experience etc of required manpower is the qualitative aspect of Manpower estimation The quality of Manpower can be judged on the basis of job analysisand job specification

COMMERCE

CAPITAL-FIXED AND WORKING

Today let us start the class by discussing the sources of finance for different types of business firms

The term lsquocapitalrsquo refers to the investment made in the enterprise for the purpose of earning profits

Requirements of capital and sources of capital for different types of business firms are

1 Capital for sole proprietorship businessA sole proprietor operates at a small scale and thereforerequires a limited amount of capital

2 Capital for partnership firmCapital requirements as well as capital base of a partnership is bigger than that of a sole trader businessThe owned capital is contributed by the partners in an agreed ratio

3 Capital for joint stock companyA joint stock company generally requires large amount of capitalA public company can raise huge capital through issue of shares In addition to share capital it can utilize retained profits

Now let u discuss the meaning of Finance PlanningFinance planning is the process of estimation the financial requirements of an organization specifying the sources of firms and ensuring that enough funds are available at the right time

1 What do you mean by Finance PlanningAnswer Finance planning is the process of estimation the financial requirements of an organization specifying the sources of firms and ensuring that enough funds are available at the right time

2Discuss the role of financial planning of an enterpriseThe role of financial planning are as followsa A sound financial plan helps a business enterprise to avaid the problems of shortage and surplus of fundsbFinancial planning serves as a guide in developing a sound capital structure so as to maximize returns to shareholders c It helps in effective utilization of fundsd It provides policies and procedures for coordinating different functional areas or departments of businesse It enables the management to exercise effective control over the financial activities of an enterprisef It helps the company to prepare for facing business shocks and surprises in future

Mathematics

Continuity and differentiability

Recall Definition of ContinuityLet f(x) be a single valued function of x and x=a be a point in the domain of definition of the function The function is said to be continuous at x=a ifi) f(c) is defined ie f(x) has a definite finite value at x=cii) lim xrarra f(x) exists andiii) lim xrarra f(x) =f(a) In other words f(x) is said to be continuous at x=a if lim xrarra+ f(x)= lim xrarra- f(x) = f(a) Or f(a+0) =f(a-0) =f(a) Or lim hrarr0 f(a+h)= f(a) Algebra of continuous functionsNow we will study some algebra of continuous functions Theorem 1 Suppose f and g be two real functions continuous at a real number c Then(1) f + g is continuous at x = c(2) f ndash g is continuous at x = c(3) f g is continuous at x = c(4) (fg) is continuous at x = c (provided g (c) ne 0)

Example 1 Prove that every rational function is continuousSolution Recall that every rational function f is given byf(x)=[p(x) q(x) ] q(x)ne0where p and q are polynomial functions The domain of f is all real numbers except points at which q is zero Since polynomial functions are continuous f is continuous by (4) of Theorem 1Example 2Discuss the continuity of sine functionSolution To see this we use the following factslim xrarr0 sin x =0Now observe that f (x) = sin x is defined for every real number Let c be a real number Put x = c + h If x rarr c we know that h rarr 0 Therefore lim xrarrc f(x)

= lim xrarrc sin x= lim hrarr0 sin(c+h) =lim hrarr0 [sin c cos h + cos c sin h ]=lim hrarr0 (sin c cos h) + lim hrarr0 (cos c sin h) = sin c +0=sin c = f(c) Thus lim xrarrc f(x) = f(c) and hence f is a constant function Exercise Prove that the function f(x) = x2 +2x is continuous for every real value of x [Hints show that lim xrarra+ f(x) = lim xrarra- f(x) = f(a) ]

Biology Reproduction in Flowering plants We will discuss about megasporoangium

megasporagenesis and female gametophyte

Q4 Describe the structure of megasporangium

Ovule is attached to the placenta by astalk called funicle

Each ovule has one two or three protectivecoverings called integuments

At the tip of integuments a small openingcalled micropyle is organised

Opposite to the micropylar end is the chalaza

Within the integuments a mass of cellsnucellusand inside it embryo sac orfemale gametophyte is present

Q5 Describe a mature embryosacamp its formation

In most of the flowering plants only oneof the 4 megaspores formed as a result ofmegasporogenesis that is functional while theother three degenerate

The

functional megaspore develops into thefemale gametophyte

Formation The nucleus

of the functional megasporedivides mitotically to form two nuclei first andthen two more sequential mitotic nucleardivisions result in the formation of four ampthen eight nucleate stages of embryo sac

Six of the eight nucleus are surrounded bycell walls and organised into cells

The remaining two nuclei called polar nuclei are found below the egg apparatus in the largecentral cell

Three cells consisting of two synergids amp one egg cell present bottom of

embryo sac Three cells

at the chalazal as antipodal cells

Two polar nuclei together present in large central cell

HISTORY

TOWARDS INDEPENDENCE AND PARTITION THE LAST PHASE(1935-1947)SUB TOPIC NATIONAL MOVEMENTS DURING THE SECOND WORLD WAR

Spread of Quit India Movement On 9th August 1942Gandhiji and other Congress leaders were arrested The Congress was declared illegal The news of the arrest of all leaders marked the beginning of a widespread movement of India It was not possible for such a movement to remain peacefulBut the arrest of the all notable congress leaders virtually left the movement in the hands of the mass The movement took the form of violent and militant outbreakBesides congressmen revolutionaries also were very active in the movement The Congress Socialist group also played a prominent role

1 Question Why did the British authority arrest the Congress leaders on 9 th August 1942Answer Congress Working committee adopted the Quit India resolution which was to be ratified at the Bombay AICC meeting in 8th August 1942 They decided to launch a mass struggle on non-violent lines Gandhiji gave a clarion call to all section of the people rdquoKarenge ya Marengerdquo (do or die) Congress leaders gave the call to driving out

the British from IndiaViceroy had taken strong action against the Quit India movement Gandhiji and all the leaders of Congress were arrested

2 Question How did Quit India Movement spread out all over IndiaAnswer The news of the leaders lsquo arrest marked the beginning of a widespread movement to remain peacefulThe movement took form of violent outbreak There were widespread cutting of telephone and Telegraph wires damaging railway lines raising barricades in cities and towns and other forms of violent demonstations

Question Name the leaders of Congress

Socialist group played a prominent part Notable among the Jayprakash Narayan Rammonohar Lohia Aruna Asaf Ali

Political science

Topic-Franchise and Representation

Summary

The election commission

The Election Commission of India is an autonomous constitutional authority responsible for administering Union and State election processes in India The body administers elections to the Lok Sabha Rajya Sabha State Legislative Assemblies in India and the offices of the President and Vice President in the country

Functions of election commission-

India is a sovereign socialist secular democratic republic Democracy runs like a golden thread in the social economic and political fabric woven by the Constitution given by lsquoWe the People of Indiarsquo unto ourselves The concept of democracy as visualised by the Constitution pre-supposes the representation of the people in Parliament and State legislatures by the method of election The Supreme Court has held that democracy is one of the inalienable basic features of the Constitution of India and forms part of its basic structure The Constitution of India adopted a Parliamentary form of government Parliament consists of the President of India and the two Houses mdash Rajya Sabha and Lok Sabha India being a Union of states has separate state legislatures for each state State legislatures consist of the Governor and two Houses mdash Legislative Council and Legislative Assembly mdash in seven states namely Andhra Pradesh Telangana Bihar Jammu amp Kashmir Karnataka Maharashtra and Uttar Pradesh and of the Governor and the state Legislative Assembly in the remaining 22 states Apart from the above two out of the seven Union Territories namely National Capital Territory of Delhi and Puducherry also have their Legislative Assemblies

ExecutionShort notes-Election commissionFunctions of election commission

Homework- Learn

Computer

Science

Computer hardware NAND Gate

A NOT-AND operation is known as NAND operation It has n input (n gt= 2) and one output

Logic diagram

Truth Table

NOR Gate

A NOT-OR operation is known as NOR operation It has n input (n gt= 2) and one output

Logic diagram

Truth Table

XOR Gate

XOR or Ex-OR gate is a special type of gate It can be used in the half

adder full adder and subtractor The exclusive-OR gate is abbreviated as EX-OR gate or sometime as X-OR gate It has n input (n gt= 2) and one output

Logic diagram

Truth Table

XNOR Gate

XNOR gate is a special type of gate It can be used in the half adder full adder and subtractor The exclusive-NOR gate is abbreviated as EX-NOR gate or sometime as X-NOR gate It has n input (n gt= 2) and one output

Logic diagram

Truth Table

Physics

Chapter 1 Electric Field ( Electric Dipole) (Summary)

Here we will derive Expression of electric field at broad side

On position of dipole

Execution

Q With the help of a labelled diagram obtain an expression for the electric field intensity E at any point on the equitorial line ( broad-side on position) of an electric dipole

Ans

E1 E1sinθ

E θ P E1 θ

( r2+L2)12 E2 E

r E2 E2sinθ

-q θ L O L +qA B

Let us consider that the point P is situated on the right bisector of the dipole AB at a distance r meter from its midpoint O

Let E1 and E2 be the electric field intensities of the electric field at P due to charge +q and ndashq of the dipole resp The distance of P from each charge is ( r2+L2)12

So E1 = 14 πϵ q

(r 2+L 2) away from +q

E2 = 14 πϵ q

(r 2+L 2) towards ndashq

The magnitudes of E1 and E2 are equal but directions are different Now resolving E1 and E2 into two components parallel and perpendicular to AB we get

The components perpendicular to AB E1sinθ and E2sinθ cancel each other because they are equal and opposite

The components parallel to AB are E1cosθ and E2 cosθ are in same direction and add up

So resultant intensity of electric field at the point P is

E = E1cosθ + E2 cosθ

E = 14 πϵ q

(r 2+L 2) 2 cosθ

Now from fig we have cosθ =BOBP = L (r2+L2)12

So we get E = 14 πϵ 2qL ( r2+L2)32

Now electric dipole moment p= 2qL

So E = 14 πϵ p ( r2+L2)32

HW Find the expression of Electric field as done here but this time take r gtgt 2L

Also find the expression of torque experience by a dipole

(Hint Electric force experienced by charges of dipole in electric field is qE each Let θ be the angle which dipole makes with electric lines of force then perpendicular distance between two charges is 2Lsinθ Then torque = force x perp distance = qE x 2L sinθ So τ=pE sinθ where p =2qL )

STUDY MATERIAL

Class XIISubject Eng Literature (The Tempest ndash William Shakespeare) Topic Act IV Scene 1 Lines 84 to 133 (Iris hellip A contract of true love Be not too late ) Date 27th April 2020 (4th Period)

[Students should read the original play and also the paraphrase given in the school prescribed textbook]Summary Questions amp Answers

o Ceres soon appears and comes to know that she has been summoned to celebrate the contract of true love

o Ceres expresses her unwillingness to meet Venus and Cupid as she has shunned their company

o Ceres and Juno both bestow their blessings upon Ferdinand and Miranda with June gifting honour riches happiness in marriage and Ceres presents plenty of earthrsquos produce

o Iris summons the water-nymphs and reapers to come and celebrate a contract

(1) IRIS Of her society (Line 91-101)

Be not afraid I met her deity

Cutting the clouds towards Pathos and her sonDove-drawn with her Here thought they to have doneSome wanton charm upon this man and maidWhose vows are that no bed-right shall be paidTill Hymens torch be lightedmdashbut in vainMarss hot minion is returned againHer waspish-headed son has broke his arrowsSwears he will shoot no more but play with sparrowsAnd be a boy right out

(i) Where were Venus and Cupid seen flying How were they travelling Why did they want to join the marriage celebration of Ferdinand and Miranda

of true love

Venus and Cupid were seen flying through the air towards Paphos the famous city which is situated on the island of Cyprus They were travelling by air-borne chariot drawn by doves They certainly wanted to come here in order to play some amorous trick upon Ferdinand and Miranda who are under a vow not to gratify their physical desires till the holy ceremony of their marriage has been performed(ii) What have Venus and Cupid done after failing in their plan

After being failure of their plan Venus who is a very passionate deity and who is the mistress of Mars (the god of war) has gone back while here ill-tempered son Cupid has broken his arrows of love in his state of desperation(iii) What has Cupid firmly decided

Cupid is feeling so disappointed that he has firmly decided to shoot no more arrows to arouse love in human hearts but to spend his time playing with sparrows Thus he would now become just a boy and would give up his original function of shooting arrows on human beings to make them fall in love(iv) What vow had Ceres taken How did Ceres feel at the abduction

After the abduction of her daughter Prosperina by Pluto Ceres had taken a vow to always keep away from the disgraceful company of Venus and her blind son Cupid the god of love Ceres felt deeply distressed when Pluto had carried off her daughter and had made her his wife by force(v) Why has Ceres not forgiven Venus and her blind son For what do Ceres want to be sure

As the abduction had been manipulated by Venus the goddess of beauty and love and her blind son Cupid Ceres has never forgiven them for their part in the whole plot Ceres wants to be sure that she would not have to meet Venus and Cupid who had engineered the abduction of her daughter Prosperina

AS THIS lsquoMASQUErsquo SCENE IS VERY IMPORTANT IN THE PLAY THE PARAPHRASE OF THE ENTIRE PORTION OF MASQUE SCENE (Act IV Lines 58 to 143) IS GIVEN BELOW

IRIS Goddess of RainbowCERES Goddess of Agriculture and all the fruits of the earth

(Nature growth prosperity rebirth ndash notions intimately connected to marriage)JUNO The majestic Queen of Heavens and wife of Jupiter (Jupiter is the king of Gods)

VENUS The Goddess of love CUPID Son of Venus PLUTO God of death (In the play referred by Shakespeare as lsquoDisrsquo which is a Roman name for Pluto)

ORIGINAL TEXT PARAPHRASEPROSPEROWellmdash

PROSPERONow come Ariel Let there be too many rather than too few

Now come my Ariel Bring a corollaryRather than want a spirit Appear and pertly[to Ferdinand and Miranda]No tongue all eyes Be silent

spirits in attendance Appear briskly

[to Ferdinand and Miranda]Look with your eyes but do not say a word

[Soft music] [Soft music][Enter Iris] [Enter Iris]

IRISCeres most bounteous lady thy rich leasOf wheat rye barley vetches oats and peasThy turfy mountains where live nibbling sheepAnd flat meads thatched with stover them to keepThy banks with pioned and twilled brimsWhich spongy April at thy hest betrimsTo make cold nymphs chaste crowns and thybroom-grovesWhose shadow the dismissegraved bachelor lovesBeing lass-lorn thy pole clipped vineyardAnd thy sea-marge sterile and rocky-hardWhere thou thyself dost airmdashthe Queen othrsquoSkyWhose watery arch and messenger am IBids thee leave these and with her sovereign grace[Juno appears] Here on this grass-plot in this very placeTo come and sport Her peacocks fly amainApproach rich Ceres her to entertain

IRISCeres most generous lady you are the cause of rich fields or fertile land where wheat rye barley beans oats and peas grow the grassy mountains where the sheep graze and the flat meadows covered with coarse hay to be used as fodder for cattleYour banks are covered with marsh-marigolds and reeds and the rainy April under your orders brings forth to make for the maids who are not in love beautiful crowns your woods where the broom flourishes and where the bachelor who has been dismissed by the maid he loved lies down being forsaken your vineyard in which the poles are embraced by the vines and the margin of the sea which is barren and rocky where you roam about to enjoy the fresh air ndash the queen of the sky (Juno) whose messenger I am besides being represented as the rainbow bids you leave all these and with her majesty here on this grassy plot in this very place come and sport her peacocks carry her fast in her chariot through the air and are making their way here approach rich Ceres to welcome her

[Enter Ariel as Ceres] [Enter Ariel as Ceres]

CERESHail many-coloured messenger that neerDost disobey the wife of JupiterWho with thy saffron wings upon my flowersDiffusest honey-drops refreshing showersAnd with each end of thy blue bow dost crownMy bosky acres and my unshrubbed downRich scarf to my proud earth Why hath thy queenSummoned me hither to this short-grassed green

CERESWelcome rainbow that never dared disobey Juno the wife of Jupiter who with your orange coloured rays spread honey-drops refreshing showers And with each end of thy blue bow drown my bushy acres and my hilly country which is free from shrubs you thus forming a rich scarf Why has your queen called me here to this place covered with short grass

IRISA contract of true love to celebrateAnd some donation freely to estateOn the blest lovers

IRISI have called you to celebrate a contract of true love and bestow some liberal gift upon the blessed lovers

ORIGINAL TEXT PARAPHRASECERESTell me heavenly bowIf Venus or her son as thou dost knowDo now attend the queen Since they did plotThe means that dusky Dis my daughter gotHer and her blind boys scandaled companyI have forsworn

CERESTell me heavenly bow if Venus the Goddess of love or Cupid her son and pedlar of passion at this time attend the heavenly queen Juno because you are sure to know Since the day they conspired against me and dark Pluto took away my daughter here and Cupidrsquos disgraceful company I have left off

IRISOf her societyBe not afraid I met her deityCutting the clouds towards Pathos and her sonDove-drawn with her Here thought they to have doneSome wanton charm upon this man and miad

IRISBe not afraid of her company I met her deity moving on the clouds towards Paphos the sacred home of Venus on the island of Cyprus along with her son on her chariot drawn by doves Here they contemplated to exercise a charm upon this man and maid producing

Whose vows are that no bed-right shall be paidTill Hymens torch be lightedmdashbut in vainMarss hot minion is returned againHer waspish-headed son has broke his arrowsSwears he will shoot no more but play with sparrowsAnd be a boy right out

wantonness before the actual marriage ceremony but did not succeed Venus has returned her irritable son has broken his arrows and swears that he will give up his practice of trying to inspire love but play with sparrows and be a boy again

[Music is heard] [Music is heard]

CERESHighst queen of stateGreat Juno comes I know her by her gait

CERESHighest queen of state Great Juno there she comes I know here by her gait

[Enter Juno] [Enter Juno]

JUNOHow does my bounteous sister Go with meTo bless this twain that they may Prosperous beAnd honoured in their issue

JUNOHow are you doing my generous sister Come with me to bless this couple so that they may be prosperous and fortunate in their children

[They sing] [They sing]

JUNOHonour riches marriage-blessingLong continuance and increasingHourly joys be still upon youJuno sings her blessings upon you

JUNOMay honour riches happiness in marriage long continuance and increase of those boons ever rest upon you as hourly joys Juno showers down upon you her blessings in song

CERESEarths increase foison plentyBarns and garners never emptyVines and clustring bunches growingPlants and goodly burden bowingSpring come to you at the farthestIn the very end of harvestScarcity and want shall shun youCeresrsquo blessing so is on you

CERESMay you have the plenty of earthrsquos produce Your barns and granaries may never be empty Your vines may grow with clustering bunches Your fruit trees may be heavily laden with their fruit May there be continuous spring and harvest May scantiness and want leave you forever Such is the blessing of Ceres upon you

FERDINANDThis is a most majestic vision andHarmoniously charmingly May I be boldTo think these spirits

FERDINANDThis is a great vision and magically melodious Should I suppose the characters (taking part in the masque) are spirits

PROSPEROSpirits which by mine artI have from their confines calld to enactMy present fancies

PROSPEROYes they are spirits whom I have summoned from the regions to which they are confined to carry into effect my fanciful designs

ORIGINAL TEXT PARAPHRASEFERDINANDLet me live here everSo rare a wondered father and a wifeMakes this place paradise

FERDINANDI should like to live here forever Such a wise and wonderful father makes this place a paradise

[Juno and Ceres whisper and send Iris on employment] [Juno and Ceres whisper and send Iris on employment]

PROSPEROSweet now silence

PROSPEROMy dear Ferdinand speak no more Juno and Ceres are

Juno and Ceres whisper seriouslyTheres something else to do Hush and be muteOr else our spell is marred

whispering with a solemn look There is something else coming Silence Or else our magic will be spoilt

IRISYour nymphs called naiads of the wandering brooksWith your sedged crowns and over-harmless looksLeave your crisp channels and on this green landAnswer your summons Juno does commandCome temperate nymphs and help to celebrateA contract of true love Be not too late

IRISYou nymphs called Naiads denizens (M inhabitants) of the running stream with your chaplets of sedge and ever-helpful looks leave your wrinkled channels and on the green land answer the summons sent to you Juno has ordered some chaste nymphs and help to celebrate a noble and true marriage Donrsquot delay

[Enter certain nymphs] [Enter certain nymphs]You sunburnt sicklemen of August wearyCome hither from the furrow and be merryMake holiday your rye-straw hats put onAnd these fresh nymphs encounter every oneIn country footing

You sunburnt harvesters weary from the effects of the heat in August come here from the furrowed land and rejoice Make holiday with your rye-straw hats upon you and meet these fresh nymphs and join in country dancing

[Enter certain reapers properly habited They join with the nymphs in a graceful dance towards the end whereof Prospero starts suddenly and speaks]

[Enter certain reapers properly habited They join with the nymphs in a graceful dance towards the end whereof Prospero starts suddenly and speaks]

PROSPERO[aside] I had forgot that foul conspiracyOf the beast Caliban and his confederatesAgainst my life The minute of their plotIs almost come [to the spirits]Well done Avoidno more

PROSPERO(Aside)I had forgotten the wicked conspiracy of the beast Caliban and his accomplices against my life the time of their plot has almost arrived ndash (To the Spirits) well done depart no more of this

[To a strange hollow and confused noise the spirits heavily vanish]

[The spirits depart]

ORIGINAL TEXT PARAPHRASEFERDINANDLet me live here everSo rare a wondered father and a wifeMakes this place paradise

FERDINANDI should like to live here forever Such a wise and wonderful father makes this place a paradise

[Juno and Ceres whisper and send Iris on employment] [Juno and Ceres whisper and send Iris on employment]

PROSPEROSweet now silenceJuno and Ceres whisper seriouslyTheres something else to do Hush and be muteOr else our spell is marred

PROSPEROMy dear Ferdinand speak no more Juno and Ceres are whispering with a solemn look There is something else coming Silence Or else our magic will be spoilt

IRISYour nymphs called naiads of the wandering brooksWith your sedged crowns and over-harmless looksLeave your crisp channels and on this green landAnswer your summons Juno does commandCome temperate nymphs and help to celebrateA contract of true love Be not too late

IRISYou nymphs called Naiads denizens (M inhabitants) of the running stream with your chaplets of sedge and ever-helpful looks leave your wrinkled channels and on the green land answer the summons sent to you Juno has ordered some chaste nymphs and help to celebrate a noble and true marriage Donrsquot delay

[Enter certain nymphs] [Enter certain nymphs]You sunburnt sicklemen of August wearyCome hither from the furrow and be merryMake holiday your rye-straw hats put onAnd these fresh nymphs encounter every oneIn country footing

You sunburnt harvesters weary from the effects of the heat in August come here from the furrowed land and rejoice Make holiday with your rye-straw hats upon you and meet these fresh nymphs and join in country dancing

[Enter certain reapers properly habited They join with the nymphs in a graceful dance towards the end whereof Prospero starts suddenly and speaks]

[Enter certain reapers properly habited They join with the nymphs in a graceful dance towards the end whereof Prospero starts suddenly and speaks]

PROSPERO[aside] I had forgot that foul conspiracyOf the beast Caliban and his confederatesAgainst my life The minute of their plotIs almost come [to the spirits]Well done Avoidno more

PROSPERO(Aside)I had forgotten the wicked conspiracy of the beast Caliban and his accomplices against my life the time of their plot has almost arrived ndash (To the Spirits) well done depart no more of this

[To a strange hollow and confused noise the spirits heavily vanish]

[The spirits depart]

Ac-12 27420 topic Revaluation of Assets and Liabilities

REVALUATION OF ASSETS AND LIABILITIES

On admission of a new partner the firm stands reconstituted and consequently the assets are revalued and liabilities are reassessed It is necessary to show the true position of the firm at the time of admission of a new partner If the values of the assets are raised gain will increase the capital of the existing partners Similarly any decrease in the value of assets ie loss will decrease the capital of the existing partners For this purpose alsquoRevaluation Accountrsquo is prepared This account is credited with all increases in the value of assets and decrease in the value of liabilities It is debited with decrease on account of value of assets and increase in the value of liabilities The balance of this account shows a gain or loss on revaluation which is transferred to the existing partnerrsquos capital account in existing profit sharing ratioAccounting for Revaluation of Assets and Liabilities when there is a Changein the Profit Sharing Ratio of Existing PartnersAssets and liabilities of a firm must also be revalued at the time of change in profit sharing ratio of existing partners The reason is that the realisable or actual value of assets and liabilities may be different from those shown in the Balance Sheet It is possible that with the passage of time some of the assets might have appreciated in value while the value of certain other assets might have decreased and no record has been made of such changes in the books of accounts Similarly there may be some unrecorded assets amp libilities that may have to be accounted for Revaluation of assets and reassessments of liabilities becomes necessary because the change in the

value of assets and liabilities belongs to the period to change in profit sharing ratio and hence must be shared by the partners in their old profit sharing ratio Revaluation of assets and reassessment of liabilities may be given effect to in two different ways (a) When revised values are to be recorded in the books and(b) When revised values are not to be recorded in the books

When revised values are to be recorded in the booksIn such a case revaluation of assets and reassessment of liabilities is done with the help of a new account called lsquoRevaluation Accountrsquo Sometimes this account is also called as lsquoProfit amp Loss Adjustment Acrsquo If there is a loss due to revaluation revaluation account is debited and if the revaluation results in a profit the revaluation account is credited The following journal entries made for this purpose are

(i) For increase in the value of assetsAsset Ac Dr (individually)To Revaluation Ac(ii) For decrease in the value of AssetRevaluation Ac Dr (individually)To Asset Ac[Decrease in the value of assets](iii) For increase in the value of LiabilitiesRevaluation Ac Dr (individually)To Liabilities Ac[Increase in the value of Liabilities](iv) For decrease in the value of LiabilitiesLiabilities Ac DrTo Revaluation Ac[Decrease in the value of Liabilities](v) For unrecorded AssetsAsset Ac [unrecorded] DrTo Revaluation Ac[Unrecorded asset recorded at actual value](vi) For unrecorded Liability Revaluation Ac DrTo Liability Ac [unrecorded][Unrecorded Liability recorded at actual value](vii) For transfer of gain on revaluationRevaluation Ac DrTo Existing Partnerrsquos CapitalCurrent Ac[Profit on revaluation transferred to capital account in existing ratio](viii) For transfer of loss on revaluationExisting Partnerrsquos CapitalCurrent Ac DrTo Revaluation Ac[Loss on revaluation transferred to capital account in existing ratio](a) When revaluation account shows gain Revaluation Ac DrTo Partnerrsquos Capital Ac (Old Profit Sharing Ratio)(Profit on revaluation credited to Partnerrsquos Capital Ac)(b) Above entry is reversed when revaluation account shows loss Partners Capital Acs (Old Profit Sharing Ratio) DrTo Revaluation Ac(Loss on revaluation debited to Partnerrsquos Capital Acs)

Proforma of Revaluation Account is given as under

Revaluation Account

Dr Cr Particulars ` Amount Particulars ` Amount To Decrease in value of assets By Increase in value of assets To Increase in value of liabilities By Decrease in value of liabilities To Unrecorded liabilities By Unrecorded assets To Gain on Revaluation (Transferred) By Loss on Revalution (Transferred)

ECO ndash12 2742020Topic- ELASTICITY OF DEMAND

CHAPTER - ELASTICITY OF DEMANDMEANINGDemand for a commodity is affected by many factors such as its price price of related goods income of its buyer tastes and preferences etc Elasticity means degree of response Elasticity of demand means degree of responsiveness of demand Demand for a commodity responds to change in price price of related goods income etc So we have three dimensions of elasticity of demandDIMENSION OF ELASTICITY OF DEMAND TYPES OF ELASTICITY OF DEMAND

Price elasticity of demand Income elasticity of demand Cross Elasticity of demand

Price elasticity of demand Price elasticity of demand means degree of responsiveness of demand for a commodity to the change in its price For example if demand for a commodity rises by 10 due to 5 fall in its price Price elasticity of demand (ep)=Percentage change in quantity demanded Percentage change in price of the commodity = 10 ( -)5 = ( - )2Note that ep will always be negative due to inverse relationship of price and quantity demanded

(ii) Income elasticity of demand Income elasticity of demand refers to the degree of responsiveness of demand for a commodity to the change in income of its buyer Suppose income of buyer rises by 10 and his demand for a commodity rises by 20 then Income elasticity of demand (ey)= change in quantity demanded change in price of the commodity =20 10 = 2

Cross Elasticity of demandCross elasticity of demand means the degree of responsiveness of demand for a commodity to the change in price of its related goods (substitute goods or complementary goods) Suppose demand for a commodity rises by 10 due to 5 rise in price of its substitute good then Cross elasticity of demand (ec) = change in quantity demanded change in price of related good = 10 2 = 5 (Tastes and preferences cannot be expressed numerically So elasticity ofdemand cannot be numerically expressed)

  • Chapter 1 Force (Summary)
  • Distinguish between external sovereignty and internal sovereignty
    • NAND Gate
      • Logic diagram
      • Truth Table
        • NOR Gate
          • Logic diagram
          • Truth Table
            • XOR Gate
              • Logic diagram
              • Truth Table
                • XNOR Gate
                  • Logic diagram
                  • Truth Table
                      • Physics
                      • Chapter 1 Electric Field ( Electric Dipole) (Summary)
Page 17:  · Web viewWe all know that Nouns are divided into two parts: common noun and proper noun.Apart from common and proper noun, we will also study about collective noun and compound

of productionbSince the product is produced by an expert workerbest quality of product is producedcIt saves time and toolsd it promotes inventions in the methods and techniques of productioneIt leads to reduction in costs fAll workers get work according to their abilities and choices

Now let us discuss the disadvantages of Division of labour

a Since many workers are involved in the production of a commodity no one has the sense of responsibility

b The constant and repetition of the same work again and again make the work monotonous

c Division of labour facilitates production on large scale Hencethere is fear of over production

d Because of territorial division of labour some areasregions become more developed than others

It is based on labour-intensive techniques of production

It is based on capital-intensive techniques

It is generally found in small enterprises

It is generally found in large enterprises

Example Indian farmers doing all farm activities

Example A modern garmet factory where one person takes the measurementanother does the cuttingsome sew the clothes while a few workers button them and other iron them

2 What are the advantages of Division of labouraIt increases the level of productionbSince the product is produced by an expert workerbest quality of product is producedcIt saves time and toolsd it promotes inventions in the methods and techniques of productioneIt leads to reduction in costs fAll workers get work according to their abilities and choices

3Discuss the disadvantages of Division of laboura Since many workers are involved in the

production of a commodity no one has the sense of responsibility

b The constant and repetition of the same work again and again make the work monotonous

c Division of labour facilitates production on large scale Hence there is fear of over production

d Because of territorial division of labour some areasregions become more developed than others

English 1 Transformation of sentences

Sentences A sentence is a group of words which makes complete sense

a Assertive sentences

Exercise 6Rewrite the following sentences according to the instructions given below without changing their meanings

1 As soon as he saw the beer he jumped into

b Imperative sentences

c Interrogative sentences

d Exclamatory sentences

Sentences can be changed from one grammatical form to another without changing the meaning of the sentence This is known as transformation of sentences

the river ( Begin No sooner)2 None but brave deserve the fair (Begin the

bravehellip)3 This box is too heavy for me to lift ( Use so hellip

That instead of too)4 No one other than a king can live like James

Luxurious ( Begin only James)5 Oh for the wings of a dove (Begin I wishhellip)

Math Topic Commercial MathematicsChapter Shares and Dividends

Study item Discuss about shares and Dividends1) What is share

Ans To start any big business (company or Industry) a large sum of money is needed But it is not possible for an individual to invest such a large amount Then some persons interested in the business join together and from a company They divide the estimated money required into small parts Each such part is called a share

2) What do you mean by the term shareholder

Ans A person who purchases one or more shares is called shareholder3) Some terms related with a share

(i) Nominal value or face value or printed value The original value of a share is called its nominal value or face value or printed value

Note The nominal value of a share always remains same(ii) Market value or cash value The price of a share at any

time is called its market value or cash value

Note The market value of a share changes from time to time(iii) At par If the market value of a share is the same as its

nominal value the share is called at par(iv) At Premium or above Par If the market value of a share

is more than its nominal value the share is called at premium or above par

Example If a share of Rs 100 is selling at Rs 150 then it is said to be selling at a premium of Rs 50 or Rs 50 above par

(v) At Discount or below par If the market value of a share is less than its nominal value the share is called at discount or below par

Example If a share of Rs 100 is selling at Rs80 then it is said to be selling at a discount of Rs 20 or at Rs 20 below par

4) What is Dividend

Ans The profit which a shareholder gets for hisher investment from the company is called dividendNote (i) The dividend is always expressed as the percentage of the face value of the share(ii) The dividend is always given( by the company ) on the face value of the share

irrespective of the market value of the shareBENGALI(2ND LANGUAGE)

ldquoদেবতোর জণমrdquoলিবরোম চকরবত

পরথম লিসর পোঠ-চোর পসথ একটি পোথর লিবপলি ঘটোয় দেক যোতোয়োসতর পসথ পরলিতলিয়ত ওই পোথসর দেো োচট দেসত একলি দেতো দেক দেো োচট দেসয় দেবোমো সয় রোসতোর মোস লি0টসক পস1 লিবপরীত লিক দেথসক আো একটি দেমোটর োলি1 চোসকর কষতোয় পরোসরণ দেবোসচ যো লিকনত পরলিতবোর এমরণ দেৌভোয দেসকর োও সত পোসর তোই লিতলি দেকোো দেজোো1 কসর পোথরটিসক উপস1 দে8স পোথর উপস1 দে8োর ময় এক দেকৌতী জতো দেকসক পরশন কসর দেয লিতলি দেকোসো দেবতোর আস দেপসয়স0রণ লিকো লিকনত দেক বস লিতলি দেকোসো দেবতোর আস পোলি দেক উপলিত ক জতোর উসltসয বস কোরও ইস= স পোথরটি লিসয় দেযসত পোসর এর পর দেথসক দেক দেযসত আসত পোথরটি দেক দেসত পো একলি দেক কষয করস দেকউ পোথরটিসক ধসয়মস0 পলিরসকোর করস0 দেক ওই দেকৌতী জতোসক পোথসরর কোস0 বস থোকসত দেস এ0ো1ো আরও কষয কসর দেক ঠোৎ ওই লি1 পোথরটির োসয় লিোর োো-দেকউ পজো কসরস0 দেকৌতী দেোকটির আঙকো য় যলিসক উ পোথরটিসক লিরসয় দে8স তোর পর ঠোৎ একলি পোথরটির দেোোজ দেই দেক লিসয় দেস0 বো দেকোথোয় দেস0

বদোথ-

দেো োচট ndashচসত লিসয় দেকো লিক0র সE ধোককো দেস পস1 যোবোর উপকরমঅকসমোৎ- ঠোৎআতমমবর- লিসজসক লিয়নতরপ-পো লিপ0স প1োদেসতসসত- লিসপলিউৎোত- দেো1ো দেথসক উপস1 দে8োপরতযয় ndash লিবশবোপরতযোস- টতযোস ndash দৈবোসধসতোধলিসত- পরসপসরর পরলিত ব পরসয়ো করোপরসতরীভত- পোথসর পলিররণলিতবোনতঃকরসরণ- মস পরোসঅলিQৎ- অQো করসত ই=কদেোপ- বধমোর- দেবোস এমইতযোকোর- এইরকমরম- বময়পযসোভী-পসযর জয দেোভ আস0 যোরপোসথয় ঞচয়- পথচোর রচ জমোসোমোমোসরোস- ব ধম ধোসমর সE

তোর লি দেইhelliphellipTo be continued

Hindi 2nd lang सर क पद(सरदास)

सरदास शरी कषण भकति कावय क सरवशरषठ कगिरव ह इनक जनम और मतय क समय तथा सथान का मतभद हसरदास रवातसलय और शरार रस क अनयतम कगिरव ह इनक कावय म बालकषण क सौदय चपपल चषटा और गि7याओ की मनोहर झाकी मिमलती ह कषण और ोगिपयो क अननय परम का कतिचतरण ह सयो शरार की अपकषा उनक कावय म गिरवयो शरार का अमिBक गिरवषय और मारमिमEक कतिचतरण हआ हइन पकतियो म हम सरदास की भकति भारवनाओ का परिरचय मिमलता ह इनका सपण सगरह सरसार म गिनगिहत ह

1 जसोदा हरिर पालन झलारवहलरारवदलराईमलहारव रव जो ईसाई कछ ारव मर लाल को आई निनEदिदया काह ना आगिनसबारव त काह नाही बरवगिह आरवतोको कानहा बलारव

शबदाथ-हलरारव-गिहलती हदलराई - दलार पयार करती हमलहारव-पचकारती हनिनEदरिरया ndashनीदरवगिह-जलदी सअBर-होठमौन-चपसन-सकत

वयाखया- सरदास जी कहत ह गिक यशोदा माता बालक कषण को पालन म झल आती ह रवह उनह गिहलाती ह पयार करती ह मलहार जस कोई ीत ान लती ह और नीद स पछती ह गिक ह नीद तम मर लाल को आकर कयो नही सलाती तझ खाना बला रहा ह कभी कषण आख बद कर लत ह कभी आखफडफडान लत ह उनह सोता हआ जानकर यशोदा माता चप हो जाती ह और इशार म बात करन लती ह इसी बीच अकला कर कषण ज जात हतो गिफर यशोदा माता गिफर स ाना ान लती ह सरदास जी कहत ह गिक भरवान क दशन का सख दरवता और ऋगिष-मगिनयो को भी दलभ ह यही सख माता यशोदा को बडी सहजता स मिमल जा रही ह माता यशोदा बहत ही भागयशाली ह2)Continue to nexthellip

Physics

Chapter 1 Force

(Summary)

Question A body is acted upon by two forces each of magnitude F but in opposite directions State the effect of the forces if

(a) Both forces act at the same point of the body

(b)the two forces act at two different points of the body at a separation r

Solutions

(a) Resultant force acting on the body = 0

F ndash F = 0(b) The forces tend to rotate the body between two forces about the midpoint

Moment of forces = F times rFr

QuestionDefine moment of a couple Write its SI unit

Solutions

Moment of couple is equal to the product of both force and the perpendicular distance between the two forces

The SI unit of moment of couple is NmCommercial Studies

Advertising and sales

Business firms use several methods to

Questions1) What do you mean by advertising

promotion create demand of their product in the market and increase it sales Such methods comprises of advertising sales promotion personal selling and publicityToday we are going to discuss about one of such methods It is advertising

Meaning of advertisingAdvertising is a paid form of non-personal presentation for promotion of Ideas goods and services

Importance or merits of advertising Advertising has importance to manufacturer or traders to customer and to society as a whole

Today we will see how advertisement help the manufacturer or traders

Answer) Advertising is a means of how a company encourages people to buy their products services or ideas It is one element of marketing which also includes design Research and data mining

2) Mention any three features of advertisingAnswer)The main features of advertising are

i) It is impersonal form of presentation for promotion of products and services of Ideas

ii) It is issued by identified sponsor The advertisement contains the name of the advertiser

iii) It is a form of mass communication because the message is directed to a large number of persons simultaneously

3) Mention the main merits or importance of advertisement to manufacturer or tradersAnswer)

i) Introducing new product A business organization can introduce itself and its products to the public through advertising

ii) Increase the sale Advertising leads to increase the sale of existing product by entering into new markets and attracting new customers

iii) Create steady demand Advertising creates sustains regular demand by smoothening out seasonal and other fluctuations It enables regular production for the organisation

iv) Economics of scale Advertising facilitate mass distribution of goods and steady demand which lead to large scale and regular production

v) Goodwill Advertising helps in creating a good image of the firm and reputation for its products

Biology Chapter - 03Genetics

Today wewill start chapter and discuss about Genetics Gregor Mendel is known as father of genetics Before entering into Mendelrsquos experiment on Genetics we must know

Q1 Define the following termsi) Genetics Genetics is the study of

transmissionof body features from parents to offspringand the laws relating to such transmission

ii) Heredity It may be defined as transmissionof genetically based characteristics from parentsto offspring

iii) Character and traits Any heritable

Importance to TraderIntroducing new productIncrease the saleCreate steady demandEconomics of scaleGoodwill

some terms featureis a character The alternative forms of acharacter are called traitsex Character (Hair shape) - Traits (Curly straight)

iv) Homologous chromosomes A pair ofcorresponding chromosomes of the same shapeand size one from each parent

v) Genes Genes are the specific parts (DNA segments) of a chromosome which determinethe hereditary characteristicsNearly 30000genes present in human

vi) Alleles Alternative forms of a gene occupying the same position (locus) on homologouschromosomes and affecting the same characteristicbut in different ways

vii) Genotype ndash PhenotypeGenotype means of genes present in the cells of an organism Phenotype means the observable characteristic which is genetically controlled

viii) Mutation It is a sudden change in one or more genes or in the number or in the structure of chromosomes ex Sickle cell anaemia is a blood disease caused by a gene mutation

CLASS NOTES

Class XSubject Eng Literature (The Merchant of Venice ndash William Shakespeare)Topic Act IV Scene 1 Lines 01 to 34 ( Duke helliphelliphellip We all expect a gentle answer Jew) ate 27th April 2020 (2nd Period)

[Students should read the original play and also the paraphrase given in the school prescribed textbook]Summary Questions amp Answers

This scene may be termed as the catastrophe of the play It is the final unravelling of the complicated events which seem to threaten the happiness of Bassanio Portia and Antonio Right is justified to the fullest degree and malice falls into the trap prepared for others No one suffers here but Shylock but even then he receives a generous measure of mercy

o This is the Court-scene Initially we meet

(1)

DUKE I am sorry for thee thou art come to answer (Line 3-6)A stony adversary an inhuman wretchUncapable of pity void and emptyFrom any dram of mercy

(i) Who is addressed here Where is the person Why is the person there

Antonio is addressed hereAntonio is in the court of justice at VeniceAntoniorsquos trial is scheduled to be held here for his failure to meet the conditions of the bond he signed with Shylock

the Duke Antonio Shylock and Salerio Later we meet Bassanio Portia Gratiano and Nerissa

o The Duke says to Antonio that he has to face a very cruel opponent which Antonio admits and expresses his gratefulness to the Duke for his efforts to soften without result the heart of Shylock in order to be merciful to Antonio Antonio further says that he is ready to accept whatever cruel judgement the Court may award

o When Shylock appears in the court the Duke says that Shylock should change his decision of prosecuting Antonio and demanding the penalty specified in the bond out of consideration of the great misfortunes that Antonio has suffered If this is done by him (Shylock) the whole court would be gladdened by his merciful action

(ii) What is the Duke sorry for

The Duke is unable to change the mind of Shylock from his decision to get the bond forfeited even after he pleaded to Shylock Shylock stands firmly for his bond which when forfeited will allow him to take a pound of flesh from any part of Antoniorsquos body(iii) How does the Duke address Shylock`The Duke calls Shylock an adversary with a heart of stones He calls Shylock as an inhuman wretch without pity Shylock is quite lacking in the slightest quality of mercy (iv) How does Antonio reply to this

Antonio replies that he will meet the revenge of Shylock patiently He has prepared himself to suffer with a quiet spirit the utmost that Shylockrsquos tyranny and rage can do(v) What quality of the Duke is revealed here

The Duke is kind and benevolent He is ready to help Antonio He requests Shylock to free Antonio from the trial(vi) What are the terms of the bond that Antonio has signed

The terms of the bond that Antonio has signed were that if Antonio is unable to repay Shylock a certain sum of money specified on the paper on a certain date and in an agreed place the forfeit has to be paid The forfeiture will be an exact pound of Antoniorsquos flesh which Shylock will be a liberty to take from any part of Antoniorsquos body which pleases him

Class XI

STUDY MATERIAL

Class XISubject Eng Literature (The Tempest ndash William Shakespeare) Topic Act I Scene 2 Lines 88 to 132 (Prospero hellip Me and thy crying self) Date 27th April 2020 (3rd Period)

[Students should read the original play and also the paraphrase given in the school prescribed textbook]Summary Questions amp Answers

o Prospero now tells Miranda that he was the Duke of Milan He had been devoting himself more to studies than the affairs of the State His brother Antonio took advantage of this situation and with the help of Alonso the king of Naples seized upon him and her one midnight and shipped them in a frail bark so that they perished in the sea All this took place

(1)

MIRANDA I should sin (Line 118-132)

To think but nobly of my grandmother

Good wombs have borne bad sonsPROSPERO Now the condition

The King of Naples being an enemyTo me inveterate hearkens my brothers suitWhich was that he in lieu othrsquo premisesOf homage and I know not how much tribute

twelve years back

IMPORTANT PASSAGES EXPLAINED(Line 98-103)

PROSPERO helliphelliphelliphelliphelliphelliphellip

He being thus lorded

Not only with what my revenue yieldedBut what my power might else exact like oneWho having into truth by telling of it Made such a sinner of his memoryTo credit his own lie he did believeHe was indeed the duke

Prospero in telling the narrative of his past life here refers to his brother Antonio Prospero being with a studious bent of mind has left the administration of Milan on his younger brother Now Antonio being thus invested like a lord with all the powers derived from Prosperorsquos wealth and what the exercise of Prosperorsquos authority might secure for him regarded himself as a de facto Duke of Milan It is a well-known fact of psychology that a man who repeatedly tells a lie makes of his memory such a sinner against truth as to credit his own lie by the telling of it So Antonio by repeatedly saying to himself and others that he was the Duke came to believe that he was really the Duke Thus falsehood repeatedly asserted gained the force of truth for Antonio and he truly believed it

Should presently extirpate me and mine Out of the dukedom and confer fair MilanWith all the honours on my brother whereonA treacherous army levied one midnightFated to thrsquo purpose did Antonio openThe gates of Milan and ithrsquo dead of darkness The ministers for thrsquo purpose hurried thenceMe and thy crying self

(i) In the earlier lines of this scene what does Prospero tell about his intense interest What was the demand of his interest

In the earlier lines of this scene Prospero tells Miranda that he had an intense interest in the study of philosophy and magic arts Hence in order to improve his mind with this kind of study he kept himself isolated from worldly and state affairs His study was dearer to him than the applause and esteem that he could win from the public His study demanded too much solitude(ii) What forced Antonio to take an undue advantage over Prospero

Prosperorsquos indifferent attitude towards the statersquos affairs and his having boundless trust in Antonio gave rise to a boundless lust for power in Antoniorsquos mind Antonio felt that he must be the actual Duke instead of the part of the Duke he played Thus Antonio took an undue advantage of the situation to usurp Prosperorsquos dukedom(iii) Explain the following lines ldquoI should sin to think but nobly of my grandmother Good wombs have borne bad sonsrdquo

After hearing the treacherous act of her uncle Antonio Miranda says that Prosperorsquos mother was a noble lady and she cannot dishonour her memory by saying that the person named Antonio cannot be his (Prosperorsquos) brother She says that in honour of her grandmother she also cannot say that Antonio must have been begotten not by her grandfather but by some other man She finally concedes that it is known that good mothers have borne bad sons in their wombs and gave birth to them

(iv) Why did the King of Naples accept Antoniorsquos request to help him in usurping his dukedom What did Antonio propose to Alonso

Alonso who was the king of Naples accepted Antoniorsquos request in usurping Prosperorsquos dukedom because he (Alonso) was a sworn enemy of Prospero Antonio proposed that Alonso should immediately drive him (Prospero) and his offspring out of Milan and should confer the dukedom upon him (Antonio) with all the dignities which go with that

position In return Antonio promised that he shall give an annual tribute and also swore his allegiance to Alonso Also he agreed to hold the Dukedom of Milan as a subordinate to the state of Naples(v) How were Prospero and Miranda carried away from the city of Milan and what was the state of small Miranda at that time

In pursuance of the agreement settled between Antonio and Alonso an army of treacherous men was assembled One midnight when the occasion suited the will of destiny Antonio opened the gates of the city of Milan and in the death like silence of midnight Antoniorsquos agents who had been directed to execute his purpose carried Prospero and small Miranda away from the city in all haste They were then forced into a ship and carried some distance out to sea where they put them on a mere hulk of a boat without any rigging or ship-gear and abandoned them leaving them at the mercy of the roaring sea Miranda was a very small child of three years age and she was crying at that time

CLASS -XIDATE-270420Subject Topic Summary Execution

EVS Chapter 1 ndash Mode of Existence

Impact of mode of existence on resources

Q) Why resources are under pressure

Ans - Increase in the sophistication

of technology enabling natural resources to be extracted quickly and efficiently Eg in the past it could take long hours just to cut down one tree only using saws Due to increased technology rates of deforestation have greatly increased

The number of humans is increasing Cultures of consumerism Materialistic views

lead to the mining of gold and diamonds to produce jewelry unnecessary commodities for human life or advancement Consumerism also leads to extraction of resources for the

production of commodities necessary for human life but in amounts excessive of what is needed because people consume more than is necessary or waste what they have

Lack of awareness among the population is striking People are not aware of ways to reduce depletion and exploitation of materials

Accounts Cash Book Today we are going to start a new topic -Cash Book

The key terms used in this chapter are

bullCash book

bullSimple cash book

bullDouble column cash book bullTriple column cash book

bullPetty cash book

bullCash discount

bullContra entry

Here I will share you the meaning of each key terms

bullCash book Cash Book is a special purpose subsidiary book or journal in which cash received and cash payments are recorded

bullSimple cash book

It is a cash book in which only cash transactions are recorded It has only one column on each side

bullTriple column cash book

It is cash book which has three columns one column for each cash and Bankdiscount on each side of the cash book In this book both cash and Bank transactions are recorded together with discount allowed and received

bullPetty cash book

It is a cash book maintained for recording petty expenses

bullCash discount

Cash discount is the amount of discount received or allowed on cash payments and cash receipts Discount received is an income for the business while discount allowed isan expense

bullContra entry

It means transactions involving both cash and Bank Such transactions though recorded in the cash book are not posted into ledger The letter lsquoC is written in Ledger folio for contra entry

Business Studies

ENTREPRENEURSHIP

Now we shall discuss the second chapter

lsquoENTREPRENEURSHIPrsquo

Today before starting the chapter let us recall what

Questions

1What are the main characteristics of Intrapreneurship

Answer

The main characteristics of Intrapreneurship are

Corporate framework-it occurs within the framework of the same company

Semi-Autonomous-Intrapreneurship

we have read last day

Let s today start the class by recalling the last topic taught

Intrapreneurship is the process of discovering and exploring business opportunities within an existing company It involves launching new business ventures within the framework of a present corporation Intrapreneurship is also known as corporate entrepreneurship or corporate venturing

Now let us start with the characteristics of Intrapreneurship

The main characteristics of Intrapreneurship are

Corporate framework

Semi-Autonomous Lack of ownership Senior position Low risk taking Not own boss

Now let us discuss the meaning of enterprise

Enterprise means an undertaking or adventure that requires some innovation and investment and thus involves riskEnterprise always entails decision making coordination and risk bearing

involves crating amd nurturing a semi-autonomous business unit which may be a subsidiary a strategic business unit or a division

Lack of ownership-the intrapreneur is not the owner of the unitb he creates and nurtures

Senior position-he occupies a senior managerial position in the company

Low risk taking-An intrapreneur does not bear the full risk of failure

Not own boss-An intrapreneur is not his own bosss in legal termsHe enjoys the freedom and gets the required resources and support

2 How is Entrepreneur is different from Intrapreneur

The functions involved in both the entrepreneurship and intrapreneurship are by and large similar however there are several differences between the two

Point of distinction

Entrepreneur

Intrapreneur

status An independent business person

A senior executive within a company

Ownership Owner of

the business

An employeesometimes a share in ownership

Financing Responsible for raising finance for the business

Not responsible for raising the finance

Risk bearing

Bears the risk of the business

Does not bears the risk of the business

Reward Profit which is uncertain and irregularcan be loss

Fixed salary and fringe benefits

Need for security low high

3 What do you understand by enterprise

Answer Enterprise means an undertaking or adventure that requires some innovation and investment and thus involves riskEnterprise always entails decision making coordination and risk bearing

COMMERCE NATURE AND OBJECTIVES OF

BUSINESS

Today let us recall the last other two objectives of business by the chart given in the previous class

Firstly we would discuss Human Objectives

Business is run by people and for people Labour is a valuable business element

Human objectives of business are concerned with the well -being of labour

The human objectives are as follows

Labour welfare Developing human

resources Participative

management Labour

management cooperation

Questions

1 Explain the human objectives of a business enterprise

Answer

Business is run by people and for people Labour is a valuable business element

Human objectives of business are concerned with the well -being of labour

The human objectives are as follows

Labour welfare-Business must recognize the dignity of labour and human factors should be given the recognition

Developing human resources-Employees must be provided the opportunities for developing new skills and attitudes

Participative management-Employees should be allowed to take part in decision making process of business

Labour management cooperation-Business should strive for creating and maintaining cordial employer employee relations so as to ensure peace and progress in industry

Now let us discuss the national objectives of business

Optimum utilization of resources

National self- reliance Development of small

scale industries Development of

backward areas Control over pollution

2Explain the national objectives of a business enterprise

Answer

It is the duty of business to utilize the resources of the country properly the national objectives of business

Optimum utilization of resources ndashBusiness should use the nationrsquos resources in the best possible manner

National self- reliance-It is the duty of the business to help the government in increasing experts and in reducing dependence on imports

Development of small scale industries-Big business firms are expected to encourage growth of small scale industries which are necessary for generating employment

Development of backward areas-Business is expected to give preference to the industrialization of backward regions of the country

ECONOMICS

BASIC ECONOMIC CONCEPTS

SUB

TOPIC

Value

Wealth

Welfare

Today we shall start with a new topic of the same chapter ie lsquoValuersquo

Value of a commodity is defined as the valuation placed by a household on the consumption of this commodity

lsquoValuersquo has two different meanings and these are

a Value -in -use It refers to consumption value of a commodity It expresses the utility derived from the consumption of a particular commodity A necessity like water has a very high value ndashin ndashuse or

Question

1What is value

Answer

Value of a commodity is defined as the valuation placed by a household on the consumption of this commodity

2What is value-in use

Answer It refers to consumption value of a commodity It expresses the utility derived from the consumption of a particular commodity A necessity like water has a very high value ndashin ndashuse or consumption value

3What is value ndashin- exchange

Answer It relates to market value of a commodity

It is the rate at which a particular good or service can be exchanged for money

For example in barter system if a person is prepared to exchange 3 metres of cloth with 1 pair

consumption value

b Value ndashin-exchange It relates to market value of a commodity

It is the rate at which a particular good or service can be exchanged for moneyFor example in barter system if a person is prepared to exchange 3 metres of cloth with 1 pair of shoes then the value in exchange of 3 metres of cloth is 1 pair of shoesValue in exchange is the power of purchasing other goods In modern monetised economies the exchange value of goods are expressed in terms of money as prices

Now let us discuss the term lsquoWealthrsquo

Wealth refers to the stock of all those assets which are a source of income

Wealth is a stock concept

Wealth must possess the following features

a Utility It must possess utility or give some

of shoes then the value in exchange of 3 metres of cloth is 1 pair of shoes

Value in exchange is the power of purchasing other goods In modern monetised economies the exchange value of goods are expressed in terms of money as prices

4 What is wealth

Answer it refers to the stock of assets or goods which are a source of income and have personal or national ownership

5 What are the features of wealth

Answer The features of wealth are as follows

Wealth must possess the following features

a Utility It must possess utility or give some satisfaction

b Scarcity It must be limited in quantityc Transferability it should be transferable its

ownership can be transferred from one person to another person

d Exchange value It must possess exchange value

6 What is welfare

Answer

Welfare is defined as satisfaction and happiness a sense of well- being among the people

satisfactionb Scarcity It must be

limited in quantityc Transferability It

should be transferable its ownership can be transferred from one person to another person

d Exchange value It must possess exchange value

Now let us discuss the term lsquoWelfarersquo

Welfare is defined as satisfaction and happiness a sense of well- being among the people

Welfare is affected by factors like

a Consumption of goods and services

b Environment

c Family relations

d Degree of freedom

e Law and order situation

Mathematics Trigonometric equation

To find the general solution of the equation sinθ=0

When sin θ =0

Then θ= 0 π2π 3π-π -2π -3

i e when θ = 0 or an integral multiple of π

i e when θ= nπ where n is any integer

Therefore the general solution of the equation sin

Example1 Find the general values of θ which satisfy the equation sin2 θ =34

Solution sin2 θ= 34

Or sin θ = +34 or -34

Or sin θ = sin π3 or sin (-π3)

Therefore

θ = [nπ + (-1) n (π3)] or[ nπ+ (-1) n (-π3)]

= nπ +π3 or nπ-π3 where n= any integer

Example 2Find the values of θ which satisfy tan2 θ

θ=0 is θ= nπ where n is any integer

To find the general solution of the equation cos θ=0

When cos θ=0

Then θ=π2 3π2 5π2 -π2 -3π2 -5π3

i e when θ is an odd multiple of π2

i e when θ=(2n+1) π2 where n= any integer

Therefore the general solution of the equation cos θ =0 is θ= (2n+1) π2 where n= any integer

To find the general solution of the equation tan θ = 0

Clearly tan θ =0 implies sin θcos θ =0

Therefore θ = nπ

i e the general solution of the equation tan θ=0 is θ =nπ where n = any integer

To find the general solution of the equation cot θ =0

Clearly cot θ =0 implies (cos θsin θ) = 0

i e cos θ =0

Therefore θ = (2n+1) π2

Therefore the general solution of the equation cot θ =0 is θ = (2n+1) π2

Where n= any integer

To find the general solution of the equation sin θ= k (-

=13 -πleθleπ

Solution tan2 θ =13

Or tan θ = plusmn1radic(3) =tan(plusmnπ6)

θ=nπ plusmn π6 where n =any integer

If n=0 then θ=plusmnπ6

If n=1 then θ= π plusmn π6

If n=-1 then θ= -π plusmn π6

Therefore the required solution in -π le θ le π are θ= π6 5π6 -π6 -5π6

Exercise Find general solution of sin 2θ=cos θ [Hints Use sin 2θ= 2sin θcosθ and then take cosθ

common]

1lek le1)

Determine an angle alpha such that sin =k and -π2le αle π2

Then we have

Sin θ = k = sin α

Or sin θ - sin α =0

Or 2 cos [(θ+α) 2] sin [(θ-α) 2] =0

Therefore either cos [(θ +α) 2] =0 (1)

Or sin [(θ-α) 2] =0 (2)

Now from (1) we get (θ+α) 2= (2m+1) π2)

Or θ = (2m+1) π-α (3)

And from (2) we get (θ-α) 2 =mπ

Or θ= 2mπ+α(4)

Where m = any integer

Clearly the solution (3) amp (4) may be combined in the following form

θ= nπ+(-1) n α where n= any integer

Therefore the general solution of sin θ = sin α is θ = nπ +(-1) n α where n is any integer and -π2 le α le π2

Biology Chapter - 04Kingdom Monera

Today we will discuss about bacterial reproduction and its usefulness

Fig Binary Fission

Fig Conjugation Fig Transformation

Bacterial reproduction is mainly asexual but sexual reproduction

also takes place

Asexual reproduction takes place by i) Binary fission - from one bacteriato

two bacteria are produced in every 20 to 30mins

ii) Buddingiii) Endospore formation - during

unfourable condition

Sexual reproduction by three ways

1) Conjugation - Transfer of genetic material between cells that are in physical contact with one another

2) Transduction - Transfer of genetic materialfrom one cell to another by a bacteriophage

3) Transformation - Transfer of cell-freeor naked DNArsquo from one cell to another

Bacteria causes different diseases inplants animals and human and

it causes food spoilage and waterpollution but it also have some useful

activities

i) Bacteria are helpful in sewage water treatment

ii) It is used in antibiotic (medicine) production

iii) Anaerobic bacteria help in biogas(energy) production

iv) Many household products like yoghurt cheese are manufactured by use of bacteria

v) Rhizobium by symbiotic relationship with leguminous plant increase soil fertility

vi) Besides these bacteria is helpful in genetic engineering degradation of petroleum hydrocarbonand in dairy

industry

Physics Motion in plane Here we will introduce Projectile Motion

Execution

Projectile

Y

usinθ u h

θX

ucosθ

Suppose a body is projected with an angle θ So initial velocity u can be resolved into two components

Horizontal component - ucosθ ( for range)

Vertical component - usinθ ( for height)

usinθ changes during motion and becomes zero at maximum height position but ucosθ remain unchanged

The maximum height of projectile is h

NB If initial is upward then g = -ve and if it is downward then g = +ve Height is +ve if direction of motion does not change ( for ex a body thrown upwards but goes down ultimately then height h = -ve)

The angle of projectile θ is the angle made with horizontal

HISTORY ndash GROWTH OF NATIONALISM

SUB TOPIC- REVOLUTIONARY NATIONALISM Bengal formation of Anushilan Samity and Jugantar Group

The intensification of the Swadeshi movement and Government policy of terror and repression led to outbreak of violence Bombs were manufactured and attempts on the lives of unpopular Government officials became frequent In the gymnasium of Scottish Church College which was known as General Assemblies Institution a secret society was formed known as Anushilan Samity

Aurobindo Ghosh send from Baroda his emissary Jatindranath Banerjee to mobilize the Bengal revolutionaries

Hemchandra Qanungo and Satyen Bose published Journal Jugantar

The Jugantar group planned to assassinate oppressive magistrate Kingsford by Khudiram Bose and Prafulla Chaki in 1908 Prafulla Chaki committed suicide to avoid arrest Khudiram was tried and hanged

Afew days later the police found a bomb factory in Maniktala and arrested a large number of revolutionaries The trial of revolutionaries became famous as the Alipore Bomb Case

In the course of the trial the approver the public prosecuter and a police officer were assassinated

1 Question Name two journals which preached the cult of violence

Answer a) Yugantar edited by Bhupendranath DuttaB) Bandemataram edited by Aurobindo Ghosh2 Question Why was Khudiram arrested and hangedAnswer An attempt was made to assassinate a hated vindictive majistrate named Kingsford by Khudiram Bose and Prafulla Chaki Their attempt failed and the bomb they threw killed two English ladies Khudiram was arrested and put to trial and then hanged3Question Who was Aurubindo GhoshAnswer Aurobindo Ghosh a nationalist revolutionary who was charged for his involvement in the Alipore Bombing Case He was accused of it along with his brother Barindra nath Ghosh But Aurobindo was acquitted because of the brilliant pleading of his counsel Chittaranjan Das Then he became a spiritual reformer introducing his visions on human progress and spiritual evolution4 Qustion Who was KingsfordAnswer Kingsford was an unpopular British chief Magistrate who was the target of the bomb thrown at Muzaffarpur by Khudiram and Prafulla Chaki

Most of the accused were convicted and sentenced to

long term of imprisonmentBut

Aurobindo Ghosh was acquitted mainly owing to the brilliant pleading of his counsel Chittaranjan Das

Political science Topic-Sovereignty

Summary Sovereignty is the full right and power of a governing body over itself without any interference from outside sources or bodies In political theory sovereignty is a substantive term designating supreme legitimate authority over some polity In international law sovereignty is the exercise of power by a state

Internal Sovereignty

Internal sovereignty means supreme authority within ones territory while external sovereignty relates to the recognition on the part of all states that each possesses this power in equal measure

External sovereignty

external sovereignty relates to the recognition on the part of all states that each possesses this power in equal measure

Distinguish between

Execution

Answer the following questions

Short notes-

Sovereignty

Internal Sovereignty

External sovereignty

Homework- learn

external sovereignty and internal sovereigntySovereignty is the principle

of supreme and

unquestionable authority

reflected in the claim by the

state to be the sole author of

laws within its territory

Definition of external vs internal sovereigntyInternal sovereignty refers to

the relationship between a

sovereign power and its

subjects ndash it refers to the

location of the supreme

authority within the state In

the UK for example internal

sovereignty (supposedly)

resides within Parliament

reflected in the

constitutional principle of

parliamentary

sovereigntyBy contrast

external sovereignty refers

to the capacity of the state

to act independently and

autonomously on the world

stage This is what is

sometimes called lsquostate

sovereigntyrsquo or lsquonational

sovereigntyrsquo and implies

that states are legally equal

and that the territorial

integrity and political

independence of a state is

inviolable

Class ndash XII

Date - 2742020 STUDY MATERIALSubject Topic Summary Execution Business Studies

Job Analysis amp Manpower Planning

At first let us recall the chapter what we have discussed till nowbullJob analysisbullJob specification bullJob description bullJob enlargement bullJob enrichment

Today we will do some questions answers from the chapter

Questions 1ldquoJob analysis job description and job specification are interrelatedrdquo Comment Answer) Job analysis is a systematic and detailed examination of a job to collect all the relevant information about it The contents off the job are summarised in the job description The qualification needed for the job are summarised in job specificationThus there is close interrelationship between job analysis job description and job specification

Question 2ldquoJob enlargement is a horizontal extension of a job whereas job enrichment is a vertical extension of a jobrdquo ElucidateAnswer) Job enlargement involves adding one or more task to a job coma where as job enrichment involves adding more autonomy and responsibility to a job Job enlargement is therefore horizontal extension of a job coma whereas job enrichment is a vertical extension of a job

Question 3 )

What is manpower estimation Explain its quantitative and qualitative aspectsAnswer) Manpower estimation is the process by which management determines how an organisation should move from itrsquos current manpower positionto its desired manpower position There are two dimensions of Manpower estimation- quantitative and qualitative

Quantitative aspectThis aspect of Manpower estimation involves estimating the number of employees required in a future time period Workload analysis and workforce analysis are done to estimate the quantity of required manpower

Qualitative aspectThe estimate of the knowledge skills experience etc of required manpower is the qualitative aspect of Manpower estimation The quality of Manpower can be judged on the basis of job analysisand job specification

COMMERCE

CAPITAL-FIXED AND WORKING

Today let us start the class by discussing the sources of finance for different types of business firms

The term lsquocapitalrsquo refers to the investment made in the enterprise for the purpose of earning profits

Requirements of capital and sources of capital for different types of business firms are

1 Capital for sole proprietorship businessA sole proprietor operates at a small scale and thereforerequires a limited amount of capital

2 Capital for partnership firmCapital requirements as well as capital base of a partnership is bigger than that of a sole trader businessThe owned capital is contributed by the partners in an agreed ratio

3 Capital for joint stock companyA joint stock company generally requires large amount of capitalA public company can raise huge capital through issue of shares In addition to share capital it can utilize retained profits

Now let u discuss the meaning of Finance PlanningFinance planning is the process of estimation the financial requirements of an organization specifying the sources of firms and ensuring that enough funds are available at the right time

1 What do you mean by Finance PlanningAnswer Finance planning is the process of estimation the financial requirements of an organization specifying the sources of firms and ensuring that enough funds are available at the right time

2Discuss the role of financial planning of an enterpriseThe role of financial planning are as followsa A sound financial plan helps a business enterprise to avaid the problems of shortage and surplus of fundsbFinancial planning serves as a guide in developing a sound capital structure so as to maximize returns to shareholders c It helps in effective utilization of fundsd It provides policies and procedures for coordinating different functional areas or departments of businesse It enables the management to exercise effective control over the financial activities of an enterprisef It helps the company to prepare for facing business shocks and surprises in future

Mathematics

Continuity and differentiability

Recall Definition of ContinuityLet f(x) be a single valued function of x and x=a be a point in the domain of definition of the function The function is said to be continuous at x=a ifi) f(c) is defined ie f(x) has a definite finite value at x=cii) lim xrarra f(x) exists andiii) lim xrarra f(x) =f(a) In other words f(x) is said to be continuous at x=a if lim xrarra+ f(x)= lim xrarra- f(x) = f(a) Or f(a+0) =f(a-0) =f(a) Or lim hrarr0 f(a+h)= f(a) Algebra of continuous functionsNow we will study some algebra of continuous functions Theorem 1 Suppose f and g be two real functions continuous at a real number c Then(1) f + g is continuous at x = c(2) f ndash g is continuous at x = c(3) f g is continuous at x = c(4) (fg) is continuous at x = c (provided g (c) ne 0)

Example 1 Prove that every rational function is continuousSolution Recall that every rational function f is given byf(x)=[p(x) q(x) ] q(x)ne0where p and q are polynomial functions The domain of f is all real numbers except points at which q is zero Since polynomial functions are continuous f is continuous by (4) of Theorem 1Example 2Discuss the continuity of sine functionSolution To see this we use the following factslim xrarr0 sin x =0Now observe that f (x) = sin x is defined for every real number Let c be a real number Put x = c + h If x rarr c we know that h rarr 0 Therefore lim xrarrc f(x)

= lim xrarrc sin x= lim hrarr0 sin(c+h) =lim hrarr0 [sin c cos h + cos c sin h ]=lim hrarr0 (sin c cos h) + lim hrarr0 (cos c sin h) = sin c +0=sin c = f(c) Thus lim xrarrc f(x) = f(c) and hence f is a constant function Exercise Prove that the function f(x) = x2 +2x is continuous for every real value of x [Hints show that lim xrarra+ f(x) = lim xrarra- f(x) = f(a) ]

Biology Reproduction in Flowering plants We will discuss about megasporoangium

megasporagenesis and female gametophyte

Q4 Describe the structure of megasporangium

Ovule is attached to the placenta by astalk called funicle

Each ovule has one two or three protectivecoverings called integuments

At the tip of integuments a small openingcalled micropyle is organised

Opposite to the micropylar end is the chalaza

Within the integuments a mass of cellsnucellusand inside it embryo sac orfemale gametophyte is present

Q5 Describe a mature embryosacamp its formation

In most of the flowering plants only oneof the 4 megaspores formed as a result ofmegasporogenesis that is functional while theother three degenerate

The

functional megaspore develops into thefemale gametophyte

Formation The nucleus

of the functional megasporedivides mitotically to form two nuclei first andthen two more sequential mitotic nucleardivisions result in the formation of four ampthen eight nucleate stages of embryo sac

Six of the eight nucleus are surrounded bycell walls and organised into cells

The remaining two nuclei called polar nuclei are found below the egg apparatus in the largecentral cell

Three cells consisting of two synergids amp one egg cell present bottom of

embryo sac Three cells

at the chalazal as antipodal cells

Two polar nuclei together present in large central cell

HISTORY

TOWARDS INDEPENDENCE AND PARTITION THE LAST PHASE(1935-1947)SUB TOPIC NATIONAL MOVEMENTS DURING THE SECOND WORLD WAR

Spread of Quit India Movement On 9th August 1942Gandhiji and other Congress leaders were arrested The Congress was declared illegal The news of the arrest of all leaders marked the beginning of a widespread movement of India It was not possible for such a movement to remain peacefulBut the arrest of the all notable congress leaders virtually left the movement in the hands of the mass The movement took the form of violent and militant outbreakBesides congressmen revolutionaries also were very active in the movement The Congress Socialist group also played a prominent role

1 Question Why did the British authority arrest the Congress leaders on 9 th August 1942Answer Congress Working committee adopted the Quit India resolution which was to be ratified at the Bombay AICC meeting in 8th August 1942 They decided to launch a mass struggle on non-violent lines Gandhiji gave a clarion call to all section of the people rdquoKarenge ya Marengerdquo (do or die) Congress leaders gave the call to driving out

the British from IndiaViceroy had taken strong action against the Quit India movement Gandhiji and all the leaders of Congress were arrested

2 Question How did Quit India Movement spread out all over IndiaAnswer The news of the leaders lsquo arrest marked the beginning of a widespread movement to remain peacefulThe movement took form of violent outbreak There were widespread cutting of telephone and Telegraph wires damaging railway lines raising barricades in cities and towns and other forms of violent demonstations

Question Name the leaders of Congress

Socialist group played a prominent part Notable among the Jayprakash Narayan Rammonohar Lohia Aruna Asaf Ali

Political science

Topic-Franchise and Representation

Summary

The election commission

The Election Commission of India is an autonomous constitutional authority responsible for administering Union and State election processes in India The body administers elections to the Lok Sabha Rajya Sabha State Legislative Assemblies in India and the offices of the President and Vice President in the country

Functions of election commission-

India is a sovereign socialist secular democratic republic Democracy runs like a golden thread in the social economic and political fabric woven by the Constitution given by lsquoWe the People of Indiarsquo unto ourselves The concept of democracy as visualised by the Constitution pre-supposes the representation of the people in Parliament and State legislatures by the method of election The Supreme Court has held that democracy is one of the inalienable basic features of the Constitution of India and forms part of its basic structure The Constitution of India adopted a Parliamentary form of government Parliament consists of the President of India and the two Houses mdash Rajya Sabha and Lok Sabha India being a Union of states has separate state legislatures for each state State legislatures consist of the Governor and two Houses mdash Legislative Council and Legislative Assembly mdash in seven states namely Andhra Pradesh Telangana Bihar Jammu amp Kashmir Karnataka Maharashtra and Uttar Pradesh and of the Governor and the state Legislative Assembly in the remaining 22 states Apart from the above two out of the seven Union Territories namely National Capital Territory of Delhi and Puducherry also have their Legislative Assemblies

ExecutionShort notes-Election commissionFunctions of election commission

Homework- Learn

Computer

Science

Computer hardware NAND Gate

A NOT-AND operation is known as NAND operation It has n input (n gt= 2) and one output

Logic diagram

Truth Table

NOR Gate

A NOT-OR operation is known as NOR operation It has n input (n gt= 2) and one output

Logic diagram

Truth Table

XOR Gate

XOR or Ex-OR gate is a special type of gate It can be used in the half

adder full adder and subtractor The exclusive-OR gate is abbreviated as EX-OR gate or sometime as X-OR gate It has n input (n gt= 2) and one output

Logic diagram

Truth Table

XNOR Gate

XNOR gate is a special type of gate It can be used in the half adder full adder and subtractor The exclusive-NOR gate is abbreviated as EX-NOR gate or sometime as X-NOR gate It has n input (n gt= 2) and one output

Logic diagram

Truth Table

Physics

Chapter 1 Electric Field ( Electric Dipole) (Summary)

Here we will derive Expression of electric field at broad side

On position of dipole

Execution

Q With the help of a labelled diagram obtain an expression for the electric field intensity E at any point on the equitorial line ( broad-side on position) of an electric dipole

Ans

E1 E1sinθ

E θ P E1 θ

( r2+L2)12 E2 E

r E2 E2sinθ

-q θ L O L +qA B

Let us consider that the point P is situated on the right bisector of the dipole AB at a distance r meter from its midpoint O

Let E1 and E2 be the electric field intensities of the electric field at P due to charge +q and ndashq of the dipole resp The distance of P from each charge is ( r2+L2)12

So E1 = 14 πϵ q

(r 2+L 2) away from +q

E2 = 14 πϵ q

(r 2+L 2) towards ndashq

The magnitudes of E1 and E2 are equal but directions are different Now resolving E1 and E2 into two components parallel and perpendicular to AB we get

The components perpendicular to AB E1sinθ and E2sinθ cancel each other because they are equal and opposite

The components parallel to AB are E1cosθ and E2 cosθ are in same direction and add up

So resultant intensity of electric field at the point P is

E = E1cosθ + E2 cosθ

E = 14 πϵ q

(r 2+L 2) 2 cosθ

Now from fig we have cosθ =BOBP = L (r2+L2)12

So we get E = 14 πϵ 2qL ( r2+L2)32

Now electric dipole moment p= 2qL

So E = 14 πϵ p ( r2+L2)32

HW Find the expression of Electric field as done here but this time take r gtgt 2L

Also find the expression of torque experience by a dipole

(Hint Electric force experienced by charges of dipole in electric field is qE each Let θ be the angle which dipole makes with electric lines of force then perpendicular distance between two charges is 2Lsinθ Then torque = force x perp distance = qE x 2L sinθ So τ=pE sinθ where p =2qL )

STUDY MATERIAL

Class XIISubject Eng Literature (The Tempest ndash William Shakespeare) Topic Act IV Scene 1 Lines 84 to 133 (Iris hellip A contract of true love Be not too late ) Date 27th April 2020 (4th Period)

[Students should read the original play and also the paraphrase given in the school prescribed textbook]Summary Questions amp Answers

o Ceres soon appears and comes to know that she has been summoned to celebrate the contract of true love

o Ceres expresses her unwillingness to meet Venus and Cupid as she has shunned their company

o Ceres and Juno both bestow their blessings upon Ferdinand and Miranda with June gifting honour riches happiness in marriage and Ceres presents plenty of earthrsquos produce

o Iris summons the water-nymphs and reapers to come and celebrate a contract

(1) IRIS Of her society (Line 91-101)

Be not afraid I met her deity

Cutting the clouds towards Pathos and her sonDove-drawn with her Here thought they to have doneSome wanton charm upon this man and maidWhose vows are that no bed-right shall be paidTill Hymens torch be lightedmdashbut in vainMarss hot minion is returned againHer waspish-headed son has broke his arrowsSwears he will shoot no more but play with sparrowsAnd be a boy right out

(i) Where were Venus and Cupid seen flying How were they travelling Why did they want to join the marriage celebration of Ferdinand and Miranda

of true love

Venus and Cupid were seen flying through the air towards Paphos the famous city which is situated on the island of Cyprus They were travelling by air-borne chariot drawn by doves They certainly wanted to come here in order to play some amorous trick upon Ferdinand and Miranda who are under a vow not to gratify their physical desires till the holy ceremony of their marriage has been performed(ii) What have Venus and Cupid done after failing in their plan

After being failure of their plan Venus who is a very passionate deity and who is the mistress of Mars (the god of war) has gone back while here ill-tempered son Cupid has broken his arrows of love in his state of desperation(iii) What has Cupid firmly decided

Cupid is feeling so disappointed that he has firmly decided to shoot no more arrows to arouse love in human hearts but to spend his time playing with sparrows Thus he would now become just a boy and would give up his original function of shooting arrows on human beings to make them fall in love(iv) What vow had Ceres taken How did Ceres feel at the abduction

After the abduction of her daughter Prosperina by Pluto Ceres had taken a vow to always keep away from the disgraceful company of Venus and her blind son Cupid the god of love Ceres felt deeply distressed when Pluto had carried off her daughter and had made her his wife by force(v) Why has Ceres not forgiven Venus and her blind son For what do Ceres want to be sure

As the abduction had been manipulated by Venus the goddess of beauty and love and her blind son Cupid Ceres has never forgiven them for their part in the whole plot Ceres wants to be sure that she would not have to meet Venus and Cupid who had engineered the abduction of her daughter Prosperina

AS THIS lsquoMASQUErsquo SCENE IS VERY IMPORTANT IN THE PLAY THE PARAPHRASE OF THE ENTIRE PORTION OF MASQUE SCENE (Act IV Lines 58 to 143) IS GIVEN BELOW

IRIS Goddess of RainbowCERES Goddess of Agriculture and all the fruits of the earth

(Nature growth prosperity rebirth ndash notions intimately connected to marriage)JUNO The majestic Queen of Heavens and wife of Jupiter (Jupiter is the king of Gods)

VENUS The Goddess of love CUPID Son of Venus PLUTO God of death (In the play referred by Shakespeare as lsquoDisrsquo which is a Roman name for Pluto)

ORIGINAL TEXT PARAPHRASEPROSPEROWellmdash

PROSPERONow come Ariel Let there be too many rather than too few

Now come my Ariel Bring a corollaryRather than want a spirit Appear and pertly[to Ferdinand and Miranda]No tongue all eyes Be silent

spirits in attendance Appear briskly

[to Ferdinand and Miranda]Look with your eyes but do not say a word

[Soft music] [Soft music][Enter Iris] [Enter Iris]

IRISCeres most bounteous lady thy rich leasOf wheat rye barley vetches oats and peasThy turfy mountains where live nibbling sheepAnd flat meads thatched with stover them to keepThy banks with pioned and twilled brimsWhich spongy April at thy hest betrimsTo make cold nymphs chaste crowns and thybroom-grovesWhose shadow the dismissegraved bachelor lovesBeing lass-lorn thy pole clipped vineyardAnd thy sea-marge sterile and rocky-hardWhere thou thyself dost airmdashthe Queen othrsquoSkyWhose watery arch and messenger am IBids thee leave these and with her sovereign grace[Juno appears] Here on this grass-plot in this very placeTo come and sport Her peacocks fly amainApproach rich Ceres her to entertain

IRISCeres most generous lady you are the cause of rich fields or fertile land where wheat rye barley beans oats and peas grow the grassy mountains where the sheep graze and the flat meadows covered with coarse hay to be used as fodder for cattleYour banks are covered with marsh-marigolds and reeds and the rainy April under your orders brings forth to make for the maids who are not in love beautiful crowns your woods where the broom flourishes and where the bachelor who has been dismissed by the maid he loved lies down being forsaken your vineyard in which the poles are embraced by the vines and the margin of the sea which is barren and rocky where you roam about to enjoy the fresh air ndash the queen of the sky (Juno) whose messenger I am besides being represented as the rainbow bids you leave all these and with her majesty here on this grassy plot in this very place come and sport her peacocks carry her fast in her chariot through the air and are making their way here approach rich Ceres to welcome her

[Enter Ariel as Ceres] [Enter Ariel as Ceres]

CERESHail many-coloured messenger that neerDost disobey the wife of JupiterWho with thy saffron wings upon my flowersDiffusest honey-drops refreshing showersAnd with each end of thy blue bow dost crownMy bosky acres and my unshrubbed downRich scarf to my proud earth Why hath thy queenSummoned me hither to this short-grassed green

CERESWelcome rainbow that never dared disobey Juno the wife of Jupiter who with your orange coloured rays spread honey-drops refreshing showers And with each end of thy blue bow drown my bushy acres and my hilly country which is free from shrubs you thus forming a rich scarf Why has your queen called me here to this place covered with short grass

IRISA contract of true love to celebrateAnd some donation freely to estateOn the blest lovers

IRISI have called you to celebrate a contract of true love and bestow some liberal gift upon the blessed lovers

ORIGINAL TEXT PARAPHRASECERESTell me heavenly bowIf Venus or her son as thou dost knowDo now attend the queen Since they did plotThe means that dusky Dis my daughter gotHer and her blind boys scandaled companyI have forsworn

CERESTell me heavenly bow if Venus the Goddess of love or Cupid her son and pedlar of passion at this time attend the heavenly queen Juno because you are sure to know Since the day they conspired against me and dark Pluto took away my daughter here and Cupidrsquos disgraceful company I have left off

IRISOf her societyBe not afraid I met her deityCutting the clouds towards Pathos and her sonDove-drawn with her Here thought they to have doneSome wanton charm upon this man and miad

IRISBe not afraid of her company I met her deity moving on the clouds towards Paphos the sacred home of Venus on the island of Cyprus along with her son on her chariot drawn by doves Here they contemplated to exercise a charm upon this man and maid producing

Whose vows are that no bed-right shall be paidTill Hymens torch be lightedmdashbut in vainMarss hot minion is returned againHer waspish-headed son has broke his arrowsSwears he will shoot no more but play with sparrowsAnd be a boy right out

wantonness before the actual marriage ceremony but did not succeed Venus has returned her irritable son has broken his arrows and swears that he will give up his practice of trying to inspire love but play with sparrows and be a boy again

[Music is heard] [Music is heard]

CERESHighst queen of stateGreat Juno comes I know her by her gait

CERESHighest queen of state Great Juno there she comes I know here by her gait

[Enter Juno] [Enter Juno]

JUNOHow does my bounteous sister Go with meTo bless this twain that they may Prosperous beAnd honoured in their issue

JUNOHow are you doing my generous sister Come with me to bless this couple so that they may be prosperous and fortunate in their children

[They sing] [They sing]

JUNOHonour riches marriage-blessingLong continuance and increasingHourly joys be still upon youJuno sings her blessings upon you

JUNOMay honour riches happiness in marriage long continuance and increase of those boons ever rest upon you as hourly joys Juno showers down upon you her blessings in song

CERESEarths increase foison plentyBarns and garners never emptyVines and clustring bunches growingPlants and goodly burden bowingSpring come to you at the farthestIn the very end of harvestScarcity and want shall shun youCeresrsquo blessing so is on you

CERESMay you have the plenty of earthrsquos produce Your barns and granaries may never be empty Your vines may grow with clustering bunches Your fruit trees may be heavily laden with their fruit May there be continuous spring and harvest May scantiness and want leave you forever Such is the blessing of Ceres upon you

FERDINANDThis is a most majestic vision andHarmoniously charmingly May I be boldTo think these spirits

FERDINANDThis is a great vision and magically melodious Should I suppose the characters (taking part in the masque) are spirits

PROSPEROSpirits which by mine artI have from their confines calld to enactMy present fancies

PROSPEROYes they are spirits whom I have summoned from the regions to which they are confined to carry into effect my fanciful designs

ORIGINAL TEXT PARAPHRASEFERDINANDLet me live here everSo rare a wondered father and a wifeMakes this place paradise

FERDINANDI should like to live here forever Such a wise and wonderful father makes this place a paradise

[Juno and Ceres whisper and send Iris on employment] [Juno and Ceres whisper and send Iris on employment]

PROSPEROSweet now silence

PROSPEROMy dear Ferdinand speak no more Juno and Ceres are

Juno and Ceres whisper seriouslyTheres something else to do Hush and be muteOr else our spell is marred

whispering with a solemn look There is something else coming Silence Or else our magic will be spoilt

IRISYour nymphs called naiads of the wandering brooksWith your sedged crowns and over-harmless looksLeave your crisp channels and on this green landAnswer your summons Juno does commandCome temperate nymphs and help to celebrateA contract of true love Be not too late

IRISYou nymphs called Naiads denizens (M inhabitants) of the running stream with your chaplets of sedge and ever-helpful looks leave your wrinkled channels and on the green land answer the summons sent to you Juno has ordered some chaste nymphs and help to celebrate a noble and true marriage Donrsquot delay

[Enter certain nymphs] [Enter certain nymphs]You sunburnt sicklemen of August wearyCome hither from the furrow and be merryMake holiday your rye-straw hats put onAnd these fresh nymphs encounter every oneIn country footing

You sunburnt harvesters weary from the effects of the heat in August come here from the furrowed land and rejoice Make holiday with your rye-straw hats upon you and meet these fresh nymphs and join in country dancing

[Enter certain reapers properly habited They join with the nymphs in a graceful dance towards the end whereof Prospero starts suddenly and speaks]

[Enter certain reapers properly habited They join with the nymphs in a graceful dance towards the end whereof Prospero starts suddenly and speaks]

PROSPERO[aside] I had forgot that foul conspiracyOf the beast Caliban and his confederatesAgainst my life The minute of their plotIs almost come [to the spirits]Well done Avoidno more

PROSPERO(Aside)I had forgotten the wicked conspiracy of the beast Caliban and his accomplices against my life the time of their plot has almost arrived ndash (To the Spirits) well done depart no more of this

[To a strange hollow and confused noise the spirits heavily vanish]

[The spirits depart]

ORIGINAL TEXT PARAPHRASEFERDINANDLet me live here everSo rare a wondered father and a wifeMakes this place paradise

FERDINANDI should like to live here forever Such a wise and wonderful father makes this place a paradise

[Juno and Ceres whisper and send Iris on employment] [Juno and Ceres whisper and send Iris on employment]

PROSPEROSweet now silenceJuno and Ceres whisper seriouslyTheres something else to do Hush and be muteOr else our spell is marred

PROSPEROMy dear Ferdinand speak no more Juno and Ceres are whispering with a solemn look There is something else coming Silence Or else our magic will be spoilt

IRISYour nymphs called naiads of the wandering brooksWith your sedged crowns and over-harmless looksLeave your crisp channels and on this green landAnswer your summons Juno does commandCome temperate nymphs and help to celebrateA contract of true love Be not too late

IRISYou nymphs called Naiads denizens (M inhabitants) of the running stream with your chaplets of sedge and ever-helpful looks leave your wrinkled channels and on the green land answer the summons sent to you Juno has ordered some chaste nymphs and help to celebrate a noble and true marriage Donrsquot delay

[Enter certain nymphs] [Enter certain nymphs]You sunburnt sicklemen of August wearyCome hither from the furrow and be merryMake holiday your rye-straw hats put onAnd these fresh nymphs encounter every oneIn country footing

You sunburnt harvesters weary from the effects of the heat in August come here from the furrowed land and rejoice Make holiday with your rye-straw hats upon you and meet these fresh nymphs and join in country dancing

[Enter certain reapers properly habited They join with the nymphs in a graceful dance towards the end whereof Prospero starts suddenly and speaks]

[Enter certain reapers properly habited They join with the nymphs in a graceful dance towards the end whereof Prospero starts suddenly and speaks]

PROSPERO[aside] I had forgot that foul conspiracyOf the beast Caliban and his confederatesAgainst my life The minute of their plotIs almost come [to the spirits]Well done Avoidno more

PROSPERO(Aside)I had forgotten the wicked conspiracy of the beast Caliban and his accomplices against my life the time of their plot has almost arrived ndash (To the Spirits) well done depart no more of this

[To a strange hollow and confused noise the spirits heavily vanish]

[The spirits depart]

Ac-12 27420 topic Revaluation of Assets and Liabilities

REVALUATION OF ASSETS AND LIABILITIES

On admission of a new partner the firm stands reconstituted and consequently the assets are revalued and liabilities are reassessed It is necessary to show the true position of the firm at the time of admission of a new partner If the values of the assets are raised gain will increase the capital of the existing partners Similarly any decrease in the value of assets ie loss will decrease the capital of the existing partners For this purpose alsquoRevaluation Accountrsquo is prepared This account is credited with all increases in the value of assets and decrease in the value of liabilities It is debited with decrease on account of value of assets and increase in the value of liabilities The balance of this account shows a gain or loss on revaluation which is transferred to the existing partnerrsquos capital account in existing profit sharing ratioAccounting for Revaluation of Assets and Liabilities when there is a Changein the Profit Sharing Ratio of Existing PartnersAssets and liabilities of a firm must also be revalued at the time of change in profit sharing ratio of existing partners The reason is that the realisable or actual value of assets and liabilities may be different from those shown in the Balance Sheet It is possible that with the passage of time some of the assets might have appreciated in value while the value of certain other assets might have decreased and no record has been made of such changes in the books of accounts Similarly there may be some unrecorded assets amp libilities that may have to be accounted for Revaluation of assets and reassessments of liabilities becomes necessary because the change in the

value of assets and liabilities belongs to the period to change in profit sharing ratio and hence must be shared by the partners in their old profit sharing ratio Revaluation of assets and reassessment of liabilities may be given effect to in two different ways (a) When revised values are to be recorded in the books and(b) When revised values are not to be recorded in the books

When revised values are to be recorded in the booksIn such a case revaluation of assets and reassessment of liabilities is done with the help of a new account called lsquoRevaluation Accountrsquo Sometimes this account is also called as lsquoProfit amp Loss Adjustment Acrsquo If there is a loss due to revaluation revaluation account is debited and if the revaluation results in a profit the revaluation account is credited The following journal entries made for this purpose are

(i) For increase in the value of assetsAsset Ac Dr (individually)To Revaluation Ac(ii) For decrease in the value of AssetRevaluation Ac Dr (individually)To Asset Ac[Decrease in the value of assets](iii) For increase in the value of LiabilitiesRevaluation Ac Dr (individually)To Liabilities Ac[Increase in the value of Liabilities](iv) For decrease in the value of LiabilitiesLiabilities Ac DrTo Revaluation Ac[Decrease in the value of Liabilities](v) For unrecorded AssetsAsset Ac [unrecorded] DrTo Revaluation Ac[Unrecorded asset recorded at actual value](vi) For unrecorded Liability Revaluation Ac DrTo Liability Ac [unrecorded][Unrecorded Liability recorded at actual value](vii) For transfer of gain on revaluationRevaluation Ac DrTo Existing Partnerrsquos CapitalCurrent Ac[Profit on revaluation transferred to capital account in existing ratio](viii) For transfer of loss on revaluationExisting Partnerrsquos CapitalCurrent Ac DrTo Revaluation Ac[Loss on revaluation transferred to capital account in existing ratio](a) When revaluation account shows gain Revaluation Ac DrTo Partnerrsquos Capital Ac (Old Profit Sharing Ratio)(Profit on revaluation credited to Partnerrsquos Capital Ac)(b) Above entry is reversed when revaluation account shows loss Partners Capital Acs (Old Profit Sharing Ratio) DrTo Revaluation Ac(Loss on revaluation debited to Partnerrsquos Capital Acs)

Proforma of Revaluation Account is given as under

Revaluation Account

Dr Cr Particulars ` Amount Particulars ` Amount To Decrease in value of assets By Increase in value of assets To Increase in value of liabilities By Decrease in value of liabilities To Unrecorded liabilities By Unrecorded assets To Gain on Revaluation (Transferred) By Loss on Revalution (Transferred)

ECO ndash12 2742020Topic- ELASTICITY OF DEMAND

CHAPTER - ELASTICITY OF DEMANDMEANINGDemand for a commodity is affected by many factors such as its price price of related goods income of its buyer tastes and preferences etc Elasticity means degree of response Elasticity of demand means degree of responsiveness of demand Demand for a commodity responds to change in price price of related goods income etc So we have three dimensions of elasticity of demandDIMENSION OF ELASTICITY OF DEMAND TYPES OF ELASTICITY OF DEMAND

Price elasticity of demand Income elasticity of demand Cross Elasticity of demand

Price elasticity of demand Price elasticity of demand means degree of responsiveness of demand for a commodity to the change in its price For example if demand for a commodity rises by 10 due to 5 fall in its price Price elasticity of demand (ep)=Percentage change in quantity demanded Percentage change in price of the commodity = 10 ( -)5 = ( - )2Note that ep will always be negative due to inverse relationship of price and quantity demanded

(ii) Income elasticity of demand Income elasticity of demand refers to the degree of responsiveness of demand for a commodity to the change in income of its buyer Suppose income of buyer rises by 10 and his demand for a commodity rises by 20 then Income elasticity of demand (ey)= change in quantity demanded change in price of the commodity =20 10 = 2

Cross Elasticity of demandCross elasticity of demand means the degree of responsiveness of demand for a commodity to the change in price of its related goods (substitute goods or complementary goods) Suppose demand for a commodity rises by 10 due to 5 rise in price of its substitute good then Cross elasticity of demand (ec) = change in quantity demanded change in price of related good = 10 2 = 5 (Tastes and preferences cannot be expressed numerically So elasticity ofdemand cannot be numerically expressed)

  • Chapter 1 Force (Summary)
  • Distinguish between external sovereignty and internal sovereignty
    • NAND Gate
      • Logic diagram
      • Truth Table
        • NOR Gate
          • Logic diagram
          • Truth Table
            • XOR Gate
              • Logic diagram
              • Truth Table
                • XNOR Gate
                  • Logic diagram
                  • Truth Table
                      • Physics
                      • Chapter 1 Electric Field ( Electric Dipole) (Summary)
Page 18:  · Web viewWe all know that Nouns are divided into two parts: common noun and proper noun.Apart from common and proper noun, we will also study about collective noun and compound

b Imperative sentences

c Interrogative sentences

d Exclamatory sentences

Sentences can be changed from one grammatical form to another without changing the meaning of the sentence This is known as transformation of sentences

the river ( Begin No sooner)2 None but brave deserve the fair (Begin the

bravehellip)3 This box is too heavy for me to lift ( Use so hellip

That instead of too)4 No one other than a king can live like James

Luxurious ( Begin only James)5 Oh for the wings of a dove (Begin I wishhellip)

Math Topic Commercial MathematicsChapter Shares and Dividends

Study item Discuss about shares and Dividends1) What is share

Ans To start any big business (company or Industry) a large sum of money is needed But it is not possible for an individual to invest such a large amount Then some persons interested in the business join together and from a company They divide the estimated money required into small parts Each such part is called a share

2) What do you mean by the term shareholder

Ans A person who purchases one or more shares is called shareholder3) Some terms related with a share

(i) Nominal value or face value or printed value The original value of a share is called its nominal value or face value or printed value

Note The nominal value of a share always remains same(ii) Market value or cash value The price of a share at any

time is called its market value or cash value

Note The market value of a share changes from time to time(iii) At par If the market value of a share is the same as its

nominal value the share is called at par(iv) At Premium or above Par If the market value of a share

is more than its nominal value the share is called at premium or above par

Example If a share of Rs 100 is selling at Rs 150 then it is said to be selling at a premium of Rs 50 or Rs 50 above par

(v) At Discount or below par If the market value of a share is less than its nominal value the share is called at discount or below par

Example If a share of Rs 100 is selling at Rs80 then it is said to be selling at a discount of Rs 20 or at Rs 20 below par

4) What is Dividend

Ans The profit which a shareholder gets for hisher investment from the company is called dividendNote (i) The dividend is always expressed as the percentage of the face value of the share(ii) The dividend is always given( by the company ) on the face value of the share

irrespective of the market value of the shareBENGALI(2ND LANGUAGE)

ldquoদেবতোর জণমrdquoলিবরোম চকরবত

পরথম লিসর পোঠ-চোর পসথ একটি পোথর লিবপলি ঘটোয় দেক যোতোয়োসতর পসথ পরলিতলিয়ত ওই পোথসর দেো োচট দেসত একলি দেতো দেক দেো োচট দেসয় দেবোমো সয় রোসতোর মোস লি0টসক পস1 লিবপরীত লিক দেথসক আো একটি দেমোটর োলি1 চোসকর কষতোয় পরোসরণ দেবোসচ যো লিকনত পরলিতবোর এমরণ দেৌভোয দেসকর োও সত পোসর তোই লিতলি দেকোো দেজোো1 কসর পোথরটিসক উপস1 দে8স পোথর উপস1 দে8োর ময় এক দেকৌতী জতো দেকসক পরশন কসর দেয লিতলি দেকোসো দেবতোর আস দেপসয়স0রণ লিকো লিকনত দেক বস লিতলি দেকোসো দেবতোর আস পোলি দেক উপলিত ক জতোর উসltসয বস কোরও ইস= স পোথরটি লিসয় দেযসত পোসর এর পর দেথসক দেক দেযসত আসত পোথরটি দেক দেসত পো একলি দেক কষয করস দেকউ পোথরটিসক ধসয়মস0 পলিরসকোর করস0 দেক ওই দেকৌতী জতোসক পোথসরর কোস0 বস থোকসত দেস এ0ো1ো আরও কষয কসর দেক ঠোৎ ওই লি1 পোথরটির োসয় লিোর োো-দেকউ পজো কসরস0 দেকৌতী দেোকটির আঙকো য় যলিসক উ পোথরটিসক লিরসয় দে8স তোর পর ঠোৎ একলি পোথরটির দেোোজ দেই দেক লিসয় দেস0 বো দেকোথোয় দেস0

বদোথ-

দেো োচট ndashচসত লিসয় দেকো লিক0র সE ধোককো দেস পস1 যোবোর উপকরমঅকসমোৎ- ঠোৎআতমমবর- লিসজসক লিয়নতরপ-পো লিপ0স প1োদেসতসসত- লিসপলিউৎোত- দেো1ো দেথসক উপস1 দে8োপরতযয় ndash লিবশবোপরতযোস- টতযোস ndash দৈবোসধসতোধলিসত- পরসপসরর পরলিত ব পরসয়ো করোপরসতরীভত- পোথসর পলিররণলিতবোনতঃকরসরণ- মস পরোসঅলিQৎ- অQো করসত ই=কদেোপ- বধমোর- দেবোস এমইতযোকোর- এইরকমরম- বময়পযসোভী-পসযর জয দেোভ আস0 যোরপোসথয় ঞচয়- পথচোর রচ জমোসোমোমোসরোস- ব ধম ধোসমর সE

তোর লি দেইhelliphellipTo be continued

Hindi 2nd lang सर क पद(सरदास)

सरदास शरी कषण भकति कावय क सरवशरषठ कगिरव ह इनक जनम और मतय क समय तथा सथान का मतभद हसरदास रवातसलय और शरार रस क अनयतम कगिरव ह इनक कावय म बालकषण क सौदय चपपल चषटा और गि7याओ की मनोहर झाकी मिमलती ह कषण और ोगिपयो क अननय परम का कतिचतरण ह सयो शरार की अपकषा उनक कावय म गिरवयो शरार का अमिBक गिरवषय और मारमिमEक कतिचतरण हआ हइन पकतियो म हम सरदास की भकति भारवनाओ का परिरचय मिमलता ह इनका सपण सगरह सरसार म गिनगिहत ह

1 जसोदा हरिर पालन झलारवहलरारवदलराईमलहारव रव जो ईसाई कछ ारव मर लाल को आई निनEदिदया काह ना आगिनसबारव त काह नाही बरवगिह आरवतोको कानहा बलारव

शबदाथ-हलरारव-गिहलती हदलराई - दलार पयार करती हमलहारव-पचकारती हनिनEदरिरया ndashनीदरवगिह-जलदी सअBर-होठमौन-चपसन-सकत

वयाखया- सरदास जी कहत ह गिक यशोदा माता बालक कषण को पालन म झल आती ह रवह उनह गिहलाती ह पयार करती ह मलहार जस कोई ीत ान लती ह और नीद स पछती ह गिक ह नीद तम मर लाल को आकर कयो नही सलाती तझ खाना बला रहा ह कभी कषण आख बद कर लत ह कभी आखफडफडान लत ह उनह सोता हआ जानकर यशोदा माता चप हो जाती ह और इशार म बात करन लती ह इसी बीच अकला कर कषण ज जात हतो गिफर यशोदा माता गिफर स ाना ान लती ह सरदास जी कहत ह गिक भरवान क दशन का सख दरवता और ऋगिष-मगिनयो को भी दलभ ह यही सख माता यशोदा को बडी सहजता स मिमल जा रही ह माता यशोदा बहत ही भागयशाली ह2)Continue to nexthellip

Physics

Chapter 1 Force

(Summary)

Question A body is acted upon by two forces each of magnitude F but in opposite directions State the effect of the forces if

(a) Both forces act at the same point of the body

(b)the two forces act at two different points of the body at a separation r

Solutions

(a) Resultant force acting on the body = 0

F ndash F = 0(b) The forces tend to rotate the body between two forces about the midpoint

Moment of forces = F times rFr

QuestionDefine moment of a couple Write its SI unit

Solutions

Moment of couple is equal to the product of both force and the perpendicular distance between the two forces

The SI unit of moment of couple is NmCommercial Studies

Advertising and sales

Business firms use several methods to

Questions1) What do you mean by advertising

promotion create demand of their product in the market and increase it sales Such methods comprises of advertising sales promotion personal selling and publicityToday we are going to discuss about one of such methods It is advertising

Meaning of advertisingAdvertising is a paid form of non-personal presentation for promotion of Ideas goods and services

Importance or merits of advertising Advertising has importance to manufacturer or traders to customer and to society as a whole

Today we will see how advertisement help the manufacturer or traders

Answer) Advertising is a means of how a company encourages people to buy their products services or ideas It is one element of marketing which also includes design Research and data mining

2) Mention any three features of advertisingAnswer)The main features of advertising are

i) It is impersonal form of presentation for promotion of products and services of Ideas

ii) It is issued by identified sponsor The advertisement contains the name of the advertiser

iii) It is a form of mass communication because the message is directed to a large number of persons simultaneously

3) Mention the main merits or importance of advertisement to manufacturer or tradersAnswer)

i) Introducing new product A business organization can introduce itself and its products to the public through advertising

ii) Increase the sale Advertising leads to increase the sale of existing product by entering into new markets and attracting new customers

iii) Create steady demand Advertising creates sustains regular demand by smoothening out seasonal and other fluctuations It enables regular production for the organisation

iv) Economics of scale Advertising facilitate mass distribution of goods and steady demand which lead to large scale and regular production

v) Goodwill Advertising helps in creating a good image of the firm and reputation for its products

Biology Chapter - 03Genetics

Today wewill start chapter and discuss about Genetics Gregor Mendel is known as father of genetics Before entering into Mendelrsquos experiment on Genetics we must know

Q1 Define the following termsi) Genetics Genetics is the study of

transmissionof body features from parents to offspringand the laws relating to such transmission

ii) Heredity It may be defined as transmissionof genetically based characteristics from parentsto offspring

iii) Character and traits Any heritable

Importance to TraderIntroducing new productIncrease the saleCreate steady demandEconomics of scaleGoodwill

some terms featureis a character The alternative forms of acharacter are called traitsex Character (Hair shape) - Traits (Curly straight)

iv) Homologous chromosomes A pair ofcorresponding chromosomes of the same shapeand size one from each parent

v) Genes Genes are the specific parts (DNA segments) of a chromosome which determinethe hereditary characteristicsNearly 30000genes present in human

vi) Alleles Alternative forms of a gene occupying the same position (locus) on homologouschromosomes and affecting the same characteristicbut in different ways

vii) Genotype ndash PhenotypeGenotype means of genes present in the cells of an organism Phenotype means the observable characteristic which is genetically controlled

viii) Mutation It is a sudden change in one or more genes or in the number or in the structure of chromosomes ex Sickle cell anaemia is a blood disease caused by a gene mutation

CLASS NOTES

Class XSubject Eng Literature (The Merchant of Venice ndash William Shakespeare)Topic Act IV Scene 1 Lines 01 to 34 ( Duke helliphelliphellip We all expect a gentle answer Jew) ate 27th April 2020 (2nd Period)

[Students should read the original play and also the paraphrase given in the school prescribed textbook]Summary Questions amp Answers

This scene may be termed as the catastrophe of the play It is the final unravelling of the complicated events which seem to threaten the happiness of Bassanio Portia and Antonio Right is justified to the fullest degree and malice falls into the trap prepared for others No one suffers here but Shylock but even then he receives a generous measure of mercy

o This is the Court-scene Initially we meet

(1)

DUKE I am sorry for thee thou art come to answer (Line 3-6)A stony adversary an inhuman wretchUncapable of pity void and emptyFrom any dram of mercy

(i) Who is addressed here Where is the person Why is the person there

Antonio is addressed hereAntonio is in the court of justice at VeniceAntoniorsquos trial is scheduled to be held here for his failure to meet the conditions of the bond he signed with Shylock

the Duke Antonio Shylock and Salerio Later we meet Bassanio Portia Gratiano and Nerissa

o The Duke says to Antonio that he has to face a very cruel opponent which Antonio admits and expresses his gratefulness to the Duke for his efforts to soften without result the heart of Shylock in order to be merciful to Antonio Antonio further says that he is ready to accept whatever cruel judgement the Court may award

o When Shylock appears in the court the Duke says that Shylock should change his decision of prosecuting Antonio and demanding the penalty specified in the bond out of consideration of the great misfortunes that Antonio has suffered If this is done by him (Shylock) the whole court would be gladdened by his merciful action

(ii) What is the Duke sorry for

The Duke is unable to change the mind of Shylock from his decision to get the bond forfeited even after he pleaded to Shylock Shylock stands firmly for his bond which when forfeited will allow him to take a pound of flesh from any part of Antoniorsquos body(iii) How does the Duke address Shylock`The Duke calls Shylock an adversary with a heart of stones He calls Shylock as an inhuman wretch without pity Shylock is quite lacking in the slightest quality of mercy (iv) How does Antonio reply to this

Antonio replies that he will meet the revenge of Shylock patiently He has prepared himself to suffer with a quiet spirit the utmost that Shylockrsquos tyranny and rage can do(v) What quality of the Duke is revealed here

The Duke is kind and benevolent He is ready to help Antonio He requests Shylock to free Antonio from the trial(vi) What are the terms of the bond that Antonio has signed

The terms of the bond that Antonio has signed were that if Antonio is unable to repay Shylock a certain sum of money specified on the paper on a certain date and in an agreed place the forfeit has to be paid The forfeiture will be an exact pound of Antoniorsquos flesh which Shylock will be a liberty to take from any part of Antoniorsquos body which pleases him

Class XI

STUDY MATERIAL

Class XISubject Eng Literature (The Tempest ndash William Shakespeare) Topic Act I Scene 2 Lines 88 to 132 (Prospero hellip Me and thy crying self) Date 27th April 2020 (3rd Period)

[Students should read the original play and also the paraphrase given in the school prescribed textbook]Summary Questions amp Answers

o Prospero now tells Miranda that he was the Duke of Milan He had been devoting himself more to studies than the affairs of the State His brother Antonio took advantage of this situation and with the help of Alonso the king of Naples seized upon him and her one midnight and shipped them in a frail bark so that they perished in the sea All this took place

(1)

MIRANDA I should sin (Line 118-132)

To think but nobly of my grandmother

Good wombs have borne bad sonsPROSPERO Now the condition

The King of Naples being an enemyTo me inveterate hearkens my brothers suitWhich was that he in lieu othrsquo premisesOf homage and I know not how much tribute

twelve years back

IMPORTANT PASSAGES EXPLAINED(Line 98-103)

PROSPERO helliphelliphelliphelliphelliphelliphellip

He being thus lorded

Not only with what my revenue yieldedBut what my power might else exact like oneWho having into truth by telling of it Made such a sinner of his memoryTo credit his own lie he did believeHe was indeed the duke

Prospero in telling the narrative of his past life here refers to his brother Antonio Prospero being with a studious bent of mind has left the administration of Milan on his younger brother Now Antonio being thus invested like a lord with all the powers derived from Prosperorsquos wealth and what the exercise of Prosperorsquos authority might secure for him regarded himself as a de facto Duke of Milan It is a well-known fact of psychology that a man who repeatedly tells a lie makes of his memory such a sinner against truth as to credit his own lie by the telling of it So Antonio by repeatedly saying to himself and others that he was the Duke came to believe that he was really the Duke Thus falsehood repeatedly asserted gained the force of truth for Antonio and he truly believed it

Should presently extirpate me and mine Out of the dukedom and confer fair MilanWith all the honours on my brother whereonA treacherous army levied one midnightFated to thrsquo purpose did Antonio openThe gates of Milan and ithrsquo dead of darkness The ministers for thrsquo purpose hurried thenceMe and thy crying self

(i) In the earlier lines of this scene what does Prospero tell about his intense interest What was the demand of his interest

In the earlier lines of this scene Prospero tells Miranda that he had an intense interest in the study of philosophy and magic arts Hence in order to improve his mind with this kind of study he kept himself isolated from worldly and state affairs His study was dearer to him than the applause and esteem that he could win from the public His study demanded too much solitude(ii) What forced Antonio to take an undue advantage over Prospero

Prosperorsquos indifferent attitude towards the statersquos affairs and his having boundless trust in Antonio gave rise to a boundless lust for power in Antoniorsquos mind Antonio felt that he must be the actual Duke instead of the part of the Duke he played Thus Antonio took an undue advantage of the situation to usurp Prosperorsquos dukedom(iii) Explain the following lines ldquoI should sin to think but nobly of my grandmother Good wombs have borne bad sonsrdquo

After hearing the treacherous act of her uncle Antonio Miranda says that Prosperorsquos mother was a noble lady and she cannot dishonour her memory by saying that the person named Antonio cannot be his (Prosperorsquos) brother She says that in honour of her grandmother she also cannot say that Antonio must have been begotten not by her grandfather but by some other man She finally concedes that it is known that good mothers have borne bad sons in their wombs and gave birth to them

(iv) Why did the King of Naples accept Antoniorsquos request to help him in usurping his dukedom What did Antonio propose to Alonso

Alonso who was the king of Naples accepted Antoniorsquos request in usurping Prosperorsquos dukedom because he (Alonso) was a sworn enemy of Prospero Antonio proposed that Alonso should immediately drive him (Prospero) and his offspring out of Milan and should confer the dukedom upon him (Antonio) with all the dignities which go with that

position In return Antonio promised that he shall give an annual tribute and also swore his allegiance to Alonso Also he agreed to hold the Dukedom of Milan as a subordinate to the state of Naples(v) How were Prospero and Miranda carried away from the city of Milan and what was the state of small Miranda at that time

In pursuance of the agreement settled between Antonio and Alonso an army of treacherous men was assembled One midnight when the occasion suited the will of destiny Antonio opened the gates of the city of Milan and in the death like silence of midnight Antoniorsquos agents who had been directed to execute his purpose carried Prospero and small Miranda away from the city in all haste They were then forced into a ship and carried some distance out to sea where they put them on a mere hulk of a boat without any rigging or ship-gear and abandoned them leaving them at the mercy of the roaring sea Miranda was a very small child of three years age and she was crying at that time

CLASS -XIDATE-270420Subject Topic Summary Execution

EVS Chapter 1 ndash Mode of Existence

Impact of mode of existence on resources

Q) Why resources are under pressure

Ans - Increase in the sophistication

of technology enabling natural resources to be extracted quickly and efficiently Eg in the past it could take long hours just to cut down one tree only using saws Due to increased technology rates of deforestation have greatly increased

The number of humans is increasing Cultures of consumerism Materialistic views

lead to the mining of gold and diamonds to produce jewelry unnecessary commodities for human life or advancement Consumerism also leads to extraction of resources for the

production of commodities necessary for human life but in amounts excessive of what is needed because people consume more than is necessary or waste what they have

Lack of awareness among the population is striking People are not aware of ways to reduce depletion and exploitation of materials

Accounts Cash Book Today we are going to start a new topic -Cash Book

The key terms used in this chapter are

bullCash book

bullSimple cash book

bullDouble column cash book bullTriple column cash book

bullPetty cash book

bullCash discount

bullContra entry

Here I will share you the meaning of each key terms

bullCash book Cash Book is a special purpose subsidiary book or journal in which cash received and cash payments are recorded

bullSimple cash book

It is a cash book in which only cash transactions are recorded It has only one column on each side

bullTriple column cash book

It is cash book which has three columns one column for each cash and Bankdiscount on each side of the cash book In this book both cash and Bank transactions are recorded together with discount allowed and received

bullPetty cash book

It is a cash book maintained for recording petty expenses

bullCash discount

Cash discount is the amount of discount received or allowed on cash payments and cash receipts Discount received is an income for the business while discount allowed isan expense

bullContra entry

It means transactions involving both cash and Bank Such transactions though recorded in the cash book are not posted into ledger The letter lsquoC is written in Ledger folio for contra entry

Business Studies

ENTREPRENEURSHIP

Now we shall discuss the second chapter

lsquoENTREPRENEURSHIPrsquo

Today before starting the chapter let us recall what

Questions

1What are the main characteristics of Intrapreneurship

Answer

The main characteristics of Intrapreneurship are

Corporate framework-it occurs within the framework of the same company

Semi-Autonomous-Intrapreneurship

we have read last day

Let s today start the class by recalling the last topic taught

Intrapreneurship is the process of discovering and exploring business opportunities within an existing company It involves launching new business ventures within the framework of a present corporation Intrapreneurship is also known as corporate entrepreneurship or corporate venturing

Now let us start with the characteristics of Intrapreneurship

The main characteristics of Intrapreneurship are

Corporate framework

Semi-Autonomous Lack of ownership Senior position Low risk taking Not own boss

Now let us discuss the meaning of enterprise

Enterprise means an undertaking or adventure that requires some innovation and investment and thus involves riskEnterprise always entails decision making coordination and risk bearing

involves crating amd nurturing a semi-autonomous business unit which may be a subsidiary a strategic business unit or a division

Lack of ownership-the intrapreneur is not the owner of the unitb he creates and nurtures

Senior position-he occupies a senior managerial position in the company

Low risk taking-An intrapreneur does not bear the full risk of failure

Not own boss-An intrapreneur is not his own bosss in legal termsHe enjoys the freedom and gets the required resources and support

2 How is Entrepreneur is different from Intrapreneur

The functions involved in both the entrepreneurship and intrapreneurship are by and large similar however there are several differences between the two

Point of distinction

Entrepreneur

Intrapreneur

status An independent business person

A senior executive within a company

Ownership Owner of

the business

An employeesometimes a share in ownership

Financing Responsible for raising finance for the business

Not responsible for raising the finance

Risk bearing

Bears the risk of the business

Does not bears the risk of the business

Reward Profit which is uncertain and irregularcan be loss

Fixed salary and fringe benefits

Need for security low high

3 What do you understand by enterprise

Answer Enterprise means an undertaking or adventure that requires some innovation and investment and thus involves riskEnterprise always entails decision making coordination and risk bearing

COMMERCE NATURE AND OBJECTIVES OF

BUSINESS

Today let us recall the last other two objectives of business by the chart given in the previous class

Firstly we would discuss Human Objectives

Business is run by people and for people Labour is a valuable business element

Human objectives of business are concerned with the well -being of labour

The human objectives are as follows

Labour welfare Developing human

resources Participative

management Labour

management cooperation

Questions

1 Explain the human objectives of a business enterprise

Answer

Business is run by people and for people Labour is a valuable business element

Human objectives of business are concerned with the well -being of labour

The human objectives are as follows

Labour welfare-Business must recognize the dignity of labour and human factors should be given the recognition

Developing human resources-Employees must be provided the opportunities for developing new skills and attitudes

Participative management-Employees should be allowed to take part in decision making process of business

Labour management cooperation-Business should strive for creating and maintaining cordial employer employee relations so as to ensure peace and progress in industry

Now let us discuss the national objectives of business

Optimum utilization of resources

National self- reliance Development of small

scale industries Development of

backward areas Control over pollution

2Explain the national objectives of a business enterprise

Answer

It is the duty of business to utilize the resources of the country properly the national objectives of business

Optimum utilization of resources ndashBusiness should use the nationrsquos resources in the best possible manner

National self- reliance-It is the duty of the business to help the government in increasing experts and in reducing dependence on imports

Development of small scale industries-Big business firms are expected to encourage growth of small scale industries which are necessary for generating employment

Development of backward areas-Business is expected to give preference to the industrialization of backward regions of the country

ECONOMICS

BASIC ECONOMIC CONCEPTS

SUB

TOPIC

Value

Wealth

Welfare

Today we shall start with a new topic of the same chapter ie lsquoValuersquo

Value of a commodity is defined as the valuation placed by a household on the consumption of this commodity

lsquoValuersquo has two different meanings and these are

a Value -in -use It refers to consumption value of a commodity It expresses the utility derived from the consumption of a particular commodity A necessity like water has a very high value ndashin ndashuse or

Question

1What is value

Answer

Value of a commodity is defined as the valuation placed by a household on the consumption of this commodity

2What is value-in use

Answer It refers to consumption value of a commodity It expresses the utility derived from the consumption of a particular commodity A necessity like water has a very high value ndashin ndashuse or consumption value

3What is value ndashin- exchange

Answer It relates to market value of a commodity

It is the rate at which a particular good or service can be exchanged for money

For example in barter system if a person is prepared to exchange 3 metres of cloth with 1 pair

consumption value

b Value ndashin-exchange It relates to market value of a commodity

It is the rate at which a particular good or service can be exchanged for moneyFor example in barter system if a person is prepared to exchange 3 metres of cloth with 1 pair of shoes then the value in exchange of 3 metres of cloth is 1 pair of shoesValue in exchange is the power of purchasing other goods In modern monetised economies the exchange value of goods are expressed in terms of money as prices

Now let us discuss the term lsquoWealthrsquo

Wealth refers to the stock of all those assets which are a source of income

Wealth is a stock concept

Wealth must possess the following features

a Utility It must possess utility or give some

of shoes then the value in exchange of 3 metres of cloth is 1 pair of shoes

Value in exchange is the power of purchasing other goods In modern monetised economies the exchange value of goods are expressed in terms of money as prices

4 What is wealth

Answer it refers to the stock of assets or goods which are a source of income and have personal or national ownership

5 What are the features of wealth

Answer The features of wealth are as follows

Wealth must possess the following features

a Utility It must possess utility or give some satisfaction

b Scarcity It must be limited in quantityc Transferability it should be transferable its

ownership can be transferred from one person to another person

d Exchange value It must possess exchange value

6 What is welfare

Answer

Welfare is defined as satisfaction and happiness a sense of well- being among the people

satisfactionb Scarcity It must be

limited in quantityc Transferability It

should be transferable its ownership can be transferred from one person to another person

d Exchange value It must possess exchange value

Now let us discuss the term lsquoWelfarersquo

Welfare is defined as satisfaction and happiness a sense of well- being among the people

Welfare is affected by factors like

a Consumption of goods and services

b Environment

c Family relations

d Degree of freedom

e Law and order situation

Mathematics Trigonometric equation

To find the general solution of the equation sinθ=0

When sin θ =0

Then θ= 0 π2π 3π-π -2π -3

i e when θ = 0 or an integral multiple of π

i e when θ= nπ where n is any integer

Therefore the general solution of the equation sin

Example1 Find the general values of θ which satisfy the equation sin2 θ =34

Solution sin2 θ= 34

Or sin θ = +34 or -34

Or sin θ = sin π3 or sin (-π3)

Therefore

θ = [nπ + (-1) n (π3)] or[ nπ+ (-1) n (-π3)]

= nπ +π3 or nπ-π3 where n= any integer

Example 2Find the values of θ which satisfy tan2 θ

θ=0 is θ= nπ where n is any integer

To find the general solution of the equation cos θ=0

When cos θ=0

Then θ=π2 3π2 5π2 -π2 -3π2 -5π3

i e when θ is an odd multiple of π2

i e when θ=(2n+1) π2 where n= any integer

Therefore the general solution of the equation cos θ =0 is θ= (2n+1) π2 where n= any integer

To find the general solution of the equation tan θ = 0

Clearly tan θ =0 implies sin θcos θ =0

Therefore θ = nπ

i e the general solution of the equation tan θ=0 is θ =nπ where n = any integer

To find the general solution of the equation cot θ =0

Clearly cot θ =0 implies (cos θsin θ) = 0

i e cos θ =0

Therefore θ = (2n+1) π2

Therefore the general solution of the equation cot θ =0 is θ = (2n+1) π2

Where n= any integer

To find the general solution of the equation sin θ= k (-

=13 -πleθleπ

Solution tan2 θ =13

Or tan θ = plusmn1radic(3) =tan(plusmnπ6)

θ=nπ plusmn π6 where n =any integer

If n=0 then θ=plusmnπ6

If n=1 then θ= π plusmn π6

If n=-1 then θ= -π plusmn π6

Therefore the required solution in -π le θ le π are θ= π6 5π6 -π6 -5π6

Exercise Find general solution of sin 2θ=cos θ [Hints Use sin 2θ= 2sin θcosθ and then take cosθ

common]

1lek le1)

Determine an angle alpha such that sin =k and -π2le αle π2

Then we have

Sin θ = k = sin α

Or sin θ - sin α =0

Or 2 cos [(θ+α) 2] sin [(θ-α) 2] =0

Therefore either cos [(θ +α) 2] =0 (1)

Or sin [(θ-α) 2] =0 (2)

Now from (1) we get (θ+α) 2= (2m+1) π2)

Or θ = (2m+1) π-α (3)

And from (2) we get (θ-α) 2 =mπ

Or θ= 2mπ+α(4)

Where m = any integer

Clearly the solution (3) amp (4) may be combined in the following form

θ= nπ+(-1) n α where n= any integer

Therefore the general solution of sin θ = sin α is θ = nπ +(-1) n α where n is any integer and -π2 le α le π2

Biology Chapter - 04Kingdom Monera

Today we will discuss about bacterial reproduction and its usefulness

Fig Binary Fission

Fig Conjugation Fig Transformation

Bacterial reproduction is mainly asexual but sexual reproduction

also takes place

Asexual reproduction takes place by i) Binary fission - from one bacteriato

two bacteria are produced in every 20 to 30mins

ii) Buddingiii) Endospore formation - during

unfourable condition

Sexual reproduction by three ways

1) Conjugation - Transfer of genetic material between cells that are in physical contact with one another

2) Transduction - Transfer of genetic materialfrom one cell to another by a bacteriophage

3) Transformation - Transfer of cell-freeor naked DNArsquo from one cell to another

Bacteria causes different diseases inplants animals and human and

it causes food spoilage and waterpollution but it also have some useful

activities

i) Bacteria are helpful in sewage water treatment

ii) It is used in antibiotic (medicine) production

iii) Anaerobic bacteria help in biogas(energy) production

iv) Many household products like yoghurt cheese are manufactured by use of bacteria

v) Rhizobium by symbiotic relationship with leguminous plant increase soil fertility

vi) Besides these bacteria is helpful in genetic engineering degradation of petroleum hydrocarbonand in dairy

industry

Physics Motion in plane Here we will introduce Projectile Motion

Execution

Projectile

Y

usinθ u h

θX

ucosθ

Suppose a body is projected with an angle θ So initial velocity u can be resolved into two components

Horizontal component - ucosθ ( for range)

Vertical component - usinθ ( for height)

usinθ changes during motion and becomes zero at maximum height position but ucosθ remain unchanged

The maximum height of projectile is h

NB If initial is upward then g = -ve and if it is downward then g = +ve Height is +ve if direction of motion does not change ( for ex a body thrown upwards but goes down ultimately then height h = -ve)

The angle of projectile θ is the angle made with horizontal

HISTORY ndash GROWTH OF NATIONALISM

SUB TOPIC- REVOLUTIONARY NATIONALISM Bengal formation of Anushilan Samity and Jugantar Group

The intensification of the Swadeshi movement and Government policy of terror and repression led to outbreak of violence Bombs were manufactured and attempts on the lives of unpopular Government officials became frequent In the gymnasium of Scottish Church College which was known as General Assemblies Institution a secret society was formed known as Anushilan Samity

Aurobindo Ghosh send from Baroda his emissary Jatindranath Banerjee to mobilize the Bengal revolutionaries

Hemchandra Qanungo and Satyen Bose published Journal Jugantar

The Jugantar group planned to assassinate oppressive magistrate Kingsford by Khudiram Bose and Prafulla Chaki in 1908 Prafulla Chaki committed suicide to avoid arrest Khudiram was tried and hanged

Afew days later the police found a bomb factory in Maniktala and arrested a large number of revolutionaries The trial of revolutionaries became famous as the Alipore Bomb Case

In the course of the trial the approver the public prosecuter and a police officer were assassinated

1 Question Name two journals which preached the cult of violence

Answer a) Yugantar edited by Bhupendranath DuttaB) Bandemataram edited by Aurobindo Ghosh2 Question Why was Khudiram arrested and hangedAnswer An attempt was made to assassinate a hated vindictive majistrate named Kingsford by Khudiram Bose and Prafulla Chaki Their attempt failed and the bomb they threw killed two English ladies Khudiram was arrested and put to trial and then hanged3Question Who was Aurubindo GhoshAnswer Aurobindo Ghosh a nationalist revolutionary who was charged for his involvement in the Alipore Bombing Case He was accused of it along with his brother Barindra nath Ghosh But Aurobindo was acquitted because of the brilliant pleading of his counsel Chittaranjan Das Then he became a spiritual reformer introducing his visions on human progress and spiritual evolution4 Qustion Who was KingsfordAnswer Kingsford was an unpopular British chief Magistrate who was the target of the bomb thrown at Muzaffarpur by Khudiram and Prafulla Chaki

Most of the accused were convicted and sentenced to

long term of imprisonmentBut

Aurobindo Ghosh was acquitted mainly owing to the brilliant pleading of his counsel Chittaranjan Das

Political science Topic-Sovereignty

Summary Sovereignty is the full right and power of a governing body over itself without any interference from outside sources or bodies In political theory sovereignty is a substantive term designating supreme legitimate authority over some polity In international law sovereignty is the exercise of power by a state

Internal Sovereignty

Internal sovereignty means supreme authority within ones territory while external sovereignty relates to the recognition on the part of all states that each possesses this power in equal measure

External sovereignty

external sovereignty relates to the recognition on the part of all states that each possesses this power in equal measure

Distinguish between

Execution

Answer the following questions

Short notes-

Sovereignty

Internal Sovereignty

External sovereignty

Homework- learn

external sovereignty and internal sovereigntySovereignty is the principle

of supreme and

unquestionable authority

reflected in the claim by the

state to be the sole author of

laws within its territory

Definition of external vs internal sovereigntyInternal sovereignty refers to

the relationship between a

sovereign power and its

subjects ndash it refers to the

location of the supreme

authority within the state In

the UK for example internal

sovereignty (supposedly)

resides within Parliament

reflected in the

constitutional principle of

parliamentary

sovereigntyBy contrast

external sovereignty refers

to the capacity of the state

to act independently and

autonomously on the world

stage This is what is

sometimes called lsquostate

sovereigntyrsquo or lsquonational

sovereigntyrsquo and implies

that states are legally equal

and that the territorial

integrity and political

independence of a state is

inviolable

Class ndash XII

Date - 2742020 STUDY MATERIALSubject Topic Summary Execution Business Studies

Job Analysis amp Manpower Planning

At first let us recall the chapter what we have discussed till nowbullJob analysisbullJob specification bullJob description bullJob enlargement bullJob enrichment

Today we will do some questions answers from the chapter

Questions 1ldquoJob analysis job description and job specification are interrelatedrdquo Comment Answer) Job analysis is a systematic and detailed examination of a job to collect all the relevant information about it The contents off the job are summarised in the job description The qualification needed for the job are summarised in job specificationThus there is close interrelationship between job analysis job description and job specification

Question 2ldquoJob enlargement is a horizontal extension of a job whereas job enrichment is a vertical extension of a jobrdquo ElucidateAnswer) Job enlargement involves adding one or more task to a job coma where as job enrichment involves adding more autonomy and responsibility to a job Job enlargement is therefore horizontal extension of a job coma whereas job enrichment is a vertical extension of a job

Question 3 )

What is manpower estimation Explain its quantitative and qualitative aspectsAnswer) Manpower estimation is the process by which management determines how an organisation should move from itrsquos current manpower positionto its desired manpower position There are two dimensions of Manpower estimation- quantitative and qualitative

Quantitative aspectThis aspect of Manpower estimation involves estimating the number of employees required in a future time period Workload analysis and workforce analysis are done to estimate the quantity of required manpower

Qualitative aspectThe estimate of the knowledge skills experience etc of required manpower is the qualitative aspect of Manpower estimation The quality of Manpower can be judged on the basis of job analysisand job specification

COMMERCE

CAPITAL-FIXED AND WORKING

Today let us start the class by discussing the sources of finance for different types of business firms

The term lsquocapitalrsquo refers to the investment made in the enterprise for the purpose of earning profits

Requirements of capital and sources of capital for different types of business firms are

1 Capital for sole proprietorship businessA sole proprietor operates at a small scale and thereforerequires a limited amount of capital

2 Capital for partnership firmCapital requirements as well as capital base of a partnership is bigger than that of a sole trader businessThe owned capital is contributed by the partners in an agreed ratio

3 Capital for joint stock companyA joint stock company generally requires large amount of capitalA public company can raise huge capital through issue of shares In addition to share capital it can utilize retained profits

Now let u discuss the meaning of Finance PlanningFinance planning is the process of estimation the financial requirements of an organization specifying the sources of firms and ensuring that enough funds are available at the right time

1 What do you mean by Finance PlanningAnswer Finance planning is the process of estimation the financial requirements of an organization specifying the sources of firms and ensuring that enough funds are available at the right time

2Discuss the role of financial planning of an enterpriseThe role of financial planning are as followsa A sound financial plan helps a business enterprise to avaid the problems of shortage and surplus of fundsbFinancial planning serves as a guide in developing a sound capital structure so as to maximize returns to shareholders c It helps in effective utilization of fundsd It provides policies and procedures for coordinating different functional areas or departments of businesse It enables the management to exercise effective control over the financial activities of an enterprisef It helps the company to prepare for facing business shocks and surprises in future

Mathematics

Continuity and differentiability

Recall Definition of ContinuityLet f(x) be a single valued function of x and x=a be a point in the domain of definition of the function The function is said to be continuous at x=a ifi) f(c) is defined ie f(x) has a definite finite value at x=cii) lim xrarra f(x) exists andiii) lim xrarra f(x) =f(a) In other words f(x) is said to be continuous at x=a if lim xrarra+ f(x)= lim xrarra- f(x) = f(a) Or f(a+0) =f(a-0) =f(a) Or lim hrarr0 f(a+h)= f(a) Algebra of continuous functionsNow we will study some algebra of continuous functions Theorem 1 Suppose f and g be two real functions continuous at a real number c Then(1) f + g is continuous at x = c(2) f ndash g is continuous at x = c(3) f g is continuous at x = c(4) (fg) is continuous at x = c (provided g (c) ne 0)

Example 1 Prove that every rational function is continuousSolution Recall that every rational function f is given byf(x)=[p(x) q(x) ] q(x)ne0where p and q are polynomial functions The domain of f is all real numbers except points at which q is zero Since polynomial functions are continuous f is continuous by (4) of Theorem 1Example 2Discuss the continuity of sine functionSolution To see this we use the following factslim xrarr0 sin x =0Now observe that f (x) = sin x is defined for every real number Let c be a real number Put x = c + h If x rarr c we know that h rarr 0 Therefore lim xrarrc f(x)

= lim xrarrc sin x= lim hrarr0 sin(c+h) =lim hrarr0 [sin c cos h + cos c sin h ]=lim hrarr0 (sin c cos h) + lim hrarr0 (cos c sin h) = sin c +0=sin c = f(c) Thus lim xrarrc f(x) = f(c) and hence f is a constant function Exercise Prove that the function f(x) = x2 +2x is continuous for every real value of x [Hints show that lim xrarra+ f(x) = lim xrarra- f(x) = f(a) ]

Biology Reproduction in Flowering plants We will discuss about megasporoangium

megasporagenesis and female gametophyte

Q4 Describe the structure of megasporangium

Ovule is attached to the placenta by astalk called funicle

Each ovule has one two or three protectivecoverings called integuments

At the tip of integuments a small openingcalled micropyle is organised

Opposite to the micropylar end is the chalaza

Within the integuments a mass of cellsnucellusand inside it embryo sac orfemale gametophyte is present

Q5 Describe a mature embryosacamp its formation

In most of the flowering plants only oneof the 4 megaspores formed as a result ofmegasporogenesis that is functional while theother three degenerate

The

functional megaspore develops into thefemale gametophyte

Formation The nucleus

of the functional megasporedivides mitotically to form two nuclei first andthen two more sequential mitotic nucleardivisions result in the formation of four ampthen eight nucleate stages of embryo sac

Six of the eight nucleus are surrounded bycell walls and organised into cells

The remaining two nuclei called polar nuclei are found below the egg apparatus in the largecentral cell

Three cells consisting of two synergids amp one egg cell present bottom of

embryo sac Three cells

at the chalazal as antipodal cells

Two polar nuclei together present in large central cell

HISTORY

TOWARDS INDEPENDENCE AND PARTITION THE LAST PHASE(1935-1947)SUB TOPIC NATIONAL MOVEMENTS DURING THE SECOND WORLD WAR

Spread of Quit India Movement On 9th August 1942Gandhiji and other Congress leaders were arrested The Congress was declared illegal The news of the arrest of all leaders marked the beginning of a widespread movement of India It was not possible for such a movement to remain peacefulBut the arrest of the all notable congress leaders virtually left the movement in the hands of the mass The movement took the form of violent and militant outbreakBesides congressmen revolutionaries also were very active in the movement The Congress Socialist group also played a prominent role

1 Question Why did the British authority arrest the Congress leaders on 9 th August 1942Answer Congress Working committee adopted the Quit India resolution which was to be ratified at the Bombay AICC meeting in 8th August 1942 They decided to launch a mass struggle on non-violent lines Gandhiji gave a clarion call to all section of the people rdquoKarenge ya Marengerdquo (do or die) Congress leaders gave the call to driving out

the British from IndiaViceroy had taken strong action against the Quit India movement Gandhiji and all the leaders of Congress were arrested

2 Question How did Quit India Movement spread out all over IndiaAnswer The news of the leaders lsquo arrest marked the beginning of a widespread movement to remain peacefulThe movement took form of violent outbreak There were widespread cutting of telephone and Telegraph wires damaging railway lines raising barricades in cities and towns and other forms of violent demonstations

Question Name the leaders of Congress

Socialist group played a prominent part Notable among the Jayprakash Narayan Rammonohar Lohia Aruna Asaf Ali

Political science

Topic-Franchise and Representation

Summary

The election commission

The Election Commission of India is an autonomous constitutional authority responsible for administering Union and State election processes in India The body administers elections to the Lok Sabha Rajya Sabha State Legislative Assemblies in India and the offices of the President and Vice President in the country

Functions of election commission-

India is a sovereign socialist secular democratic republic Democracy runs like a golden thread in the social economic and political fabric woven by the Constitution given by lsquoWe the People of Indiarsquo unto ourselves The concept of democracy as visualised by the Constitution pre-supposes the representation of the people in Parliament and State legislatures by the method of election The Supreme Court has held that democracy is one of the inalienable basic features of the Constitution of India and forms part of its basic structure The Constitution of India adopted a Parliamentary form of government Parliament consists of the President of India and the two Houses mdash Rajya Sabha and Lok Sabha India being a Union of states has separate state legislatures for each state State legislatures consist of the Governor and two Houses mdash Legislative Council and Legislative Assembly mdash in seven states namely Andhra Pradesh Telangana Bihar Jammu amp Kashmir Karnataka Maharashtra and Uttar Pradesh and of the Governor and the state Legislative Assembly in the remaining 22 states Apart from the above two out of the seven Union Territories namely National Capital Territory of Delhi and Puducherry also have their Legislative Assemblies

ExecutionShort notes-Election commissionFunctions of election commission

Homework- Learn

Computer

Science

Computer hardware NAND Gate

A NOT-AND operation is known as NAND operation It has n input (n gt= 2) and one output

Logic diagram

Truth Table

NOR Gate

A NOT-OR operation is known as NOR operation It has n input (n gt= 2) and one output

Logic diagram

Truth Table

XOR Gate

XOR or Ex-OR gate is a special type of gate It can be used in the half

adder full adder and subtractor The exclusive-OR gate is abbreviated as EX-OR gate or sometime as X-OR gate It has n input (n gt= 2) and one output

Logic diagram

Truth Table

XNOR Gate

XNOR gate is a special type of gate It can be used in the half adder full adder and subtractor The exclusive-NOR gate is abbreviated as EX-NOR gate or sometime as X-NOR gate It has n input (n gt= 2) and one output

Logic diagram

Truth Table

Physics

Chapter 1 Electric Field ( Electric Dipole) (Summary)

Here we will derive Expression of electric field at broad side

On position of dipole

Execution

Q With the help of a labelled diagram obtain an expression for the electric field intensity E at any point on the equitorial line ( broad-side on position) of an electric dipole

Ans

E1 E1sinθ

E θ P E1 θ

( r2+L2)12 E2 E

r E2 E2sinθ

-q θ L O L +qA B

Let us consider that the point P is situated on the right bisector of the dipole AB at a distance r meter from its midpoint O

Let E1 and E2 be the electric field intensities of the electric field at P due to charge +q and ndashq of the dipole resp The distance of P from each charge is ( r2+L2)12

So E1 = 14 πϵ q

(r 2+L 2) away from +q

E2 = 14 πϵ q

(r 2+L 2) towards ndashq

The magnitudes of E1 and E2 are equal but directions are different Now resolving E1 and E2 into two components parallel and perpendicular to AB we get

The components perpendicular to AB E1sinθ and E2sinθ cancel each other because they are equal and opposite

The components parallel to AB are E1cosθ and E2 cosθ are in same direction and add up

So resultant intensity of electric field at the point P is

E = E1cosθ + E2 cosθ

E = 14 πϵ q

(r 2+L 2) 2 cosθ

Now from fig we have cosθ =BOBP = L (r2+L2)12

So we get E = 14 πϵ 2qL ( r2+L2)32

Now electric dipole moment p= 2qL

So E = 14 πϵ p ( r2+L2)32

HW Find the expression of Electric field as done here but this time take r gtgt 2L

Also find the expression of torque experience by a dipole

(Hint Electric force experienced by charges of dipole in electric field is qE each Let θ be the angle which dipole makes with electric lines of force then perpendicular distance between two charges is 2Lsinθ Then torque = force x perp distance = qE x 2L sinθ So τ=pE sinθ where p =2qL )

STUDY MATERIAL

Class XIISubject Eng Literature (The Tempest ndash William Shakespeare) Topic Act IV Scene 1 Lines 84 to 133 (Iris hellip A contract of true love Be not too late ) Date 27th April 2020 (4th Period)

[Students should read the original play and also the paraphrase given in the school prescribed textbook]Summary Questions amp Answers

o Ceres soon appears and comes to know that she has been summoned to celebrate the contract of true love

o Ceres expresses her unwillingness to meet Venus and Cupid as she has shunned their company

o Ceres and Juno both bestow their blessings upon Ferdinand and Miranda with June gifting honour riches happiness in marriage and Ceres presents plenty of earthrsquos produce

o Iris summons the water-nymphs and reapers to come and celebrate a contract

(1) IRIS Of her society (Line 91-101)

Be not afraid I met her deity

Cutting the clouds towards Pathos and her sonDove-drawn with her Here thought they to have doneSome wanton charm upon this man and maidWhose vows are that no bed-right shall be paidTill Hymens torch be lightedmdashbut in vainMarss hot minion is returned againHer waspish-headed son has broke his arrowsSwears he will shoot no more but play with sparrowsAnd be a boy right out

(i) Where were Venus and Cupid seen flying How were they travelling Why did they want to join the marriage celebration of Ferdinand and Miranda

of true love

Venus and Cupid were seen flying through the air towards Paphos the famous city which is situated on the island of Cyprus They were travelling by air-borne chariot drawn by doves They certainly wanted to come here in order to play some amorous trick upon Ferdinand and Miranda who are under a vow not to gratify their physical desires till the holy ceremony of their marriage has been performed(ii) What have Venus and Cupid done after failing in their plan

After being failure of their plan Venus who is a very passionate deity and who is the mistress of Mars (the god of war) has gone back while here ill-tempered son Cupid has broken his arrows of love in his state of desperation(iii) What has Cupid firmly decided

Cupid is feeling so disappointed that he has firmly decided to shoot no more arrows to arouse love in human hearts but to spend his time playing with sparrows Thus he would now become just a boy and would give up his original function of shooting arrows on human beings to make them fall in love(iv) What vow had Ceres taken How did Ceres feel at the abduction

After the abduction of her daughter Prosperina by Pluto Ceres had taken a vow to always keep away from the disgraceful company of Venus and her blind son Cupid the god of love Ceres felt deeply distressed when Pluto had carried off her daughter and had made her his wife by force(v) Why has Ceres not forgiven Venus and her blind son For what do Ceres want to be sure

As the abduction had been manipulated by Venus the goddess of beauty and love and her blind son Cupid Ceres has never forgiven them for their part in the whole plot Ceres wants to be sure that she would not have to meet Venus and Cupid who had engineered the abduction of her daughter Prosperina

AS THIS lsquoMASQUErsquo SCENE IS VERY IMPORTANT IN THE PLAY THE PARAPHRASE OF THE ENTIRE PORTION OF MASQUE SCENE (Act IV Lines 58 to 143) IS GIVEN BELOW

IRIS Goddess of RainbowCERES Goddess of Agriculture and all the fruits of the earth

(Nature growth prosperity rebirth ndash notions intimately connected to marriage)JUNO The majestic Queen of Heavens and wife of Jupiter (Jupiter is the king of Gods)

VENUS The Goddess of love CUPID Son of Venus PLUTO God of death (In the play referred by Shakespeare as lsquoDisrsquo which is a Roman name for Pluto)

ORIGINAL TEXT PARAPHRASEPROSPEROWellmdash

PROSPERONow come Ariel Let there be too many rather than too few

Now come my Ariel Bring a corollaryRather than want a spirit Appear and pertly[to Ferdinand and Miranda]No tongue all eyes Be silent

spirits in attendance Appear briskly

[to Ferdinand and Miranda]Look with your eyes but do not say a word

[Soft music] [Soft music][Enter Iris] [Enter Iris]

IRISCeres most bounteous lady thy rich leasOf wheat rye barley vetches oats and peasThy turfy mountains where live nibbling sheepAnd flat meads thatched with stover them to keepThy banks with pioned and twilled brimsWhich spongy April at thy hest betrimsTo make cold nymphs chaste crowns and thybroom-grovesWhose shadow the dismissegraved bachelor lovesBeing lass-lorn thy pole clipped vineyardAnd thy sea-marge sterile and rocky-hardWhere thou thyself dost airmdashthe Queen othrsquoSkyWhose watery arch and messenger am IBids thee leave these and with her sovereign grace[Juno appears] Here on this grass-plot in this very placeTo come and sport Her peacocks fly amainApproach rich Ceres her to entertain

IRISCeres most generous lady you are the cause of rich fields or fertile land where wheat rye barley beans oats and peas grow the grassy mountains where the sheep graze and the flat meadows covered with coarse hay to be used as fodder for cattleYour banks are covered with marsh-marigolds and reeds and the rainy April under your orders brings forth to make for the maids who are not in love beautiful crowns your woods where the broom flourishes and where the bachelor who has been dismissed by the maid he loved lies down being forsaken your vineyard in which the poles are embraced by the vines and the margin of the sea which is barren and rocky where you roam about to enjoy the fresh air ndash the queen of the sky (Juno) whose messenger I am besides being represented as the rainbow bids you leave all these and with her majesty here on this grassy plot in this very place come and sport her peacocks carry her fast in her chariot through the air and are making their way here approach rich Ceres to welcome her

[Enter Ariel as Ceres] [Enter Ariel as Ceres]

CERESHail many-coloured messenger that neerDost disobey the wife of JupiterWho with thy saffron wings upon my flowersDiffusest honey-drops refreshing showersAnd with each end of thy blue bow dost crownMy bosky acres and my unshrubbed downRich scarf to my proud earth Why hath thy queenSummoned me hither to this short-grassed green

CERESWelcome rainbow that never dared disobey Juno the wife of Jupiter who with your orange coloured rays spread honey-drops refreshing showers And with each end of thy blue bow drown my bushy acres and my hilly country which is free from shrubs you thus forming a rich scarf Why has your queen called me here to this place covered with short grass

IRISA contract of true love to celebrateAnd some donation freely to estateOn the blest lovers

IRISI have called you to celebrate a contract of true love and bestow some liberal gift upon the blessed lovers

ORIGINAL TEXT PARAPHRASECERESTell me heavenly bowIf Venus or her son as thou dost knowDo now attend the queen Since they did plotThe means that dusky Dis my daughter gotHer and her blind boys scandaled companyI have forsworn

CERESTell me heavenly bow if Venus the Goddess of love or Cupid her son and pedlar of passion at this time attend the heavenly queen Juno because you are sure to know Since the day they conspired against me and dark Pluto took away my daughter here and Cupidrsquos disgraceful company I have left off

IRISOf her societyBe not afraid I met her deityCutting the clouds towards Pathos and her sonDove-drawn with her Here thought they to have doneSome wanton charm upon this man and miad

IRISBe not afraid of her company I met her deity moving on the clouds towards Paphos the sacred home of Venus on the island of Cyprus along with her son on her chariot drawn by doves Here they contemplated to exercise a charm upon this man and maid producing

Whose vows are that no bed-right shall be paidTill Hymens torch be lightedmdashbut in vainMarss hot minion is returned againHer waspish-headed son has broke his arrowsSwears he will shoot no more but play with sparrowsAnd be a boy right out

wantonness before the actual marriage ceremony but did not succeed Venus has returned her irritable son has broken his arrows and swears that he will give up his practice of trying to inspire love but play with sparrows and be a boy again

[Music is heard] [Music is heard]

CERESHighst queen of stateGreat Juno comes I know her by her gait

CERESHighest queen of state Great Juno there she comes I know here by her gait

[Enter Juno] [Enter Juno]

JUNOHow does my bounteous sister Go with meTo bless this twain that they may Prosperous beAnd honoured in their issue

JUNOHow are you doing my generous sister Come with me to bless this couple so that they may be prosperous and fortunate in their children

[They sing] [They sing]

JUNOHonour riches marriage-blessingLong continuance and increasingHourly joys be still upon youJuno sings her blessings upon you

JUNOMay honour riches happiness in marriage long continuance and increase of those boons ever rest upon you as hourly joys Juno showers down upon you her blessings in song

CERESEarths increase foison plentyBarns and garners never emptyVines and clustring bunches growingPlants and goodly burden bowingSpring come to you at the farthestIn the very end of harvestScarcity and want shall shun youCeresrsquo blessing so is on you

CERESMay you have the plenty of earthrsquos produce Your barns and granaries may never be empty Your vines may grow with clustering bunches Your fruit trees may be heavily laden with their fruit May there be continuous spring and harvest May scantiness and want leave you forever Such is the blessing of Ceres upon you

FERDINANDThis is a most majestic vision andHarmoniously charmingly May I be boldTo think these spirits

FERDINANDThis is a great vision and magically melodious Should I suppose the characters (taking part in the masque) are spirits

PROSPEROSpirits which by mine artI have from their confines calld to enactMy present fancies

PROSPEROYes they are spirits whom I have summoned from the regions to which they are confined to carry into effect my fanciful designs

ORIGINAL TEXT PARAPHRASEFERDINANDLet me live here everSo rare a wondered father and a wifeMakes this place paradise

FERDINANDI should like to live here forever Such a wise and wonderful father makes this place a paradise

[Juno and Ceres whisper and send Iris on employment] [Juno and Ceres whisper and send Iris on employment]

PROSPEROSweet now silence

PROSPEROMy dear Ferdinand speak no more Juno and Ceres are

Juno and Ceres whisper seriouslyTheres something else to do Hush and be muteOr else our spell is marred

whispering with a solemn look There is something else coming Silence Or else our magic will be spoilt

IRISYour nymphs called naiads of the wandering brooksWith your sedged crowns and over-harmless looksLeave your crisp channels and on this green landAnswer your summons Juno does commandCome temperate nymphs and help to celebrateA contract of true love Be not too late

IRISYou nymphs called Naiads denizens (M inhabitants) of the running stream with your chaplets of sedge and ever-helpful looks leave your wrinkled channels and on the green land answer the summons sent to you Juno has ordered some chaste nymphs and help to celebrate a noble and true marriage Donrsquot delay

[Enter certain nymphs] [Enter certain nymphs]You sunburnt sicklemen of August wearyCome hither from the furrow and be merryMake holiday your rye-straw hats put onAnd these fresh nymphs encounter every oneIn country footing

You sunburnt harvesters weary from the effects of the heat in August come here from the furrowed land and rejoice Make holiday with your rye-straw hats upon you and meet these fresh nymphs and join in country dancing

[Enter certain reapers properly habited They join with the nymphs in a graceful dance towards the end whereof Prospero starts suddenly and speaks]

[Enter certain reapers properly habited They join with the nymphs in a graceful dance towards the end whereof Prospero starts suddenly and speaks]

PROSPERO[aside] I had forgot that foul conspiracyOf the beast Caliban and his confederatesAgainst my life The minute of their plotIs almost come [to the spirits]Well done Avoidno more

PROSPERO(Aside)I had forgotten the wicked conspiracy of the beast Caliban and his accomplices against my life the time of their plot has almost arrived ndash (To the Spirits) well done depart no more of this

[To a strange hollow and confused noise the spirits heavily vanish]

[The spirits depart]

ORIGINAL TEXT PARAPHRASEFERDINANDLet me live here everSo rare a wondered father and a wifeMakes this place paradise

FERDINANDI should like to live here forever Such a wise and wonderful father makes this place a paradise

[Juno and Ceres whisper and send Iris on employment] [Juno and Ceres whisper and send Iris on employment]

PROSPEROSweet now silenceJuno and Ceres whisper seriouslyTheres something else to do Hush and be muteOr else our spell is marred

PROSPEROMy dear Ferdinand speak no more Juno and Ceres are whispering with a solemn look There is something else coming Silence Or else our magic will be spoilt

IRISYour nymphs called naiads of the wandering brooksWith your sedged crowns and over-harmless looksLeave your crisp channels and on this green landAnswer your summons Juno does commandCome temperate nymphs and help to celebrateA contract of true love Be not too late

IRISYou nymphs called Naiads denizens (M inhabitants) of the running stream with your chaplets of sedge and ever-helpful looks leave your wrinkled channels and on the green land answer the summons sent to you Juno has ordered some chaste nymphs and help to celebrate a noble and true marriage Donrsquot delay

[Enter certain nymphs] [Enter certain nymphs]You sunburnt sicklemen of August wearyCome hither from the furrow and be merryMake holiday your rye-straw hats put onAnd these fresh nymphs encounter every oneIn country footing

You sunburnt harvesters weary from the effects of the heat in August come here from the furrowed land and rejoice Make holiday with your rye-straw hats upon you and meet these fresh nymphs and join in country dancing

[Enter certain reapers properly habited They join with the nymphs in a graceful dance towards the end whereof Prospero starts suddenly and speaks]

[Enter certain reapers properly habited They join with the nymphs in a graceful dance towards the end whereof Prospero starts suddenly and speaks]

PROSPERO[aside] I had forgot that foul conspiracyOf the beast Caliban and his confederatesAgainst my life The minute of their plotIs almost come [to the spirits]Well done Avoidno more

PROSPERO(Aside)I had forgotten the wicked conspiracy of the beast Caliban and his accomplices against my life the time of their plot has almost arrived ndash (To the Spirits) well done depart no more of this

[To a strange hollow and confused noise the spirits heavily vanish]

[The spirits depart]

Ac-12 27420 topic Revaluation of Assets and Liabilities

REVALUATION OF ASSETS AND LIABILITIES

On admission of a new partner the firm stands reconstituted and consequently the assets are revalued and liabilities are reassessed It is necessary to show the true position of the firm at the time of admission of a new partner If the values of the assets are raised gain will increase the capital of the existing partners Similarly any decrease in the value of assets ie loss will decrease the capital of the existing partners For this purpose alsquoRevaluation Accountrsquo is prepared This account is credited with all increases in the value of assets and decrease in the value of liabilities It is debited with decrease on account of value of assets and increase in the value of liabilities The balance of this account shows a gain or loss on revaluation which is transferred to the existing partnerrsquos capital account in existing profit sharing ratioAccounting for Revaluation of Assets and Liabilities when there is a Changein the Profit Sharing Ratio of Existing PartnersAssets and liabilities of a firm must also be revalued at the time of change in profit sharing ratio of existing partners The reason is that the realisable or actual value of assets and liabilities may be different from those shown in the Balance Sheet It is possible that with the passage of time some of the assets might have appreciated in value while the value of certain other assets might have decreased and no record has been made of such changes in the books of accounts Similarly there may be some unrecorded assets amp libilities that may have to be accounted for Revaluation of assets and reassessments of liabilities becomes necessary because the change in the

value of assets and liabilities belongs to the period to change in profit sharing ratio and hence must be shared by the partners in their old profit sharing ratio Revaluation of assets and reassessment of liabilities may be given effect to in two different ways (a) When revised values are to be recorded in the books and(b) When revised values are not to be recorded in the books

When revised values are to be recorded in the booksIn such a case revaluation of assets and reassessment of liabilities is done with the help of a new account called lsquoRevaluation Accountrsquo Sometimes this account is also called as lsquoProfit amp Loss Adjustment Acrsquo If there is a loss due to revaluation revaluation account is debited and if the revaluation results in a profit the revaluation account is credited The following journal entries made for this purpose are

(i) For increase in the value of assetsAsset Ac Dr (individually)To Revaluation Ac(ii) For decrease in the value of AssetRevaluation Ac Dr (individually)To Asset Ac[Decrease in the value of assets](iii) For increase in the value of LiabilitiesRevaluation Ac Dr (individually)To Liabilities Ac[Increase in the value of Liabilities](iv) For decrease in the value of LiabilitiesLiabilities Ac DrTo Revaluation Ac[Decrease in the value of Liabilities](v) For unrecorded AssetsAsset Ac [unrecorded] DrTo Revaluation Ac[Unrecorded asset recorded at actual value](vi) For unrecorded Liability Revaluation Ac DrTo Liability Ac [unrecorded][Unrecorded Liability recorded at actual value](vii) For transfer of gain on revaluationRevaluation Ac DrTo Existing Partnerrsquos CapitalCurrent Ac[Profit on revaluation transferred to capital account in existing ratio](viii) For transfer of loss on revaluationExisting Partnerrsquos CapitalCurrent Ac DrTo Revaluation Ac[Loss on revaluation transferred to capital account in existing ratio](a) When revaluation account shows gain Revaluation Ac DrTo Partnerrsquos Capital Ac (Old Profit Sharing Ratio)(Profit on revaluation credited to Partnerrsquos Capital Ac)(b) Above entry is reversed when revaluation account shows loss Partners Capital Acs (Old Profit Sharing Ratio) DrTo Revaluation Ac(Loss on revaluation debited to Partnerrsquos Capital Acs)

Proforma of Revaluation Account is given as under

Revaluation Account

Dr Cr Particulars ` Amount Particulars ` Amount To Decrease in value of assets By Increase in value of assets To Increase in value of liabilities By Decrease in value of liabilities To Unrecorded liabilities By Unrecorded assets To Gain on Revaluation (Transferred) By Loss on Revalution (Transferred)

ECO ndash12 2742020Topic- ELASTICITY OF DEMAND

CHAPTER - ELASTICITY OF DEMANDMEANINGDemand for a commodity is affected by many factors such as its price price of related goods income of its buyer tastes and preferences etc Elasticity means degree of response Elasticity of demand means degree of responsiveness of demand Demand for a commodity responds to change in price price of related goods income etc So we have three dimensions of elasticity of demandDIMENSION OF ELASTICITY OF DEMAND TYPES OF ELASTICITY OF DEMAND

Price elasticity of demand Income elasticity of demand Cross Elasticity of demand

Price elasticity of demand Price elasticity of demand means degree of responsiveness of demand for a commodity to the change in its price For example if demand for a commodity rises by 10 due to 5 fall in its price Price elasticity of demand (ep)=Percentage change in quantity demanded Percentage change in price of the commodity = 10 ( -)5 = ( - )2Note that ep will always be negative due to inverse relationship of price and quantity demanded

(ii) Income elasticity of demand Income elasticity of demand refers to the degree of responsiveness of demand for a commodity to the change in income of its buyer Suppose income of buyer rises by 10 and his demand for a commodity rises by 20 then Income elasticity of demand (ey)= change in quantity demanded change in price of the commodity =20 10 = 2

Cross Elasticity of demandCross elasticity of demand means the degree of responsiveness of demand for a commodity to the change in price of its related goods (substitute goods or complementary goods) Suppose demand for a commodity rises by 10 due to 5 rise in price of its substitute good then Cross elasticity of demand (ec) = change in quantity demanded change in price of related good = 10 2 = 5 (Tastes and preferences cannot be expressed numerically So elasticity ofdemand cannot be numerically expressed)

  • Chapter 1 Force (Summary)
  • Distinguish between external sovereignty and internal sovereignty
    • NAND Gate
      • Logic diagram
      • Truth Table
        • NOR Gate
          • Logic diagram
          • Truth Table
            • XOR Gate
              • Logic diagram
              • Truth Table
                • XNOR Gate
                  • Logic diagram
                  • Truth Table
                      • Physics
                      • Chapter 1 Electric Field ( Electric Dipole) (Summary)
Page 19:  · Web viewWe all know that Nouns are divided into two parts: common noun and proper noun.Apart from common and proper noun, we will also study about collective noun and compound

irrespective of the market value of the shareBENGALI(2ND LANGUAGE)

ldquoদেবতোর জণমrdquoলিবরোম চকরবত

পরথম লিসর পোঠ-চোর পসথ একটি পোথর লিবপলি ঘটোয় দেক যোতোয়োসতর পসথ পরলিতলিয়ত ওই পোথসর দেো োচট দেসত একলি দেতো দেক দেো োচট দেসয় দেবোমো সয় রোসতোর মোস লি0টসক পস1 লিবপরীত লিক দেথসক আো একটি দেমোটর োলি1 চোসকর কষতোয় পরোসরণ দেবোসচ যো লিকনত পরলিতবোর এমরণ দেৌভোয দেসকর োও সত পোসর তোই লিতলি দেকোো দেজোো1 কসর পোথরটিসক উপস1 দে8স পোথর উপস1 দে8োর ময় এক দেকৌতী জতো দেকসক পরশন কসর দেয লিতলি দেকোসো দেবতোর আস দেপসয়স0রণ লিকো লিকনত দেক বস লিতলি দেকোসো দেবতোর আস পোলি দেক উপলিত ক জতোর উসltসয বস কোরও ইস= স পোথরটি লিসয় দেযসত পোসর এর পর দেথসক দেক দেযসত আসত পোথরটি দেক দেসত পো একলি দেক কষয করস দেকউ পোথরটিসক ধসয়মস0 পলিরসকোর করস0 দেক ওই দেকৌতী জতোসক পোথসরর কোস0 বস থোকসত দেস এ0ো1ো আরও কষয কসর দেক ঠোৎ ওই লি1 পোথরটির োসয় লিোর োো-দেকউ পজো কসরস0 দেকৌতী দেোকটির আঙকো য় যলিসক উ পোথরটিসক লিরসয় দে8স তোর পর ঠোৎ একলি পোথরটির দেোোজ দেই দেক লিসয় দেস0 বো দেকোথোয় দেস0

বদোথ-

দেো োচট ndashচসত লিসয় দেকো লিক0র সE ধোককো দেস পস1 যোবোর উপকরমঅকসমোৎ- ঠোৎআতমমবর- লিসজসক লিয়নতরপ-পো লিপ0স প1োদেসতসসত- লিসপলিউৎোত- দেো1ো দেথসক উপস1 দে8োপরতযয় ndash লিবশবোপরতযোস- টতযোস ndash দৈবোসধসতোধলিসত- পরসপসরর পরলিত ব পরসয়ো করোপরসতরীভত- পোথসর পলিররণলিতবোনতঃকরসরণ- মস পরোসঅলিQৎ- অQো করসত ই=কদেোপ- বধমোর- দেবোস এমইতযোকোর- এইরকমরম- বময়পযসোভী-পসযর জয দেোভ আস0 যোরপোসথয় ঞচয়- পথচোর রচ জমোসোমোমোসরোস- ব ধম ধোসমর সE

তোর লি দেইhelliphellipTo be continued

Hindi 2nd lang सर क पद(सरदास)

सरदास शरी कषण भकति कावय क सरवशरषठ कगिरव ह इनक जनम और मतय क समय तथा सथान का मतभद हसरदास रवातसलय और शरार रस क अनयतम कगिरव ह इनक कावय म बालकषण क सौदय चपपल चषटा और गि7याओ की मनोहर झाकी मिमलती ह कषण और ोगिपयो क अननय परम का कतिचतरण ह सयो शरार की अपकषा उनक कावय म गिरवयो शरार का अमिBक गिरवषय और मारमिमEक कतिचतरण हआ हइन पकतियो म हम सरदास की भकति भारवनाओ का परिरचय मिमलता ह इनका सपण सगरह सरसार म गिनगिहत ह

1 जसोदा हरिर पालन झलारवहलरारवदलराईमलहारव रव जो ईसाई कछ ारव मर लाल को आई निनEदिदया काह ना आगिनसबारव त काह नाही बरवगिह आरवतोको कानहा बलारव

शबदाथ-हलरारव-गिहलती हदलराई - दलार पयार करती हमलहारव-पचकारती हनिनEदरिरया ndashनीदरवगिह-जलदी सअBर-होठमौन-चपसन-सकत

वयाखया- सरदास जी कहत ह गिक यशोदा माता बालक कषण को पालन म झल आती ह रवह उनह गिहलाती ह पयार करती ह मलहार जस कोई ीत ान लती ह और नीद स पछती ह गिक ह नीद तम मर लाल को आकर कयो नही सलाती तझ खाना बला रहा ह कभी कषण आख बद कर लत ह कभी आखफडफडान लत ह उनह सोता हआ जानकर यशोदा माता चप हो जाती ह और इशार म बात करन लती ह इसी बीच अकला कर कषण ज जात हतो गिफर यशोदा माता गिफर स ाना ान लती ह सरदास जी कहत ह गिक भरवान क दशन का सख दरवता और ऋगिष-मगिनयो को भी दलभ ह यही सख माता यशोदा को बडी सहजता स मिमल जा रही ह माता यशोदा बहत ही भागयशाली ह2)Continue to nexthellip

Physics

Chapter 1 Force

(Summary)

Question A body is acted upon by two forces each of magnitude F but in opposite directions State the effect of the forces if

(a) Both forces act at the same point of the body

(b)the two forces act at two different points of the body at a separation r

Solutions

(a) Resultant force acting on the body = 0

F ndash F = 0(b) The forces tend to rotate the body between two forces about the midpoint

Moment of forces = F times rFr

QuestionDefine moment of a couple Write its SI unit

Solutions

Moment of couple is equal to the product of both force and the perpendicular distance between the two forces

The SI unit of moment of couple is NmCommercial Studies

Advertising and sales

Business firms use several methods to

Questions1) What do you mean by advertising

promotion create demand of their product in the market and increase it sales Such methods comprises of advertising sales promotion personal selling and publicityToday we are going to discuss about one of such methods It is advertising

Meaning of advertisingAdvertising is a paid form of non-personal presentation for promotion of Ideas goods and services

Importance or merits of advertising Advertising has importance to manufacturer or traders to customer and to society as a whole

Today we will see how advertisement help the manufacturer or traders

Answer) Advertising is a means of how a company encourages people to buy their products services or ideas It is one element of marketing which also includes design Research and data mining

2) Mention any three features of advertisingAnswer)The main features of advertising are

i) It is impersonal form of presentation for promotion of products and services of Ideas

ii) It is issued by identified sponsor The advertisement contains the name of the advertiser

iii) It is a form of mass communication because the message is directed to a large number of persons simultaneously

3) Mention the main merits or importance of advertisement to manufacturer or tradersAnswer)

i) Introducing new product A business organization can introduce itself and its products to the public through advertising

ii) Increase the sale Advertising leads to increase the sale of existing product by entering into new markets and attracting new customers

iii) Create steady demand Advertising creates sustains regular demand by smoothening out seasonal and other fluctuations It enables regular production for the organisation

iv) Economics of scale Advertising facilitate mass distribution of goods and steady demand which lead to large scale and regular production

v) Goodwill Advertising helps in creating a good image of the firm and reputation for its products

Biology Chapter - 03Genetics

Today wewill start chapter and discuss about Genetics Gregor Mendel is known as father of genetics Before entering into Mendelrsquos experiment on Genetics we must know

Q1 Define the following termsi) Genetics Genetics is the study of

transmissionof body features from parents to offspringand the laws relating to such transmission

ii) Heredity It may be defined as transmissionof genetically based characteristics from parentsto offspring

iii) Character and traits Any heritable

Importance to TraderIntroducing new productIncrease the saleCreate steady demandEconomics of scaleGoodwill

some terms featureis a character The alternative forms of acharacter are called traitsex Character (Hair shape) - Traits (Curly straight)

iv) Homologous chromosomes A pair ofcorresponding chromosomes of the same shapeand size one from each parent

v) Genes Genes are the specific parts (DNA segments) of a chromosome which determinethe hereditary characteristicsNearly 30000genes present in human

vi) Alleles Alternative forms of a gene occupying the same position (locus) on homologouschromosomes and affecting the same characteristicbut in different ways

vii) Genotype ndash PhenotypeGenotype means of genes present in the cells of an organism Phenotype means the observable characteristic which is genetically controlled

viii) Mutation It is a sudden change in one or more genes or in the number or in the structure of chromosomes ex Sickle cell anaemia is a blood disease caused by a gene mutation

CLASS NOTES

Class XSubject Eng Literature (The Merchant of Venice ndash William Shakespeare)Topic Act IV Scene 1 Lines 01 to 34 ( Duke helliphelliphellip We all expect a gentle answer Jew) ate 27th April 2020 (2nd Period)

[Students should read the original play and also the paraphrase given in the school prescribed textbook]Summary Questions amp Answers

This scene may be termed as the catastrophe of the play It is the final unravelling of the complicated events which seem to threaten the happiness of Bassanio Portia and Antonio Right is justified to the fullest degree and malice falls into the trap prepared for others No one suffers here but Shylock but even then he receives a generous measure of mercy

o This is the Court-scene Initially we meet

(1)

DUKE I am sorry for thee thou art come to answer (Line 3-6)A stony adversary an inhuman wretchUncapable of pity void and emptyFrom any dram of mercy

(i) Who is addressed here Where is the person Why is the person there

Antonio is addressed hereAntonio is in the court of justice at VeniceAntoniorsquos trial is scheduled to be held here for his failure to meet the conditions of the bond he signed with Shylock

the Duke Antonio Shylock and Salerio Later we meet Bassanio Portia Gratiano and Nerissa

o The Duke says to Antonio that he has to face a very cruel opponent which Antonio admits and expresses his gratefulness to the Duke for his efforts to soften without result the heart of Shylock in order to be merciful to Antonio Antonio further says that he is ready to accept whatever cruel judgement the Court may award

o When Shylock appears in the court the Duke says that Shylock should change his decision of prosecuting Antonio and demanding the penalty specified in the bond out of consideration of the great misfortunes that Antonio has suffered If this is done by him (Shylock) the whole court would be gladdened by his merciful action

(ii) What is the Duke sorry for

The Duke is unable to change the mind of Shylock from his decision to get the bond forfeited even after he pleaded to Shylock Shylock stands firmly for his bond which when forfeited will allow him to take a pound of flesh from any part of Antoniorsquos body(iii) How does the Duke address Shylock`The Duke calls Shylock an adversary with a heart of stones He calls Shylock as an inhuman wretch without pity Shylock is quite lacking in the slightest quality of mercy (iv) How does Antonio reply to this

Antonio replies that he will meet the revenge of Shylock patiently He has prepared himself to suffer with a quiet spirit the utmost that Shylockrsquos tyranny and rage can do(v) What quality of the Duke is revealed here

The Duke is kind and benevolent He is ready to help Antonio He requests Shylock to free Antonio from the trial(vi) What are the terms of the bond that Antonio has signed

The terms of the bond that Antonio has signed were that if Antonio is unable to repay Shylock a certain sum of money specified on the paper on a certain date and in an agreed place the forfeit has to be paid The forfeiture will be an exact pound of Antoniorsquos flesh which Shylock will be a liberty to take from any part of Antoniorsquos body which pleases him

Class XI

STUDY MATERIAL

Class XISubject Eng Literature (The Tempest ndash William Shakespeare) Topic Act I Scene 2 Lines 88 to 132 (Prospero hellip Me and thy crying self) Date 27th April 2020 (3rd Period)

[Students should read the original play and also the paraphrase given in the school prescribed textbook]Summary Questions amp Answers

o Prospero now tells Miranda that he was the Duke of Milan He had been devoting himself more to studies than the affairs of the State His brother Antonio took advantage of this situation and with the help of Alonso the king of Naples seized upon him and her one midnight and shipped them in a frail bark so that they perished in the sea All this took place

(1)

MIRANDA I should sin (Line 118-132)

To think but nobly of my grandmother

Good wombs have borne bad sonsPROSPERO Now the condition

The King of Naples being an enemyTo me inveterate hearkens my brothers suitWhich was that he in lieu othrsquo premisesOf homage and I know not how much tribute

twelve years back

IMPORTANT PASSAGES EXPLAINED(Line 98-103)

PROSPERO helliphelliphelliphelliphelliphelliphellip

He being thus lorded

Not only with what my revenue yieldedBut what my power might else exact like oneWho having into truth by telling of it Made such a sinner of his memoryTo credit his own lie he did believeHe was indeed the duke

Prospero in telling the narrative of his past life here refers to his brother Antonio Prospero being with a studious bent of mind has left the administration of Milan on his younger brother Now Antonio being thus invested like a lord with all the powers derived from Prosperorsquos wealth and what the exercise of Prosperorsquos authority might secure for him regarded himself as a de facto Duke of Milan It is a well-known fact of psychology that a man who repeatedly tells a lie makes of his memory such a sinner against truth as to credit his own lie by the telling of it So Antonio by repeatedly saying to himself and others that he was the Duke came to believe that he was really the Duke Thus falsehood repeatedly asserted gained the force of truth for Antonio and he truly believed it

Should presently extirpate me and mine Out of the dukedom and confer fair MilanWith all the honours on my brother whereonA treacherous army levied one midnightFated to thrsquo purpose did Antonio openThe gates of Milan and ithrsquo dead of darkness The ministers for thrsquo purpose hurried thenceMe and thy crying self

(i) In the earlier lines of this scene what does Prospero tell about his intense interest What was the demand of his interest

In the earlier lines of this scene Prospero tells Miranda that he had an intense interest in the study of philosophy and magic arts Hence in order to improve his mind with this kind of study he kept himself isolated from worldly and state affairs His study was dearer to him than the applause and esteem that he could win from the public His study demanded too much solitude(ii) What forced Antonio to take an undue advantage over Prospero

Prosperorsquos indifferent attitude towards the statersquos affairs and his having boundless trust in Antonio gave rise to a boundless lust for power in Antoniorsquos mind Antonio felt that he must be the actual Duke instead of the part of the Duke he played Thus Antonio took an undue advantage of the situation to usurp Prosperorsquos dukedom(iii) Explain the following lines ldquoI should sin to think but nobly of my grandmother Good wombs have borne bad sonsrdquo

After hearing the treacherous act of her uncle Antonio Miranda says that Prosperorsquos mother was a noble lady and she cannot dishonour her memory by saying that the person named Antonio cannot be his (Prosperorsquos) brother She says that in honour of her grandmother she also cannot say that Antonio must have been begotten not by her grandfather but by some other man She finally concedes that it is known that good mothers have borne bad sons in their wombs and gave birth to them

(iv) Why did the King of Naples accept Antoniorsquos request to help him in usurping his dukedom What did Antonio propose to Alonso

Alonso who was the king of Naples accepted Antoniorsquos request in usurping Prosperorsquos dukedom because he (Alonso) was a sworn enemy of Prospero Antonio proposed that Alonso should immediately drive him (Prospero) and his offspring out of Milan and should confer the dukedom upon him (Antonio) with all the dignities which go with that

position In return Antonio promised that he shall give an annual tribute and also swore his allegiance to Alonso Also he agreed to hold the Dukedom of Milan as a subordinate to the state of Naples(v) How were Prospero and Miranda carried away from the city of Milan and what was the state of small Miranda at that time

In pursuance of the agreement settled between Antonio and Alonso an army of treacherous men was assembled One midnight when the occasion suited the will of destiny Antonio opened the gates of the city of Milan and in the death like silence of midnight Antoniorsquos agents who had been directed to execute his purpose carried Prospero and small Miranda away from the city in all haste They were then forced into a ship and carried some distance out to sea where they put them on a mere hulk of a boat without any rigging or ship-gear and abandoned them leaving them at the mercy of the roaring sea Miranda was a very small child of three years age and she was crying at that time

CLASS -XIDATE-270420Subject Topic Summary Execution

EVS Chapter 1 ndash Mode of Existence

Impact of mode of existence on resources

Q) Why resources are under pressure

Ans - Increase in the sophistication

of technology enabling natural resources to be extracted quickly and efficiently Eg in the past it could take long hours just to cut down one tree only using saws Due to increased technology rates of deforestation have greatly increased

The number of humans is increasing Cultures of consumerism Materialistic views

lead to the mining of gold and diamonds to produce jewelry unnecessary commodities for human life or advancement Consumerism also leads to extraction of resources for the

production of commodities necessary for human life but in amounts excessive of what is needed because people consume more than is necessary or waste what they have

Lack of awareness among the population is striking People are not aware of ways to reduce depletion and exploitation of materials

Accounts Cash Book Today we are going to start a new topic -Cash Book

The key terms used in this chapter are

bullCash book

bullSimple cash book

bullDouble column cash book bullTriple column cash book

bullPetty cash book

bullCash discount

bullContra entry

Here I will share you the meaning of each key terms

bullCash book Cash Book is a special purpose subsidiary book or journal in which cash received and cash payments are recorded

bullSimple cash book

It is a cash book in which only cash transactions are recorded It has only one column on each side

bullTriple column cash book

It is cash book which has three columns one column for each cash and Bankdiscount on each side of the cash book In this book both cash and Bank transactions are recorded together with discount allowed and received

bullPetty cash book

It is a cash book maintained for recording petty expenses

bullCash discount

Cash discount is the amount of discount received or allowed on cash payments and cash receipts Discount received is an income for the business while discount allowed isan expense

bullContra entry

It means transactions involving both cash and Bank Such transactions though recorded in the cash book are not posted into ledger The letter lsquoC is written in Ledger folio for contra entry

Business Studies

ENTREPRENEURSHIP

Now we shall discuss the second chapter

lsquoENTREPRENEURSHIPrsquo

Today before starting the chapter let us recall what

Questions

1What are the main characteristics of Intrapreneurship

Answer

The main characteristics of Intrapreneurship are

Corporate framework-it occurs within the framework of the same company

Semi-Autonomous-Intrapreneurship

we have read last day

Let s today start the class by recalling the last topic taught

Intrapreneurship is the process of discovering and exploring business opportunities within an existing company It involves launching new business ventures within the framework of a present corporation Intrapreneurship is also known as corporate entrepreneurship or corporate venturing

Now let us start with the characteristics of Intrapreneurship

The main characteristics of Intrapreneurship are

Corporate framework

Semi-Autonomous Lack of ownership Senior position Low risk taking Not own boss

Now let us discuss the meaning of enterprise

Enterprise means an undertaking or adventure that requires some innovation and investment and thus involves riskEnterprise always entails decision making coordination and risk bearing

involves crating amd nurturing a semi-autonomous business unit which may be a subsidiary a strategic business unit or a division

Lack of ownership-the intrapreneur is not the owner of the unitb he creates and nurtures

Senior position-he occupies a senior managerial position in the company

Low risk taking-An intrapreneur does not bear the full risk of failure

Not own boss-An intrapreneur is not his own bosss in legal termsHe enjoys the freedom and gets the required resources and support

2 How is Entrepreneur is different from Intrapreneur

The functions involved in both the entrepreneurship and intrapreneurship are by and large similar however there are several differences between the two

Point of distinction

Entrepreneur

Intrapreneur

status An independent business person

A senior executive within a company

Ownership Owner of

the business

An employeesometimes a share in ownership

Financing Responsible for raising finance for the business

Not responsible for raising the finance

Risk bearing

Bears the risk of the business

Does not bears the risk of the business

Reward Profit which is uncertain and irregularcan be loss

Fixed salary and fringe benefits

Need for security low high

3 What do you understand by enterprise

Answer Enterprise means an undertaking or adventure that requires some innovation and investment and thus involves riskEnterprise always entails decision making coordination and risk bearing

COMMERCE NATURE AND OBJECTIVES OF

BUSINESS

Today let us recall the last other two objectives of business by the chart given in the previous class

Firstly we would discuss Human Objectives

Business is run by people and for people Labour is a valuable business element

Human objectives of business are concerned with the well -being of labour

The human objectives are as follows

Labour welfare Developing human

resources Participative

management Labour

management cooperation

Questions

1 Explain the human objectives of a business enterprise

Answer

Business is run by people and for people Labour is a valuable business element

Human objectives of business are concerned with the well -being of labour

The human objectives are as follows

Labour welfare-Business must recognize the dignity of labour and human factors should be given the recognition

Developing human resources-Employees must be provided the opportunities for developing new skills and attitudes

Participative management-Employees should be allowed to take part in decision making process of business

Labour management cooperation-Business should strive for creating and maintaining cordial employer employee relations so as to ensure peace and progress in industry

Now let us discuss the national objectives of business

Optimum utilization of resources

National self- reliance Development of small

scale industries Development of

backward areas Control over pollution

2Explain the national objectives of a business enterprise

Answer

It is the duty of business to utilize the resources of the country properly the national objectives of business

Optimum utilization of resources ndashBusiness should use the nationrsquos resources in the best possible manner

National self- reliance-It is the duty of the business to help the government in increasing experts and in reducing dependence on imports

Development of small scale industries-Big business firms are expected to encourage growth of small scale industries which are necessary for generating employment

Development of backward areas-Business is expected to give preference to the industrialization of backward regions of the country

ECONOMICS

BASIC ECONOMIC CONCEPTS

SUB

TOPIC

Value

Wealth

Welfare

Today we shall start with a new topic of the same chapter ie lsquoValuersquo

Value of a commodity is defined as the valuation placed by a household on the consumption of this commodity

lsquoValuersquo has two different meanings and these are

a Value -in -use It refers to consumption value of a commodity It expresses the utility derived from the consumption of a particular commodity A necessity like water has a very high value ndashin ndashuse or

Question

1What is value

Answer

Value of a commodity is defined as the valuation placed by a household on the consumption of this commodity

2What is value-in use

Answer It refers to consumption value of a commodity It expresses the utility derived from the consumption of a particular commodity A necessity like water has a very high value ndashin ndashuse or consumption value

3What is value ndashin- exchange

Answer It relates to market value of a commodity

It is the rate at which a particular good or service can be exchanged for money

For example in barter system if a person is prepared to exchange 3 metres of cloth with 1 pair

consumption value

b Value ndashin-exchange It relates to market value of a commodity

It is the rate at which a particular good or service can be exchanged for moneyFor example in barter system if a person is prepared to exchange 3 metres of cloth with 1 pair of shoes then the value in exchange of 3 metres of cloth is 1 pair of shoesValue in exchange is the power of purchasing other goods In modern monetised economies the exchange value of goods are expressed in terms of money as prices

Now let us discuss the term lsquoWealthrsquo

Wealth refers to the stock of all those assets which are a source of income

Wealth is a stock concept

Wealth must possess the following features

a Utility It must possess utility or give some

of shoes then the value in exchange of 3 metres of cloth is 1 pair of shoes

Value in exchange is the power of purchasing other goods In modern monetised economies the exchange value of goods are expressed in terms of money as prices

4 What is wealth

Answer it refers to the stock of assets or goods which are a source of income and have personal or national ownership

5 What are the features of wealth

Answer The features of wealth are as follows

Wealth must possess the following features

a Utility It must possess utility or give some satisfaction

b Scarcity It must be limited in quantityc Transferability it should be transferable its

ownership can be transferred from one person to another person

d Exchange value It must possess exchange value

6 What is welfare

Answer

Welfare is defined as satisfaction and happiness a sense of well- being among the people

satisfactionb Scarcity It must be

limited in quantityc Transferability It

should be transferable its ownership can be transferred from one person to another person

d Exchange value It must possess exchange value

Now let us discuss the term lsquoWelfarersquo

Welfare is defined as satisfaction and happiness a sense of well- being among the people

Welfare is affected by factors like

a Consumption of goods and services

b Environment

c Family relations

d Degree of freedom

e Law and order situation

Mathematics Trigonometric equation

To find the general solution of the equation sinθ=0

When sin θ =0

Then θ= 0 π2π 3π-π -2π -3

i e when θ = 0 or an integral multiple of π

i e when θ= nπ where n is any integer

Therefore the general solution of the equation sin

Example1 Find the general values of θ which satisfy the equation sin2 θ =34

Solution sin2 θ= 34

Or sin θ = +34 or -34

Or sin θ = sin π3 or sin (-π3)

Therefore

θ = [nπ + (-1) n (π3)] or[ nπ+ (-1) n (-π3)]

= nπ +π3 or nπ-π3 where n= any integer

Example 2Find the values of θ which satisfy tan2 θ

θ=0 is θ= nπ where n is any integer

To find the general solution of the equation cos θ=0

When cos θ=0

Then θ=π2 3π2 5π2 -π2 -3π2 -5π3

i e when θ is an odd multiple of π2

i e when θ=(2n+1) π2 where n= any integer

Therefore the general solution of the equation cos θ =0 is θ= (2n+1) π2 where n= any integer

To find the general solution of the equation tan θ = 0

Clearly tan θ =0 implies sin θcos θ =0

Therefore θ = nπ

i e the general solution of the equation tan θ=0 is θ =nπ where n = any integer

To find the general solution of the equation cot θ =0

Clearly cot θ =0 implies (cos θsin θ) = 0

i e cos θ =0

Therefore θ = (2n+1) π2

Therefore the general solution of the equation cot θ =0 is θ = (2n+1) π2

Where n= any integer

To find the general solution of the equation sin θ= k (-

=13 -πleθleπ

Solution tan2 θ =13

Or tan θ = plusmn1radic(3) =tan(plusmnπ6)

θ=nπ plusmn π6 where n =any integer

If n=0 then θ=plusmnπ6

If n=1 then θ= π plusmn π6

If n=-1 then θ= -π plusmn π6

Therefore the required solution in -π le θ le π are θ= π6 5π6 -π6 -5π6

Exercise Find general solution of sin 2θ=cos θ [Hints Use sin 2θ= 2sin θcosθ and then take cosθ

common]

1lek le1)

Determine an angle alpha such that sin =k and -π2le αle π2

Then we have

Sin θ = k = sin α

Or sin θ - sin α =0

Or 2 cos [(θ+α) 2] sin [(θ-α) 2] =0

Therefore either cos [(θ +α) 2] =0 (1)

Or sin [(θ-α) 2] =0 (2)

Now from (1) we get (θ+α) 2= (2m+1) π2)

Or θ = (2m+1) π-α (3)

And from (2) we get (θ-α) 2 =mπ

Or θ= 2mπ+α(4)

Where m = any integer

Clearly the solution (3) amp (4) may be combined in the following form

θ= nπ+(-1) n α where n= any integer

Therefore the general solution of sin θ = sin α is θ = nπ +(-1) n α where n is any integer and -π2 le α le π2

Biology Chapter - 04Kingdom Monera

Today we will discuss about bacterial reproduction and its usefulness

Fig Binary Fission

Fig Conjugation Fig Transformation

Bacterial reproduction is mainly asexual but sexual reproduction

also takes place

Asexual reproduction takes place by i) Binary fission - from one bacteriato

two bacteria are produced in every 20 to 30mins

ii) Buddingiii) Endospore formation - during

unfourable condition

Sexual reproduction by three ways

1) Conjugation - Transfer of genetic material between cells that are in physical contact with one another

2) Transduction - Transfer of genetic materialfrom one cell to another by a bacteriophage

3) Transformation - Transfer of cell-freeor naked DNArsquo from one cell to another

Bacteria causes different diseases inplants animals and human and

it causes food spoilage and waterpollution but it also have some useful

activities

i) Bacteria are helpful in sewage water treatment

ii) It is used in antibiotic (medicine) production

iii) Anaerobic bacteria help in biogas(energy) production

iv) Many household products like yoghurt cheese are manufactured by use of bacteria

v) Rhizobium by symbiotic relationship with leguminous plant increase soil fertility

vi) Besides these bacteria is helpful in genetic engineering degradation of petroleum hydrocarbonand in dairy

industry

Physics Motion in plane Here we will introduce Projectile Motion

Execution

Projectile

Y

usinθ u h

θX

ucosθ

Suppose a body is projected with an angle θ So initial velocity u can be resolved into two components

Horizontal component - ucosθ ( for range)

Vertical component - usinθ ( for height)

usinθ changes during motion and becomes zero at maximum height position but ucosθ remain unchanged

The maximum height of projectile is h

NB If initial is upward then g = -ve and if it is downward then g = +ve Height is +ve if direction of motion does not change ( for ex a body thrown upwards but goes down ultimately then height h = -ve)

The angle of projectile θ is the angle made with horizontal

HISTORY ndash GROWTH OF NATIONALISM

SUB TOPIC- REVOLUTIONARY NATIONALISM Bengal formation of Anushilan Samity and Jugantar Group

The intensification of the Swadeshi movement and Government policy of terror and repression led to outbreak of violence Bombs were manufactured and attempts on the lives of unpopular Government officials became frequent In the gymnasium of Scottish Church College which was known as General Assemblies Institution a secret society was formed known as Anushilan Samity

Aurobindo Ghosh send from Baroda his emissary Jatindranath Banerjee to mobilize the Bengal revolutionaries

Hemchandra Qanungo and Satyen Bose published Journal Jugantar

The Jugantar group planned to assassinate oppressive magistrate Kingsford by Khudiram Bose and Prafulla Chaki in 1908 Prafulla Chaki committed suicide to avoid arrest Khudiram was tried and hanged

Afew days later the police found a bomb factory in Maniktala and arrested a large number of revolutionaries The trial of revolutionaries became famous as the Alipore Bomb Case

In the course of the trial the approver the public prosecuter and a police officer were assassinated

1 Question Name two journals which preached the cult of violence

Answer a) Yugantar edited by Bhupendranath DuttaB) Bandemataram edited by Aurobindo Ghosh2 Question Why was Khudiram arrested and hangedAnswer An attempt was made to assassinate a hated vindictive majistrate named Kingsford by Khudiram Bose and Prafulla Chaki Their attempt failed and the bomb they threw killed two English ladies Khudiram was arrested and put to trial and then hanged3Question Who was Aurubindo GhoshAnswer Aurobindo Ghosh a nationalist revolutionary who was charged for his involvement in the Alipore Bombing Case He was accused of it along with his brother Barindra nath Ghosh But Aurobindo was acquitted because of the brilliant pleading of his counsel Chittaranjan Das Then he became a spiritual reformer introducing his visions on human progress and spiritual evolution4 Qustion Who was KingsfordAnswer Kingsford was an unpopular British chief Magistrate who was the target of the bomb thrown at Muzaffarpur by Khudiram and Prafulla Chaki

Most of the accused were convicted and sentenced to

long term of imprisonmentBut

Aurobindo Ghosh was acquitted mainly owing to the brilliant pleading of his counsel Chittaranjan Das

Political science Topic-Sovereignty

Summary Sovereignty is the full right and power of a governing body over itself without any interference from outside sources or bodies In political theory sovereignty is a substantive term designating supreme legitimate authority over some polity In international law sovereignty is the exercise of power by a state

Internal Sovereignty

Internal sovereignty means supreme authority within ones territory while external sovereignty relates to the recognition on the part of all states that each possesses this power in equal measure

External sovereignty

external sovereignty relates to the recognition on the part of all states that each possesses this power in equal measure

Distinguish between

Execution

Answer the following questions

Short notes-

Sovereignty

Internal Sovereignty

External sovereignty

Homework- learn

external sovereignty and internal sovereigntySovereignty is the principle

of supreme and

unquestionable authority

reflected in the claim by the

state to be the sole author of

laws within its territory

Definition of external vs internal sovereigntyInternal sovereignty refers to

the relationship between a

sovereign power and its

subjects ndash it refers to the

location of the supreme

authority within the state In

the UK for example internal

sovereignty (supposedly)

resides within Parliament

reflected in the

constitutional principle of

parliamentary

sovereigntyBy contrast

external sovereignty refers

to the capacity of the state

to act independently and

autonomously on the world

stage This is what is

sometimes called lsquostate

sovereigntyrsquo or lsquonational

sovereigntyrsquo and implies

that states are legally equal

and that the territorial

integrity and political

independence of a state is

inviolable

Class ndash XII

Date - 2742020 STUDY MATERIALSubject Topic Summary Execution Business Studies

Job Analysis amp Manpower Planning

At first let us recall the chapter what we have discussed till nowbullJob analysisbullJob specification bullJob description bullJob enlargement bullJob enrichment

Today we will do some questions answers from the chapter

Questions 1ldquoJob analysis job description and job specification are interrelatedrdquo Comment Answer) Job analysis is a systematic and detailed examination of a job to collect all the relevant information about it The contents off the job are summarised in the job description The qualification needed for the job are summarised in job specificationThus there is close interrelationship between job analysis job description and job specification

Question 2ldquoJob enlargement is a horizontal extension of a job whereas job enrichment is a vertical extension of a jobrdquo ElucidateAnswer) Job enlargement involves adding one or more task to a job coma where as job enrichment involves adding more autonomy and responsibility to a job Job enlargement is therefore horizontal extension of a job coma whereas job enrichment is a vertical extension of a job

Question 3 )

What is manpower estimation Explain its quantitative and qualitative aspectsAnswer) Manpower estimation is the process by which management determines how an organisation should move from itrsquos current manpower positionto its desired manpower position There are two dimensions of Manpower estimation- quantitative and qualitative

Quantitative aspectThis aspect of Manpower estimation involves estimating the number of employees required in a future time period Workload analysis and workforce analysis are done to estimate the quantity of required manpower

Qualitative aspectThe estimate of the knowledge skills experience etc of required manpower is the qualitative aspect of Manpower estimation The quality of Manpower can be judged on the basis of job analysisand job specification

COMMERCE

CAPITAL-FIXED AND WORKING

Today let us start the class by discussing the sources of finance for different types of business firms

The term lsquocapitalrsquo refers to the investment made in the enterprise for the purpose of earning profits

Requirements of capital and sources of capital for different types of business firms are

1 Capital for sole proprietorship businessA sole proprietor operates at a small scale and thereforerequires a limited amount of capital

2 Capital for partnership firmCapital requirements as well as capital base of a partnership is bigger than that of a sole trader businessThe owned capital is contributed by the partners in an agreed ratio

3 Capital for joint stock companyA joint stock company generally requires large amount of capitalA public company can raise huge capital through issue of shares In addition to share capital it can utilize retained profits

Now let u discuss the meaning of Finance PlanningFinance planning is the process of estimation the financial requirements of an organization specifying the sources of firms and ensuring that enough funds are available at the right time

1 What do you mean by Finance PlanningAnswer Finance planning is the process of estimation the financial requirements of an organization specifying the sources of firms and ensuring that enough funds are available at the right time

2Discuss the role of financial planning of an enterpriseThe role of financial planning are as followsa A sound financial plan helps a business enterprise to avaid the problems of shortage and surplus of fundsbFinancial planning serves as a guide in developing a sound capital structure so as to maximize returns to shareholders c It helps in effective utilization of fundsd It provides policies and procedures for coordinating different functional areas or departments of businesse It enables the management to exercise effective control over the financial activities of an enterprisef It helps the company to prepare for facing business shocks and surprises in future

Mathematics

Continuity and differentiability

Recall Definition of ContinuityLet f(x) be a single valued function of x and x=a be a point in the domain of definition of the function The function is said to be continuous at x=a ifi) f(c) is defined ie f(x) has a definite finite value at x=cii) lim xrarra f(x) exists andiii) lim xrarra f(x) =f(a) In other words f(x) is said to be continuous at x=a if lim xrarra+ f(x)= lim xrarra- f(x) = f(a) Or f(a+0) =f(a-0) =f(a) Or lim hrarr0 f(a+h)= f(a) Algebra of continuous functionsNow we will study some algebra of continuous functions Theorem 1 Suppose f and g be two real functions continuous at a real number c Then(1) f + g is continuous at x = c(2) f ndash g is continuous at x = c(3) f g is continuous at x = c(4) (fg) is continuous at x = c (provided g (c) ne 0)

Example 1 Prove that every rational function is continuousSolution Recall that every rational function f is given byf(x)=[p(x) q(x) ] q(x)ne0where p and q are polynomial functions The domain of f is all real numbers except points at which q is zero Since polynomial functions are continuous f is continuous by (4) of Theorem 1Example 2Discuss the continuity of sine functionSolution To see this we use the following factslim xrarr0 sin x =0Now observe that f (x) = sin x is defined for every real number Let c be a real number Put x = c + h If x rarr c we know that h rarr 0 Therefore lim xrarrc f(x)

= lim xrarrc sin x= lim hrarr0 sin(c+h) =lim hrarr0 [sin c cos h + cos c sin h ]=lim hrarr0 (sin c cos h) + lim hrarr0 (cos c sin h) = sin c +0=sin c = f(c) Thus lim xrarrc f(x) = f(c) and hence f is a constant function Exercise Prove that the function f(x) = x2 +2x is continuous for every real value of x [Hints show that lim xrarra+ f(x) = lim xrarra- f(x) = f(a) ]

Biology Reproduction in Flowering plants We will discuss about megasporoangium

megasporagenesis and female gametophyte

Q4 Describe the structure of megasporangium

Ovule is attached to the placenta by astalk called funicle

Each ovule has one two or three protectivecoverings called integuments

At the tip of integuments a small openingcalled micropyle is organised

Opposite to the micropylar end is the chalaza

Within the integuments a mass of cellsnucellusand inside it embryo sac orfemale gametophyte is present

Q5 Describe a mature embryosacamp its formation

In most of the flowering plants only oneof the 4 megaspores formed as a result ofmegasporogenesis that is functional while theother three degenerate

The

functional megaspore develops into thefemale gametophyte

Formation The nucleus

of the functional megasporedivides mitotically to form two nuclei first andthen two more sequential mitotic nucleardivisions result in the formation of four ampthen eight nucleate stages of embryo sac

Six of the eight nucleus are surrounded bycell walls and organised into cells

The remaining two nuclei called polar nuclei are found below the egg apparatus in the largecentral cell

Three cells consisting of two synergids amp one egg cell present bottom of

embryo sac Three cells

at the chalazal as antipodal cells

Two polar nuclei together present in large central cell

HISTORY

TOWARDS INDEPENDENCE AND PARTITION THE LAST PHASE(1935-1947)SUB TOPIC NATIONAL MOVEMENTS DURING THE SECOND WORLD WAR

Spread of Quit India Movement On 9th August 1942Gandhiji and other Congress leaders were arrested The Congress was declared illegal The news of the arrest of all leaders marked the beginning of a widespread movement of India It was not possible for such a movement to remain peacefulBut the arrest of the all notable congress leaders virtually left the movement in the hands of the mass The movement took the form of violent and militant outbreakBesides congressmen revolutionaries also were very active in the movement The Congress Socialist group also played a prominent role

1 Question Why did the British authority arrest the Congress leaders on 9 th August 1942Answer Congress Working committee adopted the Quit India resolution which was to be ratified at the Bombay AICC meeting in 8th August 1942 They decided to launch a mass struggle on non-violent lines Gandhiji gave a clarion call to all section of the people rdquoKarenge ya Marengerdquo (do or die) Congress leaders gave the call to driving out

the British from IndiaViceroy had taken strong action against the Quit India movement Gandhiji and all the leaders of Congress were arrested

2 Question How did Quit India Movement spread out all over IndiaAnswer The news of the leaders lsquo arrest marked the beginning of a widespread movement to remain peacefulThe movement took form of violent outbreak There were widespread cutting of telephone and Telegraph wires damaging railway lines raising barricades in cities and towns and other forms of violent demonstations

Question Name the leaders of Congress

Socialist group played a prominent part Notable among the Jayprakash Narayan Rammonohar Lohia Aruna Asaf Ali

Political science

Topic-Franchise and Representation

Summary

The election commission

The Election Commission of India is an autonomous constitutional authority responsible for administering Union and State election processes in India The body administers elections to the Lok Sabha Rajya Sabha State Legislative Assemblies in India and the offices of the President and Vice President in the country

Functions of election commission-

India is a sovereign socialist secular democratic republic Democracy runs like a golden thread in the social economic and political fabric woven by the Constitution given by lsquoWe the People of Indiarsquo unto ourselves The concept of democracy as visualised by the Constitution pre-supposes the representation of the people in Parliament and State legislatures by the method of election The Supreme Court has held that democracy is one of the inalienable basic features of the Constitution of India and forms part of its basic structure The Constitution of India adopted a Parliamentary form of government Parliament consists of the President of India and the two Houses mdash Rajya Sabha and Lok Sabha India being a Union of states has separate state legislatures for each state State legislatures consist of the Governor and two Houses mdash Legislative Council and Legislative Assembly mdash in seven states namely Andhra Pradesh Telangana Bihar Jammu amp Kashmir Karnataka Maharashtra and Uttar Pradesh and of the Governor and the state Legislative Assembly in the remaining 22 states Apart from the above two out of the seven Union Territories namely National Capital Territory of Delhi and Puducherry also have their Legislative Assemblies

ExecutionShort notes-Election commissionFunctions of election commission

Homework- Learn

Computer

Science

Computer hardware NAND Gate

A NOT-AND operation is known as NAND operation It has n input (n gt= 2) and one output

Logic diagram

Truth Table

NOR Gate

A NOT-OR operation is known as NOR operation It has n input (n gt= 2) and one output

Logic diagram

Truth Table

XOR Gate

XOR or Ex-OR gate is a special type of gate It can be used in the half

adder full adder and subtractor The exclusive-OR gate is abbreviated as EX-OR gate or sometime as X-OR gate It has n input (n gt= 2) and one output

Logic diagram

Truth Table

XNOR Gate

XNOR gate is a special type of gate It can be used in the half adder full adder and subtractor The exclusive-NOR gate is abbreviated as EX-NOR gate or sometime as X-NOR gate It has n input (n gt= 2) and one output

Logic diagram

Truth Table

Physics

Chapter 1 Electric Field ( Electric Dipole) (Summary)

Here we will derive Expression of electric field at broad side

On position of dipole

Execution

Q With the help of a labelled diagram obtain an expression for the electric field intensity E at any point on the equitorial line ( broad-side on position) of an electric dipole

Ans

E1 E1sinθ

E θ P E1 θ

( r2+L2)12 E2 E

r E2 E2sinθ

-q θ L O L +qA B

Let us consider that the point P is situated on the right bisector of the dipole AB at a distance r meter from its midpoint O

Let E1 and E2 be the electric field intensities of the electric field at P due to charge +q and ndashq of the dipole resp The distance of P from each charge is ( r2+L2)12

So E1 = 14 πϵ q

(r 2+L 2) away from +q

E2 = 14 πϵ q

(r 2+L 2) towards ndashq

The magnitudes of E1 and E2 are equal but directions are different Now resolving E1 and E2 into two components parallel and perpendicular to AB we get

The components perpendicular to AB E1sinθ and E2sinθ cancel each other because they are equal and opposite

The components parallel to AB are E1cosθ and E2 cosθ are in same direction and add up

So resultant intensity of electric field at the point P is

E = E1cosθ + E2 cosθ

E = 14 πϵ q

(r 2+L 2) 2 cosθ

Now from fig we have cosθ =BOBP = L (r2+L2)12

So we get E = 14 πϵ 2qL ( r2+L2)32

Now electric dipole moment p= 2qL

So E = 14 πϵ p ( r2+L2)32

HW Find the expression of Electric field as done here but this time take r gtgt 2L

Also find the expression of torque experience by a dipole

(Hint Electric force experienced by charges of dipole in electric field is qE each Let θ be the angle which dipole makes with electric lines of force then perpendicular distance between two charges is 2Lsinθ Then torque = force x perp distance = qE x 2L sinθ So τ=pE sinθ where p =2qL )

STUDY MATERIAL

Class XIISubject Eng Literature (The Tempest ndash William Shakespeare) Topic Act IV Scene 1 Lines 84 to 133 (Iris hellip A contract of true love Be not too late ) Date 27th April 2020 (4th Period)

[Students should read the original play and also the paraphrase given in the school prescribed textbook]Summary Questions amp Answers

o Ceres soon appears and comes to know that she has been summoned to celebrate the contract of true love

o Ceres expresses her unwillingness to meet Venus and Cupid as she has shunned their company

o Ceres and Juno both bestow their blessings upon Ferdinand and Miranda with June gifting honour riches happiness in marriage and Ceres presents plenty of earthrsquos produce

o Iris summons the water-nymphs and reapers to come and celebrate a contract

(1) IRIS Of her society (Line 91-101)

Be not afraid I met her deity

Cutting the clouds towards Pathos and her sonDove-drawn with her Here thought they to have doneSome wanton charm upon this man and maidWhose vows are that no bed-right shall be paidTill Hymens torch be lightedmdashbut in vainMarss hot minion is returned againHer waspish-headed son has broke his arrowsSwears he will shoot no more but play with sparrowsAnd be a boy right out

(i) Where were Venus and Cupid seen flying How were they travelling Why did they want to join the marriage celebration of Ferdinand and Miranda

of true love

Venus and Cupid were seen flying through the air towards Paphos the famous city which is situated on the island of Cyprus They were travelling by air-borne chariot drawn by doves They certainly wanted to come here in order to play some amorous trick upon Ferdinand and Miranda who are under a vow not to gratify their physical desires till the holy ceremony of their marriage has been performed(ii) What have Venus and Cupid done after failing in their plan

After being failure of their plan Venus who is a very passionate deity and who is the mistress of Mars (the god of war) has gone back while here ill-tempered son Cupid has broken his arrows of love in his state of desperation(iii) What has Cupid firmly decided

Cupid is feeling so disappointed that he has firmly decided to shoot no more arrows to arouse love in human hearts but to spend his time playing with sparrows Thus he would now become just a boy and would give up his original function of shooting arrows on human beings to make them fall in love(iv) What vow had Ceres taken How did Ceres feel at the abduction

After the abduction of her daughter Prosperina by Pluto Ceres had taken a vow to always keep away from the disgraceful company of Venus and her blind son Cupid the god of love Ceres felt deeply distressed when Pluto had carried off her daughter and had made her his wife by force(v) Why has Ceres not forgiven Venus and her blind son For what do Ceres want to be sure

As the abduction had been manipulated by Venus the goddess of beauty and love and her blind son Cupid Ceres has never forgiven them for their part in the whole plot Ceres wants to be sure that she would not have to meet Venus and Cupid who had engineered the abduction of her daughter Prosperina

AS THIS lsquoMASQUErsquo SCENE IS VERY IMPORTANT IN THE PLAY THE PARAPHRASE OF THE ENTIRE PORTION OF MASQUE SCENE (Act IV Lines 58 to 143) IS GIVEN BELOW

IRIS Goddess of RainbowCERES Goddess of Agriculture and all the fruits of the earth

(Nature growth prosperity rebirth ndash notions intimately connected to marriage)JUNO The majestic Queen of Heavens and wife of Jupiter (Jupiter is the king of Gods)

VENUS The Goddess of love CUPID Son of Venus PLUTO God of death (In the play referred by Shakespeare as lsquoDisrsquo which is a Roman name for Pluto)

ORIGINAL TEXT PARAPHRASEPROSPEROWellmdash

PROSPERONow come Ariel Let there be too many rather than too few

Now come my Ariel Bring a corollaryRather than want a spirit Appear and pertly[to Ferdinand and Miranda]No tongue all eyes Be silent

spirits in attendance Appear briskly

[to Ferdinand and Miranda]Look with your eyes but do not say a word

[Soft music] [Soft music][Enter Iris] [Enter Iris]

IRISCeres most bounteous lady thy rich leasOf wheat rye barley vetches oats and peasThy turfy mountains where live nibbling sheepAnd flat meads thatched with stover them to keepThy banks with pioned and twilled brimsWhich spongy April at thy hest betrimsTo make cold nymphs chaste crowns and thybroom-grovesWhose shadow the dismissegraved bachelor lovesBeing lass-lorn thy pole clipped vineyardAnd thy sea-marge sterile and rocky-hardWhere thou thyself dost airmdashthe Queen othrsquoSkyWhose watery arch and messenger am IBids thee leave these and with her sovereign grace[Juno appears] Here on this grass-plot in this very placeTo come and sport Her peacocks fly amainApproach rich Ceres her to entertain

IRISCeres most generous lady you are the cause of rich fields or fertile land where wheat rye barley beans oats and peas grow the grassy mountains where the sheep graze and the flat meadows covered with coarse hay to be used as fodder for cattleYour banks are covered with marsh-marigolds and reeds and the rainy April under your orders brings forth to make for the maids who are not in love beautiful crowns your woods where the broom flourishes and where the bachelor who has been dismissed by the maid he loved lies down being forsaken your vineyard in which the poles are embraced by the vines and the margin of the sea which is barren and rocky where you roam about to enjoy the fresh air ndash the queen of the sky (Juno) whose messenger I am besides being represented as the rainbow bids you leave all these and with her majesty here on this grassy plot in this very place come and sport her peacocks carry her fast in her chariot through the air and are making their way here approach rich Ceres to welcome her

[Enter Ariel as Ceres] [Enter Ariel as Ceres]

CERESHail many-coloured messenger that neerDost disobey the wife of JupiterWho with thy saffron wings upon my flowersDiffusest honey-drops refreshing showersAnd with each end of thy blue bow dost crownMy bosky acres and my unshrubbed downRich scarf to my proud earth Why hath thy queenSummoned me hither to this short-grassed green

CERESWelcome rainbow that never dared disobey Juno the wife of Jupiter who with your orange coloured rays spread honey-drops refreshing showers And with each end of thy blue bow drown my bushy acres and my hilly country which is free from shrubs you thus forming a rich scarf Why has your queen called me here to this place covered with short grass

IRISA contract of true love to celebrateAnd some donation freely to estateOn the blest lovers

IRISI have called you to celebrate a contract of true love and bestow some liberal gift upon the blessed lovers

ORIGINAL TEXT PARAPHRASECERESTell me heavenly bowIf Venus or her son as thou dost knowDo now attend the queen Since they did plotThe means that dusky Dis my daughter gotHer and her blind boys scandaled companyI have forsworn

CERESTell me heavenly bow if Venus the Goddess of love or Cupid her son and pedlar of passion at this time attend the heavenly queen Juno because you are sure to know Since the day they conspired against me and dark Pluto took away my daughter here and Cupidrsquos disgraceful company I have left off

IRISOf her societyBe not afraid I met her deityCutting the clouds towards Pathos and her sonDove-drawn with her Here thought they to have doneSome wanton charm upon this man and miad

IRISBe not afraid of her company I met her deity moving on the clouds towards Paphos the sacred home of Venus on the island of Cyprus along with her son on her chariot drawn by doves Here they contemplated to exercise a charm upon this man and maid producing

Whose vows are that no bed-right shall be paidTill Hymens torch be lightedmdashbut in vainMarss hot minion is returned againHer waspish-headed son has broke his arrowsSwears he will shoot no more but play with sparrowsAnd be a boy right out

wantonness before the actual marriage ceremony but did not succeed Venus has returned her irritable son has broken his arrows and swears that he will give up his practice of trying to inspire love but play with sparrows and be a boy again

[Music is heard] [Music is heard]

CERESHighst queen of stateGreat Juno comes I know her by her gait

CERESHighest queen of state Great Juno there she comes I know here by her gait

[Enter Juno] [Enter Juno]

JUNOHow does my bounteous sister Go with meTo bless this twain that they may Prosperous beAnd honoured in their issue

JUNOHow are you doing my generous sister Come with me to bless this couple so that they may be prosperous and fortunate in their children

[They sing] [They sing]

JUNOHonour riches marriage-blessingLong continuance and increasingHourly joys be still upon youJuno sings her blessings upon you

JUNOMay honour riches happiness in marriage long continuance and increase of those boons ever rest upon you as hourly joys Juno showers down upon you her blessings in song

CERESEarths increase foison plentyBarns and garners never emptyVines and clustring bunches growingPlants and goodly burden bowingSpring come to you at the farthestIn the very end of harvestScarcity and want shall shun youCeresrsquo blessing so is on you

CERESMay you have the plenty of earthrsquos produce Your barns and granaries may never be empty Your vines may grow with clustering bunches Your fruit trees may be heavily laden with their fruit May there be continuous spring and harvest May scantiness and want leave you forever Such is the blessing of Ceres upon you

FERDINANDThis is a most majestic vision andHarmoniously charmingly May I be boldTo think these spirits

FERDINANDThis is a great vision and magically melodious Should I suppose the characters (taking part in the masque) are spirits

PROSPEROSpirits which by mine artI have from their confines calld to enactMy present fancies

PROSPEROYes they are spirits whom I have summoned from the regions to which they are confined to carry into effect my fanciful designs

ORIGINAL TEXT PARAPHRASEFERDINANDLet me live here everSo rare a wondered father and a wifeMakes this place paradise

FERDINANDI should like to live here forever Such a wise and wonderful father makes this place a paradise

[Juno and Ceres whisper and send Iris on employment] [Juno and Ceres whisper and send Iris on employment]

PROSPEROSweet now silence

PROSPEROMy dear Ferdinand speak no more Juno and Ceres are

Juno and Ceres whisper seriouslyTheres something else to do Hush and be muteOr else our spell is marred

whispering with a solemn look There is something else coming Silence Or else our magic will be spoilt

IRISYour nymphs called naiads of the wandering brooksWith your sedged crowns and over-harmless looksLeave your crisp channels and on this green landAnswer your summons Juno does commandCome temperate nymphs and help to celebrateA contract of true love Be not too late

IRISYou nymphs called Naiads denizens (M inhabitants) of the running stream with your chaplets of sedge and ever-helpful looks leave your wrinkled channels and on the green land answer the summons sent to you Juno has ordered some chaste nymphs and help to celebrate a noble and true marriage Donrsquot delay

[Enter certain nymphs] [Enter certain nymphs]You sunburnt sicklemen of August wearyCome hither from the furrow and be merryMake holiday your rye-straw hats put onAnd these fresh nymphs encounter every oneIn country footing

You sunburnt harvesters weary from the effects of the heat in August come here from the furrowed land and rejoice Make holiday with your rye-straw hats upon you and meet these fresh nymphs and join in country dancing

[Enter certain reapers properly habited They join with the nymphs in a graceful dance towards the end whereof Prospero starts suddenly and speaks]

[Enter certain reapers properly habited They join with the nymphs in a graceful dance towards the end whereof Prospero starts suddenly and speaks]

PROSPERO[aside] I had forgot that foul conspiracyOf the beast Caliban and his confederatesAgainst my life The minute of their plotIs almost come [to the spirits]Well done Avoidno more

PROSPERO(Aside)I had forgotten the wicked conspiracy of the beast Caliban and his accomplices against my life the time of their plot has almost arrived ndash (To the Spirits) well done depart no more of this

[To a strange hollow and confused noise the spirits heavily vanish]

[The spirits depart]

ORIGINAL TEXT PARAPHRASEFERDINANDLet me live here everSo rare a wondered father and a wifeMakes this place paradise

FERDINANDI should like to live here forever Such a wise and wonderful father makes this place a paradise

[Juno and Ceres whisper and send Iris on employment] [Juno and Ceres whisper and send Iris on employment]

PROSPEROSweet now silenceJuno and Ceres whisper seriouslyTheres something else to do Hush and be muteOr else our spell is marred

PROSPEROMy dear Ferdinand speak no more Juno and Ceres are whispering with a solemn look There is something else coming Silence Or else our magic will be spoilt

IRISYour nymphs called naiads of the wandering brooksWith your sedged crowns and over-harmless looksLeave your crisp channels and on this green landAnswer your summons Juno does commandCome temperate nymphs and help to celebrateA contract of true love Be not too late

IRISYou nymphs called Naiads denizens (M inhabitants) of the running stream with your chaplets of sedge and ever-helpful looks leave your wrinkled channels and on the green land answer the summons sent to you Juno has ordered some chaste nymphs and help to celebrate a noble and true marriage Donrsquot delay

[Enter certain nymphs] [Enter certain nymphs]You sunburnt sicklemen of August wearyCome hither from the furrow and be merryMake holiday your rye-straw hats put onAnd these fresh nymphs encounter every oneIn country footing

You sunburnt harvesters weary from the effects of the heat in August come here from the furrowed land and rejoice Make holiday with your rye-straw hats upon you and meet these fresh nymphs and join in country dancing

[Enter certain reapers properly habited They join with the nymphs in a graceful dance towards the end whereof Prospero starts suddenly and speaks]

[Enter certain reapers properly habited They join with the nymphs in a graceful dance towards the end whereof Prospero starts suddenly and speaks]

PROSPERO[aside] I had forgot that foul conspiracyOf the beast Caliban and his confederatesAgainst my life The minute of their plotIs almost come [to the spirits]Well done Avoidno more

PROSPERO(Aside)I had forgotten the wicked conspiracy of the beast Caliban and his accomplices against my life the time of their plot has almost arrived ndash (To the Spirits) well done depart no more of this

[To a strange hollow and confused noise the spirits heavily vanish]

[The spirits depart]

Ac-12 27420 topic Revaluation of Assets and Liabilities

REVALUATION OF ASSETS AND LIABILITIES

On admission of a new partner the firm stands reconstituted and consequently the assets are revalued and liabilities are reassessed It is necessary to show the true position of the firm at the time of admission of a new partner If the values of the assets are raised gain will increase the capital of the existing partners Similarly any decrease in the value of assets ie loss will decrease the capital of the existing partners For this purpose alsquoRevaluation Accountrsquo is prepared This account is credited with all increases in the value of assets and decrease in the value of liabilities It is debited with decrease on account of value of assets and increase in the value of liabilities The balance of this account shows a gain or loss on revaluation which is transferred to the existing partnerrsquos capital account in existing profit sharing ratioAccounting for Revaluation of Assets and Liabilities when there is a Changein the Profit Sharing Ratio of Existing PartnersAssets and liabilities of a firm must also be revalued at the time of change in profit sharing ratio of existing partners The reason is that the realisable or actual value of assets and liabilities may be different from those shown in the Balance Sheet It is possible that with the passage of time some of the assets might have appreciated in value while the value of certain other assets might have decreased and no record has been made of such changes in the books of accounts Similarly there may be some unrecorded assets amp libilities that may have to be accounted for Revaluation of assets and reassessments of liabilities becomes necessary because the change in the

value of assets and liabilities belongs to the period to change in profit sharing ratio and hence must be shared by the partners in their old profit sharing ratio Revaluation of assets and reassessment of liabilities may be given effect to in two different ways (a) When revised values are to be recorded in the books and(b) When revised values are not to be recorded in the books

When revised values are to be recorded in the booksIn such a case revaluation of assets and reassessment of liabilities is done with the help of a new account called lsquoRevaluation Accountrsquo Sometimes this account is also called as lsquoProfit amp Loss Adjustment Acrsquo If there is a loss due to revaluation revaluation account is debited and if the revaluation results in a profit the revaluation account is credited The following journal entries made for this purpose are

(i) For increase in the value of assetsAsset Ac Dr (individually)To Revaluation Ac(ii) For decrease in the value of AssetRevaluation Ac Dr (individually)To Asset Ac[Decrease in the value of assets](iii) For increase in the value of LiabilitiesRevaluation Ac Dr (individually)To Liabilities Ac[Increase in the value of Liabilities](iv) For decrease in the value of LiabilitiesLiabilities Ac DrTo Revaluation Ac[Decrease in the value of Liabilities](v) For unrecorded AssetsAsset Ac [unrecorded] DrTo Revaluation Ac[Unrecorded asset recorded at actual value](vi) For unrecorded Liability Revaluation Ac DrTo Liability Ac [unrecorded][Unrecorded Liability recorded at actual value](vii) For transfer of gain on revaluationRevaluation Ac DrTo Existing Partnerrsquos CapitalCurrent Ac[Profit on revaluation transferred to capital account in existing ratio](viii) For transfer of loss on revaluationExisting Partnerrsquos CapitalCurrent Ac DrTo Revaluation Ac[Loss on revaluation transferred to capital account in existing ratio](a) When revaluation account shows gain Revaluation Ac DrTo Partnerrsquos Capital Ac (Old Profit Sharing Ratio)(Profit on revaluation credited to Partnerrsquos Capital Ac)(b) Above entry is reversed when revaluation account shows loss Partners Capital Acs (Old Profit Sharing Ratio) DrTo Revaluation Ac(Loss on revaluation debited to Partnerrsquos Capital Acs)

Proforma of Revaluation Account is given as under

Revaluation Account

Dr Cr Particulars ` Amount Particulars ` Amount To Decrease in value of assets By Increase in value of assets To Increase in value of liabilities By Decrease in value of liabilities To Unrecorded liabilities By Unrecorded assets To Gain on Revaluation (Transferred) By Loss on Revalution (Transferred)

ECO ndash12 2742020Topic- ELASTICITY OF DEMAND

CHAPTER - ELASTICITY OF DEMANDMEANINGDemand for a commodity is affected by many factors such as its price price of related goods income of its buyer tastes and preferences etc Elasticity means degree of response Elasticity of demand means degree of responsiveness of demand Demand for a commodity responds to change in price price of related goods income etc So we have three dimensions of elasticity of demandDIMENSION OF ELASTICITY OF DEMAND TYPES OF ELASTICITY OF DEMAND

Price elasticity of demand Income elasticity of demand Cross Elasticity of demand

Price elasticity of demand Price elasticity of demand means degree of responsiveness of demand for a commodity to the change in its price For example if demand for a commodity rises by 10 due to 5 fall in its price Price elasticity of demand (ep)=Percentage change in quantity demanded Percentage change in price of the commodity = 10 ( -)5 = ( - )2Note that ep will always be negative due to inverse relationship of price and quantity demanded

(ii) Income elasticity of demand Income elasticity of demand refers to the degree of responsiveness of demand for a commodity to the change in income of its buyer Suppose income of buyer rises by 10 and his demand for a commodity rises by 20 then Income elasticity of demand (ey)= change in quantity demanded change in price of the commodity =20 10 = 2

Cross Elasticity of demandCross elasticity of demand means the degree of responsiveness of demand for a commodity to the change in price of its related goods (substitute goods or complementary goods) Suppose demand for a commodity rises by 10 due to 5 rise in price of its substitute good then Cross elasticity of demand (ec) = change in quantity demanded change in price of related good = 10 2 = 5 (Tastes and preferences cannot be expressed numerically So elasticity ofdemand cannot be numerically expressed)

  • Chapter 1 Force (Summary)
  • Distinguish between external sovereignty and internal sovereignty
    • NAND Gate
      • Logic diagram
      • Truth Table
        • NOR Gate
          • Logic diagram
          • Truth Table
            • XOR Gate
              • Logic diagram
              • Truth Table
                • XNOR Gate
                  • Logic diagram
                  • Truth Table
                      • Physics
                      • Chapter 1 Electric Field ( Electric Dipole) (Summary)
Page 20:  · Web viewWe all know that Nouns are divided into two parts: common noun and proper noun.Apart from common and proper noun, we will also study about collective noun and compound

তোর লি দেইhelliphellipTo be continued

Hindi 2nd lang सर क पद(सरदास)

सरदास शरी कषण भकति कावय क सरवशरषठ कगिरव ह इनक जनम और मतय क समय तथा सथान का मतभद हसरदास रवातसलय और शरार रस क अनयतम कगिरव ह इनक कावय म बालकषण क सौदय चपपल चषटा और गि7याओ की मनोहर झाकी मिमलती ह कषण और ोगिपयो क अननय परम का कतिचतरण ह सयो शरार की अपकषा उनक कावय म गिरवयो शरार का अमिBक गिरवषय और मारमिमEक कतिचतरण हआ हइन पकतियो म हम सरदास की भकति भारवनाओ का परिरचय मिमलता ह इनका सपण सगरह सरसार म गिनगिहत ह

1 जसोदा हरिर पालन झलारवहलरारवदलराईमलहारव रव जो ईसाई कछ ारव मर लाल को आई निनEदिदया काह ना आगिनसबारव त काह नाही बरवगिह आरवतोको कानहा बलारव

शबदाथ-हलरारव-गिहलती हदलराई - दलार पयार करती हमलहारव-पचकारती हनिनEदरिरया ndashनीदरवगिह-जलदी सअBर-होठमौन-चपसन-सकत

वयाखया- सरदास जी कहत ह गिक यशोदा माता बालक कषण को पालन म झल आती ह रवह उनह गिहलाती ह पयार करती ह मलहार जस कोई ीत ान लती ह और नीद स पछती ह गिक ह नीद तम मर लाल को आकर कयो नही सलाती तझ खाना बला रहा ह कभी कषण आख बद कर लत ह कभी आखफडफडान लत ह उनह सोता हआ जानकर यशोदा माता चप हो जाती ह और इशार म बात करन लती ह इसी बीच अकला कर कषण ज जात हतो गिफर यशोदा माता गिफर स ाना ान लती ह सरदास जी कहत ह गिक भरवान क दशन का सख दरवता और ऋगिष-मगिनयो को भी दलभ ह यही सख माता यशोदा को बडी सहजता स मिमल जा रही ह माता यशोदा बहत ही भागयशाली ह2)Continue to nexthellip

Physics

Chapter 1 Force

(Summary)

Question A body is acted upon by two forces each of magnitude F but in opposite directions State the effect of the forces if

(a) Both forces act at the same point of the body

(b)the two forces act at two different points of the body at a separation r

Solutions

(a) Resultant force acting on the body = 0

F ndash F = 0(b) The forces tend to rotate the body between two forces about the midpoint

Moment of forces = F times rFr

QuestionDefine moment of a couple Write its SI unit

Solutions

Moment of couple is equal to the product of both force and the perpendicular distance between the two forces

The SI unit of moment of couple is NmCommercial Studies

Advertising and sales

Business firms use several methods to

Questions1) What do you mean by advertising

promotion create demand of their product in the market and increase it sales Such methods comprises of advertising sales promotion personal selling and publicityToday we are going to discuss about one of such methods It is advertising

Meaning of advertisingAdvertising is a paid form of non-personal presentation for promotion of Ideas goods and services

Importance or merits of advertising Advertising has importance to manufacturer or traders to customer and to society as a whole

Today we will see how advertisement help the manufacturer or traders

Answer) Advertising is a means of how a company encourages people to buy their products services or ideas It is one element of marketing which also includes design Research and data mining

2) Mention any three features of advertisingAnswer)The main features of advertising are

i) It is impersonal form of presentation for promotion of products and services of Ideas

ii) It is issued by identified sponsor The advertisement contains the name of the advertiser

iii) It is a form of mass communication because the message is directed to a large number of persons simultaneously

3) Mention the main merits or importance of advertisement to manufacturer or tradersAnswer)

i) Introducing new product A business organization can introduce itself and its products to the public through advertising

ii) Increase the sale Advertising leads to increase the sale of existing product by entering into new markets and attracting new customers

iii) Create steady demand Advertising creates sustains regular demand by smoothening out seasonal and other fluctuations It enables regular production for the organisation

iv) Economics of scale Advertising facilitate mass distribution of goods and steady demand which lead to large scale and regular production

v) Goodwill Advertising helps in creating a good image of the firm and reputation for its products

Biology Chapter - 03Genetics

Today wewill start chapter and discuss about Genetics Gregor Mendel is known as father of genetics Before entering into Mendelrsquos experiment on Genetics we must know

Q1 Define the following termsi) Genetics Genetics is the study of

transmissionof body features from parents to offspringand the laws relating to such transmission

ii) Heredity It may be defined as transmissionof genetically based characteristics from parentsto offspring

iii) Character and traits Any heritable

Importance to TraderIntroducing new productIncrease the saleCreate steady demandEconomics of scaleGoodwill

some terms featureis a character The alternative forms of acharacter are called traitsex Character (Hair shape) - Traits (Curly straight)

iv) Homologous chromosomes A pair ofcorresponding chromosomes of the same shapeand size one from each parent

v) Genes Genes are the specific parts (DNA segments) of a chromosome which determinethe hereditary characteristicsNearly 30000genes present in human

vi) Alleles Alternative forms of a gene occupying the same position (locus) on homologouschromosomes and affecting the same characteristicbut in different ways

vii) Genotype ndash PhenotypeGenotype means of genes present in the cells of an organism Phenotype means the observable characteristic which is genetically controlled

viii) Mutation It is a sudden change in one or more genes or in the number or in the structure of chromosomes ex Sickle cell anaemia is a blood disease caused by a gene mutation

CLASS NOTES

Class XSubject Eng Literature (The Merchant of Venice ndash William Shakespeare)Topic Act IV Scene 1 Lines 01 to 34 ( Duke helliphelliphellip We all expect a gentle answer Jew) ate 27th April 2020 (2nd Period)

[Students should read the original play and also the paraphrase given in the school prescribed textbook]Summary Questions amp Answers

This scene may be termed as the catastrophe of the play It is the final unravelling of the complicated events which seem to threaten the happiness of Bassanio Portia and Antonio Right is justified to the fullest degree and malice falls into the trap prepared for others No one suffers here but Shylock but even then he receives a generous measure of mercy

o This is the Court-scene Initially we meet

(1)

DUKE I am sorry for thee thou art come to answer (Line 3-6)A stony adversary an inhuman wretchUncapable of pity void and emptyFrom any dram of mercy

(i) Who is addressed here Where is the person Why is the person there

Antonio is addressed hereAntonio is in the court of justice at VeniceAntoniorsquos trial is scheduled to be held here for his failure to meet the conditions of the bond he signed with Shylock

the Duke Antonio Shylock and Salerio Later we meet Bassanio Portia Gratiano and Nerissa

o The Duke says to Antonio that he has to face a very cruel opponent which Antonio admits and expresses his gratefulness to the Duke for his efforts to soften without result the heart of Shylock in order to be merciful to Antonio Antonio further says that he is ready to accept whatever cruel judgement the Court may award

o When Shylock appears in the court the Duke says that Shylock should change his decision of prosecuting Antonio and demanding the penalty specified in the bond out of consideration of the great misfortunes that Antonio has suffered If this is done by him (Shylock) the whole court would be gladdened by his merciful action

(ii) What is the Duke sorry for

The Duke is unable to change the mind of Shylock from his decision to get the bond forfeited even after he pleaded to Shylock Shylock stands firmly for his bond which when forfeited will allow him to take a pound of flesh from any part of Antoniorsquos body(iii) How does the Duke address Shylock`The Duke calls Shylock an adversary with a heart of stones He calls Shylock as an inhuman wretch without pity Shylock is quite lacking in the slightest quality of mercy (iv) How does Antonio reply to this

Antonio replies that he will meet the revenge of Shylock patiently He has prepared himself to suffer with a quiet spirit the utmost that Shylockrsquos tyranny and rage can do(v) What quality of the Duke is revealed here

The Duke is kind and benevolent He is ready to help Antonio He requests Shylock to free Antonio from the trial(vi) What are the terms of the bond that Antonio has signed

The terms of the bond that Antonio has signed were that if Antonio is unable to repay Shylock a certain sum of money specified on the paper on a certain date and in an agreed place the forfeit has to be paid The forfeiture will be an exact pound of Antoniorsquos flesh which Shylock will be a liberty to take from any part of Antoniorsquos body which pleases him

Class XI

STUDY MATERIAL

Class XISubject Eng Literature (The Tempest ndash William Shakespeare) Topic Act I Scene 2 Lines 88 to 132 (Prospero hellip Me and thy crying self) Date 27th April 2020 (3rd Period)

[Students should read the original play and also the paraphrase given in the school prescribed textbook]Summary Questions amp Answers

o Prospero now tells Miranda that he was the Duke of Milan He had been devoting himself more to studies than the affairs of the State His brother Antonio took advantage of this situation and with the help of Alonso the king of Naples seized upon him and her one midnight and shipped them in a frail bark so that they perished in the sea All this took place

(1)

MIRANDA I should sin (Line 118-132)

To think but nobly of my grandmother

Good wombs have borne bad sonsPROSPERO Now the condition

The King of Naples being an enemyTo me inveterate hearkens my brothers suitWhich was that he in lieu othrsquo premisesOf homage and I know not how much tribute

twelve years back

IMPORTANT PASSAGES EXPLAINED(Line 98-103)

PROSPERO helliphelliphelliphelliphelliphelliphellip

He being thus lorded

Not only with what my revenue yieldedBut what my power might else exact like oneWho having into truth by telling of it Made such a sinner of his memoryTo credit his own lie he did believeHe was indeed the duke

Prospero in telling the narrative of his past life here refers to his brother Antonio Prospero being with a studious bent of mind has left the administration of Milan on his younger brother Now Antonio being thus invested like a lord with all the powers derived from Prosperorsquos wealth and what the exercise of Prosperorsquos authority might secure for him regarded himself as a de facto Duke of Milan It is a well-known fact of psychology that a man who repeatedly tells a lie makes of his memory such a sinner against truth as to credit his own lie by the telling of it So Antonio by repeatedly saying to himself and others that he was the Duke came to believe that he was really the Duke Thus falsehood repeatedly asserted gained the force of truth for Antonio and he truly believed it

Should presently extirpate me and mine Out of the dukedom and confer fair MilanWith all the honours on my brother whereonA treacherous army levied one midnightFated to thrsquo purpose did Antonio openThe gates of Milan and ithrsquo dead of darkness The ministers for thrsquo purpose hurried thenceMe and thy crying self

(i) In the earlier lines of this scene what does Prospero tell about his intense interest What was the demand of his interest

In the earlier lines of this scene Prospero tells Miranda that he had an intense interest in the study of philosophy and magic arts Hence in order to improve his mind with this kind of study he kept himself isolated from worldly and state affairs His study was dearer to him than the applause and esteem that he could win from the public His study demanded too much solitude(ii) What forced Antonio to take an undue advantage over Prospero

Prosperorsquos indifferent attitude towards the statersquos affairs and his having boundless trust in Antonio gave rise to a boundless lust for power in Antoniorsquos mind Antonio felt that he must be the actual Duke instead of the part of the Duke he played Thus Antonio took an undue advantage of the situation to usurp Prosperorsquos dukedom(iii) Explain the following lines ldquoI should sin to think but nobly of my grandmother Good wombs have borne bad sonsrdquo

After hearing the treacherous act of her uncle Antonio Miranda says that Prosperorsquos mother was a noble lady and she cannot dishonour her memory by saying that the person named Antonio cannot be his (Prosperorsquos) brother She says that in honour of her grandmother she also cannot say that Antonio must have been begotten not by her grandfather but by some other man She finally concedes that it is known that good mothers have borne bad sons in their wombs and gave birth to them

(iv) Why did the King of Naples accept Antoniorsquos request to help him in usurping his dukedom What did Antonio propose to Alonso

Alonso who was the king of Naples accepted Antoniorsquos request in usurping Prosperorsquos dukedom because he (Alonso) was a sworn enemy of Prospero Antonio proposed that Alonso should immediately drive him (Prospero) and his offspring out of Milan and should confer the dukedom upon him (Antonio) with all the dignities which go with that

position In return Antonio promised that he shall give an annual tribute and also swore his allegiance to Alonso Also he agreed to hold the Dukedom of Milan as a subordinate to the state of Naples(v) How were Prospero and Miranda carried away from the city of Milan and what was the state of small Miranda at that time

In pursuance of the agreement settled between Antonio and Alonso an army of treacherous men was assembled One midnight when the occasion suited the will of destiny Antonio opened the gates of the city of Milan and in the death like silence of midnight Antoniorsquos agents who had been directed to execute his purpose carried Prospero and small Miranda away from the city in all haste They were then forced into a ship and carried some distance out to sea where they put them on a mere hulk of a boat without any rigging or ship-gear and abandoned them leaving them at the mercy of the roaring sea Miranda was a very small child of three years age and she was crying at that time

CLASS -XIDATE-270420Subject Topic Summary Execution

EVS Chapter 1 ndash Mode of Existence

Impact of mode of existence on resources

Q) Why resources are under pressure

Ans - Increase in the sophistication

of technology enabling natural resources to be extracted quickly and efficiently Eg in the past it could take long hours just to cut down one tree only using saws Due to increased technology rates of deforestation have greatly increased

The number of humans is increasing Cultures of consumerism Materialistic views

lead to the mining of gold and diamonds to produce jewelry unnecessary commodities for human life or advancement Consumerism also leads to extraction of resources for the

production of commodities necessary for human life but in amounts excessive of what is needed because people consume more than is necessary or waste what they have

Lack of awareness among the population is striking People are not aware of ways to reduce depletion and exploitation of materials

Accounts Cash Book Today we are going to start a new topic -Cash Book

The key terms used in this chapter are

bullCash book

bullSimple cash book

bullDouble column cash book bullTriple column cash book

bullPetty cash book

bullCash discount

bullContra entry

Here I will share you the meaning of each key terms

bullCash book Cash Book is a special purpose subsidiary book or journal in which cash received and cash payments are recorded

bullSimple cash book

It is a cash book in which only cash transactions are recorded It has only one column on each side

bullTriple column cash book

It is cash book which has three columns one column for each cash and Bankdiscount on each side of the cash book In this book both cash and Bank transactions are recorded together with discount allowed and received

bullPetty cash book

It is a cash book maintained for recording petty expenses

bullCash discount

Cash discount is the amount of discount received or allowed on cash payments and cash receipts Discount received is an income for the business while discount allowed isan expense

bullContra entry

It means transactions involving both cash and Bank Such transactions though recorded in the cash book are not posted into ledger The letter lsquoC is written in Ledger folio for contra entry

Business Studies

ENTREPRENEURSHIP

Now we shall discuss the second chapter

lsquoENTREPRENEURSHIPrsquo

Today before starting the chapter let us recall what

Questions

1What are the main characteristics of Intrapreneurship

Answer

The main characteristics of Intrapreneurship are

Corporate framework-it occurs within the framework of the same company

Semi-Autonomous-Intrapreneurship

we have read last day

Let s today start the class by recalling the last topic taught

Intrapreneurship is the process of discovering and exploring business opportunities within an existing company It involves launching new business ventures within the framework of a present corporation Intrapreneurship is also known as corporate entrepreneurship or corporate venturing

Now let us start with the characteristics of Intrapreneurship

The main characteristics of Intrapreneurship are

Corporate framework

Semi-Autonomous Lack of ownership Senior position Low risk taking Not own boss

Now let us discuss the meaning of enterprise

Enterprise means an undertaking or adventure that requires some innovation and investment and thus involves riskEnterprise always entails decision making coordination and risk bearing

involves crating amd nurturing a semi-autonomous business unit which may be a subsidiary a strategic business unit or a division

Lack of ownership-the intrapreneur is not the owner of the unitb he creates and nurtures

Senior position-he occupies a senior managerial position in the company

Low risk taking-An intrapreneur does not bear the full risk of failure

Not own boss-An intrapreneur is not his own bosss in legal termsHe enjoys the freedom and gets the required resources and support

2 How is Entrepreneur is different from Intrapreneur

The functions involved in both the entrepreneurship and intrapreneurship are by and large similar however there are several differences between the two

Point of distinction

Entrepreneur

Intrapreneur

status An independent business person

A senior executive within a company

Ownership Owner of

the business

An employeesometimes a share in ownership

Financing Responsible for raising finance for the business

Not responsible for raising the finance

Risk bearing

Bears the risk of the business

Does not bears the risk of the business

Reward Profit which is uncertain and irregularcan be loss

Fixed salary and fringe benefits

Need for security low high

3 What do you understand by enterprise

Answer Enterprise means an undertaking or adventure that requires some innovation and investment and thus involves riskEnterprise always entails decision making coordination and risk bearing

COMMERCE NATURE AND OBJECTIVES OF

BUSINESS

Today let us recall the last other two objectives of business by the chart given in the previous class

Firstly we would discuss Human Objectives

Business is run by people and for people Labour is a valuable business element

Human objectives of business are concerned with the well -being of labour

The human objectives are as follows

Labour welfare Developing human

resources Participative

management Labour

management cooperation

Questions

1 Explain the human objectives of a business enterprise

Answer

Business is run by people and for people Labour is a valuable business element

Human objectives of business are concerned with the well -being of labour

The human objectives are as follows

Labour welfare-Business must recognize the dignity of labour and human factors should be given the recognition

Developing human resources-Employees must be provided the opportunities for developing new skills and attitudes

Participative management-Employees should be allowed to take part in decision making process of business

Labour management cooperation-Business should strive for creating and maintaining cordial employer employee relations so as to ensure peace and progress in industry

Now let us discuss the national objectives of business

Optimum utilization of resources

National self- reliance Development of small

scale industries Development of

backward areas Control over pollution

2Explain the national objectives of a business enterprise

Answer

It is the duty of business to utilize the resources of the country properly the national objectives of business

Optimum utilization of resources ndashBusiness should use the nationrsquos resources in the best possible manner

National self- reliance-It is the duty of the business to help the government in increasing experts and in reducing dependence on imports

Development of small scale industries-Big business firms are expected to encourage growth of small scale industries which are necessary for generating employment

Development of backward areas-Business is expected to give preference to the industrialization of backward regions of the country

ECONOMICS

BASIC ECONOMIC CONCEPTS

SUB

TOPIC

Value

Wealth

Welfare

Today we shall start with a new topic of the same chapter ie lsquoValuersquo

Value of a commodity is defined as the valuation placed by a household on the consumption of this commodity

lsquoValuersquo has two different meanings and these are

a Value -in -use It refers to consumption value of a commodity It expresses the utility derived from the consumption of a particular commodity A necessity like water has a very high value ndashin ndashuse or

Question

1What is value

Answer

Value of a commodity is defined as the valuation placed by a household on the consumption of this commodity

2What is value-in use

Answer It refers to consumption value of a commodity It expresses the utility derived from the consumption of a particular commodity A necessity like water has a very high value ndashin ndashuse or consumption value

3What is value ndashin- exchange

Answer It relates to market value of a commodity

It is the rate at which a particular good or service can be exchanged for money

For example in barter system if a person is prepared to exchange 3 metres of cloth with 1 pair

consumption value

b Value ndashin-exchange It relates to market value of a commodity

It is the rate at which a particular good or service can be exchanged for moneyFor example in barter system if a person is prepared to exchange 3 metres of cloth with 1 pair of shoes then the value in exchange of 3 metres of cloth is 1 pair of shoesValue in exchange is the power of purchasing other goods In modern monetised economies the exchange value of goods are expressed in terms of money as prices

Now let us discuss the term lsquoWealthrsquo

Wealth refers to the stock of all those assets which are a source of income

Wealth is a stock concept

Wealth must possess the following features

a Utility It must possess utility or give some

of shoes then the value in exchange of 3 metres of cloth is 1 pair of shoes

Value in exchange is the power of purchasing other goods In modern monetised economies the exchange value of goods are expressed in terms of money as prices

4 What is wealth

Answer it refers to the stock of assets or goods which are a source of income and have personal or national ownership

5 What are the features of wealth

Answer The features of wealth are as follows

Wealth must possess the following features

a Utility It must possess utility or give some satisfaction

b Scarcity It must be limited in quantityc Transferability it should be transferable its

ownership can be transferred from one person to another person

d Exchange value It must possess exchange value

6 What is welfare

Answer

Welfare is defined as satisfaction and happiness a sense of well- being among the people

satisfactionb Scarcity It must be

limited in quantityc Transferability It

should be transferable its ownership can be transferred from one person to another person

d Exchange value It must possess exchange value

Now let us discuss the term lsquoWelfarersquo

Welfare is defined as satisfaction and happiness a sense of well- being among the people

Welfare is affected by factors like

a Consumption of goods and services

b Environment

c Family relations

d Degree of freedom

e Law and order situation

Mathematics Trigonometric equation

To find the general solution of the equation sinθ=0

When sin θ =0

Then θ= 0 π2π 3π-π -2π -3

i e when θ = 0 or an integral multiple of π

i e when θ= nπ where n is any integer

Therefore the general solution of the equation sin

Example1 Find the general values of θ which satisfy the equation sin2 θ =34

Solution sin2 θ= 34

Or sin θ = +34 or -34

Or sin θ = sin π3 or sin (-π3)

Therefore

θ = [nπ + (-1) n (π3)] or[ nπ+ (-1) n (-π3)]

= nπ +π3 or nπ-π3 where n= any integer

Example 2Find the values of θ which satisfy tan2 θ

θ=0 is θ= nπ where n is any integer

To find the general solution of the equation cos θ=0

When cos θ=0

Then θ=π2 3π2 5π2 -π2 -3π2 -5π3

i e when θ is an odd multiple of π2

i e when θ=(2n+1) π2 where n= any integer

Therefore the general solution of the equation cos θ =0 is θ= (2n+1) π2 where n= any integer

To find the general solution of the equation tan θ = 0

Clearly tan θ =0 implies sin θcos θ =0

Therefore θ = nπ

i e the general solution of the equation tan θ=0 is θ =nπ where n = any integer

To find the general solution of the equation cot θ =0

Clearly cot θ =0 implies (cos θsin θ) = 0

i e cos θ =0

Therefore θ = (2n+1) π2

Therefore the general solution of the equation cot θ =0 is θ = (2n+1) π2

Where n= any integer

To find the general solution of the equation sin θ= k (-

=13 -πleθleπ

Solution tan2 θ =13

Or tan θ = plusmn1radic(3) =tan(plusmnπ6)

θ=nπ plusmn π6 where n =any integer

If n=0 then θ=plusmnπ6

If n=1 then θ= π plusmn π6

If n=-1 then θ= -π plusmn π6

Therefore the required solution in -π le θ le π are θ= π6 5π6 -π6 -5π6

Exercise Find general solution of sin 2θ=cos θ [Hints Use sin 2θ= 2sin θcosθ and then take cosθ

common]

1lek le1)

Determine an angle alpha such that sin =k and -π2le αle π2

Then we have

Sin θ = k = sin α

Or sin θ - sin α =0

Or 2 cos [(θ+α) 2] sin [(θ-α) 2] =0

Therefore either cos [(θ +α) 2] =0 (1)

Or sin [(θ-α) 2] =0 (2)

Now from (1) we get (θ+α) 2= (2m+1) π2)

Or θ = (2m+1) π-α (3)

And from (2) we get (θ-α) 2 =mπ

Or θ= 2mπ+α(4)

Where m = any integer

Clearly the solution (3) amp (4) may be combined in the following form

θ= nπ+(-1) n α where n= any integer

Therefore the general solution of sin θ = sin α is θ = nπ +(-1) n α where n is any integer and -π2 le α le π2

Biology Chapter - 04Kingdom Monera

Today we will discuss about bacterial reproduction and its usefulness

Fig Binary Fission

Fig Conjugation Fig Transformation

Bacterial reproduction is mainly asexual but sexual reproduction

also takes place

Asexual reproduction takes place by i) Binary fission - from one bacteriato

two bacteria are produced in every 20 to 30mins

ii) Buddingiii) Endospore formation - during

unfourable condition

Sexual reproduction by three ways

1) Conjugation - Transfer of genetic material between cells that are in physical contact with one another

2) Transduction - Transfer of genetic materialfrom one cell to another by a bacteriophage

3) Transformation - Transfer of cell-freeor naked DNArsquo from one cell to another

Bacteria causes different diseases inplants animals and human and

it causes food spoilage and waterpollution but it also have some useful

activities

i) Bacteria are helpful in sewage water treatment

ii) It is used in antibiotic (medicine) production

iii) Anaerobic bacteria help in biogas(energy) production

iv) Many household products like yoghurt cheese are manufactured by use of bacteria

v) Rhizobium by symbiotic relationship with leguminous plant increase soil fertility

vi) Besides these bacteria is helpful in genetic engineering degradation of petroleum hydrocarbonand in dairy

industry

Physics Motion in plane Here we will introduce Projectile Motion

Execution

Projectile

Y

usinθ u h

θX

ucosθ

Suppose a body is projected with an angle θ So initial velocity u can be resolved into two components

Horizontal component - ucosθ ( for range)

Vertical component - usinθ ( for height)

usinθ changes during motion and becomes zero at maximum height position but ucosθ remain unchanged

The maximum height of projectile is h

NB If initial is upward then g = -ve and if it is downward then g = +ve Height is +ve if direction of motion does not change ( for ex a body thrown upwards but goes down ultimately then height h = -ve)

The angle of projectile θ is the angle made with horizontal

HISTORY ndash GROWTH OF NATIONALISM

SUB TOPIC- REVOLUTIONARY NATIONALISM Bengal formation of Anushilan Samity and Jugantar Group

The intensification of the Swadeshi movement and Government policy of terror and repression led to outbreak of violence Bombs were manufactured and attempts on the lives of unpopular Government officials became frequent In the gymnasium of Scottish Church College which was known as General Assemblies Institution a secret society was formed known as Anushilan Samity

Aurobindo Ghosh send from Baroda his emissary Jatindranath Banerjee to mobilize the Bengal revolutionaries

Hemchandra Qanungo and Satyen Bose published Journal Jugantar

The Jugantar group planned to assassinate oppressive magistrate Kingsford by Khudiram Bose and Prafulla Chaki in 1908 Prafulla Chaki committed suicide to avoid arrest Khudiram was tried and hanged

Afew days later the police found a bomb factory in Maniktala and arrested a large number of revolutionaries The trial of revolutionaries became famous as the Alipore Bomb Case

In the course of the trial the approver the public prosecuter and a police officer were assassinated

1 Question Name two journals which preached the cult of violence

Answer a) Yugantar edited by Bhupendranath DuttaB) Bandemataram edited by Aurobindo Ghosh2 Question Why was Khudiram arrested and hangedAnswer An attempt was made to assassinate a hated vindictive majistrate named Kingsford by Khudiram Bose and Prafulla Chaki Their attempt failed and the bomb they threw killed two English ladies Khudiram was arrested and put to trial and then hanged3Question Who was Aurubindo GhoshAnswer Aurobindo Ghosh a nationalist revolutionary who was charged for his involvement in the Alipore Bombing Case He was accused of it along with his brother Barindra nath Ghosh But Aurobindo was acquitted because of the brilliant pleading of his counsel Chittaranjan Das Then he became a spiritual reformer introducing his visions on human progress and spiritual evolution4 Qustion Who was KingsfordAnswer Kingsford was an unpopular British chief Magistrate who was the target of the bomb thrown at Muzaffarpur by Khudiram and Prafulla Chaki

Most of the accused were convicted and sentenced to

long term of imprisonmentBut

Aurobindo Ghosh was acquitted mainly owing to the brilliant pleading of his counsel Chittaranjan Das

Political science Topic-Sovereignty

Summary Sovereignty is the full right and power of a governing body over itself without any interference from outside sources or bodies In political theory sovereignty is a substantive term designating supreme legitimate authority over some polity In international law sovereignty is the exercise of power by a state

Internal Sovereignty

Internal sovereignty means supreme authority within ones territory while external sovereignty relates to the recognition on the part of all states that each possesses this power in equal measure

External sovereignty

external sovereignty relates to the recognition on the part of all states that each possesses this power in equal measure

Distinguish between

Execution

Answer the following questions

Short notes-

Sovereignty

Internal Sovereignty

External sovereignty

Homework- learn

external sovereignty and internal sovereigntySovereignty is the principle

of supreme and

unquestionable authority

reflected in the claim by the

state to be the sole author of

laws within its territory

Definition of external vs internal sovereigntyInternal sovereignty refers to

the relationship between a

sovereign power and its

subjects ndash it refers to the

location of the supreme

authority within the state In

the UK for example internal

sovereignty (supposedly)

resides within Parliament

reflected in the

constitutional principle of

parliamentary

sovereigntyBy contrast

external sovereignty refers

to the capacity of the state

to act independently and

autonomously on the world

stage This is what is

sometimes called lsquostate

sovereigntyrsquo or lsquonational

sovereigntyrsquo and implies

that states are legally equal

and that the territorial

integrity and political

independence of a state is

inviolable

Class ndash XII

Date - 2742020 STUDY MATERIALSubject Topic Summary Execution Business Studies

Job Analysis amp Manpower Planning

At first let us recall the chapter what we have discussed till nowbullJob analysisbullJob specification bullJob description bullJob enlargement bullJob enrichment

Today we will do some questions answers from the chapter

Questions 1ldquoJob analysis job description and job specification are interrelatedrdquo Comment Answer) Job analysis is a systematic and detailed examination of a job to collect all the relevant information about it The contents off the job are summarised in the job description The qualification needed for the job are summarised in job specificationThus there is close interrelationship between job analysis job description and job specification

Question 2ldquoJob enlargement is a horizontal extension of a job whereas job enrichment is a vertical extension of a jobrdquo ElucidateAnswer) Job enlargement involves adding one or more task to a job coma where as job enrichment involves adding more autonomy and responsibility to a job Job enlargement is therefore horizontal extension of a job coma whereas job enrichment is a vertical extension of a job

Question 3 )

What is manpower estimation Explain its quantitative and qualitative aspectsAnswer) Manpower estimation is the process by which management determines how an organisation should move from itrsquos current manpower positionto its desired manpower position There are two dimensions of Manpower estimation- quantitative and qualitative

Quantitative aspectThis aspect of Manpower estimation involves estimating the number of employees required in a future time period Workload analysis and workforce analysis are done to estimate the quantity of required manpower

Qualitative aspectThe estimate of the knowledge skills experience etc of required manpower is the qualitative aspect of Manpower estimation The quality of Manpower can be judged on the basis of job analysisand job specification

COMMERCE

CAPITAL-FIXED AND WORKING

Today let us start the class by discussing the sources of finance for different types of business firms

The term lsquocapitalrsquo refers to the investment made in the enterprise for the purpose of earning profits

Requirements of capital and sources of capital for different types of business firms are

1 Capital for sole proprietorship businessA sole proprietor operates at a small scale and thereforerequires a limited amount of capital

2 Capital for partnership firmCapital requirements as well as capital base of a partnership is bigger than that of a sole trader businessThe owned capital is contributed by the partners in an agreed ratio

3 Capital for joint stock companyA joint stock company generally requires large amount of capitalA public company can raise huge capital through issue of shares In addition to share capital it can utilize retained profits

Now let u discuss the meaning of Finance PlanningFinance planning is the process of estimation the financial requirements of an organization specifying the sources of firms and ensuring that enough funds are available at the right time

1 What do you mean by Finance PlanningAnswer Finance planning is the process of estimation the financial requirements of an organization specifying the sources of firms and ensuring that enough funds are available at the right time

2Discuss the role of financial planning of an enterpriseThe role of financial planning are as followsa A sound financial plan helps a business enterprise to avaid the problems of shortage and surplus of fundsbFinancial planning serves as a guide in developing a sound capital structure so as to maximize returns to shareholders c It helps in effective utilization of fundsd It provides policies and procedures for coordinating different functional areas or departments of businesse It enables the management to exercise effective control over the financial activities of an enterprisef It helps the company to prepare for facing business shocks and surprises in future

Mathematics

Continuity and differentiability

Recall Definition of ContinuityLet f(x) be a single valued function of x and x=a be a point in the domain of definition of the function The function is said to be continuous at x=a ifi) f(c) is defined ie f(x) has a definite finite value at x=cii) lim xrarra f(x) exists andiii) lim xrarra f(x) =f(a) In other words f(x) is said to be continuous at x=a if lim xrarra+ f(x)= lim xrarra- f(x) = f(a) Or f(a+0) =f(a-0) =f(a) Or lim hrarr0 f(a+h)= f(a) Algebra of continuous functionsNow we will study some algebra of continuous functions Theorem 1 Suppose f and g be two real functions continuous at a real number c Then(1) f + g is continuous at x = c(2) f ndash g is continuous at x = c(3) f g is continuous at x = c(4) (fg) is continuous at x = c (provided g (c) ne 0)

Example 1 Prove that every rational function is continuousSolution Recall that every rational function f is given byf(x)=[p(x) q(x) ] q(x)ne0where p and q are polynomial functions The domain of f is all real numbers except points at which q is zero Since polynomial functions are continuous f is continuous by (4) of Theorem 1Example 2Discuss the continuity of sine functionSolution To see this we use the following factslim xrarr0 sin x =0Now observe that f (x) = sin x is defined for every real number Let c be a real number Put x = c + h If x rarr c we know that h rarr 0 Therefore lim xrarrc f(x)

= lim xrarrc sin x= lim hrarr0 sin(c+h) =lim hrarr0 [sin c cos h + cos c sin h ]=lim hrarr0 (sin c cos h) + lim hrarr0 (cos c sin h) = sin c +0=sin c = f(c) Thus lim xrarrc f(x) = f(c) and hence f is a constant function Exercise Prove that the function f(x) = x2 +2x is continuous for every real value of x [Hints show that lim xrarra+ f(x) = lim xrarra- f(x) = f(a) ]

Biology Reproduction in Flowering plants We will discuss about megasporoangium

megasporagenesis and female gametophyte

Q4 Describe the structure of megasporangium

Ovule is attached to the placenta by astalk called funicle

Each ovule has one two or three protectivecoverings called integuments

At the tip of integuments a small openingcalled micropyle is organised

Opposite to the micropylar end is the chalaza

Within the integuments a mass of cellsnucellusand inside it embryo sac orfemale gametophyte is present

Q5 Describe a mature embryosacamp its formation

In most of the flowering plants only oneof the 4 megaspores formed as a result ofmegasporogenesis that is functional while theother three degenerate

The

functional megaspore develops into thefemale gametophyte

Formation The nucleus

of the functional megasporedivides mitotically to form two nuclei first andthen two more sequential mitotic nucleardivisions result in the formation of four ampthen eight nucleate stages of embryo sac

Six of the eight nucleus are surrounded bycell walls and organised into cells

The remaining two nuclei called polar nuclei are found below the egg apparatus in the largecentral cell

Three cells consisting of two synergids amp one egg cell present bottom of

embryo sac Three cells

at the chalazal as antipodal cells

Two polar nuclei together present in large central cell

HISTORY

TOWARDS INDEPENDENCE AND PARTITION THE LAST PHASE(1935-1947)SUB TOPIC NATIONAL MOVEMENTS DURING THE SECOND WORLD WAR

Spread of Quit India Movement On 9th August 1942Gandhiji and other Congress leaders were arrested The Congress was declared illegal The news of the arrest of all leaders marked the beginning of a widespread movement of India It was not possible for such a movement to remain peacefulBut the arrest of the all notable congress leaders virtually left the movement in the hands of the mass The movement took the form of violent and militant outbreakBesides congressmen revolutionaries also were very active in the movement The Congress Socialist group also played a prominent role

1 Question Why did the British authority arrest the Congress leaders on 9 th August 1942Answer Congress Working committee adopted the Quit India resolution which was to be ratified at the Bombay AICC meeting in 8th August 1942 They decided to launch a mass struggle on non-violent lines Gandhiji gave a clarion call to all section of the people rdquoKarenge ya Marengerdquo (do or die) Congress leaders gave the call to driving out

the British from IndiaViceroy had taken strong action against the Quit India movement Gandhiji and all the leaders of Congress were arrested

2 Question How did Quit India Movement spread out all over IndiaAnswer The news of the leaders lsquo arrest marked the beginning of a widespread movement to remain peacefulThe movement took form of violent outbreak There were widespread cutting of telephone and Telegraph wires damaging railway lines raising barricades in cities and towns and other forms of violent demonstations

Question Name the leaders of Congress

Socialist group played a prominent part Notable among the Jayprakash Narayan Rammonohar Lohia Aruna Asaf Ali

Political science

Topic-Franchise and Representation

Summary

The election commission

The Election Commission of India is an autonomous constitutional authority responsible for administering Union and State election processes in India The body administers elections to the Lok Sabha Rajya Sabha State Legislative Assemblies in India and the offices of the President and Vice President in the country

Functions of election commission-

India is a sovereign socialist secular democratic republic Democracy runs like a golden thread in the social economic and political fabric woven by the Constitution given by lsquoWe the People of Indiarsquo unto ourselves The concept of democracy as visualised by the Constitution pre-supposes the representation of the people in Parliament and State legislatures by the method of election The Supreme Court has held that democracy is one of the inalienable basic features of the Constitution of India and forms part of its basic structure The Constitution of India adopted a Parliamentary form of government Parliament consists of the President of India and the two Houses mdash Rajya Sabha and Lok Sabha India being a Union of states has separate state legislatures for each state State legislatures consist of the Governor and two Houses mdash Legislative Council and Legislative Assembly mdash in seven states namely Andhra Pradesh Telangana Bihar Jammu amp Kashmir Karnataka Maharashtra and Uttar Pradesh and of the Governor and the state Legislative Assembly in the remaining 22 states Apart from the above two out of the seven Union Territories namely National Capital Territory of Delhi and Puducherry also have their Legislative Assemblies

ExecutionShort notes-Election commissionFunctions of election commission

Homework- Learn

Computer

Science

Computer hardware NAND Gate

A NOT-AND operation is known as NAND operation It has n input (n gt= 2) and one output

Logic diagram

Truth Table

NOR Gate

A NOT-OR operation is known as NOR operation It has n input (n gt= 2) and one output

Logic diagram

Truth Table

XOR Gate

XOR or Ex-OR gate is a special type of gate It can be used in the half

adder full adder and subtractor The exclusive-OR gate is abbreviated as EX-OR gate or sometime as X-OR gate It has n input (n gt= 2) and one output

Logic diagram

Truth Table

XNOR Gate

XNOR gate is a special type of gate It can be used in the half adder full adder and subtractor The exclusive-NOR gate is abbreviated as EX-NOR gate or sometime as X-NOR gate It has n input (n gt= 2) and one output

Logic diagram

Truth Table

Physics

Chapter 1 Electric Field ( Electric Dipole) (Summary)

Here we will derive Expression of electric field at broad side

On position of dipole

Execution

Q With the help of a labelled diagram obtain an expression for the electric field intensity E at any point on the equitorial line ( broad-side on position) of an electric dipole

Ans

E1 E1sinθ

E θ P E1 θ

( r2+L2)12 E2 E

r E2 E2sinθ

-q θ L O L +qA B

Let us consider that the point P is situated on the right bisector of the dipole AB at a distance r meter from its midpoint O

Let E1 and E2 be the electric field intensities of the electric field at P due to charge +q and ndashq of the dipole resp The distance of P from each charge is ( r2+L2)12

So E1 = 14 πϵ q

(r 2+L 2) away from +q

E2 = 14 πϵ q

(r 2+L 2) towards ndashq

The magnitudes of E1 and E2 are equal but directions are different Now resolving E1 and E2 into two components parallel and perpendicular to AB we get

The components perpendicular to AB E1sinθ and E2sinθ cancel each other because they are equal and opposite

The components parallel to AB are E1cosθ and E2 cosθ are in same direction and add up

So resultant intensity of electric field at the point P is

E = E1cosθ + E2 cosθ

E = 14 πϵ q

(r 2+L 2) 2 cosθ

Now from fig we have cosθ =BOBP = L (r2+L2)12

So we get E = 14 πϵ 2qL ( r2+L2)32

Now electric dipole moment p= 2qL

So E = 14 πϵ p ( r2+L2)32

HW Find the expression of Electric field as done here but this time take r gtgt 2L

Also find the expression of torque experience by a dipole

(Hint Electric force experienced by charges of dipole in electric field is qE each Let θ be the angle which dipole makes with electric lines of force then perpendicular distance between two charges is 2Lsinθ Then torque = force x perp distance = qE x 2L sinθ So τ=pE sinθ where p =2qL )

STUDY MATERIAL

Class XIISubject Eng Literature (The Tempest ndash William Shakespeare) Topic Act IV Scene 1 Lines 84 to 133 (Iris hellip A contract of true love Be not too late ) Date 27th April 2020 (4th Period)

[Students should read the original play and also the paraphrase given in the school prescribed textbook]Summary Questions amp Answers

o Ceres soon appears and comes to know that she has been summoned to celebrate the contract of true love

o Ceres expresses her unwillingness to meet Venus and Cupid as she has shunned their company

o Ceres and Juno both bestow their blessings upon Ferdinand and Miranda with June gifting honour riches happiness in marriage and Ceres presents plenty of earthrsquos produce

o Iris summons the water-nymphs and reapers to come and celebrate a contract

(1) IRIS Of her society (Line 91-101)

Be not afraid I met her deity

Cutting the clouds towards Pathos and her sonDove-drawn with her Here thought they to have doneSome wanton charm upon this man and maidWhose vows are that no bed-right shall be paidTill Hymens torch be lightedmdashbut in vainMarss hot minion is returned againHer waspish-headed son has broke his arrowsSwears he will shoot no more but play with sparrowsAnd be a boy right out

(i) Where were Venus and Cupid seen flying How were they travelling Why did they want to join the marriage celebration of Ferdinand and Miranda

of true love

Venus and Cupid were seen flying through the air towards Paphos the famous city which is situated on the island of Cyprus They were travelling by air-borne chariot drawn by doves They certainly wanted to come here in order to play some amorous trick upon Ferdinand and Miranda who are under a vow not to gratify their physical desires till the holy ceremony of their marriage has been performed(ii) What have Venus and Cupid done after failing in their plan

After being failure of their plan Venus who is a very passionate deity and who is the mistress of Mars (the god of war) has gone back while here ill-tempered son Cupid has broken his arrows of love in his state of desperation(iii) What has Cupid firmly decided

Cupid is feeling so disappointed that he has firmly decided to shoot no more arrows to arouse love in human hearts but to spend his time playing with sparrows Thus he would now become just a boy and would give up his original function of shooting arrows on human beings to make them fall in love(iv) What vow had Ceres taken How did Ceres feel at the abduction

After the abduction of her daughter Prosperina by Pluto Ceres had taken a vow to always keep away from the disgraceful company of Venus and her blind son Cupid the god of love Ceres felt deeply distressed when Pluto had carried off her daughter and had made her his wife by force(v) Why has Ceres not forgiven Venus and her blind son For what do Ceres want to be sure

As the abduction had been manipulated by Venus the goddess of beauty and love and her blind son Cupid Ceres has never forgiven them for their part in the whole plot Ceres wants to be sure that she would not have to meet Venus and Cupid who had engineered the abduction of her daughter Prosperina

AS THIS lsquoMASQUErsquo SCENE IS VERY IMPORTANT IN THE PLAY THE PARAPHRASE OF THE ENTIRE PORTION OF MASQUE SCENE (Act IV Lines 58 to 143) IS GIVEN BELOW

IRIS Goddess of RainbowCERES Goddess of Agriculture and all the fruits of the earth

(Nature growth prosperity rebirth ndash notions intimately connected to marriage)JUNO The majestic Queen of Heavens and wife of Jupiter (Jupiter is the king of Gods)

VENUS The Goddess of love CUPID Son of Venus PLUTO God of death (In the play referred by Shakespeare as lsquoDisrsquo which is a Roman name for Pluto)

ORIGINAL TEXT PARAPHRASEPROSPEROWellmdash

PROSPERONow come Ariel Let there be too many rather than too few

Now come my Ariel Bring a corollaryRather than want a spirit Appear and pertly[to Ferdinand and Miranda]No tongue all eyes Be silent

spirits in attendance Appear briskly

[to Ferdinand and Miranda]Look with your eyes but do not say a word

[Soft music] [Soft music][Enter Iris] [Enter Iris]

IRISCeres most bounteous lady thy rich leasOf wheat rye barley vetches oats and peasThy turfy mountains where live nibbling sheepAnd flat meads thatched with stover them to keepThy banks with pioned and twilled brimsWhich spongy April at thy hest betrimsTo make cold nymphs chaste crowns and thybroom-grovesWhose shadow the dismissegraved bachelor lovesBeing lass-lorn thy pole clipped vineyardAnd thy sea-marge sterile and rocky-hardWhere thou thyself dost airmdashthe Queen othrsquoSkyWhose watery arch and messenger am IBids thee leave these and with her sovereign grace[Juno appears] Here on this grass-plot in this very placeTo come and sport Her peacocks fly amainApproach rich Ceres her to entertain

IRISCeres most generous lady you are the cause of rich fields or fertile land where wheat rye barley beans oats and peas grow the grassy mountains where the sheep graze and the flat meadows covered with coarse hay to be used as fodder for cattleYour banks are covered with marsh-marigolds and reeds and the rainy April under your orders brings forth to make for the maids who are not in love beautiful crowns your woods where the broom flourishes and where the bachelor who has been dismissed by the maid he loved lies down being forsaken your vineyard in which the poles are embraced by the vines and the margin of the sea which is barren and rocky where you roam about to enjoy the fresh air ndash the queen of the sky (Juno) whose messenger I am besides being represented as the rainbow bids you leave all these and with her majesty here on this grassy plot in this very place come and sport her peacocks carry her fast in her chariot through the air and are making their way here approach rich Ceres to welcome her

[Enter Ariel as Ceres] [Enter Ariel as Ceres]

CERESHail many-coloured messenger that neerDost disobey the wife of JupiterWho with thy saffron wings upon my flowersDiffusest honey-drops refreshing showersAnd with each end of thy blue bow dost crownMy bosky acres and my unshrubbed downRich scarf to my proud earth Why hath thy queenSummoned me hither to this short-grassed green

CERESWelcome rainbow that never dared disobey Juno the wife of Jupiter who with your orange coloured rays spread honey-drops refreshing showers And with each end of thy blue bow drown my bushy acres and my hilly country which is free from shrubs you thus forming a rich scarf Why has your queen called me here to this place covered with short grass

IRISA contract of true love to celebrateAnd some donation freely to estateOn the blest lovers

IRISI have called you to celebrate a contract of true love and bestow some liberal gift upon the blessed lovers

ORIGINAL TEXT PARAPHRASECERESTell me heavenly bowIf Venus or her son as thou dost knowDo now attend the queen Since they did plotThe means that dusky Dis my daughter gotHer and her blind boys scandaled companyI have forsworn

CERESTell me heavenly bow if Venus the Goddess of love or Cupid her son and pedlar of passion at this time attend the heavenly queen Juno because you are sure to know Since the day they conspired against me and dark Pluto took away my daughter here and Cupidrsquos disgraceful company I have left off

IRISOf her societyBe not afraid I met her deityCutting the clouds towards Pathos and her sonDove-drawn with her Here thought they to have doneSome wanton charm upon this man and miad

IRISBe not afraid of her company I met her deity moving on the clouds towards Paphos the sacred home of Venus on the island of Cyprus along with her son on her chariot drawn by doves Here they contemplated to exercise a charm upon this man and maid producing

Whose vows are that no bed-right shall be paidTill Hymens torch be lightedmdashbut in vainMarss hot minion is returned againHer waspish-headed son has broke his arrowsSwears he will shoot no more but play with sparrowsAnd be a boy right out

wantonness before the actual marriage ceremony but did not succeed Venus has returned her irritable son has broken his arrows and swears that he will give up his practice of trying to inspire love but play with sparrows and be a boy again

[Music is heard] [Music is heard]

CERESHighst queen of stateGreat Juno comes I know her by her gait

CERESHighest queen of state Great Juno there she comes I know here by her gait

[Enter Juno] [Enter Juno]

JUNOHow does my bounteous sister Go with meTo bless this twain that they may Prosperous beAnd honoured in their issue

JUNOHow are you doing my generous sister Come with me to bless this couple so that they may be prosperous and fortunate in their children

[They sing] [They sing]

JUNOHonour riches marriage-blessingLong continuance and increasingHourly joys be still upon youJuno sings her blessings upon you

JUNOMay honour riches happiness in marriage long continuance and increase of those boons ever rest upon you as hourly joys Juno showers down upon you her blessings in song

CERESEarths increase foison plentyBarns and garners never emptyVines and clustring bunches growingPlants and goodly burden bowingSpring come to you at the farthestIn the very end of harvestScarcity and want shall shun youCeresrsquo blessing so is on you

CERESMay you have the plenty of earthrsquos produce Your barns and granaries may never be empty Your vines may grow with clustering bunches Your fruit trees may be heavily laden with their fruit May there be continuous spring and harvest May scantiness and want leave you forever Such is the blessing of Ceres upon you

FERDINANDThis is a most majestic vision andHarmoniously charmingly May I be boldTo think these spirits

FERDINANDThis is a great vision and magically melodious Should I suppose the characters (taking part in the masque) are spirits

PROSPEROSpirits which by mine artI have from their confines calld to enactMy present fancies

PROSPEROYes they are spirits whom I have summoned from the regions to which they are confined to carry into effect my fanciful designs

ORIGINAL TEXT PARAPHRASEFERDINANDLet me live here everSo rare a wondered father and a wifeMakes this place paradise

FERDINANDI should like to live here forever Such a wise and wonderful father makes this place a paradise

[Juno and Ceres whisper and send Iris on employment] [Juno and Ceres whisper and send Iris on employment]

PROSPEROSweet now silence

PROSPEROMy dear Ferdinand speak no more Juno and Ceres are

Juno and Ceres whisper seriouslyTheres something else to do Hush and be muteOr else our spell is marred

whispering with a solemn look There is something else coming Silence Or else our magic will be spoilt

IRISYour nymphs called naiads of the wandering brooksWith your sedged crowns and over-harmless looksLeave your crisp channels and on this green landAnswer your summons Juno does commandCome temperate nymphs and help to celebrateA contract of true love Be not too late

IRISYou nymphs called Naiads denizens (M inhabitants) of the running stream with your chaplets of sedge and ever-helpful looks leave your wrinkled channels and on the green land answer the summons sent to you Juno has ordered some chaste nymphs and help to celebrate a noble and true marriage Donrsquot delay

[Enter certain nymphs] [Enter certain nymphs]You sunburnt sicklemen of August wearyCome hither from the furrow and be merryMake holiday your rye-straw hats put onAnd these fresh nymphs encounter every oneIn country footing

You sunburnt harvesters weary from the effects of the heat in August come here from the furrowed land and rejoice Make holiday with your rye-straw hats upon you and meet these fresh nymphs and join in country dancing

[Enter certain reapers properly habited They join with the nymphs in a graceful dance towards the end whereof Prospero starts suddenly and speaks]

[Enter certain reapers properly habited They join with the nymphs in a graceful dance towards the end whereof Prospero starts suddenly and speaks]

PROSPERO[aside] I had forgot that foul conspiracyOf the beast Caliban and his confederatesAgainst my life The minute of their plotIs almost come [to the spirits]Well done Avoidno more

PROSPERO(Aside)I had forgotten the wicked conspiracy of the beast Caliban and his accomplices against my life the time of their plot has almost arrived ndash (To the Spirits) well done depart no more of this

[To a strange hollow and confused noise the spirits heavily vanish]

[The spirits depart]

ORIGINAL TEXT PARAPHRASEFERDINANDLet me live here everSo rare a wondered father and a wifeMakes this place paradise

FERDINANDI should like to live here forever Such a wise and wonderful father makes this place a paradise

[Juno and Ceres whisper and send Iris on employment] [Juno and Ceres whisper and send Iris on employment]

PROSPEROSweet now silenceJuno and Ceres whisper seriouslyTheres something else to do Hush and be muteOr else our spell is marred

PROSPEROMy dear Ferdinand speak no more Juno and Ceres are whispering with a solemn look There is something else coming Silence Or else our magic will be spoilt

IRISYour nymphs called naiads of the wandering brooksWith your sedged crowns and over-harmless looksLeave your crisp channels and on this green landAnswer your summons Juno does commandCome temperate nymphs and help to celebrateA contract of true love Be not too late

IRISYou nymphs called Naiads denizens (M inhabitants) of the running stream with your chaplets of sedge and ever-helpful looks leave your wrinkled channels and on the green land answer the summons sent to you Juno has ordered some chaste nymphs and help to celebrate a noble and true marriage Donrsquot delay

[Enter certain nymphs] [Enter certain nymphs]You sunburnt sicklemen of August wearyCome hither from the furrow and be merryMake holiday your rye-straw hats put onAnd these fresh nymphs encounter every oneIn country footing

You sunburnt harvesters weary from the effects of the heat in August come here from the furrowed land and rejoice Make holiday with your rye-straw hats upon you and meet these fresh nymphs and join in country dancing

[Enter certain reapers properly habited They join with the nymphs in a graceful dance towards the end whereof Prospero starts suddenly and speaks]

[Enter certain reapers properly habited They join with the nymphs in a graceful dance towards the end whereof Prospero starts suddenly and speaks]

PROSPERO[aside] I had forgot that foul conspiracyOf the beast Caliban and his confederatesAgainst my life The minute of their plotIs almost come [to the spirits]Well done Avoidno more

PROSPERO(Aside)I had forgotten the wicked conspiracy of the beast Caliban and his accomplices against my life the time of their plot has almost arrived ndash (To the Spirits) well done depart no more of this

[To a strange hollow and confused noise the spirits heavily vanish]

[The spirits depart]

Ac-12 27420 topic Revaluation of Assets and Liabilities

REVALUATION OF ASSETS AND LIABILITIES

On admission of a new partner the firm stands reconstituted and consequently the assets are revalued and liabilities are reassessed It is necessary to show the true position of the firm at the time of admission of a new partner If the values of the assets are raised gain will increase the capital of the existing partners Similarly any decrease in the value of assets ie loss will decrease the capital of the existing partners For this purpose alsquoRevaluation Accountrsquo is prepared This account is credited with all increases in the value of assets and decrease in the value of liabilities It is debited with decrease on account of value of assets and increase in the value of liabilities The balance of this account shows a gain or loss on revaluation which is transferred to the existing partnerrsquos capital account in existing profit sharing ratioAccounting for Revaluation of Assets and Liabilities when there is a Changein the Profit Sharing Ratio of Existing PartnersAssets and liabilities of a firm must also be revalued at the time of change in profit sharing ratio of existing partners The reason is that the realisable or actual value of assets and liabilities may be different from those shown in the Balance Sheet It is possible that with the passage of time some of the assets might have appreciated in value while the value of certain other assets might have decreased and no record has been made of such changes in the books of accounts Similarly there may be some unrecorded assets amp libilities that may have to be accounted for Revaluation of assets and reassessments of liabilities becomes necessary because the change in the

value of assets and liabilities belongs to the period to change in profit sharing ratio and hence must be shared by the partners in their old profit sharing ratio Revaluation of assets and reassessment of liabilities may be given effect to in two different ways (a) When revised values are to be recorded in the books and(b) When revised values are not to be recorded in the books

When revised values are to be recorded in the booksIn such a case revaluation of assets and reassessment of liabilities is done with the help of a new account called lsquoRevaluation Accountrsquo Sometimes this account is also called as lsquoProfit amp Loss Adjustment Acrsquo If there is a loss due to revaluation revaluation account is debited and if the revaluation results in a profit the revaluation account is credited The following journal entries made for this purpose are

(i) For increase in the value of assetsAsset Ac Dr (individually)To Revaluation Ac(ii) For decrease in the value of AssetRevaluation Ac Dr (individually)To Asset Ac[Decrease in the value of assets](iii) For increase in the value of LiabilitiesRevaluation Ac Dr (individually)To Liabilities Ac[Increase in the value of Liabilities](iv) For decrease in the value of LiabilitiesLiabilities Ac DrTo Revaluation Ac[Decrease in the value of Liabilities](v) For unrecorded AssetsAsset Ac [unrecorded] DrTo Revaluation Ac[Unrecorded asset recorded at actual value](vi) For unrecorded Liability Revaluation Ac DrTo Liability Ac [unrecorded][Unrecorded Liability recorded at actual value](vii) For transfer of gain on revaluationRevaluation Ac DrTo Existing Partnerrsquos CapitalCurrent Ac[Profit on revaluation transferred to capital account in existing ratio](viii) For transfer of loss on revaluationExisting Partnerrsquos CapitalCurrent Ac DrTo Revaluation Ac[Loss on revaluation transferred to capital account in existing ratio](a) When revaluation account shows gain Revaluation Ac DrTo Partnerrsquos Capital Ac (Old Profit Sharing Ratio)(Profit on revaluation credited to Partnerrsquos Capital Ac)(b) Above entry is reversed when revaluation account shows loss Partners Capital Acs (Old Profit Sharing Ratio) DrTo Revaluation Ac(Loss on revaluation debited to Partnerrsquos Capital Acs)

Proforma of Revaluation Account is given as under

Revaluation Account

Dr Cr Particulars ` Amount Particulars ` Amount To Decrease in value of assets By Increase in value of assets To Increase in value of liabilities By Decrease in value of liabilities To Unrecorded liabilities By Unrecorded assets To Gain on Revaluation (Transferred) By Loss on Revalution (Transferred)

ECO ndash12 2742020Topic- ELASTICITY OF DEMAND

CHAPTER - ELASTICITY OF DEMANDMEANINGDemand for a commodity is affected by many factors such as its price price of related goods income of its buyer tastes and preferences etc Elasticity means degree of response Elasticity of demand means degree of responsiveness of demand Demand for a commodity responds to change in price price of related goods income etc So we have three dimensions of elasticity of demandDIMENSION OF ELASTICITY OF DEMAND TYPES OF ELASTICITY OF DEMAND

Price elasticity of demand Income elasticity of demand Cross Elasticity of demand

Price elasticity of demand Price elasticity of demand means degree of responsiveness of demand for a commodity to the change in its price For example if demand for a commodity rises by 10 due to 5 fall in its price Price elasticity of demand (ep)=Percentage change in quantity demanded Percentage change in price of the commodity = 10 ( -)5 = ( - )2Note that ep will always be negative due to inverse relationship of price and quantity demanded

(ii) Income elasticity of demand Income elasticity of demand refers to the degree of responsiveness of demand for a commodity to the change in income of its buyer Suppose income of buyer rises by 10 and his demand for a commodity rises by 20 then Income elasticity of demand (ey)= change in quantity demanded change in price of the commodity =20 10 = 2

Cross Elasticity of demandCross elasticity of demand means the degree of responsiveness of demand for a commodity to the change in price of its related goods (substitute goods or complementary goods) Suppose demand for a commodity rises by 10 due to 5 rise in price of its substitute good then Cross elasticity of demand (ec) = change in quantity demanded change in price of related good = 10 2 = 5 (Tastes and preferences cannot be expressed numerically So elasticity ofdemand cannot be numerically expressed)

  • Chapter 1 Force (Summary)
  • Distinguish between external sovereignty and internal sovereignty
    • NAND Gate
      • Logic diagram
      • Truth Table
        • NOR Gate
          • Logic diagram
          • Truth Table
            • XOR Gate
              • Logic diagram
              • Truth Table
                • XNOR Gate
                  • Logic diagram
                  • Truth Table
                      • Physics
                      • Chapter 1 Electric Field ( Electric Dipole) (Summary)
Page 21:  · Web viewWe all know that Nouns are divided into two parts: common noun and proper noun.Apart from common and proper noun, we will also study about collective noun and compound

promotion create demand of their product in the market and increase it sales Such methods comprises of advertising sales promotion personal selling and publicityToday we are going to discuss about one of such methods It is advertising

Meaning of advertisingAdvertising is a paid form of non-personal presentation for promotion of Ideas goods and services

Importance or merits of advertising Advertising has importance to manufacturer or traders to customer and to society as a whole

Today we will see how advertisement help the manufacturer or traders

Answer) Advertising is a means of how a company encourages people to buy their products services or ideas It is one element of marketing which also includes design Research and data mining

2) Mention any three features of advertisingAnswer)The main features of advertising are

i) It is impersonal form of presentation for promotion of products and services of Ideas

ii) It is issued by identified sponsor The advertisement contains the name of the advertiser

iii) It is a form of mass communication because the message is directed to a large number of persons simultaneously

3) Mention the main merits or importance of advertisement to manufacturer or tradersAnswer)

i) Introducing new product A business organization can introduce itself and its products to the public through advertising

ii) Increase the sale Advertising leads to increase the sale of existing product by entering into new markets and attracting new customers

iii) Create steady demand Advertising creates sustains regular demand by smoothening out seasonal and other fluctuations It enables regular production for the organisation

iv) Economics of scale Advertising facilitate mass distribution of goods and steady demand which lead to large scale and regular production

v) Goodwill Advertising helps in creating a good image of the firm and reputation for its products

Biology Chapter - 03Genetics

Today wewill start chapter and discuss about Genetics Gregor Mendel is known as father of genetics Before entering into Mendelrsquos experiment on Genetics we must know

Q1 Define the following termsi) Genetics Genetics is the study of

transmissionof body features from parents to offspringand the laws relating to such transmission

ii) Heredity It may be defined as transmissionof genetically based characteristics from parentsto offspring

iii) Character and traits Any heritable

Importance to TraderIntroducing new productIncrease the saleCreate steady demandEconomics of scaleGoodwill

some terms featureis a character The alternative forms of acharacter are called traitsex Character (Hair shape) - Traits (Curly straight)

iv) Homologous chromosomes A pair ofcorresponding chromosomes of the same shapeand size one from each parent

v) Genes Genes are the specific parts (DNA segments) of a chromosome which determinethe hereditary characteristicsNearly 30000genes present in human

vi) Alleles Alternative forms of a gene occupying the same position (locus) on homologouschromosomes and affecting the same characteristicbut in different ways

vii) Genotype ndash PhenotypeGenotype means of genes present in the cells of an organism Phenotype means the observable characteristic which is genetically controlled

viii) Mutation It is a sudden change in one or more genes or in the number or in the structure of chromosomes ex Sickle cell anaemia is a blood disease caused by a gene mutation

CLASS NOTES

Class XSubject Eng Literature (The Merchant of Venice ndash William Shakespeare)Topic Act IV Scene 1 Lines 01 to 34 ( Duke helliphelliphellip We all expect a gentle answer Jew) ate 27th April 2020 (2nd Period)

[Students should read the original play and also the paraphrase given in the school prescribed textbook]Summary Questions amp Answers

This scene may be termed as the catastrophe of the play It is the final unravelling of the complicated events which seem to threaten the happiness of Bassanio Portia and Antonio Right is justified to the fullest degree and malice falls into the trap prepared for others No one suffers here but Shylock but even then he receives a generous measure of mercy

o This is the Court-scene Initially we meet

(1)

DUKE I am sorry for thee thou art come to answer (Line 3-6)A stony adversary an inhuman wretchUncapable of pity void and emptyFrom any dram of mercy

(i) Who is addressed here Where is the person Why is the person there

Antonio is addressed hereAntonio is in the court of justice at VeniceAntoniorsquos trial is scheduled to be held here for his failure to meet the conditions of the bond he signed with Shylock

the Duke Antonio Shylock and Salerio Later we meet Bassanio Portia Gratiano and Nerissa

o The Duke says to Antonio that he has to face a very cruel opponent which Antonio admits and expresses his gratefulness to the Duke for his efforts to soften without result the heart of Shylock in order to be merciful to Antonio Antonio further says that he is ready to accept whatever cruel judgement the Court may award

o When Shylock appears in the court the Duke says that Shylock should change his decision of prosecuting Antonio and demanding the penalty specified in the bond out of consideration of the great misfortunes that Antonio has suffered If this is done by him (Shylock) the whole court would be gladdened by his merciful action

(ii) What is the Duke sorry for

The Duke is unable to change the mind of Shylock from his decision to get the bond forfeited even after he pleaded to Shylock Shylock stands firmly for his bond which when forfeited will allow him to take a pound of flesh from any part of Antoniorsquos body(iii) How does the Duke address Shylock`The Duke calls Shylock an adversary with a heart of stones He calls Shylock as an inhuman wretch without pity Shylock is quite lacking in the slightest quality of mercy (iv) How does Antonio reply to this

Antonio replies that he will meet the revenge of Shylock patiently He has prepared himself to suffer with a quiet spirit the utmost that Shylockrsquos tyranny and rage can do(v) What quality of the Duke is revealed here

The Duke is kind and benevolent He is ready to help Antonio He requests Shylock to free Antonio from the trial(vi) What are the terms of the bond that Antonio has signed

The terms of the bond that Antonio has signed were that if Antonio is unable to repay Shylock a certain sum of money specified on the paper on a certain date and in an agreed place the forfeit has to be paid The forfeiture will be an exact pound of Antoniorsquos flesh which Shylock will be a liberty to take from any part of Antoniorsquos body which pleases him

Class XI

STUDY MATERIAL

Class XISubject Eng Literature (The Tempest ndash William Shakespeare) Topic Act I Scene 2 Lines 88 to 132 (Prospero hellip Me and thy crying self) Date 27th April 2020 (3rd Period)

[Students should read the original play and also the paraphrase given in the school prescribed textbook]Summary Questions amp Answers

o Prospero now tells Miranda that he was the Duke of Milan He had been devoting himself more to studies than the affairs of the State His brother Antonio took advantage of this situation and with the help of Alonso the king of Naples seized upon him and her one midnight and shipped them in a frail bark so that they perished in the sea All this took place

(1)

MIRANDA I should sin (Line 118-132)

To think but nobly of my grandmother

Good wombs have borne bad sonsPROSPERO Now the condition

The King of Naples being an enemyTo me inveterate hearkens my brothers suitWhich was that he in lieu othrsquo premisesOf homage and I know not how much tribute

twelve years back

IMPORTANT PASSAGES EXPLAINED(Line 98-103)

PROSPERO helliphelliphelliphelliphelliphelliphellip

He being thus lorded

Not only with what my revenue yieldedBut what my power might else exact like oneWho having into truth by telling of it Made such a sinner of his memoryTo credit his own lie he did believeHe was indeed the duke

Prospero in telling the narrative of his past life here refers to his brother Antonio Prospero being with a studious bent of mind has left the administration of Milan on his younger brother Now Antonio being thus invested like a lord with all the powers derived from Prosperorsquos wealth and what the exercise of Prosperorsquos authority might secure for him regarded himself as a de facto Duke of Milan It is a well-known fact of psychology that a man who repeatedly tells a lie makes of his memory such a sinner against truth as to credit his own lie by the telling of it So Antonio by repeatedly saying to himself and others that he was the Duke came to believe that he was really the Duke Thus falsehood repeatedly asserted gained the force of truth for Antonio and he truly believed it

Should presently extirpate me and mine Out of the dukedom and confer fair MilanWith all the honours on my brother whereonA treacherous army levied one midnightFated to thrsquo purpose did Antonio openThe gates of Milan and ithrsquo dead of darkness The ministers for thrsquo purpose hurried thenceMe and thy crying self

(i) In the earlier lines of this scene what does Prospero tell about his intense interest What was the demand of his interest

In the earlier lines of this scene Prospero tells Miranda that he had an intense interest in the study of philosophy and magic arts Hence in order to improve his mind with this kind of study he kept himself isolated from worldly and state affairs His study was dearer to him than the applause and esteem that he could win from the public His study demanded too much solitude(ii) What forced Antonio to take an undue advantage over Prospero

Prosperorsquos indifferent attitude towards the statersquos affairs and his having boundless trust in Antonio gave rise to a boundless lust for power in Antoniorsquos mind Antonio felt that he must be the actual Duke instead of the part of the Duke he played Thus Antonio took an undue advantage of the situation to usurp Prosperorsquos dukedom(iii) Explain the following lines ldquoI should sin to think but nobly of my grandmother Good wombs have borne bad sonsrdquo

After hearing the treacherous act of her uncle Antonio Miranda says that Prosperorsquos mother was a noble lady and she cannot dishonour her memory by saying that the person named Antonio cannot be his (Prosperorsquos) brother She says that in honour of her grandmother she also cannot say that Antonio must have been begotten not by her grandfather but by some other man She finally concedes that it is known that good mothers have borne bad sons in their wombs and gave birth to them

(iv) Why did the King of Naples accept Antoniorsquos request to help him in usurping his dukedom What did Antonio propose to Alonso

Alonso who was the king of Naples accepted Antoniorsquos request in usurping Prosperorsquos dukedom because he (Alonso) was a sworn enemy of Prospero Antonio proposed that Alonso should immediately drive him (Prospero) and his offspring out of Milan and should confer the dukedom upon him (Antonio) with all the dignities which go with that

position In return Antonio promised that he shall give an annual tribute and also swore his allegiance to Alonso Also he agreed to hold the Dukedom of Milan as a subordinate to the state of Naples(v) How were Prospero and Miranda carried away from the city of Milan and what was the state of small Miranda at that time

In pursuance of the agreement settled between Antonio and Alonso an army of treacherous men was assembled One midnight when the occasion suited the will of destiny Antonio opened the gates of the city of Milan and in the death like silence of midnight Antoniorsquos agents who had been directed to execute his purpose carried Prospero and small Miranda away from the city in all haste They were then forced into a ship and carried some distance out to sea where they put them on a mere hulk of a boat without any rigging or ship-gear and abandoned them leaving them at the mercy of the roaring sea Miranda was a very small child of three years age and she was crying at that time

CLASS -XIDATE-270420Subject Topic Summary Execution

EVS Chapter 1 ndash Mode of Existence

Impact of mode of existence on resources

Q) Why resources are under pressure

Ans - Increase in the sophistication

of technology enabling natural resources to be extracted quickly and efficiently Eg in the past it could take long hours just to cut down one tree only using saws Due to increased technology rates of deforestation have greatly increased

The number of humans is increasing Cultures of consumerism Materialistic views

lead to the mining of gold and diamonds to produce jewelry unnecessary commodities for human life or advancement Consumerism also leads to extraction of resources for the

production of commodities necessary for human life but in amounts excessive of what is needed because people consume more than is necessary or waste what they have

Lack of awareness among the population is striking People are not aware of ways to reduce depletion and exploitation of materials

Accounts Cash Book Today we are going to start a new topic -Cash Book

The key terms used in this chapter are

bullCash book

bullSimple cash book

bullDouble column cash book bullTriple column cash book

bullPetty cash book

bullCash discount

bullContra entry

Here I will share you the meaning of each key terms

bullCash book Cash Book is a special purpose subsidiary book or journal in which cash received and cash payments are recorded

bullSimple cash book

It is a cash book in which only cash transactions are recorded It has only one column on each side

bullTriple column cash book

It is cash book which has three columns one column for each cash and Bankdiscount on each side of the cash book In this book both cash and Bank transactions are recorded together with discount allowed and received

bullPetty cash book

It is a cash book maintained for recording petty expenses

bullCash discount

Cash discount is the amount of discount received or allowed on cash payments and cash receipts Discount received is an income for the business while discount allowed isan expense

bullContra entry

It means transactions involving both cash and Bank Such transactions though recorded in the cash book are not posted into ledger The letter lsquoC is written in Ledger folio for contra entry

Business Studies

ENTREPRENEURSHIP

Now we shall discuss the second chapter

lsquoENTREPRENEURSHIPrsquo

Today before starting the chapter let us recall what

Questions

1What are the main characteristics of Intrapreneurship

Answer

The main characteristics of Intrapreneurship are

Corporate framework-it occurs within the framework of the same company

Semi-Autonomous-Intrapreneurship

we have read last day

Let s today start the class by recalling the last topic taught

Intrapreneurship is the process of discovering and exploring business opportunities within an existing company It involves launching new business ventures within the framework of a present corporation Intrapreneurship is also known as corporate entrepreneurship or corporate venturing

Now let us start with the characteristics of Intrapreneurship

The main characteristics of Intrapreneurship are

Corporate framework

Semi-Autonomous Lack of ownership Senior position Low risk taking Not own boss

Now let us discuss the meaning of enterprise

Enterprise means an undertaking or adventure that requires some innovation and investment and thus involves riskEnterprise always entails decision making coordination and risk bearing

involves crating amd nurturing a semi-autonomous business unit which may be a subsidiary a strategic business unit or a division

Lack of ownership-the intrapreneur is not the owner of the unitb he creates and nurtures

Senior position-he occupies a senior managerial position in the company

Low risk taking-An intrapreneur does not bear the full risk of failure

Not own boss-An intrapreneur is not his own bosss in legal termsHe enjoys the freedom and gets the required resources and support

2 How is Entrepreneur is different from Intrapreneur

The functions involved in both the entrepreneurship and intrapreneurship are by and large similar however there are several differences between the two

Point of distinction

Entrepreneur

Intrapreneur

status An independent business person

A senior executive within a company

Ownership Owner of

the business

An employeesometimes a share in ownership

Financing Responsible for raising finance for the business

Not responsible for raising the finance

Risk bearing

Bears the risk of the business

Does not bears the risk of the business

Reward Profit which is uncertain and irregularcan be loss

Fixed salary and fringe benefits

Need for security low high

3 What do you understand by enterprise

Answer Enterprise means an undertaking or adventure that requires some innovation and investment and thus involves riskEnterprise always entails decision making coordination and risk bearing

COMMERCE NATURE AND OBJECTIVES OF

BUSINESS

Today let us recall the last other two objectives of business by the chart given in the previous class

Firstly we would discuss Human Objectives

Business is run by people and for people Labour is a valuable business element

Human objectives of business are concerned with the well -being of labour

The human objectives are as follows

Labour welfare Developing human

resources Participative

management Labour

management cooperation

Questions

1 Explain the human objectives of a business enterprise

Answer

Business is run by people and for people Labour is a valuable business element

Human objectives of business are concerned with the well -being of labour

The human objectives are as follows

Labour welfare-Business must recognize the dignity of labour and human factors should be given the recognition

Developing human resources-Employees must be provided the opportunities for developing new skills and attitudes

Participative management-Employees should be allowed to take part in decision making process of business

Labour management cooperation-Business should strive for creating and maintaining cordial employer employee relations so as to ensure peace and progress in industry

Now let us discuss the national objectives of business

Optimum utilization of resources

National self- reliance Development of small

scale industries Development of

backward areas Control over pollution

2Explain the national objectives of a business enterprise

Answer

It is the duty of business to utilize the resources of the country properly the national objectives of business

Optimum utilization of resources ndashBusiness should use the nationrsquos resources in the best possible manner

National self- reliance-It is the duty of the business to help the government in increasing experts and in reducing dependence on imports

Development of small scale industries-Big business firms are expected to encourage growth of small scale industries which are necessary for generating employment

Development of backward areas-Business is expected to give preference to the industrialization of backward regions of the country

ECONOMICS

BASIC ECONOMIC CONCEPTS

SUB

TOPIC

Value

Wealth

Welfare

Today we shall start with a new topic of the same chapter ie lsquoValuersquo

Value of a commodity is defined as the valuation placed by a household on the consumption of this commodity

lsquoValuersquo has two different meanings and these are

a Value -in -use It refers to consumption value of a commodity It expresses the utility derived from the consumption of a particular commodity A necessity like water has a very high value ndashin ndashuse or

Question

1What is value

Answer

Value of a commodity is defined as the valuation placed by a household on the consumption of this commodity

2What is value-in use

Answer It refers to consumption value of a commodity It expresses the utility derived from the consumption of a particular commodity A necessity like water has a very high value ndashin ndashuse or consumption value

3What is value ndashin- exchange

Answer It relates to market value of a commodity

It is the rate at which a particular good or service can be exchanged for money

For example in barter system if a person is prepared to exchange 3 metres of cloth with 1 pair

consumption value

b Value ndashin-exchange It relates to market value of a commodity

It is the rate at which a particular good or service can be exchanged for moneyFor example in barter system if a person is prepared to exchange 3 metres of cloth with 1 pair of shoes then the value in exchange of 3 metres of cloth is 1 pair of shoesValue in exchange is the power of purchasing other goods In modern monetised economies the exchange value of goods are expressed in terms of money as prices

Now let us discuss the term lsquoWealthrsquo

Wealth refers to the stock of all those assets which are a source of income

Wealth is a stock concept

Wealth must possess the following features

a Utility It must possess utility or give some

of shoes then the value in exchange of 3 metres of cloth is 1 pair of shoes

Value in exchange is the power of purchasing other goods In modern monetised economies the exchange value of goods are expressed in terms of money as prices

4 What is wealth

Answer it refers to the stock of assets or goods which are a source of income and have personal or national ownership

5 What are the features of wealth

Answer The features of wealth are as follows

Wealth must possess the following features

a Utility It must possess utility or give some satisfaction

b Scarcity It must be limited in quantityc Transferability it should be transferable its

ownership can be transferred from one person to another person

d Exchange value It must possess exchange value

6 What is welfare

Answer

Welfare is defined as satisfaction and happiness a sense of well- being among the people

satisfactionb Scarcity It must be

limited in quantityc Transferability It

should be transferable its ownership can be transferred from one person to another person

d Exchange value It must possess exchange value

Now let us discuss the term lsquoWelfarersquo

Welfare is defined as satisfaction and happiness a sense of well- being among the people

Welfare is affected by factors like

a Consumption of goods and services

b Environment

c Family relations

d Degree of freedom

e Law and order situation

Mathematics Trigonometric equation

To find the general solution of the equation sinθ=0

When sin θ =0

Then θ= 0 π2π 3π-π -2π -3

i e when θ = 0 or an integral multiple of π

i e when θ= nπ where n is any integer

Therefore the general solution of the equation sin

Example1 Find the general values of θ which satisfy the equation sin2 θ =34

Solution sin2 θ= 34

Or sin θ = +34 or -34

Or sin θ = sin π3 or sin (-π3)

Therefore

θ = [nπ + (-1) n (π3)] or[ nπ+ (-1) n (-π3)]

= nπ +π3 or nπ-π3 where n= any integer

Example 2Find the values of θ which satisfy tan2 θ

θ=0 is θ= nπ where n is any integer

To find the general solution of the equation cos θ=0

When cos θ=0

Then θ=π2 3π2 5π2 -π2 -3π2 -5π3

i e when θ is an odd multiple of π2

i e when θ=(2n+1) π2 where n= any integer

Therefore the general solution of the equation cos θ =0 is θ= (2n+1) π2 where n= any integer

To find the general solution of the equation tan θ = 0

Clearly tan θ =0 implies sin θcos θ =0

Therefore θ = nπ

i e the general solution of the equation tan θ=0 is θ =nπ where n = any integer

To find the general solution of the equation cot θ =0

Clearly cot θ =0 implies (cos θsin θ) = 0

i e cos θ =0

Therefore θ = (2n+1) π2

Therefore the general solution of the equation cot θ =0 is θ = (2n+1) π2

Where n= any integer

To find the general solution of the equation sin θ= k (-

=13 -πleθleπ

Solution tan2 θ =13

Or tan θ = plusmn1radic(3) =tan(plusmnπ6)

θ=nπ plusmn π6 where n =any integer

If n=0 then θ=plusmnπ6

If n=1 then θ= π plusmn π6

If n=-1 then θ= -π plusmn π6

Therefore the required solution in -π le θ le π are θ= π6 5π6 -π6 -5π6

Exercise Find general solution of sin 2θ=cos θ [Hints Use sin 2θ= 2sin θcosθ and then take cosθ

common]

1lek le1)

Determine an angle alpha such that sin =k and -π2le αle π2

Then we have

Sin θ = k = sin α

Or sin θ - sin α =0

Or 2 cos [(θ+α) 2] sin [(θ-α) 2] =0

Therefore either cos [(θ +α) 2] =0 (1)

Or sin [(θ-α) 2] =0 (2)

Now from (1) we get (θ+α) 2= (2m+1) π2)

Or θ = (2m+1) π-α (3)

And from (2) we get (θ-α) 2 =mπ

Or θ= 2mπ+α(4)

Where m = any integer

Clearly the solution (3) amp (4) may be combined in the following form

θ= nπ+(-1) n α where n= any integer

Therefore the general solution of sin θ = sin α is θ = nπ +(-1) n α where n is any integer and -π2 le α le π2

Biology Chapter - 04Kingdom Monera

Today we will discuss about bacterial reproduction and its usefulness

Fig Binary Fission

Fig Conjugation Fig Transformation

Bacterial reproduction is mainly asexual but sexual reproduction

also takes place

Asexual reproduction takes place by i) Binary fission - from one bacteriato

two bacteria are produced in every 20 to 30mins

ii) Buddingiii) Endospore formation - during

unfourable condition

Sexual reproduction by three ways

1) Conjugation - Transfer of genetic material between cells that are in physical contact with one another

2) Transduction - Transfer of genetic materialfrom one cell to another by a bacteriophage

3) Transformation - Transfer of cell-freeor naked DNArsquo from one cell to another

Bacteria causes different diseases inplants animals and human and

it causes food spoilage and waterpollution but it also have some useful

activities

i) Bacteria are helpful in sewage water treatment

ii) It is used in antibiotic (medicine) production

iii) Anaerobic bacteria help in biogas(energy) production

iv) Many household products like yoghurt cheese are manufactured by use of bacteria

v) Rhizobium by symbiotic relationship with leguminous plant increase soil fertility

vi) Besides these bacteria is helpful in genetic engineering degradation of petroleum hydrocarbonand in dairy

industry

Physics Motion in plane Here we will introduce Projectile Motion

Execution

Projectile

Y

usinθ u h

θX

ucosθ

Suppose a body is projected with an angle θ So initial velocity u can be resolved into two components

Horizontal component - ucosθ ( for range)

Vertical component - usinθ ( for height)

usinθ changes during motion and becomes zero at maximum height position but ucosθ remain unchanged

The maximum height of projectile is h

NB If initial is upward then g = -ve and if it is downward then g = +ve Height is +ve if direction of motion does not change ( for ex a body thrown upwards but goes down ultimately then height h = -ve)

The angle of projectile θ is the angle made with horizontal

HISTORY ndash GROWTH OF NATIONALISM

SUB TOPIC- REVOLUTIONARY NATIONALISM Bengal formation of Anushilan Samity and Jugantar Group

The intensification of the Swadeshi movement and Government policy of terror and repression led to outbreak of violence Bombs were manufactured and attempts on the lives of unpopular Government officials became frequent In the gymnasium of Scottish Church College which was known as General Assemblies Institution a secret society was formed known as Anushilan Samity

Aurobindo Ghosh send from Baroda his emissary Jatindranath Banerjee to mobilize the Bengal revolutionaries

Hemchandra Qanungo and Satyen Bose published Journal Jugantar

The Jugantar group planned to assassinate oppressive magistrate Kingsford by Khudiram Bose and Prafulla Chaki in 1908 Prafulla Chaki committed suicide to avoid arrest Khudiram was tried and hanged

Afew days later the police found a bomb factory in Maniktala and arrested a large number of revolutionaries The trial of revolutionaries became famous as the Alipore Bomb Case

In the course of the trial the approver the public prosecuter and a police officer were assassinated

1 Question Name two journals which preached the cult of violence

Answer a) Yugantar edited by Bhupendranath DuttaB) Bandemataram edited by Aurobindo Ghosh2 Question Why was Khudiram arrested and hangedAnswer An attempt was made to assassinate a hated vindictive majistrate named Kingsford by Khudiram Bose and Prafulla Chaki Their attempt failed and the bomb they threw killed two English ladies Khudiram was arrested and put to trial and then hanged3Question Who was Aurubindo GhoshAnswer Aurobindo Ghosh a nationalist revolutionary who was charged for his involvement in the Alipore Bombing Case He was accused of it along with his brother Barindra nath Ghosh But Aurobindo was acquitted because of the brilliant pleading of his counsel Chittaranjan Das Then he became a spiritual reformer introducing his visions on human progress and spiritual evolution4 Qustion Who was KingsfordAnswer Kingsford was an unpopular British chief Magistrate who was the target of the bomb thrown at Muzaffarpur by Khudiram and Prafulla Chaki

Most of the accused were convicted and sentenced to

long term of imprisonmentBut

Aurobindo Ghosh was acquitted mainly owing to the brilliant pleading of his counsel Chittaranjan Das

Political science Topic-Sovereignty

Summary Sovereignty is the full right and power of a governing body over itself without any interference from outside sources or bodies In political theory sovereignty is a substantive term designating supreme legitimate authority over some polity In international law sovereignty is the exercise of power by a state

Internal Sovereignty

Internal sovereignty means supreme authority within ones territory while external sovereignty relates to the recognition on the part of all states that each possesses this power in equal measure

External sovereignty

external sovereignty relates to the recognition on the part of all states that each possesses this power in equal measure

Distinguish between

Execution

Answer the following questions

Short notes-

Sovereignty

Internal Sovereignty

External sovereignty

Homework- learn

external sovereignty and internal sovereigntySovereignty is the principle

of supreme and

unquestionable authority

reflected in the claim by the

state to be the sole author of

laws within its territory

Definition of external vs internal sovereigntyInternal sovereignty refers to

the relationship between a

sovereign power and its

subjects ndash it refers to the

location of the supreme

authority within the state In

the UK for example internal

sovereignty (supposedly)

resides within Parliament

reflected in the

constitutional principle of

parliamentary

sovereigntyBy contrast

external sovereignty refers

to the capacity of the state

to act independently and

autonomously on the world

stage This is what is

sometimes called lsquostate

sovereigntyrsquo or lsquonational

sovereigntyrsquo and implies

that states are legally equal

and that the territorial

integrity and political

independence of a state is

inviolable

Class ndash XII

Date - 2742020 STUDY MATERIALSubject Topic Summary Execution Business Studies

Job Analysis amp Manpower Planning

At first let us recall the chapter what we have discussed till nowbullJob analysisbullJob specification bullJob description bullJob enlargement bullJob enrichment

Today we will do some questions answers from the chapter

Questions 1ldquoJob analysis job description and job specification are interrelatedrdquo Comment Answer) Job analysis is a systematic and detailed examination of a job to collect all the relevant information about it The contents off the job are summarised in the job description The qualification needed for the job are summarised in job specificationThus there is close interrelationship between job analysis job description and job specification

Question 2ldquoJob enlargement is a horizontal extension of a job whereas job enrichment is a vertical extension of a jobrdquo ElucidateAnswer) Job enlargement involves adding one or more task to a job coma where as job enrichment involves adding more autonomy and responsibility to a job Job enlargement is therefore horizontal extension of a job coma whereas job enrichment is a vertical extension of a job

Question 3 )

What is manpower estimation Explain its quantitative and qualitative aspectsAnswer) Manpower estimation is the process by which management determines how an organisation should move from itrsquos current manpower positionto its desired manpower position There are two dimensions of Manpower estimation- quantitative and qualitative

Quantitative aspectThis aspect of Manpower estimation involves estimating the number of employees required in a future time period Workload analysis and workforce analysis are done to estimate the quantity of required manpower

Qualitative aspectThe estimate of the knowledge skills experience etc of required manpower is the qualitative aspect of Manpower estimation The quality of Manpower can be judged on the basis of job analysisand job specification

COMMERCE

CAPITAL-FIXED AND WORKING

Today let us start the class by discussing the sources of finance for different types of business firms

The term lsquocapitalrsquo refers to the investment made in the enterprise for the purpose of earning profits

Requirements of capital and sources of capital for different types of business firms are

1 Capital for sole proprietorship businessA sole proprietor operates at a small scale and thereforerequires a limited amount of capital

2 Capital for partnership firmCapital requirements as well as capital base of a partnership is bigger than that of a sole trader businessThe owned capital is contributed by the partners in an agreed ratio

3 Capital for joint stock companyA joint stock company generally requires large amount of capitalA public company can raise huge capital through issue of shares In addition to share capital it can utilize retained profits

Now let u discuss the meaning of Finance PlanningFinance planning is the process of estimation the financial requirements of an organization specifying the sources of firms and ensuring that enough funds are available at the right time

1 What do you mean by Finance PlanningAnswer Finance planning is the process of estimation the financial requirements of an organization specifying the sources of firms and ensuring that enough funds are available at the right time

2Discuss the role of financial planning of an enterpriseThe role of financial planning are as followsa A sound financial plan helps a business enterprise to avaid the problems of shortage and surplus of fundsbFinancial planning serves as a guide in developing a sound capital structure so as to maximize returns to shareholders c It helps in effective utilization of fundsd It provides policies and procedures for coordinating different functional areas or departments of businesse It enables the management to exercise effective control over the financial activities of an enterprisef It helps the company to prepare for facing business shocks and surprises in future

Mathematics

Continuity and differentiability

Recall Definition of ContinuityLet f(x) be a single valued function of x and x=a be a point in the domain of definition of the function The function is said to be continuous at x=a ifi) f(c) is defined ie f(x) has a definite finite value at x=cii) lim xrarra f(x) exists andiii) lim xrarra f(x) =f(a) In other words f(x) is said to be continuous at x=a if lim xrarra+ f(x)= lim xrarra- f(x) = f(a) Or f(a+0) =f(a-0) =f(a) Or lim hrarr0 f(a+h)= f(a) Algebra of continuous functionsNow we will study some algebra of continuous functions Theorem 1 Suppose f and g be two real functions continuous at a real number c Then(1) f + g is continuous at x = c(2) f ndash g is continuous at x = c(3) f g is continuous at x = c(4) (fg) is continuous at x = c (provided g (c) ne 0)

Example 1 Prove that every rational function is continuousSolution Recall that every rational function f is given byf(x)=[p(x) q(x) ] q(x)ne0where p and q are polynomial functions The domain of f is all real numbers except points at which q is zero Since polynomial functions are continuous f is continuous by (4) of Theorem 1Example 2Discuss the continuity of sine functionSolution To see this we use the following factslim xrarr0 sin x =0Now observe that f (x) = sin x is defined for every real number Let c be a real number Put x = c + h If x rarr c we know that h rarr 0 Therefore lim xrarrc f(x)

= lim xrarrc sin x= lim hrarr0 sin(c+h) =lim hrarr0 [sin c cos h + cos c sin h ]=lim hrarr0 (sin c cos h) + lim hrarr0 (cos c sin h) = sin c +0=sin c = f(c) Thus lim xrarrc f(x) = f(c) and hence f is a constant function Exercise Prove that the function f(x) = x2 +2x is continuous for every real value of x [Hints show that lim xrarra+ f(x) = lim xrarra- f(x) = f(a) ]

Biology Reproduction in Flowering plants We will discuss about megasporoangium

megasporagenesis and female gametophyte

Q4 Describe the structure of megasporangium

Ovule is attached to the placenta by astalk called funicle

Each ovule has one two or three protectivecoverings called integuments

At the tip of integuments a small openingcalled micropyle is organised

Opposite to the micropylar end is the chalaza

Within the integuments a mass of cellsnucellusand inside it embryo sac orfemale gametophyte is present

Q5 Describe a mature embryosacamp its formation

In most of the flowering plants only oneof the 4 megaspores formed as a result ofmegasporogenesis that is functional while theother three degenerate

The

functional megaspore develops into thefemale gametophyte

Formation The nucleus

of the functional megasporedivides mitotically to form two nuclei first andthen two more sequential mitotic nucleardivisions result in the formation of four ampthen eight nucleate stages of embryo sac

Six of the eight nucleus are surrounded bycell walls and organised into cells

The remaining two nuclei called polar nuclei are found below the egg apparatus in the largecentral cell

Three cells consisting of two synergids amp one egg cell present bottom of

embryo sac Three cells

at the chalazal as antipodal cells

Two polar nuclei together present in large central cell

HISTORY

TOWARDS INDEPENDENCE AND PARTITION THE LAST PHASE(1935-1947)SUB TOPIC NATIONAL MOVEMENTS DURING THE SECOND WORLD WAR

Spread of Quit India Movement On 9th August 1942Gandhiji and other Congress leaders were arrested The Congress was declared illegal The news of the arrest of all leaders marked the beginning of a widespread movement of India It was not possible for such a movement to remain peacefulBut the arrest of the all notable congress leaders virtually left the movement in the hands of the mass The movement took the form of violent and militant outbreakBesides congressmen revolutionaries also were very active in the movement The Congress Socialist group also played a prominent role

1 Question Why did the British authority arrest the Congress leaders on 9 th August 1942Answer Congress Working committee adopted the Quit India resolution which was to be ratified at the Bombay AICC meeting in 8th August 1942 They decided to launch a mass struggle on non-violent lines Gandhiji gave a clarion call to all section of the people rdquoKarenge ya Marengerdquo (do or die) Congress leaders gave the call to driving out

the British from IndiaViceroy had taken strong action against the Quit India movement Gandhiji and all the leaders of Congress were arrested

2 Question How did Quit India Movement spread out all over IndiaAnswer The news of the leaders lsquo arrest marked the beginning of a widespread movement to remain peacefulThe movement took form of violent outbreak There were widespread cutting of telephone and Telegraph wires damaging railway lines raising barricades in cities and towns and other forms of violent demonstations

Question Name the leaders of Congress

Socialist group played a prominent part Notable among the Jayprakash Narayan Rammonohar Lohia Aruna Asaf Ali

Political science

Topic-Franchise and Representation

Summary

The election commission

The Election Commission of India is an autonomous constitutional authority responsible for administering Union and State election processes in India The body administers elections to the Lok Sabha Rajya Sabha State Legislative Assemblies in India and the offices of the President and Vice President in the country

Functions of election commission-

India is a sovereign socialist secular democratic republic Democracy runs like a golden thread in the social economic and political fabric woven by the Constitution given by lsquoWe the People of Indiarsquo unto ourselves The concept of democracy as visualised by the Constitution pre-supposes the representation of the people in Parliament and State legislatures by the method of election The Supreme Court has held that democracy is one of the inalienable basic features of the Constitution of India and forms part of its basic structure The Constitution of India adopted a Parliamentary form of government Parliament consists of the President of India and the two Houses mdash Rajya Sabha and Lok Sabha India being a Union of states has separate state legislatures for each state State legislatures consist of the Governor and two Houses mdash Legislative Council and Legislative Assembly mdash in seven states namely Andhra Pradesh Telangana Bihar Jammu amp Kashmir Karnataka Maharashtra and Uttar Pradesh and of the Governor and the state Legislative Assembly in the remaining 22 states Apart from the above two out of the seven Union Territories namely National Capital Territory of Delhi and Puducherry also have their Legislative Assemblies

ExecutionShort notes-Election commissionFunctions of election commission

Homework- Learn

Computer

Science

Computer hardware NAND Gate

A NOT-AND operation is known as NAND operation It has n input (n gt= 2) and one output

Logic diagram

Truth Table

NOR Gate

A NOT-OR operation is known as NOR operation It has n input (n gt= 2) and one output

Logic diagram

Truth Table

XOR Gate

XOR or Ex-OR gate is a special type of gate It can be used in the half

adder full adder and subtractor The exclusive-OR gate is abbreviated as EX-OR gate or sometime as X-OR gate It has n input (n gt= 2) and one output

Logic diagram

Truth Table

XNOR Gate

XNOR gate is a special type of gate It can be used in the half adder full adder and subtractor The exclusive-NOR gate is abbreviated as EX-NOR gate or sometime as X-NOR gate It has n input (n gt= 2) and one output

Logic diagram

Truth Table

Physics

Chapter 1 Electric Field ( Electric Dipole) (Summary)

Here we will derive Expression of electric field at broad side

On position of dipole

Execution

Q With the help of a labelled diagram obtain an expression for the electric field intensity E at any point on the equitorial line ( broad-side on position) of an electric dipole

Ans

E1 E1sinθ

E θ P E1 θ

( r2+L2)12 E2 E

r E2 E2sinθ

-q θ L O L +qA B

Let us consider that the point P is situated on the right bisector of the dipole AB at a distance r meter from its midpoint O

Let E1 and E2 be the electric field intensities of the electric field at P due to charge +q and ndashq of the dipole resp The distance of P from each charge is ( r2+L2)12

So E1 = 14 πϵ q

(r 2+L 2) away from +q

E2 = 14 πϵ q

(r 2+L 2) towards ndashq

The magnitudes of E1 and E2 are equal but directions are different Now resolving E1 and E2 into two components parallel and perpendicular to AB we get

The components perpendicular to AB E1sinθ and E2sinθ cancel each other because they are equal and opposite

The components parallel to AB are E1cosθ and E2 cosθ are in same direction and add up

So resultant intensity of electric field at the point P is

E = E1cosθ + E2 cosθ

E = 14 πϵ q

(r 2+L 2) 2 cosθ

Now from fig we have cosθ =BOBP = L (r2+L2)12

So we get E = 14 πϵ 2qL ( r2+L2)32

Now electric dipole moment p= 2qL

So E = 14 πϵ p ( r2+L2)32

HW Find the expression of Electric field as done here but this time take r gtgt 2L

Also find the expression of torque experience by a dipole

(Hint Electric force experienced by charges of dipole in electric field is qE each Let θ be the angle which dipole makes with electric lines of force then perpendicular distance between two charges is 2Lsinθ Then torque = force x perp distance = qE x 2L sinθ So τ=pE sinθ where p =2qL )

STUDY MATERIAL

Class XIISubject Eng Literature (The Tempest ndash William Shakespeare) Topic Act IV Scene 1 Lines 84 to 133 (Iris hellip A contract of true love Be not too late ) Date 27th April 2020 (4th Period)

[Students should read the original play and also the paraphrase given in the school prescribed textbook]Summary Questions amp Answers

o Ceres soon appears and comes to know that she has been summoned to celebrate the contract of true love

o Ceres expresses her unwillingness to meet Venus and Cupid as she has shunned their company

o Ceres and Juno both bestow their blessings upon Ferdinand and Miranda with June gifting honour riches happiness in marriage and Ceres presents plenty of earthrsquos produce

o Iris summons the water-nymphs and reapers to come and celebrate a contract

(1) IRIS Of her society (Line 91-101)

Be not afraid I met her deity

Cutting the clouds towards Pathos and her sonDove-drawn with her Here thought they to have doneSome wanton charm upon this man and maidWhose vows are that no bed-right shall be paidTill Hymens torch be lightedmdashbut in vainMarss hot minion is returned againHer waspish-headed son has broke his arrowsSwears he will shoot no more but play with sparrowsAnd be a boy right out

(i) Where were Venus and Cupid seen flying How were they travelling Why did they want to join the marriage celebration of Ferdinand and Miranda

of true love

Venus and Cupid were seen flying through the air towards Paphos the famous city which is situated on the island of Cyprus They were travelling by air-borne chariot drawn by doves They certainly wanted to come here in order to play some amorous trick upon Ferdinand and Miranda who are under a vow not to gratify their physical desires till the holy ceremony of their marriage has been performed(ii) What have Venus and Cupid done after failing in their plan

After being failure of their plan Venus who is a very passionate deity and who is the mistress of Mars (the god of war) has gone back while here ill-tempered son Cupid has broken his arrows of love in his state of desperation(iii) What has Cupid firmly decided

Cupid is feeling so disappointed that he has firmly decided to shoot no more arrows to arouse love in human hearts but to spend his time playing with sparrows Thus he would now become just a boy and would give up his original function of shooting arrows on human beings to make them fall in love(iv) What vow had Ceres taken How did Ceres feel at the abduction

After the abduction of her daughter Prosperina by Pluto Ceres had taken a vow to always keep away from the disgraceful company of Venus and her blind son Cupid the god of love Ceres felt deeply distressed when Pluto had carried off her daughter and had made her his wife by force(v) Why has Ceres not forgiven Venus and her blind son For what do Ceres want to be sure

As the abduction had been manipulated by Venus the goddess of beauty and love and her blind son Cupid Ceres has never forgiven them for their part in the whole plot Ceres wants to be sure that she would not have to meet Venus and Cupid who had engineered the abduction of her daughter Prosperina

AS THIS lsquoMASQUErsquo SCENE IS VERY IMPORTANT IN THE PLAY THE PARAPHRASE OF THE ENTIRE PORTION OF MASQUE SCENE (Act IV Lines 58 to 143) IS GIVEN BELOW

IRIS Goddess of RainbowCERES Goddess of Agriculture and all the fruits of the earth

(Nature growth prosperity rebirth ndash notions intimately connected to marriage)JUNO The majestic Queen of Heavens and wife of Jupiter (Jupiter is the king of Gods)

VENUS The Goddess of love CUPID Son of Venus PLUTO God of death (In the play referred by Shakespeare as lsquoDisrsquo which is a Roman name for Pluto)

ORIGINAL TEXT PARAPHRASEPROSPEROWellmdash

PROSPERONow come Ariel Let there be too many rather than too few

Now come my Ariel Bring a corollaryRather than want a spirit Appear and pertly[to Ferdinand and Miranda]No tongue all eyes Be silent

spirits in attendance Appear briskly

[to Ferdinand and Miranda]Look with your eyes but do not say a word

[Soft music] [Soft music][Enter Iris] [Enter Iris]

IRISCeres most bounteous lady thy rich leasOf wheat rye barley vetches oats and peasThy turfy mountains where live nibbling sheepAnd flat meads thatched with stover them to keepThy banks with pioned and twilled brimsWhich spongy April at thy hest betrimsTo make cold nymphs chaste crowns and thybroom-grovesWhose shadow the dismissegraved bachelor lovesBeing lass-lorn thy pole clipped vineyardAnd thy sea-marge sterile and rocky-hardWhere thou thyself dost airmdashthe Queen othrsquoSkyWhose watery arch and messenger am IBids thee leave these and with her sovereign grace[Juno appears] Here on this grass-plot in this very placeTo come and sport Her peacocks fly amainApproach rich Ceres her to entertain

IRISCeres most generous lady you are the cause of rich fields or fertile land where wheat rye barley beans oats and peas grow the grassy mountains where the sheep graze and the flat meadows covered with coarse hay to be used as fodder for cattleYour banks are covered with marsh-marigolds and reeds and the rainy April under your orders brings forth to make for the maids who are not in love beautiful crowns your woods where the broom flourishes and where the bachelor who has been dismissed by the maid he loved lies down being forsaken your vineyard in which the poles are embraced by the vines and the margin of the sea which is barren and rocky where you roam about to enjoy the fresh air ndash the queen of the sky (Juno) whose messenger I am besides being represented as the rainbow bids you leave all these and with her majesty here on this grassy plot in this very place come and sport her peacocks carry her fast in her chariot through the air and are making their way here approach rich Ceres to welcome her

[Enter Ariel as Ceres] [Enter Ariel as Ceres]

CERESHail many-coloured messenger that neerDost disobey the wife of JupiterWho with thy saffron wings upon my flowersDiffusest honey-drops refreshing showersAnd with each end of thy blue bow dost crownMy bosky acres and my unshrubbed downRich scarf to my proud earth Why hath thy queenSummoned me hither to this short-grassed green

CERESWelcome rainbow that never dared disobey Juno the wife of Jupiter who with your orange coloured rays spread honey-drops refreshing showers And with each end of thy blue bow drown my bushy acres and my hilly country which is free from shrubs you thus forming a rich scarf Why has your queen called me here to this place covered with short grass

IRISA contract of true love to celebrateAnd some donation freely to estateOn the blest lovers

IRISI have called you to celebrate a contract of true love and bestow some liberal gift upon the blessed lovers

ORIGINAL TEXT PARAPHRASECERESTell me heavenly bowIf Venus or her son as thou dost knowDo now attend the queen Since they did plotThe means that dusky Dis my daughter gotHer and her blind boys scandaled companyI have forsworn

CERESTell me heavenly bow if Venus the Goddess of love or Cupid her son and pedlar of passion at this time attend the heavenly queen Juno because you are sure to know Since the day they conspired against me and dark Pluto took away my daughter here and Cupidrsquos disgraceful company I have left off

IRISOf her societyBe not afraid I met her deityCutting the clouds towards Pathos and her sonDove-drawn with her Here thought they to have doneSome wanton charm upon this man and miad

IRISBe not afraid of her company I met her deity moving on the clouds towards Paphos the sacred home of Venus on the island of Cyprus along with her son on her chariot drawn by doves Here they contemplated to exercise a charm upon this man and maid producing

Whose vows are that no bed-right shall be paidTill Hymens torch be lightedmdashbut in vainMarss hot minion is returned againHer waspish-headed son has broke his arrowsSwears he will shoot no more but play with sparrowsAnd be a boy right out

wantonness before the actual marriage ceremony but did not succeed Venus has returned her irritable son has broken his arrows and swears that he will give up his practice of trying to inspire love but play with sparrows and be a boy again

[Music is heard] [Music is heard]

CERESHighst queen of stateGreat Juno comes I know her by her gait

CERESHighest queen of state Great Juno there she comes I know here by her gait

[Enter Juno] [Enter Juno]

JUNOHow does my bounteous sister Go with meTo bless this twain that they may Prosperous beAnd honoured in their issue

JUNOHow are you doing my generous sister Come with me to bless this couple so that they may be prosperous and fortunate in their children

[They sing] [They sing]

JUNOHonour riches marriage-blessingLong continuance and increasingHourly joys be still upon youJuno sings her blessings upon you

JUNOMay honour riches happiness in marriage long continuance and increase of those boons ever rest upon you as hourly joys Juno showers down upon you her blessings in song

CERESEarths increase foison plentyBarns and garners never emptyVines and clustring bunches growingPlants and goodly burden bowingSpring come to you at the farthestIn the very end of harvestScarcity and want shall shun youCeresrsquo blessing so is on you

CERESMay you have the plenty of earthrsquos produce Your barns and granaries may never be empty Your vines may grow with clustering bunches Your fruit trees may be heavily laden with their fruit May there be continuous spring and harvest May scantiness and want leave you forever Such is the blessing of Ceres upon you

FERDINANDThis is a most majestic vision andHarmoniously charmingly May I be boldTo think these spirits

FERDINANDThis is a great vision and magically melodious Should I suppose the characters (taking part in the masque) are spirits

PROSPEROSpirits which by mine artI have from their confines calld to enactMy present fancies

PROSPEROYes they are spirits whom I have summoned from the regions to which they are confined to carry into effect my fanciful designs

ORIGINAL TEXT PARAPHRASEFERDINANDLet me live here everSo rare a wondered father and a wifeMakes this place paradise

FERDINANDI should like to live here forever Such a wise and wonderful father makes this place a paradise

[Juno and Ceres whisper and send Iris on employment] [Juno and Ceres whisper and send Iris on employment]

PROSPEROSweet now silence

PROSPEROMy dear Ferdinand speak no more Juno and Ceres are

Juno and Ceres whisper seriouslyTheres something else to do Hush and be muteOr else our spell is marred

whispering with a solemn look There is something else coming Silence Or else our magic will be spoilt

IRISYour nymphs called naiads of the wandering brooksWith your sedged crowns and over-harmless looksLeave your crisp channels and on this green landAnswer your summons Juno does commandCome temperate nymphs and help to celebrateA contract of true love Be not too late

IRISYou nymphs called Naiads denizens (M inhabitants) of the running stream with your chaplets of sedge and ever-helpful looks leave your wrinkled channels and on the green land answer the summons sent to you Juno has ordered some chaste nymphs and help to celebrate a noble and true marriage Donrsquot delay

[Enter certain nymphs] [Enter certain nymphs]You sunburnt sicklemen of August wearyCome hither from the furrow and be merryMake holiday your rye-straw hats put onAnd these fresh nymphs encounter every oneIn country footing

You sunburnt harvesters weary from the effects of the heat in August come here from the furrowed land and rejoice Make holiday with your rye-straw hats upon you and meet these fresh nymphs and join in country dancing

[Enter certain reapers properly habited They join with the nymphs in a graceful dance towards the end whereof Prospero starts suddenly and speaks]

[Enter certain reapers properly habited They join with the nymphs in a graceful dance towards the end whereof Prospero starts suddenly and speaks]

PROSPERO[aside] I had forgot that foul conspiracyOf the beast Caliban and his confederatesAgainst my life The minute of their plotIs almost come [to the spirits]Well done Avoidno more

PROSPERO(Aside)I had forgotten the wicked conspiracy of the beast Caliban and his accomplices against my life the time of their plot has almost arrived ndash (To the Spirits) well done depart no more of this

[To a strange hollow and confused noise the spirits heavily vanish]

[The spirits depart]

ORIGINAL TEXT PARAPHRASEFERDINANDLet me live here everSo rare a wondered father and a wifeMakes this place paradise

FERDINANDI should like to live here forever Such a wise and wonderful father makes this place a paradise

[Juno and Ceres whisper and send Iris on employment] [Juno and Ceres whisper and send Iris on employment]

PROSPEROSweet now silenceJuno and Ceres whisper seriouslyTheres something else to do Hush and be muteOr else our spell is marred

PROSPEROMy dear Ferdinand speak no more Juno and Ceres are whispering with a solemn look There is something else coming Silence Or else our magic will be spoilt

IRISYour nymphs called naiads of the wandering brooksWith your sedged crowns and over-harmless looksLeave your crisp channels and on this green landAnswer your summons Juno does commandCome temperate nymphs and help to celebrateA contract of true love Be not too late

IRISYou nymphs called Naiads denizens (M inhabitants) of the running stream with your chaplets of sedge and ever-helpful looks leave your wrinkled channels and on the green land answer the summons sent to you Juno has ordered some chaste nymphs and help to celebrate a noble and true marriage Donrsquot delay

[Enter certain nymphs] [Enter certain nymphs]You sunburnt sicklemen of August wearyCome hither from the furrow and be merryMake holiday your rye-straw hats put onAnd these fresh nymphs encounter every oneIn country footing

You sunburnt harvesters weary from the effects of the heat in August come here from the furrowed land and rejoice Make holiday with your rye-straw hats upon you and meet these fresh nymphs and join in country dancing

[Enter certain reapers properly habited They join with the nymphs in a graceful dance towards the end whereof Prospero starts suddenly and speaks]

[Enter certain reapers properly habited They join with the nymphs in a graceful dance towards the end whereof Prospero starts suddenly and speaks]

PROSPERO[aside] I had forgot that foul conspiracyOf the beast Caliban and his confederatesAgainst my life The minute of their plotIs almost come [to the spirits]Well done Avoidno more

PROSPERO(Aside)I had forgotten the wicked conspiracy of the beast Caliban and his accomplices against my life the time of their plot has almost arrived ndash (To the Spirits) well done depart no more of this

[To a strange hollow and confused noise the spirits heavily vanish]

[The spirits depart]

Ac-12 27420 topic Revaluation of Assets and Liabilities

REVALUATION OF ASSETS AND LIABILITIES

On admission of a new partner the firm stands reconstituted and consequently the assets are revalued and liabilities are reassessed It is necessary to show the true position of the firm at the time of admission of a new partner If the values of the assets are raised gain will increase the capital of the existing partners Similarly any decrease in the value of assets ie loss will decrease the capital of the existing partners For this purpose alsquoRevaluation Accountrsquo is prepared This account is credited with all increases in the value of assets and decrease in the value of liabilities It is debited with decrease on account of value of assets and increase in the value of liabilities The balance of this account shows a gain or loss on revaluation which is transferred to the existing partnerrsquos capital account in existing profit sharing ratioAccounting for Revaluation of Assets and Liabilities when there is a Changein the Profit Sharing Ratio of Existing PartnersAssets and liabilities of a firm must also be revalued at the time of change in profit sharing ratio of existing partners The reason is that the realisable or actual value of assets and liabilities may be different from those shown in the Balance Sheet It is possible that with the passage of time some of the assets might have appreciated in value while the value of certain other assets might have decreased and no record has been made of such changes in the books of accounts Similarly there may be some unrecorded assets amp libilities that may have to be accounted for Revaluation of assets and reassessments of liabilities becomes necessary because the change in the

value of assets and liabilities belongs to the period to change in profit sharing ratio and hence must be shared by the partners in their old profit sharing ratio Revaluation of assets and reassessment of liabilities may be given effect to in two different ways (a) When revised values are to be recorded in the books and(b) When revised values are not to be recorded in the books

When revised values are to be recorded in the booksIn such a case revaluation of assets and reassessment of liabilities is done with the help of a new account called lsquoRevaluation Accountrsquo Sometimes this account is also called as lsquoProfit amp Loss Adjustment Acrsquo If there is a loss due to revaluation revaluation account is debited and if the revaluation results in a profit the revaluation account is credited The following journal entries made for this purpose are

(i) For increase in the value of assetsAsset Ac Dr (individually)To Revaluation Ac(ii) For decrease in the value of AssetRevaluation Ac Dr (individually)To Asset Ac[Decrease in the value of assets](iii) For increase in the value of LiabilitiesRevaluation Ac Dr (individually)To Liabilities Ac[Increase in the value of Liabilities](iv) For decrease in the value of LiabilitiesLiabilities Ac DrTo Revaluation Ac[Decrease in the value of Liabilities](v) For unrecorded AssetsAsset Ac [unrecorded] DrTo Revaluation Ac[Unrecorded asset recorded at actual value](vi) For unrecorded Liability Revaluation Ac DrTo Liability Ac [unrecorded][Unrecorded Liability recorded at actual value](vii) For transfer of gain on revaluationRevaluation Ac DrTo Existing Partnerrsquos CapitalCurrent Ac[Profit on revaluation transferred to capital account in existing ratio](viii) For transfer of loss on revaluationExisting Partnerrsquos CapitalCurrent Ac DrTo Revaluation Ac[Loss on revaluation transferred to capital account in existing ratio](a) When revaluation account shows gain Revaluation Ac DrTo Partnerrsquos Capital Ac (Old Profit Sharing Ratio)(Profit on revaluation credited to Partnerrsquos Capital Ac)(b) Above entry is reversed when revaluation account shows loss Partners Capital Acs (Old Profit Sharing Ratio) DrTo Revaluation Ac(Loss on revaluation debited to Partnerrsquos Capital Acs)

Proforma of Revaluation Account is given as under

Revaluation Account

Dr Cr Particulars ` Amount Particulars ` Amount To Decrease in value of assets By Increase in value of assets To Increase in value of liabilities By Decrease in value of liabilities To Unrecorded liabilities By Unrecorded assets To Gain on Revaluation (Transferred) By Loss on Revalution (Transferred)

ECO ndash12 2742020Topic- ELASTICITY OF DEMAND

CHAPTER - ELASTICITY OF DEMANDMEANINGDemand for a commodity is affected by many factors such as its price price of related goods income of its buyer tastes and preferences etc Elasticity means degree of response Elasticity of demand means degree of responsiveness of demand Demand for a commodity responds to change in price price of related goods income etc So we have three dimensions of elasticity of demandDIMENSION OF ELASTICITY OF DEMAND TYPES OF ELASTICITY OF DEMAND

Price elasticity of demand Income elasticity of demand Cross Elasticity of demand

Price elasticity of demand Price elasticity of demand means degree of responsiveness of demand for a commodity to the change in its price For example if demand for a commodity rises by 10 due to 5 fall in its price Price elasticity of demand (ep)=Percentage change in quantity demanded Percentage change in price of the commodity = 10 ( -)5 = ( - )2Note that ep will always be negative due to inverse relationship of price and quantity demanded

(ii) Income elasticity of demand Income elasticity of demand refers to the degree of responsiveness of demand for a commodity to the change in income of its buyer Suppose income of buyer rises by 10 and his demand for a commodity rises by 20 then Income elasticity of demand (ey)= change in quantity demanded change in price of the commodity =20 10 = 2

Cross Elasticity of demandCross elasticity of demand means the degree of responsiveness of demand for a commodity to the change in price of its related goods (substitute goods or complementary goods) Suppose demand for a commodity rises by 10 due to 5 rise in price of its substitute good then Cross elasticity of demand (ec) = change in quantity demanded change in price of related good = 10 2 = 5 (Tastes and preferences cannot be expressed numerically So elasticity ofdemand cannot be numerically expressed)

  • Chapter 1 Force (Summary)
  • Distinguish between external sovereignty and internal sovereignty
    • NAND Gate
      • Logic diagram
      • Truth Table
        • NOR Gate
          • Logic diagram
          • Truth Table
            • XOR Gate
              • Logic diagram
              • Truth Table
                • XNOR Gate
                  • Logic diagram
                  • Truth Table
                      • Physics
                      • Chapter 1 Electric Field ( Electric Dipole) (Summary)
Page 22:  · Web viewWe all know that Nouns are divided into two parts: common noun and proper noun.Apart from common and proper noun, we will also study about collective noun and compound

some terms featureis a character The alternative forms of acharacter are called traitsex Character (Hair shape) - Traits (Curly straight)

iv) Homologous chromosomes A pair ofcorresponding chromosomes of the same shapeand size one from each parent

v) Genes Genes are the specific parts (DNA segments) of a chromosome which determinethe hereditary characteristicsNearly 30000genes present in human

vi) Alleles Alternative forms of a gene occupying the same position (locus) on homologouschromosomes and affecting the same characteristicbut in different ways

vii) Genotype ndash PhenotypeGenotype means of genes present in the cells of an organism Phenotype means the observable characteristic which is genetically controlled

viii) Mutation It is a sudden change in one or more genes or in the number or in the structure of chromosomes ex Sickle cell anaemia is a blood disease caused by a gene mutation

CLASS NOTES

Class XSubject Eng Literature (The Merchant of Venice ndash William Shakespeare)Topic Act IV Scene 1 Lines 01 to 34 ( Duke helliphelliphellip We all expect a gentle answer Jew) ate 27th April 2020 (2nd Period)

[Students should read the original play and also the paraphrase given in the school prescribed textbook]Summary Questions amp Answers

This scene may be termed as the catastrophe of the play It is the final unravelling of the complicated events which seem to threaten the happiness of Bassanio Portia and Antonio Right is justified to the fullest degree and malice falls into the trap prepared for others No one suffers here but Shylock but even then he receives a generous measure of mercy

o This is the Court-scene Initially we meet

(1)

DUKE I am sorry for thee thou art come to answer (Line 3-6)A stony adversary an inhuman wretchUncapable of pity void and emptyFrom any dram of mercy

(i) Who is addressed here Where is the person Why is the person there

Antonio is addressed hereAntonio is in the court of justice at VeniceAntoniorsquos trial is scheduled to be held here for his failure to meet the conditions of the bond he signed with Shylock

the Duke Antonio Shylock and Salerio Later we meet Bassanio Portia Gratiano and Nerissa

o The Duke says to Antonio that he has to face a very cruel opponent which Antonio admits and expresses his gratefulness to the Duke for his efforts to soften without result the heart of Shylock in order to be merciful to Antonio Antonio further says that he is ready to accept whatever cruel judgement the Court may award

o When Shylock appears in the court the Duke says that Shylock should change his decision of prosecuting Antonio and demanding the penalty specified in the bond out of consideration of the great misfortunes that Antonio has suffered If this is done by him (Shylock) the whole court would be gladdened by his merciful action

(ii) What is the Duke sorry for

The Duke is unable to change the mind of Shylock from his decision to get the bond forfeited even after he pleaded to Shylock Shylock stands firmly for his bond which when forfeited will allow him to take a pound of flesh from any part of Antoniorsquos body(iii) How does the Duke address Shylock`The Duke calls Shylock an adversary with a heart of stones He calls Shylock as an inhuman wretch without pity Shylock is quite lacking in the slightest quality of mercy (iv) How does Antonio reply to this

Antonio replies that he will meet the revenge of Shylock patiently He has prepared himself to suffer with a quiet spirit the utmost that Shylockrsquos tyranny and rage can do(v) What quality of the Duke is revealed here

The Duke is kind and benevolent He is ready to help Antonio He requests Shylock to free Antonio from the trial(vi) What are the terms of the bond that Antonio has signed

The terms of the bond that Antonio has signed were that if Antonio is unable to repay Shylock a certain sum of money specified on the paper on a certain date and in an agreed place the forfeit has to be paid The forfeiture will be an exact pound of Antoniorsquos flesh which Shylock will be a liberty to take from any part of Antoniorsquos body which pleases him

Class XI

STUDY MATERIAL

Class XISubject Eng Literature (The Tempest ndash William Shakespeare) Topic Act I Scene 2 Lines 88 to 132 (Prospero hellip Me and thy crying self) Date 27th April 2020 (3rd Period)

[Students should read the original play and also the paraphrase given in the school prescribed textbook]Summary Questions amp Answers

o Prospero now tells Miranda that he was the Duke of Milan He had been devoting himself more to studies than the affairs of the State His brother Antonio took advantage of this situation and with the help of Alonso the king of Naples seized upon him and her one midnight and shipped them in a frail bark so that they perished in the sea All this took place

(1)

MIRANDA I should sin (Line 118-132)

To think but nobly of my grandmother

Good wombs have borne bad sonsPROSPERO Now the condition

The King of Naples being an enemyTo me inveterate hearkens my brothers suitWhich was that he in lieu othrsquo premisesOf homage and I know not how much tribute

twelve years back

IMPORTANT PASSAGES EXPLAINED(Line 98-103)

PROSPERO helliphelliphelliphelliphelliphelliphellip

He being thus lorded

Not only with what my revenue yieldedBut what my power might else exact like oneWho having into truth by telling of it Made such a sinner of his memoryTo credit his own lie he did believeHe was indeed the duke

Prospero in telling the narrative of his past life here refers to his brother Antonio Prospero being with a studious bent of mind has left the administration of Milan on his younger brother Now Antonio being thus invested like a lord with all the powers derived from Prosperorsquos wealth and what the exercise of Prosperorsquos authority might secure for him regarded himself as a de facto Duke of Milan It is a well-known fact of psychology that a man who repeatedly tells a lie makes of his memory such a sinner against truth as to credit his own lie by the telling of it So Antonio by repeatedly saying to himself and others that he was the Duke came to believe that he was really the Duke Thus falsehood repeatedly asserted gained the force of truth for Antonio and he truly believed it

Should presently extirpate me and mine Out of the dukedom and confer fair MilanWith all the honours on my brother whereonA treacherous army levied one midnightFated to thrsquo purpose did Antonio openThe gates of Milan and ithrsquo dead of darkness The ministers for thrsquo purpose hurried thenceMe and thy crying self

(i) In the earlier lines of this scene what does Prospero tell about his intense interest What was the demand of his interest

In the earlier lines of this scene Prospero tells Miranda that he had an intense interest in the study of philosophy and magic arts Hence in order to improve his mind with this kind of study he kept himself isolated from worldly and state affairs His study was dearer to him than the applause and esteem that he could win from the public His study demanded too much solitude(ii) What forced Antonio to take an undue advantage over Prospero

Prosperorsquos indifferent attitude towards the statersquos affairs and his having boundless trust in Antonio gave rise to a boundless lust for power in Antoniorsquos mind Antonio felt that he must be the actual Duke instead of the part of the Duke he played Thus Antonio took an undue advantage of the situation to usurp Prosperorsquos dukedom(iii) Explain the following lines ldquoI should sin to think but nobly of my grandmother Good wombs have borne bad sonsrdquo

After hearing the treacherous act of her uncle Antonio Miranda says that Prosperorsquos mother was a noble lady and she cannot dishonour her memory by saying that the person named Antonio cannot be his (Prosperorsquos) brother She says that in honour of her grandmother she also cannot say that Antonio must have been begotten not by her grandfather but by some other man She finally concedes that it is known that good mothers have borne bad sons in their wombs and gave birth to them

(iv) Why did the King of Naples accept Antoniorsquos request to help him in usurping his dukedom What did Antonio propose to Alonso

Alonso who was the king of Naples accepted Antoniorsquos request in usurping Prosperorsquos dukedom because he (Alonso) was a sworn enemy of Prospero Antonio proposed that Alonso should immediately drive him (Prospero) and his offspring out of Milan and should confer the dukedom upon him (Antonio) with all the dignities which go with that

position In return Antonio promised that he shall give an annual tribute and also swore his allegiance to Alonso Also he agreed to hold the Dukedom of Milan as a subordinate to the state of Naples(v) How were Prospero and Miranda carried away from the city of Milan and what was the state of small Miranda at that time

In pursuance of the agreement settled between Antonio and Alonso an army of treacherous men was assembled One midnight when the occasion suited the will of destiny Antonio opened the gates of the city of Milan and in the death like silence of midnight Antoniorsquos agents who had been directed to execute his purpose carried Prospero and small Miranda away from the city in all haste They were then forced into a ship and carried some distance out to sea where they put them on a mere hulk of a boat without any rigging or ship-gear and abandoned them leaving them at the mercy of the roaring sea Miranda was a very small child of three years age and she was crying at that time

CLASS -XIDATE-270420Subject Topic Summary Execution

EVS Chapter 1 ndash Mode of Existence

Impact of mode of existence on resources

Q) Why resources are under pressure

Ans - Increase in the sophistication

of technology enabling natural resources to be extracted quickly and efficiently Eg in the past it could take long hours just to cut down one tree only using saws Due to increased technology rates of deforestation have greatly increased

The number of humans is increasing Cultures of consumerism Materialistic views

lead to the mining of gold and diamonds to produce jewelry unnecessary commodities for human life or advancement Consumerism also leads to extraction of resources for the

production of commodities necessary for human life but in amounts excessive of what is needed because people consume more than is necessary or waste what they have

Lack of awareness among the population is striking People are not aware of ways to reduce depletion and exploitation of materials

Accounts Cash Book Today we are going to start a new topic -Cash Book

The key terms used in this chapter are

bullCash book

bullSimple cash book

bullDouble column cash book bullTriple column cash book

bullPetty cash book

bullCash discount

bullContra entry

Here I will share you the meaning of each key terms

bullCash book Cash Book is a special purpose subsidiary book or journal in which cash received and cash payments are recorded

bullSimple cash book

It is a cash book in which only cash transactions are recorded It has only one column on each side

bullTriple column cash book

It is cash book which has three columns one column for each cash and Bankdiscount on each side of the cash book In this book both cash and Bank transactions are recorded together with discount allowed and received

bullPetty cash book

It is a cash book maintained for recording petty expenses

bullCash discount

Cash discount is the amount of discount received or allowed on cash payments and cash receipts Discount received is an income for the business while discount allowed isan expense

bullContra entry

It means transactions involving both cash and Bank Such transactions though recorded in the cash book are not posted into ledger The letter lsquoC is written in Ledger folio for contra entry

Business Studies

ENTREPRENEURSHIP

Now we shall discuss the second chapter

lsquoENTREPRENEURSHIPrsquo

Today before starting the chapter let us recall what

Questions

1What are the main characteristics of Intrapreneurship

Answer

The main characteristics of Intrapreneurship are

Corporate framework-it occurs within the framework of the same company

Semi-Autonomous-Intrapreneurship

we have read last day

Let s today start the class by recalling the last topic taught

Intrapreneurship is the process of discovering and exploring business opportunities within an existing company It involves launching new business ventures within the framework of a present corporation Intrapreneurship is also known as corporate entrepreneurship or corporate venturing

Now let us start with the characteristics of Intrapreneurship

The main characteristics of Intrapreneurship are

Corporate framework

Semi-Autonomous Lack of ownership Senior position Low risk taking Not own boss

Now let us discuss the meaning of enterprise

Enterprise means an undertaking or adventure that requires some innovation and investment and thus involves riskEnterprise always entails decision making coordination and risk bearing

involves crating amd nurturing a semi-autonomous business unit which may be a subsidiary a strategic business unit or a division

Lack of ownership-the intrapreneur is not the owner of the unitb he creates and nurtures

Senior position-he occupies a senior managerial position in the company

Low risk taking-An intrapreneur does not bear the full risk of failure

Not own boss-An intrapreneur is not his own bosss in legal termsHe enjoys the freedom and gets the required resources and support

2 How is Entrepreneur is different from Intrapreneur

The functions involved in both the entrepreneurship and intrapreneurship are by and large similar however there are several differences between the two

Point of distinction

Entrepreneur

Intrapreneur

status An independent business person

A senior executive within a company

Ownership Owner of

the business

An employeesometimes a share in ownership

Financing Responsible for raising finance for the business

Not responsible for raising the finance

Risk bearing

Bears the risk of the business

Does not bears the risk of the business

Reward Profit which is uncertain and irregularcan be loss

Fixed salary and fringe benefits

Need for security low high

3 What do you understand by enterprise

Answer Enterprise means an undertaking or adventure that requires some innovation and investment and thus involves riskEnterprise always entails decision making coordination and risk bearing

COMMERCE NATURE AND OBJECTIVES OF

BUSINESS

Today let us recall the last other two objectives of business by the chart given in the previous class

Firstly we would discuss Human Objectives

Business is run by people and for people Labour is a valuable business element

Human objectives of business are concerned with the well -being of labour

The human objectives are as follows

Labour welfare Developing human

resources Participative

management Labour

management cooperation

Questions

1 Explain the human objectives of a business enterprise

Answer

Business is run by people and for people Labour is a valuable business element

Human objectives of business are concerned with the well -being of labour

The human objectives are as follows

Labour welfare-Business must recognize the dignity of labour and human factors should be given the recognition

Developing human resources-Employees must be provided the opportunities for developing new skills and attitudes

Participative management-Employees should be allowed to take part in decision making process of business

Labour management cooperation-Business should strive for creating and maintaining cordial employer employee relations so as to ensure peace and progress in industry

Now let us discuss the national objectives of business

Optimum utilization of resources

National self- reliance Development of small

scale industries Development of

backward areas Control over pollution

2Explain the national objectives of a business enterprise

Answer

It is the duty of business to utilize the resources of the country properly the national objectives of business

Optimum utilization of resources ndashBusiness should use the nationrsquos resources in the best possible manner

National self- reliance-It is the duty of the business to help the government in increasing experts and in reducing dependence on imports

Development of small scale industries-Big business firms are expected to encourage growth of small scale industries which are necessary for generating employment

Development of backward areas-Business is expected to give preference to the industrialization of backward regions of the country

ECONOMICS

BASIC ECONOMIC CONCEPTS

SUB

TOPIC

Value

Wealth

Welfare

Today we shall start with a new topic of the same chapter ie lsquoValuersquo

Value of a commodity is defined as the valuation placed by a household on the consumption of this commodity

lsquoValuersquo has two different meanings and these are

a Value -in -use It refers to consumption value of a commodity It expresses the utility derived from the consumption of a particular commodity A necessity like water has a very high value ndashin ndashuse or

Question

1What is value

Answer

Value of a commodity is defined as the valuation placed by a household on the consumption of this commodity

2What is value-in use

Answer It refers to consumption value of a commodity It expresses the utility derived from the consumption of a particular commodity A necessity like water has a very high value ndashin ndashuse or consumption value

3What is value ndashin- exchange

Answer It relates to market value of a commodity

It is the rate at which a particular good or service can be exchanged for money

For example in barter system if a person is prepared to exchange 3 metres of cloth with 1 pair

consumption value

b Value ndashin-exchange It relates to market value of a commodity

It is the rate at which a particular good or service can be exchanged for moneyFor example in barter system if a person is prepared to exchange 3 metres of cloth with 1 pair of shoes then the value in exchange of 3 metres of cloth is 1 pair of shoesValue in exchange is the power of purchasing other goods In modern monetised economies the exchange value of goods are expressed in terms of money as prices

Now let us discuss the term lsquoWealthrsquo

Wealth refers to the stock of all those assets which are a source of income

Wealth is a stock concept

Wealth must possess the following features

a Utility It must possess utility or give some

of shoes then the value in exchange of 3 metres of cloth is 1 pair of shoes

Value in exchange is the power of purchasing other goods In modern monetised economies the exchange value of goods are expressed in terms of money as prices

4 What is wealth

Answer it refers to the stock of assets or goods which are a source of income and have personal or national ownership

5 What are the features of wealth

Answer The features of wealth are as follows

Wealth must possess the following features

a Utility It must possess utility or give some satisfaction

b Scarcity It must be limited in quantityc Transferability it should be transferable its

ownership can be transferred from one person to another person

d Exchange value It must possess exchange value

6 What is welfare

Answer

Welfare is defined as satisfaction and happiness a sense of well- being among the people

satisfactionb Scarcity It must be

limited in quantityc Transferability It

should be transferable its ownership can be transferred from one person to another person

d Exchange value It must possess exchange value

Now let us discuss the term lsquoWelfarersquo

Welfare is defined as satisfaction and happiness a sense of well- being among the people

Welfare is affected by factors like

a Consumption of goods and services

b Environment

c Family relations

d Degree of freedom

e Law and order situation

Mathematics Trigonometric equation

To find the general solution of the equation sinθ=0

When sin θ =0

Then θ= 0 π2π 3π-π -2π -3

i e when θ = 0 or an integral multiple of π

i e when θ= nπ where n is any integer

Therefore the general solution of the equation sin

Example1 Find the general values of θ which satisfy the equation sin2 θ =34

Solution sin2 θ= 34

Or sin θ = +34 or -34

Or sin θ = sin π3 or sin (-π3)

Therefore

θ = [nπ + (-1) n (π3)] or[ nπ+ (-1) n (-π3)]

= nπ +π3 or nπ-π3 where n= any integer

Example 2Find the values of θ which satisfy tan2 θ

θ=0 is θ= nπ where n is any integer

To find the general solution of the equation cos θ=0

When cos θ=0

Then θ=π2 3π2 5π2 -π2 -3π2 -5π3

i e when θ is an odd multiple of π2

i e when θ=(2n+1) π2 where n= any integer

Therefore the general solution of the equation cos θ =0 is θ= (2n+1) π2 where n= any integer

To find the general solution of the equation tan θ = 0

Clearly tan θ =0 implies sin θcos θ =0

Therefore θ = nπ

i e the general solution of the equation tan θ=0 is θ =nπ where n = any integer

To find the general solution of the equation cot θ =0

Clearly cot θ =0 implies (cos θsin θ) = 0

i e cos θ =0

Therefore θ = (2n+1) π2

Therefore the general solution of the equation cot θ =0 is θ = (2n+1) π2

Where n= any integer

To find the general solution of the equation sin θ= k (-

=13 -πleθleπ

Solution tan2 θ =13

Or tan θ = plusmn1radic(3) =tan(plusmnπ6)

θ=nπ plusmn π6 where n =any integer

If n=0 then θ=plusmnπ6

If n=1 then θ= π plusmn π6

If n=-1 then θ= -π plusmn π6

Therefore the required solution in -π le θ le π are θ= π6 5π6 -π6 -5π6

Exercise Find general solution of sin 2θ=cos θ [Hints Use sin 2θ= 2sin θcosθ and then take cosθ

common]

1lek le1)

Determine an angle alpha such that sin =k and -π2le αle π2

Then we have

Sin θ = k = sin α

Or sin θ - sin α =0

Or 2 cos [(θ+α) 2] sin [(θ-α) 2] =0

Therefore either cos [(θ +α) 2] =0 (1)

Or sin [(θ-α) 2] =0 (2)

Now from (1) we get (θ+α) 2= (2m+1) π2)

Or θ = (2m+1) π-α (3)

And from (2) we get (θ-α) 2 =mπ

Or θ= 2mπ+α(4)

Where m = any integer

Clearly the solution (3) amp (4) may be combined in the following form

θ= nπ+(-1) n α where n= any integer

Therefore the general solution of sin θ = sin α is θ = nπ +(-1) n α where n is any integer and -π2 le α le π2

Biology Chapter - 04Kingdom Monera

Today we will discuss about bacterial reproduction and its usefulness

Fig Binary Fission

Fig Conjugation Fig Transformation

Bacterial reproduction is mainly asexual but sexual reproduction

also takes place

Asexual reproduction takes place by i) Binary fission - from one bacteriato

two bacteria are produced in every 20 to 30mins

ii) Buddingiii) Endospore formation - during

unfourable condition

Sexual reproduction by three ways

1) Conjugation - Transfer of genetic material between cells that are in physical contact with one another

2) Transduction - Transfer of genetic materialfrom one cell to another by a bacteriophage

3) Transformation - Transfer of cell-freeor naked DNArsquo from one cell to another

Bacteria causes different diseases inplants animals and human and

it causes food spoilage and waterpollution but it also have some useful

activities

i) Bacteria are helpful in sewage water treatment

ii) It is used in antibiotic (medicine) production

iii) Anaerobic bacteria help in biogas(energy) production

iv) Many household products like yoghurt cheese are manufactured by use of bacteria

v) Rhizobium by symbiotic relationship with leguminous plant increase soil fertility

vi) Besides these bacteria is helpful in genetic engineering degradation of petroleum hydrocarbonand in dairy

industry

Physics Motion in plane Here we will introduce Projectile Motion

Execution

Projectile

Y

usinθ u h

θX

ucosθ

Suppose a body is projected with an angle θ So initial velocity u can be resolved into two components

Horizontal component - ucosθ ( for range)

Vertical component - usinθ ( for height)

usinθ changes during motion and becomes zero at maximum height position but ucosθ remain unchanged

The maximum height of projectile is h

NB If initial is upward then g = -ve and if it is downward then g = +ve Height is +ve if direction of motion does not change ( for ex a body thrown upwards but goes down ultimately then height h = -ve)

The angle of projectile θ is the angle made with horizontal

HISTORY ndash GROWTH OF NATIONALISM

SUB TOPIC- REVOLUTIONARY NATIONALISM Bengal formation of Anushilan Samity and Jugantar Group

The intensification of the Swadeshi movement and Government policy of terror and repression led to outbreak of violence Bombs were manufactured and attempts on the lives of unpopular Government officials became frequent In the gymnasium of Scottish Church College which was known as General Assemblies Institution a secret society was formed known as Anushilan Samity

Aurobindo Ghosh send from Baroda his emissary Jatindranath Banerjee to mobilize the Bengal revolutionaries

Hemchandra Qanungo and Satyen Bose published Journal Jugantar

The Jugantar group planned to assassinate oppressive magistrate Kingsford by Khudiram Bose and Prafulla Chaki in 1908 Prafulla Chaki committed suicide to avoid arrest Khudiram was tried and hanged

Afew days later the police found a bomb factory in Maniktala and arrested a large number of revolutionaries The trial of revolutionaries became famous as the Alipore Bomb Case

In the course of the trial the approver the public prosecuter and a police officer were assassinated

1 Question Name two journals which preached the cult of violence

Answer a) Yugantar edited by Bhupendranath DuttaB) Bandemataram edited by Aurobindo Ghosh2 Question Why was Khudiram arrested and hangedAnswer An attempt was made to assassinate a hated vindictive majistrate named Kingsford by Khudiram Bose and Prafulla Chaki Their attempt failed and the bomb they threw killed two English ladies Khudiram was arrested and put to trial and then hanged3Question Who was Aurubindo GhoshAnswer Aurobindo Ghosh a nationalist revolutionary who was charged for his involvement in the Alipore Bombing Case He was accused of it along with his brother Barindra nath Ghosh But Aurobindo was acquitted because of the brilliant pleading of his counsel Chittaranjan Das Then he became a spiritual reformer introducing his visions on human progress and spiritual evolution4 Qustion Who was KingsfordAnswer Kingsford was an unpopular British chief Magistrate who was the target of the bomb thrown at Muzaffarpur by Khudiram and Prafulla Chaki

Most of the accused were convicted and sentenced to

long term of imprisonmentBut

Aurobindo Ghosh was acquitted mainly owing to the brilliant pleading of his counsel Chittaranjan Das

Political science Topic-Sovereignty

Summary Sovereignty is the full right and power of a governing body over itself without any interference from outside sources or bodies In political theory sovereignty is a substantive term designating supreme legitimate authority over some polity In international law sovereignty is the exercise of power by a state

Internal Sovereignty

Internal sovereignty means supreme authority within ones territory while external sovereignty relates to the recognition on the part of all states that each possesses this power in equal measure

External sovereignty

external sovereignty relates to the recognition on the part of all states that each possesses this power in equal measure

Distinguish between

Execution

Answer the following questions

Short notes-

Sovereignty

Internal Sovereignty

External sovereignty

Homework- learn

external sovereignty and internal sovereigntySovereignty is the principle

of supreme and

unquestionable authority

reflected in the claim by the

state to be the sole author of

laws within its territory

Definition of external vs internal sovereigntyInternal sovereignty refers to

the relationship between a

sovereign power and its

subjects ndash it refers to the

location of the supreme

authority within the state In

the UK for example internal

sovereignty (supposedly)

resides within Parliament

reflected in the

constitutional principle of

parliamentary

sovereigntyBy contrast

external sovereignty refers

to the capacity of the state

to act independently and

autonomously on the world

stage This is what is

sometimes called lsquostate

sovereigntyrsquo or lsquonational

sovereigntyrsquo and implies

that states are legally equal

and that the territorial

integrity and political

independence of a state is

inviolable

Class ndash XII

Date - 2742020 STUDY MATERIALSubject Topic Summary Execution Business Studies

Job Analysis amp Manpower Planning

At first let us recall the chapter what we have discussed till nowbullJob analysisbullJob specification bullJob description bullJob enlargement bullJob enrichment

Today we will do some questions answers from the chapter

Questions 1ldquoJob analysis job description and job specification are interrelatedrdquo Comment Answer) Job analysis is a systematic and detailed examination of a job to collect all the relevant information about it The contents off the job are summarised in the job description The qualification needed for the job are summarised in job specificationThus there is close interrelationship between job analysis job description and job specification

Question 2ldquoJob enlargement is a horizontal extension of a job whereas job enrichment is a vertical extension of a jobrdquo ElucidateAnswer) Job enlargement involves adding one or more task to a job coma where as job enrichment involves adding more autonomy and responsibility to a job Job enlargement is therefore horizontal extension of a job coma whereas job enrichment is a vertical extension of a job

Question 3 )

What is manpower estimation Explain its quantitative and qualitative aspectsAnswer) Manpower estimation is the process by which management determines how an organisation should move from itrsquos current manpower positionto its desired manpower position There are two dimensions of Manpower estimation- quantitative and qualitative

Quantitative aspectThis aspect of Manpower estimation involves estimating the number of employees required in a future time period Workload analysis and workforce analysis are done to estimate the quantity of required manpower

Qualitative aspectThe estimate of the knowledge skills experience etc of required manpower is the qualitative aspect of Manpower estimation The quality of Manpower can be judged on the basis of job analysisand job specification

COMMERCE

CAPITAL-FIXED AND WORKING

Today let us start the class by discussing the sources of finance for different types of business firms

The term lsquocapitalrsquo refers to the investment made in the enterprise for the purpose of earning profits

Requirements of capital and sources of capital for different types of business firms are

1 Capital for sole proprietorship businessA sole proprietor operates at a small scale and thereforerequires a limited amount of capital

2 Capital for partnership firmCapital requirements as well as capital base of a partnership is bigger than that of a sole trader businessThe owned capital is contributed by the partners in an agreed ratio

3 Capital for joint stock companyA joint stock company generally requires large amount of capitalA public company can raise huge capital through issue of shares In addition to share capital it can utilize retained profits

Now let u discuss the meaning of Finance PlanningFinance planning is the process of estimation the financial requirements of an organization specifying the sources of firms and ensuring that enough funds are available at the right time

1 What do you mean by Finance PlanningAnswer Finance planning is the process of estimation the financial requirements of an organization specifying the sources of firms and ensuring that enough funds are available at the right time

2Discuss the role of financial planning of an enterpriseThe role of financial planning are as followsa A sound financial plan helps a business enterprise to avaid the problems of shortage and surplus of fundsbFinancial planning serves as a guide in developing a sound capital structure so as to maximize returns to shareholders c It helps in effective utilization of fundsd It provides policies and procedures for coordinating different functional areas or departments of businesse It enables the management to exercise effective control over the financial activities of an enterprisef It helps the company to prepare for facing business shocks and surprises in future

Mathematics

Continuity and differentiability

Recall Definition of ContinuityLet f(x) be a single valued function of x and x=a be a point in the domain of definition of the function The function is said to be continuous at x=a ifi) f(c) is defined ie f(x) has a definite finite value at x=cii) lim xrarra f(x) exists andiii) lim xrarra f(x) =f(a) In other words f(x) is said to be continuous at x=a if lim xrarra+ f(x)= lim xrarra- f(x) = f(a) Or f(a+0) =f(a-0) =f(a) Or lim hrarr0 f(a+h)= f(a) Algebra of continuous functionsNow we will study some algebra of continuous functions Theorem 1 Suppose f and g be two real functions continuous at a real number c Then(1) f + g is continuous at x = c(2) f ndash g is continuous at x = c(3) f g is continuous at x = c(4) (fg) is continuous at x = c (provided g (c) ne 0)

Example 1 Prove that every rational function is continuousSolution Recall that every rational function f is given byf(x)=[p(x) q(x) ] q(x)ne0where p and q are polynomial functions The domain of f is all real numbers except points at which q is zero Since polynomial functions are continuous f is continuous by (4) of Theorem 1Example 2Discuss the continuity of sine functionSolution To see this we use the following factslim xrarr0 sin x =0Now observe that f (x) = sin x is defined for every real number Let c be a real number Put x = c + h If x rarr c we know that h rarr 0 Therefore lim xrarrc f(x)

= lim xrarrc sin x= lim hrarr0 sin(c+h) =lim hrarr0 [sin c cos h + cos c sin h ]=lim hrarr0 (sin c cos h) + lim hrarr0 (cos c sin h) = sin c +0=sin c = f(c) Thus lim xrarrc f(x) = f(c) and hence f is a constant function Exercise Prove that the function f(x) = x2 +2x is continuous for every real value of x [Hints show that lim xrarra+ f(x) = lim xrarra- f(x) = f(a) ]

Biology Reproduction in Flowering plants We will discuss about megasporoangium

megasporagenesis and female gametophyte

Q4 Describe the structure of megasporangium

Ovule is attached to the placenta by astalk called funicle

Each ovule has one two or three protectivecoverings called integuments

At the tip of integuments a small openingcalled micropyle is organised

Opposite to the micropylar end is the chalaza

Within the integuments a mass of cellsnucellusand inside it embryo sac orfemale gametophyte is present

Q5 Describe a mature embryosacamp its formation

In most of the flowering plants only oneof the 4 megaspores formed as a result ofmegasporogenesis that is functional while theother three degenerate

The

functional megaspore develops into thefemale gametophyte

Formation The nucleus

of the functional megasporedivides mitotically to form two nuclei first andthen two more sequential mitotic nucleardivisions result in the formation of four ampthen eight nucleate stages of embryo sac

Six of the eight nucleus are surrounded bycell walls and organised into cells

The remaining two nuclei called polar nuclei are found below the egg apparatus in the largecentral cell

Three cells consisting of two synergids amp one egg cell present bottom of

embryo sac Three cells

at the chalazal as antipodal cells

Two polar nuclei together present in large central cell

HISTORY

TOWARDS INDEPENDENCE AND PARTITION THE LAST PHASE(1935-1947)SUB TOPIC NATIONAL MOVEMENTS DURING THE SECOND WORLD WAR

Spread of Quit India Movement On 9th August 1942Gandhiji and other Congress leaders were arrested The Congress was declared illegal The news of the arrest of all leaders marked the beginning of a widespread movement of India It was not possible for such a movement to remain peacefulBut the arrest of the all notable congress leaders virtually left the movement in the hands of the mass The movement took the form of violent and militant outbreakBesides congressmen revolutionaries also were very active in the movement The Congress Socialist group also played a prominent role

1 Question Why did the British authority arrest the Congress leaders on 9 th August 1942Answer Congress Working committee adopted the Quit India resolution which was to be ratified at the Bombay AICC meeting in 8th August 1942 They decided to launch a mass struggle on non-violent lines Gandhiji gave a clarion call to all section of the people rdquoKarenge ya Marengerdquo (do or die) Congress leaders gave the call to driving out

the British from IndiaViceroy had taken strong action against the Quit India movement Gandhiji and all the leaders of Congress were arrested

2 Question How did Quit India Movement spread out all over IndiaAnswer The news of the leaders lsquo arrest marked the beginning of a widespread movement to remain peacefulThe movement took form of violent outbreak There were widespread cutting of telephone and Telegraph wires damaging railway lines raising barricades in cities and towns and other forms of violent demonstations

Question Name the leaders of Congress

Socialist group played a prominent part Notable among the Jayprakash Narayan Rammonohar Lohia Aruna Asaf Ali

Political science

Topic-Franchise and Representation

Summary

The election commission

The Election Commission of India is an autonomous constitutional authority responsible for administering Union and State election processes in India The body administers elections to the Lok Sabha Rajya Sabha State Legislative Assemblies in India and the offices of the President and Vice President in the country

Functions of election commission-

India is a sovereign socialist secular democratic republic Democracy runs like a golden thread in the social economic and political fabric woven by the Constitution given by lsquoWe the People of Indiarsquo unto ourselves The concept of democracy as visualised by the Constitution pre-supposes the representation of the people in Parliament and State legislatures by the method of election The Supreme Court has held that democracy is one of the inalienable basic features of the Constitution of India and forms part of its basic structure The Constitution of India adopted a Parliamentary form of government Parliament consists of the President of India and the two Houses mdash Rajya Sabha and Lok Sabha India being a Union of states has separate state legislatures for each state State legislatures consist of the Governor and two Houses mdash Legislative Council and Legislative Assembly mdash in seven states namely Andhra Pradesh Telangana Bihar Jammu amp Kashmir Karnataka Maharashtra and Uttar Pradesh and of the Governor and the state Legislative Assembly in the remaining 22 states Apart from the above two out of the seven Union Territories namely National Capital Territory of Delhi and Puducherry also have their Legislative Assemblies

ExecutionShort notes-Election commissionFunctions of election commission

Homework- Learn

Computer

Science

Computer hardware NAND Gate

A NOT-AND operation is known as NAND operation It has n input (n gt= 2) and one output

Logic diagram

Truth Table

NOR Gate

A NOT-OR operation is known as NOR operation It has n input (n gt= 2) and one output

Logic diagram

Truth Table

XOR Gate

XOR or Ex-OR gate is a special type of gate It can be used in the half

adder full adder and subtractor The exclusive-OR gate is abbreviated as EX-OR gate or sometime as X-OR gate It has n input (n gt= 2) and one output

Logic diagram

Truth Table

XNOR Gate

XNOR gate is a special type of gate It can be used in the half adder full adder and subtractor The exclusive-NOR gate is abbreviated as EX-NOR gate or sometime as X-NOR gate It has n input (n gt= 2) and one output

Logic diagram

Truth Table

Physics

Chapter 1 Electric Field ( Electric Dipole) (Summary)

Here we will derive Expression of electric field at broad side

On position of dipole

Execution

Q With the help of a labelled diagram obtain an expression for the electric field intensity E at any point on the equitorial line ( broad-side on position) of an electric dipole

Ans

E1 E1sinθ

E θ P E1 θ

( r2+L2)12 E2 E

r E2 E2sinθ

-q θ L O L +qA B

Let us consider that the point P is situated on the right bisector of the dipole AB at a distance r meter from its midpoint O

Let E1 and E2 be the electric field intensities of the electric field at P due to charge +q and ndashq of the dipole resp The distance of P from each charge is ( r2+L2)12

So E1 = 14 πϵ q

(r 2+L 2) away from +q

E2 = 14 πϵ q

(r 2+L 2) towards ndashq

The magnitudes of E1 and E2 are equal but directions are different Now resolving E1 and E2 into two components parallel and perpendicular to AB we get

The components perpendicular to AB E1sinθ and E2sinθ cancel each other because they are equal and opposite

The components parallel to AB are E1cosθ and E2 cosθ are in same direction and add up

So resultant intensity of electric field at the point P is

E = E1cosθ + E2 cosθ

E = 14 πϵ q

(r 2+L 2) 2 cosθ

Now from fig we have cosθ =BOBP = L (r2+L2)12

So we get E = 14 πϵ 2qL ( r2+L2)32

Now electric dipole moment p= 2qL

So E = 14 πϵ p ( r2+L2)32

HW Find the expression of Electric field as done here but this time take r gtgt 2L

Also find the expression of torque experience by a dipole

(Hint Electric force experienced by charges of dipole in electric field is qE each Let θ be the angle which dipole makes with electric lines of force then perpendicular distance between two charges is 2Lsinθ Then torque = force x perp distance = qE x 2L sinθ So τ=pE sinθ where p =2qL )

STUDY MATERIAL

Class XIISubject Eng Literature (The Tempest ndash William Shakespeare) Topic Act IV Scene 1 Lines 84 to 133 (Iris hellip A contract of true love Be not too late ) Date 27th April 2020 (4th Period)

[Students should read the original play and also the paraphrase given in the school prescribed textbook]Summary Questions amp Answers

o Ceres soon appears and comes to know that she has been summoned to celebrate the contract of true love

o Ceres expresses her unwillingness to meet Venus and Cupid as she has shunned their company

o Ceres and Juno both bestow their blessings upon Ferdinand and Miranda with June gifting honour riches happiness in marriage and Ceres presents plenty of earthrsquos produce

o Iris summons the water-nymphs and reapers to come and celebrate a contract

(1) IRIS Of her society (Line 91-101)

Be not afraid I met her deity

Cutting the clouds towards Pathos and her sonDove-drawn with her Here thought they to have doneSome wanton charm upon this man and maidWhose vows are that no bed-right shall be paidTill Hymens torch be lightedmdashbut in vainMarss hot minion is returned againHer waspish-headed son has broke his arrowsSwears he will shoot no more but play with sparrowsAnd be a boy right out

(i) Where were Venus and Cupid seen flying How were they travelling Why did they want to join the marriage celebration of Ferdinand and Miranda

of true love

Venus and Cupid were seen flying through the air towards Paphos the famous city which is situated on the island of Cyprus They were travelling by air-borne chariot drawn by doves They certainly wanted to come here in order to play some amorous trick upon Ferdinand and Miranda who are under a vow not to gratify their physical desires till the holy ceremony of their marriage has been performed(ii) What have Venus and Cupid done after failing in their plan

After being failure of their plan Venus who is a very passionate deity and who is the mistress of Mars (the god of war) has gone back while here ill-tempered son Cupid has broken his arrows of love in his state of desperation(iii) What has Cupid firmly decided

Cupid is feeling so disappointed that he has firmly decided to shoot no more arrows to arouse love in human hearts but to spend his time playing with sparrows Thus he would now become just a boy and would give up his original function of shooting arrows on human beings to make them fall in love(iv) What vow had Ceres taken How did Ceres feel at the abduction

After the abduction of her daughter Prosperina by Pluto Ceres had taken a vow to always keep away from the disgraceful company of Venus and her blind son Cupid the god of love Ceres felt deeply distressed when Pluto had carried off her daughter and had made her his wife by force(v) Why has Ceres not forgiven Venus and her blind son For what do Ceres want to be sure

As the abduction had been manipulated by Venus the goddess of beauty and love and her blind son Cupid Ceres has never forgiven them for their part in the whole plot Ceres wants to be sure that she would not have to meet Venus and Cupid who had engineered the abduction of her daughter Prosperina

AS THIS lsquoMASQUErsquo SCENE IS VERY IMPORTANT IN THE PLAY THE PARAPHRASE OF THE ENTIRE PORTION OF MASQUE SCENE (Act IV Lines 58 to 143) IS GIVEN BELOW

IRIS Goddess of RainbowCERES Goddess of Agriculture and all the fruits of the earth

(Nature growth prosperity rebirth ndash notions intimately connected to marriage)JUNO The majestic Queen of Heavens and wife of Jupiter (Jupiter is the king of Gods)

VENUS The Goddess of love CUPID Son of Venus PLUTO God of death (In the play referred by Shakespeare as lsquoDisrsquo which is a Roman name for Pluto)

ORIGINAL TEXT PARAPHRASEPROSPEROWellmdash

PROSPERONow come Ariel Let there be too many rather than too few

Now come my Ariel Bring a corollaryRather than want a spirit Appear and pertly[to Ferdinand and Miranda]No tongue all eyes Be silent

spirits in attendance Appear briskly

[to Ferdinand and Miranda]Look with your eyes but do not say a word

[Soft music] [Soft music][Enter Iris] [Enter Iris]

IRISCeres most bounteous lady thy rich leasOf wheat rye barley vetches oats and peasThy turfy mountains where live nibbling sheepAnd flat meads thatched with stover them to keepThy banks with pioned and twilled brimsWhich spongy April at thy hest betrimsTo make cold nymphs chaste crowns and thybroom-grovesWhose shadow the dismissegraved bachelor lovesBeing lass-lorn thy pole clipped vineyardAnd thy sea-marge sterile and rocky-hardWhere thou thyself dost airmdashthe Queen othrsquoSkyWhose watery arch and messenger am IBids thee leave these and with her sovereign grace[Juno appears] Here on this grass-plot in this very placeTo come and sport Her peacocks fly amainApproach rich Ceres her to entertain

IRISCeres most generous lady you are the cause of rich fields or fertile land where wheat rye barley beans oats and peas grow the grassy mountains where the sheep graze and the flat meadows covered with coarse hay to be used as fodder for cattleYour banks are covered with marsh-marigolds and reeds and the rainy April under your orders brings forth to make for the maids who are not in love beautiful crowns your woods where the broom flourishes and where the bachelor who has been dismissed by the maid he loved lies down being forsaken your vineyard in which the poles are embraced by the vines and the margin of the sea which is barren and rocky where you roam about to enjoy the fresh air ndash the queen of the sky (Juno) whose messenger I am besides being represented as the rainbow bids you leave all these and with her majesty here on this grassy plot in this very place come and sport her peacocks carry her fast in her chariot through the air and are making their way here approach rich Ceres to welcome her

[Enter Ariel as Ceres] [Enter Ariel as Ceres]

CERESHail many-coloured messenger that neerDost disobey the wife of JupiterWho with thy saffron wings upon my flowersDiffusest honey-drops refreshing showersAnd with each end of thy blue bow dost crownMy bosky acres and my unshrubbed downRich scarf to my proud earth Why hath thy queenSummoned me hither to this short-grassed green

CERESWelcome rainbow that never dared disobey Juno the wife of Jupiter who with your orange coloured rays spread honey-drops refreshing showers And with each end of thy blue bow drown my bushy acres and my hilly country which is free from shrubs you thus forming a rich scarf Why has your queen called me here to this place covered with short grass

IRISA contract of true love to celebrateAnd some donation freely to estateOn the blest lovers

IRISI have called you to celebrate a contract of true love and bestow some liberal gift upon the blessed lovers

ORIGINAL TEXT PARAPHRASECERESTell me heavenly bowIf Venus or her son as thou dost knowDo now attend the queen Since they did plotThe means that dusky Dis my daughter gotHer and her blind boys scandaled companyI have forsworn

CERESTell me heavenly bow if Venus the Goddess of love or Cupid her son and pedlar of passion at this time attend the heavenly queen Juno because you are sure to know Since the day they conspired against me and dark Pluto took away my daughter here and Cupidrsquos disgraceful company I have left off

IRISOf her societyBe not afraid I met her deityCutting the clouds towards Pathos and her sonDove-drawn with her Here thought they to have doneSome wanton charm upon this man and miad

IRISBe not afraid of her company I met her deity moving on the clouds towards Paphos the sacred home of Venus on the island of Cyprus along with her son on her chariot drawn by doves Here they contemplated to exercise a charm upon this man and maid producing

Whose vows are that no bed-right shall be paidTill Hymens torch be lightedmdashbut in vainMarss hot minion is returned againHer waspish-headed son has broke his arrowsSwears he will shoot no more but play with sparrowsAnd be a boy right out

wantonness before the actual marriage ceremony but did not succeed Venus has returned her irritable son has broken his arrows and swears that he will give up his practice of trying to inspire love but play with sparrows and be a boy again

[Music is heard] [Music is heard]

CERESHighst queen of stateGreat Juno comes I know her by her gait

CERESHighest queen of state Great Juno there she comes I know here by her gait

[Enter Juno] [Enter Juno]

JUNOHow does my bounteous sister Go with meTo bless this twain that they may Prosperous beAnd honoured in their issue

JUNOHow are you doing my generous sister Come with me to bless this couple so that they may be prosperous and fortunate in their children

[They sing] [They sing]

JUNOHonour riches marriage-blessingLong continuance and increasingHourly joys be still upon youJuno sings her blessings upon you

JUNOMay honour riches happiness in marriage long continuance and increase of those boons ever rest upon you as hourly joys Juno showers down upon you her blessings in song

CERESEarths increase foison plentyBarns and garners never emptyVines and clustring bunches growingPlants and goodly burden bowingSpring come to you at the farthestIn the very end of harvestScarcity and want shall shun youCeresrsquo blessing so is on you

CERESMay you have the plenty of earthrsquos produce Your barns and granaries may never be empty Your vines may grow with clustering bunches Your fruit trees may be heavily laden with their fruit May there be continuous spring and harvest May scantiness and want leave you forever Such is the blessing of Ceres upon you

FERDINANDThis is a most majestic vision andHarmoniously charmingly May I be boldTo think these spirits

FERDINANDThis is a great vision and magically melodious Should I suppose the characters (taking part in the masque) are spirits

PROSPEROSpirits which by mine artI have from their confines calld to enactMy present fancies

PROSPEROYes they are spirits whom I have summoned from the regions to which they are confined to carry into effect my fanciful designs

ORIGINAL TEXT PARAPHRASEFERDINANDLet me live here everSo rare a wondered father and a wifeMakes this place paradise

FERDINANDI should like to live here forever Such a wise and wonderful father makes this place a paradise

[Juno and Ceres whisper and send Iris on employment] [Juno and Ceres whisper and send Iris on employment]

PROSPEROSweet now silence

PROSPEROMy dear Ferdinand speak no more Juno and Ceres are

Juno and Ceres whisper seriouslyTheres something else to do Hush and be muteOr else our spell is marred

whispering with a solemn look There is something else coming Silence Or else our magic will be spoilt

IRISYour nymphs called naiads of the wandering brooksWith your sedged crowns and over-harmless looksLeave your crisp channels and on this green landAnswer your summons Juno does commandCome temperate nymphs and help to celebrateA contract of true love Be not too late

IRISYou nymphs called Naiads denizens (M inhabitants) of the running stream with your chaplets of sedge and ever-helpful looks leave your wrinkled channels and on the green land answer the summons sent to you Juno has ordered some chaste nymphs and help to celebrate a noble and true marriage Donrsquot delay

[Enter certain nymphs] [Enter certain nymphs]You sunburnt sicklemen of August wearyCome hither from the furrow and be merryMake holiday your rye-straw hats put onAnd these fresh nymphs encounter every oneIn country footing

You sunburnt harvesters weary from the effects of the heat in August come here from the furrowed land and rejoice Make holiday with your rye-straw hats upon you and meet these fresh nymphs and join in country dancing

[Enter certain reapers properly habited They join with the nymphs in a graceful dance towards the end whereof Prospero starts suddenly and speaks]

[Enter certain reapers properly habited They join with the nymphs in a graceful dance towards the end whereof Prospero starts suddenly and speaks]

PROSPERO[aside] I had forgot that foul conspiracyOf the beast Caliban and his confederatesAgainst my life The minute of their plotIs almost come [to the spirits]Well done Avoidno more

PROSPERO(Aside)I had forgotten the wicked conspiracy of the beast Caliban and his accomplices against my life the time of their plot has almost arrived ndash (To the Spirits) well done depart no more of this

[To a strange hollow and confused noise the spirits heavily vanish]

[The spirits depart]

ORIGINAL TEXT PARAPHRASEFERDINANDLet me live here everSo rare a wondered father and a wifeMakes this place paradise

FERDINANDI should like to live here forever Such a wise and wonderful father makes this place a paradise

[Juno and Ceres whisper and send Iris on employment] [Juno and Ceres whisper and send Iris on employment]

PROSPEROSweet now silenceJuno and Ceres whisper seriouslyTheres something else to do Hush and be muteOr else our spell is marred

PROSPEROMy dear Ferdinand speak no more Juno and Ceres are whispering with a solemn look There is something else coming Silence Or else our magic will be spoilt

IRISYour nymphs called naiads of the wandering brooksWith your sedged crowns and over-harmless looksLeave your crisp channels and on this green landAnswer your summons Juno does commandCome temperate nymphs and help to celebrateA contract of true love Be not too late

IRISYou nymphs called Naiads denizens (M inhabitants) of the running stream with your chaplets of sedge and ever-helpful looks leave your wrinkled channels and on the green land answer the summons sent to you Juno has ordered some chaste nymphs and help to celebrate a noble and true marriage Donrsquot delay

[Enter certain nymphs] [Enter certain nymphs]You sunburnt sicklemen of August wearyCome hither from the furrow and be merryMake holiday your rye-straw hats put onAnd these fresh nymphs encounter every oneIn country footing

You sunburnt harvesters weary from the effects of the heat in August come here from the furrowed land and rejoice Make holiday with your rye-straw hats upon you and meet these fresh nymphs and join in country dancing

[Enter certain reapers properly habited They join with the nymphs in a graceful dance towards the end whereof Prospero starts suddenly and speaks]

[Enter certain reapers properly habited They join with the nymphs in a graceful dance towards the end whereof Prospero starts suddenly and speaks]

PROSPERO[aside] I had forgot that foul conspiracyOf the beast Caliban and his confederatesAgainst my life The minute of their plotIs almost come [to the spirits]Well done Avoidno more

PROSPERO(Aside)I had forgotten the wicked conspiracy of the beast Caliban and his accomplices against my life the time of their plot has almost arrived ndash (To the Spirits) well done depart no more of this

[To a strange hollow and confused noise the spirits heavily vanish]

[The spirits depart]

Ac-12 27420 topic Revaluation of Assets and Liabilities

REVALUATION OF ASSETS AND LIABILITIES

On admission of a new partner the firm stands reconstituted and consequently the assets are revalued and liabilities are reassessed It is necessary to show the true position of the firm at the time of admission of a new partner If the values of the assets are raised gain will increase the capital of the existing partners Similarly any decrease in the value of assets ie loss will decrease the capital of the existing partners For this purpose alsquoRevaluation Accountrsquo is prepared This account is credited with all increases in the value of assets and decrease in the value of liabilities It is debited with decrease on account of value of assets and increase in the value of liabilities The balance of this account shows a gain or loss on revaluation which is transferred to the existing partnerrsquos capital account in existing profit sharing ratioAccounting for Revaluation of Assets and Liabilities when there is a Changein the Profit Sharing Ratio of Existing PartnersAssets and liabilities of a firm must also be revalued at the time of change in profit sharing ratio of existing partners The reason is that the realisable or actual value of assets and liabilities may be different from those shown in the Balance Sheet It is possible that with the passage of time some of the assets might have appreciated in value while the value of certain other assets might have decreased and no record has been made of such changes in the books of accounts Similarly there may be some unrecorded assets amp libilities that may have to be accounted for Revaluation of assets and reassessments of liabilities becomes necessary because the change in the

value of assets and liabilities belongs to the period to change in profit sharing ratio and hence must be shared by the partners in their old profit sharing ratio Revaluation of assets and reassessment of liabilities may be given effect to in two different ways (a) When revised values are to be recorded in the books and(b) When revised values are not to be recorded in the books

When revised values are to be recorded in the booksIn such a case revaluation of assets and reassessment of liabilities is done with the help of a new account called lsquoRevaluation Accountrsquo Sometimes this account is also called as lsquoProfit amp Loss Adjustment Acrsquo If there is a loss due to revaluation revaluation account is debited and if the revaluation results in a profit the revaluation account is credited The following journal entries made for this purpose are

(i) For increase in the value of assetsAsset Ac Dr (individually)To Revaluation Ac(ii) For decrease in the value of AssetRevaluation Ac Dr (individually)To Asset Ac[Decrease in the value of assets](iii) For increase in the value of LiabilitiesRevaluation Ac Dr (individually)To Liabilities Ac[Increase in the value of Liabilities](iv) For decrease in the value of LiabilitiesLiabilities Ac DrTo Revaluation Ac[Decrease in the value of Liabilities](v) For unrecorded AssetsAsset Ac [unrecorded] DrTo Revaluation Ac[Unrecorded asset recorded at actual value](vi) For unrecorded Liability Revaluation Ac DrTo Liability Ac [unrecorded][Unrecorded Liability recorded at actual value](vii) For transfer of gain on revaluationRevaluation Ac DrTo Existing Partnerrsquos CapitalCurrent Ac[Profit on revaluation transferred to capital account in existing ratio](viii) For transfer of loss on revaluationExisting Partnerrsquos CapitalCurrent Ac DrTo Revaluation Ac[Loss on revaluation transferred to capital account in existing ratio](a) When revaluation account shows gain Revaluation Ac DrTo Partnerrsquos Capital Ac (Old Profit Sharing Ratio)(Profit on revaluation credited to Partnerrsquos Capital Ac)(b) Above entry is reversed when revaluation account shows loss Partners Capital Acs (Old Profit Sharing Ratio) DrTo Revaluation Ac(Loss on revaluation debited to Partnerrsquos Capital Acs)

Proforma of Revaluation Account is given as under

Revaluation Account

Dr Cr Particulars ` Amount Particulars ` Amount To Decrease in value of assets By Increase in value of assets To Increase in value of liabilities By Decrease in value of liabilities To Unrecorded liabilities By Unrecorded assets To Gain on Revaluation (Transferred) By Loss on Revalution (Transferred)

ECO ndash12 2742020Topic- ELASTICITY OF DEMAND

CHAPTER - ELASTICITY OF DEMANDMEANINGDemand for a commodity is affected by many factors such as its price price of related goods income of its buyer tastes and preferences etc Elasticity means degree of response Elasticity of demand means degree of responsiveness of demand Demand for a commodity responds to change in price price of related goods income etc So we have three dimensions of elasticity of demandDIMENSION OF ELASTICITY OF DEMAND TYPES OF ELASTICITY OF DEMAND

Price elasticity of demand Income elasticity of demand Cross Elasticity of demand

Price elasticity of demand Price elasticity of demand means degree of responsiveness of demand for a commodity to the change in its price For example if demand for a commodity rises by 10 due to 5 fall in its price Price elasticity of demand (ep)=Percentage change in quantity demanded Percentage change in price of the commodity = 10 ( -)5 = ( - )2Note that ep will always be negative due to inverse relationship of price and quantity demanded

(ii) Income elasticity of demand Income elasticity of demand refers to the degree of responsiveness of demand for a commodity to the change in income of its buyer Suppose income of buyer rises by 10 and his demand for a commodity rises by 20 then Income elasticity of demand (ey)= change in quantity demanded change in price of the commodity =20 10 = 2

Cross Elasticity of demandCross elasticity of demand means the degree of responsiveness of demand for a commodity to the change in price of its related goods (substitute goods or complementary goods) Suppose demand for a commodity rises by 10 due to 5 rise in price of its substitute good then Cross elasticity of demand (ec) = change in quantity demanded change in price of related good = 10 2 = 5 (Tastes and preferences cannot be expressed numerically So elasticity ofdemand cannot be numerically expressed)

  • Chapter 1 Force (Summary)
  • Distinguish between external sovereignty and internal sovereignty
    • NAND Gate
      • Logic diagram
      • Truth Table
        • NOR Gate
          • Logic diagram
          • Truth Table
            • XOR Gate
              • Logic diagram
              • Truth Table
                • XNOR Gate
                  • Logic diagram
                  • Truth Table
                      • Physics
                      • Chapter 1 Electric Field ( Electric Dipole) (Summary)
Page 23:  · Web viewWe all know that Nouns are divided into two parts: common noun and proper noun.Apart from common and proper noun, we will also study about collective noun and compound

the Duke Antonio Shylock and Salerio Later we meet Bassanio Portia Gratiano and Nerissa

o The Duke says to Antonio that he has to face a very cruel opponent which Antonio admits and expresses his gratefulness to the Duke for his efforts to soften without result the heart of Shylock in order to be merciful to Antonio Antonio further says that he is ready to accept whatever cruel judgement the Court may award

o When Shylock appears in the court the Duke says that Shylock should change his decision of prosecuting Antonio and demanding the penalty specified in the bond out of consideration of the great misfortunes that Antonio has suffered If this is done by him (Shylock) the whole court would be gladdened by his merciful action

(ii) What is the Duke sorry for

The Duke is unable to change the mind of Shylock from his decision to get the bond forfeited even after he pleaded to Shylock Shylock stands firmly for his bond which when forfeited will allow him to take a pound of flesh from any part of Antoniorsquos body(iii) How does the Duke address Shylock`The Duke calls Shylock an adversary with a heart of stones He calls Shylock as an inhuman wretch without pity Shylock is quite lacking in the slightest quality of mercy (iv) How does Antonio reply to this

Antonio replies that he will meet the revenge of Shylock patiently He has prepared himself to suffer with a quiet spirit the utmost that Shylockrsquos tyranny and rage can do(v) What quality of the Duke is revealed here

The Duke is kind and benevolent He is ready to help Antonio He requests Shylock to free Antonio from the trial(vi) What are the terms of the bond that Antonio has signed

The terms of the bond that Antonio has signed were that if Antonio is unable to repay Shylock a certain sum of money specified on the paper on a certain date and in an agreed place the forfeit has to be paid The forfeiture will be an exact pound of Antoniorsquos flesh which Shylock will be a liberty to take from any part of Antoniorsquos body which pleases him

Class XI

STUDY MATERIAL

Class XISubject Eng Literature (The Tempest ndash William Shakespeare) Topic Act I Scene 2 Lines 88 to 132 (Prospero hellip Me and thy crying self) Date 27th April 2020 (3rd Period)

[Students should read the original play and also the paraphrase given in the school prescribed textbook]Summary Questions amp Answers

o Prospero now tells Miranda that he was the Duke of Milan He had been devoting himself more to studies than the affairs of the State His brother Antonio took advantage of this situation and with the help of Alonso the king of Naples seized upon him and her one midnight and shipped them in a frail bark so that they perished in the sea All this took place

(1)

MIRANDA I should sin (Line 118-132)

To think but nobly of my grandmother

Good wombs have borne bad sonsPROSPERO Now the condition

The King of Naples being an enemyTo me inveterate hearkens my brothers suitWhich was that he in lieu othrsquo premisesOf homage and I know not how much tribute

twelve years back

IMPORTANT PASSAGES EXPLAINED(Line 98-103)

PROSPERO helliphelliphelliphelliphelliphelliphellip

He being thus lorded

Not only with what my revenue yieldedBut what my power might else exact like oneWho having into truth by telling of it Made such a sinner of his memoryTo credit his own lie he did believeHe was indeed the duke

Prospero in telling the narrative of his past life here refers to his brother Antonio Prospero being with a studious bent of mind has left the administration of Milan on his younger brother Now Antonio being thus invested like a lord with all the powers derived from Prosperorsquos wealth and what the exercise of Prosperorsquos authority might secure for him regarded himself as a de facto Duke of Milan It is a well-known fact of psychology that a man who repeatedly tells a lie makes of his memory such a sinner against truth as to credit his own lie by the telling of it So Antonio by repeatedly saying to himself and others that he was the Duke came to believe that he was really the Duke Thus falsehood repeatedly asserted gained the force of truth for Antonio and he truly believed it

Should presently extirpate me and mine Out of the dukedom and confer fair MilanWith all the honours on my brother whereonA treacherous army levied one midnightFated to thrsquo purpose did Antonio openThe gates of Milan and ithrsquo dead of darkness The ministers for thrsquo purpose hurried thenceMe and thy crying self

(i) In the earlier lines of this scene what does Prospero tell about his intense interest What was the demand of his interest

In the earlier lines of this scene Prospero tells Miranda that he had an intense interest in the study of philosophy and magic arts Hence in order to improve his mind with this kind of study he kept himself isolated from worldly and state affairs His study was dearer to him than the applause and esteem that he could win from the public His study demanded too much solitude(ii) What forced Antonio to take an undue advantage over Prospero

Prosperorsquos indifferent attitude towards the statersquos affairs and his having boundless trust in Antonio gave rise to a boundless lust for power in Antoniorsquos mind Antonio felt that he must be the actual Duke instead of the part of the Duke he played Thus Antonio took an undue advantage of the situation to usurp Prosperorsquos dukedom(iii) Explain the following lines ldquoI should sin to think but nobly of my grandmother Good wombs have borne bad sonsrdquo

After hearing the treacherous act of her uncle Antonio Miranda says that Prosperorsquos mother was a noble lady and she cannot dishonour her memory by saying that the person named Antonio cannot be his (Prosperorsquos) brother She says that in honour of her grandmother she also cannot say that Antonio must have been begotten not by her grandfather but by some other man She finally concedes that it is known that good mothers have borne bad sons in their wombs and gave birth to them

(iv) Why did the King of Naples accept Antoniorsquos request to help him in usurping his dukedom What did Antonio propose to Alonso

Alonso who was the king of Naples accepted Antoniorsquos request in usurping Prosperorsquos dukedom because he (Alonso) was a sworn enemy of Prospero Antonio proposed that Alonso should immediately drive him (Prospero) and his offspring out of Milan and should confer the dukedom upon him (Antonio) with all the dignities which go with that

position In return Antonio promised that he shall give an annual tribute and also swore his allegiance to Alonso Also he agreed to hold the Dukedom of Milan as a subordinate to the state of Naples(v) How were Prospero and Miranda carried away from the city of Milan and what was the state of small Miranda at that time

In pursuance of the agreement settled between Antonio and Alonso an army of treacherous men was assembled One midnight when the occasion suited the will of destiny Antonio opened the gates of the city of Milan and in the death like silence of midnight Antoniorsquos agents who had been directed to execute his purpose carried Prospero and small Miranda away from the city in all haste They were then forced into a ship and carried some distance out to sea where they put them on a mere hulk of a boat without any rigging or ship-gear and abandoned them leaving them at the mercy of the roaring sea Miranda was a very small child of three years age and she was crying at that time

CLASS -XIDATE-270420Subject Topic Summary Execution

EVS Chapter 1 ndash Mode of Existence

Impact of mode of existence on resources

Q) Why resources are under pressure

Ans - Increase in the sophistication

of technology enabling natural resources to be extracted quickly and efficiently Eg in the past it could take long hours just to cut down one tree only using saws Due to increased technology rates of deforestation have greatly increased

The number of humans is increasing Cultures of consumerism Materialistic views

lead to the mining of gold and diamonds to produce jewelry unnecessary commodities for human life or advancement Consumerism also leads to extraction of resources for the

production of commodities necessary for human life but in amounts excessive of what is needed because people consume more than is necessary or waste what they have

Lack of awareness among the population is striking People are not aware of ways to reduce depletion and exploitation of materials

Accounts Cash Book Today we are going to start a new topic -Cash Book

The key terms used in this chapter are

bullCash book

bullSimple cash book

bullDouble column cash book bullTriple column cash book

bullPetty cash book

bullCash discount

bullContra entry

Here I will share you the meaning of each key terms

bullCash book Cash Book is a special purpose subsidiary book or journal in which cash received and cash payments are recorded

bullSimple cash book

It is a cash book in which only cash transactions are recorded It has only one column on each side

bullTriple column cash book

It is cash book which has three columns one column for each cash and Bankdiscount on each side of the cash book In this book both cash and Bank transactions are recorded together with discount allowed and received

bullPetty cash book

It is a cash book maintained for recording petty expenses

bullCash discount

Cash discount is the amount of discount received or allowed on cash payments and cash receipts Discount received is an income for the business while discount allowed isan expense

bullContra entry

It means transactions involving both cash and Bank Such transactions though recorded in the cash book are not posted into ledger The letter lsquoC is written in Ledger folio for contra entry

Business Studies

ENTREPRENEURSHIP

Now we shall discuss the second chapter

lsquoENTREPRENEURSHIPrsquo

Today before starting the chapter let us recall what

Questions

1What are the main characteristics of Intrapreneurship

Answer

The main characteristics of Intrapreneurship are

Corporate framework-it occurs within the framework of the same company

Semi-Autonomous-Intrapreneurship

we have read last day

Let s today start the class by recalling the last topic taught

Intrapreneurship is the process of discovering and exploring business opportunities within an existing company It involves launching new business ventures within the framework of a present corporation Intrapreneurship is also known as corporate entrepreneurship or corporate venturing

Now let us start with the characteristics of Intrapreneurship

The main characteristics of Intrapreneurship are

Corporate framework

Semi-Autonomous Lack of ownership Senior position Low risk taking Not own boss

Now let us discuss the meaning of enterprise

Enterprise means an undertaking or adventure that requires some innovation and investment and thus involves riskEnterprise always entails decision making coordination and risk bearing

involves crating amd nurturing a semi-autonomous business unit which may be a subsidiary a strategic business unit or a division

Lack of ownership-the intrapreneur is not the owner of the unitb he creates and nurtures

Senior position-he occupies a senior managerial position in the company

Low risk taking-An intrapreneur does not bear the full risk of failure

Not own boss-An intrapreneur is not his own bosss in legal termsHe enjoys the freedom and gets the required resources and support

2 How is Entrepreneur is different from Intrapreneur

The functions involved in both the entrepreneurship and intrapreneurship are by and large similar however there are several differences between the two

Point of distinction

Entrepreneur

Intrapreneur

status An independent business person

A senior executive within a company

Ownership Owner of

the business

An employeesometimes a share in ownership

Financing Responsible for raising finance for the business

Not responsible for raising the finance

Risk bearing

Bears the risk of the business

Does not bears the risk of the business

Reward Profit which is uncertain and irregularcan be loss

Fixed salary and fringe benefits

Need for security low high

3 What do you understand by enterprise

Answer Enterprise means an undertaking or adventure that requires some innovation and investment and thus involves riskEnterprise always entails decision making coordination and risk bearing

COMMERCE NATURE AND OBJECTIVES OF

BUSINESS

Today let us recall the last other two objectives of business by the chart given in the previous class

Firstly we would discuss Human Objectives

Business is run by people and for people Labour is a valuable business element

Human objectives of business are concerned with the well -being of labour

The human objectives are as follows

Labour welfare Developing human

resources Participative

management Labour

management cooperation

Questions

1 Explain the human objectives of a business enterprise

Answer

Business is run by people and for people Labour is a valuable business element

Human objectives of business are concerned with the well -being of labour

The human objectives are as follows

Labour welfare-Business must recognize the dignity of labour and human factors should be given the recognition

Developing human resources-Employees must be provided the opportunities for developing new skills and attitudes

Participative management-Employees should be allowed to take part in decision making process of business

Labour management cooperation-Business should strive for creating and maintaining cordial employer employee relations so as to ensure peace and progress in industry

Now let us discuss the national objectives of business

Optimum utilization of resources

National self- reliance Development of small

scale industries Development of

backward areas Control over pollution

2Explain the national objectives of a business enterprise

Answer

It is the duty of business to utilize the resources of the country properly the national objectives of business

Optimum utilization of resources ndashBusiness should use the nationrsquos resources in the best possible manner

National self- reliance-It is the duty of the business to help the government in increasing experts and in reducing dependence on imports

Development of small scale industries-Big business firms are expected to encourage growth of small scale industries which are necessary for generating employment

Development of backward areas-Business is expected to give preference to the industrialization of backward regions of the country

ECONOMICS

BASIC ECONOMIC CONCEPTS

SUB

TOPIC

Value

Wealth

Welfare

Today we shall start with a new topic of the same chapter ie lsquoValuersquo

Value of a commodity is defined as the valuation placed by a household on the consumption of this commodity

lsquoValuersquo has two different meanings and these are

a Value -in -use It refers to consumption value of a commodity It expresses the utility derived from the consumption of a particular commodity A necessity like water has a very high value ndashin ndashuse or

Question

1What is value

Answer

Value of a commodity is defined as the valuation placed by a household on the consumption of this commodity

2What is value-in use

Answer It refers to consumption value of a commodity It expresses the utility derived from the consumption of a particular commodity A necessity like water has a very high value ndashin ndashuse or consumption value

3What is value ndashin- exchange

Answer It relates to market value of a commodity

It is the rate at which a particular good or service can be exchanged for money

For example in barter system if a person is prepared to exchange 3 metres of cloth with 1 pair

consumption value

b Value ndashin-exchange It relates to market value of a commodity

It is the rate at which a particular good or service can be exchanged for moneyFor example in barter system if a person is prepared to exchange 3 metres of cloth with 1 pair of shoes then the value in exchange of 3 metres of cloth is 1 pair of shoesValue in exchange is the power of purchasing other goods In modern monetised economies the exchange value of goods are expressed in terms of money as prices

Now let us discuss the term lsquoWealthrsquo

Wealth refers to the stock of all those assets which are a source of income

Wealth is a stock concept

Wealth must possess the following features

a Utility It must possess utility or give some

of shoes then the value in exchange of 3 metres of cloth is 1 pair of shoes

Value in exchange is the power of purchasing other goods In modern monetised economies the exchange value of goods are expressed in terms of money as prices

4 What is wealth

Answer it refers to the stock of assets or goods which are a source of income and have personal or national ownership

5 What are the features of wealth

Answer The features of wealth are as follows

Wealth must possess the following features

a Utility It must possess utility or give some satisfaction

b Scarcity It must be limited in quantityc Transferability it should be transferable its

ownership can be transferred from one person to another person

d Exchange value It must possess exchange value

6 What is welfare

Answer

Welfare is defined as satisfaction and happiness a sense of well- being among the people

satisfactionb Scarcity It must be

limited in quantityc Transferability It

should be transferable its ownership can be transferred from one person to another person

d Exchange value It must possess exchange value

Now let us discuss the term lsquoWelfarersquo

Welfare is defined as satisfaction and happiness a sense of well- being among the people

Welfare is affected by factors like

a Consumption of goods and services

b Environment

c Family relations

d Degree of freedom

e Law and order situation

Mathematics Trigonometric equation

To find the general solution of the equation sinθ=0

When sin θ =0

Then θ= 0 π2π 3π-π -2π -3

i e when θ = 0 or an integral multiple of π

i e when θ= nπ where n is any integer

Therefore the general solution of the equation sin

Example1 Find the general values of θ which satisfy the equation sin2 θ =34

Solution sin2 θ= 34

Or sin θ = +34 or -34

Or sin θ = sin π3 or sin (-π3)

Therefore

θ = [nπ + (-1) n (π3)] or[ nπ+ (-1) n (-π3)]

= nπ +π3 or nπ-π3 where n= any integer

Example 2Find the values of θ which satisfy tan2 θ

θ=0 is θ= nπ where n is any integer

To find the general solution of the equation cos θ=0

When cos θ=0

Then θ=π2 3π2 5π2 -π2 -3π2 -5π3

i e when θ is an odd multiple of π2

i e when θ=(2n+1) π2 where n= any integer

Therefore the general solution of the equation cos θ =0 is θ= (2n+1) π2 where n= any integer

To find the general solution of the equation tan θ = 0

Clearly tan θ =0 implies sin θcos θ =0

Therefore θ = nπ

i e the general solution of the equation tan θ=0 is θ =nπ where n = any integer

To find the general solution of the equation cot θ =0

Clearly cot θ =0 implies (cos θsin θ) = 0

i e cos θ =0

Therefore θ = (2n+1) π2

Therefore the general solution of the equation cot θ =0 is θ = (2n+1) π2

Where n= any integer

To find the general solution of the equation sin θ= k (-

=13 -πleθleπ

Solution tan2 θ =13

Or tan θ = plusmn1radic(3) =tan(plusmnπ6)

θ=nπ plusmn π6 where n =any integer

If n=0 then θ=plusmnπ6

If n=1 then θ= π plusmn π6

If n=-1 then θ= -π plusmn π6

Therefore the required solution in -π le θ le π are θ= π6 5π6 -π6 -5π6

Exercise Find general solution of sin 2θ=cos θ [Hints Use sin 2θ= 2sin θcosθ and then take cosθ

common]

1lek le1)

Determine an angle alpha such that sin =k and -π2le αle π2

Then we have

Sin θ = k = sin α

Or sin θ - sin α =0

Or 2 cos [(θ+α) 2] sin [(θ-α) 2] =0

Therefore either cos [(θ +α) 2] =0 (1)

Or sin [(θ-α) 2] =0 (2)

Now from (1) we get (θ+α) 2= (2m+1) π2)

Or θ = (2m+1) π-α (3)

And from (2) we get (θ-α) 2 =mπ

Or θ= 2mπ+α(4)

Where m = any integer

Clearly the solution (3) amp (4) may be combined in the following form

θ= nπ+(-1) n α where n= any integer

Therefore the general solution of sin θ = sin α is θ = nπ +(-1) n α where n is any integer and -π2 le α le π2

Biology Chapter - 04Kingdom Monera

Today we will discuss about bacterial reproduction and its usefulness

Fig Binary Fission

Fig Conjugation Fig Transformation

Bacterial reproduction is mainly asexual but sexual reproduction

also takes place

Asexual reproduction takes place by i) Binary fission - from one bacteriato

two bacteria are produced in every 20 to 30mins

ii) Buddingiii) Endospore formation - during

unfourable condition

Sexual reproduction by three ways

1) Conjugation - Transfer of genetic material between cells that are in physical contact with one another

2) Transduction - Transfer of genetic materialfrom one cell to another by a bacteriophage

3) Transformation - Transfer of cell-freeor naked DNArsquo from one cell to another

Bacteria causes different diseases inplants animals and human and

it causes food spoilage and waterpollution but it also have some useful

activities

i) Bacteria are helpful in sewage water treatment

ii) It is used in antibiotic (medicine) production

iii) Anaerobic bacteria help in biogas(energy) production

iv) Many household products like yoghurt cheese are manufactured by use of bacteria

v) Rhizobium by symbiotic relationship with leguminous plant increase soil fertility

vi) Besides these bacteria is helpful in genetic engineering degradation of petroleum hydrocarbonand in dairy

industry

Physics Motion in plane Here we will introduce Projectile Motion

Execution

Projectile

Y

usinθ u h

θX

ucosθ

Suppose a body is projected with an angle θ So initial velocity u can be resolved into two components

Horizontal component - ucosθ ( for range)

Vertical component - usinθ ( for height)

usinθ changes during motion and becomes zero at maximum height position but ucosθ remain unchanged

The maximum height of projectile is h

NB If initial is upward then g = -ve and if it is downward then g = +ve Height is +ve if direction of motion does not change ( for ex a body thrown upwards but goes down ultimately then height h = -ve)

The angle of projectile θ is the angle made with horizontal

HISTORY ndash GROWTH OF NATIONALISM

SUB TOPIC- REVOLUTIONARY NATIONALISM Bengal formation of Anushilan Samity and Jugantar Group

The intensification of the Swadeshi movement and Government policy of terror and repression led to outbreak of violence Bombs were manufactured and attempts on the lives of unpopular Government officials became frequent In the gymnasium of Scottish Church College which was known as General Assemblies Institution a secret society was formed known as Anushilan Samity

Aurobindo Ghosh send from Baroda his emissary Jatindranath Banerjee to mobilize the Bengal revolutionaries

Hemchandra Qanungo and Satyen Bose published Journal Jugantar

The Jugantar group planned to assassinate oppressive magistrate Kingsford by Khudiram Bose and Prafulla Chaki in 1908 Prafulla Chaki committed suicide to avoid arrest Khudiram was tried and hanged

Afew days later the police found a bomb factory in Maniktala and arrested a large number of revolutionaries The trial of revolutionaries became famous as the Alipore Bomb Case

In the course of the trial the approver the public prosecuter and a police officer were assassinated

1 Question Name two journals which preached the cult of violence

Answer a) Yugantar edited by Bhupendranath DuttaB) Bandemataram edited by Aurobindo Ghosh2 Question Why was Khudiram arrested and hangedAnswer An attempt was made to assassinate a hated vindictive majistrate named Kingsford by Khudiram Bose and Prafulla Chaki Their attempt failed and the bomb they threw killed two English ladies Khudiram was arrested and put to trial and then hanged3Question Who was Aurubindo GhoshAnswer Aurobindo Ghosh a nationalist revolutionary who was charged for his involvement in the Alipore Bombing Case He was accused of it along with his brother Barindra nath Ghosh But Aurobindo was acquitted because of the brilliant pleading of his counsel Chittaranjan Das Then he became a spiritual reformer introducing his visions on human progress and spiritual evolution4 Qustion Who was KingsfordAnswer Kingsford was an unpopular British chief Magistrate who was the target of the bomb thrown at Muzaffarpur by Khudiram and Prafulla Chaki

Most of the accused were convicted and sentenced to

long term of imprisonmentBut

Aurobindo Ghosh was acquitted mainly owing to the brilliant pleading of his counsel Chittaranjan Das

Political science Topic-Sovereignty

Summary Sovereignty is the full right and power of a governing body over itself without any interference from outside sources or bodies In political theory sovereignty is a substantive term designating supreme legitimate authority over some polity In international law sovereignty is the exercise of power by a state

Internal Sovereignty

Internal sovereignty means supreme authority within ones territory while external sovereignty relates to the recognition on the part of all states that each possesses this power in equal measure

External sovereignty

external sovereignty relates to the recognition on the part of all states that each possesses this power in equal measure

Distinguish between

Execution

Answer the following questions

Short notes-

Sovereignty

Internal Sovereignty

External sovereignty

Homework- learn

external sovereignty and internal sovereigntySovereignty is the principle

of supreme and

unquestionable authority

reflected in the claim by the

state to be the sole author of

laws within its territory

Definition of external vs internal sovereigntyInternal sovereignty refers to

the relationship between a

sovereign power and its

subjects ndash it refers to the

location of the supreme

authority within the state In

the UK for example internal

sovereignty (supposedly)

resides within Parliament

reflected in the

constitutional principle of

parliamentary

sovereigntyBy contrast

external sovereignty refers

to the capacity of the state

to act independently and

autonomously on the world

stage This is what is

sometimes called lsquostate

sovereigntyrsquo or lsquonational

sovereigntyrsquo and implies

that states are legally equal

and that the territorial

integrity and political

independence of a state is

inviolable

Class ndash XII

Date - 2742020 STUDY MATERIALSubject Topic Summary Execution Business Studies

Job Analysis amp Manpower Planning

At first let us recall the chapter what we have discussed till nowbullJob analysisbullJob specification bullJob description bullJob enlargement bullJob enrichment

Today we will do some questions answers from the chapter

Questions 1ldquoJob analysis job description and job specification are interrelatedrdquo Comment Answer) Job analysis is a systematic and detailed examination of a job to collect all the relevant information about it The contents off the job are summarised in the job description The qualification needed for the job are summarised in job specificationThus there is close interrelationship between job analysis job description and job specification

Question 2ldquoJob enlargement is a horizontal extension of a job whereas job enrichment is a vertical extension of a jobrdquo ElucidateAnswer) Job enlargement involves adding one or more task to a job coma where as job enrichment involves adding more autonomy and responsibility to a job Job enlargement is therefore horizontal extension of a job coma whereas job enrichment is a vertical extension of a job

Question 3 )

What is manpower estimation Explain its quantitative and qualitative aspectsAnswer) Manpower estimation is the process by which management determines how an organisation should move from itrsquos current manpower positionto its desired manpower position There are two dimensions of Manpower estimation- quantitative and qualitative

Quantitative aspectThis aspect of Manpower estimation involves estimating the number of employees required in a future time period Workload analysis and workforce analysis are done to estimate the quantity of required manpower

Qualitative aspectThe estimate of the knowledge skills experience etc of required manpower is the qualitative aspect of Manpower estimation The quality of Manpower can be judged on the basis of job analysisand job specification

COMMERCE

CAPITAL-FIXED AND WORKING

Today let us start the class by discussing the sources of finance for different types of business firms

The term lsquocapitalrsquo refers to the investment made in the enterprise for the purpose of earning profits

Requirements of capital and sources of capital for different types of business firms are

1 Capital for sole proprietorship businessA sole proprietor operates at a small scale and thereforerequires a limited amount of capital

2 Capital for partnership firmCapital requirements as well as capital base of a partnership is bigger than that of a sole trader businessThe owned capital is contributed by the partners in an agreed ratio

3 Capital for joint stock companyA joint stock company generally requires large amount of capitalA public company can raise huge capital through issue of shares In addition to share capital it can utilize retained profits

Now let u discuss the meaning of Finance PlanningFinance planning is the process of estimation the financial requirements of an organization specifying the sources of firms and ensuring that enough funds are available at the right time

1 What do you mean by Finance PlanningAnswer Finance planning is the process of estimation the financial requirements of an organization specifying the sources of firms and ensuring that enough funds are available at the right time

2Discuss the role of financial planning of an enterpriseThe role of financial planning are as followsa A sound financial plan helps a business enterprise to avaid the problems of shortage and surplus of fundsbFinancial planning serves as a guide in developing a sound capital structure so as to maximize returns to shareholders c It helps in effective utilization of fundsd It provides policies and procedures for coordinating different functional areas or departments of businesse It enables the management to exercise effective control over the financial activities of an enterprisef It helps the company to prepare for facing business shocks and surprises in future

Mathematics

Continuity and differentiability

Recall Definition of ContinuityLet f(x) be a single valued function of x and x=a be a point in the domain of definition of the function The function is said to be continuous at x=a ifi) f(c) is defined ie f(x) has a definite finite value at x=cii) lim xrarra f(x) exists andiii) lim xrarra f(x) =f(a) In other words f(x) is said to be continuous at x=a if lim xrarra+ f(x)= lim xrarra- f(x) = f(a) Or f(a+0) =f(a-0) =f(a) Or lim hrarr0 f(a+h)= f(a) Algebra of continuous functionsNow we will study some algebra of continuous functions Theorem 1 Suppose f and g be two real functions continuous at a real number c Then(1) f + g is continuous at x = c(2) f ndash g is continuous at x = c(3) f g is continuous at x = c(4) (fg) is continuous at x = c (provided g (c) ne 0)

Example 1 Prove that every rational function is continuousSolution Recall that every rational function f is given byf(x)=[p(x) q(x) ] q(x)ne0where p and q are polynomial functions The domain of f is all real numbers except points at which q is zero Since polynomial functions are continuous f is continuous by (4) of Theorem 1Example 2Discuss the continuity of sine functionSolution To see this we use the following factslim xrarr0 sin x =0Now observe that f (x) = sin x is defined for every real number Let c be a real number Put x = c + h If x rarr c we know that h rarr 0 Therefore lim xrarrc f(x)

= lim xrarrc sin x= lim hrarr0 sin(c+h) =lim hrarr0 [sin c cos h + cos c sin h ]=lim hrarr0 (sin c cos h) + lim hrarr0 (cos c sin h) = sin c +0=sin c = f(c) Thus lim xrarrc f(x) = f(c) and hence f is a constant function Exercise Prove that the function f(x) = x2 +2x is continuous for every real value of x [Hints show that lim xrarra+ f(x) = lim xrarra- f(x) = f(a) ]

Biology Reproduction in Flowering plants We will discuss about megasporoangium

megasporagenesis and female gametophyte

Q4 Describe the structure of megasporangium

Ovule is attached to the placenta by astalk called funicle

Each ovule has one two or three protectivecoverings called integuments

At the tip of integuments a small openingcalled micropyle is organised

Opposite to the micropylar end is the chalaza

Within the integuments a mass of cellsnucellusand inside it embryo sac orfemale gametophyte is present

Q5 Describe a mature embryosacamp its formation

In most of the flowering plants only oneof the 4 megaspores formed as a result ofmegasporogenesis that is functional while theother three degenerate

The

functional megaspore develops into thefemale gametophyte

Formation The nucleus

of the functional megasporedivides mitotically to form two nuclei first andthen two more sequential mitotic nucleardivisions result in the formation of four ampthen eight nucleate stages of embryo sac

Six of the eight nucleus are surrounded bycell walls and organised into cells

The remaining two nuclei called polar nuclei are found below the egg apparatus in the largecentral cell

Three cells consisting of two synergids amp one egg cell present bottom of

embryo sac Three cells

at the chalazal as antipodal cells

Two polar nuclei together present in large central cell

HISTORY

TOWARDS INDEPENDENCE AND PARTITION THE LAST PHASE(1935-1947)SUB TOPIC NATIONAL MOVEMENTS DURING THE SECOND WORLD WAR

Spread of Quit India Movement On 9th August 1942Gandhiji and other Congress leaders were arrested The Congress was declared illegal The news of the arrest of all leaders marked the beginning of a widespread movement of India It was not possible for such a movement to remain peacefulBut the arrest of the all notable congress leaders virtually left the movement in the hands of the mass The movement took the form of violent and militant outbreakBesides congressmen revolutionaries also were very active in the movement The Congress Socialist group also played a prominent role

1 Question Why did the British authority arrest the Congress leaders on 9 th August 1942Answer Congress Working committee adopted the Quit India resolution which was to be ratified at the Bombay AICC meeting in 8th August 1942 They decided to launch a mass struggle on non-violent lines Gandhiji gave a clarion call to all section of the people rdquoKarenge ya Marengerdquo (do or die) Congress leaders gave the call to driving out

the British from IndiaViceroy had taken strong action against the Quit India movement Gandhiji and all the leaders of Congress were arrested

2 Question How did Quit India Movement spread out all over IndiaAnswer The news of the leaders lsquo arrest marked the beginning of a widespread movement to remain peacefulThe movement took form of violent outbreak There were widespread cutting of telephone and Telegraph wires damaging railway lines raising barricades in cities and towns and other forms of violent demonstations

Question Name the leaders of Congress

Socialist group played a prominent part Notable among the Jayprakash Narayan Rammonohar Lohia Aruna Asaf Ali

Political science

Topic-Franchise and Representation

Summary

The election commission

The Election Commission of India is an autonomous constitutional authority responsible for administering Union and State election processes in India The body administers elections to the Lok Sabha Rajya Sabha State Legislative Assemblies in India and the offices of the President and Vice President in the country

Functions of election commission-

India is a sovereign socialist secular democratic republic Democracy runs like a golden thread in the social economic and political fabric woven by the Constitution given by lsquoWe the People of Indiarsquo unto ourselves The concept of democracy as visualised by the Constitution pre-supposes the representation of the people in Parliament and State legislatures by the method of election The Supreme Court has held that democracy is one of the inalienable basic features of the Constitution of India and forms part of its basic structure The Constitution of India adopted a Parliamentary form of government Parliament consists of the President of India and the two Houses mdash Rajya Sabha and Lok Sabha India being a Union of states has separate state legislatures for each state State legislatures consist of the Governor and two Houses mdash Legislative Council and Legislative Assembly mdash in seven states namely Andhra Pradesh Telangana Bihar Jammu amp Kashmir Karnataka Maharashtra and Uttar Pradesh and of the Governor and the state Legislative Assembly in the remaining 22 states Apart from the above two out of the seven Union Territories namely National Capital Territory of Delhi and Puducherry also have their Legislative Assemblies

ExecutionShort notes-Election commissionFunctions of election commission

Homework- Learn

Computer

Science

Computer hardware NAND Gate

A NOT-AND operation is known as NAND operation It has n input (n gt= 2) and one output

Logic diagram

Truth Table

NOR Gate

A NOT-OR operation is known as NOR operation It has n input (n gt= 2) and one output

Logic diagram

Truth Table

XOR Gate

XOR or Ex-OR gate is a special type of gate It can be used in the half

adder full adder and subtractor The exclusive-OR gate is abbreviated as EX-OR gate or sometime as X-OR gate It has n input (n gt= 2) and one output

Logic diagram

Truth Table

XNOR Gate

XNOR gate is a special type of gate It can be used in the half adder full adder and subtractor The exclusive-NOR gate is abbreviated as EX-NOR gate or sometime as X-NOR gate It has n input (n gt= 2) and one output

Logic diagram

Truth Table

Physics

Chapter 1 Electric Field ( Electric Dipole) (Summary)

Here we will derive Expression of electric field at broad side

On position of dipole

Execution

Q With the help of a labelled diagram obtain an expression for the electric field intensity E at any point on the equitorial line ( broad-side on position) of an electric dipole

Ans

E1 E1sinθ

E θ P E1 θ

( r2+L2)12 E2 E

r E2 E2sinθ

-q θ L O L +qA B

Let us consider that the point P is situated on the right bisector of the dipole AB at a distance r meter from its midpoint O

Let E1 and E2 be the electric field intensities of the electric field at P due to charge +q and ndashq of the dipole resp The distance of P from each charge is ( r2+L2)12

So E1 = 14 πϵ q

(r 2+L 2) away from +q

E2 = 14 πϵ q

(r 2+L 2) towards ndashq

The magnitudes of E1 and E2 are equal but directions are different Now resolving E1 and E2 into two components parallel and perpendicular to AB we get

The components perpendicular to AB E1sinθ and E2sinθ cancel each other because they are equal and opposite

The components parallel to AB are E1cosθ and E2 cosθ are in same direction and add up

So resultant intensity of electric field at the point P is

E = E1cosθ + E2 cosθ

E = 14 πϵ q

(r 2+L 2) 2 cosθ

Now from fig we have cosθ =BOBP = L (r2+L2)12

So we get E = 14 πϵ 2qL ( r2+L2)32

Now electric dipole moment p= 2qL

So E = 14 πϵ p ( r2+L2)32

HW Find the expression of Electric field as done here but this time take r gtgt 2L

Also find the expression of torque experience by a dipole

(Hint Electric force experienced by charges of dipole in electric field is qE each Let θ be the angle which dipole makes with electric lines of force then perpendicular distance between two charges is 2Lsinθ Then torque = force x perp distance = qE x 2L sinθ So τ=pE sinθ where p =2qL )

STUDY MATERIAL

Class XIISubject Eng Literature (The Tempest ndash William Shakespeare) Topic Act IV Scene 1 Lines 84 to 133 (Iris hellip A contract of true love Be not too late ) Date 27th April 2020 (4th Period)

[Students should read the original play and also the paraphrase given in the school prescribed textbook]Summary Questions amp Answers

o Ceres soon appears and comes to know that she has been summoned to celebrate the contract of true love

o Ceres expresses her unwillingness to meet Venus and Cupid as she has shunned their company

o Ceres and Juno both bestow their blessings upon Ferdinand and Miranda with June gifting honour riches happiness in marriage and Ceres presents plenty of earthrsquos produce

o Iris summons the water-nymphs and reapers to come and celebrate a contract

(1) IRIS Of her society (Line 91-101)

Be not afraid I met her deity

Cutting the clouds towards Pathos and her sonDove-drawn with her Here thought they to have doneSome wanton charm upon this man and maidWhose vows are that no bed-right shall be paidTill Hymens torch be lightedmdashbut in vainMarss hot minion is returned againHer waspish-headed son has broke his arrowsSwears he will shoot no more but play with sparrowsAnd be a boy right out

(i) Where were Venus and Cupid seen flying How were they travelling Why did they want to join the marriage celebration of Ferdinand and Miranda

of true love

Venus and Cupid were seen flying through the air towards Paphos the famous city which is situated on the island of Cyprus They were travelling by air-borne chariot drawn by doves They certainly wanted to come here in order to play some amorous trick upon Ferdinand and Miranda who are under a vow not to gratify their physical desires till the holy ceremony of their marriage has been performed(ii) What have Venus and Cupid done after failing in their plan

After being failure of their plan Venus who is a very passionate deity and who is the mistress of Mars (the god of war) has gone back while here ill-tempered son Cupid has broken his arrows of love in his state of desperation(iii) What has Cupid firmly decided

Cupid is feeling so disappointed that he has firmly decided to shoot no more arrows to arouse love in human hearts but to spend his time playing with sparrows Thus he would now become just a boy and would give up his original function of shooting arrows on human beings to make them fall in love(iv) What vow had Ceres taken How did Ceres feel at the abduction

After the abduction of her daughter Prosperina by Pluto Ceres had taken a vow to always keep away from the disgraceful company of Venus and her blind son Cupid the god of love Ceres felt deeply distressed when Pluto had carried off her daughter and had made her his wife by force(v) Why has Ceres not forgiven Venus and her blind son For what do Ceres want to be sure

As the abduction had been manipulated by Venus the goddess of beauty and love and her blind son Cupid Ceres has never forgiven them for their part in the whole plot Ceres wants to be sure that she would not have to meet Venus and Cupid who had engineered the abduction of her daughter Prosperina

AS THIS lsquoMASQUErsquo SCENE IS VERY IMPORTANT IN THE PLAY THE PARAPHRASE OF THE ENTIRE PORTION OF MASQUE SCENE (Act IV Lines 58 to 143) IS GIVEN BELOW

IRIS Goddess of RainbowCERES Goddess of Agriculture and all the fruits of the earth

(Nature growth prosperity rebirth ndash notions intimately connected to marriage)JUNO The majestic Queen of Heavens and wife of Jupiter (Jupiter is the king of Gods)

VENUS The Goddess of love CUPID Son of Venus PLUTO God of death (In the play referred by Shakespeare as lsquoDisrsquo which is a Roman name for Pluto)

ORIGINAL TEXT PARAPHRASEPROSPEROWellmdash

PROSPERONow come Ariel Let there be too many rather than too few

Now come my Ariel Bring a corollaryRather than want a spirit Appear and pertly[to Ferdinand and Miranda]No tongue all eyes Be silent

spirits in attendance Appear briskly

[to Ferdinand and Miranda]Look with your eyes but do not say a word

[Soft music] [Soft music][Enter Iris] [Enter Iris]

IRISCeres most bounteous lady thy rich leasOf wheat rye barley vetches oats and peasThy turfy mountains where live nibbling sheepAnd flat meads thatched with stover them to keepThy banks with pioned and twilled brimsWhich spongy April at thy hest betrimsTo make cold nymphs chaste crowns and thybroom-grovesWhose shadow the dismissegraved bachelor lovesBeing lass-lorn thy pole clipped vineyardAnd thy sea-marge sterile and rocky-hardWhere thou thyself dost airmdashthe Queen othrsquoSkyWhose watery arch and messenger am IBids thee leave these and with her sovereign grace[Juno appears] Here on this grass-plot in this very placeTo come and sport Her peacocks fly amainApproach rich Ceres her to entertain

IRISCeres most generous lady you are the cause of rich fields or fertile land where wheat rye barley beans oats and peas grow the grassy mountains where the sheep graze and the flat meadows covered with coarse hay to be used as fodder for cattleYour banks are covered with marsh-marigolds and reeds and the rainy April under your orders brings forth to make for the maids who are not in love beautiful crowns your woods where the broom flourishes and where the bachelor who has been dismissed by the maid he loved lies down being forsaken your vineyard in which the poles are embraced by the vines and the margin of the sea which is barren and rocky where you roam about to enjoy the fresh air ndash the queen of the sky (Juno) whose messenger I am besides being represented as the rainbow bids you leave all these and with her majesty here on this grassy plot in this very place come and sport her peacocks carry her fast in her chariot through the air and are making their way here approach rich Ceres to welcome her

[Enter Ariel as Ceres] [Enter Ariel as Ceres]

CERESHail many-coloured messenger that neerDost disobey the wife of JupiterWho with thy saffron wings upon my flowersDiffusest honey-drops refreshing showersAnd with each end of thy blue bow dost crownMy bosky acres and my unshrubbed downRich scarf to my proud earth Why hath thy queenSummoned me hither to this short-grassed green

CERESWelcome rainbow that never dared disobey Juno the wife of Jupiter who with your orange coloured rays spread honey-drops refreshing showers And with each end of thy blue bow drown my bushy acres and my hilly country which is free from shrubs you thus forming a rich scarf Why has your queen called me here to this place covered with short grass

IRISA contract of true love to celebrateAnd some donation freely to estateOn the blest lovers

IRISI have called you to celebrate a contract of true love and bestow some liberal gift upon the blessed lovers

ORIGINAL TEXT PARAPHRASECERESTell me heavenly bowIf Venus or her son as thou dost knowDo now attend the queen Since they did plotThe means that dusky Dis my daughter gotHer and her blind boys scandaled companyI have forsworn

CERESTell me heavenly bow if Venus the Goddess of love or Cupid her son and pedlar of passion at this time attend the heavenly queen Juno because you are sure to know Since the day they conspired against me and dark Pluto took away my daughter here and Cupidrsquos disgraceful company I have left off

IRISOf her societyBe not afraid I met her deityCutting the clouds towards Pathos and her sonDove-drawn with her Here thought they to have doneSome wanton charm upon this man and miad

IRISBe not afraid of her company I met her deity moving on the clouds towards Paphos the sacred home of Venus on the island of Cyprus along with her son on her chariot drawn by doves Here they contemplated to exercise a charm upon this man and maid producing

Whose vows are that no bed-right shall be paidTill Hymens torch be lightedmdashbut in vainMarss hot minion is returned againHer waspish-headed son has broke his arrowsSwears he will shoot no more but play with sparrowsAnd be a boy right out

wantonness before the actual marriage ceremony but did not succeed Venus has returned her irritable son has broken his arrows and swears that he will give up his practice of trying to inspire love but play with sparrows and be a boy again

[Music is heard] [Music is heard]

CERESHighst queen of stateGreat Juno comes I know her by her gait

CERESHighest queen of state Great Juno there she comes I know here by her gait

[Enter Juno] [Enter Juno]

JUNOHow does my bounteous sister Go with meTo bless this twain that they may Prosperous beAnd honoured in their issue

JUNOHow are you doing my generous sister Come with me to bless this couple so that they may be prosperous and fortunate in their children

[They sing] [They sing]

JUNOHonour riches marriage-blessingLong continuance and increasingHourly joys be still upon youJuno sings her blessings upon you

JUNOMay honour riches happiness in marriage long continuance and increase of those boons ever rest upon you as hourly joys Juno showers down upon you her blessings in song

CERESEarths increase foison plentyBarns and garners never emptyVines and clustring bunches growingPlants and goodly burden bowingSpring come to you at the farthestIn the very end of harvestScarcity and want shall shun youCeresrsquo blessing so is on you

CERESMay you have the plenty of earthrsquos produce Your barns and granaries may never be empty Your vines may grow with clustering bunches Your fruit trees may be heavily laden with their fruit May there be continuous spring and harvest May scantiness and want leave you forever Such is the blessing of Ceres upon you

FERDINANDThis is a most majestic vision andHarmoniously charmingly May I be boldTo think these spirits

FERDINANDThis is a great vision and magically melodious Should I suppose the characters (taking part in the masque) are spirits

PROSPEROSpirits which by mine artI have from their confines calld to enactMy present fancies

PROSPEROYes they are spirits whom I have summoned from the regions to which they are confined to carry into effect my fanciful designs

ORIGINAL TEXT PARAPHRASEFERDINANDLet me live here everSo rare a wondered father and a wifeMakes this place paradise

FERDINANDI should like to live here forever Such a wise and wonderful father makes this place a paradise

[Juno and Ceres whisper and send Iris on employment] [Juno and Ceres whisper and send Iris on employment]

PROSPEROSweet now silence

PROSPEROMy dear Ferdinand speak no more Juno and Ceres are

Juno and Ceres whisper seriouslyTheres something else to do Hush and be muteOr else our spell is marred

whispering with a solemn look There is something else coming Silence Or else our magic will be spoilt

IRISYour nymphs called naiads of the wandering brooksWith your sedged crowns and over-harmless looksLeave your crisp channels and on this green landAnswer your summons Juno does commandCome temperate nymphs and help to celebrateA contract of true love Be not too late

IRISYou nymphs called Naiads denizens (M inhabitants) of the running stream with your chaplets of sedge and ever-helpful looks leave your wrinkled channels and on the green land answer the summons sent to you Juno has ordered some chaste nymphs and help to celebrate a noble and true marriage Donrsquot delay

[Enter certain nymphs] [Enter certain nymphs]You sunburnt sicklemen of August wearyCome hither from the furrow and be merryMake holiday your rye-straw hats put onAnd these fresh nymphs encounter every oneIn country footing

You sunburnt harvesters weary from the effects of the heat in August come here from the furrowed land and rejoice Make holiday with your rye-straw hats upon you and meet these fresh nymphs and join in country dancing

[Enter certain reapers properly habited They join with the nymphs in a graceful dance towards the end whereof Prospero starts suddenly and speaks]

[Enter certain reapers properly habited They join with the nymphs in a graceful dance towards the end whereof Prospero starts suddenly and speaks]

PROSPERO[aside] I had forgot that foul conspiracyOf the beast Caliban and his confederatesAgainst my life The minute of their plotIs almost come [to the spirits]Well done Avoidno more

PROSPERO(Aside)I had forgotten the wicked conspiracy of the beast Caliban and his accomplices against my life the time of their plot has almost arrived ndash (To the Spirits) well done depart no more of this

[To a strange hollow and confused noise the spirits heavily vanish]

[The spirits depart]

ORIGINAL TEXT PARAPHRASEFERDINANDLet me live here everSo rare a wondered father and a wifeMakes this place paradise

FERDINANDI should like to live here forever Such a wise and wonderful father makes this place a paradise

[Juno and Ceres whisper and send Iris on employment] [Juno and Ceres whisper and send Iris on employment]

PROSPEROSweet now silenceJuno and Ceres whisper seriouslyTheres something else to do Hush and be muteOr else our spell is marred

PROSPEROMy dear Ferdinand speak no more Juno and Ceres are whispering with a solemn look There is something else coming Silence Or else our magic will be spoilt

IRISYour nymphs called naiads of the wandering brooksWith your sedged crowns and over-harmless looksLeave your crisp channels and on this green landAnswer your summons Juno does commandCome temperate nymphs and help to celebrateA contract of true love Be not too late

IRISYou nymphs called Naiads denizens (M inhabitants) of the running stream with your chaplets of sedge and ever-helpful looks leave your wrinkled channels and on the green land answer the summons sent to you Juno has ordered some chaste nymphs and help to celebrate a noble and true marriage Donrsquot delay

[Enter certain nymphs] [Enter certain nymphs]You sunburnt sicklemen of August wearyCome hither from the furrow and be merryMake holiday your rye-straw hats put onAnd these fresh nymphs encounter every oneIn country footing

You sunburnt harvesters weary from the effects of the heat in August come here from the furrowed land and rejoice Make holiday with your rye-straw hats upon you and meet these fresh nymphs and join in country dancing

[Enter certain reapers properly habited They join with the nymphs in a graceful dance towards the end whereof Prospero starts suddenly and speaks]

[Enter certain reapers properly habited They join with the nymphs in a graceful dance towards the end whereof Prospero starts suddenly and speaks]

PROSPERO[aside] I had forgot that foul conspiracyOf the beast Caliban and his confederatesAgainst my life The minute of their plotIs almost come [to the spirits]Well done Avoidno more

PROSPERO(Aside)I had forgotten the wicked conspiracy of the beast Caliban and his accomplices against my life the time of their plot has almost arrived ndash (To the Spirits) well done depart no more of this

[To a strange hollow and confused noise the spirits heavily vanish]

[The spirits depart]

Ac-12 27420 topic Revaluation of Assets and Liabilities

REVALUATION OF ASSETS AND LIABILITIES

On admission of a new partner the firm stands reconstituted and consequently the assets are revalued and liabilities are reassessed It is necessary to show the true position of the firm at the time of admission of a new partner If the values of the assets are raised gain will increase the capital of the existing partners Similarly any decrease in the value of assets ie loss will decrease the capital of the existing partners For this purpose alsquoRevaluation Accountrsquo is prepared This account is credited with all increases in the value of assets and decrease in the value of liabilities It is debited with decrease on account of value of assets and increase in the value of liabilities The balance of this account shows a gain or loss on revaluation which is transferred to the existing partnerrsquos capital account in existing profit sharing ratioAccounting for Revaluation of Assets and Liabilities when there is a Changein the Profit Sharing Ratio of Existing PartnersAssets and liabilities of a firm must also be revalued at the time of change in profit sharing ratio of existing partners The reason is that the realisable or actual value of assets and liabilities may be different from those shown in the Balance Sheet It is possible that with the passage of time some of the assets might have appreciated in value while the value of certain other assets might have decreased and no record has been made of such changes in the books of accounts Similarly there may be some unrecorded assets amp libilities that may have to be accounted for Revaluation of assets and reassessments of liabilities becomes necessary because the change in the

value of assets and liabilities belongs to the period to change in profit sharing ratio and hence must be shared by the partners in their old profit sharing ratio Revaluation of assets and reassessment of liabilities may be given effect to in two different ways (a) When revised values are to be recorded in the books and(b) When revised values are not to be recorded in the books

When revised values are to be recorded in the booksIn such a case revaluation of assets and reassessment of liabilities is done with the help of a new account called lsquoRevaluation Accountrsquo Sometimes this account is also called as lsquoProfit amp Loss Adjustment Acrsquo If there is a loss due to revaluation revaluation account is debited and if the revaluation results in a profit the revaluation account is credited The following journal entries made for this purpose are

(i) For increase in the value of assetsAsset Ac Dr (individually)To Revaluation Ac(ii) For decrease in the value of AssetRevaluation Ac Dr (individually)To Asset Ac[Decrease in the value of assets](iii) For increase in the value of LiabilitiesRevaluation Ac Dr (individually)To Liabilities Ac[Increase in the value of Liabilities](iv) For decrease in the value of LiabilitiesLiabilities Ac DrTo Revaluation Ac[Decrease in the value of Liabilities](v) For unrecorded AssetsAsset Ac [unrecorded] DrTo Revaluation Ac[Unrecorded asset recorded at actual value](vi) For unrecorded Liability Revaluation Ac DrTo Liability Ac [unrecorded][Unrecorded Liability recorded at actual value](vii) For transfer of gain on revaluationRevaluation Ac DrTo Existing Partnerrsquos CapitalCurrent Ac[Profit on revaluation transferred to capital account in existing ratio](viii) For transfer of loss on revaluationExisting Partnerrsquos CapitalCurrent Ac DrTo Revaluation Ac[Loss on revaluation transferred to capital account in existing ratio](a) When revaluation account shows gain Revaluation Ac DrTo Partnerrsquos Capital Ac (Old Profit Sharing Ratio)(Profit on revaluation credited to Partnerrsquos Capital Ac)(b) Above entry is reversed when revaluation account shows loss Partners Capital Acs (Old Profit Sharing Ratio) DrTo Revaluation Ac(Loss on revaluation debited to Partnerrsquos Capital Acs)

Proforma of Revaluation Account is given as under

Revaluation Account

Dr Cr Particulars ` Amount Particulars ` Amount To Decrease in value of assets By Increase in value of assets To Increase in value of liabilities By Decrease in value of liabilities To Unrecorded liabilities By Unrecorded assets To Gain on Revaluation (Transferred) By Loss on Revalution (Transferred)

ECO ndash12 2742020Topic- ELASTICITY OF DEMAND

CHAPTER - ELASTICITY OF DEMANDMEANINGDemand for a commodity is affected by many factors such as its price price of related goods income of its buyer tastes and preferences etc Elasticity means degree of response Elasticity of demand means degree of responsiveness of demand Demand for a commodity responds to change in price price of related goods income etc So we have three dimensions of elasticity of demandDIMENSION OF ELASTICITY OF DEMAND TYPES OF ELASTICITY OF DEMAND

Price elasticity of demand Income elasticity of demand Cross Elasticity of demand

Price elasticity of demand Price elasticity of demand means degree of responsiveness of demand for a commodity to the change in its price For example if demand for a commodity rises by 10 due to 5 fall in its price Price elasticity of demand (ep)=Percentage change in quantity demanded Percentage change in price of the commodity = 10 ( -)5 = ( - )2Note that ep will always be negative due to inverse relationship of price and quantity demanded

(ii) Income elasticity of demand Income elasticity of demand refers to the degree of responsiveness of demand for a commodity to the change in income of its buyer Suppose income of buyer rises by 10 and his demand for a commodity rises by 20 then Income elasticity of demand (ey)= change in quantity demanded change in price of the commodity =20 10 = 2

Cross Elasticity of demandCross elasticity of demand means the degree of responsiveness of demand for a commodity to the change in price of its related goods (substitute goods or complementary goods) Suppose demand for a commodity rises by 10 due to 5 rise in price of its substitute good then Cross elasticity of demand (ec) = change in quantity demanded change in price of related good = 10 2 = 5 (Tastes and preferences cannot be expressed numerically So elasticity ofdemand cannot be numerically expressed)

  • Chapter 1 Force (Summary)
  • Distinguish between external sovereignty and internal sovereignty
    • NAND Gate
      • Logic diagram
      • Truth Table
        • NOR Gate
          • Logic diagram
          • Truth Table
            • XOR Gate
              • Logic diagram
              • Truth Table
                • XNOR Gate
                  • Logic diagram
                  • Truth Table
                      • Physics
                      • Chapter 1 Electric Field ( Electric Dipole) (Summary)
Page 24:  · Web viewWe all know that Nouns are divided into two parts: common noun and proper noun.Apart from common and proper noun, we will also study about collective noun and compound

twelve years back

IMPORTANT PASSAGES EXPLAINED(Line 98-103)

PROSPERO helliphelliphelliphelliphelliphelliphellip

He being thus lorded

Not only with what my revenue yieldedBut what my power might else exact like oneWho having into truth by telling of it Made such a sinner of his memoryTo credit his own lie he did believeHe was indeed the duke

Prospero in telling the narrative of his past life here refers to his brother Antonio Prospero being with a studious bent of mind has left the administration of Milan on his younger brother Now Antonio being thus invested like a lord with all the powers derived from Prosperorsquos wealth and what the exercise of Prosperorsquos authority might secure for him regarded himself as a de facto Duke of Milan It is a well-known fact of psychology that a man who repeatedly tells a lie makes of his memory such a sinner against truth as to credit his own lie by the telling of it So Antonio by repeatedly saying to himself and others that he was the Duke came to believe that he was really the Duke Thus falsehood repeatedly asserted gained the force of truth for Antonio and he truly believed it

Should presently extirpate me and mine Out of the dukedom and confer fair MilanWith all the honours on my brother whereonA treacherous army levied one midnightFated to thrsquo purpose did Antonio openThe gates of Milan and ithrsquo dead of darkness The ministers for thrsquo purpose hurried thenceMe and thy crying self

(i) In the earlier lines of this scene what does Prospero tell about his intense interest What was the demand of his interest

In the earlier lines of this scene Prospero tells Miranda that he had an intense interest in the study of philosophy and magic arts Hence in order to improve his mind with this kind of study he kept himself isolated from worldly and state affairs His study was dearer to him than the applause and esteem that he could win from the public His study demanded too much solitude(ii) What forced Antonio to take an undue advantage over Prospero

Prosperorsquos indifferent attitude towards the statersquos affairs and his having boundless trust in Antonio gave rise to a boundless lust for power in Antoniorsquos mind Antonio felt that he must be the actual Duke instead of the part of the Duke he played Thus Antonio took an undue advantage of the situation to usurp Prosperorsquos dukedom(iii) Explain the following lines ldquoI should sin to think but nobly of my grandmother Good wombs have borne bad sonsrdquo

After hearing the treacherous act of her uncle Antonio Miranda says that Prosperorsquos mother was a noble lady and she cannot dishonour her memory by saying that the person named Antonio cannot be his (Prosperorsquos) brother She says that in honour of her grandmother she also cannot say that Antonio must have been begotten not by her grandfather but by some other man She finally concedes that it is known that good mothers have borne bad sons in their wombs and gave birth to them

(iv) Why did the King of Naples accept Antoniorsquos request to help him in usurping his dukedom What did Antonio propose to Alonso

Alonso who was the king of Naples accepted Antoniorsquos request in usurping Prosperorsquos dukedom because he (Alonso) was a sworn enemy of Prospero Antonio proposed that Alonso should immediately drive him (Prospero) and his offspring out of Milan and should confer the dukedom upon him (Antonio) with all the dignities which go with that

position In return Antonio promised that he shall give an annual tribute and also swore his allegiance to Alonso Also he agreed to hold the Dukedom of Milan as a subordinate to the state of Naples(v) How were Prospero and Miranda carried away from the city of Milan and what was the state of small Miranda at that time

In pursuance of the agreement settled between Antonio and Alonso an army of treacherous men was assembled One midnight when the occasion suited the will of destiny Antonio opened the gates of the city of Milan and in the death like silence of midnight Antoniorsquos agents who had been directed to execute his purpose carried Prospero and small Miranda away from the city in all haste They were then forced into a ship and carried some distance out to sea where they put them on a mere hulk of a boat without any rigging or ship-gear and abandoned them leaving them at the mercy of the roaring sea Miranda was a very small child of three years age and she was crying at that time

CLASS -XIDATE-270420Subject Topic Summary Execution

EVS Chapter 1 ndash Mode of Existence

Impact of mode of existence on resources

Q) Why resources are under pressure

Ans - Increase in the sophistication

of technology enabling natural resources to be extracted quickly and efficiently Eg in the past it could take long hours just to cut down one tree only using saws Due to increased technology rates of deforestation have greatly increased

The number of humans is increasing Cultures of consumerism Materialistic views

lead to the mining of gold and diamonds to produce jewelry unnecessary commodities for human life or advancement Consumerism also leads to extraction of resources for the

production of commodities necessary for human life but in amounts excessive of what is needed because people consume more than is necessary or waste what they have

Lack of awareness among the population is striking People are not aware of ways to reduce depletion and exploitation of materials

Accounts Cash Book Today we are going to start a new topic -Cash Book

The key terms used in this chapter are

bullCash book

bullSimple cash book

bullDouble column cash book bullTriple column cash book

bullPetty cash book

bullCash discount

bullContra entry

Here I will share you the meaning of each key terms

bullCash book Cash Book is a special purpose subsidiary book or journal in which cash received and cash payments are recorded

bullSimple cash book

It is a cash book in which only cash transactions are recorded It has only one column on each side

bullTriple column cash book

It is cash book which has three columns one column for each cash and Bankdiscount on each side of the cash book In this book both cash and Bank transactions are recorded together with discount allowed and received

bullPetty cash book

It is a cash book maintained for recording petty expenses

bullCash discount

Cash discount is the amount of discount received or allowed on cash payments and cash receipts Discount received is an income for the business while discount allowed isan expense

bullContra entry

It means transactions involving both cash and Bank Such transactions though recorded in the cash book are not posted into ledger The letter lsquoC is written in Ledger folio for contra entry

Business Studies

ENTREPRENEURSHIP

Now we shall discuss the second chapter

lsquoENTREPRENEURSHIPrsquo

Today before starting the chapter let us recall what

Questions

1What are the main characteristics of Intrapreneurship

Answer

The main characteristics of Intrapreneurship are

Corporate framework-it occurs within the framework of the same company

Semi-Autonomous-Intrapreneurship

we have read last day

Let s today start the class by recalling the last topic taught

Intrapreneurship is the process of discovering and exploring business opportunities within an existing company It involves launching new business ventures within the framework of a present corporation Intrapreneurship is also known as corporate entrepreneurship or corporate venturing

Now let us start with the characteristics of Intrapreneurship

The main characteristics of Intrapreneurship are

Corporate framework

Semi-Autonomous Lack of ownership Senior position Low risk taking Not own boss

Now let us discuss the meaning of enterprise

Enterprise means an undertaking or adventure that requires some innovation and investment and thus involves riskEnterprise always entails decision making coordination and risk bearing

involves crating amd nurturing a semi-autonomous business unit which may be a subsidiary a strategic business unit or a division

Lack of ownership-the intrapreneur is not the owner of the unitb he creates and nurtures

Senior position-he occupies a senior managerial position in the company

Low risk taking-An intrapreneur does not bear the full risk of failure

Not own boss-An intrapreneur is not his own bosss in legal termsHe enjoys the freedom and gets the required resources and support

2 How is Entrepreneur is different from Intrapreneur

The functions involved in both the entrepreneurship and intrapreneurship are by and large similar however there are several differences between the two

Point of distinction

Entrepreneur

Intrapreneur

status An independent business person

A senior executive within a company

Ownership Owner of

the business

An employeesometimes a share in ownership

Financing Responsible for raising finance for the business

Not responsible for raising the finance

Risk bearing

Bears the risk of the business

Does not bears the risk of the business

Reward Profit which is uncertain and irregularcan be loss

Fixed salary and fringe benefits

Need for security low high

3 What do you understand by enterprise

Answer Enterprise means an undertaking or adventure that requires some innovation and investment and thus involves riskEnterprise always entails decision making coordination and risk bearing

COMMERCE NATURE AND OBJECTIVES OF

BUSINESS

Today let us recall the last other two objectives of business by the chart given in the previous class

Firstly we would discuss Human Objectives

Business is run by people and for people Labour is a valuable business element

Human objectives of business are concerned with the well -being of labour

The human objectives are as follows

Labour welfare Developing human

resources Participative

management Labour

management cooperation

Questions

1 Explain the human objectives of a business enterprise

Answer

Business is run by people and for people Labour is a valuable business element

Human objectives of business are concerned with the well -being of labour

The human objectives are as follows

Labour welfare-Business must recognize the dignity of labour and human factors should be given the recognition

Developing human resources-Employees must be provided the opportunities for developing new skills and attitudes

Participative management-Employees should be allowed to take part in decision making process of business

Labour management cooperation-Business should strive for creating and maintaining cordial employer employee relations so as to ensure peace and progress in industry

Now let us discuss the national objectives of business

Optimum utilization of resources

National self- reliance Development of small

scale industries Development of

backward areas Control over pollution

2Explain the national objectives of a business enterprise

Answer

It is the duty of business to utilize the resources of the country properly the national objectives of business

Optimum utilization of resources ndashBusiness should use the nationrsquos resources in the best possible manner

National self- reliance-It is the duty of the business to help the government in increasing experts and in reducing dependence on imports

Development of small scale industries-Big business firms are expected to encourage growth of small scale industries which are necessary for generating employment

Development of backward areas-Business is expected to give preference to the industrialization of backward regions of the country

ECONOMICS

BASIC ECONOMIC CONCEPTS

SUB

TOPIC

Value

Wealth

Welfare

Today we shall start with a new topic of the same chapter ie lsquoValuersquo

Value of a commodity is defined as the valuation placed by a household on the consumption of this commodity

lsquoValuersquo has two different meanings and these are

a Value -in -use It refers to consumption value of a commodity It expresses the utility derived from the consumption of a particular commodity A necessity like water has a very high value ndashin ndashuse or

Question

1What is value

Answer

Value of a commodity is defined as the valuation placed by a household on the consumption of this commodity

2What is value-in use

Answer It refers to consumption value of a commodity It expresses the utility derived from the consumption of a particular commodity A necessity like water has a very high value ndashin ndashuse or consumption value

3What is value ndashin- exchange

Answer It relates to market value of a commodity

It is the rate at which a particular good or service can be exchanged for money

For example in barter system if a person is prepared to exchange 3 metres of cloth with 1 pair

consumption value

b Value ndashin-exchange It relates to market value of a commodity

It is the rate at which a particular good or service can be exchanged for moneyFor example in barter system if a person is prepared to exchange 3 metres of cloth with 1 pair of shoes then the value in exchange of 3 metres of cloth is 1 pair of shoesValue in exchange is the power of purchasing other goods In modern monetised economies the exchange value of goods are expressed in terms of money as prices

Now let us discuss the term lsquoWealthrsquo

Wealth refers to the stock of all those assets which are a source of income

Wealth is a stock concept

Wealth must possess the following features

a Utility It must possess utility or give some

of shoes then the value in exchange of 3 metres of cloth is 1 pair of shoes

Value in exchange is the power of purchasing other goods In modern monetised economies the exchange value of goods are expressed in terms of money as prices

4 What is wealth

Answer it refers to the stock of assets or goods which are a source of income and have personal or national ownership

5 What are the features of wealth

Answer The features of wealth are as follows

Wealth must possess the following features

a Utility It must possess utility or give some satisfaction

b Scarcity It must be limited in quantityc Transferability it should be transferable its

ownership can be transferred from one person to another person

d Exchange value It must possess exchange value

6 What is welfare

Answer

Welfare is defined as satisfaction and happiness a sense of well- being among the people

satisfactionb Scarcity It must be

limited in quantityc Transferability It

should be transferable its ownership can be transferred from one person to another person

d Exchange value It must possess exchange value

Now let us discuss the term lsquoWelfarersquo

Welfare is defined as satisfaction and happiness a sense of well- being among the people

Welfare is affected by factors like

a Consumption of goods and services

b Environment

c Family relations

d Degree of freedom

e Law and order situation

Mathematics Trigonometric equation

To find the general solution of the equation sinθ=0

When sin θ =0

Then θ= 0 π2π 3π-π -2π -3

i e when θ = 0 or an integral multiple of π

i e when θ= nπ where n is any integer

Therefore the general solution of the equation sin

Example1 Find the general values of θ which satisfy the equation sin2 θ =34

Solution sin2 θ= 34

Or sin θ = +34 or -34

Or sin θ = sin π3 or sin (-π3)

Therefore

θ = [nπ + (-1) n (π3)] or[ nπ+ (-1) n (-π3)]

= nπ +π3 or nπ-π3 where n= any integer

Example 2Find the values of θ which satisfy tan2 θ

θ=0 is θ= nπ where n is any integer

To find the general solution of the equation cos θ=0

When cos θ=0

Then θ=π2 3π2 5π2 -π2 -3π2 -5π3

i e when θ is an odd multiple of π2

i e when θ=(2n+1) π2 where n= any integer

Therefore the general solution of the equation cos θ =0 is θ= (2n+1) π2 where n= any integer

To find the general solution of the equation tan θ = 0

Clearly tan θ =0 implies sin θcos θ =0

Therefore θ = nπ

i e the general solution of the equation tan θ=0 is θ =nπ where n = any integer

To find the general solution of the equation cot θ =0

Clearly cot θ =0 implies (cos θsin θ) = 0

i e cos θ =0

Therefore θ = (2n+1) π2

Therefore the general solution of the equation cot θ =0 is θ = (2n+1) π2

Where n= any integer

To find the general solution of the equation sin θ= k (-

=13 -πleθleπ

Solution tan2 θ =13

Or tan θ = plusmn1radic(3) =tan(plusmnπ6)

θ=nπ plusmn π6 where n =any integer

If n=0 then θ=plusmnπ6

If n=1 then θ= π plusmn π6

If n=-1 then θ= -π plusmn π6

Therefore the required solution in -π le θ le π are θ= π6 5π6 -π6 -5π6

Exercise Find general solution of sin 2θ=cos θ [Hints Use sin 2θ= 2sin θcosθ and then take cosθ

common]

1lek le1)

Determine an angle alpha such that sin =k and -π2le αle π2

Then we have

Sin θ = k = sin α

Or sin θ - sin α =0

Or 2 cos [(θ+α) 2] sin [(θ-α) 2] =0

Therefore either cos [(θ +α) 2] =0 (1)

Or sin [(θ-α) 2] =0 (2)

Now from (1) we get (θ+α) 2= (2m+1) π2)

Or θ = (2m+1) π-α (3)

And from (2) we get (θ-α) 2 =mπ

Or θ= 2mπ+α(4)

Where m = any integer

Clearly the solution (3) amp (4) may be combined in the following form

θ= nπ+(-1) n α where n= any integer

Therefore the general solution of sin θ = sin α is θ = nπ +(-1) n α where n is any integer and -π2 le α le π2

Biology Chapter - 04Kingdom Monera

Today we will discuss about bacterial reproduction and its usefulness

Fig Binary Fission

Fig Conjugation Fig Transformation

Bacterial reproduction is mainly asexual but sexual reproduction

also takes place

Asexual reproduction takes place by i) Binary fission - from one bacteriato

two bacteria are produced in every 20 to 30mins

ii) Buddingiii) Endospore formation - during

unfourable condition

Sexual reproduction by three ways

1) Conjugation - Transfer of genetic material between cells that are in physical contact with one another

2) Transduction - Transfer of genetic materialfrom one cell to another by a bacteriophage

3) Transformation - Transfer of cell-freeor naked DNArsquo from one cell to another

Bacteria causes different diseases inplants animals and human and

it causes food spoilage and waterpollution but it also have some useful

activities

i) Bacteria are helpful in sewage water treatment

ii) It is used in antibiotic (medicine) production

iii) Anaerobic bacteria help in biogas(energy) production

iv) Many household products like yoghurt cheese are manufactured by use of bacteria

v) Rhizobium by symbiotic relationship with leguminous plant increase soil fertility

vi) Besides these bacteria is helpful in genetic engineering degradation of petroleum hydrocarbonand in dairy

industry

Physics Motion in plane Here we will introduce Projectile Motion

Execution

Projectile

Y

usinθ u h

θX

ucosθ

Suppose a body is projected with an angle θ So initial velocity u can be resolved into two components

Horizontal component - ucosθ ( for range)

Vertical component - usinθ ( for height)

usinθ changes during motion and becomes zero at maximum height position but ucosθ remain unchanged

The maximum height of projectile is h

NB If initial is upward then g = -ve and if it is downward then g = +ve Height is +ve if direction of motion does not change ( for ex a body thrown upwards but goes down ultimately then height h = -ve)

The angle of projectile θ is the angle made with horizontal

HISTORY ndash GROWTH OF NATIONALISM

SUB TOPIC- REVOLUTIONARY NATIONALISM Bengal formation of Anushilan Samity and Jugantar Group

The intensification of the Swadeshi movement and Government policy of terror and repression led to outbreak of violence Bombs were manufactured and attempts on the lives of unpopular Government officials became frequent In the gymnasium of Scottish Church College which was known as General Assemblies Institution a secret society was formed known as Anushilan Samity

Aurobindo Ghosh send from Baroda his emissary Jatindranath Banerjee to mobilize the Bengal revolutionaries

Hemchandra Qanungo and Satyen Bose published Journal Jugantar

The Jugantar group planned to assassinate oppressive magistrate Kingsford by Khudiram Bose and Prafulla Chaki in 1908 Prafulla Chaki committed suicide to avoid arrest Khudiram was tried and hanged

Afew days later the police found a bomb factory in Maniktala and arrested a large number of revolutionaries The trial of revolutionaries became famous as the Alipore Bomb Case

In the course of the trial the approver the public prosecuter and a police officer were assassinated

1 Question Name two journals which preached the cult of violence

Answer a) Yugantar edited by Bhupendranath DuttaB) Bandemataram edited by Aurobindo Ghosh2 Question Why was Khudiram arrested and hangedAnswer An attempt was made to assassinate a hated vindictive majistrate named Kingsford by Khudiram Bose and Prafulla Chaki Their attempt failed and the bomb they threw killed two English ladies Khudiram was arrested and put to trial and then hanged3Question Who was Aurubindo GhoshAnswer Aurobindo Ghosh a nationalist revolutionary who was charged for his involvement in the Alipore Bombing Case He was accused of it along with his brother Barindra nath Ghosh But Aurobindo was acquitted because of the brilliant pleading of his counsel Chittaranjan Das Then he became a spiritual reformer introducing his visions on human progress and spiritual evolution4 Qustion Who was KingsfordAnswer Kingsford was an unpopular British chief Magistrate who was the target of the bomb thrown at Muzaffarpur by Khudiram and Prafulla Chaki

Most of the accused were convicted and sentenced to

long term of imprisonmentBut

Aurobindo Ghosh was acquitted mainly owing to the brilliant pleading of his counsel Chittaranjan Das

Political science Topic-Sovereignty

Summary Sovereignty is the full right and power of a governing body over itself without any interference from outside sources or bodies In political theory sovereignty is a substantive term designating supreme legitimate authority over some polity In international law sovereignty is the exercise of power by a state

Internal Sovereignty

Internal sovereignty means supreme authority within ones territory while external sovereignty relates to the recognition on the part of all states that each possesses this power in equal measure

External sovereignty

external sovereignty relates to the recognition on the part of all states that each possesses this power in equal measure

Distinguish between

Execution

Answer the following questions

Short notes-

Sovereignty

Internal Sovereignty

External sovereignty

Homework- learn

external sovereignty and internal sovereigntySovereignty is the principle

of supreme and

unquestionable authority

reflected in the claim by the

state to be the sole author of

laws within its territory

Definition of external vs internal sovereigntyInternal sovereignty refers to

the relationship between a

sovereign power and its

subjects ndash it refers to the

location of the supreme

authority within the state In

the UK for example internal

sovereignty (supposedly)

resides within Parliament

reflected in the

constitutional principle of

parliamentary

sovereigntyBy contrast

external sovereignty refers

to the capacity of the state

to act independently and

autonomously on the world

stage This is what is

sometimes called lsquostate

sovereigntyrsquo or lsquonational

sovereigntyrsquo and implies

that states are legally equal

and that the territorial

integrity and political

independence of a state is

inviolable

Class ndash XII

Date - 2742020 STUDY MATERIALSubject Topic Summary Execution Business Studies

Job Analysis amp Manpower Planning

At first let us recall the chapter what we have discussed till nowbullJob analysisbullJob specification bullJob description bullJob enlargement bullJob enrichment

Today we will do some questions answers from the chapter

Questions 1ldquoJob analysis job description and job specification are interrelatedrdquo Comment Answer) Job analysis is a systematic and detailed examination of a job to collect all the relevant information about it The contents off the job are summarised in the job description The qualification needed for the job are summarised in job specificationThus there is close interrelationship between job analysis job description and job specification

Question 2ldquoJob enlargement is a horizontal extension of a job whereas job enrichment is a vertical extension of a jobrdquo ElucidateAnswer) Job enlargement involves adding one or more task to a job coma where as job enrichment involves adding more autonomy and responsibility to a job Job enlargement is therefore horizontal extension of a job coma whereas job enrichment is a vertical extension of a job

Question 3 )

What is manpower estimation Explain its quantitative and qualitative aspectsAnswer) Manpower estimation is the process by which management determines how an organisation should move from itrsquos current manpower positionto its desired manpower position There are two dimensions of Manpower estimation- quantitative and qualitative

Quantitative aspectThis aspect of Manpower estimation involves estimating the number of employees required in a future time period Workload analysis and workforce analysis are done to estimate the quantity of required manpower

Qualitative aspectThe estimate of the knowledge skills experience etc of required manpower is the qualitative aspect of Manpower estimation The quality of Manpower can be judged on the basis of job analysisand job specification

COMMERCE

CAPITAL-FIXED AND WORKING

Today let us start the class by discussing the sources of finance for different types of business firms

The term lsquocapitalrsquo refers to the investment made in the enterprise for the purpose of earning profits

Requirements of capital and sources of capital for different types of business firms are

1 Capital for sole proprietorship businessA sole proprietor operates at a small scale and thereforerequires a limited amount of capital

2 Capital for partnership firmCapital requirements as well as capital base of a partnership is bigger than that of a sole trader businessThe owned capital is contributed by the partners in an agreed ratio

3 Capital for joint stock companyA joint stock company generally requires large amount of capitalA public company can raise huge capital through issue of shares In addition to share capital it can utilize retained profits

Now let u discuss the meaning of Finance PlanningFinance planning is the process of estimation the financial requirements of an organization specifying the sources of firms and ensuring that enough funds are available at the right time

1 What do you mean by Finance PlanningAnswer Finance planning is the process of estimation the financial requirements of an organization specifying the sources of firms and ensuring that enough funds are available at the right time

2Discuss the role of financial planning of an enterpriseThe role of financial planning are as followsa A sound financial plan helps a business enterprise to avaid the problems of shortage and surplus of fundsbFinancial planning serves as a guide in developing a sound capital structure so as to maximize returns to shareholders c It helps in effective utilization of fundsd It provides policies and procedures for coordinating different functional areas or departments of businesse It enables the management to exercise effective control over the financial activities of an enterprisef It helps the company to prepare for facing business shocks and surprises in future

Mathematics

Continuity and differentiability

Recall Definition of ContinuityLet f(x) be a single valued function of x and x=a be a point in the domain of definition of the function The function is said to be continuous at x=a ifi) f(c) is defined ie f(x) has a definite finite value at x=cii) lim xrarra f(x) exists andiii) lim xrarra f(x) =f(a) In other words f(x) is said to be continuous at x=a if lim xrarra+ f(x)= lim xrarra- f(x) = f(a) Or f(a+0) =f(a-0) =f(a) Or lim hrarr0 f(a+h)= f(a) Algebra of continuous functionsNow we will study some algebra of continuous functions Theorem 1 Suppose f and g be two real functions continuous at a real number c Then(1) f + g is continuous at x = c(2) f ndash g is continuous at x = c(3) f g is continuous at x = c(4) (fg) is continuous at x = c (provided g (c) ne 0)

Example 1 Prove that every rational function is continuousSolution Recall that every rational function f is given byf(x)=[p(x) q(x) ] q(x)ne0where p and q are polynomial functions The domain of f is all real numbers except points at which q is zero Since polynomial functions are continuous f is continuous by (4) of Theorem 1Example 2Discuss the continuity of sine functionSolution To see this we use the following factslim xrarr0 sin x =0Now observe that f (x) = sin x is defined for every real number Let c be a real number Put x = c + h If x rarr c we know that h rarr 0 Therefore lim xrarrc f(x)

= lim xrarrc sin x= lim hrarr0 sin(c+h) =lim hrarr0 [sin c cos h + cos c sin h ]=lim hrarr0 (sin c cos h) + lim hrarr0 (cos c sin h) = sin c +0=sin c = f(c) Thus lim xrarrc f(x) = f(c) and hence f is a constant function Exercise Prove that the function f(x) = x2 +2x is continuous for every real value of x [Hints show that lim xrarra+ f(x) = lim xrarra- f(x) = f(a) ]

Biology Reproduction in Flowering plants We will discuss about megasporoangium

megasporagenesis and female gametophyte

Q4 Describe the structure of megasporangium

Ovule is attached to the placenta by astalk called funicle

Each ovule has one two or three protectivecoverings called integuments

At the tip of integuments a small openingcalled micropyle is organised

Opposite to the micropylar end is the chalaza

Within the integuments a mass of cellsnucellusand inside it embryo sac orfemale gametophyte is present

Q5 Describe a mature embryosacamp its formation

In most of the flowering plants only oneof the 4 megaspores formed as a result ofmegasporogenesis that is functional while theother three degenerate

The

functional megaspore develops into thefemale gametophyte

Formation The nucleus

of the functional megasporedivides mitotically to form two nuclei first andthen two more sequential mitotic nucleardivisions result in the formation of four ampthen eight nucleate stages of embryo sac

Six of the eight nucleus are surrounded bycell walls and organised into cells

The remaining two nuclei called polar nuclei are found below the egg apparatus in the largecentral cell

Three cells consisting of two synergids amp one egg cell present bottom of

embryo sac Three cells

at the chalazal as antipodal cells

Two polar nuclei together present in large central cell

HISTORY

TOWARDS INDEPENDENCE AND PARTITION THE LAST PHASE(1935-1947)SUB TOPIC NATIONAL MOVEMENTS DURING THE SECOND WORLD WAR

Spread of Quit India Movement On 9th August 1942Gandhiji and other Congress leaders were arrested The Congress was declared illegal The news of the arrest of all leaders marked the beginning of a widespread movement of India It was not possible for such a movement to remain peacefulBut the arrest of the all notable congress leaders virtually left the movement in the hands of the mass The movement took the form of violent and militant outbreakBesides congressmen revolutionaries also were very active in the movement The Congress Socialist group also played a prominent role

1 Question Why did the British authority arrest the Congress leaders on 9 th August 1942Answer Congress Working committee adopted the Quit India resolution which was to be ratified at the Bombay AICC meeting in 8th August 1942 They decided to launch a mass struggle on non-violent lines Gandhiji gave a clarion call to all section of the people rdquoKarenge ya Marengerdquo (do or die) Congress leaders gave the call to driving out

the British from IndiaViceroy had taken strong action against the Quit India movement Gandhiji and all the leaders of Congress were arrested

2 Question How did Quit India Movement spread out all over IndiaAnswer The news of the leaders lsquo arrest marked the beginning of a widespread movement to remain peacefulThe movement took form of violent outbreak There were widespread cutting of telephone and Telegraph wires damaging railway lines raising barricades in cities and towns and other forms of violent demonstations

Question Name the leaders of Congress

Socialist group played a prominent part Notable among the Jayprakash Narayan Rammonohar Lohia Aruna Asaf Ali

Political science

Topic-Franchise and Representation

Summary

The election commission

The Election Commission of India is an autonomous constitutional authority responsible for administering Union and State election processes in India The body administers elections to the Lok Sabha Rajya Sabha State Legislative Assemblies in India and the offices of the President and Vice President in the country

Functions of election commission-

India is a sovereign socialist secular democratic republic Democracy runs like a golden thread in the social economic and political fabric woven by the Constitution given by lsquoWe the People of Indiarsquo unto ourselves The concept of democracy as visualised by the Constitution pre-supposes the representation of the people in Parliament and State legislatures by the method of election The Supreme Court has held that democracy is one of the inalienable basic features of the Constitution of India and forms part of its basic structure The Constitution of India adopted a Parliamentary form of government Parliament consists of the President of India and the two Houses mdash Rajya Sabha and Lok Sabha India being a Union of states has separate state legislatures for each state State legislatures consist of the Governor and two Houses mdash Legislative Council and Legislative Assembly mdash in seven states namely Andhra Pradesh Telangana Bihar Jammu amp Kashmir Karnataka Maharashtra and Uttar Pradesh and of the Governor and the state Legislative Assembly in the remaining 22 states Apart from the above two out of the seven Union Territories namely National Capital Territory of Delhi and Puducherry also have their Legislative Assemblies

ExecutionShort notes-Election commissionFunctions of election commission

Homework- Learn

Computer

Science

Computer hardware NAND Gate

A NOT-AND operation is known as NAND operation It has n input (n gt= 2) and one output

Logic diagram

Truth Table

NOR Gate

A NOT-OR operation is known as NOR operation It has n input (n gt= 2) and one output

Logic diagram

Truth Table

XOR Gate

XOR or Ex-OR gate is a special type of gate It can be used in the half

adder full adder and subtractor The exclusive-OR gate is abbreviated as EX-OR gate or sometime as X-OR gate It has n input (n gt= 2) and one output

Logic diagram

Truth Table

XNOR Gate

XNOR gate is a special type of gate It can be used in the half adder full adder and subtractor The exclusive-NOR gate is abbreviated as EX-NOR gate or sometime as X-NOR gate It has n input (n gt= 2) and one output

Logic diagram

Truth Table

Physics

Chapter 1 Electric Field ( Electric Dipole) (Summary)

Here we will derive Expression of electric field at broad side

On position of dipole

Execution

Q With the help of a labelled diagram obtain an expression for the electric field intensity E at any point on the equitorial line ( broad-side on position) of an electric dipole

Ans

E1 E1sinθ

E θ P E1 θ

( r2+L2)12 E2 E

r E2 E2sinθ

-q θ L O L +qA B

Let us consider that the point P is situated on the right bisector of the dipole AB at a distance r meter from its midpoint O

Let E1 and E2 be the electric field intensities of the electric field at P due to charge +q and ndashq of the dipole resp The distance of P from each charge is ( r2+L2)12

So E1 = 14 πϵ q

(r 2+L 2) away from +q

E2 = 14 πϵ q

(r 2+L 2) towards ndashq

The magnitudes of E1 and E2 are equal but directions are different Now resolving E1 and E2 into two components parallel and perpendicular to AB we get

The components perpendicular to AB E1sinθ and E2sinθ cancel each other because they are equal and opposite

The components parallel to AB are E1cosθ and E2 cosθ are in same direction and add up

So resultant intensity of electric field at the point P is

E = E1cosθ + E2 cosθ

E = 14 πϵ q

(r 2+L 2) 2 cosθ

Now from fig we have cosθ =BOBP = L (r2+L2)12

So we get E = 14 πϵ 2qL ( r2+L2)32

Now electric dipole moment p= 2qL

So E = 14 πϵ p ( r2+L2)32

HW Find the expression of Electric field as done here but this time take r gtgt 2L

Also find the expression of torque experience by a dipole

(Hint Electric force experienced by charges of dipole in electric field is qE each Let θ be the angle which dipole makes with electric lines of force then perpendicular distance between two charges is 2Lsinθ Then torque = force x perp distance = qE x 2L sinθ So τ=pE sinθ where p =2qL )

STUDY MATERIAL

Class XIISubject Eng Literature (The Tempest ndash William Shakespeare) Topic Act IV Scene 1 Lines 84 to 133 (Iris hellip A contract of true love Be not too late ) Date 27th April 2020 (4th Period)

[Students should read the original play and also the paraphrase given in the school prescribed textbook]Summary Questions amp Answers

o Ceres soon appears and comes to know that she has been summoned to celebrate the contract of true love

o Ceres expresses her unwillingness to meet Venus and Cupid as she has shunned their company

o Ceres and Juno both bestow their blessings upon Ferdinand and Miranda with June gifting honour riches happiness in marriage and Ceres presents plenty of earthrsquos produce

o Iris summons the water-nymphs and reapers to come and celebrate a contract

(1) IRIS Of her society (Line 91-101)

Be not afraid I met her deity

Cutting the clouds towards Pathos and her sonDove-drawn with her Here thought they to have doneSome wanton charm upon this man and maidWhose vows are that no bed-right shall be paidTill Hymens torch be lightedmdashbut in vainMarss hot minion is returned againHer waspish-headed son has broke his arrowsSwears he will shoot no more but play with sparrowsAnd be a boy right out

(i) Where were Venus and Cupid seen flying How were they travelling Why did they want to join the marriage celebration of Ferdinand and Miranda

of true love

Venus and Cupid were seen flying through the air towards Paphos the famous city which is situated on the island of Cyprus They were travelling by air-borne chariot drawn by doves They certainly wanted to come here in order to play some amorous trick upon Ferdinand and Miranda who are under a vow not to gratify their physical desires till the holy ceremony of their marriage has been performed(ii) What have Venus and Cupid done after failing in their plan

After being failure of their plan Venus who is a very passionate deity and who is the mistress of Mars (the god of war) has gone back while here ill-tempered son Cupid has broken his arrows of love in his state of desperation(iii) What has Cupid firmly decided

Cupid is feeling so disappointed that he has firmly decided to shoot no more arrows to arouse love in human hearts but to spend his time playing with sparrows Thus he would now become just a boy and would give up his original function of shooting arrows on human beings to make them fall in love(iv) What vow had Ceres taken How did Ceres feel at the abduction

After the abduction of her daughter Prosperina by Pluto Ceres had taken a vow to always keep away from the disgraceful company of Venus and her blind son Cupid the god of love Ceres felt deeply distressed when Pluto had carried off her daughter and had made her his wife by force(v) Why has Ceres not forgiven Venus and her blind son For what do Ceres want to be sure

As the abduction had been manipulated by Venus the goddess of beauty and love and her blind son Cupid Ceres has never forgiven them for their part in the whole plot Ceres wants to be sure that she would not have to meet Venus and Cupid who had engineered the abduction of her daughter Prosperina

AS THIS lsquoMASQUErsquo SCENE IS VERY IMPORTANT IN THE PLAY THE PARAPHRASE OF THE ENTIRE PORTION OF MASQUE SCENE (Act IV Lines 58 to 143) IS GIVEN BELOW

IRIS Goddess of RainbowCERES Goddess of Agriculture and all the fruits of the earth

(Nature growth prosperity rebirth ndash notions intimately connected to marriage)JUNO The majestic Queen of Heavens and wife of Jupiter (Jupiter is the king of Gods)

VENUS The Goddess of love CUPID Son of Venus PLUTO God of death (In the play referred by Shakespeare as lsquoDisrsquo which is a Roman name for Pluto)

ORIGINAL TEXT PARAPHRASEPROSPEROWellmdash

PROSPERONow come Ariel Let there be too many rather than too few

Now come my Ariel Bring a corollaryRather than want a spirit Appear and pertly[to Ferdinand and Miranda]No tongue all eyes Be silent

spirits in attendance Appear briskly

[to Ferdinand and Miranda]Look with your eyes but do not say a word

[Soft music] [Soft music][Enter Iris] [Enter Iris]

IRISCeres most bounteous lady thy rich leasOf wheat rye barley vetches oats and peasThy turfy mountains where live nibbling sheepAnd flat meads thatched with stover them to keepThy banks with pioned and twilled brimsWhich spongy April at thy hest betrimsTo make cold nymphs chaste crowns and thybroom-grovesWhose shadow the dismissegraved bachelor lovesBeing lass-lorn thy pole clipped vineyardAnd thy sea-marge sterile and rocky-hardWhere thou thyself dost airmdashthe Queen othrsquoSkyWhose watery arch and messenger am IBids thee leave these and with her sovereign grace[Juno appears] Here on this grass-plot in this very placeTo come and sport Her peacocks fly amainApproach rich Ceres her to entertain

IRISCeres most generous lady you are the cause of rich fields or fertile land where wheat rye barley beans oats and peas grow the grassy mountains where the sheep graze and the flat meadows covered with coarse hay to be used as fodder for cattleYour banks are covered with marsh-marigolds and reeds and the rainy April under your orders brings forth to make for the maids who are not in love beautiful crowns your woods where the broom flourishes and where the bachelor who has been dismissed by the maid he loved lies down being forsaken your vineyard in which the poles are embraced by the vines and the margin of the sea which is barren and rocky where you roam about to enjoy the fresh air ndash the queen of the sky (Juno) whose messenger I am besides being represented as the rainbow bids you leave all these and with her majesty here on this grassy plot in this very place come and sport her peacocks carry her fast in her chariot through the air and are making their way here approach rich Ceres to welcome her

[Enter Ariel as Ceres] [Enter Ariel as Ceres]

CERESHail many-coloured messenger that neerDost disobey the wife of JupiterWho with thy saffron wings upon my flowersDiffusest honey-drops refreshing showersAnd with each end of thy blue bow dost crownMy bosky acres and my unshrubbed downRich scarf to my proud earth Why hath thy queenSummoned me hither to this short-grassed green

CERESWelcome rainbow that never dared disobey Juno the wife of Jupiter who with your orange coloured rays spread honey-drops refreshing showers And with each end of thy blue bow drown my bushy acres and my hilly country which is free from shrubs you thus forming a rich scarf Why has your queen called me here to this place covered with short grass

IRISA contract of true love to celebrateAnd some donation freely to estateOn the blest lovers

IRISI have called you to celebrate a contract of true love and bestow some liberal gift upon the blessed lovers

ORIGINAL TEXT PARAPHRASECERESTell me heavenly bowIf Venus or her son as thou dost knowDo now attend the queen Since they did plotThe means that dusky Dis my daughter gotHer and her blind boys scandaled companyI have forsworn

CERESTell me heavenly bow if Venus the Goddess of love or Cupid her son and pedlar of passion at this time attend the heavenly queen Juno because you are sure to know Since the day they conspired against me and dark Pluto took away my daughter here and Cupidrsquos disgraceful company I have left off

IRISOf her societyBe not afraid I met her deityCutting the clouds towards Pathos and her sonDove-drawn with her Here thought they to have doneSome wanton charm upon this man and miad

IRISBe not afraid of her company I met her deity moving on the clouds towards Paphos the sacred home of Venus on the island of Cyprus along with her son on her chariot drawn by doves Here they contemplated to exercise a charm upon this man and maid producing

Whose vows are that no bed-right shall be paidTill Hymens torch be lightedmdashbut in vainMarss hot minion is returned againHer waspish-headed son has broke his arrowsSwears he will shoot no more but play with sparrowsAnd be a boy right out

wantonness before the actual marriage ceremony but did not succeed Venus has returned her irritable son has broken his arrows and swears that he will give up his practice of trying to inspire love but play with sparrows and be a boy again

[Music is heard] [Music is heard]

CERESHighst queen of stateGreat Juno comes I know her by her gait

CERESHighest queen of state Great Juno there she comes I know here by her gait

[Enter Juno] [Enter Juno]

JUNOHow does my bounteous sister Go with meTo bless this twain that they may Prosperous beAnd honoured in their issue

JUNOHow are you doing my generous sister Come with me to bless this couple so that they may be prosperous and fortunate in their children

[They sing] [They sing]

JUNOHonour riches marriage-blessingLong continuance and increasingHourly joys be still upon youJuno sings her blessings upon you

JUNOMay honour riches happiness in marriage long continuance and increase of those boons ever rest upon you as hourly joys Juno showers down upon you her blessings in song

CERESEarths increase foison plentyBarns and garners never emptyVines and clustring bunches growingPlants and goodly burden bowingSpring come to you at the farthestIn the very end of harvestScarcity and want shall shun youCeresrsquo blessing so is on you

CERESMay you have the plenty of earthrsquos produce Your barns and granaries may never be empty Your vines may grow with clustering bunches Your fruit trees may be heavily laden with their fruit May there be continuous spring and harvest May scantiness and want leave you forever Such is the blessing of Ceres upon you

FERDINANDThis is a most majestic vision andHarmoniously charmingly May I be boldTo think these spirits

FERDINANDThis is a great vision and magically melodious Should I suppose the characters (taking part in the masque) are spirits

PROSPEROSpirits which by mine artI have from their confines calld to enactMy present fancies

PROSPEROYes they are spirits whom I have summoned from the regions to which they are confined to carry into effect my fanciful designs

ORIGINAL TEXT PARAPHRASEFERDINANDLet me live here everSo rare a wondered father and a wifeMakes this place paradise

FERDINANDI should like to live here forever Such a wise and wonderful father makes this place a paradise

[Juno and Ceres whisper and send Iris on employment] [Juno and Ceres whisper and send Iris on employment]

PROSPEROSweet now silence

PROSPEROMy dear Ferdinand speak no more Juno and Ceres are

Juno and Ceres whisper seriouslyTheres something else to do Hush and be muteOr else our spell is marred

whispering with a solemn look There is something else coming Silence Or else our magic will be spoilt

IRISYour nymphs called naiads of the wandering brooksWith your sedged crowns and over-harmless looksLeave your crisp channels and on this green landAnswer your summons Juno does commandCome temperate nymphs and help to celebrateA contract of true love Be not too late

IRISYou nymphs called Naiads denizens (M inhabitants) of the running stream with your chaplets of sedge and ever-helpful looks leave your wrinkled channels and on the green land answer the summons sent to you Juno has ordered some chaste nymphs and help to celebrate a noble and true marriage Donrsquot delay

[Enter certain nymphs] [Enter certain nymphs]You sunburnt sicklemen of August wearyCome hither from the furrow and be merryMake holiday your rye-straw hats put onAnd these fresh nymphs encounter every oneIn country footing

You sunburnt harvesters weary from the effects of the heat in August come here from the furrowed land and rejoice Make holiday with your rye-straw hats upon you and meet these fresh nymphs and join in country dancing

[Enter certain reapers properly habited They join with the nymphs in a graceful dance towards the end whereof Prospero starts suddenly and speaks]

[Enter certain reapers properly habited They join with the nymphs in a graceful dance towards the end whereof Prospero starts suddenly and speaks]

PROSPERO[aside] I had forgot that foul conspiracyOf the beast Caliban and his confederatesAgainst my life The minute of their plotIs almost come [to the spirits]Well done Avoidno more

PROSPERO(Aside)I had forgotten the wicked conspiracy of the beast Caliban and his accomplices against my life the time of their plot has almost arrived ndash (To the Spirits) well done depart no more of this

[To a strange hollow and confused noise the spirits heavily vanish]

[The spirits depart]

ORIGINAL TEXT PARAPHRASEFERDINANDLet me live here everSo rare a wondered father and a wifeMakes this place paradise

FERDINANDI should like to live here forever Such a wise and wonderful father makes this place a paradise

[Juno and Ceres whisper and send Iris on employment] [Juno and Ceres whisper and send Iris on employment]

PROSPEROSweet now silenceJuno and Ceres whisper seriouslyTheres something else to do Hush and be muteOr else our spell is marred

PROSPEROMy dear Ferdinand speak no more Juno and Ceres are whispering with a solemn look There is something else coming Silence Or else our magic will be spoilt

IRISYour nymphs called naiads of the wandering brooksWith your sedged crowns and over-harmless looksLeave your crisp channels and on this green landAnswer your summons Juno does commandCome temperate nymphs and help to celebrateA contract of true love Be not too late

IRISYou nymphs called Naiads denizens (M inhabitants) of the running stream with your chaplets of sedge and ever-helpful looks leave your wrinkled channels and on the green land answer the summons sent to you Juno has ordered some chaste nymphs and help to celebrate a noble and true marriage Donrsquot delay

[Enter certain nymphs] [Enter certain nymphs]You sunburnt sicklemen of August wearyCome hither from the furrow and be merryMake holiday your rye-straw hats put onAnd these fresh nymphs encounter every oneIn country footing

You sunburnt harvesters weary from the effects of the heat in August come here from the furrowed land and rejoice Make holiday with your rye-straw hats upon you and meet these fresh nymphs and join in country dancing

[Enter certain reapers properly habited They join with the nymphs in a graceful dance towards the end whereof Prospero starts suddenly and speaks]

[Enter certain reapers properly habited They join with the nymphs in a graceful dance towards the end whereof Prospero starts suddenly and speaks]

PROSPERO[aside] I had forgot that foul conspiracyOf the beast Caliban and his confederatesAgainst my life The minute of their plotIs almost come [to the spirits]Well done Avoidno more

PROSPERO(Aside)I had forgotten the wicked conspiracy of the beast Caliban and his accomplices against my life the time of their plot has almost arrived ndash (To the Spirits) well done depart no more of this

[To a strange hollow and confused noise the spirits heavily vanish]

[The spirits depart]

Ac-12 27420 topic Revaluation of Assets and Liabilities

REVALUATION OF ASSETS AND LIABILITIES

On admission of a new partner the firm stands reconstituted and consequently the assets are revalued and liabilities are reassessed It is necessary to show the true position of the firm at the time of admission of a new partner If the values of the assets are raised gain will increase the capital of the existing partners Similarly any decrease in the value of assets ie loss will decrease the capital of the existing partners For this purpose alsquoRevaluation Accountrsquo is prepared This account is credited with all increases in the value of assets and decrease in the value of liabilities It is debited with decrease on account of value of assets and increase in the value of liabilities The balance of this account shows a gain or loss on revaluation which is transferred to the existing partnerrsquos capital account in existing profit sharing ratioAccounting for Revaluation of Assets and Liabilities when there is a Changein the Profit Sharing Ratio of Existing PartnersAssets and liabilities of a firm must also be revalued at the time of change in profit sharing ratio of existing partners The reason is that the realisable or actual value of assets and liabilities may be different from those shown in the Balance Sheet It is possible that with the passage of time some of the assets might have appreciated in value while the value of certain other assets might have decreased and no record has been made of such changes in the books of accounts Similarly there may be some unrecorded assets amp libilities that may have to be accounted for Revaluation of assets and reassessments of liabilities becomes necessary because the change in the

value of assets and liabilities belongs to the period to change in profit sharing ratio and hence must be shared by the partners in their old profit sharing ratio Revaluation of assets and reassessment of liabilities may be given effect to in two different ways (a) When revised values are to be recorded in the books and(b) When revised values are not to be recorded in the books

When revised values are to be recorded in the booksIn such a case revaluation of assets and reassessment of liabilities is done with the help of a new account called lsquoRevaluation Accountrsquo Sometimes this account is also called as lsquoProfit amp Loss Adjustment Acrsquo If there is a loss due to revaluation revaluation account is debited and if the revaluation results in a profit the revaluation account is credited The following journal entries made for this purpose are

(i) For increase in the value of assetsAsset Ac Dr (individually)To Revaluation Ac(ii) For decrease in the value of AssetRevaluation Ac Dr (individually)To Asset Ac[Decrease in the value of assets](iii) For increase in the value of LiabilitiesRevaluation Ac Dr (individually)To Liabilities Ac[Increase in the value of Liabilities](iv) For decrease in the value of LiabilitiesLiabilities Ac DrTo Revaluation Ac[Decrease in the value of Liabilities](v) For unrecorded AssetsAsset Ac [unrecorded] DrTo Revaluation Ac[Unrecorded asset recorded at actual value](vi) For unrecorded Liability Revaluation Ac DrTo Liability Ac [unrecorded][Unrecorded Liability recorded at actual value](vii) For transfer of gain on revaluationRevaluation Ac DrTo Existing Partnerrsquos CapitalCurrent Ac[Profit on revaluation transferred to capital account in existing ratio](viii) For transfer of loss on revaluationExisting Partnerrsquos CapitalCurrent Ac DrTo Revaluation Ac[Loss on revaluation transferred to capital account in existing ratio](a) When revaluation account shows gain Revaluation Ac DrTo Partnerrsquos Capital Ac (Old Profit Sharing Ratio)(Profit on revaluation credited to Partnerrsquos Capital Ac)(b) Above entry is reversed when revaluation account shows loss Partners Capital Acs (Old Profit Sharing Ratio) DrTo Revaluation Ac(Loss on revaluation debited to Partnerrsquos Capital Acs)

Proforma of Revaluation Account is given as under

Revaluation Account

Dr Cr Particulars ` Amount Particulars ` Amount To Decrease in value of assets By Increase in value of assets To Increase in value of liabilities By Decrease in value of liabilities To Unrecorded liabilities By Unrecorded assets To Gain on Revaluation (Transferred) By Loss on Revalution (Transferred)

ECO ndash12 2742020Topic- ELASTICITY OF DEMAND

CHAPTER - ELASTICITY OF DEMANDMEANINGDemand for a commodity is affected by many factors such as its price price of related goods income of its buyer tastes and preferences etc Elasticity means degree of response Elasticity of demand means degree of responsiveness of demand Demand for a commodity responds to change in price price of related goods income etc So we have three dimensions of elasticity of demandDIMENSION OF ELASTICITY OF DEMAND TYPES OF ELASTICITY OF DEMAND

Price elasticity of demand Income elasticity of demand Cross Elasticity of demand

Price elasticity of demand Price elasticity of demand means degree of responsiveness of demand for a commodity to the change in its price For example if demand for a commodity rises by 10 due to 5 fall in its price Price elasticity of demand (ep)=Percentage change in quantity demanded Percentage change in price of the commodity = 10 ( -)5 = ( - )2Note that ep will always be negative due to inverse relationship of price and quantity demanded

(ii) Income elasticity of demand Income elasticity of demand refers to the degree of responsiveness of demand for a commodity to the change in income of its buyer Suppose income of buyer rises by 10 and his demand for a commodity rises by 20 then Income elasticity of demand (ey)= change in quantity demanded change in price of the commodity =20 10 = 2

Cross Elasticity of demandCross elasticity of demand means the degree of responsiveness of demand for a commodity to the change in price of its related goods (substitute goods or complementary goods) Suppose demand for a commodity rises by 10 due to 5 rise in price of its substitute good then Cross elasticity of demand (ec) = change in quantity demanded change in price of related good = 10 2 = 5 (Tastes and preferences cannot be expressed numerically So elasticity ofdemand cannot be numerically expressed)

  • Chapter 1 Force (Summary)
  • Distinguish between external sovereignty and internal sovereignty
    • NAND Gate
      • Logic diagram
      • Truth Table
        • NOR Gate
          • Logic diagram
          • Truth Table
            • XOR Gate
              • Logic diagram
              • Truth Table
                • XNOR Gate
                  • Logic diagram
                  • Truth Table
                      • Physics
                      • Chapter 1 Electric Field ( Electric Dipole) (Summary)
Page 25:  · Web viewWe all know that Nouns are divided into two parts: common noun and proper noun.Apart from common and proper noun, we will also study about collective noun and compound

position In return Antonio promised that he shall give an annual tribute and also swore his allegiance to Alonso Also he agreed to hold the Dukedom of Milan as a subordinate to the state of Naples(v) How were Prospero and Miranda carried away from the city of Milan and what was the state of small Miranda at that time

In pursuance of the agreement settled between Antonio and Alonso an army of treacherous men was assembled One midnight when the occasion suited the will of destiny Antonio opened the gates of the city of Milan and in the death like silence of midnight Antoniorsquos agents who had been directed to execute his purpose carried Prospero and small Miranda away from the city in all haste They were then forced into a ship and carried some distance out to sea where they put them on a mere hulk of a boat without any rigging or ship-gear and abandoned them leaving them at the mercy of the roaring sea Miranda was a very small child of three years age and she was crying at that time

CLASS -XIDATE-270420Subject Topic Summary Execution

EVS Chapter 1 ndash Mode of Existence

Impact of mode of existence on resources

Q) Why resources are under pressure

Ans - Increase in the sophistication

of technology enabling natural resources to be extracted quickly and efficiently Eg in the past it could take long hours just to cut down one tree only using saws Due to increased technology rates of deforestation have greatly increased

The number of humans is increasing Cultures of consumerism Materialistic views

lead to the mining of gold and diamonds to produce jewelry unnecessary commodities for human life or advancement Consumerism also leads to extraction of resources for the

production of commodities necessary for human life but in amounts excessive of what is needed because people consume more than is necessary or waste what they have

Lack of awareness among the population is striking People are not aware of ways to reduce depletion and exploitation of materials

Accounts Cash Book Today we are going to start a new topic -Cash Book

The key terms used in this chapter are

bullCash book

bullSimple cash book

bullDouble column cash book bullTriple column cash book

bullPetty cash book

bullCash discount

bullContra entry

Here I will share you the meaning of each key terms

bullCash book Cash Book is a special purpose subsidiary book or journal in which cash received and cash payments are recorded

bullSimple cash book

It is a cash book in which only cash transactions are recorded It has only one column on each side

bullTriple column cash book

It is cash book which has three columns one column for each cash and Bankdiscount on each side of the cash book In this book both cash and Bank transactions are recorded together with discount allowed and received

bullPetty cash book

It is a cash book maintained for recording petty expenses

bullCash discount

Cash discount is the amount of discount received or allowed on cash payments and cash receipts Discount received is an income for the business while discount allowed isan expense

bullContra entry

It means transactions involving both cash and Bank Such transactions though recorded in the cash book are not posted into ledger The letter lsquoC is written in Ledger folio for contra entry

Business Studies

ENTREPRENEURSHIP

Now we shall discuss the second chapter

lsquoENTREPRENEURSHIPrsquo

Today before starting the chapter let us recall what

Questions

1What are the main characteristics of Intrapreneurship

Answer

The main characteristics of Intrapreneurship are

Corporate framework-it occurs within the framework of the same company

Semi-Autonomous-Intrapreneurship

we have read last day

Let s today start the class by recalling the last topic taught

Intrapreneurship is the process of discovering and exploring business opportunities within an existing company It involves launching new business ventures within the framework of a present corporation Intrapreneurship is also known as corporate entrepreneurship or corporate venturing

Now let us start with the characteristics of Intrapreneurship

The main characteristics of Intrapreneurship are

Corporate framework

Semi-Autonomous Lack of ownership Senior position Low risk taking Not own boss

Now let us discuss the meaning of enterprise

Enterprise means an undertaking or adventure that requires some innovation and investment and thus involves riskEnterprise always entails decision making coordination and risk bearing

involves crating amd nurturing a semi-autonomous business unit which may be a subsidiary a strategic business unit or a division

Lack of ownership-the intrapreneur is not the owner of the unitb he creates and nurtures

Senior position-he occupies a senior managerial position in the company

Low risk taking-An intrapreneur does not bear the full risk of failure

Not own boss-An intrapreneur is not his own bosss in legal termsHe enjoys the freedom and gets the required resources and support

2 How is Entrepreneur is different from Intrapreneur

The functions involved in both the entrepreneurship and intrapreneurship are by and large similar however there are several differences between the two

Point of distinction

Entrepreneur

Intrapreneur

status An independent business person

A senior executive within a company

Ownership Owner of

the business

An employeesometimes a share in ownership

Financing Responsible for raising finance for the business

Not responsible for raising the finance

Risk bearing

Bears the risk of the business

Does not bears the risk of the business

Reward Profit which is uncertain and irregularcan be loss

Fixed salary and fringe benefits

Need for security low high

3 What do you understand by enterprise

Answer Enterprise means an undertaking or adventure that requires some innovation and investment and thus involves riskEnterprise always entails decision making coordination and risk bearing

COMMERCE NATURE AND OBJECTIVES OF

BUSINESS

Today let us recall the last other two objectives of business by the chart given in the previous class

Firstly we would discuss Human Objectives

Business is run by people and for people Labour is a valuable business element

Human objectives of business are concerned with the well -being of labour

The human objectives are as follows

Labour welfare Developing human

resources Participative

management Labour

management cooperation

Questions

1 Explain the human objectives of a business enterprise

Answer

Business is run by people and for people Labour is a valuable business element

Human objectives of business are concerned with the well -being of labour

The human objectives are as follows

Labour welfare-Business must recognize the dignity of labour and human factors should be given the recognition

Developing human resources-Employees must be provided the opportunities for developing new skills and attitudes

Participative management-Employees should be allowed to take part in decision making process of business

Labour management cooperation-Business should strive for creating and maintaining cordial employer employee relations so as to ensure peace and progress in industry

Now let us discuss the national objectives of business

Optimum utilization of resources

National self- reliance Development of small

scale industries Development of

backward areas Control over pollution

2Explain the national objectives of a business enterprise

Answer

It is the duty of business to utilize the resources of the country properly the national objectives of business

Optimum utilization of resources ndashBusiness should use the nationrsquos resources in the best possible manner

National self- reliance-It is the duty of the business to help the government in increasing experts and in reducing dependence on imports

Development of small scale industries-Big business firms are expected to encourage growth of small scale industries which are necessary for generating employment

Development of backward areas-Business is expected to give preference to the industrialization of backward regions of the country

ECONOMICS

BASIC ECONOMIC CONCEPTS

SUB

TOPIC

Value

Wealth

Welfare

Today we shall start with a new topic of the same chapter ie lsquoValuersquo

Value of a commodity is defined as the valuation placed by a household on the consumption of this commodity

lsquoValuersquo has two different meanings and these are

a Value -in -use It refers to consumption value of a commodity It expresses the utility derived from the consumption of a particular commodity A necessity like water has a very high value ndashin ndashuse or

Question

1What is value

Answer

Value of a commodity is defined as the valuation placed by a household on the consumption of this commodity

2What is value-in use

Answer It refers to consumption value of a commodity It expresses the utility derived from the consumption of a particular commodity A necessity like water has a very high value ndashin ndashuse or consumption value

3What is value ndashin- exchange

Answer It relates to market value of a commodity

It is the rate at which a particular good or service can be exchanged for money

For example in barter system if a person is prepared to exchange 3 metres of cloth with 1 pair

consumption value

b Value ndashin-exchange It relates to market value of a commodity

It is the rate at which a particular good or service can be exchanged for moneyFor example in barter system if a person is prepared to exchange 3 metres of cloth with 1 pair of shoes then the value in exchange of 3 metres of cloth is 1 pair of shoesValue in exchange is the power of purchasing other goods In modern monetised economies the exchange value of goods are expressed in terms of money as prices

Now let us discuss the term lsquoWealthrsquo

Wealth refers to the stock of all those assets which are a source of income

Wealth is a stock concept

Wealth must possess the following features

a Utility It must possess utility or give some

of shoes then the value in exchange of 3 metres of cloth is 1 pair of shoes

Value in exchange is the power of purchasing other goods In modern monetised economies the exchange value of goods are expressed in terms of money as prices

4 What is wealth

Answer it refers to the stock of assets or goods which are a source of income and have personal or national ownership

5 What are the features of wealth

Answer The features of wealth are as follows

Wealth must possess the following features

a Utility It must possess utility or give some satisfaction

b Scarcity It must be limited in quantityc Transferability it should be transferable its

ownership can be transferred from one person to another person

d Exchange value It must possess exchange value

6 What is welfare

Answer

Welfare is defined as satisfaction and happiness a sense of well- being among the people

satisfactionb Scarcity It must be

limited in quantityc Transferability It

should be transferable its ownership can be transferred from one person to another person

d Exchange value It must possess exchange value

Now let us discuss the term lsquoWelfarersquo

Welfare is defined as satisfaction and happiness a sense of well- being among the people

Welfare is affected by factors like

a Consumption of goods and services

b Environment

c Family relations

d Degree of freedom

e Law and order situation

Mathematics Trigonometric equation

To find the general solution of the equation sinθ=0

When sin θ =0

Then θ= 0 π2π 3π-π -2π -3

i e when θ = 0 or an integral multiple of π

i e when θ= nπ where n is any integer

Therefore the general solution of the equation sin

Example1 Find the general values of θ which satisfy the equation sin2 θ =34

Solution sin2 θ= 34

Or sin θ = +34 or -34

Or sin θ = sin π3 or sin (-π3)

Therefore

θ = [nπ + (-1) n (π3)] or[ nπ+ (-1) n (-π3)]

= nπ +π3 or nπ-π3 where n= any integer

Example 2Find the values of θ which satisfy tan2 θ

θ=0 is θ= nπ where n is any integer

To find the general solution of the equation cos θ=0

When cos θ=0

Then θ=π2 3π2 5π2 -π2 -3π2 -5π3

i e when θ is an odd multiple of π2

i e when θ=(2n+1) π2 where n= any integer

Therefore the general solution of the equation cos θ =0 is θ= (2n+1) π2 where n= any integer

To find the general solution of the equation tan θ = 0

Clearly tan θ =0 implies sin θcos θ =0

Therefore θ = nπ

i e the general solution of the equation tan θ=0 is θ =nπ where n = any integer

To find the general solution of the equation cot θ =0

Clearly cot θ =0 implies (cos θsin θ) = 0

i e cos θ =0

Therefore θ = (2n+1) π2

Therefore the general solution of the equation cot θ =0 is θ = (2n+1) π2

Where n= any integer

To find the general solution of the equation sin θ= k (-

=13 -πleθleπ

Solution tan2 θ =13

Or tan θ = plusmn1radic(3) =tan(plusmnπ6)

θ=nπ plusmn π6 where n =any integer

If n=0 then θ=plusmnπ6

If n=1 then θ= π plusmn π6

If n=-1 then θ= -π plusmn π6

Therefore the required solution in -π le θ le π are θ= π6 5π6 -π6 -5π6

Exercise Find general solution of sin 2θ=cos θ [Hints Use sin 2θ= 2sin θcosθ and then take cosθ

common]

1lek le1)

Determine an angle alpha such that sin =k and -π2le αle π2

Then we have

Sin θ = k = sin α

Or sin θ - sin α =0

Or 2 cos [(θ+α) 2] sin [(θ-α) 2] =0

Therefore either cos [(θ +α) 2] =0 (1)

Or sin [(θ-α) 2] =0 (2)

Now from (1) we get (θ+α) 2= (2m+1) π2)

Or θ = (2m+1) π-α (3)

And from (2) we get (θ-α) 2 =mπ

Or θ= 2mπ+α(4)

Where m = any integer

Clearly the solution (3) amp (4) may be combined in the following form

θ= nπ+(-1) n α where n= any integer

Therefore the general solution of sin θ = sin α is θ = nπ +(-1) n α where n is any integer and -π2 le α le π2

Biology Chapter - 04Kingdom Monera

Today we will discuss about bacterial reproduction and its usefulness

Fig Binary Fission

Fig Conjugation Fig Transformation

Bacterial reproduction is mainly asexual but sexual reproduction

also takes place

Asexual reproduction takes place by i) Binary fission - from one bacteriato

two bacteria are produced in every 20 to 30mins

ii) Buddingiii) Endospore formation - during

unfourable condition

Sexual reproduction by three ways

1) Conjugation - Transfer of genetic material between cells that are in physical contact with one another

2) Transduction - Transfer of genetic materialfrom one cell to another by a bacteriophage

3) Transformation - Transfer of cell-freeor naked DNArsquo from one cell to another

Bacteria causes different diseases inplants animals and human and

it causes food spoilage and waterpollution but it also have some useful

activities

i) Bacteria are helpful in sewage water treatment

ii) It is used in antibiotic (medicine) production

iii) Anaerobic bacteria help in biogas(energy) production

iv) Many household products like yoghurt cheese are manufactured by use of bacteria

v) Rhizobium by symbiotic relationship with leguminous plant increase soil fertility

vi) Besides these bacteria is helpful in genetic engineering degradation of petroleum hydrocarbonand in dairy

industry

Physics Motion in plane Here we will introduce Projectile Motion

Execution

Projectile

Y

usinθ u h

θX

ucosθ

Suppose a body is projected with an angle θ So initial velocity u can be resolved into two components

Horizontal component - ucosθ ( for range)

Vertical component - usinθ ( for height)

usinθ changes during motion and becomes zero at maximum height position but ucosθ remain unchanged

The maximum height of projectile is h

NB If initial is upward then g = -ve and if it is downward then g = +ve Height is +ve if direction of motion does not change ( for ex a body thrown upwards but goes down ultimately then height h = -ve)

The angle of projectile θ is the angle made with horizontal

HISTORY ndash GROWTH OF NATIONALISM

SUB TOPIC- REVOLUTIONARY NATIONALISM Bengal formation of Anushilan Samity and Jugantar Group

The intensification of the Swadeshi movement and Government policy of terror and repression led to outbreak of violence Bombs were manufactured and attempts on the lives of unpopular Government officials became frequent In the gymnasium of Scottish Church College which was known as General Assemblies Institution a secret society was formed known as Anushilan Samity

Aurobindo Ghosh send from Baroda his emissary Jatindranath Banerjee to mobilize the Bengal revolutionaries

Hemchandra Qanungo and Satyen Bose published Journal Jugantar

The Jugantar group planned to assassinate oppressive magistrate Kingsford by Khudiram Bose and Prafulla Chaki in 1908 Prafulla Chaki committed suicide to avoid arrest Khudiram was tried and hanged

Afew days later the police found a bomb factory in Maniktala and arrested a large number of revolutionaries The trial of revolutionaries became famous as the Alipore Bomb Case

In the course of the trial the approver the public prosecuter and a police officer were assassinated

1 Question Name two journals which preached the cult of violence

Answer a) Yugantar edited by Bhupendranath DuttaB) Bandemataram edited by Aurobindo Ghosh2 Question Why was Khudiram arrested and hangedAnswer An attempt was made to assassinate a hated vindictive majistrate named Kingsford by Khudiram Bose and Prafulla Chaki Their attempt failed and the bomb they threw killed two English ladies Khudiram was arrested and put to trial and then hanged3Question Who was Aurubindo GhoshAnswer Aurobindo Ghosh a nationalist revolutionary who was charged for his involvement in the Alipore Bombing Case He was accused of it along with his brother Barindra nath Ghosh But Aurobindo was acquitted because of the brilliant pleading of his counsel Chittaranjan Das Then he became a spiritual reformer introducing his visions on human progress and spiritual evolution4 Qustion Who was KingsfordAnswer Kingsford was an unpopular British chief Magistrate who was the target of the bomb thrown at Muzaffarpur by Khudiram and Prafulla Chaki

Most of the accused were convicted and sentenced to

long term of imprisonmentBut

Aurobindo Ghosh was acquitted mainly owing to the brilliant pleading of his counsel Chittaranjan Das

Political science Topic-Sovereignty

Summary Sovereignty is the full right and power of a governing body over itself without any interference from outside sources or bodies In political theory sovereignty is a substantive term designating supreme legitimate authority over some polity In international law sovereignty is the exercise of power by a state

Internal Sovereignty

Internal sovereignty means supreme authority within ones territory while external sovereignty relates to the recognition on the part of all states that each possesses this power in equal measure

External sovereignty

external sovereignty relates to the recognition on the part of all states that each possesses this power in equal measure

Distinguish between

Execution

Answer the following questions

Short notes-

Sovereignty

Internal Sovereignty

External sovereignty

Homework- learn

external sovereignty and internal sovereigntySovereignty is the principle

of supreme and

unquestionable authority

reflected in the claim by the

state to be the sole author of

laws within its territory

Definition of external vs internal sovereigntyInternal sovereignty refers to

the relationship between a

sovereign power and its

subjects ndash it refers to the

location of the supreme

authority within the state In

the UK for example internal

sovereignty (supposedly)

resides within Parliament

reflected in the

constitutional principle of

parliamentary

sovereigntyBy contrast

external sovereignty refers

to the capacity of the state

to act independently and

autonomously on the world

stage This is what is

sometimes called lsquostate

sovereigntyrsquo or lsquonational

sovereigntyrsquo and implies

that states are legally equal

and that the territorial

integrity and political

independence of a state is

inviolable

Class ndash XII

Date - 2742020 STUDY MATERIALSubject Topic Summary Execution Business Studies

Job Analysis amp Manpower Planning

At first let us recall the chapter what we have discussed till nowbullJob analysisbullJob specification bullJob description bullJob enlargement bullJob enrichment

Today we will do some questions answers from the chapter

Questions 1ldquoJob analysis job description and job specification are interrelatedrdquo Comment Answer) Job analysis is a systematic and detailed examination of a job to collect all the relevant information about it The contents off the job are summarised in the job description The qualification needed for the job are summarised in job specificationThus there is close interrelationship between job analysis job description and job specification

Question 2ldquoJob enlargement is a horizontal extension of a job whereas job enrichment is a vertical extension of a jobrdquo ElucidateAnswer) Job enlargement involves adding one or more task to a job coma where as job enrichment involves adding more autonomy and responsibility to a job Job enlargement is therefore horizontal extension of a job coma whereas job enrichment is a vertical extension of a job

Question 3 )

What is manpower estimation Explain its quantitative and qualitative aspectsAnswer) Manpower estimation is the process by which management determines how an organisation should move from itrsquos current manpower positionto its desired manpower position There are two dimensions of Manpower estimation- quantitative and qualitative

Quantitative aspectThis aspect of Manpower estimation involves estimating the number of employees required in a future time period Workload analysis and workforce analysis are done to estimate the quantity of required manpower

Qualitative aspectThe estimate of the knowledge skills experience etc of required manpower is the qualitative aspect of Manpower estimation The quality of Manpower can be judged on the basis of job analysisand job specification

COMMERCE

CAPITAL-FIXED AND WORKING

Today let us start the class by discussing the sources of finance for different types of business firms

The term lsquocapitalrsquo refers to the investment made in the enterprise for the purpose of earning profits

Requirements of capital and sources of capital for different types of business firms are

1 Capital for sole proprietorship businessA sole proprietor operates at a small scale and thereforerequires a limited amount of capital

2 Capital for partnership firmCapital requirements as well as capital base of a partnership is bigger than that of a sole trader businessThe owned capital is contributed by the partners in an agreed ratio

3 Capital for joint stock companyA joint stock company generally requires large amount of capitalA public company can raise huge capital through issue of shares In addition to share capital it can utilize retained profits

Now let u discuss the meaning of Finance PlanningFinance planning is the process of estimation the financial requirements of an organization specifying the sources of firms and ensuring that enough funds are available at the right time

1 What do you mean by Finance PlanningAnswer Finance planning is the process of estimation the financial requirements of an organization specifying the sources of firms and ensuring that enough funds are available at the right time

2Discuss the role of financial planning of an enterpriseThe role of financial planning are as followsa A sound financial plan helps a business enterprise to avaid the problems of shortage and surplus of fundsbFinancial planning serves as a guide in developing a sound capital structure so as to maximize returns to shareholders c It helps in effective utilization of fundsd It provides policies and procedures for coordinating different functional areas or departments of businesse It enables the management to exercise effective control over the financial activities of an enterprisef It helps the company to prepare for facing business shocks and surprises in future

Mathematics

Continuity and differentiability

Recall Definition of ContinuityLet f(x) be a single valued function of x and x=a be a point in the domain of definition of the function The function is said to be continuous at x=a ifi) f(c) is defined ie f(x) has a definite finite value at x=cii) lim xrarra f(x) exists andiii) lim xrarra f(x) =f(a) In other words f(x) is said to be continuous at x=a if lim xrarra+ f(x)= lim xrarra- f(x) = f(a) Or f(a+0) =f(a-0) =f(a) Or lim hrarr0 f(a+h)= f(a) Algebra of continuous functionsNow we will study some algebra of continuous functions Theorem 1 Suppose f and g be two real functions continuous at a real number c Then(1) f + g is continuous at x = c(2) f ndash g is continuous at x = c(3) f g is continuous at x = c(4) (fg) is continuous at x = c (provided g (c) ne 0)

Example 1 Prove that every rational function is continuousSolution Recall that every rational function f is given byf(x)=[p(x) q(x) ] q(x)ne0where p and q are polynomial functions The domain of f is all real numbers except points at which q is zero Since polynomial functions are continuous f is continuous by (4) of Theorem 1Example 2Discuss the continuity of sine functionSolution To see this we use the following factslim xrarr0 sin x =0Now observe that f (x) = sin x is defined for every real number Let c be a real number Put x = c + h If x rarr c we know that h rarr 0 Therefore lim xrarrc f(x)

= lim xrarrc sin x= lim hrarr0 sin(c+h) =lim hrarr0 [sin c cos h + cos c sin h ]=lim hrarr0 (sin c cos h) + lim hrarr0 (cos c sin h) = sin c +0=sin c = f(c) Thus lim xrarrc f(x) = f(c) and hence f is a constant function Exercise Prove that the function f(x) = x2 +2x is continuous for every real value of x [Hints show that lim xrarra+ f(x) = lim xrarra- f(x) = f(a) ]

Biology Reproduction in Flowering plants We will discuss about megasporoangium

megasporagenesis and female gametophyte

Q4 Describe the structure of megasporangium

Ovule is attached to the placenta by astalk called funicle

Each ovule has one two or three protectivecoverings called integuments

At the tip of integuments a small openingcalled micropyle is organised

Opposite to the micropylar end is the chalaza

Within the integuments a mass of cellsnucellusand inside it embryo sac orfemale gametophyte is present

Q5 Describe a mature embryosacamp its formation

In most of the flowering plants only oneof the 4 megaspores formed as a result ofmegasporogenesis that is functional while theother three degenerate

The

functional megaspore develops into thefemale gametophyte

Formation The nucleus

of the functional megasporedivides mitotically to form two nuclei first andthen two more sequential mitotic nucleardivisions result in the formation of four ampthen eight nucleate stages of embryo sac

Six of the eight nucleus are surrounded bycell walls and organised into cells

The remaining two nuclei called polar nuclei are found below the egg apparatus in the largecentral cell

Three cells consisting of two synergids amp one egg cell present bottom of

embryo sac Three cells

at the chalazal as antipodal cells

Two polar nuclei together present in large central cell

HISTORY

TOWARDS INDEPENDENCE AND PARTITION THE LAST PHASE(1935-1947)SUB TOPIC NATIONAL MOVEMENTS DURING THE SECOND WORLD WAR

Spread of Quit India Movement On 9th August 1942Gandhiji and other Congress leaders were arrested The Congress was declared illegal The news of the arrest of all leaders marked the beginning of a widespread movement of India It was not possible for such a movement to remain peacefulBut the arrest of the all notable congress leaders virtually left the movement in the hands of the mass The movement took the form of violent and militant outbreakBesides congressmen revolutionaries also were very active in the movement The Congress Socialist group also played a prominent role

1 Question Why did the British authority arrest the Congress leaders on 9 th August 1942Answer Congress Working committee adopted the Quit India resolution which was to be ratified at the Bombay AICC meeting in 8th August 1942 They decided to launch a mass struggle on non-violent lines Gandhiji gave a clarion call to all section of the people rdquoKarenge ya Marengerdquo (do or die) Congress leaders gave the call to driving out

the British from IndiaViceroy had taken strong action against the Quit India movement Gandhiji and all the leaders of Congress were arrested

2 Question How did Quit India Movement spread out all over IndiaAnswer The news of the leaders lsquo arrest marked the beginning of a widespread movement to remain peacefulThe movement took form of violent outbreak There were widespread cutting of telephone and Telegraph wires damaging railway lines raising barricades in cities and towns and other forms of violent demonstations

Question Name the leaders of Congress

Socialist group played a prominent part Notable among the Jayprakash Narayan Rammonohar Lohia Aruna Asaf Ali

Political science

Topic-Franchise and Representation

Summary

The election commission

The Election Commission of India is an autonomous constitutional authority responsible for administering Union and State election processes in India The body administers elections to the Lok Sabha Rajya Sabha State Legislative Assemblies in India and the offices of the President and Vice President in the country

Functions of election commission-

India is a sovereign socialist secular democratic republic Democracy runs like a golden thread in the social economic and political fabric woven by the Constitution given by lsquoWe the People of Indiarsquo unto ourselves The concept of democracy as visualised by the Constitution pre-supposes the representation of the people in Parliament and State legislatures by the method of election The Supreme Court has held that democracy is one of the inalienable basic features of the Constitution of India and forms part of its basic structure The Constitution of India adopted a Parliamentary form of government Parliament consists of the President of India and the two Houses mdash Rajya Sabha and Lok Sabha India being a Union of states has separate state legislatures for each state State legislatures consist of the Governor and two Houses mdash Legislative Council and Legislative Assembly mdash in seven states namely Andhra Pradesh Telangana Bihar Jammu amp Kashmir Karnataka Maharashtra and Uttar Pradesh and of the Governor and the state Legislative Assembly in the remaining 22 states Apart from the above two out of the seven Union Territories namely National Capital Territory of Delhi and Puducherry also have their Legislative Assemblies

ExecutionShort notes-Election commissionFunctions of election commission

Homework- Learn

Computer

Science

Computer hardware NAND Gate

A NOT-AND operation is known as NAND operation It has n input (n gt= 2) and one output

Logic diagram

Truth Table

NOR Gate

A NOT-OR operation is known as NOR operation It has n input (n gt= 2) and one output

Logic diagram

Truth Table

XOR Gate

XOR or Ex-OR gate is a special type of gate It can be used in the half

adder full adder and subtractor The exclusive-OR gate is abbreviated as EX-OR gate or sometime as X-OR gate It has n input (n gt= 2) and one output

Logic diagram

Truth Table

XNOR Gate

XNOR gate is a special type of gate It can be used in the half adder full adder and subtractor The exclusive-NOR gate is abbreviated as EX-NOR gate or sometime as X-NOR gate It has n input (n gt= 2) and one output

Logic diagram

Truth Table

Physics

Chapter 1 Electric Field ( Electric Dipole) (Summary)

Here we will derive Expression of electric field at broad side

On position of dipole

Execution

Q With the help of a labelled diagram obtain an expression for the electric field intensity E at any point on the equitorial line ( broad-side on position) of an electric dipole

Ans

E1 E1sinθ

E θ P E1 θ

( r2+L2)12 E2 E

r E2 E2sinθ

-q θ L O L +qA B

Let us consider that the point P is situated on the right bisector of the dipole AB at a distance r meter from its midpoint O

Let E1 and E2 be the electric field intensities of the electric field at P due to charge +q and ndashq of the dipole resp The distance of P from each charge is ( r2+L2)12

So E1 = 14 πϵ q

(r 2+L 2) away from +q

E2 = 14 πϵ q

(r 2+L 2) towards ndashq

The magnitudes of E1 and E2 are equal but directions are different Now resolving E1 and E2 into two components parallel and perpendicular to AB we get

The components perpendicular to AB E1sinθ and E2sinθ cancel each other because they are equal and opposite

The components parallel to AB are E1cosθ and E2 cosθ are in same direction and add up

So resultant intensity of electric field at the point P is

E = E1cosθ + E2 cosθ

E = 14 πϵ q

(r 2+L 2) 2 cosθ

Now from fig we have cosθ =BOBP = L (r2+L2)12

So we get E = 14 πϵ 2qL ( r2+L2)32

Now electric dipole moment p= 2qL

So E = 14 πϵ p ( r2+L2)32

HW Find the expression of Electric field as done here but this time take r gtgt 2L

Also find the expression of torque experience by a dipole

(Hint Electric force experienced by charges of dipole in electric field is qE each Let θ be the angle which dipole makes with electric lines of force then perpendicular distance between two charges is 2Lsinθ Then torque = force x perp distance = qE x 2L sinθ So τ=pE sinθ where p =2qL )

STUDY MATERIAL

Class XIISubject Eng Literature (The Tempest ndash William Shakespeare) Topic Act IV Scene 1 Lines 84 to 133 (Iris hellip A contract of true love Be not too late ) Date 27th April 2020 (4th Period)

[Students should read the original play and also the paraphrase given in the school prescribed textbook]Summary Questions amp Answers

o Ceres soon appears and comes to know that she has been summoned to celebrate the contract of true love

o Ceres expresses her unwillingness to meet Venus and Cupid as she has shunned their company

o Ceres and Juno both bestow their blessings upon Ferdinand and Miranda with June gifting honour riches happiness in marriage and Ceres presents plenty of earthrsquos produce

o Iris summons the water-nymphs and reapers to come and celebrate a contract

(1) IRIS Of her society (Line 91-101)

Be not afraid I met her deity

Cutting the clouds towards Pathos and her sonDove-drawn with her Here thought they to have doneSome wanton charm upon this man and maidWhose vows are that no bed-right shall be paidTill Hymens torch be lightedmdashbut in vainMarss hot minion is returned againHer waspish-headed son has broke his arrowsSwears he will shoot no more but play with sparrowsAnd be a boy right out

(i) Where were Venus and Cupid seen flying How were they travelling Why did they want to join the marriage celebration of Ferdinand and Miranda

of true love

Venus and Cupid were seen flying through the air towards Paphos the famous city which is situated on the island of Cyprus They were travelling by air-borne chariot drawn by doves They certainly wanted to come here in order to play some amorous trick upon Ferdinand and Miranda who are under a vow not to gratify their physical desires till the holy ceremony of their marriage has been performed(ii) What have Venus and Cupid done after failing in their plan

After being failure of their plan Venus who is a very passionate deity and who is the mistress of Mars (the god of war) has gone back while here ill-tempered son Cupid has broken his arrows of love in his state of desperation(iii) What has Cupid firmly decided

Cupid is feeling so disappointed that he has firmly decided to shoot no more arrows to arouse love in human hearts but to spend his time playing with sparrows Thus he would now become just a boy and would give up his original function of shooting arrows on human beings to make them fall in love(iv) What vow had Ceres taken How did Ceres feel at the abduction

After the abduction of her daughter Prosperina by Pluto Ceres had taken a vow to always keep away from the disgraceful company of Venus and her blind son Cupid the god of love Ceres felt deeply distressed when Pluto had carried off her daughter and had made her his wife by force(v) Why has Ceres not forgiven Venus and her blind son For what do Ceres want to be sure

As the abduction had been manipulated by Venus the goddess of beauty and love and her blind son Cupid Ceres has never forgiven them for their part in the whole plot Ceres wants to be sure that she would not have to meet Venus and Cupid who had engineered the abduction of her daughter Prosperina

AS THIS lsquoMASQUErsquo SCENE IS VERY IMPORTANT IN THE PLAY THE PARAPHRASE OF THE ENTIRE PORTION OF MASQUE SCENE (Act IV Lines 58 to 143) IS GIVEN BELOW

IRIS Goddess of RainbowCERES Goddess of Agriculture and all the fruits of the earth

(Nature growth prosperity rebirth ndash notions intimately connected to marriage)JUNO The majestic Queen of Heavens and wife of Jupiter (Jupiter is the king of Gods)

VENUS The Goddess of love CUPID Son of Venus PLUTO God of death (In the play referred by Shakespeare as lsquoDisrsquo which is a Roman name for Pluto)

ORIGINAL TEXT PARAPHRASEPROSPEROWellmdash

PROSPERONow come Ariel Let there be too many rather than too few

Now come my Ariel Bring a corollaryRather than want a spirit Appear and pertly[to Ferdinand and Miranda]No tongue all eyes Be silent

spirits in attendance Appear briskly

[to Ferdinand and Miranda]Look with your eyes but do not say a word

[Soft music] [Soft music][Enter Iris] [Enter Iris]

IRISCeres most bounteous lady thy rich leasOf wheat rye barley vetches oats and peasThy turfy mountains where live nibbling sheepAnd flat meads thatched with stover them to keepThy banks with pioned and twilled brimsWhich spongy April at thy hest betrimsTo make cold nymphs chaste crowns and thybroom-grovesWhose shadow the dismissegraved bachelor lovesBeing lass-lorn thy pole clipped vineyardAnd thy sea-marge sterile and rocky-hardWhere thou thyself dost airmdashthe Queen othrsquoSkyWhose watery arch and messenger am IBids thee leave these and with her sovereign grace[Juno appears] Here on this grass-plot in this very placeTo come and sport Her peacocks fly amainApproach rich Ceres her to entertain

IRISCeres most generous lady you are the cause of rich fields or fertile land where wheat rye barley beans oats and peas grow the grassy mountains where the sheep graze and the flat meadows covered with coarse hay to be used as fodder for cattleYour banks are covered with marsh-marigolds and reeds and the rainy April under your orders brings forth to make for the maids who are not in love beautiful crowns your woods where the broom flourishes and where the bachelor who has been dismissed by the maid he loved lies down being forsaken your vineyard in which the poles are embraced by the vines and the margin of the sea which is barren and rocky where you roam about to enjoy the fresh air ndash the queen of the sky (Juno) whose messenger I am besides being represented as the rainbow bids you leave all these and with her majesty here on this grassy plot in this very place come and sport her peacocks carry her fast in her chariot through the air and are making their way here approach rich Ceres to welcome her

[Enter Ariel as Ceres] [Enter Ariel as Ceres]

CERESHail many-coloured messenger that neerDost disobey the wife of JupiterWho with thy saffron wings upon my flowersDiffusest honey-drops refreshing showersAnd with each end of thy blue bow dost crownMy bosky acres and my unshrubbed downRich scarf to my proud earth Why hath thy queenSummoned me hither to this short-grassed green

CERESWelcome rainbow that never dared disobey Juno the wife of Jupiter who with your orange coloured rays spread honey-drops refreshing showers And with each end of thy blue bow drown my bushy acres and my hilly country which is free from shrubs you thus forming a rich scarf Why has your queen called me here to this place covered with short grass

IRISA contract of true love to celebrateAnd some donation freely to estateOn the blest lovers

IRISI have called you to celebrate a contract of true love and bestow some liberal gift upon the blessed lovers

ORIGINAL TEXT PARAPHRASECERESTell me heavenly bowIf Venus or her son as thou dost knowDo now attend the queen Since they did plotThe means that dusky Dis my daughter gotHer and her blind boys scandaled companyI have forsworn

CERESTell me heavenly bow if Venus the Goddess of love or Cupid her son and pedlar of passion at this time attend the heavenly queen Juno because you are sure to know Since the day they conspired against me and dark Pluto took away my daughter here and Cupidrsquos disgraceful company I have left off

IRISOf her societyBe not afraid I met her deityCutting the clouds towards Pathos and her sonDove-drawn with her Here thought they to have doneSome wanton charm upon this man and miad

IRISBe not afraid of her company I met her deity moving on the clouds towards Paphos the sacred home of Venus on the island of Cyprus along with her son on her chariot drawn by doves Here they contemplated to exercise a charm upon this man and maid producing

Whose vows are that no bed-right shall be paidTill Hymens torch be lightedmdashbut in vainMarss hot minion is returned againHer waspish-headed son has broke his arrowsSwears he will shoot no more but play with sparrowsAnd be a boy right out

wantonness before the actual marriage ceremony but did not succeed Venus has returned her irritable son has broken his arrows and swears that he will give up his practice of trying to inspire love but play with sparrows and be a boy again

[Music is heard] [Music is heard]

CERESHighst queen of stateGreat Juno comes I know her by her gait

CERESHighest queen of state Great Juno there she comes I know here by her gait

[Enter Juno] [Enter Juno]

JUNOHow does my bounteous sister Go with meTo bless this twain that they may Prosperous beAnd honoured in their issue

JUNOHow are you doing my generous sister Come with me to bless this couple so that they may be prosperous and fortunate in their children

[They sing] [They sing]

JUNOHonour riches marriage-blessingLong continuance and increasingHourly joys be still upon youJuno sings her blessings upon you

JUNOMay honour riches happiness in marriage long continuance and increase of those boons ever rest upon you as hourly joys Juno showers down upon you her blessings in song

CERESEarths increase foison plentyBarns and garners never emptyVines and clustring bunches growingPlants and goodly burden bowingSpring come to you at the farthestIn the very end of harvestScarcity and want shall shun youCeresrsquo blessing so is on you

CERESMay you have the plenty of earthrsquos produce Your barns and granaries may never be empty Your vines may grow with clustering bunches Your fruit trees may be heavily laden with their fruit May there be continuous spring and harvest May scantiness and want leave you forever Such is the blessing of Ceres upon you

FERDINANDThis is a most majestic vision andHarmoniously charmingly May I be boldTo think these spirits

FERDINANDThis is a great vision and magically melodious Should I suppose the characters (taking part in the masque) are spirits

PROSPEROSpirits which by mine artI have from their confines calld to enactMy present fancies

PROSPEROYes they are spirits whom I have summoned from the regions to which they are confined to carry into effect my fanciful designs

ORIGINAL TEXT PARAPHRASEFERDINANDLet me live here everSo rare a wondered father and a wifeMakes this place paradise

FERDINANDI should like to live here forever Such a wise and wonderful father makes this place a paradise

[Juno and Ceres whisper and send Iris on employment] [Juno and Ceres whisper and send Iris on employment]

PROSPEROSweet now silence

PROSPEROMy dear Ferdinand speak no more Juno and Ceres are

Juno and Ceres whisper seriouslyTheres something else to do Hush and be muteOr else our spell is marred

whispering with a solemn look There is something else coming Silence Or else our magic will be spoilt

IRISYour nymphs called naiads of the wandering brooksWith your sedged crowns and over-harmless looksLeave your crisp channels and on this green landAnswer your summons Juno does commandCome temperate nymphs and help to celebrateA contract of true love Be not too late

IRISYou nymphs called Naiads denizens (M inhabitants) of the running stream with your chaplets of sedge and ever-helpful looks leave your wrinkled channels and on the green land answer the summons sent to you Juno has ordered some chaste nymphs and help to celebrate a noble and true marriage Donrsquot delay

[Enter certain nymphs] [Enter certain nymphs]You sunburnt sicklemen of August wearyCome hither from the furrow and be merryMake holiday your rye-straw hats put onAnd these fresh nymphs encounter every oneIn country footing

You sunburnt harvesters weary from the effects of the heat in August come here from the furrowed land and rejoice Make holiday with your rye-straw hats upon you and meet these fresh nymphs and join in country dancing

[Enter certain reapers properly habited They join with the nymphs in a graceful dance towards the end whereof Prospero starts suddenly and speaks]

[Enter certain reapers properly habited They join with the nymphs in a graceful dance towards the end whereof Prospero starts suddenly and speaks]

PROSPERO[aside] I had forgot that foul conspiracyOf the beast Caliban and his confederatesAgainst my life The minute of their plotIs almost come [to the spirits]Well done Avoidno more

PROSPERO(Aside)I had forgotten the wicked conspiracy of the beast Caliban and his accomplices against my life the time of their plot has almost arrived ndash (To the Spirits) well done depart no more of this

[To a strange hollow and confused noise the spirits heavily vanish]

[The spirits depart]

ORIGINAL TEXT PARAPHRASEFERDINANDLet me live here everSo rare a wondered father and a wifeMakes this place paradise

FERDINANDI should like to live here forever Such a wise and wonderful father makes this place a paradise

[Juno and Ceres whisper and send Iris on employment] [Juno and Ceres whisper and send Iris on employment]

PROSPEROSweet now silenceJuno and Ceres whisper seriouslyTheres something else to do Hush and be muteOr else our spell is marred

PROSPEROMy dear Ferdinand speak no more Juno and Ceres are whispering with a solemn look There is something else coming Silence Or else our magic will be spoilt

IRISYour nymphs called naiads of the wandering brooksWith your sedged crowns and over-harmless looksLeave your crisp channels and on this green landAnswer your summons Juno does commandCome temperate nymphs and help to celebrateA contract of true love Be not too late

IRISYou nymphs called Naiads denizens (M inhabitants) of the running stream with your chaplets of sedge and ever-helpful looks leave your wrinkled channels and on the green land answer the summons sent to you Juno has ordered some chaste nymphs and help to celebrate a noble and true marriage Donrsquot delay

[Enter certain nymphs] [Enter certain nymphs]You sunburnt sicklemen of August wearyCome hither from the furrow and be merryMake holiday your rye-straw hats put onAnd these fresh nymphs encounter every oneIn country footing

You sunburnt harvesters weary from the effects of the heat in August come here from the furrowed land and rejoice Make holiday with your rye-straw hats upon you and meet these fresh nymphs and join in country dancing

[Enter certain reapers properly habited They join with the nymphs in a graceful dance towards the end whereof Prospero starts suddenly and speaks]

[Enter certain reapers properly habited They join with the nymphs in a graceful dance towards the end whereof Prospero starts suddenly and speaks]

PROSPERO[aside] I had forgot that foul conspiracyOf the beast Caliban and his confederatesAgainst my life The minute of their plotIs almost come [to the spirits]Well done Avoidno more

PROSPERO(Aside)I had forgotten the wicked conspiracy of the beast Caliban and his accomplices against my life the time of their plot has almost arrived ndash (To the Spirits) well done depart no more of this

[To a strange hollow and confused noise the spirits heavily vanish]

[The spirits depart]

Ac-12 27420 topic Revaluation of Assets and Liabilities

REVALUATION OF ASSETS AND LIABILITIES

On admission of a new partner the firm stands reconstituted and consequently the assets are revalued and liabilities are reassessed It is necessary to show the true position of the firm at the time of admission of a new partner If the values of the assets are raised gain will increase the capital of the existing partners Similarly any decrease in the value of assets ie loss will decrease the capital of the existing partners For this purpose alsquoRevaluation Accountrsquo is prepared This account is credited with all increases in the value of assets and decrease in the value of liabilities It is debited with decrease on account of value of assets and increase in the value of liabilities The balance of this account shows a gain or loss on revaluation which is transferred to the existing partnerrsquos capital account in existing profit sharing ratioAccounting for Revaluation of Assets and Liabilities when there is a Changein the Profit Sharing Ratio of Existing PartnersAssets and liabilities of a firm must also be revalued at the time of change in profit sharing ratio of existing partners The reason is that the realisable or actual value of assets and liabilities may be different from those shown in the Balance Sheet It is possible that with the passage of time some of the assets might have appreciated in value while the value of certain other assets might have decreased and no record has been made of such changes in the books of accounts Similarly there may be some unrecorded assets amp libilities that may have to be accounted for Revaluation of assets and reassessments of liabilities becomes necessary because the change in the

value of assets and liabilities belongs to the period to change in profit sharing ratio and hence must be shared by the partners in their old profit sharing ratio Revaluation of assets and reassessment of liabilities may be given effect to in two different ways (a) When revised values are to be recorded in the books and(b) When revised values are not to be recorded in the books

When revised values are to be recorded in the booksIn such a case revaluation of assets and reassessment of liabilities is done with the help of a new account called lsquoRevaluation Accountrsquo Sometimes this account is also called as lsquoProfit amp Loss Adjustment Acrsquo If there is a loss due to revaluation revaluation account is debited and if the revaluation results in a profit the revaluation account is credited The following journal entries made for this purpose are

(i) For increase in the value of assetsAsset Ac Dr (individually)To Revaluation Ac(ii) For decrease in the value of AssetRevaluation Ac Dr (individually)To Asset Ac[Decrease in the value of assets](iii) For increase in the value of LiabilitiesRevaluation Ac Dr (individually)To Liabilities Ac[Increase in the value of Liabilities](iv) For decrease in the value of LiabilitiesLiabilities Ac DrTo Revaluation Ac[Decrease in the value of Liabilities](v) For unrecorded AssetsAsset Ac [unrecorded] DrTo Revaluation Ac[Unrecorded asset recorded at actual value](vi) For unrecorded Liability Revaluation Ac DrTo Liability Ac [unrecorded][Unrecorded Liability recorded at actual value](vii) For transfer of gain on revaluationRevaluation Ac DrTo Existing Partnerrsquos CapitalCurrent Ac[Profit on revaluation transferred to capital account in existing ratio](viii) For transfer of loss on revaluationExisting Partnerrsquos CapitalCurrent Ac DrTo Revaluation Ac[Loss on revaluation transferred to capital account in existing ratio](a) When revaluation account shows gain Revaluation Ac DrTo Partnerrsquos Capital Ac (Old Profit Sharing Ratio)(Profit on revaluation credited to Partnerrsquos Capital Ac)(b) Above entry is reversed when revaluation account shows loss Partners Capital Acs (Old Profit Sharing Ratio) DrTo Revaluation Ac(Loss on revaluation debited to Partnerrsquos Capital Acs)

Proforma of Revaluation Account is given as under

Revaluation Account

Dr Cr Particulars ` Amount Particulars ` Amount To Decrease in value of assets By Increase in value of assets To Increase in value of liabilities By Decrease in value of liabilities To Unrecorded liabilities By Unrecorded assets To Gain on Revaluation (Transferred) By Loss on Revalution (Transferred)

ECO ndash12 2742020Topic- ELASTICITY OF DEMAND

CHAPTER - ELASTICITY OF DEMANDMEANINGDemand for a commodity is affected by many factors such as its price price of related goods income of its buyer tastes and preferences etc Elasticity means degree of response Elasticity of demand means degree of responsiveness of demand Demand for a commodity responds to change in price price of related goods income etc So we have three dimensions of elasticity of demandDIMENSION OF ELASTICITY OF DEMAND TYPES OF ELASTICITY OF DEMAND

Price elasticity of demand Income elasticity of demand Cross Elasticity of demand

Price elasticity of demand Price elasticity of demand means degree of responsiveness of demand for a commodity to the change in its price For example if demand for a commodity rises by 10 due to 5 fall in its price Price elasticity of demand (ep)=Percentage change in quantity demanded Percentage change in price of the commodity = 10 ( -)5 = ( - )2Note that ep will always be negative due to inverse relationship of price and quantity demanded

(ii) Income elasticity of demand Income elasticity of demand refers to the degree of responsiveness of demand for a commodity to the change in income of its buyer Suppose income of buyer rises by 10 and his demand for a commodity rises by 20 then Income elasticity of demand (ey)= change in quantity demanded change in price of the commodity =20 10 = 2

Cross Elasticity of demandCross elasticity of demand means the degree of responsiveness of demand for a commodity to the change in price of its related goods (substitute goods or complementary goods) Suppose demand for a commodity rises by 10 due to 5 rise in price of its substitute good then Cross elasticity of demand (ec) = change in quantity demanded change in price of related good = 10 2 = 5 (Tastes and preferences cannot be expressed numerically So elasticity ofdemand cannot be numerically expressed)

  • Chapter 1 Force (Summary)
  • Distinguish between external sovereignty and internal sovereignty
    • NAND Gate
      • Logic diagram
      • Truth Table
        • NOR Gate
          • Logic diagram
          • Truth Table
            • XOR Gate
              • Logic diagram
              • Truth Table
                • XNOR Gate
                  • Logic diagram
                  • Truth Table
                      • Physics
                      • Chapter 1 Electric Field ( Electric Dipole) (Summary)
Page 26:  · Web viewWe all know that Nouns are divided into two parts: common noun and proper noun.Apart from common and proper noun, we will also study about collective noun and compound

production of commodities necessary for human life but in amounts excessive of what is needed because people consume more than is necessary or waste what they have

Lack of awareness among the population is striking People are not aware of ways to reduce depletion and exploitation of materials

Accounts Cash Book Today we are going to start a new topic -Cash Book

The key terms used in this chapter are

bullCash book

bullSimple cash book

bullDouble column cash book bullTriple column cash book

bullPetty cash book

bullCash discount

bullContra entry

Here I will share you the meaning of each key terms

bullCash book Cash Book is a special purpose subsidiary book or journal in which cash received and cash payments are recorded

bullSimple cash book

It is a cash book in which only cash transactions are recorded It has only one column on each side

bullTriple column cash book

It is cash book which has three columns one column for each cash and Bankdiscount on each side of the cash book In this book both cash and Bank transactions are recorded together with discount allowed and received

bullPetty cash book

It is a cash book maintained for recording petty expenses

bullCash discount

Cash discount is the amount of discount received or allowed on cash payments and cash receipts Discount received is an income for the business while discount allowed isan expense

bullContra entry

It means transactions involving both cash and Bank Such transactions though recorded in the cash book are not posted into ledger The letter lsquoC is written in Ledger folio for contra entry

Business Studies

ENTREPRENEURSHIP

Now we shall discuss the second chapter

lsquoENTREPRENEURSHIPrsquo

Today before starting the chapter let us recall what

Questions

1What are the main characteristics of Intrapreneurship

Answer

The main characteristics of Intrapreneurship are

Corporate framework-it occurs within the framework of the same company

Semi-Autonomous-Intrapreneurship

we have read last day

Let s today start the class by recalling the last topic taught

Intrapreneurship is the process of discovering and exploring business opportunities within an existing company It involves launching new business ventures within the framework of a present corporation Intrapreneurship is also known as corporate entrepreneurship or corporate venturing

Now let us start with the characteristics of Intrapreneurship

The main characteristics of Intrapreneurship are

Corporate framework

Semi-Autonomous Lack of ownership Senior position Low risk taking Not own boss

Now let us discuss the meaning of enterprise

Enterprise means an undertaking or adventure that requires some innovation and investment and thus involves riskEnterprise always entails decision making coordination and risk bearing

involves crating amd nurturing a semi-autonomous business unit which may be a subsidiary a strategic business unit or a division

Lack of ownership-the intrapreneur is not the owner of the unitb he creates and nurtures

Senior position-he occupies a senior managerial position in the company

Low risk taking-An intrapreneur does not bear the full risk of failure

Not own boss-An intrapreneur is not his own bosss in legal termsHe enjoys the freedom and gets the required resources and support

2 How is Entrepreneur is different from Intrapreneur

The functions involved in both the entrepreneurship and intrapreneurship are by and large similar however there are several differences between the two

Point of distinction

Entrepreneur

Intrapreneur

status An independent business person

A senior executive within a company

Ownership Owner of

the business

An employeesometimes a share in ownership

Financing Responsible for raising finance for the business

Not responsible for raising the finance

Risk bearing

Bears the risk of the business

Does not bears the risk of the business

Reward Profit which is uncertain and irregularcan be loss

Fixed salary and fringe benefits

Need for security low high

3 What do you understand by enterprise

Answer Enterprise means an undertaking or adventure that requires some innovation and investment and thus involves riskEnterprise always entails decision making coordination and risk bearing

COMMERCE NATURE AND OBJECTIVES OF

BUSINESS

Today let us recall the last other two objectives of business by the chart given in the previous class

Firstly we would discuss Human Objectives

Business is run by people and for people Labour is a valuable business element

Human objectives of business are concerned with the well -being of labour

The human objectives are as follows

Labour welfare Developing human

resources Participative

management Labour

management cooperation

Questions

1 Explain the human objectives of a business enterprise

Answer

Business is run by people and for people Labour is a valuable business element

Human objectives of business are concerned with the well -being of labour

The human objectives are as follows

Labour welfare-Business must recognize the dignity of labour and human factors should be given the recognition

Developing human resources-Employees must be provided the opportunities for developing new skills and attitudes

Participative management-Employees should be allowed to take part in decision making process of business

Labour management cooperation-Business should strive for creating and maintaining cordial employer employee relations so as to ensure peace and progress in industry

Now let us discuss the national objectives of business

Optimum utilization of resources

National self- reliance Development of small

scale industries Development of

backward areas Control over pollution

2Explain the national objectives of a business enterprise

Answer

It is the duty of business to utilize the resources of the country properly the national objectives of business

Optimum utilization of resources ndashBusiness should use the nationrsquos resources in the best possible manner

National self- reliance-It is the duty of the business to help the government in increasing experts and in reducing dependence on imports

Development of small scale industries-Big business firms are expected to encourage growth of small scale industries which are necessary for generating employment

Development of backward areas-Business is expected to give preference to the industrialization of backward regions of the country

ECONOMICS

BASIC ECONOMIC CONCEPTS

SUB

TOPIC

Value

Wealth

Welfare

Today we shall start with a new topic of the same chapter ie lsquoValuersquo

Value of a commodity is defined as the valuation placed by a household on the consumption of this commodity

lsquoValuersquo has two different meanings and these are

a Value -in -use It refers to consumption value of a commodity It expresses the utility derived from the consumption of a particular commodity A necessity like water has a very high value ndashin ndashuse or

Question

1What is value

Answer

Value of a commodity is defined as the valuation placed by a household on the consumption of this commodity

2What is value-in use

Answer It refers to consumption value of a commodity It expresses the utility derived from the consumption of a particular commodity A necessity like water has a very high value ndashin ndashuse or consumption value

3What is value ndashin- exchange

Answer It relates to market value of a commodity

It is the rate at which a particular good or service can be exchanged for money

For example in barter system if a person is prepared to exchange 3 metres of cloth with 1 pair

consumption value

b Value ndashin-exchange It relates to market value of a commodity

It is the rate at which a particular good or service can be exchanged for moneyFor example in barter system if a person is prepared to exchange 3 metres of cloth with 1 pair of shoes then the value in exchange of 3 metres of cloth is 1 pair of shoesValue in exchange is the power of purchasing other goods In modern monetised economies the exchange value of goods are expressed in terms of money as prices

Now let us discuss the term lsquoWealthrsquo

Wealth refers to the stock of all those assets which are a source of income

Wealth is a stock concept

Wealth must possess the following features

a Utility It must possess utility or give some

of shoes then the value in exchange of 3 metres of cloth is 1 pair of shoes

Value in exchange is the power of purchasing other goods In modern monetised economies the exchange value of goods are expressed in terms of money as prices

4 What is wealth

Answer it refers to the stock of assets or goods which are a source of income and have personal or national ownership

5 What are the features of wealth

Answer The features of wealth are as follows

Wealth must possess the following features

a Utility It must possess utility or give some satisfaction

b Scarcity It must be limited in quantityc Transferability it should be transferable its

ownership can be transferred from one person to another person

d Exchange value It must possess exchange value

6 What is welfare

Answer

Welfare is defined as satisfaction and happiness a sense of well- being among the people

satisfactionb Scarcity It must be

limited in quantityc Transferability It

should be transferable its ownership can be transferred from one person to another person

d Exchange value It must possess exchange value

Now let us discuss the term lsquoWelfarersquo

Welfare is defined as satisfaction and happiness a sense of well- being among the people

Welfare is affected by factors like

a Consumption of goods and services

b Environment

c Family relations

d Degree of freedom

e Law and order situation

Mathematics Trigonometric equation

To find the general solution of the equation sinθ=0

When sin θ =0

Then θ= 0 π2π 3π-π -2π -3

i e when θ = 0 or an integral multiple of π

i e when θ= nπ where n is any integer

Therefore the general solution of the equation sin

Example1 Find the general values of θ which satisfy the equation sin2 θ =34

Solution sin2 θ= 34

Or sin θ = +34 or -34

Or sin θ = sin π3 or sin (-π3)

Therefore

θ = [nπ + (-1) n (π3)] or[ nπ+ (-1) n (-π3)]

= nπ +π3 or nπ-π3 where n= any integer

Example 2Find the values of θ which satisfy tan2 θ

θ=0 is θ= nπ where n is any integer

To find the general solution of the equation cos θ=0

When cos θ=0

Then θ=π2 3π2 5π2 -π2 -3π2 -5π3

i e when θ is an odd multiple of π2

i e when θ=(2n+1) π2 where n= any integer

Therefore the general solution of the equation cos θ =0 is θ= (2n+1) π2 where n= any integer

To find the general solution of the equation tan θ = 0

Clearly tan θ =0 implies sin θcos θ =0

Therefore θ = nπ

i e the general solution of the equation tan θ=0 is θ =nπ where n = any integer

To find the general solution of the equation cot θ =0

Clearly cot θ =0 implies (cos θsin θ) = 0

i e cos θ =0

Therefore θ = (2n+1) π2

Therefore the general solution of the equation cot θ =0 is θ = (2n+1) π2

Where n= any integer

To find the general solution of the equation sin θ= k (-

=13 -πleθleπ

Solution tan2 θ =13

Or tan θ = plusmn1radic(3) =tan(plusmnπ6)

θ=nπ plusmn π6 where n =any integer

If n=0 then θ=plusmnπ6

If n=1 then θ= π plusmn π6

If n=-1 then θ= -π plusmn π6

Therefore the required solution in -π le θ le π are θ= π6 5π6 -π6 -5π6

Exercise Find general solution of sin 2θ=cos θ [Hints Use sin 2θ= 2sin θcosθ and then take cosθ

common]

1lek le1)

Determine an angle alpha such that sin =k and -π2le αle π2

Then we have

Sin θ = k = sin α

Or sin θ - sin α =0

Or 2 cos [(θ+α) 2] sin [(θ-α) 2] =0

Therefore either cos [(θ +α) 2] =0 (1)

Or sin [(θ-α) 2] =0 (2)

Now from (1) we get (θ+α) 2= (2m+1) π2)

Or θ = (2m+1) π-α (3)

And from (2) we get (θ-α) 2 =mπ

Or θ= 2mπ+α(4)

Where m = any integer

Clearly the solution (3) amp (4) may be combined in the following form

θ= nπ+(-1) n α where n= any integer

Therefore the general solution of sin θ = sin α is θ = nπ +(-1) n α where n is any integer and -π2 le α le π2

Biology Chapter - 04Kingdom Monera

Today we will discuss about bacterial reproduction and its usefulness

Fig Binary Fission

Fig Conjugation Fig Transformation

Bacterial reproduction is mainly asexual but sexual reproduction

also takes place

Asexual reproduction takes place by i) Binary fission - from one bacteriato

two bacteria are produced in every 20 to 30mins

ii) Buddingiii) Endospore formation - during

unfourable condition

Sexual reproduction by three ways

1) Conjugation - Transfer of genetic material between cells that are in physical contact with one another

2) Transduction - Transfer of genetic materialfrom one cell to another by a bacteriophage

3) Transformation - Transfer of cell-freeor naked DNArsquo from one cell to another

Bacteria causes different diseases inplants animals and human and

it causes food spoilage and waterpollution but it also have some useful

activities

i) Bacteria are helpful in sewage water treatment

ii) It is used in antibiotic (medicine) production

iii) Anaerobic bacteria help in biogas(energy) production

iv) Many household products like yoghurt cheese are manufactured by use of bacteria

v) Rhizobium by symbiotic relationship with leguminous plant increase soil fertility

vi) Besides these bacteria is helpful in genetic engineering degradation of petroleum hydrocarbonand in dairy

industry

Physics Motion in plane Here we will introduce Projectile Motion

Execution

Projectile

Y

usinθ u h

θX

ucosθ

Suppose a body is projected with an angle θ So initial velocity u can be resolved into two components

Horizontal component - ucosθ ( for range)

Vertical component - usinθ ( for height)

usinθ changes during motion and becomes zero at maximum height position but ucosθ remain unchanged

The maximum height of projectile is h

NB If initial is upward then g = -ve and if it is downward then g = +ve Height is +ve if direction of motion does not change ( for ex a body thrown upwards but goes down ultimately then height h = -ve)

The angle of projectile θ is the angle made with horizontal

HISTORY ndash GROWTH OF NATIONALISM

SUB TOPIC- REVOLUTIONARY NATIONALISM Bengal formation of Anushilan Samity and Jugantar Group

The intensification of the Swadeshi movement and Government policy of terror and repression led to outbreak of violence Bombs were manufactured and attempts on the lives of unpopular Government officials became frequent In the gymnasium of Scottish Church College which was known as General Assemblies Institution a secret society was formed known as Anushilan Samity

Aurobindo Ghosh send from Baroda his emissary Jatindranath Banerjee to mobilize the Bengal revolutionaries

Hemchandra Qanungo and Satyen Bose published Journal Jugantar

The Jugantar group planned to assassinate oppressive magistrate Kingsford by Khudiram Bose and Prafulla Chaki in 1908 Prafulla Chaki committed suicide to avoid arrest Khudiram was tried and hanged

Afew days later the police found a bomb factory in Maniktala and arrested a large number of revolutionaries The trial of revolutionaries became famous as the Alipore Bomb Case

In the course of the trial the approver the public prosecuter and a police officer were assassinated

1 Question Name two journals which preached the cult of violence

Answer a) Yugantar edited by Bhupendranath DuttaB) Bandemataram edited by Aurobindo Ghosh2 Question Why was Khudiram arrested and hangedAnswer An attempt was made to assassinate a hated vindictive majistrate named Kingsford by Khudiram Bose and Prafulla Chaki Their attempt failed and the bomb they threw killed two English ladies Khudiram was arrested and put to trial and then hanged3Question Who was Aurubindo GhoshAnswer Aurobindo Ghosh a nationalist revolutionary who was charged for his involvement in the Alipore Bombing Case He was accused of it along with his brother Barindra nath Ghosh But Aurobindo was acquitted because of the brilliant pleading of his counsel Chittaranjan Das Then he became a spiritual reformer introducing his visions on human progress and spiritual evolution4 Qustion Who was KingsfordAnswer Kingsford was an unpopular British chief Magistrate who was the target of the bomb thrown at Muzaffarpur by Khudiram and Prafulla Chaki

Most of the accused were convicted and sentenced to

long term of imprisonmentBut

Aurobindo Ghosh was acquitted mainly owing to the brilliant pleading of his counsel Chittaranjan Das

Political science Topic-Sovereignty

Summary Sovereignty is the full right and power of a governing body over itself without any interference from outside sources or bodies In political theory sovereignty is a substantive term designating supreme legitimate authority over some polity In international law sovereignty is the exercise of power by a state

Internal Sovereignty

Internal sovereignty means supreme authority within ones territory while external sovereignty relates to the recognition on the part of all states that each possesses this power in equal measure

External sovereignty

external sovereignty relates to the recognition on the part of all states that each possesses this power in equal measure

Distinguish between

Execution

Answer the following questions

Short notes-

Sovereignty

Internal Sovereignty

External sovereignty

Homework- learn

external sovereignty and internal sovereigntySovereignty is the principle

of supreme and

unquestionable authority

reflected in the claim by the

state to be the sole author of

laws within its territory

Definition of external vs internal sovereigntyInternal sovereignty refers to

the relationship between a

sovereign power and its

subjects ndash it refers to the

location of the supreme

authority within the state In

the UK for example internal

sovereignty (supposedly)

resides within Parliament

reflected in the

constitutional principle of

parliamentary

sovereigntyBy contrast

external sovereignty refers

to the capacity of the state

to act independently and

autonomously on the world

stage This is what is

sometimes called lsquostate

sovereigntyrsquo or lsquonational

sovereigntyrsquo and implies

that states are legally equal

and that the territorial

integrity and political

independence of a state is

inviolable

Class ndash XII

Date - 2742020 STUDY MATERIALSubject Topic Summary Execution Business Studies

Job Analysis amp Manpower Planning

At first let us recall the chapter what we have discussed till nowbullJob analysisbullJob specification bullJob description bullJob enlargement bullJob enrichment

Today we will do some questions answers from the chapter

Questions 1ldquoJob analysis job description and job specification are interrelatedrdquo Comment Answer) Job analysis is a systematic and detailed examination of a job to collect all the relevant information about it The contents off the job are summarised in the job description The qualification needed for the job are summarised in job specificationThus there is close interrelationship between job analysis job description and job specification

Question 2ldquoJob enlargement is a horizontal extension of a job whereas job enrichment is a vertical extension of a jobrdquo ElucidateAnswer) Job enlargement involves adding one or more task to a job coma where as job enrichment involves adding more autonomy and responsibility to a job Job enlargement is therefore horizontal extension of a job coma whereas job enrichment is a vertical extension of a job

Question 3 )

What is manpower estimation Explain its quantitative and qualitative aspectsAnswer) Manpower estimation is the process by which management determines how an organisation should move from itrsquos current manpower positionto its desired manpower position There are two dimensions of Manpower estimation- quantitative and qualitative

Quantitative aspectThis aspect of Manpower estimation involves estimating the number of employees required in a future time period Workload analysis and workforce analysis are done to estimate the quantity of required manpower

Qualitative aspectThe estimate of the knowledge skills experience etc of required manpower is the qualitative aspect of Manpower estimation The quality of Manpower can be judged on the basis of job analysisand job specification

COMMERCE

CAPITAL-FIXED AND WORKING

Today let us start the class by discussing the sources of finance for different types of business firms

The term lsquocapitalrsquo refers to the investment made in the enterprise for the purpose of earning profits

Requirements of capital and sources of capital for different types of business firms are

1 Capital for sole proprietorship businessA sole proprietor operates at a small scale and thereforerequires a limited amount of capital

2 Capital for partnership firmCapital requirements as well as capital base of a partnership is bigger than that of a sole trader businessThe owned capital is contributed by the partners in an agreed ratio

3 Capital for joint stock companyA joint stock company generally requires large amount of capitalA public company can raise huge capital through issue of shares In addition to share capital it can utilize retained profits

Now let u discuss the meaning of Finance PlanningFinance planning is the process of estimation the financial requirements of an organization specifying the sources of firms and ensuring that enough funds are available at the right time

1 What do you mean by Finance PlanningAnswer Finance planning is the process of estimation the financial requirements of an organization specifying the sources of firms and ensuring that enough funds are available at the right time

2Discuss the role of financial planning of an enterpriseThe role of financial planning are as followsa A sound financial plan helps a business enterprise to avaid the problems of shortage and surplus of fundsbFinancial planning serves as a guide in developing a sound capital structure so as to maximize returns to shareholders c It helps in effective utilization of fundsd It provides policies and procedures for coordinating different functional areas or departments of businesse It enables the management to exercise effective control over the financial activities of an enterprisef It helps the company to prepare for facing business shocks and surprises in future

Mathematics

Continuity and differentiability

Recall Definition of ContinuityLet f(x) be a single valued function of x and x=a be a point in the domain of definition of the function The function is said to be continuous at x=a ifi) f(c) is defined ie f(x) has a definite finite value at x=cii) lim xrarra f(x) exists andiii) lim xrarra f(x) =f(a) In other words f(x) is said to be continuous at x=a if lim xrarra+ f(x)= lim xrarra- f(x) = f(a) Or f(a+0) =f(a-0) =f(a) Or lim hrarr0 f(a+h)= f(a) Algebra of continuous functionsNow we will study some algebra of continuous functions Theorem 1 Suppose f and g be two real functions continuous at a real number c Then(1) f + g is continuous at x = c(2) f ndash g is continuous at x = c(3) f g is continuous at x = c(4) (fg) is continuous at x = c (provided g (c) ne 0)

Example 1 Prove that every rational function is continuousSolution Recall that every rational function f is given byf(x)=[p(x) q(x) ] q(x)ne0where p and q are polynomial functions The domain of f is all real numbers except points at which q is zero Since polynomial functions are continuous f is continuous by (4) of Theorem 1Example 2Discuss the continuity of sine functionSolution To see this we use the following factslim xrarr0 sin x =0Now observe that f (x) = sin x is defined for every real number Let c be a real number Put x = c + h If x rarr c we know that h rarr 0 Therefore lim xrarrc f(x)

= lim xrarrc sin x= lim hrarr0 sin(c+h) =lim hrarr0 [sin c cos h + cos c sin h ]=lim hrarr0 (sin c cos h) + lim hrarr0 (cos c sin h) = sin c +0=sin c = f(c) Thus lim xrarrc f(x) = f(c) and hence f is a constant function Exercise Prove that the function f(x) = x2 +2x is continuous for every real value of x [Hints show that lim xrarra+ f(x) = lim xrarra- f(x) = f(a) ]

Biology Reproduction in Flowering plants We will discuss about megasporoangium

megasporagenesis and female gametophyte

Q4 Describe the structure of megasporangium

Ovule is attached to the placenta by astalk called funicle

Each ovule has one two or three protectivecoverings called integuments

At the tip of integuments a small openingcalled micropyle is organised

Opposite to the micropylar end is the chalaza

Within the integuments a mass of cellsnucellusand inside it embryo sac orfemale gametophyte is present

Q5 Describe a mature embryosacamp its formation

In most of the flowering plants only oneof the 4 megaspores formed as a result ofmegasporogenesis that is functional while theother three degenerate

The

functional megaspore develops into thefemale gametophyte

Formation The nucleus

of the functional megasporedivides mitotically to form two nuclei first andthen two more sequential mitotic nucleardivisions result in the formation of four ampthen eight nucleate stages of embryo sac

Six of the eight nucleus are surrounded bycell walls and organised into cells

The remaining two nuclei called polar nuclei are found below the egg apparatus in the largecentral cell

Three cells consisting of two synergids amp one egg cell present bottom of

embryo sac Three cells

at the chalazal as antipodal cells

Two polar nuclei together present in large central cell

HISTORY

TOWARDS INDEPENDENCE AND PARTITION THE LAST PHASE(1935-1947)SUB TOPIC NATIONAL MOVEMENTS DURING THE SECOND WORLD WAR

Spread of Quit India Movement On 9th August 1942Gandhiji and other Congress leaders were arrested The Congress was declared illegal The news of the arrest of all leaders marked the beginning of a widespread movement of India It was not possible for such a movement to remain peacefulBut the arrest of the all notable congress leaders virtually left the movement in the hands of the mass The movement took the form of violent and militant outbreakBesides congressmen revolutionaries also were very active in the movement The Congress Socialist group also played a prominent role

1 Question Why did the British authority arrest the Congress leaders on 9 th August 1942Answer Congress Working committee adopted the Quit India resolution which was to be ratified at the Bombay AICC meeting in 8th August 1942 They decided to launch a mass struggle on non-violent lines Gandhiji gave a clarion call to all section of the people rdquoKarenge ya Marengerdquo (do or die) Congress leaders gave the call to driving out

the British from IndiaViceroy had taken strong action against the Quit India movement Gandhiji and all the leaders of Congress were arrested

2 Question How did Quit India Movement spread out all over IndiaAnswer The news of the leaders lsquo arrest marked the beginning of a widespread movement to remain peacefulThe movement took form of violent outbreak There were widespread cutting of telephone and Telegraph wires damaging railway lines raising barricades in cities and towns and other forms of violent demonstations

Question Name the leaders of Congress

Socialist group played a prominent part Notable among the Jayprakash Narayan Rammonohar Lohia Aruna Asaf Ali

Political science

Topic-Franchise and Representation

Summary

The election commission

The Election Commission of India is an autonomous constitutional authority responsible for administering Union and State election processes in India The body administers elections to the Lok Sabha Rajya Sabha State Legislative Assemblies in India and the offices of the President and Vice President in the country

Functions of election commission-

India is a sovereign socialist secular democratic republic Democracy runs like a golden thread in the social economic and political fabric woven by the Constitution given by lsquoWe the People of Indiarsquo unto ourselves The concept of democracy as visualised by the Constitution pre-supposes the representation of the people in Parliament and State legislatures by the method of election The Supreme Court has held that democracy is one of the inalienable basic features of the Constitution of India and forms part of its basic structure The Constitution of India adopted a Parliamentary form of government Parliament consists of the President of India and the two Houses mdash Rajya Sabha and Lok Sabha India being a Union of states has separate state legislatures for each state State legislatures consist of the Governor and two Houses mdash Legislative Council and Legislative Assembly mdash in seven states namely Andhra Pradesh Telangana Bihar Jammu amp Kashmir Karnataka Maharashtra and Uttar Pradesh and of the Governor and the state Legislative Assembly in the remaining 22 states Apart from the above two out of the seven Union Territories namely National Capital Territory of Delhi and Puducherry also have their Legislative Assemblies

ExecutionShort notes-Election commissionFunctions of election commission

Homework- Learn

Computer

Science

Computer hardware NAND Gate

A NOT-AND operation is known as NAND operation It has n input (n gt= 2) and one output

Logic diagram

Truth Table

NOR Gate

A NOT-OR operation is known as NOR operation It has n input (n gt= 2) and one output

Logic diagram

Truth Table

XOR Gate

XOR or Ex-OR gate is a special type of gate It can be used in the half

adder full adder and subtractor The exclusive-OR gate is abbreviated as EX-OR gate or sometime as X-OR gate It has n input (n gt= 2) and one output

Logic diagram

Truth Table

XNOR Gate

XNOR gate is a special type of gate It can be used in the half adder full adder and subtractor The exclusive-NOR gate is abbreviated as EX-NOR gate or sometime as X-NOR gate It has n input (n gt= 2) and one output

Logic diagram

Truth Table

Physics

Chapter 1 Electric Field ( Electric Dipole) (Summary)

Here we will derive Expression of electric field at broad side

On position of dipole

Execution

Q With the help of a labelled diagram obtain an expression for the electric field intensity E at any point on the equitorial line ( broad-side on position) of an electric dipole

Ans

E1 E1sinθ

E θ P E1 θ

( r2+L2)12 E2 E

r E2 E2sinθ

-q θ L O L +qA B

Let us consider that the point P is situated on the right bisector of the dipole AB at a distance r meter from its midpoint O

Let E1 and E2 be the electric field intensities of the electric field at P due to charge +q and ndashq of the dipole resp The distance of P from each charge is ( r2+L2)12

So E1 = 14 πϵ q

(r 2+L 2) away from +q

E2 = 14 πϵ q

(r 2+L 2) towards ndashq

The magnitudes of E1 and E2 are equal but directions are different Now resolving E1 and E2 into two components parallel and perpendicular to AB we get

The components perpendicular to AB E1sinθ and E2sinθ cancel each other because they are equal and opposite

The components parallel to AB are E1cosθ and E2 cosθ are in same direction and add up

So resultant intensity of electric field at the point P is

E = E1cosθ + E2 cosθ

E = 14 πϵ q

(r 2+L 2) 2 cosθ

Now from fig we have cosθ =BOBP = L (r2+L2)12

So we get E = 14 πϵ 2qL ( r2+L2)32

Now electric dipole moment p= 2qL

So E = 14 πϵ p ( r2+L2)32

HW Find the expression of Electric field as done here but this time take r gtgt 2L

Also find the expression of torque experience by a dipole

(Hint Electric force experienced by charges of dipole in electric field is qE each Let θ be the angle which dipole makes with electric lines of force then perpendicular distance between two charges is 2Lsinθ Then torque = force x perp distance = qE x 2L sinθ So τ=pE sinθ where p =2qL )

STUDY MATERIAL

Class XIISubject Eng Literature (The Tempest ndash William Shakespeare) Topic Act IV Scene 1 Lines 84 to 133 (Iris hellip A contract of true love Be not too late ) Date 27th April 2020 (4th Period)

[Students should read the original play and also the paraphrase given in the school prescribed textbook]Summary Questions amp Answers

o Ceres soon appears and comes to know that she has been summoned to celebrate the contract of true love

o Ceres expresses her unwillingness to meet Venus and Cupid as she has shunned their company

o Ceres and Juno both bestow their blessings upon Ferdinand and Miranda with June gifting honour riches happiness in marriage and Ceres presents plenty of earthrsquos produce

o Iris summons the water-nymphs and reapers to come and celebrate a contract

(1) IRIS Of her society (Line 91-101)

Be not afraid I met her deity

Cutting the clouds towards Pathos and her sonDove-drawn with her Here thought they to have doneSome wanton charm upon this man and maidWhose vows are that no bed-right shall be paidTill Hymens torch be lightedmdashbut in vainMarss hot minion is returned againHer waspish-headed son has broke his arrowsSwears he will shoot no more but play with sparrowsAnd be a boy right out

(i) Where were Venus and Cupid seen flying How were they travelling Why did they want to join the marriage celebration of Ferdinand and Miranda

of true love

Venus and Cupid were seen flying through the air towards Paphos the famous city which is situated on the island of Cyprus They were travelling by air-borne chariot drawn by doves They certainly wanted to come here in order to play some amorous trick upon Ferdinand and Miranda who are under a vow not to gratify their physical desires till the holy ceremony of their marriage has been performed(ii) What have Venus and Cupid done after failing in their plan

After being failure of their plan Venus who is a very passionate deity and who is the mistress of Mars (the god of war) has gone back while here ill-tempered son Cupid has broken his arrows of love in his state of desperation(iii) What has Cupid firmly decided

Cupid is feeling so disappointed that he has firmly decided to shoot no more arrows to arouse love in human hearts but to spend his time playing with sparrows Thus he would now become just a boy and would give up his original function of shooting arrows on human beings to make them fall in love(iv) What vow had Ceres taken How did Ceres feel at the abduction

After the abduction of her daughter Prosperina by Pluto Ceres had taken a vow to always keep away from the disgraceful company of Venus and her blind son Cupid the god of love Ceres felt deeply distressed when Pluto had carried off her daughter and had made her his wife by force(v) Why has Ceres not forgiven Venus and her blind son For what do Ceres want to be sure

As the abduction had been manipulated by Venus the goddess of beauty and love and her blind son Cupid Ceres has never forgiven them for their part in the whole plot Ceres wants to be sure that she would not have to meet Venus and Cupid who had engineered the abduction of her daughter Prosperina

AS THIS lsquoMASQUErsquo SCENE IS VERY IMPORTANT IN THE PLAY THE PARAPHRASE OF THE ENTIRE PORTION OF MASQUE SCENE (Act IV Lines 58 to 143) IS GIVEN BELOW

IRIS Goddess of RainbowCERES Goddess of Agriculture and all the fruits of the earth

(Nature growth prosperity rebirth ndash notions intimately connected to marriage)JUNO The majestic Queen of Heavens and wife of Jupiter (Jupiter is the king of Gods)

VENUS The Goddess of love CUPID Son of Venus PLUTO God of death (In the play referred by Shakespeare as lsquoDisrsquo which is a Roman name for Pluto)

ORIGINAL TEXT PARAPHRASEPROSPEROWellmdash

PROSPERONow come Ariel Let there be too many rather than too few

Now come my Ariel Bring a corollaryRather than want a spirit Appear and pertly[to Ferdinand and Miranda]No tongue all eyes Be silent

spirits in attendance Appear briskly

[to Ferdinand and Miranda]Look with your eyes but do not say a word

[Soft music] [Soft music][Enter Iris] [Enter Iris]

IRISCeres most bounteous lady thy rich leasOf wheat rye barley vetches oats and peasThy turfy mountains where live nibbling sheepAnd flat meads thatched with stover them to keepThy banks with pioned and twilled brimsWhich spongy April at thy hest betrimsTo make cold nymphs chaste crowns and thybroom-grovesWhose shadow the dismissegraved bachelor lovesBeing lass-lorn thy pole clipped vineyardAnd thy sea-marge sterile and rocky-hardWhere thou thyself dost airmdashthe Queen othrsquoSkyWhose watery arch and messenger am IBids thee leave these and with her sovereign grace[Juno appears] Here on this grass-plot in this very placeTo come and sport Her peacocks fly amainApproach rich Ceres her to entertain

IRISCeres most generous lady you are the cause of rich fields or fertile land where wheat rye barley beans oats and peas grow the grassy mountains where the sheep graze and the flat meadows covered with coarse hay to be used as fodder for cattleYour banks are covered with marsh-marigolds and reeds and the rainy April under your orders brings forth to make for the maids who are not in love beautiful crowns your woods where the broom flourishes and where the bachelor who has been dismissed by the maid he loved lies down being forsaken your vineyard in which the poles are embraced by the vines and the margin of the sea which is barren and rocky where you roam about to enjoy the fresh air ndash the queen of the sky (Juno) whose messenger I am besides being represented as the rainbow bids you leave all these and with her majesty here on this grassy plot in this very place come and sport her peacocks carry her fast in her chariot through the air and are making their way here approach rich Ceres to welcome her

[Enter Ariel as Ceres] [Enter Ariel as Ceres]

CERESHail many-coloured messenger that neerDost disobey the wife of JupiterWho with thy saffron wings upon my flowersDiffusest honey-drops refreshing showersAnd with each end of thy blue bow dost crownMy bosky acres and my unshrubbed downRich scarf to my proud earth Why hath thy queenSummoned me hither to this short-grassed green

CERESWelcome rainbow that never dared disobey Juno the wife of Jupiter who with your orange coloured rays spread honey-drops refreshing showers And with each end of thy blue bow drown my bushy acres and my hilly country which is free from shrubs you thus forming a rich scarf Why has your queen called me here to this place covered with short grass

IRISA contract of true love to celebrateAnd some donation freely to estateOn the blest lovers

IRISI have called you to celebrate a contract of true love and bestow some liberal gift upon the blessed lovers

ORIGINAL TEXT PARAPHRASECERESTell me heavenly bowIf Venus or her son as thou dost knowDo now attend the queen Since they did plotThe means that dusky Dis my daughter gotHer and her blind boys scandaled companyI have forsworn

CERESTell me heavenly bow if Venus the Goddess of love or Cupid her son and pedlar of passion at this time attend the heavenly queen Juno because you are sure to know Since the day they conspired against me and dark Pluto took away my daughter here and Cupidrsquos disgraceful company I have left off

IRISOf her societyBe not afraid I met her deityCutting the clouds towards Pathos and her sonDove-drawn with her Here thought they to have doneSome wanton charm upon this man and miad

IRISBe not afraid of her company I met her deity moving on the clouds towards Paphos the sacred home of Venus on the island of Cyprus along with her son on her chariot drawn by doves Here they contemplated to exercise a charm upon this man and maid producing

Whose vows are that no bed-right shall be paidTill Hymens torch be lightedmdashbut in vainMarss hot minion is returned againHer waspish-headed son has broke his arrowsSwears he will shoot no more but play with sparrowsAnd be a boy right out

wantonness before the actual marriage ceremony but did not succeed Venus has returned her irritable son has broken his arrows and swears that he will give up his practice of trying to inspire love but play with sparrows and be a boy again

[Music is heard] [Music is heard]

CERESHighst queen of stateGreat Juno comes I know her by her gait

CERESHighest queen of state Great Juno there she comes I know here by her gait

[Enter Juno] [Enter Juno]

JUNOHow does my bounteous sister Go with meTo bless this twain that they may Prosperous beAnd honoured in their issue

JUNOHow are you doing my generous sister Come with me to bless this couple so that they may be prosperous and fortunate in their children

[They sing] [They sing]

JUNOHonour riches marriage-blessingLong continuance and increasingHourly joys be still upon youJuno sings her blessings upon you

JUNOMay honour riches happiness in marriage long continuance and increase of those boons ever rest upon you as hourly joys Juno showers down upon you her blessings in song

CERESEarths increase foison plentyBarns and garners never emptyVines and clustring bunches growingPlants and goodly burden bowingSpring come to you at the farthestIn the very end of harvestScarcity and want shall shun youCeresrsquo blessing so is on you

CERESMay you have the plenty of earthrsquos produce Your barns and granaries may never be empty Your vines may grow with clustering bunches Your fruit trees may be heavily laden with their fruit May there be continuous spring and harvest May scantiness and want leave you forever Such is the blessing of Ceres upon you

FERDINANDThis is a most majestic vision andHarmoniously charmingly May I be boldTo think these spirits

FERDINANDThis is a great vision and magically melodious Should I suppose the characters (taking part in the masque) are spirits

PROSPEROSpirits which by mine artI have from their confines calld to enactMy present fancies

PROSPEROYes they are spirits whom I have summoned from the regions to which they are confined to carry into effect my fanciful designs

ORIGINAL TEXT PARAPHRASEFERDINANDLet me live here everSo rare a wondered father and a wifeMakes this place paradise

FERDINANDI should like to live here forever Such a wise and wonderful father makes this place a paradise

[Juno and Ceres whisper and send Iris on employment] [Juno and Ceres whisper and send Iris on employment]

PROSPEROSweet now silence

PROSPEROMy dear Ferdinand speak no more Juno and Ceres are

Juno and Ceres whisper seriouslyTheres something else to do Hush and be muteOr else our spell is marred

whispering with a solemn look There is something else coming Silence Or else our magic will be spoilt

IRISYour nymphs called naiads of the wandering brooksWith your sedged crowns and over-harmless looksLeave your crisp channels and on this green landAnswer your summons Juno does commandCome temperate nymphs and help to celebrateA contract of true love Be not too late

IRISYou nymphs called Naiads denizens (M inhabitants) of the running stream with your chaplets of sedge and ever-helpful looks leave your wrinkled channels and on the green land answer the summons sent to you Juno has ordered some chaste nymphs and help to celebrate a noble and true marriage Donrsquot delay

[Enter certain nymphs] [Enter certain nymphs]You sunburnt sicklemen of August wearyCome hither from the furrow and be merryMake holiday your rye-straw hats put onAnd these fresh nymphs encounter every oneIn country footing

You sunburnt harvesters weary from the effects of the heat in August come here from the furrowed land and rejoice Make holiday with your rye-straw hats upon you and meet these fresh nymphs and join in country dancing

[Enter certain reapers properly habited They join with the nymphs in a graceful dance towards the end whereof Prospero starts suddenly and speaks]

[Enter certain reapers properly habited They join with the nymphs in a graceful dance towards the end whereof Prospero starts suddenly and speaks]

PROSPERO[aside] I had forgot that foul conspiracyOf the beast Caliban and his confederatesAgainst my life The minute of their plotIs almost come [to the spirits]Well done Avoidno more

PROSPERO(Aside)I had forgotten the wicked conspiracy of the beast Caliban and his accomplices against my life the time of their plot has almost arrived ndash (To the Spirits) well done depart no more of this

[To a strange hollow and confused noise the spirits heavily vanish]

[The spirits depart]

ORIGINAL TEXT PARAPHRASEFERDINANDLet me live here everSo rare a wondered father and a wifeMakes this place paradise

FERDINANDI should like to live here forever Such a wise and wonderful father makes this place a paradise

[Juno and Ceres whisper and send Iris on employment] [Juno and Ceres whisper and send Iris on employment]

PROSPEROSweet now silenceJuno and Ceres whisper seriouslyTheres something else to do Hush and be muteOr else our spell is marred

PROSPEROMy dear Ferdinand speak no more Juno and Ceres are whispering with a solemn look There is something else coming Silence Or else our magic will be spoilt

IRISYour nymphs called naiads of the wandering brooksWith your sedged crowns and over-harmless looksLeave your crisp channels and on this green landAnswer your summons Juno does commandCome temperate nymphs and help to celebrateA contract of true love Be not too late

IRISYou nymphs called Naiads denizens (M inhabitants) of the running stream with your chaplets of sedge and ever-helpful looks leave your wrinkled channels and on the green land answer the summons sent to you Juno has ordered some chaste nymphs and help to celebrate a noble and true marriage Donrsquot delay

[Enter certain nymphs] [Enter certain nymphs]You sunburnt sicklemen of August wearyCome hither from the furrow and be merryMake holiday your rye-straw hats put onAnd these fresh nymphs encounter every oneIn country footing

You sunburnt harvesters weary from the effects of the heat in August come here from the furrowed land and rejoice Make holiday with your rye-straw hats upon you and meet these fresh nymphs and join in country dancing

[Enter certain reapers properly habited They join with the nymphs in a graceful dance towards the end whereof Prospero starts suddenly and speaks]

[Enter certain reapers properly habited They join with the nymphs in a graceful dance towards the end whereof Prospero starts suddenly and speaks]

PROSPERO[aside] I had forgot that foul conspiracyOf the beast Caliban and his confederatesAgainst my life The minute of their plotIs almost come [to the spirits]Well done Avoidno more

PROSPERO(Aside)I had forgotten the wicked conspiracy of the beast Caliban and his accomplices against my life the time of their plot has almost arrived ndash (To the Spirits) well done depart no more of this

[To a strange hollow and confused noise the spirits heavily vanish]

[The spirits depart]

Ac-12 27420 topic Revaluation of Assets and Liabilities

REVALUATION OF ASSETS AND LIABILITIES

On admission of a new partner the firm stands reconstituted and consequently the assets are revalued and liabilities are reassessed It is necessary to show the true position of the firm at the time of admission of a new partner If the values of the assets are raised gain will increase the capital of the existing partners Similarly any decrease in the value of assets ie loss will decrease the capital of the existing partners For this purpose alsquoRevaluation Accountrsquo is prepared This account is credited with all increases in the value of assets and decrease in the value of liabilities It is debited with decrease on account of value of assets and increase in the value of liabilities The balance of this account shows a gain or loss on revaluation which is transferred to the existing partnerrsquos capital account in existing profit sharing ratioAccounting for Revaluation of Assets and Liabilities when there is a Changein the Profit Sharing Ratio of Existing PartnersAssets and liabilities of a firm must also be revalued at the time of change in profit sharing ratio of existing partners The reason is that the realisable or actual value of assets and liabilities may be different from those shown in the Balance Sheet It is possible that with the passage of time some of the assets might have appreciated in value while the value of certain other assets might have decreased and no record has been made of such changes in the books of accounts Similarly there may be some unrecorded assets amp libilities that may have to be accounted for Revaluation of assets and reassessments of liabilities becomes necessary because the change in the

value of assets and liabilities belongs to the period to change in profit sharing ratio and hence must be shared by the partners in their old profit sharing ratio Revaluation of assets and reassessment of liabilities may be given effect to in two different ways (a) When revised values are to be recorded in the books and(b) When revised values are not to be recorded in the books

When revised values are to be recorded in the booksIn such a case revaluation of assets and reassessment of liabilities is done with the help of a new account called lsquoRevaluation Accountrsquo Sometimes this account is also called as lsquoProfit amp Loss Adjustment Acrsquo If there is a loss due to revaluation revaluation account is debited and if the revaluation results in a profit the revaluation account is credited The following journal entries made for this purpose are

(i) For increase in the value of assetsAsset Ac Dr (individually)To Revaluation Ac(ii) For decrease in the value of AssetRevaluation Ac Dr (individually)To Asset Ac[Decrease in the value of assets](iii) For increase in the value of LiabilitiesRevaluation Ac Dr (individually)To Liabilities Ac[Increase in the value of Liabilities](iv) For decrease in the value of LiabilitiesLiabilities Ac DrTo Revaluation Ac[Decrease in the value of Liabilities](v) For unrecorded AssetsAsset Ac [unrecorded] DrTo Revaluation Ac[Unrecorded asset recorded at actual value](vi) For unrecorded Liability Revaluation Ac DrTo Liability Ac [unrecorded][Unrecorded Liability recorded at actual value](vii) For transfer of gain on revaluationRevaluation Ac DrTo Existing Partnerrsquos CapitalCurrent Ac[Profit on revaluation transferred to capital account in existing ratio](viii) For transfer of loss on revaluationExisting Partnerrsquos CapitalCurrent Ac DrTo Revaluation Ac[Loss on revaluation transferred to capital account in existing ratio](a) When revaluation account shows gain Revaluation Ac DrTo Partnerrsquos Capital Ac (Old Profit Sharing Ratio)(Profit on revaluation credited to Partnerrsquos Capital Ac)(b) Above entry is reversed when revaluation account shows loss Partners Capital Acs (Old Profit Sharing Ratio) DrTo Revaluation Ac(Loss on revaluation debited to Partnerrsquos Capital Acs)

Proforma of Revaluation Account is given as under

Revaluation Account

Dr Cr Particulars ` Amount Particulars ` Amount To Decrease in value of assets By Increase in value of assets To Increase in value of liabilities By Decrease in value of liabilities To Unrecorded liabilities By Unrecorded assets To Gain on Revaluation (Transferred) By Loss on Revalution (Transferred)

ECO ndash12 2742020Topic- ELASTICITY OF DEMAND

CHAPTER - ELASTICITY OF DEMANDMEANINGDemand for a commodity is affected by many factors such as its price price of related goods income of its buyer tastes and preferences etc Elasticity means degree of response Elasticity of demand means degree of responsiveness of demand Demand for a commodity responds to change in price price of related goods income etc So we have three dimensions of elasticity of demandDIMENSION OF ELASTICITY OF DEMAND TYPES OF ELASTICITY OF DEMAND

Price elasticity of demand Income elasticity of demand Cross Elasticity of demand

Price elasticity of demand Price elasticity of demand means degree of responsiveness of demand for a commodity to the change in its price For example if demand for a commodity rises by 10 due to 5 fall in its price Price elasticity of demand (ep)=Percentage change in quantity demanded Percentage change in price of the commodity = 10 ( -)5 = ( - )2Note that ep will always be negative due to inverse relationship of price and quantity demanded

(ii) Income elasticity of demand Income elasticity of demand refers to the degree of responsiveness of demand for a commodity to the change in income of its buyer Suppose income of buyer rises by 10 and his demand for a commodity rises by 20 then Income elasticity of demand (ey)= change in quantity demanded change in price of the commodity =20 10 = 2

Cross Elasticity of demandCross elasticity of demand means the degree of responsiveness of demand for a commodity to the change in price of its related goods (substitute goods or complementary goods) Suppose demand for a commodity rises by 10 due to 5 rise in price of its substitute good then Cross elasticity of demand (ec) = change in quantity demanded change in price of related good = 10 2 = 5 (Tastes and preferences cannot be expressed numerically So elasticity ofdemand cannot be numerically expressed)

  • Chapter 1 Force (Summary)
  • Distinguish between external sovereignty and internal sovereignty
    • NAND Gate
      • Logic diagram
      • Truth Table
        • NOR Gate
          • Logic diagram
          • Truth Table
            • XOR Gate
              • Logic diagram
              • Truth Table
                • XNOR Gate
                  • Logic diagram
                  • Truth Table
                      • Physics
                      • Chapter 1 Electric Field ( Electric Dipole) (Summary)
Page 27:  · Web viewWe all know that Nouns are divided into two parts: common noun and proper noun.Apart from common and proper noun, we will also study about collective noun and compound

we have read last day

Let s today start the class by recalling the last topic taught

Intrapreneurship is the process of discovering and exploring business opportunities within an existing company It involves launching new business ventures within the framework of a present corporation Intrapreneurship is also known as corporate entrepreneurship or corporate venturing

Now let us start with the characteristics of Intrapreneurship

The main characteristics of Intrapreneurship are

Corporate framework

Semi-Autonomous Lack of ownership Senior position Low risk taking Not own boss

Now let us discuss the meaning of enterprise

Enterprise means an undertaking or adventure that requires some innovation and investment and thus involves riskEnterprise always entails decision making coordination and risk bearing

involves crating amd nurturing a semi-autonomous business unit which may be a subsidiary a strategic business unit or a division

Lack of ownership-the intrapreneur is not the owner of the unitb he creates and nurtures

Senior position-he occupies a senior managerial position in the company

Low risk taking-An intrapreneur does not bear the full risk of failure

Not own boss-An intrapreneur is not his own bosss in legal termsHe enjoys the freedom and gets the required resources and support

2 How is Entrepreneur is different from Intrapreneur

The functions involved in both the entrepreneurship and intrapreneurship are by and large similar however there are several differences between the two

Point of distinction

Entrepreneur

Intrapreneur

status An independent business person

A senior executive within a company

Ownership Owner of

the business

An employeesometimes a share in ownership

Financing Responsible for raising finance for the business

Not responsible for raising the finance

Risk bearing

Bears the risk of the business

Does not bears the risk of the business

Reward Profit which is uncertain and irregularcan be loss

Fixed salary and fringe benefits

Need for security low high

3 What do you understand by enterprise

Answer Enterprise means an undertaking or adventure that requires some innovation and investment and thus involves riskEnterprise always entails decision making coordination and risk bearing

COMMERCE NATURE AND OBJECTIVES OF

BUSINESS

Today let us recall the last other two objectives of business by the chart given in the previous class

Firstly we would discuss Human Objectives

Business is run by people and for people Labour is a valuable business element

Human objectives of business are concerned with the well -being of labour

The human objectives are as follows

Labour welfare Developing human

resources Participative

management Labour

management cooperation

Questions

1 Explain the human objectives of a business enterprise

Answer

Business is run by people and for people Labour is a valuable business element

Human objectives of business are concerned with the well -being of labour

The human objectives are as follows

Labour welfare-Business must recognize the dignity of labour and human factors should be given the recognition

Developing human resources-Employees must be provided the opportunities for developing new skills and attitudes

Participative management-Employees should be allowed to take part in decision making process of business

Labour management cooperation-Business should strive for creating and maintaining cordial employer employee relations so as to ensure peace and progress in industry

Now let us discuss the national objectives of business

Optimum utilization of resources

National self- reliance Development of small

scale industries Development of

backward areas Control over pollution

2Explain the national objectives of a business enterprise

Answer

It is the duty of business to utilize the resources of the country properly the national objectives of business

Optimum utilization of resources ndashBusiness should use the nationrsquos resources in the best possible manner

National self- reliance-It is the duty of the business to help the government in increasing experts and in reducing dependence on imports

Development of small scale industries-Big business firms are expected to encourage growth of small scale industries which are necessary for generating employment

Development of backward areas-Business is expected to give preference to the industrialization of backward regions of the country

ECONOMICS

BASIC ECONOMIC CONCEPTS

SUB

TOPIC

Value

Wealth

Welfare

Today we shall start with a new topic of the same chapter ie lsquoValuersquo

Value of a commodity is defined as the valuation placed by a household on the consumption of this commodity

lsquoValuersquo has two different meanings and these are

a Value -in -use It refers to consumption value of a commodity It expresses the utility derived from the consumption of a particular commodity A necessity like water has a very high value ndashin ndashuse or

Question

1What is value

Answer

Value of a commodity is defined as the valuation placed by a household on the consumption of this commodity

2What is value-in use

Answer It refers to consumption value of a commodity It expresses the utility derived from the consumption of a particular commodity A necessity like water has a very high value ndashin ndashuse or consumption value

3What is value ndashin- exchange

Answer It relates to market value of a commodity

It is the rate at which a particular good or service can be exchanged for money

For example in barter system if a person is prepared to exchange 3 metres of cloth with 1 pair

consumption value

b Value ndashin-exchange It relates to market value of a commodity

It is the rate at which a particular good or service can be exchanged for moneyFor example in barter system if a person is prepared to exchange 3 metres of cloth with 1 pair of shoes then the value in exchange of 3 metres of cloth is 1 pair of shoesValue in exchange is the power of purchasing other goods In modern monetised economies the exchange value of goods are expressed in terms of money as prices

Now let us discuss the term lsquoWealthrsquo

Wealth refers to the stock of all those assets which are a source of income

Wealth is a stock concept

Wealth must possess the following features

a Utility It must possess utility or give some

of shoes then the value in exchange of 3 metres of cloth is 1 pair of shoes

Value in exchange is the power of purchasing other goods In modern monetised economies the exchange value of goods are expressed in terms of money as prices

4 What is wealth

Answer it refers to the stock of assets or goods which are a source of income and have personal or national ownership

5 What are the features of wealth

Answer The features of wealth are as follows

Wealth must possess the following features

a Utility It must possess utility or give some satisfaction

b Scarcity It must be limited in quantityc Transferability it should be transferable its

ownership can be transferred from one person to another person

d Exchange value It must possess exchange value

6 What is welfare

Answer

Welfare is defined as satisfaction and happiness a sense of well- being among the people

satisfactionb Scarcity It must be

limited in quantityc Transferability It

should be transferable its ownership can be transferred from one person to another person

d Exchange value It must possess exchange value

Now let us discuss the term lsquoWelfarersquo

Welfare is defined as satisfaction and happiness a sense of well- being among the people

Welfare is affected by factors like

a Consumption of goods and services

b Environment

c Family relations

d Degree of freedom

e Law and order situation

Mathematics Trigonometric equation

To find the general solution of the equation sinθ=0

When sin θ =0

Then θ= 0 π2π 3π-π -2π -3

i e when θ = 0 or an integral multiple of π

i e when θ= nπ where n is any integer

Therefore the general solution of the equation sin

Example1 Find the general values of θ which satisfy the equation sin2 θ =34

Solution sin2 θ= 34

Or sin θ = +34 or -34

Or sin θ = sin π3 or sin (-π3)

Therefore

θ = [nπ + (-1) n (π3)] or[ nπ+ (-1) n (-π3)]

= nπ +π3 or nπ-π3 where n= any integer

Example 2Find the values of θ which satisfy tan2 θ

θ=0 is θ= nπ where n is any integer

To find the general solution of the equation cos θ=0

When cos θ=0

Then θ=π2 3π2 5π2 -π2 -3π2 -5π3

i e when θ is an odd multiple of π2

i e when θ=(2n+1) π2 where n= any integer

Therefore the general solution of the equation cos θ =0 is θ= (2n+1) π2 where n= any integer

To find the general solution of the equation tan θ = 0

Clearly tan θ =0 implies sin θcos θ =0

Therefore θ = nπ

i e the general solution of the equation tan θ=0 is θ =nπ where n = any integer

To find the general solution of the equation cot θ =0

Clearly cot θ =0 implies (cos θsin θ) = 0

i e cos θ =0

Therefore θ = (2n+1) π2

Therefore the general solution of the equation cot θ =0 is θ = (2n+1) π2

Where n= any integer

To find the general solution of the equation sin θ= k (-

=13 -πleθleπ

Solution tan2 θ =13

Or tan θ = plusmn1radic(3) =tan(plusmnπ6)

θ=nπ plusmn π6 where n =any integer

If n=0 then θ=plusmnπ6

If n=1 then θ= π plusmn π6

If n=-1 then θ= -π plusmn π6

Therefore the required solution in -π le θ le π are θ= π6 5π6 -π6 -5π6

Exercise Find general solution of sin 2θ=cos θ [Hints Use sin 2θ= 2sin θcosθ and then take cosθ

common]

1lek le1)

Determine an angle alpha such that sin =k and -π2le αle π2

Then we have

Sin θ = k = sin α

Or sin θ - sin α =0

Or 2 cos [(θ+α) 2] sin [(θ-α) 2] =0

Therefore either cos [(θ +α) 2] =0 (1)

Or sin [(θ-α) 2] =0 (2)

Now from (1) we get (θ+α) 2= (2m+1) π2)

Or θ = (2m+1) π-α (3)

And from (2) we get (θ-α) 2 =mπ

Or θ= 2mπ+α(4)

Where m = any integer

Clearly the solution (3) amp (4) may be combined in the following form

θ= nπ+(-1) n α where n= any integer

Therefore the general solution of sin θ = sin α is θ = nπ +(-1) n α where n is any integer and -π2 le α le π2

Biology Chapter - 04Kingdom Monera

Today we will discuss about bacterial reproduction and its usefulness

Fig Binary Fission

Fig Conjugation Fig Transformation

Bacterial reproduction is mainly asexual but sexual reproduction

also takes place

Asexual reproduction takes place by i) Binary fission - from one bacteriato

two bacteria are produced in every 20 to 30mins

ii) Buddingiii) Endospore formation - during

unfourable condition

Sexual reproduction by three ways

1) Conjugation - Transfer of genetic material between cells that are in physical contact with one another

2) Transduction - Transfer of genetic materialfrom one cell to another by a bacteriophage

3) Transformation - Transfer of cell-freeor naked DNArsquo from one cell to another

Bacteria causes different diseases inplants animals and human and

it causes food spoilage and waterpollution but it also have some useful

activities

i) Bacteria are helpful in sewage water treatment

ii) It is used in antibiotic (medicine) production

iii) Anaerobic bacteria help in biogas(energy) production

iv) Many household products like yoghurt cheese are manufactured by use of bacteria

v) Rhizobium by symbiotic relationship with leguminous plant increase soil fertility

vi) Besides these bacteria is helpful in genetic engineering degradation of petroleum hydrocarbonand in dairy

industry

Physics Motion in plane Here we will introduce Projectile Motion

Execution

Projectile

Y

usinθ u h

θX

ucosθ

Suppose a body is projected with an angle θ So initial velocity u can be resolved into two components

Horizontal component - ucosθ ( for range)

Vertical component - usinθ ( for height)

usinθ changes during motion and becomes zero at maximum height position but ucosθ remain unchanged

The maximum height of projectile is h

NB If initial is upward then g = -ve and if it is downward then g = +ve Height is +ve if direction of motion does not change ( for ex a body thrown upwards but goes down ultimately then height h = -ve)

The angle of projectile θ is the angle made with horizontal

HISTORY ndash GROWTH OF NATIONALISM

SUB TOPIC- REVOLUTIONARY NATIONALISM Bengal formation of Anushilan Samity and Jugantar Group

The intensification of the Swadeshi movement and Government policy of terror and repression led to outbreak of violence Bombs were manufactured and attempts on the lives of unpopular Government officials became frequent In the gymnasium of Scottish Church College which was known as General Assemblies Institution a secret society was formed known as Anushilan Samity

Aurobindo Ghosh send from Baroda his emissary Jatindranath Banerjee to mobilize the Bengal revolutionaries

Hemchandra Qanungo and Satyen Bose published Journal Jugantar

The Jugantar group planned to assassinate oppressive magistrate Kingsford by Khudiram Bose and Prafulla Chaki in 1908 Prafulla Chaki committed suicide to avoid arrest Khudiram was tried and hanged

Afew days later the police found a bomb factory in Maniktala and arrested a large number of revolutionaries The trial of revolutionaries became famous as the Alipore Bomb Case

In the course of the trial the approver the public prosecuter and a police officer were assassinated

1 Question Name two journals which preached the cult of violence

Answer a) Yugantar edited by Bhupendranath DuttaB) Bandemataram edited by Aurobindo Ghosh2 Question Why was Khudiram arrested and hangedAnswer An attempt was made to assassinate a hated vindictive majistrate named Kingsford by Khudiram Bose and Prafulla Chaki Their attempt failed and the bomb they threw killed two English ladies Khudiram was arrested and put to trial and then hanged3Question Who was Aurubindo GhoshAnswer Aurobindo Ghosh a nationalist revolutionary who was charged for his involvement in the Alipore Bombing Case He was accused of it along with his brother Barindra nath Ghosh But Aurobindo was acquitted because of the brilliant pleading of his counsel Chittaranjan Das Then he became a spiritual reformer introducing his visions on human progress and spiritual evolution4 Qustion Who was KingsfordAnswer Kingsford was an unpopular British chief Magistrate who was the target of the bomb thrown at Muzaffarpur by Khudiram and Prafulla Chaki

Most of the accused were convicted and sentenced to

long term of imprisonmentBut

Aurobindo Ghosh was acquitted mainly owing to the brilliant pleading of his counsel Chittaranjan Das

Political science Topic-Sovereignty

Summary Sovereignty is the full right and power of a governing body over itself without any interference from outside sources or bodies In political theory sovereignty is a substantive term designating supreme legitimate authority over some polity In international law sovereignty is the exercise of power by a state

Internal Sovereignty

Internal sovereignty means supreme authority within ones territory while external sovereignty relates to the recognition on the part of all states that each possesses this power in equal measure

External sovereignty

external sovereignty relates to the recognition on the part of all states that each possesses this power in equal measure

Distinguish between

Execution

Answer the following questions

Short notes-

Sovereignty

Internal Sovereignty

External sovereignty

Homework- learn

external sovereignty and internal sovereigntySovereignty is the principle

of supreme and

unquestionable authority

reflected in the claim by the

state to be the sole author of

laws within its territory

Definition of external vs internal sovereigntyInternal sovereignty refers to

the relationship between a

sovereign power and its

subjects ndash it refers to the

location of the supreme

authority within the state In

the UK for example internal

sovereignty (supposedly)

resides within Parliament

reflected in the

constitutional principle of

parliamentary

sovereigntyBy contrast

external sovereignty refers

to the capacity of the state

to act independently and

autonomously on the world

stage This is what is

sometimes called lsquostate

sovereigntyrsquo or lsquonational

sovereigntyrsquo and implies

that states are legally equal

and that the territorial

integrity and political

independence of a state is

inviolable

Class ndash XII

Date - 2742020 STUDY MATERIALSubject Topic Summary Execution Business Studies

Job Analysis amp Manpower Planning

At first let us recall the chapter what we have discussed till nowbullJob analysisbullJob specification bullJob description bullJob enlargement bullJob enrichment

Today we will do some questions answers from the chapter

Questions 1ldquoJob analysis job description and job specification are interrelatedrdquo Comment Answer) Job analysis is a systematic and detailed examination of a job to collect all the relevant information about it The contents off the job are summarised in the job description The qualification needed for the job are summarised in job specificationThus there is close interrelationship between job analysis job description and job specification

Question 2ldquoJob enlargement is a horizontal extension of a job whereas job enrichment is a vertical extension of a jobrdquo ElucidateAnswer) Job enlargement involves adding one or more task to a job coma where as job enrichment involves adding more autonomy and responsibility to a job Job enlargement is therefore horizontal extension of a job coma whereas job enrichment is a vertical extension of a job

Question 3 )

What is manpower estimation Explain its quantitative and qualitative aspectsAnswer) Manpower estimation is the process by which management determines how an organisation should move from itrsquos current manpower positionto its desired manpower position There are two dimensions of Manpower estimation- quantitative and qualitative

Quantitative aspectThis aspect of Manpower estimation involves estimating the number of employees required in a future time period Workload analysis and workforce analysis are done to estimate the quantity of required manpower

Qualitative aspectThe estimate of the knowledge skills experience etc of required manpower is the qualitative aspect of Manpower estimation The quality of Manpower can be judged on the basis of job analysisand job specification

COMMERCE

CAPITAL-FIXED AND WORKING

Today let us start the class by discussing the sources of finance for different types of business firms

The term lsquocapitalrsquo refers to the investment made in the enterprise for the purpose of earning profits

Requirements of capital and sources of capital for different types of business firms are

1 Capital for sole proprietorship businessA sole proprietor operates at a small scale and thereforerequires a limited amount of capital

2 Capital for partnership firmCapital requirements as well as capital base of a partnership is bigger than that of a sole trader businessThe owned capital is contributed by the partners in an agreed ratio

3 Capital for joint stock companyA joint stock company generally requires large amount of capitalA public company can raise huge capital through issue of shares In addition to share capital it can utilize retained profits

Now let u discuss the meaning of Finance PlanningFinance planning is the process of estimation the financial requirements of an organization specifying the sources of firms and ensuring that enough funds are available at the right time

1 What do you mean by Finance PlanningAnswer Finance planning is the process of estimation the financial requirements of an organization specifying the sources of firms and ensuring that enough funds are available at the right time

2Discuss the role of financial planning of an enterpriseThe role of financial planning are as followsa A sound financial plan helps a business enterprise to avaid the problems of shortage and surplus of fundsbFinancial planning serves as a guide in developing a sound capital structure so as to maximize returns to shareholders c It helps in effective utilization of fundsd It provides policies and procedures for coordinating different functional areas or departments of businesse It enables the management to exercise effective control over the financial activities of an enterprisef It helps the company to prepare for facing business shocks and surprises in future

Mathematics

Continuity and differentiability

Recall Definition of ContinuityLet f(x) be a single valued function of x and x=a be a point in the domain of definition of the function The function is said to be continuous at x=a ifi) f(c) is defined ie f(x) has a definite finite value at x=cii) lim xrarra f(x) exists andiii) lim xrarra f(x) =f(a) In other words f(x) is said to be continuous at x=a if lim xrarra+ f(x)= lim xrarra- f(x) = f(a) Or f(a+0) =f(a-0) =f(a) Or lim hrarr0 f(a+h)= f(a) Algebra of continuous functionsNow we will study some algebra of continuous functions Theorem 1 Suppose f and g be two real functions continuous at a real number c Then(1) f + g is continuous at x = c(2) f ndash g is continuous at x = c(3) f g is continuous at x = c(4) (fg) is continuous at x = c (provided g (c) ne 0)

Example 1 Prove that every rational function is continuousSolution Recall that every rational function f is given byf(x)=[p(x) q(x) ] q(x)ne0where p and q are polynomial functions The domain of f is all real numbers except points at which q is zero Since polynomial functions are continuous f is continuous by (4) of Theorem 1Example 2Discuss the continuity of sine functionSolution To see this we use the following factslim xrarr0 sin x =0Now observe that f (x) = sin x is defined for every real number Let c be a real number Put x = c + h If x rarr c we know that h rarr 0 Therefore lim xrarrc f(x)

= lim xrarrc sin x= lim hrarr0 sin(c+h) =lim hrarr0 [sin c cos h + cos c sin h ]=lim hrarr0 (sin c cos h) + lim hrarr0 (cos c sin h) = sin c +0=sin c = f(c) Thus lim xrarrc f(x) = f(c) and hence f is a constant function Exercise Prove that the function f(x) = x2 +2x is continuous for every real value of x [Hints show that lim xrarra+ f(x) = lim xrarra- f(x) = f(a) ]

Biology Reproduction in Flowering plants We will discuss about megasporoangium

megasporagenesis and female gametophyte

Q4 Describe the structure of megasporangium

Ovule is attached to the placenta by astalk called funicle

Each ovule has one two or three protectivecoverings called integuments

At the tip of integuments a small openingcalled micropyle is organised

Opposite to the micropylar end is the chalaza

Within the integuments a mass of cellsnucellusand inside it embryo sac orfemale gametophyte is present

Q5 Describe a mature embryosacamp its formation

In most of the flowering plants only oneof the 4 megaspores formed as a result ofmegasporogenesis that is functional while theother three degenerate

The

functional megaspore develops into thefemale gametophyte

Formation The nucleus

of the functional megasporedivides mitotically to form two nuclei first andthen two more sequential mitotic nucleardivisions result in the formation of four ampthen eight nucleate stages of embryo sac

Six of the eight nucleus are surrounded bycell walls and organised into cells

The remaining two nuclei called polar nuclei are found below the egg apparatus in the largecentral cell

Three cells consisting of two synergids amp one egg cell present bottom of

embryo sac Three cells

at the chalazal as antipodal cells

Two polar nuclei together present in large central cell

HISTORY

TOWARDS INDEPENDENCE AND PARTITION THE LAST PHASE(1935-1947)SUB TOPIC NATIONAL MOVEMENTS DURING THE SECOND WORLD WAR

Spread of Quit India Movement On 9th August 1942Gandhiji and other Congress leaders were arrested The Congress was declared illegal The news of the arrest of all leaders marked the beginning of a widespread movement of India It was not possible for such a movement to remain peacefulBut the arrest of the all notable congress leaders virtually left the movement in the hands of the mass The movement took the form of violent and militant outbreakBesides congressmen revolutionaries also were very active in the movement The Congress Socialist group also played a prominent role

1 Question Why did the British authority arrest the Congress leaders on 9 th August 1942Answer Congress Working committee adopted the Quit India resolution which was to be ratified at the Bombay AICC meeting in 8th August 1942 They decided to launch a mass struggle on non-violent lines Gandhiji gave a clarion call to all section of the people rdquoKarenge ya Marengerdquo (do or die) Congress leaders gave the call to driving out

the British from IndiaViceroy had taken strong action against the Quit India movement Gandhiji and all the leaders of Congress were arrested

2 Question How did Quit India Movement spread out all over IndiaAnswer The news of the leaders lsquo arrest marked the beginning of a widespread movement to remain peacefulThe movement took form of violent outbreak There were widespread cutting of telephone and Telegraph wires damaging railway lines raising barricades in cities and towns and other forms of violent demonstations

Question Name the leaders of Congress

Socialist group played a prominent part Notable among the Jayprakash Narayan Rammonohar Lohia Aruna Asaf Ali

Political science

Topic-Franchise and Representation

Summary

The election commission

The Election Commission of India is an autonomous constitutional authority responsible for administering Union and State election processes in India The body administers elections to the Lok Sabha Rajya Sabha State Legislative Assemblies in India and the offices of the President and Vice President in the country

Functions of election commission-

India is a sovereign socialist secular democratic republic Democracy runs like a golden thread in the social economic and political fabric woven by the Constitution given by lsquoWe the People of Indiarsquo unto ourselves The concept of democracy as visualised by the Constitution pre-supposes the representation of the people in Parliament and State legislatures by the method of election The Supreme Court has held that democracy is one of the inalienable basic features of the Constitution of India and forms part of its basic structure The Constitution of India adopted a Parliamentary form of government Parliament consists of the President of India and the two Houses mdash Rajya Sabha and Lok Sabha India being a Union of states has separate state legislatures for each state State legislatures consist of the Governor and two Houses mdash Legislative Council and Legislative Assembly mdash in seven states namely Andhra Pradesh Telangana Bihar Jammu amp Kashmir Karnataka Maharashtra and Uttar Pradesh and of the Governor and the state Legislative Assembly in the remaining 22 states Apart from the above two out of the seven Union Territories namely National Capital Territory of Delhi and Puducherry also have their Legislative Assemblies

ExecutionShort notes-Election commissionFunctions of election commission

Homework- Learn

Computer

Science

Computer hardware NAND Gate

A NOT-AND operation is known as NAND operation It has n input (n gt= 2) and one output

Logic diagram

Truth Table

NOR Gate

A NOT-OR operation is known as NOR operation It has n input (n gt= 2) and one output

Logic diagram

Truth Table

XOR Gate

XOR or Ex-OR gate is a special type of gate It can be used in the half

adder full adder and subtractor The exclusive-OR gate is abbreviated as EX-OR gate or sometime as X-OR gate It has n input (n gt= 2) and one output

Logic diagram

Truth Table

XNOR Gate

XNOR gate is a special type of gate It can be used in the half adder full adder and subtractor The exclusive-NOR gate is abbreviated as EX-NOR gate or sometime as X-NOR gate It has n input (n gt= 2) and one output

Logic diagram

Truth Table

Physics

Chapter 1 Electric Field ( Electric Dipole) (Summary)

Here we will derive Expression of electric field at broad side

On position of dipole

Execution

Q With the help of a labelled diagram obtain an expression for the electric field intensity E at any point on the equitorial line ( broad-side on position) of an electric dipole

Ans

E1 E1sinθ

E θ P E1 θ

( r2+L2)12 E2 E

r E2 E2sinθ

-q θ L O L +qA B

Let us consider that the point P is situated on the right bisector of the dipole AB at a distance r meter from its midpoint O

Let E1 and E2 be the electric field intensities of the electric field at P due to charge +q and ndashq of the dipole resp The distance of P from each charge is ( r2+L2)12

So E1 = 14 πϵ q

(r 2+L 2) away from +q

E2 = 14 πϵ q

(r 2+L 2) towards ndashq

The magnitudes of E1 and E2 are equal but directions are different Now resolving E1 and E2 into two components parallel and perpendicular to AB we get

The components perpendicular to AB E1sinθ and E2sinθ cancel each other because they are equal and opposite

The components parallel to AB are E1cosθ and E2 cosθ are in same direction and add up

So resultant intensity of electric field at the point P is

E = E1cosθ + E2 cosθ

E = 14 πϵ q

(r 2+L 2) 2 cosθ

Now from fig we have cosθ =BOBP = L (r2+L2)12

So we get E = 14 πϵ 2qL ( r2+L2)32

Now electric dipole moment p= 2qL

So E = 14 πϵ p ( r2+L2)32

HW Find the expression of Electric field as done here but this time take r gtgt 2L

Also find the expression of torque experience by a dipole

(Hint Electric force experienced by charges of dipole in electric field is qE each Let θ be the angle which dipole makes with electric lines of force then perpendicular distance between two charges is 2Lsinθ Then torque = force x perp distance = qE x 2L sinθ So τ=pE sinθ where p =2qL )

STUDY MATERIAL

Class XIISubject Eng Literature (The Tempest ndash William Shakespeare) Topic Act IV Scene 1 Lines 84 to 133 (Iris hellip A contract of true love Be not too late ) Date 27th April 2020 (4th Period)

[Students should read the original play and also the paraphrase given in the school prescribed textbook]Summary Questions amp Answers

o Ceres soon appears and comes to know that she has been summoned to celebrate the contract of true love

o Ceres expresses her unwillingness to meet Venus and Cupid as she has shunned their company

o Ceres and Juno both bestow their blessings upon Ferdinand and Miranda with June gifting honour riches happiness in marriage and Ceres presents plenty of earthrsquos produce

o Iris summons the water-nymphs and reapers to come and celebrate a contract

(1) IRIS Of her society (Line 91-101)

Be not afraid I met her deity

Cutting the clouds towards Pathos and her sonDove-drawn with her Here thought they to have doneSome wanton charm upon this man and maidWhose vows are that no bed-right shall be paidTill Hymens torch be lightedmdashbut in vainMarss hot minion is returned againHer waspish-headed son has broke his arrowsSwears he will shoot no more but play with sparrowsAnd be a boy right out

(i) Where were Venus and Cupid seen flying How were they travelling Why did they want to join the marriage celebration of Ferdinand and Miranda

of true love

Venus and Cupid were seen flying through the air towards Paphos the famous city which is situated on the island of Cyprus They were travelling by air-borne chariot drawn by doves They certainly wanted to come here in order to play some amorous trick upon Ferdinand and Miranda who are under a vow not to gratify their physical desires till the holy ceremony of their marriage has been performed(ii) What have Venus and Cupid done after failing in their plan

After being failure of their plan Venus who is a very passionate deity and who is the mistress of Mars (the god of war) has gone back while here ill-tempered son Cupid has broken his arrows of love in his state of desperation(iii) What has Cupid firmly decided

Cupid is feeling so disappointed that he has firmly decided to shoot no more arrows to arouse love in human hearts but to spend his time playing with sparrows Thus he would now become just a boy and would give up his original function of shooting arrows on human beings to make them fall in love(iv) What vow had Ceres taken How did Ceres feel at the abduction

After the abduction of her daughter Prosperina by Pluto Ceres had taken a vow to always keep away from the disgraceful company of Venus and her blind son Cupid the god of love Ceres felt deeply distressed when Pluto had carried off her daughter and had made her his wife by force(v) Why has Ceres not forgiven Venus and her blind son For what do Ceres want to be sure

As the abduction had been manipulated by Venus the goddess of beauty and love and her blind son Cupid Ceres has never forgiven them for their part in the whole plot Ceres wants to be sure that she would not have to meet Venus and Cupid who had engineered the abduction of her daughter Prosperina

AS THIS lsquoMASQUErsquo SCENE IS VERY IMPORTANT IN THE PLAY THE PARAPHRASE OF THE ENTIRE PORTION OF MASQUE SCENE (Act IV Lines 58 to 143) IS GIVEN BELOW

IRIS Goddess of RainbowCERES Goddess of Agriculture and all the fruits of the earth

(Nature growth prosperity rebirth ndash notions intimately connected to marriage)JUNO The majestic Queen of Heavens and wife of Jupiter (Jupiter is the king of Gods)

VENUS The Goddess of love CUPID Son of Venus PLUTO God of death (In the play referred by Shakespeare as lsquoDisrsquo which is a Roman name for Pluto)

ORIGINAL TEXT PARAPHRASEPROSPEROWellmdash

PROSPERONow come Ariel Let there be too many rather than too few

Now come my Ariel Bring a corollaryRather than want a spirit Appear and pertly[to Ferdinand and Miranda]No tongue all eyes Be silent

spirits in attendance Appear briskly

[to Ferdinand and Miranda]Look with your eyes but do not say a word

[Soft music] [Soft music][Enter Iris] [Enter Iris]

IRISCeres most bounteous lady thy rich leasOf wheat rye barley vetches oats and peasThy turfy mountains where live nibbling sheepAnd flat meads thatched with stover them to keepThy banks with pioned and twilled brimsWhich spongy April at thy hest betrimsTo make cold nymphs chaste crowns and thybroom-grovesWhose shadow the dismissegraved bachelor lovesBeing lass-lorn thy pole clipped vineyardAnd thy sea-marge sterile and rocky-hardWhere thou thyself dost airmdashthe Queen othrsquoSkyWhose watery arch and messenger am IBids thee leave these and with her sovereign grace[Juno appears] Here on this grass-plot in this very placeTo come and sport Her peacocks fly amainApproach rich Ceres her to entertain

IRISCeres most generous lady you are the cause of rich fields or fertile land where wheat rye barley beans oats and peas grow the grassy mountains where the sheep graze and the flat meadows covered with coarse hay to be used as fodder for cattleYour banks are covered with marsh-marigolds and reeds and the rainy April under your orders brings forth to make for the maids who are not in love beautiful crowns your woods where the broom flourishes and where the bachelor who has been dismissed by the maid he loved lies down being forsaken your vineyard in which the poles are embraced by the vines and the margin of the sea which is barren and rocky where you roam about to enjoy the fresh air ndash the queen of the sky (Juno) whose messenger I am besides being represented as the rainbow bids you leave all these and with her majesty here on this grassy plot in this very place come and sport her peacocks carry her fast in her chariot through the air and are making their way here approach rich Ceres to welcome her

[Enter Ariel as Ceres] [Enter Ariel as Ceres]

CERESHail many-coloured messenger that neerDost disobey the wife of JupiterWho with thy saffron wings upon my flowersDiffusest honey-drops refreshing showersAnd with each end of thy blue bow dost crownMy bosky acres and my unshrubbed downRich scarf to my proud earth Why hath thy queenSummoned me hither to this short-grassed green

CERESWelcome rainbow that never dared disobey Juno the wife of Jupiter who with your orange coloured rays spread honey-drops refreshing showers And with each end of thy blue bow drown my bushy acres and my hilly country which is free from shrubs you thus forming a rich scarf Why has your queen called me here to this place covered with short grass

IRISA contract of true love to celebrateAnd some donation freely to estateOn the blest lovers

IRISI have called you to celebrate a contract of true love and bestow some liberal gift upon the blessed lovers

ORIGINAL TEXT PARAPHRASECERESTell me heavenly bowIf Venus or her son as thou dost knowDo now attend the queen Since they did plotThe means that dusky Dis my daughter gotHer and her blind boys scandaled companyI have forsworn

CERESTell me heavenly bow if Venus the Goddess of love or Cupid her son and pedlar of passion at this time attend the heavenly queen Juno because you are sure to know Since the day they conspired against me and dark Pluto took away my daughter here and Cupidrsquos disgraceful company I have left off

IRISOf her societyBe not afraid I met her deityCutting the clouds towards Pathos and her sonDove-drawn with her Here thought they to have doneSome wanton charm upon this man and miad

IRISBe not afraid of her company I met her deity moving on the clouds towards Paphos the sacred home of Venus on the island of Cyprus along with her son on her chariot drawn by doves Here they contemplated to exercise a charm upon this man and maid producing

Whose vows are that no bed-right shall be paidTill Hymens torch be lightedmdashbut in vainMarss hot minion is returned againHer waspish-headed son has broke his arrowsSwears he will shoot no more but play with sparrowsAnd be a boy right out

wantonness before the actual marriage ceremony but did not succeed Venus has returned her irritable son has broken his arrows and swears that he will give up his practice of trying to inspire love but play with sparrows and be a boy again

[Music is heard] [Music is heard]

CERESHighst queen of stateGreat Juno comes I know her by her gait

CERESHighest queen of state Great Juno there she comes I know here by her gait

[Enter Juno] [Enter Juno]

JUNOHow does my bounteous sister Go with meTo bless this twain that they may Prosperous beAnd honoured in their issue

JUNOHow are you doing my generous sister Come with me to bless this couple so that they may be prosperous and fortunate in their children

[They sing] [They sing]

JUNOHonour riches marriage-blessingLong continuance and increasingHourly joys be still upon youJuno sings her blessings upon you

JUNOMay honour riches happiness in marriage long continuance and increase of those boons ever rest upon you as hourly joys Juno showers down upon you her blessings in song

CERESEarths increase foison plentyBarns and garners never emptyVines and clustring bunches growingPlants and goodly burden bowingSpring come to you at the farthestIn the very end of harvestScarcity and want shall shun youCeresrsquo blessing so is on you

CERESMay you have the plenty of earthrsquos produce Your barns and granaries may never be empty Your vines may grow with clustering bunches Your fruit trees may be heavily laden with their fruit May there be continuous spring and harvest May scantiness and want leave you forever Such is the blessing of Ceres upon you

FERDINANDThis is a most majestic vision andHarmoniously charmingly May I be boldTo think these spirits

FERDINANDThis is a great vision and magically melodious Should I suppose the characters (taking part in the masque) are spirits

PROSPEROSpirits which by mine artI have from their confines calld to enactMy present fancies

PROSPEROYes they are spirits whom I have summoned from the regions to which they are confined to carry into effect my fanciful designs

ORIGINAL TEXT PARAPHRASEFERDINANDLet me live here everSo rare a wondered father and a wifeMakes this place paradise

FERDINANDI should like to live here forever Such a wise and wonderful father makes this place a paradise

[Juno and Ceres whisper and send Iris on employment] [Juno and Ceres whisper and send Iris on employment]

PROSPEROSweet now silence

PROSPEROMy dear Ferdinand speak no more Juno and Ceres are

Juno and Ceres whisper seriouslyTheres something else to do Hush and be muteOr else our spell is marred

whispering with a solemn look There is something else coming Silence Or else our magic will be spoilt

IRISYour nymphs called naiads of the wandering brooksWith your sedged crowns and over-harmless looksLeave your crisp channels and on this green landAnswer your summons Juno does commandCome temperate nymphs and help to celebrateA contract of true love Be not too late

IRISYou nymphs called Naiads denizens (M inhabitants) of the running stream with your chaplets of sedge and ever-helpful looks leave your wrinkled channels and on the green land answer the summons sent to you Juno has ordered some chaste nymphs and help to celebrate a noble and true marriage Donrsquot delay

[Enter certain nymphs] [Enter certain nymphs]You sunburnt sicklemen of August wearyCome hither from the furrow and be merryMake holiday your rye-straw hats put onAnd these fresh nymphs encounter every oneIn country footing

You sunburnt harvesters weary from the effects of the heat in August come here from the furrowed land and rejoice Make holiday with your rye-straw hats upon you and meet these fresh nymphs and join in country dancing

[Enter certain reapers properly habited They join with the nymphs in a graceful dance towards the end whereof Prospero starts suddenly and speaks]

[Enter certain reapers properly habited They join with the nymphs in a graceful dance towards the end whereof Prospero starts suddenly and speaks]

PROSPERO[aside] I had forgot that foul conspiracyOf the beast Caliban and his confederatesAgainst my life The minute of their plotIs almost come [to the spirits]Well done Avoidno more

PROSPERO(Aside)I had forgotten the wicked conspiracy of the beast Caliban and his accomplices against my life the time of their plot has almost arrived ndash (To the Spirits) well done depart no more of this

[To a strange hollow and confused noise the spirits heavily vanish]

[The spirits depart]

ORIGINAL TEXT PARAPHRASEFERDINANDLet me live here everSo rare a wondered father and a wifeMakes this place paradise

FERDINANDI should like to live here forever Such a wise and wonderful father makes this place a paradise

[Juno and Ceres whisper and send Iris on employment] [Juno and Ceres whisper and send Iris on employment]

PROSPEROSweet now silenceJuno and Ceres whisper seriouslyTheres something else to do Hush and be muteOr else our spell is marred

PROSPEROMy dear Ferdinand speak no more Juno and Ceres are whispering with a solemn look There is something else coming Silence Or else our magic will be spoilt

IRISYour nymphs called naiads of the wandering brooksWith your sedged crowns and over-harmless looksLeave your crisp channels and on this green landAnswer your summons Juno does commandCome temperate nymphs and help to celebrateA contract of true love Be not too late

IRISYou nymphs called Naiads denizens (M inhabitants) of the running stream with your chaplets of sedge and ever-helpful looks leave your wrinkled channels and on the green land answer the summons sent to you Juno has ordered some chaste nymphs and help to celebrate a noble and true marriage Donrsquot delay

[Enter certain nymphs] [Enter certain nymphs]You sunburnt sicklemen of August wearyCome hither from the furrow and be merryMake holiday your rye-straw hats put onAnd these fresh nymphs encounter every oneIn country footing

You sunburnt harvesters weary from the effects of the heat in August come here from the furrowed land and rejoice Make holiday with your rye-straw hats upon you and meet these fresh nymphs and join in country dancing

[Enter certain reapers properly habited They join with the nymphs in a graceful dance towards the end whereof Prospero starts suddenly and speaks]

[Enter certain reapers properly habited They join with the nymphs in a graceful dance towards the end whereof Prospero starts suddenly and speaks]

PROSPERO[aside] I had forgot that foul conspiracyOf the beast Caliban and his confederatesAgainst my life The minute of their plotIs almost come [to the spirits]Well done Avoidno more

PROSPERO(Aside)I had forgotten the wicked conspiracy of the beast Caliban and his accomplices against my life the time of their plot has almost arrived ndash (To the Spirits) well done depart no more of this

[To a strange hollow and confused noise the spirits heavily vanish]

[The spirits depart]

Ac-12 27420 topic Revaluation of Assets and Liabilities

REVALUATION OF ASSETS AND LIABILITIES

On admission of a new partner the firm stands reconstituted and consequently the assets are revalued and liabilities are reassessed It is necessary to show the true position of the firm at the time of admission of a new partner If the values of the assets are raised gain will increase the capital of the existing partners Similarly any decrease in the value of assets ie loss will decrease the capital of the existing partners For this purpose alsquoRevaluation Accountrsquo is prepared This account is credited with all increases in the value of assets and decrease in the value of liabilities It is debited with decrease on account of value of assets and increase in the value of liabilities The balance of this account shows a gain or loss on revaluation which is transferred to the existing partnerrsquos capital account in existing profit sharing ratioAccounting for Revaluation of Assets and Liabilities when there is a Changein the Profit Sharing Ratio of Existing PartnersAssets and liabilities of a firm must also be revalued at the time of change in profit sharing ratio of existing partners The reason is that the realisable or actual value of assets and liabilities may be different from those shown in the Balance Sheet It is possible that with the passage of time some of the assets might have appreciated in value while the value of certain other assets might have decreased and no record has been made of such changes in the books of accounts Similarly there may be some unrecorded assets amp libilities that may have to be accounted for Revaluation of assets and reassessments of liabilities becomes necessary because the change in the

value of assets and liabilities belongs to the period to change in profit sharing ratio and hence must be shared by the partners in their old profit sharing ratio Revaluation of assets and reassessment of liabilities may be given effect to in two different ways (a) When revised values are to be recorded in the books and(b) When revised values are not to be recorded in the books

When revised values are to be recorded in the booksIn such a case revaluation of assets and reassessment of liabilities is done with the help of a new account called lsquoRevaluation Accountrsquo Sometimes this account is also called as lsquoProfit amp Loss Adjustment Acrsquo If there is a loss due to revaluation revaluation account is debited and if the revaluation results in a profit the revaluation account is credited The following journal entries made for this purpose are

(i) For increase in the value of assetsAsset Ac Dr (individually)To Revaluation Ac(ii) For decrease in the value of AssetRevaluation Ac Dr (individually)To Asset Ac[Decrease in the value of assets](iii) For increase in the value of LiabilitiesRevaluation Ac Dr (individually)To Liabilities Ac[Increase in the value of Liabilities](iv) For decrease in the value of LiabilitiesLiabilities Ac DrTo Revaluation Ac[Decrease in the value of Liabilities](v) For unrecorded AssetsAsset Ac [unrecorded] DrTo Revaluation Ac[Unrecorded asset recorded at actual value](vi) For unrecorded Liability Revaluation Ac DrTo Liability Ac [unrecorded][Unrecorded Liability recorded at actual value](vii) For transfer of gain on revaluationRevaluation Ac DrTo Existing Partnerrsquos CapitalCurrent Ac[Profit on revaluation transferred to capital account in existing ratio](viii) For transfer of loss on revaluationExisting Partnerrsquos CapitalCurrent Ac DrTo Revaluation Ac[Loss on revaluation transferred to capital account in existing ratio](a) When revaluation account shows gain Revaluation Ac DrTo Partnerrsquos Capital Ac (Old Profit Sharing Ratio)(Profit on revaluation credited to Partnerrsquos Capital Ac)(b) Above entry is reversed when revaluation account shows loss Partners Capital Acs (Old Profit Sharing Ratio) DrTo Revaluation Ac(Loss on revaluation debited to Partnerrsquos Capital Acs)

Proforma of Revaluation Account is given as under

Revaluation Account

Dr Cr Particulars ` Amount Particulars ` Amount To Decrease in value of assets By Increase in value of assets To Increase in value of liabilities By Decrease in value of liabilities To Unrecorded liabilities By Unrecorded assets To Gain on Revaluation (Transferred) By Loss on Revalution (Transferred)

ECO ndash12 2742020Topic- ELASTICITY OF DEMAND

CHAPTER - ELASTICITY OF DEMANDMEANINGDemand for a commodity is affected by many factors such as its price price of related goods income of its buyer tastes and preferences etc Elasticity means degree of response Elasticity of demand means degree of responsiveness of demand Demand for a commodity responds to change in price price of related goods income etc So we have three dimensions of elasticity of demandDIMENSION OF ELASTICITY OF DEMAND TYPES OF ELASTICITY OF DEMAND

Price elasticity of demand Income elasticity of demand Cross Elasticity of demand

Price elasticity of demand Price elasticity of demand means degree of responsiveness of demand for a commodity to the change in its price For example if demand for a commodity rises by 10 due to 5 fall in its price Price elasticity of demand (ep)=Percentage change in quantity demanded Percentage change in price of the commodity = 10 ( -)5 = ( - )2Note that ep will always be negative due to inverse relationship of price and quantity demanded

(ii) Income elasticity of demand Income elasticity of demand refers to the degree of responsiveness of demand for a commodity to the change in income of its buyer Suppose income of buyer rises by 10 and his demand for a commodity rises by 20 then Income elasticity of demand (ey)= change in quantity demanded change in price of the commodity =20 10 = 2

Cross Elasticity of demandCross elasticity of demand means the degree of responsiveness of demand for a commodity to the change in price of its related goods (substitute goods or complementary goods) Suppose demand for a commodity rises by 10 due to 5 rise in price of its substitute good then Cross elasticity of demand (ec) = change in quantity demanded change in price of related good = 10 2 = 5 (Tastes and preferences cannot be expressed numerically So elasticity ofdemand cannot be numerically expressed)

  • Chapter 1 Force (Summary)
  • Distinguish between external sovereignty and internal sovereignty
    • NAND Gate
      • Logic diagram
      • Truth Table
        • NOR Gate
          • Logic diagram
          • Truth Table
            • XOR Gate
              • Logic diagram
              • Truth Table
                • XNOR Gate
                  • Logic diagram
                  • Truth Table
                      • Physics
                      • Chapter 1 Electric Field ( Electric Dipole) (Summary)
Page 28:  · Web viewWe all know that Nouns are divided into two parts: common noun and proper noun.Apart from common and proper noun, we will also study about collective noun and compound

Reward Profit which is uncertain and irregularcan be loss

Fixed salary and fringe benefits

Need for security low high

3 What do you understand by enterprise

Answer Enterprise means an undertaking or adventure that requires some innovation and investment and thus involves riskEnterprise always entails decision making coordination and risk bearing

COMMERCE NATURE AND OBJECTIVES OF

BUSINESS

Today let us recall the last other two objectives of business by the chart given in the previous class

Firstly we would discuss Human Objectives

Business is run by people and for people Labour is a valuable business element

Human objectives of business are concerned with the well -being of labour

The human objectives are as follows

Labour welfare Developing human

resources Participative

management Labour

management cooperation

Questions

1 Explain the human objectives of a business enterprise

Answer

Business is run by people and for people Labour is a valuable business element

Human objectives of business are concerned with the well -being of labour

The human objectives are as follows

Labour welfare-Business must recognize the dignity of labour and human factors should be given the recognition

Developing human resources-Employees must be provided the opportunities for developing new skills and attitudes

Participative management-Employees should be allowed to take part in decision making process of business

Labour management cooperation-Business should strive for creating and maintaining cordial employer employee relations so as to ensure peace and progress in industry

Now let us discuss the national objectives of business

Optimum utilization of resources

National self- reliance Development of small

scale industries Development of

backward areas Control over pollution

2Explain the national objectives of a business enterprise

Answer

It is the duty of business to utilize the resources of the country properly the national objectives of business

Optimum utilization of resources ndashBusiness should use the nationrsquos resources in the best possible manner

National self- reliance-It is the duty of the business to help the government in increasing experts and in reducing dependence on imports

Development of small scale industries-Big business firms are expected to encourage growth of small scale industries which are necessary for generating employment

Development of backward areas-Business is expected to give preference to the industrialization of backward regions of the country

ECONOMICS

BASIC ECONOMIC CONCEPTS

SUB

TOPIC

Value

Wealth

Welfare

Today we shall start with a new topic of the same chapter ie lsquoValuersquo

Value of a commodity is defined as the valuation placed by a household on the consumption of this commodity

lsquoValuersquo has two different meanings and these are

a Value -in -use It refers to consumption value of a commodity It expresses the utility derived from the consumption of a particular commodity A necessity like water has a very high value ndashin ndashuse or

Question

1What is value

Answer

Value of a commodity is defined as the valuation placed by a household on the consumption of this commodity

2What is value-in use

Answer It refers to consumption value of a commodity It expresses the utility derived from the consumption of a particular commodity A necessity like water has a very high value ndashin ndashuse or consumption value

3What is value ndashin- exchange

Answer It relates to market value of a commodity

It is the rate at which a particular good or service can be exchanged for money

For example in barter system if a person is prepared to exchange 3 metres of cloth with 1 pair

consumption value

b Value ndashin-exchange It relates to market value of a commodity

It is the rate at which a particular good or service can be exchanged for moneyFor example in barter system if a person is prepared to exchange 3 metres of cloth with 1 pair of shoes then the value in exchange of 3 metres of cloth is 1 pair of shoesValue in exchange is the power of purchasing other goods In modern monetised economies the exchange value of goods are expressed in terms of money as prices

Now let us discuss the term lsquoWealthrsquo

Wealth refers to the stock of all those assets which are a source of income

Wealth is a stock concept

Wealth must possess the following features

a Utility It must possess utility or give some

of shoes then the value in exchange of 3 metres of cloth is 1 pair of shoes

Value in exchange is the power of purchasing other goods In modern monetised economies the exchange value of goods are expressed in terms of money as prices

4 What is wealth

Answer it refers to the stock of assets or goods which are a source of income and have personal or national ownership

5 What are the features of wealth

Answer The features of wealth are as follows

Wealth must possess the following features

a Utility It must possess utility or give some satisfaction

b Scarcity It must be limited in quantityc Transferability it should be transferable its

ownership can be transferred from one person to another person

d Exchange value It must possess exchange value

6 What is welfare

Answer

Welfare is defined as satisfaction and happiness a sense of well- being among the people

satisfactionb Scarcity It must be

limited in quantityc Transferability It

should be transferable its ownership can be transferred from one person to another person

d Exchange value It must possess exchange value

Now let us discuss the term lsquoWelfarersquo

Welfare is defined as satisfaction and happiness a sense of well- being among the people

Welfare is affected by factors like

a Consumption of goods and services

b Environment

c Family relations

d Degree of freedom

e Law and order situation

Mathematics Trigonometric equation

To find the general solution of the equation sinθ=0

When sin θ =0

Then θ= 0 π2π 3π-π -2π -3

i e when θ = 0 or an integral multiple of π

i e when θ= nπ where n is any integer

Therefore the general solution of the equation sin

Example1 Find the general values of θ which satisfy the equation sin2 θ =34

Solution sin2 θ= 34

Or sin θ = +34 or -34

Or sin θ = sin π3 or sin (-π3)

Therefore

θ = [nπ + (-1) n (π3)] or[ nπ+ (-1) n (-π3)]

= nπ +π3 or nπ-π3 where n= any integer

Example 2Find the values of θ which satisfy tan2 θ

θ=0 is θ= nπ where n is any integer

To find the general solution of the equation cos θ=0

When cos θ=0

Then θ=π2 3π2 5π2 -π2 -3π2 -5π3

i e when θ is an odd multiple of π2

i e when θ=(2n+1) π2 where n= any integer

Therefore the general solution of the equation cos θ =0 is θ= (2n+1) π2 where n= any integer

To find the general solution of the equation tan θ = 0

Clearly tan θ =0 implies sin θcos θ =0

Therefore θ = nπ

i e the general solution of the equation tan θ=0 is θ =nπ where n = any integer

To find the general solution of the equation cot θ =0

Clearly cot θ =0 implies (cos θsin θ) = 0

i e cos θ =0

Therefore θ = (2n+1) π2

Therefore the general solution of the equation cot θ =0 is θ = (2n+1) π2

Where n= any integer

To find the general solution of the equation sin θ= k (-

=13 -πleθleπ

Solution tan2 θ =13

Or tan θ = plusmn1radic(3) =tan(plusmnπ6)

θ=nπ plusmn π6 where n =any integer

If n=0 then θ=plusmnπ6

If n=1 then θ= π plusmn π6

If n=-1 then θ= -π plusmn π6

Therefore the required solution in -π le θ le π are θ= π6 5π6 -π6 -5π6

Exercise Find general solution of sin 2θ=cos θ [Hints Use sin 2θ= 2sin θcosθ and then take cosθ

common]

1lek le1)

Determine an angle alpha such that sin =k and -π2le αle π2

Then we have

Sin θ = k = sin α

Or sin θ - sin α =0

Or 2 cos [(θ+α) 2] sin [(θ-α) 2] =0

Therefore either cos [(θ +α) 2] =0 (1)

Or sin [(θ-α) 2] =0 (2)

Now from (1) we get (θ+α) 2= (2m+1) π2)

Or θ = (2m+1) π-α (3)

And from (2) we get (θ-α) 2 =mπ

Or θ= 2mπ+α(4)

Where m = any integer

Clearly the solution (3) amp (4) may be combined in the following form

θ= nπ+(-1) n α where n= any integer

Therefore the general solution of sin θ = sin α is θ = nπ +(-1) n α where n is any integer and -π2 le α le π2

Biology Chapter - 04Kingdom Monera

Today we will discuss about bacterial reproduction and its usefulness

Fig Binary Fission

Fig Conjugation Fig Transformation

Bacterial reproduction is mainly asexual but sexual reproduction

also takes place

Asexual reproduction takes place by i) Binary fission - from one bacteriato

two bacteria are produced in every 20 to 30mins

ii) Buddingiii) Endospore formation - during

unfourable condition

Sexual reproduction by three ways

1) Conjugation - Transfer of genetic material between cells that are in physical contact with one another

2) Transduction - Transfer of genetic materialfrom one cell to another by a bacteriophage

3) Transformation - Transfer of cell-freeor naked DNArsquo from one cell to another

Bacteria causes different diseases inplants animals and human and

it causes food spoilage and waterpollution but it also have some useful

activities

i) Bacteria are helpful in sewage water treatment

ii) It is used in antibiotic (medicine) production

iii) Anaerobic bacteria help in biogas(energy) production

iv) Many household products like yoghurt cheese are manufactured by use of bacteria

v) Rhizobium by symbiotic relationship with leguminous plant increase soil fertility

vi) Besides these bacteria is helpful in genetic engineering degradation of petroleum hydrocarbonand in dairy

industry

Physics Motion in plane Here we will introduce Projectile Motion

Execution

Projectile

Y

usinθ u h

θX

ucosθ

Suppose a body is projected with an angle θ So initial velocity u can be resolved into two components

Horizontal component - ucosθ ( for range)

Vertical component - usinθ ( for height)

usinθ changes during motion and becomes zero at maximum height position but ucosθ remain unchanged

The maximum height of projectile is h

NB If initial is upward then g = -ve and if it is downward then g = +ve Height is +ve if direction of motion does not change ( for ex a body thrown upwards but goes down ultimately then height h = -ve)

The angle of projectile θ is the angle made with horizontal

HISTORY ndash GROWTH OF NATIONALISM

SUB TOPIC- REVOLUTIONARY NATIONALISM Bengal formation of Anushilan Samity and Jugantar Group

The intensification of the Swadeshi movement and Government policy of terror and repression led to outbreak of violence Bombs were manufactured and attempts on the lives of unpopular Government officials became frequent In the gymnasium of Scottish Church College which was known as General Assemblies Institution a secret society was formed known as Anushilan Samity

Aurobindo Ghosh send from Baroda his emissary Jatindranath Banerjee to mobilize the Bengal revolutionaries

Hemchandra Qanungo and Satyen Bose published Journal Jugantar

The Jugantar group planned to assassinate oppressive magistrate Kingsford by Khudiram Bose and Prafulla Chaki in 1908 Prafulla Chaki committed suicide to avoid arrest Khudiram was tried and hanged

Afew days later the police found a bomb factory in Maniktala and arrested a large number of revolutionaries The trial of revolutionaries became famous as the Alipore Bomb Case

In the course of the trial the approver the public prosecuter and a police officer were assassinated

1 Question Name two journals which preached the cult of violence

Answer a) Yugantar edited by Bhupendranath DuttaB) Bandemataram edited by Aurobindo Ghosh2 Question Why was Khudiram arrested and hangedAnswer An attempt was made to assassinate a hated vindictive majistrate named Kingsford by Khudiram Bose and Prafulla Chaki Their attempt failed and the bomb they threw killed two English ladies Khudiram was arrested and put to trial and then hanged3Question Who was Aurubindo GhoshAnswer Aurobindo Ghosh a nationalist revolutionary who was charged for his involvement in the Alipore Bombing Case He was accused of it along with his brother Barindra nath Ghosh But Aurobindo was acquitted because of the brilliant pleading of his counsel Chittaranjan Das Then he became a spiritual reformer introducing his visions on human progress and spiritual evolution4 Qustion Who was KingsfordAnswer Kingsford was an unpopular British chief Magistrate who was the target of the bomb thrown at Muzaffarpur by Khudiram and Prafulla Chaki

Most of the accused were convicted and sentenced to

long term of imprisonmentBut

Aurobindo Ghosh was acquitted mainly owing to the brilliant pleading of his counsel Chittaranjan Das

Political science Topic-Sovereignty

Summary Sovereignty is the full right and power of a governing body over itself without any interference from outside sources or bodies In political theory sovereignty is a substantive term designating supreme legitimate authority over some polity In international law sovereignty is the exercise of power by a state

Internal Sovereignty

Internal sovereignty means supreme authority within ones territory while external sovereignty relates to the recognition on the part of all states that each possesses this power in equal measure

External sovereignty

external sovereignty relates to the recognition on the part of all states that each possesses this power in equal measure

Distinguish between

Execution

Answer the following questions

Short notes-

Sovereignty

Internal Sovereignty

External sovereignty

Homework- learn

external sovereignty and internal sovereigntySovereignty is the principle

of supreme and

unquestionable authority

reflected in the claim by the

state to be the sole author of

laws within its territory

Definition of external vs internal sovereigntyInternal sovereignty refers to

the relationship between a

sovereign power and its

subjects ndash it refers to the

location of the supreme

authority within the state In

the UK for example internal

sovereignty (supposedly)

resides within Parliament

reflected in the

constitutional principle of

parliamentary

sovereigntyBy contrast

external sovereignty refers

to the capacity of the state

to act independently and

autonomously on the world

stage This is what is

sometimes called lsquostate

sovereigntyrsquo or lsquonational

sovereigntyrsquo and implies

that states are legally equal

and that the territorial

integrity and political

independence of a state is

inviolable

Class ndash XII

Date - 2742020 STUDY MATERIALSubject Topic Summary Execution Business Studies

Job Analysis amp Manpower Planning

At first let us recall the chapter what we have discussed till nowbullJob analysisbullJob specification bullJob description bullJob enlargement bullJob enrichment

Today we will do some questions answers from the chapter

Questions 1ldquoJob analysis job description and job specification are interrelatedrdquo Comment Answer) Job analysis is a systematic and detailed examination of a job to collect all the relevant information about it The contents off the job are summarised in the job description The qualification needed for the job are summarised in job specificationThus there is close interrelationship between job analysis job description and job specification

Question 2ldquoJob enlargement is a horizontal extension of a job whereas job enrichment is a vertical extension of a jobrdquo ElucidateAnswer) Job enlargement involves adding one or more task to a job coma where as job enrichment involves adding more autonomy and responsibility to a job Job enlargement is therefore horizontal extension of a job coma whereas job enrichment is a vertical extension of a job

Question 3 )

What is manpower estimation Explain its quantitative and qualitative aspectsAnswer) Manpower estimation is the process by which management determines how an organisation should move from itrsquos current manpower positionto its desired manpower position There are two dimensions of Manpower estimation- quantitative and qualitative

Quantitative aspectThis aspect of Manpower estimation involves estimating the number of employees required in a future time period Workload analysis and workforce analysis are done to estimate the quantity of required manpower

Qualitative aspectThe estimate of the knowledge skills experience etc of required manpower is the qualitative aspect of Manpower estimation The quality of Manpower can be judged on the basis of job analysisand job specification

COMMERCE

CAPITAL-FIXED AND WORKING

Today let us start the class by discussing the sources of finance for different types of business firms

The term lsquocapitalrsquo refers to the investment made in the enterprise for the purpose of earning profits

Requirements of capital and sources of capital for different types of business firms are

1 Capital for sole proprietorship businessA sole proprietor operates at a small scale and thereforerequires a limited amount of capital

2 Capital for partnership firmCapital requirements as well as capital base of a partnership is bigger than that of a sole trader businessThe owned capital is contributed by the partners in an agreed ratio

3 Capital for joint stock companyA joint stock company generally requires large amount of capitalA public company can raise huge capital through issue of shares In addition to share capital it can utilize retained profits

Now let u discuss the meaning of Finance PlanningFinance planning is the process of estimation the financial requirements of an organization specifying the sources of firms and ensuring that enough funds are available at the right time

1 What do you mean by Finance PlanningAnswer Finance planning is the process of estimation the financial requirements of an organization specifying the sources of firms and ensuring that enough funds are available at the right time

2Discuss the role of financial planning of an enterpriseThe role of financial planning are as followsa A sound financial plan helps a business enterprise to avaid the problems of shortage and surplus of fundsbFinancial planning serves as a guide in developing a sound capital structure so as to maximize returns to shareholders c It helps in effective utilization of fundsd It provides policies and procedures for coordinating different functional areas or departments of businesse It enables the management to exercise effective control over the financial activities of an enterprisef It helps the company to prepare for facing business shocks and surprises in future

Mathematics

Continuity and differentiability

Recall Definition of ContinuityLet f(x) be a single valued function of x and x=a be a point in the domain of definition of the function The function is said to be continuous at x=a ifi) f(c) is defined ie f(x) has a definite finite value at x=cii) lim xrarra f(x) exists andiii) lim xrarra f(x) =f(a) In other words f(x) is said to be continuous at x=a if lim xrarra+ f(x)= lim xrarra- f(x) = f(a) Or f(a+0) =f(a-0) =f(a) Or lim hrarr0 f(a+h)= f(a) Algebra of continuous functionsNow we will study some algebra of continuous functions Theorem 1 Suppose f and g be two real functions continuous at a real number c Then(1) f + g is continuous at x = c(2) f ndash g is continuous at x = c(3) f g is continuous at x = c(4) (fg) is continuous at x = c (provided g (c) ne 0)

Example 1 Prove that every rational function is continuousSolution Recall that every rational function f is given byf(x)=[p(x) q(x) ] q(x)ne0where p and q are polynomial functions The domain of f is all real numbers except points at which q is zero Since polynomial functions are continuous f is continuous by (4) of Theorem 1Example 2Discuss the continuity of sine functionSolution To see this we use the following factslim xrarr0 sin x =0Now observe that f (x) = sin x is defined for every real number Let c be a real number Put x = c + h If x rarr c we know that h rarr 0 Therefore lim xrarrc f(x)

= lim xrarrc sin x= lim hrarr0 sin(c+h) =lim hrarr0 [sin c cos h + cos c sin h ]=lim hrarr0 (sin c cos h) + lim hrarr0 (cos c sin h) = sin c +0=sin c = f(c) Thus lim xrarrc f(x) = f(c) and hence f is a constant function Exercise Prove that the function f(x) = x2 +2x is continuous for every real value of x [Hints show that lim xrarra+ f(x) = lim xrarra- f(x) = f(a) ]

Biology Reproduction in Flowering plants We will discuss about megasporoangium

megasporagenesis and female gametophyte

Q4 Describe the structure of megasporangium

Ovule is attached to the placenta by astalk called funicle

Each ovule has one two or three protectivecoverings called integuments

At the tip of integuments a small openingcalled micropyle is organised

Opposite to the micropylar end is the chalaza

Within the integuments a mass of cellsnucellusand inside it embryo sac orfemale gametophyte is present

Q5 Describe a mature embryosacamp its formation

In most of the flowering plants only oneof the 4 megaspores formed as a result ofmegasporogenesis that is functional while theother three degenerate

The

functional megaspore develops into thefemale gametophyte

Formation The nucleus

of the functional megasporedivides mitotically to form two nuclei first andthen two more sequential mitotic nucleardivisions result in the formation of four ampthen eight nucleate stages of embryo sac

Six of the eight nucleus are surrounded bycell walls and organised into cells

The remaining two nuclei called polar nuclei are found below the egg apparatus in the largecentral cell

Three cells consisting of two synergids amp one egg cell present bottom of

embryo sac Three cells

at the chalazal as antipodal cells

Two polar nuclei together present in large central cell

HISTORY

TOWARDS INDEPENDENCE AND PARTITION THE LAST PHASE(1935-1947)SUB TOPIC NATIONAL MOVEMENTS DURING THE SECOND WORLD WAR

Spread of Quit India Movement On 9th August 1942Gandhiji and other Congress leaders were arrested The Congress was declared illegal The news of the arrest of all leaders marked the beginning of a widespread movement of India It was not possible for such a movement to remain peacefulBut the arrest of the all notable congress leaders virtually left the movement in the hands of the mass The movement took the form of violent and militant outbreakBesides congressmen revolutionaries also were very active in the movement The Congress Socialist group also played a prominent role

1 Question Why did the British authority arrest the Congress leaders on 9 th August 1942Answer Congress Working committee adopted the Quit India resolution which was to be ratified at the Bombay AICC meeting in 8th August 1942 They decided to launch a mass struggle on non-violent lines Gandhiji gave a clarion call to all section of the people rdquoKarenge ya Marengerdquo (do or die) Congress leaders gave the call to driving out

the British from IndiaViceroy had taken strong action against the Quit India movement Gandhiji and all the leaders of Congress were arrested

2 Question How did Quit India Movement spread out all over IndiaAnswer The news of the leaders lsquo arrest marked the beginning of a widespread movement to remain peacefulThe movement took form of violent outbreak There were widespread cutting of telephone and Telegraph wires damaging railway lines raising barricades in cities and towns and other forms of violent demonstations

Question Name the leaders of Congress

Socialist group played a prominent part Notable among the Jayprakash Narayan Rammonohar Lohia Aruna Asaf Ali

Political science

Topic-Franchise and Representation

Summary

The election commission

The Election Commission of India is an autonomous constitutional authority responsible for administering Union and State election processes in India The body administers elections to the Lok Sabha Rajya Sabha State Legislative Assemblies in India and the offices of the President and Vice President in the country

Functions of election commission-

India is a sovereign socialist secular democratic republic Democracy runs like a golden thread in the social economic and political fabric woven by the Constitution given by lsquoWe the People of Indiarsquo unto ourselves The concept of democracy as visualised by the Constitution pre-supposes the representation of the people in Parliament and State legislatures by the method of election The Supreme Court has held that democracy is one of the inalienable basic features of the Constitution of India and forms part of its basic structure The Constitution of India adopted a Parliamentary form of government Parliament consists of the President of India and the two Houses mdash Rajya Sabha and Lok Sabha India being a Union of states has separate state legislatures for each state State legislatures consist of the Governor and two Houses mdash Legislative Council and Legislative Assembly mdash in seven states namely Andhra Pradesh Telangana Bihar Jammu amp Kashmir Karnataka Maharashtra and Uttar Pradesh and of the Governor and the state Legislative Assembly in the remaining 22 states Apart from the above two out of the seven Union Territories namely National Capital Territory of Delhi and Puducherry also have their Legislative Assemblies

ExecutionShort notes-Election commissionFunctions of election commission

Homework- Learn

Computer

Science

Computer hardware NAND Gate

A NOT-AND operation is known as NAND operation It has n input (n gt= 2) and one output

Logic diagram

Truth Table

NOR Gate

A NOT-OR operation is known as NOR operation It has n input (n gt= 2) and one output

Logic diagram

Truth Table

XOR Gate

XOR or Ex-OR gate is a special type of gate It can be used in the half

adder full adder and subtractor The exclusive-OR gate is abbreviated as EX-OR gate or sometime as X-OR gate It has n input (n gt= 2) and one output

Logic diagram

Truth Table

XNOR Gate

XNOR gate is a special type of gate It can be used in the half adder full adder and subtractor The exclusive-NOR gate is abbreviated as EX-NOR gate or sometime as X-NOR gate It has n input (n gt= 2) and one output

Logic diagram

Truth Table

Physics

Chapter 1 Electric Field ( Electric Dipole) (Summary)

Here we will derive Expression of electric field at broad side

On position of dipole

Execution

Q With the help of a labelled diagram obtain an expression for the electric field intensity E at any point on the equitorial line ( broad-side on position) of an electric dipole

Ans

E1 E1sinθ

E θ P E1 θ

( r2+L2)12 E2 E

r E2 E2sinθ

-q θ L O L +qA B

Let us consider that the point P is situated on the right bisector of the dipole AB at a distance r meter from its midpoint O

Let E1 and E2 be the electric field intensities of the electric field at P due to charge +q and ndashq of the dipole resp The distance of P from each charge is ( r2+L2)12

So E1 = 14 πϵ q

(r 2+L 2) away from +q

E2 = 14 πϵ q

(r 2+L 2) towards ndashq

The magnitudes of E1 and E2 are equal but directions are different Now resolving E1 and E2 into two components parallel and perpendicular to AB we get

The components perpendicular to AB E1sinθ and E2sinθ cancel each other because they are equal and opposite

The components parallel to AB are E1cosθ and E2 cosθ are in same direction and add up

So resultant intensity of electric field at the point P is

E = E1cosθ + E2 cosθ

E = 14 πϵ q

(r 2+L 2) 2 cosθ

Now from fig we have cosθ =BOBP = L (r2+L2)12

So we get E = 14 πϵ 2qL ( r2+L2)32

Now electric dipole moment p= 2qL

So E = 14 πϵ p ( r2+L2)32

HW Find the expression of Electric field as done here but this time take r gtgt 2L

Also find the expression of torque experience by a dipole

(Hint Electric force experienced by charges of dipole in electric field is qE each Let θ be the angle which dipole makes with electric lines of force then perpendicular distance between two charges is 2Lsinθ Then torque = force x perp distance = qE x 2L sinθ So τ=pE sinθ where p =2qL )

STUDY MATERIAL

Class XIISubject Eng Literature (The Tempest ndash William Shakespeare) Topic Act IV Scene 1 Lines 84 to 133 (Iris hellip A contract of true love Be not too late ) Date 27th April 2020 (4th Period)

[Students should read the original play and also the paraphrase given in the school prescribed textbook]Summary Questions amp Answers

o Ceres soon appears and comes to know that she has been summoned to celebrate the contract of true love

o Ceres expresses her unwillingness to meet Venus and Cupid as she has shunned their company

o Ceres and Juno both bestow their blessings upon Ferdinand and Miranda with June gifting honour riches happiness in marriage and Ceres presents plenty of earthrsquos produce

o Iris summons the water-nymphs and reapers to come and celebrate a contract

(1) IRIS Of her society (Line 91-101)

Be not afraid I met her deity

Cutting the clouds towards Pathos and her sonDove-drawn with her Here thought they to have doneSome wanton charm upon this man and maidWhose vows are that no bed-right shall be paidTill Hymens torch be lightedmdashbut in vainMarss hot minion is returned againHer waspish-headed son has broke his arrowsSwears he will shoot no more but play with sparrowsAnd be a boy right out

(i) Where were Venus and Cupid seen flying How were they travelling Why did they want to join the marriage celebration of Ferdinand and Miranda

of true love

Venus and Cupid were seen flying through the air towards Paphos the famous city which is situated on the island of Cyprus They were travelling by air-borne chariot drawn by doves They certainly wanted to come here in order to play some amorous trick upon Ferdinand and Miranda who are under a vow not to gratify their physical desires till the holy ceremony of their marriage has been performed(ii) What have Venus and Cupid done after failing in their plan

After being failure of their plan Venus who is a very passionate deity and who is the mistress of Mars (the god of war) has gone back while here ill-tempered son Cupid has broken his arrows of love in his state of desperation(iii) What has Cupid firmly decided

Cupid is feeling so disappointed that he has firmly decided to shoot no more arrows to arouse love in human hearts but to spend his time playing with sparrows Thus he would now become just a boy and would give up his original function of shooting arrows on human beings to make them fall in love(iv) What vow had Ceres taken How did Ceres feel at the abduction

After the abduction of her daughter Prosperina by Pluto Ceres had taken a vow to always keep away from the disgraceful company of Venus and her blind son Cupid the god of love Ceres felt deeply distressed when Pluto had carried off her daughter and had made her his wife by force(v) Why has Ceres not forgiven Venus and her blind son For what do Ceres want to be sure

As the abduction had been manipulated by Venus the goddess of beauty and love and her blind son Cupid Ceres has never forgiven them for their part in the whole plot Ceres wants to be sure that she would not have to meet Venus and Cupid who had engineered the abduction of her daughter Prosperina

AS THIS lsquoMASQUErsquo SCENE IS VERY IMPORTANT IN THE PLAY THE PARAPHRASE OF THE ENTIRE PORTION OF MASQUE SCENE (Act IV Lines 58 to 143) IS GIVEN BELOW

IRIS Goddess of RainbowCERES Goddess of Agriculture and all the fruits of the earth

(Nature growth prosperity rebirth ndash notions intimately connected to marriage)JUNO The majestic Queen of Heavens and wife of Jupiter (Jupiter is the king of Gods)

VENUS The Goddess of love CUPID Son of Venus PLUTO God of death (In the play referred by Shakespeare as lsquoDisrsquo which is a Roman name for Pluto)

ORIGINAL TEXT PARAPHRASEPROSPEROWellmdash

PROSPERONow come Ariel Let there be too many rather than too few

Now come my Ariel Bring a corollaryRather than want a spirit Appear and pertly[to Ferdinand and Miranda]No tongue all eyes Be silent

spirits in attendance Appear briskly

[to Ferdinand and Miranda]Look with your eyes but do not say a word

[Soft music] [Soft music][Enter Iris] [Enter Iris]

IRISCeres most bounteous lady thy rich leasOf wheat rye barley vetches oats and peasThy turfy mountains where live nibbling sheepAnd flat meads thatched with stover them to keepThy banks with pioned and twilled brimsWhich spongy April at thy hest betrimsTo make cold nymphs chaste crowns and thybroom-grovesWhose shadow the dismissegraved bachelor lovesBeing lass-lorn thy pole clipped vineyardAnd thy sea-marge sterile and rocky-hardWhere thou thyself dost airmdashthe Queen othrsquoSkyWhose watery arch and messenger am IBids thee leave these and with her sovereign grace[Juno appears] Here on this grass-plot in this very placeTo come and sport Her peacocks fly amainApproach rich Ceres her to entertain

IRISCeres most generous lady you are the cause of rich fields or fertile land where wheat rye barley beans oats and peas grow the grassy mountains where the sheep graze and the flat meadows covered with coarse hay to be used as fodder for cattleYour banks are covered with marsh-marigolds and reeds and the rainy April under your orders brings forth to make for the maids who are not in love beautiful crowns your woods where the broom flourishes and where the bachelor who has been dismissed by the maid he loved lies down being forsaken your vineyard in which the poles are embraced by the vines and the margin of the sea which is barren and rocky where you roam about to enjoy the fresh air ndash the queen of the sky (Juno) whose messenger I am besides being represented as the rainbow bids you leave all these and with her majesty here on this grassy plot in this very place come and sport her peacocks carry her fast in her chariot through the air and are making their way here approach rich Ceres to welcome her

[Enter Ariel as Ceres] [Enter Ariel as Ceres]

CERESHail many-coloured messenger that neerDost disobey the wife of JupiterWho with thy saffron wings upon my flowersDiffusest honey-drops refreshing showersAnd with each end of thy blue bow dost crownMy bosky acres and my unshrubbed downRich scarf to my proud earth Why hath thy queenSummoned me hither to this short-grassed green

CERESWelcome rainbow that never dared disobey Juno the wife of Jupiter who with your orange coloured rays spread honey-drops refreshing showers And with each end of thy blue bow drown my bushy acres and my hilly country which is free from shrubs you thus forming a rich scarf Why has your queen called me here to this place covered with short grass

IRISA contract of true love to celebrateAnd some donation freely to estateOn the blest lovers

IRISI have called you to celebrate a contract of true love and bestow some liberal gift upon the blessed lovers

ORIGINAL TEXT PARAPHRASECERESTell me heavenly bowIf Venus or her son as thou dost knowDo now attend the queen Since they did plotThe means that dusky Dis my daughter gotHer and her blind boys scandaled companyI have forsworn

CERESTell me heavenly bow if Venus the Goddess of love or Cupid her son and pedlar of passion at this time attend the heavenly queen Juno because you are sure to know Since the day they conspired against me and dark Pluto took away my daughter here and Cupidrsquos disgraceful company I have left off

IRISOf her societyBe not afraid I met her deityCutting the clouds towards Pathos and her sonDove-drawn with her Here thought they to have doneSome wanton charm upon this man and miad

IRISBe not afraid of her company I met her deity moving on the clouds towards Paphos the sacred home of Venus on the island of Cyprus along with her son on her chariot drawn by doves Here they contemplated to exercise a charm upon this man and maid producing

Whose vows are that no bed-right shall be paidTill Hymens torch be lightedmdashbut in vainMarss hot minion is returned againHer waspish-headed son has broke his arrowsSwears he will shoot no more but play with sparrowsAnd be a boy right out

wantonness before the actual marriage ceremony but did not succeed Venus has returned her irritable son has broken his arrows and swears that he will give up his practice of trying to inspire love but play with sparrows and be a boy again

[Music is heard] [Music is heard]

CERESHighst queen of stateGreat Juno comes I know her by her gait

CERESHighest queen of state Great Juno there she comes I know here by her gait

[Enter Juno] [Enter Juno]

JUNOHow does my bounteous sister Go with meTo bless this twain that they may Prosperous beAnd honoured in their issue

JUNOHow are you doing my generous sister Come with me to bless this couple so that they may be prosperous and fortunate in their children

[They sing] [They sing]

JUNOHonour riches marriage-blessingLong continuance and increasingHourly joys be still upon youJuno sings her blessings upon you

JUNOMay honour riches happiness in marriage long continuance and increase of those boons ever rest upon you as hourly joys Juno showers down upon you her blessings in song

CERESEarths increase foison plentyBarns and garners never emptyVines and clustring bunches growingPlants and goodly burden bowingSpring come to you at the farthestIn the very end of harvestScarcity and want shall shun youCeresrsquo blessing so is on you

CERESMay you have the plenty of earthrsquos produce Your barns and granaries may never be empty Your vines may grow with clustering bunches Your fruit trees may be heavily laden with their fruit May there be continuous spring and harvest May scantiness and want leave you forever Such is the blessing of Ceres upon you

FERDINANDThis is a most majestic vision andHarmoniously charmingly May I be boldTo think these spirits

FERDINANDThis is a great vision and magically melodious Should I suppose the characters (taking part in the masque) are spirits

PROSPEROSpirits which by mine artI have from their confines calld to enactMy present fancies

PROSPEROYes they are spirits whom I have summoned from the regions to which they are confined to carry into effect my fanciful designs

ORIGINAL TEXT PARAPHRASEFERDINANDLet me live here everSo rare a wondered father and a wifeMakes this place paradise

FERDINANDI should like to live here forever Such a wise and wonderful father makes this place a paradise

[Juno and Ceres whisper and send Iris on employment] [Juno and Ceres whisper and send Iris on employment]

PROSPEROSweet now silence

PROSPEROMy dear Ferdinand speak no more Juno and Ceres are

Juno and Ceres whisper seriouslyTheres something else to do Hush and be muteOr else our spell is marred

whispering with a solemn look There is something else coming Silence Or else our magic will be spoilt

IRISYour nymphs called naiads of the wandering brooksWith your sedged crowns and over-harmless looksLeave your crisp channels and on this green landAnswer your summons Juno does commandCome temperate nymphs and help to celebrateA contract of true love Be not too late

IRISYou nymphs called Naiads denizens (M inhabitants) of the running stream with your chaplets of sedge and ever-helpful looks leave your wrinkled channels and on the green land answer the summons sent to you Juno has ordered some chaste nymphs and help to celebrate a noble and true marriage Donrsquot delay

[Enter certain nymphs] [Enter certain nymphs]You sunburnt sicklemen of August wearyCome hither from the furrow and be merryMake holiday your rye-straw hats put onAnd these fresh nymphs encounter every oneIn country footing

You sunburnt harvesters weary from the effects of the heat in August come here from the furrowed land and rejoice Make holiday with your rye-straw hats upon you and meet these fresh nymphs and join in country dancing

[Enter certain reapers properly habited They join with the nymphs in a graceful dance towards the end whereof Prospero starts suddenly and speaks]

[Enter certain reapers properly habited They join with the nymphs in a graceful dance towards the end whereof Prospero starts suddenly and speaks]

PROSPERO[aside] I had forgot that foul conspiracyOf the beast Caliban and his confederatesAgainst my life The minute of their plotIs almost come [to the spirits]Well done Avoidno more

PROSPERO(Aside)I had forgotten the wicked conspiracy of the beast Caliban and his accomplices against my life the time of their plot has almost arrived ndash (To the Spirits) well done depart no more of this

[To a strange hollow and confused noise the spirits heavily vanish]

[The spirits depart]

ORIGINAL TEXT PARAPHRASEFERDINANDLet me live here everSo rare a wondered father and a wifeMakes this place paradise

FERDINANDI should like to live here forever Such a wise and wonderful father makes this place a paradise

[Juno and Ceres whisper and send Iris on employment] [Juno and Ceres whisper and send Iris on employment]

PROSPEROSweet now silenceJuno and Ceres whisper seriouslyTheres something else to do Hush and be muteOr else our spell is marred

PROSPEROMy dear Ferdinand speak no more Juno and Ceres are whispering with a solemn look There is something else coming Silence Or else our magic will be spoilt

IRISYour nymphs called naiads of the wandering brooksWith your sedged crowns and over-harmless looksLeave your crisp channels and on this green landAnswer your summons Juno does commandCome temperate nymphs and help to celebrateA contract of true love Be not too late

IRISYou nymphs called Naiads denizens (M inhabitants) of the running stream with your chaplets of sedge and ever-helpful looks leave your wrinkled channels and on the green land answer the summons sent to you Juno has ordered some chaste nymphs and help to celebrate a noble and true marriage Donrsquot delay

[Enter certain nymphs] [Enter certain nymphs]You sunburnt sicklemen of August wearyCome hither from the furrow and be merryMake holiday your rye-straw hats put onAnd these fresh nymphs encounter every oneIn country footing

You sunburnt harvesters weary from the effects of the heat in August come here from the furrowed land and rejoice Make holiday with your rye-straw hats upon you and meet these fresh nymphs and join in country dancing

[Enter certain reapers properly habited They join with the nymphs in a graceful dance towards the end whereof Prospero starts suddenly and speaks]

[Enter certain reapers properly habited They join with the nymphs in a graceful dance towards the end whereof Prospero starts suddenly and speaks]

PROSPERO[aside] I had forgot that foul conspiracyOf the beast Caliban and his confederatesAgainst my life The minute of their plotIs almost come [to the spirits]Well done Avoidno more

PROSPERO(Aside)I had forgotten the wicked conspiracy of the beast Caliban and his accomplices against my life the time of their plot has almost arrived ndash (To the Spirits) well done depart no more of this

[To a strange hollow and confused noise the spirits heavily vanish]

[The spirits depart]

Ac-12 27420 topic Revaluation of Assets and Liabilities

REVALUATION OF ASSETS AND LIABILITIES

On admission of a new partner the firm stands reconstituted and consequently the assets are revalued and liabilities are reassessed It is necessary to show the true position of the firm at the time of admission of a new partner If the values of the assets are raised gain will increase the capital of the existing partners Similarly any decrease in the value of assets ie loss will decrease the capital of the existing partners For this purpose alsquoRevaluation Accountrsquo is prepared This account is credited with all increases in the value of assets and decrease in the value of liabilities It is debited with decrease on account of value of assets and increase in the value of liabilities The balance of this account shows a gain or loss on revaluation which is transferred to the existing partnerrsquos capital account in existing profit sharing ratioAccounting for Revaluation of Assets and Liabilities when there is a Changein the Profit Sharing Ratio of Existing PartnersAssets and liabilities of a firm must also be revalued at the time of change in profit sharing ratio of existing partners The reason is that the realisable or actual value of assets and liabilities may be different from those shown in the Balance Sheet It is possible that with the passage of time some of the assets might have appreciated in value while the value of certain other assets might have decreased and no record has been made of such changes in the books of accounts Similarly there may be some unrecorded assets amp libilities that may have to be accounted for Revaluation of assets and reassessments of liabilities becomes necessary because the change in the

value of assets and liabilities belongs to the period to change in profit sharing ratio and hence must be shared by the partners in their old profit sharing ratio Revaluation of assets and reassessment of liabilities may be given effect to in two different ways (a) When revised values are to be recorded in the books and(b) When revised values are not to be recorded in the books

When revised values are to be recorded in the booksIn such a case revaluation of assets and reassessment of liabilities is done with the help of a new account called lsquoRevaluation Accountrsquo Sometimes this account is also called as lsquoProfit amp Loss Adjustment Acrsquo If there is a loss due to revaluation revaluation account is debited and if the revaluation results in a profit the revaluation account is credited The following journal entries made for this purpose are

(i) For increase in the value of assetsAsset Ac Dr (individually)To Revaluation Ac(ii) For decrease in the value of AssetRevaluation Ac Dr (individually)To Asset Ac[Decrease in the value of assets](iii) For increase in the value of LiabilitiesRevaluation Ac Dr (individually)To Liabilities Ac[Increase in the value of Liabilities](iv) For decrease in the value of LiabilitiesLiabilities Ac DrTo Revaluation Ac[Decrease in the value of Liabilities](v) For unrecorded AssetsAsset Ac [unrecorded] DrTo Revaluation Ac[Unrecorded asset recorded at actual value](vi) For unrecorded Liability Revaluation Ac DrTo Liability Ac [unrecorded][Unrecorded Liability recorded at actual value](vii) For transfer of gain on revaluationRevaluation Ac DrTo Existing Partnerrsquos CapitalCurrent Ac[Profit on revaluation transferred to capital account in existing ratio](viii) For transfer of loss on revaluationExisting Partnerrsquos CapitalCurrent Ac DrTo Revaluation Ac[Loss on revaluation transferred to capital account in existing ratio](a) When revaluation account shows gain Revaluation Ac DrTo Partnerrsquos Capital Ac (Old Profit Sharing Ratio)(Profit on revaluation credited to Partnerrsquos Capital Ac)(b) Above entry is reversed when revaluation account shows loss Partners Capital Acs (Old Profit Sharing Ratio) DrTo Revaluation Ac(Loss on revaluation debited to Partnerrsquos Capital Acs)

Proforma of Revaluation Account is given as under

Revaluation Account

Dr Cr Particulars ` Amount Particulars ` Amount To Decrease in value of assets By Increase in value of assets To Increase in value of liabilities By Decrease in value of liabilities To Unrecorded liabilities By Unrecorded assets To Gain on Revaluation (Transferred) By Loss on Revalution (Transferred)

ECO ndash12 2742020Topic- ELASTICITY OF DEMAND

CHAPTER - ELASTICITY OF DEMANDMEANINGDemand for a commodity is affected by many factors such as its price price of related goods income of its buyer tastes and preferences etc Elasticity means degree of response Elasticity of demand means degree of responsiveness of demand Demand for a commodity responds to change in price price of related goods income etc So we have three dimensions of elasticity of demandDIMENSION OF ELASTICITY OF DEMAND TYPES OF ELASTICITY OF DEMAND

Price elasticity of demand Income elasticity of demand Cross Elasticity of demand

Price elasticity of demand Price elasticity of demand means degree of responsiveness of demand for a commodity to the change in its price For example if demand for a commodity rises by 10 due to 5 fall in its price Price elasticity of demand (ep)=Percentage change in quantity demanded Percentage change in price of the commodity = 10 ( -)5 = ( - )2Note that ep will always be negative due to inverse relationship of price and quantity demanded

(ii) Income elasticity of demand Income elasticity of demand refers to the degree of responsiveness of demand for a commodity to the change in income of its buyer Suppose income of buyer rises by 10 and his demand for a commodity rises by 20 then Income elasticity of demand (ey)= change in quantity demanded change in price of the commodity =20 10 = 2

Cross Elasticity of demandCross elasticity of demand means the degree of responsiveness of demand for a commodity to the change in price of its related goods (substitute goods or complementary goods) Suppose demand for a commodity rises by 10 due to 5 rise in price of its substitute good then Cross elasticity of demand (ec) = change in quantity demanded change in price of related good = 10 2 = 5 (Tastes and preferences cannot be expressed numerically So elasticity ofdemand cannot be numerically expressed)

  • Chapter 1 Force (Summary)
  • Distinguish between external sovereignty and internal sovereignty
    • NAND Gate
      • Logic diagram
      • Truth Table
        • NOR Gate
          • Logic diagram
          • Truth Table
            • XOR Gate
              • Logic diagram
              • Truth Table
                • XNOR Gate
                  • Logic diagram
                  • Truth Table
                      • Physics
                      • Chapter 1 Electric Field ( Electric Dipole) (Summary)
Page 29:  · Web viewWe all know that Nouns are divided into two parts: common noun and proper noun.Apart from common and proper noun, we will also study about collective noun and compound

Now let us discuss the national objectives of business

Optimum utilization of resources

National self- reliance Development of small

scale industries Development of

backward areas Control over pollution

2Explain the national objectives of a business enterprise

Answer

It is the duty of business to utilize the resources of the country properly the national objectives of business

Optimum utilization of resources ndashBusiness should use the nationrsquos resources in the best possible manner

National self- reliance-It is the duty of the business to help the government in increasing experts and in reducing dependence on imports

Development of small scale industries-Big business firms are expected to encourage growth of small scale industries which are necessary for generating employment

Development of backward areas-Business is expected to give preference to the industrialization of backward regions of the country

ECONOMICS

BASIC ECONOMIC CONCEPTS

SUB

TOPIC

Value

Wealth

Welfare

Today we shall start with a new topic of the same chapter ie lsquoValuersquo

Value of a commodity is defined as the valuation placed by a household on the consumption of this commodity

lsquoValuersquo has two different meanings and these are

a Value -in -use It refers to consumption value of a commodity It expresses the utility derived from the consumption of a particular commodity A necessity like water has a very high value ndashin ndashuse or

Question

1What is value

Answer

Value of a commodity is defined as the valuation placed by a household on the consumption of this commodity

2What is value-in use

Answer It refers to consumption value of a commodity It expresses the utility derived from the consumption of a particular commodity A necessity like water has a very high value ndashin ndashuse or consumption value

3What is value ndashin- exchange

Answer It relates to market value of a commodity

It is the rate at which a particular good or service can be exchanged for money

For example in barter system if a person is prepared to exchange 3 metres of cloth with 1 pair

consumption value

b Value ndashin-exchange It relates to market value of a commodity

It is the rate at which a particular good or service can be exchanged for moneyFor example in barter system if a person is prepared to exchange 3 metres of cloth with 1 pair of shoes then the value in exchange of 3 metres of cloth is 1 pair of shoesValue in exchange is the power of purchasing other goods In modern monetised economies the exchange value of goods are expressed in terms of money as prices

Now let us discuss the term lsquoWealthrsquo

Wealth refers to the stock of all those assets which are a source of income

Wealth is a stock concept

Wealth must possess the following features

a Utility It must possess utility or give some

of shoes then the value in exchange of 3 metres of cloth is 1 pair of shoes

Value in exchange is the power of purchasing other goods In modern monetised economies the exchange value of goods are expressed in terms of money as prices

4 What is wealth

Answer it refers to the stock of assets or goods which are a source of income and have personal or national ownership

5 What are the features of wealth

Answer The features of wealth are as follows

Wealth must possess the following features

a Utility It must possess utility or give some satisfaction

b Scarcity It must be limited in quantityc Transferability it should be transferable its

ownership can be transferred from one person to another person

d Exchange value It must possess exchange value

6 What is welfare

Answer

Welfare is defined as satisfaction and happiness a sense of well- being among the people

satisfactionb Scarcity It must be

limited in quantityc Transferability It

should be transferable its ownership can be transferred from one person to another person

d Exchange value It must possess exchange value

Now let us discuss the term lsquoWelfarersquo

Welfare is defined as satisfaction and happiness a sense of well- being among the people

Welfare is affected by factors like

a Consumption of goods and services

b Environment

c Family relations

d Degree of freedom

e Law and order situation

Mathematics Trigonometric equation

To find the general solution of the equation sinθ=0

When sin θ =0

Then θ= 0 π2π 3π-π -2π -3

i e when θ = 0 or an integral multiple of π

i e when θ= nπ where n is any integer

Therefore the general solution of the equation sin

Example1 Find the general values of θ which satisfy the equation sin2 θ =34

Solution sin2 θ= 34

Or sin θ = +34 or -34

Or sin θ = sin π3 or sin (-π3)

Therefore

θ = [nπ + (-1) n (π3)] or[ nπ+ (-1) n (-π3)]

= nπ +π3 or nπ-π3 where n= any integer

Example 2Find the values of θ which satisfy tan2 θ

θ=0 is θ= nπ where n is any integer

To find the general solution of the equation cos θ=0

When cos θ=0

Then θ=π2 3π2 5π2 -π2 -3π2 -5π3

i e when θ is an odd multiple of π2

i e when θ=(2n+1) π2 where n= any integer

Therefore the general solution of the equation cos θ =0 is θ= (2n+1) π2 where n= any integer

To find the general solution of the equation tan θ = 0

Clearly tan θ =0 implies sin θcos θ =0

Therefore θ = nπ

i e the general solution of the equation tan θ=0 is θ =nπ where n = any integer

To find the general solution of the equation cot θ =0

Clearly cot θ =0 implies (cos θsin θ) = 0

i e cos θ =0

Therefore θ = (2n+1) π2

Therefore the general solution of the equation cot θ =0 is θ = (2n+1) π2

Where n= any integer

To find the general solution of the equation sin θ= k (-

=13 -πleθleπ

Solution tan2 θ =13

Or tan θ = plusmn1radic(3) =tan(plusmnπ6)

θ=nπ plusmn π6 where n =any integer

If n=0 then θ=plusmnπ6

If n=1 then θ= π plusmn π6

If n=-1 then θ= -π plusmn π6

Therefore the required solution in -π le θ le π are θ= π6 5π6 -π6 -5π6

Exercise Find general solution of sin 2θ=cos θ [Hints Use sin 2θ= 2sin θcosθ and then take cosθ

common]

1lek le1)

Determine an angle alpha such that sin =k and -π2le αle π2

Then we have

Sin θ = k = sin α

Or sin θ - sin α =0

Or 2 cos [(θ+α) 2] sin [(θ-α) 2] =0

Therefore either cos [(θ +α) 2] =0 (1)

Or sin [(θ-α) 2] =0 (2)

Now from (1) we get (θ+α) 2= (2m+1) π2)

Or θ = (2m+1) π-α (3)

And from (2) we get (θ-α) 2 =mπ

Or θ= 2mπ+α(4)

Where m = any integer

Clearly the solution (3) amp (4) may be combined in the following form

θ= nπ+(-1) n α where n= any integer

Therefore the general solution of sin θ = sin α is θ = nπ +(-1) n α where n is any integer and -π2 le α le π2

Biology Chapter - 04Kingdom Monera

Today we will discuss about bacterial reproduction and its usefulness

Fig Binary Fission

Fig Conjugation Fig Transformation

Bacterial reproduction is mainly asexual but sexual reproduction

also takes place

Asexual reproduction takes place by i) Binary fission - from one bacteriato

two bacteria are produced in every 20 to 30mins

ii) Buddingiii) Endospore formation - during

unfourable condition

Sexual reproduction by three ways

1) Conjugation - Transfer of genetic material between cells that are in physical contact with one another

2) Transduction - Transfer of genetic materialfrom one cell to another by a bacteriophage

3) Transformation - Transfer of cell-freeor naked DNArsquo from one cell to another

Bacteria causes different diseases inplants animals and human and

it causes food spoilage and waterpollution but it also have some useful

activities

i) Bacteria are helpful in sewage water treatment

ii) It is used in antibiotic (medicine) production

iii) Anaerobic bacteria help in biogas(energy) production

iv) Many household products like yoghurt cheese are manufactured by use of bacteria

v) Rhizobium by symbiotic relationship with leguminous plant increase soil fertility

vi) Besides these bacteria is helpful in genetic engineering degradation of petroleum hydrocarbonand in dairy

industry

Physics Motion in plane Here we will introduce Projectile Motion

Execution

Projectile

Y

usinθ u h

θX

ucosθ

Suppose a body is projected with an angle θ So initial velocity u can be resolved into two components

Horizontal component - ucosθ ( for range)

Vertical component - usinθ ( for height)

usinθ changes during motion and becomes zero at maximum height position but ucosθ remain unchanged

The maximum height of projectile is h

NB If initial is upward then g = -ve and if it is downward then g = +ve Height is +ve if direction of motion does not change ( for ex a body thrown upwards but goes down ultimately then height h = -ve)

The angle of projectile θ is the angle made with horizontal

HISTORY ndash GROWTH OF NATIONALISM

SUB TOPIC- REVOLUTIONARY NATIONALISM Bengal formation of Anushilan Samity and Jugantar Group

The intensification of the Swadeshi movement and Government policy of terror and repression led to outbreak of violence Bombs were manufactured and attempts on the lives of unpopular Government officials became frequent In the gymnasium of Scottish Church College which was known as General Assemblies Institution a secret society was formed known as Anushilan Samity

Aurobindo Ghosh send from Baroda his emissary Jatindranath Banerjee to mobilize the Bengal revolutionaries

Hemchandra Qanungo and Satyen Bose published Journal Jugantar

The Jugantar group planned to assassinate oppressive magistrate Kingsford by Khudiram Bose and Prafulla Chaki in 1908 Prafulla Chaki committed suicide to avoid arrest Khudiram was tried and hanged

Afew days later the police found a bomb factory in Maniktala and arrested a large number of revolutionaries The trial of revolutionaries became famous as the Alipore Bomb Case

In the course of the trial the approver the public prosecuter and a police officer were assassinated

1 Question Name two journals which preached the cult of violence

Answer a) Yugantar edited by Bhupendranath DuttaB) Bandemataram edited by Aurobindo Ghosh2 Question Why was Khudiram arrested and hangedAnswer An attempt was made to assassinate a hated vindictive majistrate named Kingsford by Khudiram Bose and Prafulla Chaki Their attempt failed and the bomb they threw killed two English ladies Khudiram was arrested and put to trial and then hanged3Question Who was Aurubindo GhoshAnswer Aurobindo Ghosh a nationalist revolutionary who was charged for his involvement in the Alipore Bombing Case He was accused of it along with his brother Barindra nath Ghosh But Aurobindo was acquitted because of the brilliant pleading of his counsel Chittaranjan Das Then he became a spiritual reformer introducing his visions on human progress and spiritual evolution4 Qustion Who was KingsfordAnswer Kingsford was an unpopular British chief Magistrate who was the target of the bomb thrown at Muzaffarpur by Khudiram and Prafulla Chaki

Most of the accused were convicted and sentenced to

long term of imprisonmentBut

Aurobindo Ghosh was acquitted mainly owing to the brilliant pleading of his counsel Chittaranjan Das

Political science Topic-Sovereignty

Summary Sovereignty is the full right and power of a governing body over itself without any interference from outside sources or bodies In political theory sovereignty is a substantive term designating supreme legitimate authority over some polity In international law sovereignty is the exercise of power by a state

Internal Sovereignty

Internal sovereignty means supreme authority within ones territory while external sovereignty relates to the recognition on the part of all states that each possesses this power in equal measure

External sovereignty

external sovereignty relates to the recognition on the part of all states that each possesses this power in equal measure

Distinguish between

Execution

Answer the following questions

Short notes-

Sovereignty

Internal Sovereignty

External sovereignty

Homework- learn

external sovereignty and internal sovereigntySovereignty is the principle

of supreme and

unquestionable authority

reflected in the claim by the

state to be the sole author of

laws within its territory

Definition of external vs internal sovereigntyInternal sovereignty refers to

the relationship between a

sovereign power and its

subjects ndash it refers to the

location of the supreme

authority within the state In

the UK for example internal

sovereignty (supposedly)

resides within Parliament

reflected in the

constitutional principle of

parliamentary

sovereigntyBy contrast

external sovereignty refers

to the capacity of the state

to act independently and

autonomously on the world

stage This is what is

sometimes called lsquostate

sovereigntyrsquo or lsquonational

sovereigntyrsquo and implies

that states are legally equal

and that the territorial

integrity and political

independence of a state is

inviolable

Class ndash XII

Date - 2742020 STUDY MATERIALSubject Topic Summary Execution Business Studies

Job Analysis amp Manpower Planning

At first let us recall the chapter what we have discussed till nowbullJob analysisbullJob specification bullJob description bullJob enlargement bullJob enrichment

Today we will do some questions answers from the chapter

Questions 1ldquoJob analysis job description and job specification are interrelatedrdquo Comment Answer) Job analysis is a systematic and detailed examination of a job to collect all the relevant information about it The contents off the job are summarised in the job description The qualification needed for the job are summarised in job specificationThus there is close interrelationship between job analysis job description and job specification

Question 2ldquoJob enlargement is a horizontal extension of a job whereas job enrichment is a vertical extension of a jobrdquo ElucidateAnswer) Job enlargement involves adding one or more task to a job coma where as job enrichment involves adding more autonomy and responsibility to a job Job enlargement is therefore horizontal extension of a job coma whereas job enrichment is a vertical extension of a job

Question 3 )

What is manpower estimation Explain its quantitative and qualitative aspectsAnswer) Manpower estimation is the process by which management determines how an organisation should move from itrsquos current manpower positionto its desired manpower position There are two dimensions of Manpower estimation- quantitative and qualitative

Quantitative aspectThis aspect of Manpower estimation involves estimating the number of employees required in a future time period Workload analysis and workforce analysis are done to estimate the quantity of required manpower

Qualitative aspectThe estimate of the knowledge skills experience etc of required manpower is the qualitative aspect of Manpower estimation The quality of Manpower can be judged on the basis of job analysisand job specification

COMMERCE

CAPITAL-FIXED AND WORKING

Today let us start the class by discussing the sources of finance for different types of business firms

The term lsquocapitalrsquo refers to the investment made in the enterprise for the purpose of earning profits

Requirements of capital and sources of capital for different types of business firms are

1 Capital for sole proprietorship businessA sole proprietor operates at a small scale and thereforerequires a limited amount of capital

2 Capital for partnership firmCapital requirements as well as capital base of a partnership is bigger than that of a sole trader businessThe owned capital is contributed by the partners in an agreed ratio

3 Capital for joint stock companyA joint stock company generally requires large amount of capitalA public company can raise huge capital through issue of shares In addition to share capital it can utilize retained profits

Now let u discuss the meaning of Finance PlanningFinance planning is the process of estimation the financial requirements of an organization specifying the sources of firms and ensuring that enough funds are available at the right time

1 What do you mean by Finance PlanningAnswer Finance planning is the process of estimation the financial requirements of an organization specifying the sources of firms and ensuring that enough funds are available at the right time

2Discuss the role of financial planning of an enterpriseThe role of financial planning are as followsa A sound financial plan helps a business enterprise to avaid the problems of shortage and surplus of fundsbFinancial planning serves as a guide in developing a sound capital structure so as to maximize returns to shareholders c It helps in effective utilization of fundsd It provides policies and procedures for coordinating different functional areas or departments of businesse It enables the management to exercise effective control over the financial activities of an enterprisef It helps the company to prepare for facing business shocks and surprises in future

Mathematics

Continuity and differentiability

Recall Definition of ContinuityLet f(x) be a single valued function of x and x=a be a point in the domain of definition of the function The function is said to be continuous at x=a ifi) f(c) is defined ie f(x) has a definite finite value at x=cii) lim xrarra f(x) exists andiii) lim xrarra f(x) =f(a) In other words f(x) is said to be continuous at x=a if lim xrarra+ f(x)= lim xrarra- f(x) = f(a) Or f(a+0) =f(a-0) =f(a) Or lim hrarr0 f(a+h)= f(a) Algebra of continuous functionsNow we will study some algebra of continuous functions Theorem 1 Suppose f and g be two real functions continuous at a real number c Then(1) f + g is continuous at x = c(2) f ndash g is continuous at x = c(3) f g is continuous at x = c(4) (fg) is continuous at x = c (provided g (c) ne 0)

Example 1 Prove that every rational function is continuousSolution Recall that every rational function f is given byf(x)=[p(x) q(x) ] q(x)ne0where p and q are polynomial functions The domain of f is all real numbers except points at which q is zero Since polynomial functions are continuous f is continuous by (4) of Theorem 1Example 2Discuss the continuity of sine functionSolution To see this we use the following factslim xrarr0 sin x =0Now observe that f (x) = sin x is defined for every real number Let c be a real number Put x = c + h If x rarr c we know that h rarr 0 Therefore lim xrarrc f(x)

= lim xrarrc sin x= lim hrarr0 sin(c+h) =lim hrarr0 [sin c cos h + cos c sin h ]=lim hrarr0 (sin c cos h) + lim hrarr0 (cos c sin h) = sin c +0=sin c = f(c) Thus lim xrarrc f(x) = f(c) and hence f is a constant function Exercise Prove that the function f(x) = x2 +2x is continuous for every real value of x [Hints show that lim xrarra+ f(x) = lim xrarra- f(x) = f(a) ]

Biology Reproduction in Flowering plants We will discuss about megasporoangium

megasporagenesis and female gametophyte

Q4 Describe the structure of megasporangium

Ovule is attached to the placenta by astalk called funicle

Each ovule has one two or three protectivecoverings called integuments

At the tip of integuments a small openingcalled micropyle is organised

Opposite to the micropylar end is the chalaza

Within the integuments a mass of cellsnucellusand inside it embryo sac orfemale gametophyte is present

Q5 Describe a mature embryosacamp its formation

In most of the flowering plants only oneof the 4 megaspores formed as a result ofmegasporogenesis that is functional while theother three degenerate

The

functional megaspore develops into thefemale gametophyte

Formation The nucleus

of the functional megasporedivides mitotically to form two nuclei first andthen two more sequential mitotic nucleardivisions result in the formation of four ampthen eight nucleate stages of embryo sac

Six of the eight nucleus are surrounded bycell walls and organised into cells

The remaining two nuclei called polar nuclei are found below the egg apparatus in the largecentral cell

Three cells consisting of two synergids amp one egg cell present bottom of

embryo sac Three cells

at the chalazal as antipodal cells

Two polar nuclei together present in large central cell

HISTORY

TOWARDS INDEPENDENCE AND PARTITION THE LAST PHASE(1935-1947)SUB TOPIC NATIONAL MOVEMENTS DURING THE SECOND WORLD WAR

Spread of Quit India Movement On 9th August 1942Gandhiji and other Congress leaders were arrested The Congress was declared illegal The news of the arrest of all leaders marked the beginning of a widespread movement of India It was not possible for such a movement to remain peacefulBut the arrest of the all notable congress leaders virtually left the movement in the hands of the mass The movement took the form of violent and militant outbreakBesides congressmen revolutionaries also were very active in the movement The Congress Socialist group also played a prominent role

1 Question Why did the British authority arrest the Congress leaders on 9 th August 1942Answer Congress Working committee adopted the Quit India resolution which was to be ratified at the Bombay AICC meeting in 8th August 1942 They decided to launch a mass struggle on non-violent lines Gandhiji gave a clarion call to all section of the people rdquoKarenge ya Marengerdquo (do or die) Congress leaders gave the call to driving out

the British from IndiaViceroy had taken strong action against the Quit India movement Gandhiji and all the leaders of Congress were arrested

2 Question How did Quit India Movement spread out all over IndiaAnswer The news of the leaders lsquo arrest marked the beginning of a widespread movement to remain peacefulThe movement took form of violent outbreak There were widespread cutting of telephone and Telegraph wires damaging railway lines raising barricades in cities and towns and other forms of violent demonstations

Question Name the leaders of Congress

Socialist group played a prominent part Notable among the Jayprakash Narayan Rammonohar Lohia Aruna Asaf Ali

Political science

Topic-Franchise and Representation

Summary

The election commission

The Election Commission of India is an autonomous constitutional authority responsible for administering Union and State election processes in India The body administers elections to the Lok Sabha Rajya Sabha State Legislative Assemblies in India and the offices of the President and Vice President in the country

Functions of election commission-

India is a sovereign socialist secular democratic republic Democracy runs like a golden thread in the social economic and political fabric woven by the Constitution given by lsquoWe the People of Indiarsquo unto ourselves The concept of democracy as visualised by the Constitution pre-supposes the representation of the people in Parliament and State legislatures by the method of election The Supreme Court has held that democracy is one of the inalienable basic features of the Constitution of India and forms part of its basic structure The Constitution of India adopted a Parliamentary form of government Parliament consists of the President of India and the two Houses mdash Rajya Sabha and Lok Sabha India being a Union of states has separate state legislatures for each state State legislatures consist of the Governor and two Houses mdash Legislative Council and Legislative Assembly mdash in seven states namely Andhra Pradesh Telangana Bihar Jammu amp Kashmir Karnataka Maharashtra and Uttar Pradesh and of the Governor and the state Legislative Assembly in the remaining 22 states Apart from the above two out of the seven Union Territories namely National Capital Territory of Delhi and Puducherry also have their Legislative Assemblies

ExecutionShort notes-Election commissionFunctions of election commission

Homework- Learn

Computer

Science

Computer hardware NAND Gate

A NOT-AND operation is known as NAND operation It has n input (n gt= 2) and one output

Logic diagram

Truth Table

NOR Gate

A NOT-OR operation is known as NOR operation It has n input (n gt= 2) and one output

Logic diagram

Truth Table

XOR Gate

XOR or Ex-OR gate is a special type of gate It can be used in the half

adder full adder and subtractor The exclusive-OR gate is abbreviated as EX-OR gate or sometime as X-OR gate It has n input (n gt= 2) and one output

Logic diagram

Truth Table

XNOR Gate

XNOR gate is a special type of gate It can be used in the half adder full adder and subtractor The exclusive-NOR gate is abbreviated as EX-NOR gate or sometime as X-NOR gate It has n input (n gt= 2) and one output

Logic diagram

Truth Table

Physics

Chapter 1 Electric Field ( Electric Dipole) (Summary)

Here we will derive Expression of electric field at broad side

On position of dipole

Execution

Q With the help of a labelled diagram obtain an expression for the electric field intensity E at any point on the equitorial line ( broad-side on position) of an electric dipole

Ans

E1 E1sinθ

E θ P E1 θ

( r2+L2)12 E2 E

r E2 E2sinθ

-q θ L O L +qA B

Let us consider that the point P is situated on the right bisector of the dipole AB at a distance r meter from its midpoint O

Let E1 and E2 be the electric field intensities of the electric field at P due to charge +q and ndashq of the dipole resp The distance of P from each charge is ( r2+L2)12

So E1 = 14 πϵ q

(r 2+L 2) away from +q

E2 = 14 πϵ q

(r 2+L 2) towards ndashq

The magnitudes of E1 and E2 are equal but directions are different Now resolving E1 and E2 into two components parallel and perpendicular to AB we get

The components perpendicular to AB E1sinθ and E2sinθ cancel each other because they are equal and opposite

The components parallel to AB are E1cosθ and E2 cosθ are in same direction and add up

So resultant intensity of electric field at the point P is

E = E1cosθ + E2 cosθ

E = 14 πϵ q

(r 2+L 2) 2 cosθ

Now from fig we have cosθ =BOBP = L (r2+L2)12

So we get E = 14 πϵ 2qL ( r2+L2)32

Now electric dipole moment p= 2qL

So E = 14 πϵ p ( r2+L2)32

HW Find the expression of Electric field as done here but this time take r gtgt 2L

Also find the expression of torque experience by a dipole

(Hint Electric force experienced by charges of dipole in electric field is qE each Let θ be the angle which dipole makes with electric lines of force then perpendicular distance between two charges is 2Lsinθ Then torque = force x perp distance = qE x 2L sinθ So τ=pE sinθ where p =2qL )

STUDY MATERIAL

Class XIISubject Eng Literature (The Tempest ndash William Shakespeare) Topic Act IV Scene 1 Lines 84 to 133 (Iris hellip A contract of true love Be not too late ) Date 27th April 2020 (4th Period)

[Students should read the original play and also the paraphrase given in the school prescribed textbook]Summary Questions amp Answers

o Ceres soon appears and comes to know that she has been summoned to celebrate the contract of true love

o Ceres expresses her unwillingness to meet Venus and Cupid as she has shunned their company

o Ceres and Juno both bestow their blessings upon Ferdinand and Miranda with June gifting honour riches happiness in marriage and Ceres presents plenty of earthrsquos produce

o Iris summons the water-nymphs and reapers to come and celebrate a contract

(1) IRIS Of her society (Line 91-101)

Be not afraid I met her deity

Cutting the clouds towards Pathos and her sonDove-drawn with her Here thought they to have doneSome wanton charm upon this man and maidWhose vows are that no bed-right shall be paidTill Hymens torch be lightedmdashbut in vainMarss hot minion is returned againHer waspish-headed son has broke his arrowsSwears he will shoot no more but play with sparrowsAnd be a boy right out

(i) Where were Venus and Cupid seen flying How were they travelling Why did they want to join the marriage celebration of Ferdinand and Miranda

of true love

Venus and Cupid were seen flying through the air towards Paphos the famous city which is situated on the island of Cyprus They were travelling by air-borne chariot drawn by doves They certainly wanted to come here in order to play some amorous trick upon Ferdinand and Miranda who are under a vow not to gratify their physical desires till the holy ceremony of their marriage has been performed(ii) What have Venus and Cupid done after failing in their plan

After being failure of their plan Venus who is a very passionate deity and who is the mistress of Mars (the god of war) has gone back while here ill-tempered son Cupid has broken his arrows of love in his state of desperation(iii) What has Cupid firmly decided

Cupid is feeling so disappointed that he has firmly decided to shoot no more arrows to arouse love in human hearts but to spend his time playing with sparrows Thus he would now become just a boy and would give up his original function of shooting arrows on human beings to make them fall in love(iv) What vow had Ceres taken How did Ceres feel at the abduction

After the abduction of her daughter Prosperina by Pluto Ceres had taken a vow to always keep away from the disgraceful company of Venus and her blind son Cupid the god of love Ceres felt deeply distressed when Pluto had carried off her daughter and had made her his wife by force(v) Why has Ceres not forgiven Venus and her blind son For what do Ceres want to be sure

As the abduction had been manipulated by Venus the goddess of beauty and love and her blind son Cupid Ceres has never forgiven them for their part in the whole plot Ceres wants to be sure that she would not have to meet Venus and Cupid who had engineered the abduction of her daughter Prosperina

AS THIS lsquoMASQUErsquo SCENE IS VERY IMPORTANT IN THE PLAY THE PARAPHRASE OF THE ENTIRE PORTION OF MASQUE SCENE (Act IV Lines 58 to 143) IS GIVEN BELOW

IRIS Goddess of RainbowCERES Goddess of Agriculture and all the fruits of the earth

(Nature growth prosperity rebirth ndash notions intimately connected to marriage)JUNO The majestic Queen of Heavens and wife of Jupiter (Jupiter is the king of Gods)

VENUS The Goddess of love CUPID Son of Venus PLUTO God of death (In the play referred by Shakespeare as lsquoDisrsquo which is a Roman name for Pluto)

ORIGINAL TEXT PARAPHRASEPROSPEROWellmdash

PROSPERONow come Ariel Let there be too many rather than too few

Now come my Ariel Bring a corollaryRather than want a spirit Appear and pertly[to Ferdinand and Miranda]No tongue all eyes Be silent

spirits in attendance Appear briskly

[to Ferdinand and Miranda]Look with your eyes but do not say a word

[Soft music] [Soft music][Enter Iris] [Enter Iris]

IRISCeres most bounteous lady thy rich leasOf wheat rye barley vetches oats and peasThy turfy mountains where live nibbling sheepAnd flat meads thatched with stover them to keepThy banks with pioned and twilled brimsWhich spongy April at thy hest betrimsTo make cold nymphs chaste crowns and thybroom-grovesWhose shadow the dismissegraved bachelor lovesBeing lass-lorn thy pole clipped vineyardAnd thy sea-marge sterile and rocky-hardWhere thou thyself dost airmdashthe Queen othrsquoSkyWhose watery arch and messenger am IBids thee leave these and with her sovereign grace[Juno appears] Here on this grass-plot in this very placeTo come and sport Her peacocks fly amainApproach rich Ceres her to entertain

IRISCeres most generous lady you are the cause of rich fields or fertile land where wheat rye barley beans oats and peas grow the grassy mountains where the sheep graze and the flat meadows covered with coarse hay to be used as fodder for cattleYour banks are covered with marsh-marigolds and reeds and the rainy April under your orders brings forth to make for the maids who are not in love beautiful crowns your woods where the broom flourishes and where the bachelor who has been dismissed by the maid he loved lies down being forsaken your vineyard in which the poles are embraced by the vines and the margin of the sea which is barren and rocky where you roam about to enjoy the fresh air ndash the queen of the sky (Juno) whose messenger I am besides being represented as the rainbow bids you leave all these and with her majesty here on this grassy plot in this very place come and sport her peacocks carry her fast in her chariot through the air and are making their way here approach rich Ceres to welcome her

[Enter Ariel as Ceres] [Enter Ariel as Ceres]

CERESHail many-coloured messenger that neerDost disobey the wife of JupiterWho with thy saffron wings upon my flowersDiffusest honey-drops refreshing showersAnd with each end of thy blue bow dost crownMy bosky acres and my unshrubbed downRich scarf to my proud earth Why hath thy queenSummoned me hither to this short-grassed green

CERESWelcome rainbow that never dared disobey Juno the wife of Jupiter who with your orange coloured rays spread honey-drops refreshing showers And with each end of thy blue bow drown my bushy acres and my hilly country which is free from shrubs you thus forming a rich scarf Why has your queen called me here to this place covered with short grass

IRISA contract of true love to celebrateAnd some donation freely to estateOn the blest lovers

IRISI have called you to celebrate a contract of true love and bestow some liberal gift upon the blessed lovers

ORIGINAL TEXT PARAPHRASECERESTell me heavenly bowIf Venus or her son as thou dost knowDo now attend the queen Since they did plotThe means that dusky Dis my daughter gotHer and her blind boys scandaled companyI have forsworn

CERESTell me heavenly bow if Venus the Goddess of love or Cupid her son and pedlar of passion at this time attend the heavenly queen Juno because you are sure to know Since the day they conspired against me and dark Pluto took away my daughter here and Cupidrsquos disgraceful company I have left off

IRISOf her societyBe not afraid I met her deityCutting the clouds towards Pathos and her sonDove-drawn with her Here thought they to have doneSome wanton charm upon this man and miad

IRISBe not afraid of her company I met her deity moving on the clouds towards Paphos the sacred home of Venus on the island of Cyprus along with her son on her chariot drawn by doves Here they contemplated to exercise a charm upon this man and maid producing

Whose vows are that no bed-right shall be paidTill Hymens torch be lightedmdashbut in vainMarss hot minion is returned againHer waspish-headed son has broke his arrowsSwears he will shoot no more but play with sparrowsAnd be a boy right out

wantonness before the actual marriage ceremony but did not succeed Venus has returned her irritable son has broken his arrows and swears that he will give up his practice of trying to inspire love but play with sparrows and be a boy again

[Music is heard] [Music is heard]

CERESHighst queen of stateGreat Juno comes I know her by her gait

CERESHighest queen of state Great Juno there she comes I know here by her gait

[Enter Juno] [Enter Juno]

JUNOHow does my bounteous sister Go with meTo bless this twain that they may Prosperous beAnd honoured in their issue

JUNOHow are you doing my generous sister Come with me to bless this couple so that they may be prosperous and fortunate in their children

[They sing] [They sing]

JUNOHonour riches marriage-blessingLong continuance and increasingHourly joys be still upon youJuno sings her blessings upon you

JUNOMay honour riches happiness in marriage long continuance and increase of those boons ever rest upon you as hourly joys Juno showers down upon you her blessings in song

CERESEarths increase foison plentyBarns and garners never emptyVines and clustring bunches growingPlants and goodly burden bowingSpring come to you at the farthestIn the very end of harvestScarcity and want shall shun youCeresrsquo blessing so is on you

CERESMay you have the plenty of earthrsquos produce Your barns and granaries may never be empty Your vines may grow with clustering bunches Your fruit trees may be heavily laden with their fruit May there be continuous spring and harvest May scantiness and want leave you forever Such is the blessing of Ceres upon you

FERDINANDThis is a most majestic vision andHarmoniously charmingly May I be boldTo think these spirits

FERDINANDThis is a great vision and magically melodious Should I suppose the characters (taking part in the masque) are spirits

PROSPEROSpirits which by mine artI have from their confines calld to enactMy present fancies

PROSPEROYes they are spirits whom I have summoned from the regions to which they are confined to carry into effect my fanciful designs

ORIGINAL TEXT PARAPHRASEFERDINANDLet me live here everSo rare a wondered father and a wifeMakes this place paradise

FERDINANDI should like to live here forever Such a wise and wonderful father makes this place a paradise

[Juno and Ceres whisper and send Iris on employment] [Juno and Ceres whisper and send Iris on employment]

PROSPEROSweet now silence

PROSPEROMy dear Ferdinand speak no more Juno and Ceres are

Juno and Ceres whisper seriouslyTheres something else to do Hush and be muteOr else our spell is marred

whispering with a solemn look There is something else coming Silence Or else our magic will be spoilt

IRISYour nymphs called naiads of the wandering brooksWith your sedged crowns and over-harmless looksLeave your crisp channels and on this green landAnswer your summons Juno does commandCome temperate nymphs and help to celebrateA contract of true love Be not too late

IRISYou nymphs called Naiads denizens (M inhabitants) of the running stream with your chaplets of sedge and ever-helpful looks leave your wrinkled channels and on the green land answer the summons sent to you Juno has ordered some chaste nymphs and help to celebrate a noble and true marriage Donrsquot delay

[Enter certain nymphs] [Enter certain nymphs]You sunburnt sicklemen of August wearyCome hither from the furrow and be merryMake holiday your rye-straw hats put onAnd these fresh nymphs encounter every oneIn country footing

You sunburnt harvesters weary from the effects of the heat in August come here from the furrowed land and rejoice Make holiday with your rye-straw hats upon you and meet these fresh nymphs and join in country dancing

[Enter certain reapers properly habited They join with the nymphs in a graceful dance towards the end whereof Prospero starts suddenly and speaks]

[Enter certain reapers properly habited They join with the nymphs in a graceful dance towards the end whereof Prospero starts suddenly and speaks]

PROSPERO[aside] I had forgot that foul conspiracyOf the beast Caliban and his confederatesAgainst my life The minute of their plotIs almost come [to the spirits]Well done Avoidno more

PROSPERO(Aside)I had forgotten the wicked conspiracy of the beast Caliban and his accomplices against my life the time of their plot has almost arrived ndash (To the Spirits) well done depart no more of this

[To a strange hollow and confused noise the spirits heavily vanish]

[The spirits depart]

ORIGINAL TEXT PARAPHRASEFERDINANDLet me live here everSo rare a wondered father and a wifeMakes this place paradise

FERDINANDI should like to live here forever Such a wise and wonderful father makes this place a paradise

[Juno and Ceres whisper and send Iris on employment] [Juno and Ceres whisper and send Iris on employment]

PROSPEROSweet now silenceJuno and Ceres whisper seriouslyTheres something else to do Hush and be muteOr else our spell is marred

PROSPEROMy dear Ferdinand speak no more Juno and Ceres are whispering with a solemn look There is something else coming Silence Or else our magic will be spoilt

IRISYour nymphs called naiads of the wandering brooksWith your sedged crowns and over-harmless looksLeave your crisp channels and on this green landAnswer your summons Juno does commandCome temperate nymphs and help to celebrateA contract of true love Be not too late

IRISYou nymphs called Naiads denizens (M inhabitants) of the running stream with your chaplets of sedge and ever-helpful looks leave your wrinkled channels and on the green land answer the summons sent to you Juno has ordered some chaste nymphs and help to celebrate a noble and true marriage Donrsquot delay

[Enter certain nymphs] [Enter certain nymphs]You sunburnt sicklemen of August wearyCome hither from the furrow and be merryMake holiday your rye-straw hats put onAnd these fresh nymphs encounter every oneIn country footing

You sunburnt harvesters weary from the effects of the heat in August come here from the furrowed land and rejoice Make holiday with your rye-straw hats upon you and meet these fresh nymphs and join in country dancing

[Enter certain reapers properly habited They join with the nymphs in a graceful dance towards the end whereof Prospero starts suddenly and speaks]

[Enter certain reapers properly habited They join with the nymphs in a graceful dance towards the end whereof Prospero starts suddenly and speaks]

PROSPERO[aside] I had forgot that foul conspiracyOf the beast Caliban and his confederatesAgainst my life The minute of their plotIs almost come [to the spirits]Well done Avoidno more

PROSPERO(Aside)I had forgotten the wicked conspiracy of the beast Caliban and his accomplices against my life the time of their plot has almost arrived ndash (To the Spirits) well done depart no more of this

[To a strange hollow and confused noise the spirits heavily vanish]

[The spirits depart]

Ac-12 27420 topic Revaluation of Assets and Liabilities

REVALUATION OF ASSETS AND LIABILITIES

On admission of a new partner the firm stands reconstituted and consequently the assets are revalued and liabilities are reassessed It is necessary to show the true position of the firm at the time of admission of a new partner If the values of the assets are raised gain will increase the capital of the existing partners Similarly any decrease in the value of assets ie loss will decrease the capital of the existing partners For this purpose alsquoRevaluation Accountrsquo is prepared This account is credited with all increases in the value of assets and decrease in the value of liabilities It is debited with decrease on account of value of assets and increase in the value of liabilities The balance of this account shows a gain or loss on revaluation which is transferred to the existing partnerrsquos capital account in existing profit sharing ratioAccounting for Revaluation of Assets and Liabilities when there is a Changein the Profit Sharing Ratio of Existing PartnersAssets and liabilities of a firm must also be revalued at the time of change in profit sharing ratio of existing partners The reason is that the realisable or actual value of assets and liabilities may be different from those shown in the Balance Sheet It is possible that with the passage of time some of the assets might have appreciated in value while the value of certain other assets might have decreased and no record has been made of such changes in the books of accounts Similarly there may be some unrecorded assets amp libilities that may have to be accounted for Revaluation of assets and reassessments of liabilities becomes necessary because the change in the

value of assets and liabilities belongs to the period to change in profit sharing ratio and hence must be shared by the partners in their old profit sharing ratio Revaluation of assets and reassessment of liabilities may be given effect to in two different ways (a) When revised values are to be recorded in the books and(b) When revised values are not to be recorded in the books

When revised values are to be recorded in the booksIn such a case revaluation of assets and reassessment of liabilities is done with the help of a new account called lsquoRevaluation Accountrsquo Sometimes this account is also called as lsquoProfit amp Loss Adjustment Acrsquo If there is a loss due to revaluation revaluation account is debited and if the revaluation results in a profit the revaluation account is credited The following journal entries made for this purpose are

(i) For increase in the value of assetsAsset Ac Dr (individually)To Revaluation Ac(ii) For decrease in the value of AssetRevaluation Ac Dr (individually)To Asset Ac[Decrease in the value of assets](iii) For increase in the value of LiabilitiesRevaluation Ac Dr (individually)To Liabilities Ac[Increase in the value of Liabilities](iv) For decrease in the value of LiabilitiesLiabilities Ac DrTo Revaluation Ac[Decrease in the value of Liabilities](v) For unrecorded AssetsAsset Ac [unrecorded] DrTo Revaluation Ac[Unrecorded asset recorded at actual value](vi) For unrecorded Liability Revaluation Ac DrTo Liability Ac [unrecorded][Unrecorded Liability recorded at actual value](vii) For transfer of gain on revaluationRevaluation Ac DrTo Existing Partnerrsquos CapitalCurrent Ac[Profit on revaluation transferred to capital account in existing ratio](viii) For transfer of loss on revaluationExisting Partnerrsquos CapitalCurrent Ac DrTo Revaluation Ac[Loss on revaluation transferred to capital account in existing ratio](a) When revaluation account shows gain Revaluation Ac DrTo Partnerrsquos Capital Ac (Old Profit Sharing Ratio)(Profit on revaluation credited to Partnerrsquos Capital Ac)(b) Above entry is reversed when revaluation account shows loss Partners Capital Acs (Old Profit Sharing Ratio) DrTo Revaluation Ac(Loss on revaluation debited to Partnerrsquos Capital Acs)

Proforma of Revaluation Account is given as under

Revaluation Account

Dr Cr Particulars ` Amount Particulars ` Amount To Decrease in value of assets By Increase in value of assets To Increase in value of liabilities By Decrease in value of liabilities To Unrecorded liabilities By Unrecorded assets To Gain on Revaluation (Transferred) By Loss on Revalution (Transferred)

ECO ndash12 2742020Topic- ELASTICITY OF DEMAND

CHAPTER - ELASTICITY OF DEMANDMEANINGDemand for a commodity is affected by many factors such as its price price of related goods income of its buyer tastes and preferences etc Elasticity means degree of response Elasticity of demand means degree of responsiveness of demand Demand for a commodity responds to change in price price of related goods income etc So we have three dimensions of elasticity of demandDIMENSION OF ELASTICITY OF DEMAND TYPES OF ELASTICITY OF DEMAND

Price elasticity of demand Income elasticity of demand Cross Elasticity of demand

Price elasticity of demand Price elasticity of demand means degree of responsiveness of demand for a commodity to the change in its price For example if demand for a commodity rises by 10 due to 5 fall in its price Price elasticity of demand (ep)=Percentage change in quantity demanded Percentage change in price of the commodity = 10 ( -)5 = ( - )2Note that ep will always be negative due to inverse relationship of price and quantity demanded

(ii) Income elasticity of demand Income elasticity of demand refers to the degree of responsiveness of demand for a commodity to the change in income of its buyer Suppose income of buyer rises by 10 and his demand for a commodity rises by 20 then Income elasticity of demand (ey)= change in quantity demanded change in price of the commodity =20 10 = 2

Cross Elasticity of demandCross elasticity of demand means the degree of responsiveness of demand for a commodity to the change in price of its related goods (substitute goods or complementary goods) Suppose demand for a commodity rises by 10 due to 5 rise in price of its substitute good then Cross elasticity of demand (ec) = change in quantity demanded change in price of related good = 10 2 = 5 (Tastes and preferences cannot be expressed numerically So elasticity ofdemand cannot be numerically expressed)

  • Chapter 1 Force (Summary)
  • Distinguish between external sovereignty and internal sovereignty
    • NAND Gate
      • Logic diagram
      • Truth Table
        • NOR Gate
          • Logic diagram
          • Truth Table
            • XOR Gate
              • Logic diagram
              • Truth Table
                • XNOR Gate
                  • Logic diagram
                  • Truth Table
                      • Physics
                      • Chapter 1 Electric Field ( Electric Dipole) (Summary)
Page 30:  · Web viewWe all know that Nouns are divided into two parts: common noun and proper noun.Apart from common and proper noun, we will also study about collective noun and compound

consumption value

b Value ndashin-exchange It relates to market value of a commodity

It is the rate at which a particular good or service can be exchanged for moneyFor example in barter system if a person is prepared to exchange 3 metres of cloth with 1 pair of shoes then the value in exchange of 3 metres of cloth is 1 pair of shoesValue in exchange is the power of purchasing other goods In modern monetised economies the exchange value of goods are expressed in terms of money as prices

Now let us discuss the term lsquoWealthrsquo

Wealth refers to the stock of all those assets which are a source of income

Wealth is a stock concept

Wealth must possess the following features

a Utility It must possess utility or give some

of shoes then the value in exchange of 3 metres of cloth is 1 pair of shoes

Value in exchange is the power of purchasing other goods In modern monetised economies the exchange value of goods are expressed in terms of money as prices

4 What is wealth

Answer it refers to the stock of assets or goods which are a source of income and have personal or national ownership

5 What are the features of wealth

Answer The features of wealth are as follows

Wealth must possess the following features

a Utility It must possess utility or give some satisfaction

b Scarcity It must be limited in quantityc Transferability it should be transferable its

ownership can be transferred from one person to another person

d Exchange value It must possess exchange value

6 What is welfare

Answer

Welfare is defined as satisfaction and happiness a sense of well- being among the people

satisfactionb Scarcity It must be

limited in quantityc Transferability It

should be transferable its ownership can be transferred from one person to another person

d Exchange value It must possess exchange value

Now let us discuss the term lsquoWelfarersquo

Welfare is defined as satisfaction and happiness a sense of well- being among the people

Welfare is affected by factors like

a Consumption of goods and services

b Environment

c Family relations

d Degree of freedom

e Law and order situation

Mathematics Trigonometric equation

To find the general solution of the equation sinθ=0

When sin θ =0

Then θ= 0 π2π 3π-π -2π -3

i e when θ = 0 or an integral multiple of π

i e when θ= nπ where n is any integer

Therefore the general solution of the equation sin

Example1 Find the general values of θ which satisfy the equation sin2 θ =34

Solution sin2 θ= 34

Or sin θ = +34 or -34

Or sin θ = sin π3 or sin (-π3)

Therefore

θ = [nπ + (-1) n (π3)] or[ nπ+ (-1) n (-π3)]

= nπ +π3 or nπ-π3 where n= any integer

Example 2Find the values of θ which satisfy tan2 θ

θ=0 is θ= nπ where n is any integer

To find the general solution of the equation cos θ=0

When cos θ=0

Then θ=π2 3π2 5π2 -π2 -3π2 -5π3

i e when θ is an odd multiple of π2

i e when θ=(2n+1) π2 where n= any integer

Therefore the general solution of the equation cos θ =0 is θ= (2n+1) π2 where n= any integer

To find the general solution of the equation tan θ = 0

Clearly tan θ =0 implies sin θcos θ =0

Therefore θ = nπ

i e the general solution of the equation tan θ=0 is θ =nπ where n = any integer

To find the general solution of the equation cot θ =0

Clearly cot θ =0 implies (cos θsin θ) = 0

i e cos θ =0

Therefore θ = (2n+1) π2

Therefore the general solution of the equation cot θ =0 is θ = (2n+1) π2

Where n= any integer

To find the general solution of the equation sin θ= k (-

=13 -πleθleπ

Solution tan2 θ =13

Or tan θ = plusmn1radic(3) =tan(plusmnπ6)

θ=nπ plusmn π6 where n =any integer

If n=0 then θ=plusmnπ6

If n=1 then θ= π plusmn π6

If n=-1 then θ= -π plusmn π6

Therefore the required solution in -π le θ le π are θ= π6 5π6 -π6 -5π6

Exercise Find general solution of sin 2θ=cos θ [Hints Use sin 2θ= 2sin θcosθ and then take cosθ

common]

1lek le1)

Determine an angle alpha such that sin =k and -π2le αle π2

Then we have

Sin θ = k = sin α

Or sin θ - sin α =0

Or 2 cos [(θ+α) 2] sin [(θ-α) 2] =0

Therefore either cos [(θ +α) 2] =0 (1)

Or sin [(θ-α) 2] =0 (2)

Now from (1) we get (θ+α) 2= (2m+1) π2)

Or θ = (2m+1) π-α (3)

And from (2) we get (θ-α) 2 =mπ

Or θ= 2mπ+α(4)

Where m = any integer

Clearly the solution (3) amp (4) may be combined in the following form

θ= nπ+(-1) n α where n= any integer

Therefore the general solution of sin θ = sin α is θ = nπ +(-1) n α where n is any integer and -π2 le α le π2

Biology Chapter - 04Kingdom Monera

Today we will discuss about bacterial reproduction and its usefulness

Fig Binary Fission

Fig Conjugation Fig Transformation

Bacterial reproduction is mainly asexual but sexual reproduction

also takes place

Asexual reproduction takes place by i) Binary fission - from one bacteriato

two bacteria are produced in every 20 to 30mins

ii) Buddingiii) Endospore formation - during

unfourable condition

Sexual reproduction by three ways

1) Conjugation - Transfer of genetic material between cells that are in physical contact with one another

2) Transduction - Transfer of genetic materialfrom one cell to another by a bacteriophage

3) Transformation - Transfer of cell-freeor naked DNArsquo from one cell to another

Bacteria causes different diseases inplants animals and human and

it causes food spoilage and waterpollution but it also have some useful

activities

i) Bacteria are helpful in sewage water treatment

ii) It is used in antibiotic (medicine) production

iii) Anaerobic bacteria help in biogas(energy) production

iv) Many household products like yoghurt cheese are manufactured by use of bacteria

v) Rhizobium by symbiotic relationship with leguminous plant increase soil fertility

vi) Besides these bacteria is helpful in genetic engineering degradation of petroleum hydrocarbonand in dairy

industry

Physics Motion in plane Here we will introduce Projectile Motion

Execution

Projectile

Y

usinθ u h

θX

ucosθ

Suppose a body is projected with an angle θ So initial velocity u can be resolved into two components

Horizontal component - ucosθ ( for range)

Vertical component - usinθ ( for height)

usinθ changes during motion and becomes zero at maximum height position but ucosθ remain unchanged

The maximum height of projectile is h

NB If initial is upward then g = -ve and if it is downward then g = +ve Height is +ve if direction of motion does not change ( for ex a body thrown upwards but goes down ultimately then height h = -ve)

The angle of projectile θ is the angle made with horizontal

HISTORY ndash GROWTH OF NATIONALISM

SUB TOPIC- REVOLUTIONARY NATIONALISM Bengal formation of Anushilan Samity and Jugantar Group

The intensification of the Swadeshi movement and Government policy of terror and repression led to outbreak of violence Bombs were manufactured and attempts on the lives of unpopular Government officials became frequent In the gymnasium of Scottish Church College which was known as General Assemblies Institution a secret society was formed known as Anushilan Samity

Aurobindo Ghosh send from Baroda his emissary Jatindranath Banerjee to mobilize the Bengal revolutionaries

Hemchandra Qanungo and Satyen Bose published Journal Jugantar

The Jugantar group planned to assassinate oppressive magistrate Kingsford by Khudiram Bose and Prafulla Chaki in 1908 Prafulla Chaki committed suicide to avoid arrest Khudiram was tried and hanged

Afew days later the police found a bomb factory in Maniktala and arrested a large number of revolutionaries The trial of revolutionaries became famous as the Alipore Bomb Case

In the course of the trial the approver the public prosecuter and a police officer were assassinated

1 Question Name two journals which preached the cult of violence

Answer a) Yugantar edited by Bhupendranath DuttaB) Bandemataram edited by Aurobindo Ghosh2 Question Why was Khudiram arrested and hangedAnswer An attempt was made to assassinate a hated vindictive majistrate named Kingsford by Khudiram Bose and Prafulla Chaki Their attempt failed and the bomb they threw killed two English ladies Khudiram was arrested and put to trial and then hanged3Question Who was Aurubindo GhoshAnswer Aurobindo Ghosh a nationalist revolutionary who was charged for his involvement in the Alipore Bombing Case He was accused of it along with his brother Barindra nath Ghosh But Aurobindo was acquitted because of the brilliant pleading of his counsel Chittaranjan Das Then he became a spiritual reformer introducing his visions on human progress and spiritual evolution4 Qustion Who was KingsfordAnswer Kingsford was an unpopular British chief Magistrate who was the target of the bomb thrown at Muzaffarpur by Khudiram and Prafulla Chaki

Most of the accused were convicted and sentenced to

long term of imprisonmentBut

Aurobindo Ghosh was acquitted mainly owing to the brilliant pleading of his counsel Chittaranjan Das

Political science Topic-Sovereignty

Summary Sovereignty is the full right and power of a governing body over itself without any interference from outside sources or bodies In political theory sovereignty is a substantive term designating supreme legitimate authority over some polity In international law sovereignty is the exercise of power by a state

Internal Sovereignty

Internal sovereignty means supreme authority within ones territory while external sovereignty relates to the recognition on the part of all states that each possesses this power in equal measure

External sovereignty

external sovereignty relates to the recognition on the part of all states that each possesses this power in equal measure

Distinguish between

Execution

Answer the following questions

Short notes-

Sovereignty

Internal Sovereignty

External sovereignty

Homework- learn

external sovereignty and internal sovereigntySovereignty is the principle

of supreme and

unquestionable authority

reflected in the claim by the

state to be the sole author of

laws within its territory

Definition of external vs internal sovereigntyInternal sovereignty refers to

the relationship between a

sovereign power and its

subjects ndash it refers to the

location of the supreme

authority within the state In

the UK for example internal

sovereignty (supposedly)

resides within Parliament

reflected in the

constitutional principle of

parliamentary

sovereigntyBy contrast

external sovereignty refers

to the capacity of the state

to act independently and

autonomously on the world

stage This is what is

sometimes called lsquostate

sovereigntyrsquo or lsquonational

sovereigntyrsquo and implies

that states are legally equal

and that the territorial

integrity and political

independence of a state is

inviolable

Class ndash XII

Date - 2742020 STUDY MATERIALSubject Topic Summary Execution Business Studies

Job Analysis amp Manpower Planning

At first let us recall the chapter what we have discussed till nowbullJob analysisbullJob specification bullJob description bullJob enlargement bullJob enrichment

Today we will do some questions answers from the chapter

Questions 1ldquoJob analysis job description and job specification are interrelatedrdquo Comment Answer) Job analysis is a systematic and detailed examination of a job to collect all the relevant information about it The contents off the job are summarised in the job description The qualification needed for the job are summarised in job specificationThus there is close interrelationship between job analysis job description and job specification

Question 2ldquoJob enlargement is a horizontal extension of a job whereas job enrichment is a vertical extension of a jobrdquo ElucidateAnswer) Job enlargement involves adding one or more task to a job coma where as job enrichment involves adding more autonomy and responsibility to a job Job enlargement is therefore horizontal extension of a job coma whereas job enrichment is a vertical extension of a job

Question 3 )

What is manpower estimation Explain its quantitative and qualitative aspectsAnswer) Manpower estimation is the process by which management determines how an organisation should move from itrsquos current manpower positionto its desired manpower position There are two dimensions of Manpower estimation- quantitative and qualitative

Quantitative aspectThis aspect of Manpower estimation involves estimating the number of employees required in a future time period Workload analysis and workforce analysis are done to estimate the quantity of required manpower

Qualitative aspectThe estimate of the knowledge skills experience etc of required manpower is the qualitative aspect of Manpower estimation The quality of Manpower can be judged on the basis of job analysisand job specification

COMMERCE

CAPITAL-FIXED AND WORKING

Today let us start the class by discussing the sources of finance for different types of business firms

The term lsquocapitalrsquo refers to the investment made in the enterprise for the purpose of earning profits

Requirements of capital and sources of capital for different types of business firms are

1 Capital for sole proprietorship businessA sole proprietor operates at a small scale and thereforerequires a limited amount of capital

2 Capital for partnership firmCapital requirements as well as capital base of a partnership is bigger than that of a sole trader businessThe owned capital is contributed by the partners in an agreed ratio

3 Capital for joint stock companyA joint stock company generally requires large amount of capitalA public company can raise huge capital through issue of shares In addition to share capital it can utilize retained profits

Now let u discuss the meaning of Finance PlanningFinance planning is the process of estimation the financial requirements of an organization specifying the sources of firms and ensuring that enough funds are available at the right time

1 What do you mean by Finance PlanningAnswer Finance planning is the process of estimation the financial requirements of an organization specifying the sources of firms and ensuring that enough funds are available at the right time

2Discuss the role of financial planning of an enterpriseThe role of financial planning are as followsa A sound financial plan helps a business enterprise to avaid the problems of shortage and surplus of fundsbFinancial planning serves as a guide in developing a sound capital structure so as to maximize returns to shareholders c It helps in effective utilization of fundsd It provides policies and procedures for coordinating different functional areas or departments of businesse It enables the management to exercise effective control over the financial activities of an enterprisef It helps the company to prepare for facing business shocks and surprises in future

Mathematics

Continuity and differentiability

Recall Definition of ContinuityLet f(x) be a single valued function of x and x=a be a point in the domain of definition of the function The function is said to be continuous at x=a ifi) f(c) is defined ie f(x) has a definite finite value at x=cii) lim xrarra f(x) exists andiii) lim xrarra f(x) =f(a) In other words f(x) is said to be continuous at x=a if lim xrarra+ f(x)= lim xrarra- f(x) = f(a) Or f(a+0) =f(a-0) =f(a) Or lim hrarr0 f(a+h)= f(a) Algebra of continuous functionsNow we will study some algebra of continuous functions Theorem 1 Suppose f and g be two real functions continuous at a real number c Then(1) f + g is continuous at x = c(2) f ndash g is continuous at x = c(3) f g is continuous at x = c(4) (fg) is continuous at x = c (provided g (c) ne 0)

Example 1 Prove that every rational function is continuousSolution Recall that every rational function f is given byf(x)=[p(x) q(x) ] q(x)ne0where p and q are polynomial functions The domain of f is all real numbers except points at which q is zero Since polynomial functions are continuous f is continuous by (4) of Theorem 1Example 2Discuss the continuity of sine functionSolution To see this we use the following factslim xrarr0 sin x =0Now observe that f (x) = sin x is defined for every real number Let c be a real number Put x = c + h If x rarr c we know that h rarr 0 Therefore lim xrarrc f(x)

= lim xrarrc sin x= lim hrarr0 sin(c+h) =lim hrarr0 [sin c cos h + cos c sin h ]=lim hrarr0 (sin c cos h) + lim hrarr0 (cos c sin h) = sin c +0=sin c = f(c) Thus lim xrarrc f(x) = f(c) and hence f is a constant function Exercise Prove that the function f(x) = x2 +2x is continuous for every real value of x [Hints show that lim xrarra+ f(x) = lim xrarra- f(x) = f(a) ]

Biology Reproduction in Flowering plants We will discuss about megasporoangium

megasporagenesis and female gametophyte

Q4 Describe the structure of megasporangium

Ovule is attached to the placenta by astalk called funicle

Each ovule has one two or three protectivecoverings called integuments

At the tip of integuments a small openingcalled micropyle is organised

Opposite to the micropylar end is the chalaza

Within the integuments a mass of cellsnucellusand inside it embryo sac orfemale gametophyte is present

Q5 Describe a mature embryosacamp its formation

In most of the flowering plants only oneof the 4 megaspores formed as a result ofmegasporogenesis that is functional while theother three degenerate

The

functional megaspore develops into thefemale gametophyte

Formation The nucleus

of the functional megasporedivides mitotically to form two nuclei first andthen two more sequential mitotic nucleardivisions result in the formation of four ampthen eight nucleate stages of embryo sac

Six of the eight nucleus are surrounded bycell walls and organised into cells

The remaining two nuclei called polar nuclei are found below the egg apparatus in the largecentral cell

Three cells consisting of two synergids amp one egg cell present bottom of

embryo sac Three cells

at the chalazal as antipodal cells

Two polar nuclei together present in large central cell

HISTORY

TOWARDS INDEPENDENCE AND PARTITION THE LAST PHASE(1935-1947)SUB TOPIC NATIONAL MOVEMENTS DURING THE SECOND WORLD WAR

Spread of Quit India Movement On 9th August 1942Gandhiji and other Congress leaders were arrested The Congress was declared illegal The news of the arrest of all leaders marked the beginning of a widespread movement of India It was not possible for such a movement to remain peacefulBut the arrest of the all notable congress leaders virtually left the movement in the hands of the mass The movement took the form of violent and militant outbreakBesides congressmen revolutionaries also were very active in the movement The Congress Socialist group also played a prominent role

1 Question Why did the British authority arrest the Congress leaders on 9 th August 1942Answer Congress Working committee adopted the Quit India resolution which was to be ratified at the Bombay AICC meeting in 8th August 1942 They decided to launch a mass struggle on non-violent lines Gandhiji gave a clarion call to all section of the people rdquoKarenge ya Marengerdquo (do or die) Congress leaders gave the call to driving out

the British from IndiaViceroy had taken strong action against the Quit India movement Gandhiji and all the leaders of Congress were arrested

2 Question How did Quit India Movement spread out all over IndiaAnswer The news of the leaders lsquo arrest marked the beginning of a widespread movement to remain peacefulThe movement took form of violent outbreak There were widespread cutting of telephone and Telegraph wires damaging railway lines raising barricades in cities and towns and other forms of violent demonstations

Question Name the leaders of Congress

Socialist group played a prominent part Notable among the Jayprakash Narayan Rammonohar Lohia Aruna Asaf Ali

Political science

Topic-Franchise and Representation

Summary

The election commission

The Election Commission of India is an autonomous constitutional authority responsible for administering Union and State election processes in India The body administers elections to the Lok Sabha Rajya Sabha State Legislative Assemblies in India and the offices of the President and Vice President in the country

Functions of election commission-

India is a sovereign socialist secular democratic republic Democracy runs like a golden thread in the social economic and political fabric woven by the Constitution given by lsquoWe the People of Indiarsquo unto ourselves The concept of democracy as visualised by the Constitution pre-supposes the representation of the people in Parliament and State legislatures by the method of election The Supreme Court has held that democracy is one of the inalienable basic features of the Constitution of India and forms part of its basic structure The Constitution of India adopted a Parliamentary form of government Parliament consists of the President of India and the two Houses mdash Rajya Sabha and Lok Sabha India being a Union of states has separate state legislatures for each state State legislatures consist of the Governor and two Houses mdash Legislative Council and Legislative Assembly mdash in seven states namely Andhra Pradesh Telangana Bihar Jammu amp Kashmir Karnataka Maharashtra and Uttar Pradesh and of the Governor and the state Legislative Assembly in the remaining 22 states Apart from the above two out of the seven Union Territories namely National Capital Territory of Delhi and Puducherry also have their Legislative Assemblies

ExecutionShort notes-Election commissionFunctions of election commission

Homework- Learn

Computer

Science

Computer hardware NAND Gate

A NOT-AND operation is known as NAND operation It has n input (n gt= 2) and one output

Logic diagram

Truth Table

NOR Gate

A NOT-OR operation is known as NOR operation It has n input (n gt= 2) and one output

Logic diagram

Truth Table

XOR Gate

XOR or Ex-OR gate is a special type of gate It can be used in the half

adder full adder and subtractor The exclusive-OR gate is abbreviated as EX-OR gate or sometime as X-OR gate It has n input (n gt= 2) and one output

Logic diagram

Truth Table

XNOR Gate

XNOR gate is a special type of gate It can be used in the half adder full adder and subtractor The exclusive-NOR gate is abbreviated as EX-NOR gate or sometime as X-NOR gate It has n input (n gt= 2) and one output

Logic diagram

Truth Table

Physics

Chapter 1 Electric Field ( Electric Dipole) (Summary)

Here we will derive Expression of electric field at broad side

On position of dipole

Execution

Q With the help of a labelled diagram obtain an expression for the electric field intensity E at any point on the equitorial line ( broad-side on position) of an electric dipole

Ans

E1 E1sinθ

E θ P E1 θ

( r2+L2)12 E2 E

r E2 E2sinθ

-q θ L O L +qA B

Let us consider that the point P is situated on the right bisector of the dipole AB at a distance r meter from its midpoint O

Let E1 and E2 be the electric field intensities of the electric field at P due to charge +q and ndashq of the dipole resp The distance of P from each charge is ( r2+L2)12

So E1 = 14 πϵ q

(r 2+L 2) away from +q

E2 = 14 πϵ q

(r 2+L 2) towards ndashq

The magnitudes of E1 and E2 are equal but directions are different Now resolving E1 and E2 into two components parallel and perpendicular to AB we get

The components perpendicular to AB E1sinθ and E2sinθ cancel each other because they are equal and opposite

The components parallel to AB are E1cosθ and E2 cosθ are in same direction and add up

So resultant intensity of electric field at the point P is

E = E1cosθ + E2 cosθ

E = 14 πϵ q

(r 2+L 2) 2 cosθ

Now from fig we have cosθ =BOBP = L (r2+L2)12

So we get E = 14 πϵ 2qL ( r2+L2)32

Now electric dipole moment p= 2qL

So E = 14 πϵ p ( r2+L2)32

HW Find the expression of Electric field as done here but this time take r gtgt 2L

Also find the expression of torque experience by a dipole

(Hint Electric force experienced by charges of dipole in electric field is qE each Let θ be the angle which dipole makes with electric lines of force then perpendicular distance between two charges is 2Lsinθ Then torque = force x perp distance = qE x 2L sinθ So τ=pE sinθ where p =2qL )

STUDY MATERIAL

Class XIISubject Eng Literature (The Tempest ndash William Shakespeare) Topic Act IV Scene 1 Lines 84 to 133 (Iris hellip A contract of true love Be not too late ) Date 27th April 2020 (4th Period)

[Students should read the original play and also the paraphrase given in the school prescribed textbook]Summary Questions amp Answers

o Ceres soon appears and comes to know that she has been summoned to celebrate the contract of true love

o Ceres expresses her unwillingness to meet Venus and Cupid as she has shunned their company

o Ceres and Juno both bestow their blessings upon Ferdinand and Miranda with June gifting honour riches happiness in marriage and Ceres presents plenty of earthrsquos produce

o Iris summons the water-nymphs and reapers to come and celebrate a contract

(1) IRIS Of her society (Line 91-101)

Be not afraid I met her deity

Cutting the clouds towards Pathos and her sonDove-drawn with her Here thought they to have doneSome wanton charm upon this man and maidWhose vows are that no bed-right shall be paidTill Hymens torch be lightedmdashbut in vainMarss hot minion is returned againHer waspish-headed son has broke his arrowsSwears he will shoot no more but play with sparrowsAnd be a boy right out

(i) Where were Venus and Cupid seen flying How were they travelling Why did they want to join the marriage celebration of Ferdinand and Miranda

of true love

Venus and Cupid were seen flying through the air towards Paphos the famous city which is situated on the island of Cyprus They were travelling by air-borne chariot drawn by doves They certainly wanted to come here in order to play some amorous trick upon Ferdinand and Miranda who are under a vow not to gratify their physical desires till the holy ceremony of their marriage has been performed(ii) What have Venus and Cupid done after failing in their plan

After being failure of their plan Venus who is a very passionate deity and who is the mistress of Mars (the god of war) has gone back while here ill-tempered son Cupid has broken his arrows of love in his state of desperation(iii) What has Cupid firmly decided

Cupid is feeling so disappointed that he has firmly decided to shoot no more arrows to arouse love in human hearts but to spend his time playing with sparrows Thus he would now become just a boy and would give up his original function of shooting arrows on human beings to make them fall in love(iv) What vow had Ceres taken How did Ceres feel at the abduction

After the abduction of her daughter Prosperina by Pluto Ceres had taken a vow to always keep away from the disgraceful company of Venus and her blind son Cupid the god of love Ceres felt deeply distressed when Pluto had carried off her daughter and had made her his wife by force(v) Why has Ceres not forgiven Venus and her blind son For what do Ceres want to be sure

As the abduction had been manipulated by Venus the goddess of beauty and love and her blind son Cupid Ceres has never forgiven them for their part in the whole plot Ceres wants to be sure that she would not have to meet Venus and Cupid who had engineered the abduction of her daughter Prosperina

AS THIS lsquoMASQUErsquo SCENE IS VERY IMPORTANT IN THE PLAY THE PARAPHRASE OF THE ENTIRE PORTION OF MASQUE SCENE (Act IV Lines 58 to 143) IS GIVEN BELOW

IRIS Goddess of RainbowCERES Goddess of Agriculture and all the fruits of the earth

(Nature growth prosperity rebirth ndash notions intimately connected to marriage)JUNO The majestic Queen of Heavens and wife of Jupiter (Jupiter is the king of Gods)

VENUS The Goddess of love CUPID Son of Venus PLUTO God of death (In the play referred by Shakespeare as lsquoDisrsquo which is a Roman name for Pluto)

ORIGINAL TEXT PARAPHRASEPROSPEROWellmdash

PROSPERONow come Ariel Let there be too many rather than too few

Now come my Ariel Bring a corollaryRather than want a spirit Appear and pertly[to Ferdinand and Miranda]No tongue all eyes Be silent

spirits in attendance Appear briskly

[to Ferdinand and Miranda]Look with your eyes but do not say a word

[Soft music] [Soft music][Enter Iris] [Enter Iris]

IRISCeres most bounteous lady thy rich leasOf wheat rye barley vetches oats and peasThy turfy mountains where live nibbling sheepAnd flat meads thatched with stover them to keepThy banks with pioned and twilled brimsWhich spongy April at thy hest betrimsTo make cold nymphs chaste crowns and thybroom-grovesWhose shadow the dismissegraved bachelor lovesBeing lass-lorn thy pole clipped vineyardAnd thy sea-marge sterile and rocky-hardWhere thou thyself dost airmdashthe Queen othrsquoSkyWhose watery arch and messenger am IBids thee leave these and with her sovereign grace[Juno appears] Here on this grass-plot in this very placeTo come and sport Her peacocks fly amainApproach rich Ceres her to entertain

IRISCeres most generous lady you are the cause of rich fields or fertile land where wheat rye barley beans oats and peas grow the grassy mountains where the sheep graze and the flat meadows covered with coarse hay to be used as fodder for cattleYour banks are covered with marsh-marigolds and reeds and the rainy April under your orders brings forth to make for the maids who are not in love beautiful crowns your woods where the broom flourishes and where the bachelor who has been dismissed by the maid he loved lies down being forsaken your vineyard in which the poles are embraced by the vines and the margin of the sea which is barren and rocky where you roam about to enjoy the fresh air ndash the queen of the sky (Juno) whose messenger I am besides being represented as the rainbow bids you leave all these and with her majesty here on this grassy plot in this very place come and sport her peacocks carry her fast in her chariot through the air and are making their way here approach rich Ceres to welcome her

[Enter Ariel as Ceres] [Enter Ariel as Ceres]

CERESHail many-coloured messenger that neerDost disobey the wife of JupiterWho with thy saffron wings upon my flowersDiffusest honey-drops refreshing showersAnd with each end of thy blue bow dost crownMy bosky acres and my unshrubbed downRich scarf to my proud earth Why hath thy queenSummoned me hither to this short-grassed green

CERESWelcome rainbow that never dared disobey Juno the wife of Jupiter who with your orange coloured rays spread honey-drops refreshing showers And with each end of thy blue bow drown my bushy acres and my hilly country which is free from shrubs you thus forming a rich scarf Why has your queen called me here to this place covered with short grass

IRISA contract of true love to celebrateAnd some donation freely to estateOn the blest lovers

IRISI have called you to celebrate a contract of true love and bestow some liberal gift upon the blessed lovers

ORIGINAL TEXT PARAPHRASECERESTell me heavenly bowIf Venus or her son as thou dost knowDo now attend the queen Since they did plotThe means that dusky Dis my daughter gotHer and her blind boys scandaled companyI have forsworn

CERESTell me heavenly bow if Venus the Goddess of love or Cupid her son and pedlar of passion at this time attend the heavenly queen Juno because you are sure to know Since the day they conspired against me and dark Pluto took away my daughter here and Cupidrsquos disgraceful company I have left off

IRISOf her societyBe not afraid I met her deityCutting the clouds towards Pathos and her sonDove-drawn with her Here thought they to have doneSome wanton charm upon this man and miad

IRISBe not afraid of her company I met her deity moving on the clouds towards Paphos the sacred home of Venus on the island of Cyprus along with her son on her chariot drawn by doves Here they contemplated to exercise a charm upon this man and maid producing

Whose vows are that no bed-right shall be paidTill Hymens torch be lightedmdashbut in vainMarss hot minion is returned againHer waspish-headed son has broke his arrowsSwears he will shoot no more but play with sparrowsAnd be a boy right out

wantonness before the actual marriage ceremony but did not succeed Venus has returned her irritable son has broken his arrows and swears that he will give up his practice of trying to inspire love but play with sparrows and be a boy again

[Music is heard] [Music is heard]

CERESHighst queen of stateGreat Juno comes I know her by her gait

CERESHighest queen of state Great Juno there she comes I know here by her gait

[Enter Juno] [Enter Juno]

JUNOHow does my bounteous sister Go with meTo bless this twain that they may Prosperous beAnd honoured in their issue

JUNOHow are you doing my generous sister Come with me to bless this couple so that they may be prosperous and fortunate in their children

[They sing] [They sing]

JUNOHonour riches marriage-blessingLong continuance and increasingHourly joys be still upon youJuno sings her blessings upon you

JUNOMay honour riches happiness in marriage long continuance and increase of those boons ever rest upon you as hourly joys Juno showers down upon you her blessings in song

CERESEarths increase foison plentyBarns and garners never emptyVines and clustring bunches growingPlants and goodly burden bowingSpring come to you at the farthestIn the very end of harvestScarcity and want shall shun youCeresrsquo blessing so is on you

CERESMay you have the plenty of earthrsquos produce Your barns and granaries may never be empty Your vines may grow with clustering bunches Your fruit trees may be heavily laden with their fruit May there be continuous spring and harvest May scantiness and want leave you forever Such is the blessing of Ceres upon you

FERDINANDThis is a most majestic vision andHarmoniously charmingly May I be boldTo think these spirits

FERDINANDThis is a great vision and magically melodious Should I suppose the characters (taking part in the masque) are spirits

PROSPEROSpirits which by mine artI have from their confines calld to enactMy present fancies

PROSPEROYes they are spirits whom I have summoned from the regions to which they are confined to carry into effect my fanciful designs

ORIGINAL TEXT PARAPHRASEFERDINANDLet me live here everSo rare a wondered father and a wifeMakes this place paradise

FERDINANDI should like to live here forever Such a wise and wonderful father makes this place a paradise

[Juno and Ceres whisper and send Iris on employment] [Juno and Ceres whisper and send Iris on employment]

PROSPEROSweet now silence

PROSPEROMy dear Ferdinand speak no more Juno and Ceres are

Juno and Ceres whisper seriouslyTheres something else to do Hush and be muteOr else our spell is marred

whispering with a solemn look There is something else coming Silence Or else our magic will be spoilt

IRISYour nymphs called naiads of the wandering brooksWith your sedged crowns and over-harmless looksLeave your crisp channels and on this green landAnswer your summons Juno does commandCome temperate nymphs and help to celebrateA contract of true love Be not too late

IRISYou nymphs called Naiads denizens (M inhabitants) of the running stream with your chaplets of sedge and ever-helpful looks leave your wrinkled channels and on the green land answer the summons sent to you Juno has ordered some chaste nymphs and help to celebrate a noble and true marriage Donrsquot delay

[Enter certain nymphs] [Enter certain nymphs]You sunburnt sicklemen of August wearyCome hither from the furrow and be merryMake holiday your rye-straw hats put onAnd these fresh nymphs encounter every oneIn country footing

You sunburnt harvesters weary from the effects of the heat in August come here from the furrowed land and rejoice Make holiday with your rye-straw hats upon you and meet these fresh nymphs and join in country dancing

[Enter certain reapers properly habited They join with the nymphs in a graceful dance towards the end whereof Prospero starts suddenly and speaks]

[Enter certain reapers properly habited They join with the nymphs in a graceful dance towards the end whereof Prospero starts suddenly and speaks]

PROSPERO[aside] I had forgot that foul conspiracyOf the beast Caliban and his confederatesAgainst my life The minute of their plotIs almost come [to the spirits]Well done Avoidno more

PROSPERO(Aside)I had forgotten the wicked conspiracy of the beast Caliban and his accomplices against my life the time of their plot has almost arrived ndash (To the Spirits) well done depart no more of this

[To a strange hollow and confused noise the spirits heavily vanish]

[The spirits depart]

ORIGINAL TEXT PARAPHRASEFERDINANDLet me live here everSo rare a wondered father and a wifeMakes this place paradise

FERDINANDI should like to live here forever Such a wise and wonderful father makes this place a paradise

[Juno and Ceres whisper and send Iris on employment] [Juno and Ceres whisper and send Iris on employment]

PROSPEROSweet now silenceJuno and Ceres whisper seriouslyTheres something else to do Hush and be muteOr else our spell is marred

PROSPEROMy dear Ferdinand speak no more Juno and Ceres are whispering with a solemn look There is something else coming Silence Or else our magic will be spoilt

IRISYour nymphs called naiads of the wandering brooksWith your sedged crowns and over-harmless looksLeave your crisp channels and on this green landAnswer your summons Juno does commandCome temperate nymphs and help to celebrateA contract of true love Be not too late

IRISYou nymphs called Naiads denizens (M inhabitants) of the running stream with your chaplets of sedge and ever-helpful looks leave your wrinkled channels and on the green land answer the summons sent to you Juno has ordered some chaste nymphs and help to celebrate a noble and true marriage Donrsquot delay

[Enter certain nymphs] [Enter certain nymphs]You sunburnt sicklemen of August wearyCome hither from the furrow and be merryMake holiday your rye-straw hats put onAnd these fresh nymphs encounter every oneIn country footing

You sunburnt harvesters weary from the effects of the heat in August come here from the furrowed land and rejoice Make holiday with your rye-straw hats upon you and meet these fresh nymphs and join in country dancing

[Enter certain reapers properly habited They join with the nymphs in a graceful dance towards the end whereof Prospero starts suddenly and speaks]

[Enter certain reapers properly habited They join with the nymphs in a graceful dance towards the end whereof Prospero starts suddenly and speaks]

PROSPERO[aside] I had forgot that foul conspiracyOf the beast Caliban and his confederatesAgainst my life The minute of their plotIs almost come [to the spirits]Well done Avoidno more

PROSPERO(Aside)I had forgotten the wicked conspiracy of the beast Caliban and his accomplices against my life the time of their plot has almost arrived ndash (To the Spirits) well done depart no more of this

[To a strange hollow and confused noise the spirits heavily vanish]

[The spirits depart]

Ac-12 27420 topic Revaluation of Assets and Liabilities

REVALUATION OF ASSETS AND LIABILITIES

On admission of a new partner the firm stands reconstituted and consequently the assets are revalued and liabilities are reassessed It is necessary to show the true position of the firm at the time of admission of a new partner If the values of the assets are raised gain will increase the capital of the existing partners Similarly any decrease in the value of assets ie loss will decrease the capital of the existing partners For this purpose alsquoRevaluation Accountrsquo is prepared This account is credited with all increases in the value of assets and decrease in the value of liabilities It is debited with decrease on account of value of assets and increase in the value of liabilities The balance of this account shows a gain or loss on revaluation which is transferred to the existing partnerrsquos capital account in existing profit sharing ratioAccounting for Revaluation of Assets and Liabilities when there is a Changein the Profit Sharing Ratio of Existing PartnersAssets and liabilities of a firm must also be revalued at the time of change in profit sharing ratio of existing partners The reason is that the realisable or actual value of assets and liabilities may be different from those shown in the Balance Sheet It is possible that with the passage of time some of the assets might have appreciated in value while the value of certain other assets might have decreased and no record has been made of such changes in the books of accounts Similarly there may be some unrecorded assets amp libilities that may have to be accounted for Revaluation of assets and reassessments of liabilities becomes necessary because the change in the

value of assets and liabilities belongs to the period to change in profit sharing ratio and hence must be shared by the partners in their old profit sharing ratio Revaluation of assets and reassessment of liabilities may be given effect to in two different ways (a) When revised values are to be recorded in the books and(b) When revised values are not to be recorded in the books

When revised values are to be recorded in the booksIn such a case revaluation of assets and reassessment of liabilities is done with the help of a new account called lsquoRevaluation Accountrsquo Sometimes this account is also called as lsquoProfit amp Loss Adjustment Acrsquo If there is a loss due to revaluation revaluation account is debited and if the revaluation results in a profit the revaluation account is credited The following journal entries made for this purpose are

(i) For increase in the value of assetsAsset Ac Dr (individually)To Revaluation Ac(ii) For decrease in the value of AssetRevaluation Ac Dr (individually)To Asset Ac[Decrease in the value of assets](iii) For increase in the value of LiabilitiesRevaluation Ac Dr (individually)To Liabilities Ac[Increase in the value of Liabilities](iv) For decrease in the value of LiabilitiesLiabilities Ac DrTo Revaluation Ac[Decrease in the value of Liabilities](v) For unrecorded AssetsAsset Ac [unrecorded] DrTo Revaluation Ac[Unrecorded asset recorded at actual value](vi) For unrecorded Liability Revaluation Ac DrTo Liability Ac [unrecorded][Unrecorded Liability recorded at actual value](vii) For transfer of gain on revaluationRevaluation Ac DrTo Existing Partnerrsquos CapitalCurrent Ac[Profit on revaluation transferred to capital account in existing ratio](viii) For transfer of loss on revaluationExisting Partnerrsquos CapitalCurrent Ac DrTo Revaluation Ac[Loss on revaluation transferred to capital account in existing ratio](a) When revaluation account shows gain Revaluation Ac DrTo Partnerrsquos Capital Ac (Old Profit Sharing Ratio)(Profit on revaluation credited to Partnerrsquos Capital Ac)(b) Above entry is reversed when revaluation account shows loss Partners Capital Acs (Old Profit Sharing Ratio) DrTo Revaluation Ac(Loss on revaluation debited to Partnerrsquos Capital Acs)

Proforma of Revaluation Account is given as under

Revaluation Account

Dr Cr Particulars ` Amount Particulars ` Amount To Decrease in value of assets By Increase in value of assets To Increase in value of liabilities By Decrease in value of liabilities To Unrecorded liabilities By Unrecorded assets To Gain on Revaluation (Transferred) By Loss on Revalution (Transferred)

ECO ndash12 2742020Topic- ELASTICITY OF DEMAND

CHAPTER - ELASTICITY OF DEMANDMEANINGDemand for a commodity is affected by many factors such as its price price of related goods income of its buyer tastes and preferences etc Elasticity means degree of response Elasticity of demand means degree of responsiveness of demand Demand for a commodity responds to change in price price of related goods income etc So we have three dimensions of elasticity of demandDIMENSION OF ELASTICITY OF DEMAND TYPES OF ELASTICITY OF DEMAND

Price elasticity of demand Income elasticity of demand Cross Elasticity of demand

Price elasticity of demand Price elasticity of demand means degree of responsiveness of demand for a commodity to the change in its price For example if demand for a commodity rises by 10 due to 5 fall in its price Price elasticity of demand (ep)=Percentage change in quantity demanded Percentage change in price of the commodity = 10 ( -)5 = ( - )2Note that ep will always be negative due to inverse relationship of price and quantity demanded

(ii) Income elasticity of demand Income elasticity of demand refers to the degree of responsiveness of demand for a commodity to the change in income of its buyer Suppose income of buyer rises by 10 and his demand for a commodity rises by 20 then Income elasticity of demand (ey)= change in quantity demanded change in price of the commodity =20 10 = 2

Cross Elasticity of demandCross elasticity of demand means the degree of responsiveness of demand for a commodity to the change in price of its related goods (substitute goods or complementary goods) Suppose demand for a commodity rises by 10 due to 5 rise in price of its substitute good then Cross elasticity of demand (ec) = change in quantity demanded change in price of related good = 10 2 = 5 (Tastes and preferences cannot be expressed numerically So elasticity ofdemand cannot be numerically expressed)

  • Chapter 1 Force (Summary)
  • Distinguish between external sovereignty and internal sovereignty
    • NAND Gate
      • Logic diagram
      • Truth Table
        • NOR Gate
          • Logic diagram
          • Truth Table
            • XOR Gate
              • Logic diagram
              • Truth Table
                • XNOR Gate
                  • Logic diagram
                  • Truth Table
                      • Physics
                      • Chapter 1 Electric Field ( Electric Dipole) (Summary)
Page 31:  · Web viewWe all know that Nouns are divided into two parts: common noun and proper noun.Apart from common and proper noun, we will also study about collective noun and compound

satisfactionb Scarcity It must be

limited in quantityc Transferability It

should be transferable its ownership can be transferred from one person to another person

d Exchange value It must possess exchange value

Now let us discuss the term lsquoWelfarersquo

Welfare is defined as satisfaction and happiness a sense of well- being among the people

Welfare is affected by factors like

a Consumption of goods and services

b Environment

c Family relations

d Degree of freedom

e Law and order situation

Mathematics Trigonometric equation

To find the general solution of the equation sinθ=0

When sin θ =0

Then θ= 0 π2π 3π-π -2π -3

i e when θ = 0 or an integral multiple of π

i e when θ= nπ where n is any integer

Therefore the general solution of the equation sin

Example1 Find the general values of θ which satisfy the equation sin2 θ =34

Solution sin2 θ= 34

Or sin θ = +34 or -34

Or sin θ = sin π3 or sin (-π3)

Therefore

θ = [nπ + (-1) n (π3)] or[ nπ+ (-1) n (-π3)]

= nπ +π3 or nπ-π3 where n= any integer

Example 2Find the values of θ which satisfy tan2 θ

θ=0 is θ= nπ where n is any integer

To find the general solution of the equation cos θ=0

When cos θ=0

Then θ=π2 3π2 5π2 -π2 -3π2 -5π3

i e when θ is an odd multiple of π2

i e when θ=(2n+1) π2 where n= any integer

Therefore the general solution of the equation cos θ =0 is θ= (2n+1) π2 where n= any integer

To find the general solution of the equation tan θ = 0

Clearly tan θ =0 implies sin θcos θ =0

Therefore θ = nπ

i e the general solution of the equation tan θ=0 is θ =nπ where n = any integer

To find the general solution of the equation cot θ =0

Clearly cot θ =0 implies (cos θsin θ) = 0

i e cos θ =0

Therefore θ = (2n+1) π2

Therefore the general solution of the equation cot θ =0 is θ = (2n+1) π2

Where n= any integer

To find the general solution of the equation sin θ= k (-

=13 -πleθleπ

Solution tan2 θ =13

Or tan θ = plusmn1radic(3) =tan(plusmnπ6)

θ=nπ plusmn π6 where n =any integer

If n=0 then θ=plusmnπ6

If n=1 then θ= π plusmn π6

If n=-1 then θ= -π plusmn π6

Therefore the required solution in -π le θ le π are θ= π6 5π6 -π6 -5π6

Exercise Find general solution of sin 2θ=cos θ [Hints Use sin 2θ= 2sin θcosθ and then take cosθ

common]

1lek le1)

Determine an angle alpha such that sin =k and -π2le αle π2

Then we have

Sin θ = k = sin α

Or sin θ - sin α =0

Or 2 cos [(θ+α) 2] sin [(θ-α) 2] =0

Therefore either cos [(θ +α) 2] =0 (1)

Or sin [(θ-α) 2] =0 (2)

Now from (1) we get (θ+α) 2= (2m+1) π2)

Or θ = (2m+1) π-α (3)

And from (2) we get (θ-α) 2 =mπ

Or θ= 2mπ+α(4)

Where m = any integer

Clearly the solution (3) amp (4) may be combined in the following form

θ= nπ+(-1) n α where n= any integer

Therefore the general solution of sin θ = sin α is θ = nπ +(-1) n α where n is any integer and -π2 le α le π2

Biology Chapter - 04Kingdom Monera

Today we will discuss about bacterial reproduction and its usefulness

Fig Binary Fission

Fig Conjugation Fig Transformation

Bacterial reproduction is mainly asexual but sexual reproduction

also takes place

Asexual reproduction takes place by i) Binary fission - from one bacteriato

two bacteria are produced in every 20 to 30mins

ii) Buddingiii) Endospore formation - during

unfourable condition

Sexual reproduction by three ways

1) Conjugation - Transfer of genetic material between cells that are in physical contact with one another

2) Transduction - Transfer of genetic materialfrom one cell to another by a bacteriophage

3) Transformation - Transfer of cell-freeor naked DNArsquo from one cell to another

Bacteria causes different diseases inplants animals and human and

it causes food spoilage and waterpollution but it also have some useful

activities

i) Bacteria are helpful in sewage water treatment

ii) It is used in antibiotic (medicine) production

iii) Anaerobic bacteria help in biogas(energy) production

iv) Many household products like yoghurt cheese are manufactured by use of bacteria

v) Rhizobium by symbiotic relationship with leguminous plant increase soil fertility

vi) Besides these bacteria is helpful in genetic engineering degradation of petroleum hydrocarbonand in dairy

industry

Physics Motion in plane Here we will introduce Projectile Motion

Execution

Projectile

Y

usinθ u h

θX

ucosθ

Suppose a body is projected with an angle θ So initial velocity u can be resolved into two components

Horizontal component - ucosθ ( for range)

Vertical component - usinθ ( for height)

usinθ changes during motion and becomes zero at maximum height position but ucosθ remain unchanged

The maximum height of projectile is h

NB If initial is upward then g = -ve and if it is downward then g = +ve Height is +ve if direction of motion does not change ( for ex a body thrown upwards but goes down ultimately then height h = -ve)

The angle of projectile θ is the angle made with horizontal

HISTORY ndash GROWTH OF NATIONALISM

SUB TOPIC- REVOLUTIONARY NATIONALISM Bengal formation of Anushilan Samity and Jugantar Group

The intensification of the Swadeshi movement and Government policy of terror and repression led to outbreak of violence Bombs were manufactured and attempts on the lives of unpopular Government officials became frequent In the gymnasium of Scottish Church College which was known as General Assemblies Institution a secret society was formed known as Anushilan Samity

Aurobindo Ghosh send from Baroda his emissary Jatindranath Banerjee to mobilize the Bengal revolutionaries

Hemchandra Qanungo and Satyen Bose published Journal Jugantar

The Jugantar group planned to assassinate oppressive magistrate Kingsford by Khudiram Bose and Prafulla Chaki in 1908 Prafulla Chaki committed suicide to avoid arrest Khudiram was tried and hanged

Afew days later the police found a bomb factory in Maniktala and arrested a large number of revolutionaries The trial of revolutionaries became famous as the Alipore Bomb Case

In the course of the trial the approver the public prosecuter and a police officer were assassinated

1 Question Name two journals which preached the cult of violence

Answer a) Yugantar edited by Bhupendranath DuttaB) Bandemataram edited by Aurobindo Ghosh2 Question Why was Khudiram arrested and hangedAnswer An attempt was made to assassinate a hated vindictive majistrate named Kingsford by Khudiram Bose and Prafulla Chaki Their attempt failed and the bomb they threw killed two English ladies Khudiram was arrested and put to trial and then hanged3Question Who was Aurubindo GhoshAnswer Aurobindo Ghosh a nationalist revolutionary who was charged for his involvement in the Alipore Bombing Case He was accused of it along with his brother Barindra nath Ghosh But Aurobindo was acquitted because of the brilliant pleading of his counsel Chittaranjan Das Then he became a spiritual reformer introducing his visions on human progress and spiritual evolution4 Qustion Who was KingsfordAnswer Kingsford was an unpopular British chief Magistrate who was the target of the bomb thrown at Muzaffarpur by Khudiram and Prafulla Chaki

Most of the accused were convicted and sentenced to

long term of imprisonmentBut

Aurobindo Ghosh was acquitted mainly owing to the brilliant pleading of his counsel Chittaranjan Das

Political science Topic-Sovereignty

Summary Sovereignty is the full right and power of a governing body over itself without any interference from outside sources or bodies In political theory sovereignty is a substantive term designating supreme legitimate authority over some polity In international law sovereignty is the exercise of power by a state

Internal Sovereignty

Internal sovereignty means supreme authority within ones territory while external sovereignty relates to the recognition on the part of all states that each possesses this power in equal measure

External sovereignty

external sovereignty relates to the recognition on the part of all states that each possesses this power in equal measure

Distinguish between

Execution

Answer the following questions

Short notes-

Sovereignty

Internal Sovereignty

External sovereignty

Homework- learn

external sovereignty and internal sovereigntySovereignty is the principle

of supreme and

unquestionable authority

reflected in the claim by the

state to be the sole author of

laws within its territory

Definition of external vs internal sovereigntyInternal sovereignty refers to

the relationship between a

sovereign power and its

subjects ndash it refers to the

location of the supreme

authority within the state In

the UK for example internal

sovereignty (supposedly)

resides within Parliament

reflected in the

constitutional principle of

parliamentary

sovereigntyBy contrast

external sovereignty refers

to the capacity of the state

to act independently and

autonomously on the world

stage This is what is

sometimes called lsquostate

sovereigntyrsquo or lsquonational

sovereigntyrsquo and implies

that states are legally equal

and that the territorial

integrity and political

independence of a state is

inviolable

Class ndash XII

Date - 2742020 STUDY MATERIALSubject Topic Summary Execution Business Studies

Job Analysis amp Manpower Planning

At first let us recall the chapter what we have discussed till nowbullJob analysisbullJob specification bullJob description bullJob enlargement bullJob enrichment

Today we will do some questions answers from the chapter

Questions 1ldquoJob analysis job description and job specification are interrelatedrdquo Comment Answer) Job analysis is a systematic and detailed examination of a job to collect all the relevant information about it The contents off the job are summarised in the job description The qualification needed for the job are summarised in job specificationThus there is close interrelationship between job analysis job description and job specification

Question 2ldquoJob enlargement is a horizontal extension of a job whereas job enrichment is a vertical extension of a jobrdquo ElucidateAnswer) Job enlargement involves adding one or more task to a job coma where as job enrichment involves adding more autonomy and responsibility to a job Job enlargement is therefore horizontal extension of a job coma whereas job enrichment is a vertical extension of a job

Question 3 )

What is manpower estimation Explain its quantitative and qualitative aspectsAnswer) Manpower estimation is the process by which management determines how an organisation should move from itrsquos current manpower positionto its desired manpower position There are two dimensions of Manpower estimation- quantitative and qualitative

Quantitative aspectThis aspect of Manpower estimation involves estimating the number of employees required in a future time period Workload analysis and workforce analysis are done to estimate the quantity of required manpower

Qualitative aspectThe estimate of the knowledge skills experience etc of required manpower is the qualitative aspect of Manpower estimation The quality of Manpower can be judged on the basis of job analysisand job specification

COMMERCE

CAPITAL-FIXED AND WORKING

Today let us start the class by discussing the sources of finance for different types of business firms

The term lsquocapitalrsquo refers to the investment made in the enterprise for the purpose of earning profits

Requirements of capital and sources of capital for different types of business firms are

1 Capital for sole proprietorship businessA sole proprietor operates at a small scale and thereforerequires a limited amount of capital

2 Capital for partnership firmCapital requirements as well as capital base of a partnership is bigger than that of a sole trader businessThe owned capital is contributed by the partners in an agreed ratio

3 Capital for joint stock companyA joint stock company generally requires large amount of capitalA public company can raise huge capital through issue of shares In addition to share capital it can utilize retained profits

Now let u discuss the meaning of Finance PlanningFinance planning is the process of estimation the financial requirements of an organization specifying the sources of firms and ensuring that enough funds are available at the right time

1 What do you mean by Finance PlanningAnswer Finance planning is the process of estimation the financial requirements of an organization specifying the sources of firms and ensuring that enough funds are available at the right time

2Discuss the role of financial planning of an enterpriseThe role of financial planning are as followsa A sound financial plan helps a business enterprise to avaid the problems of shortage and surplus of fundsbFinancial planning serves as a guide in developing a sound capital structure so as to maximize returns to shareholders c It helps in effective utilization of fundsd It provides policies and procedures for coordinating different functional areas or departments of businesse It enables the management to exercise effective control over the financial activities of an enterprisef It helps the company to prepare for facing business shocks and surprises in future

Mathematics

Continuity and differentiability

Recall Definition of ContinuityLet f(x) be a single valued function of x and x=a be a point in the domain of definition of the function The function is said to be continuous at x=a ifi) f(c) is defined ie f(x) has a definite finite value at x=cii) lim xrarra f(x) exists andiii) lim xrarra f(x) =f(a) In other words f(x) is said to be continuous at x=a if lim xrarra+ f(x)= lim xrarra- f(x) = f(a) Or f(a+0) =f(a-0) =f(a) Or lim hrarr0 f(a+h)= f(a) Algebra of continuous functionsNow we will study some algebra of continuous functions Theorem 1 Suppose f and g be two real functions continuous at a real number c Then(1) f + g is continuous at x = c(2) f ndash g is continuous at x = c(3) f g is continuous at x = c(4) (fg) is continuous at x = c (provided g (c) ne 0)

Example 1 Prove that every rational function is continuousSolution Recall that every rational function f is given byf(x)=[p(x) q(x) ] q(x)ne0where p and q are polynomial functions The domain of f is all real numbers except points at which q is zero Since polynomial functions are continuous f is continuous by (4) of Theorem 1Example 2Discuss the continuity of sine functionSolution To see this we use the following factslim xrarr0 sin x =0Now observe that f (x) = sin x is defined for every real number Let c be a real number Put x = c + h If x rarr c we know that h rarr 0 Therefore lim xrarrc f(x)

= lim xrarrc sin x= lim hrarr0 sin(c+h) =lim hrarr0 [sin c cos h + cos c sin h ]=lim hrarr0 (sin c cos h) + lim hrarr0 (cos c sin h) = sin c +0=sin c = f(c) Thus lim xrarrc f(x) = f(c) and hence f is a constant function Exercise Prove that the function f(x) = x2 +2x is continuous for every real value of x [Hints show that lim xrarra+ f(x) = lim xrarra- f(x) = f(a) ]

Biology Reproduction in Flowering plants We will discuss about megasporoangium

megasporagenesis and female gametophyte

Q4 Describe the structure of megasporangium

Ovule is attached to the placenta by astalk called funicle

Each ovule has one two or three protectivecoverings called integuments

At the tip of integuments a small openingcalled micropyle is organised

Opposite to the micropylar end is the chalaza

Within the integuments a mass of cellsnucellusand inside it embryo sac orfemale gametophyte is present

Q5 Describe a mature embryosacamp its formation

In most of the flowering plants only oneof the 4 megaspores formed as a result ofmegasporogenesis that is functional while theother three degenerate

The

functional megaspore develops into thefemale gametophyte

Formation The nucleus

of the functional megasporedivides mitotically to form two nuclei first andthen two more sequential mitotic nucleardivisions result in the formation of four ampthen eight nucleate stages of embryo sac

Six of the eight nucleus are surrounded bycell walls and organised into cells

The remaining two nuclei called polar nuclei are found below the egg apparatus in the largecentral cell

Three cells consisting of two synergids amp one egg cell present bottom of

embryo sac Three cells

at the chalazal as antipodal cells

Two polar nuclei together present in large central cell

HISTORY

TOWARDS INDEPENDENCE AND PARTITION THE LAST PHASE(1935-1947)SUB TOPIC NATIONAL MOVEMENTS DURING THE SECOND WORLD WAR

Spread of Quit India Movement On 9th August 1942Gandhiji and other Congress leaders were arrested The Congress was declared illegal The news of the arrest of all leaders marked the beginning of a widespread movement of India It was not possible for such a movement to remain peacefulBut the arrest of the all notable congress leaders virtually left the movement in the hands of the mass The movement took the form of violent and militant outbreakBesides congressmen revolutionaries also were very active in the movement The Congress Socialist group also played a prominent role

1 Question Why did the British authority arrest the Congress leaders on 9 th August 1942Answer Congress Working committee adopted the Quit India resolution which was to be ratified at the Bombay AICC meeting in 8th August 1942 They decided to launch a mass struggle on non-violent lines Gandhiji gave a clarion call to all section of the people rdquoKarenge ya Marengerdquo (do or die) Congress leaders gave the call to driving out

the British from IndiaViceroy had taken strong action against the Quit India movement Gandhiji and all the leaders of Congress were arrested

2 Question How did Quit India Movement spread out all over IndiaAnswer The news of the leaders lsquo arrest marked the beginning of a widespread movement to remain peacefulThe movement took form of violent outbreak There were widespread cutting of telephone and Telegraph wires damaging railway lines raising barricades in cities and towns and other forms of violent demonstations

Question Name the leaders of Congress

Socialist group played a prominent part Notable among the Jayprakash Narayan Rammonohar Lohia Aruna Asaf Ali

Political science

Topic-Franchise and Representation

Summary

The election commission

The Election Commission of India is an autonomous constitutional authority responsible for administering Union and State election processes in India The body administers elections to the Lok Sabha Rajya Sabha State Legislative Assemblies in India and the offices of the President and Vice President in the country

Functions of election commission-

India is a sovereign socialist secular democratic republic Democracy runs like a golden thread in the social economic and political fabric woven by the Constitution given by lsquoWe the People of Indiarsquo unto ourselves The concept of democracy as visualised by the Constitution pre-supposes the representation of the people in Parliament and State legislatures by the method of election The Supreme Court has held that democracy is one of the inalienable basic features of the Constitution of India and forms part of its basic structure The Constitution of India adopted a Parliamentary form of government Parliament consists of the President of India and the two Houses mdash Rajya Sabha and Lok Sabha India being a Union of states has separate state legislatures for each state State legislatures consist of the Governor and two Houses mdash Legislative Council and Legislative Assembly mdash in seven states namely Andhra Pradesh Telangana Bihar Jammu amp Kashmir Karnataka Maharashtra and Uttar Pradesh and of the Governor and the state Legislative Assembly in the remaining 22 states Apart from the above two out of the seven Union Territories namely National Capital Territory of Delhi and Puducherry also have their Legislative Assemblies

ExecutionShort notes-Election commissionFunctions of election commission

Homework- Learn

Computer

Science

Computer hardware NAND Gate

A NOT-AND operation is known as NAND operation It has n input (n gt= 2) and one output

Logic diagram

Truth Table

NOR Gate

A NOT-OR operation is known as NOR operation It has n input (n gt= 2) and one output

Logic diagram

Truth Table

XOR Gate

XOR or Ex-OR gate is a special type of gate It can be used in the half

adder full adder and subtractor The exclusive-OR gate is abbreviated as EX-OR gate or sometime as X-OR gate It has n input (n gt= 2) and one output

Logic diagram

Truth Table

XNOR Gate

XNOR gate is a special type of gate It can be used in the half adder full adder and subtractor The exclusive-NOR gate is abbreviated as EX-NOR gate or sometime as X-NOR gate It has n input (n gt= 2) and one output

Logic diagram

Truth Table

Physics

Chapter 1 Electric Field ( Electric Dipole) (Summary)

Here we will derive Expression of electric field at broad side

On position of dipole

Execution

Q With the help of a labelled diagram obtain an expression for the electric field intensity E at any point on the equitorial line ( broad-side on position) of an electric dipole

Ans

E1 E1sinθ

E θ P E1 θ

( r2+L2)12 E2 E

r E2 E2sinθ

-q θ L O L +qA B

Let us consider that the point P is situated on the right bisector of the dipole AB at a distance r meter from its midpoint O

Let E1 and E2 be the electric field intensities of the electric field at P due to charge +q and ndashq of the dipole resp The distance of P from each charge is ( r2+L2)12

So E1 = 14 πϵ q

(r 2+L 2) away from +q

E2 = 14 πϵ q

(r 2+L 2) towards ndashq

The magnitudes of E1 and E2 are equal but directions are different Now resolving E1 and E2 into two components parallel and perpendicular to AB we get

The components perpendicular to AB E1sinθ and E2sinθ cancel each other because they are equal and opposite

The components parallel to AB are E1cosθ and E2 cosθ are in same direction and add up

So resultant intensity of electric field at the point P is

E = E1cosθ + E2 cosθ

E = 14 πϵ q

(r 2+L 2) 2 cosθ

Now from fig we have cosθ =BOBP = L (r2+L2)12

So we get E = 14 πϵ 2qL ( r2+L2)32

Now electric dipole moment p= 2qL

So E = 14 πϵ p ( r2+L2)32

HW Find the expression of Electric field as done here but this time take r gtgt 2L

Also find the expression of torque experience by a dipole

(Hint Electric force experienced by charges of dipole in electric field is qE each Let θ be the angle which dipole makes with electric lines of force then perpendicular distance between two charges is 2Lsinθ Then torque = force x perp distance = qE x 2L sinθ So τ=pE sinθ where p =2qL )

STUDY MATERIAL

Class XIISubject Eng Literature (The Tempest ndash William Shakespeare) Topic Act IV Scene 1 Lines 84 to 133 (Iris hellip A contract of true love Be not too late ) Date 27th April 2020 (4th Period)

[Students should read the original play and also the paraphrase given in the school prescribed textbook]Summary Questions amp Answers

o Ceres soon appears and comes to know that she has been summoned to celebrate the contract of true love

o Ceres expresses her unwillingness to meet Venus and Cupid as she has shunned their company

o Ceres and Juno both bestow their blessings upon Ferdinand and Miranda with June gifting honour riches happiness in marriage and Ceres presents plenty of earthrsquos produce

o Iris summons the water-nymphs and reapers to come and celebrate a contract

(1) IRIS Of her society (Line 91-101)

Be not afraid I met her deity

Cutting the clouds towards Pathos and her sonDove-drawn with her Here thought they to have doneSome wanton charm upon this man and maidWhose vows are that no bed-right shall be paidTill Hymens torch be lightedmdashbut in vainMarss hot minion is returned againHer waspish-headed son has broke his arrowsSwears he will shoot no more but play with sparrowsAnd be a boy right out

(i) Where were Venus and Cupid seen flying How were they travelling Why did they want to join the marriage celebration of Ferdinand and Miranda

of true love

Venus and Cupid were seen flying through the air towards Paphos the famous city which is situated on the island of Cyprus They were travelling by air-borne chariot drawn by doves They certainly wanted to come here in order to play some amorous trick upon Ferdinand and Miranda who are under a vow not to gratify their physical desires till the holy ceremony of their marriage has been performed(ii) What have Venus and Cupid done after failing in their plan

After being failure of their plan Venus who is a very passionate deity and who is the mistress of Mars (the god of war) has gone back while here ill-tempered son Cupid has broken his arrows of love in his state of desperation(iii) What has Cupid firmly decided

Cupid is feeling so disappointed that he has firmly decided to shoot no more arrows to arouse love in human hearts but to spend his time playing with sparrows Thus he would now become just a boy and would give up his original function of shooting arrows on human beings to make them fall in love(iv) What vow had Ceres taken How did Ceres feel at the abduction

After the abduction of her daughter Prosperina by Pluto Ceres had taken a vow to always keep away from the disgraceful company of Venus and her blind son Cupid the god of love Ceres felt deeply distressed when Pluto had carried off her daughter and had made her his wife by force(v) Why has Ceres not forgiven Venus and her blind son For what do Ceres want to be sure

As the abduction had been manipulated by Venus the goddess of beauty and love and her blind son Cupid Ceres has never forgiven them for their part in the whole plot Ceres wants to be sure that she would not have to meet Venus and Cupid who had engineered the abduction of her daughter Prosperina

AS THIS lsquoMASQUErsquo SCENE IS VERY IMPORTANT IN THE PLAY THE PARAPHRASE OF THE ENTIRE PORTION OF MASQUE SCENE (Act IV Lines 58 to 143) IS GIVEN BELOW

IRIS Goddess of RainbowCERES Goddess of Agriculture and all the fruits of the earth

(Nature growth prosperity rebirth ndash notions intimately connected to marriage)JUNO The majestic Queen of Heavens and wife of Jupiter (Jupiter is the king of Gods)

VENUS The Goddess of love CUPID Son of Venus PLUTO God of death (In the play referred by Shakespeare as lsquoDisrsquo which is a Roman name for Pluto)

ORIGINAL TEXT PARAPHRASEPROSPEROWellmdash

PROSPERONow come Ariel Let there be too many rather than too few

Now come my Ariel Bring a corollaryRather than want a spirit Appear and pertly[to Ferdinand and Miranda]No tongue all eyes Be silent

spirits in attendance Appear briskly

[to Ferdinand and Miranda]Look with your eyes but do not say a word

[Soft music] [Soft music][Enter Iris] [Enter Iris]

IRISCeres most bounteous lady thy rich leasOf wheat rye barley vetches oats and peasThy turfy mountains where live nibbling sheepAnd flat meads thatched with stover them to keepThy banks with pioned and twilled brimsWhich spongy April at thy hest betrimsTo make cold nymphs chaste crowns and thybroom-grovesWhose shadow the dismissegraved bachelor lovesBeing lass-lorn thy pole clipped vineyardAnd thy sea-marge sterile and rocky-hardWhere thou thyself dost airmdashthe Queen othrsquoSkyWhose watery arch and messenger am IBids thee leave these and with her sovereign grace[Juno appears] Here on this grass-plot in this very placeTo come and sport Her peacocks fly amainApproach rich Ceres her to entertain

IRISCeres most generous lady you are the cause of rich fields or fertile land where wheat rye barley beans oats and peas grow the grassy mountains where the sheep graze and the flat meadows covered with coarse hay to be used as fodder for cattleYour banks are covered with marsh-marigolds and reeds and the rainy April under your orders brings forth to make for the maids who are not in love beautiful crowns your woods where the broom flourishes and where the bachelor who has been dismissed by the maid he loved lies down being forsaken your vineyard in which the poles are embraced by the vines and the margin of the sea which is barren and rocky where you roam about to enjoy the fresh air ndash the queen of the sky (Juno) whose messenger I am besides being represented as the rainbow bids you leave all these and with her majesty here on this grassy plot in this very place come and sport her peacocks carry her fast in her chariot through the air and are making their way here approach rich Ceres to welcome her

[Enter Ariel as Ceres] [Enter Ariel as Ceres]

CERESHail many-coloured messenger that neerDost disobey the wife of JupiterWho with thy saffron wings upon my flowersDiffusest honey-drops refreshing showersAnd with each end of thy blue bow dost crownMy bosky acres and my unshrubbed downRich scarf to my proud earth Why hath thy queenSummoned me hither to this short-grassed green

CERESWelcome rainbow that never dared disobey Juno the wife of Jupiter who with your orange coloured rays spread honey-drops refreshing showers And with each end of thy blue bow drown my bushy acres and my hilly country which is free from shrubs you thus forming a rich scarf Why has your queen called me here to this place covered with short grass

IRISA contract of true love to celebrateAnd some donation freely to estateOn the blest lovers

IRISI have called you to celebrate a contract of true love and bestow some liberal gift upon the blessed lovers

ORIGINAL TEXT PARAPHRASECERESTell me heavenly bowIf Venus or her son as thou dost knowDo now attend the queen Since they did plotThe means that dusky Dis my daughter gotHer and her blind boys scandaled companyI have forsworn

CERESTell me heavenly bow if Venus the Goddess of love or Cupid her son and pedlar of passion at this time attend the heavenly queen Juno because you are sure to know Since the day they conspired against me and dark Pluto took away my daughter here and Cupidrsquos disgraceful company I have left off

IRISOf her societyBe not afraid I met her deityCutting the clouds towards Pathos and her sonDove-drawn with her Here thought they to have doneSome wanton charm upon this man and miad

IRISBe not afraid of her company I met her deity moving on the clouds towards Paphos the sacred home of Venus on the island of Cyprus along with her son on her chariot drawn by doves Here they contemplated to exercise a charm upon this man and maid producing

Whose vows are that no bed-right shall be paidTill Hymens torch be lightedmdashbut in vainMarss hot minion is returned againHer waspish-headed son has broke his arrowsSwears he will shoot no more but play with sparrowsAnd be a boy right out

wantonness before the actual marriage ceremony but did not succeed Venus has returned her irritable son has broken his arrows and swears that he will give up his practice of trying to inspire love but play with sparrows and be a boy again

[Music is heard] [Music is heard]

CERESHighst queen of stateGreat Juno comes I know her by her gait

CERESHighest queen of state Great Juno there she comes I know here by her gait

[Enter Juno] [Enter Juno]

JUNOHow does my bounteous sister Go with meTo bless this twain that they may Prosperous beAnd honoured in their issue

JUNOHow are you doing my generous sister Come with me to bless this couple so that they may be prosperous and fortunate in their children

[They sing] [They sing]

JUNOHonour riches marriage-blessingLong continuance and increasingHourly joys be still upon youJuno sings her blessings upon you

JUNOMay honour riches happiness in marriage long continuance and increase of those boons ever rest upon you as hourly joys Juno showers down upon you her blessings in song

CERESEarths increase foison plentyBarns and garners never emptyVines and clustring bunches growingPlants and goodly burden bowingSpring come to you at the farthestIn the very end of harvestScarcity and want shall shun youCeresrsquo blessing so is on you

CERESMay you have the plenty of earthrsquos produce Your barns and granaries may never be empty Your vines may grow with clustering bunches Your fruit trees may be heavily laden with their fruit May there be continuous spring and harvest May scantiness and want leave you forever Such is the blessing of Ceres upon you

FERDINANDThis is a most majestic vision andHarmoniously charmingly May I be boldTo think these spirits

FERDINANDThis is a great vision and magically melodious Should I suppose the characters (taking part in the masque) are spirits

PROSPEROSpirits which by mine artI have from their confines calld to enactMy present fancies

PROSPEROYes they are spirits whom I have summoned from the regions to which they are confined to carry into effect my fanciful designs

ORIGINAL TEXT PARAPHRASEFERDINANDLet me live here everSo rare a wondered father and a wifeMakes this place paradise

FERDINANDI should like to live here forever Such a wise and wonderful father makes this place a paradise

[Juno and Ceres whisper and send Iris on employment] [Juno and Ceres whisper and send Iris on employment]

PROSPEROSweet now silence

PROSPEROMy dear Ferdinand speak no more Juno and Ceres are

Juno and Ceres whisper seriouslyTheres something else to do Hush and be muteOr else our spell is marred

whispering with a solemn look There is something else coming Silence Or else our magic will be spoilt

IRISYour nymphs called naiads of the wandering brooksWith your sedged crowns and over-harmless looksLeave your crisp channels and on this green landAnswer your summons Juno does commandCome temperate nymphs and help to celebrateA contract of true love Be not too late

IRISYou nymphs called Naiads denizens (M inhabitants) of the running stream with your chaplets of sedge and ever-helpful looks leave your wrinkled channels and on the green land answer the summons sent to you Juno has ordered some chaste nymphs and help to celebrate a noble and true marriage Donrsquot delay

[Enter certain nymphs] [Enter certain nymphs]You sunburnt sicklemen of August wearyCome hither from the furrow and be merryMake holiday your rye-straw hats put onAnd these fresh nymphs encounter every oneIn country footing

You sunburnt harvesters weary from the effects of the heat in August come here from the furrowed land and rejoice Make holiday with your rye-straw hats upon you and meet these fresh nymphs and join in country dancing

[Enter certain reapers properly habited They join with the nymphs in a graceful dance towards the end whereof Prospero starts suddenly and speaks]

[Enter certain reapers properly habited They join with the nymphs in a graceful dance towards the end whereof Prospero starts suddenly and speaks]

PROSPERO[aside] I had forgot that foul conspiracyOf the beast Caliban and his confederatesAgainst my life The minute of their plotIs almost come [to the spirits]Well done Avoidno more

PROSPERO(Aside)I had forgotten the wicked conspiracy of the beast Caliban and his accomplices against my life the time of their plot has almost arrived ndash (To the Spirits) well done depart no more of this

[To a strange hollow and confused noise the spirits heavily vanish]

[The spirits depart]

ORIGINAL TEXT PARAPHRASEFERDINANDLet me live here everSo rare a wondered father and a wifeMakes this place paradise

FERDINANDI should like to live here forever Such a wise and wonderful father makes this place a paradise

[Juno and Ceres whisper and send Iris on employment] [Juno and Ceres whisper and send Iris on employment]

PROSPEROSweet now silenceJuno and Ceres whisper seriouslyTheres something else to do Hush and be muteOr else our spell is marred

PROSPEROMy dear Ferdinand speak no more Juno and Ceres are whispering with a solemn look There is something else coming Silence Or else our magic will be spoilt

IRISYour nymphs called naiads of the wandering brooksWith your sedged crowns and over-harmless looksLeave your crisp channels and on this green landAnswer your summons Juno does commandCome temperate nymphs and help to celebrateA contract of true love Be not too late

IRISYou nymphs called Naiads denizens (M inhabitants) of the running stream with your chaplets of sedge and ever-helpful looks leave your wrinkled channels and on the green land answer the summons sent to you Juno has ordered some chaste nymphs and help to celebrate a noble and true marriage Donrsquot delay

[Enter certain nymphs] [Enter certain nymphs]You sunburnt sicklemen of August wearyCome hither from the furrow and be merryMake holiday your rye-straw hats put onAnd these fresh nymphs encounter every oneIn country footing

You sunburnt harvesters weary from the effects of the heat in August come here from the furrowed land and rejoice Make holiday with your rye-straw hats upon you and meet these fresh nymphs and join in country dancing

[Enter certain reapers properly habited They join with the nymphs in a graceful dance towards the end whereof Prospero starts suddenly and speaks]

[Enter certain reapers properly habited They join with the nymphs in a graceful dance towards the end whereof Prospero starts suddenly and speaks]

PROSPERO[aside] I had forgot that foul conspiracyOf the beast Caliban and his confederatesAgainst my life The minute of their plotIs almost come [to the spirits]Well done Avoidno more

PROSPERO(Aside)I had forgotten the wicked conspiracy of the beast Caliban and his accomplices against my life the time of their plot has almost arrived ndash (To the Spirits) well done depart no more of this

[To a strange hollow and confused noise the spirits heavily vanish]

[The spirits depart]

Ac-12 27420 topic Revaluation of Assets and Liabilities

REVALUATION OF ASSETS AND LIABILITIES

On admission of a new partner the firm stands reconstituted and consequently the assets are revalued and liabilities are reassessed It is necessary to show the true position of the firm at the time of admission of a new partner If the values of the assets are raised gain will increase the capital of the existing partners Similarly any decrease in the value of assets ie loss will decrease the capital of the existing partners For this purpose alsquoRevaluation Accountrsquo is prepared This account is credited with all increases in the value of assets and decrease in the value of liabilities It is debited with decrease on account of value of assets and increase in the value of liabilities The balance of this account shows a gain or loss on revaluation which is transferred to the existing partnerrsquos capital account in existing profit sharing ratioAccounting for Revaluation of Assets and Liabilities when there is a Changein the Profit Sharing Ratio of Existing PartnersAssets and liabilities of a firm must also be revalued at the time of change in profit sharing ratio of existing partners The reason is that the realisable or actual value of assets and liabilities may be different from those shown in the Balance Sheet It is possible that with the passage of time some of the assets might have appreciated in value while the value of certain other assets might have decreased and no record has been made of such changes in the books of accounts Similarly there may be some unrecorded assets amp libilities that may have to be accounted for Revaluation of assets and reassessments of liabilities becomes necessary because the change in the

value of assets and liabilities belongs to the period to change in profit sharing ratio and hence must be shared by the partners in their old profit sharing ratio Revaluation of assets and reassessment of liabilities may be given effect to in two different ways (a) When revised values are to be recorded in the books and(b) When revised values are not to be recorded in the books

When revised values are to be recorded in the booksIn such a case revaluation of assets and reassessment of liabilities is done with the help of a new account called lsquoRevaluation Accountrsquo Sometimes this account is also called as lsquoProfit amp Loss Adjustment Acrsquo If there is a loss due to revaluation revaluation account is debited and if the revaluation results in a profit the revaluation account is credited The following journal entries made for this purpose are

(i) For increase in the value of assetsAsset Ac Dr (individually)To Revaluation Ac(ii) For decrease in the value of AssetRevaluation Ac Dr (individually)To Asset Ac[Decrease in the value of assets](iii) For increase in the value of LiabilitiesRevaluation Ac Dr (individually)To Liabilities Ac[Increase in the value of Liabilities](iv) For decrease in the value of LiabilitiesLiabilities Ac DrTo Revaluation Ac[Decrease in the value of Liabilities](v) For unrecorded AssetsAsset Ac [unrecorded] DrTo Revaluation Ac[Unrecorded asset recorded at actual value](vi) For unrecorded Liability Revaluation Ac DrTo Liability Ac [unrecorded][Unrecorded Liability recorded at actual value](vii) For transfer of gain on revaluationRevaluation Ac DrTo Existing Partnerrsquos CapitalCurrent Ac[Profit on revaluation transferred to capital account in existing ratio](viii) For transfer of loss on revaluationExisting Partnerrsquos CapitalCurrent Ac DrTo Revaluation Ac[Loss on revaluation transferred to capital account in existing ratio](a) When revaluation account shows gain Revaluation Ac DrTo Partnerrsquos Capital Ac (Old Profit Sharing Ratio)(Profit on revaluation credited to Partnerrsquos Capital Ac)(b) Above entry is reversed when revaluation account shows loss Partners Capital Acs (Old Profit Sharing Ratio) DrTo Revaluation Ac(Loss on revaluation debited to Partnerrsquos Capital Acs)

Proforma of Revaluation Account is given as under

Revaluation Account

Dr Cr Particulars ` Amount Particulars ` Amount To Decrease in value of assets By Increase in value of assets To Increase in value of liabilities By Decrease in value of liabilities To Unrecorded liabilities By Unrecorded assets To Gain on Revaluation (Transferred) By Loss on Revalution (Transferred)

ECO ndash12 2742020Topic- ELASTICITY OF DEMAND

CHAPTER - ELASTICITY OF DEMANDMEANINGDemand for a commodity is affected by many factors such as its price price of related goods income of its buyer tastes and preferences etc Elasticity means degree of response Elasticity of demand means degree of responsiveness of demand Demand for a commodity responds to change in price price of related goods income etc So we have three dimensions of elasticity of demandDIMENSION OF ELASTICITY OF DEMAND TYPES OF ELASTICITY OF DEMAND

Price elasticity of demand Income elasticity of demand Cross Elasticity of demand

Price elasticity of demand Price elasticity of demand means degree of responsiveness of demand for a commodity to the change in its price For example if demand for a commodity rises by 10 due to 5 fall in its price Price elasticity of demand (ep)=Percentage change in quantity demanded Percentage change in price of the commodity = 10 ( -)5 = ( - )2Note that ep will always be negative due to inverse relationship of price and quantity demanded

(ii) Income elasticity of demand Income elasticity of demand refers to the degree of responsiveness of demand for a commodity to the change in income of its buyer Suppose income of buyer rises by 10 and his demand for a commodity rises by 20 then Income elasticity of demand (ey)= change in quantity demanded change in price of the commodity =20 10 = 2

Cross Elasticity of demandCross elasticity of demand means the degree of responsiveness of demand for a commodity to the change in price of its related goods (substitute goods or complementary goods) Suppose demand for a commodity rises by 10 due to 5 rise in price of its substitute good then Cross elasticity of demand (ec) = change in quantity demanded change in price of related good = 10 2 = 5 (Tastes and preferences cannot be expressed numerically So elasticity ofdemand cannot be numerically expressed)

  • Chapter 1 Force (Summary)
  • Distinguish between external sovereignty and internal sovereignty
    • NAND Gate
      • Logic diagram
      • Truth Table
        • NOR Gate
          • Logic diagram
          • Truth Table
            • XOR Gate
              • Logic diagram
              • Truth Table
                • XNOR Gate
                  • Logic diagram
                  • Truth Table
                      • Physics
                      • Chapter 1 Electric Field ( Electric Dipole) (Summary)
Page 32:  · Web viewWe all know that Nouns are divided into two parts: common noun and proper noun.Apart from common and proper noun, we will also study about collective noun and compound

θ=0 is θ= nπ where n is any integer

To find the general solution of the equation cos θ=0

When cos θ=0

Then θ=π2 3π2 5π2 -π2 -3π2 -5π3

i e when θ is an odd multiple of π2

i e when θ=(2n+1) π2 where n= any integer

Therefore the general solution of the equation cos θ =0 is θ= (2n+1) π2 where n= any integer

To find the general solution of the equation tan θ = 0

Clearly tan θ =0 implies sin θcos θ =0

Therefore θ = nπ

i e the general solution of the equation tan θ=0 is θ =nπ where n = any integer

To find the general solution of the equation cot θ =0

Clearly cot θ =0 implies (cos θsin θ) = 0

i e cos θ =0

Therefore θ = (2n+1) π2

Therefore the general solution of the equation cot θ =0 is θ = (2n+1) π2

Where n= any integer

To find the general solution of the equation sin θ= k (-

=13 -πleθleπ

Solution tan2 θ =13

Or tan θ = plusmn1radic(3) =tan(plusmnπ6)

θ=nπ plusmn π6 where n =any integer

If n=0 then θ=plusmnπ6

If n=1 then θ= π plusmn π6

If n=-1 then θ= -π plusmn π6

Therefore the required solution in -π le θ le π are θ= π6 5π6 -π6 -5π6

Exercise Find general solution of sin 2θ=cos θ [Hints Use sin 2θ= 2sin θcosθ and then take cosθ

common]

1lek le1)

Determine an angle alpha such that sin =k and -π2le αle π2

Then we have

Sin θ = k = sin α

Or sin θ - sin α =0

Or 2 cos [(θ+α) 2] sin [(θ-α) 2] =0

Therefore either cos [(θ +α) 2] =0 (1)

Or sin [(θ-α) 2] =0 (2)

Now from (1) we get (θ+α) 2= (2m+1) π2)

Or θ = (2m+1) π-α (3)

And from (2) we get (θ-α) 2 =mπ

Or θ= 2mπ+α(4)

Where m = any integer

Clearly the solution (3) amp (4) may be combined in the following form

θ= nπ+(-1) n α where n= any integer

Therefore the general solution of sin θ = sin α is θ = nπ +(-1) n α where n is any integer and -π2 le α le π2

Biology Chapter - 04Kingdom Monera

Today we will discuss about bacterial reproduction and its usefulness

Fig Binary Fission

Fig Conjugation Fig Transformation

Bacterial reproduction is mainly asexual but sexual reproduction

also takes place

Asexual reproduction takes place by i) Binary fission - from one bacteriato

two bacteria are produced in every 20 to 30mins

ii) Buddingiii) Endospore formation - during

unfourable condition

Sexual reproduction by three ways

1) Conjugation - Transfer of genetic material between cells that are in physical contact with one another

2) Transduction - Transfer of genetic materialfrom one cell to another by a bacteriophage

3) Transformation - Transfer of cell-freeor naked DNArsquo from one cell to another

Bacteria causes different diseases inplants animals and human and

it causes food spoilage and waterpollution but it also have some useful

activities

i) Bacteria are helpful in sewage water treatment

ii) It is used in antibiotic (medicine) production

iii) Anaerobic bacteria help in biogas(energy) production

iv) Many household products like yoghurt cheese are manufactured by use of bacteria

v) Rhizobium by symbiotic relationship with leguminous plant increase soil fertility

vi) Besides these bacteria is helpful in genetic engineering degradation of petroleum hydrocarbonand in dairy

industry

Physics Motion in plane Here we will introduce Projectile Motion

Execution

Projectile

Y

usinθ u h

θX

ucosθ

Suppose a body is projected with an angle θ So initial velocity u can be resolved into two components

Horizontal component - ucosθ ( for range)

Vertical component - usinθ ( for height)

usinθ changes during motion and becomes zero at maximum height position but ucosθ remain unchanged

The maximum height of projectile is h

NB If initial is upward then g = -ve and if it is downward then g = +ve Height is +ve if direction of motion does not change ( for ex a body thrown upwards but goes down ultimately then height h = -ve)

The angle of projectile θ is the angle made with horizontal

HISTORY ndash GROWTH OF NATIONALISM

SUB TOPIC- REVOLUTIONARY NATIONALISM Bengal formation of Anushilan Samity and Jugantar Group

The intensification of the Swadeshi movement and Government policy of terror and repression led to outbreak of violence Bombs were manufactured and attempts on the lives of unpopular Government officials became frequent In the gymnasium of Scottish Church College which was known as General Assemblies Institution a secret society was formed known as Anushilan Samity

Aurobindo Ghosh send from Baroda his emissary Jatindranath Banerjee to mobilize the Bengal revolutionaries

Hemchandra Qanungo and Satyen Bose published Journal Jugantar

The Jugantar group planned to assassinate oppressive magistrate Kingsford by Khudiram Bose and Prafulla Chaki in 1908 Prafulla Chaki committed suicide to avoid arrest Khudiram was tried and hanged

Afew days later the police found a bomb factory in Maniktala and arrested a large number of revolutionaries The trial of revolutionaries became famous as the Alipore Bomb Case

In the course of the trial the approver the public prosecuter and a police officer were assassinated

1 Question Name two journals which preached the cult of violence

Answer a) Yugantar edited by Bhupendranath DuttaB) Bandemataram edited by Aurobindo Ghosh2 Question Why was Khudiram arrested and hangedAnswer An attempt was made to assassinate a hated vindictive majistrate named Kingsford by Khudiram Bose and Prafulla Chaki Their attempt failed and the bomb they threw killed two English ladies Khudiram was arrested and put to trial and then hanged3Question Who was Aurubindo GhoshAnswer Aurobindo Ghosh a nationalist revolutionary who was charged for his involvement in the Alipore Bombing Case He was accused of it along with his brother Barindra nath Ghosh But Aurobindo was acquitted because of the brilliant pleading of his counsel Chittaranjan Das Then he became a spiritual reformer introducing his visions on human progress and spiritual evolution4 Qustion Who was KingsfordAnswer Kingsford was an unpopular British chief Magistrate who was the target of the bomb thrown at Muzaffarpur by Khudiram and Prafulla Chaki

Most of the accused were convicted and sentenced to

long term of imprisonmentBut

Aurobindo Ghosh was acquitted mainly owing to the brilliant pleading of his counsel Chittaranjan Das

Political science Topic-Sovereignty

Summary Sovereignty is the full right and power of a governing body over itself without any interference from outside sources or bodies In political theory sovereignty is a substantive term designating supreme legitimate authority over some polity In international law sovereignty is the exercise of power by a state

Internal Sovereignty

Internal sovereignty means supreme authority within ones territory while external sovereignty relates to the recognition on the part of all states that each possesses this power in equal measure

External sovereignty

external sovereignty relates to the recognition on the part of all states that each possesses this power in equal measure

Distinguish between

Execution

Answer the following questions

Short notes-

Sovereignty

Internal Sovereignty

External sovereignty

Homework- learn

external sovereignty and internal sovereigntySovereignty is the principle

of supreme and

unquestionable authority

reflected in the claim by the

state to be the sole author of

laws within its territory

Definition of external vs internal sovereigntyInternal sovereignty refers to

the relationship between a

sovereign power and its

subjects ndash it refers to the

location of the supreme

authority within the state In

the UK for example internal

sovereignty (supposedly)

resides within Parliament

reflected in the

constitutional principle of

parliamentary

sovereigntyBy contrast

external sovereignty refers

to the capacity of the state

to act independently and

autonomously on the world

stage This is what is

sometimes called lsquostate

sovereigntyrsquo or lsquonational

sovereigntyrsquo and implies

that states are legally equal

and that the territorial

integrity and political

independence of a state is

inviolable

Class ndash XII

Date - 2742020 STUDY MATERIALSubject Topic Summary Execution Business Studies

Job Analysis amp Manpower Planning

At first let us recall the chapter what we have discussed till nowbullJob analysisbullJob specification bullJob description bullJob enlargement bullJob enrichment

Today we will do some questions answers from the chapter

Questions 1ldquoJob analysis job description and job specification are interrelatedrdquo Comment Answer) Job analysis is a systematic and detailed examination of a job to collect all the relevant information about it The contents off the job are summarised in the job description The qualification needed for the job are summarised in job specificationThus there is close interrelationship between job analysis job description and job specification

Question 2ldquoJob enlargement is a horizontal extension of a job whereas job enrichment is a vertical extension of a jobrdquo ElucidateAnswer) Job enlargement involves adding one or more task to a job coma where as job enrichment involves adding more autonomy and responsibility to a job Job enlargement is therefore horizontal extension of a job coma whereas job enrichment is a vertical extension of a job

Question 3 )

What is manpower estimation Explain its quantitative and qualitative aspectsAnswer) Manpower estimation is the process by which management determines how an organisation should move from itrsquos current manpower positionto its desired manpower position There are two dimensions of Manpower estimation- quantitative and qualitative

Quantitative aspectThis aspect of Manpower estimation involves estimating the number of employees required in a future time period Workload analysis and workforce analysis are done to estimate the quantity of required manpower

Qualitative aspectThe estimate of the knowledge skills experience etc of required manpower is the qualitative aspect of Manpower estimation The quality of Manpower can be judged on the basis of job analysisand job specification

COMMERCE

CAPITAL-FIXED AND WORKING

Today let us start the class by discussing the sources of finance for different types of business firms

The term lsquocapitalrsquo refers to the investment made in the enterprise for the purpose of earning profits

Requirements of capital and sources of capital for different types of business firms are

1 Capital for sole proprietorship businessA sole proprietor operates at a small scale and thereforerequires a limited amount of capital

2 Capital for partnership firmCapital requirements as well as capital base of a partnership is bigger than that of a sole trader businessThe owned capital is contributed by the partners in an agreed ratio

3 Capital for joint stock companyA joint stock company generally requires large amount of capitalA public company can raise huge capital through issue of shares In addition to share capital it can utilize retained profits

Now let u discuss the meaning of Finance PlanningFinance planning is the process of estimation the financial requirements of an organization specifying the sources of firms and ensuring that enough funds are available at the right time

1 What do you mean by Finance PlanningAnswer Finance planning is the process of estimation the financial requirements of an organization specifying the sources of firms and ensuring that enough funds are available at the right time

2Discuss the role of financial planning of an enterpriseThe role of financial planning are as followsa A sound financial plan helps a business enterprise to avaid the problems of shortage and surplus of fundsbFinancial planning serves as a guide in developing a sound capital structure so as to maximize returns to shareholders c It helps in effective utilization of fundsd It provides policies and procedures for coordinating different functional areas or departments of businesse It enables the management to exercise effective control over the financial activities of an enterprisef It helps the company to prepare for facing business shocks and surprises in future

Mathematics

Continuity and differentiability

Recall Definition of ContinuityLet f(x) be a single valued function of x and x=a be a point in the domain of definition of the function The function is said to be continuous at x=a ifi) f(c) is defined ie f(x) has a definite finite value at x=cii) lim xrarra f(x) exists andiii) lim xrarra f(x) =f(a) In other words f(x) is said to be continuous at x=a if lim xrarra+ f(x)= lim xrarra- f(x) = f(a) Or f(a+0) =f(a-0) =f(a) Or lim hrarr0 f(a+h)= f(a) Algebra of continuous functionsNow we will study some algebra of continuous functions Theorem 1 Suppose f and g be two real functions continuous at a real number c Then(1) f + g is continuous at x = c(2) f ndash g is continuous at x = c(3) f g is continuous at x = c(4) (fg) is continuous at x = c (provided g (c) ne 0)

Example 1 Prove that every rational function is continuousSolution Recall that every rational function f is given byf(x)=[p(x) q(x) ] q(x)ne0where p and q are polynomial functions The domain of f is all real numbers except points at which q is zero Since polynomial functions are continuous f is continuous by (4) of Theorem 1Example 2Discuss the continuity of sine functionSolution To see this we use the following factslim xrarr0 sin x =0Now observe that f (x) = sin x is defined for every real number Let c be a real number Put x = c + h If x rarr c we know that h rarr 0 Therefore lim xrarrc f(x)

= lim xrarrc sin x= lim hrarr0 sin(c+h) =lim hrarr0 [sin c cos h + cos c sin h ]=lim hrarr0 (sin c cos h) + lim hrarr0 (cos c sin h) = sin c +0=sin c = f(c) Thus lim xrarrc f(x) = f(c) and hence f is a constant function Exercise Prove that the function f(x) = x2 +2x is continuous for every real value of x [Hints show that lim xrarra+ f(x) = lim xrarra- f(x) = f(a) ]

Biology Reproduction in Flowering plants We will discuss about megasporoangium

megasporagenesis and female gametophyte

Q4 Describe the structure of megasporangium

Ovule is attached to the placenta by astalk called funicle

Each ovule has one two or three protectivecoverings called integuments

At the tip of integuments a small openingcalled micropyle is organised

Opposite to the micropylar end is the chalaza

Within the integuments a mass of cellsnucellusand inside it embryo sac orfemale gametophyte is present

Q5 Describe a mature embryosacamp its formation

In most of the flowering plants only oneof the 4 megaspores formed as a result ofmegasporogenesis that is functional while theother three degenerate

The

functional megaspore develops into thefemale gametophyte

Formation The nucleus

of the functional megasporedivides mitotically to form two nuclei first andthen two more sequential mitotic nucleardivisions result in the formation of four ampthen eight nucleate stages of embryo sac

Six of the eight nucleus are surrounded bycell walls and organised into cells

The remaining two nuclei called polar nuclei are found below the egg apparatus in the largecentral cell

Three cells consisting of two synergids amp one egg cell present bottom of

embryo sac Three cells

at the chalazal as antipodal cells

Two polar nuclei together present in large central cell

HISTORY

TOWARDS INDEPENDENCE AND PARTITION THE LAST PHASE(1935-1947)SUB TOPIC NATIONAL MOVEMENTS DURING THE SECOND WORLD WAR

Spread of Quit India Movement On 9th August 1942Gandhiji and other Congress leaders were arrested The Congress was declared illegal The news of the arrest of all leaders marked the beginning of a widespread movement of India It was not possible for such a movement to remain peacefulBut the arrest of the all notable congress leaders virtually left the movement in the hands of the mass The movement took the form of violent and militant outbreakBesides congressmen revolutionaries also were very active in the movement The Congress Socialist group also played a prominent role

1 Question Why did the British authority arrest the Congress leaders on 9 th August 1942Answer Congress Working committee adopted the Quit India resolution which was to be ratified at the Bombay AICC meeting in 8th August 1942 They decided to launch a mass struggle on non-violent lines Gandhiji gave a clarion call to all section of the people rdquoKarenge ya Marengerdquo (do or die) Congress leaders gave the call to driving out

the British from IndiaViceroy had taken strong action against the Quit India movement Gandhiji and all the leaders of Congress were arrested

2 Question How did Quit India Movement spread out all over IndiaAnswer The news of the leaders lsquo arrest marked the beginning of a widespread movement to remain peacefulThe movement took form of violent outbreak There were widespread cutting of telephone and Telegraph wires damaging railway lines raising barricades in cities and towns and other forms of violent demonstations

Question Name the leaders of Congress

Socialist group played a prominent part Notable among the Jayprakash Narayan Rammonohar Lohia Aruna Asaf Ali

Political science

Topic-Franchise and Representation

Summary

The election commission

The Election Commission of India is an autonomous constitutional authority responsible for administering Union and State election processes in India The body administers elections to the Lok Sabha Rajya Sabha State Legislative Assemblies in India and the offices of the President and Vice President in the country

Functions of election commission-

India is a sovereign socialist secular democratic republic Democracy runs like a golden thread in the social economic and political fabric woven by the Constitution given by lsquoWe the People of Indiarsquo unto ourselves The concept of democracy as visualised by the Constitution pre-supposes the representation of the people in Parliament and State legislatures by the method of election The Supreme Court has held that democracy is one of the inalienable basic features of the Constitution of India and forms part of its basic structure The Constitution of India adopted a Parliamentary form of government Parliament consists of the President of India and the two Houses mdash Rajya Sabha and Lok Sabha India being a Union of states has separate state legislatures for each state State legislatures consist of the Governor and two Houses mdash Legislative Council and Legislative Assembly mdash in seven states namely Andhra Pradesh Telangana Bihar Jammu amp Kashmir Karnataka Maharashtra and Uttar Pradesh and of the Governor and the state Legislative Assembly in the remaining 22 states Apart from the above two out of the seven Union Territories namely National Capital Territory of Delhi and Puducherry also have their Legislative Assemblies

ExecutionShort notes-Election commissionFunctions of election commission

Homework- Learn

Computer

Science

Computer hardware NAND Gate

A NOT-AND operation is known as NAND operation It has n input (n gt= 2) and one output

Logic diagram

Truth Table

NOR Gate

A NOT-OR operation is known as NOR operation It has n input (n gt= 2) and one output

Logic diagram

Truth Table

XOR Gate

XOR or Ex-OR gate is a special type of gate It can be used in the half

adder full adder and subtractor The exclusive-OR gate is abbreviated as EX-OR gate or sometime as X-OR gate It has n input (n gt= 2) and one output

Logic diagram

Truth Table

XNOR Gate

XNOR gate is a special type of gate It can be used in the half adder full adder and subtractor The exclusive-NOR gate is abbreviated as EX-NOR gate or sometime as X-NOR gate It has n input (n gt= 2) and one output

Logic diagram

Truth Table

Physics

Chapter 1 Electric Field ( Electric Dipole) (Summary)

Here we will derive Expression of electric field at broad side

On position of dipole

Execution

Q With the help of a labelled diagram obtain an expression for the electric field intensity E at any point on the equitorial line ( broad-side on position) of an electric dipole

Ans

E1 E1sinθ

E θ P E1 θ

( r2+L2)12 E2 E

r E2 E2sinθ

-q θ L O L +qA B

Let us consider that the point P is situated on the right bisector of the dipole AB at a distance r meter from its midpoint O

Let E1 and E2 be the electric field intensities of the electric field at P due to charge +q and ndashq of the dipole resp The distance of P from each charge is ( r2+L2)12

So E1 = 14 πϵ q

(r 2+L 2) away from +q

E2 = 14 πϵ q

(r 2+L 2) towards ndashq

The magnitudes of E1 and E2 are equal but directions are different Now resolving E1 and E2 into two components parallel and perpendicular to AB we get

The components perpendicular to AB E1sinθ and E2sinθ cancel each other because they are equal and opposite

The components parallel to AB are E1cosθ and E2 cosθ are in same direction and add up

So resultant intensity of electric field at the point P is

E = E1cosθ + E2 cosθ

E = 14 πϵ q

(r 2+L 2) 2 cosθ

Now from fig we have cosθ =BOBP = L (r2+L2)12

So we get E = 14 πϵ 2qL ( r2+L2)32

Now electric dipole moment p= 2qL

So E = 14 πϵ p ( r2+L2)32

HW Find the expression of Electric field as done here but this time take r gtgt 2L

Also find the expression of torque experience by a dipole

(Hint Electric force experienced by charges of dipole in electric field is qE each Let θ be the angle which dipole makes with electric lines of force then perpendicular distance between two charges is 2Lsinθ Then torque = force x perp distance = qE x 2L sinθ So τ=pE sinθ where p =2qL )

STUDY MATERIAL

Class XIISubject Eng Literature (The Tempest ndash William Shakespeare) Topic Act IV Scene 1 Lines 84 to 133 (Iris hellip A contract of true love Be not too late ) Date 27th April 2020 (4th Period)

[Students should read the original play and also the paraphrase given in the school prescribed textbook]Summary Questions amp Answers

o Ceres soon appears and comes to know that she has been summoned to celebrate the contract of true love

o Ceres expresses her unwillingness to meet Venus and Cupid as she has shunned their company

o Ceres and Juno both bestow their blessings upon Ferdinand and Miranda with June gifting honour riches happiness in marriage and Ceres presents plenty of earthrsquos produce

o Iris summons the water-nymphs and reapers to come and celebrate a contract

(1) IRIS Of her society (Line 91-101)

Be not afraid I met her deity

Cutting the clouds towards Pathos and her sonDove-drawn with her Here thought they to have doneSome wanton charm upon this man and maidWhose vows are that no bed-right shall be paidTill Hymens torch be lightedmdashbut in vainMarss hot minion is returned againHer waspish-headed son has broke his arrowsSwears he will shoot no more but play with sparrowsAnd be a boy right out

(i) Where were Venus and Cupid seen flying How were they travelling Why did they want to join the marriage celebration of Ferdinand and Miranda

of true love

Venus and Cupid were seen flying through the air towards Paphos the famous city which is situated on the island of Cyprus They were travelling by air-borne chariot drawn by doves They certainly wanted to come here in order to play some amorous trick upon Ferdinand and Miranda who are under a vow not to gratify their physical desires till the holy ceremony of their marriage has been performed(ii) What have Venus and Cupid done after failing in their plan

After being failure of their plan Venus who is a very passionate deity and who is the mistress of Mars (the god of war) has gone back while here ill-tempered son Cupid has broken his arrows of love in his state of desperation(iii) What has Cupid firmly decided

Cupid is feeling so disappointed that he has firmly decided to shoot no more arrows to arouse love in human hearts but to spend his time playing with sparrows Thus he would now become just a boy and would give up his original function of shooting arrows on human beings to make them fall in love(iv) What vow had Ceres taken How did Ceres feel at the abduction

After the abduction of her daughter Prosperina by Pluto Ceres had taken a vow to always keep away from the disgraceful company of Venus and her blind son Cupid the god of love Ceres felt deeply distressed when Pluto had carried off her daughter and had made her his wife by force(v) Why has Ceres not forgiven Venus and her blind son For what do Ceres want to be sure

As the abduction had been manipulated by Venus the goddess of beauty and love and her blind son Cupid Ceres has never forgiven them for their part in the whole plot Ceres wants to be sure that she would not have to meet Venus and Cupid who had engineered the abduction of her daughter Prosperina

AS THIS lsquoMASQUErsquo SCENE IS VERY IMPORTANT IN THE PLAY THE PARAPHRASE OF THE ENTIRE PORTION OF MASQUE SCENE (Act IV Lines 58 to 143) IS GIVEN BELOW

IRIS Goddess of RainbowCERES Goddess of Agriculture and all the fruits of the earth

(Nature growth prosperity rebirth ndash notions intimately connected to marriage)JUNO The majestic Queen of Heavens and wife of Jupiter (Jupiter is the king of Gods)

VENUS The Goddess of love CUPID Son of Venus PLUTO God of death (In the play referred by Shakespeare as lsquoDisrsquo which is a Roman name for Pluto)

ORIGINAL TEXT PARAPHRASEPROSPEROWellmdash

PROSPERONow come Ariel Let there be too many rather than too few

Now come my Ariel Bring a corollaryRather than want a spirit Appear and pertly[to Ferdinand and Miranda]No tongue all eyes Be silent

spirits in attendance Appear briskly

[to Ferdinand and Miranda]Look with your eyes but do not say a word

[Soft music] [Soft music][Enter Iris] [Enter Iris]

IRISCeres most bounteous lady thy rich leasOf wheat rye barley vetches oats and peasThy turfy mountains where live nibbling sheepAnd flat meads thatched with stover them to keepThy banks with pioned and twilled brimsWhich spongy April at thy hest betrimsTo make cold nymphs chaste crowns and thybroom-grovesWhose shadow the dismissegraved bachelor lovesBeing lass-lorn thy pole clipped vineyardAnd thy sea-marge sterile and rocky-hardWhere thou thyself dost airmdashthe Queen othrsquoSkyWhose watery arch and messenger am IBids thee leave these and with her sovereign grace[Juno appears] Here on this grass-plot in this very placeTo come and sport Her peacocks fly amainApproach rich Ceres her to entertain

IRISCeres most generous lady you are the cause of rich fields or fertile land where wheat rye barley beans oats and peas grow the grassy mountains where the sheep graze and the flat meadows covered with coarse hay to be used as fodder for cattleYour banks are covered with marsh-marigolds and reeds and the rainy April under your orders brings forth to make for the maids who are not in love beautiful crowns your woods where the broom flourishes and where the bachelor who has been dismissed by the maid he loved lies down being forsaken your vineyard in which the poles are embraced by the vines and the margin of the sea which is barren and rocky where you roam about to enjoy the fresh air ndash the queen of the sky (Juno) whose messenger I am besides being represented as the rainbow bids you leave all these and with her majesty here on this grassy plot in this very place come and sport her peacocks carry her fast in her chariot through the air and are making their way here approach rich Ceres to welcome her

[Enter Ariel as Ceres] [Enter Ariel as Ceres]

CERESHail many-coloured messenger that neerDost disobey the wife of JupiterWho with thy saffron wings upon my flowersDiffusest honey-drops refreshing showersAnd with each end of thy blue bow dost crownMy bosky acres and my unshrubbed downRich scarf to my proud earth Why hath thy queenSummoned me hither to this short-grassed green

CERESWelcome rainbow that never dared disobey Juno the wife of Jupiter who with your orange coloured rays spread honey-drops refreshing showers And with each end of thy blue bow drown my bushy acres and my hilly country which is free from shrubs you thus forming a rich scarf Why has your queen called me here to this place covered with short grass

IRISA contract of true love to celebrateAnd some donation freely to estateOn the blest lovers

IRISI have called you to celebrate a contract of true love and bestow some liberal gift upon the blessed lovers

ORIGINAL TEXT PARAPHRASECERESTell me heavenly bowIf Venus or her son as thou dost knowDo now attend the queen Since they did plotThe means that dusky Dis my daughter gotHer and her blind boys scandaled companyI have forsworn

CERESTell me heavenly bow if Venus the Goddess of love or Cupid her son and pedlar of passion at this time attend the heavenly queen Juno because you are sure to know Since the day they conspired against me and dark Pluto took away my daughter here and Cupidrsquos disgraceful company I have left off

IRISOf her societyBe not afraid I met her deityCutting the clouds towards Pathos and her sonDove-drawn with her Here thought they to have doneSome wanton charm upon this man and miad

IRISBe not afraid of her company I met her deity moving on the clouds towards Paphos the sacred home of Venus on the island of Cyprus along with her son on her chariot drawn by doves Here they contemplated to exercise a charm upon this man and maid producing

Whose vows are that no bed-right shall be paidTill Hymens torch be lightedmdashbut in vainMarss hot minion is returned againHer waspish-headed son has broke his arrowsSwears he will shoot no more but play with sparrowsAnd be a boy right out

wantonness before the actual marriage ceremony but did not succeed Venus has returned her irritable son has broken his arrows and swears that he will give up his practice of trying to inspire love but play with sparrows and be a boy again

[Music is heard] [Music is heard]

CERESHighst queen of stateGreat Juno comes I know her by her gait

CERESHighest queen of state Great Juno there she comes I know here by her gait

[Enter Juno] [Enter Juno]

JUNOHow does my bounteous sister Go with meTo bless this twain that they may Prosperous beAnd honoured in their issue

JUNOHow are you doing my generous sister Come with me to bless this couple so that they may be prosperous and fortunate in their children

[They sing] [They sing]

JUNOHonour riches marriage-blessingLong continuance and increasingHourly joys be still upon youJuno sings her blessings upon you

JUNOMay honour riches happiness in marriage long continuance and increase of those boons ever rest upon you as hourly joys Juno showers down upon you her blessings in song

CERESEarths increase foison plentyBarns and garners never emptyVines and clustring bunches growingPlants and goodly burden bowingSpring come to you at the farthestIn the very end of harvestScarcity and want shall shun youCeresrsquo blessing so is on you

CERESMay you have the plenty of earthrsquos produce Your barns and granaries may never be empty Your vines may grow with clustering bunches Your fruit trees may be heavily laden with their fruit May there be continuous spring and harvest May scantiness and want leave you forever Such is the blessing of Ceres upon you

FERDINANDThis is a most majestic vision andHarmoniously charmingly May I be boldTo think these spirits

FERDINANDThis is a great vision and magically melodious Should I suppose the characters (taking part in the masque) are spirits

PROSPEROSpirits which by mine artI have from their confines calld to enactMy present fancies

PROSPEROYes they are spirits whom I have summoned from the regions to which they are confined to carry into effect my fanciful designs

ORIGINAL TEXT PARAPHRASEFERDINANDLet me live here everSo rare a wondered father and a wifeMakes this place paradise

FERDINANDI should like to live here forever Such a wise and wonderful father makes this place a paradise

[Juno and Ceres whisper and send Iris on employment] [Juno and Ceres whisper and send Iris on employment]

PROSPEROSweet now silence

PROSPEROMy dear Ferdinand speak no more Juno and Ceres are

Juno and Ceres whisper seriouslyTheres something else to do Hush and be muteOr else our spell is marred

whispering with a solemn look There is something else coming Silence Or else our magic will be spoilt

IRISYour nymphs called naiads of the wandering brooksWith your sedged crowns and over-harmless looksLeave your crisp channels and on this green landAnswer your summons Juno does commandCome temperate nymphs and help to celebrateA contract of true love Be not too late

IRISYou nymphs called Naiads denizens (M inhabitants) of the running stream with your chaplets of sedge and ever-helpful looks leave your wrinkled channels and on the green land answer the summons sent to you Juno has ordered some chaste nymphs and help to celebrate a noble and true marriage Donrsquot delay

[Enter certain nymphs] [Enter certain nymphs]You sunburnt sicklemen of August wearyCome hither from the furrow and be merryMake holiday your rye-straw hats put onAnd these fresh nymphs encounter every oneIn country footing

You sunburnt harvesters weary from the effects of the heat in August come here from the furrowed land and rejoice Make holiday with your rye-straw hats upon you and meet these fresh nymphs and join in country dancing

[Enter certain reapers properly habited They join with the nymphs in a graceful dance towards the end whereof Prospero starts suddenly and speaks]

[Enter certain reapers properly habited They join with the nymphs in a graceful dance towards the end whereof Prospero starts suddenly and speaks]

PROSPERO[aside] I had forgot that foul conspiracyOf the beast Caliban and his confederatesAgainst my life The minute of their plotIs almost come [to the spirits]Well done Avoidno more

PROSPERO(Aside)I had forgotten the wicked conspiracy of the beast Caliban and his accomplices against my life the time of their plot has almost arrived ndash (To the Spirits) well done depart no more of this

[To a strange hollow and confused noise the spirits heavily vanish]

[The spirits depart]

ORIGINAL TEXT PARAPHRASEFERDINANDLet me live here everSo rare a wondered father and a wifeMakes this place paradise

FERDINANDI should like to live here forever Such a wise and wonderful father makes this place a paradise

[Juno and Ceres whisper and send Iris on employment] [Juno and Ceres whisper and send Iris on employment]

PROSPEROSweet now silenceJuno and Ceres whisper seriouslyTheres something else to do Hush and be muteOr else our spell is marred

PROSPEROMy dear Ferdinand speak no more Juno and Ceres are whispering with a solemn look There is something else coming Silence Or else our magic will be spoilt

IRISYour nymphs called naiads of the wandering brooksWith your sedged crowns and over-harmless looksLeave your crisp channels and on this green landAnswer your summons Juno does commandCome temperate nymphs and help to celebrateA contract of true love Be not too late

IRISYou nymphs called Naiads denizens (M inhabitants) of the running stream with your chaplets of sedge and ever-helpful looks leave your wrinkled channels and on the green land answer the summons sent to you Juno has ordered some chaste nymphs and help to celebrate a noble and true marriage Donrsquot delay

[Enter certain nymphs] [Enter certain nymphs]You sunburnt sicklemen of August wearyCome hither from the furrow and be merryMake holiday your rye-straw hats put onAnd these fresh nymphs encounter every oneIn country footing

You sunburnt harvesters weary from the effects of the heat in August come here from the furrowed land and rejoice Make holiday with your rye-straw hats upon you and meet these fresh nymphs and join in country dancing

[Enter certain reapers properly habited They join with the nymphs in a graceful dance towards the end whereof Prospero starts suddenly and speaks]

[Enter certain reapers properly habited They join with the nymphs in a graceful dance towards the end whereof Prospero starts suddenly and speaks]

PROSPERO[aside] I had forgot that foul conspiracyOf the beast Caliban and his confederatesAgainst my life The minute of their plotIs almost come [to the spirits]Well done Avoidno more

PROSPERO(Aside)I had forgotten the wicked conspiracy of the beast Caliban and his accomplices against my life the time of their plot has almost arrived ndash (To the Spirits) well done depart no more of this

[To a strange hollow and confused noise the spirits heavily vanish]

[The spirits depart]

Ac-12 27420 topic Revaluation of Assets and Liabilities

REVALUATION OF ASSETS AND LIABILITIES

On admission of a new partner the firm stands reconstituted and consequently the assets are revalued and liabilities are reassessed It is necessary to show the true position of the firm at the time of admission of a new partner If the values of the assets are raised gain will increase the capital of the existing partners Similarly any decrease in the value of assets ie loss will decrease the capital of the existing partners For this purpose alsquoRevaluation Accountrsquo is prepared This account is credited with all increases in the value of assets and decrease in the value of liabilities It is debited with decrease on account of value of assets and increase in the value of liabilities The balance of this account shows a gain or loss on revaluation which is transferred to the existing partnerrsquos capital account in existing profit sharing ratioAccounting for Revaluation of Assets and Liabilities when there is a Changein the Profit Sharing Ratio of Existing PartnersAssets and liabilities of a firm must also be revalued at the time of change in profit sharing ratio of existing partners The reason is that the realisable or actual value of assets and liabilities may be different from those shown in the Balance Sheet It is possible that with the passage of time some of the assets might have appreciated in value while the value of certain other assets might have decreased and no record has been made of such changes in the books of accounts Similarly there may be some unrecorded assets amp libilities that may have to be accounted for Revaluation of assets and reassessments of liabilities becomes necessary because the change in the

value of assets and liabilities belongs to the period to change in profit sharing ratio and hence must be shared by the partners in their old profit sharing ratio Revaluation of assets and reassessment of liabilities may be given effect to in two different ways (a) When revised values are to be recorded in the books and(b) When revised values are not to be recorded in the books

When revised values are to be recorded in the booksIn such a case revaluation of assets and reassessment of liabilities is done with the help of a new account called lsquoRevaluation Accountrsquo Sometimes this account is also called as lsquoProfit amp Loss Adjustment Acrsquo If there is a loss due to revaluation revaluation account is debited and if the revaluation results in a profit the revaluation account is credited The following journal entries made for this purpose are

(i) For increase in the value of assetsAsset Ac Dr (individually)To Revaluation Ac(ii) For decrease in the value of AssetRevaluation Ac Dr (individually)To Asset Ac[Decrease in the value of assets](iii) For increase in the value of LiabilitiesRevaluation Ac Dr (individually)To Liabilities Ac[Increase in the value of Liabilities](iv) For decrease in the value of LiabilitiesLiabilities Ac DrTo Revaluation Ac[Decrease in the value of Liabilities](v) For unrecorded AssetsAsset Ac [unrecorded] DrTo Revaluation Ac[Unrecorded asset recorded at actual value](vi) For unrecorded Liability Revaluation Ac DrTo Liability Ac [unrecorded][Unrecorded Liability recorded at actual value](vii) For transfer of gain on revaluationRevaluation Ac DrTo Existing Partnerrsquos CapitalCurrent Ac[Profit on revaluation transferred to capital account in existing ratio](viii) For transfer of loss on revaluationExisting Partnerrsquos CapitalCurrent Ac DrTo Revaluation Ac[Loss on revaluation transferred to capital account in existing ratio](a) When revaluation account shows gain Revaluation Ac DrTo Partnerrsquos Capital Ac (Old Profit Sharing Ratio)(Profit on revaluation credited to Partnerrsquos Capital Ac)(b) Above entry is reversed when revaluation account shows loss Partners Capital Acs (Old Profit Sharing Ratio) DrTo Revaluation Ac(Loss on revaluation debited to Partnerrsquos Capital Acs)

Proforma of Revaluation Account is given as under

Revaluation Account

Dr Cr Particulars ` Amount Particulars ` Amount To Decrease in value of assets By Increase in value of assets To Increase in value of liabilities By Decrease in value of liabilities To Unrecorded liabilities By Unrecorded assets To Gain on Revaluation (Transferred) By Loss on Revalution (Transferred)

ECO ndash12 2742020Topic- ELASTICITY OF DEMAND

CHAPTER - ELASTICITY OF DEMANDMEANINGDemand for a commodity is affected by many factors such as its price price of related goods income of its buyer tastes and preferences etc Elasticity means degree of response Elasticity of demand means degree of responsiveness of demand Demand for a commodity responds to change in price price of related goods income etc So we have three dimensions of elasticity of demandDIMENSION OF ELASTICITY OF DEMAND TYPES OF ELASTICITY OF DEMAND

Price elasticity of demand Income elasticity of demand Cross Elasticity of demand

Price elasticity of demand Price elasticity of demand means degree of responsiveness of demand for a commodity to the change in its price For example if demand for a commodity rises by 10 due to 5 fall in its price Price elasticity of demand (ep)=Percentage change in quantity demanded Percentage change in price of the commodity = 10 ( -)5 = ( - )2Note that ep will always be negative due to inverse relationship of price and quantity demanded

(ii) Income elasticity of demand Income elasticity of demand refers to the degree of responsiveness of demand for a commodity to the change in income of its buyer Suppose income of buyer rises by 10 and his demand for a commodity rises by 20 then Income elasticity of demand (ey)= change in quantity demanded change in price of the commodity =20 10 = 2

Cross Elasticity of demandCross elasticity of demand means the degree of responsiveness of demand for a commodity to the change in price of its related goods (substitute goods or complementary goods) Suppose demand for a commodity rises by 10 due to 5 rise in price of its substitute good then Cross elasticity of demand (ec) = change in quantity demanded change in price of related good = 10 2 = 5 (Tastes and preferences cannot be expressed numerically So elasticity ofdemand cannot be numerically expressed)

  • Chapter 1 Force (Summary)
  • Distinguish between external sovereignty and internal sovereignty
    • NAND Gate
      • Logic diagram
      • Truth Table
        • NOR Gate
          • Logic diagram
          • Truth Table
            • XOR Gate
              • Logic diagram
              • Truth Table
                • XNOR Gate
                  • Logic diagram
                  • Truth Table
                      • Physics
                      • Chapter 1 Electric Field ( Electric Dipole) (Summary)
Page 33:  · Web viewWe all know that Nouns are divided into two parts: common noun and proper noun.Apart from common and proper noun, we will also study about collective noun and compound

1lek le1)

Determine an angle alpha such that sin =k and -π2le αle π2

Then we have

Sin θ = k = sin α

Or sin θ - sin α =0

Or 2 cos [(θ+α) 2] sin [(θ-α) 2] =0

Therefore either cos [(θ +α) 2] =0 (1)

Or sin [(θ-α) 2] =0 (2)

Now from (1) we get (θ+α) 2= (2m+1) π2)

Or θ = (2m+1) π-α (3)

And from (2) we get (θ-α) 2 =mπ

Or θ= 2mπ+α(4)

Where m = any integer

Clearly the solution (3) amp (4) may be combined in the following form

θ= nπ+(-1) n α where n= any integer

Therefore the general solution of sin θ = sin α is θ = nπ +(-1) n α where n is any integer and -π2 le α le π2

Biology Chapter - 04Kingdom Monera

Today we will discuss about bacterial reproduction and its usefulness

Fig Binary Fission

Fig Conjugation Fig Transformation

Bacterial reproduction is mainly asexual but sexual reproduction

also takes place

Asexual reproduction takes place by i) Binary fission - from one bacteriato

two bacteria are produced in every 20 to 30mins

ii) Buddingiii) Endospore formation - during

unfourable condition

Sexual reproduction by three ways

1) Conjugation - Transfer of genetic material between cells that are in physical contact with one another

2) Transduction - Transfer of genetic materialfrom one cell to another by a bacteriophage

3) Transformation - Transfer of cell-freeor naked DNArsquo from one cell to another

Bacteria causes different diseases inplants animals and human and

it causes food spoilage and waterpollution but it also have some useful

activities

i) Bacteria are helpful in sewage water treatment

ii) It is used in antibiotic (medicine) production

iii) Anaerobic bacteria help in biogas(energy) production

iv) Many household products like yoghurt cheese are manufactured by use of bacteria

v) Rhizobium by symbiotic relationship with leguminous plant increase soil fertility

vi) Besides these bacteria is helpful in genetic engineering degradation of petroleum hydrocarbonand in dairy

industry

Physics Motion in plane Here we will introduce Projectile Motion

Execution

Projectile

Y

usinθ u h

θX

ucosθ

Suppose a body is projected with an angle θ So initial velocity u can be resolved into two components

Horizontal component - ucosθ ( for range)

Vertical component - usinθ ( for height)

usinθ changes during motion and becomes zero at maximum height position but ucosθ remain unchanged

The maximum height of projectile is h

NB If initial is upward then g = -ve and if it is downward then g = +ve Height is +ve if direction of motion does not change ( for ex a body thrown upwards but goes down ultimately then height h = -ve)

The angle of projectile θ is the angle made with horizontal

HISTORY ndash GROWTH OF NATIONALISM

SUB TOPIC- REVOLUTIONARY NATIONALISM Bengal formation of Anushilan Samity and Jugantar Group

The intensification of the Swadeshi movement and Government policy of terror and repression led to outbreak of violence Bombs were manufactured and attempts on the lives of unpopular Government officials became frequent In the gymnasium of Scottish Church College which was known as General Assemblies Institution a secret society was formed known as Anushilan Samity

Aurobindo Ghosh send from Baroda his emissary Jatindranath Banerjee to mobilize the Bengal revolutionaries

Hemchandra Qanungo and Satyen Bose published Journal Jugantar

The Jugantar group planned to assassinate oppressive magistrate Kingsford by Khudiram Bose and Prafulla Chaki in 1908 Prafulla Chaki committed suicide to avoid arrest Khudiram was tried and hanged

Afew days later the police found a bomb factory in Maniktala and arrested a large number of revolutionaries The trial of revolutionaries became famous as the Alipore Bomb Case

In the course of the trial the approver the public prosecuter and a police officer were assassinated

1 Question Name two journals which preached the cult of violence

Answer a) Yugantar edited by Bhupendranath DuttaB) Bandemataram edited by Aurobindo Ghosh2 Question Why was Khudiram arrested and hangedAnswer An attempt was made to assassinate a hated vindictive majistrate named Kingsford by Khudiram Bose and Prafulla Chaki Their attempt failed and the bomb they threw killed two English ladies Khudiram was arrested and put to trial and then hanged3Question Who was Aurubindo GhoshAnswer Aurobindo Ghosh a nationalist revolutionary who was charged for his involvement in the Alipore Bombing Case He was accused of it along with his brother Barindra nath Ghosh But Aurobindo was acquitted because of the brilliant pleading of his counsel Chittaranjan Das Then he became a spiritual reformer introducing his visions on human progress and spiritual evolution4 Qustion Who was KingsfordAnswer Kingsford was an unpopular British chief Magistrate who was the target of the bomb thrown at Muzaffarpur by Khudiram and Prafulla Chaki

Most of the accused were convicted and sentenced to

long term of imprisonmentBut

Aurobindo Ghosh was acquitted mainly owing to the brilliant pleading of his counsel Chittaranjan Das

Political science Topic-Sovereignty

Summary Sovereignty is the full right and power of a governing body over itself without any interference from outside sources or bodies In political theory sovereignty is a substantive term designating supreme legitimate authority over some polity In international law sovereignty is the exercise of power by a state

Internal Sovereignty

Internal sovereignty means supreme authority within ones territory while external sovereignty relates to the recognition on the part of all states that each possesses this power in equal measure

External sovereignty

external sovereignty relates to the recognition on the part of all states that each possesses this power in equal measure

Distinguish between

Execution

Answer the following questions

Short notes-

Sovereignty

Internal Sovereignty

External sovereignty

Homework- learn

external sovereignty and internal sovereigntySovereignty is the principle

of supreme and

unquestionable authority

reflected in the claim by the

state to be the sole author of

laws within its territory

Definition of external vs internal sovereigntyInternal sovereignty refers to

the relationship between a

sovereign power and its

subjects ndash it refers to the

location of the supreme

authority within the state In

the UK for example internal

sovereignty (supposedly)

resides within Parliament

reflected in the

constitutional principle of

parliamentary

sovereigntyBy contrast

external sovereignty refers

to the capacity of the state

to act independently and

autonomously on the world

stage This is what is

sometimes called lsquostate

sovereigntyrsquo or lsquonational

sovereigntyrsquo and implies

that states are legally equal

and that the territorial

integrity and political

independence of a state is

inviolable

Class ndash XII

Date - 2742020 STUDY MATERIALSubject Topic Summary Execution Business Studies

Job Analysis amp Manpower Planning

At first let us recall the chapter what we have discussed till nowbullJob analysisbullJob specification bullJob description bullJob enlargement bullJob enrichment

Today we will do some questions answers from the chapter

Questions 1ldquoJob analysis job description and job specification are interrelatedrdquo Comment Answer) Job analysis is a systematic and detailed examination of a job to collect all the relevant information about it The contents off the job are summarised in the job description The qualification needed for the job are summarised in job specificationThus there is close interrelationship between job analysis job description and job specification

Question 2ldquoJob enlargement is a horizontal extension of a job whereas job enrichment is a vertical extension of a jobrdquo ElucidateAnswer) Job enlargement involves adding one or more task to a job coma where as job enrichment involves adding more autonomy and responsibility to a job Job enlargement is therefore horizontal extension of a job coma whereas job enrichment is a vertical extension of a job

Question 3 )

What is manpower estimation Explain its quantitative and qualitative aspectsAnswer) Manpower estimation is the process by which management determines how an organisation should move from itrsquos current manpower positionto its desired manpower position There are two dimensions of Manpower estimation- quantitative and qualitative

Quantitative aspectThis aspect of Manpower estimation involves estimating the number of employees required in a future time period Workload analysis and workforce analysis are done to estimate the quantity of required manpower

Qualitative aspectThe estimate of the knowledge skills experience etc of required manpower is the qualitative aspect of Manpower estimation The quality of Manpower can be judged on the basis of job analysisand job specification

COMMERCE

CAPITAL-FIXED AND WORKING

Today let us start the class by discussing the sources of finance for different types of business firms

The term lsquocapitalrsquo refers to the investment made in the enterprise for the purpose of earning profits

Requirements of capital and sources of capital for different types of business firms are

1 Capital for sole proprietorship businessA sole proprietor operates at a small scale and thereforerequires a limited amount of capital

2 Capital for partnership firmCapital requirements as well as capital base of a partnership is bigger than that of a sole trader businessThe owned capital is contributed by the partners in an agreed ratio

3 Capital for joint stock companyA joint stock company generally requires large amount of capitalA public company can raise huge capital through issue of shares In addition to share capital it can utilize retained profits

Now let u discuss the meaning of Finance PlanningFinance planning is the process of estimation the financial requirements of an organization specifying the sources of firms and ensuring that enough funds are available at the right time

1 What do you mean by Finance PlanningAnswer Finance planning is the process of estimation the financial requirements of an organization specifying the sources of firms and ensuring that enough funds are available at the right time

2Discuss the role of financial planning of an enterpriseThe role of financial planning are as followsa A sound financial plan helps a business enterprise to avaid the problems of shortage and surplus of fundsbFinancial planning serves as a guide in developing a sound capital structure so as to maximize returns to shareholders c It helps in effective utilization of fundsd It provides policies and procedures for coordinating different functional areas or departments of businesse It enables the management to exercise effective control over the financial activities of an enterprisef It helps the company to prepare for facing business shocks and surprises in future

Mathematics

Continuity and differentiability

Recall Definition of ContinuityLet f(x) be a single valued function of x and x=a be a point in the domain of definition of the function The function is said to be continuous at x=a ifi) f(c) is defined ie f(x) has a definite finite value at x=cii) lim xrarra f(x) exists andiii) lim xrarra f(x) =f(a) In other words f(x) is said to be continuous at x=a if lim xrarra+ f(x)= lim xrarra- f(x) = f(a) Or f(a+0) =f(a-0) =f(a) Or lim hrarr0 f(a+h)= f(a) Algebra of continuous functionsNow we will study some algebra of continuous functions Theorem 1 Suppose f and g be two real functions continuous at a real number c Then(1) f + g is continuous at x = c(2) f ndash g is continuous at x = c(3) f g is continuous at x = c(4) (fg) is continuous at x = c (provided g (c) ne 0)

Example 1 Prove that every rational function is continuousSolution Recall that every rational function f is given byf(x)=[p(x) q(x) ] q(x)ne0where p and q are polynomial functions The domain of f is all real numbers except points at which q is zero Since polynomial functions are continuous f is continuous by (4) of Theorem 1Example 2Discuss the continuity of sine functionSolution To see this we use the following factslim xrarr0 sin x =0Now observe that f (x) = sin x is defined for every real number Let c be a real number Put x = c + h If x rarr c we know that h rarr 0 Therefore lim xrarrc f(x)

= lim xrarrc sin x= lim hrarr0 sin(c+h) =lim hrarr0 [sin c cos h + cos c sin h ]=lim hrarr0 (sin c cos h) + lim hrarr0 (cos c sin h) = sin c +0=sin c = f(c) Thus lim xrarrc f(x) = f(c) and hence f is a constant function Exercise Prove that the function f(x) = x2 +2x is continuous for every real value of x [Hints show that lim xrarra+ f(x) = lim xrarra- f(x) = f(a) ]

Biology Reproduction in Flowering plants We will discuss about megasporoangium

megasporagenesis and female gametophyte

Q4 Describe the structure of megasporangium

Ovule is attached to the placenta by astalk called funicle

Each ovule has one two or three protectivecoverings called integuments

At the tip of integuments a small openingcalled micropyle is organised

Opposite to the micropylar end is the chalaza

Within the integuments a mass of cellsnucellusand inside it embryo sac orfemale gametophyte is present

Q5 Describe a mature embryosacamp its formation

In most of the flowering plants only oneof the 4 megaspores formed as a result ofmegasporogenesis that is functional while theother three degenerate

The

functional megaspore develops into thefemale gametophyte

Formation The nucleus

of the functional megasporedivides mitotically to form two nuclei first andthen two more sequential mitotic nucleardivisions result in the formation of four ampthen eight nucleate stages of embryo sac

Six of the eight nucleus are surrounded bycell walls and organised into cells

The remaining two nuclei called polar nuclei are found below the egg apparatus in the largecentral cell

Three cells consisting of two synergids amp one egg cell present bottom of

embryo sac Three cells

at the chalazal as antipodal cells

Two polar nuclei together present in large central cell

HISTORY

TOWARDS INDEPENDENCE AND PARTITION THE LAST PHASE(1935-1947)SUB TOPIC NATIONAL MOVEMENTS DURING THE SECOND WORLD WAR

Spread of Quit India Movement On 9th August 1942Gandhiji and other Congress leaders were arrested The Congress was declared illegal The news of the arrest of all leaders marked the beginning of a widespread movement of India It was not possible for such a movement to remain peacefulBut the arrest of the all notable congress leaders virtually left the movement in the hands of the mass The movement took the form of violent and militant outbreakBesides congressmen revolutionaries also were very active in the movement The Congress Socialist group also played a prominent role

1 Question Why did the British authority arrest the Congress leaders on 9 th August 1942Answer Congress Working committee adopted the Quit India resolution which was to be ratified at the Bombay AICC meeting in 8th August 1942 They decided to launch a mass struggle on non-violent lines Gandhiji gave a clarion call to all section of the people rdquoKarenge ya Marengerdquo (do or die) Congress leaders gave the call to driving out

the British from IndiaViceroy had taken strong action against the Quit India movement Gandhiji and all the leaders of Congress were arrested

2 Question How did Quit India Movement spread out all over IndiaAnswer The news of the leaders lsquo arrest marked the beginning of a widespread movement to remain peacefulThe movement took form of violent outbreak There were widespread cutting of telephone and Telegraph wires damaging railway lines raising barricades in cities and towns and other forms of violent demonstations

Question Name the leaders of Congress

Socialist group played a prominent part Notable among the Jayprakash Narayan Rammonohar Lohia Aruna Asaf Ali

Political science

Topic-Franchise and Representation

Summary

The election commission

The Election Commission of India is an autonomous constitutional authority responsible for administering Union and State election processes in India The body administers elections to the Lok Sabha Rajya Sabha State Legislative Assemblies in India and the offices of the President and Vice President in the country

Functions of election commission-

India is a sovereign socialist secular democratic republic Democracy runs like a golden thread in the social economic and political fabric woven by the Constitution given by lsquoWe the People of Indiarsquo unto ourselves The concept of democracy as visualised by the Constitution pre-supposes the representation of the people in Parliament and State legislatures by the method of election The Supreme Court has held that democracy is one of the inalienable basic features of the Constitution of India and forms part of its basic structure The Constitution of India adopted a Parliamentary form of government Parliament consists of the President of India and the two Houses mdash Rajya Sabha and Lok Sabha India being a Union of states has separate state legislatures for each state State legislatures consist of the Governor and two Houses mdash Legislative Council and Legislative Assembly mdash in seven states namely Andhra Pradesh Telangana Bihar Jammu amp Kashmir Karnataka Maharashtra and Uttar Pradesh and of the Governor and the state Legislative Assembly in the remaining 22 states Apart from the above two out of the seven Union Territories namely National Capital Territory of Delhi and Puducherry also have their Legislative Assemblies

ExecutionShort notes-Election commissionFunctions of election commission

Homework- Learn

Computer

Science

Computer hardware NAND Gate

A NOT-AND operation is known as NAND operation It has n input (n gt= 2) and one output

Logic diagram

Truth Table

NOR Gate

A NOT-OR operation is known as NOR operation It has n input (n gt= 2) and one output

Logic diagram

Truth Table

XOR Gate

XOR or Ex-OR gate is a special type of gate It can be used in the half

adder full adder and subtractor The exclusive-OR gate is abbreviated as EX-OR gate or sometime as X-OR gate It has n input (n gt= 2) and one output

Logic diagram

Truth Table

XNOR Gate

XNOR gate is a special type of gate It can be used in the half adder full adder and subtractor The exclusive-NOR gate is abbreviated as EX-NOR gate or sometime as X-NOR gate It has n input (n gt= 2) and one output

Logic diagram

Truth Table

Physics

Chapter 1 Electric Field ( Electric Dipole) (Summary)

Here we will derive Expression of electric field at broad side

On position of dipole

Execution

Q With the help of a labelled diagram obtain an expression for the electric field intensity E at any point on the equitorial line ( broad-side on position) of an electric dipole

Ans

E1 E1sinθ

E θ P E1 θ

( r2+L2)12 E2 E

r E2 E2sinθ

-q θ L O L +qA B

Let us consider that the point P is situated on the right bisector of the dipole AB at a distance r meter from its midpoint O

Let E1 and E2 be the electric field intensities of the electric field at P due to charge +q and ndashq of the dipole resp The distance of P from each charge is ( r2+L2)12

So E1 = 14 πϵ q

(r 2+L 2) away from +q

E2 = 14 πϵ q

(r 2+L 2) towards ndashq

The magnitudes of E1 and E2 are equal but directions are different Now resolving E1 and E2 into two components parallel and perpendicular to AB we get

The components perpendicular to AB E1sinθ and E2sinθ cancel each other because they are equal and opposite

The components parallel to AB are E1cosθ and E2 cosθ are in same direction and add up

So resultant intensity of electric field at the point P is

E = E1cosθ + E2 cosθ

E = 14 πϵ q

(r 2+L 2) 2 cosθ

Now from fig we have cosθ =BOBP = L (r2+L2)12

So we get E = 14 πϵ 2qL ( r2+L2)32

Now electric dipole moment p= 2qL

So E = 14 πϵ p ( r2+L2)32

HW Find the expression of Electric field as done here but this time take r gtgt 2L

Also find the expression of torque experience by a dipole

(Hint Electric force experienced by charges of dipole in electric field is qE each Let θ be the angle which dipole makes with electric lines of force then perpendicular distance between two charges is 2Lsinθ Then torque = force x perp distance = qE x 2L sinθ So τ=pE sinθ where p =2qL )

STUDY MATERIAL

Class XIISubject Eng Literature (The Tempest ndash William Shakespeare) Topic Act IV Scene 1 Lines 84 to 133 (Iris hellip A contract of true love Be not too late ) Date 27th April 2020 (4th Period)

[Students should read the original play and also the paraphrase given in the school prescribed textbook]Summary Questions amp Answers

o Ceres soon appears and comes to know that she has been summoned to celebrate the contract of true love

o Ceres expresses her unwillingness to meet Venus and Cupid as she has shunned their company

o Ceres and Juno both bestow their blessings upon Ferdinand and Miranda with June gifting honour riches happiness in marriage and Ceres presents plenty of earthrsquos produce

o Iris summons the water-nymphs and reapers to come and celebrate a contract

(1) IRIS Of her society (Line 91-101)

Be not afraid I met her deity

Cutting the clouds towards Pathos and her sonDove-drawn with her Here thought they to have doneSome wanton charm upon this man and maidWhose vows are that no bed-right shall be paidTill Hymens torch be lightedmdashbut in vainMarss hot minion is returned againHer waspish-headed son has broke his arrowsSwears he will shoot no more but play with sparrowsAnd be a boy right out

(i) Where were Venus and Cupid seen flying How were they travelling Why did they want to join the marriage celebration of Ferdinand and Miranda

of true love

Venus and Cupid were seen flying through the air towards Paphos the famous city which is situated on the island of Cyprus They were travelling by air-borne chariot drawn by doves They certainly wanted to come here in order to play some amorous trick upon Ferdinand and Miranda who are under a vow not to gratify their physical desires till the holy ceremony of their marriage has been performed(ii) What have Venus and Cupid done after failing in their plan

After being failure of their plan Venus who is a very passionate deity and who is the mistress of Mars (the god of war) has gone back while here ill-tempered son Cupid has broken his arrows of love in his state of desperation(iii) What has Cupid firmly decided

Cupid is feeling so disappointed that he has firmly decided to shoot no more arrows to arouse love in human hearts but to spend his time playing with sparrows Thus he would now become just a boy and would give up his original function of shooting arrows on human beings to make them fall in love(iv) What vow had Ceres taken How did Ceres feel at the abduction

After the abduction of her daughter Prosperina by Pluto Ceres had taken a vow to always keep away from the disgraceful company of Venus and her blind son Cupid the god of love Ceres felt deeply distressed when Pluto had carried off her daughter and had made her his wife by force(v) Why has Ceres not forgiven Venus and her blind son For what do Ceres want to be sure

As the abduction had been manipulated by Venus the goddess of beauty and love and her blind son Cupid Ceres has never forgiven them for their part in the whole plot Ceres wants to be sure that she would not have to meet Venus and Cupid who had engineered the abduction of her daughter Prosperina

AS THIS lsquoMASQUErsquo SCENE IS VERY IMPORTANT IN THE PLAY THE PARAPHRASE OF THE ENTIRE PORTION OF MASQUE SCENE (Act IV Lines 58 to 143) IS GIVEN BELOW

IRIS Goddess of RainbowCERES Goddess of Agriculture and all the fruits of the earth

(Nature growth prosperity rebirth ndash notions intimately connected to marriage)JUNO The majestic Queen of Heavens and wife of Jupiter (Jupiter is the king of Gods)

VENUS The Goddess of love CUPID Son of Venus PLUTO God of death (In the play referred by Shakespeare as lsquoDisrsquo which is a Roman name for Pluto)

ORIGINAL TEXT PARAPHRASEPROSPEROWellmdash

PROSPERONow come Ariel Let there be too many rather than too few

Now come my Ariel Bring a corollaryRather than want a spirit Appear and pertly[to Ferdinand and Miranda]No tongue all eyes Be silent

spirits in attendance Appear briskly

[to Ferdinand and Miranda]Look with your eyes but do not say a word

[Soft music] [Soft music][Enter Iris] [Enter Iris]

IRISCeres most bounteous lady thy rich leasOf wheat rye barley vetches oats and peasThy turfy mountains where live nibbling sheepAnd flat meads thatched with stover them to keepThy banks with pioned and twilled brimsWhich spongy April at thy hest betrimsTo make cold nymphs chaste crowns and thybroom-grovesWhose shadow the dismissegraved bachelor lovesBeing lass-lorn thy pole clipped vineyardAnd thy sea-marge sterile and rocky-hardWhere thou thyself dost airmdashthe Queen othrsquoSkyWhose watery arch and messenger am IBids thee leave these and with her sovereign grace[Juno appears] Here on this grass-plot in this very placeTo come and sport Her peacocks fly amainApproach rich Ceres her to entertain

IRISCeres most generous lady you are the cause of rich fields or fertile land where wheat rye barley beans oats and peas grow the grassy mountains where the sheep graze and the flat meadows covered with coarse hay to be used as fodder for cattleYour banks are covered with marsh-marigolds and reeds and the rainy April under your orders brings forth to make for the maids who are not in love beautiful crowns your woods where the broom flourishes and where the bachelor who has been dismissed by the maid he loved lies down being forsaken your vineyard in which the poles are embraced by the vines and the margin of the sea which is barren and rocky where you roam about to enjoy the fresh air ndash the queen of the sky (Juno) whose messenger I am besides being represented as the rainbow bids you leave all these and with her majesty here on this grassy plot in this very place come and sport her peacocks carry her fast in her chariot through the air and are making their way here approach rich Ceres to welcome her

[Enter Ariel as Ceres] [Enter Ariel as Ceres]

CERESHail many-coloured messenger that neerDost disobey the wife of JupiterWho with thy saffron wings upon my flowersDiffusest honey-drops refreshing showersAnd with each end of thy blue bow dost crownMy bosky acres and my unshrubbed downRich scarf to my proud earth Why hath thy queenSummoned me hither to this short-grassed green

CERESWelcome rainbow that never dared disobey Juno the wife of Jupiter who with your orange coloured rays spread honey-drops refreshing showers And with each end of thy blue bow drown my bushy acres and my hilly country which is free from shrubs you thus forming a rich scarf Why has your queen called me here to this place covered with short grass

IRISA contract of true love to celebrateAnd some donation freely to estateOn the blest lovers

IRISI have called you to celebrate a contract of true love and bestow some liberal gift upon the blessed lovers

ORIGINAL TEXT PARAPHRASECERESTell me heavenly bowIf Venus or her son as thou dost knowDo now attend the queen Since they did plotThe means that dusky Dis my daughter gotHer and her blind boys scandaled companyI have forsworn

CERESTell me heavenly bow if Venus the Goddess of love or Cupid her son and pedlar of passion at this time attend the heavenly queen Juno because you are sure to know Since the day they conspired against me and dark Pluto took away my daughter here and Cupidrsquos disgraceful company I have left off

IRISOf her societyBe not afraid I met her deityCutting the clouds towards Pathos and her sonDove-drawn with her Here thought they to have doneSome wanton charm upon this man and miad

IRISBe not afraid of her company I met her deity moving on the clouds towards Paphos the sacred home of Venus on the island of Cyprus along with her son on her chariot drawn by doves Here they contemplated to exercise a charm upon this man and maid producing

Whose vows are that no bed-right shall be paidTill Hymens torch be lightedmdashbut in vainMarss hot minion is returned againHer waspish-headed son has broke his arrowsSwears he will shoot no more but play with sparrowsAnd be a boy right out

wantonness before the actual marriage ceremony but did not succeed Venus has returned her irritable son has broken his arrows and swears that he will give up his practice of trying to inspire love but play with sparrows and be a boy again

[Music is heard] [Music is heard]

CERESHighst queen of stateGreat Juno comes I know her by her gait

CERESHighest queen of state Great Juno there she comes I know here by her gait

[Enter Juno] [Enter Juno]

JUNOHow does my bounteous sister Go with meTo bless this twain that they may Prosperous beAnd honoured in their issue

JUNOHow are you doing my generous sister Come with me to bless this couple so that they may be prosperous and fortunate in their children

[They sing] [They sing]

JUNOHonour riches marriage-blessingLong continuance and increasingHourly joys be still upon youJuno sings her blessings upon you

JUNOMay honour riches happiness in marriage long continuance and increase of those boons ever rest upon you as hourly joys Juno showers down upon you her blessings in song

CERESEarths increase foison plentyBarns and garners never emptyVines and clustring bunches growingPlants and goodly burden bowingSpring come to you at the farthestIn the very end of harvestScarcity and want shall shun youCeresrsquo blessing so is on you

CERESMay you have the plenty of earthrsquos produce Your barns and granaries may never be empty Your vines may grow with clustering bunches Your fruit trees may be heavily laden with their fruit May there be continuous spring and harvest May scantiness and want leave you forever Such is the blessing of Ceres upon you

FERDINANDThis is a most majestic vision andHarmoniously charmingly May I be boldTo think these spirits

FERDINANDThis is a great vision and magically melodious Should I suppose the characters (taking part in the masque) are spirits

PROSPEROSpirits which by mine artI have from their confines calld to enactMy present fancies

PROSPEROYes they are spirits whom I have summoned from the regions to which they are confined to carry into effect my fanciful designs

ORIGINAL TEXT PARAPHRASEFERDINANDLet me live here everSo rare a wondered father and a wifeMakes this place paradise

FERDINANDI should like to live here forever Such a wise and wonderful father makes this place a paradise

[Juno and Ceres whisper and send Iris on employment] [Juno and Ceres whisper and send Iris on employment]

PROSPEROSweet now silence

PROSPEROMy dear Ferdinand speak no more Juno and Ceres are

Juno and Ceres whisper seriouslyTheres something else to do Hush and be muteOr else our spell is marred

whispering with a solemn look There is something else coming Silence Or else our magic will be spoilt

IRISYour nymphs called naiads of the wandering brooksWith your sedged crowns and over-harmless looksLeave your crisp channels and on this green landAnswer your summons Juno does commandCome temperate nymphs and help to celebrateA contract of true love Be not too late

IRISYou nymphs called Naiads denizens (M inhabitants) of the running stream with your chaplets of sedge and ever-helpful looks leave your wrinkled channels and on the green land answer the summons sent to you Juno has ordered some chaste nymphs and help to celebrate a noble and true marriage Donrsquot delay

[Enter certain nymphs] [Enter certain nymphs]You sunburnt sicklemen of August wearyCome hither from the furrow and be merryMake holiday your rye-straw hats put onAnd these fresh nymphs encounter every oneIn country footing

You sunburnt harvesters weary from the effects of the heat in August come here from the furrowed land and rejoice Make holiday with your rye-straw hats upon you and meet these fresh nymphs and join in country dancing

[Enter certain reapers properly habited They join with the nymphs in a graceful dance towards the end whereof Prospero starts suddenly and speaks]

[Enter certain reapers properly habited They join with the nymphs in a graceful dance towards the end whereof Prospero starts suddenly and speaks]

PROSPERO[aside] I had forgot that foul conspiracyOf the beast Caliban and his confederatesAgainst my life The minute of their plotIs almost come [to the spirits]Well done Avoidno more

PROSPERO(Aside)I had forgotten the wicked conspiracy of the beast Caliban and his accomplices against my life the time of their plot has almost arrived ndash (To the Spirits) well done depart no more of this

[To a strange hollow and confused noise the spirits heavily vanish]

[The spirits depart]

ORIGINAL TEXT PARAPHRASEFERDINANDLet me live here everSo rare a wondered father and a wifeMakes this place paradise

FERDINANDI should like to live here forever Such a wise and wonderful father makes this place a paradise

[Juno and Ceres whisper and send Iris on employment] [Juno and Ceres whisper and send Iris on employment]

PROSPEROSweet now silenceJuno and Ceres whisper seriouslyTheres something else to do Hush and be muteOr else our spell is marred

PROSPEROMy dear Ferdinand speak no more Juno and Ceres are whispering with a solemn look There is something else coming Silence Or else our magic will be spoilt

IRISYour nymphs called naiads of the wandering brooksWith your sedged crowns and over-harmless looksLeave your crisp channels and on this green landAnswer your summons Juno does commandCome temperate nymphs and help to celebrateA contract of true love Be not too late

IRISYou nymphs called Naiads denizens (M inhabitants) of the running stream with your chaplets of sedge and ever-helpful looks leave your wrinkled channels and on the green land answer the summons sent to you Juno has ordered some chaste nymphs and help to celebrate a noble and true marriage Donrsquot delay

[Enter certain nymphs] [Enter certain nymphs]You sunburnt sicklemen of August wearyCome hither from the furrow and be merryMake holiday your rye-straw hats put onAnd these fresh nymphs encounter every oneIn country footing

You sunburnt harvesters weary from the effects of the heat in August come here from the furrowed land and rejoice Make holiday with your rye-straw hats upon you and meet these fresh nymphs and join in country dancing

[Enter certain reapers properly habited They join with the nymphs in a graceful dance towards the end whereof Prospero starts suddenly and speaks]

[Enter certain reapers properly habited They join with the nymphs in a graceful dance towards the end whereof Prospero starts suddenly and speaks]

PROSPERO[aside] I had forgot that foul conspiracyOf the beast Caliban and his confederatesAgainst my life The minute of their plotIs almost come [to the spirits]Well done Avoidno more

PROSPERO(Aside)I had forgotten the wicked conspiracy of the beast Caliban and his accomplices against my life the time of their plot has almost arrived ndash (To the Spirits) well done depart no more of this

[To a strange hollow and confused noise the spirits heavily vanish]

[The spirits depart]

Ac-12 27420 topic Revaluation of Assets and Liabilities

REVALUATION OF ASSETS AND LIABILITIES

On admission of a new partner the firm stands reconstituted and consequently the assets are revalued and liabilities are reassessed It is necessary to show the true position of the firm at the time of admission of a new partner If the values of the assets are raised gain will increase the capital of the existing partners Similarly any decrease in the value of assets ie loss will decrease the capital of the existing partners For this purpose alsquoRevaluation Accountrsquo is prepared This account is credited with all increases in the value of assets and decrease in the value of liabilities It is debited with decrease on account of value of assets and increase in the value of liabilities The balance of this account shows a gain or loss on revaluation which is transferred to the existing partnerrsquos capital account in existing profit sharing ratioAccounting for Revaluation of Assets and Liabilities when there is a Changein the Profit Sharing Ratio of Existing PartnersAssets and liabilities of a firm must also be revalued at the time of change in profit sharing ratio of existing partners The reason is that the realisable or actual value of assets and liabilities may be different from those shown in the Balance Sheet It is possible that with the passage of time some of the assets might have appreciated in value while the value of certain other assets might have decreased and no record has been made of such changes in the books of accounts Similarly there may be some unrecorded assets amp libilities that may have to be accounted for Revaluation of assets and reassessments of liabilities becomes necessary because the change in the

value of assets and liabilities belongs to the period to change in profit sharing ratio and hence must be shared by the partners in their old profit sharing ratio Revaluation of assets and reassessment of liabilities may be given effect to in two different ways (a) When revised values are to be recorded in the books and(b) When revised values are not to be recorded in the books

When revised values are to be recorded in the booksIn such a case revaluation of assets and reassessment of liabilities is done with the help of a new account called lsquoRevaluation Accountrsquo Sometimes this account is also called as lsquoProfit amp Loss Adjustment Acrsquo If there is a loss due to revaluation revaluation account is debited and if the revaluation results in a profit the revaluation account is credited The following journal entries made for this purpose are

(i) For increase in the value of assetsAsset Ac Dr (individually)To Revaluation Ac(ii) For decrease in the value of AssetRevaluation Ac Dr (individually)To Asset Ac[Decrease in the value of assets](iii) For increase in the value of LiabilitiesRevaluation Ac Dr (individually)To Liabilities Ac[Increase in the value of Liabilities](iv) For decrease in the value of LiabilitiesLiabilities Ac DrTo Revaluation Ac[Decrease in the value of Liabilities](v) For unrecorded AssetsAsset Ac [unrecorded] DrTo Revaluation Ac[Unrecorded asset recorded at actual value](vi) For unrecorded Liability Revaluation Ac DrTo Liability Ac [unrecorded][Unrecorded Liability recorded at actual value](vii) For transfer of gain on revaluationRevaluation Ac DrTo Existing Partnerrsquos CapitalCurrent Ac[Profit on revaluation transferred to capital account in existing ratio](viii) For transfer of loss on revaluationExisting Partnerrsquos CapitalCurrent Ac DrTo Revaluation Ac[Loss on revaluation transferred to capital account in existing ratio](a) When revaluation account shows gain Revaluation Ac DrTo Partnerrsquos Capital Ac (Old Profit Sharing Ratio)(Profit on revaluation credited to Partnerrsquos Capital Ac)(b) Above entry is reversed when revaluation account shows loss Partners Capital Acs (Old Profit Sharing Ratio) DrTo Revaluation Ac(Loss on revaluation debited to Partnerrsquos Capital Acs)

Proforma of Revaluation Account is given as under

Revaluation Account

Dr Cr Particulars ` Amount Particulars ` Amount To Decrease in value of assets By Increase in value of assets To Increase in value of liabilities By Decrease in value of liabilities To Unrecorded liabilities By Unrecorded assets To Gain on Revaluation (Transferred) By Loss on Revalution (Transferred)

ECO ndash12 2742020Topic- ELASTICITY OF DEMAND

CHAPTER - ELASTICITY OF DEMANDMEANINGDemand for a commodity is affected by many factors such as its price price of related goods income of its buyer tastes and preferences etc Elasticity means degree of response Elasticity of demand means degree of responsiveness of demand Demand for a commodity responds to change in price price of related goods income etc So we have three dimensions of elasticity of demandDIMENSION OF ELASTICITY OF DEMAND TYPES OF ELASTICITY OF DEMAND

Price elasticity of demand Income elasticity of demand Cross Elasticity of demand

Price elasticity of demand Price elasticity of demand means degree of responsiveness of demand for a commodity to the change in its price For example if demand for a commodity rises by 10 due to 5 fall in its price Price elasticity of demand (ep)=Percentage change in quantity demanded Percentage change in price of the commodity = 10 ( -)5 = ( - )2Note that ep will always be negative due to inverse relationship of price and quantity demanded

(ii) Income elasticity of demand Income elasticity of demand refers to the degree of responsiveness of demand for a commodity to the change in income of its buyer Suppose income of buyer rises by 10 and his demand for a commodity rises by 20 then Income elasticity of demand (ey)= change in quantity demanded change in price of the commodity =20 10 = 2

Cross Elasticity of demandCross elasticity of demand means the degree of responsiveness of demand for a commodity to the change in price of its related goods (substitute goods or complementary goods) Suppose demand for a commodity rises by 10 due to 5 rise in price of its substitute good then Cross elasticity of demand (ec) = change in quantity demanded change in price of related good = 10 2 = 5 (Tastes and preferences cannot be expressed numerically So elasticity ofdemand cannot be numerically expressed)

  • Chapter 1 Force (Summary)
  • Distinguish between external sovereignty and internal sovereignty
    • NAND Gate
      • Logic diagram
      • Truth Table
        • NOR Gate
          • Logic diagram
          • Truth Table
            • XOR Gate
              • Logic diagram
              • Truth Table
                • XNOR Gate
                  • Logic diagram
                  • Truth Table
                      • Physics
                      • Chapter 1 Electric Field ( Electric Dipole) (Summary)
Page 34:  · Web viewWe all know that Nouns are divided into two parts: common noun and proper noun.Apart from common and proper noun, we will also study about collective noun and compound

Biology Chapter - 04Kingdom Monera

Today we will discuss about bacterial reproduction and its usefulness

Fig Binary Fission

Fig Conjugation Fig Transformation

Bacterial reproduction is mainly asexual but sexual reproduction

also takes place

Asexual reproduction takes place by i) Binary fission - from one bacteriato

two bacteria are produced in every 20 to 30mins

ii) Buddingiii) Endospore formation - during

unfourable condition

Sexual reproduction by three ways

1) Conjugation - Transfer of genetic material between cells that are in physical contact with one another

2) Transduction - Transfer of genetic materialfrom one cell to another by a bacteriophage

3) Transformation - Transfer of cell-freeor naked DNArsquo from one cell to another

Bacteria causes different diseases inplants animals and human and

it causes food spoilage and waterpollution but it also have some useful

activities

i) Bacteria are helpful in sewage water treatment

ii) It is used in antibiotic (medicine) production

iii) Anaerobic bacteria help in biogas(energy) production

iv) Many household products like yoghurt cheese are manufactured by use of bacteria

v) Rhizobium by symbiotic relationship with leguminous plant increase soil fertility

vi) Besides these bacteria is helpful in genetic engineering degradation of petroleum hydrocarbonand in dairy

industry

Physics Motion in plane Here we will introduce Projectile Motion

Execution

Projectile

Y

usinθ u h

θX

ucosθ

Suppose a body is projected with an angle θ So initial velocity u can be resolved into two components

Horizontal component - ucosθ ( for range)

Vertical component - usinθ ( for height)

usinθ changes during motion and becomes zero at maximum height position but ucosθ remain unchanged

The maximum height of projectile is h

NB If initial is upward then g = -ve and if it is downward then g = +ve Height is +ve if direction of motion does not change ( for ex a body thrown upwards but goes down ultimately then height h = -ve)

The angle of projectile θ is the angle made with horizontal

HISTORY ndash GROWTH OF NATIONALISM

SUB TOPIC- REVOLUTIONARY NATIONALISM Bengal formation of Anushilan Samity and Jugantar Group

The intensification of the Swadeshi movement and Government policy of terror and repression led to outbreak of violence Bombs were manufactured and attempts on the lives of unpopular Government officials became frequent In the gymnasium of Scottish Church College which was known as General Assemblies Institution a secret society was formed known as Anushilan Samity

Aurobindo Ghosh send from Baroda his emissary Jatindranath Banerjee to mobilize the Bengal revolutionaries

Hemchandra Qanungo and Satyen Bose published Journal Jugantar

The Jugantar group planned to assassinate oppressive magistrate Kingsford by Khudiram Bose and Prafulla Chaki in 1908 Prafulla Chaki committed suicide to avoid arrest Khudiram was tried and hanged

Afew days later the police found a bomb factory in Maniktala and arrested a large number of revolutionaries The trial of revolutionaries became famous as the Alipore Bomb Case

In the course of the trial the approver the public prosecuter and a police officer were assassinated

1 Question Name two journals which preached the cult of violence

Answer a) Yugantar edited by Bhupendranath DuttaB) Bandemataram edited by Aurobindo Ghosh2 Question Why was Khudiram arrested and hangedAnswer An attempt was made to assassinate a hated vindictive majistrate named Kingsford by Khudiram Bose and Prafulla Chaki Their attempt failed and the bomb they threw killed two English ladies Khudiram was arrested and put to trial and then hanged3Question Who was Aurubindo GhoshAnswer Aurobindo Ghosh a nationalist revolutionary who was charged for his involvement in the Alipore Bombing Case He was accused of it along with his brother Barindra nath Ghosh But Aurobindo was acquitted because of the brilliant pleading of his counsel Chittaranjan Das Then he became a spiritual reformer introducing his visions on human progress and spiritual evolution4 Qustion Who was KingsfordAnswer Kingsford was an unpopular British chief Magistrate who was the target of the bomb thrown at Muzaffarpur by Khudiram and Prafulla Chaki

Most of the accused were convicted and sentenced to

long term of imprisonmentBut

Aurobindo Ghosh was acquitted mainly owing to the brilliant pleading of his counsel Chittaranjan Das

Political science Topic-Sovereignty

Summary Sovereignty is the full right and power of a governing body over itself without any interference from outside sources or bodies In political theory sovereignty is a substantive term designating supreme legitimate authority over some polity In international law sovereignty is the exercise of power by a state

Internal Sovereignty

Internal sovereignty means supreme authority within ones territory while external sovereignty relates to the recognition on the part of all states that each possesses this power in equal measure

External sovereignty

external sovereignty relates to the recognition on the part of all states that each possesses this power in equal measure

Distinguish between

Execution

Answer the following questions

Short notes-

Sovereignty

Internal Sovereignty

External sovereignty

Homework- learn

external sovereignty and internal sovereigntySovereignty is the principle

of supreme and

unquestionable authority

reflected in the claim by the

state to be the sole author of

laws within its territory

Definition of external vs internal sovereigntyInternal sovereignty refers to

the relationship between a

sovereign power and its

subjects ndash it refers to the

location of the supreme

authority within the state In

the UK for example internal

sovereignty (supposedly)

resides within Parliament

reflected in the

constitutional principle of

parliamentary

sovereigntyBy contrast

external sovereignty refers

to the capacity of the state

to act independently and

autonomously on the world

stage This is what is

sometimes called lsquostate

sovereigntyrsquo or lsquonational

sovereigntyrsquo and implies

that states are legally equal

and that the territorial

integrity and political

independence of a state is

inviolable

Class ndash XII

Date - 2742020 STUDY MATERIALSubject Topic Summary Execution Business Studies

Job Analysis amp Manpower Planning

At first let us recall the chapter what we have discussed till nowbullJob analysisbullJob specification bullJob description bullJob enlargement bullJob enrichment

Today we will do some questions answers from the chapter

Questions 1ldquoJob analysis job description and job specification are interrelatedrdquo Comment Answer) Job analysis is a systematic and detailed examination of a job to collect all the relevant information about it The contents off the job are summarised in the job description The qualification needed for the job are summarised in job specificationThus there is close interrelationship between job analysis job description and job specification

Question 2ldquoJob enlargement is a horizontal extension of a job whereas job enrichment is a vertical extension of a jobrdquo ElucidateAnswer) Job enlargement involves adding one or more task to a job coma where as job enrichment involves adding more autonomy and responsibility to a job Job enlargement is therefore horizontal extension of a job coma whereas job enrichment is a vertical extension of a job

Question 3 )

What is manpower estimation Explain its quantitative and qualitative aspectsAnswer) Manpower estimation is the process by which management determines how an organisation should move from itrsquos current manpower positionto its desired manpower position There are two dimensions of Manpower estimation- quantitative and qualitative

Quantitative aspectThis aspect of Manpower estimation involves estimating the number of employees required in a future time period Workload analysis and workforce analysis are done to estimate the quantity of required manpower

Qualitative aspectThe estimate of the knowledge skills experience etc of required manpower is the qualitative aspect of Manpower estimation The quality of Manpower can be judged on the basis of job analysisand job specification

COMMERCE

CAPITAL-FIXED AND WORKING

Today let us start the class by discussing the sources of finance for different types of business firms

The term lsquocapitalrsquo refers to the investment made in the enterprise for the purpose of earning profits

Requirements of capital and sources of capital for different types of business firms are

1 Capital for sole proprietorship businessA sole proprietor operates at a small scale and thereforerequires a limited amount of capital

2 Capital for partnership firmCapital requirements as well as capital base of a partnership is bigger than that of a sole trader businessThe owned capital is contributed by the partners in an agreed ratio

3 Capital for joint stock companyA joint stock company generally requires large amount of capitalA public company can raise huge capital through issue of shares In addition to share capital it can utilize retained profits

Now let u discuss the meaning of Finance PlanningFinance planning is the process of estimation the financial requirements of an organization specifying the sources of firms and ensuring that enough funds are available at the right time

1 What do you mean by Finance PlanningAnswer Finance planning is the process of estimation the financial requirements of an organization specifying the sources of firms and ensuring that enough funds are available at the right time

2Discuss the role of financial planning of an enterpriseThe role of financial planning are as followsa A sound financial plan helps a business enterprise to avaid the problems of shortage and surplus of fundsbFinancial planning serves as a guide in developing a sound capital structure so as to maximize returns to shareholders c It helps in effective utilization of fundsd It provides policies and procedures for coordinating different functional areas or departments of businesse It enables the management to exercise effective control over the financial activities of an enterprisef It helps the company to prepare for facing business shocks and surprises in future

Mathematics

Continuity and differentiability

Recall Definition of ContinuityLet f(x) be a single valued function of x and x=a be a point in the domain of definition of the function The function is said to be continuous at x=a ifi) f(c) is defined ie f(x) has a definite finite value at x=cii) lim xrarra f(x) exists andiii) lim xrarra f(x) =f(a) In other words f(x) is said to be continuous at x=a if lim xrarra+ f(x)= lim xrarra- f(x) = f(a) Or f(a+0) =f(a-0) =f(a) Or lim hrarr0 f(a+h)= f(a) Algebra of continuous functionsNow we will study some algebra of continuous functions Theorem 1 Suppose f and g be two real functions continuous at a real number c Then(1) f + g is continuous at x = c(2) f ndash g is continuous at x = c(3) f g is continuous at x = c(4) (fg) is continuous at x = c (provided g (c) ne 0)

Example 1 Prove that every rational function is continuousSolution Recall that every rational function f is given byf(x)=[p(x) q(x) ] q(x)ne0where p and q are polynomial functions The domain of f is all real numbers except points at which q is zero Since polynomial functions are continuous f is continuous by (4) of Theorem 1Example 2Discuss the continuity of sine functionSolution To see this we use the following factslim xrarr0 sin x =0Now observe that f (x) = sin x is defined for every real number Let c be a real number Put x = c + h If x rarr c we know that h rarr 0 Therefore lim xrarrc f(x)

= lim xrarrc sin x= lim hrarr0 sin(c+h) =lim hrarr0 [sin c cos h + cos c sin h ]=lim hrarr0 (sin c cos h) + lim hrarr0 (cos c sin h) = sin c +0=sin c = f(c) Thus lim xrarrc f(x) = f(c) and hence f is a constant function Exercise Prove that the function f(x) = x2 +2x is continuous for every real value of x [Hints show that lim xrarra+ f(x) = lim xrarra- f(x) = f(a) ]

Biology Reproduction in Flowering plants We will discuss about megasporoangium

megasporagenesis and female gametophyte

Q4 Describe the structure of megasporangium

Ovule is attached to the placenta by astalk called funicle

Each ovule has one two or three protectivecoverings called integuments

At the tip of integuments a small openingcalled micropyle is organised

Opposite to the micropylar end is the chalaza

Within the integuments a mass of cellsnucellusand inside it embryo sac orfemale gametophyte is present

Q5 Describe a mature embryosacamp its formation

In most of the flowering plants only oneof the 4 megaspores formed as a result ofmegasporogenesis that is functional while theother three degenerate

The

functional megaspore develops into thefemale gametophyte

Formation The nucleus

of the functional megasporedivides mitotically to form two nuclei first andthen two more sequential mitotic nucleardivisions result in the formation of four ampthen eight nucleate stages of embryo sac

Six of the eight nucleus are surrounded bycell walls and organised into cells

The remaining two nuclei called polar nuclei are found below the egg apparatus in the largecentral cell

Three cells consisting of two synergids amp one egg cell present bottom of

embryo sac Three cells

at the chalazal as antipodal cells

Two polar nuclei together present in large central cell

HISTORY

TOWARDS INDEPENDENCE AND PARTITION THE LAST PHASE(1935-1947)SUB TOPIC NATIONAL MOVEMENTS DURING THE SECOND WORLD WAR

Spread of Quit India Movement On 9th August 1942Gandhiji and other Congress leaders were arrested The Congress was declared illegal The news of the arrest of all leaders marked the beginning of a widespread movement of India It was not possible for such a movement to remain peacefulBut the arrest of the all notable congress leaders virtually left the movement in the hands of the mass The movement took the form of violent and militant outbreakBesides congressmen revolutionaries also were very active in the movement The Congress Socialist group also played a prominent role

1 Question Why did the British authority arrest the Congress leaders on 9 th August 1942Answer Congress Working committee adopted the Quit India resolution which was to be ratified at the Bombay AICC meeting in 8th August 1942 They decided to launch a mass struggle on non-violent lines Gandhiji gave a clarion call to all section of the people rdquoKarenge ya Marengerdquo (do or die) Congress leaders gave the call to driving out

the British from IndiaViceroy had taken strong action against the Quit India movement Gandhiji and all the leaders of Congress were arrested

2 Question How did Quit India Movement spread out all over IndiaAnswer The news of the leaders lsquo arrest marked the beginning of a widespread movement to remain peacefulThe movement took form of violent outbreak There were widespread cutting of telephone and Telegraph wires damaging railway lines raising barricades in cities and towns and other forms of violent demonstations

Question Name the leaders of Congress

Socialist group played a prominent part Notable among the Jayprakash Narayan Rammonohar Lohia Aruna Asaf Ali

Political science

Topic-Franchise and Representation

Summary

The election commission

The Election Commission of India is an autonomous constitutional authority responsible for administering Union and State election processes in India The body administers elections to the Lok Sabha Rajya Sabha State Legislative Assemblies in India and the offices of the President and Vice President in the country

Functions of election commission-

India is a sovereign socialist secular democratic republic Democracy runs like a golden thread in the social economic and political fabric woven by the Constitution given by lsquoWe the People of Indiarsquo unto ourselves The concept of democracy as visualised by the Constitution pre-supposes the representation of the people in Parliament and State legislatures by the method of election The Supreme Court has held that democracy is one of the inalienable basic features of the Constitution of India and forms part of its basic structure The Constitution of India adopted a Parliamentary form of government Parliament consists of the President of India and the two Houses mdash Rajya Sabha and Lok Sabha India being a Union of states has separate state legislatures for each state State legislatures consist of the Governor and two Houses mdash Legislative Council and Legislative Assembly mdash in seven states namely Andhra Pradesh Telangana Bihar Jammu amp Kashmir Karnataka Maharashtra and Uttar Pradesh and of the Governor and the state Legislative Assembly in the remaining 22 states Apart from the above two out of the seven Union Territories namely National Capital Territory of Delhi and Puducherry also have their Legislative Assemblies

ExecutionShort notes-Election commissionFunctions of election commission

Homework- Learn

Computer

Science

Computer hardware NAND Gate

A NOT-AND operation is known as NAND operation It has n input (n gt= 2) and one output

Logic diagram

Truth Table

NOR Gate

A NOT-OR operation is known as NOR operation It has n input (n gt= 2) and one output

Logic diagram

Truth Table

XOR Gate

XOR or Ex-OR gate is a special type of gate It can be used in the half

adder full adder and subtractor The exclusive-OR gate is abbreviated as EX-OR gate or sometime as X-OR gate It has n input (n gt= 2) and one output

Logic diagram

Truth Table

XNOR Gate

XNOR gate is a special type of gate It can be used in the half adder full adder and subtractor The exclusive-NOR gate is abbreviated as EX-NOR gate or sometime as X-NOR gate It has n input (n gt= 2) and one output

Logic diagram

Truth Table

Physics

Chapter 1 Electric Field ( Electric Dipole) (Summary)

Here we will derive Expression of electric field at broad side

On position of dipole

Execution

Q With the help of a labelled diagram obtain an expression for the electric field intensity E at any point on the equitorial line ( broad-side on position) of an electric dipole

Ans

E1 E1sinθ

E θ P E1 θ

( r2+L2)12 E2 E

r E2 E2sinθ

-q θ L O L +qA B

Let us consider that the point P is situated on the right bisector of the dipole AB at a distance r meter from its midpoint O

Let E1 and E2 be the electric field intensities of the electric field at P due to charge +q and ndashq of the dipole resp The distance of P from each charge is ( r2+L2)12

So E1 = 14 πϵ q

(r 2+L 2) away from +q

E2 = 14 πϵ q

(r 2+L 2) towards ndashq

The magnitudes of E1 and E2 are equal but directions are different Now resolving E1 and E2 into two components parallel and perpendicular to AB we get

The components perpendicular to AB E1sinθ and E2sinθ cancel each other because they are equal and opposite

The components parallel to AB are E1cosθ and E2 cosθ are in same direction and add up

So resultant intensity of electric field at the point P is

E = E1cosθ + E2 cosθ

E = 14 πϵ q

(r 2+L 2) 2 cosθ

Now from fig we have cosθ =BOBP = L (r2+L2)12

So we get E = 14 πϵ 2qL ( r2+L2)32

Now electric dipole moment p= 2qL

So E = 14 πϵ p ( r2+L2)32

HW Find the expression of Electric field as done here but this time take r gtgt 2L

Also find the expression of torque experience by a dipole

(Hint Electric force experienced by charges of dipole in electric field is qE each Let θ be the angle which dipole makes with electric lines of force then perpendicular distance between two charges is 2Lsinθ Then torque = force x perp distance = qE x 2L sinθ So τ=pE sinθ where p =2qL )

STUDY MATERIAL

Class XIISubject Eng Literature (The Tempest ndash William Shakespeare) Topic Act IV Scene 1 Lines 84 to 133 (Iris hellip A contract of true love Be not too late ) Date 27th April 2020 (4th Period)

[Students should read the original play and also the paraphrase given in the school prescribed textbook]Summary Questions amp Answers

o Ceres soon appears and comes to know that she has been summoned to celebrate the contract of true love

o Ceres expresses her unwillingness to meet Venus and Cupid as she has shunned their company

o Ceres and Juno both bestow their blessings upon Ferdinand and Miranda with June gifting honour riches happiness in marriage and Ceres presents plenty of earthrsquos produce

o Iris summons the water-nymphs and reapers to come and celebrate a contract

(1) IRIS Of her society (Line 91-101)

Be not afraid I met her deity

Cutting the clouds towards Pathos and her sonDove-drawn with her Here thought they to have doneSome wanton charm upon this man and maidWhose vows are that no bed-right shall be paidTill Hymens torch be lightedmdashbut in vainMarss hot minion is returned againHer waspish-headed son has broke his arrowsSwears he will shoot no more but play with sparrowsAnd be a boy right out

(i) Where were Venus and Cupid seen flying How were they travelling Why did they want to join the marriage celebration of Ferdinand and Miranda

of true love

Venus and Cupid were seen flying through the air towards Paphos the famous city which is situated on the island of Cyprus They were travelling by air-borne chariot drawn by doves They certainly wanted to come here in order to play some amorous trick upon Ferdinand and Miranda who are under a vow not to gratify their physical desires till the holy ceremony of their marriage has been performed(ii) What have Venus and Cupid done after failing in their plan

After being failure of their plan Venus who is a very passionate deity and who is the mistress of Mars (the god of war) has gone back while here ill-tempered son Cupid has broken his arrows of love in his state of desperation(iii) What has Cupid firmly decided

Cupid is feeling so disappointed that he has firmly decided to shoot no more arrows to arouse love in human hearts but to spend his time playing with sparrows Thus he would now become just a boy and would give up his original function of shooting arrows on human beings to make them fall in love(iv) What vow had Ceres taken How did Ceres feel at the abduction

After the abduction of her daughter Prosperina by Pluto Ceres had taken a vow to always keep away from the disgraceful company of Venus and her blind son Cupid the god of love Ceres felt deeply distressed when Pluto had carried off her daughter and had made her his wife by force(v) Why has Ceres not forgiven Venus and her blind son For what do Ceres want to be sure

As the abduction had been manipulated by Venus the goddess of beauty and love and her blind son Cupid Ceres has never forgiven them for their part in the whole plot Ceres wants to be sure that she would not have to meet Venus and Cupid who had engineered the abduction of her daughter Prosperina

AS THIS lsquoMASQUErsquo SCENE IS VERY IMPORTANT IN THE PLAY THE PARAPHRASE OF THE ENTIRE PORTION OF MASQUE SCENE (Act IV Lines 58 to 143) IS GIVEN BELOW

IRIS Goddess of RainbowCERES Goddess of Agriculture and all the fruits of the earth

(Nature growth prosperity rebirth ndash notions intimately connected to marriage)JUNO The majestic Queen of Heavens and wife of Jupiter (Jupiter is the king of Gods)

VENUS The Goddess of love CUPID Son of Venus PLUTO God of death (In the play referred by Shakespeare as lsquoDisrsquo which is a Roman name for Pluto)

ORIGINAL TEXT PARAPHRASEPROSPEROWellmdash

PROSPERONow come Ariel Let there be too many rather than too few

Now come my Ariel Bring a corollaryRather than want a spirit Appear and pertly[to Ferdinand and Miranda]No tongue all eyes Be silent

spirits in attendance Appear briskly

[to Ferdinand and Miranda]Look with your eyes but do not say a word

[Soft music] [Soft music][Enter Iris] [Enter Iris]

IRISCeres most bounteous lady thy rich leasOf wheat rye barley vetches oats and peasThy turfy mountains where live nibbling sheepAnd flat meads thatched with stover them to keepThy banks with pioned and twilled brimsWhich spongy April at thy hest betrimsTo make cold nymphs chaste crowns and thybroom-grovesWhose shadow the dismissegraved bachelor lovesBeing lass-lorn thy pole clipped vineyardAnd thy sea-marge sterile and rocky-hardWhere thou thyself dost airmdashthe Queen othrsquoSkyWhose watery arch and messenger am IBids thee leave these and with her sovereign grace[Juno appears] Here on this grass-plot in this very placeTo come and sport Her peacocks fly amainApproach rich Ceres her to entertain

IRISCeres most generous lady you are the cause of rich fields or fertile land where wheat rye barley beans oats and peas grow the grassy mountains where the sheep graze and the flat meadows covered with coarse hay to be used as fodder for cattleYour banks are covered with marsh-marigolds and reeds and the rainy April under your orders brings forth to make for the maids who are not in love beautiful crowns your woods where the broom flourishes and where the bachelor who has been dismissed by the maid he loved lies down being forsaken your vineyard in which the poles are embraced by the vines and the margin of the sea which is barren and rocky where you roam about to enjoy the fresh air ndash the queen of the sky (Juno) whose messenger I am besides being represented as the rainbow bids you leave all these and with her majesty here on this grassy plot in this very place come and sport her peacocks carry her fast in her chariot through the air and are making their way here approach rich Ceres to welcome her

[Enter Ariel as Ceres] [Enter Ariel as Ceres]

CERESHail many-coloured messenger that neerDost disobey the wife of JupiterWho with thy saffron wings upon my flowersDiffusest honey-drops refreshing showersAnd with each end of thy blue bow dost crownMy bosky acres and my unshrubbed downRich scarf to my proud earth Why hath thy queenSummoned me hither to this short-grassed green

CERESWelcome rainbow that never dared disobey Juno the wife of Jupiter who with your orange coloured rays spread honey-drops refreshing showers And with each end of thy blue bow drown my bushy acres and my hilly country which is free from shrubs you thus forming a rich scarf Why has your queen called me here to this place covered with short grass

IRISA contract of true love to celebrateAnd some donation freely to estateOn the blest lovers

IRISI have called you to celebrate a contract of true love and bestow some liberal gift upon the blessed lovers

ORIGINAL TEXT PARAPHRASECERESTell me heavenly bowIf Venus or her son as thou dost knowDo now attend the queen Since they did plotThe means that dusky Dis my daughter gotHer and her blind boys scandaled companyI have forsworn

CERESTell me heavenly bow if Venus the Goddess of love or Cupid her son and pedlar of passion at this time attend the heavenly queen Juno because you are sure to know Since the day they conspired against me and dark Pluto took away my daughter here and Cupidrsquos disgraceful company I have left off

IRISOf her societyBe not afraid I met her deityCutting the clouds towards Pathos and her sonDove-drawn with her Here thought they to have doneSome wanton charm upon this man and miad

IRISBe not afraid of her company I met her deity moving on the clouds towards Paphos the sacred home of Venus on the island of Cyprus along with her son on her chariot drawn by doves Here they contemplated to exercise a charm upon this man and maid producing

Whose vows are that no bed-right shall be paidTill Hymens torch be lightedmdashbut in vainMarss hot minion is returned againHer waspish-headed son has broke his arrowsSwears he will shoot no more but play with sparrowsAnd be a boy right out

wantonness before the actual marriage ceremony but did not succeed Venus has returned her irritable son has broken his arrows and swears that he will give up his practice of trying to inspire love but play with sparrows and be a boy again

[Music is heard] [Music is heard]

CERESHighst queen of stateGreat Juno comes I know her by her gait

CERESHighest queen of state Great Juno there she comes I know here by her gait

[Enter Juno] [Enter Juno]

JUNOHow does my bounteous sister Go with meTo bless this twain that they may Prosperous beAnd honoured in their issue

JUNOHow are you doing my generous sister Come with me to bless this couple so that they may be prosperous and fortunate in their children

[They sing] [They sing]

JUNOHonour riches marriage-blessingLong continuance and increasingHourly joys be still upon youJuno sings her blessings upon you

JUNOMay honour riches happiness in marriage long continuance and increase of those boons ever rest upon you as hourly joys Juno showers down upon you her blessings in song

CERESEarths increase foison plentyBarns and garners never emptyVines and clustring bunches growingPlants and goodly burden bowingSpring come to you at the farthestIn the very end of harvestScarcity and want shall shun youCeresrsquo blessing so is on you

CERESMay you have the plenty of earthrsquos produce Your barns and granaries may never be empty Your vines may grow with clustering bunches Your fruit trees may be heavily laden with their fruit May there be continuous spring and harvest May scantiness and want leave you forever Such is the blessing of Ceres upon you

FERDINANDThis is a most majestic vision andHarmoniously charmingly May I be boldTo think these spirits

FERDINANDThis is a great vision and magically melodious Should I suppose the characters (taking part in the masque) are spirits

PROSPEROSpirits which by mine artI have from their confines calld to enactMy present fancies

PROSPEROYes they are spirits whom I have summoned from the regions to which they are confined to carry into effect my fanciful designs

ORIGINAL TEXT PARAPHRASEFERDINANDLet me live here everSo rare a wondered father and a wifeMakes this place paradise

FERDINANDI should like to live here forever Such a wise and wonderful father makes this place a paradise

[Juno and Ceres whisper and send Iris on employment] [Juno and Ceres whisper and send Iris on employment]

PROSPEROSweet now silence

PROSPEROMy dear Ferdinand speak no more Juno and Ceres are

Juno and Ceres whisper seriouslyTheres something else to do Hush and be muteOr else our spell is marred

whispering with a solemn look There is something else coming Silence Or else our magic will be spoilt

IRISYour nymphs called naiads of the wandering brooksWith your sedged crowns and over-harmless looksLeave your crisp channels and on this green landAnswer your summons Juno does commandCome temperate nymphs and help to celebrateA contract of true love Be not too late

IRISYou nymphs called Naiads denizens (M inhabitants) of the running stream with your chaplets of sedge and ever-helpful looks leave your wrinkled channels and on the green land answer the summons sent to you Juno has ordered some chaste nymphs and help to celebrate a noble and true marriage Donrsquot delay

[Enter certain nymphs] [Enter certain nymphs]You sunburnt sicklemen of August wearyCome hither from the furrow and be merryMake holiday your rye-straw hats put onAnd these fresh nymphs encounter every oneIn country footing

You sunburnt harvesters weary from the effects of the heat in August come here from the furrowed land and rejoice Make holiday with your rye-straw hats upon you and meet these fresh nymphs and join in country dancing

[Enter certain reapers properly habited They join with the nymphs in a graceful dance towards the end whereof Prospero starts suddenly and speaks]

[Enter certain reapers properly habited They join with the nymphs in a graceful dance towards the end whereof Prospero starts suddenly and speaks]

PROSPERO[aside] I had forgot that foul conspiracyOf the beast Caliban and his confederatesAgainst my life The minute of their plotIs almost come [to the spirits]Well done Avoidno more

PROSPERO(Aside)I had forgotten the wicked conspiracy of the beast Caliban and his accomplices against my life the time of their plot has almost arrived ndash (To the Spirits) well done depart no more of this

[To a strange hollow and confused noise the spirits heavily vanish]

[The spirits depart]

ORIGINAL TEXT PARAPHRASEFERDINANDLet me live here everSo rare a wondered father and a wifeMakes this place paradise

FERDINANDI should like to live here forever Such a wise and wonderful father makes this place a paradise

[Juno and Ceres whisper and send Iris on employment] [Juno and Ceres whisper and send Iris on employment]

PROSPEROSweet now silenceJuno and Ceres whisper seriouslyTheres something else to do Hush and be muteOr else our spell is marred

PROSPEROMy dear Ferdinand speak no more Juno and Ceres are whispering with a solemn look There is something else coming Silence Or else our magic will be spoilt

IRISYour nymphs called naiads of the wandering brooksWith your sedged crowns and over-harmless looksLeave your crisp channels and on this green landAnswer your summons Juno does commandCome temperate nymphs and help to celebrateA contract of true love Be not too late

IRISYou nymphs called Naiads denizens (M inhabitants) of the running stream with your chaplets of sedge and ever-helpful looks leave your wrinkled channels and on the green land answer the summons sent to you Juno has ordered some chaste nymphs and help to celebrate a noble and true marriage Donrsquot delay

[Enter certain nymphs] [Enter certain nymphs]You sunburnt sicklemen of August wearyCome hither from the furrow and be merryMake holiday your rye-straw hats put onAnd these fresh nymphs encounter every oneIn country footing

You sunburnt harvesters weary from the effects of the heat in August come here from the furrowed land and rejoice Make holiday with your rye-straw hats upon you and meet these fresh nymphs and join in country dancing

[Enter certain reapers properly habited They join with the nymphs in a graceful dance towards the end whereof Prospero starts suddenly and speaks]

[Enter certain reapers properly habited They join with the nymphs in a graceful dance towards the end whereof Prospero starts suddenly and speaks]

PROSPERO[aside] I had forgot that foul conspiracyOf the beast Caliban and his confederatesAgainst my life The minute of their plotIs almost come [to the spirits]Well done Avoidno more

PROSPERO(Aside)I had forgotten the wicked conspiracy of the beast Caliban and his accomplices against my life the time of their plot has almost arrived ndash (To the Spirits) well done depart no more of this

[To a strange hollow and confused noise the spirits heavily vanish]

[The spirits depart]

Ac-12 27420 topic Revaluation of Assets and Liabilities

REVALUATION OF ASSETS AND LIABILITIES

On admission of a new partner the firm stands reconstituted and consequently the assets are revalued and liabilities are reassessed It is necessary to show the true position of the firm at the time of admission of a new partner If the values of the assets are raised gain will increase the capital of the existing partners Similarly any decrease in the value of assets ie loss will decrease the capital of the existing partners For this purpose alsquoRevaluation Accountrsquo is prepared This account is credited with all increases in the value of assets and decrease in the value of liabilities It is debited with decrease on account of value of assets and increase in the value of liabilities The balance of this account shows a gain or loss on revaluation which is transferred to the existing partnerrsquos capital account in existing profit sharing ratioAccounting for Revaluation of Assets and Liabilities when there is a Changein the Profit Sharing Ratio of Existing PartnersAssets and liabilities of a firm must also be revalued at the time of change in profit sharing ratio of existing partners The reason is that the realisable or actual value of assets and liabilities may be different from those shown in the Balance Sheet It is possible that with the passage of time some of the assets might have appreciated in value while the value of certain other assets might have decreased and no record has been made of such changes in the books of accounts Similarly there may be some unrecorded assets amp libilities that may have to be accounted for Revaluation of assets and reassessments of liabilities becomes necessary because the change in the

value of assets and liabilities belongs to the period to change in profit sharing ratio and hence must be shared by the partners in their old profit sharing ratio Revaluation of assets and reassessment of liabilities may be given effect to in two different ways (a) When revised values are to be recorded in the books and(b) When revised values are not to be recorded in the books

When revised values are to be recorded in the booksIn such a case revaluation of assets and reassessment of liabilities is done with the help of a new account called lsquoRevaluation Accountrsquo Sometimes this account is also called as lsquoProfit amp Loss Adjustment Acrsquo If there is a loss due to revaluation revaluation account is debited and if the revaluation results in a profit the revaluation account is credited The following journal entries made for this purpose are

(i) For increase in the value of assetsAsset Ac Dr (individually)To Revaluation Ac(ii) For decrease in the value of AssetRevaluation Ac Dr (individually)To Asset Ac[Decrease in the value of assets](iii) For increase in the value of LiabilitiesRevaluation Ac Dr (individually)To Liabilities Ac[Increase in the value of Liabilities](iv) For decrease in the value of LiabilitiesLiabilities Ac DrTo Revaluation Ac[Decrease in the value of Liabilities](v) For unrecorded AssetsAsset Ac [unrecorded] DrTo Revaluation Ac[Unrecorded asset recorded at actual value](vi) For unrecorded Liability Revaluation Ac DrTo Liability Ac [unrecorded][Unrecorded Liability recorded at actual value](vii) For transfer of gain on revaluationRevaluation Ac DrTo Existing Partnerrsquos CapitalCurrent Ac[Profit on revaluation transferred to capital account in existing ratio](viii) For transfer of loss on revaluationExisting Partnerrsquos CapitalCurrent Ac DrTo Revaluation Ac[Loss on revaluation transferred to capital account in existing ratio](a) When revaluation account shows gain Revaluation Ac DrTo Partnerrsquos Capital Ac (Old Profit Sharing Ratio)(Profit on revaluation credited to Partnerrsquos Capital Ac)(b) Above entry is reversed when revaluation account shows loss Partners Capital Acs (Old Profit Sharing Ratio) DrTo Revaluation Ac(Loss on revaluation debited to Partnerrsquos Capital Acs)

Proforma of Revaluation Account is given as under

Revaluation Account

Dr Cr Particulars ` Amount Particulars ` Amount To Decrease in value of assets By Increase in value of assets To Increase in value of liabilities By Decrease in value of liabilities To Unrecorded liabilities By Unrecorded assets To Gain on Revaluation (Transferred) By Loss on Revalution (Transferred)

ECO ndash12 2742020Topic- ELASTICITY OF DEMAND

CHAPTER - ELASTICITY OF DEMANDMEANINGDemand for a commodity is affected by many factors such as its price price of related goods income of its buyer tastes and preferences etc Elasticity means degree of response Elasticity of demand means degree of responsiveness of demand Demand for a commodity responds to change in price price of related goods income etc So we have three dimensions of elasticity of demandDIMENSION OF ELASTICITY OF DEMAND TYPES OF ELASTICITY OF DEMAND

Price elasticity of demand Income elasticity of demand Cross Elasticity of demand

Price elasticity of demand Price elasticity of demand means degree of responsiveness of demand for a commodity to the change in its price For example if demand for a commodity rises by 10 due to 5 fall in its price Price elasticity of demand (ep)=Percentage change in quantity demanded Percentage change in price of the commodity = 10 ( -)5 = ( - )2Note that ep will always be negative due to inverse relationship of price and quantity demanded

(ii) Income elasticity of demand Income elasticity of demand refers to the degree of responsiveness of demand for a commodity to the change in income of its buyer Suppose income of buyer rises by 10 and his demand for a commodity rises by 20 then Income elasticity of demand (ey)= change in quantity demanded change in price of the commodity =20 10 = 2

Cross Elasticity of demandCross elasticity of demand means the degree of responsiveness of demand for a commodity to the change in price of its related goods (substitute goods or complementary goods) Suppose demand for a commodity rises by 10 due to 5 rise in price of its substitute good then Cross elasticity of demand (ec) = change in quantity demanded change in price of related good = 10 2 = 5 (Tastes and preferences cannot be expressed numerically So elasticity ofdemand cannot be numerically expressed)

  • Chapter 1 Force (Summary)
  • Distinguish between external sovereignty and internal sovereignty
    • NAND Gate
      • Logic diagram
      • Truth Table
        • NOR Gate
          • Logic diagram
          • Truth Table
            • XOR Gate
              • Logic diagram
              • Truth Table
                • XNOR Gate
                  • Logic diagram
                  • Truth Table
                      • Physics
                      • Chapter 1 Electric Field ( Electric Dipole) (Summary)
Page 35:  · Web viewWe all know that Nouns are divided into two parts: common noun and proper noun.Apart from common and proper noun, we will also study about collective noun and compound

industry

Physics Motion in plane Here we will introduce Projectile Motion

Execution

Projectile

Y

usinθ u h

θX

ucosθ

Suppose a body is projected with an angle θ So initial velocity u can be resolved into two components

Horizontal component - ucosθ ( for range)

Vertical component - usinθ ( for height)

usinθ changes during motion and becomes zero at maximum height position but ucosθ remain unchanged

The maximum height of projectile is h

NB If initial is upward then g = -ve and if it is downward then g = +ve Height is +ve if direction of motion does not change ( for ex a body thrown upwards but goes down ultimately then height h = -ve)

The angle of projectile θ is the angle made with horizontal

HISTORY ndash GROWTH OF NATIONALISM

SUB TOPIC- REVOLUTIONARY NATIONALISM Bengal formation of Anushilan Samity and Jugantar Group

The intensification of the Swadeshi movement and Government policy of terror and repression led to outbreak of violence Bombs were manufactured and attempts on the lives of unpopular Government officials became frequent In the gymnasium of Scottish Church College which was known as General Assemblies Institution a secret society was formed known as Anushilan Samity

Aurobindo Ghosh send from Baroda his emissary Jatindranath Banerjee to mobilize the Bengal revolutionaries

Hemchandra Qanungo and Satyen Bose published Journal Jugantar

The Jugantar group planned to assassinate oppressive magistrate Kingsford by Khudiram Bose and Prafulla Chaki in 1908 Prafulla Chaki committed suicide to avoid arrest Khudiram was tried and hanged

Afew days later the police found a bomb factory in Maniktala and arrested a large number of revolutionaries The trial of revolutionaries became famous as the Alipore Bomb Case

In the course of the trial the approver the public prosecuter and a police officer were assassinated

1 Question Name two journals which preached the cult of violence

Answer a) Yugantar edited by Bhupendranath DuttaB) Bandemataram edited by Aurobindo Ghosh2 Question Why was Khudiram arrested and hangedAnswer An attempt was made to assassinate a hated vindictive majistrate named Kingsford by Khudiram Bose and Prafulla Chaki Their attempt failed and the bomb they threw killed two English ladies Khudiram was arrested and put to trial and then hanged3Question Who was Aurubindo GhoshAnswer Aurobindo Ghosh a nationalist revolutionary who was charged for his involvement in the Alipore Bombing Case He was accused of it along with his brother Barindra nath Ghosh But Aurobindo was acquitted because of the brilliant pleading of his counsel Chittaranjan Das Then he became a spiritual reformer introducing his visions on human progress and spiritual evolution4 Qustion Who was KingsfordAnswer Kingsford was an unpopular British chief Magistrate who was the target of the bomb thrown at Muzaffarpur by Khudiram and Prafulla Chaki

Most of the accused were convicted and sentenced to

long term of imprisonmentBut

Aurobindo Ghosh was acquitted mainly owing to the brilliant pleading of his counsel Chittaranjan Das

Political science Topic-Sovereignty

Summary Sovereignty is the full right and power of a governing body over itself without any interference from outside sources or bodies In political theory sovereignty is a substantive term designating supreme legitimate authority over some polity In international law sovereignty is the exercise of power by a state

Internal Sovereignty

Internal sovereignty means supreme authority within ones territory while external sovereignty relates to the recognition on the part of all states that each possesses this power in equal measure

External sovereignty

external sovereignty relates to the recognition on the part of all states that each possesses this power in equal measure

Distinguish between

Execution

Answer the following questions

Short notes-

Sovereignty

Internal Sovereignty

External sovereignty

Homework- learn

external sovereignty and internal sovereigntySovereignty is the principle

of supreme and

unquestionable authority

reflected in the claim by the

state to be the sole author of

laws within its territory

Definition of external vs internal sovereigntyInternal sovereignty refers to

the relationship between a

sovereign power and its

subjects ndash it refers to the

location of the supreme

authority within the state In

the UK for example internal

sovereignty (supposedly)

resides within Parliament

reflected in the

constitutional principle of

parliamentary

sovereigntyBy contrast

external sovereignty refers

to the capacity of the state

to act independently and

autonomously on the world

stage This is what is

sometimes called lsquostate

sovereigntyrsquo or lsquonational

sovereigntyrsquo and implies

that states are legally equal

and that the territorial

integrity and political

independence of a state is

inviolable

Class ndash XII

Date - 2742020 STUDY MATERIALSubject Topic Summary Execution Business Studies

Job Analysis amp Manpower Planning

At first let us recall the chapter what we have discussed till nowbullJob analysisbullJob specification bullJob description bullJob enlargement bullJob enrichment

Today we will do some questions answers from the chapter

Questions 1ldquoJob analysis job description and job specification are interrelatedrdquo Comment Answer) Job analysis is a systematic and detailed examination of a job to collect all the relevant information about it The contents off the job are summarised in the job description The qualification needed for the job are summarised in job specificationThus there is close interrelationship between job analysis job description and job specification

Question 2ldquoJob enlargement is a horizontal extension of a job whereas job enrichment is a vertical extension of a jobrdquo ElucidateAnswer) Job enlargement involves adding one or more task to a job coma where as job enrichment involves adding more autonomy and responsibility to a job Job enlargement is therefore horizontal extension of a job coma whereas job enrichment is a vertical extension of a job

Question 3 )

What is manpower estimation Explain its quantitative and qualitative aspectsAnswer) Manpower estimation is the process by which management determines how an organisation should move from itrsquos current manpower positionto its desired manpower position There are two dimensions of Manpower estimation- quantitative and qualitative

Quantitative aspectThis aspect of Manpower estimation involves estimating the number of employees required in a future time period Workload analysis and workforce analysis are done to estimate the quantity of required manpower

Qualitative aspectThe estimate of the knowledge skills experience etc of required manpower is the qualitative aspect of Manpower estimation The quality of Manpower can be judged on the basis of job analysisand job specification

COMMERCE

CAPITAL-FIXED AND WORKING

Today let us start the class by discussing the sources of finance for different types of business firms

The term lsquocapitalrsquo refers to the investment made in the enterprise for the purpose of earning profits

Requirements of capital and sources of capital for different types of business firms are

1 Capital for sole proprietorship businessA sole proprietor operates at a small scale and thereforerequires a limited amount of capital

2 Capital for partnership firmCapital requirements as well as capital base of a partnership is bigger than that of a sole trader businessThe owned capital is contributed by the partners in an agreed ratio

3 Capital for joint stock companyA joint stock company generally requires large amount of capitalA public company can raise huge capital through issue of shares In addition to share capital it can utilize retained profits

Now let u discuss the meaning of Finance PlanningFinance planning is the process of estimation the financial requirements of an organization specifying the sources of firms and ensuring that enough funds are available at the right time

1 What do you mean by Finance PlanningAnswer Finance planning is the process of estimation the financial requirements of an organization specifying the sources of firms and ensuring that enough funds are available at the right time

2Discuss the role of financial planning of an enterpriseThe role of financial planning are as followsa A sound financial plan helps a business enterprise to avaid the problems of shortage and surplus of fundsbFinancial planning serves as a guide in developing a sound capital structure so as to maximize returns to shareholders c It helps in effective utilization of fundsd It provides policies and procedures for coordinating different functional areas or departments of businesse It enables the management to exercise effective control over the financial activities of an enterprisef It helps the company to prepare for facing business shocks and surprises in future

Mathematics

Continuity and differentiability

Recall Definition of ContinuityLet f(x) be a single valued function of x and x=a be a point in the domain of definition of the function The function is said to be continuous at x=a ifi) f(c) is defined ie f(x) has a definite finite value at x=cii) lim xrarra f(x) exists andiii) lim xrarra f(x) =f(a) In other words f(x) is said to be continuous at x=a if lim xrarra+ f(x)= lim xrarra- f(x) = f(a) Or f(a+0) =f(a-0) =f(a) Or lim hrarr0 f(a+h)= f(a) Algebra of continuous functionsNow we will study some algebra of continuous functions Theorem 1 Suppose f and g be two real functions continuous at a real number c Then(1) f + g is continuous at x = c(2) f ndash g is continuous at x = c(3) f g is continuous at x = c(4) (fg) is continuous at x = c (provided g (c) ne 0)

Example 1 Prove that every rational function is continuousSolution Recall that every rational function f is given byf(x)=[p(x) q(x) ] q(x)ne0where p and q are polynomial functions The domain of f is all real numbers except points at which q is zero Since polynomial functions are continuous f is continuous by (4) of Theorem 1Example 2Discuss the continuity of sine functionSolution To see this we use the following factslim xrarr0 sin x =0Now observe that f (x) = sin x is defined for every real number Let c be a real number Put x = c + h If x rarr c we know that h rarr 0 Therefore lim xrarrc f(x)

= lim xrarrc sin x= lim hrarr0 sin(c+h) =lim hrarr0 [sin c cos h + cos c sin h ]=lim hrarr0 (sin c cos h) + lim hrarr0 (cos c sin h) = sin c +0=sin c = f(c) Thus lim xrarrc f(x) = f(c) and hence f is a constant function Exercise Prove that the function f(x) = x2 +2x is continuous for every real value of x [Hints show that lim xrarra+ f(x) = lim xrarra- f(x) = f(a) ]

Biology Reproduction in Flowering plants We will discuss about megasporoangium

megasporagenesis and female gametophyte

Q4 Describe the structure of megasporangium

Ovule is attached to the placenta by astalk called funicle

Each ovule has one two or three protectivecoverings called integuments

At the tip of integuments a small openingcalled micropyle is organised

Opposite to the micropylar end is the chalaza

Within the integuments a mass of cellsnucellusand inside it embryo sac orfemale gametophyte is present

Q5 Describe a mature embryosacamp its formation

In most of the flowering plants only oneof the 4 megaspores formed as a result ofmegasporogenesis that is functional while theother three degenerate

The

functional megaspore develops into thefemale gametophyte

Formation The nucleus

of the functional megasporedivides mitotically to form two nuclei first andthen two more sequential mitotic nucleardivisions result in the formation of four ampthen eight nucleate stages of embryo sac

Six of the eight nucleus are surrounded bycell walls and organised into cells

The remaining two nuclei called polar nuclei are found below the egg apparatus in the largecentral cell

Three cells consisting of two synergids amp one egg cell present bottom of

embryo sac Three cells

at the chalazal as antipodal cells

Two polar nuclei together present in large central cell

HISTORY

TOWARDS INDEPENDENCE AND PARTITION THE LAST PHASE(1935-1947)SUB TOPIC NATIONAL MOVEMENTS DURING THE SECOND WORLD WAR

Spread of Quit India Movement On 9th August 1942Gandhiji and other Congress leaders were arrested The Congress was declared illegal The news of the arrest of all leaders marked the beginning of a widespread movement of India It was not possible for such a movement to remain peacefulBut the arrest of the all notable congress leaders virtually left the movement in the hands of the mass The movement took the form of violent and militant outbreakBesides congressmen revolutionaries also were very active in the movement The Congress Socialist group also played a prominent role

1 Question Why did the British authority arrest the Congress leaders on 9 th August 1942Answer Congress Working committee adopted the Quit India resolution which was to be ratified at the Bombay AICC meeting in 8th August 1942 They decided to launch a mass struggle on non-violent lines Gandhiji gave a clarion call to all section of the people rdquoKarenge ya Marengerdquo (do or die) Congress leaders gave the call to driving out

the British from IndiaViceroy had taken strong action against the Quit India movement Gandhiji and all the leaders of Congress were arrested

2 Question How did Quit India Movement spread out all over IndiaAnswer The news of the leaders lsquo arrest marked the beginning of a widespread movement to remain peacefulThe movement took form of violent outbreak There were widespread cutting of telephone and Telegraph wires damaging railway lines raising barricades in cities and towns and other forms of violent demonstations

Question Name the leaders of Congress

Socialist group played a prominent part Notable among the Jayprakash Narayan Rammonohar Lohia Aruna Asaf Ali

Political science

Topic-Franchise and Representation

Summary

The election commission

The Election Commission of India is an autonomous constitutional authority responsible for administering Union and State election processes in India The body administers elections to the Lok Sabha Rajya Sabha State Legislative Assemblies in India and the offices of the President and Vice President in the country

Functions of election commission-

India is a sovereign socialist secular democratic republic Democracy runs like a golden thread in the social economic and political fabric woven by the Constitution given by lsquoWe the People of Indiarsquo unto ourselves The concept of democracy as visualised by the Constitution pre-supposes the representation of the people in Parliament and State legislatures by the method of election The Supreme Court has held that democracy is one of the inalienable basic features of the Constitution of India and forms part of its basic structure The Constitution of India adopted a Parliamentary form of government Parliament consists of the President of India and the two Houses mdash Rajya Sabha and Lok Sabha India being a Union of states has separate state legislatures for each state State legislatures consist of the Governor and two Houses mdash Legislative Council and Legislative Assembly mdash in seven states namely Andhra Pradesh Telangana Bihar Jammu amp Kashmir Karnataka Maharashtra and Uttar Pradesh and of the Governor and the state Legislative Assembly in the remaining 22 states Apart from the above two out of the seven Union Territories namely National Capital Territory of Delhi and Puducherry also have their Legislative Assemblies

ExecutionShort notes-Election commissionFunctions of election commission

Homework- Learn

Computer

Science

Computer hardware NAND Gate

A NOT-AND operation is known as NAND operation It has n input (n gt= 2) and one output

Logic diagram

Truth Table

NOR Gate

A NOT-OR operation is known as NOR operation It has n input (n gt= 2) and one output

Logic diagram

Truth Table

XOR Gate

XOR or Ex-OR gate is a special type of gate It can be used in the half

adder full adder and subtractor The exclusive-OR gate is abbreviated as EX-OR gate or sometime as X-OR gate It has n input (n gt= 2) and one output

Logic diagram

Truth Table

XNOR Gate

XNOR gate is a special type of gate It can be used in the half adder full adder and subtractor The exclusive-NOR gate is abbreviated as EX-NOR gate or sometime as X-NOR gate It has n input (n gt= 2) and one output

Logic diagram

Truth Table

Physics

Chapter 1 Electric Field ( Electric Dipole) (Summary)

Here we will derive Expression of electric field at broad side

On position of dipole

Execution

Q With the help of a labelled diagram obtain an expression for the electric field intensity E at any point on the equitorial line ( broad-side on position) of an electric dipole

Ans

E1 E1sinθ

E θ P E1 θ

( r2+L2)12 E2 E

r E2 E2sinθ

-q θ L O L +qA B

Let us consider that the point P is situated on the right bisector of the dipole AB at a distance r meter from its midpoint O

Let E1 and E2 be the electric field intensities of the electric field at P due to charge +q and ndashq of the dipole resp The distance of P from each charge is ( r2+L2)12

So E1 = 14 πϵ q

(r 2+L 2) away from +q

E2 = 14 πϵ q

(r 2+L 2) towards ndashq

The magnitudes of E1 and E2 are equal but directions are different Now resolving E1 and E2 into two components parallel and perpendicular to AB we get

The components perpendicular to AB E1sinθ and E2sinθ cancel each other because they are equal and opposite

The components parallel to AB are E1cosθ and E2 cosθ are in same direction and add up

So resultant intensity of electric field at the point P is

E = E1cosθ + E2 cosθ

E = 14 πϵ q

(r 2+L 2) 2 cosθ

Now from fig we have cosθ =BOBP = L (r2+L2)12

So we get E = 14 πϵ 2qL ( r2+L2)32

Now electric dipole moment p= 2qL

So E = 14 πϵ p ( r2+L2)32

HW Find the expression of Electric field as done here but this time take r gtgt 2L

Also find the expression of torque experience by a dipole

(Hint Electric force experienced by charges of dipole in electric field is qE each Let θ be the angle which dipole makes with electric lines of force then perpendicular distance between two charges is 2Lsinθ Then torque = force x perp distance = qE x 2L sinθ So τ=pE sinθ where p =2qL )

STUDY MATERIAL

Class XIISubject Eng Literature (The Tempest ndash William Shakespeare) Topic Act IV Scene 1 Lines 84 to 133 (Iris hellip A contract of true love Be not too late ) Date 27th April 2020 (4th Period)

[Students should read the original play and also the paraphrase given in the school prescribed textbook]Summary Questions amp Answers

o Ceres soon appears and comes to know that she has been summoned to celebrate the contract of true love

o Ceres expresses her unwillingness to meet Venus and Cupid as she has shunned their company

o Ceres and Juno both bestow their blessings upon Ferdinand and Miranda with June gifting honour riches happiness in marriage and Ceres presents plenty of earthrsquos produce

o Iris summons the water-nymphs and reapers to come and celebrate a contract

(1) IRIS Of her society (Line 91-101)

Be not afraid I met her deity

Cutting the clouds towards Pathos and her sonDove-drawn with her Here thought they to have doneSome wanton charm upon this man and maidWhose vows are that no bed-right shall be paidTill Hymens torch be lightedmdashbut in vainMarss hot minion is returned againHer waspish-headed son has broke his arrowsSwears he will shoot no more but play with sparrowsAnd be a boy right out

(i) Where were Venus and Cupid seen flying How were they travelling Why did they want to join the marriage celebration of Ferdinand and Miranda

of true love

Venus and Cupid were seen flying through the air towards Paphos the famous city which is situated on the island of Cyprus They were travelling by air-borne chariot drawn by doves They certainly wanted to come here in order to play some amorous trick upon Ferdinand and Miranda who are under a vow not to gratify their physical desires till the holy ceremony of their marriage has been performed(ii) What have Venus and Cupid done after failing in their plan

After being failure of their plan Venus who is a very passionate deity and who is the mistress of Mars (the god of war) has gone back while here ill-tempered son Cupid has broken his arrows of love in his state of desperation(iii) What has Cupid firmly decided

Cupid is feeling so disappointed that he has firmly decided to shoot no more arrows to arouse love in human hearts but to spend his time playing with sparrows Thus he would now become just a boy and would give up his original function of shooting arrows on human beings to make them fall in love(iv) What vow had Ceres taken How did Ceres feel at the abduction

After the abduction of her daughter Prosperina by Pluto Ceres had taken a vow to always keep away from the disgraceful company of Venus and her blind son Cupid the god of love Ceres felt deeply distressed when Pluto had carried off her daughter and had made her his wife by force(v) Why has Ceres not forgiven Venus and her blind son For what do Ceres want to be sure

As the abduction had been manipulated by Venus the goddess of beauty and love and her blind son Cupid Ceres has never forgiven them for their part in the whole plot Ceres wants to be sure that she would not have to meet Venus and Cupid who had engineered the abduction of her daughter Prosperina

AS THIS lsquoMASQUErsquo SCENE IS VERY IMPORTANT IN THE PLAY THE PARAPHRASE OF THE ENTIRE PORTION OF MASQUE SCENE (Act IV Lines 58 to 143) IS GIVEN BELOW

IRIS Goddess of RainbowCERES Goddess of Agriculture and all the fruits of the earth

(Nature growth prosperity rebirth ndash notions intimately connected to marriage)JUNO The majestic Queen of Heavens and wife of Jupiter (Jupiter is the king of Gods)

VENUS The Goddess of love CUPID Son of Venus PLUTO God of death (In the play referred by Shakespeare as lsquoDisrsquo which is a Roman name for Pluto)

ORIGINAL TEXT PARAPHRASEPROSPEROWellmdash

PROSPERONow come Ariel Let there be too many rather than too few

Now come my Ariel Bring a corollaryRather than want a spirit Appear and pertly[to Ferdinand and Miranda]No tongue all eyes Be silent

spirits in attendance Appear briskly

[to Ferdinand and Miranda]Look with your eyes but do not say a word

[Soft music] [Soft music][Enter Iris] [Enter Iris]

IRISCeres most bounteous lady thy rich leasOf wheat rye barley vetches oats and peasThy turfy mountains where live nibbling sheepAnd flat meads thatched with stover them to keepThy banks with pioned and twilled brimsWhich spongy April at thy hest betrimsTo make cold nymphs chaste crowns and thybroom-grovesWhose shadow the dismissegraved bachelor lovesBeing lass-lorn thy pole clipped vineyardAnd thy sea-marge sterile and rocky-hardWhere thou thyself dost airmdashthe Queen othrsquoSkyWhose watery arch and messenger am IBids thee leave these and with her sovereign grace[Juno appears] Here on this grass-plot in this very placeTo come and sport Her peacocks fly amainApproach rich Ceres her to entertain

IRISCeres most generous lady you are the cause of rich fields or fertile land where wheat rye barley beans oats and peas grow the grassy mountains where the sheep graze and the flat meadows covered with coarse hay to be used as fodder for cattleYour banks are covered with marsh-marigolds and reeds and the rainy April under your orders brings forth to make for the maids who are not in love beautiful crowns your woods where the broom flourishes and where the bachelor who has been dismissed by the maid he loved lies down being forsaken your vineyard in which the poles are embraced by the vines and the margin of the sea which is barren and rocky where you roam about to enjoy the fresh air ndash the queen of the sky (Juno) whose messenger I am besides being represented as the rainbow bids you leave all these and with her majesty here on this grassy plot in this very place come and sport her peacocks carry her fast in her chariot through the air and are making their way here approach rich Ceres to welcome her

[Enter Ariel as Ceres] [Enter Ariel as Ceres]

CERESHail many-coloured messenger that neerDost disobey the wife of JupiterWho with thy saffron wings upon my flowersDiffusest honey-drops refreshing showersAnd with each end of thy blue bow dost crownMy bosky acres and my unshrubbed downRich scarf to my proud earth Why hath thy queenSummoned me hither to this short-grassed green

CERESWelcome rainbow that never dared disobey Juno the wife of Jupiter who with your orange coloured rays spread honey-drops refreshing showers And with each end of thy blue bow drown my bushy acres and my hilly country which is free from shrubs you thus forming a rich scarf Why has your queen called me here to this place covered with short grass

IRISA contract of true love to celebrateAnd some donation freely to estateOn the blest lovers

IRISI have called you to celebrate a contract of true love and bestow some liberal gift upon the blessed lovers

ORIGINAL TEXT PARAPHRASECERESTell me heavenly bowIf Venus or her son as thou dost knowDo now attend the queen Since they did plotThe means that dusky Dis my daughter gotHer and her blind boys scandaled companyI have forsworn

CERESTell me heavenly bow if Venus the Goddess of love or Cupid her son and pedlar of passion at this time attend the heavenly queen Juno because you are sure to know Since the day they conspired against me and dark Pluto took away my daughter here and Cupidrsquos disgraceful company I have left off

IRISOf her societyBe not afraid I met her deityCutting the clouds towards Pathos and her sonDove-drawn with her Here thought they to have doneSome wanton charm upon this man and miad

IRISBe not afraid of her company I met her deity moving on the clouds towards Paphos the sacred home of Venus on the island of Cyprus along with her son on her chariot drawn by doves Here they contemplated to exercise a charm upon this man and maid producing

Whose vows are that no bed-right shall be paidTill Hymens torch be lightedmdashbut in vainMarss hot minion is returned againHer waspish-headed son has broke his arrowsSwears he will shoot no more but play with sparrowsAnd be a boy right out

wantonness before the actual marriage ceremony but did not succeed Venus has returned her irritable son has broken his arrows and swears that he will give up his practice of trying to inspire love but play with sparrows and be a boy again

[Music is heard] [Music is heard]

CERESHighst queen of stateGreat Juno comes I know her by her gait

CERESHighest queen of state Great Juno there she comes I know here by her gait

[Enter Juno] [Enter Juno]

JUNOHow does my bounteous sister Go with meTo bless this twain that they may Prosperous beAnd honoured in their issue

JUNOHow are you doing my generous sister Come with me to bless this couple so that they may be prosperous and fortunate in their children

[They sing] [They sing]

JUNOHonour riches marriage-blessingLong continuance and increasingHourly joys be still upon youJuno sings her blessings upon you

JUNOMay honour riches happiness in marriage long continuance and increase of those boons ever rest upon you as hourly joys Juno showers down upon you her blessings in song

CERESEarths increase foison plentyBarns and garners never emptyVines and clustring bunches growingPlants and goodly burden bowingSpring come to you at the farthestIn the very end of harvestScarcity and want shall shun youCeresrsquo blessing so is on you

CERESMay you have the plenty of earthrsquos produce Your barns and granaries may never be empty Your vines may grow with clustering bunches Your fruit trees may be heavily laden with their fruit May there be continuous spring and harvest May scantiness and want leave you forever Such is the blessing of Ceres upon you

FERDINANDThis is a most majestic vision andHarmoniously charmingly May I be boldTo think these spirits

FERDINANDThis is a great vision and magically melodious Should I suppose the characters (taking part in the masque) are spirits

PROSPEROSpirits which by mine artI have from their confines calld to enactMy present fancies

PROSPEROYes they are spirits whom I have summoned from the regions to which they are confined to carry into effect my fanciful designs

ORIGINAL TEXT PARAPHRASEFERDINANDLet me live here everSo rare a wondered father and a wifeMakes this place paradise

FERDINANDI should like to live here forever Such a wise and wonderful father makes this place a paradise

[Juno and Ceres whisper and send Iris on employment] [Juno and Ceres whisper and send Iris on employment]

PROSPEROSweet now silence

PROSPEROMy dear Ferdinand speak no more Juno and Ceres are

Juno and Ceres whisper seriouslyTheres something else to do Hush and be muteOr else our spell is marred

whispering with a solemn look There is something else coming Silence Or else our magic will be spoilt

IRISYour nymphs called naiads of the wandering brooksWith your sedged crowns and over-harmless looksLeave your crisp channels and on this green landAnswer your summons Juno does commandCome temperate nymphs and help to celebrateA contract of true love Be not too late

IRISYou nymphs called Naiads denizens (M inhabitants) of the running stream with your chaplets of sedge and ever-helpful looks leave your wrinkled channels and on the green land answer the summons sent to you Juno has ordered some chaste nymphs and help to celebrate a noble and true marriage Donrsquot delay

[Enter certain nymphs] [Enter certain nymphs]You sunburnt sicklemen of August wearyCome hither from the furrow and be merryMake holiday your rye-straw hats put onAnd these fresh nymphs encounter every oneIn country footing

You sunburnt harvesters weary from the effects of the heat in August come here from the furrowed land and rejoice Make holiday with your rye-straw hats upon you and meet these fresh nymphs and join in country dancing

[Enter certain reapers properly habited They join with the nymphs in a graceful dance towards the end whereof Prospero starts suddenly and speaks]

[Enter certain reapers properly habited They join with the nymphs in a graceful dance towards the end whereof Prospero starts suddenly and speaks]

PROSPERO[aside] I had forgot that foul conspiracyOf the beast Caliban and his confederatesAgainst my life The minute of their plotIs almost come [to the spirits]Well done Avoidno more

PROSPERO(Aside)I had forgotten the wicked conspiracy of the beast Caliban and his accomplices against my life the time of their plot has almost arrived ndash (To the Spirits) well done depart no more of this

[To a strange hollow and confused noise the spirits heavily vanish]

[The spirits depart]

ORIGINAL TEXT PARAPHRASEFERDINANDLet me live here everSo rare a wondered father and a wifeMakes this place paradise

FERDINANDI should like to live here forever Such a wise and wonderful father makes this place a paradise

[Juno and Ceres whisper and send Iris on employment] [Juno and Ceres whisper and send Iris on employment]

PROSPEROSweet now silenceJuno and Ceres whisper seriouslyTheres something else to do Hush and be muteOr else our spell is marred

PROSPEROMy dear Ferdinand speak no more Juno and Ceres are whispering with a solemn look There is something else coming Silence Or else our magic will be spoilt

IRISYour nymphs called naiads of the wandering brooksWith your sedged crowns and over-harmless looksLeave your crisp channels and on this green landAnswer your summons Juno does commandCome temperate nymphs and help to celebrateA contract of true love Be not too late

IRISYou nymphs called Naiads denizens (M inhabitants) of the running stream with your chaplets of sedge and ever-helpful looks leave your wrinkled channels and on the green land answer the summons sent to you Juno has ordered some chaste nymphs and help to celebrate a noble and true marriage Donrsquot delay

[Enter certain nymphs] [Enter certain nymphs]You sunburnt sicklemen of August wearyCome hither from the furrow and be merryMake holiday your rye-straw hats put onAnd these fresh nymphs encounter every oneIn country footing

You sunburnt harvesters weary from the effects of the heat in August come here from the furrowed land and rejoice Make holiday with your rye-straw hats upon you and meet these fresh nymphs and join in country dancing

[Enter certain reapers properly habited They join with the nymphs in a graceful dance towards the end whereof Prospero starts suddenly and speaks]

[Enter certain reapers properly habited They join with the nymphs in a graceful dance towards the end whereof Prospero starts suddenly and speaks]

PROSPERO[aside] I had forgot that foul conspiracyOf the beast Caliban and his confederatesAgainst my life The minute of their plotIs almost come [to the spirits]Well done Avoidno more

PROSPERO(Aside)I had forgotten the wicked conspiracy of the beast Caliban and his accomplices against my life the time of their plot has almost arrived ndash (To the Spirits) well done depart no more of this

[To a strange hollow and confused noise the spirits heavily vanish]

[The spirits depart]

Ac-12 27420 topic Revaluation of Assets and Liabilities

REVALUATION OF ASSETS AND LIABILITIES

On admission of a new partner the firm stands reconstituted and consequently the assets are revalued and liabilities are reassessed It is necessary to show the true position of the firm at the time of admission of a new partner If the values of the assets are raised gain will increase the capital of the existing partners Similarly any decrease in the value of assets ie loss will decrease the capital of the existing partners For this purpose alsquoRevaluation Accountrsquo is prepared This account is credited with all increases in the value of assets and decrease in the value of liabilities It is debited with decrease on account of value of assets and increase in the value of liabilities The balance of this account shows a gain or loss on revaluation which is transferred to the existing partnerrsquos capital account in existing profit sharing ratioAccounting for Revaluation of Assets and Liabilities when there is a Changein the Profit Sharing Ratio of Existing PartnersAssets and liabilities of a firm must also be revalued at the time of change in profit sharing ratio of existing partners The reason is that the realisable or actual value of assets and liabilities may be different from those shown in the Balance Sheet It is possible that with the passage of time some of the assets might have appreciated in value while the value of certain other assets might have decreased and no record has been made of such changes in the books of accounts Similarly there may be some unrecorded assets amp libilities that may have to be accounted for Revaluation of assets and reassessments of liabilities becomes necessary because the change in the

value of assets and liabilities belongs to the period to change in profit sharing ratio and hence must be shared by the partners in their old profit sharing ratio Revaluation of assets and reassessment of liabilities may be given effect to in two different ways (a) When revised values are to be recorded in the books and(b) When revised values are not to be recorded in the books

When revised values are to be recorded in the booksIn such a case revaluation of assets and reassessment of liabilities is done with the help of a new account called lsquoRevaluation Accountrsquo Sometimes this account is also called as lsquoProfit amp Loss Adjustment Acrsquo If there is a loss due to revaluation revaluation account is debited and if the revaluation results in a profit the revaluation account is credited The following journal entries made for this purpose are

(i) For increase in the value of assetsAsset Ac Dr (individually)To Revaluation Ac(ii) For decrease in the value of AssetRevaluation Ac Dr (individually)To Asset Ac[Decrease in the value of assets](iii) For increase in the value of LiabilitiesRevaluation Ac Dr (individually)To Liabilities Ac[Increase in the value of Liabilities](iv) For decrease in the value of LiabilitiesLiabilities Ac DrTo Revaluation Ac[Decrease in the value of Liabilities](v) For unrecorded AssetsAsset Ac [unrecorded] DrTo Revaluation Ac[Unrecorded asset recorded at actual value](vi) For unrecorded Liability Revaluation Ac DrTo Liability Ac [unrecorded][Unrecorded Liability recorded at actual value](vii) For transfer of gain on revaluationRevaluation Ac DrTo Existing Partnerrsquos CapitalCurrent Ac[Profit on revaluation transferred to capital account in existing ratio](viii) For transfer of loss on revaluationExisting Partnerrsquos CapitalCurrent Ac DrTo Revaluation Ac[Loss on revaluation transferred to capital account in existing ratio](a) When revaluation account shows gain Revaluation Ac DrTo Partnerrsquos Capital Ac (Old Profit Sharing Ratio)(Profit on revaluation credited to Partnerrsquos Capital Ac)(b) Above entry is reversed when revaluation account shows loss Partners Capital Acs (Old Profit Sharing Ratio) DrTo Revaluation Ac(Loss on revaluation debited to Partnerrsquos Capital Acs)

Proforma of Revaluation Account is given as under

Revaluation Account

Dr Cr Particulars ` Amount Particulars ` Amount To Decrease in value of assets By Increase in value of assets To Increase in value of liabilities By Decrease in value of liabilities To Unrecorded liabilities By Unrecorded assets To Gain on Revaluation (Transferred) By Loss on Revalution (Transferred)

ECO ndash12 2742020Topic- ELASTICITY OF DEMAND

CHAPTER - ELASTICITY OF DEMANDMEANINGDemand for a commodity is affected by many factors such as its price price of related goods income of its buyer tastes and preferences etc Elasticity means degree of response Elasticity of demand means degree of responsiveness of demand Demand for a commodity responds to change in price price of related goods income etc So we have three dimensions of elasticity of demandDIMENSION OF ELASTICITY OF DEMAND TYPES OF ELASTICITY OF DEMAND

Price elasticity of demand Income elasticity of demand Cross Elasticity of demand

Price elasticity of demand Price elasticity of demand means degree of responsiveness of demand for a commodity to the change in its price For example if demand for a commodity rises by 10 due to 5 fall in its price Price elasticity of demand (ep)=Percentage change in quantity demanded Percentage change in price of the commodity = 10 ( -)5 = ( - )2Note that ep will always be negative due to inverse relationship of price and quantity demanded

(ii) Income elasticity of demand Income elasticity of demand refers to the degree of responsiveness of demand for a commodity to the change in income of its buyer Suppose income of buyer rises by 10 and his demand for a commodity rises by 20 then Income elasticity of demand (ey)= change in quantity demanded change in price of the commodity =20 10 = 2

Cross Elasticity of demandCross elasticity of demand means the degree of responsiveness of demand for a commodity to the change in price of its related goods (substitute goods or complementary goods) Suppose demand for a commodity rises by 10 due to 5 rise in price of its substitute good then Cross elasticity of demand (ec) = change in quantity demanded change in price of related good = 10 2 = 5 (Tastes and preferences cannot be expressed numerically So elasticity ofdemand cannot be numerically expressed)

  • Chapter 1 Force (Summary)
  • Distinguish between external sovereignty and internal sovereignty
    • NAND Gate
      • Logic diagram
      • Truth Table
        • NOR Gate
          • Logic diagram
          • Truth Table
            • XOR Gate
              • Logic diagram
              • Truth Table
                • XNOR Gate
                  • Logic diagram
                  • Truth Table
                      • Physics
                      • Chapter 1 Electric Field ( Electric Dipole) (Summary)
Page 36:  · Web viewWe all know that Nouns are divided into two parts: common noun and proper noun.Apart from common and proper noun, we will also study about collective noun and compound

HISTORY ndash GROWTH OF NATIONALISM

SUB TOPIC- REVOLUTIONARY NATIONALISM Bengal formation of Anushilan Samity and Jugantar Group

The intensification of the Swadeshi movement and Government policy of terror and repression led to outbreak of violence Bombs were manufactured and attempts on the lives of unpopular Government officials became frequent In the gymnasium of Scottish Church College which was known as General Assemblies Institution a secret society was formed known as Anushilan Samity

Aurobindo Ghosh send from Baroda his emissary Jatindranath Banerjee to mobilize the Bengal revolutionaries

Hemchandra Qanungo and Satyen Bose published Journal Jugantar

The Jugantar group planned to assassinate oppressive magistrate Kingsford by Khudiram Bose and Prafulla Chaki in 1908 Prafulla Chaki committed suicide to avoid arrest Khudiram was tried and hanged

Afew days later the police found a bomb factory in Maniktala and arrested a large number of revolutionaries The trial of revolutionaries became famous as the Alipore Bomb Case

In the course of the trial the approver the public prosecuter and a police officer were assassinated

1 Question Name two journals which preached the cult of violence

Answer a) Yugantar edited by Bhupendranath DuttaB) Bandemataram edited by Aurobindo Ghosh2 Question Why was Khudiram arrested and hangedAnswer An attempt was made to assassinate a hated vindictive majistrate named Kingsford by Khudiram Bose and Prafulla Chaki Their attempt failed and the bomb they threw killed two English ladies Khudiram was arrested and put to trial and then hanged3Question Who was Aurubindo GhoshAnswer Aurobindo Ghosh a nationalist revolutionary who was charged for his involvement in the Alipore Bombing Case He was accused of it along with his brother Barindra nath Ghosh But Aurobindo was acquitted because of the brilliant pleading of his counsel Chittaranjan Das Then he became a spiritual reformer introducing his visions on human progress and spiritual evolution4 Qustion Who was KingsfordAnswer Kingsford was an unpopular British chief Magistrate who was the target of the bomb thrown at Muzaffarpur by Khudiram and Prafulla Chaki

Most of the accused were convicted and sentenced to

long term of imprisonmentBut

Aurobindo Ghosh was acquitted mainly owing to the brilliant pleading of his counsel Chittaranjan Das

Political science Topic-Sovereignty

Summary Sovereignty is the full right and power of a governing body over itself without any interference from outside sources or bodies In political theory sovereignty is a substantive term designating supreme legitimate authority over some polity In international law sovereignty is the exercise of power by a state

Internal Sovereignty

Internal sovereignty means supreme authority within ones territory while external sovereignty relates to the recognition on the part of all states that each possesses this power in equal measure

External sovereignty

external sovereignty relates to the recognition on the part of all states that each possesses this power in equal measure

Distinguish between

Execution

Answer the following questions

Short notes-

Sovereignty

Internal Sovereignty

External sovereignty

Homework- learn

external sovereignty and internal sovereigntySovereignty is the principle

of supreme and

unquestionable authority

reflected in the claim by the

state to be the sole author of

laws within its territory

Definition of external vs internal sovereigntyInternal sovereignty refers to

the relationship between a

sovereign power and its

subjects ndash it refers to the

location of the supreme

authority within the state In

the UK for example internal

sovereignty (supposedly)

resides within Parliament

reflected in the

constitutional principle of

parliamentary

sovereigntyBy contrast

external sovereignty refers

to the capacity of the state

to act independently and

autonomously on the world

stage This is what is

sometimes called lsquostate

sovereigntyrsquo or lsquonational

sovereigntyrsquo and implies

that states are legally equal

and that the territorial

integrity and political

independence of a state is

inviolable

Class ndash XII

Date - 2742020 STUDY MATERIALSubject Topic Summary Execution Business Studies

Job Analysis amp Manpower Planning

At first let us recall the chapter what we have discussed till nowbullJob analysisbullJob specification bullJob description bullJob enlargement bullJob enrichment

Today we will do some questions answers from the chapter

Questions 1ldquoJob analysis job description and job specification are interrelatedrdquo Comment Answer) Job analysis is a systematic and detailed examination of a job to collect all the relevant information about it The contents off the job are summarised in the job description The qualification needed for the job are summarised in job specificationThus there is close interrelationship between job analysis job description and job specification

Question 2ldquoJob enlargement is a horizontal extension of a job whereas job enrichment is a vertical extension of a jobrdquo ElucidateAnswer) Job enlargement involves adding one or more task to a job coma where as job enrichment involves adding more autonomy and responsibility to a job Job enlargement is therefore horizontal extension of a job coma whereas job enrichment is a vertical extension of a job

Question 3 )

What is manpower estimation Explain its quantitative and qualitative aspectsAnswer) Manpower estimation is the process by which management determines how an organisation should move from itrsquos current manpower positionto its desired manpower position There are two dimensions of Manpower estimation- quantitative and qualitative

Quantitative aspectThis aspect of Manpower estimation involves estimating the number of employees required in a future time period Workload analysis and workforce analysis are done to estimate the quantity of required manpower

Qualitative aspectThe estimate of the knowledge skills experience etc of required manpower is the qualitative aspect of Manpower estimation The quality of Manpower can be judged on the basis of job analysisand job specification

COMMERCE

CAPITAL-FIXED AND WORKING

Today let us start the class by discussing the sources of finance for different types of business firms

The term lsquocapitalrsquo refers to the investment made in the enterprise for the purpose of earning profits

Requirements of capital and sources of capital for different types of business firms are

1 Capital for sole proprietorship businessA sole proprietor operates at a small scale and thereforerequires a limited amount of capital

2 Capital for partnership firmCapital requirements as well as capital base of a partnership is bigger than that of a sole trader businessThe owned capital is contributed by the partners in an agreed ratio

3 Capital for joint stock companyA joint stock company generally requires large amount of capitalA public company can raise huge capital through issue of shares In addition to share capital it can utilize retained profits

Now let u discuss the meaning of Finance PlanningFinance planning is the process of estimation the financial requirements of an organization specifying the sources of firms and ensuring that enough funds are available at the right time

1 What do you mean by Finance PlanningAnswer Finance planning is the process of estimation the financial requirements of an organization specifying the sources of firms and ensuring that enough funds are available at the right time

2Discuss the role of financial planning of an enterpriseThe role of financial planning are as followsa A sound financial plan helps a business enterprise to avaid the problems of shortage and surplus of fundsbFinancial planning serves as a guide in developing a sound capital structure so as to maximize returns to shareholders c It helps in effective utilization of fundsd It provides policies and procedures for coordinating different functional areas or departments of businesse It enables the management to exercise effective control over the financial activities of an enterprisef It helps the company to prepare for facing business shocks and surprises in future

Mathematics

Continuity and differentiability

Recall Definition of ContinuityLet f(x) be a single valued function of x and x=a be a point in the domain of definition of the function The function is said to be continuous at x=a ifi) f(c) is defined ie f(x) has a definite finite value at x=cii) lim xrarra f(x) exists andiii) lim xrarra f(x) =f(a) In other words f(x) is said to be continuous at x=a if lim xrarra+ f(x)= lim xrarra- f(x) = f(a) Or f(a+0) =f(a-0) =f(a) Or lim hrarr0 f(a+h)= f(a) Algebra of continuous functionsNow we will study some algebra of continuous functions Theorem 1 Suppose f and g be two real functions continuous at a real number c Then(1) f + g is continuous at x = c(2) f ndash g is continuous at x = c(3) f g is continuous at x = c(4) (fg) is continuous at x = c (provided g (c) ne 0)

Example 1 Prove that every rational function is continuousSolution Recall that every rational function f is given byf(x)=[p(x) q(x) ] q(x)ne0where p and q are polynomial functions The domain of f is all real numbers except points at which q is zero Since polynomial functions are continuous f is continuous by (4) of Theorem 1Example 2Discuss the continuity of sine functionSolution To see this we use the following factslim xrarr0 sin x =0Now observe that f (x) = sin x is defined for every real number Let c be a real number Put x = c + h If x rarr c we know that h rarr 0 Therefore lim xrarrc f(x)

= lim xrarrc sin x= lim hrarr0 sin(c+h) =lim hrarr0 [sin c cos h + cos c sin h ]=lim hrarr0 (sin c cos h) + lim hrarr0 (cos c sin h) = sin c +0=sin c = f(c) Thus lim xrarrc f(x) = f(c) and hence f is a constant function Exercise Prove that the function f(x) = x2 +2x is continuous for every real value of x [Hints show that lim xrarra+ f(x) = lim xrarra- f(x) = f(a) ]

Biology Reproduction in Flowering plants We will discuss about megasporoangium

megasporagenesis and female gametophyte

Q4 Describe the structure of megasporangium

Ovule is attached to the placenta by astalk called funicle

Each ovule has one two or three protectivecoverings called integuments

At the tip of integuments a small openingcalled micropyle is organised

Opposite to the micropylar end is the chalaza

Within the integuments a mass of cellsnucellusand inside it embryo sac orfemale gametophyte is present

Q5 Describe a mature embryosacamp its formation

In most of the flowering plants only oneof the 4 megaspores formed as a result ofmegasporogenesis that is functional while theother three degenerate

The

functional megaspore develops into thefemale gametophyte

Formation The nucleus

of the functional megasporedivides mitotically to form two nuclei first andthen two more sequential mitotic nucleardivisions result in the formation of four ampthen eight nucleate stages of embryo sac

Six of the eight nucleus are surrounded bycell walls and organised into cells

The remaining two nuclei called polar nuclei are found below the egg apparatus in the largecentral cell

Three cells consisting of two synergids amp one egg cell present bottom of

embryo sac Three cells

at the chalazal as antipodal cells

Two polar nuclei together present in large central cell

HISTORY

TOWARDS INDEPENDENCE AND PARTITION THE LAST PHASE(1935-1947)SUB TOPIC NATIONAL MOVEMENTS DURING THE SECOND WORLD WAR

Spread of Quit India Movement On 9th August 1942Gandhiji and other Congress leaders were arrested The Congress was declared illegal The news of the arrest of all leaders marked the beginning of a widespread movement of India It was not possible for such a movement to remain peacefulBut the arrest of the all notable congress leaders virtually left the movement in the hands of the mass The movement took the form of violent and militant outbreakBesides congressmen revolutionaries also were very active in the movement The Congress Socialist group also played a prominent role

1 Question Why did the British authority arrest the Congress leaders on 9 th August 1942Answer Congress Working committee adopted the Quit India resolution which was to be ratified at the Bombay AICC meeting in 8th August 1942 They decided to launch a mass struggle on non-violent lines Gandhiji gave a clarion call to all section of the people rdquoKarenge ya Marengerdquo (do or die) Congress leaders gave the call to driving out

the British from IndiaViceroy had taken strong action against the Quit India movement Gandhiji and all the leaders of Congress were arrested

2 Question How did Quit India Movement spread out all over IndiaAnswer The news of the leaders lsquo arrest marked the beginning of a widespread movement to remain peacefulThe movement took form of violent outbreak There were widespread cutting of telephone and Telegraph wires damaging railway lines raising barricades in cities and towns and other forms of violent demonstations

Question Name the leaders of Congress

Socialist group played a prominent part Notable among the Jayprakash Narayan Rammonohar Lohia Aruna Asaf Ali

Political science

Topic-Franchise and Representation

Summary

The election commission

The Election Commission of India is an autonomous constitutional authority responsible for administering Union and State election processes in India The body administers elections to the Lok Sabha Rajya Sabha State Legislative Assemblies in India and the offices of the President and Vice President in the country

Functions of election commission-

India is a sovereign socialist secular democratic republic Democracy runs like a golden thread in the social economic and political fabric woven by the Constitution given by lsquoWe the People of Indiarsquo unto ourselves The concept of democracy as visualised by the Constitution pre-supposes the representation of the people in Parliament and State legislatures by the method of election The Supreme Court has held that democracy is one of the inalienable basic features of the Constitution of India and forms part of its basic structure The Constitution of India adopted a Parliamentary form of government Parliament consists of the President of India and the two Houses mdash Rajya Sabha and Lok Sabha India being a Union of states has separate state legislatures for each state State legislatures consist of the Governor and two Houses mdash Legislative Council and Legislative Assembly mdash in seven states namely Andhra Pradesh Telangana Bihar Jammu amp Kashmir Karnataka Maharashtra and Uttar Pradesh and of the Governor and the state Legislative Assembly in the remaining 22 states Apart from the above two out of the seven Union Territories namely National Capital Territory of Delhi and Puducherry also have their Legislative Assemblies

ExecutionShort notes-Election commissionFunctions of election commission

Homework- Learn

Computer

Science

Computer hardware NAND Gate

A NOT-AND operation is known as NAND operation It has n input (n gt= 2) and one output

Logic diagram

Truth Table

NOR Gate

A NOT-OR operation is known as NOR operation It has n input (n gt= 2) and one output

Logic diagram

Truth Table

XOR Gate

XOR or Ex-OR gate is a special type of gate It can be used in the half

adder full adder and subtractor The exclusive-OR gate is abbreviated as EX-OR gate or sometime as X-OR gate It has n input (n gt= 2) and one output

Logic diagram

Truth Table

XNOR Gate

XNOR gate is a special type of gate It can be used in the half adder full adder and subtractor The exclusive-NOR gate is abbreviated as EX-NOR gate or sometime as X-NOR gate It has n input (n gt= 2) and one output

Logic diagram

Truth Table

Physics

Chapter 1 Electric Field ( Electric Dipole) (Summary)

Here we will derive Expression of electric field at broad side

On position of dipole

Execution

Q With the help of a labelled diagram obtain an expression for the electric field intensity E at any point on the equitorial line ( broad-side on position) of an electric dipole

Ans

E1 E1sinθ

E θ P E1 θ

( r2+L2)12 E2 E

r E2 E2sinθ

-q θ L O L +qA B

Let us consider that the point P is situated on the right bisector of the dipole AB at a distance r meter from its midpoint O

Let E1 and E2 be the electric field intensities of the electric field at P due to charge +q and ndashq of the dipole resp The distance of P from each charge is ( r2+L2)12

So E1 = 14 πϵ q

(r 2+L 2) away from +q

E2 = 14 πϵ q

(r 2+L 2) towards ndashq

The magnitudes of E1 and E2 are equal but directions are different Now resolving E1 and E2 into two components parallel and perpendicular to AB we get

The components perpendicular to AB E1sinθ and E2sinθ cancel each other because they are equal and opposite

The components parallel to AB are E1cosθ and E2 cosθ are in same direction and add up

So resultant intensity of electric field at the point P is

E = E1cosθ + E2 cosθ

E = 14 πϵ q

(r 2+L 2) 2 cosθ

Now from fig we have cosθ =BOBP = L (r2+L2)12

So we get E = 14 πϵ 2qL ( r2+L2)32

Now electric dipole moment p= 2qL

So E = 14 πϵ p ( r2+L2)32

HW Find the expression of Electric field as done here but this time take r gtgt 2L

Also find the expression of torque experience by a dipole

(Hint Electric force experienced by charges of dipole in electric field is qE each Let θ be the angle which dipole makes with electric lines of force then perpendicular distance between two charges is 2Lsinθ Then torque = force x perp distance = qE x 2L sinθ So τ=pE sinθ where p =2qL )

STUDY MATERIAL

Class XIISubject Eng Literature (The Tempest ndash William Shakespeare) Topic Act IV Scene 1 Lines 84 to 133 (Iris hellip A contract of true love Be not too late ) Date 27th April 2020 (4th Period)

[Students should read the original play and also the paraphrase given in the school prescribed textbook]Summary Questions amp Answers

o Ceres soon appears and comes to know that she has been summoned to celebrate the contract of true love

o Ceres expresses her unwillingness to meet Venus and Cupid as she has shunned their company

o Ceres and Juno both bestow their blessings upon Ferdinand and Miranda with June gifting honour riches happiness in marriage and Ceres presents plenty of earthrsquos produce

o Iris summons the water-nymphs and reapers to come and celebrate a contract

(1) IRIS Of her society (Line 91-101)

Be not afraid I met her deity

Cutting the clouds towards Pathos and her sonDove-drawn with her Here thought they to have doneSome wanton charm upon this man and maidWhose vows are that no bed-right shall be paidTill Hymens torch be lightedmdashbut in vainMarss hot minion is returned againHer waspish-headed son has broke his arrowsSwears he will shoot no more but play with sparrowsAnd be a boy right out

(i) Where were Venus and Cupid seen flying How were they travelling Why did they want to join the marriage celebration of Ferdinand and Miranda

of true love

Venus and Cupid were seen flying through the air towards Paphos the famous city which is situated on the island of Cyprus They were travelling by air-borne chariot drawn by doves They certainly wanted to come here in order to play some amorous trick upon Ferdinand and Miranda who are under a vow not to gratify their physical desires till the holy ceremony of their marriage has been performed(ii) What have Venus and Cupid done after failing in their plan

After being failure of their plan Venus who is a very passionate deity and who is the mistress of Mars (the god of war) has gone back while here ill-tempered son Cupid has broken his arrows of love in his state of desperation(iii) What has Cupid firmly decided

Cupid is feeling so disappointed that he has firmly decided to shoot no more arrows to arouse love in human hearts but to spend his time playing with sparrows Thus he would now become just a boy and would give up his original function of shooting arrows on human beings to make them fall in love(iv) What vow had Ceres taken How did Ceres feel at the abduction

After the abduction of her daughter Prosperina by Pluto Ceres had taken a vow to always keep away from the disgraceful company of Venus and her blind son Cupid the god of love Ceres felt deeply distressed when Pluto had carried off her daughter and had made her his wife by force(v) Why has Ceres not forgiven Venus and her blind son For what do Ceres want to be sure

As the abduction had been manipulated by Venus the goddess of beauty and love and her blind son Cupid Ceres has never forgiven them for their part in the whole plot Ceres wants to be sure that she would not have to meet Venus and Cupid who had engineered the abduction of her daughter Prosperina

AS THIS lsquoMASQUErsquo SCENE IS VERY IMPORTANT IN THE PLAY THE PARAPHRASE OF THE ENTIRE PORTION OF MASQUE SCENE (Act IV Lines 58 to 143) IS GIVEN BELOW

IRIS Goddess of RainbowCERES Goddess of Agriculture and all the fruits of the earth

(Nature growth prosperity rebirth ndash notions intimately connected to marriage)JUNO The majestic Queen of Heavens and wife of Jupiter (Jupiter is the king of Gods)

VENUS The Goddess of love CUPID Son of Venus PLUTO God of death (In the play referred by Shakespeare as lsquoDisrsquo which is a Roman name for Pluto)

ORIGINAL TEXT PARAPHRASEPROSPEROWellmdash

PROSPERONow come Ariel Let there be too many rather than too few

Now come my Ariel Bring a corollaryRather than want a spirit Appear and pertly[to Ferdinand and Miranda]No tongue all eyes Be silent

spirits in attendance Appear briskly

[to Ferdinand and Miranda]Look with your eyes but do not say a word

[Soft music] [Soft music][Enter Iris] [Enter Iris]

IRISCeres most bounteous lady thy rich leasOf wheat rye barley vetches oats and peasThy turfy mountains where live nibbling sheepAnd flat meads thatched with stover them to keepThy banks with pioned and twilled brimsWhich spongy April at thy hest betrimsTo make cold nymphs chaste crowns and thybroom-grovesWhose shadow the dismissegraved bachelor lovesBeing lass-lorn thy pole clipped vineyardAnd thy sea-marge sterile and rocky-hardWhere thou thyself dost airmdashthe Queen othrsquoSkyWhose watery arch and messenger am IBids thee leave these and with her sovereign grace[Juno appears] Here on this grass-plot in this very placeTo come and sport Her peacocks fly amainApproach rich Ceres her to entertain

IRISCeres most generous lady you are the cause of rich fields or fertile land where wheat rye barley beans oats and peas grow the grassy mountains where the sheep graze and the flat meadows covered with coarse hay to be used as fodder for cattleYour banks are covered with marsh-marigolds and reeds and the rainy April under your orders brings forth to make for the maids who are not in love beautiful crowns your woods where the broom flourishes and where the bachelor who has been dismissed by the maid he loved lies down being forsaken your vineyard in which the poles are embraced by the vines and the margin of the sea which is barren and rocky where you roam about to enjoy the fresh air ndash the queen of the sky (Juno) whose messenger I am besides being represented as the rainbow bids you leave all these and with her majesty here on this grassy plot in this very place come and sport her peacocks carry her fast in her chariot through the air and are making their way here approach rich Ceres to welcome her

[Enter Ariel as Ceres] [Enter Ariel as Ceres]

CERESHail many-coloured messenger that neerDost disobey the wife of JupiterWho with thy saffron wings upon my flowersDiffusest honey-drops refreshing showersAnd with each end of thy blue bow dost crownMy bosky acres and my unshrubbed downRich scarf to my proud earth Why hath thy queenSummoned me hither to this short-grassed green

CERESWelcome rainbow that never dared disobey Juno the wife of Jupiter who with your orange coloured rays spread honey-drops refreshing showers And with each end of thy blue bow drown my bushy acres and my hilly country which is free from shrubs you thus forming a rich scarf Why has your queen called me here to this place covered with short grass

IRISA contract of true love to celebrateAnd some donation freely to estateOn the blest lovers

IRISI have called you to celebrate a contract of true love and bestow some liberal gift upon the blessed lovers

ORIGINAL TEXT PARAPHRASECERESTell me heavenly bowIf Venus or her son as thou dost knowDo now attend the queen Since they did plotThe means that dusky Dis my daughter gotHer and her blind boys scandaled companyI have forsworn

CERESTell me heavenly bow if Venus the Goddess of love or Cupid her son and pedlar of passion at this time attend the heavenly queen Juno because you are sure to know Since the day they conspired against me and dark Pluto took away my daughter here and Cupidrsquos disgraceful company I have left off

IRISOf her societyBe not afraid I met her deityCutting the clouds towards Pathos and her sonDove-drawn with her Here thought they to have doneSome wanton charm upon this man and miad

IRISBe not afraid of her company I met her deity moving on the clouds towards Paphos the sacred home of Venus on the island of Cyprus along with her son on her chariot drawn by doves Here they contemplated to exercise a charm upon this man and maid producing

Whose vows are that no bed-right shall be paidTill Hymens torch be lightedmdashbut in vainMarss hot minion is returned againHer waspish-headed son has broke his arrowsSwears he will shoot no more but play with sparrowsAnd be a boy right out

wantonness before the actual marriage ceremony but did not succeed Venus has returned her irritable son has broken his arrows and swears that he will give up his practice of trying to inspire love but play with sparrows and be a boy again

[Music is heard] [Music is heard]

CERESHighst queen of stateGreat Juno comes I know her by her gait

CERESHighest queen of state Great Juno there she comes I know here by her gait

[Enter Juno] [Enter Juno]

JUNOHow does my bounteous sister Go with meTo bless this twain that they may Prosperous beAnd honoured in their issue

JUNOHow are you doing my generous sister Come with me to bless this couple so that they may be prosperous and fortunate in their children

[They sing] [They sing]

JUNOHonour riches marriage-blessingLong continuance and increasingHourly joys be still upon youJuno sings her blessings upon you

JUNOMay honour riches happiness in marriage long continuance and increase of those boons ever rest upon you as hourly joys Juno showers down upon you her blessings in song

CERESEarths increase foison plentyBarns and garners never emptyVines and clustring bunches growingPlants and goodly burden bowingSpring come to you at the farthestIn the very end of harvestScarcity and want shall shun youCeresrsquo blessing so is on you

CERESMay you have the plenty of earthrsquos produce Your barns and granaries may never be empty Your vines may grow with clustering bunches Your fruit trees may be heavily laden with their fruit May there be continuous spring and harvest May scantiness and want leave you forever Such is the blessing of Ceres upon you

FERDINANDThis is a most majestic vision andHarmoniously charmingly May I be boldTo think these spirits

FERDINANDThis is a great vision and magically melodious Should I suppose the characters (taking part in the masque) are spirits

PROSPEROSpirits which by mine artI have from their confines calld to enactMy present fancies

PROSPEROYes they are spirits whom I have summoned from the regions to which they are confined to carry into effect my fanciful designs

ORIGINAL TEXT PARAPHRASEFERDINANDLet me live here everSo rare a wondered father and a wifeMakes this place paradise

FERDINANDI should like to live here forever Such a wise and wonderful father makes this place a paradise

[Juno and Ceres whisper and send Iris on employment] [Juno and Ceres whisper and send Iris on employment]

PROSPEROSweet now silence

PROSPEROMy dear Ferdinand speak no more Juno and Ceres are

Juno and Ceres whisper seriouslyTheres something else to do Hush and be muteOr else our spell is marred

whispering with a solemn look There is something else coming Silence Or else our magic will be spoilt

IRISYour nymphs called naiads of the wandering brooksWith your sedged crowns and over-harmless looksLeave your crisp channels and on this green landAnswer your summons Juno does commandCome temperate nymphs and help to celebrateA contract of true love Be not too late

IRISYou nymphs called Naiads denizens (M inhabitants) of the running stream with your chaplets of sedge and ever-helpful looks leave your wrinkled channels and on the green land answer the summons sent to you Juno has ordered some chaste nymphs and help to celebrate a noble and true marriage Donrsquot delay

[Enter certain nymphs] [Enter certain nymphs]You sunburnt sicklemen of August wearyCome hither from the furrow and be merryMake holiday your rye-straw hats put onAnd these fresh nymphs encounter every oneIn country footing

You sunburnt harvesters weary from the effects of the heat in August come here from the furrowed land and rejoice Make holiday with your rye-straw hats upon you and meet these fresh nymphs and join in country dancing

[Enter certain reapers properly habited They join with the nymphs in a graceful dance towards the end whereof Prospero starts suddenly and speaks]

[Enter certain reapers properly habited They join with the nymphs in a graceful dance towards the end whereof Prospero starts suddenly and speaks]

PROSPERO[aside] I had forgot that foul conspiracyOf the beast Caliban and his confederatesAgainst my life The minute of their plotIs almost come [to the spirits]Well done Avoidno more

PROSPERO(Aside)I had forgotten the wicked conspiracy of the beast Caliban and his accomplices against my life the time of their plot has almost arrived ndash (To the Spirits) well done depart no more of this

[To a strange hollow and confused noise the spirits heavily vanish]

[The spirits depart]

ORIGINAL TEXT PARAPHRASEFERDINANDLet me live here everSo rare a wondered father and a wifeMakes this place paradise

FERDINANDI should like to live here forever Such a wise and wonderful father makes this place a paradise

[Juno and Ceres whisper and send Iris on employment] [Juno and Ceres whisper and send Iris on employment]

PROSPEROSweet now silenceJuno and Ceres whisper seriouslyTheres something else to do Hush and be muteOr else our spell is marred

PROSPEROMy dear Ferdinand speak no more Juno and Ceres are whispering with a solemn look There is something else coming Silence Or else our magic will be spoilt

IRISYour nymphs called naiads of the wandering brooksWith your sedged crowns and over-harmless looksLeave your crisp channels and on this green landAnswer your summons Juno does commandCome temperate nymphs and help to celebrateA contract of true love Be not too late

IRISYou nymphs called Naiads denizens (M inhabitants) of the running stream with your chaplets of sedge and ever-helpful looks leave your wrinkled channels and on the green land answer the summons sent to you Juno has ordered some chaste nymphs and help to celebrate a noble and true marriage Donrsquot delay

[Enter certain nymphs] [Enter certain nymphs]You sunburnt sicklemen of August wearyCome hither from the furrow and be merryMake holiday your rye-straw hats put onAnd these fresh nymphs encounter every oneIn country footing

You sunburnt harvesters weary from the effects of the heat in August come here from the furrowed land and rejoice Make holiday with your rye-straw hats upon you and meet these fresh nymphs and join in country dancing

[Enter certain reapers properly habited They join with the nymphs in a graceful dance towards the end whereof Prospero starts suddenly and speaks]

[Enter certain reapers properly habited They join with the nymphs in a graceful dance towards the end whereof Prospero starts suddenly and speaks]

PROSPERO[aside] I had forgot that foul conspiracyOf the beast Caliban and his confederatesAgainst my life The minute of their plotIs almost come [to the spirits]Well done Avoidno more

PROSPERO(Aside)I had forgotten the wicked conspiracy of the beast Caliban and his accomplices against my life the time of their plot has almost arrived ndash (To the Spirits) well done depart no more of this

[To a strange hollow and confused noise the spirits heavily vanish]

[The spirits depart]

Ac-12 27420 topic Revaluation of Assets and Liabilities

REVALUATION OF ASSETS AND LIABILITIES

On admission of a new partner the firm stands reconstituted and consequently the assets are revalued and liabilities are reassessed It is necessary to show the true position of the firm at the time of admission of a new partner If the values of the assets are raised gain will increase the capital of the existing partners Similarly any decrease in the value of assets ie loss will decrease the capital of the existing partners For this purpose alsquoRevaluation Accountrsquo is prepared This account is credited with all increases in the value of assets and decrease in the value of liabilities It is debited with decrease on account of value of assets and increase in the value of liabilities The balance of this account shows a gain or loss on revaluation which is transferred to the existing partnerrsquos capital account in existing profit sharing ratioAccounting for Revaluation of Assets and Liabilities when there is a Changein the Profit Sharing Ratio of Existing PartnersAssets and liabilities of a firm must also be revalued at the time of change in profit sharing ratio of existing partners The reason is that the realisable or actual value of assets and liabilities may be different from those shown in the Balance Sheet It is possible that with the passage of time some of the assets might have appreciated in value while the value of certain other assets might have decreased and no record has been made of such changes in the books of accounts Similarly there may be some unrecorded assets amp libilities that may have to be accounted for Revaluation of assets and reassessments of liabilities becomes necessary because the change in the

value of assets and liabilities belongs to the period to change in profit sharing ratio and hence must be shared by the partners in their old profit sharing ratio Revaluation of assets and reassessment of liabilities may be given effect to in two different ways (a) When revised values are to be recorded in the books and(b) When revised values are not to be recorded in the books

When revised values are to be recorded in the booksIn such a case revaluation of assets and reassessment of liabilities is done with the help of a new account called lsquoRevaluation Accountrsquo Sometimes this account is also called as lsquoProfit amp Loss Adjustment Acrsquo If there is a loss due to revaluation revaluation account is debited and if the revaluation results in a profit the revaluation account is credited The following journal entries made for this purpose are

(i) For increase in the value of assetsAsset Ac Dr (individually)To Revaluation Ac(ii) For decrease in the value of AssetRevaluation Ac Dr (individually)To Asset Ac[Decrease in the value of assets](iii) For increase in the value of LiabilitiesRevaluation Ac Dr (individually)To Liabilities Ac[Increase in the value of Liabilities](iv) For decrease in the value of LiabilitiesLiabilities Ac DrTo Revaluation Ac[Decrease in the value of Liabilities](v) For unrecorded AssetsAsset Ac [unrecorded] DrTo Revaluation Ac[Unrecorded asset recorded at actual value](vi) For unrecorded Liability Revaluation Ac DrTo Liability Ac [unrecorded][Unrecorded Liability recorded at actual value](vii) For transfer of gain on revaluationRevaluation Ac DrTo Existing Partnerrsquos CapitalCurrent Ac[Profit on revaluation transferred to capital account in existing ratio](viii) For transfer of loss on revaluationExisting Partnerrsquos CapitalCurrent Ac DrTo Revaluation Ac[Loss on revaluation transferred to capital account in existing ratio](a) When revaluation account shows gain Revaluation Ac DrTo Partnerrsquos Capital Ac (Old Profit Sharing Ratio)(Profit on revaluation credited to Partnerrsquos Capital Ac)(b) Above entry is reversed when revaluation account shows loss Partners Capital Acs (Old Profit Sharing Ratio) DrTo Revaluation Ac(Loss on revaluation debited to Partnerrsquos Capital Acs)

Proforma of Revaluation Account is given as under

Revaluation Account

Dr Cr Particulars ` Amount Particulars ` Amount To Decrease in value of assets By Increase in value of assets To Increase in value of liabilities By Decrease in value of liabilities To Unrecorded liabilities By Unrecorded assets To Gain on Revaluation (Transferred) By Loss on Revalution (Transferred)

ECO ndash12 2742020Topic- ELASTICITY OF DEMAND

CHAPTER - ELASTICITY OF DEMANDMEANINGDemand for a commodity is affected by many factors such as its price price of related goods income of its buyer tastes and preferences etc Elasticity means degree of response Elasticity of demand means degree of responsiveness of demand Demand for a commodity responds to change in price price of related goods income etc So we have three dimensions of elasticity of demandDIMENSION OF ELASTICITY OF DEMAND TYPES OF ELASTICITY OF DEMAND

Price elasticity of demand Income elasticity of demand Cross Elasticity of demand

Price elasticity of demand Price elasticity of demand means degree of responsiveness of demand for a commodity to the change in its price For example if demand for a commodity rises by 10 due to 5 fall in its price Price elasticity of demand (ep)=Percentage change in quantity demanded Percentage change in price of the commodity = 10 ( -)5 = ( - )2Note that ep will always be negative due to inverse relationship of price and quantity demanded

(ii) Income elasticity of demand Income elasticity of demand refers to the degree of responsiveness of demand for a commodity to the change in income of its buyer Suppose income of buyer rises by 10 and his demand for a commodity rises by 20 then Income elasticity of demand (ey)= change in quantity demanded change in price of the commodity =20 10 = 2

Cross Elasticity of demandCross elasticity of demand means the degree of responsiveness of demand for a commodity to the change in price of its related goods (substitute goods or complementary goods) Suppose demand for a commodity rises by 10 due to 5 rise in price of its substitute good then Cross elasticity of demand (ec) = change in quantity demanded change in price of related good = 10 2 = 5 (Tastes and preferences cannot be expressed numerically So elasticity ofdemand cannot be numerically expressed)

  • Chapter 1 Force (Summary)
  • Distinguish between external sovereignty and internal sovereignty
    • NAND Gate
      • Logic diagram
      • Truth Table
        • NOR Gate
          • Logic diagram
          • Truth Table
            • XOR Gate
              • Logic diagram
              • Truth Table
                • XNOR Gate
                  • Logic diagram
                  • Truth Table
                      • Physics
                      • Chapter 1 Electric Field ( Electric Dipole) (Summary)
Page 37:  · Web viewWe all know that Nouns are divided into two parts: common noun and proper noun.Apart from common and proper noun, we will also study about collective noun and compound

Most of the accused were convicted and sentenced to

long term of imprisonmentBut

Aurobindo Ghosh was acquitted mainly owing to the brilliant pleading of his counsel Chittaranjan Das

Political science Topic-Sovereignty

Summary Sovereignty is the full right and power of a governing body over itself without any interference from outside sources or bodies In political theory sovereignty is a substantive term designating supreme legitimate authority over some polity In international law sovereignty is the exercise of power by a state

Internal Sovereignty

Internal sovereignty means supreme authority within ones territory while external sovereignty relates to the recognition on the part of all states that each possesses this power in equal measure

External sovereignty

external sovereignty relates to the recognition on the part of all states that each possesses this power in equal measure

Distinguish between

Execution

Answer the following questions

Short notes-

Sovereignty

Internal Sovereignty

External sovereignty

Homework- learn

external sovereignty and internal sovereigntySovereignty is the principle

of supreme and

unquestionable authority

reflected in the claim by the

state to be the sole author of

laws within its territory

Definition of external vs internal sovereigntyInternal sovereignty refers to

the relationship between a

sovereign power and its

subjects ndash it refers to the

location of the supreme

authority within the state In

the UK for example internal

sovereignty (supposedly)

resides within Parliament

reflected in the

constitutional principle of

parliamentary

sovereigntyBy contrast

external sovereignty refers

to the capacity of the state

to act independently and

autonomously on the world

stage This is what is

sometimes called lsquostate

sovereigntyrsquo or lsquonational

sovereigntyrsquo and implies

that states are legally equal

and that the territorial

integrity and political

independence of a state is

inviolable

Class ndash XII

Date - 2742020 STUDY MATERIALSubject Topic Summary Execution Business Studies

Job Analysis amp Manpower Planning

At first let us recall the chapter what we have discussed till nowbullJob analysisbullJob specification bullJob description bullJob enlargement bullJob enrichment

Today we will do some questions answers from the chapter

Questions 1ldquoJob analysis job description and job specification are interrelatedrdquo Comment Answer) Job analysis is a systematic and detailed examination of a job to collect all the relevant information about it The contents off the job are summarised in the job description The qualification needed for the job are summarised in job specificationThus there is close interrelationship between job analysis job description and job specification

Question 2ldquoJob enlargement is a horizontal extension of a job whereas job enrichment is a vertical extension of a jobrdquo ElucidateAnswer) Job enlargement involves adding one or more task to a job coma where as job enrichment involves adding more autonomy and responsibility to a job Job enlargement is therefore horizontal extension of a job coma whereas job enrichment is a vertical extension of a job

Question 3 )

What is manpower estimation Explain its quantitative and qualitative aspectsAnswer) Manpower estimation is the process by which management determines how an organisation should move from itrsquos current manpower positionto its desired manpower position There are two dimensions of Manpower estimation- quantitative and qualitative

Quantitative aspectThis aspect of Manpower estimation involves estimating the number of employees required in a future time period Workload analysis and workforce analysis are done to estimate the quantity of required manpower

Qualitative aspectThe estimate of the knowledge skills experience etc of required manpower is the qualitative aspect of Manpower estimation The quality of Manpower can be judged on the basis of job analysisand job specification

COMMERCE

CAPITAL-FIXED AND WORKING

Today let us start the class by discussing the sources of finance for different types of business firms

The term lsquocapitalrsquo refers to the investment made in the enterprise for the purpose of earning profits

Requirements of capital and sources of capital for different types of business firms are

1 Capital for sole proprietorship businessA sole proprietor operates at a small scale and thereforerequires a limited amount of capital

2 Capital for partnership firmCapital requirements as well as capital base of a partnership is bigger than that of a sole trader businessThe owned capital is contributed by the partners in an agreed ratio

3 Capital for joint stock companyA joint stock company generally requires large amount of capitalA public company can raise huge capital through issue of shares In addition to share capital it can utilize retained profits

Now let u discuss the meaning of Finance PlanningFinance planning is the process of estimation the financial requirements of an organization specifying the sources of firms and ensuring that enough funds are available at the right time

1 What do you mean by Finance PlanningAnswer Finance planning is the process of estimation the financial requirements of an organization specifying the sources of firms and ensuring that enough funds are available at the right time

2Discuss the role of financial planning of an enterpriseThe role of financial planning are as followsa A sound financial plan helps a business enterprise to avaid the problems of shortage and surplus of fundsbFinancial planning serves as a guide in developing a sound capital structure so as to maximize returns to shareholders c It helps in effective utilization of fundsd It provides policies and procedures for coordinating different functional areas or departments of businesse It enables the management to exercise effective control over the financial activities of an enterprisef It helps the company to prepare for facing business shocks and surprises in future

Mathematics

Continuity and differentiability

Recall Definition of ContinuityLet f(x) be a single valued function of x and x=a be a point in the domain of definition of the function The function is said to be continuous at x=a ifi) f(c) is defined ie f(x) has a definite finite value at x=cii) lim xrarra f(x) exists andiii) lim xrarra f(x) =f(a) In other words f(x) is said to be continuous at x=a if lim xrarra+ f(x)= lim xrarra- f(x) = f(a) Or f(a+0) =f(a-0) =f(a) Or lim hrarr0 f(a+h)= f(a) Algebra of continuous functionsNow we will study some algebra of continuous functions Theorem 1 Suppose f and g be two real functions continuous at a real number c Then(1) f + g is continuous at x = c(2) f ndash g is continuous at x = c(3) f g is continuous at x = c(4) (fg) is continuous at x = c (provided g (c) ne 0)

Example 1 Prove that every rational function is continuousSolution Recall that every rational function f is given byf(x)=[p(x) q(x) ] q(x)ne0where p and q are polynomial functions The domain of f is all real numbers except points at which q is zero Since polynomial functions are continuous f is continuous by (4) of Theorem 1Example 2Discuss the continuity of sine functionSolution To see this we use the following factslim xrarr0 sin x =0Now observe that f (x) = sin x is defined for every real number Let c be a real number Put x = c + h If x rarr c we know that h rarr 0 Therefore lim xrarrc f(x)

= lim xrarrc sin x= lim hrarr0 sin(c+h) =lim hrarr0 [sin c cos h + cos c sin h ]=lim hrarr0 (sin c cos h) + lim hrarr0 (cos c sin h) = sin c +0=sin c = f(c) Thus lim xrarrc f(x) = f(c) and hence f is a constant function Exercise Prove that the function f(x) = x2 +2x is continuous for every real value of x [Hints show that lim xrarra+ f(x) = lim xrarra- f(x) = f(a) ]

Biology Reproduction in Flowering plants We will discuss about megasporoangium

megasporagenesis and female gametophyte

Q4 Describe the structure of megasporangium

Ovule is attached to the placenta by astalk called funicle

Each ovule has one two or three protectivecoverings called integuments

At the tip of integuments a small openingcalled micropyle is organised

Opposite to the micropylar end is the chalaza

Within the integuments a mass of cellsnucellusand inside it embryo sac orfemale gametophyte is present

Q5 Describe a mature embryosacamp its formation

In most of the flowering plants only oneof the 4 megaspores formed as a result ofmegasporogenesis that is functional while theother three degenerate

The

functional megaspore develops into thefemale gametophyte

Formation The nucleus

of the functional megasporedivides mitotically to form two nuclei first andthen two more sequential mitotic nucleardivisions result in the formation of four ampthen eight nucleate stages of embryo sac

Six of the eight nucleus are surrounded bycell walls and organised into cells

The remaining two nuclei called polar nuclei are found below the egg apparatus in the largecentral cell

Three cells consisting of two synergids amp one egg cell present bottom of

embryo sac Three cells

at the chalazal as antipodal cells

Two polar nuclei together present in large central cell

HISTORY

TOWARDS INDEPENDENCE AND PARTITION THE LAST PHASE(1935-1947)SUB TOPIC NATIONAL MOVEMENTS DURING THE SECOND WORLD WAR

Spread of Quit India Movement On 9th August 1942Gandhiji and other Congress leaders were arrested The Congress was declared illegal The news of the arrest of all leaders marked the beginning of a widespread movement of India It was not possible for such a movement to remain peacefulBut the arrest of the all notable congress leaders virtually left the movement in the hands of the mass The movement took the form of violent and militant outbreakBesides congressmen revolutionaries also were very active in the movement The Congress Socialist group also played a prominent role

1 Question Why did the British authority arrest the Congress leaders on 9 th August 1942Answer Congress Working committee adopted the Quit India resolution which was to be ratified at the Bombay AICC meeting in 8th August 1942 They decided to launch a mass struggle on non-violent lines Gandhiji gave a clarion call to all section of the people rdquoKarenge ya Marengerdquo (do or die) Congress leaders gave the call to driving out

the British from IndiaViceroy had taken strong action against the Quit India movement Gandhiji and all the leaders of Congress were arrested

2 Question How did Quit India Movement spread out all over IndiaAnswer The news of the leaders lsquo arrest marked the beginning of a widespread movement to remain peacefulThe movement took form of violent outbreak There were widespread cutting of telephone and Telegraph wires damaging railway lines raising barricades in cities and towns and other forms of violent demonstations

Question Name the leaders of Congress

Socialist group played a prominent part Notable among the Jayprakash Narayan Rammonohar Lohia Aruna Asaf Ali

Political science

Topic-Franchise and Representation

Summary

The election commission

The Election Commission of India is an autonomous constitutional authority responsible for administering Union and State election processes in India The body administers elections to the Lok Sabha Rajya Sabha State Legislative Assemblies in India and the offices of the President and Vice President in the country

Functions of election commission-

India is a sovereign socialist secular democratic republic Democracy runs like a golden thread in the social economic and political fabric woven by the Constitution given by lsquoWe the People of Indiarsquo unto ourselves The concept of democracy as visualised by the Constitution pre-supposes the representation of the people in Parliament and State legislatures by the method of election The Supreme Court has held that democracy is one of the inalienable basic features of the Constitution of India and forms part of its basic structure The Constitution of India adopted a Parliamentary form of government Parliament consists of the President of India and the two Houses mdash Rajya Sabha and Lok Sabha India being a Union of states has separate state legislatures for each state State legislatures consist of the Governor and two Houses mdash Legislative Council and Legislative Assembly mdash in seven states namely Andhra Pradesh Telangana Bihar Jammu amp Kashmir Karnataka Maharashtra and Uttar Pradesh and of the Governor and the state Legislative Assembly in the remaining 22 states Apart from the above two out of the seven Union Territories namely National Capital Territory of Delhi and Puducherry also have their Legislative Assemblies

ExecutionShort notes-Election commissionFunctions of election commission

Homework- Learn

Computer

Science

Computer hardware NAND Gate

A NOT-AND operation is known as NAND operation It has n input (n gt= 2) and one output

Logic diagram

Truth Table

NOR Gate

A NOT-OR operation is known as NOR operation It has n input (n gt= 2) and one output

Logic diagram

Truth Table

XOR Gate

XOR or Ex-OR gate is a special type of gate It can be used in the half

adder full adder and subtractor The exclusive-OR gate is abbreviated as EX-OR gate or sometime as X-OR gate It has n input (n gt= 2) and one output

Logic diagram

Truth Table

XNOR Gate

XNOR gate is a special type of gate It can be used in the half adder full adder and subtractor The exclusive-NOR gate is abbreviated as EX-NOR gate or sometime as X-NOR gate It has n input (n gt= 2) and one output

Logic diagram

Truth Table

Physics

Chapter 1 Electric Field ( Electric Dipole) (Summary)

Here we will derive Expression of electric field at broad side

On position of dipole

Execution

Q With the help of a labelled diagram obtain an expression for the electric field intensity E at any point on the equitorial line ( broad-side on position) of an electric dipole

Ans

E1 E1sinθ

E θ P E1 θ

( r2+L2)12 E2 E

r E2 E2sinθ

-q θ L O L +qA B

Let us consider that the point P is situated on the right bisector of the dipole AB at a distance r meter from its midpoint O

Let E1 and E2 be the electric field intensities of the electric field at P due to charge +q and ndashq of the dipole resp The distance of P from each charge is ( r2+L2)12

So E1 = 14 πϵ q

(r 2+L 2) away from +q

E2 = 14 πϵ q

(r 2+L 2) towards ndashq

The magnitudes of E1 and E2 are equal but directions are different Now resolving E1 and E2 into two components parallel and perpendicular to AB we get

The components perpendicular to AB E1sinθ and E2sinθ cancel each other because they are equal and opposite

The components parallel to AB are E1cosθ and E2 cosθ are in same direction and add up

So resultant intensity of electric field at the point P is

E = E1cosθ + E2 cosθ

E = 14 πϵ q

(r 2+L 2) 2 cosθ

Now from fig we have cosθ =BOBP = L (r2+L2)12

So we get E = 14 πϵ 2qL ( r2+L2)32

Now electric dipole moment p= 2qL

So E = 14 πϵ p ( r2+L2)32

HW Find the expression of Electric field as done here but this time take r gtgt 2L

Also find the expression of torque experience by a dipole

(Hint Electric force experienced by charges of dipole in electric field is qE each Let θ be the angle which dipole makes with electric lines of force then perpendicular distance between two charges is 2Lsinθ Then torque = force x perp distance = qE x 2L sinθ So τ=pE sinθ where p =2qL )

STUDY MATERIAL

Class XIISubject Eng Literature (The Tempest ndash William Shakespeare) Topic Act IV Scene 1 Lines 84 to 133 (Iris hellip A contract of true love Be not too late ) Date 27th April 2020 (4th Period)

[Students should read the original play and also the paraphrase given in the school prescribed textbook]Summary Questions amp Answers

o Ceres soon appears and comes to know that she has been summoned to celebrate the contract of true love

o Ceres expresses her unwillingness to meet Venus and Cupid as she has shunned their company

o Ceres and Juno both bestow their blessings upon Ferdinand and Miranda with June gifting honour riches happiness in marriage and Ceres presents plenty of earthrsquos produce

o Iris summons the water-nymphs and reapers to come and celebrate a contract

(1) IRIS Of her society (Line 91-101)

Be not afraid I met her deity

Cutting the clouds towards Pathos and her sonDove-drawn with her Here thought they to have doneSome wanton charm upon this man and maidWhose vows are that no bed-right shall be paidTill Hymens torch be lightedmdashbut in vainMarss hot minion is returned againHer waspish-headed son has broke his arrowsSwears he will shoot no more but play with sparrowsAnd be a boy right out

(i) Where were Venus and Cupid seen flying How were they travelling Why did they want to join the marriage celebration of Ferdinand and Miranda

of true love

Venus and Cupid were seen flying through the air towards Paphos the famous city which is situated on the island of Cyprus They were travelling by air-borne chariot drawn by doves They certainly wanted to come here in order to play some amorous trick upon Ferdinand and Miranda who are under a vow not to gratify their physical desires till the holy ceremony of their marriage has been performed(ii) What have Venus and Cupid done after failing in their plan

After being failure of their plan Venus who is a very passionate deity and who is the mistress of Mars (the god of war) has gone back while here ill-tempered son Cupid has broken his arrows of love in his state of desperation(iii) What has Cupid firmly decided

Cupid is feeling so disappointed that he has firmly decided to shoot no more arrows to arouse love in human hearts but to spend his time playing with sparrows Thus he would now become just a boy and would give up his original function of shooting arrows on human beings to make them fall in love(iv) What vow had Ceres taken How did Ceres feel at the abduction

After the abduction of her daughter Prosperina by Pluto Ceres had taken a vow to always keep away from the disgraceful company of Venus and her blind son Cupid the god of love Ceres felt deeply distressed when Pluto had carried off her daughter and had made her his wife by force(v) Why has Ceres not forgiven Venus and her blind son For what do Ceres want to be sure

As the abduction had been manipulated by Venus the goddess of beauty and love and her blind son Cupid Ceres has never forgiven them for their part in the whole plot Ceres wants to be sure that she would not have to meet Venus and Cupid who had engineered the abduction of her daughter Prosperina

AS THIS lsquoMASQUErsquo SCENE IS VERY IMPORTANT IN THE PLAY THE PARAPHRASE OF THE ENTIRE PORTION OF MASQUE SCENE (Act IV Lines 58 to 143) IS GIVEN BELOW

IRIS Goddess of RainbowCERES Goddess of Agriculture and all the fruits of the earth

(Nature growth prosperity rebirth ndash notions intimately connected to marriage)JUNO The majestic Queen of Heavens and wife of Jupiter (Jupiter is the king of Gods)

VENUS The Goddess of love CUPID Son of Venus PLUTO God of death (In the play referred by Shakespeare as lsquoDisrsquo which is a Roman name for Pluto)

ORIGINAL TEXT PARAPHRASEPROSPEROWellmdash

PROSPERONow come Ariel Let there be too many rather than too few

Now come my Ariel Bring a corollaryRather than want a spirit Appear and pertly[to Ferdinand and Miranda]No tongue all eyes Be silent

spirits in attendance Appear briskly

[to Ferdinand and Miranda]Look with your eyes but do not say a word

[Soft music] [Soft music][Enter Iris] [Enter Iris]

IRISCeres most bounteous lady thy rich leasOf wheat rye barley vetches oats and peasThy turfy mountains where live nibbling sheepAnd flat meads thatched with stover them to keepThy banks with pioned and twilled brimsWhich spongy April at thy hest betrimsTo make cold nymphs chaste crowns and thybroom-grovesWhose shadow the dismissegraved bachelor lovesBeing lass-lorn thy pole clipped vineyardAnd thy sea-marge sterile and rocky-hardWhere thou thyself dost airmdashthe Queen othrsquoSkyWhose watery arch and messenger am IBids thee leave these and with her sovereign grace[Juno appears] Here on this grass-plot in this very placeTo come and sport Her peacocks fly amainApproach rich Ceres her to entertain

IRISCeres most generous lady you are the cause of rich fields or fertile land where wheat rye barley beans oats and peas grow the grassy mountains where the sheep graze and the flat meadows covered with coarse hay to be used as fodder for cattleYour banks are covered with marsh-marigolds and reeds and the rainy April under your orders brings forth to make for the maids who are not in love beautiful crowns your woods where the broom flourishes and where the bachelor who has been dismissed by the maid he loved lies down being forsaken your vineyard in which the poles are embraced by the vines and the margin of the sea which is barren and rocky where you roam about to enjoy the fresh air ndash the queen of the sky (Juno) whose messenger I am besides being represented as the rainbow bids you leave all these and with her majesty here on this grassy plot in this very place come and sport her peacocks carry her fast in her chariot through the air and are making their way here approach rich Ceres to welcome her

[Enter Ariel as Ceres] [Enter Ariel as Ceres]

CERESHail many-coloured messenger that neerDost disobey the wife of JupiterWho with thy saffron wings upon my flowersDiffusest honey-drops refreshing showersAnd with each end of thy blue bow dost crownMy bosky acres and my unshrubbed downRich scarf to my proud earth Why hath thy queenSummoned me hither to this short-grassed green

CERESWelcome rainbow that never dared disobey Juno the wife of Jupiter who with your orange coloured rays spread honey-drops refreshing showers And with each end of thy blue bow drown my bushy acres and my hilly country which is free from shrubs you thus forming a rich scarf Why has your queen called me here to this place covered with short grass

IRISA contract of true love to celebrateAnd some donation freely to estateOn the blest lovers

IRISI have called you to celebrate a contract of true love and bestow some liberal gift upon the blessed lovers

ORIGINAL TEXT PARAPHRASECERESTell me heavenly bowIf Venus or her son as thou dost knowDo now attend the queen Since they did plotThe means that dusky Dis my daughter gotHer and her blind boys scandaled companyI have forsworn

CERESTell me heavenly bow if Venus the Goddess of love or Cupid her son and pedlar of passion at this time attend the heavenly queen Juno because you are sure to know Since the day they conspired against me and dark Pluto took away my daughter here and Cupidrsquos disgraceful company I have left off

IRISOf her societyBe not afraid I met her deityCutting the clouds towards Pathos and her sonDove-drawn with her Here thought they to have doneSome wanton charm upon this man and miad

IRISBe not afraid of her company I met her deity moving on the clouds towards Paphos the sacred home of Venus on the island of Cyprus along with her son on her chariot drawn by doves Here they contemplated to exercise a charm upon this man and maid producing

Whose vows are that no bed-right shall be paidTill Hymens torch be lightedmdashbut in vainMarss hot minion is returned againHer waspish-headed son has broke his arrowsSwears he will shoot no more but play with sparrowsAnd be a boy right out

wantonness before the actual marriage ceremony but did not succeed Venus has returned her irritable son has broken his arrows and swears that he will give up his practice of trying to inspire love but play with sparrows and be a boy again

[Music is heard] [Music is heard]

CERESHighst queen of stateGreat Juno comes I know her by her gait

CERESHighest queen of state Great Juno there she comes I know here by her gait

[Enter Juno] [Enter Juno]

JUNOHow does my bounteous sister Go with meTo bless this twain that they may Prosperous beAnd honoured in their issue

JUNOHow are you doing my generous sister Come with me to bless this couple so that they may be prosperous and fortunate in their children

[They sing] [They sing]

JUNOHonour riches marriage-blessingLong continuance and increasingHourly joys be still upon youJuno sings her blessings upon you

JUNOMay honour riches happiness in marriage long continuance and increase of those boons ever rest upon you as hourly joys Juno showers down upon you her blessings in song

CERESEarths increase foison plentyBarns and garners never emptyVines and clustring bunches growingPlants and goodly burden bowingSpring come to you at the farthestIn the very end of harvestScarcity and want shall shun youCeresrsquo blessing so is on you

CERESMay you have the plenty of earthrsquos produce Your barns and granaries may never be empty Your vines may grow with clustering bunches Your fruit trees may be heavily laden with their fruit May there be continuous spring and harvest May scantiness and want leave you forever Such is the blessing of Ceres upon you

FERDINANDThis is a most majestic vision andHarmoniously charmingly May I be boldTo think these spirits

FERDINANDThis is a great vision and magically melodious Should I suppose the characters (taking part in the masque) are spirits

PROSPEROSpirits which by mine artI have from their confines calld to enactMy present fancies

PROSPEROYes they are spirits whom I have summoned from the regions to which they are confined to carry into effect my fanciful designs

ORIGINAL TEXT PARAPHRASEFERDINANDLet me live here everSo rare a wondered father and a wifeMakes this place paradise

FERDINANDI should like to live here forever Such a wise and wonderful father makes this place a paradise

[Juno and Ceres whisper and send Iris on employment] [Juno and Ceres whisper and send Iris on employment]

PROSPEROSweet now silence

PROSPEROMy dear Ferdinand speak no more Juno and Ceres are

Juno and Ceres whisper seriouslyTheres something else to do Hush and be muteOr else our spell is marred

whispering with a solemn look There is something else coming Silence Or else our magic will be spoilt

IRISYour nymphs called naiads of the wandering brooksWith your sedged crowns and over-harmless looksLeave your crisp channels and on this green landAnswer your summons Juno does commandCome temperate nymphs and help to celebrateA contract of true love Be not too late

IRISYou nymphs called Naiads denizens (M inhabitants) of the running stream with your chaplets of sedge and ever-helpful looks leave your wrinkled channels and on the green land answer the summons sent to you Juno has ordered some chaste nymphs and help to celebrate a noble and true marriage Donrsquot delay

[Enter certain nymphs] [Enter certain nymphs]You sunburnt sicklemen of August wearyCome hither from the furrow and be merryMake holiday your rye-straw hats put onAnd these fresh nymphs encounter every oneIn country footing

You sunburnt harvesters weary from the effects of the heat in August come here from the furrowed land and rejoice Make holiday with your rye-straw hats upon you and meet these fresh nymphs and join in country dancing

[Enter certain reapers properly habited They join with the nymphs in a graceful dance towards the end whereof Prospero starts suddenly and speaks]

[Enter certain reapers properly habited They join with the nymphs in a graceful dance towards the end whereof Prospero starts suddenly and speaks]

PROSPERO[aside] I had forgot that foul conspiracyOf the beast Caliban and his confederatesAgainst my life The minute of their plotIs almost come [to the spirits]Well done Avoidno more

PROSPERO(Aside)I had forgotten the wicked conspiracy of the beast Caliban and his accomplices against my life the time of their plot has almost arrived ndash (To the Spirits) well done depart no more of this

[To a strange hollow and confused noise the spirits heavily vanish]

[The spirits depart]

ORIGINAL TEXT PARAPHRASEFERDINANDLet me live here everSo rare a wondered father and a wifeMakes this place paradise

FERDINANDI should like to live here forever Such a wise and wonderful father makes this place a paradise

[Juno and Ceres whisper and send Iris on employment] [Juno and Ceres whisper and send Iris on employment]

PROSPEROSweet now silenceJuno and Ceres whisper seriouslyTheres something else to do Hush and be muteOr else our spell is marred

PROSPEROMy dear Ferdinand speak no more Juno and Ceres are whispering with a solemn look There is something else coming Silence Or else our magic will be spoilt

IRISYour nymphs called naiads of the wandering brooksWith your sedged crowns and over-harmless looksLeave your crisp channels and on this green landAnswer your summons Juno does commandCome temperate nymphs and help to celebrateA contract of true love Be not too late

IRISYou nymphs called Naiads denizens (M inhabitants) of the running stream with your chaplets of sedge and ever-helpful looks leave your wrinkled channels and on the green land answer the summons sent to you Juno has ordered some chaste nymphs and help to celebrate a noble and true marriage Donrsquot delay

[Enter certain nymphs] [Enter certain nymphs]You sunburnt sicklemen of August wearyCome hither from the furrow and be merryMake holiday your rye-straw hats put onAnd these fresh nymphs encounter every oneIn country footing

You sunburnt harvesters weary from the effects of the heat in August come here from the furrowed land and rejoice Make holiday with your rye-straw hats upon you and meet these fresh nymphs and join in country dancing

[Enter certain reapers properly habited They join with the nymphs in a graceful dance towards the end whereof Prospero starts suddenly and speaks]

[Enter certain reapers properly habited They join with the nymphs in a graceful dance towards the end whereof Prospero starts suddenly and speaks]

PROSPERO[aside] I had forgot that foul conspiracyOf the beast Caliban and his confederatesAgainst my life The minute of their plotIs almost come [to the spirits]Well done Avoidno more

PROSPERO(Aside)I had forgotten the wicked conspiracy of the beast Caliban and his accomplices against my life the time of their plot has almost arrived ndash (To the Spirits) well done depart no more of this

[To a strange hollow and confused noise the spirits heavily vanish]

[The spirits depart]

Ac-12 27420 topic Revaluation of Assets and Liabilities

REVALUATION OF ASSETS AND LIABILITIES

On admission of a new partner the firm stands reconstituted and consequently the assets are revalued and liabilities are reassessed It is necessary to show the true position of the firm at the time of admission of a new partner If the values of the assets are raised gain will increase the capital of the existing partners Similarly any decrease in the value of assets ie loss will decrease the capital of the existing partners For this purpose alsquoRevaluation Accountrsquo is prepared This account is credited with all increases in the value of assets and decrease in the value of liabilities It is debited with decrease on account of value of assets and increase in the value of liabilities The balance of this account shows a gain or loss on revaluation which is transferred to the existing partnerrsquos capital account in existing profit sharing ratioAccounting for Revaluation of Assets and Liabilities when there is a Changein the Profit Sharing Ratio of Existing PartnersAssets and liabilities of a firm must also be revalued at the time of change in profit sharing ratio of existing partners The reason is that the realisable or actual value of assets and liabilities may be different from those shown in the Balance Sheet It is possible that with the passage of time some of the assets might have appreciated in value while the value of certain other assets might have decreased and no record has been made of such changes in the books of accounts Similarly there may be some unrecorded assets amp libilities that may have to be accounted for Revaluation of assets and reassessments of liabilities becomes necessary because the change in the

value of assets and liabilities belongs to the period to change in profit sharing ratio and hence must be shared by the partners in their old profit sharing ratio Revaluation of assets and reassessment of liabilities may be given effect to in two different ways (a) When revised values are to be recorded in the books and(b) When revised values are not to be recorded in the books

When revised values are to be recorded in the booksIn such a case revaluation of assets and reassessment of liabilities is done with the help of a new account called lsquoRevaluation Accountrsquo Sometimes this account is also called as lsquoProfit amp Loss Adjustment Acrsquo If there is a loss due to revaluation revaluation account is debited and if the revaluation results in a profit the revaluation account is credited The following journal entries made for this purpose are

(i) For increase in the value of assetsAsset Ac Dr (individually)To Revaluation Ac(ii) For decrease in the value of AssetRevaluation Ac Dr (individually)To Asset Ac[Decrease in the value of assets](iii) For increase in the value of LiabilitiesRevaluation Ac Dr (individually)To Liabilities Ac[Increase in the value of Liabilities](iv) For decrease in the value of LiabilitiesLiabilities Ac DrTo Revaluation Ac[Decrease in the value of Liabilities](v) For unrecorded AssetsAsset Ac [unrecorded] DrTo Revaluation Ac[Unrecorded asset recorded at actual value](vi) For unrecorded Liability Revaluation Ac DrTo Liability Ac [unrecorded][Unrecorded Liability recorded at actual value](vii) For transfer of gain on revaluationRevaluation Ac DrTo Existing Partnerrsquos CapitalCurrent Ac[Profit on revaluation transferred to capital account in existing ratio](viii) For transfer of loss on revaluationExisting Partnerrsquos CapitalCurrent Ac DrTo Revaluation Ac[Loss on revaluation transferred to capital account in existing ratio](a) When revaluation account shows gain Revaluation Ac DrTo Partnerrsquos Capital Ac (Old Profit Sharing Ratio)(Profit on revaluation credited to Partnerrsquos Capital Ac)(b) Above entry is reversed when revaluation account shows loss Partners Capital Acs (Old Profit Sharing Ratio) DrTo Revaluation Ac(Loss on revaluation debited to Partnerrsquos Capital Acs)

Proforma of Revaluation Account is given as under

Revaluation Account

Dr Cr Particulars ` Amount Particulars ` Amount To Decrease in value of assets By Increase in value of assets To Increase in value of liabilities By Decrease in value of liabilities To Unrecorded liabilities By Unrecorded assets To Gain on Revaluation (Transferred) By Loss on Revalution (Transferred)

ECO ndash12 2742020Topic- ELASTICITY OF DEMAND

CHAPTER - ELASTICITY OF DEMANDMEANINGDemand for a commodity is affected by many factors such as its price price of related goods income of its buyer tastes and preferences etc Elasticity means degree of response Elasticity of demand means degree of responsiveness of demand Demand for a commodity responds to change in price price of related goods income etc So we have three dimensions of elasticity of demandDIMENSION OF ELASTICITY OF DEMAND TYPES OF ELASTICITY OF DEMAND

Price elasticity of demand Income elasticity of demand Cross Elasticity of demand

Price elasticity of demand Price elasticity of demand means degree of responsiveness of demand for a commodity to the change in its price For example if demand for a commodity rises by 10 due to 5 fall in its price Price elasticity of demand (ep)=Percentage change in quantity demanded Percentage change in price of the commodity = 10 ( -)5 = ( - )2Note that ep will always be negative due to inverse relationship of price and quantity demanded

(ii) Income elasticity of demand Income elasticity of demand refers to the degree of responsiveness of demand for a commodity to the change in income of its buyer Suppose income of buyer rises by 10 and his demand for a commodity rises by 20 then Income elasticity of demand (ey)= change in quantity demanded change in price of the commodity =20 10 = 2

Cross Elasticity of demandCross elasticity of demand means the degree of responsiveness of demand for a commodity to the change in price of its related goods (substitute goods or complementary goods) Suppose demand for a commodity rises by 10 due to 5 rise in price of its substitute good then Cross elasticity of demand (ec) = change in quantity demanded change in price of related good = 10 2 = 5 (Tastes and preferences cannot be expressed numerically So elasticity ofdemand cannot be numerically expressed)

  • Chapter 1 Force (Summary)
  • Distinguish between external sovereignty and internal sovereignty
    • NAND Gate
      • Logic diagram
      • Truth Table
        • NOR Gate
          • Logic diagram
          • Truth Table
            • XOR Gate
              • Logic diagram
              • Truth Table
                • XNOR Gate
                  • Logic diagram
                  • Truth Table
                      • Physics
                      • Chapter 1 Electric Field ( Electric Dipole) (Summary)
Page 38:  · Web viewWe all know that Nouns are divided into two parts: common noun and proper noun.Apart from common and proper noun, we will also study about collective noun and compound

external sovereignty and internal sovereigntySovereignty is the principle

of supreme and

unquestionable authority

reflected in the claim by the

state to be the sole author of

laws within its territory

Definition of external vs internal sovereigntyInternal sovereignty refers to

the relationship between a

sovereign power and its

subjects ndash it refers to the

location of the supreme

authority within the state In

the UK for example internal

sovereignty (supposedly)

resides within Parliament

reflected in the

constitutional principle of

parliamentary

sovereigntyBy contrast

external sovereignty refers

to the capacity of the state

to act independently and

autonomously on the world

stage This is what is

sometimes called lsquostate

sovereigntyrsquo or lsquonational

sovereigntyrsquo and implies

that states are legally equal

and that the territorial

integrity and political

independence of a state is

inviolable

Class ndash XII

Date - 2742020 STUDY MATERIALSubject Topic Summary Execution Business Studies

Job Analysis amp Manpower Planning

At first let us recall the chapter what we have discussed till nowbullJob analysisbullJob specification bullJob description bullJob enlargement bullJob enrichment

Today we will do some questions answers from the chapter

Questions 1ldquoJob analysis job description and job specification are interrelatedrdquo Comment Answer) Job analysis is a systematic and detailed examination of a job to collect all the relevant information about it The contents off the job are summarised in the job description The qualification needed for the job are summarised in job specificationThus there is close interrelationship between job analysis job description and job specification

Question 2ldquoJob enlargement is a horizontal extension of a job whereas job enrichment is a vertical extension of a jobrdquo ElucidateAnswer) Job enlargement involves adding one or more task to a job coma where as job enrichment involves adding more autonomy and responsibility to a job Job enlargement is therefore horizontal extension of a job coma whereas job enrichment is a vertical extension of a job

Question 3 )

What is manpower estimation Explain its quantitative and qualitative aspectsAnswer) Manpower estimation is the process by which management determines how an organisation should move from itrsquos current manpower positionto its desired manpower position There are two dimensions of Manpower estimation- quantitative and qualitative

Quantitative aspectThis aspect of Manpower estimation involves estimating the number of employees required in a future time period Workload analysis and workforce analysis are done to estimate the quantity of required manpower

Qualitative aspectThe estimate of the knowledge skills experience etc of required manpower is the qualitative aspect of Manpower estimation The quality of Manpower can be judged on the basis of job analysisand job specification

COMMERCE

CAPITAL-FIXED AND WORKING

Today let us start the class by discussing the sources of finance for different types of business firms

The term lsquocapitalrsquo refers to the investment made in the enterprise for the purpose of earning profits

Requirements of capital and sources of capital for different types of business firms are

1 Capital for sole proprietorship businessA sole proprietor operates at a small scale and thereforerequires a limited amount of capital

2 Capital for partnership firmCapital requirements as well as capital base of a partnership is bigger than that of a sole trader businessThe owned capital is contributed by the partners in an agreed ratio

3 Capital for joint stock companyA joint stock company generally requires large amount of capitalA public company can raise huge capital through issue of shares In addition to share capital it can utilize retained profits

Now let u discuss the meaning of Finance PlanningFinance planning is the process of estimation the financial requirements of an organization specifying the sources of firms and ensuring that enough funds are available at the right time

1 What do you mean by Finance PlanningAnswer Finance planning is the process of estimation the financial requirements of an organization specifying the sources of firms and ensuring that enough funds are available at the right time

2Discuss the role of financial planning of an enterpriseThe role of financial planning are as followsa A sound financial plan helps a business enterprise to avaid the problems of shortage and surplus of fundsbFinancial planning serves as a guide in developing a sound capital structure so as to maximize returns to shareholders c It helps in effective utilization of fundsd It provides policies and procedures for coordinating different functional areas or departments of businesse It enables the management to exercise effective control over the financial activities of an enterprisef It helps the company to prepare for facing business shocks and surprises in future

Mathematics

Continuity and differentiability

Recall Definition of ContinuityLet f(x) be a single valued function of x and x=a be a point in the domain of definition of the function The function is said to be continuous at x=a ifi) f(c) is defined ie f(x) has a definite finite value at x=cii) lim xrarra f(x) exists andiii) lim xrarra f(x) =f(a) In other words f(x) is said to be continuous at x=a if lim xrarra+ f(x)= lim xrarra- f(x) = f(a) Or f(a+0) =f(a-0) =f(a) Or lim hrarr0 f(a+h)= f(a) Algebra of continuous functionsNow we will study some algebra of continuous functions Theorem 1 Suppose f and g be two real functions continuous at a real number c Then(1) f + g is continuous at x = c(2) f ndash g is continuous at x = c(3) f g is continuous at x = c(4) (fg) is continuous at x = c (provided g (c) ne 0)

Example 1 Prove that every rational function is continuousSolution Recall that every rational function f is given byf(x)=[p(x) q(x) ] q(x)ne0where p and q are polynomial functions The domain of f is all real numbers except points at which q is zero Since polynomial functions are continuous f is continuous by (4) of Theorem 1Example 2Discuss the continuity of sine functionSolution To see this we use the following factslim xrarr0 sin x =0Now observe that f (x) = sin x is defined for every real number Let c be a real number Put x = c + h If x rarr c we know that h rarr 0 Therefore lim xrarrc f(x)

= lim xrarrc sin x= lim hrarr0 sin(c+h) =lim hrarr0 [sin c cos h + cos c sin h ]=lim hrarr0 (sin c cos h) + lim hrarr0 (cos c sin h) = sin c +0=sin c = f(c) Thus lim xrarrc f(x) = f(c) and hence f is a constant function Exercise Prove that the function f(x) = x2 +2x is continuous for every real value of x [Hints show that lim xrarra+ f(x) = lim xrarra- f(x) = f(a) ]

Biology Reproduction in Flowering plants We will discuss about megasporoangium

megasporagenesis and female gametophyte

Q4 Describe the structure of megasporangium

Ovule is attached to the placenta by astalk called funicle

Each ovule has one two or three protectivecoverings called integuments

At the tip of integuments a small openingcalled micropyle is organised

Opposite to the micropylar end is the chalaza

Within the integuments a mass of cellsnucellusand inside it embryo sac orfemale gametophyte is present

Q5 Describe a mature embryosacamp its formation

In most of the flowering plants only oneof the 4 megaspores formed as a result ofmegasporogenesis that is functional while theother three degenerate

The

functional megaspore develops into thefemale gametophyte

Formation The nucleus

of the functional megasporedivides mitotically to form two nuclei first andthen two more sequential mitotic nucleardivisions result in the formation of four ampthen eight nucleate stages of embryo sac

Six of the eight nucleus are surrounded bycell walls and organised into cells

The remaining two nuclei called polar nuclei are found below the egg apparatus in the largecentral cell

Three cells consisting of two synergids amp one egg cell present bottom of

embryo sac Three cells

at the chalazal as antipodal cells

Two polar nuclei together present in large central cell

HISTORY

TOWARDS INDEPENDENCE AND PARTITION THE LAST PHASE(1935-1947)SUB TOPIC NATIONAL MOVEMENTS DURING THE SECOND WORLD WAR

Spread of Quit India Movement On 9th August 1942Gandhiji and other Congress leaders were arrested The Congress was declared illegal The news of the arrest of all leaders marked the beginning of a widespread movement of India It was not possible for such a movement to remain peacefulBut the arrest of the all notable congress leaders virtually left the movement in the hands of the mass The movement took the form of violent and militant outbreakBesides congressmen revolutionaries also were very active in the movement The Congress Socialist group also played a prominent role

1 Question Why did the British authority arrest the Congress leaders on 9 th August 1942Answer Congress Working committee adopted the Quit India resolution which was to be ratified at the Bombay AICC meeting in 8th August 1942 They decided to launch a mass struggle on non-violent lines Gandhiji gave a clarion call to all section of the people rdquoKarenge ya Marengerdquo (do or die) Congress leaders gave the call to driving out

the British from IndiaViceroy had taken strong action against the Quit India movement Gandhiji and all the leaders of Congress were arrested

2 Question How did Quit India Movement spread out all over IndiaAnswer The news of the leaders lsquo arrest marked the beginning of a widespread movement to remain peacefulThe movement took form of violent outbreak There were widespread cutting of telephone and Telegraph wires damaging railway lines raising barricades in cities and towns and other forms of violent demonstations

Question Name the leaders of Congress

Socialist group played a prominent part Notable among the Jayprakash Narayan Rammonohar Lohia Aruna Asaf Ali

Political science

Topic-Franchise and Representation

Summary

The election commission

The Election Commission of India is an autonomous constitutional authority responsible for administering Union and State election processes in India The body administers elections to the Lok Sabha Rajya Sabha State Legislative Assemblies in India and the offices of the President and Vice President in the country

Functions of election commission-

India is a sovereign socialist secular democratic republic Democracy runs like a golden thread in the social economic and political fabric woven by the Constitution given by lsquoWe the People of Indiarsquo unto ourselves The concept of democracy as visualised by the Constitution pre-supposes the representation of the people in Parliament and State legislatures by the method of election The Supreme Court has held that democracy is one of the inalienable basic features of the Constitution of India and forms part of its basic structure The Constitution of India adopted a Parliamentary form of government Parliament consists of the President of India and the two Houses mdash Rajya Sabha and Lok Sabha India being a Union of states has separate state legislatures for each state State legislatures consist of the Governor and two Houses mdash Legislative Council and Legislative Assembly mdash in seven states namely Andhra Pradesh Telangana Bihar Jammu amp Kashmir Karnataka Maharashtra and Uttar Pradesh and of the Governor and the state Legislative Assembly in the remaining 22 states Apart from the above two out of the seven Union Territories namely National Capital Territory of Delhi and Puducherry also have their Legislative Assemblies

ExecutionShort notes-Election commissionFunctions of election commission

Homework- Learn

Computer

Science

Computer hardware NAND Gate

A NOT-AND operation is known as NAND operation It has n input (n gt= 2) and one output

Logic diagram

Truth Table

NOR Gate

A NOT-OR operation is known as NOR operation It has n input (n gt= 2) and one output

Logic diagram

Truth Table

XOR Gate

XOR or Ex-OR gate is a special type of gate It can be used in the half

adder full adder and subtractor The exclusive-OR gate is abbreviated as EX-OR gate or sometime as X-OR gate It has n input (n gt= 2) and one output

Logic diagram

Truth Table

XNOR Gate

XNOR gate is a special type of gate It can be used in the half adder full adder and subtractor The exclusive-NOR gate is abbreviated as EX-NOR gate or sometime as X-NOR gate It has n input (n gt= 2) and one output

Logic diagram

Truth Table

Physics

Chapter 1 Electric Field ( Electric Dipole) (Summary)

Here we will derive Expression of electric field at broad side

On position of dipole

Execution

Q With the help of a labelled diagram obtain an expression for the electric field intensity E at any point on the equitorial line ( broad-side on position) of an electric dipole

Ans

E1 E1sinθ

E θ P E1 θ

( r2+L2)12 E2 E

r E2 E2sinθ

-q θ L O L +qA B

Let us consider that the point P is situated on the right bisector of the dipole AB at a distance r meter from its midpoint O

Let E1 and E2 be the electric field intensities of the electric field at P due to charge +q and ndashq of the dipole resp The distance of P from each charge is ( r2+L2)12

So E1 = 14 πϵ q

(r 2+L 2) away from +q

E2 = 14 πϵ q

(r 2+L 2) towards ndashq

The magnitudes of E1 and E2 are equal but directions are different Now resolving E1 and E2 into two components parallel and perpendicular to AB we get

The components perpendicular to AB E1sinθ and E2sinθ cancel each other because they are equal and opposite

The components parallel to AB are E1cosθ and E2 cosθ are in same direction and add up

So resultant intensity of electric field at the point P is

E = E1cosθ + E2 cosθ

E = 14 πϵ q

(r 2+L 2) 2 cosθ

Now from fig we have cosθ =BOBP = L (r2+L2)12

So we get E = 14 πϵ 2qL ( r2+L2)32

Now electric dipole moment p= 2qL

So E = 14 πϵ p ( r2+L2)32

HW Find the expression of Electric field as done here but this time take r gtgt 2L

Also find the expression of torque experience by a dipole

(Hint Electric force experienced by charges of dipole in electric field is qE each Let θ be the angle which dipole makes with electric lines of force then perpendicular distance between two charges is 2Lsinθ Then torque = force x perp distance = qE x 2L sinθ So τ=pE sinθ where p =2qL )

STUDY MATERIAL

Class XIISubject Eng Literature (The Tempest ndash William Shakespeare) Topic Act IV Scene 1 Lines 84 to 133 (Iris hellip A contract of true love Be not too late ) Date 27th April 2020 (4th Period)

[Students should read the original play and also the paraphrase given in the school prescribed textbook]Summary Questions amp Answers

o Ceres soon appears and comes to know that she has been summoned to celebrate the contract of true love

o Ceres expresses her unwillingness to meet Venus and Cupid as she has shunned their company

o Ceres and Juno both bestow their blessings upon Ferdinand and Miranda with June gifting honour riches happiness in marriage and Ceres presents plenty of earthrsquos produce

o Iris summons the water-nymphs and reapers to come and celebrate a contract

(1) IRIS Of her society (Line 91-101)

Be not afraid I met her deity

Cutting the clouds towards Pathos and her sonDove-drawn with her Here thought they to have doneSome wanton charm upon this man and maidWhose vows are that no bed-right shall be paidTill Hymens torch be lightedmdashbut in vainMarss hot minion is returned againHer waspish-headed son has broke his arrowsSwears he will shoot no more but play with sparrowsAnd be a boy right out

(i) Where were Venus and Cupid seen flying How were they travelling Why did they want to join the marriage celebration of Ferdinand and Miranda

of true love

Venus and Cupid were seen flying through the air towards Paphos the famous city which is situated on the island of Cyprus They were travelling by air-borne chariot drawn by doves They certainly wanted to come here in order to play some amorous trick upon Ferdinand and Miranda who are under a vow not to gratify their physical desires till the holy ceremony of their marriage has been performed(ii) What have Venus and Cupid done after failing in their plan

After being failure of their plan Venus who is a very passionate deity and who is the mistress of Mars (the god of war) has gone back while here ill-tempered son Cupid has broken his arrows of love in his state of desperation(iii) What has Cupid firmly decided

Cupid is feeling so disappointed that he has firmly decided to shoot no more arrows to arouse love in human hearts but to spend his time playing with sparrows Thus he would now become just a boy and would give up his original function of shooting arrows on human beings to make them fall in love(iv) What vow had Ceres taken How did Ceres feel at the abduction

After the abduction of her daughter Prosperina by Pluto Ceres had taken a vow to always keep away from the disgraceful company of Venus and her blind son Cupid the god of love Ceres felt deeply distressed when Pluto had carried off her daughter and had made her his wife by force(v) Why has Ceres not forgiven Venus and her blind son For what do Ceres want to be sure

As the abduction had been manipulated by Venus the goddess of beauty and love and her blind son Cupid Ceres has never forgiven them for their part in the whole plot Ceres wants to be sure that she would not have to meet Venus and Cupid who had engineered the abduction of her daughter Prosperina

AS THIS lsquoMASQUErsquo SCENE IS VERY IMPORTANT IN THE PLAY THE PARAPHRASE OF THE ENTIRE PORTION OF MASQUE SCENE (Act IV Lines 58 to 143) IS GIVEN BELOW

IRIS Goddess of RainbowCERES Goddess of Agriculture and all the fruits of the earth

(Nature growth prosperity rebirth ndash notions intimately connected to marriage)JUNO The majestic Queen of Heavens and wife of Jupiter (Jupiter is the king of Gods)

VENUS The Goddess of love CUPID Son of Venus PLUTO God of death (In the play referred by Shakespeare as lsquoDisrsquo which is a Roman name for Pluto)

ORIGINAL TEXT PARAPHRASEPROSPEROWellmdash

PROSPERONow come Ariel Let there be too many rather than too few

Now come my Ariel Bring a corollaryRather than want a spirit Appear and pertly[to Ferdinand and Miranda]No tongue all eyes Be silent

spirits in attendance Appear briskly

[to Ferdinand and Miranda]Look with your eyes but do not say a word

[Soft music] [Soft music][Enter Iris] [Enter Iris]

IRISCeres most bounteous lady thy rich leasOf wheat rye barley vetches oats and peasThy turfy mountains where live nibbling sheepAnd flat meads thatched with stover them to keepThy banks with pioned and twilled brimsWhich spongy April at thy hest betrimsTo make cold nymphs chaste crowns and thybroom-grovesWhose shadow the dismissegraved bachelor lovesBeing lass-lorn thy pole clipped vineyardAnd thy sea-marge sterile and rocky-hardWhere thou thyself dost airmdashthe Queen othrsquoSkyWhose watery arch and messenger am IBids thee leave these and with her sovereign grace[Juno appears] Here on this grass-plot in this very placeTo come and sport Her peacocks fly amainApproach rich Ceres her to entertain

IRISCeres most generous lady you are the cause of rich fields or fertile land where wheat rye barley beans oats and peas grow the grassy mountains where the sheep graze and the flat meadows covered with coarse hay to be used as fodder for cattleYour banks are covered with marsh-marigolds and reeds and the rainy April under your orders brings forth to make for the maids who are not in love beautiful crowns your woods where the broom flourishes and where the bachelor who has been dismissed by the maid he loved lies down being forsaken your vineyard in which the poles are embraced by the vines and the margin of the sea which is barren and rocky where you roam about to enjoy the fresh air ndash the queen of the sky (Juno) whose messenger I am besides being represented as the rainbow bids you leave all these and with her majesty here on this grassy plot in this very place come and sport her peacocks carry her fast in her chariot through the air and are making their way here approach rich Ceres to welcome her

[Enter Ariel as Ceres] [Enter Ariel as Ceres]

CERESHail many-coloured messenger that neerDost disobey the wife of JupiterWho with thy saffron wings upon my flowersDiffusest honey-drops refreshing showersAnd with each end of thy blue bow dost crownMy bosky acres and my unshrubbed downRich scarf to my proud earth Why hath thy queenSummoned me hither to this short-grassed green

CERESWelcome rainbow that never dared disobey Juno the wife of Jupiter who with your orange coloured rays spread honey-drops refreshing showers And with each end of thy blue bow drown my bushy acres and my hilly country which is free from shrubs you thus forming a rich scarf Why has your queen called me here to this place covered with short grass

IRISA contract of true love to celebrateAnd some donation freely to estateOn the blest lovers

IRISI have called you to celebrate a contract of true love and bestow some liberal gift upon the blessed lovers

ORIGINAL TEXT PARAPHRASECERESTell me heavenly bowIf Venus or her son as thou dost knowDo now attend the queen Since they did plotThe means that dusky Dis my daughter gotHer and her blind boys scandaled companyI have forsworn

CERESTell me heavenly bow if Venus the Goddess of love or Cupid her son and pedlar of passion at this time attend the heavenly queen Juno because you are sure to know Since the day they conspired against me and dark Pluto took away my daughter here and Cupidrsquos disgraceful company I have left off

IRISOf her societyBe not afraid I met her deityCutting the clouds towards Pathos and her sonDove-drawn with her Here thought they to have doneSome wanton charm upon this man and miad

IRISBe not afraid of her company I met her deity moving on the clouds towards Paphos the sacred home of Venus on the island of Cyprus along with her son on her chariot drawn by doves Here they contemplated to exercise a charm upon this man and maid producing

Whose vows are that no bed-right shall be paidTill Hymens torch be lightedmdashbut in vainMarss hot minion is returned againHer waspish-headed son has broke his arrowsSwears he will shoot no more but play with sparrowsAnd be a boy right out

wantonness before the actual marriage ceremony but did not succeed Venus has returned her irritable son has broken his arrows and swears that he will give up his practice of trying to inspire love but play with sparrows and be a boy again

[Music is heard] [Music is heard]

CERESHighst queen of stateGreat Juno comes I know her by her gait

CERESHighest queen of state Great Juno there she comes I know here by her gait

[Enter Juno] [Enter Juno]

JUNOHow does my bounteous sister Go with meTo bless this twain that they may Prosperous beAnd honoured in their issue

JUNOHow are you doing my generous sister Come with me to bless this couple so that they may be prosperous and fortunate in their children

[They sing] [They sing]

JUNOHonour riches marriage-blessingLong continuance and increasingHourly joys be still upon youJuno sings her blessings upon you

JUNOMay honour riches happiness in marriage long continuance and increase of those boons ever rest upon you as hourly joys Juno showers down upon you her blessings in song

CERESEarths increase foison plentyBarns and garners never emptyVines and clustring bunches growingPlants and goodly burden bowingSpring come to you at the farthestIn the very end of harvestScarcity and want shall shun youCeresrsquo blessing so is on you

CERESMay you have the plenty of earthrsquos produce Your barns and granaries may never be empty Your vines may grow with clustering bunches Your fruit trees may be heavily laden with their fruit May there be continuous spring and harvest May scantiness and want leave you forever Such is the blessing of Ceres upon you

FERDINANDThis is a most majestic vision andHarmoniously charmingly May I be boldTo think these spirits

FERDINANDThis is a great vision and magically melodious Should I suppose the characters (taking part in the masque) are spirits

PROSPEROSpirits which by mine artI have from their confines calld to enactMy present fancies

PROSPEROYes they are spirits whom I have summoned from the regions to which they are confined to carry into effect my fanciful designs

ORIGINAL TEXT PARAPHRASEFERDINANDLet me live here everSo rare a wondered father and a wifeMakes this place paradise

FERDINANDI should like to live here forever Such a wise and wonderful father makes this place a paradise

[Juno and Ceres whisper and send Iris on employment] [Juno and Ceres whisper and send Iris on employment]

PROSPEROSweet now silence

PROSPEROMy dear Ferdinand speak no more Juno and Ceres are

Juno and Ceres whisper seriouslyTheres something else to do Hush and be muteOr else our spell is marred

whispering with a solemn look There is something else coming Silence Or else our magic will be spoilt

IRISYour nymphs called naiads of the wandering brooksWith your sedged crowns and over-harmless looksLeave your crisp channels and on this green landAnswer your summons Juno does commandCome temperate nymphs and help to celebrateA contract of true love Be not too late

IRISYou nymphs called Naiads denizens (M inhabitants) of the running stream with your chaplets of sedge and ever-helpful looks leave your wrinkled channels and on the green land answer the summons sent to you Juno has ordered some chaste nymphs and help to celebrate a noble and true marriage Donrsquot delay

[Enter certain nymphs] [Enter certain nymphs]You sunburnt sicklemen of August wearyCome hither from the furrow and be merryMake holiday your rye-straw hats put onAnd these fresh nymphs encounter every oneIn country footing

You sunburnt harvesters weary from the effects of the heat in August come here from the furrowed land and rejoice Make holiday with your rye-straw hats upon you and meet these fresh nymphs and join in country dancing

[Enter certain reapers properly habited They join with the nymphs in a graceful dance towards the end whereof Prospero starts suddenly and speaks]

[Enter certain reapers properly habited They join with the nymphs in a graceful dance towards the end whereof Prospero starts suddenly and speaks]

PROSPERO[aside] I had forgot that foul conspiracyOf the beast Caliban and his confederatesAgainst my life The minute of their plotIs almost come [to the spirits]Well done Avoidno more

PROSPERO(Aside)I had forgotten the wicked conspiracy of the beast Caliban and his accomplices against my life the time of their plot has almost arrived ndash (To the Spirits) well done depart no more of this

[To a strange hollow and confused noise the spirits heavily vanish]

[The spirits depart]

ORIGINAL TEXT PARAPHRASEFERDINANDLet me live here everSo rare a wondered father and a wifeMakes this place paradise

FERDINANDI should like to live here forever Such a wise and wonderful father makes this place a paradise

[Juno and Ceres whisper and send Iris on employment] [Juno and Ceres whisper and send Iris on employment]

PROSPEROSweet now silenceJuno and Ceres whisper seriouslyTheres something else to do Hush and be muteOr else our spell is marred

PROSPEROMy dear Ferdinand speak no more Juno and Ceres are whispering with a solemn look There is something else coming Silence Or else our magic will be spoilt

IRISYour nymphs called naiads of the wandering brooksWith your sedged crowns and over-harmless looksLeave your crisp channels and on this green landAnswer your summons Juno does commandCome temperate nymphs and help to celebrateA contract of true love Be not too late

IRISYou nymphs called Naiads denizens (M inhabitants) of the running stream with your chaplets of sedge and ever-helpful looks leave your wrinkled channels and on the green land answer the summons sent to you Juno has ordered some chaste nymphs and help to celebrate a noble and true marriage Donrsquot delay

[Enter certain nymphs] [Enter certain nymphs]You sunburnt sicklemen of August wearyCome hither from the furrow and be merryMake holiday your rye-straw hats put onAnd these fresh nymphs encounter every oneIn country footing

You sunburnt harvesters weary from the effects of the heat in August come here from the furrowed land and rejoice Make holiday with your rye-straw hats upon you and meet these fresh nymphs and join in country dancing

[Enter certain reapers properly habited They join with the nymphs in a graceful dance towards the end whereof Prospero starts suddenly and speaks]

[Enter certain reapers properly habited They join with the nymphs in a graceful dance towards the end whereof Prospero starts suddenly and speaks]

PROSPERO[aside] I had forgot that foul conspiracyOf the beast Caliban and his confederatesAgainst my life The minute of their plotIs almost come [to the spirits]Well done Avoidno more

PROSPERO(Aside)I had forgotten the wicked conspiracy of the beast Caliban and his accomplices against my life the time of their plot has almost arrived ndash (To the Spirits) well done depart no more of this

[To a strange hollow and confused noise the spirits heavily vanish]

[The spirits depart]

Ac-12 27420 topic Revaluation of Assets and Liabilities

REVALUATION OF ASSETS AND LIABILITIES

On admission of a new partner the firm stands reconstituted and consequently the assets are revalued and liabilities are reassessed It is necessary to show the true position of the firm at the time of admission of a new partner If the values of the assets are raised gain will increase the capital of the existing partners Similarly any decrease in the value of assets ie loss will decrease the capital of the existing partners For this purpose alsquoRevaluation Accountrsquo is prepared This account is credited with all increases in the value of assets and decrease in the value of liabilities It is debited with decrease on account of value of assets and increase in the value of liabilities The balance of this account shows a gain or loss on revaluation which is transferred to the existing partnerrsquos capital account in existing profit sharing ratioAccounting for Revaluation of Assets and Liabilities when there is a Changein the Profit Sharing Ratio of Existing PartnersAssets and liabilities of a firm must also be revalued at the time of change in profit sharing ratio of existing partners The reason is that the realisable or actual value of assets and liabilities may be different from those shown in the Balance Sheet It is possible that with the passage of time some of the assets might have appreciated in value while the value of certain other assets might have decreased and no record has been made of such changes in the books of accounts Similarly there may be some unrecorded assets amp libilities that may have to be accounted for Revaluation of assets and reassessments of liabilities becomes necessary because the change in the

value of assets and liabilities belongs to the period to change in profit sharing ratio and hence must be shared by the partners in their old profit sharing ratio Revaluation of assets and reassessment of liabilities may be given effect to in two different ways (a) When revised values are to be recorded in the books and(b) When revised values are not to be recorded in the books

When revised values are to be recorded in the booksIn such a case revaluation of assets and reassessment of liabilities is done with the help of a new account called lsquoRevaluation Accountrsquo Sometimes this account is also called as lsquoProfit amp Loss Adjustment Acrsquo If there is a loss due to revaluation revaluation account is debited and if the revaluation results in a profit the revaluation account is credited The following journal entries made for this purpose are

(i) For increase in the value of assetsAsset Ac Dr (individually)To Revaluation Ac(ii) For decrease in the value of AssetRevaluation Ac Dr (individually)To Asset Ac[Decrease in the value of assets](iii) For increase in the value of LiabilitiesRevaluation Ac Dr (individually)To Liabilities Ac[Increase in the value of Liabilities](iv) For decrease in the value of LiabilitiesLiabilities Ac DrTo Revaluation Ac[Decrease in the value of Liabilities](v) For unrecorded AssetsAsset Ac [unrecorded] DrTo Revaluation Ac[Unrecorded asset recorded at actual value](vi) For unrecorded Liability Revaluation Ac DrTo Liability Ac [unrecorded][Unrecorded Liability recorded at actual value](vii) For transfer of gain on revaluationRevaluation Ac DrTo Existing Partnerrsquos CapitalCurrent Ac[Profit on revaluation transferred to capital account in existing ratio](viii) For transfer of loss on revaluationExisting Partnerrsquos CapitalCurrent Ac DrTo Revaluation Ac[Loss on revaluation transferred to capital account in existing ratio](a) When revaluation account shows gain Revaluation Ac DrTo Partnerrsquos Capital Ac (Old Profit Sharing Ratio)(Profit on revaluation credited to Partnerrsquos Capital Ac)(b) Above entry is reversed when revaluation account shows loss Partners Capital Acs (Old Profit Sharing Ratio) DrTo Revaluation Ac(Loss on revaluation debited to Partnerrsquos Capital Acs)

Proforma of Revaluation Account is given as under

Revaluation Account

Dr Cr Particulars ` Amount Particulars ` Amount To Decrease in value of assets By Increase in value of assets To Increase in value of liabilities By Decrease in value of liabilities To Unrecorded liabilities By Unrecorded assets To Gain on Revaluation (Transferred) By Loss on Revalution (Transferred)

ECO ndash12 2742020Topic- ELASTICITY OF DEMAND

CHAPTER - ELASTICITY OF DEMANDMEANINGDemand for a commodity is affected by many factors such as its price price of related goods income of its buyer tastes and preferences etc Elasticity means degree of response Elasticity of demand means degree of responsiveness of demand Demand for a commodity responds to change in price price of related goods income etc So we have three dimensions of elasticity of demandDIMENSION OF ELASTICITY OF DEMAND TYPES OF ELASTICITY OF DEMAND

Price elasticity of demand Income elasticity of demand Cross Elasticity of demand

Price elasticity of demand Price elasticity of demand means degree of responsiveness of demand for a commodity to the change in its price For example if demand for a commodity rises by 10 due to 5 fall in its price Price elasticity of demand (ep)=Percentage change in quantity demanded Percentage change in price of the commodity = 10 ( -)5 = ( - )2Note that ep will always be negative due to inverse relationship of price and quantity demanded

(ii) Income elasticity of demand Income elasticity of demand refers to the degree of responsiveness of demand for a commodity to the change in income of its buyer Suppose income of buyer rises by 10 and his demand for a commodity rises by 20 then Income elasticity of demand (ey)= change in quantity demanded change in price of the commodity =20 10 = 2

Cross Elasticity of demandCross elasticity of demand means the degree of responsiveness of demand for a commodity to the change in price of its related goods (substitute goods or complementary goods) Suppose demand for a commodity rises by 10 due to 5 rise in price of its substitute good then Cross elasticity of demand (ec) = change in quantity demanded change in price of related good = 10 2 = 5 (Tastes and preferences cannot be expressed numerically So elasticity ofdemand cannot be numerically expressed)

  • Chapter 1 Force (Summary)
  • Distinguish between external sovereignty and internal sovereignty
    • NAND Gate
      • Logic diagram
      • Truth Table
        • NOR Gate
          • Logic diagram
          • Truth Table
            • XOR Gate
              • Logic diagram
              • Truth Table
                • XNOR Gate
                  • Logic diagram
                  • Truth Table
                      • Physics
                      • Chapter 1 Electric Field ( Electric Dipole) (Summary)
Page 39:  · Web viewWe all know that Nouns are divided into two parts: common noun and proper noun.Apart from common and proper noun, we will also study about collective noun and compound

Date - 2742020 STUDY MATERIALSubject Topic Summary Execution Business Studies

Job Analysis amp Manpower Planning

At first let us recall the chapter what we have discussed till nowbullJob analysisbullJob specification bullJob description bullJob enlargement bullJob enrichment

Today we will do some questions answers from the chapter

Questions 1ldquoJob analysis job description and job specification are interrelatedrdquo Comment Answer) Job analysis is a systematic and detailed examination of a job to collect all the relevant information about it The contents off the job are summarised in the job description The qualification needed for the job are summarised in job specificationThus there is close interrelationship between job analysis job description and job specification

Question 2ldquoJob enlargement is a horizontal extension of a job whereas job enrichment is a vertical extension of a jobrdquo ElucidateAnswer) Job enlargement involves adding one or more task to a job coma where as job enrichment involves adding more autonomy and responsibility to a job Job enlargement is therefore horizontal extension of a job coma whereas job enrichment is a vertical extension of a job

Question 3 )

What is manpower estimation Explain its quantitative and qualitative aspectsAnswer) Manpower estimation is the process by which management determines how an organisation should move from itrsquos current manpower positionto its desired manpower position There are two dimensions of Manpower estimation- quantitative and qualitative

Quantitative aspectThis aspect of Manpower estimation involves estimating the number of employees required in a future time period Workload analysis and workforce analysis are done to estimate the quantity of required manpower

Qualitative aspectThe estimate of the knowledge skills experience etc of required manpower is the qualitative aspect of Manpower estimation The quality of Manpower can be judged on the basis of job analysisand job specification

COMMERCE

CAPITAL-FIXED AND WORKING

Today let us start the class by discussing the sources of finance for different types of business firms

The term lsquocapitalrsquo refers to the investment made in the enterprise for the purpose of earning profits

Requirements of capital and sources of capital for different types of business firms are

1 Capital for sole proprietorship businessA sole proprietor operates at a small scale and thereforerequires a limited amount of capital

2 Capital for partnership firmCapital requirements as well as capital base of a partnership is bigger than that of a sole trader businessThe owned capital is contributed by the partners in an agreed ratio

3 Capital for joint stock companyA joint stock company generally requires large amount of capitalA public company can raise huge capital through issue of shares In addition to share capital it can utilize retained profits

Now let u discuss the meaning of Finance PlanningFinance planning is the process of estimation the financial requirements of an organization specifying the sources of firms and ensuring that enough funds are available at the right time

1 What do you mean by Finance PlanningAnswer Finance planning is the process of estimation the financial requirements of an organization specifying the sources of firms and ensuring that enough funds are available at the right time

2Discuss the role of financial planning of an enterpriseThe role of financial planning are as followsa A sound financial plan helps a business enterprise to avaid the problems of shortage and surplus of fundsbFinancial planning serves as a guide in developing a sound capital structure so as to maximize returns to shareholders c It helps in effective utilization of fundsd It provides policies and procedures for coordinating different functional areas or departments of businesse It enables the management to exercise effective control over the financial activities of an enterprisef It helps the company to prepare for facing business shocks and surprises in future

Mathematics

Continuity and differentiability

Recall Definition of ContinuityLet f(x) be a single valued function of x and x=a be a point in the domain of definition of the function The function is said to be continuous at x=a ifi) f(c) is defined ie f(x) has a definite finite value at x=cii) lim xrarra f(x) exists andiii) lim xrarra f(x) =f(a) In other words f(x) is said to be continuous at x=a if lim xrarra+ f(x)= lim xrarra- f(x) = f(a) Or f(a+0) =f(a-0) =f(a) Or lim hrarr0 f(a+h)= f(a) Algebra of continuous functionsNow we will study some algebra of continuous functions Theorem 1 Suppose f and g be two real functions continuous at a real number c Then(1) f + g is continuous at x = c(2) f ndash g is continuous at x = c(3) f g is continuous at x = c(4) (fg) is continuous at x = c (provided g (c) ne 0)

Example 1 Prove that every rational function is continuousSolution Recall that every rational function f is given byf(x)=[p(x) q(x) ] q(x)ne0where p and q are polynomial functions The domain of f is all real numbers except points at which q is zero Since polynomial functions are continuous f is continuous by (4) of Theorem 1Example 2Discuss the continuity of sine functionSolution To see this we use the following factslim xrarr0 sin x =0Now observe that f (x) = sin x is defined for every real number Let c be a real number Put x = c + h If x rarr c we know that h rarr 0 Therefore lim xrarrc f(x)

= lim xrarrc sin x= lim hrarr0 sin(c+h) =lim hrarr0 [sin c cos h + cos c sin h ]=lim hrarr0 (sin c cos h) + lim hrarr0 (cos c sin h) = sin c +0=sin c = f(c) Thus lim xrarrc f(x) = f(c) and hence f is a constant function Exercise Prove that the function f(x) = x2 +2x is continuous for every real value of x [Hints show that lim xrarra+ f(x) = lim xrarra- f(x) = f(a) ]

Biology Reproduction in Flowering plants We will discuss about megasporoangium

megasporagenesis and female gametophyte

Q4 Describe the structure of megasporangium

Ovule is attached to the placenta by astalk called funicle

Each ovule has one two or three protectivecoverings called integuments

At the tip of integuments a small openingcalled micropyle is organised

Opposite to the micropylar end is the chalaza

Within the integuments a mass of cellsnucellusand inside it embryo sac orfemale gametophyte is present

Q5 Describe a mature embryosacamp its formation

In most of the flowering plants only oneof the 4 megaspores formed as a result ofmegasporogenesis that is functional while theother three degenerate

The

functional megaspore develops into thefemale gametophyte

Formation The nucleus

of the functional megasporedivides mitotically to form two nuclei first andthen two more sequential mitotic nucleardivisions result in the formation of four ampthen eight nucleate stages of embryo sac

Six of the eight nucleus are surrounded bycell walls and organised into cells

The remaining two nuclei called polar nuclei are found below the egg apparatus in the largecentral cell

Three cells consisting of two synergids amp one egg cell present bottom of

embryo sac Three cells

at the chalazal as antipodal cells

Two polar nuclei together present in large central cell

HISTORY

TOWARDS INDEPENDENCE AND PARTITION THE LAST PHASE(1935-1947)SUB TOPIC NATIONAL MOVEMENTS DURING THE SECOND WORLD WAR

Spread of Quit India Movement On 9th August 1942Gandhiji and other Congress leaders were arrested The Congress was declared illegal The news of the arrest of all leaders marked the beginning of a widespread movement of India It was not possible for such a movement to remain peacefulBut the arrest of the all notable congress leaders virtually left the movement in the hands of the mass The movement took the form of violent and militant outbreakBesides congressmen revolutionaries also were very active in the movement The Congress Socialist group also played a prominent role

1 Question Why did the British authority arrest the Congress leaders on 9 th August 1942Answer Congress Working committee adopted the Quit India resolution which was to be ratified at the Bombay AICC meeting in 8th August 1942 They decided to launch a mass struggle on non-violent lines Gandhiji gave a clarion call to all section of the people rdquoKarenge ya Marengerdquo (do or die) Congress leaders gave the call to driving out

the British from IndiaViceroy had taken strong action against the Quit India movement Gandhiji and all the leaders of Congress were arrested

2 Question How did Quit India Movement spread out all over IndiaAnswer The news of the leaders lsquo arrest marked the beginning of a widespread movement to remain peacefulThe movement took form of violent outbreak There were widespread cutting of telephone and Telegraph wires damaging railway lines raising barricades in cities and towns and other forms of violent demonstations

Question Name the leaders of Congress

Socialist group played a prominent part Notable among the Jayprakash Narayan Rammonohar Lohia Aruna Asaf Ali

Political science

Topic-Franchise and Representation

Summary

The election commission

The Election Commission of India is an autonomous constitutional authority responsible for administering Union and State election processes in India The body administers elections to the Lok Sabha Rajya Sabha State Legislative Assemblies in India and the offices of the President and Vice President in the country

Functions of election commission-

India is a sovereign socialist secular democratic republic Democracy runs like a golden thread in the social economic and political fabric woven by the Constitution given by lsquoWe the People of Indiarsquo unto ourselves The concept of democracy as visualised by the Constitution pre-supposes the representation of the people in Parliament and State legislatures by the method of election The Supreme Court has held that democracy is one of the inalienable basic features of the Constitution of India and forms part of its basic structure The Constitution of India adopted a Parliamentary form of government Parliament consists of the President of India and the two Houses mdash Rajya Sabha and Lok Sabha India being a Union of states has separate state legislatures for each state State legislatures consist of the Governor and two Houses mdash Legislative Council and Legislative Assembly mdash in seven states namely Andhra Pradesh Telangana Bihar Jammu amp Kashmir Karnataka Maharashtra and Uttar Pradesh and of the Governor and the state Legislative Assembly in the remaining 22 states Apart from the above two out of the seven Union Territories namely National Capital Territory of Delhi and Puducherry also have their Legislative Assemblies

ExecutionShort notes-Election commissionFunctions of election commission

Homework- Learn

Computer

Science

Computer hardware NAND Gate

A NOT-AND operation is known as NAND operation It has n input (n gt= 2) and one output

Logic diagram

Truth Table

NOR Gate

A NOT-OR operation is known as NOR operation It has n input (n gt= 2) and one output

Logic diagram

Truth Table

XOR Gate

XOR or Ex-OR gate is a special type of gate It can be used in the half

adder full adder and subtractor The exclusive-OR gate is abbreviated as EX-OR gate or sometime as X-OR gate It has n input (n gt= 2) and one output

Logic diagram

Truth Table

XNOR Gate

XNOR gate is a special type of gate It can be used in the half adder full adder and subtractor The exclusive-NOR gate is abbreviated as EX-NOR gate or sometime as X-NOR gate It has n input (n gt= 2) and one output

Logic diagram

Truth Table

Physics

Chapter 1 Electric Field ( Electric Dipole) (Summary)

Here we will derive Expression of electric field at broad side

On position of dipole

Execution

Q With the help of a labelled diagram obtain an expression for the electric field intensity E at any point on the equitorial line ( broad-side on position) of an electric dipole

Ans

E1 E1sinθ

E θ P E1 θ

( r2+L2)12 E2 E

r E2 E2sinθ

-q θ L O L +qA B

Let us consider that the point P is situated on the right bisector of the dipole AB at a distance r meter from its midpoint O

Let E1 and E2 be the electric field intensities of the electric field at P due to charge +q and ndashq of the dipole resp The distance of P from each charge is ( r2+L2)12

So E1 = 14 πϵ q

(r 2+L 2) away from +q

E2 = 14 πϵ q

(r 2+L 2) towards ndashq

The magnitudes of E1 and E2 are equal but directions are different Now resolving E1 and E2 into two components parallel and perpendicular to AB we get

The components perpendicular to AB E1sinθ and E2sinθ cancel each other because they are equal and opposite

The components parallel to AB are E1cosθ and E2 cosθ are in same direction and add up

So resultant intensity of electric field at the point P is

E = E1cosθ + E2 cosθ

E = 14 πϵ q

(r 2+L 2) 2 cosθ

Now from fig we have cosθ =BOBP = L (r2+L2)12

So we get E = 14 πϵ 2qL ( r2+L2)32

Now electric dipole moment p= 2qL

So E = 14 πϵ p ( r2+L2)32

HW Find the expression of Electric field as done here but this time take r gtgt 2L

Also find the expression of torque experience by a dipole

(Hint Electric force experienced by charges of dipole in electric field is qE each Let θ be the angle which dipole makes with electric lines of force then perpendicular distance between two charges is 2Lsinθ Then torque = force x perp distance = qE x 2L sinθ So τ=pE sinθ where p =2qL )

STUDY MATERIAL

Class XIISubject Eng Literature (The Tempest ndash William Shakespeare) Topic Act IV Scene 1 Lines 84 to 133 (Iris hellip A contract of true love Be not too late ) Date 27th April 2020 (4th Period)

[Students should read the original play and also the paraphrase given in the school prescribed textbook]Summary Questions amp Answers

o Ceres soon appears and comes to know that she has been summoned to celebrate the contract of true love

o Ceres expresses her unwillingness to meet Venus and Cupid as she has shunned their company

o Ceres and Juno both bestow their blessings upon Ferdinand and Miranda with June gifting honour riches happiness in marriage and Ceres presents plenty of earthrsquos produce

o Iris summons the water-nymphs and reapers to come and celebrate a contract

(1) IRIS Of her society (Line 91-101)

Be not afraid I met her deity

Cutting the clouds towards Pathos and her sonDove-drawn with her Here thought they to have doneSome wanton charm upon this man and maidWhose vows are that no bed-right shall be paidTill Hymens torch be lightedmdashbut in vainMarss hot minion is returned againHer waspish-headed son has broke his arrowsSwears he will shoot no more but play with sparrowsAnd be a boy right out

(i) Where were Venus and Cupid seen flying How were they travelling Why did they want to join the marriage celebration of Ferdinand and Miranda

of true love

Venus and Cupid were seen flying through the air towards Paphos the famous city which is situated on the island of Cyprus They were travelling by air-borne chariot drawn by doves They certainly wanted to come here in order to play some amorous trick upon Ferdinand and Miranda who are under a vow not to gratify their physical desires till the holy ceremony of their marriage has been performed(ii) What have Venus and Cupid done after failing in their plan

After being failure of their plan Venus who is a very passionate deity and who is the mistress of Mars (the god of war) has gone back while here ill-tempered son Cupid has broken his arrows of love in his state of desperation(iii) What has Cupid firmly decided

Cupid is feeling so disappointed that he has firmly decided to shoot no more arrows to arouse love in human hearts but to spend his time playing with sparrows Thus he would now become just a boy and would give up his original function of shooting arrows on human beings to make them fall in love(iv) What vow had Ceres taken How did Ceres feel at the abduction

After the abduction of her daughter Prosperina by Pluto Ceres had taken a vow to always keep away from the disgraceful company of Venus and her blind son Cupid the god of love Ceres felt deeply distressed when Pluto had carried off her daughter and had made her his wife by force(v) Why has Ceres not forgiven Venus and her blind son For what do Ceres want to be sure

As the abduction had been manipulated by Venus the goddess of beauty and love and her blind son Cupid Ceres has never forgiven them for their part in the whole plot Ceres wants to be sure that she would not have to meet Venus and Cupid who had engineered the abduction of her daughter Prosperina

AS THIS lsquoMASQUErsquo SCENE IS VERY IMPORTANT IN THE PLAY THE PARAPHRASE OF THE ENTIRE PORTION OF MASQUE SCENE (Act IV Lines 58 to 143) IS GIVEN BELOW

IRIS Goddess of RainbowCERES Goddess of Agriculture and all the fruits of the earth

(Nature growth prosperity rebirth ndash notions intimately connected to marriage)JUNO The majestic Queen of Heavens and wife of Jupiter (Jupiter is the king of Gods)

VENUS The Goddess of love CUPID Son of Venus PLUTO God of death (In the play referred by Shakespeare as lsquoDisrsquo which is a Roman name for Pluto)

ORIGINAL TEXT PARAPHRASEPROSPEROWellmdash

PROSPERONow come Ariel Let there be too many rather than too few

Now come my Ariel Bring a corollaryRather than want a spirit Appear and pertly[to Ferdinand and Miranda]No tongue all eyes Be silent

spirits in attendance Appear briskly

[to Ferdinand and Miranda]Look with your eyes but do not say a word

[Soft music] [Soft music][Enter Iris] [Enter Iris]

IRISCeres most bounteous lady thy rich leasOf wheat rye barley vetches oats and peasThy turfy mountains where live nibbling sheepAnd flat meads thatched with stover them to keepThy banks with pioned and twilled brimsWhich spongy April at thy hest betrimsTo make cold nymphs chaste crowns and thybroom-grovesWhose shadow the dismissegraved bachelor lovesBeing lass-lorn thy pole clipped vineyardAnd thy sea-marge sterile and rocky-hardWhere thou thyself dost airmdashthe Queen othrsquoSkyWhose watery arch and messenger am IBids thee leave these and with her sovereign grace[Juno appears] Here on this grass-plot in this very placeTo come and sport Her peacocks fly amainApproach rich Ceres her to entertain

IRISCeres most generous lady you are the cause of rich fields or fertile land where wheat rye barley beans oats and peas grow the grassy mountains where the sheep graze and the flat meadows covered with coarse hay to be used as fodder for cattleYour banks are covered with marsh-marigolds and reeds and the rainy April under your orders brings forth to make for the maids who are not in love beautiful crowns your woods where the broom flourishes and where the bachelor who has been dismissed by the maid he loved lies down being forsaken your vineyard in which the poles are embraced by the vines and the margin of the sea which is barren and rocky where you roam about to enjoy the fresh air ndash the queen of the sky (Juno) whose messenger I am besides being represented as the rainbow bids you leave all these and with her majesty here on this grassy plot in this very place come and sport her peacocks carry her fast in her chariot through the air and are making their way here approach rich Ceres to welcome her

[Enter Ariel as Ceres] [Enter Ariel as Ceres]

CERESHail many-coloured messenger that neerDost disobey the wife of JupiterWho with thy saffron wings upon my flowersDiffusest honey-drops refreshing showersAnd with each end of thy blue bow dost crownMy bosky acres and my unshrubbed downRich scarf to my proud earth Why hath thy queenSummoned me hither to this short-grassed green

CERESWelcome rainbow that never dared disobey Juno the wife of Jupiter who with your orange coloured rays spread honey-drops refreshing showers And with each end of thy blue bow drown my bushy acres and my hilly country which is free from shrubs you thus forming a rich scarf Why has your queen called me here to this place covered with short grass

IRISA contract of true love to celebrateAnd some donation freely to estateOn the blest lovers

IRISI have called you to celebrate a contract of true love and bestow some liberal gift upon the blessed lovers

ORIGINAL TEXT PARAPHRASECERESTell me heavenly bowIf Venus or her son as thou dost knowDo now attend the queen Since they did plotThe means that dusky Dis my daughter gotHer and her blind boys scandaled companyI have forsworn

CERESTell me heavenly bow if Venus the Goddess of love or Cupid her son and pedlar of passion at this time attend the heavenly queen Juno because you are sure to know Since the day they conspired against me and dark Pluto took away my daughter here and Cupidrsquos disgraceful company I have left off

IRISOf her societyBe not afraid I met her deityCutting the clouds towards Pathos and her sonDove-drawn with her Here thought they to have doneSome wanton charm upon this man and miad

IRISBe not afraid of her company I met her deity moving on the clouds towards Paphos the sacred home of Venus on the island of Cyprus along with her son on her chariot drawn by doves Here they contemplated to exercise a charm upon this man and maid producing

Whose vows are that no bed-right shall be paidTill Hymens torch be lightedmdashbut in vainMarss hot minion is returned againHer waspish-headed son has broke his arrowsSwears he will shoot no more but play with sparrowsAnd be a boy right out

wantonness before the actual marriage ceremony but did not succeed Venus has returned her irritable son has broken his arrows and swears that he will give up his practice of trying to inspire love but play with sparrows and be a boy again

[Music is heard] [Music is heard]

CERESHighst queen of stateGreat Juno comes I know her by her gait

CERESHighest queen of state Great Juno there she comes I know here by her gait

[Enter Juno] [Enter Juno]

JUNOHow does my bounteous sister Go with meTo bless this twain that they may Prosperous beAnd honoured in their issue

JUNOHow are you doing my generous sister Come with me to bless this couple so that they may be prosperous and fortunate in their children

[They sing] [They sing]

JUNOHonour riches marriage-blessingLong continuance and increasingHourly joys be still upon youJuno sings her blessings upon you

JUNOMay honour riches happiness in marriage long continuance and increase of those boons ever rest upon you as hourly joys Juno showers down upon you her blessings in song

CERESEarths increase foison plentyBarns and garners never emptyVines and clustring bunches growingPlants and goodly burden bowingSpring come to you at the farthestIn the very end of harvestScarcity and want shall shun youCeresrsquo blessing so is on you

CERESMay you have the plenty of earthrsquos produce Your barns and granaries may never be empty Your vines may grow with clustering bunches Your fruit trees may be heavily laden with their fruit May there be continuous spring and harvest May scantiness and want leave you forever Such is the blessing of Ceres upon you

FERDINANDThis is a most majestic vision andHarmoniously charmingly May I be boldTo think these spirits

FERDINANDThis is a great vision and magically melodious Should I suppose the characters (taking part in the masque) are spirits

PROSPEROSpirits which by mine artI have from their confines calld to enactMy present fancies

PROSPEROYes they are spirits whom I have summoned from the regions to which they are confined to carry into effect my fanciful designs

ORIGINAL TEXT PARAPHRASEFERDINANDLet me live here everSo rare a wondered father and a wifeMakes this place paradise

FERDINANDI should like to live here forever Such a wise and wonderful father makes this place a paradise

[Juno and Ceres whisper and send Iris on employment] [Juno and Ceres whisper and send Iris on employment]

PROSPEROSweet now silence

PROSPEROMy dear Ferdinand speak no more Juno and Ceres are

Juno and Ceres whisper seriouslyTheres something else to do Hush and be muteOr else our spell is marred

whispering with a solemn look There is something else coming Silence Or else our magic will be spoilt

IRISYour nymphs called naiads of the wandering brooksWith your sedged crowns and over-harmless looksLeave your crisp channels and on this green landAnswer your summons Juno does commandCome temperate nymphs and help to celebrateA contract of true love Be not too late

IRISYou nymphs called Naiads denizens (M inhabitants) of the running stream with your chaplets of sedge and ever-helpful looks leave your wrinkled channels and on the green land answer the summons sent to you Juno has ordered some chaste nymphs and help to celebrate a noble and true marriage Donrsquot delay

[Enter certain nymphs] [Enter certain nymphs]You sunburnt sicklemen of August wearyCome hither from the furrow and be merryMake holiday your rye-straw hats put onAnd these fresh nymphs encounter every oneIn country footing

You sunburnt harvesters weary from the effects of the heat in August come here from the furrowed land and rejoice Make holiday with your rye-straw hats upon you and meet these fresh nymphs and join in country dancing

[Enter certain reapers properly habited They join with the nymphs in a graceful dance towards the end whereof Prospero starts suddenly and speaks]

[Enter certain reapers properly habited They join with the nymphs in a graceful dance towards the end whereof Prospero starts suddenly and speaks]

PROSPERO[aside] I had forgot that foul conspiracyOf the beast Caliban and his confederatesAgainst my life The minute of their plotIs almost come [to the spirits]Well done Avoidno more

PROSPERO(Aside)I had forgotten the wicked conspiracy of the beast Caliban and his accomplices against my life the time of their plot has almost arrived ndash (To the Spirits) well done depart no more of this

[To a strange hollow and confused noise the spirits heavily vanish]

[The spirits depart]

ORIGINAL TEXT PARAPHRASEFERDINANDLet me live here everSo rare a wondered father and a wifeMakes this place paradise

FERDINANDI should like to live here forever Such a wise and wonderful father makes this place a paradise

[Juno and Ceres whisper and send Iris on employment] [Juno and Ceres whisper and send Iris on employment]

PROSPEROSweet now silenceJuno and Ceres whisper seriouslyTheres something else to do Hush and be muteOr else our spell is marred

PROSPEROMy dear Ferdinand speak no more Juno and Ceres are whispering with a solemn look There is something else coming Silence Or else our magic will be spoilt

IRISYour nymphs called naiads of the wandering brooksWith your sedged crowns and over-harmless looksLeave your crisp channels and on this green landAnswer your summons Juno does commandCome temperate nymphs and help to celebrateA contract of true love Be not too late

IRISYou nymphs called Naiads denizens (M inhabitants) of the running stream with your chaplets of sedge and ever-helpful looks leave your wrinkled channels and on the green land answer the summons sent to you Juno has ordered some chaste nymphs and help to celebrate a noble and true marriage Donrsquot delay

[Enter certain nymphs] [Enter certain nymphs]You sunburnt sicklemen of August wearyCome hither from the furrow and be merryMake holiday your rye-straw hats put onAnd these fresh nymphs encounter every oneIn country footing

You sunburnt harvesters weary from the effects of the heat in August come here from the furrowed land and rejoice Make holiday with your rye-straw hats upon you and meet these fresh nymphs and join in country dancing

[Enter certain reapers properly habited They join with the nymphs in a graceful dance towards the end whereof Prospero starts suddenly and speaks]

[Enter certain reapers properly habited They join with the nymphs in a graceful dance towards the end whereof Prospero starts suddenly and speaks]

PROSPERO[aside] I had forgot that foul conspiracyOf the beast Caliban and his confederatesAgainst my life The minute of their plotIs almost come [to the spirits]Well done Avoidno more

PROSPERO(Aside)I had forgotten the wicked conspiracy of the beast Caliban and his accomplices against my life the time of their plot has almost arrived ndash (To the Spirits) well done depart no more of this

[To a strange hollow and confused noise the spirits heavily vanish]

[The spirits depart]

Ac-12 27420 topic Revaluation of Assets and Liabilities

REVALUATION OF ASSETS AND LIABILITIES

On admission of a new partner the firm stands reconstituted and consequently the assets are revalued and liabilities are reassessed It is necessary to show the true position of the firm at the time of admission of a new partner If the values of the assets are raised gain will increase the capital of the existing partners Similarly any decrease in the value of assets ie loss will decrease the capital of the existing partners For this purpose alsquoRevaluation Accountrsquo is prepared This account is credited with all increases in the value of assets and decrease in the value of liabilities It is debited with decrease on account of value of assets and increase in the value of liabilities The balance of this account shows a gain or loss on revaluation which is transferred to the existing partnerrsquos capital account in existing profit sharing ratioAccounting for Revaluation of Assets and Liabilities when there is a Changein the Profit Sharing Ratio of Existing PartnersAssets and liabilities of a firm must also be revalued at the time of change in profit sharing ratio of existing partners The reason is that the realisable or actual value of assets and liabilities may be different from those shown in the Balance Sheet It is possible that with the passage of time some of the assets might have appreciated in value while the value of certain other assets might have decreased and no record has been made of such changes in the books of accounts Similarly there may be some unrecorded assets amp libilities that may have to be accounted for Revaluation of assets and reassessments of liabilities becomes necessary because the change in the

value of assets and liabilities belongs to the period to change in profit sharing ratio and hence must be shared by the partners in their old profit sharing ratio Revaluation of assets and reassessment of liabilities may be given effect to in two different ways (a) When revised values are to be recorded in the books and(b) When revised values are not to be recorded in the books

When revised values are to be recorded in the booksIn such a case revaluation of assets and reassessment of liabilities is done with the help of a new account called lsquoRevaluation Accountrsquo Sometimes this account is also called as lsquoProfit amp Loss Adjustment Acrsquo If there is a loss due to revaluation revaluation account is debited and if the revaluation results in a profit the revaluation account is credited The following journal entries made for this purpose are

(i) For increase in the value of assetsAsset Ac Dr (individually)To Revaluation Ac(ii) For decrease in the value of AssetRevaluation Ac Dr (individually)To Asset Ac[Decrease in the value of assets](iii) For increase in the value of LiabilitiesRevaluation Ac Dr (individually)To Liabilities Ac[Increase in the value of Liabilities](iv) For decrease in the value of LiabilitiesLiabilities Ac DrTo Revaluation Ac[Decrease in the value of Liabilities](v) For unrecorded AssetsAsset Ac [unrecorded] DrTo Revaluation Ac[Unrecorded asset recorded at actual value](vi) For unrecorded Liability Revaluation Ac DrTo Liability Ac [unrecorded][Unrecorded Liability recorded at actual value](vii) For transfer of gain on revaluationRevaluation Ac DrTo Existing Partnerrsquos CapitalCurrent Ac[Profit on revaluation transferred to capital account in existing ratio](viii) For transfer of loss on revaluationExisting Partnerrsquos CapitalCurrent Ac DrTo Revaluation Ac[Loss on revaluation transferred to capital account in existing ratio](a) When revaluation account shows gain Revaluation Ac DrTo Partnerrsquos Capital Ac (Old Profit Sharing Ratio)(Profit on revaluation credited to Partnerrsquos Capital Ac)(b) Above entry is reversed when revaluation account shows loss Partners Capital Acs (Old Profit Sharing Ratio) DrTo Revaluation Ac(Loss on revaluation debited to Partnerrsquos Capital Acs)

Proforma of Revaluation Account is given as under

Revaluation Account

Dr Cr Particulars ` Amount Particulars ` Amount To Decrease in value of assets By Increase in value of assets To Increase in value of liabilities By Decrease in value of liabilities To Unrecorded liabilities By Unrecorded assets To Gain on Revaluation (Transferred) By Loss on Revalution (Transferred)

ECO ndash12 2742020Topic- ELASTICITY OF DEMAND

CHAPTER - ELASTICITY OF DEMANDMEANINGDemand for a commodity is affected by many factors such as its price price of related goods income of its buyer tastes and preferences etc Elasticity means degree of response Elasticity of demand means degree of responsiveness of demand Demand for a commodity responds to change in price price of related goods income etc So we have three dimensions of elasticity of demandDIMENSION OF ELASTICITY OF DEMAND TYPES OF ELASTICITY OF DEMAND

Price elasticity of demand Income elasticity of demand Cross Elasticity of demand

Price elasticity of demand Price elasticity of demand means degree of responsiveness of demand for a commodity to the change in its price For example if demand for a commodity rises by 10 due to 5 fall in its price Price elasticity of demand (ep)=Percentage change in quantity demanded Percentage change in price of the commodity = 10 ( -)5 = ( - )2Note that ep will always be negative due to inverse relationship of price and quantity demanded

(ii) Income elasticity of demand Income elasticity of demand refers to the degree of responsiveness of demand for a commodity to the change in income of its buyer Suppose income of buyer rises by 10 and his demand for a commodity rises by 20 then Income elasticity of demand (ey)= change in quantity demanded change in price of the commodity =20 10 = 2

Cross Elasticity of demandCross elasticity of demand means the degree of responsiveness of demand for a commodity to the change in price of its related goods (substitute goods or complementary goods) Suppose demand for a commodity rises by 10 due to 5 rise in price of its substitute good then Cross elasticity of demand (ec) = change in quantity demanded change in price of related good = 10 2 = 5 (Tastes and preferences cannot be expressed numerically So elasticity ofdemand cannot be numerically expressed)

  • Chapter 1 Force (Summary)
  • Distinguish between external sovereignty and internal sovereignty
    • NAND Gate
      • Logic diagram
      • Truth Table
        • NOR Gate
          • Logic diagram
          • Truth Table
            • XOR Gate
              • Logic diagram
              • Truth Table
                • XNOR Gate
                  • Logic diagram
                  • Truth Table
                      • Physics
                      • Chapter 1 Electric Field ( Electric Dipole) (Summary)
Page 40:  · Web viewWe all know that Nouns are divided into two parts: common noun and proper noun.Apart from common and proper noun, we will also study about collective noun and compound

What is manpower estimation Explain its quantitative and qualitative aspectsAnswer) Manpower estimation is the process by which management determines how an organisation should move from itrsquos current manpower positionto its desired manpower position There are two dimensions of Manpower estimation- quantitative and qualitative

Quantitative aspectThis aspect of Manpower estimation involves estimating the number of employees required in a future time period Workload analysis and workforce analysis are done to estimate the quantity of required manpower

Qualitative aspectThe estimate of the knowledge skills experience etc of required manpower is the qualitative aspect of Manpower estimation The quality of Manpower can be judged on the basis of job analysisand job specification

COMMERCE

CAPITAL-FIXED AND WORKING

Today let us start the class by discussing the sources of finance for different types of business firms

The term lsquocapitalrsquo refers to the investment made in the enterprise for the purpose of earning profits

Requirements of capital and sources of capital for different types of business firms are

1 Capital for sole proprietorship businessA sole proprietor operates at a small scale and thereforerequires a limited amount of capital

2 Capital for partnership firmCapital requirements as well as capital base of a partnership is bigger than that of a sole trader businessThe owned capital is contributed by the partners in an agreed ratio

3 Capital for joint stock companyA joint stock company generally requires large amount of capitalA public company can raise huge capital through issue of shares In addition to share capital it can utilize retained profits

Now let u discuss the meaning of Finance PlanningFinance planning is the process of estimation the financial requirements of an organization specifying the sources of firms and ensuring that enough funds are available at the right time

1 What do you mean by Finance PlanningAnswer Finance planning is the process of estimation the financial requirements of an organization specifying the sources of firms and ensuring that enough funds are available at the right time

2Discuss the role of financial planning of an enterpriseThe role of financial planning are as followsa A sound financial plan helps a business enterprise to avaid the problems of shortage and surplus of fundsbFinancial planning serves as a guide in developing a sound capital structure so as to maximize returns to shareholders c It helps in effective utilization of fundsd It provides policies and procedures for coordinating different functional areas or departments of businesse It enables the management to exercise effective control over the financial activities of an enterprisef It helps the company to prepare for facing business shocks and surprises in future

Mathematics

Continuity and differentiability

Recall Definition of ContinuityLet f(x) be a single valued function of x and x=a be a point in the domain of definition of the function The function is said to be continuous at x=a ifi) f(c) is defined ie f(x) has a definite finite value at x=cii) lim xrarra f(x) exists andiii) lim xrarra f(x) =f(a) In other words f(x) is said to be continuous at x=a if lim xrarra+ f(x)= lim xrarra- f(x) = f(a) Or f(a+0) =f(a-0) =f(a) Or lim hrarr0 f(a+h)= f(a) Algebra of continuous functionsNow we will study some algebra of continuous functions Theorem 1 Suppose f and g be two real functions continuous at a real number c Then(1) f + g is continuous at x = c(2) f ndash g is continuous at x = c(3) f g is continuous at x = c(4) (fg) is continuous at x = c (provided g (c) ne 0)

Example 1 Prove that every rational function is continuousSolution Recall that every rational function f is given byf(x)=[p(x) q(x) ] q(x)ne0where p and q are polynomial functions The domain of f is all real numbers except points at which q is zero Since polynomial functions are continuous f is continuous by (4) of Theorem 1Example 2Discuss the continuity of sine functionSolution To see this we use the following factslim xrarr0 sin x =0Now observe that f (x) = sin x is defined for every real number Let c be a real number Put x = c + h If x rarr c we know that h rarr 0 Therefore lim xrarrc f(x)

= lim xrarrc sin x= lim hrarr0 sin(c+h) =lim hrarr0 [sin c cos h + cos c sin h ]=lim hrarr0 (sin c cos h) + lim hrarr0 (cos c sin h) = sin c +0=sin c = f(c) Thus lim xrarrc f(x) = f(c) and hence f is a constant function Exercise Prove that the function f(x) = x2 +2x is continuous for every real value of x [Hints show that lim xrarra+ f(x) = lim xrarra- f(x) = f(a) ]

Biology Reproduction in Flowering plants We will discuss about megasporoangium

megasporagenesis and female gametophyte

Q4 Describe the structure of megasporangium

Ovule is attached to the placenta by astalk called funicle

Each ovule has one two or three protectivecoverings called integuments

At the tip of integuments a small openingcalled micropyle is organised

Opposite to the micropylar end is the chalaza

Within the integuments a mass of cellsnucellusand inside it embryo sac orfemale gametophyte is present

Q5 Describe a mature embryosacamp its formation

In most of the flowering plants only oneof the 4 megaspores formed as a result ofmegasporogenesis that is functional while theother three degenerate

The

functional megaspore develops into thefemale gametophyte

Formation The nucleus

of the functional megasporedivides mitotically to form two nuclei first andthen two more sequential mitotic nucleardivisions result in the formation of four ampthen eight nucleate stages of embryo sac

Six of the eight nucleus are surrounded bycell walls and organised into cells

The remaining two nuclei called polar nuclei are found below the egg apparatus in the largecentral cell

Three cells consisting of two synergids amp one egg cell present bottom of

embryo sac Three cells

at the chalazal as antipodal cells

Two polar nuclei together present in large central cell

HISTORY

TOWARDS INDEPENDENCE AND PARTITION THE LAST PHASE(1935-1947)SUB TOPIC NATIONAL MOVEMENTS DURING THE SECOND WORLD WAR

Spread of Quit India Movement On 9th August 1942Gandhiji and other Congress leaders were arrested The Congress was declared illegal The news of the arrest of all leaders marked the beginning of a widespread movement of India It was not possible for such a movement to remain peacefulBut the arrest of the all notable congress leaders virtually left the movement in the hands of the mass The movement took the form of violent and militant outbreakBesides congressmen revolutionaries also were very active in the movement The Congress Socialist group also played a prominent role

1 Question Why did the British authority arrest the Congress leaders on 9 th August 1942Answer Congress Working committee adopted the Quit India resolution which was to be ratified at the Bombay AICC meeting in 8th August 1942 They decided to launch a mass struggle on non-violent lines Gandhiji gave a clarion call to all section of the people rdquoKarenge ya Marengerdquo (do or die) Congress leaders gave the call to driving out

the British from IndiaViceroy had taken strong action against the Quit India movement Gandhiji and all the leaders of Congress were arrested

2 Question How did Quit India Movement spread out all over IndiaAnswer The news of the leaders lsquo arrest marked the beginning of a widespread movement to remain peacefulThe movement took form of violent outbreak There were widespread cutting of telephone and Telegraph wires damaging railway lines raising barricades in cities and towns and other forms of violent demonstations

Question Name the leaders of Congress

Socialist group played a prominent part Notable among the Jayprakash Narayan Rammonohar Lohia Aruna Asaf Ali

Political science

Topic-Franchise and Representation

Summary

The election commission

The Election Commission of India is an autonomous constitutional authority responsible for administering Union and State election processes in India The body administers elections to the Lok Sabha Rajya Sabha State Legislative Assemblies in India and the offices of the President and Vice President in the country

Functions of election commission-

India is a sovereign socialist secular democratic republic Democracy runs like a golden thread in the social economic and political fabric woven by the Constitution given by lsquoWe the People of Indiarsquo unto ourselves The concept of democracy as visualised by the Constitution pre-supposes the representation of the people in Parliament and State legislatures by the method of election The Supreme Court has held that democracy is one of the inalienable basic features of the Constitution of India and forms part of its basic structure The Constitution of India adopted a Parliamentary form of government Parliament consists of the President of India and the two Houses mdash Rajya Sabha and Lok Sabha India being a Union of states has separate state legislatures for each state State legislatures consist of the Governor and two Houses mdash Legislative Council and Legislative Assembly mdash in seven states namely Andhra Pradesh Telangana Bihar Jammu amp Kashmir Karnataka Maharashtra and Uttar Pradesh and of the Governor and the state Legislative Assembly in the remaining 22 states Apart from the above two out of the seven Union Territories namely National Capital Territory of Delhi and Puducherry also have their Legislative Assemblies

ExecutionShort notes-Election commissionFunctions of election commission

Homework- Learn

Computer

Science

Computer hardware NAND Gate

A NOT-AND operation is known as NAND operation It has n input (n gt= 2) and one output

Logic diagram

Truth Table

NOR Gate

A NOT-OR operation is known as NOR operation It has n input (n gt= 2) and one output

Logic diagram

Truth Table

XOR Gate

XOR or Ex-OR gate is a special type of gate It can be used in the half

adder full adder and subtractor The exclusive-OR gate is abbreviated as EX-OR gate or sometime as X-OR gate It has n input (n gt= 2) and one output

Logic diagram

Truth Table

XNOR Gate

XNOR gate is a special type of gate It can be used in the half adder full adder and subtractor The exclusive-NOR gate is abbreviated as EX-NOR gate or sometime as X-NOR gate It has n input (n gt= 2) and one output

Logic diagram

Truth Table

Physics

Chapter 1 Electric Field ( Electric Dipole) (Summary)

Here we will derive Expression of electric field at broad side

On position of dipole

Execution

Q With the help of a labelled diagram obtain an expression for the electric field intensity E at any point on the equitorial line ( broad-side on position) of an electric dipole

Ans

E1 E1sinθ

E θ P E1 θ

( r2+L2)12 E2 E

r E2 E2sinθ

-q θ L O L +qA B

Let us consider that the point P is situated on the right bisector of the dipole AB at a distance r meter from its midpoint O

Let E1 and E2 be the electric field intensities of the electric field at P due to charge +q and ndashq of the dipole resp The distance of P from each charge is ( r2+L2)12

So E1 = 14 πϵ q

(r 2+L 2) away from +q

E2 = 14 πϵ q

(r 2+L 2) towards ndashq

The magnitudes of E1 and E2 are equal but directions are different Now resolving E1 and E2 into two components parallel and perpendicular to AB we get

The components perpendicular to AB E1sinθ and E2sinθ cancel each other because they are equal and opposite

The components parallel to AB are E1cosθ and E2 cosθ are in same direction and add up

So resultant intensity of electric field at the point P is

E = E1cosθ + E2 cosθ

E = 14 πϵ q

(r 2+L 2) 2 cosθ

Now from fig we have cosθ =BOBP = L (r2+L2)12

So we get E = 14 πϵ 2qL ( r2+L2)32

Now electric dipole moment p= 2qL

So E = 14 πϵ p ( r2+L2)32

HW Find the expression of Electric field as done here but this time take r gtgt 2L

Also find the expression of torque experience by a dipole

(Hint Electric force experienced by charges of dipole in electric field is qE each Let θ be the angle which dipole makes with electric lines of force then perpendicular distance between two charges is 2Lsinθ Then torque = force x perp distance = qE x 2L sinθ So τ=pE sinθ where p =2qL )

STUDY MATERIAL

Class XIISubject Eng Literature (The Tempest ndash William Shakespeare) Topic Act IV Scene 1 Lines 84 to 133 (Iris hellip A contract of true love Be not too late ) Date 27th April 2020 (4th Period)

[Students should read the original play and also the paraphrase given in the school prescribed textbook]Summary Questions amp Answers

o Ceres soon appears and comes to know that she has been summoned to celebrate the contract of true love

o Ceres expresses her unwillingness to meet Venus and Cupid as she has shunned their company

o Ceres and Juno both bestow their blessings upon Ferdinand and Miranda with June gifting honour riches happiness in marriage and Ceres presents plenty of earthrsquos produce

o Iris summons the water-nymphs and reapers to come and celebrate a contract

(1) IRIS Of her society (Line 91-101)

Be not afraid I met her deity

Cutting the clouds towards Pathos and her sonDove-drawn with her Here thought they to have doneSome wanton charm upon this man and maidWhose vows are that no bed-right shall be paidTill Hymens torch be lightedmdashbut in vainMarss hot minion is returned againHer waspish-headed son has broke his arrowsSwears he will shoot no more but play with sparrowsAnd be a boy right out

(i) Where were Venus and Cupid seen flying How were they travelling Why did they want to join the marriage celebration of Ferdinand and Miranda

of true love

Venus and Cupid were seen flying through the air towards Paphos the famous city which is situated on the island of Cyprus They were travelling by air-borne chariot drawn by doves They certainly wanted to come here in order to play some amorous trick upon Ferdinand and Miranda who are under a vow not to gratify their physical desires till the holy ceremony of their marriage has been performed(ii) What have Venus and Cupid done after failing in their plan

After being failure of their plan Venus who is a very passionate deity and who is the mistress of Mars (the god of war) has gone back while here ill-tempered son Cupid has broken his arrows of love in his state of desperation(iii) What has Cupid firmly decided

Cupid is feeling so disappointed that he has firmly decided to shoot no more arrows to arouse love in human hearts but to spend his time playing with sparrows Thus he would now become just a boy and would give up his original function of shooting arrows on human beings to make them fall in love(iv) What vow had Ceres taken How did Ceres feel at the abduction

After the abduction of her daughter Prosperina by Pluto Ceres had taken a vow to always keep away from the disgraceful company of Venus and her blind son Cupid the god of love Ceres felt deeply distressed when Pluto had carried off her daughter and had made her his wife by force(v) Why has Ceres not forgiven Venus and her blind son For what do Ceres want to be sure

As the abduction had been manipulated by Venus the goddess of beauty and love and her blind son Cupid Ceres has never forgiven them for their part in the whole plot Ceres wants to be sure that she would not have to meet Venus and Cupid who had engineered the abduction of her daughter Prosperina

AS THIS lsquoMASQUErsquo SCENE IS VERY IMPORTANT IN THE PLAY THE PARAPHRASE OF THE ENTIRE PORTION OF MASQUE SCENE (Act IV Lines 58 to 143) IS GIVEN BELOW

IRIS Goddess of RainbowCERES Goddess of Agriculture and all the fruits of the earth

(Nature growth prosperity rebirth ndash notions intimately connected to marriage)JUNO The majestic Queen of Heavens and wife of Jupiter (Jupiter is the king of Gods)

VENUS The Goddess of love CUPID Son of Venus PLUTO God of death (In the play referred by Shakespeare as lsquoDisrsquo which is a Roman name for Pluto)

ORIGINAL TEXT PARAPHRASEPROSPEROWellmdash

PROSPERONow come Ariel Let there be too many rather than too few

Now come my Ariel Bring a corollaryRather than want a spirit Appear and pertly[to Ferdinand and Miranda]No tongue all eyes Be silent

spirits in attendance Appear briskly

[to Ferdinand and Miranda]Look with your eyes but do not say a word

[Soft music] [Soft music][Enter Iris] [Enter Iris]

IRISCeres most bounteous lady thy rich leasOf wheat rye barley vetches oats and peasThy turfy mountains where live nibbling sheepAnd flat meads thatched with stover them to keepThy banks with pioned and twilled brimsWhich spongy April at thy hest betrimsTo make cold nymphs chaste crowns and thybroom-grovesWhose shadow the dismissegraved bachelor lovesBeing lass-lorn thy pole clipped vineyardAnd thy sea-marge sterile and rocky-hardWhere thou thyself dost airmdashthe Queen othrsquoSkyWhose watery arch and messenger am IBids thee leave these and with her sovereign grace[Juno appears] Here on this grass-plot in this very placeTo come and sport Her peacocks fly amainApproach rich Ceres her to entertain

IRISCeres most generous lady you are the cause of rich fields or fertile land where wheat rye barley beans oats and peas grow the grassy mountains where the sheep graze and the flat meadows covered with coarse hay to be used as fodder for cattleYour banks are covered with marsh-marigolds and reeds and the rainy April under your orders brings forth to make for the maids who are not in love beautiful crowns your woods where the broom flourishes and where the bachelor who has been dismissed by the maid he loved lies down being forsaken your vineyard in which the poles are embraced by the vines and the margin of the sea which is barren and rocky where you roam about to enjoy the fresh air ndash the queen of the sky (Juno) whose messenger I am besides being represented as the rainbow bids you leave all these and with her majesty here on this grassy plot in this very place come and sport her peacocks carry her fast in her chariot through the air and are making their way here approach rich Ceres to welcome her

[Enter Ariel as Ceres] [Enter Ariel as Ceres]

CERESHail many-coloured messenger that neerDost disobey the wife of JupiterWho with thy saffron wings upon my flowersDiffusest honey-drops refreshing showersAnd with each end of thy blue bow dost crownMy bosky acres and my unshrubbed downRich scarf to my proud earth Why hath thy queenSummoned me hither to this short-grassed green

CERESWelcome rainbow that never dared disobey Juno the wife of Jupiter who with your orange coloured rays spread honey-drops refreshing showers And with each end of thy blue bow drown my bushy acres and my hilly country which is free from shrubs you thus forming a rich scarf Why has your queen called me here to this place covered with short grass

IRISA contract of true love to celebrateAnd some donation freely to estateOn the blest lovers

IRISI have called you to celebrate a contract of true love and bestow some liberal gift upon the blessed lovers

ORIGINAL TEXT PARAPHRASECERESTell me heavenly bowIf Venus or her son as thou dost knowDo now attend the queen Since they did plotThe means that dusky Dis my daughter gotHer and her blind boys scandaled companyI have forsworn

CERESTell me heavenly bow if Venus the Goddess of love or Cupid her son and pedlar of passion at this time attend the heavenly queen Juno because you are sure to know Since the day they conspired against me and dark Pluto took away my daughter here and Cupidrsquos disgraceful company I have left off

IRISOf her societyBe not afraid I met her deityCutting the clouds towards Pathos and her sonDove-drawn with her Here thought they to have doneSome wanton charm upon this man and miad

IRISBe not afraid of her company I met her deity moving on the clouds towards Paphos the sacred home of Venus on the island of Cyprus along with her son on her chariot drawn by doves Here they contemplated to exercise a charm upon this man and maid producing

Whose vows are that no bed-right shall be paidTill Hymens torch be lightedmdashbut in vainMarss hot minion is returned againHer waspish-headed son has broke his arrowsSwears he will shoot no more but play with sparrowsAnd be a boy right out

wantonness before the actual marriage ceremony but did not succeed Venus has returned her irritable son has broken his arrows and swears that he will give up his practice of trying to inspire love but play with sparrows and be a boy again

[Music is heard] [Music is heard]

CERESHighst queen of stateGreat Juno comes I know her by her gait

CERESHighest queen of state Great Juno there she comes I know here by her gait

[Enter Juno] [Enter Juno]

JUNOHow does my bounteous sister Go with meTo bless this twain that they may Prosperous beAnd honoured in their issue

JUNOHow are you doing my generous sister Come with me to bless this couple so that they may be prosperous and fortunate in their children

[They sing] [They sing]

JUNOHonour riches marriage-blessingLong continuance and increasingHourly joys be still upon youJuno sings her blessings upon you

JUNOMay honour riches happiness in marriage long continuance and increase of those boons ever rest upon you as hourly joys Juno showers down upon you her blessings in song

CERESEarths increase foison plentyBarns and garners never emptyVines and clustring bunches growingPlants and goodly burden bowingSpring come to you at the farthestIn the very end of harvestScarcity and want shall shun youCeresrsquo blessing so is on you

CERESMay you have the plenty of earthrsquos produce Your barns and granaries may never be empty Your vines may grow with clustering bunches Your fruit trees may be heavily laden with their fruit May there be continuous spring and harvest May scantiness and want leave you forever Such is the blessing of Ceres upon you

FERDINANDThis is a most majestic vision andHarmoniously charmingly May I be boldTo think these spirits

FERDINANDThis is a great vision and magically melodious Should I suppose the characters (taking part in the masque) are spirits

PROSPEROSpirits which by mine artI have from their confines calld to enactMy present fancies

PROSPEROYes they are spirits whom I have summoned from the regions to which they are confined to carry into effect my fanciful designs

ORIGINAL TEXT PARAPHRASEFERDINANDLet me live here everSo rare a wondered father and a wifeMakes this place paradise

FERDINANDI should like to live here forever Such a wise and wonderful father makes this place a paradise

[Juno and Ceres whisper and send Iris on employment] [Juno and Ceres whisper and send Iris on employment]

PROSPEROSweet now silence

PROSPEROMy dear Ferdinand speak no more Juno and Ceres are

Juno and Ceres whisper seriouslyTheres something else to do Hush and be muteOr else our spell is marred

whispering with a solemn look There is something else coming Silence Or else our magic will be spoilt

IRISYour nymphs called naiads of the wandering brooksWith your sedged crowns and over-harmless looksLeave your crisp channels and on this green landAnswer your summons Juno does commandCome temperate nymphs and help to celebrateA contract of true love Be not too late

IRISYou nymphs called Naiads denizens (M inhabitants) of the running stream with your chaplets of sedge and ever-helpful looks leave your wrinkled channels and on the green land answer the summons sent to you Juno has ordered some chaste nymphs and help to celebrate a noble and true marriage Donrsquot delay

[Enter certain nymphs] [Enter certain nymphs]You sunburnt sicklemen of August wearyCome hither from the furrow and be merryMake holiday your rye-straw hats put onAnd these fresh nymphs encounter every oneIn country footing

You sunburnt harvesters weary from the effects of the heat in August come here from the furrowed land and rejoice Make holiday with your rye-straw hats upon you and meet these fresh nymphs and join in country dancing

[Enter certain reapers properly habited They join with the nymphs in a graceful dance towards the end whereof Prospero starts suddenly and speaks]

[Enter certain reapers properly habited They join with the nymphs in a graceful dance towards the end whereof Prospero starts suddenly and speaks]

PROSPERO[aside] I had forgot that foul conspiracyOf the beast Caliban and his confederatesAgainst my life The minute of their plotIs almost come [to the spirits]Well done Avoidno more

PROSPERO(Aside)I had forgotten the wicked conspiracy of the beast Caliban and his accomplices against my life the time of their plot has almost arrived ndash (To the Spirits) well done depart no more of this

[To a strange hollow and confused noise the spirits heavily vanish]

[The spirits depart]

ORIGINAL TEXT PARAPHRASEFERDINANDLet me live here everSo rare a wondered father and a wifeMakes this place paradise

FERDINANDI should like to live here forever Such a wise and wonderful father makes this place a paradise

[Juno and Ceres whisper and send Iris on employment] [Juno and Ceres whisper and send Iris on employment]

PROSPEROSweet now silenceJuno and Ceres whisper seriouslyTheres something else to do Hush and be muteOr else our spell is marred

PROSPEROMy dear Ferdinand speak no more Juno and Ceres are whispering with a solemn look There is something else coming Silence Or else our magic will be spoilt

IRISYour nymphs called naiads of the wandering brooksWith your sedged crowns and over-harmless looksLeave your crisp channels and on this green landAnswer your summons Juno does commandCome temperate nymphs and help to celebrateA contract of true love Be not too late

IRISYou nymphs called Naiads denizens (M inhabitants) of the running stream with your chaplets of sedge and ever-helpful looks leave your wrinkled channels and on the green land answer the summons sent to you Juno has ordered some chaste nymphs and help to celebrate a noble and true marriage Donrsquot delay

[Enter certain nymphs] [Enter certain nymphs]You sunburnt sicklemen of August wearyCome hither from the furrow and be merryMake holiday your rye-straw hats put onAnd these fresh nymphs encounter every oneIn country footing

You sunburnt harvesters weary from the effects of the heat in August come here from the furrowed land and rejoice Make holiday with your rye-straw hats upon you and meet these fresh nymphs and join in country dancing

[Enter certain reapers properly habited They join with the nymphs in a graceful dance towards the end whereof Prospero starts suddenly and speaks]

[Enter certain reapers properly habited They join with the nymphs in a graceful dance towards the end whereof Prospero starts suddenly and speaks]

PROSPERO[aside] I had forgot that foul conspiracyOf the beast Caliban and his confederatesAgainst my life The minute of their plotIs almost come [to the spirits]Well done Avoidno more

PROSPERO(Aside)I had forgotten the wicked conspiracy of the beast Caliban and his accomplices against my life the time of their plot has almost arrived ndash (To the Spirits) well done depart no more of this

[To a strange hollow and confused noise the spirits heavily vanish]

[The spirits depart]

Ac-12 27420 topic Revaluation of Assets and Liabilities

REVALUATION OF ASSETS AND LIABILITIES

On admission of a new partner the firm stands reconstituted and consequently the assets are revalued and liabilities are reassessed It is necessary to show the true position of the firm at the time of admission of a new partner If the values of the assets are raised gain will increase the capital of the existing partners Similarly any decrease in the value of assets ie loss will decrease the capital of the existing partners For this purpose alsquoRevaluation Accountrsquo is prepared This account is credited with all increases in the value of assets and decrease in the value of liabilities It is debited with decrease on account of value of assets and increase in the value of liabilities The balance of this account shows a gain or loss on revaluation which is transferred to the existing partnerrsquos capital account in existing profit sharing ratioAccounting for Revaluation of Assets and Liabilities when there is a Changein the Profit Sharing Ratio of Existing PartnersAssets and liabilities of a firm must also be revalued at the time of change in profit sharing ratio of existing partners The reason is that the realisable or actual value of assets and liabilities may be different from those shown in the Balance Sheet It is possible that with the passage of time some of the assets might have appreciated in value while the value of certain other assets might have decreased and no record has been made of such changes in the books of accounts Similarly there may be some unrecorded assets amp libilities that may have to be accounted for Revaluation of assets and reassessments of liabilities becomes necessary because the change in the

value of assets and liabilities belongs to the period to change in profit sharing ratio and hence must be shared by the partners in their old profit sharing ratio Revaluation of assets and reassessment of liabilities may be given effect to in two different ways (a) When revised values are to be recorded in the books and(b) When revised values are not to be recorded in the books

When revised values are to be recorded in the booksIn such a case revaluation of assets and reassessment of liabilities is done with the help of a new account called lsquoRevaluation Accountrsquo Sometimes this account is also called as lsquoProfit amp Loss Adjustment Acrsquo If there is a loss due to revaluation revaluation account is debited and if the revaluation results in a profit the revaluation account is credited The following journal entries made for this purpose are

(i) For increase in the value of assetsAsset Ac Dr (individually)To Revaluation Ac(ii) For decrease in the value of AssetRevaluation Ac Dr (individually)To Asset Ac[Decrease in the value of assets](iii) For increase in the value of LiabilitiesRevaluation Ac Dr (individually)To Liabilities Ac[Increase in the value of Liabilities](iv) For decrease in the value of LiabilitiesLiabilities Ac DrTo Revaluation Ac[Decrease in the value of Liabilities](v) For unrecorded AssetsAsset Ac [unrecorded] DrTo Revaluation Ac[Unrecorded asset recorded at actual value](vi) For unrecorded Liability Revaluation Ac DrTo Liability Ac [unrecorded][Unrecorded Liability recorded at actual value](vii) For transfer of gain on revaluationRevaluation Ac DrTo Existing Partnerrsquos CapitalCurrent Ac[Profit on revaluation transferred to capital account in existing ratio](viii) For transfer of loss on revaluationExisting Partnerrsquos CapitalCurrent Ac DrTo Revaluation Ac[Loss on revaluation transferred to capital account in existing ratio](a) When revaluation account shows gain Revaluation Ac DrTo Partnerrsquos Capital Ac (Old Profit Sharing Ratio)(Profit on revaluation credited to Partnerrsquos Capital Ac)(b) Above entry is reversed when revaluation account shows loss Partners Capital Acs (Old Profit Sharing Ratio) DrTo Revaluation Ac(Loss on revaluation debited to Partnerrsquos Capital Acs)

Proforma of Revaluation Account is given as under

Revaluation Account

Dr Cr Particulars ` Amount Particulars ` Amount To Decrease in value of assets By Increase in value of assets To Increase in value of liabilities By Decrease in value of liabilities To Unrecorded liabilities By Unrecorded assets To Gain on Revaluation (Transferred) By Loss on Revalution (Transferred)

ECO ndash12 2742020Topic- ELASTICITY OF DEMAND

CHAPTER - ELASTICITY OF DEMANDMEANINGDemand for a commodity is affected by many factors such as its price price of related goods income of its buyer tastes and preferences etc Elasticity means degree of response Elasticity of demand means degree of responsiveness of demand Demand for a commodity responds to change in price price of related goods income etc So we have three dimensions of elasticity of demandDIMENSION OF ELASTICITY OF DEMAND TYPES OF ELASTICITY OF DEMAND

Price elasticity of demand Income elasticity of demand Cross Elasticity of demand

Price elasticity of demand Price elasticity of demand means degree of responsiveness of demand for a commodity to the change in its price For example if demand for a commodity rises by 10 due to 5 fall in its price Price elasticity of demand (ep)=Percentage change in quantity demanded Percentage change in price of the commodity = 10 ( -)5 = ( - )2Note that ep will always be negative due to inverse relationship of price and quantity demanded

(ii) Income elasticity of demand Income elasticity of demand refers to the degree of responsiveness of demand for a commodity to the change in income of its buyer Suppose income of buyer rises by 10 and his demand for a commodity rises by 20 then Income elasticity of demand (ey)= change in quantity demanded change in price of the commodity =20 10 = 2

Cross Elasticity of demandCross elasticity of demand means the degree of responsiveness of demand for a commodity to the change in price of its related goods (substitute goods or complementary goods) Suppose demand for a commodity rises by 10 due to 5 rise in price of its substitute good then Cross elasticity of demand (ec) = change in quantity demanded change in price of related good = 10 2 = 5 (Tastes and preferences cannot be expressed numerically So elasticity ofdemand cannot be numerically expressed)

  • Chapter 1 Force (Summary)
  • Distinguish between external sovereignty and internal sovereignty
    • NAND Gate
      • Logic diagram
      • Truth Table
        • NOR Gate
          • Logic diagram
          • Truth Table
            • XOR Gate
              • Logic diagram
              • Truth Table
                • XNOR Gate
                  • Logic diagram
                  • Truth Table
                      • Physics
                      • Chapter 1 Electric Field ( Electric Dipole) (Summary)
Page 41:  · Web viewWe all know that Nouns are divided into two parts: common noun and proper noun.Apart from common and proper noun, we will also study about collective noun and compound

COMMERCE

CAPITAL-FIXED AND WORKING

Today let us start the class by discussing the sources of finance for different types of business firms

The term lsquocapitalrsquo refers to the investment made in the enterprise for the purpose of earning profits

Requirements of capital and sources of capital for different types of business firms are

1 Capital for sole proprietorship businessA sole proprietor operates at a small scale and thereforerequires a limited amount of capital

2 Capital for partnership firmCapital requirements as well as capital base of a partnership is bigger than that of a sole trader businessThe owned capital is contributed by the partners in an agreed ratio

3 Capital for joint stock companyA joint stock company generally requires large amount of capitalA public company can raise huge capital through issue of shares In addition to share capital it can utilize retained profits

Now let u discuss the meaning of Finance PlanningFinance planning is the process of estimation the financial requirements of an organization specifying the sources of firms and ensuring that enough funds are available at the right time

1 What do you mean by Finance PlanningAnswer Finance planning is the process of estimation the financial requirements of an organization specifying the sources of firms and ensuring that enough funds are available at the right time

2Discuss the role of financial planning of an enterpriseThe role of financial planning are as followsa A sound financial plan helps a business enterprise to avaid the problems of shortage and surplus of fundsbFinancial planning serves as a guide in developing a sound capital structure so as to maximize returns to shareholders c It helps in effective utilization of fundsd It provides policies and procedures for coordinating different functional areas or departments of businesse It enables the management to exercise effective control over the financial activities of an enterprisef It helps the company to prepare for facing business shocks and surprises in future

Mathematics

Continuity and differentiability

Recall Definition of ContinuityLet f(x) be a single valued function of x and x=a be a point in the domain of definition of the function The function is said to be continuous at x=a ifi) f(c) is defined ie f(x) has a definite finite value at x=cii) lim xrarra f(x) exists andiii) lim xrarra f(x) =f(a) In other words f(x) is said to be continuous at x=a if lim xrarra+ f(x)= lim xrarra- f(x) = f(a) Or f(a+0) =f(a-0) =f(a) Or lim hrarr0 f(a+h)= f(a) Algebra of continuous functionsNow we will study some algebra of continuous functions Theorem 1 Suppose f and g be two real functions continuous at a real number c Then(1) f + g is continuous at x = c(2) f ndash g is continuous at x = c(3) f g is continuous at x = c(4) (fg) is continuous at x = c (provided g (c) ne 0)

Example 1 Prove that every rational function is continuousSolution Recall that every rational function f is given byf(x)=[p(x) q(x) ] q(x)ne0where p and q are polynomial functions The domain of f is all real numbers except points at which q is zero Since polynomial functions are continuous f is continuous by (4) of Theorem 1Example 2Discuss the continuity of sine functionSolution To see this we use the following factslim xrarr0 sin x =0Now observe that f (x) = sin x is defined for every real number Let c be a real number Put x = c + h If x rarr c we know that h rarr 0 Therefore lim xrarrc f(x)

= lim xrarrc sin x= lim hrarr0 sin(c+h) =lim hrarr0 [sin c cos h + cos c sin h ]=lim hrarr0 (sin c cos h) + lim hrarr0 (cos c sin h) = sin c +0=sin c = f(c) Thus lim xrarrc f(x) = f(c) and hence f is a constant function Exercise Prove that the function f(x) = x2 +2x is continuous for every real value of x [Hints show that lim xrarra+ f(x) = lim xrarra- f(x) = f(a) ]

Biology Reproduction in Flowering plants We will discuss about megasporoangium

megasporagenesis and female gametophyte

Q4 Describe the structure of megasporangium

Ovule is attached to the placenta by astalk called funicle

Each ovule has one two or three protectivecoverings called integuments

At the tip of integuments a small openingcalled micropyle is organised

Opposite to the micropylar end is the chalaza

Within the integuments a mass of cellsnucellusand inside it embryo sac orfemale gametophyte is present

Q5 Describe a mature embryosacamp its formation

In most of the flowering plants only oneof the 4 megaspores formed as a result ofmegasporogenesis that is functional while theother three degenerate

The

functional megaspore develops into thefemale gametophyte

Formation The nucleus

of the functional megasporedivides mitotically to form two nuclei first andthen two more sequential mitotic nucleardivisions result in the formation of four ampthen eight nucleate stages of embryo sac

Six of the eight nucleus are surrounded bycell walls and organised into cells

The remaining two nuclei called polar nuclei are found below the egg apparatus in the largecentral cell

Three cells consisting of two synergids amp one egg cell present bottom of

embryo sac Three cells

at the chalazal as antipodal cells

Two polar nuclei together present in large central cell

HISTORY

TOWARDS INDEPENDENCE AND PARTITION THE LAST PHASE(1935-1947)SUB TOPIC NATIONAL MOVEMENTS DURING THE SECOND WORLD WAR

Spread of Quit India Movement On 9th August 1942Gandhiji and other Congress leaders were arrested The Congress was declared illegal The news of the arrest of all leaders marked the beginning of a widespread movement of India It was not possible for such a movement to remain peacefulBut the arrest of the all notable congress leaders virtually left the movement in the hands of the mass The movement took the form of violent and militant outbreakBesides congressmen revolutionaries also were very active in the movement The Congress Socialist group also played a prominent role

1 Question Why did the British authority arrest the Congress leaders on 9 th August 1942Answer Congress Working committee adopted the Quit India resolution which was to be ratified at the Bombay AICC meeting in 8th August 1942 They decided to launch a mass struggle on non-violent lines Gandhiji gave a clarion call to all section of the people rdquoKarenge ya Marengerdquo (do or die) Congress leaders gave the call to driving out

the British from IndiaViceroy had taken strong action against the Quit India movement Gandhiji and all the leaders of Congress were arrested

2 Question How did Quit India Movement spread out all over IndiaAnswer The news of the leaders lsquo arrest marked the beginning of a widespread movement to remain peacefulThe movement took form of violent outbreak There were widespread cutting of telephone and Telegraph wires damaging railway lines raising barricades in cities and towns and other forms of violent demonstations

Question Name the leaders of Congress

Socialist group played a prominent part Notable among the Jayprakash Narayan Rammonohar Lohia Aruna Asaf Ali

Political science

Topic-Franchise and Representation

Summary

The election commission

The Election Commission of India is an autonomous constitutional authority responsible for administering Union and State election processes in India The body administers elections to the Lok Sabha Rajya Sabha State Legislative Assemblies in India and the offices of the President and Vice President in the country

Functions of election commission-

India is a sovereign socialist secular democratic republic Democracy runs like a golden thread in the social economic and political fabric woven by the Constitution given by lsquoWe the People of Indiarsquo unto ourselves The concept of democracy as visualised by the Constitution pre-supposes the representation of the people in Parliament and State legislatures by the method of election The Supreme Court has held that democracy is one of the inalienable basic features of the Constitution of India and forms part of its basic structure The Constitution of India adopted a Parliamentary form of government Parliament consists of the President of India and the two Houses mdash Rajya Sabha and Lok Sabha India being a Union of states has separate state legislatures for each state State legislatures consist of the Governor and two Houses mdash Legislative Council and Legislative Assembly mdash in seven states namely Andhra Pradesh Telangana Bihar Jammu amp Kashmir Karnataka Maharashtra and Uttar Pradesh and of the Governor and the state Legislative Assembly in the remaining 22 states Apart from the above two out of the seven Union Territories namely National Capital Territory of Delhi and Puducherry also have their Legislative Assemblies

ExecutionShort notes-Election commissionFunctions of election commission

Homework- Learn

Computer

Science

Computer hardware NAND Gate

A NOT-AND operation is known as NAND operation It has n input (n gt= 2) and one output

Logic diagram

Truth Table

NOR Gate

A NOT-OR operation is known as NOR operation It has n input (n gt= 2) and one output

Logic diagram

Truth Table

XOR Gate

XOR or Ex-OR gate is a special type of gate It can be used in the half

adder full adder and subtractor The exclusive-OR gate is abbreviated as EX-OR gate or sometime as X-OR gate It has n input (n gt= 2) and one output

Logic diagram

Truth Table

XNOR Gate

XNOR gate is a special type of gate It can be used in the half adder full adder and subtractor The exclusive-NOR gate is abbreviated as EX-NOR gate or sometime as X-NOR gate It has n input (n gt= 2) and one output

Logic diagram

Truth Table

Physics

Chapter 1 Electric Field ( Electric Dipole) (Summary)

Here we will derive Expression of electric field at broad side

On position of dipole

Execution

Q With the help of a labelled diagram obtain an expression for the electric field intensity E at any point on the equitorial line ( broad-side on position) of an electric dipole

Ans

E1 E1sinθ

E θ P E1 θ

( r2+L2)12 E2 E

r E2 E2sinθ

-q θ L O L +qA B

Let us consider that the point P is situated on the right bisector of the dipole AB at a distance r meter from its midpoint O

Let E1 and E2 be the electric field intensities of the electric field at P due to charge +q and ndashq of the dipole resp The distance of P from each charge is ( r2+L2)12

So E1 = 14 πϵ q

(r 2+L 2) away from +q

E2 = 14 πϵ q

(r 2+L 2) towards ndashq

The magnitudes of E1 and E2 are equal but directions are different Now resolving E1 and E2 into two components parallel and perpendicular to AB we get

The components perpendicular to AB E1sinθ and E2sinθ cancel each other because they are equal and opposite

The components parallel to AB are E1cosθ and E2 cosθ are in same direction and add up

So resultant intensity of electric field at the point P is

E = E1cosθ + E2 cosθ

E = 14 πϵ q

(r 2+L 2) 2 cosθ

Now from fig we have cosθ =BOBP = L (r2+L2)12

So we get E = 14 πϵ 2qL ( r2+L2)32

Now electric dipole moment p= 2qL

So E = 14 πϵ p ( r2+L2)32

HW Find the expression of Electric field as done here but this time take r gtgt 2L

Also find the expression of torque experience by a dipole

(Hint Electric force experienced by charges of dipole in electric field is qE each Let θ be the angle which dipole makes with electric lines of force then perpendicular distance between two charges is 2Lsinθ Then torque = force x perp distance = qE x 2L sinθ So τ=pE sinθ where p =2qL )

STUDY MATERIAL

Class XIISubject Eng Literature (The Tempest ndash William Shakespeare) Topic Act IV Scene 1 Lines 84 to 133 (Iris hellip A contract of true love Be not too late ) Date 27th April 2020 (4th Period)

[Students should read the original play and also the paraphrase given in the school prescribed textbook]Summary Questions amp Answers

o Ceres soon appears and comes to know that she has been summoned to celebrate the contract of true love

o Ceres expresses her unwillingness to meet Venus and Cupid as she has shunned their company

o Ceres and Juno both bestow their blessings upon Ferdinand and Miranda with June gifting honour riches happiness in marriage and Ceres presents plenty of earthrsquos produce

o Iris summons the water-nymphs and reapers to come and celebrate a contract

(1) IRIS Of her society (Line 91-101)

Be not afraid I met her deity

Cutting the clouds towards Pathos and her sonDove-drawn with her Here thought they to have doneSome wanton charm upon this man and maidWhose vows are that no bed-right shall be paidTill Hymens torch be lightedmdashbut in vainMarss hot minion is returned againHer waspish-headed son has broke his arrowsSwears he will shoot no more but play with sparrowsAnd be a boy right out

(i) Where were Venus and Cupid seen flying How were they travelling Why did they want to join the marriage celebration of Ferdinand and Miranda

of true love

Venus and Cupid were seen flying through the air towards Paphos the famous city which is situated on the island of Cyprus They were travelling by air-borne chariot drawn by doves They certainly wanted to come here in order to play some amorous trick upon Ferdinand and Miranda who are under a vow not to gratify their physical desires till the holy ceremony of their marriage has been performed(ii) What have Venus and Cupid done after failing in their plan

After being failure of their plan Venus who is a very passionate deity and who is the mistress of Mars (the god of war) has gone back while here ill-tempered son Cupid has broken his arrows of love in his state of desperation(iii) What has Cupid firmly decided

Cupid is feeling so disappointed that he has firmly decided to shoot no more arrows to arouse love in human hearts but to spend his time playing with sparrows Thus he would now become just a boy and would give up his original function of shooting arrows on human beings to make them fall in love(iv) What vow had Ceres taken How did Ceres feel at the abduction

After the abduction of her daughter Prosperina by Pluto Ceres had taken a vow to always keep away from the disgraceful company of Venus and her blind son Cupid the god of love Ceres felt deeply distressed when Pluto had carried off her daughter and had made her his wife by force(v) Why has Ceres not forgiven Venus and her blind son For what do Ceres want to be sure

As the abduction had been manipulated by Venus the goddess of beauty and love and her blind son Cupid Ceres has never forgiven them for their part in the whole plot Ceres wants to be sure that she would not have to meet Venus and Cupid who had engineered the abduction of her daughter Prosperina

AS THIS lsquoMASQUErsquo SCENE IS VERY IMPORTANT IN THE PLAY THE PARAPHRASE OF THE ENTIRE PORTION OF MASQUE SCENE (Act IV Lines 58 to 143) IS GIVEN BELOW

IRIS Goddess of RainbowCERES Goddess of Agriculture and all the fruits of the earth

(Nature growth prosperity rebirth ndash notions intimately connected to marriage)JUNO The majestic Queen of Heavens and wife of Jupiter (Jupiter is the king of Gods)

VENUS The Goddess of love CUPID Son of Venus PLUTO God of death (In the play referred by Shakespeare as lsquoDisrsquo which is a Roman name for Pluto)

ORIGINAL TEXT PARAPHRASEPROSPEROWellmdash

PROSPERONow come Ariel Let there be too many rather than too few

Now come my Ariel Bring a corollaryRather than want a spirit Appear and pertly[to Ferdinand and Miranda]No tongue all eyes Be silent

spirits in attendance Appear briskly

[to Ferdinand and Miranda]Look with your eyes but do not say a word

[Soft music] [Soft music][Enter Iris] [Enter Iris]

IRISCeres most bounteous lady thy rich leasOf wheat rye barley vetches oats and peasThy turfy mountains where live nibbling sheepAnd flat meads thatched with stover them to keepThy banks with pioned and twilled brimsWhich spongy April at thy hest betrimsTo make cold nymphs chaste crowns and thybroom-grovesWhose shadow the dismissegraved bachelor lovesBeing lass-lorn thy pole clipped vineyardAnd thy sea-marge sterile and rocky-hardWhere thou thyself dost airmdashthe Queen othrsquoSkyWhose watery arch and messenger am IBids thee leave these and with her sovereign grace[Juno appears] Here on this grass-plot in this very placeTo come and sport Her peacocks fly amainApproach rich Ceres her to entertain

IRISCeres most generous lady you are the cause of rich fields or fertile land where wheat rye barley beans oats and peas grow the grassy mountains where the sheep graze and the flat meadows covered with coarse hay to be used as fodder for cattleYour banks are covered with marsh-marigolds and reeds and the rainy April under your orders brings forth to make for the maids who are not in love beautiful crowns your woods where the broom flourishes and where the bachelor who has been dismissed by the maid he loved lies down being forsaken your vineyard in which the poles are embraced by the vines and the margin of the sea which is barren and rocky where you roam about to enjoy the fresh air ndash the queen of the sky (Juno) whose messenger I am besides being represented as the rainbow bids you leave all these and with her majesty here on this grassy plot in this very place come and sport her peacocks carry her fast in her chariot through the air and are making their way here approach rich Ceres to welcome her

[Enter Ariel as Ceres] [Enter Ariel as Ceres]

CERESHail many-coloured messenger that neerDost disobey the wife of JupiterWho with thy saffron wings upon my flowersDiffusest honey-drops refreshing showersAnd with each end of thy blue bow dost crownMy bosky acres and my unshrubbed downRich scarf to my proud earth Why hath thy queenSummoned me hither to this short-grassed green

CERESWelcome rainbow that never dared disobey Juno the wife of Jupiter who with your orange coloured rays spread honey-drops refreshing showers And with each end of thy blue bow drown my bushy acres and my hilly country which is free from shrubs you thus forming a rich scarf Why has your queen called me here to this place covered with short grass

IRISA contract of true love to celebrateAnd some donation freely to estateOn the blest lovers

IRISI have called you to celebrate a contract of true love and bestow some liberal gift upon the blessed lovers

ORIGINAL TEXT PARAPHRASECERESTell me heavenly bowIf Venus or her son as thou dost knowDo now attend the queen Since they did plotThe means that dusky Dis my daughter gotHer and her blind boys scandaled companyI have forsworn

CERESTell me heavenly bow if Venus the Goddess of love or Cupid her son and pedlar of passion at this time attend the heavenly queen Juno because you are sure to know Since the day they conspired against me and dark Pluto took away my daughter here and Cupidrsquos disgraceful company I have left off

IRISOf her societyBe not afraid I met her deityCutting the clouds towards Pathos and her sonDove-drawn with her Here thought they to have doneSome wanton charm upon this man and miad

IRISBe not afraid of her company I met her deity moving on the clouds towards Paphos the sacred home of Venus on the island of Cyprus along with her son on her chariot drawn by doves Here they contemplated to exercise a charm upon this man and maid producing

Whose vows are that no bed-right shall be paidTill Hymens torch be lightedmdashbut in vainMarss hot minion is returned againHer waspish-headed son has broke his arrowsSwears he will shoot no more but play with sparrowsAnd be a boy right out

wantonness before the actual marriage ceremony but did not succeed Venus has returned her irritable son has broken his arrows and swears that he will give up his practice of trying to inspire love but play with sparrows and be a boy again

[Music is heard] [Music is heard]

CERESHighst queen of stateGreat Juno comes I know her by her gait

CERESHighest queen of state Great Juno there she comes I know here by her gait

[Enter Juno] [Enter Juno]

JUNOHow does my bounteous sister Go with meTo bless this twain that they may Prosperous beAnd honoured in their issue

JUNOHow are you doing my generous sister Come with me to bless this couple so that they may be prosperous and fortunate in their children

[They sing] [They sing]

JUNOHonour riches marriage-blessingLong continuance and increasingHourly joys be still upon youJuno sings her blessings upon you

JUNOMay honour riches happiness in marriage long continuance and increase of those boons ever rest upon you as hourly joys Juno showers down upon you her blessings in song

CERESEarths increase foison plentyBarns and garners never emptyVines and clustring bunches growingPlants and goodly burden bowingSpring come to you at the farthestIn the very end of harvestScarcity and want shall shun youCeresrsquo blessing so is on you

CERESMay you have the plenty of earthrsquos produce Your barns and granaries may never be empty Your vines may grow with clustering bunches Your fruit trees may be heavily laden with their fruit May there be continuous spring and harvest May scantiness and want leave you forever Such is the blessing of Ceres upon you

FERDINANDThis is a most majestic vision andHarmoniously charmingly May I be boldTo think these spirits

FERDINANDThis is a great vision and magically melodious Should I suppose the characters (taking part in the masque) are spirits

PROSPEROSpirits which by mine artI have from their confines calld to enactMy present fancies

PROSPEROYes they are spirits whom I have summoned from the regions to which they are confined to carry into effect my fanciful designs

ORIGINAL TEXT PARAPHRASEFERDINANDLet me live here everSo rare a wondered father and a wifeMakes this place paradise

FERDINANDI should like to live here forever Such a wise and wonderful father makes this place a paradise

[Juno and Ceres whisper and send Iris on employment] [Juno and Ceres whisper and send Iris on employment]

PROSPEROSweet now silence

PROSPEROMy dear Ferdinand speak no more Juno and Ceres are

Juno and Ceres whisper seriouslyTheres something else to do Hush and be muteOr else our spell is marred

whispering with a solemn look There is something else coming Silence Or else our magic will be spoilt

IRISYour nymphs called naiads of the wandering brooksWith your sedged crowns and over-harmless looksLeave your crisp channels and on this green landAnswer your summons Juno does commandCome temperate nymphs and help to celebrateA contract of true love Be not too late

IRISYou nymphs called Naiads denizens (M inhabitants) of the running stream with your chaplets of sedge and ever-helpful looks leave your wrinkled channels and on the green land answer the summons sent to you Juno has ordered some chaste nymphs and help to celebrate a noble and true marriage Donrsquot delay

[Enter certain nymphs] [Enter certain nymphs]You sunburnt sicklemen of August wearyCome hither from the furrow and be merryMake holiday your rye-straw hats put onAnd these fresh nymphs encounter every oneIn country footing

You sunburnt harvesters weary from the effects of the heat in August come here from the furrowed land and rejoice Make holiday with your rye-straw hats upon you and meet these fresh nymphs and join in country dancing

[Enter certain reapers properly habited They join with the nymphs in a graceful dance towards the end whereof Prospero starts suddenly and speaks]

[Enter certain reapers properly habited They join with the nymphs in a graceful dance towards the end whereof Prospero starts suddenly and speaks]

PROSPERO[aside] I had forgot that foul conspiracyOf the beast Caliban and his confederatesAgainst my life The minute of their plotIs almost come [to the spirits]Well done Avoidno more

PROSPERO(Aside)I had forgotten the wicked conspiracy of the beast Caliban and his accomplices against my life the time of their plot has almost arrived ndash (To the Spirits) well done depart no more of this

[To a strange hollow and confused noise the spirits heavily vanish]

[The spirits depart]

ORIGINAL TEXT PARAPHRASEFERDINANDLet me live here everSo rare a wondered father and a wifeMakes this place paradise

FERDINANDI should like to live here forever Such a wise and wonderful father makes this place a paradise

[Juno and Ceres whisper and send Iris on employment] [Juno and Ceres whisper and send Iris on employment]

PROSPEROSweet now silenceJuno and Ceres whisper seriouslyTheres something else to do Hush and be muteOr else our spell is marred

PROSPEROMy dear Ferdinand speak no more Juno and Ceres are whispering with a solemn look There is something else coming Silence Or else our magic will be spoilt

IRISYour nymphs called naiads of the wandering brooksWith your sedged crowns and over-harmless looksLeave your crisp channels and on this green landAnswer your summons Juno does commandCome temperate nymphs and help to celebrateA contract of true love Be not too late

IRISYou nymphs called Naiads denizens (M inhabitants) of the running stream with your chaplets of sedge and ever-helpful looks leave your wrinkled channels and on the green land answer the summons sent to you Juno has ordered some chaste nymphs and help to celebrate a noble and true marriage Donrsquot delay

[Enter certain nymphs] [Enter certain nymphs]You sunburnt sicklemen of August wearyCome hither from the furrow and be merryMake holiday your rye-straw hats put onAnd these fresh nymphs encounter every oneIn country footing

You sunburnt harvesters weary from the effects of the heat in August come here from the furrowed land and rejoice Make holiday with your rye-straw hats upon you and meet these fresh nymphs and join in country dancing

[Enter certain reapers properly habited They join with the nymphs in a graceful dance towards the end whereof Prospero starts suddenly and speaks]

[Enter certain reapers properly habited They join with the nymphs in a graceful dance towards the end whereof Prospero starts suddenly and speaks]

PROSPERO[aside] I had forgot that foul conspiracyOf the beast Caliban and his confederatesAgainst my life The minute of their plotIs almost come [to the spirits]Well done Avoidno more

PROSPERO(Aside)I had forgotten the wicked conspiracy of the beast Caliban and his accomplices against my life the time of their plot has almost arrived ndash (To the Spirits) well done depart no more of this

[To a strange hollow and confused noise the spirits heavily vanish]

[The spirits depart]

Ac-12 27420 topic Revaluation of Assets and Liabilities

REVALUATION OF ASSETS AND LIABILITIES

On admission of a new partner the firm stands reconstituted and consequently the assets are revalued and liabilities are reassessed It is necessary to show the true position of the firm at the time of admission of a new partner If the values of the assets are raised gain will increase the capital of the existing partners Similarly any decrease in the value of assets ie loss will decrease the capital of the existing partners For this purpose alsquoRevaluation Accountrsquo is prepared This account is credited with all increases in the value of assets and decrease in the value of liabilities It is debited with decrease on account of value of assets and increase in the value of liabilities The balance of this account shows a gain or loss on revaluation which is transferred to the existing partnerrsquos capital account in existing profit sharing ratioAccounting for Revaluation of Assets and Liabilities when there is a Changein the Profit Sharing Ratio of Existing PartnersAssets and liabilities of a firm must also be revalued at the time of change in profit sharing ratio of existing partners The reason is that the realisable or actual value of assets and liabilities may be different from those shown in the Balance Sheet It is possible that with the passage of time some of the assets might have appreciated in value while the value of certain other assets might have decreased and no record has been made of such changes in the books of accounts Similarly there may be some unrecorded assets amp libilities that may have to be accounted for Revaluation of assets and reassessments of liabilities becomes necessary because the change in the

value of assets and liabilities belongs to the period to change in profit sharing ratio and hence must be shared by the partners in their old profit sharing ratio Revaluation of assets and reassessment of liabilities may be given effect to in two different ways (a) When revised values are to be recorded in the books and(b) When revised values are not to be recorded in the books

When revised values are to be recorded in the booksIn such a case revaluation of assets and reassessment of liabilities is done with the help of a new account called lsquoRevaluation Accountrsquo Sometimes this account is also called as lsquoProfit amp Loss Adjustment Acrsquo If there is a loss due to revaluation revaluation account is debited and if the revaluation results in a profit the revaluation account is credited The following journal entries made for this purpose are

(i) For increase in the value of assetsAsset Ac Dr (individually)To Revaluation Ac(ii) For decrease in the value of AssetRevaluation Ac Dr (individually)To Asset Ac[Decrease in the value of assets](iii) For increase in the value of LiabilitiesRevaluation Ac Dr (individually)To Liabilities Ac[Increase in the value of Liabilities](iv) For decrease in the value of LiabilitiesLiabilities Ac DrTo Revaluation Ac[Decrease in the value of Liabilities](v) For unrecorded AssetsAsset Ac [unrecorded] DrTo Revaluation Ac[Unrecorded asset recorded at actual value](vi) For unrecorded Liability Revaluation Ac DrTo Liability Ac [unrecorded][Unrecorded Liability recorded at actual value](vii) For transfer of gain on revaluationRevaluation Ac DrTo Existing Partnerrsquos CapitalCurrent Ac[Profit on revaluation transferred to capital account in existing ratio](viii) For transfer of loss on revaluationExisting Partnerrsquos CapitalCurrent Ac DrTo Revaluation Ac[Loss on revaluation transferred to capital account in existing ratio](a) When revaluation account shows gain Revaluation Ac DrTo Partnerrsquos Capital Ac (Old Profit Sharing Ratio)(Profit on revaluation credited to Partnerrsquos Capital Ac)(b) Above entry is reversed when revaluation account shows loss Partners Capital Acs (Old Profit Sharing Ratio) DrTo Revaluation Ac(Loss on revaluation debited to Partnerrsquos Capital Acs)

Proforma of Revaluation Account is given as under

Revaluation Account

Dr Cr Particulars ` Amount Particulars ` Amount To Decrease in value of assets By Increase in value of assets To Increase in value of liabilities By Decrease in value of liabilities To Unrecorded liabilities By Unrecorded assets To Gain on Revaluation (Transferred) By Loss on Revalution (Transferred)

ECO ndash12 2742020Topic- ELASTICITY OF DEMAND

CHAPTER - ELASTICITY OF DEMANDMEANINGDemand for a commodity is affected by many factors such as its price price of related goods income of its buyer tastes and preferences etc Elasticity means degree of response Elasticity of demand means degree of responsiveness of demand Demand for a commodity responds to change in price price of related goods income etc So we have three dimensions of elasticity of demandDIMENSION OF ELASTICITY OF DEMAND TYPES OF ELASTICITY OF DEMAND

Price elasticity of demand Income elasticity of demand Cross Elasticity of demand

Price elasticity of demand Price elasticity of demand means degree of responsiveness of demand for a commodity to the change in its price For example if demand for a commodity rises by 10 due to 5 fall in its price Price elasticity of demand (ep)=Percentage change in quantity demanded Percentage change in price of the commodity = 10 ( -)5 = ( - )2Note that ep will always be negative due to inverse relationship of price and quantity demanded

(ii) Income elasticity of demand Income elasticity of demand refers to the degree of responsiveness of demand for a commodity to the change in income of its buyer Suppose income of buyer rises by 10 and his demand for a commodity rises by 20 then Income elasticity of demand (ey)= change in quantity demanded change in price of the commodity =20 10 = 2

Cross Elasticity of demandCross elasticity of demand means the degree of responsiveness of demand for a commodity to the change in price of its related goods (substitute goods or complementary goods) Suppose demand for a commodity rises by 10 due to 5 rise in price of its substitute good then Cross elasticity of demand (ec) = change in quantity demanded change in price of related good = 10 2 = 5 (Tastes and preferences cannot be expressed numerically So elasticity ofdemand cannot be numerically expressed)

  • Chapter 1 Force (Summary)
  • Distinguish between external sovereignty and internal sovereignty
    • NAND Gate
      • Logic diagram
      • Truth Table
        • NOR Gate
          • Logic diagram
          • Truth Table
            • XOR Gate
              • Logic diagram
              • Truth Table
                • XNOR Gate
                  • Logic diagram
                  • Truth Table
                      • Physics
                      • Chapter 1 Electric Field ( Electric Dipole) (Summary)
Page 42:  · Web viewWe all know that Nouns are divided into two parts: common noun and proper noun.Apart from common and proper noun, we will also study about collective noun and compound

Mathematics

Continuity and differentiability

Recall Definition of ContinuityLet f(x) be a single valued function of x and x=a be a point in the domain of definition of the function The function is said to be continuous at x=a ifi) f(c) is defined ie f(x) has a definite finite value at x=cii) lim xrarra f(x) exists andiii) lim xrarra f(x) =f(a) In other words f(x) is said to be continuous at x=a if lim xrarra+ f(x)= lim xrarra- f(x) = f(a) Or f(a+0) =f(a-0) =f(a) Or lim hrarr0 f(a+h)= f(a) Algebra of continuous functionsNow we will study some algebra of continuous functions Theorem 1 Suppose f and g be two real functions continuous at a real number c Then(1) f + g is continuous at x = c(2) f ndash g is continuous at x = c(3) f g is continuous at x = c(4) (fg) is continuous at x = c (provided g (c) ne 0)

Example 1 Prove that every rational function is continuousSolution Recall that every rational function f is given byf(x)=[p(x) q(x) ] q(x)ne0where p and q are polynomial functions The domain of f is all real numbers except points at which q is zero Since polynomial functions are continuous f is continuous by (4) of Theorem 1Example 2Discuss the continuity of sine functionSolution To see this we use the following factslim xrarr0 sin x =0Now observe that f (x) = sin x is defined for every real number Let c be a real number Put x = c + h If x rarr c we know that h rarr 0 Therefore lim xrarrc f(x)

= lim xrarrc sin x= lim hrarr0 sin(c+h) =lim hrarr0 [sin c cos h + cos c sin h ]=lim hrarr0 (sin c cos h) + lim hrarr0 (cos c sin h) = sin c +0=sin c = f(c) Thus lim xrarrc f(x) = f(c) and hence f is a constant function Exercise Prove that the function f(x) = x2 +2x is continuous for every real value of x [Hints show that lim xrarra+ f(x) = lim xrarra- f(x) = f(a) ]

Biology Reproduction in Flowering plants We will discuss about megasporoangium

megasporagenesis and female gametophyte

Q4 Describe the structure of megasporangium

Ovule is attached to the placenta by astalk called funicle

Each ovule has one two or three protectivecoverings called integuments

At the tip of integuments a small openingcalled micropyle is organised

Opposite to the micropylar end is the chalaza

Within the integuments a mass of cellsnucellusand inside it embryo sac orfemale gametophyte is present

Q5 Describe a mature embryosacamp its formation

In most of the flowering plants only oneof the 4 megaspores formed as a result ofmegasporogenesis that is functional while theother three degenerate

The

functional megaspore develops into thefemale gametophyte

Formation The nucleus

of the functional megasporedivides mitotically to form two nuclei first andthen two more sequential mitotic nucleardivisions result in the formation of four ampthen eight nucleate stages of embryo sac

Six of the eight nucleus are surrounded bycell walls and organised into cells

The remaining two nuclei called polar nuclei are found below the egg apparatus in the largecentral cell

Three cells consisting of two synergids amp one egg cell present bottom of

embryo sac Three cells

at the chalazal as antipodal cells

Two polar nuclei together present in large central cell

HISTORY

TOWARDS INDEPENDENCE AND PARTITION THE LAST PHASE(1935-1947)SUB TOPIC NATIONAL MOVEMENTS DURING THE SECOND WORLD WAR

Spread of Quit India Movement On 9th August 1942Gandhiji and other Congress leaders were arrested The Congress was declared illegal The news of the arrest of all leaders marked the beginning of a widespread movement of India It was not possible for such a movement to remain peacefulBut the arrest of the all notable congress leaders virtually left the movement in the hands of the mass The movement took the form of violent and militant outbreakBesides congressmen revolutionaries also were very active in the movement The Congress Socialist group also played a prominent role

1 Question Why did the British authority arrest the Congress leaders on 9 th August 1942Answer Congress Working committee adopted the Quit India resolution which was to be ratified at the Bombay AICC meeting in 8th August 1942 They decided to launch a mass struggle on non-violent lines Gandhiji gave a clarion call to all section of the people rdquoKarenge ya Marengerdquo (do or die) Congress leaders gave the call to driving out

the British from IndiaViceroy had taken strong action against the Quit India movement Gandhiji and all the leaders of Congress were arrested

2 Question How did Quit India Movement spread out all over IndiaAnswer The news of the leaders lsquo arrest marked the beginning of a widespread movement to remain peacefulThe movement took form of violent outbreak There were widespread cutting of telephone and Telegraph wires damaging railway lines raising barricades in cities and towns and other forms of violent demonstations

Question Name the leaders of Congress

Socialist group played a prominent part Notable among the Jayprakash Narayan Rammonohar Lohia Aruna Asaf Ali

Political science

Topic-Franchise and Representation

Summary

The election commission

The Election Commission of India is an autonomous constitutional authority responsible for administering Union and State election processes in India The body administers elections to the Lok Sabha Rajya Sabha State Legislative Assemblies in India and the offices of the President and Vice President in the country

Functions of election commission-

India is a sovereign socialist secular democratic republic Democracy runs like a golden thread in the social economic and political fabric woven by the Constitution given by lsquoWe the People of Indiarsquo unto ourselves The concept of democracy as visualised by the Constitution pre-supposes the representation of the people in Parliament and State legislatures by the method of election The Supreme Court has held that democracy is one of the inalienable basic features of the Constitution of India and forms part of its basic structure The Constitution of India adopted a Parliamentary form of government Parliament consists of the President of India and the two Houses mdash Rajya Sabha and Lok Sabha India being a Union of states has separate state legislatures for each state State legislatures consist of the Governor and two Houses mdash Legislative Council and Legislative Assembly mdash in seven states namely Andhra Pradesh Telangana Bihar Jammu amp Kashmir Karnataka Maharashtra and Uttar Pradesh and of the Governor and the state Legislative Assembly in the remaining 22 states Apart from the above two out of the seven Union Territories namely National Capital Territory of Delhi and Puducherry also have their Legislative Assemblies

ExecutionShort notes-Election commissionFunctions of election commission

Homework- Learn

Computer

Science

Computer hardware NAND Gate

A NOT-AND operation is known as NAND operation It has n input (n gt= 2) and one output

Logic diagram

Truth Table

NOR Gate

A NOT-OR operation is known as NOR operation It has n input (n gt= 2) and one output

Logic diagram

Truth Table

XOR Gate

XOR or Ex-OR gate is a special type of gate It can be used in the half

adder full adder and subtractor The exclusive-OR gate is abbreviated as EX-OR gate or sometime as X-OR gate It has n input (n gt= 2) and one output

Logic diagram

Truth Table

XNOR Gate

XNOR gate is a special type of gate It can be used in the half adder full adder and subtractor The exclusive-NOR gate is abbreviated as EX-NOR gate or sometime as X-NOR gate It has n input (n gt= 2) and one output

Logic diagram

Truth Table

Physics

Chapter 1 Electric Field ( Electric Dipole) (Summary)

Here we will derive Expression of electric field at broad side

On position of dipole

Execution

Q With the help of a labelled diagram obtain an expression for the electric field intensity E at any point on the equitorial line ( broad-side on position) of an electric dipole

Ans

E1 E1sinθ

E θ P E1 θ

( r2+L2)12 E2 E

r E2 E2sinθ

-q θ L O L +qA B

Let us consider that the point P is situated on the right bisector of the dipole AB at a distance r meter from its midpoint O

Let E1 and E2 be the electric field intensities of the electric field at P due to charge +q and ndashq of the dipole resp The distance of P from each charge is ( r2+L2)12

So E1 = 14 πϵ q

(r 2+L 2) away from +q

E2 = 14 πϵ q

(r 2+L 2) towards ndashq

The magnitudes of E1 and E2 are equal but directions are different Now resolving E1 and E2 into two components parallel and perpendicular to AB we get

The components perpendicular to AB E1sinθ and E2sinθ cancel each other because they are equal and opposite

The components parallel to AB are E1cosθ and E2 cosθ are in same direction and add up

So resultant intensity of electric field at the point P is

E = E1cosθ + E2 cosθ

E = 14 πϵ q

(r 2+L 2) 2 cosθ

Now from fig we have cosθ =BOBP = L (r2+L2)12

So we get E = 14 πϵ 2qL ( r2+L2)32

Now electric dipole moment p= 2qL

So E = 14 πϵ p ( r2+L2)32

HW Find the expression of Electric field as done here but this time take r gtgt 2L

Also find the expression of torque experience by a dipole

(Hint Electric force experienced by charges of dipole in electric field is qE each Let θ be the angle which dipole makes with electric lines of force then perpendicular distance between two charges is 2Lsinθ Then torque = force x perp distance = qE x 2L sinθ So τ=pE sinθ where p =2qL )

STUDY MATERIAL

Class XIISubject Eng Literature (The Tempest ndash William Shakespeare) Topic Act IV Scene 1 Lines 84 to 133 (Iris hellip A contract of true love Be not too late ) Date 27th April 2020 (4th Period)

[Students should read the original play and also the paraphrase given in the school prescribed textbook]Summary Questions amp Answers

o Ceres soon appears and comes to know that she has been summoned to celebrate the contract of true love

o Ceres expresses her unwillingness to meet Venus and Cupid as she has shunned their company

o Ceres and Juno both bestow their blessings upon Ferdinand and Miranda with June gifting honour riches happiness in marriage and Ceres presents plenty of earthrsquos produce

o Iris summons the water-nymphs and reapers to come and celebrate a contract

(1) IRIS Of her society (Line 91-101)

Be not afraid I met her deity

Cutting the clouds towards Pathos and her sonDove-drawn with her Here thought they to have doneSome wanton charm upon this man and maidWhose vows are that no bed-right shall be paidTill Hymens torch be lightedmdashbut in vainMarss hot minion is returned againHer waspish-headed son has broke his arrowsSwears he will shoot no more but play with sparrowsAnd be a boy right out

(i) Where were Venus and Cupid seen flying How were they travelling Why did they want to join the marriage celebration of Ferdinand and Miranda

of true love

Venus and Cupid were seen flying through the air towards Paphos the famous city which is situated on the island of Cyprus They were travelling by air-borne chariot drawn by doves They certainly wanted to come here in order to play some amorous trick upon Ferdinand and Miranda who are under a vow not to gratify their physical desires till the holy ceremony of their marriage has been performed(ii) What have Venus and Cupid done after failing in their plan

After being failure of their plan Venus who is a very passionate deity and who is the mistress of Mars (the god of war) has gone back while here ill-tempered son Cupid has broken his arrows of love in his state of desperation(iii) What has Cupid firmly decided

Cupid is feeling so disappointed that he has firmly decided to shoot no more arrows to arouse love in human hearts but to spend his time playing with sparrows Thus he would now become just a boy and would give up his original function of shooting arrows on human beings to make them fall in love(iv) What vow had Ceres taken How did Ceres feel at the abduction

After the abduction of her daughter Prosperina by Pluto Ceres had taken a vow to always keep away from the disgraceful company of Venus and her blind son Cupid the god of love Ceres felt deeply distressed when Pluto had carried off her daughter and had made her his wife by force(v) Why has Ceres not forgiven Venus and her blind son For what do Ceres want to be sure

As the abduction had been manipulated by Venus the goddess of beauty and love and her blind son Cupid Ceres has never forgiven them for their part in the whole plot Ceres wants to be sure that she would not have to meet Venus and Cupid who had engineered the abduction of her daughter Prosperina

AS THIS lsquoMASQUErsquo SCENE IS VERY IMPORTANT IN THE PLAY THE PARAPHRASE OF THE ENTIRE PORTION OF MASQUE SCENE (Act IV Lines 58 to 143) IS GIVEN BELOW

IRIS Goddess of RainbowCERES Goddess of Agriculture and all the fruits of the earth

(Nature growth prosperity rebirth ndash notions intimately connected to marriage)JUNO The majestic Queen of Heavens and wife of Jupiter (Jupiter is the king of Gods)

VENUS The Goddess of love CUPID Son of Venus PLUTO God of death (In the play referred by Shakespeare as lsquoDisrsquo which is a Roman name for Pluto)

ORIGINAL TEXT PARAPHRASEPROSPEROWellmdash

PROSPERONow come Ariel Let there be too many rather than too few

Now come my Ariel Bring a corollaryRather than want a spirit Appear and pertly[to Ferdinand and Miranda]No tongue all eyes Be silent

spirits in attendance Appear briskly

[to Ferdinand and Miranda]Look with your eyes but do not say a word

[Soft music] [Soft music][Enter Iris] [Enter Iris]

IRISCeres most bounteous lady thy rich leasOf wheat rye barley vetches oats and peasThy turfy mountains where live nibbling sheepAnd flat meads thatched with stover them to keepThy banks with pioned and twilled brimsWhich spongy April at thy hest betrimsTo make cold nymphs chaste crowns and thybroom-grovesWhose shadow the dismissegraved bachelor lovesBeing lass-lorn thy pole clipped vineyardAnd thy sea-marge sterile and rocky-hardWhere thou thyself dost airmdashthe Queen othrsquoSkyWhose watery arch and messenger am IBids thee leave these and with her sovereign grace[Juno appears] Here on this grass-plot in this very placeTo come and sport Her peacocks fly amainApproach rich Ceres her to entertain

IRISCeres most generous lady you are the cause of rich fields or fertile land where wheat rye barley beans oats and peas grow the grassy mountains where the sheep graze and the flat meadows covered with coarse hay to be used as fodder for cattleYour banks are covered with marsh-marigolds and reeds and the rainy April under your orders brings forth to make for the maids who are not in love beautiful crowns your woods where the broom flourishes and where the bachelor who has been dismissed by the maid he loved lies down being forsaken your vineyard in which the poles are embraced by the vines and the margin of the sea which is barren and rocky where you roam about to enjoy the fresh air ndash the queen of the sky (Juno) whose messenger I am besides being represented as the rainbow bids you leave all these and with her majesty here on this grassy plot in this very place come and sport her peacocks carry her fast in her chariot through the air and are making their way here approach rich Ceres to welcome her

[Enter Ariel as Ceres] [Enter Ariel as Ceres]

CERESHail many-coloured messenger that neerDost disobey the wife of JupiterWho with thy saffron wings upon my flowersDiffusest honey-drops refreshing showersAnd with each end of thy blue bow dost crownMy bosky acres and my unshrubbed downRich scarf to my proud earth Why hath thy queenSummoned me hither to this short-grassed green

CERESWelcome rainbow that never dared disobey Juno the wife of Jupiter who with your orange coloured rays spread honey-drops refreshing showers And with each end of thy blue bow drown my bushy acres and my hilly country which is free from shrubs you thus forming a rich scarf Why has your queen called me here to this place covered with short grass

IRISA contract of true love to celebrateAnd some donation freely to estateOn the blest lovers

IRISI have called you to celebrate a contract of true love and bestow some liberal gift upon the blessed lovers

ORIGINAL TEXT PARAPHRASECERESTell me heavenly bowIf Venus or her son as thou dost knowDo now attend the queen Since they did plotThe means that dusky Dis my daughter gotHer and her blind boys scandaled companyI have forsworn

CERESTell me heavenly bow if Venus the Goddess of love or Cupid her son and pedlar of passion at this time attend the heavenly queen Juno because you are sure to know Since the day they conspired against me and dark Pluto took away my daughter here and Cupidrsquos disgraceful company I have left off

IRISOf her societyBe not afraid I met her deityCutting the clouds towards Pathos and her sonDove-drawn with her Here thought they to have doneSome wanton charm upon this man and miad

IRISBe not afraid of her company I met her deity moving on the clouds towards Paphos the sacred home of Venus on the island of Cyprus along with her son on her chariot drawn by doves Here they contemplated to exercise a charm upon this man and maid producing

Whose vows are that no bed-right shall be paidTill Hymens torch be lightedmdashbut in vainMarss hot minion is returned againHer waspish-headed son has broke his arrowsSwears he will shoot no more but play with sparrowsAnd be a boy right out

wantonness before the actual marriage ceremony but did not succeed Venus has returned her irritable son has broken his arrows and swears that he will give up his practice of trying to inspire love but play with sparrows and be a boy again

[Music is heard] [Music is heard]

CERESHighst queen of stateGreat Juno comes I know her by her gait

CERESHighest queen of state Great Juno there she comes I know here by her gait

[Enter Juno] [Enter Juno]

JUNOHow does my bounteous sister Go with meTo bless this twain that they may Prosperous beAnd honoured in their issue

JUNOHow are you doing my generous sister Come with me to bless this couple so that they may be prosperous and fortunate in their children

[They sing] [They sing]

JUNOHonour riches marriage-blessingLong continuance and increasingHourly joys be still upon youJuno sings her blessings upon you

JUNOMay honour riches happiness in marriage long continuance and increase of those boons ever rest upon you as hourly joys Juno showers down upon you her blessings in song

CERESEarths increase foison plentyBarns and garners never emptyVines and clustring bunches growingPlants and goodly burden bowingSpring come to you at the farthestIn the very end of harvestScarcity and want shall shun youCeresrsquo blessing so is on you

CERESMay you have the plenty of earthrsquos produce Your barns and granaries may never be empty Your vines may grow with clustering bunches Your fruit trees may be heavily laden with their fruit May there be continuous spring and harvest May scantiness and want leave you forever Such is the blessing of Ceres upon you

FERDINANDThis is a most majestic vision andHarmoniously charmingly May I be boldTo think these spirits

FERDINANDThis is a great vision and magically melodious Should I suppose the characters (taking part in the masque) are spirits

PROSPEROSpirits which by mine artI have from their confines calld to enactMy present fancies

PROSPEROYes they are spirits whom I have summoned from the regions to which they are confined to carry into effect my fanciful designs

ORIGINAL TEXT PARAPHRASEFERDINANDLet me live here everSo rare a wondered father and a wifeMakes this place paradise

FERDINANDI should like to live here forever Such a wise and wonderful father makes this place a paradise

[Juno and Ceres whisper and send Iris on employment] [Juno and Ceres whisper and send Iris on employment]

PROSPEROSweet now silence

PROSPEROMy dear Ferdinand speak no more Juno and Ceres are

Juno and Ceres whisper seriouslyTheres something else to do Hush and be muteOr else our spell is marred

whispering with a solemn look There is something else coming Silence Or else our magic will be spoilt

IRISYour nymphs called naiads of the wandering brooksWith your sedged crowns and over-harmless looksLeave your crisp channels and on this green landAnswer your summons Juno does commandCome temperate nymphs and help to celebrateA contract of true love Be not too late

IRISYou nymphs called Naiads denizens (M inhabitants) of the running stream with your chaplets of sedge and ever-helpful looks leave your wrinkled channels and on the green land answer the summons sent to you Juno has ordered some chaste nymphs and help to celebrate a noble and true marriage Donrsquot delay

[Enter certain nymphs] [Enter certain nymphs]You sunburnt sicklemen of August wearyCome hither from the furrow and be merryMake holiday your rye-straw hats put onAnd these fresh nymphs encounter every oneIn country footing

You sunburnt harvesters weary from the effects of the heat in August come here from the furrowed land and rejoice Make holiday with your rye-straw hats upon you and meet these fresh nymphs and join in country dancing

[Enter certain reapers properly habited They join with the nymphs in a graceful dance towards the end whereof Prospero starts suddenly and speaks]

[Enter certain reapers properly habited They join with the nymphs in a graceful dance towards the end whereof Prospero starts suddenly and speaks]

PROSPERO[aside] I had forgot that foul conspiracyOf the beast Caliban and his confederatesAgainst my life The minute of their plotIs almost come [to the spirits]Well done Avoidno more

PROSPERO(Aside)I had forgotten the wicked conspiracy of the beast Caliban and his accomplices against my life the time of their plot has almost arrived ndash (To the Spirits) well done depart no more of this

[To a strange hollow and confused noise the spirits heavily vanish]

[The spirits depart]

ORIGINAL TEXT PARAPHRASEFERDINANDLet me live here everSo rare a wondered father and a wifeMakes this place paradise

FERDINANDI should like to live here forever Such a wise and wonderful father makes this place a paradise

[Juno and Ceres whisper and send Iris on employment] [Juno and Ceres whisper and send Iris on employment]

PROSPEROSweet now silenceJuno and Ceres whisper seriouslyTheres something else to do Hush and be muteOr else our spell is marred

PROSPEROMy dear Ferdinand speak no more Juno and Ceres are whispering with a solemn look There is something else coming Silence Or else our magic will be spoilt

IRISYour nymphs called naiads of the wandering brooksWith your sedged crowns and over-harmless looksLeave your crisp channels and on this green landAnswer your summons Juno does commandCome temperate nymphs and help to celebrateA contract of true love Be not too late

IRISYou nymphs called Naiads denizens (M inhabitants) of the running stream with your chaplets of sedge and ever-helpful looks leave your wrinkled channels and on the green land answer the summons sent to you Juno has ordered some chaste nymphs and help to celebrate a noble and true marriage Donrsquot delay

[Enter certain nymphs] [Enter certain nymphs]You sunburnt sicklemen of August wearyCome hither from the furrow and be merryMake holiday your rye-straw hats put onAnd these fresh nymphs encounter every oneIn country footing

You sunburnt harvesters weary from the effects of the heat in August come here from the furrowed land and rejoice Make holiday with your rye-straw hats upon you and meet these fresh nymphs and join in country dancing

[Enter certain reapers properly habited They join with the nymphs in a graceful dance towards the end whereof Prospero starts suddenly and speaks]

[Enter certain reapers properly habited They join with the nymphs in a graceful dance towards the end whereof Prospero starts suddenly and speaks]

PROSPERO[aside] I had forgot that foul conspiracyOf the beast Caliban and his confederatesAgainst my life The minute of their plotIs almost come [to the spirits]Well done Avoidno more

PROSPERO(Aside)I had forgotten the wicked conspiracy of the beast Caliban and his accomplices against my life the time of their plot has almost arrived ndash (To the Spirits) well done depart no more of this

[To a strange hollow and confused noise the spirits heavily vanish]

[The spirits depart]

Ac-12 27420 topic Revaluation of Assets and Liabilities

REVALUATION OF ASSETS AND LIABILITIES

On admission of a new partner the firm stands reconstituted and consequently the assets are revalued and liabilities are reassessed It is necessary to show the true position of the firm at the time of admission of a new partner If the values of the assets are raised gain will increase the capital of the existing partners Similarly any decrease in the value of assets ie loss will decrease the capital of the existing partners For this purpose alsquoRevaluation Accountrsquo is prepared This account is credited with all increases in the value of assets and decrease in the value of liabilities It is debited with decrease on account of value of assets and increase in the value of liabilities The balance of this account shows a gain or loss on revaluation which is transferred to the existing partnerrsquos capital account in existing profit sharing ratioAccounting for Revaluation of Assets and Liabilities when there is a Changein the Profit Sharing Ratio of Existing PartnersAssets and liabilities of a firm must also be revalued at the time of change in profit sharing ratio of existing partners The reason is that the realisable or actual value of assets and liabilities may be different from those shown in the Balance Sheet It is possible that with the passage of time some of the assets might have appreciated in value while the value of certain other assets might have decreased and no record has been made of such changes in the books of accounts Similarly there may be some unrecorded assets amp libilities that may have to be accounted for Revaluation of assets and reassessments of liabilities becomes necessary because the change in the

value of assets and liabilities belongs to the period to change in profit sharing ratio and hence must be shared by the partners in their old profit sharing ratio Revaluation of assets and reassessment of liabilities may be given effect to in two different ways (a) When revised values are to be recorded in the books and(b) When revised values are not to be recorded in the books

When revised values are to be recorded in the booksIn such a case revaluation of assets and reassessment of liabilities is done with the help of a new account called lsquoRevaluation Accountrsquo Sometimes this account is also called as lsquoProfit amp Loss Adjustment Acrsquo If there is a loss due to revaluation revaluation account is debited and if the revaluation results in a profit the revaluation account is credited The following journal entries made for this purpose are

(i) For increase in the value of assetsAsset Ac Dr (individually)To Revaluation Ac(ii) For decrease in the value of AssetRevaluation Ac Dr (individually)To Asset Ac[Decrease in the value of assets](iii) For increase in the value of LiabilitiesRevaluation Ac Dr (individually)To Liabilities Ac[Increase in the value of Liabilities](iv) For decrease in the value of LiabilitiesLiabilities Ac DrTo Revaluation Ac[Decrease in the value of Liabilities](v) For unrecorded AssetsAsset Ac [unrecorded] DrTo Revaluation Ac[Unrecorded asset recorded at actual value](vi) For unrecorded Liability Revaluation Ac DrTo Liability Ac [unrecorded][Unrecorded Liability recorded at actual value](vii) For transfer of gain on revaluationRevaluation Ac DrTo Existing Partnerrsquos CapitalCurrent Ac[Profit on revaluation transferred to capital account in existing ratio](viii) For transfer of loss on revaluationExisting Partnerrsquos CapitalCurrent Ac DrTo Revaluation Ac[Loss on revaluation transferred to capital account in existing ratio](a) When revaluation account shows gain Revaluation Ac DrTo Partnerrsquos Capital Ac (Old Profit Sharing Ratio)(Profit on revaluation credited to Partnerrsquos Capital Ac)(b) Above entry is reversed when revaluation account shows loss Partners Capital Acs (Old Profit Sharing Ratio) DrTo Revaluation Ac(Loss on revaluation debited to Partnerrsquos Capital Acs)

Proforma of Revaluation Account is given as under

Revaluation Account

Dr Cr Particulars ` Amount Particulars ` Amount To Decrease in value of assets By Increase in value of assets To Increase in value of liabilities By Decrease in value of liabilities To Unrecorded liabilities By Unrecorded assets To Gain on Revaluation (Transferred) By Loss on Revalution (Transferred)

ECO ndash12 2742020Topic- ELASTICITY OF DEMAND

CHAPTER - ELASTICITY OF DEMANDMEANINGDemand for a commodity is affected by many factors such as its price price of related goods income of its buyer tastes and preferences etc Elasticity means degree of response Elasticity of demand means degree of responsiveness of demand Demand for a commodity responds to change in price price of related goods income etc So we have three dimensions of elasticity of demandDIMENSION OF ELASTICITY OF DEMAND TYPES OF ELASTICITY OF DEMAND

Price elasticity of demand Income elasticity of demand Cross Elasticity of demand

Price elasticity of demand Price elasticity of demand means degree of responsiveness of demand for a commodity to the change in its price For example if demand for a commodity rises by 10 due to 5 fall in its price Price elasticity of demand (ep)=Percentage change in quantity demanded Percentage change in price of the commodity = 10 ( -)5 = ( - )2Note that ep will always be negative due to inverse relationship of price and quantity demanded

(ii) Income elasticity of demand Income elasticity of demand refers to the degree of responsiveness of demand for a commodity to the change in income of its buyer Suppose income of buyer rises by 10 and his demand for a commodity rises by 20 then Income elasticity of demand (ey)= change in quantity demanded change in price of the commodity =20 10 = 2

Cross Elasticity of demandCross elasticity of demand means the degree of responsiveness of demand for a commodity to the change in price of its related goods (substitute goods or complementary goods) Suppose demand for a commodity rises by 10 due to 5 rise in price of its substitute good then Cross elasticity of demand (ec) = change in quantity demanded change in price of related good = 10 2 = 5 (Tastes and preferences cannot be expressed numerically So elasticity ofdemand cannot be numerically expressed)

  • Chapter 1 Force (Summary)
  • Distinguish between external sovereignty and internal sovereignty
    • NAND Gate
      • Logic diagram
      • Truth Table
        • NOR Gate
          • Logic diagram
          • Truth Table
            • XOR Gate
              • Logic diagram
              • Truth Table
                • XNOR Gate
                  • Logic diagram
                  • Truth Table
                      • Physics
                      • Chapter 1 Electric Field ( Electric Dipole) (Summary)
Page 43:  · Web viewWe all know that Nouns are divided into two parts: common noun and proper noun.Apart from common and proper noun, we will also study about collective noun and compound

Biology Reproduction in Flowering plants We will discuss about megasporoangium

megasporagenesis and female gametophyte

Q4 Describe the structure of megasporangium

Ovule is attached to the placenta by astalk called funicle

Each ovule has one two or three protectivecoverings called integuments

At the tip of integuments a small openingcalled micropyle is organised

Opposite to the micropylar end is the chalaza

Within the integuments a mass of cellsnucellusand inside it embryo sac orfemale gametophyte is present

Q5 Describe a mature embryosacamp its formation

In most of the flowering plants only oneof the 4 megaspores formed as a result ofmegasporogenesis that is functional while theother three degenerate

The

functional megaspore develops into thefemale gametophyte

Formation The nucleus

of the functional megasporedivides mitotically to form two nuclei first andthen two more sequential mitotic nucleardivisions result in the formation of four ampthen eight nucleate stages of embryo sac

Six of the eight nucleus are surrounded bycell walls and organised into cells

The remaining two nuclei called polar nuclei are found below the egg apparatus in the largecentral cell

Three cells consisting of two synergids amp one egg cell present bottom of

embryo sac Three cells

at the chalazal as antipodal cells

Two polar nuclei together present in large central cell

HISTORY

TOWARDS INDEPENDENCE AND PARTITION THE LAST PHASE(1935-1947)SUB TOPIC NATIONAL MOVEMENTS DURING THE SECOND WORLD WAR

Spread of Quit India Movement On 9th August 1942Gandhiji and other Congress leaders were arrested The Congress was declared illegal The news of the arrest of all leaders marked the beginning of a widespread movement of India It was not possible for such a movement to remain peacefulBut the arrest of the all notable congress leaders virtually left the movement in the hands of the mass The movement took the form of violent and militant outbreakBesides congressmen revolutionaries also were very active in the movement The Congress Socialist group also played a prominent role

1 Question Why did the British authority arrest the Congress leaders on 9 th August 1942Answer Congress Working committee adopted the Quit India resolution which was to be ratified at the Bombay AICC meeting in 8th August 1942 They decided to launch a mass struggle on non-violent lines Gandhiji gave a clarion call to all section of the people rdquoKarenge ya Marengerdquo (do or die) Congress leaders gave the call to driving out

the British from IndiaViceroy had taken strong action against the Quit India movement Gandhiji and all the leaders of Congress were arrested

2 Question How did Quit India Movement spread out all over IndiaAnswer The news of the leaders lsquo arrest marked the beginning of a widespread movement to remain peacefulThe movement took form of violent outbreak There were widespread cutting of telephone and Telegraph wires damaging railway lines raising barricades in cities and towns and other forms of violent demonstations

Question Name the leaders of Congress

Socialist group played a prominent part Notable among the Jayprakash Narayan Rammonohar Lohia Aruna Asaf Ali

Political science

Topic-Franchise and Representation

Summary

The election commission

The Election Commission of India is an autonomous constitutional authority responsible for administering Union and State election processes in India The body administers elections to the Lok Sabha Rajya Sabha State Legislative Assemblies in India and the offices of the President and Vice President in the country

Functions of election commission-

India is a sovereign socialist secular democratic republic Democracy runs like a golden thread in the social economic and political fabric woven by the Constitution given by lsquoWe the People of Indiarsquo unto ourselves The concept of democracy as visualised by the Constitution pre-supposes the representation of the people in Parliament and State legislatures by the method of election The Supreme Court has held that democracy is one of the inalienable basic features of the Constitution of India and forms part of its basic structure The Constitution of India adopted a Parliamentary form of government Parliament consists of the President of India and the two Houses mdash Rajya Sabha and Lok Sabha India being a Union of states has separate state legislatures for each state State legislatures consist of the Governor and two Houses mdash Legislative Council and Legislative Assembly mdash in seven states namely Andhra Pradesh Telangana Bihar Jammu amp Kashmir Karnataka Maharashtra and Uttar Pradesh and of the Governor and the state Legislative Assembly in the remaining 22 states Apart from the above two out of the seven Union Territories namely National Capital Territory of Delhi and Puducherry also have their Legislative Assemblies

ExecutionShort notes-Election commissionFunctions of election commission

Homework- Learn

Computer

Science

Computer hardware NAND Gate

A NOT-AND operation is known as NAND operation It has n input (n gt= 2) and one output

Logic diagram

Truth Table

NOR Gate

A NOT-OR operation is known as NOR operation It has n input (n gt= 2) and one output

Logic diagram

Truth Table

XOR Gate

XOR or Ex-OR gate is a special type of gate It can be used in the half

adder full adder and subtractor The exclusive-OR gate is abbreviated as EX-OR gate or sometime as X-OR gate It has n input (n gt= 2) and one output

Logic diagram

Truth Table

XNOR Gate

XNOR gate is a special type of gate It can be used in the half adder full adder and subtractor The exclusive-NOR gate is abbreviated as EX-NOR gate or sometime as X-NOR gate It has n input (n gt= 2) and one output

Logic diagram

Truth Table

Physics

Chapter 1 Electric Field ( Electric Dipole) (Summary)

Here we will derive Expression of electric field at broad side

On position of dipole

Execution

Q With the help of a labelled diagram obtain an expression for the electric field intensity E at any point on the equitorial line ( broad-side on position) of an electric dipole

Ans

E1 E1sinθ

E θ P E1 θ

( r2+L2)12 E2 E

r E2 E2sinθ

-q θ L O L +qA B

Let us consider that the point P is situated on the right bisector of the dipole AB at a distance r meter from its midpoint O

Let E1 and E2 be the electric field intensities of the electric field at P due to charge +q and ndashq of the dipole resp The distance of P from each charge is ( r2+L2)12

So E1 = 14 πϵ q

(r 2+L 2) away from +q

E2 = 14 πϵ q

(r 2+L 2) towards ndashq

The magnitudes of E1 and E2 are equal but directions are different Now resolving E1 and E2 into two components parallel and perpendicular to AB we get

The components perpendicular to AB E1sinθ and E2sinθ cancel each other because they are equal and opposite

The components parallel to AB are E1cosθ and E2 cosθ are in same direction and add up

So resultant intensity of electric field at the point P is

E = E1cosθ + E2 cosθ

E = 14 πϵ q

(r 2+L 2) 2 cosθ

Now from fig we have cosθ =BOBP = L (r2+L2)12

So we get E = 14 πϵ 2qL ( r2+L2)32

Now electric dipole moment p= 2qL

So E = 14 πϵ p ( r2+L2)32

HW Find the expression of Electric field as done here but this time take r gtgt 2L

Also find the expression of torque experience by a dipole

(Hint Electric force experienced by charges of dipole in electric field is qE each Let θ be the angle which dipole makes with electric lines of force then perpendicular distance between two charges is 2Lsinθ Then torque = force x perp distance = qE x 2L sinθ So τ=pE sinθ where p =2qL )

STUDY MATERIAL

Class XIISubject Eng Literature (The Tempest ndash William Shakespeare) Topic Act IV Scene 1 Lines 84 to 133 (Iris hellip A contract of true love Be not too late ) Date 27th April 2020 (4th Period)

[Students should read the original play and also the paraphrase given in the school prescribed textbook]Summary Questions amp Answers

o Ceres soon appears and comes to know that she has been summoned to celebrate the contract of true love

o Ceres expresses her unwillingness to meet Venus and Cupid as she has shunned their company

o Ceres and Juno both bestow their blessings upon Ferdinand and Miranda with June gifting honour riches happiness in marriage and Ceres presents plenty of earthrsquos produce

o Iris summons the water-nymphs and reapers to come and celebrate a contract

(1) IRIS Of her society (Line 91-101)

Be not afraid I met her deity

Cutting the clouds towards Pathos and her sonDove-drawn with her Here thought they to have doneSome wanton charm upon this man and maidWhose vows are that no bed-right shall be paidTill Hymens torch be lightedmdashbut in vainMarss hot minion is returned againHer waspish-headed son has broke his arrowsSwears he will shoot no more but play with sparrowsAnd be a boy right out

(i) Where were Venus and Cupid seen flying How were they travelling Why did they want to join the marriage celebration of Ferdinand and Miranda

of true love

Venus and Cupid were seen flying through the air towards Paphos the famous city which is situated on the island of Cyprus They were travelling by air-borne chariot drawn by doves They certainly wanted to come here in order to play some amorous trick upon Ferdinand and Miranda who are under a vow not to gratify their physical desires till the holy ceremony of their marriage has been performed(ii) What have Venus and Cupid done after failing in their plan

After being failure of their plan Venus who is a very passionate deity and who is the mistress of Mars (the god of war) has gone back while here ill-tempered son Cupid has broken his arrows of love in his state of desperation(iii) What has Cupid firmly decided

Cupid is feeling so disappointed that he has firmly decided to shoot no more arrows to arouse love in human hearts but to spend his time playing with sparrows Thus he would now become just a boy and would give up his original function of shooting arrows on human beings to make them fall in love(iv) What vow had Ceres taken How did Ceres feel at the abduction

After the abduction of her daughter Prosperina by Pluto Ceres had taken a vow to always keep away from the disgraceful company of Venus and her blind son Cupid the god of love Ceres felt deeply distressed when Pluto had carried off her daughter and had made her his wife by force(v) Why has Ceres not forgiven Venus and her blind son For what do Ceres want to be sure

As the abduction had been manipulated by Venus the goddess of beauty and love and her blind son Cupid Ceres has never forgiven them for their part in the whole plot Ceres wants to be sure that she would not have to meet Venus and Cupid who had engineered the abduction of her daughter Prosperina

AS THIS lsquoMASQUErsquo SCENE IS VERY IMPORTANT IN THE PLAY THE PARAPHRASE OF THE ENTIRE PORTION OF MASQUE SCENE (Act IV Lines 58 to 143) IS GIVEN BELOW

IRIS Goddess of RainbowCERES Goddess of Agriculture and all the fruits of the earth

(Nature growth prosperity rebirth ndash notions intimately connected to marriage)JUNO The majestic Queen of Heavens and wife of Jupiter (Jupiter is the king of Gods)

VENUS The Goddess of love CUPID Son of Venus PLUTO God of death (In the play referred by Shakespeare as lsquoDisrsquo which is a Roman name for Pluto)

ORIGINAL TEXT PARAPHRASEPROSPEROWellmdash

PROSPERONow come Ariel Let there be too many rather than too few

Now come my Ariel Bring a corollaryRather than want a spirit Appear and pertly[to Ferdinand and Miranda]No tongue all eyes Be silent

spirits in attendance Appear briskly

[to Ferdinand and Miranda]Look with your eyes but do not say a word

[Soft music] [Soft music][Enter Iris] [Enter Iris]

IRISCeres most bounteous lady thy rich leasOf wheat rye barley vetches oats and peasThy turfy mountains where live nibbling sheepAnd flat meads thatched with stover them to keepThy banks with pioned and twilled brimsWhich spongy April at thy hest betrimsTo make cold nymphs chaste crowns and thybroom-grovesWhose shadow the dismissegraved bachelor lovesBeing lass-lorn thy pole clipped vineyardAnd thy sea-marge sterile and rocky-hardWhere thou thyself dost airmdashthe Queen othrsquoSkyWhose watery arch and messenger am IBids thee leave these and with her sovereign grace[Juno appears] Here on this grass-plot in this very placeTo come and sport Her peacocks fly amainApproach rich Ceres her to entertain

IRISCeres most generous lady you are the cause of rich fields or fertile land where wheat rye barley beans oats and peas grow the grassy mountains where the sheep graze and the flat meadows covered with coarse hay to be used as fodder for cattleYour banks are covered with marsh-marigolds and reeds and the rainy April under your orders brings forth to make for the maids who are not in love beautiful crowns your woods where the broom flourishes and where the bachelor who has been dismissed by the maid he loved lies down being forsaken your vineyard in which the poles are embraced by the vines and the margin of the sea which is barren and rocky where you roam about to enjoy the fresh air ndash the queen of the sky (Juno) whose messenger I am besides being represented as the rainbow bids you leave all these and with her majesty here on this grassy plot in this very place come and sport her peacocks carry her fast in her chariot through the air and are making their way here approach rich Ceres to welcome her

[Enter Ariel as Ceres] [Enter Ariel as Ceres]

CERESHail many-coloured messenger that neerDost disobey the wife of JupiterWho with thy saffron wings upon my flowersDiffusest honey-drops refreshing showersAnd with each end of thy blue bow dost crownMy bosky acres and my unshrubbed downRich scarf to my proud earth Why hath thy queenSummoned me hither to this short-grassed green

CERESWelcome rainbow that never dared disobey Juno the wife of Jupiter who with your orange coloured rays spread honey-drops refreshing showers And with each end of thy blue bow drown my bushy acres and my hilly country which is free from shrubs you thus forming a rich scarf Why has your queen called me here to this place covered with short grass

IRISA contract of true love to celebrateAnd some donation freely to estateOn the blest lovers

IRISI have called you to celebrate a contract of true love and bestow some liberal gift upon the blessed lovers

ORIGINAL TEXT PARAPHRASECERESTell me heavenly bowIf Venus or her son as thou dost knowDo now attend the queen Since they did plotThe means that dusky Dis my daughter gotHer and her blind boys scandaled companyI have forsworn

CERESTell me heavenly bow if Venus the Goddess of love or Cupid her son and pedlar of passion at this time attend the heavenly queen Juno because you are sure to know Since the day they conspired against me and dark Pluto took away my daughter here and Cupidrsquos disgraceful company I have left off

IRISOf her societyBe not afraid I met her deityCutting the clouds towards Pathos and her sonDove-drawn with her Here thought they to have doneSome wanton charm upon this man and miad

IRISBe not afraid of her company I met her deity moving on the clouds towards Paphos the sacred home of Venus on the island of Cyprus along with her son on her chariot drawn by doves Here they contemplated to exercise a charm upon this man and maid producing

Whose vows are that no bed-right shall be paidTill Hymens torch be lightedmdashbut in vainMarss hot minion is returned againHer waspish-headed son has broke his arrowsSwears he will shoot no more but play with sparrowsAnd be a boy right out

wantonness before the actual marriage ceremony but did not succeed Venus has returned her irritable son has broken his arrows and swears that he will give up his practice of trying to inspire love but play with sparrows and be a boy again

[Music is heard] [Music is heard]

CERESHighst queen of stateGreat Juno comes I know her by her gait

CERESHighest queen of state Great Juno there she comes I know here by her gait

[Enter Juno] [Enter Juno]

JUNOHow does my bounteous sister Go with meTo bless this twain that they may Prosperous beAnd honoured in their issue

JUNOHow are you doing my generous sister Come with me to bless this couple so that they may be prosperous and fortunate in their children

[They sing] [They sing]

JUNOHonour riches marriage-blessingLong continuance and increasingHourly joys be still upon youJuno sings her blessings upon you

JUNOMay honour riches happiness in marriage long continuance and increase of those boons ever rest upon you as hourly joys Juno showers down upon you her blessings in song

CERESEarths increase foison plentyBarns and garners never emptyVines and clustring bunches growingPlants and goodly burden bowingSpring come to you at the farthestIn the very end of harvestScarcity and want shall shun youCeresrsquo blessing so is on you

CERESMay you have the plenty of earthrsquos produce Your barns and granaries may never be empty Your vines may grow with clustering bunches Your fruit trees may be heavily laden with their fruit May there be continuous spring and harvest May scantiness and want leave you forever Such is the blessing of Ceres upon you

FERDINANDThis is a most majestic vision andHarmoniously charmingly May I be boldTo think these spirits

FERDINANDThis is a great vision and magically melodious Should I suppose the characters (taking part in the masque) are spirits

PROSPEROSpirits which by mine artI have from their confines calld to enactMy present fancies

PROSPEROYes they are spirits whom I have summoned from the regions to which they are confined to carry into effect my fanciful designs

ORIGINAL TEXT PARAPHRASEFERDINANDLet me live here everSo rare a wondered father and a wifeMakes this place paradise

FERDINANDI should like to live here forever Such a wise and wonderful father makes this place a paradise

[Juno and Ceres whisper and send Iris on employment] [Juno and Ceres whisper and send Iris on employment]

PROSPEROSweet now silence

PROSPEROMy dear Ferdinand speak no more Juno and Ceres are

Juno and Ceres whisper seriouslyTheres something else to do Hush and be muteOr else our spell is marred

whispering with a solemn look There is something else coming Silence Or else our magic will be spoilt

IRISYour nymphs called naiads of the wandering brooksWith your sedged crowns and over-harmless looksLeave your crisp channels and on this green landAnswer your summons Juno does commandCome temperate nymphs and help to celebrateA contract of true love Be not too late

IRISYou nymphs called Naiads denizens (M inhabitants) of the running stream with your chaplets of sedge and ever-helpful looks leave your wrinkled channels and on the green land answer the summons sent to you Juno has ordered some chaste nymphs and help to celebrate a noble and true marriage Donrsquot delay

[Enter certain nymphs] [Enter certain nymphs]You sunburnt sicklemen of August wearyCome hither from the furrow and be merryMake holiday your rye-straw hats put onAnd these fresh nymphs encounter every oneIn country footing

You sunburnt harvesters weary from the effects of the heat in August come here from the furrowed land and rejoice Make holiday with your rye-straw hats upon you and meet these fresh nymphs and join in country dancing

[Enter certain reapers properly habited They join with the nymphs in a graceful dance towards the end whereof Prospero starts suddenly and speaks]

[Enter certain reapers properly habited They join with the nymphs in a graceful dance towards the end whereof Prospero starts suddenly and speaks]

PROSPERO[aside] I had forgot that foul conspiracyOf the beast Caliban and his confederatesAgainst my life The minute of their plotIs almost come [to the spirits]Well done Avoidno more

PROSPERO(Aside)I had forgotten the wicked conspiracy of the beast Caliban and his accomplices against my life the time of their plot has almost arrived ndash (To the Spirits) well done depart no more of this

[To a strange hollow and confused noise the spirits heavily vanish]

[The spirits depart]

ORIGINAL TEXT PARAPHRASEFERDINANDLet me live here everSo rare a wondered father and a wifeMakes this place paradise

FERDINANDI should like to live here forever Such a wise and wonderful father makes this place a paradise

[Juno and Ceres whisper and send Iris on employment] [Juno and Ceres whisper and send Iris on employment]

PROSPEROSweet now silenceJuno and Ceres whisper seriouslyTheres something else to do Hush and be muteOr else our spell is marred

PROSPEROMy dear Ferdinand speak no more Juno and Ceres are whispering with a solemn look There is something else coming Silence Or else our magic will be spoilt

IRISYour nymphs called naiads of the wandering brooksWith your sedged crowns and over-harmless looksLeave your crisp channels and on this green landAnswer your summons Juno does commandCome temperate nymphs and help to celebrateA contract of true love Be not too late

IRISYou nymphs called Naiads denizens (M inhabitants) of the running stream with your chaplets of sedge and ever-helpful looks leave your wrinkled channels and on the green land answer the summons sent to you Juno has ordered some chaste nymphs and help to celebrate a noble and true marriage Donrsquot delay

[Enter certain nymphs] [Enter certain nymphs]You sunburnt sicklemen of August wearyCome hither from the furrow and be merryMake holiday your rye-straw hats put onAnd these fresh nymphs encounter every oneIn country footing

You sunburnt harvesters weary from the effects of the heat in August come here from the furrowed land and rejoice Make holiday with your rye-straw hats upon you and meet these fresh nymphs and join in country dancing

[Enter certain reapers properly habited They join with the nymphs in a graceful dance towards the end whereof Prospero starts suddenly and speaks]

[Enter certain reapers properly habited They join with the nymphs in a graceful dance towards the end whereof Prospero starts suddenly and speaks]

PROSPERO[aside] I had forgot that foul conspiracyOf the beast Caliban and his confederatesAgainst my life The minute of their plotIs almost come [to the spirits]Well done Avoidno more

PROSPERO(Aside)I had forgotten the wicked conspiracy of the beast Caliban and his accomplices against my life the time of their plot has almost arrived ndash (To the Spirits) well done depart no more of this

[To a strange hollow and confused noise the spirits heavily vanish]

[The spirits depart]

Ac-12 27420 topic Revaluation of Assets and Liabilities

REVALUATION OF ASSETS AND LIABILITIES

On admission of a new partner the firm stands reconstituted and consequently the assets are revalued and liabilities are reassessed It is necessary to show the true position of the firm at the time of admission of a new partner If the values of the assets are raised gain will increase the capital of the existing partners Similarly any decrease in the value of assets ie loss will decrease the capital of the existing partners For this purpose alsquoRevaluation Accountrsquo is prepared This account is credited with all increases in the value of assets and decrease in the value of liabilities It is debited with decrease on account of value of assets and increase in the value of liabilities The balance of this account shows a gain or loss on revaluation which is transferred to the existing partnerrsquos capital account in existing profit sharing ratioAccounting for Revaluation of Assets and Liabilities when there is a Changein the Profit Sharing Ratio of Existing PartnersAssets and liabilities of a firm must also be revalued at the time of change in profit sharing ratio of existing partners The reason is that the realisable or actual value of assets and liabilities may be different from those shown in the Balance Sheet It is possible that with the passage of time some of the assets might have appreciated in value while the value of certain other assets might have decreased and no record has been made of such changes in the books of accounts Similarly there may be some unrecorded assets amp libilities that may have to be accounted for Revaluation of assets and reassessments of liabilities becomes necessary because the change in the

value of assets and liabilities belongs to the period to change in profit sharing ratio and hence must be shared by the partners in their old profit sharing ratio Revaluation of assets and reassessment of liabilities may be given effect to in two different ways (a) When revised values are to be recorded in the books and(b) When revised values are not to be recorded in the books

When revised values are to be recorded in the booksIn such a case revaluation of assets and reassessment of liabilities is done with the help of a new account called lsquoRevaluation Accountrsquo Sometimes this account is also called as lsquoProfit amp Loss Adjustment Acrsquo If there is a loss due to revaluation revaluation account is debited and if the revaluation results in a profit the revaluation account is credited The following journal entries made for this purpose are

(i) For increase in the value of assetsAsset Ac Dr (individually)To Revaluation Ac(ii) For decrease in the value of AssetRevaluation Ac Dr (individually)To Asset Ac[Decrease in the value of assets](iii) For increase in the value of LiabilitiesRevaluation Ac Dr (individually)To Liabilities Ac[Increase in the value of Liabilities](iv) For decrease in the value of LiabilitiesLiabilities Ac DrTo Revaluation Ac[Decrease in the value of Liabilities](v) For unrecorded AssetsAsset Ac [unrecorded] DrTo Revaluation Ac[Unrecorded asset recorded at actual value](vi) For unrecorded Liability Revaluation Ac DrTo Liability Ac [unrecorded][Unrecorded Liability recorded at actual value](vii) For transfer of gain on revaluationRevaluation Ac DrTo Existing Partnerrsquos CapitalCurrent Ac[Profit on revaluation transferred to capital account in existing ratio](viii) For transfer of loss on revaluationExisting Partnerrsquos CapitalCurrent Ac DrTo Revaluation Ac[Loss on revaluation transferred to capital account in existing ratio](a) When revaluation account shows gain Revaluation Ac DrTo Partnerrsquos Capital Ac (Old Profit Sharing Ratio)(Profit on revaluation credited to Partnerrsquos Capital Ac)(b) Above entry is reversed when revaluation account shows loss Partners Capital Acs (Old Profit Sharing Ratio) DrTo Revaluation Ac(Loss on revaluation debited to Partnerrsquos Capital Acs)

Proforma of Revaluation Account is given as under

Revaluation Account

Dr Cr Particulars ` Amount Particulars ` Amount To Decrease in value of assets By Increase in value of assets To Increase in value of liabilities By Decrease in value of liabilities To Unrecorded liabilities By Unrecorded assets To Gain on Revaluation (Transferred) By Loss on Revalution (Transferred)

ECO ndash12 2742020Topic- ELASTICITY OF DEMAND

CHAPTER - ELASTICITY OF DEMANDMEANINGDemand for a commodity is affected by many factors such as its price price of related goods income of its buyer tastes and preferences etc Elasticity means degree of response Elasticity of demand means degree of responsiveness of demand Demand for a commodity responds to change in price price of related goods income etc So we have three dimensions of elasticity of demandDIMENSION OF ELASTICITY OF DEMAND TYPES OF ELASTICITY OF DEMAND

Price elasticity of demand Income elasticity of demand Cross Elasticity of demand

Price elasticity of demand Price elasticity of demand means degree of responsiveness of demand for a commodity to the change in its price For example if demand for a commodity rises by 10 due to 5 fall in its price Price elasticity of demand (ep)=Percentage change in quantity demanded Percentage change in price of the commodity = 10 ( -)5 = ( - )2Note that ep will always be negative due to inverse relationship of price and quantity demanded

(ii) Income elasticity of demand Income elasticity of demand refers to the degree of responsiveness of demand for a commodity to the change in income of its buyer Suppose income of buyer rises by 10 and his demand for a commodity rises by 20 then Income elasticity of demand (ey)= change in quantity demanded change in price of the commodity =20 10 = 2

Cross Elasticity of demandCross elasticity of demand means the degree of responsiveness of demand for a commodity to the change in price of its related goods (substitute goods or complementary goods) Suppose demand for a commodity rises by 10 due to 5 rise in price of its substitute good then Cross elasticity of demand (ec) = change in quantity demanded change in price of related good = 10 2 = 5 (Tastes and preferences cannot be expressed numerically So elasticity ofdemand cannot be numerically expressed)

  • Chapter 1 Force (Summary)
  • Distinguish between external sovereignty and internal sovereignty
    • NAND Gate
      • Logic diagram
      • Truth Table
        • NOR Gate
          • Logic diagram
          • Truth Table
            • XOR Gate
              • Logic diagram
              • Truth Table
                • XNOR Gate
                  • Logic diagram
                  • Truth Table
                      • Physics
                      • Chapter 1 Electric Field ( Electric Dipole) (Summary)
Page 44:  · Web viewWe all know that Nouns are divided into two parts: common noun and proper noun.Apart from common and proper noun, we will also study about collective noun and compound

functional megaspore develops into thefemale gametophyte

Formation The nucleus

of the functional megasporedivides mitotically to form two nuclei first andthen two more sequential mitotic nucleardivisions result in the formation of four ampthen eight nucleate stages of embryo sac

Six of the eight nucleus are surrounded bycell walls and organised into cells

The remaining two nuclei called polar nuclei are found below the egg apparatus in the largecentral cell

Three cells consisting of two synergids amp one egg cell present bottom of

embryo sac Three cells

at the chalazal as antipodal cells

Two polar nuclei together present in large central cell

HISTORY

TOWARDS INDEPENDENCE AND PARTITION THE LAST PHASE(1935-1947)SUB TOPIC NATIONAL MOVEMENTS DURING THE SECOND WORLD WAR

Spread of Quit India Movement On 9th August 1942Gandhiji and other Congress leaders were arrested The Congress was declared illegal The news of the arrest of all leaders marked the beginning of a widespread movement of India It was not possible for such a movement to remain peacefulBut the arrest of the all notable congress leaders virtually left the movement in the hands of the mass The movement took the form of violent and militant outbreakBesides congressmen revolutionaries also were very active in the movement The Congress Socialist group also played a prominent role

1 Question Why did the British authority arrest the Congress leaders on 9 th August 1942Answer Congress Working committee adopted the Quit India resolution which was to be ratified at the Bombay AICC meeting in 8th August 1942 They decided to launch a mass struggle on non-violent lines Gandhiji gave a clarion call to all section of the people rdquoKarenge ya Marengerdquo (do or die) Congress leaders gave the call to driving out

the British from IndiaViceroy had taken strong action against the Quit India movement Gandhiji and all the leaders of Congress were arrested

2 Question How did Quit India Movement spread out all over IndiaAnswer The news of the leaders lsquo arrest marked the beginning of a widespread movement to remain peacefulThe movement took form of violent outbreak There were widespread cutting of telephone and Telegraph wires damaging railway lines raising barricades in cities and towns and other forms of violent demonstations

Question Name the leaders of Congress

Socialist group played a prominent part Notable among the Jayprakash Narayan Rammonohar Lohia Aruna Asaf Ali

Political science

Topic-Franchise and Representation

Summary

The election commission

The Election Commission of India is an autonomous constitutional authority responsible for administering Union and State election processes in India The body administers elections to the Lok Sabha Rajya Sabha State Legislative Assemblies in India and the offices of the President and Vice President in the country

Functions of election commission-

India is a sovereign socialist secular democratic republic Democracy runs like a golden thread in the social economic and political fabric woven by the Constitution given by lsquoWe the People of Indiarsquo unto ourselves The concept of democracy as visualised by the Constitution pre-supposes the representation of the people in Parliament and State legislatures by the method of election The Supreme Court has held that democracy is one of the inalienable basic features of the Constitution of India and forms part of its basic structure The Constitution of India adopted a Parliamentary form of government Parliament consists of the President of India and the two Houses mdash Rajya Sabha and Lok Sabha India being a Union of states has separate state legislatures for each state State legislatures consist of the Governor and two Houses mdash Legislative Council and Legislative Assembly mdash in seven states namely Andhra Pradesh Telangana Bihar Jammu amp Kashmir Karnataka Maharashtra and Uttar Pradesh and of the Governor and the state Legislative Assembly in the remaining 22 states Apart from the above two out of the seven Union Territories namely National Capital Territory of Delhi and Puducherry also have their Legislative Assemblies

ExecutionShort notes-Election commissionFunctions of election commission

Homework- Learn

Computer

Science

Computer hardware NAND Gate

A NOT-AND operation is known as NAND operation It has n input (n gt= 2) and one output

Logic diagram

Truth Table

NOR Gate

A NOT-OR operation is known as NOR operation It has n input (n gt= 2) and one output

Logic diagram

Truth Table

XOR Gate

XOR or Ex-OR gate is a special type of gate It can be used in the half

adder full adder and subtractor The exclusive-OR gate is abbreviated as EX-OR gate or sometime as X-OR gate It has n input (n gt= 2) and one output

Logic diagram

Truth Table

XNOR Gate

XNOR gate is a special type of gate It can be used in the half adder full adder and subtractor The exclusive-NOR gate is abbreviated as EX-NOR gate or sometime as X-NOR gate It has n input (n gt= 2) and one output

Logic diagram

Truth Table

Physics

Chapter 1 Electric Field ( Electric Dipole) (Summary)

Here we will derive Expression of electric field at broad side

On position of dipole

Execution

Q With the help of a labelled diagram obtain an expression for the electric field intensity E at any point on the equitorial line ( broad-side on position) of an electric dipole

Ans

E1 E1sinθ

E θ P E1 θ

( r2+L2)12 E2 E

r E2 E2sinθ

-q θ L O L +qA B

Let us consider that the point P is situated on the right bisector of the dipole AB at a distance r meter from its midpoint O

Let E1 and E2 be the electric field intensities of the electric field at P due to charge +q and ndashq of the dipole resp The distance of P from each charge is ( r2+L2)12

So E1 = 14 πϵ q

(r 2+L 2) away from +q

E2 = 14 πϵ q

(r 2+L 2) towards ndashq

The magnitudes of E1 and E2 are equal but directions are different Now resolving E1 and E2 into two components parallel and perpendicular to AB we get

The components perpendicular to AB E1sinθ and E2sinθ cancel each other because they are equal and opposite

The components parallel to AB are E1cosθ and E2 cosθ are in same direction and add up

So resultant intensity of electric field at the point P is

E = E1cosθ + E2 cosθ

E = 14 πϵ q

(r 2+L 2) 2 cosθ

Now from fig we have cosθ =BOBP = L (r2+L2)12

So we get E = 14 πϵ 2qL ( r2+L2)32

Now electric dipole moment p= 2qL

So E = 14 πϵ p ( r2+L2)32

HW Find the expression of Electric field as done here but this time take r gtgt 2L

Also find the expression of torque experience by a dipole

(Hint Electric force experienced by charges of dipole in electric field is qE each Let θ be the angle which dipole makes with electric lines of force then perpendicular distance between two charges is 2Lsinθ Then torque = force x perp distance = qE x 2L sinθ So τ=pE sinθ where p =2qL )

STUDY MATERIAL

Class XIISubject Eng Literature (The Tempest ndash William Shakespeare) Topic Act IV Scene 1 Lines 84 to 133 (Iris hellip A contract of true love Be not too late ) Date 27th April 2020 (4th Period)

[Students should read the original play and also the paraphrase given in the school prescribed textbook]Summary Questions amp Answers

o Ceres soon appears and comes to know that she has been summoned to celebrate the contract of true love

o Ceres expresses her unwillingness to meet Venus and Cupid as she has shunned their company

o Ceres and Juno both bestow their blessings upon Ferdinand and Miranda with June gifting honour riches happiness in marriage and Ceres presents plenty of earthrsquos produce

o Iris summons the water-nymphs and reapers to come and celebrate a contract

(1) IRIS Of her society (Line 91-101)

Be not afraid I met her deity

Cutting the clouds towards Pathos and her sonDove-drawn with her Here thought they to have doneSome wanton charm upon this man and maidWhose vows are that no bed-right shall be paidTill Hymens torch be lightedmdashbut in vainMarss hot minion is returned againHer waspish-headed son has broke his arrowsSwears he will shoot no more but play with sparrowsAnd be a boy right out

(i) Where were Venus and Cupid seen flying How were they travelling Why did they want to join the marriage celebration of Ferdinand and Miranda

of true love

Venus and Cupid were seen flying through the air towards Paphos the famous city which is situated on the island of Cyprus They were travelling by air-borne chariot drawn by doves They certainly wanted to come here in order to play some amorous trick upon Ferdinand and Miranda who are under a vow not to gratify their physical desires till the holy ceremony of their marriage has been performed(ii) What have Venus and Cupid done after failing in their plan

After being failure of their plan Venus who is a very passionate deity and who is the mistress of Mars (the god of war) has gone back while here ill-tempered son Cupid has broken his arrows of love in his state of desperation(iii) What has Cupid firmly decided

Cupid is feeling so disappointed that he has firmly decided to shoot no more arrows to arouse love in human hearts but to spend his time playing with sparrows Thus he would now become just a boy and would give up his original function of shooting arrows on human beings to make them fall in love(iv) What vow had Ceres taken How did Ceres feel at the abduction

After the abduction of her daughter Prosperina by Pluto Ceres had taken a vow to always keep away from the disgraceful company of Venus and her blind son Cupid the god of love Ceres felt deeply distressed when Pluto had carried off her daughter and had made her his wife by force(v) Why has Ceres not forgiven Venus and her blind son For what do Ceres want to be sure

As the abduction had been manipulated by Venus the goddess of beauty and love and her blind son Cupid Ceres has never forgiven them for their part in the whole plot Ceres wants to be sure that she would not have to meet Venus and Cupid who had engineered the abduction of her daughter Prosperina

AS THIS lsquoMASQUErsquo SCENE IS VERY IMPORTANT IN THE PLAY THE PARAPHRASE OF THE ENTIRE PORTION OF MASQUE SCENE (Act IV Lines 58 to 143) IS GIVEN BELOW

IRIS Goddess of RainbowCERES Goddess of Agriculture and all the fruits of the earth

(Nature growth prosperity rebirth ndash notions intimately connected to marriage)JUNO The majestic Queen of Heavens and wife of Jupiter (Jupiter is the king of Gods)

VENUS The Goddess of love CUPID Son of Venus PLUTO God of death (In the play referred by Shakespeare as lsquoDisrsquo which is a Roman name for Pluto)

ORIGINAL TEXT PARAPHRASEPROSPEROWellmdash

PROSPERONow come Ariel Let there be too many rather than too few

Now come my Ariel Bring a corollaryRather than want a spirit Appear and pertly[to Ferdinand and Miranda]No tongue all eyes Be silent

spirits in attendance Appear briskly

[to Ferdinand and Miranda]Look with your eyes but do not say a word

[Soft music] [Soft music][Enter Iris] [Enter Iris]

IRISCeres most bounteous lady thy rich leasOf wheat rye barley vetches oats and peasThy turfy mountains where live nibbling sheepAnd flat meads thatched with stover them to keepThy banks with pioned and twilled brimsWhich spongy April at thy hest betrimsTo make cold nymphs chaste crowns and thybroom-grovesWhose shadow the dismissegraved bachelor lovesBeing lass-lorn thy pole clipped vineyardAnd thy sea-marge sterile and rocky-hardWhere thou thyself dost airmdashthe Queen othrsquoSkyWhose watery arch and messenger am IBids thee leave these and with her sovereign grace[Juno appears] Here on this grass-plot in this very placeTo come and sport Her peacocks fly amainApproach rich Ceres her to entertain

IRISCeres most generous lady you are the cause of rich fields or fertile land where wheat rye barley beans oats and peas grow the grassy mountains where the sheep graze and the flat meadows covered with coarse hay to be used as fodder for cattleYour banks are covered with marsh-marigolds and reeds and the rainy April under your orders brings forth to make for the maids who are not in love beautiful crowns your woods where the broom flourishes and where the bachelor who has been dismissed by the maid he loved lies down being forsaken your vineyard in which the poles are embraced by the vines and the margin of the sea which is barren and rocky where you roam about to enjoy the fresh air ndash the queen of the sky (Juno) whose messenger I am besides being represented as the rainbow bids you leave all these and with her majesty here on this grassy plot in this very place come and sport her peacocks carry her fast in her chariot through the air and are making their way here approach rich Ceres to welcome her

[Enter Ariel as Ceres] [Enter Ariel as Ceres]

CERESHail many-coloured messenger that neerDost disobey the wife of JupiterWho with thy saffron wings upon my flowersDiffusest honey-drops refreshing showersAnd with each end of thy blue bow dost crownMy bosky acres and my unshrubbed downRich scarf to my proud earth Why hath thy queenSummoned me hither to this short-grassed green

CERESWelcome rainbow that never dared disobey Juno the wife of Jupiter who with your orange coloured rays spread honey-drops refreshing showers And with each end of thy blue bow drown my bushy acres and my hilly country which is free from shrubs you thus forming a rich scarf Why has your queen called me here to this place covered with short grass

IRISA contract of true love to celebrateAnd some donation freely to estateOn the blest lovers

IRISI have called you to celebrate a contract of true love and bestow some liberal gift upon the blessed lovers

ORIGINAL TEXT PARAPHRASECERESTell me heavenly bowIf Venus or her son as thou dost knowDo now attend the queen Since they did plotThe means that dusky Dis my daughter gotHer and her blind boys scandaled companyI have forsworn

CERESTell me heavenly bow if Venus the Goddess of love or Cupid her son and pedlar of passion at this time attend the heavenly queen Juno because you are sure to know Since the day they conspired against me and dark Pluto took away my daughter here and Cupidrsquos disgraceful company I have left off

IRISOf her societyBe not afraid I met her deityCutting the clouds towards Pathos and her sonDove-drawn with her Here thought they to have doneSome wanton charm upon this man and miad

IRISBe not afraid of her company I met her deity moving on the clouds towards Paphos the sacred home of Venus on the island of Cyprus along with her son on her chariot drawn by doves Here they contemplated to exercise a charm upon this man and maid producing

Whose vows are that no bed-right shall be paidTill Hymens torch be lightedmdashbut in vainMarss hot minion is returned againHer waspish-headed son has broke his arrowsSwears he will shoot no more but play with sparrowsAnd be a boy right out

wantonness before the actual marriage ceremony but did not succeed Venus has returned her irritable son has broken his arrows and swears that he will give up his practice of trying to inspire love but play with sparrows and be a boy again

[Music is heard] [Music is heard]

CERESHighst queen of stateGreat Juno comes I know her by her gait

CERESHighest queen of state Great Juno there she comes I know here by her gait

[Enter Juno] [Enter Juno]

JUNOHow does my bounteous sister Go with meTo bless this twain that they may Prosperous beAnd honoured in their issue

JUNOHow are you doing my generous sister Come with me to bless this couple so that they may be prosperous and fortunate in their children

[They sing] [They sing]

JUNOHonour riches marriage-blessingLong continuance and increasingHourly joys be still upon youJuno sings her blessings upon you

JUNOMay honour riches happiness in marriage long continuance and increase of those boons ever rest upon you as hourly joys Juno showers down upon you her blessings in song

CERESEarths increase foison plentyBarns and garners never emptyVines and clustring bunches growingPlants and goodly burden bowingSpring come to you at the farthestIn the very end of harvestScarcity and want shall shun youCeresrsquo blessing so is on you

CERESMay you have the plenty of earthrsquos produce Your barns and granaries may never be empty Your vines may grow with clustering bunches Your fruit trees may be heavily laden with their fruit May there be continuous spring and harvest May scantiness and want leave you forever Such is the blessing of Ceres upon you

FERDINANDThis is a most majestic vision andHarmoniously charmingly May I be boldTo think these spirits

FERDINANDThis is a great vision and magically melodious Should I suppose the characters (taking part in the masque) are spirits

PROSPEROSpirits which by mine artI have from their confines calld to enactMy present fancies

PROSPEROYes they are spirits whom I have summoned from the regions to which they are confined to carry into effect my fanciful designs

ORIGINAL TEXT PARAPHRASEFERDINANDLet me live here everSo rare a wondered father and a wifeMakes this place paradise

FERDINANDI should like to live here forever Such a wise and wonderful father makes this place a paradise

[Juno and Ceres whisper and send Iris on employment] [Juno and Ceres whisper and send Iris on employment]

PROSPEROSweet now silence

PROSPEROMy dear Ferdinand speak no more Juno and Ceres are

Juno and Ceres whisper seriouslyTheres something else to do Hush and be muteOr else our spell is marred

whispering with a solemn look There is something else coming Silence Or else our magic will be spoilt

IRISYour nymphs called naiads of the wandering brooksWith your sedged crowns and over-harmless looksLeave your crisp channels and on this green landAnswer your summons Juno does commandCome temperate nymphs and help to celebrateA contract of true love Be not too late

IRISYou nymphs called Naiads denizens (M inhabitants) of the running stream with your chaplets of sedge and ever-helpful looks leave your wrinkled channels and on the green land answer the summons sent to you Juno has ordered some chaste nymphs and help to celebrate a noble and true marriage Donrsquot delay

[Enter certain nymphs] [Enter certain nymphs]You sunburnt sicklemen of August wearyCome hither from the furrow and be merryMake holiday your rye-straw hats put onAnd these fresh nymphs encounter every oneIn country footing

You sunburnt harvesters weary from the effects of the heat in August come here from the furrowed land and rejoice Make holiday with your rye-straw hats upon you and meet these fresh nymphs and join in country dancing

[Enter certain reapers properly habited They join with the nymphs in a graceful dance towards the end whereof Prospero starts suddenly and speaks]

[Enter certain reapers properly habited They join with the nymphs in a graceful dance towards the end whereof Prospero starts suddenly and speaks]

PROSPERO[aside] I had forgot that foul conspiracyOf the beast Caliban and his confederatesAgainst my life The minute of their plotIs almost come [to the spirits]Well done Avoidno more

PROSPERO(Aside)I had forgotten the wicked conspiracy of the beast Caliban and his accomplices against my life the time of their plot has almost arrived ndash (To the Spirits) well done depart no more of this

[To a strange hollow and confused noise the spirits heavily vanish]

[The spirits depart]

ORIGINAL TEXT PARAPHRASEFERDINANDLet me live here everSo rare a wondered father and a wifeMakes this place paradise

FERDINANDI should like to live here forever Such a wise and wonderful father makes this place a paradise

[Juno and Ceres whisper and send Iris on employment] [Juno and Ceres whisper and send Iris on employment]

PROSPEROSweet now silenceJuno and Ceres whisper seriouslyTheres something else to do Hush and be muteOr else our spell is marred

PROSPEROMy dear Ferdinand speak no more Juno and Ceres are whispering with a solemn look There is something else coming Silence Or else our magic will be spoilt

IRISYour nymphs called naiads of the wandering brooksWith your sedged crowns and over-harmless looksLeave your crisp channels and on this green landAnswer your summons Juno does commandCome temperate nymphs and help to celebrateA contract of true love Be not too late

IRISYou nymphs called Naiads denizens (M inhabitants) of the running stream with your chaplets of sedge and ever-helpful looks leave your wrinkled channels and on the green land answer the summons sent to you Juno has ordered some chaste nymphs and help to celebrate a noble and true marriage Donrsquot delay

[Enter certain nymphs] [Enter certain nymphs]You sunburnt sicklemen of August wearyCome hither from the furrow and be merryMake holiday your rye-straw hats put onAnd these fresh nymphs encounter every oneIn country footing

You sunburnt harvesters weary from the effects of the heat in August come here from the furrowed land and rejoice Make holiday with your rye-straw hats upon you and meet these fresh nymphs and join in country dancing

[Enter certain reapers properly habited They join with the nymphs in a graceful dance towards the end whereof Prospero starts suddenly and speaks]

[Enter certain reapers properly habited They join with the nymphs in a graceful dance towards the end whereof Prospero starts suddenly and speaks]

PROSPERO[aside] I had forgot that foul conspiracyOf the beast Caliban and his confederatesAgainst my life The minute of their plotIs almost come [to the spirits]Well done Avoidno more

PROSPERO(Aside)I had forgotten the wicked conspiracy of the beast Caliban and his accomplices against my life the time of their plot has almost arrived ndash (To the Spirits) well done depart no more of this

[To a strange hollow and confused noise the spirits heavily vanish]

[The spirits depart]

Ac-12 27420 topic Revaluation of Assets and Liabilities

REVALUATION OF ASSETS AND LIABILITIES

On admission of a new partner the firm stands reconstituted and consequently the assets are revalued and liabilities are reassessed It is necessary to show the true position of the firm at the time of admission of a new partner If the values of the assets are raised gain will increase the capital of the existing partners Similarly any decrease in the value of assets ie loss will decrease the capital of the existing partners For this purpose alsquoRevaluation Accountrsquo is prepared This account is credited with all increases in the value of assets and decrease in the value of liabilities It is debited with decrease on account of value of assets and increase in the value of liabilities The balance of this account shows a gain or loss on revaluation which is transferred to the existing partnerrsquos capital account in existing profit sharing ratioAccounting for Revaluation of Assets and Liabilities when there is a Changein the Profit Sharing Ratio of Existing PartnersAssets and liabilities of a firm must also be revalued at the time of change in profit sharing ratio of existing partners The reason is that the realisable or actual value of assets and liabilities may be different from those shown in the Balance Sheet It is possible that with the passage of time some of the assets might have appreciated in value while the value of certain other assets might have decreased and no record has been made of such changes in the books of accounts Similarly there may be some unrecorded assets amp libilities that may have to be accounted for Revaluation of assets and reassessments of liabilities becomes necessary because the change in the

value of assets and liabilities belongs to the period to change in profit sharing ratio and hence must be shared by the partners in their old profit sharing ratio Revaluation of assets and reassessment of liabilities may be given effect to in two different ways (a) When revised values are to be recorded in the books and(b) When revised values are not to be recorded in the books

When revised values are to be recorded in the booksIn such a case revaluation of assets and reassessment of liabilities is done with the help of a new account called lsquoRevaluation Accountrsquo Sometimes this account is also called as lsquoProfit amp Loss Adjustment Acrsquo If there is a loss due to revaluation revaluation account is debited and if the revaluation results in a profit the revaluation account is credited The following journal entries made for this purpose are

(i) For increase in the value of assetsAsset Ac Dr (individually)To Revaluation Ac(ii) For decrease in the value of AssetRevaluation Ac Dr (individually)To Asset Ac[Decrease in the value of assets](iii) For increase in the value of LiabilitiesRevaluation Ac Dr (individually)To Liabilities Ac[Increase in the value of Liabilities](iv) For decrease in the value of LiabilitiesLiabilities Ac DrTo Revaluation Ac[Decrease in the value of Liabilities](v) For unrecorded AssetsAsset Ac [unrecorded] DrTo Revaluation Ac[Unrecorded asset recorded at actual value](vi) For unrecorded Liability Revaluation Ac DrTo Liability Ac [unrecorded][Unrecorded Liability recorded at actual value](vii) For transfer of gain on revaluationRevaluation Ac DrTo Existing Partnerrsquos CapitalCurrent Ac[Profit on revaluation transferred to capital account in existing ratio](viii) For transfer of loss on revaluationExisting Partnerrsquos CapitalCurrent Ac DrTo Revaluation Ac[Loss on revaluation transferred to capital account in existing ratio](a) When revaluation account shows gain Revaluation Ac DrTo Partnerrsquos Capital Ac (Old Profit Sharing Ratio)(Profit on revaluation credited to Partnerrsquos Capital Ac)(b) Above entry is reversed when revaluation account shows loss Partners Capital Acs (Old Profit Sharing Ratio) DrTo Revaluation Ac(Loss on revaluation debited to Partnerrsquos Capital Acs)

Proforma of Revaluation Account is given as under

Revaluation Account

Dr Cr Particulars ` Amount Particulars ` Amount To Decrease in value of assets By Increase in value of assets To Increase in value of liabilities By Decrease in value of liabilities To Unrecorded liabilities By Unrecorded assets To Gain on Revaluation (Transferred) By Loss on Revalution (Transferred)

ECO ndash12 2742020Topic- ELASTICITY OF DEMAND

CHAPTER - ELASTICITY OF DEMANDMEANINGDemand for a commodity is affected by many factors such as its price price of related goods income of its buyer tastes and preferences etc Elasticity means degree of response Elasticity of demand means degree of responsiveness of demand Demand for a commodity responds to change in price price of related goods income etc So we have three dimensions of elasticity of demandDIMENSION OF ELASTICITY OF DEMAND TYPES OF ELASTICITY OF DEMAND

Price elasticity of demand Income elasticity of demand Cross Elasticity of demand

Price elasticity of demand Price elasticity of demand means degree of responsiveness of demand for a commodity to the change in its price For example if demand for a commodity rises by 10 due to 5 fall in its price Price elasticity of demand (ep)=Percentage change in quantity demanded Percentage change in price of the commodity = 10 ( -)5 = ( - )2Note that ep will always be negative due to inverse relationship of price and quantity demanded

(ii) Income elasticity of demand Income elasticity of demand refers to the degree of responsiveness of demand for a commodity to the change in income of its buyer Suppose income of buyer rises by 10 and his demand for a commodity rises by 20 then Income elasticity of demand (ey)= change in quantity demanded change in price of the commodity =20 10 = 2

Cross Elasticity of demandCross elasticity of demand means the degree of responsiveness of demand for a commodity to the change in price of its related goods (substitute goods or complementary goods) Suppose demand for a commodity rises by 10 due to 5 rise in price of its substitute good then Cross elasticity of demand (ec) = change in quantity demanded change in price of related good = 10 2 = 5 (Tastes and preferences cannot be expressed numerically So elasticity ofdemand cannot be numerically expressed)

  • Chapter 1 Force (Summary)
  • Distinguish between external sovereignty and internal sovereignty
    • NAND Gate
      • Logic diagram
      • Truth Table
        • NOR Gate
          • Logic diagram
          • Truth Table
            • XOR Gate
              • Logic diagram
              • Truth Table
                • XNOR Gate
                  • Logic diagram
                  • Truth Table
                      • Physics
                      • Chapter 1 Electric Field ( Electric Dipole) (Summary)
Page 45:  · Web viewWe all know that Nouns are divided into two parts: common noun and proper noun.Apart from common and proper noun, we will also study about collective noun and compound

embryo sac Three cells

at the chalazal as antipodal cells

Two polar nuclei together present in large central cell

HISTORY

TOWARDS INDEPENDENCE AND PARTITION THE LAST PHASE(1935-1947)SUB TOPIC NATIONAL MOVEMENTS DURING THE SECOND WORLD WAR

Spread of Quit India Movement On 9th August 1942Gandhiji and other Congress leaders were arrested The Congress was declared illegal The news of the arrest of all leaders marked the beginning of a widespread movement of India It was not possible for such a movement to remain peacefulBut the arrest of the all notable congress leaders virtually left the movement in the hands of the mass The movement took the form of violent and militant outbreakBesides congressmen revolutionaries also were very active in the movement The Congress Socialist group also played a prominent role

1 Question Why did the British authority arrest the Congress leaders on 9 th August 1942Answer Congress Working committee adopted the Quit India resolution which was to be ratified at the Bombay AICC meeting in 8th August 1942 They decided to launch a mass struggle on non-violent lines Gandhiji gave a clarion call to all section of the people rdquoKarenge ya Marengerdquo (do or die) Congress leaders gave the call to driving out

the British from IndiaViceroy had taken strong action against the Quit India movement Gandhiji and all the leaders of Congress were arrested

2 Question How did Quit India Movement spread out all over IndiaAnswer The news of the leaders lsquo arrest marked the beginning of a widespread movement to remain peacefulThe movement took form of violent outbreak There were widespread cutting of telephone and Telegraph wires damaging railway lines raising barricades in cities and towns and other forms of violent demonstations

Question Name the leaders of Congress

Socialist group played a prominent part Notable among the Jayprakash Narayan Rammonohar Lohia Aruna Asaf Ali

Political science

Topic-Franchise and Representation

Summary

The election commission

The Election Commission of India is an autonomous constitutional authority responsible for administering Union and State election processes in India The body administers elections to the Lok Sabha Rajya Sabha State Legislative Assemblies in India and the offices of the President and Vice President in the country

Functions of election commission-

India is a sovereign socialist secular democratic republic Democracy runs like a golden thread in the social economic and political fabric woven by the Constitution given by lsquoWe the People of Indiarsquo unto ourselves The concept of democracy as visualised by the Constitution pre-supposes the representation of the people in Parliament and State legislatures by the method of election The Supreme Court has held that democracy is one of the inalienable basic features of the Constitution of India and forms part of its basic structure The Constitution of India adopted a Parliamentary form of government Parliament consists of the President of India and the two Houses mdash Rajya Sabha and Lok Sabha India being a Union of states has separate state legislatures for each state State legislatures consist of the Governor and two Houses mdash Legislative Council and Legislative Assembly mdash in seven states namely Andhra Pradesh Telangana Bihar Jammu amp Kashmir Karnataka Maharashtra and Uttar Pradesh and of the Governor and the state Legislative Assembly in the remaining 22 states Apart from the above two out of the seven Union Territories namely National Capital Territory of Delhi and Puducherry also have their Legislative Assemblies

ExecutionShort notes-Election commissionFunctions of election commission

Homework- Learn

Computer

Science

Computer hardware NAND Gate

A NOT-AND operation is known as NAND operation It has n input (n gt= 2) and one output

Logic diagram

Truth Table

NOR Gate

A NOT-OR operation is known as NOR operation It has n input (n gt= 2) and one output

Logic diagram

Truth Table

XOR Gate

XOR or Ex-OR gate is a special type of gate It can be used in the half

adder full adder and subtractor The exclusive-OR gate is abbreviated as EX-OR gate or sometime as X-OR gate It has n input (n gt= 2) and one output

Logic diagram

Truth Table

XNOR Gate

XNOR gate is a special type of gate It can be used in the half adder full adder and subtractor The exclusive-NOR gate is abbreviated as EX-NOR gate or sometime as X-NOR gate It has n input (n gt= 2) and one output

Logic diagram

Truth Table

Physics

Chapter 1 Electric Field ( Electric Dipole) (Summary)

Here we will derive Expression of electric field at broad side

On position of dipole

Execution

Q With the help of a labelled diagram obtain an expression for the electric field intensity E at any point on the equitorial line ( broad-side on position) of an electric dipole

Ans

E1 E1sinθ

E θ P E1 θ

( r2+L2)12 E2 E

r E2 E2sinθ

-q θ L O L +qA B

Let us consider that the point P is situated on the right bisector of the dipole AB at a distance r meter from its midpoint O

Let E1 and E2 be the electric field intensities of the electric field at P due to charge +q and ndashq of the dipole resp The distance of P from each charge is ( r2+L2)12

So E1 = 14 πϵ q

(r 2+L 2) away from +q

E2 = 14 πϵ q

(r 2+L 2) towards ndashq

The magnitudes of E1 and E2 are equal but directions are different Now resolving E1 and E2 into two components parallel and perpendicular to AB we get

The components perpendicular to AB E1sinθ and E2sinθ cancel each other because they are equal and opposite

The components parallel to AB are E1cosθ and E2 cosθ are in same direction and add up

So resultant intensity of electric field at the point P is

E = E1cosθ + E2 cosθ

E = 14 πϵ q

(r 2+L 2) 2 cosθ

Now from fig we have cosθ =BOBP = L (r2+L2)12

So we get E = 14 πϵ 2qL ( r2+L2)32

Now electric dipole moment p= 2qL

So E = 14 πϵ p ( r2+L2)32

HW Find the expression of Electric field as done here but this time take r gtgt 2L

Also find the expression of torque experience by a dipole

(Hint Electric force experienced by charges of dipole in electric field is qE each Let θ be the angle which dipole makes with electric lines of force then perpendicular distance between two charges is 2Lsinθ Then torque = force x perp distance = qE x 2L sinθ So τ=pE sinθ where p =2qL )

STUDY MATERIAL

Class XIISubject Eng Literature (The Tempest ndash William Shakespeare) Topic Act IV Scene 1 Lines 84 to 133 (Iris hellip A contract of true love Be not too late ) Date 27th April 2020 (4th Period)

[Students should read the original play and also the paraphrase given in the school prescribed textbook]Summary Questions amp Answers

o Ceres soon appears and comes to know that she has been summoned to celebrate the contract of true love

o Ceres expresses her unwillingness to meet Venus and Cupid as she has shunned their company

o Ceres and Juno both bestow their blessings upon Ferdinand and Miranda with June gifting honour riches happiness in marriage and Ceres presents plenty of earthrsquos produce

o Iris summons the water-nymphs and reapers to come and celebrate a contract

(1) IRIS Of her society (Line 91-101)

Be not afraid I met her deity

Cutting the clouds towards Pathos and her sonDove-drawn with her Here thought they to have doneSome wanton charm upon this man and maidWhose vows are that no bed-right shall be paidTill Hymens torch be lightedmdashbut in vainMarss hot minion is returned againHer waspish-headed son has broke his arrowsSwears he will shoot no more but play with sparrowsAnd be a boy right out

(i) Where were Venus and Cupid seen flying How were they travelling Why did they want to join the marriage celebration of Ferdinand and Miranda

of true love

Venus and Cupid were seen flying through the air towards Paphos the famous city which is situated on the island of Cyprus They were travelling by air-borne chariot drawn by doves They certainly wanted to come here in order to play some amorous trick upon Ferdinand and Miranda who are under a vow not to gratify their physical desires till the holy ceremony of their marriage has been performed(ii) What have Venus and Cupid done after failing in their plan

After being failure of their plan Venus who is a very passionate deity and who is the mistress of Mars (the god of war) has gone back while here ill-tempered son Cupid has broken his arrows of love in his state of desperation(iii) What has Cupid firmly decided

Cupid is feeling so disappointed that he has firmly decided to shoot no more arrows to arouse love in human hearts but to spend his time playing with sparrows Thus he would now become just a boy and would give up his original function of shooting arrows on human beings to make them fall in love(iv) What vow had Ceres taken How did Ceres feel at the abduction

After the abduction of her daughter Prosperina by Pluto Ceres had taken a vow to always keep away from the disgraceful company of Venus and her blind son Cupid the god of love Ceres felt deeply distressed when Pluto had carried off her daughter and had made her his wife by force(v) Why has Ceres not forgiven Venus and her blind son For what do Ceres want to be sure

As the abduction had been manipulated by Venus the goddess of beauty and love and her blind son Cupid Ceres has never forgiven them for their part in the whole plot Ceres wants to be sure that she would not have to meet Venus and Cupid who had engineered the abduction of her daughter Prosperina

AS THIS lsquoMASQUErsquo SCENE IS VERY IMPORTANT IN THE PLAY THE PARAPHRASE OF THE ENTIRE PORTION OF MASQUE SCENE (Act IV Lines 58 to 143) IS GIVEN BELOW

IRIS Goddess of RainbowCERES Goddess of Agriculture and all the fruits of the earth

(Nature growth prosperity rebirth ndash notions intimately connected to marriage)JUNO The majestic Queen of Heavens and wife of Jupiter (Jupiter is the king of Gods)

VENUS The Goddess of love CUPID Son of Venus PLUTO God of death (In the play referred by Shakespeare as lsquoDisrsquo which is a Roman name for Pluto)

ORIGINAL TEXT PARAPHRASEPROSPEROWellmdash

PROSPERONow come Ariel Let there be too many rather than too few

Now come my Ariel Bring a corollaryRather than want a spirit Appear and pertly[to Ferdinand and Miranda]No tongue all eyes Be silent

spirits in attendance Appear briskly

[to Ferdinand and Miranda]Look with your eyes but do not say a word

[Soft music] [Soft music][Enter Iris] [Enter Iris]

IRISCeres most bounteous lady thy rich leasOf wheat rye barley vetches oats and peasThy turfy mountains where live nibbling sheepAnd flat meads thatched with stover them to keepThy banks with pioned and twilled brimsWhich spongy April at thy hest betrimsTo make cold nymphs chaste crowns and thybroom-grovesWhose shadow the dismissegraved bachelor lovesBeing lass-lorn thy pole clipped vineyardAnd thy sea-marge sterile and rocky-hardWhere thou thyself dost airmdashthe Queen othrsquoSkyWhose watery arch and messenger am IBids thee leave these and with her sovereign grace[Juno appears] Here on this grass-plot in this very placeTo come and sport Her peacocks fly amainApproach rich Ceres her to entertain

IRISCeres most generous lady you are the cause of rich fields or fertile land where wheat rye barley beans oats and peas grow the grassy mountains where the sheep graze and the flat meadows covered with coarse hay to be used as fodder for cattleYour banks are covered with marsh-marigolds and reeds and the rainy April under your orders brings forth to make for the maids who are not in love beautiful crowns your woods where the broom flourishes and where the bachelor who has been dismissed by the maid he loved lies down being forsaken your vineyard in which the poles are embraced by the vines and the margin of the sea which is barren and rocky where you roam about to enjoy the fresh air ndash the queen of the sky (Juno) whose messenger I am besides being represented as the rainbow bids you leave all these and with her majesty here on this grassy plot in this very place come and sport her peacocks carry her fast in her chariot through the air and are making their way here approach rich Ceres to welcome her

[Enter Ariel as Ceres] [Enter Ariel as Ceres]

CERESHail many-coloured messenger that neerDost disobey the wife of JupiterWho with thy saffron wings upon my flowersDiffusest honey-drops refreshing showersAnd with each end of thy blue bow dost crownMy bosky acres and my unshrubbed downRich scarf to my proud earth Why hath thy queenSummoned me hither to this short-grassed green

CERESWelcome rainbow that never dared disobey Juno the wife of Jupiter who with your orange coloured rays spread honey-drops refreshing showers And with each end of thy blue bow drown my bushy acres and my hilly country which is free from shrubs you thus forming a rich scarf Why has your queen called me here to this place covered with short grass

IRISA contract of true love to celebrateAnd some donation freely to estateOn the blest lovers

IRISI have called you to celebrate a contract of true love and bestow some liberal gift upon the blessed lovers

ORIGINAL TEXT PARAPHRASECERESTell me heavenly bowIf Venus or her son as thou dost knowDo now attend the queen Since they did plotThe means that dusky Dis my daughter gotHer and her blind boys scandaled companyI have forsworn

CERESTell me heavenly bow if Venus the Goddess of love or Cupid her son and pedlar of passion at this time attend the heavenly queen Juno because you are sure to know Since the day they conspired against me and dark Pluto took away my daughter here and Cupidrsquos disgraceful company I have left off

IRISOf her societyBe not afraid I met her deityCutting the clouds towards Pathos and her sonDove-drawn with her Here thought they to have doneSome wanton charm upon this man and miad

IRISBe not afraid of her company I met her deity moving on the clouds towards Paphos the sacred home of Venus on the island of Cyprus along with her son on her chariot drawn by doves Here they contemplated to exercise a charm upon this man and maid producing

Whose vows are that no bed-right shall be paidTill Hymens torch be lightedmdashbut in vainMarss hot minion is returned againHer waspish-headed son has broke his arrowsSwears he will shoot no more but play with sparrowsAnd be a boy right out

wantonness before the actual marriage ceremony but did not succeed Venus has returned her irritable son has broken his arrows and swears that he will give up his practice of trying to inspire love but play with sparrows and be a boy again

[Music is heard] [Music is heard]

CERESHighst queen of stateGreat Juno comes I know her by her gait

CERESHighest queen of state Great Juno there she comes I know here by her gait

[Enter Juno] [Enter Juno]

JUNOHow does my bounteous sister Go with meTo bless this twain that they may Prosperous beAnd honoured in their issue

JUNOHow are you doing my generous sister Come with me to bless this couple so that they may be prosperous and fortunate in their children

[They sing] [They sing]

JUNOHonour riches marriage-blessingLong continuance and increasingHourly joys be still upon youJuno sings her blessings upon you

JUNOMay honour riches happiness in marriage long continuance and increase of those boons ever rest upon you as hourly joys Juno showers down upon you her blessings in song

CERESEarths increase foison plentyBarns and garners never emptyVines and clustring bunches growingPlants and goodly burden bowingSpring come to you at the farthestIn the very end of harvestScarcity and want shall shun youCeresrsquo blessing so is on you

CERESMay you have the plenty of earthrsquos produce Your barns and granaries may never be empty Your vines may grow with clustering bunches Your fruit trees may be heavily laden with their fruit May there be continuous spring and harvest May scantiness and want leave you forever Such is the blessing of Ceres upon you

FERDINANDThis is a most majestic vision andHarmoniously charmingly May I be boldTo think these spirits

FERDINANDThis is a great vision and magically melodious Should I suppose the characters (taking part in the masque) are spirits

PROSPEROSpirits which by mine artI have from their confines calld to enactMy present fancies

PROSPEROYes they are spirits whom I have summoned from the regions to which they are confined to carry into effect my fanciful designs

ORIGINAL TEXT PARAPHRASEFERDINANDLet me live here everSo rare a wondered father and a wifeMakes this place paradise

FERDINANDI should like to live here forever Such a wise and wonderful father makes this place a paradise

[Juno and Ceres whisper and send Iris on employment] [Juno and Ceres whisper and send Iris on employment]

PROSPEROSweet now silence

PROSPEROMy dear Ferdinand speak no more Juno and Ceres are

Juno and Ceres whisper seriouslyTheres something else to do Hush and be muteOr else our spell is marred

whispering with a solemn look There is something else coming Silence Or else our magic will be spoilt

IRISYour nymphs called naiads of the wandering brooksWith your sedged crowns and over-harmless looksLeave your crisp channels and on this green landAnswer your summons Juno does commandCome temperate nymphs and help to celebrateA contract of true love Be not too late

IRISYou nymphs called Naiads denizens (M inhabitants) of the running stream with your chaplets of sedge and ever-helpful looks leave your wrinkled channels and on the green land answer the summons sent to you Juno has ordered some chaste nymphs and help to celebrate a noble and true marriage Donrsquot delay

[Enter certain nymphs] [Enter certain nymphs]You sunburnt sicklemen of August wearyCome hither from the furrow and be merryMake holiday your rye-straw hats put onAnd these fresh nymphs encounter every oneIn country footing

You sunburnt harvesters weary from the effects of the heat in August come here from the furrowed land and rejoice Make holiday with your rye-straw hats upon you and meet these fresh nymphs and join in country dancing

[Enter certain reapers properly habited They join with the nymphs in a graceful dance towards the end whereof Prospero starts suddenly and speaks]

[Enter certain reapers properly habited They join with the nymphs in a graceful dance towards the end whereof Prospero starts suddenly and speaks]

PROSPERO[aside] I had forgot that foul conspiracyOf the beast Caliban and his confederatesAgainst my life The minute of their plotIs almost come [to the spirits]Well done Avoidno more

PROSPERO(Aside)I had forgotten the wicked conspiracy of the beast Caliban and his accomplices against my life the time of their plot has almost arrived ndash (To the Spirits) well done depart no more of this

[To a strange hollow and confused noise the spirits heavily vanish]

[The spirits depart]

ORIGINAL TEXT PARAPHRASEFERDINANDLet me live here everSo rare a wondered father and a wifeMakes this place paradise

FERDINANDI should like to live here forever Such a wise and wonderful father makes this place a paradise

[Juno and Ceres whisper and send Iris on employment] [Juno and Ceres whisper and send Iris on employment]

PROSPEROSweet now silenceJuno and Ceres whisper seriouslyTheres something else to do Hush and be muteOr else our spell is marred

PROSPEROMy dear Ferdinand speak no more Juno and Ceres are whispering with a solemn look There is something else coming Silence Or else our magic will be spoilt

IRISYour nymphs called naiads of the wandering brooksWith your sedged crowns and over-harmless looksLeave your crisp channels and on this green landAnswer your summons Juno does commandCome temperate nymphs and help to celebrateA contract of true love Be not too late

IRISYou nymphs called Naiads denizens (M inhabitants) of the running stream with your chaplets of sedge and ever-helpful looks leave your wrinkled channels and on the green land answer the summons sent to you Juno has ordered some chaste nymphs and help to celebrate a noble and true marriage Donrsquot delay

[Enter certain nymphs] [Enter certain nymphs]You sunburnt sicklemen of August wearyCome hither from the furrow and be merryMake holiday your rye-straw hats put onAnd these fresh nymphs encounter every oneIn country footing

You sunburnt harvesters weary from the effects of the heat in August come here from the furrowed land and rejoice Make holiday with your rye-straw hats upon you and meet these fresh nymphs and join in country dancing

[Enter certain reapers properly habited They join with the nymphs in a graceful dance towards the end whereof Prospero starts suddenly and speaks]

[Enter certain reapers properly habited They join with the nymphs in a graceful dance towards the end whereof Prospero starts suddenly and speaks]

PROSPERO[aside] I had forgot that foul conspiracyOf the beast Caliban and his confederatesAgainst my life The minute of their plotIs almost come [to the spirits]Well done Avoidno more

PROSPERO(Aside)I had forgotten the wicked conspiracy of the beast Caliban and his accomplices against my life the time of their plot has almost arrived ndash (To the Spirits) well done depart no more of this

[To a strange hollow and confused noise the spirits heavily vanish]

[The spirits depart]

Ac-12 27420 topic Revaluation of Assets and Liabilities

REVALUATION OF ASSETS AND LIABILITIES

On admission of a new partner the firm stands reconstituted and consequently the assets are revalued and liabilities are reassessed It is necessary to show the true position of the firm at the time of admission of a new partner If the values of the assets are raised gain will increase the capital of the existing partners Similarly any decrease in the value of assets ie loss will decrease the capital of the existing partners For this purpose alsquoRevaluation Accountrsquo is prepared This account is credited with all increases in the value of assets and decrease in the value of liabilities It is debited with decrease on account of value of assets and increase in the value of liabilities The balance of this account shows a gain or loss on revaluation which is transferred to the existing partnerrsquos capital account in existing profit sharing ratioAccounting for Revaluation of Assets and Liabilities when there is a Changein the Profit Sharing Ratio of Existing PartnersAssets and liabilities of a firm must also be revalued at the time of change in profit sharing ratio of existing partners The reason is that the realisable or actual value of assets and liabilities may be different from those shown in the Balance Sheet It is possible that with the passage of time some of the assets might have appreciated in value while the value of certain other assets might have decreased and no record has been made of such changes in the books of accounts Similarly there may be some unrecorded assets amp libilities that may have to be accounted for Revaluation of assets and reassessments of liabilities becomes necessary because the change in the

value of assets and liabilities belongs to the period to change in profit sharing ratio and hence must be shared by the partners in their old profit sharing ratio Revaluation of assets and reassessment of liabilities may be given effect to in two different ways (a) When revised values are to be recorded in the books and(b) When revised values are not to be recorded in the books

When revised values are to be recorded in the booksIn such a case revaluation of assets and reassessment of liabilities is done with the help of a new account called lsquoRevaluation Accountrsquo Sometimes this account is also called as lsquoProfit amp Loss Adjustment Acrsquo If there is a loss due to revaluation revaluation account is debited and if the revaluation results in a profit the revaluation account is credited The following journal entries made for this purpose are

(i) For increase in the value of assetsAsset Ac Dr (individually)To Revaluation Ac(ii) For decrease in the value of AssetRevaluation Ac Dr (individually)To Asset Ac[Decrease in the value of assets](iii) For increase in the value of LiabilitiesRevaluation Ac Dr (individually)To Liabilities Ac[Increase in the value of Liabilities](iv) For decrease in the value of LiabilitiesLiabilities Ac DrTo Revaluation Ac[Decrease in the value of Liabilities](v) For unrecorded AssetsAsset Ac [unrecorded] DrTo Revaluation Ac[Unrecorded asset recorded at actual value](vi) For unrecorded Liability Revaluation Ac DrTo Liability Ac [unrecorded][Unrecorded Liability recorded at actual value](vii) For transfer of gain on revaluationRevaluation Ac DrTo Existing Partnerrsquos CapitalCurrent Ac[Profit on revaluation transferred to capital account in existing ratio](viii) For transfer of loss on revaluationExisting Partnerrsquos CapitalCurrent Ac DrTo Revaluation Ac[Loss on revaluation transferred to capital account in existing ratio](a) When revaluation account shows gain Revaluation Ac DrTo Partnerrsquos Capital Ac (Old Profit Sharing Ratio)(Profit on revaluation credited to Partnerrsquos Capital Ac)(b) Above entry is reversed when revaluation account shows loss Partners Capital Acs (Old Profit Sharing Ratio) DrTo Revaluation Ac(Loss on revaluation debited to Partnerrsquos Capital Acs)

Proforma of Revaluation Account is given as under

Revaluation Account

Dr Cr Particulars ` Amount Particulars ` Amount To Decrease in value of assets By Increase in value of assets To Increase in value of liabilities By Decrease in value of liabilities To Unrecorded liabilities By Unrecorded assets To Gain on Revaluation (Transferred) By Loss on Revalution (Transferred)

ECO ndash12 2742020Topic- ELASTICITY OF DEMAND

CHAPTER - ELASTICITY OF DEMANDMEANINGDemand for a commodity is affected by many factors such as its price price of related goods income of its buyer tastes and preferences etc Elasticity means degree of response Elasticity of demand means degree of responsiveness of demand Demand for a commodity responds to change in price price of related goods income etc So we have three dimensions of elasticity of demandDIMENSION OF ELASTICITY OF DEMAND TYPES OF ELASTICITY OF DEMAND

Price elasticity of demand Income elasticity of demand Cross Elasticity of demand

Price elasticity of demand Price elasticity of demand means degree of responsiveness of demand for a commodity to the change in its price For example if demand for a commodity rises by 10 due to 5 fall in its price Price elasticity of demand (ep)=Percentage change in quantity demanded Percentage change in price of the commodity = 10 ( -)5 = ( - )2Note that ep will always be negative due to inverse relationship of price and quantity demanded

(ii) Income elasticity of demand Income elasticity of demand refers to the degree of responsiveness of demand for a commodity to the change in income of its buyer Suppose income of buyer rises by 10 and his demand for a commodity rises by 20 then Income elasticity of demand (ey)= change in quantity demanded change in price of the commodity =20 10 = 2

Cross Elasticity of demandCross elasticity of demand means the degree of responsiveness of demand for a commodity to the change in price of its related goods (substitute goods or complementary goods) Suppose demand for a commodity rises by 10 due to 5 rise in price of its substitute good then Cross elasticity of demand (ec) = change in quantity demanded change in price of related good = 10 2 = 5 (Tastes and preferences cannot be expressed numerically So elasticity ofdemand cannot be numerically expressed)

  • Chapter 1 Force (Summary)
  • Distinguish between external sovereignty and internal sovereignty
    • NAND Gate
      • Logic diagram
      • Truth Table
        • NOR Gate
          • Logic diagram
          • Truth Table
            • XOR Gate
              • Logic diagram
              • Truth Table
                • XNOR Gate
                  • Logic diagram
                  • Truth Table
                      • Physics
                      • Chapter 1 Electric Field ( Electric Dipole) (Summary)
Page 46:  · Web viewWe all know that Nouns are divided into two parts: common noun and proper noun.Apart from common and proper noun, we will also study about collective noun and compound

the British from IndiaViceroy had taken strong action against the Quit India movement Gandhiji and all the leaders of Congress were arrested

2 Question How did Quit India Movement spread out all over IndiaAnswer The news of the leaders lsquo arrest marked the beginning of a widespread movement to remain peacefulThe movement took form of violent outbreak There were widespread cutting of telephone and Telegraph wires damaging railway lines raising barricades in cities and towns and other forms of violent demonstations

Question Name the leaders of Congress

Socialist group played a prominent part Notable among the Jayprakash Narayan Rammonohar Lohia Aruna Asaf Ali

Political science

Topic-Franchise and Representation

Summary

The election commission

The Election Commission of India is an autonomous constitutional authority responsible for administering Union and State election processes in India The body administers elections to the Lok Sabha Rajya Sabha State Legislative Assemblies in India and the offices of the President and Vice President in the country

Functions of election commission-

India is a sovereign socialist secular democratic republic Democracy runs like a golden thread in the social economic and political fabric woven by the Constitution given by lsquoWe the People of Indiarsquo unto ourselves The concept of democracy as visualised by the Constitution pre-supposes the representation of the people in Parliament and State legislatures by the method of election The Supreme Court has held that democracy is one of the inalienable basic features of the Constitution of India and forms part of its basic structure The Constitution of India adopted a Parliamentary form of government Parliament consists of the President of India and the two Houses mdash Rajya Sabha and Lok Sabha India being a Union of states has separate state legislatures for each state State legislatures consist of the Governor and two Houses mdash Legislative Council and Legislative Assembly mdash in seven states namely Andhra Pradesh Telangana Bihar Jammu amp Kashmir Karnataka Maharashtra and Uttar Pradesh and of the Governor and the state Legislative Assembly in the remaining 22 states Apart from the above two out of the seven Union Territories namely National Capital Territory of Delhi and Puducherry also have their Legislative Assemblies

ExecutionShort notes-Election commissionFunctions of election commission

Homework- Learn

Computer

Science

Computer hardware NAND Gate

A NOT-AND operation is known as NAND operation It has n input (n gt= 2) and one output

Logic diagram

Truth Table

NOR Gate

A NOT-OR operation is known as NOR operation It has n input (n gt= 2) and one output

Logic diagram

Truth Table

XOR Gate

XOR or Ex-OR gate is a special type of gate It can be used in the half

adder full adder and subtractor The exclusive-OR gate is abbreviated as EX-OR gate or sometime as X-OR gate It has n input (n gt= 2) and one output

Logic diagram

Truth Table

XNOR Gate

XNOR gate is a special type of gate It can be used in the half adder full adder and subtractor The exclusive-NOR gate is abbreviated as EX-NOR gate or sometime as X-NOR gate It has n input (n gt= 2) and one output

Logic diagram

Truth Table

Physics

Chapter 1 Electric Field ( Electric Dipole) (Summary)

Here we will derive Expression of electric field at broad side

On position of dipole

Execution

Q With the help of a labelled diagram obtain an expression for the electric field intensity E at any point on the equitorial line ( broad-side on position) of an electric dipole

Ans

E1 E1sinθ

E θ P E1 θ

( r2+L2)12 E2 E

r E2 E2sinθ

-q θ L O L +qA B

Let us consider that the point P is situated on the right bisector of the dipole AB at a distance r meter from its midpoint O

Let E1 and E2 be the electric field intensities of the electric field at P due to charge +q and ndashq of the dipole resp The distance of P from each charge is ( r2+L2)12

So E1 = 14 πϵ q

(r 2+L 2) away from +q

E2 = 14 πϵ q

(r 2+L 2) towards ndashq

The magnitudes of E1 and E2 are equal but directions are different Now resolving E1 and E2 into two components parallel and perpendicular to AB we get

The components perpendicular to AB E1sinθ and E2sinθ cancel each other because they are equal and opposite

The components parallel to AB are E1cosθ and E2 cosθ are in same direction and add up

So resultant intensity of electric field at the point P is

E = E1cosθ + E2 cosθ

E = 14 πϵ q

(r 2+L 2) 2 cosθ

Now from fig we have cosθ =BOBP = L (r2+L2)12

So we get E = 14 πϵ 2qL ( r2+L2)32

Now electric dipole moment p= 2qL

So E = 14 πϵ p ( r2+L2)32

HW Find the expression of Electric field as done here but this time take r gtgt 2L

Also find the expression of torque experience by a dipole

(Hint Electric force experienced by charges of dipole in electric field is qE each Let θ be the angle which dipole makes with electric lines of force then perpendicular distance between two charges is 2Lsinθ Then torque = force x perp distance = qE x 2L sinθ So τ=pE sinθ where p =2qL )

STUDY MATERIAL

Class XIISubject Eng Literature (The Tempest ndash William Shakespeare) Topic Act IV Scene 1 Lines 84 to 133 (Iris hellip A contract of true love Be not too late ) Date 27th April 2020 (4th Period)

[Students should read the original play and also the paraphrase given in the school prescribed textbook]Summary Questions amp Answers

o Ceres soon appears and comes to know that she has been summoned to celebrate the contract of true love

o Ceres expresses her unwillingness to meet Venus and Cupid as she has shunned their company

o Ceres and Juno both bestow their blessings upon Ferdinand and Miranda with June gifting honour riches happiness in marriage and Ceres presents plenty of earthrsquos produce

o Iris summons the water-nymphs and reapers to come and celebrate a contract

(1) IRIS Of her society (Line 91-101)

Be not afraid I met her deity

Cutting the clouds towards Pathos and her sonDove-drawn with her Here thought they to have doneSome wanton charm upon this man and maidWhose vows are that no bed-right shall be paidTill Hymens torch be lightedmdashbut in vainMarss hot minion is returned againHer waspish-headed son has broke his arrowsSwears he will shoot no more but play with sparrowsAnd be a boy right out

(i) Where were Venus and Cupid seen flying How were they travelling Why did they want to join the marriage celebration of Ferdinand and Miranda

of true love

Venus and Cupid were seen flying through the air towards Paphos the famous city which is situated on the island of Cyprus They were travelling by air-borne chariot drawn by doves They certainly wanted to come here in order to play some amorous trick upon Ferdinand and Miranda who are under a vow not to gratify their physical desires till the holy ceremony of their marriage has been performed(ii) What have Venus and Cupid done after failing in their plan

After being failure of their plan Venus who is a very passionate deity and who is the mistress of Mars (the god of war) has gone back while here ill-tempered son Cupid has broken his arrows of love in his state of desperation(iii) What has Cupid firmly decided

Cupid is feeling so disappointed that he has firmly decided to shoot no more arrows to arouse love in human hearts but to spend his time playing with sparrows Thus he would now become just a boy and would give up his original function of shooting arrows on human beings to make them fall in love(iv) What vow had Ceres taken How did Ceres feel at the abduction

After the abduction of her daughter Prosperina by Pluto Ceres had taken a vow to always keep away from the disgraceful company of Venus and her blind son Cupid the god of love Ceres felt deeply distressed when Pluto had carried off her daughter and had made her his wife by force(v) Why has Ceres not forgiven Venus and her blind son For what do Ceres want to be sure

As the abduction had been manipulated by Venus the goddess of beauty and love and her blind son Cupid Ceres has never forgiven them for their part in the whole plot Ceres wants to be sure that she would not have to meet Venus and Cupid who had engineered the abduction of her daughter Prosperina

AS THIS lsquoMASQUErsquo SCENE IS VERY IMPORTANT IN THE PLAY THE PARAPHRASE OF THE ENTIRE PORTION OF MASQUE SCENE (Act IV Lines 58 to 143) IS GIVEN BELOW

IRIS Goddess of RainbowCERES Goddess of Agriculture and all the fruits of the earth

(Nature growth prosperity rebirth ndash notions intimately connected to marriage)JUNO The majestic Queen of Heavens and wife of Jupiter (Jupiter is the king of Gods)

VENUS The Goddess of love CUPID Son of Venus PLUTO God of death (In the play referred by Shakespeare as lsquoDisrsquo which is a Roman name for Pluto)

ORIGINAL TEXT PARAPHRASEPROSPEROWellmdash

PROSPERONow come Ariel Let there be too many rather than too few

Now come my Ariel Bring a corollaryRather than want a spirit Appear and pertly[to Ferdinand and Miranda]No tongue all eyes Be silent

spirits in attendance Appear briskly

[to Ferdinand and Miranda]Look with your eyes but do not say a word

[Soft music] [Soft music][Enter Iris] [Enter Iris]

IRISCeres most bounteous lady thy rich leasOf wheat rye barley vetches oats and peasThy turfy mountains where live nibbling sheepAnd flat meads thatched with stover them to keepThy banks with pioned and twilled brimsWhich spongy April at thy hest betrimsTo make cold nymphs chaste crowns and thybroom-grovesWhose shadow the dismissegraved bachelor lovesBeing lass-lorn thy pole clipped vineyardAnd thy sea-marge sterile and rocky-hardWhere thou thyself dost airmdashthe Queen othrsquoSkyWhose watery arch and messenger am IBids thee leave these and with her sovereign grace[Juno appears] Here on this grass-plot in this very placeTo come and sport Her peacocks fly amainApproach rich Ceres her to entertain

IRISCeres most generous lady you are the cause of rich fields or fertile land where wheat rye barley beans oats and peas grow the grassy mountains where the sheep graze and the flat meadows covered with coarse hay to be used as fodder for cattleYour banks are covered with marsh-marigolds and reeds and the rainy April under your orders brings forth to make for the maids who are not in love beautiful crowns your woods where the broom flourishes and where the bachelor who has been dismissed by the maid he loved lies down being forsaken your vineyard in which the poles are embraced by the vines and the margin of the sea which is barren and rocky where you roam about to enjoy the fresh air ndash the queen of the sky (Juno) whose messenger I am besides being represented as the rainbow bids you leave all these and with her majesty here on this grassy plot in this very place come and sport her peacocks carry her fast in her chariot through the air and are making their way here approach rich Ceres to welcome her

[Enter Ariel as Ceres] [Enter Ariel as Ceres]

CERESHail many-coloured messenger that neerDost disobey the wife of JupiterWho with thy saffron wings upon my flowersDiffusest honey-drops refreshing showersAnd with each end of thy blue bow dost crownMy bosky acres and my unshrubbed downRich scarf to my proud earth Why hath thy queenSummoned me hither to this short-grassed green

CERESWelcome rainbow that never dared disobey Juno the wife of Jupiter who with your orange coloured rays spread honey-drops refreshing showers And with each end of thy blue bow drown my bushy acres and my hilly country which is free from shrubs you thus forming a rich scarf Why has your queen called me here to this place covered with short grass

IRISA contract of true love to celebrateAnd some donation freely to estateOn the blest lovers

IRISI have called you to celebrate a contract of true love and bestow some liberal gift upon the blessed lovers

ORIGINAL TEXT PARAPHRASECERESTell me heavenly bowIf Venus or her son as thou dost knowDo now attend the queen Since they did plotThe means that dusky Dis my daughter gotHer and her blind boys scandaled companyI have forsworn

CERESTell me heavenly bow if Venus the Goddess of love or Cupid her son and pedlar of passion at this time attend the heavenly queen Juno because you are sure to know Since the day they conspired against me and dark Pluto took away my daughter here and Cupidrsquos disgraceful company I have left off

IRISOf her societyBe not afraid I met her deityCutting the clouds towards Pathos and her sonDove-drawn with her Here thought they to have doneSome wanton charm upon this man and miad

IRISBe not afraid of her company I met her deity moving on the clouds towards Paphos the sacred home of Venus on the island of Cyprus along with her son on her chariot drawn by doves Here they contemplated to exercise a charm upon this man and maid producing

Whose vows are that no bed-right shall be paidTill Hymens torch be lightedmdashbut in vainMarss hot minion is returned againHer waspish-headed son has broke his arrowsSwears he will shoot no more but play with sparrowsAnd be a boy right out

wantonness before the actual marriage ceremony but did not succeed Venus has returned her irritable son has broken his arrows and swears that he will give up his practice of trying to inspire love but play with sparrows and be a boy again

[Music is heard] [Music is heard]

CERESHighst queen of stateGreat Juno comes I know her by her gait

CERESHighest queen of state Great Juno there she comes I know here by her gait

[Enter Juno] [Enter Juno]

JUNOHow does my bounteous sister Go with meTo bless this twain that they may Prosperous beAnd honoured in their issue

JUNOHow are you doing my generous sister Come with me to bless this couple so that they may be prosperous and fortunate in their children

[They sing] [They sing]

JUNOHonour riches marriage-blessingLong continuance and increasingHourly joys be still upon youJuno sings her blessings upon you

JUNOMay honour riches happiness in marriage long continuance and increase of those boons ever rest upon you as hourly joys Juno showers down upon you her blessings in song

CERESEarths increase foison plentyBarns and garners never emptyVines and clustring bunches growingPlants and goodly burden bowingSpring come to you at the farthestIn the very end of harvestScarcity and want shall shun youCeresrsquo blessing so is on you

CERESMay you have the plenty of earthrsquos produce Your barns and granaries may never be empty Your vines may grow with clustering bunches Your fruit trees may be heavily laden with their fruit May there be continuous spring and harvest May scantiness and want leave you forever Such is the blessing of Ceres upon you

FERDINANDThis is a most majestic vision andHarmoniously charmingly May I be boldTo think these spirits

FERDINANDThis is a great vision and magically melodious Should I suppose the characters (taking part in the masque) are spirits

PROSPEROSpirits which by mine artI have from their confines calld to enactMy present fancies

PROSPEROYes they are spirits whom I have summoned from the regions to which they are confined to carry into effect my fanciful designs

ORIGINAL TEXT PARAPHRASEFERDINANDLet me live here everSo rare a wondered father and a wifeMakes this place paradise

FERDINANDI should like to live here forever Such a wise and wonderful father makes this place a paradise

[Juno and Ceres whisper and send Iris on employment] [Juno and Ceres whisper and send Iris on employment]

PROSPEROSweet now silence

PROSPEROMy dear Ferdinand speak no more Juno and Ceres are

Juno and Ceres whisper seriouslyTheres something else to do Hush and be muteOr else our spell is marred

whispering with a solemn look There is something else coming Silence Or else our magic will be spoilt

IRISYour nymphs called naiads of the wandering brooksWith your sedged crowns and over-harmless looksLeave your crisp channels and on this green landAnswer your summons Juno does commandCome temperate nymphs and help to celebrateA contract of true love Be not too late

IRISYou nymphs called Naiads denizens (M inhabitants) of the running stream with your chaplets of sedge and ever-helpful looks leave your wrinkled channels and on the green land answer the summons sent to you Juno has ordered some chaste nymphs and help to celebrate a noble and true marriage Donrsquot delay

[Enter certain nymphs] [Enter certain nymphs]You sunburnt sicklemen of August wearyCome hither from the furrow and be merryMake holiday your rye-straw hats put onAnd these fresh nymphs encounter every oneIn country footing

You sunburnt harvesters weary from the effects of the heat in August come here from the furrowed land and rejoice Make holiday with your rye-straw hats upon you and meet these fresh nymphs and join in country dancing

[Enter certain reapers properly habited They join with the nymphs in a graceful dance towards the end whereof Prospero starts suddenly and speaks]

[Enter certain reapers properly habited They join with the nymphs in a graceful dance towards the end whereof Prospero starts suddenly and speaks]

PROSPERO[aside] I had forgot that foul conspiracyOf the beast Caliban and his confederatesAgainst my life The minute of their plotIs almost come [to the spirits]Well done Avoidno more

PROSPERO(Aside)I had forgotten the wicked conspiracy of the beast Caliban and his accomplices against my life the time of their plot has almost arrived ndash (To the Spirits) well done depart no more of this

[To a strange hollow and confused noise the spirits heavily vanish]

[The spirits depart]

ORIGINAL TEXT PARAPHRASEFERDINANDLet me live here everSo rare a wondered father and a wifeMakes this place paradise

FERDINANDI should like to live here forever Such a wise and wonderful father makes this place a paradise

[Juno and Ceres whisper and send Iris on employment] [Juno and Ceres whisper and send Iris on employment]

PROSPEROSweet now silenceJuno and Ceres whisper seriouslyTheres something else to do Hush and be muteOr else our spell is marred

PROSPEROMy dear Ferdinand speak no more Juno and Ceres are whispering with a solemn look There is something else coming Silence Or else our magic will be spoilt

IRISYour nymphs called naiads of the wandering brooksWith your sedged crowns and over-harmless looksLeave your crisp channels and on this green landAnswer your summons Juno does commandCome temperate nymphs and help to celebrateA contract of true love Be not too late

IRISYou nymphs called Naiads denizens (M inhabitants) of the running stream with your chaplets of sedge and ever-helpful looks leave your wrinkled channels and on the green land answer the summons sent to you Juno has ordered some chaste nymphs and help to celebrate a noble and true marriage Donrsquot delay

[Enter certain nymphs] [Enter certain nymphs]You sunburnt sicklemen of August wearyCome hither from the furrow and be merryMake holiday your rye-straw hats put onAnd these fresh nymphs encounter every oneIn country footing

You sunburnt harvesters weary from the effects of the heat in August come here from the furrowed land and rejoice Make holiday with your rye-straw hats upon you and meet these fresh nymphs and join in country dancing

[Enter certain reapers properly habited They join with the nymphs in a graceful dance towards the end whereof Prospero starts suddenly and speaks]

[Enter certain reapers properly habited They join with the nymphs in a graceful dance towards the end whereof Prospero starts suddenly and speaks]

PROSPERO[aside] I had forgot that foul conspiracyOf the beast Caliban and his confederatesAgainst my life The minute of their plotIs almost come [to the spirits]Well done Avoidno more

PROSPERO(Aside)I had forgotten the wicked conspiracy of the beast Caliban and his accomplices against my life the time of their plot has almost arrived ndash (To the Spirits) well done depart no more of this

[To a strange hollow and confused noise the spirits heavily vanish]

[The spirits depart]

Ac-12 27420 topic Revaluation of Assets and Liabilities

REVALUATION OF ASSETS AND LIABILITIES

On admission of a new partner the firm stands reconstituted and consequently the assets are revalued and liabilities are reassessed It is necessary to show the true position of the firm at the time of admission of a new partner If the values of the assets are raised gain will increase the capital of the existing partners Similarly any decrease in the value of assets ie loss will decrease the capital of the existing partners For this purpose alsquoRevaluation Accountrsquo is prepared This account is credited with all increases in the value of assets and decrease in the value of liabilities It is debited with decrease on account of value of assets and increase in the value of liabilities The balance of this account shows a gain or loss on revaluation which is transferred to the existing partnerrsquos capital account in existing profit sharing ratioAccounting for Revaluation of Assets and Liabilities when there is a Changein the Profit Sharing Ratio of Existing PartnersAssets and liabilities of a firm must also be revalued at the time of change in profit sharing ratio of existing partners The reason is that the realisable or actual value of assets and liabilities may be different from those shown in the Balance Sheet It is possible that with the passage of time some of the assets might have appreciated in value while the value of certain other assets might have decreased and no record has been made of such changes in the books of accounts Similarly there may be some unrecorded assets amp libilities that may have to be accounted for Revaluation of assets and reassessments of liabilities becomes necessary because the change in the

value of assets and liabilities belongs to the period to change in profit sharing ratio and hence must be shared by the partners in their old profit sharing ratio Revaluation of assets and reassessment of liabilities may be given effect to in two different ways (a) When revised values are to be recorded in the books and(b) When revised values are not to be recorded in the books

When revised values are to be recorded in the booksIn such a case revaluation of assets and reassessment of liabilities is done with the help of a new account called lsquoRevaluation Accountrsquo Sometimes this account is also called as lsquoProfit amp Loss Adjustment Acrsquo If there is a loss due to revaluation revaluation account is debited and if the revaluation results in a profit the revaluation account is credited The following journal entries made for this purpose are

(i) For increase in the value of assetsAsset Ac Dr (individually)To Revaluation Ac(ii) For decrease in the value of AssetRevaluation Ac Dr (individually)To Asset Ac[Decrease in the value of assets](iii) For increase in the value of LiabilitiesRevaluation Ac Dr (individually)To Liabilities Ac[Increase in the value of Liabilities](iv) For decrease in the value of LiabilitiesLiabilities Ac DrTo Revaluation Ac[Decrease in the value of Liabilities](v) For unrecorded AssetsAsset Ac [unrecorded] DrTo Revaluation Ac[Unrecorded asset recorded at actual value](vi) For unrecorded Liability Revaluation Ac DrTo Liability Ac [unrecorded][Unrecorded Liability recorded at actual value](vii) For transfer of gain on revaluationRevaluation Ac DrTo Existing Partnerrsquos CapitalCurrent Ac[Profit on revaluation transferred to capital account in existing ratio](viii) For transfer of loss on revaluationExisting Partnerrsquos CapitalCurrent Ac DrTo Revaluation Ac[Loss on revaluation transferred to capital account in existing ratio](a) When revaluation account shows gain Revaluation Ac DrTo Partnerrsquos Capital Ac (Old Profit Sharing Ratio)(Profit on revaluation credited to Partnerrsquos Capital Ac)(b) Above entry is reversed when revaluation account shows loss Partners Capital Acs (Old Profit Sharing Ratio) DrTo Revaluation Ac(Loss on revaluation debited to Partnerrsquos Capital Acs)

Proforma of Revaluation Account is given as under

Revaluation Account

Dr Cr Particulars ` Amount Particulars ` Amount To Decrease in value of assets By Increase in value of assets To Increase in value of liabilities By Decrease in value of liabilities To Unrecorded liabilities By Unrecorded assets To Gain on Revaluation (Transferred) By Loss on Revalution (Transferred)

ECO ndash12 2742020Topic- ELASTICITY OF DEMAND

CHAPTER - ELASTICITY OF DEMANDMEANINGDemand for a commodity is affected by many factors such as its price price of related goods income of its buyer tastes and preferences etc Elasticity means degree of response Elasticity of demand means degree of responsiveness of demand Demand for a commodity responds to change in price price of related goods income etc So we have three dimensions of elasticity of demandDIMENSION OF ELASTICITY OF DEMAND TYPES OF ELASTICITY OF DEMAND

Price elasticity of demand Income elasticity of demand Cross Elasticity of demand

Price elasticity of demand Price elasticity of demand means degree of responsiveness of demand for a commodity to the change in its price For example if demand for a commodity rises by 10 due to 5 fall in its price Price elasticity of demand (ep)=Percentage change in quantity demanded Percentage change in price of the commodity = 10 ( -)5 = ( - )2Note that ep will always be negative due to inverse relationship of price and quantity demanded

(ii) Income elasticity of demand Income elasticity of demand refers to the degree of responsiveness of demand for a commodity to the change in income of its buyer Suppose income of buyer rises by 10 and his demand for a commodity rises by 20 then Income elasticity of demand (ey)= change in quantity demanded change in price of the commodity =20 10 = 2

Cross Elasticity of demandCross elasticity of demand means the degree of responsiveness of demand for a commodity to the change in price of its related goods (substitute goods or complementary goods) Suppose demand for a commodity rises by 10 due to 5 rise in price of its substitute good then Cross elasticity of demand (ec) = change in quantity demanded change in price of related good = 10 2 = 5 (Tastes and preferences cannot be expressed numerically So elasticity ofdemand cannot be numerically expressed)

  • Chapter 1 Force (Summary)
  • Distinguish between external sovereignty and internal sovereignty
    • NAND Gate
      • Logic diagram
      • Truth Table
        • NOR Gate
          • Logic diagram
          • Truth Table
            • XOR Gate
              • Logic diagram
              • Truth Table
                • XNOR Gate
                  • Logic diagram
                  • Truth Table
                      • Physics
                      • Chapter 1 Electric Field ( Electric Dipole) (Summary)
Page 47:  · Web viewWe all know that Nouns are divided into two parts: common noun and proper noun.Apart from common and proper noun, we will also study about collective noun and compound

Socialist group played a prominent part Notable among the Jayprakash Narayan Rammonohar Lohia Aruna Asaf Ali

Political science

Topic-Franchise and Representation

Summary

The election commission

The Election Commission of India is an autonomous constitutional authority responsible for administering Union and State election processes in India The body administers elections to the Lok Sabha Rajya Sabha State Legislative Assemblies in India and the offices of the President and Vice President in the country

Functions of election commission-

India is a sovereign socialist secular democratic republic Democracy runs like a golden thread in the social economic and political fabric woven by the Constitution given by lsquoWe the People of Indiarsquo unto ourselves The concept of democracy as visualised by the Constitution pre-supposes the representation of the people in Parliament and State legislatures by the method of election The Supreme Court has held that democracy is one of the inalienable basic features of the Constitution of India and forms part of its basic structure The Constitution of India adopted a Parliamentary form of government Parliament consists of the President of India and the two Houses mdash Rajya Sabha and Lok Sabha India being a Union of states has separate state legislatures for each state State legislatures consist of the Governor and two Houses mdash Legislative Council and Legislative Assembly mdash in seven states namely Andhra Pradesh Telangana Bihar Jammu amp Kashmir Karnataka Maharashtra and Uttar Pradesh and of the Governor and the state Legislative Assembly in the remaining 22 states Apart from the above two out of the seven Union Territories namely National Capital Territory of Delhi and Puducherry also have their Legislative Assemblies

ExecutionShort notes-Election commissionFunctions of election commission

Homework- Learn

Computer

Science

Computer hardware NAND Gate

A NOT-AND operation is known as NAND operation It has n input (n gt= 2) and one output

Logic diagram

Truth Table

NOR Gate

A NOT-OR operation is known as NOR operation It has n input (n gt= 2) and one output

Logic diagram

Truth Table

XOR Gate

XOR or Ex-OR gate is a special type of gate It can be used in the half

adder full adder and subtractor The exclusive-OR gate is abbreviated as EX-OR gate or sometime as X-OR gate It has n input (n gt= 2) and one output

Logic diagram

Truth Table

XNOR Gate

XNOR gate is a special type of gate It can be used in the half adder full adder and subtractor The exclusive-NOR gate is abbreviated as EX-NOR gate or sometime as X-NOR gate It has n input (n gt= 2) and one output

Logic diagram

Truth Table

Physics

Chapter 1 Electric Field ( Electric Dipole) (Summary)

Here we will derive Expression of electric field at broad side

On position of dipole

Execution

Q With the help of a labelled diagram obtain an expression for the electric field intensity E at any point on the equitorial line ( broad-side on position) of an electric dipole

Ans

E1 E1sinθ

E θ P E1 θ

( r2+L2)12 E2 E

r E2 E2sinθ

-q θ L O L +qA B

Let us consider that the point P is situated on the right bisector of the dipole AB at a distance r meter from its midpoint O

Let E1 and E2 be the electric field intensities of the electric field at P due to charge +q and ndashq of the dipole resp The distance of P from each charge is ( r2+L2)12

So E1 = 14 πϵ q

(r 2+L 2) away from +q

E2 = 14 πϵ q

(r 2+L 2) towards ndashq

The magnitudes of E1 and E2 are equal but directions are different Now resolving E1 and E2 into two components parallel and perpendicular to AB we get

The components perpendicular to AB E1sinθ and E2sinθ cancel each other because they are equal and opposite

The components parallel to AB are E1cosθ and E2 cosθ are in same direction and add up

So resultant intensity of electric field at the point P is

E = E1cosθ + E2 cosθ

E = 14 πϵ q

(r 2+L 2) 2 cosθ

Now from fig we have cosθ =BOBP = L (r2+L2)12

So we get E = 14 πϵ 2qL ( r2+L2)32

Now electric dipole moment p= 2qL

So E = 14 πϵ p ( r2+L2)32

HW Find the expression of Electric field as done here but this time take r gtgt 2L

Also find the expression of torque experience by a dipole

(Hint Electric force experienced by charges of dipole in electric field is qE each Let θ be the angle which dipole makes with electric lines of force then perpendicular distance between two charges is 2Lsinθ Then torque = force x perp distance = qE x 2L sinθ So τ=pE sinθ where p =2qL )

STUDY MATERIAL

Class XIISubject Eng Literature (The Tempest ndash William Shakespeare) Topic Act IV Scene 1 Lines 84 to 133 (Iris hellip A contract of true love Be not too late ) Date 27th April 2020 (4th Period)

[Students should read the original play and also the paraphrase given in the school prescribed textbook]Summary Questions amp Answers

o Ceres soon appears and comes to know that she has been summoned to celebrate the contract of true love

o Ceres expresses her unwillingness to meet Venus and Cupid as she has shunned their company

o Ceres and Juno both bestow their blessings upon Ferdinand and Miranda with June gifting honour riches happiness in marriage and Ceres presents plenty of earthrsquos produce

o Iris summons the water-nymphs and reapers to come and celebrate a contract

(1) IRIS Of her society (Line 91-101)

Be not afraid I met her deity

Cutting the clouds towards Pathos and her sonDove-drawn with her Here thought they to have doneSome wanton charm upon this man and maidWhose vows are that no bed-right shall be paidTill Hymens torch be lightedmdashbut in vainMarss hot minion is returned againHer waspish-headed son has broke his arrowsSwears he will shoot no more but play with sparrowsAnd be a boy right out

(i) Where were Venus and Cupid seen flying How were they travelling Why did they want to join the marriage celebration of Ferdinand and Miranda

of true love

Venus and Cupid were seen flying through the air towards Paphos the famous city which is situated on the island of Cyprus They were travelling by air-borne chariot drawn by doves They certainly wanted to come here in order to play some amorous trick upon Ferdinand and Miranda who are under a vow not to gratify their physical desires till the holy ceremony of their marriage has been performed(ii) What have Venus and Cupid done after failing in their plan

After being failure of their plan Venus who is a very passionate deity and who is the mistress of Mars (the god of war) has gone back while here ill-tempered son Cupid has broken his arrows of love in his state of desperation(iii) What has Cupid firmly decided

Cupid is feeling so disappointed that he has firmly decided to shoot no more arrows to arouse love in human hearts but to spend his time playing with sparrows Thus he would now become just a boy and would give up his original function of shooting arrows on human beings to make them fall in love(iv) What vow had Ceres taken How did Ceres feel at the abduction

After the abduction of her daughter Prosperina by Pluto Ceres had taken a vow to always keep away from the disgraceful company of Venus and her blind son Cupid the god of love Ceres felt deeply distressed when Pluto had carried off her daughter and had made her his wife by force(v) Why has Ceres not forgiven Venus and her blind son For what do Ceres want to be sure

As the abduction had been manipulated by Venus the goddess of beauty and love and her blind son Cupid Ceres has never forgiven them for their part in the whole plot Ceres wants to be sure that she would not have to meet Venus and Cupid who had engineered the abduction of her daughter Prosperina

AS THIS lsquoMASQUErsquo SCENE IS VERY IMPORTANT IN THE PLAY THE PARAPHRASE OF THE ENTIRE PORTION OF MASQUE SCENE (Act IV Lines 58 to 143) IS GIVEN BELOW

IRIS Goddess of RainbowCERES Goddess of Agriculture and all the fruits of the earth

(Nature growth prosperity rebirth ndash notions intimately connected to marriage)JUNO The majestic Queen of Heavens and wife of Jupiter (Jupiter is the king of Gods)

VENUS The Goddess of love CUPID Son of Venus PLUTO God of death (In the play referred by Shakespeare as lsquoDisrsquo which is a Roman name for Pluto)

ORIGINAL TEXT PARAPHRASEPROSPEROWellmdash

PROSPERONow come Ariel Let there be too many rather than too few

Now come my Ariel Bring a corollaryRather than want a spirit Appear and pertly[to Ferdinand and Miranda]No tongue all eyes Be silent

spirits in attendance Appear briskly

[to Ferdinand and Miranda]Look with your eyes but do not say a word

[Soft music] [Soft music][Enter Iris] [Enter Iris]

IRISCeres most bounteous lady thy rich leasOf wheat rye barley vetches oats and peasThy turfy mountains where live nibbling sheepAnd flat meads thatched with stover them to keepThy banks with pioned and twilled brimsWhich spongy April at thy hest betrimsTo make cold nymphs chaste crowns and thybroom-grovesWhose shadow the dismissegraved bachelor lovesBeing lass-lorn thy pole clipped vineyardAnd thy sea-marge sterile and rocky-hardWhere thou thyself dost airmdashthe Queen othrsquoSkyWhose watery arch and messenger am IBids thee leave these and with her sovereign grace[Juno appears] Here on this grass-plot in this very placeTo come and sport Her peacocks fly amainApproach rich Ceres her to entertain

IRISCeres most generous lady you are the cause of rich fields or fertile land where wheat rye barley beans oats and peas grow the grassy mountains where the sheep graze and the flat meadows covered with coarse hay to be used as fodder for cattleYour banks are covered with marsh-marigolds and reeds and the rainy April under your orders brings forth to make for the maids who are not in love beautiful crowns your woods where the broom flourishes and where the bachelor who has been dismissed by the maid he loved lies down being forsaken your vineyard in which the poles are embraced by the vines and the margin of the sea which is barren and rocky where you roam about to enjoy the fresh air ndash the queen of the sky (Juno) whose messenger I am besides being represented as the rainbow bids you leave all these and with her majesty here on this grassy plot in this very place come and sport her peacocks carry her fast in her chariot through the air and are making their way here approach rich Ceres to welcome her

[Enter Ariel as Ceres] [Enter Ariel as Ceres]

CERESHail many-coloured messenger that neerDost disobey the wife of JupiterWho with thy saffron wings upon my flowersDiffusest honey-drops refreshing showersAnd with each end of thy blue bow dost crownMy bosky acres and my unshrubbed downRich scarf to my proud earth Why hath thy queenSummoned me hither to this short-grassed green

CERESWelcome rainbow that never dared disobey Juno the wife of Jupiter who with your orange coloured rays spread honey-drops refreshing showers And with each end of thy blue bow drown my bushy acres and my hilly country which is free from shrubs you thus forming a rich scarf Why has your queen called me here to this place covered with short grass

IRISA contract of true love to celebrateAnd some donation freely to estateOn the blest lovers

IRISI have called you to celebrate a contract of true love and bestow some liberal gift upon the blessed lovers

ORIGINAL TEXT PARAPHRASECERESTell me heavenly bowIf Venus or her son as thou dost knowDo now attend the queen Since they did plotThe means that dusky Dis my daughter gotHer and her blind boys scandaled companyI have forsworn

CERESTell me heavenly bow if Venus the Goddess of love or Cupid her son and pedlar of passion at this time attend the heavenly queen Juno because you are sure to know Since the day they conspired against me and dark Pluto took away my daughter here and Cupidrsquos disgraceful company I have left off

IRISOf her societyBe not afraid I met her deityCutting the clouds towards Pathos and her sonDove-drawn with her Here thought they to have doneSome wanton charm upon this man and miad

IRISBe not afraid of her company I met her deity moving on the clouds towards Paphos the sacred home of Venus on the island of Cyprus along with her son on her chariot drawn by doves Here they contemplated to exercise a charm upon this man and maid producing

Whose vows are that no bed-right shall be paidTill Hymens torch be lightedmdashbut in vainMarss hot minion is returned againHer waspish-headed son has broke his arrowsSwears he will shoot no more but play with sparrowsAnd be a boy right out

wantonness before the actual marriage ceremony but did not succeed Venus has returned her irritable son has broken his arrows and swears that he will give up his practice of trying to inspire love but play with sparrows and be a boy again

[Music is heard] [Music is heard]

CERESHighst queen of stateGreat Juno comes I know her by her gait

CERESHighest queen of state Great Juno there she comes I know here by her gait

[Enter Juno] [Enter Juno]

JUNOHow does my bounteous sister Go with meTo bless this twain that they may Prosperous beAnd honoured in their issue

JUNOHow are you doing my generous sister Come with me to bless this couple so that they may be prosperous and fortunate in their children

[They sing] [They sing]

JUNOHonour riches marriage-blessingLong continuance and increasingHourly joys be still upon youJuno sings her blessings upon you

JUNOMay honour riches happiness in marriage long continuance and increase of those boons ever rest upon you as hourly joys Juno showers down upon you her blessings in song

CERESEarths increase foison plentyBarns and garners never emptyVines and clustring bunches growingPlants and goodly burden bowingSpring come to you at the farthestIn the very end of harvestScarcity and want shall shun youCeresrsquo blessing so is on you

CERESMay you have the plenty of earthrsquos produce Your barns and granaries may never be empty Your vines may grow with clustering bunches Your fruit trees may be heavily laden with their fruit May there be continuous spring and harvest May scantiness and want leave you forever Such is the blessing of Ceres upon you

FERDINANDThis is a most majestic vision andHarmoniously charmingly May I be boldTo think these spirits

FERDINANDThis is a great vision and magically melodious Should I suppose the characters (taking part in the masque) are spirits

PROSPEROSpirits which by mine artI have from their confines calld to enactMy present fancies

PROSPEROYes they are spirits whom I have summoned from the regions to which they are confined to carry into effect my fanciful designs

ORIGINAL TEXT PARAPHRASEFERDINANDLet me live here everSo rare a wondered father and a wifeMakes this place paradise

FERDINANDI should like to live here forever Such a wise and wonderful father makes this place a paradise

[Juno and Ceres whisper and send Iris on employment] [Juno and Ceres whisper and send Iris on employment]

PROSPEROSweet now silence

PROSPEROMy dear Ferdinand speak no more Juno and Ceres are

Juno and Ceres whisper seriouslyTheres something else to do Hush and be muteOr else our spell is marred

whispering with a solemn look There is something else coming Silence Or else our magic will be spoilt

IRISYour nymphs called naiads of the wandering brooksWith your sedged crowns and over-harmless looksLeave your crisp channels and on this green landAnswer your summons Juno does commandCome temperate nymphs and help to celebrateA contract of true love Be not too late

IRISYou nymphs called Naiads denizens (M inhabitants) of the running stream with your chaplets of sedge and ever-helpful looks leave your wrinkled channels and on the green land answer the summons sent to you Juno has ordered some chaste nymphs and help to celebrate a noble and true marriage Donrsquot delay

[Enter certain nymphs] [Enter certain nymphs]You sunburnt sicklemen of August wearyCome hither from the furrow and be merryMake holiday your rye-straw hats put onAnd these fresh nymphs encounter every oneIn country footing

You sunburnt harvesters weary from the effects of the heat in August come here from the furrowed land and rejoice Make holiday with your rye-straw hats upon you and meet these fresh nymphs and join in country dancing

[Enter certain reapers properly habited They join with the nymphs in a graceful dance towards the end whereof Prospero starts suddenly and speaks]

[Enter certain reapers properly habited They join with the nymphs in a graceful dance towards the end whereof Prospero starts suddenly and speaks]

PROSPERO[aside] I had forgot that foul conspiracyOf the beast Caliban and his confederatesAgainst my life The minute of their plotIs almost come [to the spirits]Well done Avoidno more

PROSPERO(Aside)I had forgotten the wicked conspiracy of the beast Caliban and his accomplices against my life the time of their plot has almost arrived ndash (To the Spirits) well done depart no more of this

[To a strange hollow and confused noise the spirits heavily vanish]

[The spirits depart]

ORIGINAL TEXT PARAPHRASEFERDINANDLet me live here everSo rare a wondered father and a wifeMakes this place paradise

FERDINANDI should like to live here forever Such a wise and wonderful father makes this place a paradise

[Juno and Ceres whisper and send Iris on employment] [Juno and Ceres whisper and send Iris on employment]

PROSPEROSweet now silenceJuno and Ceres whisper seriouslyTheres something else to do Hush and be muteOr else our spell is marred

PROSPEROMy dear Ferdinand speak no more Juno and Ceres are whispering with a solemn look There is something else coming Silence Or else our magic will be spoilt

IRISYour nymphs called naiads of the wandering brooksWith your sedged crowns and over-harmless looksLeave your crisp channels and on this green landAnswer your summons Juno does commandCome temperate nymphs and help to celebrateA contract of true love Be not too late

IRISYou nymphs called Naiads denizens (M inhabitants) of the running stream with your chaplets of sedge and ever-helpful looks leave your wrinkled channels and on the green land answer the summons sent to you Juno has ordered some chaste nymphs and help to celebrate a noble and true marriage Donrsquot delay

[Enter certain nymphs] [Enter certain nymphs]You sunburnt sicklemen of August wearyCome hither from the furrow and be merryMake holiday your rye-straw hats put onAnd these fresh nymphs encounter every oneIn country footing

You sunburnt harvesters weary from the effects of the heat in August come here from the furrowed land and rejoice Make holiday with your rye-straw hats upon you and meet these fresh nymphs and join in country dancing

[Enter certain reapers properly habited They join with the nymphs in a graceful dance towards the end whereof Prospero starts suddenly and speaks]

[Enter certain reapers properly habited They join with the nymphs in a graceful dance towards the end whereof Prospero starts suddenly and speaks]

PROSPERO[aside] I had forgot that foul conspiracyOf the beast Caliban and his confederatesAgainst my life The minute of their plotIs almost come [to the spirits]Well done Avoidno more

PROSPERO(Aside)I had forgotten the wicked conspiracy of the beast Caliban and his accomplices against my life the time of their plot has almost arrived ndash (To the Spirits) well done depart no more of this

[To a strange hollow and confused noise the spirits heavily vanish]

[The spirits depart]

Ac-12 27420 topic Revaluation of Assets and Liabilities

REVALUATION OF ASSETS AND LIABILITIES

On admission of a new partner the firm stands reconstituted and consequently the assets are revalued and liabilities are reassessed It is necessary to show the true position of the firm at the time of admission of a new partner If the values of the assets are raised gain will increase the capital of the existing partners Similarly any decrease in the value of assets ie loss will decrease the capital of the existing partners For this purpose alsquoRevaluation Accountrsquo is prepared This account is credited with all increases in the value of assets and decrease in the value of liabilities It is debited with decrease on account of value of assets and increase in the value of liabilities The balance of this account shows a gain or loss on revaluation which is transferred to the existing partnerrsquos capital account in existing profit sharing ratioAccounting for Revaluation of Assets and Liabilities when there is a Changein the Profit Sharing Ratio of Existing PartnersAssets and liabilities of a firm must also be revalued at the time of change in profit sharing ratio of existing partners The reason is that the realisable or actual value of assets and liabilities may be different from those shown in the Balance Sheet It is possible that with the passage of time some of the assets might have appreciated in value while the value of certain other assets might have decreased and no record has been made of such changes in the books of accounts Similarly there may be some unrecorded assets amp libilities that may have to be accounted for Revaluation of assets and reassessments of liabilities becomes necessary because the change in the

value of assets and liabilities belongs to the period to change in profit sharing ratio and hence must be shared by the partners in their old profit sharing ratio Revaluation of assets and reassessment of liabilities may be given effect to in two different ways (a) When revised values are to be recorded in the books and(b) When revised values are not to be recorded in the books

When revised values are to be recorded in the booksIn such a case revaluation of assets and reassessment of liabilities is done with the help of a new account called lsquoRevaluation Accountrsquo Sometimes this account is also called as lsquoProfit amp Loss Adjustment Acrsquo If there is a loss due to revaluation revaluation account is debited and if the revaluation results in a profit the revaluation account is credited The following journal entries made for this purpose are

(i) For increase in the value of assetsAsset Ac Dr (individually)To Revaluation Ac(ii) For decrease in the value of AssetRevaluation Ac Dr (individually)To Asset Ac[Decrease in the value of assets](iii) For increase in the value of LiabilitiesRevaluation Ac Dr (individually)To Liabilities Ac[Increase in the value of Liabilities](iv) For decrease in the value of LiabilitiesLiabilities Ac DrTo Revaluation Ac[Decrease in the value of Liabilities](v) For unrecorded AssetsAsset Ac [unrecorded] DrTo Revaluation Ac[Unrecorded asset recorded at actual value](vi) For unrecorded Liability Revaluation Ac DrTo Liability Ac [unrecorded][Unrecorded Liability recorded at actual value](vii) For transfer of gain on revaluationRevaluation Ac DrTo Existing Partnerrsquos CapitalCurrent Ac[Profit on revaluation transferred to capital account in existing ratio](viii) For transfer of loss on revaluationExisting Partnerrsquos CapitalCurrent Ac DrTo Revaluation Ac[Loss on revaluation transferred to capital account in existing ratio](a) When revaluation account shows gain Revaluation Ac DrTo Partnerrsquos Capital Ac (Old Profit Sharing Ratio)(Profit on revaluation credited to Partnerrsquos Capital Ac)(b) Above entry is reversed when revaluation account shows loss Partners Capital Acs (Old Profit Sharing Ratio) DrTo Revaluation Ac(Loss on revaluation debited to Partnerrsquos Capital Acs)

Proforma of Revaluation Account is given as under

Revaluation Account

Dr Cr Particulars ` Amount Particulars ` Amount To Decrease in value of assets By Increase in value of assets To Increase in value of liabilities By Decrease in value of liabilities To Unrecorded liabilities By Unrecorded assets To Gain on Revaluation (Transferred) By Loss on Revalution (Transferred)

ECO ndash12 2742020Topic- ELASTICITY OF DEMAND

CHAPTER - ELASTICITY OF DEMANDMEANINGDemand for a commodity is affected by many factors such as its price price of related goods income of its buyer tastes and preferences etc Elasticity means degree of response Elasticity of demand means degree of responsiveness of demand Demand for a commodity responds to change in price price of related goods income etc So we have three dimensions of elasticity of demandDIMENSION OF ELASTICITY OF DEMAND TYPES OF ELASTICITY OF DEMAND

Price elasticity of demand Income elasticity of demand Cross Elasticity of demand

Price elasticity of demand Price elasticity of demand means degree of responsiveness of demand for a commodity to the change in its price For example if demand for a commodity rises by 10 due to 5 fall in its price Price elasticity of demand (ep)=Percentage change in quantity demanded Percentage change in price of the commodity = 10 ( -)5 = ( - )2Note that ep will always be negative due to inverse relationship of price and quantity demanded

(ii) Income elasticity of demand Income elasticity of demand refers to the degree of responsiveness of demand for a commodity to the change in income of its buyer Suppose income of buyer rises by 10 and his demand for a commodity rises by 20 then Income elasticity of demand (ey)= change in quantity demanded change in price of the commodity =20 10 = 2

Cross Elasticity of demandCross elasticity of demand means the degree of responsiveness of demand for a commodity to the change in price of its related goods (substitute goods or complementary goods) Suppose demand for a commodity rises by 10 due to 5 rise in price of its substitute good then Cross elasticity of demand (ec) = change in quantity demanded change in price of related good = 10 2 = 5 (Tastes and preferences cannot be expressed numerically So elasticity ofdemand cannot be numerically expressed)

  • Chapter 1 Force (Summary)
  • Distinguish between external sovereignty and internal sovereignty
    • NAND Gate
      • Logic diagram
      • Truth Table
        • NOR Gate
          • Logic diagram
          • Truth Table
            • XOR Gate
              • Logic diagram
              • Truth Table
                • XNOR Gate
                  • Logic diagram
                  • Truth Table
                      • Physics
                      • Chapter 1 Electric Field ( Electric Dipole) (Summary)
Page 48:  · Web viewWe all know that Nouns are divided into two parts: common noun and proper noun.Apart from common and proper noun, we will also study about collective noun and compound

Computer

Science

Computer hardware NAND Gate

A NOT-AND operation is known as NAND operation It has n input (n gt= 2) and one output

Logic diagram

Truth Table

NOR Gate

A NOT-OR operation is known as NOR operation It has n input (n gt= 2) and one output

Logic diagram

Truth Table

XOR Gate

XOR or Ex-OR gate is a special type of gate It can be used in the half

adder full adder and subtractor The exclusive-OR gate is abbreviated as EX-OR gate or sometime as X-OR gate It has n input (n gt= 2) and one output

Logic diagram

Truth Table

XNOR Gate

XNOR gate is a special type of gate It can be used in the half adder full adder and subtractor The exclusive-NOR gate is abbreviated as EX-NOR gate or sometime as X-NOR gate It has n input (n gt= 2) and one output

Logic diagram

Truth Table

Physics

Chapter 1 Electric Field ( Electric Dipole) (Summary)

Here we will derive Expression of electric field at broad side

On position of dipole

Execution

Q With the help of a labelled diagram obtain an expression for the electric field intensity E at any point on the equitorial line ( broad-side on position) of an electric dipole

Ans

E1 E1sinθ

E θ P E1 θ

( r2+L2)12 E2 E

r E2 E2sinθ

-q θ L O L +qA B

Let us consider that the point P is situated on the right bisector of the dipole AB at a distance r meter from its midpoint O

Let E1 and E2 be the electric field intensities of the electric field at P due to charge +q and ndashq of the dipole resp The distance of P from each charge is ( r2+L2)12

So E1 = 14 πϵ q

(r 2+L 2) away from +q

E2 = 14 πϵ q

(r 2+L 2) towards ndashq

The magnitudes of E1 and E2 are equal but directions are different Now resolving E1 and E2 into two components parallel and perpendicular to AB we get

The components perpendicular to AB E1sinθ and E2sinθ cancel each other because they are equal and opposite

The components parallel to AB are E1cosθ and E2 cosθ are in same direction and add up

So resultant intensity of electric field at the point P is

E = E1cosθ + E2 cosθ

E = 14 πϵ q

(r 2+L 2) 2 cosθ

Now from fig we have cosθ =BOBP = L (r2+L2)12

So we get E = 14 πϵ 2qL ( r2+L2)32

Now electric dipole moment p= 2qL

So E = 14 πϵ p ( r2+L2)32

HW Find the expression of Electric field as done here but this time take r gtgt 2L

Also find the expression of torque experience by a dipole

(Hint Electric force experienced by charges of dipole in electric field is qE each Let θ be the angle which dipole makes with electric lines of force then perpendicular distance between two charges is 2Lsinθ Then torque = force x perp distance = qE x 2L sinθ So τ=pE sinθ where p =2qL )

STUDY MATERIAL

Class XIISubject Eng Literature (The Tempest ndash William Shakespeare) Topic Act IV Scene 1 Lines 84 to 133 (Iris hellip A contract of true love Be not too late ) Date 27th April 2020 (4th Period)

[Students should read the original play and also the paraphrase given in the school prescribed textbook]Summary Questions amp Answers

o Ceres soon appears and comes to know that she has been summoned to celebrate the contract of true love

o Ceres expresses her unwillingness to meet Venus and Cupid as she has shunned their company

o Ceres and Juno both bestow their blessings upon Ferdinand and Miranda with June gifting honour riches happiness in marriage and Ceres presents plenty of earthrsquos produce

o Iris summons the water-nymphs and reapers to come and celebrate a contract

(1) IRIS Of her society (Line 91-101)

Be not afraid I met her deity

Cutting the clouds towards Pathos and her sonDove-drawn with her Here thought they to have doneSome wanton charm upon this man and maidWhose vows are that no bed-right shall be paidTill Hymens torch be lightedmdashbut in vainMarss hot minion is returned againHer waspish-headed son has broke his arrowsSwears he will shoot no more but play with sparrowsAnd be a boy right out

(i) Where were Venus and Cupid seen flying How were they travelling Why did they want to join the marriage celebration of Ferdinand and Miranda

of true love

Venus and Cupid were seen flying through the air towards Paphos the famous city which is situated on the island of Cyprus They were travelling by air-borne chariot drawn by doves They certainly wanted to come here in order to play some amorous trick upon Ferdinand and Miranda who are under a vow not to gratify their physical desires till the holy ceremony of their marriage has been performed(ii) What have Venus and Cupid done after failing in their plan

After being failure of their plan Venus who is a very passionate deity and who is the mistress of Mars (the god of war) has gone back while here ill-tempered son Cupid has broken his arrows of love in his state of desperation(iii) What has Cupid firmly decided

Cupid is feeling so disappointed that he has firmly decided to shoot no more arrows to arouse love in human hearts but to spend his time playing with sparrows Thus he would now become just a boy and would give up his original function of shooting arrows on human beings to make them fall in love(iv) What vow had Ceres taken How did Ceres feel at the abduction

After the abduction of her daughter Prosperina by Pluto Ceres had taken a vow to always keep away from the disgraceful company of Venus and her blind son Cupid the god of love Ceres felt deeply distressed when Pluto had carried off her daughter and had made her his wife by force(v) Why has Ceres not forgiven Venus and her blind son For what do Ceres want to be sure

As the abduction had been manipulated by Venus the goddess of beauty and love and her blind son Cupid Ceres has never forgiven them for their part in the whole plot Ceres wants to be sure that she would not have to meet Venus and Cupid who had engineered the abduction of her daughter Prosperina

AS THIS lsquoMASQUErsquo SCENE IS VERY IMPORTANT IN THE PLAY THE PARAPHRASE OF THE ENTIRE PORTION OF MASQUE SCENE (Act IV Lines 58 to 143) IS GIVEN BELOW

IRIS Goddess of RainbowCERES Goddess of Agriculture and all the fruits of the earth

(Nature growth prosperity rebirth ndash notions intimately connected to marriage)JUNO The majestic Queen of Heavens and wife of Jupiter (Jupiter is the king of Gods)

VENUS The Goddess of love CUPID Son of Venus PLUTO God of death (In the play referred by Shakespeare as lsquoDisrsquo which is a Roman name for Pluto)

ORIGINAL TEXT PARAPHRASEPROSPEROWellmdash

PROSPERONow come Ariel Let there be too many rather than too few

Now come my Ariel Bring a corollaryRather than want a spirit Appear and pertly[to Ferdinand and Miranda]No tongue all eyes Be silent

spirits in attendance Appear briskly

[to Ferdinand and Miranda]Look with your eyes but do not say a word

[Soft music] [Soft music][Enter Iris] [Enter Iris]

IRISCeres most bounteous lady thy rich leasOf wheat rye barley vetches oats and peasThy turfy mountains where live nibbling sheepAnd flat meads thatched with stover them to keepThy banks with pioned and twilled brimsWhich spongy April at thy hest betrimsTo make cold nymphs chaste crowns and thybroom-grovesWhose shadow the dismissegraved bachelor lovesBeing lass-lorn thy pole clipped vineyardAnd thy sea-marge sterile and rocky-hardWhere thou thyself dost airmdashthe Queen othrsquoSkyWhose watery arch and messenger am IBids thee leave these and with her sovereign grace[Juno appears] Here on this grass-plot in this very placeTo come and sport Her peacocks fly amainApproach rich Ceres her to entertain

IRISCeres most generous lady you are the cause of rich fields or fertile land where wheat rye barley beans oats and peas grow the grassy mountains where the sheep graze and the flat meadows covered with coarse hay to be used as fodder for cattleYour banks are covered with marsh-marigolds and reeds and the rainy April under your orders brings forth to make for the maids who are not in love beautiful crowns your woods where the broom flourishes and where the bachelor who has been dismissed by the maid he loved lies down being forsaken your vineyard in which the poles are embraced by the vines and the margin of the sea which is barren and rocky where you roam about to enjoy the fresh air ndash the queen of the sky (Juno) whose messenger I am besides being represented as the rainbow bids you leave all these and with her majesty here on this grassy plot in this very place come and sport her peacocks carry her fast in her chariot through the air and are making their way here approach rich Ceres to welcome her

[Enter Ariel as Ceres] [Enter Ariel as Ceres]

CERESHail many-coloured messenger that neerDost disobey the wife of JupiterWho with thy saffron wings upon my flowersDiffusest honey-drops refreshing showersAnd with each end of thy blue bow dost crownMy bosky acres and my unshrubbed downRich scarf to my proud earth Why hath thy queenSummoned me hither to this short-grassed green

CERESWelcome rainbow that never dared disobey Juno the wife of Jupiter who with your orange coloured rays spread honey-drops refreshing showers And with each end of thy blue bow drown my bushy acres and my hilly country which is free from shrubs you thus forming a rich scarf Why has your queen called me here to this place covered with short grass

IRISA contract of true love to celebrateAnd some donation freely to estateOn the blest lovers

IRISI have called you to celebrate a contract of true love and bestow some liberal gift upon the blessed lovers

ORIGINAL TEXT PARAPHRASECERESTell me heavenly bowIf Venus or her son as thou dost knowDo now attend the queen Since they did plotThe means that dusky Dis my daughter gotHer and her blind boys scandaled companyI have forsworn

CERESTell me heavenly bow if Venus the Goddess of love or Cupid her son and pedlar of passion at this time attend the heavenly queen Juno because you are sure to know Since the day they conspired against me and dark Pluto took away my daughter here and Cupidrsquos disgraceful company I have left off

IRISOf her societyBe not afraid I met her deityCutting the clouds towards Pathos and her sonDove-drawn with her Here thought they to have doneSome wanton charm upon this man and miad

IRISBe not afraid of her company I met her deity moving on the clouds towards Paphos the sacred home of Venus on the island of Cyprus along with her son on her chariot drawn by doves Here they contemplated to exercise a charm upon this man and maid producing

Whose vows are that no bed-right shall be paidTill Hymens torch be lightedmdashbut in vainMarss hot minion is returned againHer waspish-headed son has broke his arrowsSwears he will shoot no more but play with sparrowsAnd be a boy right out

wantonness before the actual marriage ceremony but did not succeed Venus has returned her irritable son has broken his arrows and swears that he will give up his practice of trying to inspire love but play with sparrows and be a boy again

[Music is heard] [Music is heard]

CERESHighst queen of stateGreat Juno comes I know her by her gait

CERESHighest queen of state Great Juno there she comes I know here by her gait

[Enter Juno] [Enter Juno]

JUNOHow does my bounteous sister Go with meTo bless this twain that they may Prosperous beAnd honoured in their issue

JUNOHow are you doing my generous sister Come with me to bless this couple so that they may be prosperous and fortunate in their children

[They sing] [They sing]

JUNOHonour riches marriage-blessingLong continuance and increasingHourly joys be still upon youJuno sings her blessings upon you

JUNOMay honour riches happiness in marriage long continuance and increase of those boons ever rest upon you as hourly joys Juno showers down upon you her blessings in song

CERESEarths increase foison plentyBarns and garners never emptyVines and clustring bunches growingPlants and goodly burden bowingSpring come to you at the farthestIn the very end of harvestScarcity and want shall shun youCeresrsquo blessing so is on you

CERESMay you have the plenty of earthrsquos produce Your barns and granaries may never be empty Your vines may grow with clustering bunches Your fruit trees may be heavily laden with their fruit May there be continuous spring and harvest May scantiness and want leave you forever Such is the blessing of Ceres upon you

FERDINANDThis is a most majestic vision andHarmoniously charmingly May I be boldTo think these spirits

FERDINANDThis is a great vision and magically melodious Should I suppose the characters (taking part in the masque) are spirits

PROSPEROSpirits which by mine artI have from their confines calld to enactMy present fancies

PROSPEROYes they are spirits whom I have summoned from the regions to which they are confined to carry into effect my fanciful designs

ORIGINAL TEXT PARAPHRASEFERDINANDLet me live here everSo rare a wondered father and a wifeMakes this place paradise

FERDINANDI should like to live here forever Such a wise and wonderful father makes this place a paradise

[Juno and Ceres whisper and send Iris on employment] [Juno and Ceres whisper and send Iris on employment]

PROSPEROSweet now silence

PROSPEROMy dear Ferdinand speak no more Juno and Ceres are

Juno and Ceres whisper seriouslyTheres something else to do Hush and be muteOr else our spell is marred

whispering with a solemn look There is something else coming Silence Or else our magic will be spoilt

IRISYour nymphs called naiads of the wandering brooksWith your sedged crowns and over-harmless looksLeave your crisp channels and on this green landAnswer your summons Juno does commandCome temperate nymphs and help to celebrateA contract of true love Be not too late

IRISYou nymphs called Naiads denizens (M inhabitants) of the running stream with your chaplets of sedge and ever-helpful looks leave your wrinkled channels and on the green land answer the summons sent to you Juno has ordered some chaste nymphs and help to celebrate a noble and true marriage Donrsquot delay

[Enter certain nymphs] [Enter certain nymphs]You sunburnt sicklemen of August wearyCome hither from the furrow and be merryMake holiday your rye-straw hats put onAnd these fresh nymphs encounter every oneIn country footing

You sunburnt harvesters weary from the effects of the heat in August come here from the furrowed land and rejoice Make holiday with your rye-straw hats upon you and meet these fresh nymphs and join in country dancing

[Enter certain reapers properly habited They join with the nymphs in a graceful dance towards the end whereof Prospero starts suddenly and speaks]

[Enter certain reapers properly habited They join with the nymphs in a graceful dance towards the end whereof Prospero starts suddenly and speaks]

PROSPERO[aside] I had forgot that foul conspiracyOf the beast Caliban and his confederatesAgainst my life The minute of their plotIs almost come [to the spirits]Well done Avoidno more

PROSPERO(Aside)I had forgotten the wicked conspiracy of the beast Caliban and his accomplices against my life the time of their plot has almost arrived ndash (To the Spirits) well done depart no more of this

[To a strange hollow and confused noise the spirits heavily vanish]

[The spirits depart]

ORIGINAL TEXT PARAPHRASEFERDINANDLet me live here everSo rare a wondered father and a wifeMakes this place paradise

FERDINANDI should like to live here forever Such a wise and wonderful father makes this place a paradise

[Juno and Ceres whisper and send Iris on employment] [Juno and Ceres whisper and send Iris on employment]

PROSPEROSweet now silenceJuno and Ceres whisper seriouslyTheres something else to do Hush and be muteOr else our spell is marred

PROSPEROMy dear Ferdinand speak no more Juno and Ceres are whispering with a solemn look There is something else coming Silence Or else our magic will be spoilt

IRISYour nymphs called naiads of the wandering brooksWith your sedged crowns and over-harmless looksLeave your crisp channels and on this green landAnswer your summons Juno does commandCome temperate nymphs and help to celebrateA contract of true love Be not too late

IRISYou nymphs called Naiads denizens (M inhabitants) of the running stream with your chaplets of sedge and ever-helpful looks leave your wrinkled channels and on the green land answer the summons sent to you Juno has ordered some chaste nymphs and help to celebrate a noble and true marriage Donrsquot delay

[Enter certain nymphs] [Enter certain nymphs]You sunburnt sicklemen of August wearyCome hither from the furrow and be merryMake holiday your rye-straw hats put onAnd these fresh nymphs encounter every oneIn country footing

You sunburnt harvesters weary from the effects of the heat in August come here from the furrowed land and rejoice Make holiday with your rye-straw hats upon you and meet these fresh nymphs and join in country dancing

[Enter certain reapers properly habited They join with the nymphs in a graceful dance towards the end whereof Prospero starts suddenly and speaks]

[Enter certain reapers properly habited They join with the nymphs in a graceful dance towards the end whereof Prospero starts suddenly and speaks]

PROSPERO[aside] I had forgot that foul conspiracyOf the beast Caliban and his confederatesAgainst my life The minute of their plotIs almost come [to the spirits]Well done Avoidno more

PROSPERO(Aside)I had forgotten the wicked conspiracy of the beast Caliban and his accomplices against my life the time of their plot has almost arrived ndash (To the Spirits) well done depart no more of this

[To a strange hollow and confused noise the spirits heavily vanish]

[The spirits depart]

Ac-12 27420 topic Revaluation of Assets and Liabilities

REVALUATION OF ASSETS AND LIABILITIES

On admission of a new partner the firm stands reconstituted and consequently the assets are revalued and liabilities are reassessed It is necessary to show the true position of the firm at the time of admission of a new partner If the values of the assets are raised gain will increase the capital of the existing partners Similarly any decrease in the value of assets ie loss will decrease the capital of the existing partners For this purpose alsquoRevaluation Accountrsquo is prepared This account is credited with all increases in the value of assets and decrease in the value of liabilities It is debited with decrease on account of value of assets and increase in the value of liabilities The balance of this account shows a gain or loss on revaluation which is transferred to the existing partnerrsquos capital account in existing profit sharing ratioAccounting for Revaluation of Assets and Liabilities when there is a Changein the Profit Sharing Ratio of Existing PartnersAssets and liabilities of a firm must also be revalued at the time of change in profit sharing ratio of existing partners The reason is that the realisable or actual value of assets and liabilities may be different from those shown in the Balance Sheet It is possible that with the passage of time some of the assets might have appreciated in value while the value of certain other assets might have decreased and no record has been made of such changes in the books of accounts Similarly there may be some unrecorded assets amp libilities that may have to be accounted for Revaluation of assets and reassessments of liabilities becomes necessary because the change in the

value of assets and liabilities belongs to the period to change in profit sharing ratio and hence must be shared by the partners in their old profit sharing ratio Revaluation of assets and reassessment of liabilities may be given effect to in two different ways (a) When revised values are to be recorded in the books and(b) When revised values are not to be recorded in the books

When revised values are to be recorded in the booksIn such a case revaluation of assets and reassessment of liabilities is done with the help of a new account called lsquoRevaluation Accountrsquo Sometimes this account is also called as lsquoProfit amp Loss Adjustment Acrsquo If there is a loss due to revaluation revaluation account is debited and if the revaluation results in a profit the revaluation account is credited The following journal entries made for this purpose are

(i) For increase in the value of assetsAsset Ac Dr (individually)To Revaluation Ac(ii) For decrease in the value of AssetRevaluation Ac Dr (individually)To Asset Ac[Decrease in the value of assets](iii) For increase in the value of LiabilitiesRevaluation Ac Dr (individually)To Liabilities Ac[Increase in the value of Liabilities](iv) For decrease in the value of LiabilitiesLiabilities Ac DrTo Revaluation Ac[Decrease in the value of Liabilities](v) For unrecorded AssetsAsset Ac [unrecorded] DrTo Revaluation Ac[Unrecorded asset recorded at actual value](vi) For unrecorded Liability Revaluation Ac DrTo Liability Ac [unrecorded][Unrecorded Liability recorded at actual value](vii) For transfer of gain on revaluationRevaluation Ac DrTo Existing Partnerrsquos CapitalCurrent Ac[Profit on revaluation transferred to capital account in existing ratio](viii) For transfer of loss on revaluationExisting Partnerrsquos CapitalCurrent Ac DrTo Revaluation Ac[Loss on revaluation transferred to capital account in existing ratio](a) When revaluation account shows gain Revaluation Ac DrTo Partnerrsquos Capital Ac (Old Profit Sharing Ratio)(Profit on revaluation credited to Partnerrsquos Capital Ac)(b) Above entry is reversed when revaluation account shows loss Partners Capital Acs (Old Profit Sharing Ratio) DrTo Revaluation Ac(Loss on revaluation debited to Partnerrsquos Capital Acs)

Proforma of Revaluation Account is given as under

Revaluation Account

Dr Cr Particulars ` Amount Particulars ` Amount To Decrease in value of assets By Increase in value of assets To Increase in value of liabilities By Decrease in value of liabilities To Unrecorded liabilities By Unrecorded assets To Gain on Revaluation (Transferred) By Loss on Revalution (Transferred)

ECO ndash12 2742020Topic- ELASTICITY OF DEMAND

CHAPTER - ELASTICITY OF DEMANDMEANINGDemand for a commodity is affected by many factors such as its price price of related goods income of its buyer tastes and preferences etc Elasticity means degree of response Elasticity of demand means degree of responsiveness of demand Demand for a commodity responds to change in price price of related goods income etc So we have three dimensions of elasticity of demandDIMENSION OF ELASTICITY OF DEMAND TYPES OF ELASTICITY OF DEMAND

Price elasticity of demand Income elasticity of demand Cross Elasticity of demand

Price elasticity of demand Price elasticity of demand means degree of responsiveness of demand for a commodity to the change in its price For example if demand for a commodity rises by 10 due to 5 fall in its price Price elasticity of demand (ep)=Percentage change in quantity demanded Percentage change in price of the commodity = 10 ( -)5 = ( - )2Note that ep will always be negative due to inverse relationship of price and quantity demanded

(ii) Income elasticity of demand Income elasticity of demand refers to the degree of responsiveness of demand for a commodity to the change in income of its buyer Suppose income of buyer rises by 10 and his demand for a commodity rises by 20 then Income elasticity of demand (ey)= change in quantity demanded change in price of the commodity =20 10 = 2

Cross Elasticity of demandCross elasticity of demand means the degree of responsiveness of demand for a commodity to the change in price of its related goods (substitute goods or complementary goods) Suppose demand for a commodity rises by 10 due to 5 rise in price of its substitute good then Cross elasticity of demand (ec) = change in quantity demanded change in price of related good = 10 2 = 5 (Tastes and preferences cannot be expressed numerically So elasticity ofdemand cannot be numerically expressed)

  • Chapter 1 Force (Summary)
  • Distinguish between external sovereignty and internal sovereignty
    • NAND Gate
      • Logic diagram
      • Truth Table
        • NOR Gate
          • Logic diagram
          • Truth Table
            • XOR Gate
              • Logic diagram
              • Truth Table
                • XNOR Gate
                  • Logic diagram
                  • Truth Table
                      • Physics
                      • Chapter 1 Electric Field ( Electric Dipole) (Summary)
Page 49:  · Web viewWe all know that Nouns are divided into two parts: common noun and proper noun.Apart from common and proper noun, we will also study about collective noun and compound

adder full adder and subtractor The exclusive-OR gate is abbreviated as EX-OR gate or sometime as X-OR gate It has n input (n gt= 2) and one output

Logic diagram

Truth Table

XNOR Gate

XNOR gate is a special type of gate It can be used in the half adder full adder and subtractor The exclusive-NOR gate is abbreviated as EX-NOR gate or sometime as X-NOR gate It has n input (n gt= 2) and one output

Logic diagram

Truth Table

Physics

Chapter 1 Electric Field ( Electric Dipole) (Summary)

Here we will derive Expression of electric field at broad side

On position of dipole

Execution

Q With the help of a labelled diagram obtain an expression for the electric field intensity E at any point on the equitorial line ( broad-side on position) of an electric dipole

Ans

E1 E1sinθ

E θ P E1 θ

( r2+L2)12 E2 E

r E2 E2sinθ

-q θ L O L +qA B

Let us consider that the point P is situated on the right bisector of the dipole AB at a distance r meter from its midpoint O

Let E1 and E2 be the electric field intensities of the electric field at P due to charge +q and ndashq of the dipole resp The distance of P from each charge is ( r2+L2)12

So E1 = 14 πϵ q

(r 2+L 2) away from +q

E2 = 14 πϵ q

(r 2+L 2) towards ndashq

The magnitudes of E1 and E2 are equal but directions are different Now resolving E1 and E2 into two components parallel and perpendicular to AB we get

The components perpendicular to AB E1sinθ and E2sinθ cancel each other because they are equal and opposite

The components parallel to AB are E1cosθ and E2 cosθ are in same direction and add up

So resultant intensity of electric field at the point P is

E = E1cosθ + E2 cosθ

E = 14 πϵ q

(r 2+L 2) 2 cosθ

Now from fig we have cosθ =BOBP = L (r2+L2)12

So we get E = 14 πϵ 2qL ( r2+L2)32

Now electric dipole moment p= 2qL

So E = 14 πϵ p ( r2+L2)32

HW Find the expression of Electric field as done here but this time take r gtgt 2L

Also find the expression of torque experience by a dipole

(Hint Electric force experienced by charges of dipole in electric field is qE each Let θ be the angle which dipole makes with electric lines of force then perpendicular distance between two charges is 2Lsinθ Then torque = force x perp distance = qE x 2L sinθ So τ=pE sinθ where p =2qL )

STUDY MATERIAL

Class XIISubject Eng Literature (The Tempest ndash William Shakespeare) Topic Act IV Scene 1 Lines 84 to 133 (Iris hellip A contract of true love Be not too late ) Date 27th April 2020 (4th Period)

[Students should read the original play and also the paraphrase given in the school prescribed textbook]Summary Questions amp Answers

o Ceres soon appears and comes to know that she has been summoned to celebrate the contract of true love

o Ceres expresses her unwillingness to meet Venus and Cupid as she has shunned their company

o Ceres and Juno both bestow their blessings upon Ferdinand and Miranda with June gifting honour riches happiness in marriage and Ceres presents plenty of earthrsquos produce

o Iris summons the water-nymphs and reapers to come and celebrate a contract

(1) IRIS Of her society (Line 91-101)

Be not afraid I met her deity

Cutting the clouds towards Pathos and her sonDove-drawn with her Here thought they to have doneSome wanton charm upon this man and maidWhose vows are that no bed-right shall be paidTill Hymens torch be lightedmdashbut in vainMarss hot minion is returned againHer waspish-headed son has broke his arrowsSwears he will shoot no more but play with sparrowsAnd be a boy right out

(i) Where were Venus and Cupid seen flying How were they travelling Why did they want to join the marriage celebration of Ferdinand and Miranda

of true love

Venus and Cupid were seen flying through the air towards Paphos the famous city which is situated on the island of Cyprus They were travelling by air-borne chariot drawn by doves They certainly wanted to come here in order to play some amorous trick upon Ferdinand and Miranda who are under a vow not to gratify their physical desires till the holy ceremony of their marriage has been performed(ii) What have Venus and Cupid done after failing in their plan

After being failure of their plan Venus who is a very passionate deity and who is the mistress of Mars (the god of war) has gone back while here ill-tempered son Cupid has broken his arrows of love in his state of desperation(iii) What has Cupid firmly decided

Cupid is feeling so disappointed that he has firmly decided to shoot no more arrows to arouse love in human hearts but to spend his time playing with sparrows Thus he would now become just a boy and would give up his original function of shooting arrows on human beings to make them fall in love(iv) What vow had Ceres taken How did Ceres feel at the abduction

After the abduction of her daughter Prosperina by Pluto Ceres had taken a vow to always keep away from the disgraceful company of Venus and her blind son Cupid the god of love Ceres felt deeply distressed when Pluto had carried off her daughter and had made her his wife by force(v) Why has Ceres not forgiven Venus and her blind son For what do Ceres want to be sure

As the abduction had been manipulated by Venus the goddess of beauty and love and her blind son Cupid Ceres has never forgiven them for their part in the whole plot Ceres wants to be sure that she would not have to meet Venus and Cupid who had engineered the abduction of her daughter Prosperina

AS THIS lsquoMASQUErsquo SCENE IS VERY IMPORTANT IN THE PLAY THE PARAPHRASE OF THE ENTIRE PORTION OF MASQUE SCENE (Act IV Lines 58 to 143) IS GIVEN BELOW

IRIS Goddess of RainbowCERES Goddess of Agriculture and all the fruits of the earth

(Nature growth prosperity rebirth ndash notions intimately connected to marriage)JUNO The majestic Queen of Heavens and wife of Jupiter (Jupiter is the king of Gods)

VENUS The Goddess of love CUPID Son of Venus PLUTO God of death (In the play referred by Shakespeare as lsquoDisrsquo which is a Roman name for Pluto)

ORIGINAL TEXT PARAPHRASEPROSPEROWellmdash

PROSPERONow come Ariel Let there be too many rather than too few

Now come my Ariel Bring a corollaryRather than want a spirit Appear and pertly[to Ferdinand and Miranda]No tongue all eyes Be silent

spirits in attendance Appear briskly

[to Ferdinand and Miranda]Look with your eyes but do not say a word

[Soft music] [Soft music][Enter Iris] [Enter Iris]

IRISCeres most bounteous lady thy rich leasOf wheat rye barley vetches oats and peasThy turfy mountains where live nibbling sheepAnd flat meads thatched with stover them to keepThy banks with pioned and twilled brimsWhich spongy April at thy hest betrimsTo make cold nymphs chaste crowns and thybroom-grovesWhose shadow the dismissegraved bachelor lovesBeing lass-lorn thy pole clipped vineyardAnd thy sea-marge sterile and rocky-hardWhere thou thyself dost airmdashthe Queen othrsquoSkyWhose watery arch and messenger am IBids thee leave these and with her sovereign grace[Juno appears] Here on this grass-plot in this very placeTo come and sport Her peacocks fly amainApproach rich Ceres her to entertain

IRISCeres most generous lady you are the cause of rich fields or fertile land where wheat rye barley beans oats and peas grow the grassy mountains where the sheep graze and the flat meadows covered with coarse hay to be used as fodder for cattleYour banks are covered with marsh-marigolds and reeds and the rainy April under your orders brings forth to make for the maids who are not in love beautiful crowns your woods where the broom flourishes and where the bachelor who has been dismissed by the maid he loved lies down being forsaken your vineyard in which the poles are embraced by the vines and the margin of the sea which is barren and rocky where you roam about to enjoy the fresh air ndash the queen of the sky (Juno) whose messenger I am besides being represented as the rainbow bids you leave all these and with her majesty here on this grassy plot in this very place come and sport her peacocks carry her fast in her chariot through the air and are making their way here approach rich Ceres to welcome her

[Enter Ariel as Ceres] [Enter Ariel as Ceres]

CERESHail many-coloured messenger that neerDost disobey the wife of JupiterWho with thy saffron wings upon my flowersDiffusest honey-drops refreshing showersAnd with each end of thy blue bow dost crownMy bosky acres and my unshrubbed downRich scarf to my proud earth Why hath thy queenSummoned me hither to this short-grassed green

CERESWelcome rainbow that never dared disobey Juno the wife of Jupiter who with your orange coloured rays spread honey-drops refreshing showers And with each end of thy blue bow drown my bushy acres and my hilly country which is free from shrubs you thus forming a rich scarf Why has your queen called me here to this place covered with short grass

IRISA contract of true love to celebrateAnd some donation freely to estateOn the blest lovers

IRISI have called you to celebrate a contract of true love and bestow some liberal gift upon the blessed lovers

ORIGINAL TEXT PARAPHRASECERESTell me heavenly bowIf Venus or her son as thou dost knowDo now attend the queen Since they did plotThe means that dusky Dis my daughter gotHer and her blind boys scandaled companyI have forsworn

CERESTell me heavenly bow if Venus the Goddess of love or Cupid her son and pedlar of passion at this time attend the heavenly queen Juno because you are sure to know Since the day they conspired against me and dark Pluto took away my daughter here and Cupidrsquos disgraceful company I have left off

IRISOf her societyBe not afraid I met her deityCutting the clouds towards Pathos and her sonDove-drawn with her Here thought they to have doneSome wanton charm upon this man and miad

IRISBe not afraid of her company I met her deity moving on the clouds towards Paphos the sacred home of Venus on the island of Cyprus along with her son on her chariot drawn by doves Here they contemplated to exercise a charm upon this man and maid producing

Whose vows are that no bed-right shall be paidTill Hymens torch be lightedmdashbut in vainMarss hot minion is returned againHer waspish-headed son has broke his arrowsSwears he will shoot no more but play with sparrowsAnd be a boy right out

wantonness before the actual marriage ceremony but did not succeed Venus has returned her irritable son has broken his arrows and swears that he will give up his practice of trying to inspire love but play with sparrows and be a boy again

[Music is heard] [Music is heard]

CERESHighst queen of stateGreat Juno comes I know her by her gait

CERESHighest queen of state Great Juno there she comes I know here by her gait

[Enter Juno] [Enter Juno]

JUNOHow does my bounteous sister Go with meTo bless this twain that they may Prosperous beAnd honoured in their issue

JUNOHow are you doing my generous sister Come with me to bless this couple so that they may be prosperous and fortunate in their children

[They sing] [They sing]

JUNOHonour riches marriage-blessingLong continuance and increasingHourly joys be still upon youJuno sings her blessings upon you

JUNOMay honour riches happiness in marriage long continuance and increase of those boons ever rest upon you as hourly joys Juno showers down upon you her blessings in song

CERESEarths increase foison plentyBarns and garners never emptyVines and clustring bunches growingPlants and goodly burden bowingSpring come to you at the farthestIn the very end of harvestScarcity and want shall shun youCeresrsquo blessing so is on you

CERESMay you have the plenty of earthrsquos produce Your barns and granaries may never be empty Your vines may grow with clustering bunches Your fruit trees may be heavily laden with their fruit May there be continuous spring and harvest May scantiness and want leave you forever Such is the blessing of Ceres upon you

FERDINANDThis is a most majestic vision andHarmoniously charmingly May I be boldTo think these spirits

FERDINANDThis is a great vision and magically melodious Should I suppose the characters (taking part in the masque) are spirits

PROSPEROSpirits which by mine artI have from their confines calld to enactMy present fancies

PROSPEROYes they are spirits whom I have summoned from the regions to which they are confined to carry into effect my fanciful designs

ORIGINAL TEXT PARAPHRASEFERDINANDLet me live here everSo rare a wondered father and a wifeMakes this place paradise

FERDINANDI should like to live here forever Such a wise and wonderful father makes this place a paradise

[Juno and Ceres whisper and send Iris on employment] [Juno and Ceres whisper and send Iris on employment]

PROSPEROSweet now silence

PROSPEROMy dear Ferdinand speak no more Juno and Ceres are

Juno and Ceres whisper seriouslyTheres something else to do Hush and be muteOr else our spell is marred

whispering with a solemn look There is something else coming Silence Or else our magic will be spoilt

IRISYour nymphs called naiads of the wandering brooksWith your sedged crowns and over-harmless looksLeave your crisp channels and on this green landAnswer your summons Juno does commandCome temperate nymphs and help to celebrateA contract of true love Be not too late

IRISYou nymphs called Naiads denizens (M inhabitants) of the running stream with your chaplets of sedge and ever-helpful looks leave your wrinkled channels and on the green land answer the summons sent to you Juno has ordered some chaste nymphs and help to celebrate a noble and true marriage Donrsquot delay

[Enter certain nymphs] [Enter certain nymphs]You sunburnt sicklemen of August wearyCome hither from the furrow and be merryMake holiday your rye-straw hats put onAnd these fresh nymphs encounter every oneIn country footing

You sunburnt harvesters weary from the effects of the heat in August come here from the furrowed land and rejoice Make holiday with your rye-straw hats upon you and meet these fresh nymphs and join in country dancing

[Enter certain reapers properly habited They join with the nymphs in a graceful dance towards the end whereof Prospero starts suddenly and speaks]

[Enter certain reapers properly habited They join with the nymphs in a graceful dance towards the end whereof Prospero starts suddenly and speaks]

PROSPERO[aside] I had forgot that foul conspiracyOf the beast Caliban and his confederatesAgainst my life The minute of their plotIs almost come [to the spirits]Well done Avoidno more

PROSPERO(Aside)I had forgotten the wicked conspiracy of the beast Caliban and his accomplices against my life the time of their plot has almost arrived ndash (To the Spirits) well done depart no more of this

[To a strange hollow and confused noise the spirits heavily vanish]

[The spirits depart]

ORIGINAL TEXT PARAPHRASEFERDINANDLet me live here everSo rare a wondered father and a wifeMakes this place paradise

FERDINANDI should like to live here forever Such a wise and wonderful father makes this place a paradise

[Juno and Ceres whisper and send Iris on employment] [Juno and Ceres whisper and send Iris on employment]

PROSPEROSweet now silenceJuno and Ceres whisper seriouslyTheres something else to do Hush and be muteOr else our spell is marred

PROSPEROMy dear Ferdinand speak no more Juno and Ceres are whispering with a solemn look There is something else coming Silence Or else our magic will be spoilt

IRISYour nymphs called naiads of the wandering brooksWith your sedged crowns and over-harmless looksLeave your crisp channels and on this green landAnswer your summons Juno does commandCome temperate nymphs and help to celebrateA contract of true love Be not too late

IRISYou nymphs called Naiads denizens (M inhabitants) of the running stream with your chaplets of sedge and ever-helpful looks leave your wrinkled channels and on the green land answer the summons sent to you Juno has ordered some chaste nymphs and help to celebrate a noble and true marriage Donrsquot delay

[Enter certain nymphs] [Enter certain nymphs]You sunburnt sicklemen of August wearyCome hither from the furrow and be merryMake holiday your rye-straw hats put onAnd these fresh nymphs encounter every oneIn country footing

You sunburnt harvesters weary from the effects of the heat in August come here from the furrowed land and rejoice Make holiday with your rye-straw hats upon you and meet these fresh nymphs and join in country dancing

[Enter certain reapers properly habited They join with the nymphs in a graceful dance towards the end whereof Prospero starts suddenly and speaks]

[Enter certain reapers properly habited They join with the nymphs in a graceful dance towards the end whereof Prospero starts suddenly and speaks]

PROSPERO[aside] I had forgot that foul conspiracyOf the beast Caliban and his confederatesAgainst my life The minute of their plotIs almost come [to the spirits]Well done Avoidno more

PROSPERO(Aside)I had forgotten the wicked conspiracy of the beast Caliban and his accomplices against my life the time of their plot has almost arrived ndash (To the Spirits) well done depart no more of this

[To a strange hollow and confused noise the spirits heavily vanish]

[The spirits depart]

Ac-12 27420 topic Revaluation of Assets and Liabilities

REVALUATION OF ASSETS AND LIABILITIES

On admission of a new partner the firm stands reconstituted and consequently the assets are revalued and liabilities are reassessed It is necessary to show the true position of the firm at the time of admission of a new partner If the values of the assets are raised gain will increase the capital of the existing partners Similarly any decrease in the value of assets ie loss will decrease the capital of the existing partners For this purpose alsquoRevaluation Accountrsquo is prepared This account is credited with all increases in the value of assets and decrease in the value of liabilities It is debited with decrease on account of value of assets and increase in the value of liabilities The balance of this account shows a gain or loss on revaluation which is transferred to the existing partnerrsquos capital account in existing profit sharing ratioAccounting for Revaluation of Assets and Liabilities when there is a Changein the Profit Sharing Ratio of Existing PartnersAssets and liabilities of a firm must also be revalued at the time of change in profit sharing ratio of existing partners The reason is that the realisable or actual value of assets and liabilities may be different from those shown in the Balance Sheet It is possible that with the passage of time some of the assets might have appreciated in value while the value of certain other assets might have decreased and no record has been made of such changes in the books of accounts Similarly there may be some unrecorded assets amp libilities that may have to be accounted for Revaluation of assets and reassessments of liabilities becomes necessary because the change in the

value of assets and liabilities belongs to the period to change in profit sharing ratio and hence must be shared by the partners in their old profit sharing ratio Revaluation of assets and reassessment of liabilities may be given effect to in two different ways (a) When revised values are to be recorded in the books and(b) When revised values are not to be recorded in the books

When revised values are to be recorded in the booksIn such a case revaluation of assets and reassessment of liabilities is done with the help of a new account called lsquoRevaluation Accountrsquo Sometimes this account is also called as lsquoProfit amp Loss Adjustment Acrsquo If there is a loss due to revaluation revaluation account is debited and if the revaluation results in a profit the revaluation account is credited The following journal entries made for this purpose are

(i) For increase in the value of assetsAsset Ac Dr (individually)To Revaluation Ac(ii) For decrease in the value of AssetRevaluation Ac Dr (individually)To Asset Ac[Decrease in the value of assets](iii) For increase in the value of LiabilitiesRevaluation Ac Dr (individually)To Liabilities Ac[Increase in the value of Liabilities](iv) For decrease in the value of LiabilitiesLiabilities Ac DrTo Revaluation Ac[Decrease in the value of Liabilities](v) For unrecorded AssetsAsset Ac [unrecorded] DrTo Revaluation Ac[Unrecorded asset recorded at actual value](vi) For unrecorded Liability Revaluation Ac DrTo Liability Ac [unrecorded][Unrecorded Liability recorded at actual value](vii) For transfer of gain on revaluationRevaluation Ac DrTo Existing Partnerrsquos CapitalCurrent Ac[Profit on revaluation transferred to capital account in existing ratio](viii) For transfer of loss on revaluationExisting Partnerrsquos CapitalCurrent Ac DrTo Revaluation Ac[Loss on revaluation transferred to capital account in existing ratio](a) When revaluation account shows gain Revaluation Ac DrTo Partnerrsquos Capital Ac (Old Profit Sharing Ratio)(Profit on revaluation credited to Partnerrsquos Capital Ac)(b) Above entry is reversed when revaluation account shows loss Partners Capital Acs (Old Profit Sharing Ratio) DrTo Revaluation Ac(Loss on revaluation debited to Partnerrsquos Capital Acs)

Proforma of Revaluation Account is given as under

Revaluation Account

Dr Cr Particulars ` Amount Particulars ` Amount To Decrease in value of assets By Increase in value of assets To Increase in value of liabilities By Decrease in value of liabilities To Unrecorded liabilities By Unrecorded assets To Gain on Revaluation (Transferred) By Loss on Revalution (Transferred)

ECO ndash12 2742020Topic- ELASTICITY OF DEMAND

CHAPTER - ELASTICITY OF DEMANDMEANINGDemand for a commodity is affected by many factors such as its price price of related goods income of its buyer tastes and preferences etc Elasticity means degree of response Elasticity of demand means degree of responsiveness of demand Demand for a commodity responds to change in price price of related goods income etc So we have three dimensions of elasticity of demandDIMENSION OF ELASTICITY OF DEMAND TYPES OF ELASTICITY OF DEMAND

Price elasticity of demand Income elasticity of demand Cross Elasticity of demand

Price elasticity of demand Price elasticity of demand means degree of responsiveness of demand for a commodity to the change in its price For example if demand for a commodity rises by 10 due to 5 fall in its price Price elasticity of demand (ep)=Percentage change in quantity demanded Percentage change in price of the commodity = 10 ( -)5 = ( - )2Note that ep will always be negative due to inverse relationship of price and quantity demanded

(ii) Income elasticity of demand Income elasticity of demand refers to the degree of responsiveness of demand for a commodity to the change in income of its buyer Suppose income of buyer rises by 10 and his demand for a commodity rises by 20 then Income elasticity of demand (ey)= change in quantity demanded change in price of the commodity =20 10 = 2

Cross Elasticity of demandCross elasticity of demand means the degree of responsiveness of demand for a commodity to the change in price of its related goods (substitute goods or complementary goods) Suppose demand for a commodity rises by 10 due to 5 rise in price of its substitute good then Cross elasticity of demand (ec) = change in quantity demanded change in price of related good = 10 2 = 5 (Tastes and preferences cannot be expressed numerically So elasticity ofdemand cannot be numerically expressed)

  • Chapter 1 Force (Summary)
  • Distinguish between external sovereignty and internal sovereignty
    • NAND Gate
      • Logic diagram
      • Truth Table
        • NOR Gate
          • Logic diagram
          • Truth Table
            • XOR Gate
              • Logic diagram
              • Truth Table
                • XNOR Gate
                  • Logic diagram
                  • Truth Table
                      • Physics
                      • Chapter 1 Electric Field ( Electric Dipole) (Summary)
Page 50:  · Web viewWe all know that Nouns are divided into two parts: common noun and proper noun.Apart from common and proper noun, we will also study about collective noun and compound

Physics

Chapter 1 Electric Field ( Electric Dipole) (Summary)

Here we will derive Expression of electric field at broad side

On position of dipole

Execution

Q With the help of a labelled diagram obtain an expression for the electric field intensity E at any point on the equitorial line ( broad-side on position) of an electric dipole

Ans

E1 E1sinθ

E θ P E1 θ

( r2+L2)12 E2 E

r E2 E2sinθ

-q θ L O L +qA B

Let us consider that the point P is situated on the right bisector of the dipole AB at a distance r meter from its midpoint O

Let E1 and E2 be the electric field intensities of the electric field at P due to charge +q and ndashq of the dipole resp The distance of P from each charge is ( r2+L2)12

So E1 = 14 πϵ q

(r 2+L 2) away from +q

E2 = 14 πϵ q

(r 2+L 2) towards ndashq

The magnitudes of E1 and E2 are equal but directions are different Now resolving E1 and E2 into two components parallel and perpendicular to AB we get

The components perpendicular to AB E1sinθ and E2sinθ cancel each other because they are equal and opposite

The components parallel to AB are E1cosθ and E2 cosθ are in same direction and add up

So resultant intensity of electric field at the point P is

E = E1cosθ + E2 cosθ

E = 14 πϵ q

(r 2+L 2) 2 cosθ

Now from fig we have cosθ =BOBP = L (r2+L2)12

So we get E = 14 πϵ 2qL ( r2+L2)32

Now electric dipole moment p= 2qL

So E = 14 πϵ p ( r2+L2)32

HW Find the expression of Electric field as done here but this time take r gtgt 2L

Also find the expression of torque experience by a dipole

(Hint Electric force experienced by charges of dipole in electric field is qE each Let θ be the angle which dipole makes with electric lines of force then perpendicular distance between two charges is 2Lsinθ Then torque = force x perp distance = qE x 2L sinθ So τ=pE sinθ where p =2qL )

STUDY MATERIAL

Class XIISubject Eng Literature (The Tempest ndash William Shakespeare) Topic Act IV Scene 1 Lines 84 to 133 (Iris hellip A contract of true love Be not too late ) Date 27th April 2020 (4th Period)

[Students should read the original play and also the paraphrase given in the school prescribed textbook]Summary Questions amp Answers

o Ceres soon appears and comes to know that she has been summoned to celebrate the contract of true love

o Ceres expresses her unwillingness to meet Venus and Cupid as she has shunned their company

o Ceres and Juno both bestow their blessings upon Ferdinand and Miranda with June gifting honour riches happiness in marriage and Ceres presents plenty of earthrsquos produce

o Iris summons the water-nymphs and reapers to come and celebrate a contract

(1) IRIS Of her society (Line 91-101)

Be not afraid I met her deity

Cutting the clouds towards Pathos and her sonDove-drawn with her Here thought they to have doneSome wanton charm upon this man and maidWhose vows are that no bed-right shall be paidTill Hymens torch be lightedmdashbut in vainMarss hot minion is returned againHer waspish-headed son has broke his arrowsSwears he will shoot no more but play with sparrowsAnd be a boy right out

(i) Where were Venus and Cupid seen flying How were they travelling Why did they want to join the marriage celebration of Ferdinand and Miranda

of true love

Venus and Cupid were seen flying through the air towards Paphos the famous city which is situated on the island of Cyprus They were travelling by air-borne chariot drawn by doves They certainly wanted to come here in order to play some amorous trick upon Ferdinand and Miranda who are under a vow not to gratify their physical desires till the holy ceremony of their marriage has been performed(ii) What have Venus and Cupid done after failing in their plan

After being failure of their plan Venus who is a very passionate deity and who is the mistress of Mars (the god of war) has gone back while here ill-tempered son Cupid has broken his arrows of love in his state of desperation(iii) What has Cupid firmly decided

Cupid is feeling so disappointed that he has firmly decided to shoot no more arrows to arouse love in human hearts but to spend his time playing with sparrows Thus he would now become just a boy and would give up his original function of shooting arrows on human beings to make them fall in love(iv) What vow had Ceres taken How did Ceres feel at the abduction

After the abduction of her daughter Prosperina by Pluto Ceres had taken a vow to always keep away from the disgraceful company of Venus and her blind son Cupid the god of love Ceres felt deeply distressed when Pluto had carried off her daughter and had made her his wife by force(v) Why has Ceres not forgiven Venus and her blind son For what do Ceres want to be sure

As the abduction had been manipulated by Venus the goddess of beauty and love and her blind son Cupid Ceres has never forgiven them for their part in the whole plot Ceres wants to be sure that she would not have to meet Venus and Cupid who had engineered the abduction of her daughter Prosperina

AS THIS lsquoMASQUErsquo SCENE IS VERY IMPORTANT IN THE PLAY THE PARAPHRASE OF THE ENTIRE PORTION OF MASQUE SCENE (Act IV Lines 58 to 143) IS GIVEN BELOW

IRIS Goddess of RainbowCERES Goddess of Agriculture and all the fruits of the earth

(Nature growth prosperity rebirth ndash notions intimately connected to marriage)JUNO The majestic Queen of Heavens and wife of Jupiter (Jupiter is the king of Gods)

VENUS The Goddess of love CUPID Son of Venus PLUTO God of death (In the play referred by Shakespeare as lsquoDisrsquo which is a Roman name for Pluto)

ORIGINAL TEXT PARAPHRASEPROSPEROWellmdash

PROSPERONow come Ariel Let there be too many rather than too few

Now come my Ariel Bring a corollaryRather than want a spirit Appear and pertly[to Ferdinand and Miranda]No tongue all eyes Be silent

spirits in attendance Appear briskly

[to Ferdinand and Miranda]Look with your eyes but do not say a word

[Soft music] [Soft music][Enter Iris] [Enter Iris]

IRISCeres most bounteous lady thy rich leasOf wheat rye barley vetches oats and peasThy turfy mountains where live nibbling sheepAnd flat meads thatched with stover them to keepThy banks with pioned and twilled brimsWhich spongy April at thy hest betrimsTo make cold nymphs chaste crowns and thybroom-grovesWhose shadow the dismissegraved bachelor lovesBeing lass-lorn thy pole clipped vineyardAnd thy sea-marge sterile and rocky-hardWhere thou thyself dost airmdashthe Queen othrsquoSkyWhose watery arch and messenger am IBids thee leave these and with her sovereign grace[Juno appears] Here on this grass-plot in this very placeTo come and sport Her peacocks fly amainApproach rich Ceres her to entertain

IRISCeres most generous lady you are the cause of rich fields or fertile land where wheat rye barley beans oats and peas grow the grassy mountains where the sheep graze and the flat meadows covered with coarse hay to be used as fodder for cattleYour banks are covered with marsh-marigolds and reeds and the rainy April under your orders brings forth to make for the maids who are not in love beautiful crowns your woods where the broom flourishes and where the bachelor who has been dismissed by the maid he loved lies down being forsaken your vineyard in which the poles are embraced by the vines and the margin of the sea which is barren and rocky where you roam about to enjoy the fresh air ndash the queen of the sky (Juno) whose messenger I am besides being represented as the rainbow bids you leave all these and with her majesty here on this grassy plot in this very place come and sport her peacocks carry her fast in her chariot through the air and are making their way here approach rich Ceres to welcome her

[Enter Ariel as Ceres] [Enter Ariel as Ceres]

CERESHail many-coloured messenger that neerDost disobey the wife of JupiterWho with thy saffron wings upon my flowersDiffusest honey-drops refreshing showersAnd with each end of thy blue bow dost crownMy bosky acres and my unshrubbed downRich scarf to my proud earth Why hath thy queenSummoned me hither to this short-grassed green

CERESWelcome rainbow that never dared disobey Juno the wife of Jupiter who with your orange coloured rays spread honey-drops refreshing showers And with each end of thy blue bow drown my bushy acres and my hilly country which is free from shrubs you thus forming a rich scarf Why has your queen called me here to this place covered with short grass

IRISA contract of true love to celebrateAnd some donation freely to estateOn the blest lovers

IRISI have called you to celebrate a contract of true love and bestow some liberal gift upon the blessed lovers

ORIGINAL TEXT PARAPHRASECERESTell me heavenly bowIf Venus or her son as thou dost knowDo now attend the queen Since they did plotThe means that dusky Dis my daughter gotHer and her blind boys scandaled companyI have forsworn

CERESTell me heavenly bow if Venus the Goddess of love or Cupid her son and pedlar of passion at this time attend the heavenly queen Juno because you are sure to know Since the day they conspired against me and dark Pluto took away my daughter here and Cupidrsquos disgraceful company I have left off

IRISOf her societyBe not afraid I met her deityCutting the clouds towards Pathos and her sonDove-drawn with her Here thought they to have doneSome wanton charm upon this man and miad

IRISBe not afraid of her company I met her deity moving on the clouds towards Paphos the sacred home of Venus on the island of Cyprus along with her son on her chariot drawn by doves Here they contemplated to exercise a charm upon this man and maid producing

Whose vows are that no bed-right shall be paidTill Hymens torch be lightedmdashbut in vainMarss hot minion is returned againHer waspish-headed son has broke his arrowsSwears he will shoot no more but play with sparrowsAnd be a boy right out

wantonness before the actual marriage ceremony but did not succeed Venus has returned her irritable son has broken his arrows and swears that he will give up his practice of trying to inspire love but play with sparrows and be a boy again

[Music is heard] [Music is heard]

CERESHighst queen of stateGreat Juno comes I know her by her gait

CERESHighest queen of state Great Juno there she comes I know here by her gait

[Enter Juno] [Enter Juno]

JUNOHow does my bounteous sister Go with meTo bless this twain that they may Prosperous beAnd honoured in their issue

JUNOHow are you doing my generous sister Come with me to bless this couple so that they may be prosperous and fortunate in their children

[They sing] [They sing]

JUNOHonour riches marriage-blessingLong continuance and increasingHourly joys be still upon youJuno sings her blessings upon you

JUNOMay honour riches happiness in marriage long continuance and increase of those boons ever rest upon you as hourly joys Juno showers down upon you her blessings in song

CERESEarths increase foison plentyBarns and garners never emptyVines and clustring bunches growingPlants and goodly burden bowingSpring come to you at the farthestIn the very end of harvestScarcity and want shall shun youCeresrsquo blessing so is on you

CERESMay you have the plenty of earthrsquos produce Your barns and granaries may never be empty Your vines may grow with clustering bunches Your fruit trees may be heavily laden with their fruit May there be continuous spring and harvest May scantiness and want leave you forever Such is the blessing of Ceres upon you

FERDINANDThis is a most majestic vision andHarmoniously charmingly May I be boldTo think these spirits

FERDINANDThis is a great vision and magically melodious Should I suppose the characters (taking part in the masque) are spirits

PROSPEROSpirits which by mine artI have from their confines calld to enactMy present fancies

PROSPEROYes they are spirits whom I have summoned from the regions to which they are confined to carry into effect my fanciful designs

ORIGINAL TEXT PARAPHRASEFERDINANDLet me live here everSo rare a wondered father and a wifeMakes this place paradise

FERDINANDI should like to live here forever Such a wise and wonderful father makes this place a paradise

[Juno and Ceres whisper and send Iris on employment] [Juno and Ceres whisper and send Iris on employment]

PROSPEROSweet now silence

PROSPEROMy dear Ferdinand speak no more Juno and Ceres are

Juno and Ceres whisper seriouslyTheres something else to do Hush and be muteOr else our spell is marred

whispering with a solemn look There is something else coming Silence Or else our magic will be spoilt

IRISYour nymphs called naiads of the wandering brooksWith your sedged crowns and over-harmless looksLeave your crisp channels and on this green landAnswer your summons Juno does commandCome temperate nymphs and help to celebrateA contract of true love Be not too late

IRISYou nymphs called Naiads denizens (M inhabitants) of the running stream with your chaplets of sedge and ever-helpful looks leave your wrinkled channels and on the green land answer the summons sent to you Juno has ordered some chaste nymphs and help to celebrate a noble and true marriage Donrsquot delay

[Enter certain nymphs] [Enter certain nymphs]You sunburnt sicklemen of August wearyCome hither from the furrow and be merryMake holiday your rye-straw hats put onAnd these fresh nymphs encounter every oneIn country footing

You sunburnt harvesters weary from the effects of the heat in August come here from the furrowed land and rejoice Make holiday with your rye-straw hats upon you and meet these fresh nymphs and join in country dancing

[Enter certain reapers properly habited They join with the nymphs in a graceful dance towards the end whereof Prospero starts suddenly and speaks]

[Enter certain reapers properly habited They join with the nymphs in a graceful dance towards the end whereof Prospero starts suddenly and speaks]

PROSPERO[aside] I had forgot that foul conspiracyOf the beast Caliban and his confederatesAgainst my life The minute of their plotIs almost come [to the spirits]Well done Avoidno more

PROSPERO(Aside)I had forgotten the wicked conspiracy of the beast Caliban and his accomplices against my life the time of their plot has almost arrived ndash (To the Spirits) well done depart no more of this

[To a strange hollow and confused noise the spirits heavily vanish]

[The spirits depart]

ORIGINAL TEXT PARAPHRASEFERDINANDLet me live here everSo rare a wondered father and a wifeMakes this place paradise

FERDINANDI should like to live here forever Such a wise and wonderful father makes this place a paradise

[Juno and Ceres whisper and send Iris on employment] [Juno and Ceres whisper and send Iris on employment]

PROSPEROSweet now silenceJuno and Ceres whisper seriouslyTheres something else to do Hush and be muteOr else our spell is marred

PROSPEROMy dear Ferdinand speak no more Juno and Ceres are whispering with a solemn look There is something else coming Silence Or else our magic will be spoilt

IRISYour nymphs called naiads of the wandering brooksWith your sedged crowns and over-harmless looksLeave your crisp channels and on this green landAnswer your summons Juno does commandCome temperate nymphs and help to celebrateA contract of true love Be not too late

IRISYou nymphs called Naiads denizens (M inhabitants) of the running stream with your chaplets of sedge and ever-helpful looks leave your wrinkled channels and on the green land answer the summons sent to you Juno has ordered some chaste nymphs and help to celebrate a noble and true marriage Donrsquot delay

[Enter certain nymphs] [Enter certain nymphs]You sunburnt sicklemen of August wearyCome hither from the furrow and be merryMake holiday your rye-straw hats put onAnd these fresh nymphs encounter every oneIn country footing

You sunburnt harvesters weary from the effects of the heat in August come here from the furrowed land and rejoice Make holiday with your rye-straw hats upon you and meet these fresh nymphs and join in country dancing

[Enter certain reapers properly habited They join with the nymphs in a graceful dance towards the end whereof Prospero starts suddenly and speaks]

[Enter certain reapers properly habited They join with the nymphs in a graceful dance towards the end whereof Prospero starts suddenly and speaks]

PROSPERO[aside] I had forgot that foul conspiracyOf the beast Caliban and his confederatesAgainst my life The minute of their plotIs almost come [to the spirits]Well done Avoidno more

PROSPERO(Aside)I had forgotten the wicked conspiracy of the beast Caliban and his accomplices against my life the time of their plot has almost arrived ndash (To the Spirits) well done depart no more of this

[To a strange hollow and confused noise the spirits heavily vanish]

[The spirits depart]

Ac-12 27420 topic Revaluation of Assets and Liabilities

REVALUATION OF ASSETS AND LIABILITIES

On admission of a new partner the firm stands reconstituted and consequently the assets are revalued and liabilities are reassessed It is necessary to show the true position of the firm at the time of admission of a new partner If the values of the assets are raised gain will increase the capital of the existing partners Similarly any decrease in the value of assets ie loss will decrease the capital of the existing partners For this purpose alsquoRevaluation Accountrsquo is prepared This account is credited with all increases in the value of assets and decrease in the value of liabilities It is debited with decrease on account of value of assets and increase in the value of liabilities The balance of this account shows a gain or loss on revaluation which is transferred to the existing partnerrsquos capital account in existing profit sharing ratioAccounting for Revaluation of Assets and Liabilities when there is a Changein the Profit Sharing Ratio of Existing PartnersAssets and liabilities of a firm must also be revalued at the time of change in profit sharing ratio of existing partners The reason is that the realisable or actual value of assets and liabilities may be different from those shown in the Balance Sheet It is possible that with the passage of time some of the assets might have appreciated in value while the value of certain other assets might have decreased and no record has been made of such changes in the books of accounts Similarly there may be some unrecorded assets amp libilities that may have to be accounted for Revaluation of assets and reassessments of liabilities becomes necessary because the change in the

value of assets and liabilities belongs to the period to change in profit sharing ratio and hence must be shared by the partners in their old profit sharing ratio Revaluation of assets and reassessment of liabilities may be given effect to in two different ways (a) When revised values are to be recorded in the books and(b) When revised values are not to be recorded in the books

When revised values are to be recorded in the booksIn such a case revaluation of assets and reassessment of liabilities is done with the help of a new account called lsquoRevaluation Accountrsquo Sometimes this account is also called as lsquoProfit amp Loss Adjustment Acrsquo If there is a loss due to revaluation revaluation account is debited and if the revaluation results in a profit the revaluation account is credited The following journal entries made for this purpose are

(i) For increase in the value of assetsAsset Ac Dr (individually)To Revaluation Ac(ii) For decrease in the value of AssetRevaluation Ac Dr (individually)To Asset Ac[Decrease in the value of assets](iii) For increase in the value of LiabilitiesRevaluation Ac Dr (individually)To Liabilities Ac[Increase in the value of Liabilities](iv) For decrease in the value of LiabilitiesLiabilities Ac DrTo Revaluation Ac[Decrease in the value of Liabilities](v) For unrecorded AssetsAsset Ac [unrecorded] DrTo Revaluation Ac[Unrecorded asset recorded at actual value](vi) For unrecorded Liability Revaluation Ac DrTo Liability Ac [unrecorded][Unrecorded Liability recorded at actual value](vii) For transfer of gain on revaluationRevaluation Ac DrTo Existing Partnerrsquos CapitalCurrent Ac[Profit on revaluation transferred to capital account in existing ratio](viii) For transfer of loss on revaluationExisting Partnerrsquos CapitalCurrent Ac DrTo Revaluation Ac[Loss on revaluation transferred to capital account in existing ratio](a) When revaluation account shows gain Revaluation Ac DrTo Partnerrsquos Capital Ac (Old Profit Sharing Ratio)(Profit on revaluation credited to Partnerrsquos Capital Ac)(b) Above entry is reversed when revaluation account shows loss Partners Capital Acs (Old Profit Sharing Ratio) DrTo Revaluation Ac(Loss on revaluation debited to Partnerrsquos Capital Acs)

Proforma of Revaluation Account is given as under

Revaluation Account

Dr Cr Particulars ` Amount Particulars ` Amount To Decrease in value of assets By Increase in value of assets To Increase in value of liabilities By Decrease in value of liabilities To Unrecorded liabilities By Unrecorded assets To Gain on Revaluation (Transferred) By Loss on Revalution (Transferred)

ECO ndash12 2742020Topic- ELASTICITY OF DEMAND

CHAPTER - ELASTICITY OF DEMANDMEANINGDemand for a commodity is affected by many factors such as its price price of related goods income of its buyer tastes and preferences etc Elasticity means degree of response Elasticity of demand means degree of responsiveness of demand Demand for a commodity responds to change in price price of related goods income etc So we have three dimensions of elasticity of demandDIMENSION OF ELASTICITY OF DEMAND TYPES OF ELASTICITY OF DEMAND

Price elasticity of demand Income elasticity of demand Cross Elasticity of demand

Price elasticity of demand Price elasticity of demand means degree of responsiveness of demand for a commodity to the change in its price For example if demand for a commodity rises by 10 due to 5 fall in its price Price elasticity of demand (ep)=Percentage change in quantity demanded Percentage change in price of the commodity = 10 ( -)5 = ( - )2Note that ep will always be negative due to inverse relationship of price and quantity demanded

(ii) Income elasticity of demand Income elasticity of demand refers to the degree of responsiveness of demand for a commodity to the change in income of its buyer Suppose income of buyer rises by 10 and his demand for a commodity rises by 20 then Income elasticity of demand (ey)= change in quantity demanded change in price of the commodity =20 10 = 2

Cross Elasticity of demandCross elasticity of demand means the degree of responsiveness of demand for a commodity to the change in price of its related goods (substitute goods or complementary goods) Suppose demand for a commodity rises by 10 due to 5 rise in price of its substitute good then Cross elasticity of demand (ec) = change in quantity demanded change in price of related good = 10 2 = 5 (Tastes and preferences cannot be expressed numerically So elasticity ofdemand cannot be numerically expressed)

  • Chapter 1 Force (Summary)
  • Distinguish between external sovereignty and internal sovereignty
    • NAND Gate
      • Logic diagram
      • Truth Table
        • NOR Gate
          • Logic diagram
          • Truth Table
            • XOR Gate
              • Logic diagram
              • Truth Table
                • XNOR Gate
                  • Logic diagram
                  • Truth Table
                      • Physics
                      • Chapter 1 Electric Field ( Electric Dipole) (Summary)
Page 51:  · Web viewWe all know that Nouns are divided into two parts: common noun and proper noun.Apart from common and proper noun, we will also study about collective noun and compound

Let E1 and E2 be the electric field intensities of the electric field at P due to charge +q and ndashq of the dipole resp The distance of P from each charge is ( r2+L2)12

So E1 = 14 πϵ q

(r 2+L 2) away from +q

E2 = 14 πϵ q

(r 2+L 2) towards ndashq

The magnitudes of E1 and E2 are equal but directions are different Now resolving E1 and E2 into two components parallel and perpendicular to AB we get

The components perpendicular to AB E1sinθ and E2sinθ cancel each other because they are equal and opposite

The components parallel to AB are E1cosθ and E2 cosθ are in same direction and add up

So resultant intensity of electric field at the point P is

E = E1cosθ + E2 cosθ

E = 14 πϵ q

(r 2+L 2) 2 cosθ

Now from fig we have cosθ =BOBP = L (r2+L2)12

So we get E = 14 πϵ 2qL ( r2+L2)32

Now electric dipole moment p= 2qL

So E = 14 πϵ p ( r2+L2)32

HW Find the expression of Electric field as done here but this time take r gtgt 2L

Also find the expression of torque experience by a dipole

(Hint Electric force experienced by charges of dipole in electric field is qE each Let θ be the angle which dipole makes with electric lines of force then perpendicular distance between two charges is 2Lsinθ Then torque = force x perp distance = qE x 2L sinθ So τ=pE sinθ where p =2qL )

STUDY MATERIAL

Class XIISubject Eng Literature (The Tempest ndash William Shakespeare) Topic Act IV Scene 1 Lines 84 to 133 (Iris hellip A contract of true love Be not too late ) Date 27th April 2020 (4th Period)

[Students should read the original play and also the paraphrase given in the school prescribed textbook]Summary Questions amp Answers

o Ceres soon appears and comes to know that she has been summoned to celebrate the contract of true love

o Ceres expresses her unwillingness to meet Venus and Cupid as she has shunned their company

o Ceres and Juno both bestow their blessings upon Ferdinand and Miranda with June gifting honour riches happiness in marriage and Ceres presents plenty of earthrsquos produce

o Iris summons the water-nymphs and reapers to come and celebrate a contract

(1) IRIS Of her society (Line 91-101)

Be not afraid I met her deity

Cutting the clouds towards Pathos and her sonDove-drawn with her Here thought they to have doneSome wanton charm upon this man and maidWhose vows are that no bed-right shall be paidTill Hymens torch be lightedmdashbut in vainMarss hot minion is returned againHer waspish-headed son has broke his arrowsSwears he will shoot no more but play with sparrowsAnd be a boy right out

(i) Where were Venus and Cupid seen flying How were they travelling Why did they want to join the marriage celebration of Ferdinand and Miranda

of true love

Venus and Cupid were seen flying through the air towards Paphos the famous city which is situated on the island of Cyprus They were travelling by air-borne chariot drawn by doves They certainly wanted to come here in order to play some amorous trick upon Ferdinand and Miranda who are under a vow not to gratify their physical desires till the holy ceremony of their marriage has been performed(ii) What have Venus and Cupid done after failing in their plan

After being failure of their plan Venus who is a very passionate deity and who is the mistress of Mars (the god of war) has gone back while here ill-tempered son Cupid has broken his arrows of love in his state of desperation(iii) What has Cupid firmly decided

Cupid is feeling so disappointed that he has firmly decided to shoot no more arrows to arouse love in human hearts but to spend his time playing with sparrows Thus he would now become just a boy and would give up his original function of shooting arrows on human beings to make them fall in love(iv) What vow had Ceres taken How did Ceres feel at the abduction

After the abduction of her daughter Prosperina by Pluto Ceres had taken a vow to always keep away from the disgraceful company of Venus and her blind son Cupid the god of love Ceres felt deeply distressed when Pluto had carried off her daughter and had made her his wife by force(v) Why has Ceres not forgiven Venus and her blind son For what do Ceres want to be sure

As the abduction had been manipulated by Venus the goddess of beauty and love and her blind son Cupid Ceres has never forgiven them for their part in the whole plot Ceres wants to be sure that she would not have to meet Venus and Cupid who had engineered the abduction of her daughter Prosperina

AS THIS lsquoMASQUErsquo SCENE IS VERY IMPORTANT IN THE PLAY THE PARAPHRASE OF THE ENTIRE PORTION OF MASQUE SCENE (Act IV Lines 58 to 143) IS GIVEN BELOW

IRIS Goddess of RainbowCERES Goddess of Agriculture and all the fruits of the earth

(Nature growth prosperity rebirth ndash notions intimately connected to marriage)JUNO The majestic Queen of Heavens and wife of Jupiter (Jupiter is the king of Gods)

VENUS The Goddess of love CUPID Son of Venus PLUTO God of death (In the play referred by Shakespeare as lsquoDisrsquo which is a Roman name for Pluto)

ORIGINAL TEXT PARAPHRASEPROSPEROWellmdash

PROSPERONow come Ariel Let there be too many rather than too few

Now come my Ariel Bring a corollaryRather than want a spirit Appear and pertly[to Ferdinand and Miranda]No tongue all eyes Be silent

spirits in attendance Appear briskly

[to Ferdinand and Miranda]Look with your eyes but do not say a word

[Soft music] [Soft music][Enter Iris] [Enter Iris]

IRISCeres most bounteous lady thy rich leasOf wheat rye barley vetches oats and peasThy turfy mountains where live nibbling sheepAnd flat meads thatched with stover them to keepThy banks with pioned and twilled brimsWhich spongy April at thy hest betrimsTo make cold nymphs chaste crowns and thybroom-grovesWhose shadow the dismissegraved bachelor lovesBeing lass-lorn thy pole clipped vineyardAnd thy sea-marge sterile and rocky-hardWhere thou thyself dost airmdashthe Queen othrsquoSkyWhose watery arch and messenger am IBids thee leave these and with her sovereign grace[Juno appears] Here on this grass-plot in this very placeTo come and sport Her peacocks fly amainApproach rich Ceres her to entertain

IRISCeres most generous lady you are the cause of rich fields or fertile land where wheat rye barley beans oats and peas grow the grassy mountains where the sheep graze and the flat meadows covered with coarse hay to be used as fodder for cattleYour banks are covered with marsh-marigolds and reeds and the rainy April under your orders brings forth to make for the maids who are not in love beautiful crowns your woods where the broom flourishes and where the bachelor who has been dismissed by the maid he loved lies down being forsaken your vineyard in which the poles are embraced by the vines and the margin of the sea which is barren and rocky where you roam about to enjoy the fresh air ndash the queen of the sky (Juno) whose messenger I am besides being represented as the rainbow bids you leave all these and with her majesty here on this grassy plot in this very place come and sport her peacocks carry her fast in her chariot through the air and are making their way here approach rich Ceres to welcome her

[Enter Ariel as Ceres] [Enter Ariel as Ceres]

CERESHail many-coloured messenger that neerDost disobey the wife of JupiterWho with thy saffron wings upon my flowersDiffusest honey-drops refreshing showersAnd with each end of thy blue bow dost crownMy bosky acres and my unshrubbed downRich scarf to my proud earth Why hath thy queenSummoned me hither to this short-grassed green

CERESWelcome rainbow that never dared disobey Juno the wife of Jupiter who with your orange coloured rays spread honey-drops refreshing showers And with each end of thy blue bow drown my bushy acres and my hilly country which is free from shrubs you thus forming a rich scarf Why has your queen called me here to this place covered with short grass

IRISA contract of true love to celebrateAnd some donation freely to estateOn the blest lovers

IRISI have called you to celebrate a contract of true love and bestow some liberal gift upon the blessed lovers

ORIGINAL TEXT PARAPHRASECERESTell me heavenly bowIf Venus or her son as thou dost knowDo now attend the queen Since they did plotThe means that dusky Dis my daughter gotHer and her blind boys scandaled companyI have forsworn

CERESTell me heavenly bow if Venus the Goddess of love or Cupid her son and pedlar of passion at this time attend the heavenly queen Juno because you are sure to know Since the day they conspired against me and dark Pluto took away my daughter here and Cupidrsquos disgraceful company I have left off

IRISOf her societyBe not afraid I met her deityCutting the clouds towards Pathos and her sonDove-drawn with her Here thought they to have doneSome wanton charm upon this man and miad

IRISBe not afraid of her company I met her deity moving on the clouds towards Paphos the sacred home of Venus on the island of Cyprus along with her son on her chariot drawn by doves Here they contemplated to exercise a charm upon this man and maid producing

Whose vows are that no bed-right shall be paidTill Hymens torch be lightedmdashbut in vainMarss hot minion is returned againHer waspish-headed son has broke his arrowsSwears he will shoot no more but play with sparrowsAnd be a boy right out

wantonness before the actual marriage ceremony but did not succeed Venus has returned her irritable son has broken his arrows and swears that he will give up his practice of trying to inspire love but play with sparrows and be a boy again

[Music is heard] [Music is heard]

CERESHighst queen of stateGreat Juno comes I know her by her gait

CERESHighest queen of state Great Juno there she comes I know here by her gait

[Enter Juno] [Enter Juno]

JUNOHow does my bounteous sister Go with meTo bless this twain that they may Prosperous beAnd honoured in their issue

JUNOHow are you doing my generous sister Come with me to bless this couple so that they may be prosperous and fortunate in their children

[They sing] [They sing]

JUNOHonour riches marriage-blessingLong continuance and increasingHourly joys be still upon youJuno sings her blessings upon you

JUNOMay honour riches happiness in marriage long continuance and increase of those boons ever rest upon you as hourly joys Juno showers down upon you her blessings in song

CERESEarths increase foison plentyBarns and garners never emptyVines and clustring bunches growingPlants and goodly burden bowingSpring come to you at the farthestIn the very end of harvestScarcity and want shall shun youCeresrsquo blessing so is on you

CERESMay you have the plenty of earthrsquos produce Your barns and granaries may never be empty Your vines may grow with clustering bunches Your fruit trees may be heavily laden with their fruit May there be continuous spring and harvest May scantiness and want leave you forever Such is the blessing of Ceres upon you

FERDINANDThis is a most majestic vision andHarmoniously charmingly May I be boldTo think these spirits

FERDINANDThis is a great vision and magically melodious Should I suppose the characters (taking part in the masque) are spirits

PROSPEROSpirits which by mine artI have from their confines calld to enactMy present fancies

PROSPEROYes they are spirits whom I have summoned from the regions to which they are confined to carry into effect my fanciful designs

ORIGINAL TEXT PARAPHRASEFERDINANDLet me live here everSo rare a wondered father and a wifeMakes this place paradise

FERDINANDI should like to live here forever Such a wise and wonderful father makes this place a paradise

[Juno and Ceres whisper and send Iris on employment] [Juno and Ceres whisper and send Iris on employment]

PROSPEROSweet now silence

PROSPEROMy dear Ferdinand speak no more Juno and Ceres are

Juno and Ceres whisper seriouslyTheres something else to do Hush and be muteOr else our spell is marred

whispering with a solemn look There is something else coming Silence Or else our magic will be spoilt

IRISYour nymphs called naiads of the wandering brooksWith your sedged crowns and over-harmless looksLeave your crisp channels and on this green landAnswer your summons Juno does commandCome temperate nymphs and help to celebrateA contract of true love Be not too late

IRISYou nymphs called Naiads denizens (M inhabitants) of the running stream with your chaplets of sedge and ever-helpful looks leave your wrinkled channels and on the green land answer the summons sent to you Juno has ordered some chaste nymphs and help to celebrate a noble and true marriage Donrsquot delay

[Enter certain nymphs] [Enter certain nymphs]You sunburnt sicklemen of August wearyCome hither from the furrow and be merryMake holiday your rye-straw hats put onAnd these fresh nymphs encounter every oneIn country footing

You sunburnt harvesters weary from the effects of the heat in August come here from the furrowed land and rejoice Make holiday with your rye-straw hats upon you and meet these fresh nymphs and join in country dancing

[Enter certain reapers properly habited They join with the nymphs in a graceful dance towards the end whereof Prospero starts suddenly and speaks]

[Enter certain reapers properly habited They join with the nymphs in a graceful dance towards the end whereof Prospero starts suddenly and speaks]

PROSPERO[aside] I had forgot that foul conspiracyOf the beast Caliban and his confederatesAgainst my life The minute of their plotIs almost come [to the spirits]Well done Avoidno more

PROSPERO(Aside)I had forgotten the wicked conspiracy of the beast Caliban and his accomplices against my life the time of their plot has almost arrived ndash (To the Spirits) well done depart no more of this

[To a strange hollow and confused noise the spirits heavily vanish]

[The spirits depart]

ORIGINAL TEXT PARAPHRASEFERDINANDLet me live here everSo rare a wondered father and a wifeMakes this place paradise

FERDINANDI should like to live here forever Such a wise and wonderful father makes this place a paradise

[Juno and Ceres whisper and send Iris on employment] [Juno and Ceres whisper and send Iris on employment]

PROSPEROSweet now silenceJuno and Ceres whisper seriouslyTheres something else to do Hush and be muteOr else our spell is marred

PROSPEROMy dear Ferdinand speak no more Juno and Ceres are whispering with a solemn look There is something else coming Silence Or else our magic will be spoilt

IRISYour nymphs called naiads of the wandering brooksWith your sedged crowns and over-harmless looksLeave your crisp channels and on this green landAnswer your summons Juno does commandCome temperate nymphs and help to celebrateA contract of true love Be not too late

IRISYou nymphs called Naiads denizens (M inhabitants) of the running stream with your chaplets of sedge and ever-helpful looks leave your wrinkled channels and on the green land answer the summons sent to you Juno has ordered some chaste nymphs and help to celebrate a noble and true marriage Donrsquot delay

[Enter certain nymphs] [Enter certain nymphs]You sunburnt sicklemen of August wearyCome hither from the furrow and be merryMake holiday your rye-straw hats put onAnd these fresh nymphs encounter every oneIn country footing

You sunburnt harvesters weary from the effects of the heat in August come here from the furrowed land and rejoice Make holiday with your rye-straw hats upon you and meet these fresh nymphs and join in country dancing

[Enter certain reapers properly habited They join with the nymphs in a graceful dance towards the end whereof Prospero starts suddenly and speaks]

[Enter certain reapers properly habited They join with the nymphs in a graceful dance towards the end whereof Prospero starts suddenly and speaks]

PROSPERO[aside] I had forgot that foul conspiracyOf the beast Caliban and his confederatesAgainst my life The minute of their plotIs almost come [to the spirits]Well done Avoidno more

PROSPERO(Aside)I had forgotten the wicked conspiracy of the beast Caliban and his accomplices against my life the time of their plot has almost arrived ndash (To the Spirits) well done depart no more of this

[To a strange hollow and confused noise the spirits heavily vanish]

[The spirits depart]

Ac-12 27420 topic Revaluation of Assets and Liabilities

REVALUATION OF ASSETS AND LIABILITIES

On admission of a new partner the firm stands reconstituted and consequently the assets are revalued and liabilities are reassessed It is necessary to show the true position of the firm at the time of admission of a new partner If the values of the assets are raised gain will increase the capital of the existing partners Similarly any decrease in the value of assets ie loss will decrease the capital of the existing partners For this purpose alsquoRevaluation Accountrsquo is prepared This account is credited with all increases in the value of assets and decrease in the value of liabilities It is debited with decrease on account of value of assets and increase in the value of liabilities The balance of this account shows a gain or loss on revaluation which is transferred to the existing partnerrsquos capital account in existing profit sharing ratioAccounting for Revaluation of Assets and Liabilities when there is a Changein the Profit Sharing Ratio of Existing PartnersAssets and liabilities of a firm must also be revalued at the time of change in profit sharing ratio of existing partners The reason is that the realisable or actual value of assets and liabilities may be different from those shown in the Balance Sheet It is possible that with the passage of time some of the assets might have appreciated in value while the value of certain other assets might have decreased and no record has been made of such changes in the books of accounts Similarly there may be some unrecorded assets amp libilities that may have to be accounted for Revaluation of assets and reassessments of liabilities becomes necessary because the change in the

value of assets and liabilities belongs to the period to change in profit sharing ratio and hence must be shared by the partners in their old profit sharing ratio Revaluation of assets and reassessment of liabilities may be given effect to in two different ways (a) When revised values are to be recorded in the books and(b) When revised values are not to be recorded in the books

When revised values are to be recorded in the booksIn such a case revaluation of assets and reassessment of liabilities is done with the help of a new account called lsquoRevaluation Accountrsquo Sometimes this account is also called as lsquoProfit amp Loss Adjustment Acrsquo If there is a loss due to revaluation revaluation account is debited and if the revaluation results in a profit the revaluation account is credited The following journal entries made for this purpose are

(i) For increase in the value of assetsAsset Ac Dr (individually)To Revaluation Ac(ii) For decrease in the value of AssetRevaluation Ac Dr (individually)To Asset Ac[Decrease in the value of assets](iii) For increase in the value of LiabilitiesRevaluation Ac Dr (individually)To Liabilities Ac[Increase in the value of Liabilities](iv) For decrease in the value of LiabilitiesLiabilities Ac DrTo Revaluation Ac[Decrease in the value of Liabilities](v) For unrecorded AssetsAsset Ac [unrecorded] DrTo Revaluation Ac[Unrecorded asset recorded at actual value](vi) For unrecorded Liability Revaluation Ac DrTo Liability Ac [unrecorded][Unrecorded Liability recorded at actual value](vii) For transfer of gain on revaluationRevaluation Ac DrTo Existing Partnerrsquos CapitalCurrent Ac[Profit on revaluation transferred to capital account in existing ratio](viii) For transfer of loss on revaluationExisting Partnerrsquos CapitalCurrent Ac DrTo Revaluation Ac[Loss on revaluation transferred to capital account in existing ratio](a) When revaluation account shows gain Revaluation Ac DrTo Partnerrsquos Capital Ac (Old Profit Sharing Ratio)(Profit on revaluation credited to Partnerrsquos Capital Ac)(b) Above entry is reversed when revaluation account shows loss Partners Capital Acs (Old Profit Sharing Ratio) DrTo Revaluation Ac(Loss on revaluation debited to Partnerrsquos Capital Acs)

Proforma of Revaluation Account is given as under

Revaluation Account

Dr Cr Particulars ` Amount Particulars ` Amount To Decrease in value of assets By Increase in value of assets To Increase in value of liabilities By Decrease in value of liabilities To Unrecorded liabilities By Unrecorded assets To Gain on Revaluation (Transferred) By Loss on Revalution (Transferred)

ECO ndash12 2742020Topic- ELASTICITY OF DEMAND

CHAPTER - ELASTICITY OF DEMANDMEANINGDemand for a commodity is affected by many factors such as its price price of related goods income of its buyer tastes and preferences etc Elasticity means degree of response Elasticity of demand means degree of responsiveness of demand Demand for a commodity responds to change in price price of related goods income etc So we have three dimensions of elasticity of demandDIMENSION OF ELASTICITY OF DEMAND TYPES OF ELASTICITY OF DEMAND

Price elasticity of demand Income elasticity of demand Cross Elasticity of demand

Price elasticity of demand Price elasticity of demand means degree of responsiveness of demand for a commodity to the change in its price For example if demand for a commodity rises by 10 due to 5 fall in its price Price elasticity of demand (ep)=Percentage change in quantity demanded Percentage change in price of the commodity = 10 ( -)5 = ( - )2Note that ep will always be negative due to inverse relationship of price and quantity demanded

(ii) Income elasticity of demand Income elasticity of demand refers to the degree of responsiveness of demand for a commodity to the change in income of its buyer Suppose income of buyer rises by 10 and his demand for a commodity rises by 20 then Income elasticity of demand (ey)= change in quantity demanded change in price of the commodity =20 10 = 2

Cross Elasticity of demandCross elasticity of demand means the degree of responsiveness of demand for a commodity to the change in price of its related goods (substitute goods or complementary goods) Suppose demand for a commodity rises by 10 due to 5 rise in price of its substitute good then Cross elasticity of demand (ec) = change in quantity demanded change in price of related good = 10 2 = 5 (Tastes and preferences cannot be expressed numerically So elasticity ofdemand cannot be numerically expressed)

  • Chapter 1 Force (Summary)
  • Distinguish between external sovereignty and internal sovereignty
    • NAND Gate
      • Logic diagram
      • Truth Table
        • NOR Gate
          • Logic diagram
          • Truth Table
            • XOR Gate
              • Logic diagram
              • Truth Table
                • XNOR Gate
                  • Logic diagram
                  • Truth Table
                      • Physics
                      • Chapter 1 Electric Field ( Electric Dipole) (Summary)
Page 52:  · Web viewWe all know that Nouns are divided into two parts: common noun and proper noun.Apart from common and proper noun, we will also study about collective noun and compound

Also find the expression of torque experience by a dipole

(Hint Electric force experienced by charges of dipole in electric field is qE each Let θ be the angle which dipole makes with electric lines of force then perpendicular distance between two charges is 2Lsinθ Then torque = force x perp distance = qE x 2L sinθ So τ=pE sinθ where p =2qL )

STUDY MATERIAL

Class XIISubject Eng Literature (The Tempest ndash William Shakespeare) Topic Act IV Scene 1 Lines 84 to 133 (Iris hellip A contract of true love Be not too late ) Date 27th April 2020 (4th Period)

[Students should read the original play and also the paraphrase given in the school prescribed textbook]Summary Questions amp Answers

o Ceres soon appears and comes to know that she has been summoned to celebrate the contract of true love

o Ceres expresses her unwillingness to meet Venus and Cupid as she has shunned their company

o Ceres and Juno both bestow their blessings upon Ferdinand and Miranda with June gifting honour riches happiness in marriage and Ceres presents plenty of earthrsquos produce

o Iris summons the water-nymphs and reapers to come and celebrate a contract

(1) IRIS Of her society (Line 91-101)

Be not afraid I met her deity

Cutting the clouds towards Pathos and her sonDove-drawn with her Here thought they to have doneSome wanton charm upon this man and maidWhose vows are that no bed-right shall be paidTill Hymens torch be lightedmdashbut in vainMarss hot minion is returned againHer waspish-headed son has broke his arrowsSwears he will shoot no more but play with sparrowsAnd be a boy right out

(i) Where were Venus and Cupid seen flying How were they travelling Why did they want to join the marriage celebration of Ferdinand and Miranda

of true love

Venus and Cupid were seen flying through the air towards Paphos the famous city which is situated on the island of Cyprus They were travelling by air-borne chariot drawn by doves They certainly wanted to come here in order to play some amorous trick upon Ferdinand and Miranda who are under a vow not to gratify their physical desires till the holy ceremony of their marriage has been performed(ii) What have Venus and Cupid done after failing in their plan

After being failure of their plan Venus who is a very passionate deity and who is the mistress of Mars (the god of war) has gone back while here ill-tempered son Cupid has broken his arrows of love in his state of desperation(iii) What has Cupid firmly decided

Cupid is feeling so disappointed that he has firmly decided to shoot no more arrows to arouse love in human hearts but to spend his time playing with sparrows Thus he would now become just a boy and would give up his original function of shooting arrows on human beings to make them fall in love(iv) What vow had Ceres taken How did Ceres feel at the abduction

After the abduction of her daughter Prosperina by Pluto Ceres had taken a vow to always keep away from the disgraceful company of Venus and her blind son Cupid the god of love Ceres felt deeply distressed when Pluto had carried off her daughter and had made her his wife by force(v) Why has Ceres not forgiven Venus and her blind son For what do Ceres want to be sure

As the abduction had been manipulated by Venus the goddess of beauty and love and her blind son Cupid Ceres has never forgiven them for their part in the whole plot Ceres wants to be sure that she would not have to meet Venus and Cupid who had engineered the abduction of her daughter Prosperina

AS THIS lsquoMASQUErsquo SCENE IS VERY IMPORTANT IN THE PLAY THE PARAPHRASE OF THE ENTIRE PORTION OF MASQUE SCENE (Act IV Lines 58 to 143) IS GIVEN BELOW

IRIS Goddess of RainbowCERES Goddess of Agriculture and all the fruits of the earth

(Nature growth prosperity rebirth ndash notions intimately connected to marriage)JUNO The majestic Queen of Heavens and wife of Jupiter (Jupiter is the king of Gods)

VENUS The Goddess of love CUPID Son of Venus PLUTO God of death (In the play referred by Shakespeare as lsquoDisrsquo which is a Roman name for Pluto)

ORIGINAL TEXT PARAPHRASEPROSPEROWellmdash

PROSPERONow come Ariel Let there be too many rather than too few

Now come my Ariel Bring a corollaryRather than want a spirit Appear and pertly[to Ferdinand and Miranda]No tongue all eyes Be silent

spirits in attendance Appear briskly

[to Ferdinand and Miranda]Look with your eyes but do not say a word

[Soft music] [Soft music][Enter Iris] [Enter Iris]

IRISCeres most bounteous lady thy rich leasOf wheat rye barley vetches oats and peasThy turfy mountains where live nibbling sheepAnd flat meads thatched with stover them to keepThy banks with pioned and twilled brimsWhich spongy April at thy hest betrimsTo make cold nymphs chaste crowns and thybroom-grovesWhose shadow the dismissegraved bachelor lovesBeing lass-lorn thy pole clipped vineyardAnd thy sea-marge sterile and rocky-hardWhere thou thyself dost airmdashthe Queen othrsquoSkyWhose watery arch and messenger am IBids thee leave these and with her sovereign grace[Juno appears] Here on this grass-plot in this very placeTo come and sport Her peacocks fly amainApproach rich Ceres her to entertain

IRISCeres most generous lady you are the cause of rich fields or fertile land where wheat rye barley beans oats and peas grow the grassy mountains where the sheep graze and the flat meadows covered with coarse hay to be used as fodder for cattleYour banks are covered with marsh-marigolds and reeds and the rainy April under your orders brings forth to make for the maids who are not in love beautiful crowns your woods where the broom flourishes and where the bachelor who has been dismissed by the maid he loved lies down being forsaken your vineyard in which the poles are embraced by the vines and the margin of the sea which is barren and rocky where you roam about to enjoy the fresh air ndash the queen of the sky (Juno) whose messenger I am besides being represented as the rainbow bids you leave all these and with her majesty here on this grassy plot in this very place come and sport her peacocks carry her fast in her chariot through the air and are making their way here approach rich Ceres to welcome her

[Enter Ariel as Ceres] [Enter Ariel as Ceres]

CERESHail many-coloured messenger that neerDost disobey the wife of JupiterWho with thy saffron wings upon my flowersDiffusest honey-drops refreshing showersAnd with each end of thy blue bow dost crownMy bosky acres and my unshrubbed downRich scarf to my proud earth Why hath thy queenSummoned me hither to this short-grassed green

CERESWelcome rainbow that never dared disobey Juno the wife of Jupiter who with your orange coloured rays spread honey-drops refreshing showers And with each end of thy blue bow drown my bushy acres and my hilly country which is free from shrubs you thus forming a rich scarf Why has your queen called me here to this place covered with short grass

IRISA contract of true love to celebrateAnd some donation freely to estateOn the blest lovers

IRISI have called you to celebrate a contract of true love and bestow some liberal gift upon the blessed lovers

ORIGINAL TEXT PARAPHRASECERESTell me heavenly bowIf Venus or her son as thou dost knowDo now attend the queen Since they did plotThe means that dusky Dis my daughter gotHer and her blind boys scandaled companyI have forsworn

CERESTell me heavenly bow if Venus the Goddess of love or Cupid her son and pedlar of passion at this time attend the heavenly queen Juno because you are sure to know Since the day they conspired against me and dark Pluto took away my daughter here and Cupidrsquos disgraceful company I have left off

IRISOf her societyBe not afraid I met her deityCutting the clouds towards Pathos and her sonDove-drawn with her Here thought they to have doneSome wanton charm upon this man and miad

IRISBe not afraid of her company I met her deity moving on the clouds towards Paphos the sacred home of Venus on the island of Cyprus along with her son on her chariot drawn by doves Here they contemplated to exercise a charm upon this man and maid producing

Whose vows are that no bed-right shall be paidTill Hymens torch be lightedmdashbut in vainMarss hot minion is returned againHer waspish-headed son has broke his arrowsSwears he will shoot no more but play with sparrowsAnd be a boy right out

wantonness before the actual marriage ceremony but did not succeed Venus has returned her irritable son has broken his arrows and swears that he will give up his practice of trying to inspire love but play with sparrows and be a boy again

[Music is heard] [Music is heard]

CERESHighst queen of stateGreat Juno comes I know her by her gait

CERESHighest queen of state Great Juno there she comes I know here by her gait

[Enter Juno] [Enter Juno]

JUNOHow does my bounteous sister Go with meTo bless this twain that they may Prosperous beAnd honoured in their issue

JUNOHow are you doing my generous sister Come with me to bless this couple so that they may be prosperous and fortunate in their children

[They sing] [They sing]

JUNOHonour riches marriage-blessingLong continuance and increasingHourly joys be still upon youJuno sings her blessings upon you

JUNOMay honour riches happiness in marriage long continuance and increase of those boons ever rest upon you as hourly joys Juno showers down upon you her blessings in song

CERESEarths increase foison plentyBarns and garners never emptyVines and clustring bunches growingPlants and goodly burden bowingSpring come to you at the farthestIn the very end of harvestScarcity and want shall shun youCeresrsquo blessing so is on you

CERESMay you have the plenty of earthrsquos produce Your barns and granaries may never be empty Your vines may grow with clustering bunches Your fruit trees may be heavily laden with their fruit May there be continuous spring and harvest May scantiness and want leave you forever Such is the blessing of Ceres upon you

FERDINANDThis is a most majestic vision andHarmoniously charmingly May I be boldTo think these spirits

FERDINANDThis is a great vision and magically melodious Should I suppose the characters (taking part in the masque) are spirits

PROSPEROSpirits which by mine artI have from their confines calld to enactMy present fancies

PROSPEROYes they are spirits whom I have summoned from the regions to which they are confined to carry into effect my fanciful designs

ORIGINAL TEXT PARAPHRASEFERDINANDLet me live here everSo rare a wondered father and a wifeMakes this place paradise

FERDINANDI should like to live here forever Such a wise and wonderful father makes this place a paradise

[Juno and Ceres whisper and send Iris on employment] [Juno and Ceres whisper and send Iris on employment]

PROSPEROSweet now silence

PROSPEROMy dear Ferdinand speak no more Juno and Ceres are

Juno and Ceres whisper seriouslyTheres something else to do Hush and be muteOr else our spell is marred

whispering with a solemn look There is something else coming Silence Or else our magic will be spoilt

IRISYour nymphs called naiads of the wandering brooksWith your sedged crowns and over-harmless looksLeave your crisp channels and on this green landAnswer your summons Juno does commandCome temperate nymphs and help to celebrateA contract of true love Be not too late

IRISYou nymphs called Naiads denizens (M inhabitants) of the running stream with your chaplets of sedge and ever-helpful looks leave your wrinkled channels and on the green land answer the summons sent to you Juno has ordered some chaste nymphs and help to celebrate a noble and true marriage Donrsquot delay

[Enter certain nymphs] [Enter certain nymphs]You sunburnt sicklemen of August wearyCome hither from the furrow and be merryMake holiday your rye-straw hats put onAnd these fresh nymphs encounter every oneIn country footing

You sunburnt harvesters weary from the effects of the heat in August come here from the furrowed land and rejoice Make holiday with your rye-straw hats upon you and meet these fresh nymphs and join in country dancing

[Enter certain reapers properly habited They join with the nymphs in a graceful dance towards the end whereof Prospero starts suddenly and speaks]

[Enter certain reapers properly habited They join with the nymphs in a graceful dance towards the end whereof Prospero starts suddenly and speaks]

PROSPERO[aside] I had forgot that foul conspiracyOf the beast Caliban and his confederatesAgainst my life The minute of their plotIs almost come [to the spirits]Well done Avoidno more

PROSPERO(Aside)I had forgotten the wicked conspiracy of the beast Caliban and his accomplices against my life the time of their plot has almost arrived ndash (To the Spirits) well done depart no more of this

[To a strange hollow and confused noise the spirits heavily vanish]

[The spirits depart]

ORIGINAL TEXT PARAPHRASEFERDINANDLet me live here everSo rare a wondered father and a wifeMakes this place paradise

FERDINANDI should like to live here forever Such a wise and wonderful father makes this place a paradise

[Juno and Ceres whisper and send Iris on employment] [Juno and Ceres whisper and send Iris on employment]

PROSPEROSweet now silenceJuno and Ceres whisper seriouslyTheres something else to do Hush and be muteOr else our spell is marred

PROSPEROMy dear Ferdinand speak no more Juno and Ceres are whispering with a solemn look There is something else coming Silence Or else our magic will be spoilt

IRISYour nymphs called naiads of the wandering brooksWith your sedged crowns and over-harmless looksLeave your crisp channels and on this green landAnswer your summons Juno does commandCome temperate nymphs and help to celebrateA contract of true love Be not too late

IRISYou nymphs called Naiads denizens (M inhabitants) of the running stream with your chaplets of sedge and ever-helpful looks leave your wrinkled channels and on the green land answer the summons sent to you Juno has ordered some chaste nymphs and help to celebrate a noble and true marriage Donrsquot delay

[Enter certain nymphs] [Enter certain nymphs]You sunburnt sicklemen of August wearyCome hither from the furrow and be merryMake holiday your rye-straw hats put onAnd these fresh nymphs encounter every oneIn country footing

You sunburnt harvesters weary from the effects of the heat in August come here from the furrowed land and rejoice Make holiday with your rye-straw hats upon you and meet these fresh nymphs and join in country dancing

[Enter certain reapers properly habited They join with the nymphs in a graceful dance towards the end whereof Prospero starts suddenly and speaks]

[Enter certain reapers properly habited They join with the nymphs in a graceful dance towards the end whereof Prospero starts suddenly and speaks]

PROSPERO[aside] I had forgot that foul conspiracyOf the beast Caliban and his confederatesAgainst my life The minute of their plotIs almost come [to the spirits]Well done Avoidno more

PROSPERO(Aside)I had forgotten the wicked conspiracy of the beast Caliban and his accomplices against my life the time of their plot has almost arrived ndash (To the Spirits) well done depart no more of this

[To a strange hollow and confused noise the spirits heavily vanish]

[The spirits depart]

Ac-12 27420 topic Revaluation of Assets and Liabilities

REVALUATION OF ASSETS AND LIABILITIES

On admission of a new partner the firm stands reconstituted and consequently the assets are revalued and liabilities are reassessed It is necessary to show the true position of the firm at the time of admission of a new partner If the values of the assets are raised gain will increase the capital of the existing partners Similarly any decrease in the value of assets ie loss will decrease the capital of the existing partners For this purpose alsquoRevaluation Accountrsquo is prepared This account is credited with all increases in the value of assets and decrease in the value of liabilities It is debited with decrease on account of value of assets and increase in the value of liabilities The balance of this account shows a gain or loss on revaluation which is transferred to the existing partnerrsquos capital account in existing profit sharing ratioAccounting for Revaluation of Assets and Liabilities when there is a Changein the Profit Sharing Ratio of Existing PartnersAssets and liabilities of a firm must also be revalued at the time of change in profit sharing ratio of existing partners The reason is that the realisable or actual value of assets and liabilities may be different from those shown in the Balance Sheet It is possible that with the passage of time some of the assets might have appreciated in value while the value of certain other assets might have decreased and no record has been made of such changes in the books of accounts Similarly there may be some unrecorded assets amp libilities that may have to be accounted for Revaluation of assets and reassessments of liabilities becomes necessary because the change in the

value of assets and liabilities belongs to the period to change in profit sharing ratio and hence must be shared by the partners in their old profit sharing ratio Revaluation of assets and reassessment of liabilities may be given effect to in two different ways (a) When revised values are to be recorded in the books and(b) When revised values are not to be recorded in the books

When revised values are to be recorded in the booksIn such a case revaluation of assets and reassessment of liabilities is done with the help of a new account called lsquoRevaluation Accountrsquo Sometimes this account is also called as lsquoProfit amp Loss Adjustment Acrsquo If there is a loss due to revaluation revaluation account is debited and if the revaluation results in a profit the revaluation account is credited The following journal entries made for this purpose are

(i) For increase in the value of assetsAsset Ac Dr (individually)To Revaluation Ac(ii) For decrease in the value of AssetRevaluation Ac Dr (individually)To Asset Ac[Decrease in the value of assets](iii) For increase in the value of LiabilitiesRevaluation Ac Dr (individually)To Liabilities Ac[Increase in the value of Liabilities](iv) For decrease in the value of LiabilitiesLiabilities Ac DrTo Revaluation Ac[Decrease in the value of Liabilities](v) For unrecorded AssetsAsset Ac [unrecorded] DrTo Revaluation Ac[Unrecorded asset recorded at actual value](vi) For unrecorded Liability Revaluation Ac DrTo Liability Ac [unrecorded][Unrecorded Liability recorded at actual value](vii) For transfer of gain on revaluationRevaluation Ac DrTo Existing Partnerrsquos CapitalCurrent Ac[Profit on revaluation transferred to capital account in existing ratio](viii) For transfer of loss on revaluationExisting Partnerrsquos CapitalCurrent Ac DrTo Revaluation Ac[Loss on revaluation transferred to capital account in existing ratio](a) When revaluation account shows gain Revaluation Ac DrTo Partnerrsquos Capital Ac (Old Profit Sharing Ratio)(Profit on revaluation credited to Partnerrsquos Capital Ac)(b) Above entry is reversed when revaluation account shows loss Partners Capital Acs (Old Profit Sharing Ratio) DrTo Revaluation Ac(Loss on revaluation debited to Partnerrsquos Capital Acs)

Proforma of Revaluation Account is given as under

Revaluation Account

Dr Cr Particulars ` Amount Particulars ` Amount To Decrease in value of assets By Increase in value of assets To Increase in value of liabilities By Decrease in value of liabilities To Unrecorded liabilities By Unrecorded assets To Gain on Revaluation (Transferred) By Loss on Revalution (Transferred)

ECO ndash12 2742020Topic- ELASTICITY OF DEMAND

CHAPTER - ELASTICITY OF DEMANDMEANINGDemand for a commodity is affected by many factors such as its price price of related goods income of its buyer tastes and preferences etc Elasticity means degree of response Elasticity of demand means degree of responsiveness of demand Demand for a commodity responds to change in price price of related goods income etc So we have three dimensions of elasticity of demandDIMENSION OF ELASTICITY OF DEMAND TYPES OF ELASTICITY OF DEMAND

Price elasticity of demand Income elasticity of demand Cross Elasticity of demand

Price elasticity of demand Price elasticity of demand means degree of responsiveness of demand for a commodity to the change in its price For example if demand for a commodity rises by 10 due to 5 fall in its price Price elasticity of demand (ep)=Percentage change in quantity demanded Percentage change in price of the commodity = 10 ( -)5 = ( - )2Note that ep will always be negative due to inverse relationship of price and quantity demanded

(ii) Income elasticity of demand Income elasticity of demand refers to the degree of responsiveness of demand for a commodity to the change in income of its buyer Suppose income of buyer rises by 10 and his demand for a commodity rises by 20 then Income elasticity of demand (ey)= change in quantity demanded change in price of the commodity =20 10 = 2

Cross Elasticity of demandCross elasticity of demand means the degree of responsiveness of demand for a commodity to the change in price of its related goods (substitute goods or complementary goods) Suppose demand for a commodity rises by 10 due to 5 rise in price of its substitute good then Cross elasticity of demand (ec) = change in quantity demanded change in price of related good = 10 2 = 5 (Tastes and preferences cannot be expressed numerically So elasticity ofdemand cannot be numerically expressed)

  • Chapter 1 Force (Summary)
  • Distinguish between external sovereignty and internal sovereignty
    • NAND Gate
      • Logic diagram
      • Truth Table
        • NOR Gate
          • Logic diagram
          • Truth Table
            • XOR Gate
              • Logic diagram
              • Truth Table
                • XNOR Gate
                  • Logic diagram
                  • Truth Table
                      • Physics
                      • Chapter 1 Electric Field ( Electric Dipole) (Summary)
Page 53:  · Web viewWe all know that Nouns are divided into two parts: common noun and proper noun.Apart from common and proper noun, we will also study about collective noun and compound

of true love

Venus and Cupid were seen flying through the air towards Paphos the famous city which is situated on the island of Cyprus They were travelling by air-borne chariot drawn by doves They certainly wanted to come here in order to play some amorous trick upon Ferdinand and Miranda who are under a vow not to gratify their physical desires till the holy ceremony of their marriage has been performed(ii) What have Venus and Cupid done after failing in their plan

After being failure of their plan Venus who is a very passionate deity and who is the mistress of Mars (the god of war) has gone back while here ill-tempered son Cupid has broken his arrows of love in his state of desperation(iii) What has Cupid firmly decided

Cupid is feeling so disappointed that he has firmly decided to shoot no more arrows to arouse love in human hearts but to spend his time playing with sparrows Thus he would now become just a boy and would give up his original function of shooting arrows on human beings to make them fall in love(iv) What vow had Ceres taken How did Ceres feel at the abduction

After the abduction of her daughter Prosperina by Pluto Ceres had taken a vow to always keep away from the disgraceful company of Venus and her blind son Cupid the god of love Ceres felt deeply distressed when Pluto had carried off her daughter and had made her his wife by force(v) Why has Ceres not forgiven Venus and her blind son For what do Ceres want to be sure

As the abduction had been manipulated by Venus the goddess of beauty and love and her blind son Cupid Ceres has never forgiven them for their part in the whole plot Ceres wants to be sure that she would not have to meet Venus and Cupid who had engineered the abduction of her daughter Prosperina

AS THIS lsquoMASQUErsquo SCENE IS VERY IMPORTANT IN THE PLAY THE PARAPHRASE OF THE ENTIRE PORTION OF MASQUE SCENE (Act IV Lines 58 to 143) IS GIVEN BELOW

IRIS Goddess of RainbowCERES Goddess of Agriculture and all the fruits of the earth

(Nature growth prosperity rebirth ndash notions intimately connected to marriage)JUNO The majestic Queen of Heavens and wife of Jupiter (Jupiter is the king of Gods)

VENUS The Goddess of love CUPID Son of Venus PLUTO God of death (In the play referred by Shakespeare as lsquoDisrsquo which is a Roman name for Pluto)

ORIGINAL TEXT PARAPHRASEPROSPEROWellmdash

PROSPERONow come Ariel Let there be too many rather than too few

Now come my Ariel Bring a corollaryRather than want a spirit Appear and pertly[to Ferdinand and Miranda]No tongue all eyes Be silent

spirits in attendance Appear briskly

[to Ferdinand and Miranda]Look with your eyes but do not say a word

[Soft music] [Soft music][Enter Iris] [Enter Iris]

IRISCeres most bounteous lady thy rich leasOf wheat rye barley vetches oats and peasThy turfy mountains where live nibbling sheepAnd flat meads thatched with stover them to keepThy banks with pioned and twilled brimsWhich spongy April at thy hest betrimsTo make cold nymphs chaste crowns and thybroom-grovesWhose shadow the dismissegraved bachelor lovesBeing lass-lorn thy pole clipped vineyardAnd thy sea-marge sterile and rocky-hardWhere thou thyself dost airmdashthe Queen othrsquoSkyWhose watery arch and messenger am IBids thee leave these and with her sovereign grace[Juno appears] Here on this grass-plot in this very placeTo come and sport Her peacocks fly amainApproach rich Ceres her to entertain

IRISCeres most generous lady you are the cause of rich fields or fertile land where wheat rye barley beans oats and peas grow the grassy mountains where the sheep graze and the flat meadows covered with coarse hay to be used as fodder for cattleYour banks are covered with marsh-marigolds and reeds and the rainy April under your orders brings forth to make for the maids who are not in love beautiful crowns your woods where the broom flourishes and where the bachelor who has been dismissed by the maid he loved lies down being forsaken your vineyard in which the poles are embraced by the vines and the margin of the sea which is barren and rocky where you roam about to enjoy the fresh air ndash the queen of the sky (Juno) whose messenger I am besides being represented as the rainbow bids you leave all these and with her majesty here on this grassy plot in this very place come and sport her peacocks carry her fast in her chariot through the air and are making their way here approach rich Ceres to welcome her

[Enter Ariel as Ceres] [Enter Ariel as Ceres]

CERESHail many-coloured messenger that neerDost disobey the wife of JupiterWho with thy saffron wings upon my flowersDiffusest honey-drops refreshing showersAnd with each end of thy blue bow dost crownMy bosky acres and my unshrubbed downRich scarf to my proud earth Why hath thy queenSummoned me hither to this short-grassed green

CERESWelcome rainbow that never dared disobey Juno the wife of Jupiter who with your orange coloured rays spread honey-drops refreshing showers And with each end of thy blue bow drown my bushy acres and my hilly country which is free from shrubs you thus forming a rich scarf Why has your queen called me here to this place covered with short grass

IRISA contract of true love to celebrateAnd some donation freely to estateOn the blest lovers

IRISI have called you to celebrate a contract of true love and bestow some liberal gift upon the blessed lovers

ORIGINAL TEXT PARAPHRASECERESTell me heavenly bowIf Venus or her son as thou dost knowDo now attend the queen Since they did plotThe means that dusky Dis my daughter gotHer and her blind boys scandaled companyI have forsworn

CERESTell me heavenly bow if Venus the Goddess of love or Cupid her son and pedlar of passion at this time attend the heavenly queen Juno because you are sure to know Since the day they conspired against me and dark Pluto took away my daughter here and Cupidrsquos disgraceful company I have left off

IRISOf her societyBe not afraid I met her deityCutting the clouds towards Pathos and her sonDove-drawn with her Here thought they to have doneSome wanton charm upon this man and miad

IRISBe not afraid of her company I met her deity moving on the clouds towards Paphos the sacred home of Venus on the island of Cyprus along with her son on her chariot drawn by doves Here they contemplated to exercise a charm upon this man and maid producing

Whose vows are that no bed-right shall be paidTill Hymens torch be lightedmdashbut in vainMarss hot minion is returned againHer waspish-headed son has broke his arrowsSwears he will shoot no more but play with sparrowsAnd be a boy right out

wantonness before the actual marriage ceremony but did not succeed Venus has returned her irritable son has broken his arrows and swears that he will give up his practice of trying to inspire love but play with sparrows and be a boy again

[Music is heard] [Music is heard]

CERESHighst queen of stateGreat Juno comes I know her by her gait

CERESHighest queen of state Great Juno there she comes I know here by her gait

[Enter Juno] [Enter Juno]

JUNOHow does my bounteous sister Go with meTo bless this twain that they may Prosperous beAnd honoured in their issue

JUNOHow are you doing my generous sister Come with me to bless this couple so that they may be prosperous and fortunate in their children

[They sing] [They sing]

JUNOHonour riches marriage-blessingLong continuance and increasingHourly joys be still upon youJuno sings her blessings upon you

JUNOMay honour riches happiness in marriage long continuance and increase of those boons ever rest upon you as hourly joys Juno showers down upon you her blessings in song

CERESEarths increase foison plentyBarns and garners never emptyVines and clustring bunches growingPlants and goodly burden bowingSpring come to you at the farthestIn the very end of harvestScarcity and want shall shun youCeresrsquo blessing so is on you

CERESMay you have the plenty of earthrsquos produce Your barns and granaries may never be empty Your vines may grow with clustering bunches Your fruit trees may be heavily laden with their fruit May there be continuous spring and harvest May scantiness and want leave you forever Such is the blessing of Ceres upon you

FERDINANDThis is a most majestic vision andHarmoniously charmingly May I be boldTo think these spirits

FERDINANDThis is a great vision and magically melodious Should I suppose the characters (taking part in the masque) are spirits

PROSPEROSpirits which by mine artI have from their confines calld to enactMy present fancies

PROSPEROYes they are spirits whom I have summoned from the regions to which they are confined to carry into effect my fanciful designs

ORIGINAL TEXT PARAPHRASEFERDINANDLet me live here everSo rare a wondered father and a wifeMakes this place paradise

FERDINANDI should like to live here forever Such a wise and wonderful father makes this place a paradise

[Juno and Ceres whisper and send Iris on employment] [Juno and Ceres whisper and send Iris on employment]

PROSPEROSweet now silence

PROSPEROMy dear Ferdinand speak no more Juno and Ceres are

Juno and Ceres whisper seriouslyTheres something else to do Hush and be muteOr else our spell is marred

whispering with a solemn look There is something else coming Silence Or else our magic will be spoilt

IRISYour nymphs called naiads of the wandering brooksWith your sedged crowns and over-harmless looksLeave your crisp channels and on this green landAnswer your summons Juno does commandCome temperate nymphs and help to celebrateA contract of true love Be not too late

IRISYou nymphs called Naiads denizens (M inhabitants) of the running stream with your chaplets of sedge and ever-helpful looks leave your wrinkled channels and on the green land answer the summons sent to you Juno has ordered some chaste nymphs and help to celebrate a noble and true marriage Donrsquot delay

[Enter certain nymphs] [Enter certain nymphs]You sunburnt sicklemen of August wearyCome hither from the furrow and be merryMake holiday your rye-straw hats put onAnd these fresh nymphs encounter every oneIn country footing

You sunburnt harvesters weary from the effects of the heat in August come here from the furrowed land and rejoice Make holiday with your rye-straw hats upon you and meet these fresh nymphs and join in country dancing

[Enter certain reapers properly habited They join with the nymphs in a graceful dance towards the end whereof Prospero starts suddenly and speaks]

[Enter certain reapers properly habited They join with the nymphs in a graceful dance towards the end whereof Prospero starts suddenly and speaks]

PROSPERO[aside] I had forgot that foul conspiracyOf the beast Caliban and his confederatesAgainst my life The minute of their plotIs almost come [to the spirits]Well done Avoidno more

PROSPERO(Aside)I had forgotten the wicked conspiracy of the beast Caliban and his accomplices against my life the time of their plot has almost arrived ndash (To the Spirits) well done depart no more of this

[To a strange hollow and confused noise the spirits heavily vanish]

[The spirits depart]

ORIGINAL TEXT PARAPHRASEFERDINANDLet me live here everSo rare a wondered father and a wifeMakes this place paradise

FERDINANDI should like to live here forever Such a wise and wonderful father makes this place a paradise

[Juno and Ceres whisper and send Iris on employment] [Juno and Ceres whisper and send Iris on employment]

PROSPEROSweet now silenceJuno and Ceres whisper seriouslyTheres something else to do Hush and be muteOr else our spell is marred

PROSPEROMy dear Ferdinand speak no more Juno and Ceres are whispering with a solemn look There is something else coming Silence Or else our magic will be spoilt

IRISYour nymphs called naiads of the wandering brooksWith your sedged crowns and over-harmless looksLeave your crisp channels and on this green landAnswer your summons Juno does commandCome temperate nymphs and help to celebrateA contract of true love Be not too late

IRISYou nymphs called Naiads denizens (M inhabitants) of the running stream with your chaplets of sedge and ever-helpful looks leave your wrinkled channels and on the green land answer the summons sent to you Juno has ordered some chaste nymphs and help to celebrate a noble and true marriage Donrsquot delay

[Enter certain nymphs] [Enter certain nymphs]You sunburnt sicklemen of August wearyCome hither from the furrow and be merryMake holiday your rye-straw hats put onAnd these fresh nymphs encounter every oneIn country footing

You sunburnt harvesters weary from the effects of the heat in August come here from the furrowed land and rejoice Make holiday with your rye-straw hats upon you and meet these fresh nymphs and join in country dancing

[Enter certain reapers properly habited They join with the nymphs in a graceful dance towards the end whereof Prospero starts suddenly and speaks]

[Enter certain reapers properly habited They join with the nymphs in a graceful dance towards the end whereof Prospero starts suddenly and speaks]

PROSPERO[aside] I had forgot that foul conspiracyOf the beast Caliban and his confederatesAgainst my life The minute of their plotIs almost come [to the spirits]Well done Avoidno more

PROSPERO(Aside)I had forgotten the wicked conspiracy of the beast Caliban and his accomplices against my life the time of their plot has almost arrived ndash (To the Spirits) well done depart no more of this

[To a strange hollow and confused noise the spirits heavily vanish]

[The spirits depart]

Ac-12 27420 topic Revaluation of Assets and Liabilities

REVALUATION OF ASSETS AND LIABILITIES

On admission of a new partner the firm stands reconstituted and consequently the assets are revalued and liabilities are reassessed It is necessary to show the true position of the firm at the time of admission of a new partner If the values of the assets are raised gain will increase the capital of the existing partners Similarly any decrease in the value of assets ie loss will decrease the capital of the existing partners For this purpose alsquoRevaluation Accountrsquo is prepared This account is credited with all increases in the value of assets and decrease in the value of liabilities It is debited with decrease on account of value of assets and increase in the value of liabilities The balance of this account shows a gain or loss on revaluation which is transferred to the existing partnerrsquos capital account in existing profit sharing ratioAccounting for Revaluation of Assets and Liabilities when there is a Changein the Profit Sharing Ratio of Existing PartnersAssets and liabilities of a firm must also be revalued at the time of change in profit sharing ratio of existing partners The reason is that the realisable or actual value of assets and liabilities may be different from those shown in the Balance Sheet It is possible that with the passage of time some of the assets might have appreciated in value while the value of certain other assets might have decreased and no record has been made of such changes in the books of accounts Similarly there may be some unrecorded assets amp libilities that may have to be accounted for Revaluation of assets and reassessments of liabilities becomes necessary because the change in the

value of assets and liabilities belongs to the period to change in profit sharing ratio and hence must be shared by the partners in their old profit sharing ratio Revaluation of assets and reassessment of liabilities may be given effect to in two different ways (a) When revised values are to be recorded in the books and(b) When revised values are not to be recorded in the books

When revised values are to be recorded in the booksIn such a case revaluation of assets and reassessment of liabilities is done with the help of a new account called lsquoRevaluation Accountrsquo Sometimes this account is also called as lsquoProfit amp Loss Adjustment Acrsquo If there is a loss due to revaluation revaluation account is debited and if the revaluation results in a profit the revaluation account is credited The following journal entries made for this purpose are

(i) For increase in the value of assetsAsset Ac Dr (individually)To Revaluation Ac(ii) For decrease in the value of AssetRevaluation Ac Dr (individually)To Asset Ac[Decrease in the value of assets](iii) For increase in the value of LiabilitiesRevaluation Ac Dr (individually)To Liabilities Ac[Increase in the value of Liabilities](iv) For decrease in the value of LiabilitiesLiabilities Ac DrTo Revaluation Ac[Decrease in the value of Liabilities](v) For unrecorded AssetsAsset Ac [unrecorded] DrTo Revaluation Ac[Unrecorded asset recorded at actual value](vi) For unrecorded Liability Revaluation Ac DrTo Liability Ac [unrecorded][Unrecorded Liability recorded at actual value](vii) For transfer of gain on revaluationRevaluation Ac DrTo Existing Partnerrsquos CapitalCurrent Ac[Profit on revaluation transferred to capital account in existing ratio](viii) For transfer of loss on revaluationExisting Partnerrsquos CapitalCurrent Ac DrTo Revaluation Ac[Loss on revaluation transferred to capital account in existing ratio](a) When revaluation account shows gain Revaluation Ac DrTo Partnerrsquos Capital Ac (Old Profit Sharing Ratio)(Profit on revaluation credited to Partnerrsquos Capital Ac)(b) Above entry is reversed when revaluation account shows loss Partners Capital Acs (Old Profit Sharing Ratio) DrTo Revaluation Ac(Loss on revaluation debited to Partnerrsquos Capital Acs)

Proforma of Revaluation Account is given as under

Revaluation Account

Dr Cr Particulars ` Amount Particulars ` Amount To Decrease in value of assets By Increase in value of assets To Increase in value of liabilities By Decrease in value of liabilities To Unrecorded liabilities By Unrecorded assets To Gain on Revaluation (Transferred) By Loss on Revalution (Transferred)

ECO ndash12 2742020Topic- ELASTICITY OF DEMAND

CHAPTER - ELASTICITY OF DEMANDMEANINGDemand for a commodity is affected by many factors such as its price price of related goods income of its buyer tastes and preferences etc Elasticity means degree of response Elasticity of demand means degree of responsiveness of demand Demand for a commodity responds to change in price price of related goods income etc So we have three dimensions of elasticity of demandDIMENSION OF ELASTICITY OF DEMAND TYPES OF ELASTICITY OF DEMAND

Price elasticity of demand Income elasticity of demand Cross Elasticity of demand

Price elasticity of demand Price elasticity of demand means degree of responsiveness of demand for a commodity to the change in its price For example if demand for a commodity rises by 10 due to 5 fall in its price Price elasticity of demand (ep)=Percentage change in quantity demanded Percentage change in price of the commodity = 10 ( -)5 = ( - )2Note that ep will always be negative due to inverse relationship of price and quantity demanded

(ii) Income elasticity of demand Income elasticity of demand refers to the degree of responsiveness of demand for a commodity to the change in income of its buyer Suppose income of buyer rises by 10 and his demand for a commodity rises by 20 then Income elasticity of demand (ey)= change in quantity demanded change in price of the commodity =20 10 = 2

Cross Elasticity of demandCross elasticity of demand means the degree of responsiveness of demand for a commodity to the change in price of its related goods (substitute goods or complementary goods) Suppose demand for a commodity rises by 10 due to 5 rise in price of its substitute good then Cross elasticity of demand (ec) = change in quantity demanded change in price of related good = 10 2 = 5 (Tastes and preferences cannot be expressed numerically So elasticity ofdemand cannot be numerically expressed)

  • Chapter 1 Force (Summary)
  • Distinguish between external sovereignty and internal sovereignty
    • NAND Gate
      • Logic diagram
      • Truth Table
        • NOR Gate
          • Logic diagram
          • Truth Table
            • XOR Gate
              • Logic diagram
              • Truth Table
                • XNOR Gate
                  • Logic diagram
                  • Truth Table
                      • Physics
                      • Chapter 1 Electric Field ( Electric Dipole) (Summary)
Page 54:  · Web viewWe all know that Nouns are divided into two parts: common noun and proper noun.Apart from common and proper noun, we will also study about collective noun and compound

Now come my Ariel Bring a corollaryRather than want a spirit Appear and pertly[to Ferdinand and Miranda]No tongue all eyes Be silent

spirits in attendance Appear briskly

[to Ferdinand and Miranda]Look with your eyes but do not say a word

[Soft music] [Soft music][Enter Iris] [Enter Iris]

IRISCeres most bounteous lady thy rich leasOf wheat rye barley vetches oats and peasThy turfy mountains where live nibbling sheepAnd flat meads thatched with stover them to keepThy banks with pioned and twilled brimsWhich spongy April at thy hest betrimsTo make cold nymphs chaste crowns and thybroom-grovesWhose shadow the dismissegraved bachelor lovesBeing lass-lorn thy pole clipped vineyardAnd thy sea-marge sterile and rocky-hardWhere thou thyself dost airmdashthe Queen othrsquoSkyWhose watery arch and messenger am IBids thee leave these and with her sovereign grace[Juno appears] Here on this grass-plot in this very placeTo come and sport Her peacocks fly amainApproach rich Ceres her to entertain

IRISCeres most generous lady you are the cause of rich fields or fertile land where wheat rye barley beans oats and peas grow the grassy mountains where the sheep graze and the flat meadows covered with coarse hay to be used as fodder for cattleYour banks are covered with marsh-marigolds and reeds and the rainy April under your orders brings forth to make for the maids who are not in love beautiful crowns your woods where the broom flourishes and where the bachelor who has been dismissed by the maid he loved lies down being forsaken your vineyard in which the poles are embraced by the vines and the margin of the sea which is barren and rocky where you roam about to enjoy the fresh air ndash the queen of the sky (Juno) whose messenger I am besides being represented as the rainbow bids you leave all these and with her majesty here on this grassy plot in this very place come and sport her peacocks carry her fast in her chariot through the air and are making their way here approach rich Ceres to welcome her

[Enter Ariel as Ceres] [Enter Ariel as Ceres]

CERESHail many-coloured messenger that neerDost disobey the wife of JupiterWho with thy saffron wings upon my flowersDiffusest honey-drops refreshing showersAnd with each end of thy blue bow dost crownMy bosky acres and my unshrubbed downRich scarf to my proud earth Why hath thy queenSummoned me hither to this short-grassed green

CERESWelcome rainbow that never dared disobey Juno the wife of Jupiter who with your orange coloured rays spread honey-drops refreshing showers And with each end of thy blue bow drown my bushy acres and my hilly country which is free from shrubs you thus forming a rich scarf Why has your queen called me here to this place covered with short grass

IRISA contract of true love to celebrateAnd some donation freely to estateOn the blest lovers

IRISI have called you to celebrate a contract of true love and bestow some liberal gift upon the blessed lovers

ORIGINAL TEXT PARAPHRASECERESTell me heavenly bowIf Venus or her son as thou dost knowDo now attend the queen Since they did plotThe means that dusky Dis my daughter gotHer and her blind boys scandaled companyI have forsworn

CERESTell me heavenly bow if Venus the Goddess of love or Cupid her son and pedlar of passion at this time attend the heavenly queen Juno because you are sure to know Since the day they conspired against me and dark Pluto took away my daughter here and Cupidrsquos disgraceful company I have left off

IRISOf her societyBe not afraid I met her deityCutting the clouds towards Pathos and her sonDove-drawn with her Here thought they to have doneSome wanton charm upon this man and miad

IRISBe not afraid of her company I met her deity moving on the clouds towards Paphos the sacred home of Venus on the island of Cyprus along with her son on her chariot drawn by doves Here they contemplated to exercise a charm upon this man and maid producing

Whose vows are that no bed-right shall be paidTill Hymens torch be lightedmdashbut in vainMarss hot minion is returned againHer waspish-headed son has broke his arrowsSwears he will shoot no more but play with sparrowsAnd be a boy right out

wantonness before the actual marriage ceremony but did not succeed Venus has returned her irritable son has broken his arrows and swears that he will give up his practice of trying to inspire love but play with sparrows and be a boy again

[Music is heard] [Music is heard]

CERESHighst queen of stateGreat Juno comes I know her by her gait

CERESHighest queen of state Great Juno there she comes I know here by her gait

[Enter Juno] [Enter Juno]

JUNOHow does my bounteous sister Go with meTo bless this twain that they may Prosperous beAnd honoured in their issue

JUNOHow are you doing my generous sister Come with me to bless this couple so that they may be prosperous and fortunate in their children

[They sing] [They sing]

JUNOHonour riches marriage-blessingLong continuance and increasingHourly joys be still upon youJuno sings her blessings upon you

JUNOMay honour riches happiness in marriage long continuance and increase of those boons ever rest upon you as hourly joys Juno showers down upon you her blessings in song

CERESEarths increase foison plentyBarns and garners never emptyVines and clustring bunches growingPlants and goodly burden bowingSpring come to you at the farthestIn the very end of harvestScarcity and want shall shun youCeresrsquo blessing so is on you

CERESMay you have the plenty of earthrsquos produce Your barns and granaries may never be empty Your vines may grow with clustering bunches Your fruit trees may be heavily laden with their fruit May there be continuous spring and harvest May scantiness and want leave you forever Such is the blessing of Ceres upon you

FERDINANDThis is a most majestic vision andHarmoniously charmingly May I be boldTo think these spirits

FERDINANDThis is a great vision and magically melodious Should I suppose the characters (taking part in the masque) are spirits

PROSPEROSpirits which by mine artI have from their confines calld to enactMy present fancies

PROSPEROYes they are spirits whom I have summoned from the regions to which they are confined to carry into effect my fanciful designs

ORIGINAL TEXT PARAPHRASEFERDINANDLet me live here everSo rare a wondered father and a wifeMakes this place paradise

FERDINANDI should like to live here forever Such a wise and wonderful father makes this place a paradise

[Juno and Ceres whisper and send Iris on employment] [Juno and Ceres whisper and send Iris on employment]

PROSPEROSweet now silence

PROSPEROMy dear Ferdinand speak no more Juno and Ceres are

Juno and Ceres whisper seriouslyTheres something else to do Hush and be muteOr else our spell is marred

whispering with a solemn look There is something else coming Silence Or else our magic will be spoilt

IRISYour nymphs called naiads of the wandering brooksWith your sedged crowns and over-harmless looksLeave your crisp channels and on this green landAnswer your summons Juno does commandCome temperate nymphs and help to celebrateA contract of true love Be not too late

IRISYou nymphs called Naiads denizens (M inhabitants) of the running stream with your chaplets of sedge and ever-helpful looks leave your wrinkled channels and on the green land answer the summons sent to you Juno has ordered some chaste nymphs and help to celebrate a noble and true marriage Donrsquot delay

[Enter certain nymphs] [Enter certain nymphs]You sunburnt sicklemen of August wearyCome hither from the furrow and be merryMake holiday your rye-straw hats put onAnd these fresh nymphs encounter every oneIn country footing

You sunburnt harvesters weary from the effects of the heat in August come here from the furrowed land and rejoice Make holiday with your rye-straw hats upon you and meet these fresh nymphs and join in country dancing

[Enter certain reapers properly habited They join with the nymphs in a graceful dance towards the end whereof Prospero starts suddenly and speaks]

[Enter certain reapers properly habited They join with the nymphs in a graceful dance towards the end whereof Prospero starts suddenly and speaks]

PROSPERO[aside] I had forgot that foul conspiracyOf the beast Caliban and his confederatesAgainst my life The minute of their plotIs almost come [to the spirits]Well done Avoidno more

PROSPERO(Aside)I had forgotten the wicked conspiracy of the beast Caliban and his accomplices against my life the time of their plot has almost arrived ndash (To the Spirits) well done depart no more of this

[To a strange hollow and confused noise the spirits heavily vanish]

[The spirits depart]

ORIGINAL TEXT PARAPHRASEFERDINANDLet me live here everSo rare a wondered father and a wifeMakes this place paradise

FERDINANDI should like to live here forever Such a wise and wonderful father makes this place a paradise

[Juno and Ceres whisper and send Iris on employment] [Juno and Ceres whisper and send Iris on employment]

PROSPEROSweet now silenceJuno and Ceres whisper seriouslyTheres something else to do Hush and be muteOr else our spell is marred

PROSPEROMy dear Ferdinand speak no more Juno and Ceres are whispering with a solemn look There is something else coming Silence Or else our magic will be spoilt

IRISYour nymphs called naiads of the wandering brooksWith your sedged crowns and over-harmless looksLeave your crisp channels and on this green landAnswer your summons Juno does commandCome temperate nymphs and help to celebrateA contract of true love Be not too late

IRISYou nymphs called Naiads denizens (M inhabitants) of the running stream with your chaplets of sedge and ever-helpful looks leave your wrinkled channels and on the green land answer the summons sent to you Juno has ordered some chaste nymphs and help to celebrate a noble and true marriage Donrsquot delay

[Enter certain nymphs] [Enter certain nymphs]You sunburnt sicklemen of August wearyCome hither from the furrow and be merryMake holiday your rye-straw hats put onAnd these fresh nymphs encounter every oneIn country footing

You sunburnt harvesters weary from the effects of the heat in August come here from the furrowed land and rejoice Make holiday with your rye-straw hats upon you and meet these fresh nymphs and join in country dancing

[Enter certain reapers properly habited They join with the nymphs in a graceful dance towards the end whereof Prospero starts suddenly and speaks]

[Enter certain reapers properly habited They join with the nymphs in a graceful dance towards the end whereof Prospero starts suddenly and speaks]

PROSPERO[aside] I had forgot that foul conspiracyOf the beast Caliban and his confederatesAgainst my life The minute of their plotIs almost come [to the spirits]Well done Avoidno more

PROSPERO(Aside)I had forgotten the wicked conspiracy of the beast Caliban and his accomplices against my life the time of their plot has almost arrived ndash (To the Spirits) well done depart no more of this

[To a strange hollow and confused noise the spirits heavily vanish]

[The spirits depart]

Ac-12 27420 topic Revaluation of Assets and Liabilities

REVALUATION OF ASSETS AND LIABILITIES

On admission of a new partner the firm stands reconstituted and consequently the assets are revalued and liabilities are reassessed It is necessary to show the true position of the firm at the time of admission of a new partner If the values of the assets are raised gain will increase the capital of the existing partners Similarly any decrease in the value of assets ie loss will decrease the capital of the existing partners For this purpose alsquoRevaluation Accountrsquo is prepared This account is credited with all increases in the value of assets and decrease in the value of liabilities It is debited with decrease on account of value of assets and increase in the value of liabilities The balance of this account shows a gain or loss on revaluation which is transferred to the existing partnerrsquos capital account in existing profit sharing ratioAccounting for Revaluation of Assets and Liabilities when there is a Changein the Profit Sharing Ratio of Existing PartnersAssets and liabilities of a firm must also be revalued at the time of change in profit sharing ratio of existing partners The reason is that the realisable or actual value of assets and liabilities may be different from those shown in the Balance Sheet It is possible that with the passage of time some of the assets might have appreciated in value while the value of certain other assets might have decreased and no record has been made of such changes in the books of accounts Similarly there may be some unrecorded assets amp libilities that may have to be accounted for Revaluation of assets and reassessments of liabilities becomes necessary because the change in the

value of assets and liabilities belongs to the period to change in profit sharing ratio and hence must be shared by the partners in their old profit sharing ratio Revaluation of assets and reassessment of liabilities may be given effect to in two different ways (a) When revised values are to be recorded in the books and(b) When revised values are not to be recorded in the books

When revised values are to be recorded in the booksIn such a case revaluation of assets and reassessment of liabilities is done with the help of a new account called lsquoRevaluation Accountrsquo Sometimes this account is also called as lsquoProfit amp Loss Adjustment Acrsquo If there is a loss due to revaluation revaluation account is debited and if the revaluation results in a profit the revaluation account is credited The following journal entries made for this purpose are

(i) For increase in the value of assetsAsset Ac Dr (individually)To Revaluation Ac(ii) For decrease in the value of AssetRevaluation Ac Dr (individually)To Asset Ac[Decrease in the value of assets](iii) For increase in the value of LiabilitiesRevaluation Ac Dr (individually)To Liabilities Ac[Increase in the value of Liabilities](iv) For decrease in the value of LiabilitiesLiabilities Ac DrTo Revaluation Ac[Decrease in the value of Liabilities](v) For unrecorded AssetsAsset Ac [unrecorded] DrTo Revaluation Ac[Unrecorded asset recorded at actual value](vi) For unrecorded Liability Revaluation Ac DrTo Liability Ac [unrecorded][Unrecorded Liability recorded at actual value](vii) For transfer of gain on revaluationRevaluation Ac DrTo Existing Partnerrsquos CapitalCurrent Ac[Profit on revaluation transferred to capital account in existing ratio](viii) For transfer of loss on revaluationExisting Partnerrsquos CapitalCurrent Ac DrTo Revaluation Ac[Loss on revaluation transferred to capital account in existing ratio](a) When revaluation account shows gain Revaluation Ac DrTo Partnerrsquos Capital Ac (Old Profit Sharing Ratio)(Profit on revaluation credited to Partnerrsquos Capital Ac)(b) Above entry is reversed when revaluation account shows loss Partners Capital Acs (Old Profit Sharing Ratio) DrTo Revaluation Ac(Loss on revaluation debited to Partnerrsquos Capital Acs)

Proforma of Revaluation Account is given as under

Revaluation Account

Dr Cr Particulars ` Amount Particulars ` Amount To Decrease in value of assets By Increase in value of assets To Increase in value of liabilities By Decrease in value of liabilities To Unrecorded liabilities By Unrecorded assets To Gain on Revaluation (Transferred) By Loss on Revalution (Transferred)

ECO ndash12 2742020Topic- ELASTICITY OF DEMAND

CHAPTER - ELASTICITY OF DEMANDMEANINGDemand for a commodity is affected by many factors such as its price price of related goods income of its buyer tastes and preferences etc Elasticity means degree of response Elasticity of demand means degree of responsiveness of demand Demand for a commodity responds to change in price price of related goods income etc So we have three dimensions of elasticity of demandDIMENSION OF ELASTICITY OF DEMAND TYPES OF ELASTICITY OF DEMAND

Price elasticity of demand Income elasticity of demand Cross Elasticity of demand

Price elasticity of demand Price elasticity of demand means degree of responsiveness of demand for a commodity to the change in its price For example if demand for a commodity rises by 10 due to 5 fall in its price Price elasticity of demand (ep)=Percentage change in quantity demanded Percentage change in price of the commodity = 10 ( -)5 = ( - )2Note that ep will always be negative due to inverse relationship of price and quantity demanded

(ii) Income elasticity of demand Income elasticity of demand refers to the degree of responsiveness of demand for a commodity to the change in income of its buyer Suppose income of buyer rises by 10 and his demand for a commodity rises by 20 then Income elasticity of demand (ey)= change in quantity demanded change in price of the commodity =20 10 = 2

Cross Elasticity of demandCross elasticity of demand means the degree of responsiveness of demand for a commodity to the change in price of its related goods (substitute goods or complementary goods) Suppose demand for a commodity rises by 10 due to 5 rise in price of its substitute good then Cross elasticity of demand (ec) = change in quantity demanded change in price of related good = 10 2 = 5 (Tastes and preferences cannot be expressed numerically So elasticity ofdemand cannot be numerically expressed)

  • Chapter 1 Force (Summary)
  • Distinguish between external sovereignty and internal sovereignty
    • NAND Gate
      • Logic diagram
      • Truth Table
        • NOR Gate
          • Logic diagram
          • Truth Table
            • XOR Gate
              • Logic diagram
              • Truth Table
                • XNOR Gate
                  • Logic diagram
                  • Truth Table
                      • Physics
                      • Chapter 1 Electric Field ( Electric Dipole) (Summary)
Page 55:  · Web viewWe all know that Nouns are divided into two parts: common noun and proper noun.Apart from common and proper noun, we will also study about collective noun and compound

Whose vows are that no bed-right shall be paidTill Hymens torch be lightedmdashbut in vainMarss hot minion is returned againHer waspish-headed son has broke his arrowsSwears he will shoot no more but play with sparrowsAnd be a boy right out

wantonness before the actual marriage ceremony but did not succeed Venus has returned her irritable son has broken his arrows and swears that he will give up his practice of trying to inspire love but play with sparrows and be a boy again

[Music is heard] [Music is heard]

CERESHighst queen of stateGreat Juno comes I know her by her gait

CERESHighest queen of state Great Juno there she comes I know here by her gait

[Enter Juno] [Enter Juno]

JUNOHow does my bounteous sister Go with meTo bless this twain that they may Prosperous beAnd honoured in their issue

JUNOHow are you doing my generous sister Come with me to bless this couple so that they may be prosperous and fortunate in their children

[They sing] [They sing]

JUNOHonour riches marriage-blessingLong continuance and increasingHourly joys be still upon youJuno sings her blessings upon you

JUNOMay honour riches happiness in marriage long continuance and increase of those boons ever rest upon you as hourly joys Juno showers down upon you her blessings in song

CERESEarths increase foison plentyBarns and garners never emptyVines and clustring bunches growingPlants and goodly burden bowingSpring come to you at the farthestIn the very end of harvestScarcity and want shall shun youCeresrsquo blessing so is on you

CERESMay you have the plenty of earthrsquos produce Your barns and granaries may never be empty Your vines may grow with clustering bunches Your fruit trees may be heavily laden with their fruit May there be continuous spring and harvest May scantiness and want leave you forever Such is the blessing of Ceres upon you

FERDINANDThis is a most majestic vision andHarmoniously charmingly May I be boldTo think these spirits

FERDINANDThis is a great vision and magically melodious Should I suppose the characters (taking part in the masque) are spirits

PROSPEROSpirits which by mine artI have from their confines calld to enactMy present fancies

PROSPEROYes they are spirits whom I have summoned from the regions to which they are confined to carry into effect my fanciful designs

ORIGINAL TEXT PARAPHRASEFERDINANDLet me live here everSo rare a wondered father and a wifeMakes this place paradise

FERDINANDI should like to live here forever Such a wise and wonderful father makes this place a paradise

[Juno and Ceres whisper and send Iris on employment] [Juno and Ceres whisper and send Iris on employment]

PROSPEROSweet now silence

PROSPEROMy dear Ferdinand speak no more Juno and Ceres are

Juno and Ceres whisper seriouslyTheres something else to do Hush and be muteOr else our spell is marred

whispering with a solemn look There is something else coming Silence Or else our magic will be spoilt

IRISYour nymphs called naiads of the wandering brooksWith your sedged crowns and over-harmless looksLeave your crisp channels and on this green landAnswer your summons Juno does commandCome temperate nymphs and help to celebrateA contract of true love Be not too late

IRISYou nymphs called Naiads denizens (M inhabitants) of the running stream with your chaplets of sedge and ever-helpful looks leave your wrinkled channels and on the green land answer the summons sent to you Juno has ordered some chaste nymphs and help to celebrate a noble and true marriage Donrsquot delay

[Enter certain nymphs] [Enter certain nymphs]You sunburnt sicklemen of August wearyCome hither from the furrow and be merryMake holiday your rye-straw hats put onAnd these fresh nymphs encounter every oneIn country footing

You sunburnt harvesters weary from the effects of the heat in August come here from the furrowed land and rejoice Make holiday with your rye-straw hats upon you and meet these fresh nymphs and join in country dancing

[Enter certain reapers properly habited They join with the nymphs in a graceful dance towards the end whereof Prospero starts suddenly and speaks]

[Enter certain reapers properly habited They join with the nymphs in a graceful dance towards the end whereof Prospero starts suddenly and speaks]

PROSPERO[aside] I had forgot that foul conspiracyOf the beast Caliban and his confederatesAgainst my life The minute of their plotIs almost come [to the spirits]Well done Avoidno more

PROSPERO(Aside)I had forgotten the wicked conspiracy of the beast Caliban and his accomplices against my life the time of their plot has almost arrived ndash (To the Spirits) well done depart no more of this

[To a strange hollow and confused noise the spirits heavily vanish]

[The spirits depart]

ORIGINAL TEXT PARAPHRASEFERDINANDLet me live here everSo rare a wondered father and a wifeMakes this place paradise

FERDINANDI should like to live here forever Such a wise and wonderful father makes this place a paradise

[Juno and Ceres whisper and send Iris on employment] [Juno and Ceres whisper and send Iris on employment]

PROSPEROSweet now silenceJuno and Ceres whisper seriouslyTheres something else to do Hush and be muteOr else our spell is marred

PROSPEROMy dear Ferdinand speak no more Juno and Ceres are whispering with a solemn look There is something else coming Silence Or else our magic will be spoilt

IRISYour nymphs called naiads of the wandering brooksWith your sedged crowns and over-harmless looksLeave your crisp channels and on this green landAnswer your summons Juno does commandCome temperate nymphs and help to celebrateA contract of true love Be not too late

IRISYou nymphs called Naiads denizens (M inhabitants) of the running stream with your chaplets of sedge and ever-helpful looks leave your wrinkled channels and on the green land answer the summons sent to you Juno has ordered some chaste nymphs and help to celebrate a noble and true marriage Donrsquot delay

[Enter certain nymphs] [Enter certain nymphs]You sunburnt sicklemen of August wearyCome hither from the furrow and be merryMake holiday your rye-straw hats put onAnd these fresh nymphs encounter every oneIn country footing

You sunburnt harvesters weary from the effects of the heat in August come here from the furrowed land and rejoice Make holiday with your rye-straw hats upon you and meet these fresh nymphs and join in country dancing

[Enter certain reapers properly habited They join with the nymphs in a graceful dance towards the end whereof Prospero starts suddenly and speaks]

[Enter certain reapers properly habited They join with the nymphs in a graceful dance towards the end whereof Prospero starts suddenly and speaks]

PROSPERO[aside] I had forgot that foul conspiracyOf the beast Caliban and his confederatesAgainst my life The minute of their plotIs almost come [to the spirits]Well done Avoidno more

PROSPERO(Aside)I had forgotten the wicked conspiracy of the beast Caliban and his accomplices against my life the time of their plot has almost arrived ndash (To the Spirits) well done depart no more of this

[To a strange hollow and confused noise the spirits heavily vanish]

[The spirits depart]

Ac-12 27420 topic Revaluation of Assets and Liabilities

REVALUATION OF ASSETS AND LIABILITIES

On admission of a new partner the firm stands reconstituted and consequently the assets are revalued and liabilities are reassessed It is necessary to show the true position of the firm at the time of admission of a new partner If the values of the assets are raised gain will increase the capital of the existing partners Similarly any decrease in the value of assets ie loss will decrease the capital of the existing partners For this purpose alsquoRevaluation Accountrsquo is prepared This account is credited with all increases in the value of assets and decrease in the value of liabilities It is debited with decrease on account of value of assets and increase in the value of liabilities The balance of this account shows a gain or loss on revaluation which is transferred to the existing partnerrsquos capital account in existing profit sharing ratioAccounting for Revaluation of Assets and Liabilities when there is a Changein the Profit Sharing Ratio of Existing PartnersAssets and liabilities of a firm must also be revalued at the time of change in profit sharing ratio of existing partners The reason is that the realisable or actual value of assets and liabilities may be different from those shown in the Balance Sheet It is possible that with the passage of time some of the assets might have appreciated in value while the value of certain other assets might have decreased and no record has been made of such changes in the books of accounts Similarly there may be some unrecorded assets amp libilities that may have to be accounted for Revaluation of assets and reassessments of liabilities becomes necessary because the change in the

value of assets and liabilities belongs to the period to change in profit sharing ratio and hence must be shared by the partners in their old profit sharing ratio Revaluation of assets and reassessment of liabilities may be given effect to in two different ways (a) When revised values are to be recorded in the books and(b) When revised values are not to be recorded in the books

When revised values are to be recorded in the booksIn such a case revaluation of assets and reassessment of liabilities is done with the help of a new account called lsquoRevaluation Accountrsquo Sometimes this account is also called as lsquoProfit amp Loss Adjustment Acrsquo If there is a loss due to revaluation revaluation account is debited and if the revaluation results in a profit the revaluation account is credited The following journal entries made for this purpose are

(i) For increase in the value of assetsAsset Ac Dr (individually)To Revaluation Ac(ii) For decrease in the value of AssetRevaluation Ac Dr (individually)To Asset Ac[Decrease in the value of assets](iii) For increase in the value of LiabilitiesRevaluation Ac Dr (individually)To Liabilities Ac[Increase in the value of Liabilities](iv) For decrease in the value of LiabilitiesLiabilities Ac DrTo Revaluation Ac[Decrease in the value of Liabilities](v) For unrecorded AssetsAsset Ac [unrecorded] DrTo Revaluation Ac[Unrecorded asset recorded at actual value](vi) For unrecorded Liability Revaluation Ac DrTo Liability Ac [unrecorded][Unrecorded Liability recorded at actual value](vii) For transfer of gain on revaluationRevaluation Ac DrTo Existing Partnerrsquos CapitalCurrent Ac[Profit on revaluation transferred to capital account in existing ratio](viii) For transfer of loss on revaluationExisting Partnerrsquos CapitalCurrent Ac DrTo Revaluation Ac[Loss on revaluation transferred to capital account in existing ratio](a) When revaluation account shows gain Revaluation Ac DrTo Partnerrsquos Capital Ac (Old Profit Sharing Ratio)(Profit on revaluation credited to Partnerrsquos Capital Ac)(b) Above entry is reversed when revaluation account shows loss Partners Capital Acs (Old Profit Sharing Ratio) DrTo Revaluation Ac(Loss on revaluation debited to Partnerrsquos Capital Acs)

Proforma of Revaluation Account is given as under

Revaluation Account

Dr Cr Particulars ` Amount Particulars ` Amount To Decrease in value of assets By Increase in value of assets To Increase in value of liabilities By Decrease in value of liabilities To Unrecorded liabilities By Unrecorded assets To Gain on Revaluation (Transferred) By Loss on Revalution (Transferred)

ECO ndash12 2742020Topic- ELASTICITY OF DEMAND

CHAPTER - ELASTICITY OF DEMANDMEANINGDemand for a commodity is affected by many factors such as its price price of related goods income of its buyer tastes and preferences etc Elasticity means degree of response Elasticity of demand means degree of responsiveness of demand Demand for a commodity responds to change in price price of related goods income etc So we have three dimensions of elasticity of demandDIMENSION OF ELASTICITY OF DEMAND TYPES OF ELASTICITY OF DEMAND

Price elasticity of demand Income elasticity of demand Cross Elasticity of demand

Price elasticity of demand Price elasticity of demand means degree of responsiveness of demand for a commodity to the change in its price For example if demand for a commodity rises by 10 due to 5 fall in its price Price elasticity of demand (ep)=Percentage change in quantity demanded Percentage change in price of the commodity = 10 ( -)5 = ( - )2Note that ep will always be negative due to inverse relationship of price and quantity demanded

(ii) Income elasticity of demand Income elasticity of demand refers to the degree of responsiveness of demand for a commodity to the change in income of its buyer Suppose income of buyer rises by 10 and his demand for a commodity rises by 20 then Income elasticity of demand (ey)= change in quantity demanded change in price of the commodity =20 10 = 2

Cross Elasticity of demandCross elasticity of demand means the degree of responsiveness of demand for a commodity to the change in price of its related goods (substitute goods or complementary goods) Suppose demand for a commodity rises by 10 due to 5 rise in price of its substitute good then Cross elasticity of demand (ec) = change in quantity demanded change in price of related good = 10 2 = 5 (Tastes and preferences cannot be expressed numerically So elasticity ofdemand cannot be numerically expressed)

  • Chapter 1 Force (Summary)
  • Distinguish between external sovereignty and internal sovereignty
    • NAND Gate
      • Logic diagram
      • Truth Table
        • NOR Gate
          • Logic diagram
          • Truth Table
            • XOR Gate
              • Logic diagram
              • Truth Table
                • XNOR Gate
                  • Logic diagram
                  • Truth Table
                      • Physics
                      • Chapter 1 Electric Field ( Electric Dipole) (Summary)
Page 56:  · Web viewWe all know that Nouns are divided into two parts: common noun and proper noun.Apart from common and proper noun, we will also study about collective noun and compound

Juno and Ceres whisper seriouslyTheres something else to do Hush and be muteOr else our spell is marred

whispering with a solemn look There is something else coming Silence Or else our magic will be spoilt

IRISYour nymphs called naiads of the wandering brooksWith your sedged crowns and over-harmless looksLeave your crisp channels and on this green landAnswer your summons Juno does commandCome temperate nymphs and help to celebrateA contract of true love Be not too late

IRISYou nymphs called Naiads denizens (M inhabitants) of the running stream with your chaplets of sedge and ever-helpful looks leave your wrinkled channels and on the green land answer the summons sent to you Juno has ordered some chaste nymphs and help to celebrate a noble and true marriage Donrsquot delay

[Enter certain nymphs] [Enter certain nymphs]You sunburnt sicklemen of August wearyCome hither from the furrow and be merryMake holiday your rye-straw hats put onAnd these fresh nymphs encounter every oneIn country footing

You sunburnt harvesters weary from the effects of the heat in August come here from the furrowed land and rejoice Make holiday with your rye-straw hats upon you and meet these fresh nymphs and join in country dancing

[Enter certain reapers properly habited They join with the nymphs in a graceful dance towards the end whereof Prospero starts suddenly and speaks]

[Enter certain reapers properly habited They join with the nymphs in a graceful dance towards the end whereof Prospero starts suddenly and speaks]

PROSPERO[aside] I had forgot that foul conspiracyOf the beast Caliban and his confederatesAgainst my life The minute of their plotIs almost come [to the spirits]Well done Avoidno more

PROSPERO(Aside)I had forgotten the wicked conspiracy of the beast Caliban and his accomplices against my life the time of their plot has almost arrived ndash (To the Spirits) well done depart no more of this

[To a strange hollow and confused noise the spirits heavily vanish]

[The spirits depart]

ORIGINAL TEXT PARAPHRASEFERDINANDLet me live here everSo rare a wondered father and a wifeMakes this place paradise

FERDINANDI should like to live here forever Such a wise and wonderful father makes this place a paradise

[Juno and Ceres whisper and send Iris on employment] [Juno and Ceres whisper and send Iris on employment]

PROSPEROSweet now silenceJuno and Ceres whisper seriouslyTheres something else to do Hush and be muteOr else our spell is marred

PROSPEROMy dear Ferdinand speak no more Juno and Ceres are whispering with a solemn look There is something else coming Silence Or else our magic will be spoilt

IRISYour nymphs called naiads of the wandering brooksWith your sedged crowns and over-harmless looksLeave your crisp channels and on this green landAnswer your summons Juno does commandCome temperate nymphs and help to celebrateA contract of true love Be not too late

IRISYou nymphs called Naiads denizens (M inhabitants) of the running stream with your chaplets of sedge and ever-helpful looks leave your wrinkled channels and on the green land answer the summons sent to you Juno has ordered some chaste nymphs and help to celebrate a noble and true marriage Donrsquot delay

[Enter certain nymphs] [Enter certain nymphs]You sunburnt sicklemen of August wearyCome hither from the furrow and be merryMake holiday your rye-straw hats put onAnd these fresh nymphs encounter every oneIn country footing

You sunburnt harvesters weary from the effects of the heat in August come here from the furrowed land and rejoice Make holiday with your rye-straw hats upon you and meet these fresh nymphs and join in country dancing

[Enter certain reapers properly habited They join with the nymphs in a graceful dance towards the end whereof Prospero starts suddenly and speaks]

[Enter certain reapers properly habited They join with the nymphs in a graceful dance towards the end whereof Prospero starts suddenly and speaks]

PROSPERO[aside] I had forgot that foul conspiracyOf the beast Caliban and his confederatesAgainst my life The minute of their plotIs almost come [to the spirits]Well done Avoidno more

PROSPERO(Aside)I had forgotten the wicked conspiracy of the beast Caliban and his accomplices against my life the time of their plot has almost arrived ndash (To the Spirits) well done depart no more of this

[To a strange hollow and confused noise the spirits heavily vanish]

[The spirits depart]

Ac-12 27420 topic Revaluation of Assets and Liabilities

REVALUATION OF ASSETS AND LIABILITIES

On admission of a new partner the firm stands reconstituted and consequently the assets are revalued and liabilities are reassessed It is necessary to show the true position of the firm at the time of admission of a new partner If the values of the assets are raised gain will increase the capital of the existing partners Similarly any decrease in the value of assets ie loss will decrease the capital of the existing partners For this purpose alsquoRevaluation Accountrsquo is prepared This account is credited with all increases in the value of assets and decrease in the value of liabilities It is debited with decrease on account of value of assets and increase in the value of liabilities The balance of this account shows a gain or loss on revaluation which is transferred to the existing partnerrsquos capital account in existing profit sharing ratioAccounting for Revaluation of Assets and Liabilities when there is a Changein the Profit Sharing Ratio of Existing PartnersAssets and liabilities of a firm must also be revalued at the time of change in profit sharing ratio of existing partners The reason is that the realisable or actual value of assets and liabilities may be different from those shown in the Balance Sheet It is possible that with the passage of time some of the assets might have appreciated in value while the value of certain other assets might have decreased and no record has been made of such changes in the books of accounts Similarly there may be some unrecorded assets amp libilities that may have to be accounted for Revaluation of assets and reassessments of liabilities becomes necessary because the change in the

value of assets and liabilities belongs to the period to change in profit sharing ratio and hence must be shared by the partners in their old profit sharing ratio Revaluation of assets and reassessment of liabilities may be given effect to in two different ways (a) When revised values are to be recorded in the books and(b) When revised values are not to be recorded in the books

When revised values are to be recorded in the booksIn such a case revaluation of assets and reassessment of liabilities is done with the help of a new account called lsquoRevaluation Accountrsquo Sometimes this account is also called as lsquoProfit amp Loss Adjustment Acrsquo If there is a loss due to revaluation revaluation account is debited and if the revaluation results in a profit the revaluation account is credited The following journal entries made for this purpose are

(i) For increase in the value of assetsAsset Ac Dr (individually)To Revaluation Ac(ii) For decrease in the value of AssetRevaluation Ac Dr (individually)To Asset Ac[Decrease in the value of assets](iii) For increase in the value of LiabilitiesRevaluation Ac Dr (individually)To Liabilities Ac[Increase in the value of Liabilities](iv) For decrease in the value of LiabilitiesLiabilities Ac DrTo Revaluation Ac[Decrease in the value of Liabilities](v) For unrecorded AssetsAsset Ac [unrecorded] DrTo Revaluation Ac[Unrecorded asset recorded at actual value](vi) For unrecorded Liability Revaluation Ac DrTo Liability Ac [unrecorded][Unrecorded Liability recorded at actual value](vii) For transfer of gain on revaluationRevaluation Ac DrTo Existing Partnerrsquos CapitalCurrent Ac[Profit on revaluation transferred to capital account in existing ratio](viii) For transfer of loss on revaluationExisting Partnerrsquos CapitalCurrent Ac DrTo Revaluation Ac[Loss on revaluation transferred to capital account in existing ratio](a) When revaluation account shows gain Revaluation Ac DrTo Partnerrsquos Capital Ac (Old Profit Sharing Ratio)(Profit on revaluation credited to Partnerrsquos Capital Ac)(b) Above entry is reversed when revaluation account shows loss Partners Capital Acs (Old Profit Sharing Ratio) DrTo Revaluation Ac(Loss on revaluation debited to Partnerrsquos Capital Acs)

Proforma of Revaluation Account is given as under

Revaluation Account

Dr Cr Particulars ` Amount Particulars ` Amount To Decrease in value of assets By Increase in value of assets To Increase in value of liabilities By Decrease in value of liabilities To Unrecorded liabilities By Unrecorded assets To Gain on Revaluation (Transferred) By Loss on Revalution (Transferred)

ECO ndash12 2742020Topic- ELASTICITY OF DEMAND

CHAPTER - ELASTICITY OF DEMANDMEANINGDemand for a commodity is affected by many factors such as its price price of related goods income of its buyer tastes and preferences etc Elasticity means degree of response Elasticity of demand means degree of responsiveness of demand Demand for a commodity responds to change in price price of related goods income etc So we have three dimensions of elasticity of demandDIMENSION OF ELASTICITY OF DEMAND TYPES OF ELASTICITY OF DEMAND

Price elasticity of demand Income elasticity of demand Cross Elasticity of demand

Price elasticity of demand Price elasticity of demand means degree of responsiveness of demand for a commodity to the change in its price For example if demand for a commodity rises by 10 due to 5 fall in its price Price elasticity of demand (ep)=Percentage change in quantity demanded Percentage change in price of the commodity = 10 ( -)5 = ( - )2Note that ep will always be negative due to inverse relationship of price and quantity demanded

(ii) Income elasticity of demand Income elasticity of demand refers to the degree of responsiveness of demand for a commodity to the change in income of its buyer Suppose income of buyer rises by 10 and his demand for a commodity rises by 20 then Income elasticity of demand (ey)= change in quantity demanded change in price of the commodity =20 10 = 2

Cross Elasticity of demandCross elasticity of demand means the degree of responsiveness of demand for a commodity to the change in price of its related goods (substitute goods or complementary goods) Suppose demand for a commodity rises by 10 due to 5 rise in price of its substitute good then Cross elasticity of demand (ec) = change in quantity demanded change in price of related good = 10 2 = 5 (Tastes and preferences cannot be expressed numerically So elasticity ofdemand cannot be numerically expressed)

  • Chapter 1 Force (Summary)
  • Distinguish between external sovereignty and internal sovereignty
    • NAND Gate
      • Logic diagram
      • Truth Table
        • NOR Gate
          • Logic diagram
          • Truth Table
            • XOR Gate
              • Logic diagram
              • Truth Table
                • XNOR Gate
                  • Logic diagram
                  • Truth Table
                      • Physics
                      • Chapter 1 Electric Field ( Electric Dipole) (Summary)
Page 57:  · Web viewWe all know that Nouns are divided into two parts: common noun and proper noun.Apart from common and proper noun, we will also study about collective noun and compound

[Enter certain reapers properly habited They join with the nymphs in a graceful dance towards the end whereof Prospero starts suddenly and speaks]

[Enter certain reapers properly habited They join with the nymphs in a graceful dance towards the end whereof Prospero starts suddenly and speaks]

PROSPERO[aside] I had forgot that foul conspiracyOf the beast Caliban and his confederatesAgainst my life The minute of their plotIs almost come [to the spirits]Well done Avoidno more

PROSPERO(Aside)I had forgotten the wicked conspiracy of the beast Caliban and his accomplices against my life the time of their plot has almost arrived ndash (To the Spirits) well done depart no more of this

[To a strange hollow and confused noise the spirits heavily vanish]

[The spirits depart]

Ac-12 27420 topic Revaluation of Assets and Liabilities

REVALUATION OF ASSETS AND LIABILITIES

On admission of a new partner the firm stands reconstituted and consequently the assets are revalued and liabilities are reassessed It is necessary to show the true position of the firm at the time of admission of a new partner If the values of the assets are raised gain will increase the capital of the existing partners Similarly any decrease in the value of assets ie loss will decrease the capital of the existing partners For this purpose alsquoRevaluation Accountrsquo is prepared This account is credited with all increases in the value of assets and decrease in the value of liabilities It is debited with decrease on account of value of assets and increase in the value of liabilities The balance of this account shows a gain or loss on revaluation which is transferred to the existing partnerrsquos capital account in existing profit sharing ratioAccounting for Revaluation of Assets and Liabilities when there is a Changein the Profit Sharing Ratio of Existing PartnersAssets and liabilities of a firm must also be revalued at the time of change in profit sharing ratio of existing partners The reason is that the realisable or actual value of assets and liabilities may be different from those shown in the Balance Sheet It is possible that with the passage of time some of the assets might have appreciated in value while the value of certain other assets might have decreased and no record has been made of such changes in the books of accounts Similarly there may be some unrecorded assets amp libilities that may have to be accounted for Revaluation of assets and reassessments of liabilities becomes necessary because the change in the

value of assets and liabilities belongs to the period to change in profit sharing ratio and hence must be shared by the partners in their old profit sharing ratio Revaluation of assets and reassessment of liabilities may be given effect to in two different ways (a) When revised values are to be recorded in the books and(b) When revised values are not to be recorded in the books

When revised values are to be recorded in the booksIn such a case revaluation of assets and reassessment of liabilities is done with the help of a new account called lsquoRevaluation Accountrsquo Sometimes this account is also called as lsquoProfit amp Loss Adjustment Acrsquo If there is a loss due to revaluation revaluation account is debited and if the revaluation results in a profit the revaluation account is credited The following journal entries made for this purpose are

(i) For increase in the value of assetsAsset Ac Dr (individually)To Revaluation Ac(ii) For decrease in the value of AssetRevaluation Ac Dr (individually)To Asset Ac[Decrease in the value of assets](iii) For increase in the value of LiabilitiesRevaluation Ac Dr (individually)To Liabilities Ac[Increase in the value of Liabilities](iv) For decrease in the value of LiabilitiesLiabilities Ac DrTo Revaluation Ac[Decrease in the value of Liabilities](v) For unrecorded AssetsAsset Ac [unrecorded] DrTo Revaluation Ac[Unrecorded asset recorded at actual value](vi) For unrecorded Liability Revaluation Ac DrTo Liability Ac [unrecorded][Unrecorded Liability recorded at actual value](vii) For transfer of gain on revaluationRevaluation Ac DrTo Existing Partnerrsquos CapitalCurrent Ac[Profit on revaluation transferred to capital account in existing ratio](viii) For transfer of loss on revaluationExisting Partnerrsquos CapitalCurrent Ac DrTo Revaluation Ac[Loss on revaluation transferred to capital account in existing ratio](a) When revaluation account shows gain Revaluation Ac DrTo Partnerrsquos Capital Ac (Old Profit Sharing Ratio)(Profit on revaluation credited to Partnerrsquos Capital Ac)(b) Above entry is reversed when revaluation account shows loss Partners Capital Acs (Old Profit Sharing Ratio) DrTo Revaluation Ac(Loss on revaluation debited to Partnerrsquos Capital Acs)

Proforma of Revaluation Account is given as under

Revaluation Account

Dr Cr Particulars ` Amount Particulars ` Amount To Decrease in value of assets By Increase in value of assets To Increase in value of liabilities By Decrease in value of liabilities To Unrecorded liabilities By Unrecorded assets To Gain on Revaluation (Transferred) By Loss on Revalution (Transferred)

ECO ndash12 2742020Topic- ELASTICITY OF DEMAND

CHAPTER - ELASTICITY OF DEMANDMEANINGDemand for a commodity is affected by many factors such as its price price of related goods income of its buyer tastes and preferences etc Elasticity means degree of response Elasticity of demand means degree of responsiveness of demand Demand for a commodity responds to change in price price of related goods income etc So we have three dimensions of elasticity of demandDIMENSION OF ELASTICITY OF DEMAND TYPES OF ELASTICITY OF DEMAND

Price elasticity of demand Income elasticity of demand Cross Elasticity of demand

Price elasticity of demand Price elasticity of demand means degree of responsiveness of demand for a commodity to the change in its price For example if demand for a commodity rises by 10 due to 5 fall in its price Price elasticity of demand (ep)=Percentage change in quantity demanded Percentage change in price of the commodity = 10 ( -)5 = ( - )2Note that ep will always be negative due to inverse relationship of price and quantity demanded

(ii) Income elasticity of demand Income elasticity of demand refers to the degree of responsiveness of demand for a commodity to the change in income of its buyer Suppose income of buyer rises by 10 and his demand for a commodity rises by 20 then Income elasticity of demand (ey)= change in quantity demanded change in price of the commodity =20 10 = 2

Cross Elasticity of demandCross elasticity of demand means the degree of responsiveness of demand for a commodity to the change in price of its related goods (substitute goods or complementary goods) Suppose demand for a commodity rises by 10 due to 5 rise in price of its substitute good then Cross elasticity of demand (ec) = change in quantity demanded change in price of related good = 10 2 = 5 (Tastes and preferences cannot be expressed numerically So elasticity ofdemand cannot be numerically expressed)

  • Chapter 1 Force (Summary)
  • Distinguish between external sovereignty and internal sovereignty
    • NAND Gate
      • Logic diagram
      • Truth Table
        • NOR Gate
          • Logic diagram
          • Truth Table
            • XOR Gate
              • Logic diagram
              • Truth Table
                • XNOR Gate
                  • Logic diagram
                  • Truth Table
                      • Physics
                      • Chapter 1 Electric Field ( Electric Dipole) (Summary)
Page 58:  · Web viewWe all know that Nouns are divided into two parts: common noun and proper noun.Apart from common and proper noun, we will also study about collective noun and compound

value of assets and liabilities belongs to the period to change in profit sharing ratio and hence must be shared by the partners in their old profit sharing ratio Revaluation of assets and reassessment of liabilities may be given effect to in two different ways (a) When revised values are to be recorded in the books and(b) When revised values are not to be recorded in the books

When revised values are to be recorded in the booksIn such a case revaluation of assets and reassessment of liabilities is done with the help of a new account called lsquoRevaluation Accountrsquo Sometimes this account is also called as lsquoProfit amp Loss Adjustment Acrsquo If there is a loss due to revaluation revaluation account is debited and if the revaluation results in a profit the revaluation account is credited The following journal entries made for this purpose are

(i) For increase in the value of assetsAsset Ac Dr (individually)To Revaluation Ac(ii) For decrease in the value of AssetRevaluation Ac Dr (individually)To Asset Ac[Decrease in the value of assets](iii) For increase in the value of LiabilitiesRevaluation Ac Dr (individually)To Liabilities Ac[Increase in the value of Liabilities](iv) For decrease in the value of LiabilitiesLiabilities Ac DrTo Revaluation Ac[Decrease in the value of Liabilities](v) For unrecorded AssetsAsset Ac [unrecorded] DrTo Revaluation Ac[Unrecorded asset recorded at actual value](vi) For unrecorded Liability Revaluation Ac DrTo Liability Ac [unrecorded][Unrecorded Liability recorded at actual value](vii) For transfer of gain on revaluationRevaluation Ac DrTo Existing Partnerrsquos CapitalCurrent Ac[Profit on revaluation transferred to capital account in existing ratio](viii) For transfer of loss on revaluationExisting Partnerrsquos CapitalCurrent Ac DrTo Revaluation Ac[Loss on revaluation transferred to capital account in existing ratio](a) When revaluation account shows gain Revaluation Ac DrTo Partnerrsquos Capital Ac (Old Profit Sharing Ratio)(Profit on revaluation credited to Partnerrsquos Capital Ac)(b) Above entry is reversed when revaluation account shows loss Partners Capital Acs (Old Profit Sharing Ratio) DrTo Revaluation Ac(Loss on revaluation debited to Partnerrsquos Capital Acs)

Proforma of Revaluation Account is given as under

Revaluation Account

Dr Cr Particulars ` Amount Particulars ` Amount To Decrease in value of assets By Increase in value of assets To Increase in value of liabilities By Decrease in value of liabilities To Unrecorded liabilities By Unrecorded assets To Gain on Revaluation (Transferred) By Loss on Revalution (Transferred)

ECO ndash12 2742020Topic- ELASTICITY OF DEMAND

CHAPTER - ELASTICITY OF DEMANDMEANINGDemand for a commodity is affected by many factors such as its price price of related goods income of its buyer tastes and preferences etc Elasticity means degree of response Elasticity of demand means degree of responsiveness of demand Demand for a commodity responds to change in price price of related goods income etc So we have three dimensions of elasticity of demandDIMENSION OF ELASTICITY OF DEMAND TYPES OF ELASTICITY OF DEMAND

Price elasticity of demand Income elasticity of demand Cross Elasticity of demand

Price elasticity of demand Price elasticity of demand means degree of responsiveness of demand for a commodity to the change in its price For example if demand for a commodity rises by 10 due to 5 fall in its price Price elasticity of demand (ep)=Percentage change in quantity demanded Percentage change in price of the commodity = 10 ( -)5 = ( - )2Note that ep will always be negative due to inverse relationship of price and quantity demanded

(ii) Income elasticity of demand Income elasticity of demand refers to the degree of responsiveness of demand for a commodity to the change in income of its buyer Suppose income of buyer rises by 10 and his demand for a commodity rises by 20 then Income elasticity of demand (ey)= change in quantity demanded change in price of the commodity =20 10 = 2

Cross Elasticity of demandCross elasticity of demand means the degree of responsiveness of demand for a commodity to the change in price of its related goods (substitute goods or complementary goods) Suppose demand for a commodity rises by 10 due to 5 rise in price of its substitute good then Cross elasticity of demand (ec) = change in quantity demanded change in price of related good = 10 2 = 5 (Tastes and preferences cannot be expressed numerically So elasticity ofdemand cannot be numerically expressed)

  • Chapter 1 Force (Summary)
  • Distinguish between external sovereignty and internal sovereignty
    • NAND Gate
      • Logic diagram
      • Truth Table
        • NOR Gate
          • Logic diagram
          • Truth Table
            • XOR Gate
              • Logic diagram
              • Truth Table
                • XNOR Gate
                  • Logic diagram
                  • Truth Table
                      • Physics
                      • Chapter 1 Electric Field ( Electric Dipole) (Summary)
Page 59:  · Web viewWe all know that Nouns are divided into two parts: common noun and proper noun.Apart from common and proper noun, we will also study about collective noun and compound

Proforma of Revaluation Account is given as under

Revaluation Account

Dr Cr Particulars ` Amount Particulars ` Amount To Decrease in value of assets By Increase in value of assets To Increase in value of liabilities By Decrease in value of liabilities To Unrecorded liabilities By Unrecorded assets To Gain on Revaluation (Transferred) By Loss on Revalution (Transferred)

ECO ndash12 2742020Topic- ELASTICITY OF DEMAND

CHAPTER - ELASTICITY OF DEMANDMEANINGDemand for a commodity is affected by many factors such as its price price of related goods income of its buyer tastes and preferences etc Elasticity means degree of response Elasticity of demand means degree of responsiveness of demand Demand for a commodity responds to change in price price of related goods income etc So we have three dimensions of elasticity of demandDIMENSION OF ELASTICITY OF DEMAND TYPES OF ELASTICITY OF DEMAND

Price elasticity of demand Income elasticity of demand Cross Elasticity of demand

Price elasticity of demand Price elasticity of demand means degree of responsiveness of demand for a commodity to the change in its price For example if demand for a commodity rises by 10 due to 5 fall in its price Price elasticity of demand (ep)=Percentage change in quantity demanded Percentage change in price of the commodity = 10 ( -)5 = ( - )2Note that ep will always be negative due to inverse relationship of price and quantity demanded

(ii) Income elasticity of demand Income elasticity of demand refers to the degree of responsiveness of demand for a commodity to the change in income of its buyer Suppose income of buyer rises by 10 and his demand for a commodity rises by 20 then Income elasticity of demand (ey)= change in quantity demanded change in price of the commodity =20 10 = 2

Cross Elasticity of demandCross elasticity of demand means the degree of responsiveness of demand for a commodity to the change in price of its related goods (substitute goods or complementary goods) Suppose demand for a commodity rises by 10 due to 5 rise in price of its substitute good then Cross elasticity of demand (ec) = change in quantity demanded change in price of related good = 10 2 = 5 (Tastes and preferences cannot be expressed numerically So elasticity ofdemand cannot be numerically expressed)

  • Chapter 1 Force (Summary)
  • Distinguish between external sovereignty and internal sovereignty
    • NAND Gate
      • Logic diagram
      • Truth Table
        • NOR Gate
          • Logic diagram
          • Truth Table
            • XOR Gate
              • Logic diagram
              • Truth Table
                • XNOR Gate
                  • Logic diagram
                  • Truth Table
                      • Physics
                      • Chapter 1 Electric Field ( Electric Dipole) (Summary)
Page 60:  · Web viewWe all know that Nouns are divided into two parts: common noun and proper noun.Apart from common and proper noun, we will also study about collective noun and compound

Cross Elasticity of demandCross elasticity of demand means the degree of responsiveness of demand for a commodity to the change in price of its related goods (substitute goods or complementary goods) Suppose demand for a commodity rises by 10 due to 5 rise in price of its substitute good then Cross elasticity of demand (ec) = change in quantity demanded change in price of related good = 10 2 = 5 (Tastes and preferences cannot be expressed numerically So elasticity ofdemand cannot be numerically expressed)

  • Chapter 1 Force (Summary)
  • Distinguish between external sovereignty and internal sovereignty
    • NAND Gate
      • Logic diagram
      • Truth Table
        • NOR Gate
          • Logic diagram
          • Truth Table
            • XOR Gate
              • Logic diagram
              • Truth Table
                • XNOR Gate
                  • Logic diagram
                  • Truth Table
                      • Physics
                      • Chapter 1 Electric Field ( Electric Dipole) (Summary)
Page 61:  · Web viewWe all know that Nouns are divided into two parts: common noun and proper noun.Apart from common and proper noun, we will also study about collective noun and compound
  • Chapter 1 Force (Summary)
  • Distinguish between external sovereignty and internal sovereignty
    • NAND Gate
      • Logic diagram
      • Truth Table
        • NOR Gate
          • Logic diagram
          • Truth Table
            • XOR Gate
              • Logic diagram
              • Truth Table
                • XNOR Gate
                  • Logic diagram
                  • Truth Table
                      • Physics
                      • Chapter 1 Electric Field ( Electric Dipole) (Summary)